MKSAP
PID tx?
IM CTX x1 + PO doxy (100mg) BID x14 days
What kind of acid/base disorder does milk alkali cause?
Metabolic ALKALOSIS
Tx for WPW?
Procainamide
A 67-year-old man is evaluated in the emergency department with a 3-day history of weakness and nausea and a 2-week history of difficulty swallowing. He has lost 22.7 kg (50 lb) during the past year. He has no other symptoms. History is significant for a 30-pack-year history of smoking. He quit smoking 4 years ago. He takes no medications. On physical examination, vital signs are normal. Lung examination reveals decreased tactile fremitus above the lower portion of the right lung as well as dullness to percussion and decreased breath sounds. His neurologic examination is normal. Laboratory studies reveal a serum sodium concentration of 127 mEq/L (127 mmol/L). A chest CT scan is shown.
Small cell = SIADH!!! Squamous = PTHrP (hyperCa)!!! Typical imaging findings in patients with small cell lung cancer (SCLC) include a large mediastinal mass; hyponatremia due to ectopic production of antidiuretic hormone is most often due to SCLC and is rarely seen with other lung tumors. The most likely diagnosis is small cell lung cancer (SCLC). SCLC is a neuroendocrine tumor that accounts for approximately 15% of all lung cancers and occurs predominantly in smokers. This patient has signs and symptoms of hyponatremia and chest CT scan shows a large mediastinal right hilar mass and right-lower-lobe mass (arrows). Imaging studies in SCLC commonly demonstrate a large hilar mass with bulky mediastinal lymphadenopathy; some patients may not have an obvious primary lesion. Signs and symptoms include cough, dyspnea, weight loss, and debility. Less commonly SCLC can present with endocrinologic or neurologic paraneoplastic syndromes. The syndrome of inappropriate antidiuretic hormone secretion (SIADH) due to ectopic production of antidiuretic hormone (ADH) is most often due to a SCLC and is rarely seen with other lung tumors. It occurs in approximately 10% of patients and results in hyponatremia. The severity of symptoms is related to the degree of hyponatremia and rapidity of the decrease. They may include anorexia, nausea, and vomiting, but if the decrease is rapid, cerebral edema can occur and may result in irritability, restlessness, personality changes, confusion, coma, seizures, and respiratory arrest. SCLC is more aggressive than non-small cell lung cancer (NSCLC), is usually already disseminated at presentation, and requires prompt treatment; however, it is often initially more sensitive to chemotherapy and radiotherapy but typically relapses and becomes resistant to further treatment. Staging should not delay treatment. NSCLC accounts for 80% of lung cancers, of which adenocarcinoma is the most common type, followed by squamous cell. Although NSCLC is in the differential diagnosis for this patient, hyponatremia and a large mediastinal mass are most consistent with SCLC. Malignant pleural mesothelioma typically presents as a recurrent exudative pleural effusion with pleural thickening in a patient with exposure to asbestos. This patient did not present with a pleural effusion, which makes this diagnosis less likely.
A 28-year-old woman arrives to discuss possible recent exposure to Zika virus. She is pregnant, at 8 weeks' gestation, and feels well. Her husband returned home from Brazil 4 weeks ago; while away, he experienced a 5-day febrile illness, which was accompanied by a faint rash, headache, and myalgias, all of which resolved before he arrived home. After his return, they engaged in unprotected sexual intercourse. Her only medication is a prenatal vitamin. On physical examination, vital signs are normal, and other examination findings are unremarkable. Which of the following is the most appropriate test to perform next? Dengue virus IgM and IgG antibodies test Uterine ultrasonography every 3 weeks Zika virus IgM antibody test Zika virus RNA nucleic acid amplification test
Testing for Zika virus IgM antibodies is the most appropriate management for this pregnant patient, who has had possible exposure to Zika virus through unprotected sexual activity. Asymptomatic pregnant women not living in an area with active Zika virus transmission who may have been exposed more than 2 weeks previously require testing for Zika virus IgM antibodies; a positive result would indicate probable recent infection. Men who have had symptomatic or asymptomatic infection have been proven to have detectable Zika virus RNA in their semen for up to several weeks, with subsequent sexual transmission to their female partner. Under such circumstances, condoms should be used with each sexual encounter for at least 3 months. Her husband's recent viral-like illness occurring in an endemic geographic area, although unconfirmed, must be presumed to have been Zika virus infection. Evaluating her for evidence of asymptomatic infection is of paramount importance to provide counseling and investigate the possible risk of congenital Zika syndrome, of which microcephaly is the most frequent manifestation. Patients with dengue may be asymptomatic or present with acute febrile illness associated with frontal headache, retro-orbital pain, myalgia, and arthralgia, with or without purpura, melena, or conjunctival injection. Gastrointestinal or respiratory symptoms may predominate. Severe lumbosacral pain is characteristic ("breakbone fever"). As the fever abates, a macular or scarlatiniform rash, which spares the palms and soles and evolves into areas of petechiae on extensor surfaces, may develop. The husband's symptoms are not compatible with dengue. Even if he did have dengue, he cannot transmit the infection to his wife, and testing her for dengue infection is not indicated. Pregnant women with proven or presumptive recent Zika virus infection require serial ultrasonography every 3 to 4 weeks to assess fetal anatomy and growth. Frequent ultrasonographic monitoring may be appropriate if recent Zika infection is confirmed. RNA nucleic acid amplification testing for the presence of Zika virus in serum and urine has a greatly diminished sensitivity if performed more than 2 weeks after symptomatic or asymptomatic exposure. In this case, Zika virus IgM antibody testing is preferred.
A 46-year-old woman is evaluated for new-onset fever and discomfort at the site of a peripherally inserted central catheter (PICC). She was hospitalized for acute pancreatitis and required the PICC for intravenous access and hydration. On physical examination, temperature is 37.8 °C (100.1 °F), and the remaining vital signs are normal. Right brachial PICC line site is tender, without erythema. Cardiac examination is without murmurs. Two sets of peripheral blood cultures are obtained, showing gram-positive cocci in clusters. Both sets of blood cultures are growing coagulase-negative Staphylococcus resistant to methicillin. Vancomycin is initiated. Which of the following is the most appropriate additional management? Maintain PICC line and continue vancomycin for 4 weeks Maintain PICC line and perform transesophageal echocardiography Remove PICC line Remove PICC line and repeat blood cultures
The most appropriate management is to remove the peripherally inserted central catheter (PICC) and repeat blood cultures. Central line-associated bloodstream infections (CLABSIs) can occur with any type of catheter, including short-term peripheral intravenous catheters and PICCs. They are most commonly caused by coagulase-negative Staphylococcus followed by S. aureus. Coagulase-negative Staphylococcus is less virulent than S. aureus and is less likely to cause metastatic infection or endocarditis in patients without prosthetic devices or endovascular hardware (such as prosthetic heart valves) in place. Coagulase-negative Staphylococcus CLABSIs often resolve with removal of the catheter. Blood cultures should be repeated after catheter removal to document clearance of the organism. Alternatively, patients with coagulase-negative Staphylococcus CLABSIs can be treated with antibiotics for 5 to 7 days if the catheter is removed and for 10 to 14 days in combination with antibiotic lock therapy if the catheter is not removed. Most uncomplicated CLABSIs are treated for 7 to 14 days; S. aureus CLABSIs may require treatment for 4 to 6 weeks after catheter removal. Shorter therapy durations (14 days) may be considered in select patients with S. aureus CLABSI whose catheters have been removed and who do not have diabetes mellitus, are not immunosuppressed, have no implanted prosthetic devices, have no evidence of endocarditis by echocardiography (preferably transesophageal), and have no evidence of infected thrombophlebitis; whose fever and bacteremia resolve within 72 hours of starting appropriate therapy; and whose physical examination and diagnostic tests do not suggest the presence of metastatic infection. In patients with persistent bacteremia or endovascular hardware, transesophageal echocardiography may be pursued to evaluate for endocarditis. In patients with endovascular hardware, longer durations of antibiotics are generally required. Catheters should always be removed when endocarditis, metastatic infection, hemodynamic instability, suppurative thrombophlebitis, or persistent bacteremia is evident.
A 45-year-old man is evaluated for a 3-day history of fever, myalgia, headache, and nonproductive cough. He works as a large-animal veterinarian. Medical history is unremarkable, and he takes no medications. On physical examination, vital signs are normal except for a temperature of 38.2 °C (100.8 °F). Oxygen saturation is 94% breathing ambient air. The examination is otherwise unremarkable. A chest radiograph shows a patchy right lower lobe interstitial infiltrate. Which of the following is the most likely cause of his illness? Bacillus anthracis Coxiella burnetii Chlamydia psittaci Francisella tularensis Yersinia pestis
The most likely cause of this patient's illness is Coxiella burnetii. He presents with community-acquired pneumonia not requiring hospitalization, which is most frequently caused by atypical bacteria, Streptococcus pneumoniae, or a viral pathogen. His occupation puts him at risk for zoonotic causes of pneumonia as well. Many zoonotic organisms can potentially cause pulmonary infection, but they can usually be differentiated based on illness severity and animal reservoir. In this patient, the relatively mild infection coupled with exposure to livestock makes C. burnetii the most likely culprit. C. burnetii, which causes Q fever pneumonia, is most frequently associated with exposure to farm animals, parturient animals in particular. High rates of seropositivity have been reported in farmers, veterinarians, and abattoir workers. Infection can occur without direct animal exposure; increased rates of Q fever pneumonia have been found among persons residing in proximity to livestock farms. Infection occurs after inhalation of aerosolized bodily fluids from infected animals. Acute pulmonary infection ranges from subclinical to severe, but it is rarely fatal. Treatment with doxycycline is indicated to decrease symptom duration and to prevent progression to chronic Q fever. Bacillus anthracis is the causative agent of anthrax. Cutaneous anthrax is the most common naturally occurring form of the disease. In the United States, inhalation anthrax is almost exclusively of concern as an agent of bioterrorism. In endemic areas, pulmonary infection may occur after inhalation of spores from the fur or hide of infected livestock, particularly goats and cattle. Inhalation anthrax is a fulminant, often fatal infection. Chlamydia psittaci is the causative agent of psittacosis, which typically presents as pneumonia associated with abrupt onset of fever, severe headache, and dry cough. This organism is associated with inhalation of dried bird droppings, so bird owners or breeders and poultry farmers are at particularly high risk for this infection. Pneumonic tularemia occurs either after direct inhalation or through secondary spread of Francisella tularensis into the lungs. Pneumonic tularemia is characterized by a nonproductive cough, dyspnea, and substernal or pleuritic chest pain. Chest radiographs show infiltrates, hilar lymphadenopathy, and pleural effusion. In the United States, most infections are transmitted through animal exposure. Hunters are at particularly high risk for primary tularemia pneumonia through skinning and dressing of infected rabbits or other wild game. Yersinia pestis infection of the lung causes pneumonic plague. Rodents serve as the primary reservoir for plague. Pulmonary infection occurs through droplet transmission from an infected host or secondary spread from an extrapulmonary source. Patients present with sudden high fever, pleuritic chest pain, a productive cough, and hemoptysis. Pneumonic plague is almost uniformly fatal unless recognized and treated promptly. Read Related TextNext Question
CN poisoning tx?
Hydroxycobalamin- effectively binds intracellular cyanide to form cyanocobalamin Sodium nitrite Sodium thiosulfate
When to test for G6PD def?
IN ACUTE REACTION!!! Won't be positive when not under stress Testing for glucose-6-phosphate dehydrogenase activity is reliable in men who are not experiencing an acute hemolytic episode but is less useful during acute episodes because reticulocytes produce higher levels of enzyme, resulting in a falsely normal test result. G6PD levels should not be checked during an acute hemolytic event because they are normal in newly produced erythrocytes but decrease up to 75% as the cells age. As a result, during an acute hemolytic event, the older, more deficient erythrocytes are preferentially destroyed, leaving the newer less deficient cells to circulate. This leads to falsely normal G6PD levels, so G6PD measurement should be delayed for several weeks after a hemolytic event.
Hyperhemolysis syndrome- presentation, pathophys?
In some patients with sickle cell disease, a rare condition known as hyperhemolysis is thought to be responsible for an acute hemolytic anemia and reticulocytosis. Episodes have sometimes been associated with delayed transfusion reactions and acute vasoocclusive events. Hyperhemolysis syndrome would be associated with an elevated reticulocyte count, not a reduced count.
FUO workup?
Initial studies for fever of unknown origin in most patients typically include a complete blood count with differential, complete metabolic profile with kidney and liver studies, at least three blood culture sets and cultures of other bodily fluids (such as urine or from other sources based on clinical suspicion), an erythrocyte sedimentation rate, tuberculosis testing, and serology for HIV; it is reasonable to perform chest imaging (radiography or CT) as initial diagnostic imaging - evaluate for abscess, neoplasm (hepatoma, renal cell carcinoma), splenomegaly, and lymphadenopathy
A 49-year-old woman is evaluated in the office following a recent hospitalization for an asthma exacerbation. Her symptoms have improved but she continues to have dyspnea and intermittent wheezing. She has had two other hospitalizations within the past year for asthma exacerbations despite the chronic use of oral glucocorticoids. Other than a 3-year history of asthma, her medical history is unremarkable. Medications are mometasone/formoterol, montelukast, albuterol, tiotropium, and prednisone. On physical examination, vital signs are normal. Oxygen saturation is 95% on ambient air. Pulmonary examination reveals expiratory wheezes with good air movement. The remainder of the physical examination is unremarkable. Laboratory studies reveal leukocyte count of 10,000/µL (10 × 109/L) with 650 eosinophils/µL (0.65 × 109/L). Serum IgE level is 12 U/mL (12 kU/L) (normal range, 0-90 U/mL [0-90 kU/L]). FEV1 is 56% of predicted. Chest radiograph is normal. Begin doxycycline Change mometasone/formoterol to fluticasone/salmeterol Initiate a trial of mepolizumab therapy Initiate a trial of omalizumab therapy
Treat a patient with eosinophilic asthma with mepolizumab = IL5 inhibitor The most appropriate treatment is initiation of a trial of mepolizumab therapy. This patient has signs and symptoms of severe persistent asthma that is uncontrolled (more than two exacerbations per year), despite treatment with a high-dose inhaled glucocorticoid, long-acting β2-agonist, long-acting anticholinergic agent, leukotriene-receptor antagonist, and oral glucocorticoids, so her therapy should be stepped up. Biologic therapies that target atopic pathways of asthma are indicated in appropriate patients. Currently available agents are directed against either eosinophils or their products (IgE). Interleukin (IL-5) is a pro-eosinophilic cytokine that promotes eosinophil production and contributes to eosinophilic inflammation in the airways. Mepolizumab and reslizumab are monoclonal antibodies to IL-5 and both agents reduce exacerbations of severe asthma in patients who have blood eosinophil counts of 150/µL or 300/µL, respectively, or higher. Mepolizumab is administered subcutaneously every 4 weeks, whereas reslizumab is an infusion. Although these agents are expensive, patients with uncontrolled eosinophilic asthma treated with mepolizumab have reduced emergency department visits and hospitalizations, and both agents have reduced the requirement for inhaled and oral glucocorticoids. Use of these agents is contraindicated during acute asthma exacerbations. Treatment with doxycycline is not indicated because there are no signs or symptoms of bacterial infection. Changing from one inhaled glucocorticoid/long-acting β2-agonist to another would not be expected to have a significant effect, as the inhaled glucocorticoid dose is similar and there is no apparent difference in the efficacy of the various combinations. Omalizumab, a humanized monoclonal antibody directed at IgE, was the first biologic agent approved by the FDA for use in asthma. Administered subcutaneously every 2 to 4 weeks, omalizumab is indicated in patients with moderate to severe persistent asthma with the following characteristics: (1) symptoms inadequately controlled with inhaled glucocorticoids, (2) evidence of allergies to perennial aeroallergens, and (3) serum IgE levels between 30 and 700 U/mL (30-700 kU/L) (normal range, 0-90 U/mL [0-90 kU/L]). Omalizumab has been shown to reduce exacerbations and emergency department visits; it is not indicated for use in patients other than those meeting these treatment parameters. *Omalizumab indications- (1) symptoms inadequately controlled with inhaled glucocorticoids, (2) evidence of allergies to perennial aeroallergens, and (3) serum IgE levels between 30 and 700 U/mL (30-700 kU/L) (normal range, 0-90 U/mL [0-90 kU/L]).
What is a high ankle sprain?
A high ankle sprain is when you tear or damage the high ankle ligaments that connect the tibia to the fibula.
A 78-year-old woman is evaluated for a 2-week history of pain and swelling in the left wrist. In the past year, she experienced two similar episodes, one in the left knee and one in the left wrist, each resolving after 3 weeks. On physical examination, the patient appears well. Vital signs are normal. The left wrist is swollen, warm, and tender, with decreased range of motion. Laboratory studies show an erythrocyte sedimentation rate of 53 mm/h and a serum urate level of 3.8 mg/dL (0.22 mmol/L). A plain radiograph of the left wrist is shown. Which of the following is the most likely diagnosis? Acute calcium pyrophosphate crystal arthritis Gout Infectious arthritis Palindromic rheumatism
Acute calcium pyrophosphate crystal arthritis (pseudogout) is characterized by the sudden onset of pain, warmth, tenderness, and swelling of the affected joint, usually a knee or wrist; attacks are typically longer than those of gout. The most likely diagnosis is acute calcium pyrophosphate (CPP) crystal arthritis (pseudogout). Acute CPP crystal arthritis occurs episodically and is generally a condition that occurs in the elderly. The most frequently involved joints are knees and wrists. An acute attack is characterized by pain, warmth, tenderness, and swelling; signs of systemic inflammation may be present but are not universal. Attacks can last for weeks to months (in general, longer than gout attacks). Acute CPP crystal arthritis usually occurs in patients with osteoarthritis of involved joints, and plain radiographs may demonstrate chondrocalcinosis, seen as a thin radiopaque line of CPP deposition at chondral surfaces (most easily appreciated in the knees, pubic symphysis, and wrists). Ideally, acute CPP crystal arthritis is definitively diagnosed by identifying positively refringent rhomboid-shaped CPP crystals from joint fluid. However, there is rarely fluid to drain from a wrist, and even when fluid is obtainable, CPP crystals can be difficult to identify. Gout appears less likely than acute CPP crystal arthritis given the joints involved and the lengthy nature of the attacks. Initially, gout usually affects the metatarsophalangeal joints, the mid foot, ankles, and knees, although in postmenopausal women the finger joints may be affected (especially those with pre-existing osteoarthritis). Even without treatment, most gout attacks resolve within 10 to 14 days. Additionally, the low serum urate level makes gout less likely. Infectious arthritis is highly unlikely in this patient because symptoms began 2 weeks ago, she otherwise appears well, and her radiographs demonstrate no joint destruction (which would be expected 2 weeks into an untreated joint infection). Additionally, she had two prior similar episodes, and infectious arthritis is not self-limited or recurrent. Palindromic rheumatism is an uncommon arthritis that is often a precursor to rheumatoid arthritis and is characterized by acute episodes of monoarthritis that last from 24 to 72 hours; attacks migrate between different appendicular joints, especially the metacarpophalangeal and proximal interphalangeal joints. The prolonged symptoms and joints in this case do not fit the pattern of palindromic rheumatism.
A 42-year-old man is evaluated for a 2-month history of painless gross hematuria. He was diagnosed with chronic kidney disease 4 years ago. His only medication is occasional ibuprofen. He is a recent immigrant from Bosnia. On physical examination, blood pressure is 150/86 mm Hg. The remainder of the examination is unremarkable. Laboratory studies: Hemoglobin 10.5 g/dL (105 g/L) Creatinine 4.0 mg/dL (353.6 µmol/L) Urinalysis 1+ protein; numerous nondysmorphic erythrocytes; 5-8 leukocytes/hpf; occasional granular casts Urine protein-creatinine ratio 900 mg/g Kidney ultrasound shows echogenic kidneys measuring 8.0 cm and 8.5 cm in length; an irregular bladder wall is noted. Which of the following is the most appropriate diagnostic test to perform next? CT urography Endoscopic urologic evaluation Kidney biopsy Urine cultures
Balkan endemic nephropathy is strongly associated with the development of upper tract transitional cell (urothelial) cancers, and urologic evaluation is necessary. The most appropriate next step for this patient is an endoscopic urologic evaluation with cystoscopy and upper tract imaging. The patient has Balkan endemic nephropathy (BEN), a slowly progressive tubulointerstitial disease that has been linked to aristolochic acid. Aristolochic acid is a nephrotoxic alkaloid from the plant Aristolochia clematitis. BEN has a high prevalence rate in southeastern Europe (Serbia, Bulgaria, Romania, Bosnia and Herzegovina, and Croatia) and is the cause of kidney disease in up to 70% of patients receiving dialysis in some of the most heavily affected regions. Aristolochic acid is also sometimes found as a component of herbal therapies used for weight loss. Characteristics of BEN include chronic kidney disease due to tubulointerstitial injury, tubular dysfunction (polyuria and decreased concentrating ability, glucosuria without hyperglycemia, and tubular proteinuria), and anemia. Ultrasound demonstrates small echogenic kidneys. BEN has a familial but not inherited pattern of distribution. It is thought to be caused by exposure to aristolochic acid, and, because it is mutagenic, it is strongly associated with the development of upper tract transitional cell (urothelial) cancers. Therefore, urologic evaluation is indicated. CT urography is the preferred test for patients with unexplained urologic/nonglomerular hematuria. However, CT urography involves the use of intravenous contrast, which would place this patient at a high risk of developing contrast-induced nephropathy given the severity of his kidney disease. Cystoscopy and retrograde ureteropyelography would be the preferred imaging techniques for this patient. Kidney biopsy would not explain the nondysmorphic hematuria. Furthermore, kidney biopsy is not indicated in the setting of small echogenic kidneys <9 cm in size, which signifies chronic irreversible disease. Pyuria, granular casts, and low-grade proteinuria are commonly found in patients with tubulointerstitial nephritis and contrasts with the isolated pyuria associated with most urinary tract infections. In addition, the small, echogenic kidneys, chronic hematuria, and irregular bladder wall are not consistent with a urinary tract infection and more strongly suggest the possibility of a malignancy. Therefore, urine cultures are not indicated.
A 38-year-old woman is seen in follow-up to discuss the findings of an abdominal and pelvic CT scan done to evaluate renal colic, which has since resolved. The abdominal CT scan showed two small nonobstructing renal calculi in the right kidney and a 1.6-cm left adrenal mass with a density of 21 Hounsfield units (indeterminate for adrenal adenoma). Other than nephrolithiasis, the remainder of the medical history is unremarkable, and she takes no medications. On physical examination, vital signs and the remainder of the examination are unremarkable. Laboratory studies show normal serum electrolytes. Which of the following is the most appropriate test to perform next? 24-Hour urine free cortisol measurement 24-Hour urine total metanephrine measurement Plasma aldosterone-plasma renin ratio (ARR) measurement Serum dehydroepiandrosterone sulfate (DHEAS) measurement
Biochemical testing for pheochromocytoma should be undertaken in all patients with an adrenal mass that is clearly not an adenoma, even in the absence of typical symptoms or hypertension. The most appropriate next test to perform is a 24-hour urine total metanephrine measurement to screen for pheochromocytoma. Even though this patient does not have hypertension, she should be screened for pheochromocytoma, as these tumors may exist in the absence of typical symptoms or hypertension. Approximately 50% of pheochromocytomas are now first discovered as an incidental adrenal mass. An alternative screening test for pheochromocytoma is measuring the fractionated free plasma metanephrine level. This test has a false-positive rate of approximately 11%, and, therefore, may be considered more useful when suspicion for pheochromocytoma is high. This patient should also be screened for subclinical Cushing syndrome with a 1-mg overnight dexamethasone suppression test. The prevalence of incidentally noted adrenal masses increases with age and is estimated to be about 10% in the elderly. Most lesions are benign, nonfunctioning adenomas, and approximately 10% to 15% secrete excess hormones. The 24-hour urine free cortisol test is not sensitive enough to diagnose subclinical autonomous cortisol secretion from an adrenal mass. The 24-hour urine free cortisol levels are usually within the normal range in subclinical Cushing syndrome. The patient does not require screening for primary aldosteronism with a plasma aldosterone-plasma renin ratio (ARR) as she does not have hypertension. Only patients with an incidental adrenal mass and hypertension require screening for primary aldosteronism. Hypokalemia, traditionally thought to be a key feature of primary aldosteronism, is no longer a prerequisite for diagnosis because many patients with this disorder have normal potassium levels. In women, rapid onset of hirsutism, menstrual irregularities, and virilization should raise suspicion for tumoral hyperandrogenism. Measurement of dehydroepiandrosterone sulfate (DHEAS) is not indicated in this patient, as she did not show signs of hyperandrogenism (hirsutism, deep voice, male pattern balding, clitoromegaly). Serum DHEAS may be measured if signs of significant hyperandrogenism are present in the setting of an adrenal mass that has radiologic features suspicious for malignancy (size >4 cm, heterogeneous enhancement with contrast administration, irregular margins, presence of calcifications or necrosis).
A 72-year-old man is evaluated for a 6-month history of headaches that have gradually increased in frequency and have been daily and constant for the past 6 weeks. The pain is bilateral, occipital, and described as a steady "pulling" sensation of mild to moderate intensity. Fatigue and neck discomfort are present. He has had no associated nausea, photophobia, phonophobia, or neurologic symptoms. The patient has atrial fibrillation treated with warfarin and verapamil and no other personal or family history of headache. On physical examination, blood pressure is 128/88 mm Hg and pulse rate is 88/min and irregular. Decreased cervical range of motion is noted. Neurologic examination findings are unremarkable. Results of laboratory studies show a normal erythrocyte sedimentation rate and complete blood count, a therapeutic INR, a normal comprehensive metabolic profile, and a normal serum level of C-reactive protein. Which of the following is the most appropriate next step in management? Amitriptyline administration Brain MRI Cervical spine MRI Head CT Temporal artery biopsy
Brain MRI is the most appropriate diagnostic study for patients with a suspected secondary headache and should be performed before additional testing is considered. This patient should have a brain MRI. His development of a new headache after age 50 years, use of an anticoagulant, and progressive headache pattern are all red flags indicating a potential secondary headache condition. Other red flags for secondary headache include first or worst headache, abrupt-onset or thunderclap attack, episode associated with neurologic symptoms lasting more than 1 hour, alterations in consciousness, abnormal physical examination findings, and onset after exertion, sex, or Valsalva maneuvers. Brain MRI is the most appropriate diagnostic study for patients with a suspected secondary headache and should be performed before additional testing is considered. Amitriptyline is the treatment of choice for prevention of chronic tension-type headache. Diagnosis of this condition, however, first requires exclusion of a secondary headache. Although features of the patient's clinical presentation may suggest a tension-type headache syndrome, the presence of red flags makes brain neuroimaging a priority. Neck pain is a nonspecific feature common to many primary and secondary headache syndromes. Its presence does not necessarily indicate a primary pathologic abnormality in the cervical region. Neurologic examination findings do not suggest cervical radiculopathy or myelopathy and thus an MRI of the cervical spine is not indicated. Head CT is indicated in the assessment of acute severe headache in an emergent setting. In these situations, intracranial hemorrhage often is suspected and is well-visualized by a noncontrast CT scan. Given its greater sensitivity and safety, brain MRI is recommended in the evaluation of subacute or chronic headache requiring imaging. Temporal artery biopsy is helpful in the evaluation of suspected giant cell arteritis. Most patients with this disorder are older than 50 years. Erythrocyte sedimentation rate and C-reactive protein level are frequently elevated. Brain MRI and reversal of anticoagulation are both indicated before temporal artery biopsy.
A 76-year-old woman is evaluated in the hospital for recurrent diverticulitis. She has had multiple episodes of diverticulitis treated with ciprofloxacin and metronidazole over the past 18 months. During this hospitalization, an abscess was noted on a CT scan of the abdomen and pelvis, and a percutaneous drain was placed and the fluid sent for culture. Piperacillin-tazobactam is started empirically. On physical examination, temperature is 39.2 °C (102.6 °F), blood pressure is 126/82 mm Hg, pulse rate is 100/min, and respiration rate is 15/min. She has diffuse abdominal tenderness to palpation. Laboratory studies show a leukocyte count of 22,000/µL (22 × 109/L) and a serum creatinine level of 1.3 mg/dL (115 µmol/L). Gram stain of the abscess fluid reveals 4+ gram-negative rods; fluid culture grows Pseudomonas aeruginosa resistant to ceftazidime, piperacillin-tazobactam, ciprofloxacin, meropenem, and doripenem. Blood cultures are negative. The Pseudomonas isolate should be tested for susceptibility to which of the following antibiotics? Ceftolozane-tazobactam and colistin Ertapenem and tobramycin Fosfomycin Minocycline
Ceftolozane-tazobactam is a newer antipseudomonal cephalosporin combined with a β-lactamase inhibitor that can be used in the treatment of multidrug-resistant intra-abdominal infection. The Pseudomonas isolate should be tested for susceptibility to ceftolozane-tazobactam and colistin. This patient has an intra-abdominal infection caused by carbapenem-resistant (meropenem and doripenem) Pseudomonas aeruginosa. The antimicrobial options for treating carbapenem-resistant organisms are limited. Ceftolozane-tazobactam consists of a newer antipseudomonal cephalosporin combined with a β-lactamase inhibitor. It has activity against some extended-spectrum β-lactamase (ESBL)-producing gram-negative organisms as well as carbapenem-resistant strains of Pseudomonas. Its efficacy is reduced in patients whose creatinine clearance rate is 50 mL/min or less. Ceftolozane-tazobactam is approved to treat complicated urinary tract infections and complicated intra-abdominal infections. For complicated intra-abdominal infections, ceftolozane-tazobactam must be paired with metronidazole because it lacks antianaerobic activity. Colistin is an older antimicrobial agent that has made a resurgence because of its bactericidal activity against pan-resistant gram-negative organisms (including carbapenem-resistant, gram-negative organisms). Half of patients who are administered colistin develop nephrotoxicity, which limits the drug's usefulness in many cases. Kidney function should be closely monitored during administration, and colistin should be dose adjusted for patients with kidney disease. Paresthesias are a commonly reported neurotoxicity. Unfortunately, colistin resistance has been reported and appears to be increasing. Ertapenem is a non-antipseudomonal carbapenem, so it would not be an option for this patient. Likewise, tobramycin is an aminoglycoside, which has poor activity and penetration into abscesses and would not be a consideration for this patient. Fosfomycin is used to treat urinary tract infections caused by several multidrug-resistant organisms (such as vancomycin-resistant enterococci, methicillin-resistant Staphylococcus aureus, and resistant Klebsiella pneumoniae). Fosfomycin has poor tissue penetration and is not indicated for infections other than uncomplicated urinary tract infections. Minocycline is another older antimicrobial agent that has been used recently to treat multidrug-resistant Acinetobacter as well as Stenotrophomonas maltophilia infections, but it does not have activity against Pseudomonas.
A 40-year-old woman is evaluated at a follow-up appointment for hepatitis B virus (HBV) infection, which was diagnosed at age 23 years. The patient also has psoriasis. She is married and is sexually active. Medications are topical clobetasol and an oral contraceptive agent. On physical examination, vital signs are normal. Psoriatic lesions are present on the elbows and knees. No hepatosplenomegaly is noted. Laboratory studies: Alanine aminotransferase 125 U/L Aspartate aminotransferase 112 U/L Hepatitis B surface antigen Positive Hepatitis B e antigen Positive HBV DNA 41,326 IU/mL Which of the following is the most appropriate treatment? Adefovir Lamivudine Pegylated interferon Tenofovir No treatment
Chronic hepatitis B viral infection in the immune-active, hepatitis B e antigen-positive phase should be treated with tenofovir or entecavir to decrease hepatic inflammation and the risk for progression to fibrosis. Tenofovir is the most appropriate treatment for this patient. Entecavir would be an equally suitable choice. Chronic hepatitis B virus (HBV) infection is characteristically divided into phases of disease. Not all patients go through each phase. Patients in the immune-active or reactivation phases have an elevated alanine aminotransferase (ALT) level and an elevated HBV DNA level (in the immune-active phase, >20,000 IU/mL in hepatitis B e antigen [HBeAg]-positive disease or >2,000 IU/mL in HBeAg-negative disease, and in the reactivation phase, a rise above baseline HBV DNA level). The patient has chronic HBV infection in the immune-active, HBeAg-positive phase, with ALT and aspartate aminotransferase levels exceeding two times the upper limit of normal and HBV DNA exceeding 20,000 IU/mL. Treatment is necessary to decrease hepatic inflammation, risk for progression to fibrosis, and eventual cirrhosis that will occur without treatment. In approximately 50% of untreated patients with chronic HBV infection in the United States, HBV infection will contribute to the cause of death (from hepatocellular carcinoma or other complications of end-stage liver disease, such as cirrhosis). Patients who respond to treatment have a decreased risk of liver-related complications of HBV infection. Tenofovir or entecavir is first-line treatment for immune-active HBV infection because of low rates of resistance. Treatment goals for HBV infection in the immune-active, HBeAg-positive phase are loss of HBeAg and anti-hepatitis B e antibody seroconversion. For most patients, chronic treatment is necessary because seroconversion of HBeAg or hepatitis B surface antigen is not commonly achieved. Adefovir and lamivudine are not preferred antiviral agents given the risk for HBV resistance. Resistance rates at 5 years are approximately 30% for adefovir and 70% for lamivudine. In this patient, oral antiviral regimens are preferred over pegylated interferon because pegylated interferon may exacerbate psoriasis. Pegylated interferon is most appropriate for patients who have higher ALT levels, low HBV DNA levels, and no cirrhosis.
A 55-year-old man is referred for evaluation of fatigue, weight gain, decreased libido, and difficulty maintaining an erection. Sexual functioning was normal until 6 months ago, and he has fathered two children. Medical history is significant for polysubstance abuse that is being managed with daily methadone. His medical history is otherwise unremarkable, and his only medication is methadone. On physical examination, vital signs are normal. Neurological, genitalia, and the remainder of the physical examination are normal. Laboratory studies: Follicle-stimulating hormone 5 mU/mL (5 U/L) Luteinizing hormone 4 mU/ml (4 U/L) Prolactin 12 ng/mL (12 µg/L) Testosterone 185 ng/dL (6.4 nmol/L) Thyroid-stimulating hormone 2.4 µU/mL (2.4 mU/L) Thyroxine (T4), free 1.3 ng/dL (16.8 pmol/L) Which of the following is the most likely cause of this patient's hypogonadism? Age-related decline in gonadal function Chronic opioid therapy Pituitary tumor Primary gonadal failure
Chronic opioid use suppresses gonadotroph function, resulting in hypogonadotropic hypogonadism, which is increasingly recognized as a cause of secondary hypogonadism.
Hyperbaric O2 indications for CO poisoning?
Cited indications for hyperbaric oxygen therapy include loss of consciousness, ischemic cardiac changes, neurological deficits, significant metabolic acidosis, or carboxyhemoglobin level greater than 25%.
A 39-year-old woman is evaluated for a 3-week history of malodorous vaginal discharge. She was treated with antibiotics for a urinary tract infection 3 weeks before the onset of symptoms. She is in a monogamous sexual relationship. Medical history is unremarkable. Her only medication is an oral contraceptive. On physical examination, vital signs are normal. Pelvic examination reveals thin, homogenous, grayish vaginal discharge. There is no adnexal or cervical motion tenderness. The rest of the examination is unremarkable. Laboratory testing reveals a vaginal pH of 5.6; whiff test result is negative. Which of the following is the most appropriate test to confirm the diagnosis? Culture for Gardnerella vaginalis Nucleic acid amplification test for trichomoniasis Potassium hydroxide wet mount study for yeast Saline microscopy for clue cells
Clinical diagnosis of bacterial vaginosis requires three of the following four features: vaginal pH greater than 4.5, thin and homogenous vaginal discharge, positive whiff test result, and clue cells comprising at least 20% of all squamous cells on saline microscopy; culture is not a reasonable test to confirm the diagnosis of bacterial vaginosis and would also be costly and inefficient compared with an office-based diagnosis.
A 34-year-old man is evaluated for recurrent right-sided severe, sharp periorbital headache. Episodes have been occurring nocturnally for the past 2 weeks, typically last 90 minutes, and are associated with ipsilateral nasal congestion, tearing, nausea, and photophobia. He reports having had a similar series of headaches over a 2-month period 6 months ago. The patient has been taking combination acetaminophen, aspirin, and caffeine as needed for pain. He takes no other medication and has a 10-pack-year smoking history. On physical examination, vital signs and all other physical examination findings, including those from a neurologic examination, are unremarkable. An MRI of the brain is normal. Which of the following is the most appropriate preventive treatment? Carbamazepine Fexofenadine Indomethacin Propranolol Verapamil
Cluster headache, the most common of the trigeminal autonomic cephalalgias, is best prevented with verapamil. This patient should be treated with verapamil. His history is compatible with the diagnosis of cluster headache. Cluster headache is the most common of the trigeminal autonomic cephalalgias (TACs). The term "cluster" is derived from this disorder's characteristic short cycles of headache activity (weeks) interrupted by long periods of complete remission (month or years). Male sex and tobacco use are two potential risk factors. Alcohol is a commonly reported trigger during an active cycle. Episodes are common nocturnally and may occur seasonally. Attack duration is between 15 and 180 minutes. Pain is almost always unilateral and localized near the temple or orbit. Ipsilateral autonomic symptoms, such as tearing, ptosis, nasal congestion, and rhinorrhea, are characteristic. Attacks may occur one to eight times daily for several weeks or months. Brain MRI should be performed initially to exclude structural lesions mimicking cluster headache. Oxygen inhalation and subcutaneous sumatriptan are first-line therapies for acute cluster headache. The patient should be counseled to stop smoking. Verapamil is the preventive medication of choice. Carbamazepine is the treatment of choice for trigeminal neuralgia, which most often occurs in patients older than 50 years, but has no effect on cluster headache or any of the TACs. Periorbital location of pain would be atypical for trigeminal neuralgia, as would an attack duration beyond seconds. Ipsilateral autonomic symptoms are also generally absent. Fexofenadine, a second-generation antihistamine, has no established benefit in cluster headache. This patient has no history of allergic rhinitis or other conditions that would respond to an antihistamine. Several indomethacin-responsive headache syndromes have been identified, but cluster headache is not among them. Chronic paroxysmal hemicrania is a TAC that responds to indomethacin. Episodes of this condition last only 3 to 20 minutes and can recur up to 40 times daily. The diagnosis also requires concomitant ipsilateral autonomic findings, such as tearing, nasal congestion, or rhinorrhea. Propranolol is an effective migraine preventive therapy but has no benefit for cluster headache. Migraine may present with severe periorbital headache with nausea and photophobia and occasionally with autonomic features. Unlike cluster headache, which typically lasts 15 to 180 minutes, migraine by definition extends 4 to 72 hours without treatment.
What do canon a waves indicate?
Complete heart block
A 64-year-old woman is evaluated for a 6-week history of right knee swelling and pain. She has had no recent injury, fevers, or chills. She retired as a horticulturist and moved from Massachusetts to Florida 3 months ago. She takes NSAIDs, which provide partial pain relief, but the swelling persists. On physical examination, vital signs are normal. The right knee has a moderately sized effusion but no erythema or warmth. Slight pain is present on passive movement of the knee. Results of laboratory studies show an equivocal Borrelia burgdorferi enzyme immunoassay of 1.07; leukocyte count and rheumatoid factor titer are within normal limits. Which of the following is the most appropriate next diagnostic test? Borrelia burgdorferi IgG Western blot C6 enzyme immunoassay antibody test Polymerase chain reaction of joint fluid No further testing
Confirmatory serologic testing using a two-tiered diagnostic approach that includes enzyme immunoassay and IgG Western blotting (and, early in the course of disease, IgM Western blotting) is required for definitive diagnosis of late disseminated Lyme disease. This patient should have IgG Western blotting to detect antigens to Borrelia burgdorferi. She has symptoms of monoarticular arthritis, which has a broad differential diagnosis. Her presentation is most compatible with late disseminated Lyme disease; her previous residence in a Lyme-endemic area and her occupation, with its increased likelihood of tick exposure, increase the probability of this diagnosis. Onset of symptoms of Lyme arthritis typically occurs months, and sometimes years, after the initial infection. Involvement of large joints, particularly the knees, is common. Confirmatory serologic testing using a two-tiered diagnostic approach is required for definitive diagnosis of late disseminated Lyme disease. The initial test is a Lyme enzyme immunoassay antibody titer. This test is very sensitive, and, in a patient suspected of having late disseminated Lyme disease, a negative result essentially excludes the diagnosis. In patients with a positive or equivocal result, as with this patient, a second-tier test is necessary to confirm the diagnosis. Early in the course of infection, both IgM and IgG Western blots are recommended because IgM antibody production predates IgG development. After 4 weeks or more of symptoms, IgG antibody is presumed to be positive, and thus a positive IgM Western blot with a negative IgG likely represents a false-positive result. For this reason, the testing algorithm from the Centers for Disease Control and Prevention specifies that when signs and symptoms are present for more than 30 days, only a confirmatory IgG Western blot should be performed. The C6 antibody corresponds to a highly conserved protein common to all Borrelia strains. Testing using this assay may have a role in diagnosis of infections that occurred outside of the United States. For domestically acquired infections, this test is not currently recommended to replace either component of the conventional two-tier algorithm. The sensitivity of synovial polymerase chain reaction is low, and this method is not recommended as an initial diagnostic test for Lyme arthritis.
A 36-year-old man is evaluated for a 10-day history of abdominal cramping, diarrhea, malaise, and nausea. Diarrhea is watery without mucus or blood. He returned 2 weeks ago from a 7-day trip to Lima, Peru. On physical examination, temperature is 37.7 °C (99.9 °F); the remaining vital signs are normal. On abdominal examination, bowel sounds are present with diffuse tenderness to palpation. The abdomen is not distended; no guarding or rebound is noted. Stool polymerase chain reaction assay is positive for Cyclospora. Which of the following is the most appropriate treatment? Atovaquone Metronidazole Pyrimethamine Quinacrine Trimethoprim-sulfamethoxazole
Cyclospora infection is treated with oral trimethoprim-sulfamethoxazole. This patient has travel-associated Cyclospora infection and should be treated with trimethoprim-sulfamethoxazole. Cyclospora protozoan infections are typically acquired after consumption of fecal-contaminated food or water, particularly in countries where the parasite is endemic, such as Peru, Guatemala, Haiti, and Nepal. Cyclospora infections may also be acquired through consumption of fresh produce imported from tropical areas. The incubation period is approximately 1 week (range, 2 days to ≥2 weeks). The clinical presentation usually consists of crampy abdominal pain, anorexia, bloating, decreased appetite, fatigue, flatulence, low-grade fever, malaise, nausea, watery diarrhea, and weight loss. Persons with HIV infection may have more severe symptoms associated with wasting. Diagnosis can be established microscopically by visualization of oocysts with modified acid-fast staining; fluorescence microscopy can be used as well. Several stool specimens may be required because Cyclospora oocysts may be shed intermittently and at low levels, even in persons with profuse diarrhea. Polymerase chain reaction assays appear to have the greatest sensitivity for the diagnosis of a Cyclospora infection. The recommended treatment is one double-strength tablet of trimethoprim-sulfamethoxazole taken orally twice daily for 7 to 10 days. The Centers for Disease Control and Prevention states no effective alternative treatments have been identified for persons who are allergic to or cannot tolerate trimethoprim-sulfamethoxazole; observation and symptomatic care is recommended for those patients. Atovaquone has activity against protozoans such as Pneumocystis jirovecii, Toxoplasma, Plasmodium, and Babesia, but not Cyclospora. Metronidazole has activity against some protozoans, including Giardia, Entamoeba, and Trichomonas, but not Cyclospora. Pyrimethamine has activity against protozoans such as Toxoplasma, Pneumocystis jirovecii, and Isospora belli, but not Cyclospora. Quinacrine can be used to treat Giardia but is not effective against Cyclospora.
What does the Rinne test show? -Sensorineural -Conductive
Sensorineural: Air > bone Conductive: Bone > air
A 48-year-old woman is evaluated for shortness of breath of 6 weeks' duration. She has cirrhosis due to primary biliary cholangitis. On physical examination, vital signs are normal. Spider nevi are present on the skin. The cardiopulmonary examination is normal. There is no edema. When the patient is supine, oxygen saturation is 98% breathing ambient air, but oxygen saturation drops to 92% with standing. A radiograph of the chest is normal. Bronchoscopy CT angiography Echocardiography with agitated saline Pulmonary function testing
Echocardiography with agitated saline is the most appropriate next test for this patient. Hepatopulmonary syndrome is a complication of cirrhosis caused by dilation of the pulmonary vasculature in the setting of advanced liver disease and portal hypertension. A high alveolar-arterial oxygen gradient results from functional shunting. Patients with hepatopulmonary syndrome usually have a preexisting diagnosis of liver disease and present with shortness of breath. Dilation of pulmonary vasculature occurs at the base of the lungs, so hypoxemia is most noted when patients are upright or sitting, when shunting is maximal. Classic features are platypnea (worsening shortness of breath in the upright position) and orthodeoxia (worsening arterial oxygen saturation in the upright position). Pulse oximetry is often used to screen for changes in the arterial oxygen saturation level with changes of position. The diagnosis is made by demonstrating an arterial oxygen tension less than 80 mm Hg (10.7 kPa) breathing ambient air, or an alveolar-arterial gradient of 15 mm Hg (2 kPa) or greater, along with evidence of intrapulmonary shunting on echocardiography with agitated saline or macroaggregated albumin study. The detection of intrapulmonary shunting of blood is best confirmed by echocardiography with agitated saline (also known as a bubble study), during which bubbles are identified in the left side of the heart after 5 beats, demonstrating that the shunting of blood is not intracardiac. Clinically significant hepatopulmonary syndrome is treated with supplemental oxygen and liver transplantation. Hepatopulmonary syndrome is a progressive condition that is ultimately fatal without liver transplantation. Bronchoscopy is of no value in the diagnosis of platypnea or shunting disorders. It is potentially useful in the diagnosis of a pulmonary infiltrate or relief of an airway obstruction. CT angiography can demonstrate the presence of large vascular shunts in the lungs but is rarely required to establish the diagnosis of hepatopulmonary syndrome. An additional benefit of CT angiography is its ability to show pulmonary emboli. In this patient, the presence of orthopnea is not consistent with pulmonary embolism. Transthoracic echocardiography with agitated saline is the gold standard for detecting pulmonary vascular dilatation and diagnosing hepatopulmonary syndrome. Pulmonary function testing is useful for evaluating the presence of obstructive lung disease as well as restrictive lung disease. The normal pulmonary examination and normal chest radiography suggest that neither restrictive nor obstructive lung disease is contributing to this patient's presentation.
A 63-year-old man is evaluated in follow-up for a lung mass found during evaluation for a persistent cough. He is a current smoker with a 60-pack-year smoking history. His medical history is otherwise unremarkable, and he takes no medications. Upon physical examination, vital signs are normal. The remainder of the physical examination is normal. Laboratory studies, including liver chemistry tests, sodium , and calcium , are normal. Chest radiograph shows a 2-cm nodule in the right upper lobe that was not seen on previous imaging. CT and PET scans demonstrate PET positivity of the lung lesion and in the mediastinal lymph nodes. Which of the following is the most appropriate diagnostic procedure to perform next? CT-guided needle biopsy Endobronchial ultrasound and mediastinal lymph node biopsy Sputum cytology Thoracoscopic lung biopsy with lymph node dissection
Endobronchial ultrasound-guided transbronchial needle aspiration is the procedure of choice for diagnosing and staging mediastinal and hilar lymphadenopathy in patients with suspected thoracic malignancy. The most appropriate diagnostic test for this patient with a lesion highly suspicious for lung cancer and PET-positive mediastinal lymphadenopathy is bronchoscopy with endobronchial ultrasound-guided transbronchial needle aspiration. The evaluation of a patient with suspected lung cancer aims to confirm whether the patient indeed has lung cancer, to determine the pathology (non-small cell lung cancer versus small cell lung cancer), and to assess the stage at presentation. Most patients undergo chest CT scan as the first imaging modality, either after an abnormal chest radiograph or in evaluation of a symptom. The findings on the chest CT scan determine whether a PET/CT scan is necessary. A PET/CT scan can help in staging and therefore also help guide where to biopsy. For example, if a patient has a solitary pulmonary nodule, a PET/CT scan may help determine if any lymph node involvement is present that was not visible on the chest CT scan. The next step is to obtain tissue diagnosis. The choice of initial diagnostic testing should be aimed first at identifying potential lymph node involvement or metastatic disease. Tissue diagnosis should then be targeted at the lesion that would result in the highest potential staging. In this patient, sampling the mediastinal lymph nodes is critical to both diagnose and stage the patient and will affect the clinical decision making for this patient. Endobronchial ultrasound-guided transbronchial needle aspiration is a minimally invasive way and preferred to more invasive surgical techniques. Endobronchial ultrasound-guided transbronchial needle aspiration can sample most mediastinal and some hilar lymph node stations, although some of the more posterior lymph nodes are not accessible with this technique. CT-guided needle biopsy has a high accuracy for diagnosis of lung cancers, but has higher risks of procedural complications than endobronchial ultrasound-guided transbronchial needle aspiration (mainly pneumothorax), and the patient would require a second procedure to sample the mediastinum if the lung lesion is a non-small cell lung cancer. Sputum cytology has a lower sensitivity than endobronchial ultrasound-guided transbronchial needle aspiration for diagnosis of lung cancer, does not produce sufficient sample material for molecular studies, and will not provide needed staging information. Thoracoscopic lung biopsy with lymph node dissection is less preferable than a minimally invasive approach, due to cost and risks of complications.
A 32-year-old woman is evaluated in the emergency department for a 2-day history of headache and vomiting. She was diagnosed with Raynaud phenomenon 1 year ago and gastroesophageal reflux disease 6 months ago. Her only medication is omeprazole. On physical examination, temperature is 38.0 °C (100.4 °F), blood pressure is 240/140 mm Hg, pulse rate is 88/min, respiration rate is 16/min, and oxygen saturation is 96% breathing ambient air. Skin findings are digital pitting at the ends of the fingers, thickening of the skin over the fingers and dorsum of the hands, and thickening with poikilodermic changes over the skin of the anterior chest. Laboratory studies: Hematocrit 32% Platelet count 75,000/µL (75 × 109/L) Creatinine 1.5 mg/dL (132.6 µmol/L) Urinalysis 2+ protein; no blood Peripheral blood smear shows diminished platelet numbers and schistocytes. Which of the following is the most appropriate treatment? Captopril Cyclophosphamide Methylprednisolone Nitroprusside
Features of scleroderma renal crisis include hypertensive emergency, headache, microangiopathic hemolytic anemia, thrombocytopenia, elevated serum creatinine levels, and proteinuria; treatment involves ACE inhibitors, typically captopril. The most appropriate treatment for this patient with scleroderma renal crisis is the ACE inhibitor captopril. This patient has features of systemic sclerosis (SSc), including a history of Raynaud phenomenon, gastroesophageal reflux disease, and skin changes typical of this disease. She also has features that occur in patients with scleroderma renal crisis, including hypertensive emergency, headache, microangiopathic hemolytic anemia (schistocytes on the peripheral blood smear), thrombocytopenia, elevated serum creatinine levels, and proteinuria. Approximately 75% of cases of scleroderma renal crisis occur in the first 4 years of SSc disease onset. Scleroderma renal crisis can be seen in both limited and diffuse forms of SSc but more often in those with rapidly progressive diffuse disease. Anti-RNA polymerase III antibodies serve as a marker for increased risk for scleroderma renal crisis as well as extensive skin disease. The use of ACE inhibitors has dropped the 1-year mortality of scleroderma renal crisis from 76% to 15%. The mechanism of action of ACE inhibitors is believed to be mitigation of the effect of interstitial fibrosis and vascular dysfunction in the glomerular arterial bed. Treatment with an ACE inhibitor (typically captopril) should be initiated promptly in SSc patients with even mild hypertension or otherwise unexplained elevations in serum creatinine levels or acute kidney injury. Angiotensin receptor blockers (ARBs) are an alternative for patients who cannot take ACE inhibitors, although ARBs are less effective for managing scleroderma renal crisis. Cyclophosphamide is an immunosuppressive agent and not effective in scleroderma renal crisis. Glucocorticoids, such as methylprednisolone, are implicated as potential risk factors for the development of scleroderma renal crisis and are not indicated for this patient. In patients with hypertensive emergency, blood pressure must be lowered quickly with short-acting intravenous antihypertensive infusions to limit end-organ damage. Intravenous nitroprusside is often a first choice for patients with hypertensive emergencies. It can be titrated quickly to achieve blood pressure control. However, it does nothing to address the underlying physiological cause of scleroderma renal crisis and is inferior to ACE inhibitors on reducing mortality.
An 87-year-old woman is evaluated during a follow-up visit after a recent diagnosis of breast cancer. In the 18 months before her diagnosis, she noted a generalized decline in her energy level and appetite. She no longer is able keep up with others when walking any distance, and she now requires some assistance with dressing because of generalized weakness. She has unintentionally lost 3.6 kg (8 lb) in the last 6 months. Medical history is significant for breast cancer, coronary artery bypass graft surgery at age 74 years, COPD, and hypertension. Medications are albuterol, tiotropium, salmeterol, atorvastatin, aspirin, lisinopril, and metoprolol. On physical examination, vital signs are normal. BMI is 19. The remainder of the examination is unremarkable. Which of the following is most likely to predict the patient's overall morbidity, mortality, and response to breast cancer treatment? FRAIL scale score Pharmacologic cardiac stress test Six-minute walk test Timed Up and Go test
Frailty is a quantifiable geriatric syndrome that may predict a patient's response to medical treatment. Frailty is a multifactorial geriatric syndrome that may predict a patient's response to certain treatments as well as morbidity and mortality in light of chronic illness. This patient demonstrates unintentional weight loss, low energy and activity levels, slow walking speed, and weakness, all of which are associated with frailty. Indices such as the Frailty Index, the FRAIL (Fatigue, Resistance, Ambulation, Illness, and Loss of weight) scale, and the Osteoporotic Fractures Frailty Scale have been validated for use in primary care. The Frailty Index has been in use for a longer time than other indices; however, its length and complexity limit its usefulness in routine care. The FRAIL scale consists of five self-reported measures and is easy to administer and score in an office setting. Pharmacologic cardiac stress testing is recommended for patients who cannot exercise and are experiencing symptoms suggestive of cardiac ischemia. There is no established role for pharmacologic stress testing as a predictor of response to cancer treatment. Lung function during exertion using the 6-minute walk test is helpful to assess disability and prognosis in chronic lung conditions. Simple pulse oximetry and oxygen desaturation studies performed at rest and with exertion assess the need for oxygen supplementation. During a 6-minute walk test, oxygen saturation, heart rate, dyspnea and fatigue levels, and distance walked in 6 minutes are recorded. The 6-minute walk test has no established role in predicting response to cancer chemotherapy in frail older adults. The Timed Up and Go test is used to identify patients at risk for falls. The individual components of the test (rising from the chair, gait, walking speed, balance maintenance while turning, and sitting) offer insight into the various mechanics of mobility and can guide a more focused evaluation and intervention. Results of the Timed Up and Go test do not predict response to cancer treatment.
Causes of ITP?
HIV!!!! HCV!!! Immune thrombocytopenic purpura can be idiopathic, triggered by medications, or associated with other disorders, such as systemic lupus erythematosus, chronic lymphocytic leukemia, lymphoma, HIV, hepatitis C, or Helicobacter pylori infection. *Antiplatelet antibody testing is not recommended because of low sensitivity.
A 33-year-old woman is evaluated in the emergency department in January with a 3-day history of fever, headache, stiff neck, and photophobia. She was previously well, and medical history is negative for recent travel; she takes no medications. On physical examination, temperature is 38.5 °C (101.3 °F), blood pressure is 136/86 mm Hg, pulse rate is 100/min, and respiration rate is 18/min. The general medical examination is unremarkable. On neurologic examination, she shows photophobia, and a nondilated funduscopic examination shows no papilledema. The remainder of the examination is nonfocal. Cerebrospinal fluid evaluation shows a leukocyte count of 324/µL (324 × 106/L) with 60% neutrophils, glucose level of 58 mg/dL (3.2 mmol/L), and protein level of 125 mg/dL (1250 mg/L). Gram stain of the cerebrospinal fluid is negative, and culture is pending. Which of the following is the most likely cause of this patient's symptoms? Enterovirus Herpes simplex virus type 2 Mumps virus West Nile virus
Herpes simplex virus type 2 can cause acute aseptic meningitis year round and is the most common cause of recurrent viral meningitis. The most likely diagnosis in this patient is meningitis caused by herpes simplex virus (HSV) type 2. Viral meningitis is the most common cause of "aseptic" meningitis, in which cerebrospinal fluid (CSF) Gram stain and cultures are negative. Most patients have typical meningitis symptoms, such as fever, nuchal rigidity, headache, and photophobia. HSV meningitis syndromes can be related to primary infections, with central nervous system involvement as a secondary consequence, or reactivation of latent infection presenting as aseptic meningitis. HSV-2 is more commonly associated with meningitis and is the most common cause of recurrent meningitis (recurrent benign lymphocytic meningitis). HSV-1 is associated with encephalitis. HSV can cause meningitis year round. CSF findings resemble enteroviral meningitis, with lymphocytic pleocytosis, a normal glucose level, and a mildly elevated protein level as in this patient. CSF polymerase chain reaction studies may be used for diagnosing HSV and enterovirus meningitis. Enteroviruses are the most common cause of viral meningitis, but they usually present between May and November in the Western Hemisphere, with symptoms including headache, fever, nuchal rigidity, photophobia, nausea, vomiting, myalgias, pharyngitis, maculopapular rash, and cough. This patient's presentation in the winter makes this an unlikely cause of her illness. Mumps virus can cause meningitis, with typical symptoms of fever, headache, and neck stiffness. Since the advent of universal childhood vaccination for measles, mumps, and rubella, the incidence of mumps-related meningitis has dramatically decreased. Meningitis from mumps virus can occur at any point during the course of clinical mumps infection. Parotitis or orchitis may be present. The patient's clinical presentation is not consistent with mumps meningitis. Focal motor weakness is a common finding in West Nile neuroinvasive disease (WNND), either combined with meningoencephalitis or as an isolated myelitis. In its most severe form, infection of the anterior horn cells can cause a symmetric or asymmetric flaccid paralysis, analogous to that seen with polio in the prevaccination era. The diagnosis of WNND can be confirmed through identification of the IgM antibody in CSF. West Nile virus is a mosquito-borne illness most commonly seen between June and October, making this an unlikely cause of this patient's illness.
A 50-year-old woman is evaluated for nausea and abdominal discomfort that have been present and increasing for the past 3 to 4 months. She takes no medications. On physical examination, vital signs are normal. BMI is 24. The abdominal examination reveals no masses, tenderness, or organomegaly. An esophagogastroscopy identifies a discrete 8-cm mass in the pylorus. The biopsy specimen shows a gastrointestinal stromal tumor (GIST). Immunohistochemical stain for the KIT gene is strongly positive. Contrast-enhanced CT scans of the chest, abdomen, and pelvis confirm the mass and do not identify any other abnormal findings. The patient undergoes a distal gastrectomy. Pathologic findings confirm a GIST and further note a high mitotic rate (10 mitoses per 50 high-power fields). Margins of resection and all lymph nodes examined are free of tumor. Which of the following is the most appropriate management? Intravenous rituximab Oral imatinib Radiation therapy Observation without further therapy
High-risk gastrointestinal stromal tumors should be treated with surgery and 3 years of adjuvant imatinib. The most appropriate management for this patient is oral imatinib for 3 years. Gastrointestinal stromal tumors (GISTs) most commonly occur in the stomach, although they can arise anywhere in the digestive tract. Although rare, they are the most common sarcoma of the digestive tract. Location outside of the stomach, larger size, and higher mitotic index are all factors that increase the risk of recurrence after resection and indicate use of adjuvant therapy. Almost all GISTs have an activating mutation in the c-KIT proto-oncogene, leading to constitutive activation of the KIT receptor tyrosine kinase. Therefore, she should be treated with the small-molecule receptor tyrosine kinase inhibitor imatinib, which blocks c-KIT tyrosine kinase phosphorylation. In such high-risk patients, recurrence-free survival and overall survival are superior in patients who receive 3 years of imatinib therapy compared with 1 year of therapy. Rituximab is a genetically engineered chimeric monoclonal antibody against CD20, a B-cell specific surface antigen. CD20 regulates cell cycle initiation. Rituximab binds to the antigen on the cell surface, activating complement-dependent B-cell cytotoxicity, and binds to human Fc receptors, mediating cell killing through an antibody-dependent cellular toxicity. Rituximab is used in the treatment of rheumatoid arthritis, ANCA-related vasculitis, chronic lymphocytic leukemia, and non-Hodgkin lymphoma. As predicted by its mechanism of action, it is not effective in the treatment of GIST. Because GISTs are relatively resistant to radiation therapy, such treatment is not used after a complete resection. Small gastric GISTs with low mitotic indices may often be managed with surgery alone, but because this patient has a GIST with several factors that increase the risk of recurrence, observation without further therapy is not sufficient.
A 22-year-old man is evaluated in the emergency department 1 hour after awakening with right-sided facial weakness. He reports a 2-day history of headache but has had no fever. He lives in Minnesota and has removed two embedded ticks from himself and three from his dog over the past 3 weeks. The patient is otherwise healthy and takes no medication. On physical examination, vital signs are normal. Nuchal rigidity and right facial nerve (cranial nerve VII) palsy are noted. Mental status is intact. Skin examination findings are shown. All other physical examination findings are unremarkable. Results of laboratory studies show a leukocyte count of 11,500/µL (11.5 × 109/L). Which of the following is the most appropriate initial management? Borrelia burgdorferi antibody testing Ceftriaxone administration Doxycycline administration Head CT Lumbar puncture
In a patient with Lyme disease and possible central nervous system involvement, positive findings on lumbar puncture can support the diagnosis of neuroborreliosis, which necessitates parenteral therapy with ceftriaxone, cefotaxime, or penicillin. This patient should undergo lumbar puncture. He has unilateral facial nerve palsy, headache, neck stiffness, and a circular rash with central clearing that is clinically consistent with erythema migrans. The presence of erythema migrans in a patient with risk factors for Lyme disease is diagnostic of infection. Neuroborreliosis occurs in 10% to 15% of patients with Lyme disease, and cranial nerve palsy, particularly of the facial nerve (cranial nerve VII), is the most common presentation. When unilateral or bilateral facial nerve palsy is present in isolation, oral doxycycline treatment for 14 to 28 days is sufficient for Lyme disease. However, when the central nervous system is involved, parenteral therapy with ceftriaxone, cefotaxime, or penicillin is recommended. In this patient, the presence of headache and nuchal rigidity raise concern for concomitant meningitis. Because confirmation of meningeal involvement would change therapy, a lumbar puncture is first necessary to determine appropriate therapy. Cerebrospinal fluid (CSF) findings in Lyme meningitis are indistinguishable from other forms of aseptic meningitis. Testing for antibodies to Borrelia burgdorferi adds little to the diagnosis because the presence of erythema migrans with cranial neuropathy is sufficient for diagnosis of neuroborreliosis. Serum antibody testing for B. burgdorferi infection would be important in the absence of a compatible skin lesion or with inconsistent exposure history. In neuroborreliosis, a delay in starting antimicrobial treatment is not associated with adverse outcomes as it is in bacterial meningitis, and empiric therapy can be deferred until CSF cell counts are available. Performing a head CT is unnecessary because Lyme neuroborreliosis is rarely associated with intraparenchymal lesions, and thus the risk of lumbar puncture in this previously healthy, cognitively intact patient is low.
A 55-year-old woman is evaluated during a follow-up examination. She completed six cycles of chemotherapy with rituximab plus cyclophosphamide, doxorubicin, vincristine, and prednisone (R-CHOP) for stage III diffuse large B-cell lymphoma with bulky retroperitoneal lymphadenopathy. She takes no medications. The results of the physical examination are unremarkable, with no findings of lymphadenopathy or abdominal masses. CT scan shows a decrease in the confluent retroperitoneal nodal mass, from 15 cm × 17 cm before chemotherapy to 5 cm × 4 cm. PET scan shows no metabolic activity of the mass. No other areas of disease are noted. Which of the following is the most appropriate management? Autologous stem cell transplant CT-guided biopsy of the residual mass Radiation therapy to the site of residual disease Surgical biopsy of the residual mass Observation by serial CT scanning
In a patient with a residual mass after treatment of bulky diffuse large B-cell lymphoma, observation with serial CT scanning is appropriate if diagnostic testing shows a low likelihood of active disease. For this patient with a residual mass after treatment for diffuse large B-cell lymphoma, the appropriate management is observation with serial CT scanning to ensure that the mass is stable. This patient's current CT and PET scan results demonstrate an excellent response to therapy. Her disease was characterized by bulky retroperitoneal lymphadenopathy, and it is probable that the residual mass found on CT scan represents scar tissue rather than viable tumor. Bulky lymphoma masses, when treated with effective systemic therapy, often do not show complete resolution by CT scan. The PET scan results also suggest the absence of active disease, although PET scan is not 100% sensitive or specific. In this circumstance, observation by serial CT (with or without further PET scanning) is most appropriate. An autologous stem cell transplant could be an appropriate option for a patient with relapsed or residual disease. Even if the PET scan showed higher residual uptake, given the potential for false-positive CT and PET scan results, proceeding to further therapy without tissue documentation of recurrence would not be appropriate. Although biopsy could be considered to document the presence or absence of viable lymphoma (and should be done to definitively document disease before initiation of any salvage treatment), this procedure could be deferred in a patient with a low suspicion for disease who has a favorable CT scan and metabolic response by PET, unless there are reasons to expect residual or progressive disease. Consolidative radiation therapy has been used for patients with large cell lymphoma and initial bulky disease and residual mass. This treatment can decrease the risk of local recurrence; however, routine use has not been proven to improve survival or cure, and many patients without active disease would be irradiated.
A 48-year-old woman is hospitalized for a 2-week history of cough, sputum production, fever, and dyspnea. On physical examination, temperature is 39.6 °C (103.3 °F), blood pressure is 110/63 mm Hg, pulse rate is 122/min, and respiration rate is 36/min. Oxygen saturation is 88% breathing ambient air. Lung examination reveals diminished breath sounds over the left base. Cardiac examination is notable only for tachycardia. Chest radiograph reveals a small loculated effusion on the left. A diagnostic thoracentesis is performed, which results in incomplete removal of the effusion. Pleural fluid analysis: pH 6.8 Lactate dehydrogenase 3289 U/L Total protein 3.7 g/dL (37 g/L) Glucose 9 mg/dL (0.5 mmol/L) Gram stain Gram-positive cocci in chains Appropriate intravenous antibiotics are initiated. Which of the following is the most appropriate intrapleural treatment of the effusion? Antibiotics Streptokinase Tissue plasminogen activator-deoxyribonuclease No additional therapy required
Instillation of intrapleural tissue plasminogen activator-deoxyribonuclease has been shown to decrease the radiographic pleural opacity, lower the rate of surgical referral, and decrease hospital stay of patients with empyema. Instillation of intrapleural tissue plasminogen activator-deoxyribonuclease (tPA-DNase) is the most appropriate treatment to promote drainage. This patient has a community-acquired pneumonia complicated by an empyema. The diagnostic criterion for empyema is either visualization of frankly purulent pleural fluid or a positive Gram stain. The Gram stain supports the diagnosis of group A pneumoniae pneumonia. An empyema requires drainage to resolve the infection. A small-bore (14-Fr) tube can be placed to facilitate drainage. An empyema can become loculated (divided into small cavities or compartments) and will not resolve with simple thoracostomy drainage; loculated empyemas often require thorascopic or open surgical debridement. This patient has an incompletely drained empyema due to loculation. When performed twice daily for 3 days, intrapleural administration of tPA-DNase has been shown to decrease the radiographic pleural opacity, lower the rate of surgical intervention, and decrease hospital stay of patients with empyema (MIST-2 trial). It should be noted that the tPA-DNase has not been shown to decrease mortality. In addition, video-assisted thorascopic surgery has also been shown to effectively manage empyema in greater than 90% of cases, and a delay in surgery increases the risk of open thoracotomy. Given the lack of prospective data, a multidisciplinary discussion should be undertaken for any patient presenting with an empyema who is a good surgical candidate. Instillation of intrapleural antibiotic solution may be used for postsurgical empyema but has no demonstrated efficacy in the initial management of a loculated empyema. The MIST-1 trial compared the use of intrapleural instillation of streptokinase compared to saline and found no difference in mortality, need for surgery, radiographic outcome, or length of hospitalization. Consequently, use of fibrinolytics alone in empyema is not recommended. Continued systemic antibiotic therapy without adequate drainage of the pleural space is inadequate therapy. Failure to adequately drain the pleural space of a complicated pleural effusion or empyema will result in failure to resolve the infection, increased morbidity, and the possibility of death by overwhelming infection.
Ivabradine indication?
Ivabradine is a sinus node modulator that reduces heart failure-associated hospitalizations in select heart failure patients; however, it is indicated as therapy only in patients with a reduced left ventricular ejection fraction and heart rate higher than 70/min while receiving β-blocker therapy.
A 71-year-old man is evaluated for a 2-year history of episodes of staring, confusion, and repetitive left-arm movements (grabbing at his clothes or face). These events occur every 2 to 3 months, last approximately 2 minutes, and typically are followed by a 15-minute period of disorientation and fatigue. On one occasion, the episode was followed immediately by the patient's falling down, experiencing shaking of the entire body, and having urinary incontinence. He also has hypertension, nephrolithiasis, and mild cognitive impairment. Medications are hydrochlorothiazide and metoprolol. All physical examination findings, including vital signs, are normal. A routine electroencephalogram and brain MRI are normal. Which of the following is the most appropriate treatment? Lamotrigine Oxcarbazepine Phenytoin Topiramate Valproic acid
Lamotrigine is the most appropriate treatment for epilepsy with focal seizure in older patients because of strong evidence of its safety and effectiveness. This patient should be treated with lamotrigine. He has a history of focal seizures with altered awareness (formerly known as complex partial seizures) and also has had a generalized tonic-clonic seizure. Only a few antiepileptic drugs (AEDs) (namely, lamotrigine, levetiracetam, topiramate, valproic acid, and zonisamide) are considered broad-spectrum agents and can be used to treat both generalized and partial epilepsy syndromes. Other narrow-spectrum AEDs (such as carbamazepine, gabapentin, oxcarbazepine, phenobarbital, phenytoin, and pregabalin) have the potential to exacerbate seizures in patients with generalized epilepsy. Lamotrigine is the most appropriate treatment because of the good evidence of its safety and effectiveness in epilepsy with focal seizures, especially in older patients, and is effective in generalized seizures. Lamotrigine is unlikely to cause cognitive dysfunction and does not affect blood counts, the liver, the kidney, or electrolytes. Although lamotrigine is associated with Stevens-Johnson syndrome, the drug can be titrated slowly to minimize risk; this patient's seizures are infrequent enough to allow for a 6- to 8-week titration. Although all the other drugs listed are likely effective in controlling seizures, lamotrigine also has a lower risk of adverse effects. Oxcarbazepine is known to cause hyponatremia, especially in older patients taking thiazide diuretics, and thus should be avoided in this patient. Phenytoin can lead to dizziness, ataxia, tremor, peripheral neuropathy, cerebellar atrophy, and agranulocytosis. Additionally, it has a narrow therapeutic window that is difficult to maintain, especially in older patients who are usually more sensitive to its zero-order pharmacokinetics at higher doses (constant absorption rate independent of concentration). Kidney stones and cognitive impairment can be exacerbated by topiramate. The drug should be avoided in this patient with mild cognitive impairment and nephrolithiasis. Valproic acid can cause tremor, cognitive dysfunction (including parkinsonism-associated dementia in older persons), and thrombocytopenia and thus is inappropriate for this patient.
A 51-year-old woman is evaluated during a routine follow-up visit for diabetes mellitus. She also has hypertension and hyperlipidemia. Medications are metformin, enalapril, chlorthalidone, and high-intensity rosuvastatin. She has no drug allergies. On physical examination, blood pressure is 126/74 mm Hg. The remainder of the examination is unremarkable. Her 10-year risk for atherosclerotic cardiovascular disease is 11% according to the Pooled Cohort Equations. She has been instructed in intensive lifestyle modifications. Which of the following is the most appropriate preventive measure to reduce this patient's cardiovascular risk? Clopidogrel Low-dose aspirin Low-dose aspirin and clopidogrel Regular-dose aspirin No additional therapy
Low-dose aspirin for the primary prevention of atherosclerotic cardiovascular disease (ASCVD) and colorectal cancer is recommended for adults aged 50 to 59 years with a 10-year ASCVD risk of 10% or higher who do not have an increased risk for bleeding. The most appropriate measure to reduce this patient's atherosclerotic cardiovascular disease (ASCVD) risk is low-dose aspirin. The U.S. Preventive Services Task Force (USPSTF) recommends low-dose aspirin for the primary prevention of ASCVD and colorectal cancer in adults aged 50 to 59 years with a 10-year ASCVD risk of 10% or higher who do not have an increased risk for bleeding, have a life expectancy of at least 10 years, and are willing to take low-dose aspirin daily for at least 10 years. In those aged 60 to 69 years with a 10-year ASCVD risk of 10% or higher, the benefits of aspirin use for primary prevention are smaller but still outweigh the risk for bleeding, and the decision to initiate low-dose aspirin in this population should be individualized. In contrast to the USPSTF recommendations, the American Diabetes Association (ADA) recommends consideration of low-dose aspirin therapy as a primary prevention strategy in patients with type 1 or type 2 diabetes mellitus who are at increased cardiovascular risk. Similarly, the American College of Cardiology/American Heart Association suggest that low-dose aspirin might be considered in patients aged 40 to 70 years at higher risk for ASCVD who do not have an increased risk of bleeding. The decision to initiate low-dose aspirin should be informed by a holistic approach to ASCVD risk estimation that considers risk-enhancing factors, such as strong family history of premature myocardial infarction, smoking, albuminuria, or inability to achieve lipid, blood pressure, or glucose targets. The decision must take into account bleeding risk and patient preferences after careful assessment and a frank discussion of benefits and harms. For patients older than 70 years, aspirin appears to have a greater risk than benefit. The ADA does not recommend aspirin for patients younger than 50 years without major cardiovascular risk factors, although the American College of Cardiology/American Heart Association suggest considering therapy at age 40 years in those at higher ASCVD risk. For patients with ASCVD and documented aspirin allergy, the ADA recommends clopidogrel as an alternative preventive measure. This patient does not have a documented aspirin allergy, and therapy in this patient will be initiated for primary, not secondary, prevention; therefore, clopidogrel is not recommended for this patient. The most commonly recommended dose of aspirin for primary prevention of cardiovascular events is 75 mg to 100 mg. Primary prevention trials have shown that lower doses are likely as effective as higher doses; however, observational trials and a meta-analysis have demonstrated an increased risk for bleeding with regular-dose aspirin compared with low-dose aspirin. Read Related TextNext Question
A 28-year-old woman requests a referral to an obstetrician. She has been attempting to conceive, and her menstrual period is now 3 weeks late. Her home pregnancy test was positive. She has a 2-year history of seropositive rheumatoid arthritis. Her only medication is hydroxychloroquine; methotrexate was stopped 4 months ago in anticipation of conception. On physical examination, vital signs are normal. Swelling of the second and third metacarpophalangeal joints of the right hand is noted. Pregnancy test is positive. Which of the following is the most appropriate treatment at this time? Add etanercept Add leflunomide Discontinue hydroxychloroquine No change in therapy
Methotrexate and leflunomide are contraindicated in pregnant patients because these medications are highly teratogenic and must not be used before/during pregnancy; hydroxychloroquine can be safely used during pregnancy. No change in therapy is needed for this pregnant patient with rheumatoid arthritis (RA). For women with established RA, two thirds will go into remission or achieve low disease activity during pregnancy; however, one third will not improve or will get worse. Improvement may depend on an HLA-DQ mismatch between mother and child and/or other factors such as microchimerism (sharing of maternal cells with the fetus and vice versa). The benefit begins in the first trimester and continues throughout the pregnancy. Disease activity typically returns, often with a flare, after delivery. Medication management of a patient with RA is a major issue, and pregnancy plans should be discussed with any woman of childbearing age who will be placed on therapy. Hydroxychloroquine is an antimalarial medication that appears to inhibit antigen processing. It can be used to treat RA and other forms of inflammatory arthritis. Although its efficacy in arthritis is modest, its excellent side-effect profile makes it a useful adjunctive therapy. Hydroxychloroquine crosses the placenta; however, there does not appear to be fetal toxicity with doses used for the treatment of RA. This patient currently has minimal disease activity and is likely to improve during pregnancy; therefore, she requires no change in therapy. Limited case studies suggest that use of tumor necrosis factor α inhibitors during pregnancy may be safe, but a relationship to rare birth defects has been raised. Different agents may have different potential to cross the placenta. Decisions regarding the use of any biologic agent in pregnancy should incorporate risk-benefit analysis. Etanercept could be used only if this patient has a significant flare of disease during pregnancy but is not needed at the current time. Leflunomide is contraindicated in this patient; this medication is extremely teratogenic and must not be used before/during pregnancy. Pregnancy must be avoided until the drug is no longer detectable in the serum; cholestyramine may be used to hasten the elimination of leflunomide from the body. Methotrexate was discontinued prior to attempted conception, which is highly appropriate. Methotrexate is highly teratogenic and abortifacient, and it must be discontinued at least 3 months before pregnancy.
A 65-year-old man is evaluated in the ICU for a rash limited to his back that was first noticed this morning. He was admitted to the ICU for hospital-acquired pneumonia following hip replacement surgery 3 days ago. Because of deteriorating respiratory function, he was intubated and placed on mechanical ventilation. His current medications are fentanyl and piperacillin-tazobactam. On physical examination, temperature is 38.3 °C (100.9 °F), blood pressure is 110/60 mm Hg, pulse rate is 115/min, and respiration rate is 18/min (ventilator set rate is 14/min). Pulmonary examination reveals diffuse crackles. Skin findings are shown. Which of the following is the most likely diagnosis? Acute generalized exanthematous pustulosis Candida albicans infection Miliaria Povidone iodine contact dermatitis
Miliaria or "heat rash" can appear as superficial clear vesicles or as multiple discrete red papules due to the occlusion of eccrine sweat ducts. The patient has miliaria. Miliaria or "heat rash" can appear as superficial clear vesicles or as multiple discrete red papules due to occlusion of eccrine sweat ducts. When the gland is clogged superficially, there are minute pustules that rupture easily and can be wiped off (miliaria crystallina). Miliaria rubra causes deeper red papules and some pustules when the clog is deeper and more inflammation is present. Miliaria is often seen in the setting of fever and occlusion. A typical clinical situation is a patient who is immobilized, either from pain or following surgery, and the sweat glands are occluded as a result. Therapy is guided toward cooling the affected area, allowing air circulation. Acute generalized exanthematous pustulosis (AGEP) is not an uncommon febrile drug reaction. It tends to occur 1 to 2 days after the offending agent is given. β-Lactam antibiotics are the most common cause of AGEP. It presents as punctate nonfollicular sterile pustules on a background of erythema. It starts on the face and intertriginous areas and spreads to the trunk and extremities. The pustular nature of this condition and its typical location argue against the diagnosis of AGEP. Candidiasis occurs most often in intertriginous areas. An occluded, warm, moist area is an ideal setting for Candida albicans. The skin findings tend to be bright red plaques with surrounding red satellite papules and pustules. The location and appearance of this patient's rash are not consistent with a Candida albicans infection. Postsurgical contact dermatitis to povidone iodine is not uncommon. It tends to occur at the surgical site about 1 to 2 days after surgery. It appears as pruritic, weeping vesicles with underlying erythema often in a well-delineated shape where the povidone was applied. Its appearance is quite unlike miliaria, and it is unlikely that the patient had povidone applied to his back, making this an untenable diagnosis. Read Related TextNext Question
What is mixed connective tissue disease?
Mixed connective tissue disease (MCTD) is an overlap syndrome that includes features of systemic lupus erythematosus (SLE), SSc, and/or polymyositis in the presence of anti-U1-ribonucleoprotein antibodies. More than 50% of patients with MCTD have hand edema and synovitis at disease onset, which is not present in this patient.
CLABSI tx? Duration?
Most uncomplicated CLABSIs are treated for 7 to 14 days; S. aureus CLABSIs may require treatment for 4 to 6 weeks after catheter removal. Shorter therapy durations (14 days) may be considered in select patients with S. aureus CLABSI whose catheters have been removed and who do not have diabetes mellitus, are not immunosuppressed, have no implanted prosthetic devices, have no evidence of endocarditis by echocardiography (preferably transesophageal), and have no evidence of infected thrombophlebitis; whose fever and bacteremia resolve within 72 hours of starting appropriate therapy; and whose physical examination and diagnostic tests do not suggest the presence of metastatic infection.
Difference between NSTE-ACS vs NSTEMI?
NSTE-ACS: No troponin elevation, but no N-ST elevation NSTEMI: Troponin elevation AND NST elevation
Guidelines for lung cancer screening?
Patients recommended for lung cancer screening are those aged 50 to 80 years with a greater than 20-pack-year history of tobacco use within the previous 15 years.
A 59-year-old man is evaluated for recurring epistaxis over the last several months, usually on the right side. He has been able to stop the bleeding by applying pressure to the nares, but on one occasion, he required treatment in the emergency department. Typically, blood loss has been minimal. He does not use intranasal medications and reports no substance use. He has otherwise been well. There is no family history of epistaxis or rheumatologic or bleeding disorders. His only medication is low-dose aspirin. On physical examination, vital signs are normal. On nasal examination, there are no clear lesions, erythema, petechiae, scabs, telangiectasias, ulcers, or visible bleeding. Which of the following is the most appropriate management? Discontinue aspirin Obtain coagulation studies Obtain complete blood count Perform nasal endoscopy
Recurrent unilateral epistaxis may be a sign of neoplasm and warrants referral for nasal endoscopy. The most appropriate management is nasal endoscopy. Ninety percent of episodes of epistaxis occur in the anterior nasal septum in the Kiesselbach area. Anterior bleeding can be managed with compression for at least 5 minutes. Posterior epistaxis (behind the posterior middle turbinate, requiring a nasopharyngoscope for visualization) may be more difficult to manage and is more common in older patients. Common causes of epistaxis include topical intranasal medications (such as glucocorticoids or antihistamines), dehumidification, and self-induced digital trauma. Among patients with epistaxis serious enough to require hospitalization, almost half have a causal systemic condition, such as anticoagulation, hemophilia, hematologic malignancy, neoplasm, and acquired coagulopathies from kidney or liver disease. Recurrent unilateral epistaxis may represent a neoplasm; hence, this patient should be referred for nasal endoscopy. Low-dose aspirin may be associated with a slight increase in the risk for epistaxis, although data have not been conclusive and do not support routine discontinuation in otherwise healthy patients with epistaxis. Notably, other NSAIDs have not been associated with epistaxis. Even if aspirin were stopped, this patient requires further assessment given the presence of unilateral bleeding. This patient has no symptoms of anemia or history of excessive blood loss. Although epistaxis may be associated with coagulopathies, this patient is not taking an anticoagulant and is otherwise at low risk for an acquired coagulopathy that manifests only as epistaxis. Therefore, coagulation studies are not warranted. In 80% of patients with epistaxis, results of laboratory evaluation are normal; therefore, routine laboratory testing is not required. A complete blood count, prothrombin time, and activated partial thromboplastin time might be considered in patients with symptoms or signs of a bleeding disorder and those with severe or recurrent epistaxis.
A 32-year-old woman is evaluated for a painful rash occurring bilaterally on her lower extremities, which began 2 days ago. She was admitted to the hospital 5 days ago for diagnosis and treatment of pulmonary embolism with heparin and warfarin. Her father had a pulmonary embolism after a long airplane ride. Her medical history is otherwise unremarkable, and she takes no other medications. On physical examination, vital signs are normal. She has nonblanchable macules and papules and areas of cutaneous necrosis in an angulated reticular pattern on the lower legs; findings on the feet are shown. The examination is otherwise unremarkable. Laboratory studies show a normal complete blood count, a normal peripheral blood smear, and an INR of 4. Which of the following is the most likely diagnosis? Factor V Leiden mutation Plasminogen activator inhibitor 1 deficiency Protein C deficiency Prothrombin gene mutation
Protein C or S deficiency is associated with warfarin-associated skin necrosis. The most likely diagnosis is protein C deficiency. The patient's skin condition is most likely retiform purpura. The term "retiform" describes the angulated or netlike configuration that reflects the vascular structure in the skin. The color is often a dark brick-red or purple. Retiform purpura is caused by local skin ischemia caused by occlusion or breakdown of vascular integrity that may lead to necrosis, which may become life-threatening if not aggressively treated. Various conditions can cause retiform purpura, many of which disrupt arterial blood flow. Thrombotic and embolic causes should be considered first. Thrombotic causes include alterations to the coagulation cascade such as disseminated intravascular coagulation, thrombotic thrombocytopenic purpura, and drug-induced thrombosis (warfarin or heparin). The patient's clinical presentation is consistent with warfarin-induced skin necrosis, which is a rare complication of warfarin therapy. The pathophysiology is thought to be caused by a transient hypercoagulable state resulting from protein C deficiency, which can be inherited or acquired. A family history of pulmonary embolism suggests the possibility of inherited protein C deficiency. Congenital protein C deficiency is an autosomal dominant inherited thrombophilia associated with an increased risk of venous thromboembolism. When protein C is activated, it inactivates the activated coagulation factors V and VIII, which are needed for factor X activation. When warfarin is initiated, an initial reduction in protein C activity of 50% occurs, which leads to a transient hypercoagulable state. If the patient is receiving heparin and warfarin therapy, the lesions may appear when the heparin is discontinued, which likely accounts for the appearance of the rash on day 3 of her hospitalization. Treatment involves the discontinuation of warfarin, continuation of alternate anticoagulation (such as a non-vitamin K antagonist oral anticoagulant), vitamin K for warfarin reversal, and fresh frozen plasma in an attempt to improve decreased protein C levels. Warfarin-induced skin necrosis is not associated with factor V Leiden mutation, plasminogen activator inhibitor 1 deficiency, or prothrombin gene mutation.
ITP Tx?
Recommended initial therapy includes a short course (<6 weeks) of prednisone or dexamethasone. The response to intravenous immune globulin (IVIG) is faster and may be indicated in patients with more severe thrombocytopenia and life-threatening bleeding. Complications from glucocorticoids may include mood disorders, insomnia, fluid retention, hyperglycemia, and hypertension; complications from IVIG may include infusion reactions (headache, chills, anaphylaxis), kidney disease, and thrombosis. Those patients who relapse or have ITP refractory to these treatments require second-line treatments, such as splenectomy, rituximab, or thrombopoietin receptor agonists.
A 60-year-old woman is evaluated for fatigue and weakness. She reports no nausea or vomiting. History is significant for hypertension, stage G4 chronic kidney disease, and type 2 diabetes mellitus. Medications are labetalol, amlodipine, insulin glargine, insulin lispro, and sodium bicarbonate. On physical examination, blood pressure is 140/90 mm Hg; other vital signs are normal. A mature radiocephalic arteriovenous fistula (AVF) with a strong thrill and bruit is noted. There are no lung crackles. Trace pedal edema is present. Laboratory studies show normal serum bicarbonate and potassium levels, a blood urea nitrogen level of 50 mg/dL (17.8 mmol/L), and an estimated glomerular filtration rate of 18 mL/min/1.73 m2. Which of the following is the most appropriate management? Clinical follow-up in 6 months Fistulography to evaluate patency of AVF Hemodialysis Kidney transplant evaluation
Referral for kidney transplant evaluation is indicated when the estimated glomerular filtration rate is <30 mL/min/1.73 m2 to allow for adequate time to identify suitable living donors or to be put on an early listing if no living donor is available. Kidney transplant evaluation is the most appropriate management for this patient with stage G4 chronic kidney disease (CKD). Referral to a kidney transplant center is indicated when the estimated glomerular filtration rate (eGFR) is <30 mL/min/1.73 m2. Kidney transplantation is the preferred treatment for patients with end-stage kidney disease, because it improves life expectancy and quality of life. It also provides a significant cost savings to the health care system compared with maintaining a patient on dialysis. Early referral allows adequate time to identify suitable living donors. If no living donor is available, early listing is essential to begin the waiting process for a deceased-donor kidney. Patients undergo an extensive health screening to identify potential issues that may affect the safety and/or outcome of the transplant. In the potential recipient, these include active malignancy, coronary ischemia, or active infections. An adequate social support system and financial resources also are important to ensure medication adherence and long-term survival of the transplanted allograft. Patients with stage G4 chronic kidney disease should be seen at least every 3 months, not 6 months. More importantly, clinical follow-up alone does not address the immediate problem of planning for renal replacement therapy, ideally with kidney transplantation. Fistulography is not indicated because physical examination findings suggest that the fistula is mature and may be ready to use for hemodialysis. Invasive imaging is not required to confirm patency unless physical examination findings suggest a problem. Additionally, fistulography exposes the patient to nephrotoxic iodinated contrast and may reduce residual kidney function. Although this patient has an arteriovenous fistula, hemodialysis is not indicated at this time because she has normal serum potassium and bicarbonate levels, does not have significant volume overload, and does not have uremic symptoms. Several recent studies have demonstrated no benefit in starting dialysis in asymptomatic patients early based upon an arbitrary eGFR cutoff compared with waiting until patients develop very low eGFR (<8-10 mL/min/1.73 m2) or clinical indications for dialysis are present.
A 45-year-old woman is evaluated for a 2-day history of deep boring pain in the right eye. She also describes eye redness and photophobia but no recent trauma to the eye. She has a 10-year history of rheumatoid arthritis, treated with etanercept. On physical examination, vital signs are normal. Diffuse right eye redness is noted, and there is pain on extraocular movement testing. Gentle pressure over the eye with the lid closed results in pain. There is no scleromalacia in either eye. There is diminished visual acuity in the right eye. Which of the following is the most likely diagnosis? Episcleritis Scleritis Subconjunctival hemorrhage Viral conjunctivitis
Rheumatoid arthritis is one of the most common diseases associated with scleritis, which can be vision-threatening and lead to thinning of the sclera and perforation. The most likely diagnosis in this patient with rheumatoid arthritis (RA) is scleritis. RA is one of the most common diseases associated with scleritis. Typical features include eye pain, pain with gentle palpation of the globe, and photophobia. The deep scleral vessels are involved and may lead to scleromalacia, which is characterized by thinning of the sclera and is seen as a dark area in the white sclera. Scleromalacia may lead to perforation of the sclera, called scleromalacia perforans. Scleritis can be vision-threatening and lead to blindness; it is therefore important to urgently refer the patient to an ophthalmologist for care. Episcleritis is an abrupt inflammation of the superficial vessels of the episclera, a thin membrane that lies just beneath the conjunctiva. The cause is often unclear; rarely, it is associated with systemic rheumatologic disease. Patients with episcleritis frequently present without pain or decreased visual acuity. On examination, the inflammation appears localized. White sclera can be seen between superficial dilated blood vessels. Episcleritis typically resolves spontaneously. The presence of severe pain, diffuse redness, and decreased visual acuity make episcleritis an unlikely diagnosis. Subconjunctival hemorrhage is a common disorder and typically benign in origin. It is caused by painless bleeding into the superficial portion of the eye. Examination reveals a blotchy redness (from extravascular blood) that is typically confined to one area of the conjunctiva. Subconjunctival hemorrhage is painless and not associated with loss of vision. Most cases resolve within several weeks without intervention. The patient's findings are not compatible with subconjunctival hemorrhage. Viral conjunctivitis also causes a red eye. Typically, the underlying vessels are visible, a watery discharge may be seen, and the eyelids are matted in the morning. The eye may feel irritated, but there is no pain or loss of visual acuity. In general, conjunctivitis is a diagnosis of exclusion. The presence of pain and decreased visual acuity exclude viral conjunctivitis in this patient. Read Related TextNext Question
A 42-year-old man is evaluated in the hospital for increased pain and drainage from a previously healed surgical wound over the left fibula. He underwent open reduction and internal fixation of a fracture 4 weeks ago. The patient has undergone incision and surgical debridement of the wound. A bone culture revealed methicillin-sensitive Staphylococcus aureus. Medical history is otherwise noncontributory, and his only medication is ibuprofen for pain. On physical examination, vital signs are normal. A surgical wound over the left lateral leg is well approximated with no erythema or drainage. A plain radiograph before debridement shows nonunion of the fracture with screws and K-wires in place. Which of the following is the most appropriate treatment? Cefazolin Cefazolin and rifampin Ceftaroline Vancomycin and rifampin
Rifampin should be used in combination with another antistaphylococcal agent when managing Staphylococcus aureus osteomyelitis in the setting of orthopedic hardware if the hardware cannot be removed. Cefazolin and rifampin are appropriate therapy for treatment of methicillin-sensitive Staphylococcus aureus (MSSA) osteomyelitis associated with orthopedic hardware. Identification of the causative pathogen, administration of adequate antimicrobials for a prolonged duration, surgical debridement (if warranted), and removal of orthopedic prosthetic devices (if feasible) influence the success of osteomyelitis treatment. Optimal management of this patient's infection includes hardware removal; however, this is not possible because the fracture has not yet healed. Hardware-associated infections caused by S. aureus are difficult to eradicate because of the biofilm that forms on the hardware. First-line treatment of MSSA osteomyelitis consists of a β-lactam agent such as cefazolin; a randomized controlled trial and systematic review of the literature have demonstrated that if infected hardware cannot be removed, the addition of rifampin increases the chances of therapeutic success compared with an antistaphylococcal agent alone. Although cefazolin has activity against MSSA as well as good bone penetration, it would not be an appropriate therapeutic option for the treatment of hardware-associated osteomyelitis without the addition of rifampin. Ceftaroline has coverage for MSSA, methicillin-resistant S. aureus, and Enterobacteriaceae, but it is unnecessarily broad coverage for the treatment of this patient's MSSA infection. Vancomycin, a bacteriostatic agent, is less effective than β-lactam agents for the treatment of MSSA and is typically restricted to patients with drug intolerance or allergy.
What does weber test show? -better heard in same ear as hearing loss -better heard in opposite ear as hearing loss
Same ear- conductive loss Opposite ear- sensorineural loss
A 36-year-old woman is evaluated 3 days after being hospitalized for gallstone pancreatitis. An abdominal ultrasound showed multiple gallstones in the gallbladder and a normal-diameter common bile duct. She was treated with intravenous hydration and pain medication. Over the course of 3 days, she tolerated eating and her pain subsided. On physical examination, vital signs are normal; BMI is 32. Minimal tenderness to palpation is noted on abdominal examination. All other findings are unremarkable. All laboratory studies have returned to baseline normal values. Which of the following is the most appropriate treatment? Endoscopic retrograde cholangiopancreatography Laparoscopic cholecystectomy before discharge from the hospital Laparoscopic cholecystectomy 4 weeks after discharge from the hospital MR cholangiopancreatography Ursodeoxycholic acid
Same-admission cholecystectomy reduces rates of gallstone-related complications compared with cholecystectomy after hospital discharge for patients with mild gallstone pancreatitis. Laparoscopic cholecystectomy before discharge from the hospital is the most appropriate treatment. Gallstone acute pancreatitis can be diagnosed based on elevated liver transaminases on presentation, a lipase level elevated to more than three times the upper limit of normal, characteristic severe abdominal pain, and ultrasonographic evidence of cholelithiasis. This patient showed clinical improvement within 3 days of hospitalization. Her laboratory values have normalized, suggesting spontaneous passage of a gallstone through the common bile duct, which occurs in most patients with gallstone pancreatitis. In a multicenter randomized controlled trial, same-admission cholecystectomy reduced rates of gallstone-related complications compared with interval cholecystectomy 25 to 30 days after hospital discharge for patients with mild gallstone pancreatitis. Endoscopic retrograde cholangiopancreatography (ERCP) is indicated urgently for patients with acute pancreatitis and ascending cholangitis (fever, abdominal pain, and jaundice) due to choledocholithiasis. If there is evidence of ongoing biliary obstruction in patients hospitalized with acute pancreatitis, ERCP may be indicated to remove a retained gallstone from the common bile duct. This patient's symptoms and laboratory abnormalities resolved quickly, which supports the spontaneous passage of a gallstone without evidence of ongoing biliary obstruction. MR cholangiopancreatography (MRCP) can be used to identify causes of biliary obstruction. MRCP is not needed in this patient because she has normal-caliber bile ducts on abdominal ultrasonography and normal liver chemistry test results, indicating that a biliary obstruction is unlikely. Ursodeoxycholic acid has been used to medically dissolve small cholesterol gallstones in patients who are not candidates for surgery. The medication works slowly and may take longer than 1 year to dissolve small stones, leaving patients at risk for recurrent attacks of gallstone pancreatitis or other gallstone-related complications. This patient is young and without comorbidities, making surgery a more appropriate treatment.
A 32-year-old man is evaluated for decreased libido and fatigue. His symptoms have increased over the last 6 months. His medical history is otherwise unremarkable, and he takes no medications. On physical examination, vital signs are normal. BMI is 26. He has gynecomastia. Visual field acuity testing and testicular examination are normal. Smell is intact. Laboratory studies: Follicle-stimulating hormone 4 mU/mL (4 U/L) Luteinizing hormone 5 mU/mL (5 U/L) Prolactin 100 ng/mL (100 µg/L) Testosterone 110 ng/dL (3.8 nmol/L) Which of the following is the most appropriate diagnostic test to perform next? Karyotype analysis Pituitary MRI Screening for anabolic steroid abuse Serum ferritin measurement Sex hormone-binding globulin measurement
Secondary hypogonadism is characterized by low testosterone level and low or inappropriately normal serum luteinizing hormone and follicle-stimulating hormone concentrations; MRI of the pituitary is typically performed to evaluate secondary hypogonadism in the absence of obvious reversible causes such as drugs.
A 27-year-old woman is evaluated for proteinuria identified on urinalysis performed for a life insurance examination. She reports no symptoms. History is significant for premature birth, a 2-year history of hypertriglyceridemia and prediabetes, and a 5-year history of obesity. The remainder of her medical history is unremarkable. Her only medication is gemfibrozil. On physical examination, vital signs are normal. BMI is 37. The remainder of the examination is unremarkable. Laboratory studies: Albumin 3.8 g/dL (38 g/L) Creatinine 1.0 mg/dL (88.4 µmol/L) Hemoglobin A1c 6.4% Urinalysis No blood; 3+ protein Urine protein-creatinine ratio 2100 mg/g Kidney ultrasound shows normal-appearing kidneys with no masses or hydronephrosis. Which of the following is the most likely diagnosis? Diabetic nephropathy Lipoprotein glomerulopathy Minimal change glomerulopathy Secondary focal segmental glomerulosclerosis
Secondary focal segmental glomerulosclerosis (FSGS) is the most likely diagnosis. FSGS is the most common form of the nephrotic syndrome in black persons. In the United States, FSGS currently accounts for up to 40% of idiopathic (primary) nephrotic syndromes in adults. The pathogenesis of FSGS stems from podocyte injury due to immunologic, genetic, and/or hyperfiltration causes. A large and growing proportion of FSGS cases are considered secondary forms of FSGS due to hyperfiltration injury in the setting of relatively reduced renal mass. The overworking of the glomerulus in this setting leads to adaptive podocyte injury and segmental sclerosis. This hyperfiltration form of FSGS is classically seen in patients with obesity but also can manifest in patients with a history of premature birth or solitary kidney. This patient has two risk factors for the secondary form of FSGS: obesity and a history of premature birth. In addition, her presentation is more typical of a secondary FSGS lesion, with subnephrotic proteinuria and no associated clinical findings. Electron microscopy of her kidney biopsy would be expected to show only mild to moderate effacement of the podocyte's foot processes. An immunologic route to injury is considered the main pathogenic mechanism behind primary forms of FSGS, with leukocytes producing a soluble circulating factor that directly targets podocytes. In such cases, proteinuria tends to be heavy (nephrotic range) with associated hypoalbuminemia, and edema is usually present on physical examination. Electron microscopy of a kidney biopsy with primary FSGS will typically show extensive effacement of the podocyte's foot process. Diabetic nephropathy is the sequelae of chronic glycemic-induced damage to the glomerulus. On average, it occurs 8 years after the diagnosis of overt diabetes mellitus and is typically associated with other microvascular or macrovascular complications of diabetes. This patient's short history of prediabetes and lack of other microvascular/macrovascular findings makes diabetic nephropathy an unlikely diagnosis. Lipoprotein glomerulopathy is a rare kidney disease characterized by moderate to severe proteinuria, progressive kidney failure, and distinct histopathologic findings of glomerular capillary dilatation by lipoprotein thrombi. To date, less than 100 cases have been reported in the medical literature, nearly all of which are from East Asian countries (predominantly Japan and China). This patient does not fit the profile for lipoprotein glomerulopathy. Minimal change glomerulopathy typically presents with the full nephrotic syndrome (proteinuria >3500 mg/24 h, serum albumin usually <3.0 g/dL [30 g/L], hypercholesterolemia, and edema on examination). These findings do not fit this patient's mild presentation.
Effect of smoking on RA development?
Smoking stimulates the production of enzymes that modify arginine to citrulline, and the modified proteins containing citrulline are targets for the immune system in susceptible individuals. T
Tx for kidney stones?
Stones up to 10 mm can be managed conservatively, although the likelihood of spontaneous passage decreases with increasing size. Medical expulsive therapy with α-blocker therapy (such as tamsulosin) or a calcium channel blocker (such as nifedipine) can aid the passage of small stones (≤10 mm in diameter). This large stone associated with probable obstruction is not a candidate for medical expulsive therapy with either tamsulosin or nifedipine.
A 52-year-old man is evaluated for a 9-month history of progressively worsening gait, ataxia, and paresthesia in the legs. He is a dentist who was recently barred from practice because of chronic illicit drug use, specifically amphetamines and nitrous oxide. On physical examination, vital signs are normal. Decreased vibration and position sense is noted in both feet. Reflexes are 3+ in both legs. Muscle strength is 4/5 in both hips. A complete blood count is normal, as is a routine chemistry panel. A T2-weighted MRI of the thoracic spinal cord shows hyperintensity in the posterior columns and throughout the cord. There is no associated contrast enhancement. Measurement of which of the following serum levels is the most appropriate next diagnostic test? 25-Hydroxyvitamin D Thiamine Vitamin A Vitamin B6 Vitamin B12
Subacute combined degeneration is a myelopathy manifesting as dysfunction of the corticospinal tracts and dorsal columns that is caused by vitamin B12 and copper deficiencies. This patient's serum level of vitamin B12 should be measured. His examination findings are consistent with a myelopathy localizing to both the posterior columns and the corticospinal tracts of the spinal cord, and neuroimaging confirms the diagnosis. Common entities causing this pattern of weakness and sensory deficits are vitamin B12 deficiency, copper deficiency, and neurosyphilis. When these symptoms occur as a consequence of vitamin B12 or copper deficiency, the neurologic syndrome is termed subacute combined degeneration. This patient's previous chronic abuse of nitrous oxide has resulted in a functional B12 deficiency that is due to inactivation of the vitamin. In patients who abuse nitrous oxide, the B12 level also is often low or low-normal, with elevated methylmalonic acid and homocysteine levels. Not all patients with neurologic abnormalities will have anemia or macrocytosis. Although low 25-hydroxyvitamin D levels have been associated with several conditions, including increased risk for autoimmune disease, this vitamin deficiency itself is not a cause of myelopathy. Similarly, thiamine deficiency is associated with Wernicke-Korsakoff syndrome (nystagmus, ophthalmoplegia, ataxia, and confusion) but not with myelopathy. Wernicke encephalopathy most commonly occurs in persons with alcoholism but also has been reported in patients who have undergone bariatric surgery, have had a prolonged period of fasting, have had repeated episodes of vomiting, or have been on prolonged parenteral nutrition without adequate vitamin supplementation. Vitamin A deficiency can cause blindness and hypervitaminosis and has been associated with benign idiopathic intracranial hypertension. Vitamin A deficiency is rarely seen in the United States but is the most common vitamin deficiency worldwide. When it does occur in resource-rich countries, it typically is associated with conditions causing fat malabsorption, such as pancreatic insufficiency, celiac disease, or certain forms of bariatric surgery. Alterations in vitamin A level do not result in myelopathy. Vitamin B6 deficiency is not common but may be associated with drugs that interfere with its metabolism, including isoniazid, hydralazine, carbidopa, and levodopa. The most common manifestations are stomatitis, glossitis, cheilosis, confusion, and depression. Both deficiency and toxicity of this nutrient can cause peripheral neuropathy. The upper motor neuron signs evident on physical examination of this patient, however, rule out a peripheral neuropathy.
A 72-year-old man is evaluated during a follow-up visit. He was evaluated in the emergency department 2 weeks ago for the sudden onset of chest pain. A CT scan was negative for pulmonary embolism but demonstrated an 8-mm ground-glass nodule in the right upper lobe. He has had no recurrence of chest pain. His history is significant for hypertension treated with lisinopril. Upon physical examination, vital signs are normal. The remainder of the physical examination is normal. The patient undergoes follow-up CT scans of his lung at 12 months and also at 2 years. The nodule is unchanged. Chest CT scans every 2 years for 5 years PET/CT scan Tissue sampling No further follow-up is needed
Subsolid lung nodules 6-8 mm in size should be initially followed up at 6-12 months and then every 2 years for 5 years because of the slow rate of growth if such masses are malignant. The most appropriate management of the pulmonary nodule is to perform follow-up chest CT scanning at 6-12 months and then every 2 years for 5 years, as recommended by the Fleischner Society Guidelines. Nodules are classified as solid or subsolid. Subsolid nodules are either pure ground-glass nodules (no solid component) or part-solid nodules (both ground-glass and solid components). A ground-glass nodule is defined as a focal area of increased attenuation in the lung through which normal parenchymal structures can still be seen. The classification of nodules helps in the assessment of malignant potential (for example, adenocarcinoma is more likely to present as a subsolid and part-solid nodule) and guides appropriate follow-up. This patient has a solitary pure ground-glass subsolid nodule that is larger than 6 mm. Earlier guidelines recommended initial follow-up at 3 months, but this was changed to 6-12 months because earlier follow-up is unlikely to affect the outcome of these characteristically indolent lesions. The average doubling time of subsolid, cancerous nodules typically is 3-5 years. Therefore, longer initial and total follow-up intervals are recommended for subsolid nodules than for solid nodules. Evaluation with a PET/CT scan would be recommended for a solid nodule that is greater than 8 mm in size. This test most commonly uses fluorodeoxyglucose (FDG) as a metabolic marker to identify rapidly dividing cells such as tumor cells and, to a lesser degree, any inflammatory lesion. A nodule that demonstrates no FDG uptake is unlikely to be malignant. PET/CT imaging can also be used for staging a cancer by determining the presence or absence of metastatic disease. Tissue sampling would not be appropriate at this stage because the vast majority of these lesions are not malignant.
Guidelines for colon cancer screening?
The 2016 USPSTF guideline recommends sensitive gFOBT or FIT annually or multitargeted stool DNA testing every 3 years. Flexible sigmoidoscopy is recommended every 5 years, but if combined with FIT (or possibly gFOBT), the interval can be increased to every 10 years, the same interval recommended for colonoscopy. CT colonography can be performed every 5 years. The ACP suggests colonoscopy every 10 years, flexible sigmoidoscopy every 10 years plus fecal immunochemical testing (FIT) every 2 years, or FIT or high-sensitivity guaiac-based fecal occult blood testing every 2 years as acceptable screening intervals.
A 65-year-old man is evaluated after a screening ultrasound for abdominal aortic aneurysm showed incidental gallbladder findings. He reports no symptoms. He continues to smoke cigarettes, 1 pack per day. He has no other medical problems and takes no medications. On physical examination, vital signs are normal, as is the remainder of the examination. The results of all laboratory studies, including a complete blood count and alkaline phosphatase, alanine aminotransferase, aspartate aminotransferase, and total bilirubin levels, are within normal limits. The abdominal ultrasound shows numerous layering gallstones and an immobile 8-mm gallbladder polyp. Which of the following is the most appropriate next step in management? Cholecystectomy MR cholangiopancreatography Repeat ultrasonography in 6 months Ursodeoxycholic acid
The finding of a gallbladder polyp larger than 1 cm in size, or a polyp of any size associated with gallstones, is an indication for cholecystectomy even if the patient is asymptomatic. Cholecystectomy is indicated for this patient with a gallbladder polyp and gallstones because of the increased risk for gallbladder cancer when the two conditions coexist. The finding of a gallbladder polyp larger than 1 cm in size is an indication for cholecystectomy, even if the patient is asymptomatic. An additional indication for prophylactic cholecystectomy is the presence of a gallbladder polyp larger than 8 mm in size in the setting of primary sclerosing cholangitis. Gallbladder polyps are found on approximately 5% of ultrasounds. Although only a small percentage of gallbladder polyps are neoplastic (adenoma or adenocarcinoma), the risk for neoplasia increases as polyp size increases. Further evaluation of the polyp with abdominal CT, endoscopic retrograde cholangiopancreatography, or MR cholangiopancreatography is not indicated because gallbladder ultrasonography is adequately sensitive for the detection of gallbladder lesions. These tests could be considered if this patient had symptoms or elevated liver chemistry tests suggesting bile-duct obstruction or malignancy. In a patient with an 8-mm gallbladder polyp in the absence of gallstones or primary sclerosing cholangitis, repeat ultrasonography in 6 months would be indicated. However, follow-up ultrasonography is not appropriate for this patient with a gallbladder polyp and gallstones, which increase the risk for gallbladder cancer. Bile acids, such as ursodeoxycholic acid, work by reducing biliary cholesterol secretion, thereby increasing biliary bile-acid concentrations and, as a result, reducing the cholesterol saturation index and gallstone size. Bile-acid therapy works best for small, primarily cholesterol gallstones and is indicated in patients who cannot or will not undergo laparoscopic cholecystectomy. More importantly, ursodeoxycholic acid therapy would not address this patient's gallbladder polyp. Read Related TextNext Question
What could pulsatile tinnitus indicate?
The type of tinnitus is an important factor in the evaluation. Pulsatile tinnitus, when synchronous with the heartbeat, may suggest a vascular anomaly, including atherosclerotic disease, arteriovenous fistulas, or paragangliomas, most commonly in the jugular bulb or tympanic arteries of the middle ear. A patient with pulsatile tinnitus should be examined for bruits over the neck, periauricular area, temple, orbit, and mastoid areas. If the physical examination findings do not explain the pulsatile tinnitus, noninvasive intracranial imaging, including CT angiography or MR angiography, should be performed. This patient does not have pulsatile tinnitus but does have unilateral hearing loss. Therefore, imaging of the internal auditory canal for acoustic neuroma will be of higher diagnostic yield than vascular imaging.
During a routine health examination, a patient asks about an article that recommended avoiding statin therapy because of the risk for memory loss. The findings were based on cross-sectional data analysis of a well-validated national health survey, which was conducted by random sampling of patients according to zip code of residence. The analysis showed that patients who self-reported memory loss were more likely to also report having taken statin drugs (odds ratio, 1.8; 95% CI, 1.2-2.7; P = 0.046). Which of the following is the most likely threat to the validity of this study? Confounding Selection bias Self-reported data Statistical significance
The validity of cross-sectional studies is particularly susceptible to recall bias and confounding. The most likely threat to the validity of this cross-sectional study is confounding. Cross-sectional studies evaluate the relationship between exposures and health outcomes in a population of interest. These studies are characterized by the measurement of factors and outcomes at a single point in time. The validity of cross-sectional studies is particularly susceptible to recall bias and confounding. Recall bias is a systematic error that is introduced into a study by differences in the accuracy of the recollections of study participants; participants who have unpleasant experiences may recall past events differently than those who do not have similar experiences. Because cross-sectional studies are observational and not experimental, there is also no opportunity to randomly distribute factors that might influence the relationship being studied. Although statistical techniques can be used to control for known potential confounders, unknown confounders remain a threat to the validity of the conclusions. As such, cross-sectional studies are best suited to identifying potentially significant associations that can be more rigorously tested in experimental studies. Because there is no way to verify that the purported cause (statin therapy) preceded the effect (memory loss), cross-sectional studies cannot prove cause-and-effect relationships. Therefore, the study cannot conclude that statins cause memory loss and should not recommend avoidance of statin therapy. Selection bias occurs when the study participants do not accurately reflect the population being studied, usually because the choice to participate is influenced by the clinical question. Selection bias can compromise the validity of observational study designs; however, in this study, the random sampling according to zip code of residence minimizes the possibility of selection bias. Although self-reported data are less robust than measured data, well-validated survey designs may use self-reported data to determine the presence or absence of conditions, risk factors, or behaviors in a population. Self-reported data do not necessarily result in recall bias. A well-constructed survey can minimize recall bias by focusing on the collection of the most objective data that are available (for example, by asking if the patient has consulted a physician about memory loss, has been evaluated for memory loss, is being treated for memory loss, or has been told by family members that memory loss is present). The conventional level of statistical significance is a P value less than or equal to 0.05, and an odds ratio of 1 implies the absence of a significant relationship. In this case, the confidence interval for the odds ratio does not include the value 1, which supports the statistical significance of the findings. Read Related TextNext Question
A 35-year-old woman is evaluated during a follow-up visit for anxiety. Her symptoms have been well controlled with cognitive behavioral therapy, and she has been drinking kava tea every morning and practicing mindfulness daily. Medical history is significant for nonalcoholic steatohepatitis. Other supplements are folic acid and echinacea. On physical examination, vital signs are normal. BMI is 36. The remainder of the examination is unremarkable. Which of the following is the most appropriate recommendation for this patient? Discontinue echinacea Discontinue folic acid Discontinue kava tea No changes to current therapy
This patient should be advised to discontinue drinking kava tea. Patients take dietary supplements for various reasons, such as to prevent illness, enhance health, and correct perceived deficiencies. In the United States, approximately 50% of adults report using vitamins or dietary supplements, with total consumer spending of more than $20 billion each year. Despite their prevalent use, the U.S. Preventive Services Task Force does not recommend multivitamins or herbal supplements for the prevention of cardiovascular disease or cancer. In addition to questionable efficacy, supplement use is associated with risk for considerable harms, including side effects; interactions with other drugs; and harms related to inclusion of unadvertised additives, compounds, or toxins. Despite the risks, many patients strongly believe in supplement use, and the role of the physician is to inform these patients of harmful supplements and suggest safer alternatives. This patient is taking kava, which is derived from Piper methysticum, a plant native to the western Pacific islands. It is often used to relieve stress and anxiety but has been associated with liver damage. In 2002, the FDA issued a consumer advisory regarding the potential risk for severe liver injury with kava use, especially in patients with liver disease or at risk for liver disease. Therefore, advising this patient with nonalcoholic steatohepatitis to discontinue kava tea would be the best management option. Echinacea may be slightly effective for prevention but not treatment of the common cold. The most common side effects are gastrointestinal upset and nausea. Although this patient does not need to take echinacea, this herb has a relatively safe side effect profile, and she may continue it if she wishes. The U.S. Preventive Services Task Force recommends daily folic acid supplementation (400 to 800 µg) for women of childbearing age to prevent fetal neural tube defects; therefore, folic acid should be continued in this 35-year-old patient. This patient has diagnoses of nonalcoholic steatohepatitis and obesity, and continuing an herbal supplement that is associated with known hepatotoxicity may cause harms. Read Related TextNext Question
A 75-year-old man is hospitalized for treatment of extreme agitation and delirium after developing a urinary tract infection (UTI). He has a 1-year history of dementia with Lewy bodies. According to his wife, the patient has nighttime visual hallucinations 4 or 5 times per week but is rarely bothered by them. Home medications are simvastatin and aspirin. On physical examination, blood pressure is 150/92 mm Hg and pulse rate is 98/min; other vital signs are normal. The patient is agitated and disoriented and appears to be having visual hallucinations. Although his agitation steadily increases, he does not become aggressive. His UTI is being treated appropriately. No other clear sources of the delirium are present. Environmental interventions are instituted to help abate his symptoms. Which of the following medications is most likely to be effective in treating his acute agitation? Alprazolam Diphenhydramine Donepezil Haloperidol
This patient should be treated with donepezil. Delirium is a potentially preventable syndrome associated with other medical disorders, the adverse effects of medication, or drug withdrawal. Preexisting cognitive impairment is the major predisposing factor for delirium. With treatment, delirium typically resolves within days. Environmental interventions include orienting strategies (reliance on calendars, clocks, and familiar objects in the room), promoting a normal sleep-wake cycle (having no or limited interruptions during nocturnal sleeping hours, minimizing light and noise at night, and opening curtains and encouraging activity during the daytime), identifying and correcting sensory impairments, avoiding physical restraints, and early discontinuation of catheters and intravenous lines. This patient has dementia with Lewy bodies (DLB) and has developed acute agitation and delirium associated with a concurrent urinary tract infection (UTI). He should continue to receive therapy for the UTI until it resolves. The treatment of acute agitation in patients with dementia is challenging, and no medications are FDA approved for the treatment of delirium. Atypical and typical antipsychotic agents have a black-box safety warning when used in patients with underlying dementia. Although not formally approved for DLB, donepezil can be effective in treating the behavioral and cognitive symptoms associated with the disorder; controlled clinical trials, however, remain inconclusive. Because donepezil (or any other medication) has not been shown to be efficacious for DLB in high level, well-conducted trials, its use is based on expert opinion at this time. Nevertheless, donepezil is likely the safest and most efficacious medication for this patient. Benzodiazepines (such as alprazolam) and diphenhydramine have a strong potential for worsening delirium symptoms in an older population and thus should be avoided in this patient. Haloperidol is absolutely contraindicated in DLB because of the risk of significant worsening of the dementia syndrome. In addition, the patient is not aggressive at this point and is unlikely to harm himself or others, so antipsychotic agents are unnecessary, as are medications with a strong sedating effect (such as diphenhydramine).
How to diagnose PE in pregnant patient?
V/Q scan 1st!!!! CT angiogram should follow if VQ inconclusive
A 22-year-old woman is evaluated for tachycardia, fever, agitation, and confusion 3 days following laparoscopic cholecystectomy for acute cholecystitis. Medical history is significant for type 1 diabetes mellitus well controlled on basal-bolus insulin and Graves disease previously treated with methimazole. Methimazole was discontinued 6 months ago when she was considered to be in remission. Medications are insulin glargine and insulin aspart. On physical examination, temperature is 39.0 °C (102.2 °F), blood pressure is 95/50 mm Hg, pulse rate is 132/min and irregular, and respiration rate is 24/min. The patient is confused and appears flushed and diaphoretic. Lid lag is noted. The thyroid gland is diffusely enlarged with an audible bruit. Her deep tendon reflexes are brisk. There is a fine tremor of her hands. Her examination is otherwise normal. Which of the following is the most likely diagnosis? Adrenal crisis Malignant hyperthermia Myxedema coma Thyroid storm
Thyroid storm is a severe manifestation of thyrotoxicosis with life-threatening secondary systemic decompensation; it occurs most commonly with underlying Graves disease coupled with a precipitating factor such as surgery. The most likely diagnosis is thyroid storm. Thyroid storm is characterized by severe thyrotoxicosis associated with systemic decompensation. Presentation often follows a precipitating event, such as non-thyroid surgery as in this patient's case. Clinical manifestations include high fever, altered mental status and seizures, tachycardia, atrial fibrillation and heart failure, and hepatic dysfunction. A diagnostic system, such as the Burch and Wartofsky Point Scale, can support the diagnosis with scores greater than or equal to 45 being highly suggestive. The patient's thyroid function tests should be assessed to confirm recurrent hyperthyroidism and aggressive treatment should be initiated in the ICU. Management includes treatment of any precipitant illness, supportive care, and thyrotoxicosis-directed therapy including β-adrenergic blockers (esmolol infusion), antithyroid drug therapy, intravenous glucocorticoids, and potassium iodide. Plasmapheresis and emergent thyroidectomy are utilized in patients who cannot be sufficiently managed with medical therapy alone. Adrenal crisis is not the most likely diagnosis. Although autoimmune primary adrenal failure occurs more commonly in patients with other autoimmune disorders, patients with adrenal crisis usually present with hypotension, hyponatremia, and hyperkalemia, in addition to gastrointestinal manifestations. This diagnosis cannot explain the patient's hyperthermia, lid lag, thyromegaly, brisk reflexes, or tremor. Malignant hyperthermia is an uncommon cause of severe hyperthermia that occurs in genetically susceptible individuals upon exposure to a volatile anesthetic such as halothane or isoflurane. Symptoms begin intraoperatively or in postoperative recovery, not 3 days following surgery. Features include mixed respiratory and metabolic acidosis, muscle rigidity, hyperkalemia, and rhabdomyolysis. This diagnosis does not explain the patient's thyrotoxicosis-related findings. Finally, the patient lacks muscle rigidity, a pathognomonic finding in malignant hyperthermia. Myxedema coma is also unlikely. Although patients with myxedema coma may experience cardiac dysfunction and mental status changes, this patient is presenting with signs and symptoms of thyroid hormone excess, not deficiency. She has a history of Graves disease in remission after treatment with antithyroid drug therapy. She has not received radioactive iodine or thyroidectomy and thus the development of hypothyroidism at this point would be unusual.
A 40-year-old man is evaluated during a routine examination. He has a history of cardiac murmur. He is asymptomatic and active, cycling 12 miles three or four times per week. On physical examination, vital signs are normal. There is no jugular venous distention. Cardiovascular examination reveals a point of maximal impulse that is displaced laterally and is diffuse. A grade 3/6 early diastolic decrescendo murmur is heard at the left parasternal third intercostal space. Lungs are clear. There are brisk, prominent distal extremity pulses. A transthoracic echocardiogram demonstrates a bicuspid aortic valve and severe aortic regurgitation. Left ventricular end-systolic dimension is enlarged at 58 mm. Left ventricular ejection fraction is 50%. The proximal ascending aortic diameter is 50 mm (normal, <35 mm). Which of the following is the most appropriate management? Aortic valve and root replacement Clinical and echocardiographic surveillance Initiation of nifedipine therapy Treadmill stress echocardiography
Valve replacement surgery is recommended for asymptomatic patients with a bicuspid aortic valve and severe aortic regurgitation when the left ventricular end-systolic diameter reaches 50 mm or the left ventricular ejection fraction is less than 50%. The most appropriate management is aortic valve and root replacement. This young, asymptomatic patient with a bicuspid aortic valve has significant aortic regurgitation. The transthoracic echocardiogram demonstrates a dilated left ventricle, mildly decreased systolic function, and a moderately dilated ascending aorta. Asymptomatic patients with severe aortic regurgitation who develop left ventricular dilatation or systolic dysfunction are at increased risk for sudden cardiac death, death, and heart failure. Therefore, these patients should be referred for aortic valve replacement. The prognosis of asymptomatic patients with severe aortic regurgitation but with normal left ventricular size and function is excellent. The American College of Cardiology/American Heart Association valvular heart disease guidelines recommend surgery in asymptomatic patients with severe aortic regurgitation when the left ventricular end-systolic dimension reaches 50 mm or the ejection fraction is less than 50%. In patients undergoing cardiac surgery, repair of the ascending aorta is indicated when the diameter exceeds 45 mm; thus, concomitant aortic root replacement, in addition to aortic valve replacement, is indicated in this patient. In asymptomatic patients who do not meet surgical criteria, serial echocardiography is warranted to assess for disease progression. Patients with mild regurgitation should receive an annual clinical evaluation and echocardiography every 3 to 5 years, whereas patients with severe regurgitation should undergo clinical evaluation and echocardiography every 6 to 12 months. For asymptomatic patients with normal systolic function and mild regurgitation, vasodilator therapy with nifedipine or ACE inhibitors is not recommended. For asymptomatic patients with normal systolic function and severe regurgitation, vasodilator therapy may be considered, but medical therapy does not supplant surgical intervention in these patients. In patients with significant aortic regurgitation and equivocal symptoms, exercise-induced increases in pulmonary systolic pressure to more than 60 mm Hg (or a 25-mm Hg increase above baseline) identified during treadmill stress echocardiography suggest hemodynamic significance and may lead to earlier referral for surgical intervention. In this patient, however, ventricular dilatation and decreased ejection fraction are adequate criteria for recommending surgery.
A 35-year-old man is evaluated for a 3-year history of epilepsy. Seizures typically occur twice monthly, last 2 minutes, and are characterized by staring, lip smacking, and confusion; approximately once every 6 months, the patient experiences a whole-body convulsion marked by incontinence and prolonged confusion for several hours. Treatment with oxcarbazepine and lamotrigine, although initially reducing seizure frequency, has been largely ineffective. He no longer drives or works because of the seizures. He also has migraines, which are well controlled by sumatriptan. On physical examination, vital signs are normal. All other physical examination findings, including those from a neurologic examination, are unremarkable. Results of routine outpatient electroencephalography (EEG) are normal. An MRI of the brain shows right hippocampal atrophy. Which of the following is the most appropriate next step in management? Levetiracetam Topiramate Vagus nerve stimulation Video EEG monitoring
Video electroencephalography is first step in determining candidacy for epilepsy surgery in patients with medically intractable epilepsy. This patient is having ongoing focal seizures with altered awareness (formerly known as complex partial seizures) because of temporal lobe epilepsy despite taking two antiepileptic drugs and should be referred to an epilepsy center for monitoring by video electroencephalography (EEG). Because his epilepsy is medically intractable, he may be a candidate for epilepsy surgery. Video EEG is the first step in determining candidacy for surgery by confirming that the seizures seen on video EEG match the location of abnormal findings on MRI. Temporal lobectomy leads to seizure freedom in 60% to 70% of patients with temporal lobe epilepsy not helped by medication and is the best option for treating this patient. Although levetiracetam and topiramate are reasonable options for treating this seizure type, and topiramate has the added benefit of migraine prophylaxis, the chance of seizure freedom from an additional drug is only approximately 5% to 10%. Additionally, this patient's migraines are already well controlled, so a prophylactic agent is not required. Use of a vagus nerve stimulator is a palliative measure, is unlikely to result in freedom from seizures, and should be offered only if resection is not an option.
A 26-year-old woman is evaluated for the new onset of jaundice. She reports no fever or abdominal pain. Her medical history is unremarkable and she takes no medication. On physical examination, vital signs are normal. Scleral icterus and psychomotor slowing are noted. Laboratory studies: Hemoglobin 10.2 g/dL (102 g/L) Reticulocyte count 8% of total erythrocyte count Alkaline phosphatase 26 U/L Alanine aminotransferase 78 U/L Aspartate aminotransferase 156 U/L Total bilirubin 6.4 mg/dL (109.4 µmol/L) Conjugated bilirubin 2.6 mg/dL (44.5 µmol/L) Testing for hepatitis A, B, and C viral infections is negative for acute infection. On abdominal ultrasonography, the liver is small and nodular. Splenomegaly is noted. Which of the following is the most appropriate test to perform next? Antimitochondrial antibody test Hepatitis B virus DNA measurement Serum ceruloplasmin measurement Transferrin saturation measurement
Wilson disease should be considered in all patients younger than age 40 years who have unexplained liver disease. Serum ceruloplasmin measurement is the most appropriate test for this patient. Wilson disease is an autosomal recessive disorder of copper excretion with subsequent accumulation of copper in the liver and central nervous system, cornea, kidney, joints, and cardiac muscle resulting in organ dysfunction. Children aged 10 years or younger with Wilson disease tend to present with acute liver failure; older patients (aged 30 years or younger) present with chronic liver disease and/or neurologic manifestations. Wilson disease should be considered in all patients younger than age 40 years who have unexplained liver disease. When Wilson disease causes acute hepatitis, usually in young patients, the sudden release of copper from liver cells can also induce hemolytic anemia. In this patient with evidence of hepatic encephalopathy, hemolytic anemia, low alkaline phosphatase level, and unconjugated bilirubinemia, the diagnosis of Wilson disease should be considered. The serum ceruloplasmin level is used to test for Wilson disease. Slit-lamp examination of the cornea can be performed to evaluate for Kayser-Fleischer rings, which most commonly occur in patients with neurologic manifestations of Wilson disease. A 24-hour urine copper assessment can be performed to confirm an excessive amount of copper excretion. The diagnosis of primary biliary cholangitis is generally made on the basis of a cholestatic liver enzyme profile in the setting of a positive antimitochondrial antibody test. The predominant liver enzyme abnormality is an increase in serum alkaline phosphatase level. This patient's biochemical profile reflects hepatocellular injury, not cholestasis, and an antimitochondrial antibody test is not indicated. With a negative test for hepatitis B virus surface antigen, this patient is very unlikely to have a hepatitis B viral infection and measurement of a hepatitis B virus DNA level is not indicated. Symptomatic hereditary hemochromatosis is usually diagnosed after age 40 years and is often diagnosed later in women than in men, due to iron losses associated with menstruation, pregnancy, and lactation. The most sensitive initial diagnostic study is measurement of the serum transferrin saturation. The patient's sex, young age, and presence of a hemolytic anemia make the diagnosis of hereditary hemochromatosis unlikely and the measurement of a transferrin level unnecessary.
Common complications after heart transplant?
-The most frequent complication within the first year after transplant is infection. Cytomegalovirus (CMV) infection is common, and patients at moderate risk (CMV-positive donor/CMV-positive recipient) and high risk (CMV-positive donor/CMV-negative recipient) often receive antiviral prophylaxis for 6 months. Incidence of rejection is highest in the first 6 months after transplantation. Because most patients with rejection are asymptomatic, regularly scheduled endomyocardial biopsies are often performed to detect rejection for the first few years after transplant. Severe rejection is characterized by signs of acute heart failure and atrial arrhythmias (typically atrial flutter) or conduction abnormalities. Early complications related to immunosuppressive therapy include hypertension (more than 90% of patients) and diabetes (15%-20% of patients). Long-term complications after transplantation include CAD and an increased incidence of malignancies, including skin cancer (common) and B-cell lymphoma related to immunosuppressive therapy (less common). -Coronary artery disease is a common long-term complication of cardiac transplantation, and by 5 years after transplantation, vasculopathy is present in almost 50% of patients.
A 35-year-old man is evaluated during a follow-up consultation. He was diagnosed with HIV 6 months ago; at that time, his CD4 cell count was 30/µL. He immediately began antiretroviral therapy and has been adherent to his regimen. His initial screening tests included a nonreactive tuberculin skin test (TST). He states that he has been unemployed for more than 2 years and has lived in a variety of different homeless shelters. Medications are tenofovir-emtricitabine and raltegravir. On physical examination, vital signs and other findings are unremarkable. Today, a repeat CD4 cell count is 100/µL and a repeat TST results in 5 mm of induration. A posteroanterior and lateral chest radiograph is unremarkable. Which of the following is the most appropriate management? Initiate isoniazid plus pyridoxine Initiate isoniazid, rifampin, pyrazinamide, and ethambutol Obtain an interferon-γ release assay Repeat TST in 2 weeks
A 5-mm induration on tuberculin skin testing is considered positive in persons who are immunocompromised, including those with HIV; if no other signs of tuberculosis infection are present, treatment for latent tuberculosis infection should be initiated with isoniazid.
A 35-year-old man is evaluated during a follow-up consultation. He was diagnosed with HIV 6 months ago; at that time, his CD4 cell count was 30/µL. He immediately began antiretroviral therapy and has been adherent to his regimen. His initial screening tests included a nonreactive tuberculin skin test (TST). He states that he has been unemployed for more than 2 years and has lived in a variety of different homeless shelters. Medications are tenofovir-emtricitabine and raltegravir. On physical examination, vital signs and other findings are unremarkable. Today, a repeat CD4 cell count is 100/µL and a repeat TST results in 5 mm of induration. A posteroanterior and lateral chest radiograph is unremarkable. Which of the following is the most appropriate management? Initiate isoniazid plus pyridoxine Initiate isoniazid, rifampin, pyrazinamide, and ethambutol Obtain an interferon-γ release assay Repeat TST in 2 weeks
A 5-mm induration on tuberculin skin testing is considered positive in persons who are immunocompromised, including those with HIV; if no other signs of tuberculosis infection are present, treatment for latent tuberculosis infection should be initiated with isoniazid. The most appropriate management for this patient is daily isoniazid plus pyridoxine. He has HIV, and a tuberculin skin test (TST) produces a 5-mm induration, which is considered positive in patients who are immunocompromised. Because the chest radiograph was negative, he should be treated as having latent tuberculosis infection (LTBI) with 9 months of isoniazid therapy. The 2020 CDC guideline for the treatment of latent tuberculosis offers 6 and 9 months of isoniazid for patients with and without HIV infection. The CDC recognizes 3 months of isoniazid plus rifapentine given once weekly or 3 months of isoniazid plus rifampin given daily as preferred regimens because of drug tolerability, efficacy, and high completion rates. Isoniazid given daily for 6 or 9 months is now considered an alternative therapy for patients unable to take a preferred regimen. Pyridoxine is added to isoniazid treatment in patients at risk for peripheral neuropathy (such as patients with HIV, diabetes, uremia, alcoholism, malnutrition, and pregnancy). It cannot be determined if the patient's tuberculosis infection is newly acquired or long standing because the previous negative TST was performed when his CD4 cell count was extremely low. Because this patient has no evidence of active infection, four-drug antituberculous therapy is not necessary. Routine use of a TST and interferon-γ release assay (IGRA) is not recommended. In certain circumstances, however, using a second test when the result of the initial test is negative might be helpful when the risk of infection or progression is increased or risk for poor outcome exists, such as in children younger than 5 years who have been exposed to a patient with active tuberculosis or in patients with HIV infection. Use of a second test for diagnosing infection when the result of the first test is negative can also be considered when the suspicion of tuberculosis is strong based on clinical presentation or radiographic imaging. Conversely, using both tests when the result of the initial test is positive might be helpful when a suspected false-positive result is obtained in a person at low risk for infection and progression to active disease. This patient is at high risk for tuberculosis and has a positive TST; therefore, additional testing with IGRA is not necessary. Repeating the TST would not provide additional information. The Centers for Disease Control and Prevention recommends initiating antituberculous prophylaxis in patients who are HIV positive who have a TST induration of 5 mm or greater.
A 32-year-old woman is evaluated for a 15-year history of low back pain. The pain is worse with rest, improves with movement, and can awaken her during the night. Family history is notable for three paternal uncles with back problems. She takes naproxen twice daily with some relief. On physical examination, vital signs are normal. Joint examination does not reveal any warmth, erythema, or swelling. Tenderness over the sacroiliac joints bilaterally and reduction in the range of motion of the lumbar spine are noted. Laboratory studies are notable for an erythrocyte sedimentation rate of 27 mm/h. A plain anteroposterior radiograph of the pelvis shows fusion of the sacroiliac joints. ANCA Anti-cyclic citrullinated peptide antibodies Antinuclear antibodies HLA-B27 antigen No additional testing
A diagnosis of ankylosing spondylitis can be made in a patient younger than age 45 years with symptoms of inflammatory back pain for 3 months or more and bilateral sacroiliitis on imaging. No additional tests are necessary. Different criteria have been proposed for the diagnosis of ankylosing spondylitis. Common requirements include the presence of inflammatory back pain for 3 or more months in a person younger than age 45 years, limited lumbar spine motion, elevated inflammatory markers, and evidence of bilateral sacroiliitis on imaging. The patient has a long history of inflammatory back pain (improves with exercise, worsens with sleep or inactivity), loss of range of motion of the lumbar spine, and radiographs showing fusion of the sacroiliac joints, one of the typical features of ankylosing spondylitis. It is important to establish the diagnosis of ankylosing spondylitis to assess the risk of further joint fusion and deformity in this patient. ANCA is present in some forms of medium-vessel vasculitis. In the absence of cutaneous or internal organ manifestations suggestive of vasculitis (sinus, pulmonary, kidney, cutaneous, and ophthalmologic abnormalities), it is not appropriate as the next step in this patient's evaluation. Anti-cyclic citrullinated peptide antibodies have specificity for the diagnosis of rheumatoid arthritis and are appropriately ordered when patients present with bilaterally symmetric inflammatory arthritis of the small joints of the hands. These antibodies will not establish the diagnosis of ankylosing spondylitis. Testing for antinuclear antibodies should be carried out when there is suspicion for the presence of systemic lupus erythematosus. This patient does not have rash, sun sensitivity, alopecia, oral ulcers, pleuropericarditis, kidney disease, or laboratory abnormalities that would raise suspicion for lupus. The presence of HLA-B27 antigen is not a diagnostic criterion for ankylosing spondylitis. However, such testing may be particularly helpful in patients with inflammatory back pain and other manifestations of ankylosing spondylitis but without evidence of sacroiliitis on imaging.
A 53-year-old man is evaluated for tea-colored urine. He reports no other symptoms. He was hospitalized 2 weeks ago with melena. Upper gastrointestinal endoscopy and colonoscopy at the time did not show a bleeding source, but he stabilized after the transfusion of one unit of blood. He was discharged 10 days ago and has returned to work. Medical history is otherwise significant for trauma sustained in a motor vehicle accident 5 years ago, requiring multiple surgeries. He takes no medications. On physical examination, temperature is 37.8 °C (100.1 °F), and other vital signs are normal, with no postural blood pressure or pulse changes. Scleral icterus is noted. The stool guaiac test result is negative. The remainder of the examination is unremarkable. Laboratory studies: Current 10 Days Ago Hemoglobin 6.9 g/dL (69 g/L) 8 g/dL (80 g/L) Bilirubin Total 5 mg/dL (34.2 µmol/L) 1.1 mg/dL (18.8 µmol/L) Direct 0.7 mg/dL (12 µmol/L) — Urinalysis 4+ blood, 2-3 erythrocytes/hpf, 1 leukocyte/hpf — Which of the following is the most appropriate next step in the management of this patient? Direct antiglobulin (Coombs) test Flow cytometry Repeat upper and lower endoscopy Transfusion of one unit of blood
A direct antiglobulin (Coombs) test (DAT) should be ordered for this patient who is likely experiencing a delayed hemolytic transfusion reaction (DHTR), which is an anamnestic antibody response to previous erythrocyte antigen (non-ABO) sensitization that typically occurs 7 to 14 days after the index transfusion. The primary alloimmunization event is usually a remote transfusion or pregnancy. At the time of the index transfusion, the antibody level is usually lower than the detectable threshold determined by routine blood bank screening. A DHTR is associated with jaundice, low-grade fever, and an otherwise unexplained decrease in the hemoglobin concentration; it is sometimes accompanied by hemoglobinuria, which is confirmed by urinalysis and this patient's presenting symptom of tea-colored urine. Laboratory testing in these patients shows indirect hyperbilirubinemia. The DAT will be positive, whereas the indirect antiglobulin test, which is invariably negative before transfusion, may remain negative or show an unexpected alloantibody. For patients known to have received a recent transfusion, obtaining a transfusion history from the outside blood bank may be helpful. Flow cytometry will assist in the diagnosis of paroxysmal nocturnal hemoglobinuria (PNH), an acquired clonal stem cell disorder that should be considered in patients presenting with hemolytic anemia, pancytopenia, or unprovoked atypical thrombosis. Hemolytic anemia occurring within days of a transfusion in a previously sensitized individual is more likely to represent DHTR than PNH. Patients thought to have obscure gastrointestinal bleeding should be considered for repeat upper endoscopy, colonoscopy, or both. Up to 50% of lesions can be identified using this strategy. However, this patient's decrease in hemoglobin level, hemoglobinuria, and elevated total bilirubin level suggest hemolysis, not gastrointestinal bleeding. Patients who are well compensated for their degree of anemia do not require immediate transfusion.
A 37-year-old man is evaluated in the hospital for polyuria 1 day after transsphenoidal pituitary surgery for a craniopharyngioma. The patient reports increased thirst overnight. Urine output is currently 300 mL/hour for the last 12 hours. He takes no medications. On physical examination, vital signs are normal. He has dry mucous membranes. Laboratory studies show a sodium level of 146 mEq/L (146 mmol/L). Which of the following is the most appropriate diagnostic test to perform next? Desmopressin challenge Urine and serum osmolality Urine electrolytes Water deprivation test
A low urine osmolality in the setting of a high serum osmolality and high serum sodium in a patient with polyuria is diagnostic of diabetes insipidus. The most appropriate diagnostic test for this patient is urine and serum osmolality measurement. Central diabetes insipidus (DI) results from inadequate production of antidiuretic hormone (ADH) by the posterior pituitary gland. In the presence of ADH, aquaporin water channels are inserted in the collecting tubules and allow water to be reabsorbed. In the absence of ADH, excessive water is excreted by the kidneys. Excretion of more than 3 liters of urine per day is considered polyuric. Frank hypernatremia is unusual because patients develop extreme thirst and polydipsia, and with free access to water, can maintain serum sodium in the high normal range. When patients do not drink enough to replace the water lost in the urine, due to poor or absent thirst drive or lack of free access to water, they develop hypernatremia. A low urine osmolality in the setting of a high serum osmolality and high serum sodium in a patient with polyuria is diagnostic of DI. Patients with craniopharyngiomas are at higher risk of developing central DI. Desmopressin challenge is not appropriate at this time. Prior to initiating desmopressin, one must confirm the diagnosis of DI by measuring urine and serum osmolality. Once confirmed, desmopressin can be administered and urine osmolality remeasured to assure that administration of desmopressin resulted in an increase in urine osmolality. After desmopressin is given, urine concentrates to more than 800 mOsm/kg in central DI, less than 300 mOsm/kg in nephrogenic DI, and between 300 and 800 mOsm/kg in partial DI. Measurement of urine electrolytes is not helpful in the diagnosis of DI. A water deprivation test should not be performed in this patient. He already has hypernatremia so further water deprivation could result in serious hypernatremia. In the setting of hypernatremia, a urine and serum osmolality are the best tests to confirm the diagnosis of DI. A water deprivation test can be pursued when the diagnosis is unclear. Urine osmolality above 800 mOsm/kg H2O is a normal response to water deprivation, indicating ADH production and peripheral effect are intact. A serum osmolality above 295 mOsm/kg with inappropriately hypotonic urine (urine osmolality-serum osmolality ratio <2) during fluid deprivation confirms DI.
A 30-year-old woman is evaluated for a dark spot on the lower lip for several months' duration. Medical history is unremarkable, and she takes no medications. On physical examination, vital signs are normal. There is 2 × 3-mm perfectly round, well-circumscribed brown-to-black macule on the lower mucosal lip. Oral mucosa is otherwise normal. Which of the following is the most likely diagnosis? Actinic cheilitis Amalgam tattoo Melanoma in-situ Melanotic macule
Melanotic macules are well-circumscribed, brown-to-black macules that most commonly occur on the lower lip, although they may be seen on the gingiva, buccal mucosa, or tongue. Melanotic macule or mucosal lentigo is a small, well-circumscribed, brown-to-black macule. The most common location is the lower mucosal lip, but they can occur on any mucosal surface. They are the mucosal counterpart of a lentigo on the skin. They are typically solitary, but multiple macules can occur. Numerous mucosal lentigines are associated with various syndromes such as Peutz-Jeghers and Laugier-Hunziker syndromes. Peutz-Jeghers syndrome is an autosomal dominant hamartomatous polyposis syndrome characterized by hamartomatous polyps in the gastrointestinal tract, mucocutaneous pigmentation, and an increased risk of cancer. Laugier-Hunziker syndrome is an acquired, benign disorder characterized by lentigines on the lips and buccal mucosa. It not associated with any systemic disorder. Actinic cheilitis appears as chronic red-to-tan scaly patches with erosions and characteristically involves the lower lip. Actinic cheilitis is a premalignant condition and is considered the precursor to squamous cell carcinoma in situ. Treatment with topical chemotherapy agents (5-fluorouracil), imiquimod, laser therapy, photodynamic therapy, or cryotherapy is recommended. Actinic cheilitis differs from angular cheilitis, which is inflammation involving one or both corners of the mouth and often related to a bacterial or fungal infection. Amalgam tattoos are the most common source of localized pigmentation on the buccal mucosa. Amalgam dental fillings contain metals that can become implanted into the adjacent mucosa at the time of application. They appear as blue-gray macules and do not change over time. The typical location for amalgam tattoos is the buccal mucosa, not the lip. Diagnosis can be made clinically, or confirmation can be made with biopsy and pathology. No treatment is necessary. Although it can arise anywhere in the skin or mucous membranes, malignant melanoma most commonly occurs in sun-exposed areas. The back is the most common location in men, and the legs are the most common location in women. The clinical features that suggest melanoma are asymmetry, irregular border, multiple colors, diameter greater than 6 mm, and changing size. The more of these features a pigmented lesion possesses, the more worrisome it is for malignant melanoma or melanoma in-situ, and these entities must be excluded with an excisional biopsy. This patient's lesion is small, round, well-circumscribed and has no worrisome features of a melanoma.
A 75-year-old woman is evaluated for a 2-week history of gradually increasing pain in both shoulders and hips; the pain radiates down both arms to the elbows and down both hamstrings to the knees. She reports no headache, jaw claudication, or vision changes. She was diagnosed with polymyalgia rheumatica 3 months ago. She started prednisone, 15 mg/d, with immediate and complete relief of symptoms; prednisone was weaned from 15 to 10 mg/d 2 months ago, then to 8 mg/d (current dose) 1 month ago. She remained asymptomatic until 2 weeks ago. She says that her current symptoms are just as bad as when she was first diagnosed. On physical examination, vital signs are normal; blood pressure is identical in both arms. There is no temporal tenderness or induration. Painful range of motion of both shoulders and hips is noted. Which of the following is the most appropriate management? Prednisone, 10 mg/d Prednisone, 30 mg/d Prednisone, 60 mg/d Prednisone, 20 mg/d, and methotrexate
A relapse of polymyalgia rheumatica should be treated with an increase in prednisone to the last pre-relapse dose at which the patient was doing well, followed by a gradual reduction within 4 to 8 weeks back to the relapse dose. Prednisone, 10 mg/d, is the most appropriate treatment for this patient with a relapse of polymyalgia rheumatica (PMR) after her prednisone dose was weaned to 8 mg/d. Patients with PMR experience symmetric pain and stiffness in the shoulder, neck, and hip regions, typically without synovitis. PMR is a clinical diagnosis based on the characteristic symptoms in a patient older than 50 years and is supported by an elevated erythrocyte sedimentation rate and/or C-reactive protein. Recent guidelines for the management of PMR (developed by a collaborative effort of the American College of Rheumatology and the European League Against Rheumatism) recommend increasing the prednisone to the last pre-relapse dose at which the patient was doing well, followed by a gradual reduction within 4 to 8 weeks back to the relapse dose. This patient was asymptomatic on 10 mg/d, prior to reducing to her current relapse dose; therefore, an increase to 10 mg/d is appropriate. After the flare-up subsides, a slow taper of 1 mg every 4 weeks may be better tolerated then larger increments over 2- to 4-week periods. Prednisone, 30 mg/d, would also help treat the flare-up, but this higher dose is probably unnecessary, and a lower dose should be attempted first. Prednisone, 60 mg/d, is not a typical dose used for PMR, and in the absence of concomitant giant cell arteritis is not needed to treat a PMR flare-up. Methotrexate can be added as a glucocorticoid-sparing agent for patients who cannot be successfully weaned off prednisone or who are experiencing significant glucocorticoid toxicity; this patient has only had one flare-up and may do well after a modest increase followed by a more gradual taper of prednisone, making the addition of methotrexate premature at this time.
A 25-year-old woman returns for counseling regarding results of HIV testing completed during a recent routine health maintenance visit. She reports no known exposure or risk factors for HIV infection. Medical history is unremarkable, and she takes no medications. Laboratory studies show a reactive HIV-1/2 antigen/antibody combination immunoassay, a negative HIV-1/2 antibody differentiation immunoassay, and no RNA detected on HIV-1 RNA nucleic acid amplification testing. Which of the following is the most appropriate management? Initiate combination antiretroviral therapy after counseling Order T-cell subset testing to check CD4 cell count Perform HIV genotypic resistance testing Perform Western blot HIV antibody testing Tell the patient she does not have HIV infection
A screening HIV test result that is positive on the initial antigen/antibody combination immunoassay but negative on the antibody differentiation immunoassay and nucleic acid amplification testing for HIV RNA represents a false-positive result. This woman had a false-positive result for HIV and should be reassured she does not have HIV infection. She has no symptoms to suggest acute infection. The initial HIV combination immunoassay tests for HIV-1 or HIV-2 antibody and p24 antigen. If reactive, an HIV-1/2 antibody differentiation immunoassay is performed, which differentiates between HIV-1 and HIV-2 antibodies. If the antibody differentiation assay is reactive for HIV-1 antibody, then HIV-1 infection is confirmed. If the antibody differentiation assay is negative, then testing for HIV RNA by nucleic acid amplification is performed. A negative antibody differentiation assay with a positive HIV RNA test would indicate acute HIV infection (in the "window" period after infection but before antibody development). But if the HIV RNA assay is also negative (no HIV RNA detected), then no evidence for HIV infection exists, and the initial test was a false positive. Although false-positive findings are rare, in a population with low pretest probability (such as in screening), false-positive results on the initial antigen/antibody combination immunoassay may be seen with higher frequency than true-positive results; waiting for the confirmatory testing results is crucial to avoid misdiagnosis and unnecessary additional testing and treatment. Because the patient does not have HIV infection, it would be inappropriate to begin antiretroviral therapy. Likewise, baseline HIV resistance testing is not indicated, nor would it be possible because the patient has no detectable HIV RNA to be genotyped. It is important to assess the CD4 cell count for all persons newly diagnosed with HIV infection so the level of immunocompromise and consequent risk for opportunistic infections can be determined and prophylaxis can be started, if indicated. But this patient does not have HIV infection. Testing CD4 cell count as a surrogate for HIV infection is inappropriate because the CD4 level is neither sensitive nor specific for HIV infection. Western blot testing for HIV is no longer performed as part of the laboratory protocol for diagnosing HIV infection because of problems with sensitivity in acute infection and with interpretation of indeterminate results.
A 30-year-old man is evaluated after being hospitalized for an acute flare of extensive ulcerative colitis. He reports six to eight bloody bowel movements daily with prominent urgency and lower abdominal cramping for the past 2 weeks. He has been taking prednisone daily for 1 week. His only other medication is mesalamine. On physical examination, vital signs are normal. Abdominal examination reveals lower abdominal tenderness. The abdomen is not distended and bowel sounds are normal. Blood is seen on digital rectal examination. Hemoglobin level is 10 g/dL (100 g/L). He remains hemodynamically stable. Which of the following is the most appropriate venous thromboembolism prophylaxis for this patient? Aspirin Graduated compression stockings Heparin Intermittent pneumatic compression Low-dose warfarin
All hospitalized patients with inflammatory bowel disease should be given pharmacologic venous thromboembolism prophylaxis with subcutaneous heparin. Subcutaneous heparin is the most appropriate venous thromboembolism (VTE) prophylaxis for this patient. VTE is a common clinical problem and is associated with substantial morbidity and mortality among hospitalized patients. Most medical patients have one or more risk factors for VTE. More than a quarter of patients with undiagnosed and untreated pulmonary embolism (PE) will have a subsequent fatal PE, and between 5% and 10% of all in-hospital deaths are a direct result of PE. VTE is one of the most common extraintestinal manifestations of inflammatory bowel disease (IBD); patients with IBD have a three-fold risk for VTE compared to patients without IBD. VTE represents a significant cause of morbidity and mortality in patients with IBD, and risk for VTE is highest at the time of disease flare. Although the incidence of VTE increases with age, the highest relative risk for VTE in IBD is observed in patients younger than 40 years old. Patients with IBD who develop VTE have an increased mortality rate, and the most important risk factor for development of VTE is active IBD. For these reasons, prevention of VTE in patients with IBD is essential. All hospitalized patients with IBD should be given pharmacologic VTE prophylaxis with subcutaneous heparin. Systematic reviews of trials comparing prophylactic low-molecular-weight heparin with unfractionated heparin have not shown a statistically significant difference for mortality or major bleeding events, although there was a nonsignificant trend favoring low-molecular-weight heparin in the prevention of PE. Mechanical VTE prophylaxis with graduated compression stockings or intermittent pneumatic compression devices is not recommended either with or in place of pharmacologic prophylaxis, although intermittent pneumatic compression has shown some efficacy in surgical patients and may be an option for nonsurgical patients with a contraindication to pharmacologic therapy, such as severe gastrointestinal bleeding. Compression stocking are no more effective than placebo in preventing VTE and are associated with harm in the form of increased incidence of skin breakdown. Aspirin and low-dose warfarin have no role in VTE prophylaxis for hospitalized medical patients or patients with IBD.
Essential thrombocythemia tx?
ASA + Hydroxyurea The patient has essential thrombocythemia (ET) and should be treated with hydroxyurea. The first step in evaluating thrombocytosis is to rule out a secondary elevation. Chronic infection, collagen vascular disease, malignancy, ongoing bleeding, and iron deficiency can all cause a reactive thrombocytosis. JAK2 mutation testing is positive in only about 60% of patients with ET, but other clonal mutations involving calreticulin and MPL genes have been identified in some patients with JAK2-negative ET. Approximately 10% to 15% of patients with ET have negative results for all three mutations (termed triple-negative patients). A bone marrow biopsy specimen showing hypercellularity with increased numbers of enlarged megakaryocytes is required to make the diagnosis in the few patients with ET who do not have any of the three mutations listed previously. The most significant complications that may occur during follow-up of ET are thrombosis (10%-15% 15-year cumulative risk; arterial is more common than venous), hemorrhage (8.6% 15-year cumulative risk), progression to myelofibrosis (10% 15-year cumulative risk), and transformation to acute leukemia (3% 15-year cumulative risk). The patient's age (older than 60 years) and previous thromboembolic event place her at particularly high risk for subsequent thromboembolic complications. Platelet-lowering therapy is indicated for any patient at high risk. Hydroxyurea is well tolerated in older adult patients and is considered first-line therapy for nonpregnant patients. For that reason, she should be treated with aspirin and hydroxyurea. Patients with high-risk ET, such as this patient who has a history of stroke, require antiplatelet therapy and platelet-lowering therapy with agents such as hydroxyurea. Changing antiplatelet therapy from aspirin to clopidogrel would not address this patient's need for platelet-lowering therapy nor would adding a second antiplatelet medication, such as clopidogrel, to the aspirin. Plateletpheresis may be indicated in the emergent management of patients with ET who have markedly elevated platelet counts, usually greater than 1,000,000/µL (1000 × 109/L), and acute hemorrhage. Plateletpheresis would not be indicated in this patient.
A 78-year-old woman is evaluated in the emergency department for severe pain in the left hip after a fall. History is significant for end-stage kidney disease as of 18 months ago, hypertension, and peripheral vascular disease. Medications are lisinopril, amlodipine, sevelamer, and epoetin alfa. She is also receiving morphine for the hip pain. On physical examination, blood pressure is 132/70 mm Hg, and pulse rate is 72/min; other vital signs are normal. The left lower extremity is externally rotated at the hip. Peripheral pulses are diminished. The remainder of the physical examination is noncontributory. Laboratory studies: Alkaline phosphatase 78 U/L Calcium 9.7 mg/dL (2.4 mmol/L) Phosphorus 4.2 mg/dL (1.4 mmol/L) Parathyroid hormone 62 pg/mL (62 ng/L) 25-Hydroxyvitamin D 32 ng/mL (80 nmol/L) Radiographs of the hips show a left hip fracture and calcified arteries. Which of the following is the most likely diagnosis for the underlying bone disease? Adynamic bone disease β2-Microglobulin-associated amyloidosis Osteitis fibrosis cystica Osteomalacia
Adynamic bone disease can occur in patients with chronic kidney disease or those on dialysis and is associated with fracture or bone pain; parathyroid hormone and alkaline phosphatase levels are typically normal. The most likely bone pathology is adynamic bone disease in this patient with end-stage kidney disease and normal serum calcium and phosphorus and relatively suppressed parathyroid hormone (PTH) and alkaline phosphatase levels. Adynamic bone disease can occur in patients with chronic kidney disease (CKD) or those on dialysis. It is typically associated with significant vascular calcifications. The gold standard for the diagnosis of adynamic bone disease is bone biopsy; however, this is rarely performed. Adynamic bone disease has no specific markers, but a constellation of findings may suggest this diagnosis. Patients with adynamic bone disease may present with fracture or bone pain. The latter has been attributed to the inability to repair microdamage because of low turnover. Serum calcium may be normal or elevated because the bone is unable to take up calcium. High PTH and alkaline phosphatase would exclude adynamic bone disease; in this disorder, both are typically normal. Treatment is targeted at factors that allow PTH secretion to rise. This includes avoiding calcium-based binders, conservative use of vitamin D, and decreasing the dialysate calcium concentration. It is important to note that, as with the general population, patients with CKD may also develop osteoporosis, particularly if they received glucocorticoid therapy for the primary kidney disorder or for immunosuppression in the setting of a kidney transplant. β2-Microglobulin-associated amyloidosis is usually seen in patients who have been on dialysis for at least 5 years. This disorder involves osteoarticular sites, and patients may present with carpal tunnel syndrome or shoulder pain. Bone cysts may be visible on radiograph. Osteitis fibrosa cystica is the classic pathology associated with kidney disease. This disorder is associated with increased bone turnover and elevated PTH and alkaline phosphatase levels. Mixed uremic osteodystrophy has elements of both high and low bone turnover. Osteomalacia refers to a defect with both low turnover and abnormal mineralization of bone. This disorder can be seen by vitamin D deficiency, but in the past, it was a common complication of aluminum toxicity. This patient's vitamin D level is in the "sufficient range," and there is no history of aluminum exposure. Read Related TextNext Question
Secondary causes of membranous nephropathy?
Age- and sex-appropriate cancer screening is the most appropriate management. The initial step in the management of newly diagnosed membranous glomerulopathy is to evaluate for secondary forms of the disease, which account for approximately 25% of cases. Some of this evaluation is often done in the prebiopsy screening laboratory tests (for example, screening for hepatitis B and C viruses, lupus, and syphilis). Secondary forms of membranous glomerulopathy correlate with age. Cancer screening is particularly important in evaluating for secondary forms of membranous glomerulopathy in patients over the age of 65 years. Up to 25% of such patients will have a malignancy discovered within 1 year of diagnosis, essentially accounting for all forms of secondary membranous glomerulopathy in this age group. This 68-year-old woman with newly diagnosed membranous glomerulopathy has a 50-pack-year history of smoking. She should be sent for age- and sex-appropriate cancer screening, which would include cervical cytology and human papillomavirus testing, mammography, colonoscopy, and low-dose chest CT. Immunosuppression should not be offered in this case until a secondary form has definitively been ruled out and the patient has been carefully monitored for at least 3 to 6 months to allow for the possibility, if this is a primary form of membranous glomerulopathy, for spontaneous remission, which occurs in approximately one third of primary cases. Membranous glomerulopathy is associated with a higher risk for clotting, particularly when serum albumin is <2.8 g/dL (28 g/L). However, there is no consensus on whether such patients should be offered prophylactic anticoagulation, and most experts in the United States opt for vigilant monitoring rather than prophylactic anticoagulation. The M-type phospholipase A2 receptor (PLA2R) is the specific podocyte antigen responsible for eliciting immune complex formation with circulating autoantibodies in most cases of primary membranous glomerulopathy. Anti-PLA2R antibodies are detected in approximately 75% of primary cases and rarely found in secondary forms. A negative staining on biopsy for this antigen (which is a more specific test than serum antibody assays) raises suspicion for a secondary form of the disease and does not require serum testing for confirmation. Read Related TextNext Question
A 74-year-old woman is evaluated for back pain after a fall occurring 2 weeks ago. Medical history is significant for deep venous thrombosis 3 years ago following a 12-hour airplane flight. Medications are acetaminophen as needed for back pain and calcium carbonate with vitamin D. On physical examination, vital signs are normal. She has minimal pain to percussion over T8. Her examination is otherwise normal. Laboratory studies: Alkaline phosphatase 82 U/L Calcium, serum 9.9 mg/dL (2.5 mmol/L) Creatinine, serum 1.1 mg/dL (97.2 µmol/L) 25-Hydroxyvitamin D 40 ng/mL (99.8 nmol/L) Lateral spine radiograph shows 30% compression of T8, not present on prior radiographs. Bone mineral density by DEXA shows a lumbar spine T-score of −3.0 and femur neck T-score of −2.8. Which of the following is the most appropriate treatment? Alendronate Calcitonin Denosumab Raloxifene Teriparatide
Alendronate, risedronate, zoledronic acid, and denosumab have been shown to reduce the risk for spine, hip, and nonvertebral fractures, and are generally well tolerated with low risk for serious adverse effects. The most appropriate treatment for this patient is alendronate. The American College of Physicians (ACP) recommends that clinicians offer pharmacologic treatment with alendronate, risedronate, zoledronic acid, or denosumab to reduce the risk for hip and vertebral fractures in women who have known osteoporosis. Individual patient factors and cost help decide which agent is initially used. Bisphosphonates are the most commonly prescribed first-line therapy as they have been shown to reduce the risk of fractures in large, randomized, placebo-controlled trials, and are generally well tolerated with low risk for serious adverse effects. Although calcitonin increases spine bone mineral density in clinical trials, its anti-fracture efficacy is inconsistent at the spine. The availability of intravenous bisphosphonates and denosumab negates the argument for calcitonin in patients who cannot tolerate oral osteoporosis medications. Denosumab is effective for prevention of vertebral fracture in postmenopausal women, yet it is expensive and, once started, should be continued indefinitely. Even if followed by intravenous bisphosphonate therapy, discontinuation results in loss of bone mineral density and has been associated with an increased risk of vertebral fracture. Denosumab may be used safely in the setting of compromised kidney function (KDIGO stage G3b and G4). It may also be preferred in patients with poor adherence or tolerance of oral bisphosphonates. Raloxifene is approved in postmenopausal women for the prevention and treatment of osteoporosis and the prevention of invasive breast cancer in those at high risk. However, raloxifene is contraindicated in those at increased risk of venous thromboembolism. ACP recommends against raloxifene for the treatment of osteoporosis in women. Teriparatide and abaloparatide are anabolic therapies that reduce the risk of vertebral fracture in postmenopausal osteoporosis. Each increases bone mass and strength of the spine more than antiresorptive drugs and may be preferred if spine bone mineral density is severely low (T-score ≤ −3.5), in patients who fail bisphosphonate therapy, and in glucocorticoid-induced osteoporosis. Neither drug should be prescribed for patients who are at increased risk for osteosarcoma including those with a history of radiation therapy.
A 36-year-old woman is evaluated for new-onset hirsutism noted on the face, chest, and abdomen. Hirsute hair growth has rapidly progressed over the last 6 months. Additionally, she notes frontal hair loss. Menstrual cycles have become irregular over the same time course. Medical history is otherwise unremarkable, and she takes no medications. On physical examination, her blood pressure is 142/88 mm Hg; the remainder of the vital signs is normal. BMI is 26. On skin examination, acne is noted. Dark terminal hair appears on the face, chest, and abdomen. The patient has diffuse hair thinning on top of the head. The remainder of the physical examination is noncontributory. Laboratory studies: Estradiol 68 pg/mL (249.6 pmol/L) Follicle-stimulating hormone 12 mU/mL (12 U/L) Human chorionic gonadotropin, serum Negative Prolactin Normal Testosterone , total 220 ng/dL (7.6 nmol/L) Thyroid-stimulating hormone Normal Which of the following is the most appropriate management? Adrenal vein sampling for cortisol and androgens 24-Hour urine free cortisol Oral contraceptive therapy Pelvic ultrasound
An androgen-secreting ovarian tumor should be considered in patients with abrupt, rapidly progressive, or severe hyperandrogenism as well as in women with marked hyperandrogenemia (total testosterone >150 ng/dL [5.2 nmol/L]). A pelvic ultrasound is the most appropriate management for this patient. Although androgen-secreting ovarian tumors are rare, they should be considered in patients with abrupt, rapidly progressive, or severe hyperandrogenism as well as in women with marked hyperandrogenemia (total testosterone >150 ng/dL [5.2 nmol/L]). Given this patient's rapid onset of hirsutism coupled with significantly elevated total testosterone level, a pelvic ultrasound is the best next step in evaluation to assess for a possible ovarian tumor. Adrenal vein sampling is a procedure used to confirm whether autonomous adrenal hormone production is unilateral or bilateral. It is most commonly performed in the evaluation of primary aldosteronism. Adrenal vein sampling is needed in most patients to determine the source of aldosterone secretion when imaging is unrevealing and to confirm lateralization when imaging demonstrates an adrenal adenoma. For this patient, adrenal CT is more appropriate to exclude an adrenal cortisol-secreting and/or androgen-secreting neoplasm and may be appropriate if the pelvic ultrasound is unrevealing. There is no indication for this procedure at this time in this patient. Cushing syndrome (CS) results from elevated levels of cortisol. Clinical findings that are highly specific for CS include centripetal obesity, facial plethora, abnormal fat deposition in the supraclavicular or dorsocervical ("buffalo hump") areas, and wide (>1 cm) violaceous striae. Evaluation for CS is most appropriate in patients who have specific signs and symptoms of CS, rather than in patients with diffuse obesity, those who have nonpathologic striae, or those who are having trouble losing weight because endogenous CS is such a rare condition with a costly evaluation algorithm. At least two first-line tests should be abnormal before the diagnosis is confirmed. Initial tests include the overnight low-dose dexamethasone suppression test, 24-hour urine free cortisol, and late-night salivary cortisol. Since the patient has none of the specific findings of CS, a 24-hour urine cortisol test is not indicated. In addition to weight loss, combined hormonal oral contraceptives are first-line agents for treatment of polycystic ovary syndrome. Polycystic ovary syndrome is a diagnosis of exclusion; given this patient's rapid onset of hirsutism coupled with degree of her total testosterone elevation, an androgen-secreting tumor must be considered.
A 62-year-old woman is evaluated in the emergency department for worsening dyspnea on exertion during the last 2 weeks. She has a history of severe COPD and an FEV1 of 45% of predicted. She has a cough productive of yellow sputum and wheezing during the same time period. She has a 20-pack-year history of smoking but quit 10 years ago. Her albuterol inhaler and nebulizer have provided temporary relief at home. Current medications are umeclidinium/vilanterol, mometasone, and albuterol. On physical examination, temperature is normal, blood pressure is 132/64 mm Hg, pulse rate is 110/min, and respiration rate is 30/min. Oxygen saturation is 90% on 6 L/min of oxygen by nasal cannula. Cardiopulmonary examination reveals tachycardia, tachypnea with accessory muscle use, and decreased breath sounds throughout with a prolonged expiratory phase and end expiratory wheezes. She has no jugular venous distention or edema. Laboratory studies reveal an arterial PCO2 of 46 mm Hg (6.11 kPa). Complete blood count, serum electrolytes , and blood glucose are normal. A chest radiograph shows hyperinflated lungs with flattening of the diaphragms but no infiltrate. CT pulmonary angiogram Electrocardiogram Sputum culture Sputum Gram stain
An electrocardiogram should be obtained for this patient to evaluate other causes of her acute presentation, such as acute myocardial infarction, arrhythmia, and atrial fibrillation. The first steps in managing a patient with a presumed COPD exacerbation are to confirm the diagnosis. Studies helpful in the evaluation may include pulse oximetry to assess oxygenation or guide oxygen therapy; a chest radiograph to rule out an alternative diagnosis; a complete blood count to identify the presence of polycythemia, anemia, or leukocytosis; arterial blood gas studies; a biochemical panel to assess for electrolyte and glycemic abnormalities; and an electrocardiogram to evaluate tachycardia (as in this patient) and for other possible cardiac comorbidity. This patient has already had a chest radiograph and appropriate laboratory studies but an electrocardiogram is also indicated. Patients with pulmonary embolism (PE) can present with symptoms similar to a COPD exacerbation. Because of this, pretest probability models such as the Wells criteria have been developed and validated to assist in clinical decision making. Patients who have a low pretest probability for PE using the Wells criteria, such as this patient, and who meet all Pulmonary Embolism Rule-Out Criteria do not need further testing to rule out PE. Physicians should obtain a D-dimer assay in patients with intermediate pretest probability for PE or for those with a low pretest probability for PE who do not meet all Pulmonary Embolism Rule-Out Criteria. In these patients, imaging studies should not be used for initial evaluation. A sputum culture or Gram stain is not routinely used to assess COPD exacerbations as it rarely changes management. For patients with a confirmed COPD exacerbation with cough, increased sputum, and dyspnea, antibiotics are usually initiated regardless of the results of the sputum Gram stain or culture.
A 42-year-old woman has follow-up evaluation for stage II colon cancer that was diagnosed 3 years ago. Her tumor was found to be deficient in the mismatch repair protein MLH1. Genetic evaluation showed that she had Lynch syndrome, a germline mutation of one of the DNA mismatch repair genes. Nine months later, she developed recurrent disease with liver and lung metastases. Analysis of her tumor at that time revealed a KRAS mutation. She was treated initially with 5-fluorouracil, leucovorin, and oxaliplatin (FOLFOX) and then with irinotecan, each with only a short duration of tumor control and subsequent progression of disease. She reports that she has been able to maintain her full-time job and has been walking more than a mile daily. She takes no medications. On physical examination, vital signs and the remainder of the examination are normal. Which of the following is the most appropriate treatment? An epidermal growth factor receptor inhibitor An immune checkpoint inhibitor Dual antibody therapy targeting vascular endothelial growth factor and endothelial growth factor receptor No further chemotherapy; palliation care only
An immune checkpoint inhibitor, such as pembrolizumab, shows benefit in patients with metastatic mismatch repair−deficient colorectal cancer. An immune checkpoint inhibitor is the most reasonable treatment. This patient now has metastatic colorectal cancer with a mismatch repair-deficient tumor. Tumors may be mismatch repair deficient either because of a germline mutation (Lynch syndrome) or a somatic mutation or epigenetic silencing that is limited to the tumor. In either case, these mismatch repair-deficient tumors represent the very limited subset of metastatic colorectal cancers that have shown substantial benefit in a few small studies from treatment with immune checkpoint inhibitors, such as pembrolizumab or nivolumab. Unfortunately, these mismatch repair-deficient tumors represent only a small minority of patients with metastatic cancer. For most patients with metastatic colorectal disease (greater than 95%), immune checkpoint inhibitors have thus far been essentially ineffective. Epidermal growth factor receptor inhibitors such as panitumumab or cetuximab are inappropriate for consideration in a patient with a mutation in KRAS or any other RAS gene because they are potentially active only in tumors with nonmutated KRAS, NRAS, and BRAF genes. Several trials have examined the efficacy of simultaneously targeting both vascular endothelial growth factor and the endothelial growth factor receptor with dual antibody therapy. However, this synergic approach has not been shown to be effective, and the results suggest that dual antibody therapy for this patient is not the next most reasonable treatment option. Although supportive comfort care only should be considered in patients who are either too sick for treatment or who have exhausted all reasonable treatment options, such a course of action would not be the first consideration in a fully functional patient with a meaningful treatment option.
A 42-year-old woman is evaluated prior to surgery following a diagnosis of pheochromocytoma. Her symptoms are palpitations, hypertension, and sweating for 8 months' duration. Medications are lisinopril and hydralazine. On physical examination, blood pressure is 155/98 mm Hg. Other vital signs and the remainder of the examination are normal. Which of the following is the most appropriate next step in management? Increase hydralazine Increase lisinopril Start chlorthalidone Start metoprolol Start phenoxybenzamine
An α-receptor blockade with phenoxybenzamine or another α-blocker is required prior to adrenalectomy for pheochromocytoma to prevent potential hypertensive crisis during anesthesia induction and/or manipulation of the tumor. The most appropriate preoperative management in this patient with a pheochromocytoma is an α-adrenergic blocking agent. An α-receptor blockade is required prior to adrenalectomy to prevent potential hypertensive crisis caused by catecholamine release during anesthesia induction and/or manipulation of the tumor. Phenoxybenzamine is started approximately 10 to 14 days prior to surgery, and the dose progressively increased to achieve a desired blood pressure of 130/80 mm Hg or lower when seated, and systolic pressure of 90 mm Hg or higher when standing. Because phenoxybenzamine causes vasodilation, an expected consequence of therapy is postural hypotension. To counteract this and allow appropriate dose escalation of phenoxybenzamine, patients are advised to drink plenty of fluids, eat high salt-containing foods, and to make liberal use of the salt shaker at meal times. If blood pressure is not adequately controlled with an α-receptor blockade (or prohibitive side effects occur with required higher doses), a calcium-channel blocker such as amlodipine can be added. A short-acting, selective α-blocker such as prazosin, doxazosin, or terazosin, can be considered alternatives to phenoxybenzamine, based on decreased cost and limited data suggesting similar patient outcomes. Hydralazine or lisinopril do not provide the needed α-receptor blockade, and therefore increasing the dose of these medications would not be helpful before surgery. Diuretics, such as chlorthalidone, should be avoided in patients with pheochromocytoma. These patients may have diminished intravascular volume secondary to intense vasoconstriction and further diuretic-induced volume depletion may lead to severe hypotension. Finally, diuretic treatment will not prevent hypertensive crisis during adrenalectomy. Metoprolol or other β-blockers should never be started prior to adequate α-receptor blockade in patients with pheochromocytoma, as unopposed α-receptor stimulation can precipitate a hypertensive crisis. Once adequate α-receptor blockade has been achieved, however, a β-blocker is typically added to the medication regimen 2 to 3 days prior to surgery to counteract vasodilation-induced tachycardia. In the case of very large tumors and/or significant metanephrine elevation, the catecholamine synthesis inhibitor metyrosine may also be added to the medication regimen. Metyrosine is not given routinely in the preoperative management of pheochromocytoma due to significant side effects associated with body-wide catecholamine deficiency. Read Related TextNext Question
HIV PrEP (pre-exposure prophylaxis)?
Tenofovir Emtricitabine *2 components (post-exposure = 3 components)
HIV post-exposure prophylaxis components??
Tenofovir (NTRTI) Emtricitabine (Reverse transcriptase inhibitor) Dolutegravir!!! (Integrase inhibitor) OR RALTEGRAVIR!!! DIFFERENT FROM PrEP (2 components) = tenofovir + emtricitabine
A 67-year-old man is evaluated for a 1-year history of weakness, with increased tripping on curbs and difficulty with handwriting, which is not as neat as in the past. He reports no muscle cramping. He takes no medications. On physical examination, vital signs are normal. There is no rash. There is symmetric weakness of the forearm and thigh muscles. Reduced grip strength and reduced wrist and finger flexion are noted. Reflexes are normal. There is atrophy of the muscles of the forearms and interosseous muscles of the right hand greater than the left. There are no fasciculations. Laboratory studies show a serum creatine kinase level of 1150 U/L and a normal thyroid-stimulating hormone level; antinuclear and anti-Jo-1 antibody testing is negative. Electromyogram and nerve conduction studies show myopathic changes in the proximal and distal muscles of the extremities, as well as some neurogenic changes. Which of the following is the most likely diagnosis? Amyotrophic lateral sclerosis Inclusion body myositis Mitochondrial myopathy Polymyositis
The most likely diagnosis is inclusion body myositis (IBM), an inflammatory myopathy that can involve both the proximal and distal muscles; although typically symmetric, muscle distribution may be asymmetric. Sporadic IBM is the most common muscle disease in elderly populations, is more common in men than women, and typically develops after age 50 years and does not appear to affect individuals younger than 45 years old. Serum creatine kinase levels are typically less than 10 to 12 times the upper limit of normal. Its insidious onset and distal muscle involvement help to distinguish IBM from the other inflammatory myopathies. The absence of rash helps to exclude dermatomyositis. Electromyogram may show myopathic, neurogenic, or mixed changes with both short and long duration motor unit potentials and spontaneous activity. Diagnosis of IBM is confirmed by muscle biopsy showing muscle fibers containing multiple rimmed vacuoles. Distinguishing IBM from polymyositis is important because IBM is usually not treated with immunosuppressive therapy and has an overall poor response to treatment. Amyotrophic lateral sclerosis (ALS) is a progressive, neurodegenerative disease that can have an insidious onset causing muscle weakness. Upper and lower motor neuron signs may be seen, including spasticity, abnormal gait, increased reflexes, and difficulty with coordination (including manipulation of small objects), tripping, and falling. Findings may initially be proximal or distal and asymmetric. ALS can be distinguished from IBM by the presence of fasciculations, and ALS does not raise serum creatine kinase levels. Mitochondrial myopathy can present in childhood, adolescence, or young adulthood, and a family history of mitochondrial disease is often evident. When myopathy predominates, the patient generally presents with proximal muscle weakness. However, exercise intolerance, painful and sometimes severe muscle cramping, and eye symptoms such as ptosis and diplopia are common. Serum creatine kinase levels are usually within the normal range, but serum lactate levels may be elevated. Polymyositis occurs more frequently in women than in men and has its age of onset before 50 more often than IBM. It generally presents with proximal, rather than distal, muscle weakness, and serum creatine kinase levels can be substantially higher than those seen in IBM, often greater than 10 to 50 times normal.
A 79-year-old woman is evaluated in the emergency department for worsening confusion over the past 5 days. She also reports lower back pain for the past 3 months. History is significant for hypertension and coronary artery disease with stenting of the left anterior descending artery 2 years ago. Daily medications are metoprolol, hydrochlorothiazide, atorvastatin, low-dose aspirin, and acetaminophen. Her husband confirms that the patient takes all medications as directed. On physical examination, temperature is normal, blood pressure is 128/76 mm Hg, pulse rate is 72/min, respiration rate is 20/min, and oxygen saturation is 95% on ambient air. BMI is 19. There is no abdominal pain. The patient is weak, confused to time and place, and sleepy but easily arousable. The remainder of the neurologic examination is normal. Laboratory studies: Blood urea nitrogen 35 mg/dL (12.5 mmol/L) Creatinine 1.4 mg/dL (123.8 µmol/L) Electrolytes : Sodium 138 mEq/L (138 mmol/L) Potassium 4.8 mEq/L (4.8 mmol/L) Chloride 102 mEq/L (102 mmol/L) Bicarbonate 14 mEq/L (14 mmol/L) Lactate 0.7 mEq/L (0.7 mmol/L) Arterial blood gases : pH 7.31 PCO2 29 mm Hg (3.9 kPa) Urinalysis Specific gravity 1.025; no protein, ketones, cells, or crystals Which of the following is the most likely diagnosis? D-Lactic acidosis Propylene glycol toxicity Pyroglutamic acidosis Salicylate toxicity
The most likely diagnosis is pyroglutamic acidosis. Pyroglutamic acidosis, which presents with mental status changes and an increased anion gap, occurs in selected patients receiving therapeutic doses of acetaminophen on a chronic basis. Susceptible patients are those with critical illness, poor nutrition, liver disease, or chronic kidney disease, as well as those on a strict vegetarian diet. In this context, acetaminophen leads to depletion of glutathione, altering the γ-glutamyl cycle to overproduce pyroglutamic acid (also known as 5-oxoproline). Diagnosis can be confirmed by measuring urine levels of pyroglutamic acid. D-Lactic acidosis presents with an increased anion gap metabolic acidosis and characteristic neurologic findings of intermittent confusion, slurred speech, and ataxia in patients with short-bowel syndrome. Accumulation of the D-isomer of lactate can occur in patients with short-bowel syndrome following jejunoileal bypass or small-bowel resection. In these patients, excess carbohydrates that reach the colon are metabolized to D-lactate. Laboratory studies show increased anion gap metabolic acidosis with normal plasma lactate levels, because the D-isomer is not measured by conventional laboratory assays for lactate. Diagnosis is confirmed by specifically measuring D-lactate. This patient's lack of short-bowel syndrome rules out this diagnosis. Propylene glycol, a solvent used to enhance the solubility of various intravenously administered medications, causes an increased anion gap metabolic acidosis through its acid metabolites, L-lactate and D-lactate. An increased osmolal gap accompanies the increased anion gap metabolic acidosis seen with propylene glycol. This patient's clinical history and lack of lactic acidosis are not consistent with propylene glycol toxicity. Salicylate toxicity most commonly presents in adults as respiratory alkalosis or with features of both respiratory alkalosis and increased anion gap metabolic acidosis. This patient has appropriate respiratory compensation for the metabolic acidosis, not respiratory alkalosis, making salicylate toxicity unlikely.
A 41-year-old woman is evaluated for a 3-month history of increasing nonproductive cough, fatigue, anorexia, and malaise. History is significant for hypertension. Medications are hydrochlorothiazide, lisinopril, and self-prescribed vitamin D and calcium for bone health. On physical examination, vital signs are normal. Bilateral crackles are heard on pulmonary auscultation. Trace pedal edema is present. The remainder of the examination is unremarkable. Laboratory studies: Calcium 11.3 mg/dL (2.8 mmol/L) Creatinine 1.6 mg/dL (141.4 µmol/L); 1 year ago: 1.0 mg/dL (88.4 µmol/L) Phosphorus 3.4 mg/dL (1.1 mmol/L) Parathyroid hormone 12 pg/mL (12 ng/L) 25-Hydroxyvitamin D 43 ng/mL (107.3 nmol/L) Urinalysis Specific gravity 1.010; 1+ protein; 5-20 leukocytes/hpf; occasional granular casts Urine protein-creatinine ratio 400 mg/g 24-Hour urine calcium Elevated Chest radiograph shows diffuse reticular opacities. Kidney ultrasound demonstrates nephrocalcinosis. Which of the following is the most likely cause of this patient's findings? Hydrochlorothiazide Primary hyperparathyroidism Sarcoidosis Vitamin D intoxication
The most likely diagnosis is sarcoidosis, a systemic inflammatory disease that can affect multiple organs. The most common presentations of sarcoidosis are hilar lymphadenopathy and parenchymal lung disease; common extrapulmonary organ involvement includes the skin, joints, and eyes. More than 90% of patients with kidney involvement have thoracic sarcoid identified on chest radiograph. Kidney manifestations in sarcoidosis are common and include nephrocalcinosis from hypercalcemia and hypercalciuria, nephrolithiasis, and chronic interstitial nephritis with granuloma formation. Hypercalcemia occurs due to peripheral conversion of 25-hydroxyvitamin D to 1,25-dihydroxyvitamin D by activated macrophages. Parathyroid hormone is typically suppressed in response to the hypercalcemia. The urinalysis is typical of other chronic tubulointerstitial diseases and can be normal or show only sterile pyuria or mild proteinuria, as in this patient's case. Kidney manifestations of hypercalcemia and interstitial nephritis are treated with glucocorticoids. Thiazide diuretics can be associated with a mild hypercalcemia but not nephrocalcinosis. Furthermore, they decrease urinary calcium excretion. This patient's urinary calcium excretion is increased. Primary hyperparathyroidism is characterized by hypercalcemia, hypophosphatemia, and inappropriately normal or elevated parathyroid hormone. This patient's parathyroid hormone is suppressed, making this an unlikely diagnosis. Increased levels of calcium absorption from the gut can be from markedly high vitamin D levels. Vitamin D intoxication is usually defined as a value >150 ng/mL (374.4 nmol/L). Vitamin D intoxication is not consistent with the laboratory findings because the phosphorus and 25-hydroxyvitamin D levels are normal. Finally, thiazide diuretic use, primary hyperparathyroidism, or vitamin D intoxication cannot account for the patient's pulmonary findings.
A 21-year-old man is evaluated in the emergency department for persistent convulsive status epilepticus that began 30 minutes before his arrival. An airway has been secured, and he has received intravenous glucose, thiamine, and two doses of intravenous lorazepam. After receiving the second dose of lorazepam, he continues shaking for another 5 minutes. Medications are levetiracetam and acetaminophen; the patient is allergic to phenytoin. On physical examination, temperature is normal, blood pressure is 155/89 mm Hg, pulse rate is 108/min, respiration rate is 16/min, and oxygen saturation with the patient breathing 6 L of oxygen via a nasal cannula is 97%. The pupils are reactive but he remains comatose. Results of a urine drug screen are negative. Which of the following is the most appropriate next step in management? Brain MRI Electroencephalography Fosphenytoin Lacosamide Valproic acid
The patient should now receive intravenous valproic acid. He is in generalized convulsive status epilepticus (CSE). A medical emergency that can lead to significant morbidity and mortality, CSE is defined as persistent tonic-clonic activity with impaired mental status that lasts longer than 5 minutes. Initial management of CSE requires rapidly assessing airways, breathing, and circulation; checking the blood glucose level; and administering thiamine with glucose, if needed. These steps should be performed simultaneously with initiation of drug treatment. According to a guideline from the American Epilepsy Society, intramuscular midazolam, intravenous (IV) lorazepam, and IV diazepam have been judged to be equivalent and are the first-line agents for the initial treatment of CSE. Phenytoin, a longer-acting antiseizure drug, should then be administered; if available, fosphenytoin, a prodrug of phenytoin, is preferable to phenytoin for initial treatment of CSE because it can be administered faster and does not carry the risk of thrombophlebitis or skin necrosis (purple glove syndrome) that is associated with phenytoin extravasation. Valproic acid is an alternative to phenytoin or fosphenytoin, particularly in patients with an allergy to phenytoin or primary generalized epilepsy. Therefore, intravenous valproic acid should be administered to this patient as a second-line therapy. CSE usually is diagnosed clinically. Obtaining an MRI of the brain is an unnecessary time-consuming process and would delay treatment. Once CSE is controlled, head CT usually is indicated because it takes less time and is more readily available in the acute setting. Electroencephalography could eventually be considered, but, again, the diagnosis of CSE is primarily clinical, and treatment should not be postponed. Lacosamide, which is available intravenously, sometimes is considered in the management of convulsive status epilepticus. However, evidence is insufficient to recommend it as part of the standard guidelines for initial treatment of generalized CSE, which makes it inappropriate for this patient. A 2018 systematic review found that supplemental oxygen in patients with normal oxygen saturation increases mortality in critically ill patients. A subsequent international guideline strongly recommends that oxygen therapy not be initiated for critically ill patients with an oxygen saturation of 93% or higher. If supplemental oxygen is initiated, the guideline recommends not exceeding an oxygen saturation of 96%. This patient's oxygen saturation is 97%, and thus his supplemental oxygen also should be reduced.
A 46-year-old man is evaluated in the hospital for a 6-week history of fatigue and worsening dyspnea. Medical history is significant for a bicuspid aortic valve. He takes no medications. On physical examination, temperature is 38.1 °C (100.6 °F), blood pressure is 118/58 mm Hg, pulse rate is 92/min, and respiration rate is 18/min. Oxygen saturation is 98% breathing 2 L of oxygen by nasal cannula. Cardiac examination reveals a grade 2/6 diastolic murmur heard best at the left lower sternal border. There are crackles at the lung bases bilaterally. Conjunctival hemorrhage is present in the left eye. Laboratory studies are notable for a leukocyte count of 15,000/µL (15 × 109/L). Three sets of blood cultures are positive for gram-positive cocci. An electrocardiogram shows sinus rhythm, a PR interval of 220 ms, and QRS duration of 100 ms. Which of the following is the most appropriate diagnostic test to perform next? Cardiac magnetic resonance imaging Coronary CT angiography Transesophageal echocardiography Transthoracic echocardiography
The preferred imaging modality for evaluating patients with a high pretest probability of infective endocarditis or with potential complications of endocarditis, such as abscess, is transesophageal echocardiography. Transesophageal echocardiography (TEE) is the most appropriate diagnostic test to perform next. According to the modified Duke criteria, this patient has definite endocarditis, with one major criterion (persistently positive blood cultures) and three minor criteria (predisposing valvular abnormality [bicuspid valve], fever with temperature >38 °C [100.4 °F], and a vascular phenomenon [conjunctival hemorrhage]). On physical examination, he has a wide pulse pressure and evidence of a murmur of aortic regurgitation, findings that further support the diagnosis. Additionally, the electrocardiogram shows PR-interval prolongation, which should raise suspicion for the presence of a periaortic abscess that is interfering with the conduction system and causing first-degree atrioventricular block. Perivalvular abscesses may be present in 30% to 40% of patients with infective endocarditis, and the risk may be further increased in those with a bicuspid aortic valve. The preferred imaging modality for evaluating patients with a high pretest probability of infective endocarditis or with potential complications of endocarditis is TEE, as it better visualizes vegetations and abscesses. TEE is also important for surgical planning. Cardiac magnetic resonance (CMR) imaging is not the most appropriate initial imaging choice for patients with infective endocarditis. CMR imaging is effective in identifying intramyocardial infection, although it is a more complex technology with limited availability in some areas. CMR imaging is often used in situations in which a perivalvular abscess is suspected but transesophageal echocardiographic findings are equivocal. Coronary CT angiography could be used for evaluation in patients suspected of having perivalvular infections if echocardiography cannot adequately identify the anatomy. Transthoracic echocardiography can help identify the presence of a vegetation; however, there is an increased likelihood of detecting perivalvular abscess with TEE because of the closer proximity of the ultrasound probe to the valve structures. TEE is preferred to transthoracic echocardiography if this diagnosis is a consideration. A 2018 systematic review found that supplemental oxygen in the setting of normal oxygen saturation increases mortality in critically ill patients. A subsequent international guideline strongly recommends that oxygen therapy not be initiated for critically ill patients with an oxygen saturation as measured by pulse oximetry (SpO2) of 93% or higher. If supplemental oxygen is initiated, the guideline recommends not exceeding an SpO2 of 96%. This patient's oxygen saturation is 98%, and his supplemental oxygen should be reduced.
A 60-year-old man is evaluated for an asymptomatic elevation of serum protein. Medical history is unremarkable, and he takes no medications. On physical examination, vital signs are normal. Neurologic examination reveals no deficits. Laboratory studies: Hemoglobin 14 g/dL (140 g/L) Leukocyte count 7000/µL (7 × 109/L) Platelet count 300,000/µL (300 × 109/L) Calcium 9.4 mg/dL (2.4 mmol/L) Creatinine 1.1 mg/dL (97.2 µmol/L) Serum protein electrophoresis and immunofixation show an IgA spike of 3.5 g/dL. Skeletal survey findings are negative. Which of the following is the most appropriate next test? Bone scan CT of the chest, abdomen, and pelvis Whole body MRI No further testing
Whole body MRI is the most appropriate next diagnostic test for this patient. Smoldering multiple myeloma (MM) is characterized by a serum M protein level of 3 g/dL or greater (or ≥500 mg/24 hr of urinary monoclonal free light chains) or bone marrow plasma clonal cells of 10% or greater and no evidence of myeloma-related signs or symptoms requiring therapy. All patients with MM should be assessed for skeletal lesions at diagnosis, periodically thereafter, and when any new symptoms occur. Skeletal survey with plain radiography is commonly used to assess for lytic lesions in patients with monoclonal gammopathy. However, MRI has recently been found to be more sensitive at identifying myeloma bone lesions and soft tissue lesions from plasmacytoma. The International Myeloma Working Group recommends that all patients with smoldering MM undergo whole body MRI (or spine and pelvic MRI if whole body MRI is not available). Whole body MRI is considered the gold standard for imaging of the axial skeleton, for the evaluation of painful lesions, and for distinguishing benign versus malignant osteoporotic vertebral fractures. If a patient is discovered to have more than one lesion greater than 5 mm, the patient should be considered symptomatic and requires consideration for treatment. With equivocal small lesions, a second MRI should be performed after 3 to 6 months; if the MRI shows progression, the patient should be treated as having symptomatic myeloma. In this patient with smoldering MM, negative findings on plain radiographs do not reliably rule out a skeletal lesion, and MRI is needed for further evaluation. Although bone scans are useful in detecting bone metastases from underlying cancer, the bone lesions in MM are often purely lytic and lack the enhanced osteoblastic activity that is shown by bone scan imaging. Bone scans should not be used to assess bone involvement in myeloma. CT is more sensitive than plain radiography in detecting bone lesions but is less sensitive than MRI in detecting small lesions. Additionally, patients with MM are at high risk for contrast nephropathy from CT contrast. No further testing is inappropriate because the detection of asymptomatic focal lesions on whole body MRI determines the plan of care for patients with smoldering MM.
An 18-year-old man is evaluated for a 4-day history of frequent, large-volume diarrhea, with associated abdominal cramping, emesis, fever, and nausea. He is a lifeguard at a freshwater municipal pool, and several other swimmers who use the pool have recently developed similar symptoms. On physical examination, temperature is 37.5 °C (99.5 °F); the vital signs are otherwise normal. On abdominal examination, bowel sounds are present, palpation elicits minimal tenderness, and no guarding or rebound is noted. Modified acid-fast staining of the stool reveals oocysts that are about 5 microns in diameter. Which of the following is the most likely cause of this patient's diarrhea? Cryptosporidium Enterohemorrhagic Escherichia coli Norovirus Nocardia Vibrio parahaemolyticus
The protozoan Cryptosporidium is the most common cause of swimming pool-related outbreaks of diarrhea; diagnosis is made by microscopic examination of the stool or by stool antigen testing. This patient has watery diarrhea associated with swimming pool exposure, and the oocysts observed microscopically represent Cryptosporidium. This parasitic protozoan is tolerant to chlorine and can persist for days in a chlorinated pool. Cryptosporidium has become the leading cause of swimming pool-related outbreaks of diarrheal illness. Swallowing infected water can result in infection. The incubation period is about 1 week, and the clinical presentation typically includes watery diarrhea, crampy abdominal pain, dehydration, fever, malaise, nausea, vomiting, and weight loss. The infection typically resolves in immunocompetent persons, but infection can be more serious and prolonged in those with immunocompromise, particularly in persons with AIDS who are not receiving combination antiretroviral therapy. Diagnosis can be established microscopically by visualization of oocysts with modified acid-fast staining. Because oocysts are shed intermittently, diagnosis may require stool antigen testing using polymerase chain reaction, enzyme immunoassay, or direct fluorescent antibody testing. Although enterohemorrhagic Escherichia coli (EHEC) infection can be acquired by aspiration of contaminated swimming pool water, it typically produces bloody diarrhea. EHEC is a gram-negative rod that does not exhibit modified acid-fast staining. Norovirus is the most common cause of gastroenteritis and is characterized by explosive vomiting and diarrhea. It is spread person to person through the fecal-oral route, leading to community outbreaks. But the virus is not visualized with modified acid-fast staining. A diagnostic assay for viral gastroenteritis with polymerase chain reaction testing is reserved for public health investigation. Modified acid-fast staining can detect Nocardia species, but the organisms are filamentous branching rods. Infection usually involves the lungs, central nervous system, and skin, but not the gastrointestinal tract. Vibrio parahaemolyticus lives in salt water and causes diarrhea, usually after consumption of undercooked shellfish, especially oysters. This gram-negative rod is not detected with a modified acid-fast stain and requires special culture media with high salt content for growth. Read Related TextNext Question
When is highest risk for cardiac transplant rejection? How do you monitor?
The risk for cardiac transplant rejection is highest within the first 6 months after transplantation and then within the first year; endomyocardial biopsy should be routinely performed within the first year after cardiac transplantation to diagnose rejection. It is important to note that most patients with clinical rejection will not have any symptoms of heart failure; therefore, routine endomyocardial biopsies are performed within the first year after transplantation.
A 43-year-old woman with a 7-year history of relapsing follicular lymphoma is evaluated for fevers, night sweats, and a rapidly enlarging right inguinal nodal mass. Her lymphoma has been treated on multiple occasions. Her current relapse began 1 year ago with diffuse lymphadenopathy; however, the onset of her symptoms and the right inguinal node enlargement is recent. She takes no medications. On physical examination, all vital signs are normal. There is diffuse cervical and axillary lymphadenopathy bilaterally. The left inguinal node is enlarged to 2 cm; the right inguinal node is enlarged to 5 cm. There is no splenomegaly. Laboratory results are significant only for a serum lactate dehydrogenase level of 6420 U/L. Which of the following is the most appropriate management? Biopsy of the right inguinal lymph node Oral idelalisib Radiation therapy to the right inguinal node Repeat a course of chemotherapy
Transformation of follicular lymphoma to an aggressive non-Hodgkin lymphoma may be suggested by a change in the clinical pattern of disease, such as the development of new systemic symptoms or rapid progression of a localized area of disease. This patient has a 7-year history of follicular lymphoma with a recent onset of systemic symptoms and a rapidly enlarging right inguinal lymph node. Biopsy of the right inguinal node would be appropriate to evaluate for histologic progression of disease. Transformation to diffuse large B-cell lymphoma occurs in approximately 30% of patients with follicular lymphoma. Transformation may be suggested by a change in the clinical pattern of disease, such as the development of new systemic symptoms or rapid progression of a localized area of disease. In addition, a rise in serum lactate dehydrogenase level or markedly higher areas of standardized uptake values on PET scanning should raise suspicion of transformation. In such cases, a new lymph node biopsy is required to establish the occurrence of transformation. In this patient, rapid progression in just one area of disease, the onset of systemic symptoms, and a high serum lactate dehydrogenase level raise suspicion for transformation. If transformation to another, more aggressive lymphoma is demonstrated, a more aggressive course of treatment would be appropriate. Thus, a new biopsy in this setting is important to optimize management. Idelalisib, a PI3K kinase inhibitor, is an oral agent that has shown significant activity for the treatment of relapsed follicular lymphoma. It has a reasonably high rate of partial response. It would be appropriate in this patient if transformation is excluded. Radiation for palliation to a single symptomatic site can be considered for relapsed follicular lymphoma. However, this patient has systemic symptoms, and active systemic options are available; therefore, single-site palliative radiation would not be preferred at this time. Treating with additional chemotherapy would be a reasonable option for this patient, given her excellent and prolonged initial response, if she has documented relapse of follicular lymphoma and requires retreatment. However, transformation to a large B-cell lymphoma should first be excluded because it may require a different treatment strategy.
A 63-year-old woman was diagnosed with osteoporosis 6 years ago. Initial treatment with an oral bisphosphonate resulted in upper gastrointestinal symptoms, so subcutaneous denosumab twice yearly was prescribed. The patient has now completed 5 years of denosumab therapy. Medical history is otherwise unremarkable. Denosumab was last administered 6 months ago. Vital signs and the remainder of the physical examination are normal. Which of the following is the most appropriate management? Continue denosumab Dual-energy x-ray absorptiometry (DEXA) scan Osteoporosis drug holiday Switch to zoledronic acid
When administered subcutaneously twice yearly, denosumab suppresses bone resorption, increases bone density, and reduces the incidence of osteoporotic fractures in men and women; the effects of denosumab are not sustained when treatment is stopped. The most appropriate management for this patient is to continue denosumab. Denosumab, a monoclonal antibody against the receptor activator of nuclear factor κB ligand (RANKL), reduces bone resorption by inhibiting the development of osteoclasts. It circulates in the blood for up to 9 months after subcutaneous injection, but once cleared from the circulation, bone resorption transiently but dramatically increases, resulting in an abrupt decline in bone mineral density and, in some cases, vertebral fractures. Once initiated, there is no defined endpoint for cessation of denosumab therapy. Although bone mineral density increases in response to denosumab therapy, it does not impact management with respect to dose and duration of treatment. Therefore, a DEXA scan is not necessary. In the pharmacologic management of osteoporosis, drug holidays are considered during the course of bisphosphonate therapy. Due to their binding to bone tissue, bisphosphonates have durable effects on bone remodeling and fracture risk after discontinuation. After 5 years of treatment, patients at low risk for fracture can be considered for a bisphosphonate drug holiday. One study showed no cumulative difference in the risk for nonvertebral fractures in women continuing alendronate therapy for 5 versus 10 years. Post hoc analysis of this study showed that women with femoral neck T scores of -2.5 or worse and baseline prevalent vertebral fracture had reduced fracture risk by continuing alendronate therapy for 10 years versus stopping after 5 years compared with placebo. Zoledronic acid is an intravenous bisphosphonate indicated for the treatment of osteoporosis especially in patients intolerant to oral bisphosphonates. Patients switched from zoledronic acid to denosumab experience further gains in bone mineral density suggesting additive benefit from this sequence of therapy. However, in patients receiving long-term denosumab, switching to zoledronic acid attenuated but did prevent loss of bone mineral density suggesting that denosumab therapy should be continued once initiated.
A 37-year-old woman is unable to achieve pregnancy despite 7 months of unprotected intercourse. Her menstrual cycles are normal, occurring every 28 days with associated breast tenderness and bloating. There have been no prior pregnancies or attempts to achieve pregnancy by either the patient or her male partner. There is no history of previous sexually transmitted infections. She is otherwise healthy. Medical history is significant for appendicitis at age 26 for which she had an uncomplicated appendectomy. Her only medication is a prenatal vitamin. On physical examination, vital signs are normal. She has a well-healed abdominal scar. Thyroid, skin, and pelvic examinations are all unremarkable. There is no elicitable breast discharge, no signs of hyperandrogenism, and no visual field cuts. Which of the following is the most appropriate management? Obtain midluteal phase serum progesterone level Obtain semen analysis Recommend an additional 5 months of unprotected intercourse Refer for laparoscopy
When evaluating infertility, both female and male factors should be considered concurrently; semen analysis is part of the initial diagnostic evaluation. When evaluating infertility, both female and male factors should be considered concurrently. Thus, semen analysis is part of the initial diagnostic evaluation. Collection should occur after 2 to 3 days of sexual abstinence, but no longer to avoid decreased sperm motility. If semen analysis is abnormal, it should be repeated at least 2 weeks later, and if results are abnormal, referral to a reproductive endocrinologist is recommended. Given that this patient has regular menses every 28 days with molimina symptoms (breast pain and bloating), her cycles appear to be ovulatory. Thus, laboratory assessment of ovulatory function is not needed at this time. In patients who do not have normal menstrual cycles with ovulation, laboratory assessment should be performed. A midluteal phase serum progesterone level, obtained approximately 1 week before the expected menses, is an effective way to assess ovulatory status. A progesterone level above 3 ng/mL (9.5 nmol/L) is evidence of recent ovulation. Measurement of serum thyroid stimulating hormone and prolactin levels is appropriate to exclude thyroid disease and hyperprolactinemia as causes of oligo-ovulation. In women over the age of 35 years, an infertility evaluation is initiated after 6 months of unprotected intercourse; in women under the age of 35, an infertility evaluation is initiated after 1 year of regular unprotected intercourse. Recommending an additional 5 months of unprotected intercourse is unnecessary for this 37-year-old woman. Laparoscopy for evaluation of pelvic adhesions or mild endometriosis may be warranted in patients with dysmenorrhea, history of sexually transmitted infections, or previous pelvic surgery. While assessment of tubal patency may be indicated in this patient given her prior history of appendicitis and abdominal surgery (putting her at risk for adhesions), laparoscopy is not indicated at this immediate time prior to moving forward with an initial noninvasive diagnostic evaluation.
A 23-year-old woman is evaluated for 2 weeks of painful lumps on her legs. The lumps persist for several days and make it difficult for her to go to work as a waitress. She is a college student in Ohio where she has lived her whole life. She has not traveled outside of the state for the last 2 years. She has no swollen or painful joints. She denies abdominal pain, diarrhea, weight loss, night sweats, and fever. She is sexually active with one partner for the past 2 years. She is taking oral contraceptive pills. On physical examination, vital signs are normal. There are tender faint pink-brown nodules on the shins bilaterally. The throat and tonsils appear normal. There is no joint swelling. Pregnancy test is negative. Complete blood count, erythrocyte sedimentation rate , and antistreptolysin O titers are pending. Barrier contraceptive methods are recommended in lieu of oral contraceptives. Which of the following tests should be done next? Biopsy of a nodule Chest radiography Colonoscopy Nucleic acid amplification testing for gonorrhea
A chest radiograph is recommended in the evaluation of erythema nodosum to assess for the presence of lymphoma, sarcoidosis, tuberculosis, and fungal infection such as coccidioidomycosis. This patient needs a chest radiograph to complete the evaluation of her erythema nodosum (EN). EN is the most common form of panniculitis, or inflammation of the fat, with most inflammation concentrated on the intralobular septae. Because the inflammation is deep under the skin, the clinical manifestation seen on the surface is often tender, ill-defined erythema with some substance on palpation, which may fade from an active inflammatory red-pink to dull brown. Most commonly, EN occurs bilaterally and symmetrically on the anterior shins; however, it may also appear in any fatty area. Although lesions will often come and go, most resolve over 4 to 6 weeks. EN is a nonspecific reaction pattern occurring in response to some systemic process. EN can be idiopathic, but the most common associations are streptococcal infection, hormones (including oral contraceptives, hormone replacement therapy, or pregnancy), inflammatory bowel disease, sarcoidosis, lymphoma, and medication reactions. The diagnosis of EN can be clinically based on the acute onset of tender nodules on the bilateral shins typically in a young woman. Biopsy is not necessary in typical lesions. Most authorities recommend a chest radiograph in the evaluation of EN to assess for the presence of lymphoma, sarcoidosis, tuberculosis, and fungal infection such as coccidioidomycosis. In the absence of gastrointestinal symptoms, a colonoscopy for inflammatory bowel disease is unlikely to reveal a causative diagnosis. Patients with disseminated gonococcal infection and bacteremia manifest vesiculopustular or hemorrhagic macular skin lesions, not tender subcutaneous nodules as seen in this patient. Patients with disseminated gonococcal infection present with dusky pustules or purpura, fever, chills, and polyarthralgia. Knees, elbows, and distal joints are typical sites of involvement. Subcutaneous nodules are not features of this infection
A 28-year-old woman is evaluated in the emergency department for hematuria of 12 hours' duration. She also reports upper respiratory infection symptoms for the past 2 days. She does not have dysuria or flank pain and is not currently menstruating. She reports a similar incident during college when she had the flu, but the urine cleared up after a few hours; she also had an episode of red urine after completing a half-marathon last year. She takes no medications. Physical examination and vital signs are unremarkable. Laboratory studies: C3 Normal C4 Normal Creatinine Normal Urinalysis 4+ blood; 1+ protein; no leukocyte esterase; no nitrites Pregnancy test Negative Noncontrast CT scan of the abdomen shows no renal masses, no evidence of nephrolithiasis, and no hydronephrosis. Which of the following is the most likely diagnosis? Acute postinfectious glomerulonephritis IgA nephropathy IgA vasculitis Lupus nephritis
A classic presentation of IgA nephropathy is recurrent gross hematuria that occurs concomitantly or within days after an upper respiratory infection or physical exertion and usually follows a benign course. IgA nephropathy is the most likely diagnosis. Recurrent gross hematuria, in which macroscopic hematuria occurs concomitantly or within days after an upper respiratory infection (synpharyngitic nephritis) or physical exertion, is a classic presentation of IgA nephropathy, particularly in younger patients. This nephritic presentation usually follows a benign clinical course without associated kidney insufficiency, although in a minority of cases intraluminal obstructive erythrocyte casts can be a cause of moderate to severe acute kidney injury that typically responds to supportive care measures. The diagnosis of IgA nephropathy can only be made by kidney biopsy, but some clinicians will make an empiric diagnosis for young patients with a classic history of recurrent gross hematuria, negative imaging for masses/stones, and normal kidney function with little to no proteinuria. IgA vasculitis (Henoch-Schönlein purpura) is the most common childhood vasculitis and tends to appear after upper respiratory infections. IgA vasculitis is rarer among adults. Characteristic symptoms are abdominal pain and palpable purpura. Gastrointestinal ischemia may be severe enough to cause intestinal bleeding. Arthritis is common, other organ systems may be involved, and patients may present with glomerulonephritis. The patient's recurrent symptoms, lack of additional findings, benign clinical course, and urine findings are not compatible with a diagnosis of IgA vasculitis. Lupus nephritis and postinfectious glomerulonephritis can present with macroscopic glomerular hematuria, but these disease states usually induce low complement levels (low C3 with or without low C4). Postinfectious glomerulonephritis, when associated with upper respiratory infections caused by streptococcal organisms, usually occurs 2 to 3 weeks after the resolution of the streptococcal infection. In the developed world, it is more common to see peri-infectious glomerulonephritis associated with staphylococcal infections (usually skin infections), with onset at the time of infection, but these are not usually associated with gross hematuria or a recurrent history of such episodes.
A 74-year-old man undergoes follow-up evaluation for anemia found on preoperative assessment. He reports a 3-month history of night sweats but no headaches, vision changes, or shortness of breath. He is scheduled to have hip replacement surgery. Medical history is significant for hypertension. His only medication is hydrochlorothiazide. On physical examination, vital signs are normal. Funduscopic examination is normal. No lymphadenopathy is present in the cervical, axillary, or inguinal regions. Hepatosplenomegaly is noted on abdominal examination. Laboratory studies: Hemoglobin 8.4 g/dL (84 g/L) Leukocyte count 7000/µL (7 × 109/L) with normal differential Mean corpuscular volume 102 fL Platelet count 300,000/µL (300 × 109/L) Reticulocyte count 1.9% of erythrocytes Calcium 9.4 mg/dL (2.4 mmol/L) Creatinine 1.1 mg/dL (97.2 µmol/L) Serum protein electrophoresis with immunofixation shows an IgM spike of 330 mg/dL (3.3 g/L). CT scan of the chest and abdomen shows hepatomegaly and splenomegaly. Prominent lymph nodes are seen in the para-aortic and celiac regions, the largest of which is 2.1 cm. Which of the following is the most appropriate diagnostic test to perform next? Bone marrow biopsy CT imaging-guided needle biopsy of a para-aortic lymph node Excisional biopsy of a para-aortic lymph node Serum viscosity testing
A bone marrow biopsy is the most appropriate diagnostic test to perform in this patient. He has hepatosplenomegaly, anemia, and night sweats along with an IgM spike on laboratory testing, indicating he most likely has Waldenström macroglobulinemia (WM). WM is an indolent B-cell lymphoma with clonal lymphoplasmacytic infiltration of the bone marrow that secretes IgM in the blood. Patients with WM can present with classic "B symptoms" of drenching night sweats, fever, weight loss, and anemia. Tissue infiltration of the lymphoma can cause hepatosplenomegaly and lymphadenopathy. Bone marrow biopsy to document lymphoplasmacytic infiltrate will confirm the diagnosis. For patients suspected of having a lymphoma, excisional biopsy of a lymph node is preferred to needle biopsy because it provides better architectural detail for distinguishing malignant from benign causes and for classifying the nature of the malignancy. In patients with WM, bone marrow biopsy is predictably diagnostic and a much less invasive option than is para-aortic lymph node excision. Increased IgM protein in the blood can cause hyperviscosity, with symptoms such as altered vision, headache, hearing loss, tinnitus, dizziness, nystagmus, altered mental status, and nasal and oropharyngeal bleeding. Funduscopic evaluation may show hyperviscosity-related findings, including dilated retinal veins, papilledema, and flame hemorrhages. This patient has no signs or symptoms of hyperviscosity and has an M protein level of less than 400 mg/dL (4 g/L), so serum viscosity testing is not indicated.
A 75-year old man is evaluated for ongoing management of his type 2 diabetes mellitus. He was diagnosed with diabetes 11 years ago. In addition to diabetes, medical history is significant for hypertension, hyperlipidemia, and chronic kidney disease stage G3b. He had a myocardial infarction 6 months ago with subsequent placement of two drug-eluting stents. His hemoglobin A1c level has decreased from 8.7% to 7.8% with adherence to lifestyle modifications and his basal and prandial insulin regimen. Medications are rosuvastatin, lisinopril, metoprolol, chlorthalidone, low-dose aspirin, and clopidogrel. On physical examination, vital signs are normal. BMI is 29. Other than an S4, the cardiac examination and the remainder of the physical examination are normal. According to the American Diabetes Association, which of the following is the most appropriate hemoglobin A1c goal for this patient? Less than 7% 7% to less than 7.5% 7.5% to less than 8% 8% to less than 9%
A hemoglobin A1c goal of 7.5% to 8% is recommended for older adults with complex medical history and significant comorbidities. This patient is an older adult with a complex/intermediate medical history. The American Diabetes Association (ADA) defines complex/intermediate medical history as the presence of at least three coexisting chronic illnesses serious enough to require medications or lifestyle management and may include arthritis, cancer, heart failure, depression, emphysema, falls, hypertension, incontinence, stage 3 or worse chronic kidney disease, myocardial infarction, and stroke. These patients are expected to have "intermediate life-expectancy," high treatment burden, hypoglycemia vulnerability, and fall risk. The ADA's recommended hemoglobin A1c goal for a patient with similar characteristics and health status is 7.5% to 8%, if this can be achieved without significant hypoglycemia. Tighter glycemic control in this scenario would likely require an escalation in treatment burden with a subsequent increase in risks that may outweigh any potential long-term benefits. Additionally, the patient's reduced kidney function increases his hypoglycemia risk with more intense glycemic goals. The American College of Physicians (ACP) recommends that clinicians avoid targeting an hemoglobin A1c level in patients with a life expectancy less than 10 years due to advanced age (80 years or older), residence in a nursing home, or chronic conditions (such as dementia, cancer, end-stage kidney disease, severe chronic obstructive pulmonary disease, or heart failure) because the harms outweigh the benefits in this population. Intensive glycemic control early in the disease course in patients with type 2 diabetes may decrease the incidence of long-term cardiovascular events, as suggested from the 10-year follow-up from the UK Prospective Diabetes Study (UKPDS). Similar reductions in cardiovascular disease did not occur when tight glycemic control was applied to older adults with long-standing type 2 diabetes with prior cardiovascular events or cardiovascular risk factors in three landmark studies: Action to Control Cardiovascular Risk on Diabetes (ACCORD), Action in Diabetes and Vascular Disease: Preterax and Diamicron Modified Release Controlled Evaluation (ADVANCE), and Veterans Affairs Diabetes Trial (VADT). In addition, subjects in the intensive treatment arms had increased rates of hypoglycemia in all three studies and a higher rate of death in the ACCORD trial. In an older healthy adult with little comorbidity, intact cognitive and functional status, the ADA recommends a hemoglobin A1c goal of less than 7.5% given the expected longer life expectancy for that person. This patient does not meet these criteria. In contrast, the ACP recommends hemoglobin A1c level between 7% and 8% in most patients with type 2 diabetes based on lack of mortality benefit for death or macrovascular events over 5 to 10 years and risk in substantial harms. In patients with complex or poor health, a hemoglobin A1c level less than 8.5% is recommended by the ADA, if it can be achieved without significant hypoglycemia. The ADA defines very complex or poor health requiring residence in a long-term care facility or end-stage chronic illnesses (stage 3-4 heart failure, oxygen-dependent lung disease, chronic kidney disease requiring dialysis, or uncontrolled metastatic cancer), moderate to severe cognitive impairment, or two or more activity of daily living dependencies. Hemoglobin A1c targets greater than 8.5% are generally not recommended due to an increased risk for hyperglycemia-related complications, such as dehydration from glycosuria, infections, and hyperglycemic hyperosmolar syndrome.
A 55-year-old man is evaluated in the emergency department for hematemesis. He has had intermittent epigastric discomfort for the past few weeks that does not vary with meals. He does not smoke cigarettes, drink alcohol, or take any medications. On physical examination, temperature is 37 °C (98.6 °F), blood pressure is 118/68 mm Hg without postural changes, pulse rate is 110/min, and respiration rate is 18/min. Physical examination findings are unrevealing. Laboratory studies show a hemoglobin level of 8.0 g/dL (80 g/L), platelet count of 130,000/µL (130 × 109/L), and prothrombin time of 10 s. Crystalloid resuscitation is initiated, and the patient undergoes upper endoscopy, which reveals an 8-mm duodenal ulcer with a clean ulcer base without active arterial bleeding. Which of the following is the most appropriate management? Continue crystalloid resuscitation Packed red blood cell transfusion Plasma transfusion Platelet transfusion
A hemoglobin transfusion threshold of less than 7 g/dL (70 g/L) for hemodynamically stable patients is associated with less blood use and lower mortality compared with a more liberal hemoglobin transfusion threshold of 9 g/dL (90 g/L). This patient should continue to receive crystalloid resuscitation. In the absence of symptoms of anemia, hemodynamic instability, or massive bleeding, a patient such as this one does not require a blood transfusion. A randomized controlled trial of 900 patients with acute upper gastrointestinal bleeding and a hemoglobin level less than 12 g/dL (120 g/L) compared a restrictive transfusion strategy (hemoglobin transfusion threshold of 7 g/dL [70 g/L]) to a liberal strategy (hemoglobin transfusion threshold of 9 g/dL [90 g/L]). Half of the patients had peptic ulcer disease as the source of bleeding. Mortality was lower in the restrictive group compared with the liberal transfusion group (5% versus 9%). Half of the patients in the restrictive group did not receive transfusion at all. Therefore, crystalloid volume resuscitation is appropriate and erythrocyte transfusion unnecessary in this patient. Plasma transfusion is not indicated in the absence of coagulopathy. Fresh frozen plasma (FFP) contains all of the coagulation factors. FFP use is excessive in the United States and other countries despite published guidelines regarding its use. FFP is ineffective at treating mild coagulopathies. Indications for the use of FFP and other plasma-based products include treatment or prevention of coagulopathy from massive transfusion, bleeding associated with multiple acquired clotting factor deficiencies (liver disease, disseminated intravascular coagulation), and major warfarin-associated hemorrhage when a four-factor prothrombin complex concentrate is not available. There is no reason to suspect that this patient has a coagulopathy requiring replacement of clotting factors. Platelet counts greater than 50,000/µL (50 × 109/L) do not warrant platelet transfusion in most situations when thrombocytopenia is the only hemostatic defect. The exception is neurosurgical patients, in whom platelet counts of approximately 100,000/µL (100 × 109/L) should be maintained.
A 38-year-old man is evaluated for epigastric discomfort and early satiety associated with an unintentional 4.5-kg (9.9-lb) weight loss over the preceding 5 months. His family history includes lobular breast cancer diagnosed in his mother at age 45 years, stomach cancer diagnosed in his maternal grandfather at age 48 years, and stomach cancer diagnosed in his maternal uncle at age 52 years. The patient's medical history is unremarkable, and he takes no medication. On physical examination, vital signs are normal. Abdominal examination shows epigastric tenderness to palpation and normal bowel sounds. Which of the following is the most appropriate diagnostic test to perform next? Colonoscopy Gastric emptying study Helicobacter pylori serology Upper endoscopy Upper gastrointestinal radiograph series
A history of multiple family members with gastric cancer, particularly before age 50 years, or multiple family members with lobular breast cancer with or without gastric cancer, suggest the possibility of hereditary diffuse gastric cancer and the need for upper endoscopy and testing for mutations of the CDH1 gene. Upper endoscopy is the most appropriate next test for this patient. The patient's presentation is suggestive of gastric cancer, specifically diffuse gastric cancer. Proposed criteria for selection of patients for genetic testing for hereditary diffuse gastric cancer include the following: family members with two or more documented cases of gastric cancer in first- or second-degree relatives, with at least one diffuse gastric cancer diagnosed before age 50 years; family members with multiple lobular breast cancers with or without diffuse gastric cancer in first- or second-degree relatives; and, a personal diagnosis of diffuse gastric cancer before age 35 years from a low-incidence population such as in Canada and the United States. Based on the patient's young age and history of multiple family members with gastric and lobular breast cancer, hereditary diffuse gastric cancer is a likely diagnosis. The best diagnostic test for gastric cancer is an upper endoscopy with multiple biopsies of the stomach. The syndrome of hereditary diffuse gastric cancer is associated with mutations in the CDH1 gene. The risk for diffuse gastric cancer approaches 80% in carriers of the mutation, and prophylactic gastrectomy is recommended in mutation carriers who have not developed gastric cancer. A colonoscopy is not indicated as the initial test because the patient has primarily upper gastrointestinal symptoms. Colon polyps and cancer have been associated with hereditary diffuse gastric cancer, and more frequent screening colonoscopy for colon polyps is indicated in mutation carriers. Delayed gastric emptying can present with abdominal pain and early satiety but is also often accompanied by nausea and vomiting. The patient has no risk factors for gastroparesis, such as diabetes mellitus, so a gastric emptying study is not indicated. While Helicobacter pylori can present with dyspeptic symptoms, serological testing is not the most appropriate next step for this patient because his weight loss and family history raise concern for gastric cancer. An upper gastrointestinal radiograph series might show thickened gastric folds related to diffuse gastric cancer, but it cannot be used to obtain biopsies to diagnose cancer.
A 52-year-old woman is hospitalized for rapidly progressive weakness in the arms and respiratory muscle weakness that necessitates ventilatory support. Cerebrospinal fluid analysis reveals a cell count of 3/µL (3 × 106/L) and a protein level of 190 mg/dL (1900 mg/L). Electromyographic findings, including those from a nerve conduction study, reveal slowing of nerve conduction. Guillain-Barré syndrome is diagnosed and treated with a standard 10-day course of plasma exchange; at the end of treatment, motor strength is improved by 50%, and she is extubated. On subsequent physical examination, vital signs are normal. There is full range of motion in all extremities. The patient's strength has improved in the upper and lower extremities, but residual weakness persists. Ambulation is possible only with assistance because of weakness. Deep tendon reflexes are absent throughout. The cranial nerves are intact. Which of the following is the most appropriate next step in management? Continue hospital monitoring for another week Discharge to rehabilitation Repeat plasma exchange Treat with intravenous glucocorticoids Treat with intravenous immune globulin
A hospitalized patient with Guillain-Barré syndrome who exhibits improvement after an initial course of plasma exchange should be discharged to rehabilitation.
A 52-year-old woman is hospitalized for rapidly progressive weakness in the arms and respiratory muscle weakness that necessitates ventilatory support. Cerebrospinal fluid analysis reveals a cell count of 3/µL (3 × 106/L) and a protein level of 190 mg/dL (1900 mg/L). Electromyographic findings, including those from a nerve conduction study, reveal slowing of nerve conduction. Guillain-Barré syndrome is diagnosed and treated with a standard 10-day course of plasma exchange; at the end of treatment, motor strength is improved by 50%, and she is extubated. On subsequent physical examination, vital signs are normal. There is full range of motion in all extremities. The patient's strength has improved in the upper and lower extremities, but residual weakness persists. Ambulation is possible only with assistance because of weakness. Deep tendon reflexes are absent throughout. The cranial nerves are intact. Which of the following is the most appropriate next step in management? Continue hospital monitoring for another week Discharge to rehabilitation Repeat plasma exchange Treat with intravenous glucocorticoids Treat with intravenous immune globulin
A hospitalized patient with Guillain-Barré syndrome who exhibits improvement after an initial course of plasma exchange should be discharged to rehabilitation. This patient with Guillain-Barré syndrome (GBS) who exhibits improvement after an initial course of plasma exchange should be discharged to rehabilitation. A diagnosis of GBS, an acute inflammatory demyelinating polyradiculoneuropathy, is supported by prodromal systemic infection, ascending weakness, radicular symptoms, cerebrospinal fluid albuminocytologic dissociation (elevated protein, normal leukocyte count), demyelinating changes (reduced conduction velocity) on a nerve conduction study, and areflexia. GBS is a monophasic disease with weakness reaching its nadir in less than 4 weeks. Given that this patient has already passed the nadir of weakness and is stable from a respiratory and autonomic standpoint, she should be transferred to a rehabilitation center. Rates of full recovery can be slow, but 80% of patients resume ambulation by 6 months. In a patient with GBS who is already extubated and whose motor strength has improved by 50%, continued monitoring in the hospital is unnecessary and would not be cost-effective high value care. Additionally, early initiation of an active rehabilitation program would reduce the risk of secondary weakness caused by decompensation. Repeating plasma exchange is indicated in GBS only if symptoms get worse after initial improvement or stabilization. This patient has experienced no deterioration. Treatment with intravenous glucocorticoids is contraindicated in GBS and may worsen the clinical outcome. Although intravenous immune globulin (IVIG) has the same efficacy as plasma exchange in shortening the time to recovery and reducing the duration of ventilation in GBS, serial treatment with IVIG after plasma exchange is not superior to either treatment alone and is not recommended. An exception would be if symptoms deteriorate after initial improvement or stabilization.
A 21-year-old woman is evaluated during a follow-up visit for a 1-year history of systemic lupus erythematosus. At the time of diagnosis, she presented with a malar rash and arthritis, along with positive antinuclear and anti-double-stranded DNA antibodies. Medications are hydroxychloroquine, low-dose prednisone, calcium, and vitamin D. She currently feels well and is asymptomatic. Vital signs are normal, and the physical examination is unremarkable. Laboratory studies: C3 40 mg/dL (400 mg/L) C4 8 mg/dL (80 mg/L) Anti-double-stranded DNA antibodies Positive (titer: 1:320) Urinalysis 2+ blood; 2+ protein; dysmorphic erythrocytes; no casts Urine protein-creatinine ratio 600 mg/g Kidney ultrasound shows kidneys of normal size and echogenicity. Which of the following is the most appropriate next step in management? Begin pulse glucocorticoids followed by cyclophosphamide Begin pulse glucocorticoids followed by mycophenolate mofetil Increase oral prednisone dose and add mycophenolate mofetil Schedule a kidney biopsy
A kidney biopsy should be performed in patients with known systemic lupus erythematosus with suspected significant kidney involvement to establish the diagnosis and to identify the class, which will guide treatment decisions. The most appropriate next step is a kidney biopsy. The presence of hematuria, dysmorphic erythrocytes, and proteinuria in a patient with known systemic lupus erythematosus (SLE) is highly suggestive of lupus nephritis; when serologies are positive and serum complement levels are low, this diagnosis is even more likely. Commonly cited indications for kidney biopsy include increasing serum creatinine without explanation, proteinuria >500 mg/24 h, or active urine sediment (dysmorphic erythrocytes, erythrocyte casts). Patients meeting these criteria are more likely to have focal or diffuse proliferative lupus nephritis or lupus membranous nephropathy requiring immunosuppressive therapy. Patients with proteinuria <500 mg/24 h and inactive urine sediment are more likely to have milder kidney involvement and may be followed with urinalysis, urine protein-creatinine ratio, and serum creatinine every 3 to 6 months. Unless absolutely contraindicated (for example, bleeding diathesis or inability to stop anticoagulation), both rheumatology and nephrology guidelines support performing a kidney biopsy in patients with SLE who develop evidence of significant kidney involvement to establish the diagnosis and, of equal importance with regard to treatment decisions, to identify the International Society of Nephrology/Renal Pathology Society (ISN/RPS) class of lupus nephritis. This patient's kidney biopsy could show a proliferative lupus nephritis (class III or IV), in which case treatment with glucocorticoids and mycophenolate mofetil or cyclophosphamide would be indicated (mycophenolate mofetil would likely be preferred given this patient's age). However, her biopsy could also conceivably show a milder, mesangial proliferative lupus nephritis (class II) or membranous lupus nephritis (class V), neither of which would require immunosuppression at this stage. Therefore, empiric therapy in this setting is substandard to biopsy-guided therapy according to ISN/RPS class of nephritis.
A 32-year-old woman is evaluated for a 2-year history of dry eyes and dry mouth. She wakes up in the morning with a feeling like something is in her eyes. She cannot eat dry foods without consuming large amounts of water. She has noticed a change in the shape of her face; it has gotten more round. She previously saw an ophthalmologist who found abnormal ocular surface staining consistent with dry eyes. She uses artificial tears as needed. On physical examination, vital signs are normal. There is no pooled saliva under the tongue. Bilateral parotid and lacrimal enlargement is present. The remainder of the examination is normal. Antinuclear antibodies, anti-Ro/SSA antibodies, and rheumatoid factor are negative. Screening for HIV and hepatitis B and C is negative. Serum IgG levels are normal. Chest radiograph is normal. Which of the following is the most appropriate diagnostic test to perform next? CT of the chest Lip biopsy Parotid biopsy Schirmer test Sialography
A lip biopsy should be considered in a patient with sicca and suspected Sjögren syndrome when initial serologic evaluation is uninformative. The most appropriate diagnostic test to perform next is a lip biopsy. This patient has parotid and lacrimal enlargement and sicca. The differential diagnosis of sicca with parotid/lacrimal enlargement includes Sjögren syndrome, sarcoidosis, IgG4-related disease, HIV-associated diffuse lymphocytic infiltrate syndrome, and others. The American College of Rheumatology/European League Against Rheumatism has proposed diagnostic criteria for Sjögren syndrome. The most heavily weighted criteria include focal lymphocytic sialadenitis in labial salivary gland biopsy and the presence of anti-Ro/SSA antibodies. The presence of both will establish the diagnosis. Alternatively, the diagnosis can be established with one of the two major criteria and an abnormal ocular staining score of ≥5, abnormal Schirmer test, or abnormal whole saliva flow test. In addition, entities in the differential diagnosis should be excluded. The patient meets the criteria for keratoconjunctivitis sicca, but because of the absence of anti-Ro/SSA antibodies, additional support is needed. A lip biopsy to assess minor salivary gland pathology is relatively noninvasive; it can provide evidence sufficient to meet the proposed diagnostic criteria and might assist in ruling out other entities in the differential diagnosis. CT of the chest might identify findings to support an alternative diagnosis. For example, findings of hilar lymphadenopathy or pulmonary nodules might support a diagnosis of sarcoidosis. However, the yield would be low given the normal chest radiograph. Like lip biopsy, parotid biopsy can provide evidence of Sjögren syndrome; however, it is more invasive and risks seventh cranial nerve damage. Its use is currently limited to special situations. A Schirmer test could conveniently confirm dry eyes, but dry eyes have already been confirmed by a more rigorous and reproducible ophthalmologic examination. Sialography can assess the extent of mouth dryness, but the diagnosis of sicca has already been sufficiently confirmed. Read Related TextNext Question
A 47-year-old man is evaluated for intermittent episodes of anxiety, diaphoresis, and palpitations. These episodes occur approximately 4 hours after a meal. His first episode occurred while at work. A fingerstick blood glucose reading performed by the employee health nurse was 48 mg/dL (2.7 mmol/L). His symptoms resolved with juice. He has had three similar episodes over the last month. He has increased his consumption of snacks between meals in an attempt to avoid repeat episodes. He denies use of over-the counter supplements or illicit drug use. He does not consume alcohol. He is asymptomatic today. Medical history is significant for Roux-en-Y gastric bypass surgery 3 years ago. He takes no glucose-lowering medications. On physical examination, vital signs are normal. BMI is 31. The remainder of his examination is normal. His fingerstick blood glucose level is 85 mg/dL (4.7 mmol/L). Laboratory studies show a hemoglobin A1c level of 5%. All other laboratory results are normal. Which of the following is the most appropriate diagnostic test to perform next? 72-Hour fast Mixed-meal testing Oral glucose tolerance test Pancreatic imaging
A mixed-meal test consisting of the types of food that normally induce the hypoglycemia should be performed to determine the cause of postprandial hypoglycemia. The most appropriate diagnostic test to perform next is mixed-meal testing. Hypoglycemia in persons without diabetes is extremely rare and requires additional investigation. Causes of hypoglycemia in adults without diabetes include drug or alcohol use, critical illness, hormonal deficiency, non-islet cell tumor, endogenous hyperinsulinism, accidental or intentional hypoglycemia, and prior Roux-en-Y gastric bypass surgery. Postprandial hypoglycemia can develop 2 to 3 years after Roux-en-Y gastric bypass surgery. The patient either denies or does not have clinical evidence for the presence of many of these causes. The timing of the hypoglycemia can guide the appropriate next diagnostic test. This patient is experiencing postprandial symptoms consistent with an increase in sympathetic activity within 5 hours of a meal. He meets two out of three criteria for a hypoglycemic disorder as defined by the Whipple triad: symptoms that can be attributed to hypoglycemia and resolution of symptoms with food consumption. He does not have documented hypoglycemia at the time of symptoms from a reliable laboratory method to meet the third criteria of the Whipple triad. Point-of-care fingerstick measurements lack precision to confirm hypoglycemia. An attempt should be made to recreate the scenario that is likely to cause the symptoms. Since his symptoms occur in the postprandial state, a mixed-meal test should be performed. The mixed-meal consists of food types that have induced the onset of symptoms in the past. The patient should have the following hypoglycemic laboratory tests measured at baseline and every 30 minutes for 5 hours after consuming the mixed-meal: plasma glucose, insulin, C-peptide, β-hydroxybutyrate (low in the presence of insulin), and proinsulin. He should also repeat these tests at the time of symptomatic hypoglycemia (<60 mg/dL [3.3 mmol/L]) before administering carbohydrates. If symptomatic hypoglycemia is documented during the mixed-meal testing, the patient should also be screened for insulin secretagogues (sulfonylureas and meglitinides) and insulin antibodies. A 72-hour fast involves prolonged fasting while measuring the hypoglycemic laboratory test (plasma glucose, insulin, C-peptide, β-hydroxybutyrate, and proinsulin) every 6 hours, followed by every 1 to 2 hours once the glucose is less than 60 mg/dL (3.3 mmol/L) or the patient becomes symptomatic. Given that the patient is experiencing postprandial symptoms, the 72-hour fast would likely not induce hypoglycemia. An oral glucose tolerance test (OGTT) consists of liquid caloric consumption that is usually not similar to the type of food that induces symptomatic postprandial hypoglycemia. The OGTT can produce unreliable results, particularly in those patients with altered gastric anatomy or gastric motility. Pancreatic imaging should only occur after biochemical confirmation of endogenous hyperinsulinism.
A 43-year-old woman is evaluated for a 3-day history of left knee pain and swelling. Similar episodes have occurred in either knee as well as the left wrist during the past 2 years. She also reports a sense of generalized weakness, and has experienced constipation and vague abdominal discomfort during the past year. Because of her constipation, her thyroid-stimulating hormone level was recently measured and found to be normal. She takes no medications. On physical examination, vital signs are normal. The left knee is warm with a moderate- to large-sized effusion, and decreased range of motion is noted. Proximal muscle strength is normal. Radiographs show a thin white line at the chondral surfaces of both knees and at the pubic symphysis. Joint aspiration of the left knee shows a leukocyte count of 35,000/µL (35 × 109/L), with 90% neutrophils; polarizing microscopy shows numerous positively birefringent rhomboid crystals within neutrophils. Which of the following laboratory studies will most likely identify the cause of this patient's symptoms? Anti-cyclic citrullinated peptide antibodies Serum calcium Serum creatine kinase Serum urate
A serum calcium measurement is most likely to identify the cause of this patient's symptoms. This young woman has recurrent attacks of acute calcium pyrophosphate (CPP) crystal arthritis (pseudogout). Clues to the diagnosis include acute and recurrent self-limited monoarticular arthritis, chondrocalcinosis on radiograph, and, most tellingly, identification of CPP crystals under polarizing microscopy. This condition usually affects the elderly; therefore, evidence of acute CPP crystal arthritis in a young person should always prompt an investigation for secondary causes. Secondary causes include hyperparathyroidism, hypothyroidism, hypophosphatasia, hypomagnesemia, and hemochromatosis. This patient has recent-onset signs and symptoms suggestive of hypercalcemia and hyperparathyroidism, including abdominal discomfort, constipation, and weakness. Therefore, measuring the serum calcium is appropriate (and serum parathyroid hormone, if hypercalcemia is present), because this patient appears to have CPP deposition due to hyperparathyroidism. Anti-cyclic citrullinated peptide antibodies are useful in the diagnosis of rheumatoid arthritis (RA). RA typically presents with the insidious onset of symmetric arthritis in the hands and feet along with morning stiffness. Monoarthritis is a rare presenting feature of RA, but it is usually followed by development of the symmetric arthritis. Checking serum creatine kinase would not be a first-line test for this patient. Although muscle disease is associated with weakness, myositis would not account for many of the other manifestations, including the monoarticular arthritis, radiographic chondrocalcinosis, and synovial fluid crystals. Serum urate would be an appropriate laboratory study to measure if gout was suspected, but the synovial fluid analysis is consistent with acute CPP crystal arthritis rather than gout, in which negatively birefringent crystals are seen. Gout also more typically affects the metatarsophalangeal joints, the mid foot, ankles, and knees, although the finger joints may be affected in postmenopausal women.
A 28-year-old man is evaluated for a 1-month history of right testicular swelling. His medical history is unremarkable, and he takes no medications. On physical examination, vital signs are normal. Palpation reveals an enlarged right testicle that is firm and mildly tender. Scrotal ultrasonography shows a 5-cm right testicular mass. Which of the following is the most appropriate management? Measurement of β-human chorionic gonadotropin and α-fetoprotein levels Needle biopsy PET-CT scan Radical inguinal orchiectomy
Before histologic confirmation of testicular cancer, α-fetoprotein and β-human chorionic gonadotropin levels should be measured to most accurately determine stage and prognosis. The most appropriate management is to measure β-human chorionic gonadotropin and α-fetoprotein levels. In the setting of unilateral testicular swelling or a palpable testicular mass in a young man, cancer is a diagnosis of exclusion. This patient has unilateral testicular swelling. Physical examination and ultrasonography both point to the presence of a testicular mass. Although extrinsic processes can cause testicular swelling, ultrasonography is very accurate in differentiation of intrinsic from extrinsic testicular pathology. In a patient found to have a testicular mass, the diagnosis of cancer must be confirmed histologically. Germ cell tumors are the most common type of testicular cancer. These tumors can be pure seminomas or nonseminomas. However, before histologic confirmation, α-fetoprotein and β-human chorionic gonadotropin levels should be measured to most accurately determine stage and prognosis. The serum α-fetoprotein level is never elevated in patients with pure seminomas, and β-human chorionic gonadotropin is elevated in only approximately 20% of patients with pure seminomas. Nonseminomatous germ cell tumors can contain elements of seminoma, but those elements are mixed with tumors with nonseminomatous histologies, which include yolk sac tumor, choriocarcinoma, and embryonal carcinoma. This distinction is important for determining the correct treatment. However, these measurements do not determine diagnosis and are not a substitute for histologic evaluation. Because levels will fall after orchiectomy, measurement before surgery is recommended. Failure of the levels to fall suggests inadequate therapy, and elevation of the levels after treatment suggests recurrent disease. Needle biopsy is contraindicated in the evaluation of testicular masses owing to the risk of biopsy tract seeding. Chest radiography and CT of the abdomen and pelvis are done after a diagnosis is established. CT of the chest is indicated if the chest radiograph is abnormal or lung metastases are strongly suspected. PET-CT has little role in the evaluation of men with confirmed testicular cancer in part because of a high rate of false-positive results. It would not be indicated before establishing a diagnosis. Radical inguinal orchiectomy is essential for establishing a diagnosis of testicular cancer in the most expeditious and safest way possible. However, it should be performed after tumor markers are measured.
A 23-year-old man is evaluated for a 2-day history of erythema and drainage from a surgical incision site. He reports no fever or chills. He underwent a left inguinal hernia repair 10 days ago. Medical history is otherwise noncontributory, and he takes no medications. On physical examination, vital signs are normal. The left inguinal incision is erythematous around the edges, is tender, and exudes a small amount of serosanguinous, cloudy drainage from a small area of incision breakdown. No induration is noted around the incision area. Laboratory studies show a leukocyte count of 8700/µL (8.7 × 109/L). Which of the following is the most appropriate diagnostic test to perform next? Blood culture CT of upper thigh and pelvis Gram stain and culture of incision site drainage Gram stain and culture of incision site swabbing
A superficial incisional infection involves the underlying soft tissue and presents with inflammatory changes at the incision site (erythema, tenderness), with or without purulent drainage, and few if any systemic signs of infection such as fever; therapy is guided by Gram stain and culture of the wound. Gram stain and culture of the drainage from the incision site is the most appropriate test to perform next. Most surgical site infections (SSIs) occur within 30 days of surgery (90 days for surgery involving placement of an implant). The major sources of organisms causing SSIs are from the patient's skin and possibly the alimentary tract or female genital tract, depending on the type of surgery. The organism most often isolated is Staphylococcus aureus, followed by coagulase-negative staphylococci, Escherichia coli, Enterococcus faecalis, and Pseudomonas aeruginosa. SSIs are categorized as superficial incisional, deep incisional, and organ/deep organ space infections. A superficial incisional infection involves the underlying soft tissue and presents with inflammatory changes at the incision site (erythema, tenderness), with or without purulent drainage, and few if any systemic signs of infection, such as fever. Such incisions may require reopening to determine the extent of infection, allow complete drainage, and obtain proper specimens for Gram stain and culture to guide antibiotic therapy. Obtaining a culture is important to identify the pathogen involved and obtain antibiotic sensitivity information to determine if antibiotic-resistant organisms are present. Wound drainage fluid, purulent fluid, or infected tissue is the best culture source. Deep tissue or wound cultures are preferable to superficial wound swab cultures, which are more likely to reflect skin or wound colonization and do not necessarily yield the causative pathogen. The most narrow-spectrum oral antibiotic should be used whenever possible. Deep incisional (involving fascia and/or muscle layers) SSIs usually present with some systemic signs of infection, such as fever and leukocytosis. These infections are managed with debridement and antibiotic therapy guided by results of deep-tissue cultures. Bacteremia may occur with deep or organ space infections, and blood cultures should be considered in such circumstances but are unnecessary in patients with a superficial site infection such as this one. CT is useful in cases of organ or deep space (tissue deep to the fascia) SSIs to identify abscesses and plan necessary drainage procedures. Imaging, such as ultrasonography or CT, does not provide additional information needed to manage this type of SSI. A superficial swab of the incision site is likely to pick up skin flora and make interpreting the culture results difficult.
A 26-year-old woman is evaluated for a changing mole on her left lower leg. She says it is getting bigger and darker. The patient is 5 weeks pregnant. Her only medication is a prenatal vitamin. On physical examination, vital signs are normal. There is an 8-mm black and gray asymmetric papule with irregular borders on her left medial calf. Which of the following is the most appropriate next step in management? Biopsy the lesion now Biopsy after delivery Cryotherapy Provide reassurance
A suspicious melanocytic lesion should be biopsied promptly during pregnancy because dermatologic surgery under local anesthesia is safe during pregnancy; a diagnosis of melanoma should not be delayed. The patient has a changing mole that is very suspicious for melanoma on her left leg. It needs to be biopsied promptly as not to delay the diagnosis of melanoma. The skin biopsy is an important part of the diagnostic work-up for melanoma. Ideally, an atypical pigmented lesion should be removed using an excisional biopsy, in which the entire lesion is removed. Since this is not always practical or possible, additional options are sometimes used. These include punch biopsy, in which a sample of the lesion is removed using a cylindrical blade and sent for analysis. Although this type of biopsy generally provides a sample of adequate depth, there is concern about sampling error given that the area removed may not necessarily be representative of the remainder of the lesion. Similarly, although shave biopsies are often readily available and easy to perform, this technique carries the risk of transecting the melanoma if a deep enough sample is not taken. This would result in an underestimate of the true depth of the lesion, which could have implications regarding further work-up and management. There is no reason to delay the biopsy in a suspicious melanocytic lesion during pregnancy. Lidocaine is classified by the FDA as pregnancy category B and can be safely used as local anesthesia during pregnancy. It is controversial to use epinephrine as it is FDA classified as category C. Although no adverse events have been documented in humans, in vitro studies of human uterine arteries have shown that in high doses, epinephrine can cause spasms and reduce flow through the uterine artery. Obtaining a biopsy from a changing mole in a pregnant woman should be done promptly as not to delay the diagnosis of melanoma. It is not appropriate to treat a highly suspicious lesion with cryotherapy. The depth of invasion (Breslow depth) of a melanoma is the most important prognostic feature and correlates most strongly with the risk of recurrence and metastasis. Destruction of a pigmented lesion removes the possibility of making a proper diagnosis and prognosis for a potential melanoma. Providing reassurance and thereby delaying diagnosis of melanoma could result in poor prognosis for the mother and child. Malignant melanoma is the most common malignancy during pregnancy. Studies show that changes in size in nevi during pregnancy occur on the front of the body owing to the stretching of the skin as the pregnancy progresses. Nevi on other locations, such as the leg, typically do not change during pregnancy, and this development is a red flag for possible melanoma.
A 24-year-old woman is evaluated for a 4-day history of a worsening pruritic rash on the chin. She is a kindergarten teacher. Medical history is unremarkable, and she takes no medications. On physical examination, vital signs are normal. Skin findings are shown. Which of the following is the most appropriate treatment? Oral doxycycline Topical hydrocortisone Topical mupirocin Topical neomycin/polymyxin B/bacitracin
A topical antibacterial agent, such as mupirocin, is the first-line therapy for localized impetigo. This patient has nonbullous impetigo, and the most appropriate treatment is topical mupirocin. Impetigo is most commonly caused by Staphylococcus aureus or beta-hemolytic streptococci. It is very contagious and most commonly seen in children. Impetigo can be either nonbullous or bullous impetigo. Bullous impetigo is a toxin-mediated process usually caused by production of an exfoliative toxin by S. aureus, which induces erythema and loss of the superficial layer of the epidermis. Nonbullous impetigo is the more common type and often affects the face or extremities. It appears as erythematous papules or pustules that rupture and then progress to erosions with overlying honey-colored crust. The diagnosis of impetigo often can be made based on clinical presentation; however, culture of the honey-colored crust can confirm the pathologic organism and obtain sensitivity testing, which is important when treating extensive disease or staphylococcal scalded skin syndrome. This patient has only a few lesions, so the best treatment would be topical mupirocin. Mupirocin is a topical antibiotic that inhibits bacterial protein synthesis. It has good activity against both S. aureus and beta-hemolytic streptococci. There has been limited reported antibiotic resistance to mupirocin, and it is usually well tolerated. Washes with chlorhexidine and diluted bleach baths also can be used. Oral doxycycline would be a treatment for patients with suspected or confirmed methicillin-resistant Staphylococcus aureus (MRSA) skin and soft tissue infection. Oral antibiotic therapy increases antibiotic resistance and has more potential side effects compared with topical therapy, so oral doxycycline would not be the most appropriate choice. Oral antibiotics should be reserved for patients with more extensive infection, involvement of deeper skin structures, or systemic symptoms such as fever. Cephalexin, dicloxacillin, erythromycin, amoxicillin-clavulanate, and clindamycin are the preferred initial systemic treatments for extensive impetigo. Topical hydrocortisone is not appropriate to treat impetigo caused by S. aureus or beta-hemolytic streptococci. It may be used for eczematous dermatitis, but not for impetigo. Topical neomycin/polymyxin B/bacitracin does have some activity again S. aureus and streptococci, but it is not as effective as mupirocin. Also, there is a higher risk of allergic contact dermatitis with topical antibiotics containing neomycin and bacitracin, so this would not be the most appropriate treatment in this patient. Read Related TextNext Question
A 36-year-old woman is evaluated for dry cough and progressive dyspnea that limits her ability to exercise. She initially presented 8 weeks ago with cough, fever, sputum production, and dyspnea. A chest radiograph at that time revealed left-lower-lobe opacities; she was diagnosed with pneumonia and treated with azithromycin but had little improvement in her symptoms. Her fever and sputum production have resolved. She is a nonsmoker. On physical examination, vital signs are normal. Lungs are clear to auscultation. Repeat chest radiograph reveals patchy opacities bilaterally and several nodular densities that are peripherally predominant in different locations than previous radiographs. High-resolution CT scan of the chest shows extensive ground-glass changes bilaterally with several areas of nodular consolidation that are peripherally predominant and along bronchovascular bundles. Which of the following is the most likely diagnosis? Acute HIV infection Community-acquired pneumonia Cryptogenic organizing pneumonia Idiopathic pulmonary fibrosis
A typical presentation of cryptogenic organizing pneumonia includes cough, fever, and malaise for 6 to 8 weeks that does not respond to antibiotics; patchy opacities on chest radiograph; and ground-glass opacities on CT scan that are peripherally distributed; glucocorticoids are first-line therapy.
A 36-year-old woman is evaluated for dry cough and progressive dyspnea that limits her ability to exercise. She initially presented 8 weeks ago with cough, fever, sputum production, and dyspnea. A chest radiograph at that time revealed left-lower-lobe opacities; she was diagnosed with pneumonia and treated with azithromycin but had little improvement in her symptoms. Her fever and sputum production have resolved. She is a nonsmoker. On physical examination, vital signs are normal. Lungs are clear to auscultation. Repeat chest radiograph reveals patchy opacities bilaterally and several nodular densities that are peripherally predominant in different locations than previous radiographs. High-resolution CT scan of the chest shows extensive ground-glass changes bilaterally with several areas of nodular consolidation that are peripherally predominant and along bronchovascular bundles. Which of the following is the most likely diagnosis? Acute HIV infection Community-acquired pneumonia Cryptogenic organizing pneumonia Idiopathic pulmonary fibrosis
A typical presentation of cryptogenic organizing pneumonia includes cough, fever, and malaise for 6 to 8 weeks that does not respond to antibiotics; patchy opacities on chest radiograph; and ground-glass opacities on CT scan that are peripherally distributed; glucocorticoids are first-line therapy. The most likely diagnosis is cryptogenic organizing pneumonia. This patient has a subacute history of progressive dyspnea and persistent cough after an initial history consistent with community-acquired pneumonia. Despite treatment with appropriate antibiotic therapy, she now has clear evidence of new opacities that are located in different areas, are peripherally predominant, and coalesce along broncho-vascular bundles. These findings are consistent with a diagnosis of organizing pneumonia, which involves proliferation of granulation tissue within alveolar ducts, alveolar spaces, and surrounding areas of chronic inflammation. There are many known causes of this pattern, including acute infections and autoimmune disorders like rheumatoid arthritis. The term cryptogenic organizing pneumonia is reserved for individuals who have this pattern but do not have a clear associated cause. Patients with cryptogenic organizing pneumonia will typically present with a 6-to-8-week history of symptoms that mimic community-acquired pneumonia. Typically, an initial empiric treatment of infection is given but fails; subsequently, noninfectious causes are considered. This patient has a subacute illness that began with viral symptoms, persistent and progressive cough and dyspnea that are not responsive to past antibiotics, and radiographic findings typical of cryptogenic organizing pneumonia. Patients with cryptogenic organizing pneumonia respond well to glucocorticoids. Glucocorticoids are slowly tapered during the subsequent 6 months. Acute HIV is unlikely in this patient who initially presented with symptoms of community-acquired pneumonia and whose fever has resolved. Although acute HIV symptoms are not specific, the most common presentation includes persistence of symptoms including fever. Recurrent community-acquired pneumonia is also unlikely in this patient who was appropriately treated with azithromycin and whose symptoms have partially improved with resolution of the fever and sputum production. Idiopathic pulmonary fibrosis (IPF) is a disease that affects older patients (the mean age of presentation is in the mid- to late 60s) and presents with chronic (longer than 6 months) symptoms of dry cough and shortness of breath. This patient's presentation is not consistent with these criteria, making a diagnosis of IPF unlikely.
A 48-year-old woman is evaluated for edema, dyspnea, and proteinuria. She has a 10-year history of rheumatoid arthritis. She was initially treated with NSAIDs, methotrexate, and prednisone. She stopped taking the NSAIDs 6 months ago because of gastritis. Three months ago she began noticing swelling of her legs, and two weeks ago she began experiencing dyspnea when walking. She has never received penicillamine, gold, or a biologic agent. Current medications are methotrexate, prednisone, and omeprazole. On physical examination, the patient appears chronically ill. Temperature is 37.2 °C (99.0 °F), blood pressure is 158/90 mm Hg, pulse rate is 96/min, and respiration rate is 20/min. Jugular venous pressure is elevated. Cardiac examination reveals a summation gallop. The lungs are clear. Swelling and tenderness at the metacarpophalangeal joints and wrists are noted. There is 3+ lower extremity edema to the knees. Laboratory studies: Albumin 2.8 g/dL (28 g/L) Creatinine 1.1 mg/dL (97.2 µmol/L) Serum protein electrophoresis Polyclonal gammopathy Urinalysis Specific gravity 1.0120; pH 5.5; no blood; 4+ protein Urine protein-creatinine ratio 6000 mg/g Electrocardiogram reveals low voltage, pronounced in the limb leads, but without acute changes. Which of the following is the most likely cause of this patient's proteinuria? AA amyloidosis Focal segmental glomerulosclerosis Minimal change glomerulopathy NSAID-associated interstitial nephritis Proton pump-associated interstitial nephritis
AA amyloid is formed by serum amyloid A protein, an acute phase reactant produced in various inflammatory diseases such as rheumatoid arthritis; confirmation of AA renal amyloidosis requires a kidney biopsy.
A 48-year-old woman is evaluated for edema, dyspnea, and proteinuria. She has a 10-year history of rheumatoid arthritis. She was initially treated with NSAIDs, methotrexate, and prednisone. She stopped taking the NSAIDs 6 months ago because of gastritis. Three months ago she began noticing swelling of her legs, and two weeks ago she began experiencing dyspnea when walking. She has never received penicillamine, gold, or a biologic agent. Current medications are methotrexate, prednisone, and omeprazole. On physical examination, the patient appears chronically ill. Temperature is 37.2 °C (99.0 °F), blood pressure is 158/90 mm Hg, pulse rate is 96/min, and respiration rate is 20/min. Jugular venous pressure is elevated. Cardiac examination reveals a summation gallop. The lungs are clear. Swelling and tenderness at the metacarpophalangeal joints and wrists are noted. There is 3+ lower extremity edema to the knees. Laboratory studies: Albumin 2.8 g/dL (28 g/L) Creatinine 1.1 mg/dL (97.2 µmol/L) Serum protein electrophoresis Polyclonal gammopathy Urinalysis Specific gravity 1.0120; pH 5.5; no blood; 4+ protein Urine protein-creatinine ratio 6000 mg/g Electrocardiogram reveals low voltage, pronounced in the limb leads, but without acute changes. Which of the following is the most likely cause of this patient's proteinuria? 71% A AA amyloidosis Focal segmental glomerulosclerosis Minimal change glomerulopathy NSAID-associated interstitial nephritis Proton pump-associated interstitial nephritis
AA amyloid is formed by serum amyloid A protein, an acute phase reactant produced in various inflammatory diseases such as rheumatoid arthritis; confirmation of AA renal amyloidosis requires a kidney biopsy. AA amyloidosis is the most likely cause of this patient's proteinuria. Chronic inflammatory states, particularly rheumatoid arthritis, are associated with production of amyloid protein A. This acute phase reactant deposits in numerous tissues, most commonly the kidney (80%-90%) and heart, forming β-pleated sheets. In the kidney, amyloidosis presents with a bland urinary sediment and nephrotic-range proteinuria. Progression to end-stage kidney disease is frequent. Cardiac manifestations may include systolic or diastolic dysfunction and heart failure; low voltage on electrocardiogram can be seen. Although serum amyloid P component scintigraphy can diagnose amyloid, it is not available in the United States. Confirmation would therefore require a kidney biopsy in this case. Current treatment strategies are aimed at the underlying disease. Tocilizumab, an anti-interleukin-6 antibody, has been used successfully in patients with AA amyloidosis from rheumatoid arthritis. Both focal segmental glomerulosclerosis (FSGS) and minimal change glomerulopathy (MCG) are associated with the nephrotic syndrome. However, FSGS more frequently affects black persons, and MCG is the most common cause of the nephrotic syndrome in children. Finally, this patient's history of a chronic inflammatory disease and findings of heart failure and an abnormal electrocardiogram more strongly suggest the possibility of amyloidosis. NSAIDs are associated with interstitial nephritis and the nephrotic syndrome due to MCG or membranous glomerulopathy; typical findings include hematuria, pyuria, leukocyte casts, proteinuria, and an acute rise in the plasma creatinine concentration. However, this patient has not been using NSAIDs for the past 6 months and her urinalysis is bland, making this diagnosis unlikely. Proton pump inhibitors can cause acute interstitial nephritis but are not associated with nephrotic-range proteinuria.
A 48-year-old woman is evaluated for edema, dyspnea, and proteinuria. She has a 10-year history of rheumatoid arthritis. She was initially treated with NSAIDs, methotrexate, and prednisone. She stopped taking the NSAIDs 6 months ago because of gastritis. Three months ago she began noticing swelling of her legs, and two weeks ago she began experiencing dyspnea when walking. She has never received penicillamine, gold, or a biologic agent. Current medications are methotrexate, prednisone, and omeprazole. On physical examination, the patient appears chronically ill. Temperature is 37.2 °C (99.0 °F), blood pressure is 158/90 mm Hg, pulse rate is 96/min, and respiration rate is 20/min. Jugular venous pressure is elevated. Cardiac examination reveals a summation gallop. The lungs are clear. Swelling and tenderness at the metacarpophalangeal joints and wrists are noted. There is 3+ lower extremity edema to the knees. Laboratory studies: Albumin 2.8 g/dL (28 g/L) Creatinine 1.1 mg/dL (97.2 µmol/L) Serum protein electrophoresis Polyclonal gammopathy Urinalysis Specific gravity 1.0120; pH 5.5; no blood; 4+ protein Urine protein-creatinine ratio 6000 mg/g Electrocardiogram reveals low voltage, pronounced in the limb leads, but without acute changes. Which of the following is the most likely cause of this patient's proteinuria? AA amyloidosis Focal segmental glomerulosclerosis Minimal change glomerulopathy NSAID-associated interstitial nephritis Proton pump-associated interstitial nephritis
AA amyloid is formed by serum amyloid A protein, an acute phase reactant produced in various inflammatory diseases such as rheumatoid arthritis; confirmation of AA renal amyloidosis requires a kidney biopsy. AA amyloidosis is the most likely cause of this patient's proteinuria. Chronic inflammatory states, particularly rheumatoid arthritis, are associated with production of amyloid protein A. This acute phase reactant deposits in numerous tissues, most commonly the kidney (80%-90%) and heart, forming β-pleated sheets. In the kidney, amyloidosis presents with a bland urinary sediment and nephrotic-range proteinuria. Progression to end-stage kidney disease is frequent. Cardiac manifestations may include systolic or diastolic dysfunction and heart failure; low voltage on electrocardiogram can be seen. Although serum amyloid P component scintigraphy can diagnose amyloid, it is not available in the United States. Confirmation would therefore require a kidney biopsy in this case. Current treatment strategies are aimed at the underlying disease. Tocilizumab, an anti-interleukin-6 antibody, has been used successfully in patients with AA amyloidosis from rheumatoid arthritis. Both focal segmental glomerulosclerosis (FSGS) and minimal change glomerulopathy (MCG) are associated with the nephrotic syndrome. However, FSGS more frequently affects black persons, and MCG is the most common cause of the nephrotic syndrome in children. Finally, this patient's history of a chronic inflammatory disease and findings of heart failure and an abnormal electrocardiogram more strongly suggest the possibility of amyloidosis. NSAIDs are associated with interstitial nephritis and the nephrotic syndrome due to MCG or membranous glomerulopathy; typical findings include hematuria, pyuria, leukocyte casts, proteinuria, and an acute rise in the plasma creatinine concentration. However, this patient has not been using NSAIDs for the past 6 months and her urinalysis is bland, making this diagnosis unlikely. Proton pump inhibitors can cause acute interstitial nephritis but are not associated with nephrotic-range proteinuria.
How to prevent DM neuropathy in DM1 patients?
According to high-quality evidence, enhanced glucose control significantly prevents the development of clinical neuropathy and reduces nerve conduction and vibration threshold abnormalities in type 1 diabetes mellitus. Glycemic control can delay progression of neuropathy in type 2 diabetes. Other than glucose control, no other preventive strategies are available for diabetic neuropathy. Hemoglobin A1c goals in patients with diabetes should be individually tailored taking into account the demonstrated benefits with regard to prevention and delay in microvascular complications with the risk of hypoglycemia. A reasonable goal of therapy might be a hemoglobin A1c value less than or equal to 7% for most patients, with higher targets for older adult patients and those with comorbidities or a limited life expectancy and more stringent control for those patients with type 1 diabetes and during pregnancy. The American Diabetes Association advocates for a target systolic blood pressure between 125 and 130 mm Hg in select patients (young, long life expectancy, increased risk of stroke), if this can be accomplished safely. While this patient will benefit from improved blood pressure control by reducing the risk of cardiovascular disease, it will have no impact on her risk for the development of diabetic neuropathy. While lifestyle intervention to improve the lipid profile should be undertaken in all patients with diabetes, for patients without cardiovascular disease and under age 40 years, statin therapy should be considered only in those with multiple cardiovascular disease risk factors; the American College of Cardiology/American Heart Association risk calculator can determine the 10-year atherosclerotic cardiovascular disease risk to guide therapeutic management. Improved lipid control, while indicated and beneficial for the prevention of cardiovascular disease, does not assist in the prevention of diabetic neuropathy. Although pregabalin is indicated to treat painful diabetic neuropathy, it does not prevent diabetic neuropathy. Although weight loss should be discussed as part of a therapeutic lifestyle plan, there is no data to suggest weight loss prevents the development of diabetic neuropathy.
A 66-year-old man is evaluated in the emergency department for chest pain and dyspnea. Three days ago, he experienced the sudden onset of substernal chest pain. His symptoms waxed and waned over a period of 3 hours and then spontaneously subsided. Today, he began experiencing progressive shortness of breath with symptoms occurring at rest. His medical history is significant for hypertension and hyperlipidemia. His medications are hydrochlorothiazide, atenolol, and atorvastatin. On physical examination, he appears distressed. Temperature is normal, blood pressure is 95/50 mm Hg, pulse rate is 115/min, and respiration rate is 24/min. Oxygen saturation is 90% breathing 4 L of oxygen by nasal cannula. Jugular venous pulse is elevated and visible, with exaggerated x and y descents, and the estimated central venous pressure is 12 cm H2O. Bilateral crackles are noted. A soft, early systolic murmur with an S3 is audible in the apical area. An early diastolic rumble is present. An electrocardiogram demonstrates significant Q waves in the inferolateral leads and elevated ST segments. A chest radiograph confirms the presence of heart failure with cardiomegaly and bilateral pulmonary edema. Which of the following is the most likely diagnosis? Aortic stenosis Cardiac tamponade Mitral regurgitation Tricuspid regurgitation
Acute severe mitral regurgitation is associated with papillary muscle rupture following acute myocardial infarction. The most likely diagnosis is acute mitral regurgitation. This patient has evidence of a recent inferolateral myocardial infarction and decompensated heart failure. The mitral regurgitation is likely secondary to papillary muscle dysfunction (or rupture) and/or ventricular dysfunction. In acute mitral regurgitation, heart failure symptoms may occur abruptly because there has not been time for adaptive chamber dilatation. The murmur of acute mitral regurgitation may be present only in early diastole, owing to rapid diastolic equalization of ventricular and atrial pressures caused by the high volume overload. In cases of severe mitral regurgitation, a diastolic rumble occurs because of the large regurgitant volume during diastole. If present, the systolic murmur of acute mitral regurgitation is typically soft and ends before A2. It is best heard along the left sternal border and base of the heart, generally without a thrill. In patients with aortic stenosis, the murmur characteristically gets louder and then softer (crescendo-decrescendo) during systole; it is loudest at the second right intercostal space, with radiation to the carotid arteries. These findings are not present in this patient. Left ventricular free wall rupture is an often fatal complication of myocardial infarction that occurs 3 to 7 days after the initial event. Patients most commonly present with cardiac tamponade (due to hemopericardium), pulseless electrical activity, and death. Patients with cardiac tamponade typically do not have a heart murmur or a preserved or exaggerated x descent in the jugular venous pulse, as seen in this patient. Most patients with mild to moderate tricuspid regurgitation are asymptomatic. Examination reveals a systolic murmur that is loudest at the left lower sternal border and increases with inspiration. Symptoms and signs of right-sided heart failure, such as fatigue, elevated jugular venous pulse, and lower extremity edema, may be found; however, tricuspid regurgitation does not cause volume overload in the lungs, as was observed in this patient.
An 89-year-old woman is evaluated in the office for an annual examination. She has dementia. She was brought to the appointment by her caregiver who noted "bruises" on her arms. The patient says she does not remember what happened. On physical examination, vital signs are normal. Skin findings are shown. Which is the following is the most appropriate management? Biopsy the lesion Call adult protective services Obtain coagulation studies Prescribe triamcinolone ointment No intervention needed
Actinic purpura is caused by age-related capillary fragility and bleeding under atrophic skin; minor trauma can cause impressive purpuric macules and patches, most commonly on the forearm. Actinic purpura is a common incidental finding in elderly persons, and no intervention is needed at this time; however, reassurance should be provided to the patient and caregiver. A myriad of changes occur in the skin over time; these can be due to chronologic aging alone or influenced by the cumulative effects of ultraviolet light exposure. Actinic purpura is caused by age-related capillary fragility and bleeding under atrophic skin. Minor trauma can cause impressive purpuric macules and patches, most commonly on the forearm. It is not a sign of vasculitis, a bleeding disorder, or nutritional deficiency, and it does not require additional evaluation with coagulation studies or therapy. Often associated with actinic purpura are stellate pseudoscars, jagged or linear atrophic scars that primarily occur on the forearms of patients with chronic actinic damage. Patients may or may not recall antecedent trauma. There is no treatment for actinic purpura. Good skin care measures such as emollient moisturizers can help reduce dryness and protect the skin from minor trauma such as scratching. It is unnecessary to biopsy this lesion. There is no indication that it is a new growth or that this is an undiagnosed inflammatory skin condition or vasculitis. Although it is good to be alert for abusive situations, in this case the findings are typical for actinic purpura. Triamcinolone is a medium-potency glucocorticoid. Although initial application of the ointment will not harm the patient, chronic application of a medium-potency glucocorticoid on normal skin will result in thinning of the already thin skin, the development of striae and purpura, pigmentary changes, acneiform eruptions, and an increased risk for infection, resulting in more damage.
A 60-year-old man is seen for a routine evaluation. His family history is notable for prostate cancer in his father. He takes no medications. On physical examination, vital signs are normal. The remainder of the examination is normal. After discussion of the benefits and harms of prostate cancer screening, the patient wishes to proceed with screening. Laboratory studies reveal a serum prostate-specific antigen level of 5.1 ng/mL (5.1 µg/L). Biopsy of the prostate reveals cancer in two cores of the right lobe (Gleason score, 6), with less than 10% cancer in each core. Which of the following is the most appropriate management? Active surveillance Bone scan Leuprolide Observation
Active surveillance is a reasonable option in men with low-risk or very-low-risk prostate cancer who have a life expectancy of at least 10 years; active surveillance consists of scheduled assessments that include digital rectal examination, prostate-specific antigen measurement, and prostate biopsy. The most appropriate management is active surveillance. The American College of Physicians recommends that clinicians inform men between 50 and 69 years of age about the limited potential benefits and substantial harms of screening for prostate cancer. The decision to screen should be based on the patient's general health and life expectancy and patient preferences. This patient has very-low-risk prostate cancer based on his biopsy findings and prostate-specific antigen (PSA) level at diagnosis. Management of such patients has evolved during the past several years. Although treatment is a reasonable consideration, recent literature supports the use of active surveillance, which is a program of scheduled assessments that include digital rectal examination, PSA measurement, and prostate biopsy. The purpose of active surveillance is to identify early evidence of cancer progression in an effort to limit treatment to those most likely to benefit. It is appropriate only for men with low-risk or very-low-risk prostate cancer who have a life expectancy of at least 10 years. Although no randomized data are available to support this strategy, available data indicate that the 15-year metastasis-free survival in appropriately selected patients is up to 97%. Active surveillance is currently considered appropriate standard care in this patient population. Bone scan is not indicated for this asymptomatic patient because he has a very low likelihood of occult metastatic disease. A bone scan would be indicated only if there were signs or symptoms suggestive of osseous metastatic disease. Leuprolide would be indicated for treatment of metastatic disease, and it can also be combined with radiation for treatment of high-risk or very-high-risk prostate cancer. However, it has no role in the treatment of an asymptomatic patient with very-low-risk prostate cancer. Observation, sometimes called "watchful waiting," is distinct from active surveillance. These patients are provided palliative care if and when symptomatic progression requires medical intervention for control of symptoms. Observation is appropriate for elderly men with significant medical comorbidities that limit life expectancy. Treatment is not appropriate for these men based on their relatively short predicted life expectancy.
A 30-year-old man is evaluated in the emergency department for the recent onset of confusion and jaundice of 6 weeks' duration. His medical history is unremarkable. On physical examination, the patient is confused. His temperature is 36.9 °C (98.4 °F), blood pressure is 102/60 mm Hg, pulse rate is 115/min, and respiration rate is 22/min. Jaundice and asterixis are noted. No organomegaly is noted. Laboratory studies: Hematocrit 38% Leukocyte count 11,000/μL (11 × 109/L) Platelet count 350,000/μL (350 × 109/L) INR 2.6 Alanine aminotransferase 200 U/L Aspartate aminotransferase 150 U/L Total bilirubin 18.6 mg/dL (318.1 µmol/L) Creatinine 3.0 mg/dL (265.2 µmol/L) On abdominal ultrasound, the liver appears normal with intact vasculature. No splenomegaly or dilation of bile ducts is noted. Which of the following is the most appropriate next step in management? Administer fresh frozen plasma Initiate hemodialysis Perform endoscopic retrograde cholangiopancreatography Refer for liver transplantation
Acute liver failure is an indication for immediate referral to a liver transplantation center.
A 30-year-old man is evaluated in the emergency department for the recent onset of confusion and jaundice of 6 weeks' duration. His medical history is unremarkable. On physical examination, the patient is confused. His temperature is 36.9 °C (98.4 °F), blood pressure is 102/60 mm Hg, pulse rate is 115/min, and respiration rate is 22/min. Jaundice and asterixis are noted. No organomegaly is noted. Laboratory studies: Hematocrit 38% Leukocyte count 11,000/μL (11 × 109/L) Platelet count 350,000/μL (350 × 109/L) INR 2.6 Alanine aminotransferase 200 U/L Aspartate aminotransferase 150 U/L Total bilirubin 18.6 mg/dL (318.1 µmol/L) Creatinine 3.0 mg/dL (265.2 µmol/L) On abdominal ultrasound, the liver appears normal with intact vasculature. No splenomegaly or dilation of bile ducts is noted. Which of the following is the most appropriate next step in management? Administer fresh frozen plasma Initiate hemodialysis Perform endoscopic retrograde cholangiopancreatography Refer for liver transplantation
Acute liver failure is an indication for immediate referral to a liver transplantation center. Referral for liver transplantation is the most appropriate choice for this patient with acute liver failure. Acute liver failure is defined by the manifestation of hepatic encephalopathy within 26 weeks of developing symptoms of liver disease. The development of jaundice was this patient's first symptom of liver disease. Within 6 weeks, he developed coagulopathy, with an INR of 2.6, as well as symptoms of hepatic encephalopathy (confusion and asterixis). Liver injury is distinct from acute liver failure and presents with elevated liver test results and/or jaundice in the absence of evidence of liver function failure. The most common causes of acute liver failure are medications, especially acetaminophen, and viral infections, although many cases are due to indeterminate causes. The prompt recognition of acute liver failure is essential due to high rates of mortality. Patients require frequent monitoring for hypoglycemia, hypophosphatemia, acute kidney injury, infections, and progressive hepatic encephalopathy, which can be accompanied by cerebral edema and intracranial hypertension. Liver transplantation improves the survival rate; therefore, referral to a liver transplantation center is essential upon recognition of acute liver failure. The presence of an elevated INR reflects decreased synthesis of liver-derived clotting factors, and the administration of fresh frozen plasma is not indicated in the absence of demonstrable bleeding or the need for invasive procedures. Kidney failure is common in the setting of acute liver failure, and many patients require renal replacement therapy, which should be performed in a liver transplant center. Most patients requiring hemodialysis in this setting tolerate continuous renal replacement therapy better than intermittent hemodialysis. There are several appropriate indications for endoscopic retrograde cholangiopancreatography (ERCP), and one of the most common is the evaluation of biliary obstruction. In these cases, ERCP can both be diagnostic and therapeutic if the obstruction can be removed or bypassed. In this patient, ERCP is not indicated because the ultrasound shows no signs of biliary obstruction.
A 26-year-old man is hospitalized for a 1-week history of dry cough and progressive shortness of breath. He has a 2-year history of systemic lupus erythematosus (SLE); disease manifestations have been pleuropericarditis, polyarthritis, leukopenia, and nephritis. He currently reports no fever, sputum production, or hemoptysis, as well as no other SLE manifestations. His disease has been difficult to control and has needed multiple tapering doses of prednisone. He currently takes 15 mg/d of prednisone; other medications are hydroxychloroquine and azathioprine. On physical examination, temperature is 36.4 °C (97.6 °F), blood pressure is 150/100 mm Hg, pulse rate is 98/min, respiration rate is 30/min, and oxygen saturation is 84% breathing ambient air. Diffuse crackles are heard on auscultation of the chest. Diffuse tenderness and swelling of multiple small joints of the hands are present. The remainder of the physical examination is normal. Laboratory studies: Erythrocyte sedimentation rate 68 mm/h Hematocrit 42% Leukocyte count 5500/µL (5.5 × 109/L) with normal differential Complements (C3 and C4) Low C-reactive protein Normal Creatinine 0.6 mg/dL (53 µmol/L) Procalcitonin Normal Chest radiograph and CT scan of the chest show diffuse infiltrates in both lungs with ground glass opacities in multiple lobes. Bronchoalveolar lavage shows only increased leukocytes with lymphocytic predominance. Cultures are pending. Which of the following is the most likely diagnosis? Acute lupus pneumonitis Acute pulmonary hypertension Community-acquired pneumonia Diffuse alveolar hemorrhage Shrinking lung syndrome
Acute lupus pneumonitis is the most likely diagnosis in this patient who has difficult-to-control systemic lupus erythematosus (SLE) requiring aggressive therapy with prednisone and azathioprine. Lupus pneumonitis is a rare but severe presentation of SLE characterized by shortness of breath, hypoxia, and diffuse pulmonary infiltrates. Other major entities in the differential diagnosis include infection and diffuse alveolar hemorrhage. He has clear evidence of active SLE with polyarthritis, a high erythrocyte sedimentation rate, hypocomplementemia, and diffuse pulmonary infiltrates on radiologic studies, as well as lymphocytic predominance on bronchoalveolar lavage. These findings are very suggestive of lupus pneumonitis. It usually requires rapid and aggressive therapy with glucocorticoids and/or immunosuppressive agents. Pulmonary hypertension does not present with diffuse infiltrates with ground glass opacities, as seen in this patient, and it cannot explain the bronchoalveolar lavage findings. This patient has no fever or sputum production, and testing shows a normal leukocyte count and differential, normal C-reactive protein (CRP), and normal procalcitonin, making infection less likely. CRP is a marker of inflammation, and levels are significantly increased in bacterial pneumonia but frequently normal or only slightly elevated in SLE flares. Procalcitonin is produced by cells as a response to bacterial toxins, which result in serum procalcitonin elevations in bacterial infections. The patient reports no hemoptysis, and lack of anemia and absence of erythrocytes on bronchoalveolar lavage exclude a diagnosis of diffuse alveolar hemorrhage. Shrinking lung syndrome presents with chronic insidious shortness of breath with low lung volumes possibly related to diaphragmatic dysfunction. This patient's acute presentation and diffuse infiltrates on imaging is not consistent with this syndrome.
A 56-year-old man is evaluated for a 10-day history of pain in a patchy region to the right of the umbilicus. He says that the area is sensitive to light touch. He reports no recent systemic illness, weakness, or fever. The patient has hypertension and hyperlipidemia. Medications are losartan, aspirin, and atorvastatin. On physical examination, vital signs are normal; BMI is 36. Sensory examination reveals a patchy area of paresthesia and allodynia on the right side of the abdomen that extends posteriorly but does not cross the midline. Abdominal swelling is noted on the right. No abdominal mass or tenderness is present. There is no skin rash. Results of cranial nerve, motor strength, coordination, and gait testing are normal, and deep tendon reflexes are preserved. Results of laboratory studies show a serum creatinine level of 2.1 mg/dL (186 µmol/L) and a hemoglobin A1c value of 7.8% (0.078). Results of serum and urine protein electrophoresis and serum light chain measurement are unremarkable. An MRI of the thoracic spine is normal. Which of the following is the most likely diagnosis? Diabetic mononeuropathy Paraproteinemic neuropathy Uremic neuropathy Vitamin B12 deficiency
Acute or subacute pain and paresthesia in a dermatomal pattern in the thoracic or abdominal region in a patient with diabetes mellitus and no evidence of herpes zoster is most likely due to diabetic mononeuropathy. This patient most likely has diabetic mononeuropathy. He has an elevated hemoglobin A1c value that is consistent with a diagnosis of diabetes mellitus and has no evidence of herpes zoster infection. Truncal diabetic mononeuropathy presents with acute or subacute pain and paresthesia in a dermatomal pattern in the thoracic or abdominal region. The area affected may be unilateral or cross the midline. Associated weakness of abdominal wall muscles can also be present and is detected by the presence of apparent abdominal swelling in the absence of palpable organomegaly. The differential diagnosis for this condition includes truncal radiculopathy caused by varicella zoster infection and thoracic myelopathy caused by demyelinating disease. Paraproteinemic neuropathy can present as a symmetric distal sensory neuropathy or as sensorimotor, multifocal motor, and cranial neuropathies. The normal results of serum electrophoresis and serum free light chain measurement rules out this diagnosis. Uremic neuropathy is similar to other neuropathies caused by metabolic disruptions and is associated with distal symmetric polyneuropathy. Asymmetric truncal neuropathy is not a typical presentation. Also, uremic neuropathy is often associated with a more severe degree of kidney injury than is present in this patient. Vitamin B12 deficiency can cause myeloneuropathy, distal symmetric neuropathy, and cranial neuropathy but typically does not present as truncal mononeuropathy.
A 45-year-old man is evaluated during a follow-up visit for membranous glomerulopathy diagnosed 3 weeks ago. He reports persistent lower extremity edema and no weight loss despite adhering to a low-salt diet and taking maximal-dose furosemide. He does not have shortness of breath or abdominal discomfort. Other medications are enalapril and simvastatin. On physical examination, vital signs are normal. The patient weighs 80 kg (176.4 lb), with a baseline weight of 75 kg (165.3 lb). There is no rash. Cardiac examination is normal, and there is no evidence of jugular venous distention. The lungs are clear on examination. There is pitting edema in the legs bilaterally to just below the patellae. Laboratory studies: Albumin 2.9 g/dL (29 g/L) Blood urea nitrogen Normal Creatinine 1.0 mg/dL (88.4 µmol/L) Electrolytes Normal Urinalysis No blood; 4+ protein Urine protein-creatinine ratio 6100 mg/g Doppler ultrasound of the lower extremities performed 3 weeks ago showed no evidence of deep venous thrombosis. Which of the following is the most appropriate management? Add metolazone Change furosemide to bumetanide Hospitalize for intravenous diuresis Repeat lower extremity Doppler ultrasonography
Addition of the thiazide diuretic metolazone is the most appropriate treatment. Edema management in a patient with newly diagnosed nephrotic syndrome generally starts with a salt-restricted diet and an oral loop diuretic, with a goal weight loss of 1 to 2 kg (2.2-4.4 lb) per week. Loop diuretics should be uptitrated toward this weight loss goal until a maximal dose is achieved, with close monitoring of electrolytes. It is appropriate for blood urea nitrogen, serum creatinine, and serum urate to rise slightly (≤10%) with effective diuresis. When oral loop diuretics have been maximally uptitrated, and weight loss and edema control are insufficient, it is often necessary to add a second oral diuretic (a thiazide diuretic and/or potassium-sparing diuretic) that works distal to the loop of Henle, which in this case can be accomplished via the addition of metolazone in this patient taking maximal-dose furosemide. Loop diuretics are similarly effective when administered at equipotent doses. All loop diuretics have a similar dose-response curve characterized by an increased diuresis as more of the drug is renally excreted and flattening of the curve, at which further increases in dose are not associated with increased drug excretion and resultant diuresis. Hospitalization for intravenous diuresis should be reserved for patients who have not responded to maximal doses of oral diuretics (that is, loop diuretic and thiazide diuretic) or for patients with severe symptoms of anasarca such as shortness of breath, swelling of genital regions, or gastrointestinal discomfort from ascites. Repeating lower extremity Doppler ultrasonography is not indicated at this time given this patient's prior negative test results; serum albumin level >2.8 g/dL (28 g/L) (clotting risk in membranous glomerulopathy rises when albumin is ≤2.8 g/dL [28 g/L]); and the symmetric nature of his edema.
How does adenosine help diagnose SVTs?
Adenosine can be used to interrupt AV conduction if vagal maneuvers are ineffective, such as in this patient. Adenosine may also help ascertain the nature of an arrhythmia. For example, when adenosine is administered to patients with atrial tachycardia and rapid 1:1 conduction, it demonstrates persistent rapid P waves despite AV block, which indicates the presence of an atrial tachycardia. Adenosine is metabolized by adenosine deaminase in the blood and has a very short half-life (6 seconds). It is contraindicated in preexcited atrial fibrillation with antidromic tachycardia, which presents with a QRS duration greater than 120 ms due to conduction down the accessory pathway.
A 65-year-old man is evaluated for an episode of hemoptysis. His medical history is significant for a 30-pack-year smoking history; he quit smoking 1 year ago. He takes no medications. On physical examination, vital signs are normal. BMI is 28. Oxygen saturation is 96% breathing ambient air. CT scan of the chest identifies a 3.5-cm spiculated mass in the right lower lobe and an enlarged hilar lymph node. PET/CT scans and brain MRI scans show no evidence of metastatic disease. Surgery is performed and reveals a 3.2-cm, poorly differentiated adenocarcinoma with negative margins, two hilar lymph nodes positive for metastatic adenocarcinoma, and negative mediastinal lymph nodes. Which of the following is the most appropriate management? Adjuvant chemoradiation Adjuvant chemotherapy Adjuvant radiation Observation
Adjuvant cisplatin-based chemotherapy for stage II and stage III non-small cell lung cancer provides a survival advantage. The most appropriate treatment is adjuvant chemotherapy. Most (80%) lung cancers are non-small cell lung cancers (NSCLCs), with the most common type being adenocarcinoma of the lung. Current treatment options for NSCLCs include surgical resection, radiation therapy, chemotherapy, and targeted therapies. Several studies have found a small but significant survival benefit associated with adjuvant chemotherapy in patients with stage II and stage III NSCLC. Although both cisplatin- and carboplatin-based chemotherapy regimens have been studied, only cisplatin-based chemotherapy has been shown to improve survival. At least four different cisplatin-based combination regimens can be used for adjuvant chemotherapy treatment; however, patients must be selected carefully because toxicity can be substantial. Based on this survival benefit, cisplatin-based chemotherapy is recommended after surgery for stage II and stage III NSCLC in patients such as this one without contraindications to chemotherapy. Combined chemotherapy and radiation has not been shown to improve outcomes in patients with NSCLC after surgery with negative margins. Combined chemotherapy and radiation is used, however, as a primary therapy for patients with stage III lung cancer with clinically evident mediastinal lymphadenopathy noted at the time of initial evaluation; this is because this group of patients does not typically fare well with surgery. Radiation as a single modality has no role in the adjuvant treatment of resected NSCLC in the absence of positive surgical margins. In the presence of positive margins, it is reasonable to recommend radiation. Observation is not recommended as a routine management strategy because of the survival benefit associated with chemotherapy treatment. Observation would be appropriate only for a patient who could not be treated with chemotherapy owing to medical contraindications or personal preference.
A 41-year-old woman comes to the office for follow-up evaluation of relapsing-remitting multiple sclerosis, which was diagnosed 1 year ago. She has been taking the maximum dose of interferon beta-1a in three-times-weekly injections as disease-modifying therapy since that time. The patient currently has no symptoms and takes no other medication or supplement. On physical examination, all findings are unremarkable. A routine surveillance MRI of the brain shows two new lesions in the periventricular white matter, both of which enhance with contrast. Which of the following is the most appropriate next step in management? Add dalfampridine Add glatiramer acetate Evaluate for cerebrospinal fluid JC virus antibody Measure serum 25-hydroxyvitamin D level
Adjuvant therapy with vitamin D supplementation in patients with relapsing-remitting multiple sclerosis provides additional disease activity control. This patient's serum 25-hydroxyvitamin D level should be measured to determine if she has vitamin D deficiency. The response to disease-modifying therapy is monitored with clinical evaluation and often with periodic MRI studies. Although she is clinically stable, the MRI shows possible new disease activity. Accumulating evidence suggests that disease activity in multiple sclerosis (MS) is highly correlated with serum vitamin D levels, with less frequent relapses and MRI lesion development in patients with high vitamin D levels. Furthermore, administration of vitamin D supplementation has now been shown to provide additional control of disease activity, with a recent trial finding less MRI activity in patients taking interferon plus vitamin D versus interferon alone. This patient is not taking any vitamin D supplementation, which may make her more prone to exacerbations. Supplementation would be indicated if she demonstrates deficiency on serologic testing. The addition of dalfampridine to this patient's regimen would have no benefit at this time. Although this medication has shown efficacy in improving ambulatory function in some patients with MS, it has no immunologic, disease-modifying effect. Because this patient has no ambulation difficulty and is experiencing an inflammatory disease breakthrough, addition of dalfampridine would be an inappropriate choice. Glatiramer acetate is a copolymer of four amino acids that, among other mechanisms of action, may bind major histocompatibility complex molecules and induce a shift in the immune response away from autoimmunity. Glatiramer acetate also has been shown to reduce the relapse rate by approximately one third compared with placebo and appears equivalent to the beta interferons in head-to-head studies. However, clinical trial evidence has shown that combining glatiramer acetate with an interferon beta provides no added benefit to what either drug achieves alone. Lumbar puncture to obtain cerebrospinal fluid for JC virus testing is not appropriate. Reactivation of the JC virus in the brain can result in progressive multifocal leukoencephalopathy (PML), an adverse effect of some immunosuppressive medications and chemotherapy. PML, however, is not a known adverse effect of the interferon beta preparations, and the changes on this patient's MRI are not indicative of PML. Thus, performance of this invasive test is not indicated.
A 45-year-old man underwent abdominal CT imaging for evaluation of bloating and constipation. The CT scan shows a 5-cm right adrenal mass with a density of 42 Hounsfield units and absolute contrast washout of 38% at 10 minutes. Testing for pheochromocytoma and subclinical Cushing syndrome was negative. Medical history is otherwise unremarkable, and he takes no medications. On physical examination, vital signs and the remainder of the physical examination are normal. Which of the following is the most appropriate next step in management? Adrenal biopsy Adrenalectomy Mitotane therapy Repeat CT at 6 months
Adrenalectomy is recommended for incidental adrenal masses with radiologic features that suggest increased risk of an adrenal malignancy (size >4 cm, density ≥10 Hounsfield units, and absolute contrast washout <50% at 10 minutes). The most appropriate next step in management is adrenalectomy. The patient presented with an incidental adrenal mass with radiologic features that are indeterminate for adenoma and may indicate an adrenal malignancy (size >4 cm, density ≥10 Hounsfield units, and absolute contrast washout <50% at 10 minutes). Benign adrenal adenomas tend to be small (<4 cm), often have an intracytoplasmic fat content and appear less dense on noncontrast CT scan (<10 Hounsfield units), and exhibit rapid contrast washout during delayed contrast imaging (>50% at 10 minutes). These radiologic features are not diagnostic of malignancy, as one-third of benign adrenal masses are lipid poor (≥10 Hounsfield units) and many are larger than 4 cm. However, because adrenal carcinoma is an aggressive tumor and data indicate that prognosis may be more favorable when the disease is diagnosed and treated at an earlier stage, adrenalectomy is usually recommended. Adrenal biopsy is not routinely indicated in the diagnostic evaluation of an incidentally discovered adrenal mass, even if the suspected diagnosis is primary adrenal malignancy, because adrenocortical carcinoma can be missed due to sampling error. Adrenal biopsy may be indicated when adrenal metastasis or an infiltrative disorder such as infection or lymphoma is suspected. Screening for pheochromocytoma should be performed prior to adrenal biopsy to avoid potential hypertensive crisis during the procedure. Mitotane, an adrenolytic drug, may be used as adjuvant therapy following primary resection. Adrenalectomy is the first-line treatment of choice for patients with suspected adrenocortical carcinoma. Repeat abdominal CT imaging at 6 months is suggested for adrenal masses that are small (<4 cm) and have benign radiologic features. The optimal time to repeat CT imaging in the radiologically benign-appearing, or even indeterminate-appearing, incidentally noted adrenal mass, is controversial. Repeat CT imaging is not indicated in this patient with high-risk features for adrenal carcinoma.
A 35-year-old woman is hospitalized for left-sided pleuritic chest pain and dyspnea that began 1 day ago. Four weeks ago, she began to experience fever once or twice a day, pharyngitis, intermittent rash on the trunk and proximal extremities that occurs with the fever, severe joint pain, and myalgia. She gave birth 10 weeks ago to a healthy female infant. On physical examination, temperature is 39.0 °C (102.2 °F), pulse rate is 90/min, and respiration rate is 22/min. Enlarged cervical lymph nodes, hepatomegaly, and splenomegaly are present. A pleural friction rub is heard. A pink maculopapular rash is present on the trunk. Tenderness and swelling of the wrists, knees, and ankles are noted. Laboratory studies: Erythrocyte sedimentation rate 90 mm/h Hemoglobin 10 g/dL (100 g/L) Leukocyte count 20,000/µL (20 × 109/L), 90% neutrophils Alanine aminotransferase 80 U/L Aspartate aminotransferase 70 U/L Ferritin 6000 ng/mL (6000 µg/L) Urinalysis Normal Chest radiograph shows a small left-sided pleural effusion. Which of the following is the most likely diagnosis? Adult-onset Still disease Cryoglobulinemic vasculitis Lymphoma Microscopic polyangiitis Systemic lupus erythematosus
Adult-onset Still disease is characterized by spiking fever, an evanescent salmon-colored rash on the trunk and extremities that occurs in conjunction with fever, arthritis, lymphadenopathy, and leukocytosis; an extremely high serum ferritin level is characteristic. The most likely diagnosis is adult-onset Still disease (AOSD), a systemic inflammatory disease of multiple organ systems. This patient displays the typical signs and symptoms of AOSD, with spiking fevers, pharyngitis, an evanescent salmon pink rash that occurs in conjunction with fever, adenopathy, hepatosplenomegaly with elevated liver enzymes, leukocytosis with neutrophil predominance, inflammatory arthritis, and myalgia. Patients with AOSD can also develop other manifestations such as serositis. Extremely high serum ferritin levels are a typical feature. AOSD may also be seen during pregnancy or in the postpartum period. The diagnosis is clinical, and other entities such as infection and malignancy must be excluded. Cryoglobulinemic vasculitis can cause arthritis, rash, and fever and may be associated with liver abnormalities, particularly if it is related to hepatitis C virus. However, the rash of cryoglobulinemia is related to small-vessel vasculitis characterized by palpable purpura of the lower extremities. Other organs characteristically involved include peripheral nerves and the kidneys (glomerulonephritis). Cryoglobulinemia cannot explain the elevated serum ferritin level. Lymphoma can cause arthralgia, fever, rash, lymphadenopathy, and organomegaly. Lymphoma would not explain the patient's pleural friction rub, arthritis, or elevated serum ferritin level. Microscopic polyangiitis is a form of ANCA-associated vasculitis in which patients can develop fever, rash, arthritis, and pulmonary disease, but this patient's rash (pink, evanescent, involving the trunk) is not typical for vasculitis (palpable purpura, urticaria, livedo reticularis, nodules). In addition, kidney involvement is nearly ubiquitous in microscopic polyangiitis, and the normal urinalysis argues against this diagnosis. Similarly, systemic lupus erythematosus (SLE) can cause the same symptoms, but the rash associated with SLE is unlike that of AOSD. The malar rash of acute cutaneous lupus erythematosus consists of bright erythematous patches over both cheeks and the nasal bridge, almost always sparing the nasolabial folds; the rash is unrelated to fever. In addition, SLE cannot explain the elevated ferritin level. Read Related TextNext Question
A 50-year-old man is evaluated for elevated blood pressure measurements despite an increase in his hydrochlorothiazide dose 1 month ago. History is significant for hypertension and hyperlipidemia. Medications are hydrochlorothiazide and atorvastatin. On physical examination, blood pressure is 150/92 mm Hg, and pulse rate is 69/min. BMI is 30. The remainder of the examination is normal. Laboratory studies show a serum creatinine level of 1.0 mg/dL (88.4 µmol/L), a serum potassium level of 3.4 mEq/L (3.4 mmol/L), and a urine albumin-creatinine ratio of 550 mg/g. In addition to weight loss, which of the following is the most appropriate management? Add amlodipine Add losartan Add spironolactone Schedule a follow-up visit for 3 months
An ACE inhibitor or angiotensin receptor blocker is an agent of choice for treatment of hypertension in a patient with chronic kidney disease. The addition of losartan is the most appropriate management. This patient has chronic kidney disease (CKD), given the presence of albuminuria, as well as uncontrolled hypertension. His antihypertensive medication regimen needs to be modified to control his blood pressure to target, given that uncontrolled hypertension will lead to CKD progression. The best option for this patient is to add an angiotensin receptor blocker (ARB) (such as losartan) or an ACE inhibitor, which are antihypertensive agents of choice in patients with CKD. Several studies have shown that use of an ARB or ACE inhibitor can result in decreased progression of albuminuria and CKD. Addition of losartan will also likely increase his serum potassium back to within normal range. Adding amlodipine, a calcium channel blocker, is not the most appropriate next step in hypertension management for this patient and should be reserved if treatment with a maximum-tolerated dose of an ACE inhibitor or ARB does not result in blood pressure control. Adding a low-dose aldosterone antagonist (such as spironolactone or eplerenone) may improve blood pressure control in patients with resistant hypertension. Treatment-resistant hypertension is defined as blood pressure that remains above goal despite concurrent use of three antihypertensive agents of different classes, one of which is a diuretic. Spironolactone is typically the fourth drug added to a three-drug regimen. This patient does not meet the definition of resistant hypertension. Failure to act on this patient's blood pressure and albuminuria and reevaluating in 3 months misses the opportunity to slow the progression of CKD and albuminuria. In addition, guidelines recommend that adults initiating a new or adjusted drug regimen for hypertension should have a follow-up evaluation of adherence and response to treatment at monthly intervals until control is achieved. Read Related TextNext Question
A 62-year-old woman is evaluated for fatigue and weakness. History is significant for stage G4 chronic kidney disease and hypertension. Her only medication is amlodipine. On physical examination, blood pressure is 135/85 mm Hg; other vital signs are normal. There is no jaundice. Conjunctival rim pallor is noted, and there is no scleral icterus. Laboratory studies: Hemoglobin 8.5 g/dL (85 g/L) Leukocyte count Normal Mean corpuscular volume 80 fL Platelet count Normal Reticulocyte count 1% of erythrocytes Ferritin 30 ng/mL (30 µg/L) Transferrin saturation 10% Estimated glomerular filtration rate 18 mL/min/1.73 m2 Stool testing for occult blood Negative Colonoscopy performed at age 60 years was normal. Which of the following is the most appropriate treatment? Blood transfusion Bone marrow biopsy Erythropoietin-stimulating agent Iron supplementation
All patients with chronic kidney disease and anemia should have iron profiles assessed, including transferrin saturation and ferritin levels; treatment target levels are a transferrin saturation level >30% and a serum ferritin level >500 ng/mL (500 µg/L) using either oral or intravenous iron supplementation. Iron supplementation is the most appropriate treatment for this patient with anemia associated with stage G4 chronic kidney disease (CKD). The prevalence of anemia increases as CKD progresses due to several factors, including impaired erythropoietin production, erythropoietin resistance, and reduced erythrocyte life span. Initial evaluation for the cause of anemia in patients with CKD includes laboratory studies as would be appropriate for patients without CKD. All patients with CKD and anemia should have iron profiles assessed, including total transferrin saturation (serum iron ÷ total iron-binding capacity × 100) and serum ferritin levels. The Kidney Disease: Improving Global Outcomes (KDIGO) recommendations suggest maintaining transferrin saturation levels of >30% and serum ferritin levels of >500 ng/mL (500 µg/L) using either oral or intravenous iron supplementation. Patients with stage G3 or G4 CKD can be treated with oral iron supplements but may need parenteral administration if oral iron is not effective. Patients with stage G5 CKD generally do not respond well to oral iron due to impaired gastrointestinal absorption and therefore often need intravenous administration. This patient with stage G4 CKD has iron deficiency and should have iron repletion. Although blood transfusions are effective at acutely raising the hemoglobin, they are associated with adverse side effects. Specific to the advanced CKD population, blood transfusions should be avoided if possible to prevent HLA sensitization to foreign antigens, which may increase waiting time for kidney transplantation. A bone marrow biopsy is not indicated because there is no evidence to suggest bone marrow dysfunction. Platelet count and leukocyte count are normal. The low hemoglobin and erythrocyte reticulocyte count reflect the inadequate iron stores. A brisk reticulocytosis should be observed within days of starting iron therapy. Although many patients with advanced CKD and end-stage kidney disease require an erythropoietin-stimulating agent (ESA), its use is indicated only when the patient has adequate iron stores. Iron deficiency is a common reason for ESA resistance. ESAs take several weeks to achieve full effect and are sometimes started at the same time as parenteral iron. Current KDIGO guidelines recommend consideration of ESAs for patients with CKD and hemoglobin concentrations <10 g/dL (100 g/L). However, it is premature to consider ESA in this patient for she may respond to supplemental iron administration alone with an increase in hemoglobin level. Read Related TextNext Question
A 48-year-old man is evaluated during a routine follow-up visit for gout diagnosed 2 months ago, at which time he started allopurinol and colchicine. Since then, he has had three acute attacks separately involving the left wrist, right knee, and left foot. He has no current joint pain. He also reports having diarrhea two to three times daily over the past 6 weeks. History is also significant for type 2 diabetes mellitus, for which he takes insulin. On physical examination, vital signs are normal. Musculoskeletal examination is normal. Laboratory studies show a serum creatinine level of 0.8 mg/dL (70.7 µmol/L) and a serum urate level of 5.5 mg/dL (0.32 mmol/L). Which of the following is the most appropriate next step to decrease the frequency of gout attacks? Add prednisone Add probenecid Discontinue colchicine; begin meloxicam Increase allopurinol Increase colchicine
All patients with gout beginning urate-lowering therapy should also receive a prophylactic agent such as colchicine, low-dose glucocorticoids, or low-dose NSAIDs to prevent mobilization flares; the choice of prophylactic drug is determined by patient comorbidities. Discontinuing colchicine and beginning an NSAID such as meloxicam is the most appropriate next step to decrease the frequency of this patient's gout attacks. He has experienced several acute gout attacks since beginning allopurinol 2 months ago. It is common to experience "mobilization attacks" at the initiation of urate-lowering therapy, which occur when urate body stores shift from joints and soft tissues to the vasculature. Hence, all patients beginning urate-lowering therapy should also receive a prophylactic agent to prevent against mobilization flares for 6 to 12 months, without which the average patient will experience several acute attacks. Colchicine is the primary prophylactic agent employed (once or twice daily) for this role, but low-dose glucocorticoids and low-dose NSAIDs are also effective. Choice of therapy should be tailored to the individual's comorbid conditions. This patient is currently taking colchicine, which is most likely causing the diarrhea. In this case, discontinuation of colchicine and initiating a low-dose NSAID such as meloxicam is the most appropriate choice. All NSAIDs are considered equivalent for this purpose. Adding prednisone would be a reasonable option for prophylaxis but is not ideal for this patient with insulin-dependent type 2 diabetes mellitus, because even low-dose glucocorticoids may result in hyperglycemia and increase insulin requirements. Adding the uricosuric agent probenecid or increasing the allopurinol would not afford protection against mobilization flares because they are both urate-lowering agents, not prophylactic agents. Intensifying urate-lowering therapy would be a reasonable choice if the recurrent attacks are thought to represent a failure of urate-lowering therapy, rather than mobilization attacks, as is suspected with this patient. Increasing colchicine would be a reasonable option, were it not for the diarrhea that began with the institution of colchicine 2 months ago. Diarrhea is one of the most common side effects of colchicine, and increasing the dose would worsen gastrointestinal toxicity.
A 45-year-old man is evaluated during a second follow-up visit after hospitalization for new-onset heart failure with reduced ejection fraction. His ejection fraction at the time of diagnosis 2 months ago was 42%. He reports feeling better, with improving exercise tolerance and no exertional dyspnea. Medications are lisinopril, low-dose carvedilol, and furosemide. He is Black. On physical examination, the patient is afebrile, blood pressure is 120/76 mm Hg, pulse rate is 84/min, and respiration rate is 16/min. The estimated central venous pressure is normal. There is no S3. Lungs are clear, and there is no peripheral edema. Which of the following is the most appropriate treatment? Add digoxin Add hydralazine and isosorbide dinitrate Increase carvedilol Increase furosemide
All patients with heart failure with reduced ejection fraction should be treated with an ACE inhibitor and a β-blocker; β-blocker dosage should be uptitrated every 2 to 4 weeks until the patient achieves a heart rate of approximately 60/min or has symptomatic hypotension.
A 45-year-old man is evaluated during a second follow-up visit after hospitalization for new-onset heart failure with reduced ejection fraction. His ejection fraction at the time of diagnosis 2 months ago was 42%. He reports feeling better, with improving exercise tolerance and no exertional dyspnea. Medications are lisinopril, low-dose carvedilol, and furosemide. He is Black. On physical examination, the patient is afebrile, blood pressure is 120/76 mm Hg, pulse rate is 84/min, and respiration rate is 16/min. The estimated central venous pressure is normal. There is no S3. Lungs are clear, and there is no peripheral edema. Which of the following is the most appropriate treatment? Add digoxin Add hydralazine and isosorbide dinitrate Increase carvedilol Increase furosemide
All patients with heart failure with reduced ejection fraction should be treated with an ACE inhibitor and a β-blocker; β-blocker dosage should be uptitrated every 2 to 4 weeks until the patient achieves a heart rate of approximately 60/min or has symptomatic hypotension. The dosage of carvedilol should be increased in this patient with new-onset heart failure who is symptomatically improving with standard heart failure therapy. All patients with heart failure with reduced ejection fraction should be treated with an ACE inhibitor and a β-blocker, such as carvedilol, to control heart failure symptoms, improve left ventricular ejection fraction, and decrease mortality. Data suggest improved outcomes with higher dosages of β-blockers. Therefore, in patients with new-onset heart failure, it is generally reasonable to increase the β-blocker dosage every 2 to 4 weeks until the patient achieves a heart rate of approximately 60/min or has symptomatic hypotension. Volume status should always be assessed before initiation and uptitration of a β-blocker because patients with volume overload will experience dyspnea if the dosage is increased. Because this patient has a systolic blood pressure of 120 mm Hg, a heart rate of 84/min, and no evidence of volume overload, he should be able to tolerate an increase in the β-blocker dosage. Repeat echocardiography should be performed to assess for improvement or recovery of ejection fraction once the ACE inhibitor and β-blocker have been uptitrated to maximally tolerated dosages. Digoxin reduces the incidence of hospitalizations in patients with symptomatic heart failure. This patient is asymptomatic; therefore, there is no reason to start digoxin at this time. Hydralazine and isosorbide dinitrate improve survival in patients who are Black and are already receiving maximal therapy for New York Heart Association functional class III or IV heart failure symptoms. This patient has New York Heart Association functional class I symptoms and is not yet taking optimal dosages of medical therapy; both the ACE inhibitor and β-blocker dosages should be uptitrated before other therapies are added to this patient's medication regimen. Diuretics are the primary treatment for symptoms of heart failure associated with volume overload. In general, the lowest diuretic dosage that achieves euvolemia should be used. In this patient without evidence of fluid overload, there is no reason to increase the furosemide dosage.
A 52-year-old woman is evaluated for acute onset of right-sided hearing loss that began yesterday. Soon afterward, she also noted a sensation of ear fullness and ringing in the same ear. She has no other focal neurologic symptoms. She reports no rhinorrhea, fever, pharyngitis, or ear pain. Medical history is significant for hypertension. She takes chlorthalidone. She has had no other exposures to medications or supplements. On physical examination, vital signs are normal. There is decreased hearing in the right ear; the Weber test lateralizes to the left ear, and air conduction is louder than bone conduction bilaterally. The ear canals are unobstructed, and the tympanic membranes are normal appearing. The neurologic examination is unremarkable. Which of the following is the most likely diagnosis? Meniere disease Otosclerosis Ototoxicity Sudden sensorineural hearing loss
All patients with sudden sensorineural hearing loss should undergo audiometric evaluation, and most patients will require MRI. The most likely diagnosis for this patient's acute, unilateral hearing loss is sudden sensorineural hearing loss (SSHL). The right-sided hearing loss and the finding of lateralization to the left ear on Weber testing support the diagnosis. Approximately 90% of cases of SSHL are idiopathic; however, viral infection, drug reactions, acoustic neuroma, multiple sclerosis, head injury, vascular issues, systemic immune-mediated conditions, and Meniere disease can all be causes. SSHL most commonly presents as unilateral tinnitus and ear fullness; vertigo occurs less often. Because this patient lacks other features to explain the acute hearing loss, she should undergo urgent referral to an otolaryngologist for audiometry, clinical assessment, and MRI to exclude tumors, multiple sclerosis, or vascular causes. Treatment involves oral glucocorticoids, although strong evidence of efficacy is lacking. Meniere disease, which is associated with endolymphatic hydrops (excess fluid in the endolymphatic spaces), can cause unilateral sensorineural hearing loss, but its presentation is characterized by episodic vertigo (lasting between 20 minutes and 24 hours) and tinnitus, which is often low pitched. The hearing loss may be described as fluctuating, and early in the disease it often involves low frequencies. Meniere disease may also present with a sensation of ear fullness. Otosclerosis involves bony overgrowth on the footplate of the stapes, leading to a lack of functioning of the ossicles. This middle ear process typically causes gradual, painless, bilateral conductive hearing loss, not sensorineural hearing loss, as seen in this patient. Otosclerosis occurs more often in women, and there is often a family history of the condition. Ototoxicity also can result in sensorineural hearing loss; it may be caused by a variety of medications, including antibiotics (particularly aminoglycosides), chemotherapeutic agents, loop diuretics, and aspirin or other NSAIDs. Often occurring gradually and bilaterally, ototoxicity may be reversible or permanent, depending on the agent involved. This patient is not taking and has not been exposed to drugs known to be ototoxic. Read Related TextNext Question
A 54-year-old man is evaluated in the hospital for respiratory distress. The patient has well-controlled generalized epilepsy and was admitted 5 days earlier for a cervical discectomy, laminectomy, and fusion. On hospital day 4, he developed a productive cough, chills, and dyspnea. Medications are oxycodone, levetiracetam, and docusate sodium. On physical examination, temperature is 38.4 °C (101.1 °F), blood pressure is 125/84 mm Hg, pulse rate is 108/min, respiration rate is 22/min, and oxygen saturation with the patient breathing ambient air is 93%. Crackles are heard in the right posterior thorax on pulmonary auscultation. Results of laboratory studies show a leukocyte count of 18,400/µL (18.4 × 109/L). A chest radiograph shows a right lower lobe infiltrate. Which of the following is the most appropriate treatment? Cefepime Imipenem Levofloxacin Piperacillin-tazobactam
Although evidence is limited, carbapenems, fluoroquinolones, and fourth-generation cephalosporins may lower the seizure threshold and thus should be avoided in patients with epilepsy. Given his history of epilepsy, this patient should receive an injection of piperacillin-tazobactam as treatment of his hospital-acquired pneumonia. This drug has the lowest risk of triggering seizures of the drugs listed. Although all four choices may trigger seizures, piperacillin-tazobactam and non-fourth-generation cephalosporins have the weakest association with triggering seizures in patients with epilepsy (class IV evidence that seizure association is not well established in patients with epilepsy or past seizures). Cefepime and carbapenems (such as imipenem) have class III evidence of triggering seizures in patients with epilepsy. In addition, cefepime has been known to cause encephalopathy, coma, and status epilepticus in patients with or without epilepsy, especially those with acute kidney injury. There is class IV evidence that fluoroquinolones (such as levofloxacin) and fourth-generation cephalosporins lower the seizure threshold, including in patients with epilepsy.
A 31-year-old man is evaluated near the end of a guided climb of a 3500-meter (11,482 feet) summit in the French Alps. He is confused and increasingly irritable. His only medication was prophylactic acetazolamide, which he discontinued due to bothersome nocturia. On physical examination, pulse rate is 128/min and respiration rate is 22/min. In addition to confusion, his gait is ataxic. The neurological examination is otherwise nonfocal. Supplemental oxygen is administered and arrangements are being made for descent. Which of the following is the most appropriate additional treatment? Acetazolamide Dexamethasone Nifedipine Sildenafil
Although the most important treatment of high-altitude cerebral edema is descent to lower elevation, dexamethasone should be administered immediately upon recognition of high-altitude cerebral edema. The most appropriate treatment is dexamethasone. Hypoxia and hypocapnia associated with altitude alter cerebral blood flow and oxygen delivery to the brain. When autoregulatory mechanisms are overcome, symptoms may be mild, as with acute mountain sickness, or severe, as with life-threatening cerebral edema. Acute mountain sickness is characterized by nonspecific symptoms such as headache, fatigue, nausea, and vomiting, in addition to disturbed sleep. High-altitude cerebral edema is a more extreme manifestation of acute mountain sickness. Vascular leak leads to brain swelling, resulting in manifestations that range from confusion and irritability to ataxic gait to coma and death. Recognition of cerebral edema mandates immediate intervention. This patient is exhibiting signs and symptoms of encephalopathy indicative of high-altitude cerebral edema, the risk of which increases at more extreme elevations (higher than 3000 meters [9842 feet]). Although the most important intervention is descent to lower elevation, dexamethasone should be administered immediately upon recognition of high-altitude cerebral edema. Supplemental oxygen should also be administered. High-altitude illness can be prevented by gradually ascending, which can generally be accomplished by spending one night at an intermediate altitude to allow acclimatization. Acetazolamide accelerates the acclimatization process to high altitude by inducing a slight metabolic acidosis to stimulate ventilation and enhance gas exchange; it can be used prophylactically in patients with a history of altitude illness. However, it isn't an effective treatment once symptoms develop; although it may be used as an adjunct to dexamethasone, it has no role in high-altitude cerebral edema as monotherapy. Nifedipine is used as a preventive and therapeutic agent for high-altitude pulmonary edema. Another vasodilator, sildenafil, may be used as an alternative to nifedipine in high-altitude pulmonary edema. However, neither is useful in the treatment of high-altitude cerebral edema.
A 35-year-old man is evaluated during a follow-up appointment for persistent heartburn with chronic cough. He has a 1-year history of gastroesophageal reflux disease and takes pantoprazole twice daily. He reports no nausea, vomiting, or dysphagia. Upper endoscopy performed 1 year earlier showed no abnormal findings. His vital signs and physical examination are normal. Results of an ear, nose, and throat evaluation are noncontributory. Which of the following is the most appropriate next diagnostic test? Ambulatory pH testing Barium esophagography Esophageal manometry Upper endoscopy
Ambulatory pH testing can be a helpful diagnostic test in patients with suspected extraesophageal manifestations of gastroesophageal reflux disease. Ambulatory pH testing is the most appropriate next diagnostic test for this patient. The patient has persistent cough, which may be an extraesophageal symptom of gastroesophageal reflux disease (GERD) resulting from laryngopharyngeal reflux. Additional extraesophageal symptoms of GERD include asthma, globus sensation, hoarseness, throat clearing, and chronic laryngitis. It appears that the laryngopharynx is more sensitive to the erosive effects of acid, and small amounts of reflux may produce symptoms. The selection of a diagnostic test to confirm or exclude laryngopharyngeal reflux is controversial. Ambulatory pH testing, if positive, can help to confirm the diagnosis of GERD and supports the diagnosis of laryngopharyngeal reflux. Negative ambulatory pH testing suggests that the patient does not have GERD and that proton pump inhibitor therapy should be discontinued and another cause of the persistent hoarseness sought. Other experts propose laryngoscopy as the gold standard to diagnose laryngopharyngeal reflux. Common findings during laryngoscopy include edema and erythema, but these findings are also seen in 80% of healthy controls. While laryngoscopy should not be used as the sole test to diagnose extraesophageal GERD, abnormal findings in patients with an appropriate clinical history suggest that cough is related to GERD. Other causes of cough may include allergy, smoking, and voice abuse, and these should be ruled out with an ear, nose, and throat evaluation. Barium esophagography should not be used as the initial diagnostic test for GERD and is not useful for evaluating this patient's symptoms, which primarily involve the laryngopharynx. Esophageal manometry is used for patients suspected of having an underlying motility disorder involving peristalsis or lower esophageal sphincter dysfunction. This patient reports no dysphagia, a common presenting symptom of motility disorders, so esophageal manometry is not indicated. Upper endoscopy is the primary tool used to evaluate patients with GERD for complications such as erosive esophagitis, stricture, Barrett esophagus, and esophageal cancer. Because this patient does not have esophageal symptoms and had a normal endoscopy 1 year ago, upper endoscopy is not indicated.
A 54-year-old woman is evaluated for flushing of the face of 1 year's duration. These episodes occur two or three times per week and last about 30 minutes. She went through menopause at age 50 and is on estrogen and progesterone hormone therapy. She also experiences episodes of anxiety, diaphoresis, and tachycardia. Medical history is significant for increasingly frequent migraine headaches, difficult to control hypertension, and gastroesophageal reflux disease. Medications are amitriptyline, chlorthalidone, metoprolol, conjugated estrogens, progesterone, and omeprazole. On physical examination, blood pressure is 156/92 mm Hg; the remainder of the vital signs is normal. BMI is 32. The remainder of the examination is unremarkable. Which of the following medications should be discontinued prior to screening for secondary causes of hypertension? Amitriptyline Chlorthalidone Metoprolol Omeprazole Progesterone
Amitriptyline can cause falsely elevated normetanephrine levels and should be discontinued prior to screening for pheochromocytoma. Most pheochromocytomas secrete norepinephrine, resulting in episodic or sustained hypertension. Orthostatic hypotension can also be seen and likely reflects low plasma volume. In addition to the classic triad of diaphoresis, headache, and tachycardia, common symptoms include palpitations, tremor, pallor, and anxiety. Screening for pheochromocytoma is appropriate in this patient, following discontinuation of amitriptyline. Amitriptyline acts by inhibiting norepinephrine uptake into nerve terminals, with subsequent elevation of its metabolite, normetanephrine. False-positive elevation of plasma free normetanephrine levels can occur with other tricyclic medications such as nortriptyline or combination serotonin/norepinephrine uptake inhibitors such as venlafaxine or duloxetine. False-positive elevation of plasma normetanephrine and metanephrine levels can also occur with other medications including levodopa (a substrate for catecholamine synthesis); psychoactive medications such as buspirone, prochlorperazine, amphetamines; and over-the-counter decongestant medications that contain adrenergic receptor agonists. Plasma free metanephrines can also be elevated during acute or stressful medical situations including psychiatric illness. Therefore, unless there is significant suspicion for pheochromocytoma, testing should be delayed until the acute illness has passed. Medications that can interfere with catecholamine metabolism should be discontinued (with tapering if indicated) at least 2 weeks prior to testing for pheochromocytoma. Omeprazole, chlorthalidone, metoprolol, and progesterone do not impact catecholamine metabolism and, therefore, can be continued during screening for pheochromocytoma. Read Related TextNext Question
A 34-year-old man is evaluated immediately after a dog bite to his thigh. Medical history is notable for splenectomy 5 years ago. The patient and dog are up to date on all immunizations. He takes no medications. On physical examination, vital signs are normal. A tiny puncture wound is located on the right mid-thigh with minimal surrounding erythema. Which of the following is the most appropriate management? Amoxicillin-clavulanate Ciprofloxacin Metronidazole Observation
Amoxicillin-clavulanate is recommended for patients with animal bites who are immunosuppressed (including patients with cirrhosis and asplenia); have wounds with associated edema, lymphatic or venous insufficiency, or crush injury; have wounds involving a joint or bone; have deep puncture wounds; or have moderate to severe injuries, especially when involving the face, genitalia, or hand. The most appropriate management for this patient would be administration of amoxicillin-clavulanate. Because of asplenia, he is immunodeficient and should receive prophylactic antibiotic therapy after the dog bite, even without evidence of infection. Infections typically result from the host's skin flora and the animal's mouth flora. This flora is a mix of aerobic and anaerobic organisms, including staphylococci, streptococci, Bacteroides species, Porphyromonas species, Fusobacterium species, Capnocytophaga canimorsus, and Pasteurella species. C. canimorsus is a gram-negative bacillus that can cause overwhelming sepsis in patients with functional or anatomic asplenia who have experienced a dog bite or scratch. Because of its activity against pathogens associated with animal bite wounds, a 3- to 5-day course of amoxicillin-clavulanate is recommended for patients who are immunosuppressed (including patients with cirrhosis and asplenia); have wounds with associated edema, lymphatic or venous insufficiency, or crush injury; have wounds involving a joint or bone; have deep puncture wounds; or have moderate to severe injuries, especially when involving the face, genitalia, or hand. If a patient is allergic to penicillin, a combination of trimethoprim-sulfamethoxazole or a fluoroquinolone or doxycycline plus clindamycin or metronidazole can be used. Because these infections are typically polymicrobial, consisting of aerobic and anaerobic bacteria, neither ciprofloxacin nor metronidazole alone would be adequate. Ciprofloxacin lacks anaerobic bacterial coverage, and metronidazole lacks aerobic activity. Because of this the patient's immunodeficiency, observation alone would not be recommended by the Infectious Diseases Society of America guidelines.
A 62-year-old woman is evaluated for a "sun allergy," manifesting as a rash on her scalp, eyelids, upper back, and knuckles. This started 2 years ago, but is less pronounced over the winter months. Medical history is unremarkable. Her only medication is hydrocortisone cream. On physical examination, vital signs are normal. Pink-violet edematous macules are present on the eyelids. The scalp has pink-violet diffuse scaly macules with some thinning of the hair. There is a poikilodermatous pink scaly plaque on the upper back and posterior neck. The metacarpophalangeal and proximal interphalangeal joints have pink-violet flat-topped papules on their dorsal surface. There are dilated periungual capillary loops on each finger, and all 10 cuticles are dystrophic. There are no swollen joints, no abnormalities on strength testing, and no oral ulcerations. Laboratory studies show antinuclear antibody titer of 1:640. All other laboratory values, including creatine kinase , aldolase, aspartate aminotransferase , and gamma-glutamyl transferase, are normal. Which of the following is the most likely diagnosis? Amyopathic dermatomyositis Mixed connective tissue disease Polymorphus light eruption Systemic lupus erythematosus
Amyopathic dermatomyositis presents with skin findings characteristic of dermatomyositis, but without clinical or laboratory evidence of muscle disease; it carries risks for underlying malignancy. This patient has amyopathic dermatomyositis, an underrecognized presentation of dermatomyositis that does not include muscle disease. These patients experience characteristic cutaneous features of dermatomyositis, such as the heliotrope sign, shawl sign, and Gottron papules, but muscle enzymes and strength testing are normal. Skin findings in dermatomyositis are photosensitive and tend to flare after sun exposure. Amyopathic dermatomyositis carries similar risk for underlying malignancy and pulmonary fibrosis. There is no strong consensus on how to screen for malignancy. Mixed connective tissue disease is an overlap syndrome that includes features of systemic lupus erythematosus (SLE), systemic sclerosis, and/or polymyositis in the presence of anti-U1-ribonucleoprotein antibodies. Skin manifestations include sclerodactyly, scleroderma, calcinosis, telangiectasias, photosensitivity, malar rash, and Gottron rash. The absence of findings consistent with SLE and systemic sclerosis make this diagnosis unlikely. Polymorphous light eruption (PMLE) is the most common idiopathic photosensitivity disorder. PMLE typically manifests before the age of 30 and is most common in fair-skinned women, first appearing in the spring and early summer. The rash will persist for weeks and resolve without scarring, even with continued exposure to the sun. Lesions appear within hours of sun exposure and are found on sun-exposed body parts. Although many different types of eruptions may occur, the most common are pruritic skin-colored or pink papules. PMLE does not explain the positive antinuclear antibody titer or periungual findings. The most specific skin manifestations of systemic lupus erythematosus are variations on interface dermatitis, with pink-to-violet macules or plaques and varying scale or atrophy. Cutaneous lupus as a rule spares the upper eyelids and the knuckles. Inflammatory joint involvement occurs in 90% of patients with SLE. The absence of joint findings, rash over the knuckles, and periungual findings are not consistent with SLE. ...
A 70-year-old man is evaluated for a 6-month history of right upper extremity weakness, intermittent painful spasms in both calves, and occasional brief muscle twitches. The patient reports right arm and hand weakness and says that for the past 2 weeks, his right foot drags when he walks quickly. He has had no numbness, sensory loss, paresthesia, or problems with swallowing, sphincter control, or coordination. On physical examination, vital signs are normal. Weakness and muscle wasting is noted in the thumb abductor and intrinsic hand muscles on the right side. Spontaneous muscle twitches are present in the bilateral thumb adductor, gastrocnemius, and paraspinal muscles. Minor weakness of right ankle dorsiflexion, a right Hoffman sign, and right foot clonus is noted. Deep tendon reflexes are brisk throughout. Cranial nerves are intact, as is sensory perception. Electromyography and nerve condition studies reveal evidence of lower motor neuron involvement in the upper and lower extremities and paraspinal muscles. MRIs of the brain and cervical spine are unremarkable. Which of the following is the most likely diagnosis? Amyotrophic lateral sclerosis Charcot-Marie-Tooth disease Granulomatosis with polyangiitis Miller Fisher variant of Guillain-Barré syndrome
Amyotrophic lateral sclerosis is characterized by upper motor neuron signs (hyperreflexia, spasticity, and an extensor plantar response) coexistent with lower motor neuron findings (atrophy and fasciculation); sensory deficits are characteristically absent.
A 70-year-old man is evaluated for a 6-month history of right upper extremity weakness, intermittent painful spasms in both calves, and occasional brief muscle twitches. The patient reports right arm and hand weakness and says that for the past 2 weeks, his right foot drags when he walks quickly. He has had no numbness, sensory loss, paresthesia, or problems with swallowing, sphincter control, or coordination. On physical examination, vital signs are normal. Weakness and muscle wasting is noted in the thumb abductor and intrinsic hand muscles on the right side. Spontaneous muscle twitches are present in the bilateral thumb adductor, gastrocnemius, and paraspinal muscles. Minor weakness of right ankle dorsiflexion, a right Hoffman sign, and right foot clonus is noted. Deep tendon reflexes are brisk throughout. Cranial nerves are intact, as is sensory perception. Electromyography and nerve condition studies reveal evidence of lower motor neuron involvement in the upper and lower extremities and paraspinal muscles. MRIs of the brain and cervical spine are unremarkable. Which of the following is the most likely diagnosis? Amyotrophic lateral sclerosis Charcot-Marie-Tooth disease Granulomatosis with polyangiitis Miller Fisher variant of Guillain-Barré syndrome
Amyotrophic lateral sclerosis is characterized by upper motor neuron signs (hyperreflexia, spasticity, and an extensor plantar response) coexistent with lower motor neuron findings (atrophy and fasciculation); sensory deficits are characteristically absent. This patient most likely has amyotrophic lateral sclerosis (ALS). Diagnosis requires fulfillment of positive diagnostic criteria and exclusion of mimics. Positive diagnostic criteria for ALS include the presence of clinical upper (hyperreflexia, Hoffman sign, clonus) and lower (atrophy, fasciculation, weakness) motor neuron signs and electromyographic lower motor neuron signs in two (probable ALS) or more (definitive ALS) body regions (upper and lower limbs, paraspinal, bulbar). Because hyperthyroidism can lead to a combination of upper and lower motor neuron findings that may mimic ALS, it must be excluded before diagnosis. Other important mimics include structural brain and cervical spinal lesions, vitamin B12 and copper deficiency, Lyme disease, and hyperparathyroidism. Charcot-Marie-Tooth (CMT) disease is an inherited neuropathy arising from mutations in several genes encoding for myelin formation, structure, and function. The two most common forms are demyelinating (CMT1) and axonal (CMT2). Both share the clinical features of numbness, distal extremity weakness, unsteady gait, areflexia, high arches, hammer toes, and atrophy of distal extremity muscles and foreleg muscles ("stork leg" deformity). This patient's findings are not consistent with CMT disease. Granulomatosis with polyangiitis is an antineutrophil cytoplasmic antibody-associated vasculitis. Areas most commonly affected by the vasculitis include the airways, lung parenchyma, kidneys, skin, eyes, and nervous system. The vasculitic neuropathy can present as mononeuropathy multiplex and are associated with focal or multifocal weakness. However, these entities are associated with pain and sensory deficits and not with upper motor neuron signs. The Miller Fisher variant of Guillain-Barré syndrome typically presents with subacute ataxia, areflexia, and ophthalmoplegia, with or without diffuse weakness. Antibodies to GQ1b (a ganglioside component of nerve) are present in more than 85% of patients. The patient's findings are not compatible with the Miller Fisher variant.
A 44-year-old man is evaluated during a follow-up visit for membranous glomerulopathy, which was diagnosed last week on kidney biopsy. He has no other pertinent personal or family history. His only medication is furosemide. On physical examination, vital signs are normal. There is trace bilateral lower extremity edema to the ankles. The remainder of the examination is unremarkable. Laboratory studies performed before kidney biopsy: Albumin 3.0 g/dL (30 g/L) Total cholesterol 310 mg/dL (8.0 mmol/L) Creatinine 0.8 mg/dL (70.7 µmol/L) Antinuclear antibodies Negative Anti-phospholipase A2 receptor antibodies Titer: 1:80 Hepatitis B surface Ag and Ab antibodies Negative Hepatitis C Ab antibodies Negative HIV antibodies Negative 24-Hour urine protein excretion 6500 mg/24 h Ultrasound of the kidneys shows normal appearance with no evidence of thrombus in the renal veins. An ACE inhibitor and a statin are initiated. Which of the following is the most appropriate additional management? Alternating course of glucocorticoids and alkylating agents Anti-double-stranded DNA antibody measurement Cyclosporine Hepatitis B and hepatitis C viral polymerase chain reaction testing No additional management at this time
An ACE inhibitor and statin are appropriate for this patient with recently diagnosed primary membranous glomerulopathy. His kidney biopsy findings are consistent with the diagnosis, and the presence of anti-phospholipase A2 receptor (PLA2R) antibodies has been shown to approach 100% specificity for the primary form of this disease. Approximately one third of patients with primary membranous glomerulopathy experience a spontaneous remission of disease over the first 6 to 24 months without immunosuppression. Therefore, within the first 6 months of diagnosis, barring signs of severe complications of the nephrotic syndrome (such as kidney failure, anasarca, or deep vein thrombosis), the recommended strategy is to treat patients with primary membranous glomerulopathy conservatively with renin-angiotensin system blockers, cholesterol-lowering medications (if cholesterol is above goal), and diuretics (for edema). The patient is then monitored with examinations and laboratory studies to gauge for spontaneous remission. If proteinuria increases in 6 to 12 months, a course of immunosuppression should be considered for those with persistent nephrotic-range proteinuria. With the advent of serologic testing for anti-PLA2R antibodies, these titers can now be followed during the observation period alongside traditional clinical parameters, such as proteinuria. Falling anti-PLA2R titers are associated with remission, whereas persistently high titers are associated with ongoing disease activity. Alternating months of glucocorticoids and alkylating agents is first-line immunosuppressive therapy of choice for primary membranous glomerulopathy, and substituting with a calcineurin inhibitor such as cyclosporine is now considered a viable alternative for patients with contraindications to alkylating agents. However, utilization of such immunosuppressive therapies at this point is premature and runs the risk of treating a patient who may remit spontaneously. Although hepatitis B and C virus infections, along with lupus, are well-known forms of secondary membranous glomerulopathy in adults, this patient's screening tests are negative and, with PLA2R antibody positivity, further testing is unnecessary.
A 45-year-old woman is evaluated for elevated blood pressure found for the first time at her previous visit 1 month ago. She has a 7-year history of type 2 diabetes mellitus without retinopathy, as well as hyperlipidemia. Medications are metformin and atorvastatin. On physical examination, blood pressure is 148/94 mm Hg (confirmed by home ambulatory blood pressure monitoring), and pulse rate is 74/min; other vital signs are normal. BMI is 32. The remainder of the physical examination is unremarkable. Laboratory studies show a serum creatinine level of 0.9 mg/dL (79.6 µmol/L), a serum potassium level of 3.8 mEq/L (3.8 mmol/L), and a urine albumin-creatinine ratio of 50 mg/g. The patient is instructed in appropriate lifestyle modifications. Which of the following is the most appropriate treatment? Begin amlodipine Begin chlorthalidone Begin losartan Remeasure blood pressure in 2 months
An ACE inhibitor or angiotensin receptor blocker is the initial treatment of choice for hypertension in patients with diabetes mellitus and albuminuria. In addition to lifestyle modifications, initiation of the angiotensin receptor blocker (ARB) losartan is the most appropriate treatment for this patient with newly diagnosed hypertension. She has a normal serum creatinine level but has moderately increased albuminuria. Her blood pressure has been elevated at two office visits and confirmed with home blood pressure monitoring, and she needs to be treated to target, given that uncontrolled hypertension will lead to progression of albuminuria and chronic kidney disease (CKD). A reasonable target blood pressure for this patient is <130/80 mm Hg. The 2017 high blood pressure guideline from the American College of Cardiology (ACC), American Heart Association (AHA), and nine other organizations recommends a treatment goal of <130/80 mm Hg. The ACC/AHA guideline notes that ACE inhibitors or ARBs may be considered as initial treatment choices in the presence of albuminuria. The American Diabetes Association (ADA) Standards of Medical Care in Diabetes 2018 recommends that most patients with diabetes mellitus and hypertension should be treated to a systolic blood pressure goal of <140 mm Hg and a diastolic blood pressure goal of <90 mm Hg. Lower systolic and diastolic blood pressure targets, such as 130/80 mm Hg, may be appropriate for individuals at high risk of cardiovascular disease if they can be achieved without undue treatment burden. In nonpregnant patients with diabetes and hypertension, the ADA recommends either an ACE inhibitor or an ARB for those with albuminuria. Several large randomized controlled trials and systematic reviews have shown that use of an ARB or ACE inhibitor can result in decreased progression of albuminuria and CKD in patients with diabetes. The ACC/AHA blood pressure guideline recommends the following lifestyle modifications for individuals with elevated blood pressure or hypertension: weight loss in adults with overweight or obesity; a heart-healthy diet, such as DASH (Dietary Approaches to Stop Hypertension), that facilitates achieving a desirable weight; sodium reduction; potassium supplementation, preferably in dietary modification, unless contraindicated by the presence of CKD or use of drugs that reduce potassium excretion; increased physical activity with a structured exercise program; and limiting alcohol consumption of standard drinks to two (men) and one (women) per day. Although these recommendations are appropriate for this patient, it is not the only recommended intervention to control hypertension in this patient with diabetes and albuminuria. Therefore, simply asking the patient to return in 2 months for a blood pressure measurement after initiating lifestyle changes is not sufficient. Calcium channel blockers (such as amlodipine) and thiazide diuretics (such as chlorthalidone) are not initial choices for treating hypertension in patients with diabetes and albuminuria. If this patient's blood pressure is still not controlled to target after maximizing the dose of the ARB, then one of these agents can be added.
A 24-year-old woman is evaluated for 6 months of amenorrhea, weight gain, and depressed mood. Medical history is otherwise unremarkable, and she takes no medications. On physical examination, blood pressure is 134/86 mm Hg and pulse rate is 82/min. BMI is 31. Other vital signs are normal. The patient has facial plethora. Skin examination reveals multiple ecchymoses. There are wide pigmented striae on the abdomen as well as a dorsocervical fat pad. Laboratory studies: Cortisol , free, urine Initial measurement 120 µg/24 h (330.7 nmol/24 h) Repeat measurement 240 µg/24 h (661.3 nmol/24 h) Cortisol after 1 mg dexamethasone test 6.0 µg/dL (165.6 nmol/L) Which of the following is the most appropriate diagnostic test to perform next? Adrenocorticotropic hormone (ACTH) level 8-mg Dexamethasone suppression test Inferior petrosal sinus sampling for ACTH Pituitary MRI
An adrenocorticotropic hormone (ACTH) measurement should be obtained once the diagnosis of Cushing syndrome is established to determine if it is ACTH dependent or ACTH independent. The most appropriate diagnostic test for this patient is measurement of the adrenocorticotropic hormone (ACTH) level. Cushing disease is the term used to indicate excess cortisol production due to an ACTH-secreting pituitary adenoma. Cushing syndrome refers to hypercortisolism from any cause, exogenous or endogenous, ACTH-dependent or not. The most common cause of endogenous Cushing syndrome is Cushing disease. The initial step in evaluation for Cushing disease is to seek biochemical evidence of hypercortisolism. At least two first-line tests should be diagnostically abnormal before the diagnosis is confirmed. Initial tests include the overnight low-dose dexamethasone suppression test, 24-hour urine free cortisol, and late-night salivary cortisol. The 24-hour urine free cortisol and late night salivary cortisol tests should be performed at least twice to ensure reproducibility of results. The abnormal dexamethasone suppression test and elevated urine free cortisol levels establish the diagnosis of Cushing syndrome in this patient. An ACTH measurement should be obtained once the diagnosis of Cushing syndrome is established to determine if it is ACTH dependent or ACTH independent. Once ACTH-dependent Cushing syndrome is confirmed biochemically, a pituitary MRI should be obtained. If no pituitary tumor or a tumor less than 6 mm is visualized on MRI, an 8-mg dexamethasone suppression test is used to differentiate Cushing disease from an ectopic source of ACTH. Dexamethasone is administered at 11 PM, and cortisol is tested at 8 AM. A pituitary source of ACTH will respond to negative feedback from high doses of dexamethasone, suppressing plasma cortisol at 8 AM by more than 50%, whereas an ectopic source of ACTH will not have suppressible cortisol. However, this test has low sensitivity (88%) and specificity (57%) for Cushing disease, so inferior petrosal sinus sampling (IPSS) is often recommended before exploratory pituitary surgery. In IPSS, ACTH levels in the petrosal sinus are compared with those in the periphery after the administration of corticotropin-releasing hormone (CRH). A central to peripheral gradient greater than 2.0 before CRH or greater than 3.0 after CRH is diagnostic of Cushing disease. An 8-mg dexamethasone suppression test, inferior petrosal sinus sampling, and pituitary MRI should follow, if necessary, ACTH testing. Their inclusion in the diagnostic algorithm at this point is premature and possibly unnecessary.
A 62-year-old woman with a history of moderate COPD is evaluated in the emergency department for increasing dyspnea, cough, and sputum production. She had been doing well until 4 days ago when she developed rhinorrhea and a cough productive of purulent sputum. During the past two days she has noted increasing dyspnea on exertion that responds transiently to her albuterol inhaler. This is the patient's first episode of this nature. She stopped smoking 18 months ago. Current medications are albuterol and tiotropium inhalers. On physical examination, temperature is 38.1 °C (100.6 °F), blood pressure is 130/80 mm Hg, pulse rate is 102/min, and respiration rate is 22/min. Oxygen saturation is 90% on ambient air. The patient is tachypneic and catches her breath between sentences but improves after a treatment of albuterol. She has diffuse end-expiratory wheezing. A chest radiograph shows the lungs to be clear but hyperinflated. Arterial blood gases on 2L/min of oxygen through nasal cannula show a pH 7.38, a PCO2 of 42 mm Hg (5.58 kPa), and a PO2 of 70 mm Hg (9.31 kPa). Which of the following is the most appropriate treatment? Azithromycin and prednisone Mometasone inhaler Noninvasive positive pressure ventilation Roflumilast
An exacerbation of COPD is defined as a sustained worsening of symptoms, typically cough, dyspnea, and sputum production; standard treatment of moderate to severe exacerbations includes antibiotics and oral glucocorticoids. The most appropriate treatment of the COPD exacerbation in this patient is azithromycin and prednisone. Exacerbations are marked by increased breathlessness and are usually accompanied by increased cough and sputum production. The degree of exacerbation is considered mild when a change in the clinical condition is noted but no change in medication is necessary. An exacerbation is considered moderate when medication changes are made. A severe exacerbation results in hospitalization. Short-acting bronchodilator therapy is a mainstay of therapy for treating COPD exacerbation. Glucocorticoids, such as prednisone, have been shown to reduce recovery time, improve lung function and arterial hypoxemia, decrease risk of early relapse, decrease treatment failure, and decrease length of hospital stay. Guidelines recommend 40 mg of prednisone or an oral equivalent for 5 to 7 days. The most recent Global Initiative for Chronic Obstructive Lung Disease report recommends that antibiotics should be considered in patients with moderate or severe COPD and symptoms of increased dyspnea, increased sputum, and sputum purulence. Recent studies in patients with moderate COPD have demonstrated improved patient outcomes. Commonly used regimens include an advanced macrolide (such as azithromycin), a cephalosporin, or doxycycline. This patient's COPD appears to be well-controlled at baseline, so the addition of an inhaled glucocorticoid such as mometasone is not indicated for long-term COPD management. An inhaled glucocorticoid is not an effective treatment of an exacerbation of COPD. Noninvasive positive pressure ventilation (NIPPV) has a significant role in the management of patients with very severe COPD during an acute exacerbation and may be helpful to avoid intubation. NIPPV is strongly recommended in patients with acute COPD exacerbations who have respiratory acidosis. This patient does not have an indication for NIPPV. Although roflumilast has been shown to decrease the frequency of recurrent exacerbations, there is no role for this agent in the treatment of an acute exacerbation. As this is a first exacerbation of COPD, chronic roflumilast therapy is not indicated.
A 29-year-old man is hospitalized for lower extremity edema and fatigue that has progressed over the past 6 months. Laboratory studies document kidney failure. History is notable for obesity. He has a remote history of intravenous drug use and a 5-year history of multiple sex partners (men and women). He takes no medications. On physical examination, the patient is afebrile, and blood pressure is 148/94 mm Hg; other vital signs are normal. BMI is 38. There is no rash. There is pitting edema in the lower extremities to the ankles bilaterally. The remainder of the physical examination is unremarkable. Laboratory studies: C3 60 mg/dL (600 mg/L) C4 7.0 mg/dL (70 mg/L) Creatinine 2.8 mg/dL (247.5 µmol/L) Urinalysis 3+ blood; 3+protein Urine protein-creatinine ratio 2900 mg/g Kidney biopsy shows membranoproliferative glomerulonephritis on light microscopy, with immunofluorescence microscopy showing 3+ staining for IgG, 1+ staining for IgM, 2+ staining for C1q, and 2+ staining for C3. Results of which of the following tests will most likely explain this patient's findings? Genetic mutations in alternative complement pathway proteins Hepatitis B surface antigen and surface antibodies Hepatitis C antibodies HIV antibodies
An immune-complex membranoproliferative glomerulonephritis is the classic form of kidney involvement seen in patients with hepatitis C virus infection.
A 29-year-old man is hospitalized for lower extremity edema and fatigue that has progressed over the past 6 months. Laboratory studies document kidney failure. History is notable for obesity. He has a remote history of intravenous drug use and a 5-year history of multiple sex partners (men and women). He takes no medications. On physical examination, the patient is afebrile, and blood pressure is 148/94 mm Hg; other vital signs are normal. BMI is 38. There is no rash. There is pitting edema in the lower extremities to the ankles bilaterally. The remainder of the physical examination is unremarkable. Laboratory studies: C3 60 mg/dL (600 mg/L) C4 7.0 mg/dL (70 mg/L) Creatinine 2.8 mg/dL (247.5 µmol/L) Urinalysis 3+ blood; 3+protein Urine protein-creatinine ratio 2900 mg/g Kidney biopsy shows membranoproliferative glomerulonephritis on light microscopy, with immunofluorescence microscopy showing 3+ staining for IgG, 1+ staining for IgM, 2+ staining for C1q, and 2+ staining for C3. Results of which of the following tests will most likely explain this patient's findings? Genetic mutations in alternative complement pathway proteins Hepatitis B surface antigen and surface antibodies Hepatitis C antibodies HIV antibodies
An immune-complex membranoproliferative glomerulonephritis is the classic form of kidney involvement seen in patients with hepatitis C virus infection. The most appropriate test to perform next is measurement of hepatitis C antibodies. This patient presents with glomerulonephritis (elevated serum creatinine level, hematuria, and subnephrotic proteinuria), which shows a membranoproliferative (MPGN) pattern on kidney biopsy. The new approach to MPGN lesions is a bifurcation based on the pattern of staining on immunofluorescence microscopy. The more common pattern, as seen in this patient, is immune-complex deposition with the presence of both immunoglobulin (IgG, IgM, and/or IgA) and complement (C1q and/or C3) on immunofluorescence, which infers that the classical pathway has been activated by an inciting cause or event that generally falls into one of three major categories: infectious, autoimmune, or malignancy associated. The most common is infectious, specifically infection with hepatitis C virus (HCV). When an MPGN lesion on immunofluorescence microscopy shows only C3 staining (that is, without immunoglobulin or C1q staining), this extremely rare finding is suggestive of an antibody-independent means of complement activation and points to hyperactivity of the alternative complement pathway. In these C3 glomerulopathies, named based on the isolated C3 staining pattern seen on immunofluorescence, screening for genetic abnormalities in alternative complement pathway proteins is an appropriate part of the diagnostic evaluation. Hepatitis B virus infection and HIV infection have been linked to glomerular diseases, but these are classically associated with the nephrotic syndrome, specifically membranous glomerulopathy with hepatitis B virus infection and focal segmental glomerulosclerosis with HIV infection.
A 71-year-old woman is evaluated for a 3-year history of rheumatoid arthritis. She reports burning and tingling sensations in her feet and legs that are worse at night, without exacerbation with position or activity. She has not noted back pain, lower extremity weakness, or bowel or bladder dysfunction. Medications are methotrexate, hydroxychloroquine, low-dose prednisone, and folic acid. Her rheumatoid arthritis was moderately active 6 months ago, at which time leflunomide was added with good response. On physical examination, vital signs are normal. Decreased sensation to light touch and vibration is limited to the feet. Joint examination reveals one tender and two swollen metacarpophalangeal joints. Strength testing is intact throughout, and reflexes are intact. There are no skin rashes or lesions. Laboratory studies show normal complete blood count, serum creatinine, blood glucose, and hemoglobin A1c values. Which of the following is the most appropriate next step in management? Begin gabapentin Increase prednisone Stop leflunomide Stop methotrexate
An uncommon side effect of leflunomide is peripheral neuropathy, and definitive treatment is discontinuation of the medication. The most appropriate next step in management is to stop leflunomide. Leflunomide inhibits lymphocyte activation by blocking the pyrimidine synthesis pathway. It is approved to treat rheumatoid arthritis (RA), in which its efficacy is comparable to methotrexate. Common toxicities include gastrointestinal upset, diarrhea, aminotransferase elevations, cytopenias, infection, and teratogenesis. An uncommon side effect is peripheral neuropathy, which is usually axonal and may include sensory, motor, or mixed findings. The neuropathy can be severe but is usually self-limited if the drug is discontinued, highlighting the importance of early recognition. Low-dose methotrexate in combination with leflunomide is an infrequently employed, but proven, strategy for treating refractory RA. Beginning gabapentin as a treatment for peripheral neuropathy would be an appropriate adjunctive therapy to stopping leflunomide, but the definitive treatment is to stop the offending agent. High-dose glucocorticoids would be appropriate in the setting of rheumatoid vasculitis. Although peripheral neuropathy may be present in this rare complication of RA, rheumatoid vasculitis invariably occurs in the setting of poorly controlled disease. In addition, other manifestations of vasculitis would typically be present, including cutaneous findings (ulcers) and ocular disease (scleritis, episcleritis, uveitis). This patient has well-controlled RA and has no associated systemic or cutaneous findings of vasculitis, making the diagnosis extremely unlikely and the need for increasing the prednisone unnecessary. Methotrexate inhibits folic acid metabolism and increases extracellular adenosine levels. It is the recommended initial disease-modifying antirheumatic drug for most patients with RA. Methotrexate is administered weekly along with daily folic acid supplementation, which limits toxicity without affecting efficacy. Potential toxicities include hepatitis and bone marrow suppression (leukopenia, anemia). Patients with liver disease should not receive methotrexate, and limiting alcohol intake is strongly advised. Methotrexate does not cause peripheral neuropathy and should not be discontinued.
A 57-year-old postmenopausal woman is evaluated following the diagnosis of right breast ductal carcinoma in situ after lumpectomy. Pathologic findings showed a grade 3 ductal carcinoma in situ spanning 2.5 cm with negative margins that is estrogen receptor−positive; lymph nodes were not sampled. She has received primary breast radiation therapy. She takes no medications. On physical examination, vital signs are normal. There is a healed right breast incision. There are no breast masses or lymphadenopathy. Which of the following is the most appropriate management to decrease the risk of ipsilateral and contralateral breast cancer? Initiate anastrozole Initiate raloxifene Initiate tamoxifen plus docetaxel Obtain 21-gene recurrence score testing
Anastrozole adjuvant therapy for ductal carcinoma in situ in postmenopausal patients younger than age 60 years decreases the risk of recurrent ipsilateral and contralateral breast cancer but does not decrease overall survival. Anastrozole is the most effective adjuvant therapy for ductal carcinoma in situ (DCIS) in postmenopausal women younger than 60 years. In the National Surgical Adjuvant Breast and Bowel Project (NSABP) B-35 clinical trial, 5 years of anastrozole was found to be more effective than 5 years of tamoxifen at decreasing both ipsilateral and contralateral breast cancer in postmenopausal women with DCIS who were younger than 60 years when starting treatment. Almost 95% of the anastrozole group was free from local or contralateral recurrence compared to 88% of the tamoxifen group. Overall 10-year survival, approximately 90%, was comparable between the two groups. In women age 60 years or older, anastrozole and tamoxifen yielded equivalent results. All women in this trial were postmenopausal and underwent lumpectomy and breast radiation. Unlike in invasive breast cancer, there is no survival advantage to treatment with antiestrogens for patients with DCIS, and such treatment is discussed as an option for patients who wish to decrease their risk of local recurrence and contralateral breast cancers. Because this patient is younger than 60 years, anastrozole is the most effective adjuvant antiestrogen option to achieve this goal. Raloxifene is a selective estrogen receptor modulator (SERM) that can be used for breast cancer prophylaxis in patients with atypical breast lesions or in patients with a strong family history of breast cancer, but it is not an effective option for hormonal therapy in patients who have DCIS or invasive cancers. Chemotherapy is not indicated in the treatment of DCIS. Chemotherapy is used as adjuvant treatment to decrease the risk of distant metastases in many invasive breast cancers, but in DCIS the risk of distant metastases is only 1%. Tamoxifen alone is an option for DCIS and could be discussed as an alternative to anastrozole, particularly in patients who wish to avoid potential anastrozole side effects, such as arthralgia and decreased bone density. The 21-gene recurrence score testing is used in invasive cancers to determine the benefit of adding adjuvant chemotherapy in patients with estrogen receptor-positive cancers who will be treated with antiestrogen treatment. It is not helpful at determining the benefit of antiestrogen therapy in DCIS. The use of the 21-gene recurrence score in DCIS to determine if radiation can be safely omitted has been studied, but it is not yet approved for this use.
A 42-year-old man is evaluated for an elevated blood pressure measurement of 136/86 mm Hg found during a routine screening at his workplace. He exercises 5 days a week for 45 minutes and adheres to a low fat, low salt diet. Family history is significant for hypertension in his father and mother; his father died of a stroke at the age of 55 years. He takes no medications. The patient is black. On physical examination, the patient is well developed and muscular. The average of three blood pressure measurements is 126/75 mm Hg, and pulse rate is 52/min. BMI is 24.5. The remainder of the examination is unremarkable. Which of the following is the most appropriate management? Amlodipine Annual blood pressure screening Blood pressure screening in 3 months Hydrochlorothiazide
Annual blood pressure screening is appropriate for patients who are ≥40 years of age and persons at increased risk for hypertension. Annual blood pressure screening is the most appropriate management. This patient is healthy, physically active, not overweight, and does not meet criteria for hypertension based on these office blood pressure (BP) readings measured during one visit. However, he is at risk for future hypertension given his age (>40 years), black race, and a positive family history of hypertension. In addition, according to the American College of Cardiology/American Heart Association (ACC/AHA) guideline, the patient has elevated BP, defined as systolic BP between 120-129 mm Hg and diastolic BP <80 mm Hg. Although no interventions are needed during this visit, the patient does need to be screened for high BP at least annually or more frequently. The U.S. Preventive Services Task Force (USPSTF) recommendations have not been updated since the release of the ACC/AHA guideline, with borderline and normal BP defined by older guidelines; however, the USPSTF recommends screening for hypertension in adults ≥18 years of age to identify those at increased risk for cardiovascular disease from hypertension and to begin early interventions to decrease this risk. Adults aged 18 to 39 years with normal BP and without cardiovascular risk factors should be rescreened every 3 to 5 years. Those who are ≥40 years of age and persons at increased risk for hypertension should be screened annually. Beginning antihypertensive therapy with medications such as amlodipine or hydrochlorothiazide is inappropriate for this patient who does not meet the criteria for hypertension (defined by the ACC/AHA as a systolic BP ≥130 mm Hg and/or a diastolic BP ≥80 mm Hg). According to this guideline, patients with clinical cardiovascular disease and an average systolic BP ≥130 mm Hg or an average diastolic BP ≥80 mm Hg should be treated with lifestyle changes and medications for secondary prevention of cardiovascular events. Adults without clinical cardiovascular disease but an estimated 10-year atherosclerotic cardiovascular disease risk ≥10% and an average systolic BP ≥130 mm Hg or an average diastolic BP ≥80 mm Hg should also be treated with lifestyle interventions and pharmacologic therapy for primary prevention of cardiovascular disease.
A 55-year-old woman is evaluated during a routine visit. She was previously diagnosed with spirometry-confirmed mild COPD for which she was prescribed a short-acting bronchodilator. She has a 20-pack-year history of cigarette smoking, but she quit 5 years ago. She is currently asymptomatic and has never received an influenza vaccination or pneumococcal vaccination. Her only medication is an albuterol inhaler. On physical examination, vital signs are normal; oxygen saturation is 95% on ambient air. Lungs are clear to auscultation. The remainder of the physical examination is unremarkable. Which of the following vaccinations should the patient receive at this time? High-dose influenza Pneumococcal polysaccharide (PPSV23) and standard influenza PPSV23 and pneumococcal conjugate (PCV13) and standard influenza PPSV23 and PCV13
Annual influenza vaccination and the pneumococcal polysaccharide vaccine are recommended for all patients with chronic lung disease (COPD, emphysema, asthma).
A 55-year-old woman is evaluated during a routine visit. She was previously diagnosed with spirometry-confirmed mild COPD for which she was prescribed a short-acting bronchodilator. She has a 20-pack-year history of cigarette smoking, but she quit 5 years ago. She is currently asymptomatic and has never received an influenza vaccination or pneumococcal vaccination. Her only medication is an albuterol inhaler. On physical examination, vital signs are normal; oxygen saturation is 95% on ambient air. Lungs are clear to auscultation. The remainder of the physical examination is unremarkable. Which of the following vaccinations should the patient receive at this time? High-dose influenza Pneumococcal polysaccharide (PPSV23) and standard influenza PPSV23 and pneumococcal conjugate (PCV13) and standard influenza PPSV23 and PCV13
Annual influenza vaccination and the pneumococcal polysaccharide vaccine are recommended for all patients with chronic lung disease (COPD, emphysema, asthma). This patient should receive the pneumococcal polysaccharide vaccine and an annual influenza vaccine. Influenza vaccination has been shown to reduce serious illness (such as lower respiratory tract infections that require hospitalization) and death in patients with COPD. These vaccines should be administered annually in all patients with COPD. There are currently three different types of influenza vaccine available in the United States: inactivated influenza vaccine, live attenuated influenza vaccine, and recombinant trivalent influenza vaccine. Inactivated influenza vaccine is approved for use in all adults, including immunosuppressed persons and pregnant women. A high-dose inactivated influenza vaccine is approved for use in adults age 65 years and older; it has been shown to be modestly more effective than the standard-dose inactivated influenza vaccine in this patient population. Pneumococcal vaccination is indicated for all adults aged 65 years and older and for high-risk persons younger than 65 years. Two vaccines are currently available: pneumococcal polysaccharide vaccine (PPSV23) is composed of polysaccharide capsular material from 23 pneumococcal subtypes, whereas pneumococcal conjugate vaccine (PCV13) contains capsular material from 13 subtypes conjugated to a nontoxic protein, which increases its immunogenicity. For pneumococcal vaccine-naïve adults between the ages of 19 and 65 years with certain immunocompromising conditions or who are otherwise at high risk, a single dose of PCV13 should be given. These conditions include functional or anatomic asplenia, cerebrospinal fluid leaks, cochlear implants, and conditions causing immunosuppression. All patients should also receive the 13-valent pneumococcal conjugate vaccine at age 65 years, although the polysaccharide and conjugate vaccines should be given sequentially at least a year apart for immunocompetent adults over age 65 rather than together for optimal effect. COPD alone is not an indication for PCV13 vaccination. PPSV23 has the same indications as the PCV13 vaccine, plus it is indicated in immunocompetent people with certain chronic medical conditions such as heart, liver, and lung disease (COPD, emphysema, asthma) and diabetes, as well as in cigarette smokers. PPSV23 revaccination should be given at age 65 years if 5 years have elapsed since the previous pneumococcal immunization. When possible, the PCV13 vaccine should be administered first, followed by a dose of PPSV23 at least 1 year later for most immunocompetent patients. Some patients with immunocompromising conditions, cochlear implants, or cerebrospinal fluid leaks should receive the dose of PPSV23 at least 8 weeks after the first dose of PCV13. If a patient has already received the PPSV23 vaccine, a single dose of PCV13 should be given at least 1 year after the administration of PPSV23.
A 26-year-old woman with Crohn disease is evaluated for a 2-week history of worsening abdominal pain in the right lower quadrant. She reports passage of one to two formed and nonbloody stools per day with no changes in bowel habits. The patient has required three courses of prednisone for disease flares over the past year. Her only medication is azathioprine. On physical examination, temperature is 37.7 °C (99.9 °F) and pulse rate is 115/min; other vital signs are normal. Abdominal examination shows fullness and tenderness in the right lower quadrant with no distinct mass. The remainder of the examination is unremarkable. Laboratory studies show a hemoglobin level of 10.5 g/dL (105 g/L) and a C-reactive protein level of 32 mg/dL (320 mg/L). Leukocyte count and liver chemistry tests are normal. CT enterography shows asymmetric mural thickening and mucosal inflammation of a long segment of distal ileum without luminal narrowing. Which of the following is the most appropriate treatment? Budesonide Infliximab Mesalamine Prednisone
Anti-tumor necrosis factor agents such as infliximab are effective in inducing and maintaining remission in moderate to severe Crohn disease. Infliximab is the most appropriate treatment for this patient. This patient has moderate to severe ileal Crohn disease that has required multiple courses of tapering prednisone for flares of disease over the last year, despite treatment with the immunomodulator azathioprine. Infliximab is an anti-tumor necrosis factor (TNF)-α antagonist effective in inducing and maintaining remission in moderate to severe Crohn disease. Other FDA-approved anti-TNF agents include adalimumab and certolizumab pegol. Evidence indicates that efficacy is better when an anti-TNF agent is used together with an immunomodulator. In addition, the risk for developing antibodies against the anti-TNF agent is lower with combination therapy. Patients whose disease does not respond to one anti-TNF agent are often switched to a second or third anti-TNF agent. Fibrostenosing Crohn disease in the absence of ongoing mucosal inflammation is unlikely to respond to any anti-TNF agent. Patients with no response to or intolerance of anti-TNF agents should be treated with either surgery or a leukocyte trafficking blocker (natalizumab or vedolizumab). Budesonide is a potent glucocorticoid with high first-pass metabolism in the liver, which limits systemic side effects related to conventional glucocorticoids. Budesonide can be an effective therapy for treating mild flares of ileal Crohn disease, but it is unlikely to induce remission in more severe Crohn disease and cannot be used to maintain remission. Mesalamine agents are mainly used to treat ulcerative colitis of mild to moderate severity. Mesalamine may have efficacy in treating mild to moderate Crohn colitis, but it is not efficacious in treating small-bowel Crohn disease. Reinitiating prednisone may induce remission of the patient's current disease flare, but it would not be helpful for maintenance of remission. Because this patient has required three separate tapering doses of glucocorticoids over the last year, she requires a medication such as infliximab that can both induce remission and maintain Crohn disease in remission.
A 25-year-old woman is seen for counseling. She is newly pregnant at 7 weeks' gestation and has HIV infection diagnosed 6 years ago. HIV has been well controlled with an antiretroviral regimen, which she tolerates well. Medications are tenofovir disoproxil fumarate (TDF), emtricitabine, and efavirenz. On physical examination, vital signs and other findings are normal. Laboratory studies show HIV viral load is undetectable and the CD4 cell count is normal. Which of the following is the most appropriate management? Continue TDF, emtricitabine, and efavirenz Pause antiretroviral therapy until after the first trimester Perform resistance testing Switch to zidovudine, lamivudine, and ritonavir-boosted lopinavir
Antiretroviral therapy during pregnancy is crucial and significantly decreases the risk of perinatal transmission of HIV to the baby. This pregnant woman with well-controlled HIV infection should continue the same antiretroviral regimen. Antiretroviral therapy during pregnancy is crucial and significantly decreases the risk of perinatal transmission of HIV to the baby. Although most transmission in untreated women occurs at the time of delivery, in utero transmission also occurs, and maintaining therapy throughout pregnancy or starting therapy immediately is important to significantly reduce the risk. Although concerns have been raised regarding the safety of efavirenz and tenofovir disoproxil fumarate in pregnancy, more recent data demonstrate the safety of these agents, including in the first trimester. A woman whose HIV is well controlled and is found to be pregnant should continue the same regimen unless another reason exists to change it. Pausing antiretroviral therapy would result in rebound of viral replication and viremia, which would significantly increase the risk of in utero transmission of HIV to the developing fetus. Testing for HIV drug resistance can be genotypic (looking for specific mutations associated with resistance to specific drugs) or phenotypic (assessing whether HIV can replicate in the presence of achievable levels of specific drugs). Resistance testing should always be done before an initial drug regimen is chosen and when treatment failure occurs, as indicated by failure to suppress viral load or an increase in viral load that was previously suppressed. Resistance testing is unnecessary at this time because no virologic failure is evident that would necessitate changing therapy. Moreover, it would not be possible to perform resistance testing in a patient with an undetectable viral load because not enough virus is present to test. A viral load level greater than 500 copies/mL is usually necessary to successfully perform resistance testing. Zidovudine, lamivudine, and ritonavir-boosted lopinavir is a valid alternative for treating HIV; it was previously a preferred regimen in pregnancy. However, changing this patient's therapy, which is well tolerated and controls her HIV infection well, is unnecessary and would only risk new adverse effects, poor adherence (because of more pills or more frequent dosing), or treatment failure.
A 27-year-old woman is seen in a follow-up visit for hyperthyroidism. Three months ago she was diagnosed with Graves disease, and methimazole was initiated. Today the patient reports an overall improvement in hyperthyroid symptoms, but over the past week she has developed a fever, sore throat, and painful swallowing. She reports no cough. Medications are methimazole and propranolol. On physical examination, temperature is 38.7 °C (101.6 °F), blood pressure is 112/78 mm Hg, pulse rate is 98/min, and respiration rate is 18/min. The patient's posterior oropharynx is erythematous without exudates. Her examination is otherwise normal. Which of the following is the most appropriate management? Begin empiric oral penicillin V Obtain a rapid antigen detection test for Group A streptococcus Obtain a throat culture Stop methimazole and obtain complete blood count with differential
Antithyroid drug-related agranulocytosis affects between one in 300 and one in 500 patients taking therapy and may present with fever and sore throat; initial management includes stopping the drug and assessment of the neutrophil count. The most appropriate management is to stop methimazole and order a complete blood count with differential. The patient has been receiving methimazole for treatment of Graves hyperthyroidism over the past 3 months. She has both a sore throat and a fever, which can be seen in patients with methimazole-induced agranulocytosis. Agranulocytosis affects between one in 300 and one in 500 patients taking antithyroid drug therapy. Agranulocytosis from methimazole usually occurs within the first several months of initiating therapy but generally is not seen with doses below 20 mg per day. Suspected agranulocytosis should be managed with cessation of the offending drug and assessment of the patient's neutrophil count. An absolute neutrophil count below 500/µL (0.5 × 109/L) confirms the diagnosis. Management of agranulocytosis may include hospitalization, broad-spectrum antibiotics, and hematopoietic growth factor therapy. Patients should be counseled regarding agranulocytosis as a potential side effect at the initiation of therapy and instructed to contact the prescribing physician immediately with occurrence of any suggestive symptoms. Diagnosis and treatment of patients with Group A streptococcus (GAS) pharyngitis has traditionally been aided by the four-point Centor criteria: (1) fever, (2) absence of cough, (3) tonsillar exudates, and (4) tender anterior cervical lymphadenopathy. The Centor criteria have a low positive predictive value for GAS infection; according to the Infectious Disease Society of America, these criteria may be used to determine which patients have a low likelihood of GAS pharyngitis and require no further testing. No additional testing or treatment is needed for patients who meet fewer than three criteria. Patients who meet three or more criteria should have a confirmatory test (either a rapid antigen detection test for GAS or throat culture). Penicillin or amoxicillin is first-line treatment for GAS pharyngitis. However, the clinical situation of most immediate concern is the possibility of methimazole-induced agranulocytosis, a potentially fatal condition. The most appropriate management is to stop the drug and measure the neutrophil count.
A 52-year-old woman is evaluated in the emergency department for progressive dyspnea. Medical history is notable for aortic stenosis and long-standing hypertension. Family history is unremarkable. Medications are metoprolol and chlorthalidone. On physical examination, temperature is normal, blood pressure is 190/90 mm Hg in both upper extremities, pulse rate is 80/min and regular, and respiration rate is 22/min. The jugular venous pressure is normal. The apical impulse is displaced and sustained. The S1 is normal, and the S2 is soft; an S4 is noted at the apex. An ejection click is heard at the apex and left sternal border. A systolic ejection murmur is noted along the right sternal border, and a separate systolic murmur is noted under the left clavicle and over the left posterior chest. The femoral pulses are diminished, and the radial artery-to-femoral artery pulse is delayed. No bruits are appreciated in the epigastrium or over the femoral vessels. A chest radiograph is shown. In addition to aortic stenosis, which of the following is the most likely diagnosis? Ascending aortic dissection Coarctation of the aorta Essential hypertension Renovascular hypertension
Aortic coarctation is characterized by clinical features of upper extremity hypertension and a radial artery-to-femoral artery pulse delay as well as radiographic findings of "figure 3 sign" and rib notching. The most likely diagnosis is aortic coarctation. This patient has dyspnea in the setting of aortic stenosis and hypertension, and she has clinical features of upper extremity hypertension and a radial artery-to-femoral artery pulse delay that suggest a mechanical obstruction between the radial and femoral arteries. A systolic murmur heard over the left chest is common in patients with severe aortic coarctation; these murmurs can arise from the obstruction or the collateral blood flow. The patient's chest radiograph demonstrates rib notching (arrows) affecting several of the posterior ribs; rib notching results from exaggerated collateral blood flow diverting blood around the area of obstruction. The patient additionally has aortic stenosis associated with a bicuspid aortic valve, which is present in more than 50% of patients with aortic coarctation. Cerebral artery aneurysm also occurs with increased frequency in patients with aortic coarctation. The syndrome of aortic dissection usually presents with sharp or tearing chest and/or back pain. It may be accompanied by syncope, heart failure, or stroke and is rarely asymptomatic. Chest pain with concomitant acute ischemia in a distant arterial bed (stroke, limb ischemia) should prompt consideration of dissection. The chest radiograph does not demonstrate features of aortic dilatation or mediastinal widening characteristic of dissection. Although the physical examination findings in a patient with severe essential hypertension often include an S4, blood pressure and pulse differentials between the upper and lower extremities are not expected. Renovascular hypertension is a common cause of hypertension, occurring primarily in patients with diffuse atherosclerosis. The physical examination in a patient with renovascular hypertension is usually normal, with the exception of hypertension and possible epigastric bruit; blood pressure and pulse differentials between upper and lower extremities are not expected.
When to do aortic repair for AAA?
Aortic repair should be performed in suitable patients with an AAA diameter of 5.5 cm or larger, in patients with rapid expansion in AAA size (>0.5 cm/year), and in patients presenting with symptoms resulting from AAA (abdominal or back pain/tenderness).
A 44-year-old man is evaluated after his routine yearly follow-up for ulcerative colitis showed elevated liver chemistry test results. He was diagnosed with ulcerative colitis 5 years ago. He reports that he passes solid, formed stool every 1 to 2 days without abdominal pain. His only medication is mesalamine. On physical examination, vital signs are normal; BMI is 26. Hepatomegaly and palmar erythema are noted. Laboratory studies: Alkaline phosphatase 412 U/L Alanine aminotransferase 78 U/L Aspartate aminotransferase 62 U/L Total bilirubin 2 mg/dL (34.2 µmol/L) Direct bilirubin 1.5 mg/dL (25.7 µmol/L) A complete blood count and serum creatinine level are normal, and an antimitochondrial antibody titer is negative. MR cholangiopancreatography shows hepatomegaly but is otherwise normal. Liver biopsy is pending. Which of the following is the most likely diagnosis? Antimitochondrial antibody-negative primary biliary cholangitis Drug-induced liver injury Nonalcoholic fatty liver disease Primary sclerosing cholangitis
Approximately 5% of patients with inflammatory bowel disease will develop primary sclerosing cholangitis during the course of their disease, typically presenting as cholestatic liver injury with a characteristic imaging study showing bile duct strictures and dilations ("string of beads"). Primary sclerosing cholangitis (PSC) is the most likely diagnosis in this patient. The patient has elevated liver chemistry test results in a predominantly cholestatic pattern and concomitant inflammatory bowel disease (IBD). Disorders are considered cholestatic when the most prominent abnormality of liver chemistry testing is an elevation of the serum alkaline phosphatase level. Approximately 5% of patients with IBD will develop PSC during the course of their disease. In most patients, PSC presents as a stricturing process in the medium to large bile ducts, readily identifiable by MR cholangiopancreatography (alternating strictures and dilations resulting in a "string of beads" pattern). However, a minority of patients may present with involvement of only the small bile ducts, called small-duct PSC. The cholangiogram is normal in these patients; the diagnosis of small-duct PSC can only be made by liver biopsy. Small-duct PSC is associated with a better prognosis than typical PSC, although 10% to 20% of patients with small-duct PSC eventually develop changes in the medium and large ducts. The diagnosis of primary biliary cholangitis (PBC) is generally made on the basis of a cholestatic liver enzyme profile in the setting of a positive antimitochondrial antibody test. Case reports have described PBC associated with IBD, but the association between the two conditions is not well accepted, making the diagnosis of PBC unlikely. Although 10% of patients with PBC may test negative for antimitochondrial antibody, the absence of antibody makes this diagnosis less likely. A liver biopsy is required to establish the diagnosis of antimitochondrial antibody-negative PBC. In suspected antimitochondrial antibody-negative PBC, other PBC-specific autoantibodies (sp100 or gp210) can establish the diagnosis in a patient with a compatible cholestatic liver enzyme profile. Drug-induced liver injury can occur in patients taking medications used to treat IBD, such as azathioprine and tumor necrosis factor-α inhibitors. Drug-induced liver injury is not a common adverse effect of mesalamine; therefore, this diagnosis is unlikely in this patient. Nonalcoholic fatty liver disease is the most common liver disease in the United States. It is unlikely in this patient because it usually does not present with alkaline phosphatase levels greater than 2 to 2.5 times the upper limit of normal. Furthermore, MRI has a reasonable sensitivity (>80%) for the detection of fatty liver, which was not seen in this patient.
A 66-year-old woman is evaluated after developing five dental caries over the past year. She cannot eat crackers without accompanying water. She has lost two teeth due to caries. She also reports scratchy and itchy eyes for 2 years and intermittent joint pain, particularly of the small hand joints, for 1 year. She takes ibuprofen as needed for the joint pain, which provides relief. On physical examination, vital signs are normal. Two molars are missing, and there is no salivary pooling below the tongue. Bilateral parotid and lacrimal enlargement is present. Mild tenderness without swelling of the second through fourth metacarpophalangeal joints bilaterally is noted. Laboratory studies show positive rheumatoid factor, high-titer antinuclear antibodies, and high-titer anti-Ro/SSA antibodies. A Schirmer test for ocular wetting is diminished at 3 mm. Chest radiograph is normal. Radiographs of the hands show no erosions. Which of the following is the most appropriate treatment at this time? Artificial tears and sugar-free candies Methotrexate Pilocarpine Rituximab Topical ophthalmic NSAID drops
Artificial tears and sugar-free candies are the most appropriate treatment at this time. This patient meets diagnostic criteria for Sjögren syndrome, with objective documentation of sicca in the presence of anti-Ro/SSA antibodies, antinuclear antibodies, and rheumatoid factor. Her dental carries and missing teeth are almost certainly due to her dry mouth, as evidenced by lack of salivary pooling below the tongue. She has arthralgia but no definite arthritis and, consistent with Sjögren joint involvement, no evidence of erosive joint disease on radiograph. The immediate treatment goal of sicca is the re-establishment of wetting of the eyes and mouth. For the eyes, topical application of artificial tears is usually the first strategy, potentially followed by addition of topical cyclosporine or lifitegrast if artificial tears prove insufficient. For oral dryness, sugar-free candy or chewing gum can help to increase saliva flow. Regular intake or rinsing with small sips of water is also recommended. Artificial saliva substitutes would be considered next if these approaches are insufficient. Methotrexate is a nonbiologic disease-modifying antirheumatic drug occasionally used in Sjögren syndrome for arthritis. Because the patient's joint disease is unlikely to result in future bony damage and is responsive to ibuprofen, no additional arthritis management is needed at this time. There is no documented indication for methotrexate to treat sicca symptoms associated with Sjögren syndrome. Pilocarpine is a muscarinic agonist that may stimulate saliva secretion. It is often poorly tolerated and would not be considered unless basic measures to treat oral dryness are insufficient. Rituximab depletes B cells and has shown early promise for the management of Sjögren syndrome. Early studies suggest a possible benefit for treating sicca, but it is generally reserved for patients with systemic Sjögren complications or severe disease. At the present time, its use is not warranted in this patient. Topical ophthalmic NSAID drops are absolutely contraindicated in Sjögren syndrome because they carry risk for severe corneal damage.
A 73-year-old man is evaluated for a 6-month history of right-sided chest discomfort, fatigue, nonproductive cough, and progressive dyspnea. He has lost 9 kg (20 lb) during the last 6 months. His history is significant for COPD. He was a brake mechanic for 30 years. Medications are an albuterol inhaler as needed and tiotropium inhaler. On physical examination, vital signs are normal. Lung examination reveals dullness to percussion and diminished breath sounds above the lower half of the right hemithorax. The physical examination is otherwise normal. Laboratory studies, including complete blood count, are normal. Chest radiograph shows a moderate right-sided loculated pleural effusion with pleural thickening. A thoracentesis is performed that removes 600 mL of serosanguineous fluid. Pleural fluid analysis: Cytology Atypical mesothelial cells, negative for malignancy Gram stain Negative Lactate dehydrogenase 425 U/L pH 7.35 Total protein 4.6 g/dL (46 g/L) Which of the following is the most likely diagnosis? Empyema Heart failure Malignant pleural mesothelioma Rheumatoid pleuritis
Asbestos exposure is the primary risk factor for malignant pleural mesothelioma, and patients most commonly present with symptoms of chest pain and a slowly enlarging pleural effusion.
A 73-year-old man is evaluated for a 6-month history of right-sided chest discomfort, fatigue, nonproductive cough, and progressive dyspnea. He has lost 9 kg (20 lb) during the last 6 months. His history is significant for COPD. He was a brake mechanic for 30 years. Medications are an albuterol inhaler as needed and tiotropium inhaler. On physical examination, vital signs are normal. Lung examination reveals dullness to percussion and diminished breath sounds above the lower half of the right hemithorax. The physical examination is otherwise normal. Laboratory studies, including complete blood count, are normal. Chest radiograph shows a moderate right-sided loculated pleural effusion with pleural thickening. A thoracentesis is performed that removes 600 mL of serosanguineous fluid. Pleural fluid analysis: Cytology Atypical mesothelial cells, negative for malignancy Gram stain Negative Lactate dehydrogenase 425 U/L pH 7.35 Total protein 4.6 g/dL (46 g/L) Which of the following is the most likely diagnosis? Empyema Heart failure Malignant pleural mesothelioma Rheumatoid pleuritis
Asbestos exposure is the primary risk factor for malignant pleural mesothelioma, and patients most commonly present with symptoms of chest pain and a slowly enlarging pleural effusion. The most likely diagnosis is malignant pleural mesothelioma. Asbestos exposure is the primary risk factor for mesothelioma, which has a latency period of 20 to 40 years, and this patient likely has a history of asbestos exposure from working as a brake mechanic. Occupational exposure to asbestos is most common in miners, electricians, plumbers, brake mechanics, shipyard workers, home remodelers, and selected military personnel. Patients most commonly present with symptoms of chest pain and a slowly enlarging pleural effusion. Chest imaging typically shows a unilateral pleural effusion, but patients can also present with pleural thickening, calcification, nodules, or masses. If malignant pleural mesothelioma is suspected, a thoracentesis should be performed, including pleural fluid cytology. Additional evaluation includes a chest CT scan to determine the extent of disease and evaluate for pleural lesions. Confirmation of the diagnosis requires pleural biopsy. Video-assisted thoracoscopic biopsy or open thoracotomy is required when the diagnosis remains uncertain, as diagnosis of mesothelioma cannot be made using cytology alone. Empyema often presents with unilateral loculated exudative pleural effusion; however, the patient would not have a 6-month history of symptoms in the absence of fever and with a normal complete blood count. Finally, the pleural fluid characteristics are not consistent with empyema (typically pleural fluid pH less than 7.2, purulent effusion, or positive Gram stain). Although cough, dyspnea, and pleural effusion could occur in patients with heart failure, the lack of physical examination findings consistent with volume overload and findings of a serosanguineous exudative effusion on thoracentesis would not be consistent with a diagnosis of heart failure (transudative effusion). Rheumatoid pleuritis would also present with an exudative pleural effusion, but the patient has no other signs or symptoms of rheumatoid arthritis.
An 81-year-old man is evaluated before elective hip arthroplasty. Medical history is significant for hypertension and osteoarthritis. He reports no chest pain, palpitations, exertional dyspnea, or other symptoms of cardiovascular disease. His medications are lisinopril and celecoxib. On physical examination, vital signs are normal. The cardiopulmonary examination is normal. Range of motion of the right hip is limited by pain without overlying erythema or warmth. Laboratory studies reveal normal kidney function and electrolyte levels. A 12-lead electrocardiogram is shown (bifascicular block). Findings are unchanged from 7 years ago. Which of the following is the most appropriate management? Dobutamine echocardiography Echocardiography Prophylactic pacemaker insertion No further testing or intervention
Asymptomatic first-degree atrioventricular block with bifascicular block does not require pacemaker implantation. No further testing or intervention is required at this time. This asymptomatic patient has first-degree atrioventricular (AV) block (PR interval >200 ms), right bundle branch block, and left posterior fascicular block. Right bundle branch block is diagnosed by the findings of a widened QRS complex (>120 ms); an RSR′ pattern in lead V1; and a wide negative S wave in leads I, V5, and V6. Blocks may also occur in the anterior or posterior divisions (fascicles) of the left bundle; these are termed fascicular blocks (or hemiblocks). Left anterior fascicular block is recognized by a positive QRS complex in lead I and a negative QRS complex in lead aVF. Left posterior fascicular block is recognized by a negative QRS complex in lead I and a positive QRS complex in lead aVF. Conduction disturbances involving the right bundle branch and one of the two fascicles (anterior or posterior) of the left bundle branch are commonly referred to as bifascicular block. The presence of first-degree AV block with bifascicular block is often called "trifascicular block"; however, the term is misleading because true trifascicular block would indicate complete AV block. There is no need for extensive cardiac evaluation of patients with asymptomatic bifascicular block other than a careful history and physical examination to exclude the diagnosis of occult cardiac disease. Therefore, echocardiography and dobutamine echocardiography are not indicated because occult ischemic or structural heart disease is not suspected in this patient. Pacemakers are indicated in patients with symptomatic bradycardia in the absence of a reversible cause, hence the importance of establishing symptoms when evaluating patients with bradycardia. Pacing is not indicated in asymptomatic patients with first-degree AV block accompanied by bifascicular block because the risk for progression to complete heart block is less than 2% to 3% per year. As such, pacemaker insertion is not needed in this patient.
What does afib with aberrancy look like? How to treat?
Atrial fibrillation with aberrant conduction results in an irregularly irregular rhythm and a wide-complex tachycardia with a QRS morphology (rSR pattern in lead V1, deep terminal S waves in leads I and V6) on electrocardiogram typical of right bundle branch block. *BASICALLY IRREGULARLY IRREGULAR WIDE COMPLEX TACHYCARDIA -Afib with some occasional bundle branch block (that's why it's wide?) -Aberrant conduction = conduction through AV node with delay or block Tx: B-blocker + anticoagulation
An 84-year-old woman in hospice care is evaluated for "death rattle" that is disturbing to family members. She is in the active phases of dying, and her family is distressed by her noisy respiratory secretions; they are worried that she is choking. Medications are haloperidol, hydromorphone, lactulose, and acetaminophen. On physical examination, respiration rate is 12/min. She is not responsive but does not appear uncomfortable. Extremities are cool. There are oropharyngeal secretions that produce a rattling and gurgling sound with inspiration. Which of the following is the most appropriate initial management? Atropine ophthalmic drops given sublingually Glycopyrronium Scopolamine patch Suctioning by catheter Symptom explanation and reassurance
Audible posterior oropharyngeal secretions ("death rattle") are common at the end of life and are best managed with family education and reassurance. This patient has audible posterior oropharyngeal secretions, which are most appropriately managed with family education and reassurance. Although several studies suggest that respiratory distress is not typically associated with these secretions, caregivers are often concerned by what is commonly referred to as the "death rattle." The first steps in management include caregiver education and anticipatory guidance. Additionally, repositioning often allows secretions to drain without pharmacologic intervention. Mouth hygiene with a sponge swab may also be helpful. Current literature does not support the routine use of antimuscarinic drugs in the treatment of death rattle. A 2014 literature review acknowledged that death rattle leads to distress in both relatives and professional caregivers; however, its impact on patients is unclear, and medical therapy is unproven. Studies involving atropine, glycopyrronium, scopolamine, hyoscine butylbromide, and/or octreotide were reviewed, and only one study used a placebo group. There is currently no evidence that the use of any antimuscarinic drug is superior to no treatment. In addition, the use of anticholinergic agents in patients who are awake can lead to undesirable symptoms, such as dry mouth and urinary retention. Suctioning by catheter should be avoided in managing end-of-life secretions unless the secretions are causing the patient obvious respiratory distress or cough. Suction catheters can cause local trauma.
A 51-year-old man is evaluated in the emergency department for a 12-hour history of fever, chills, headache, and weakness. He works on his farm in Maine and spends a considerable part of most days outside, but he is not aware of any tick or mosquito bites. He has had no diarrhea, cough, or rash. The patient had his spleen surgically removed 20 years ago after being involved in a motor vehicle collision. He has had no recent travel outside of the United States, and he is up to date on all immunizations. On physical examination, temperature is 39.6 °C (103.3 °F), blood pressure is 88/42 mm Hg, pulse rate is 135/min, respiration rate is 28/min, and oxygen saturation is 90% with the patient receiving 4 L/min of oxygen via a nasal cannula. Lethargy, scleral icterus, jaundiced skin, hepatomegaly, and lower extremity petechiae are noted. Laboratory studies: Haptoglobin <8 mg/dL (80 mg/L) Hematocrit 25% Leukocyte count 4500/µL (4.5 × 109/L) Platelet count 109,000/µL (109 × 109/L) Bilirubin, total 3.9 mg/dL (66.7 µmol/L) Creatinine 1.0 mg/dL (88.4 µmol/L) Lactate dehydrogenase 909 U/L A blood smear from this patient is most likely to show which of the following abnormalities? Cytoplasmic morulae in leukocytes Intraerythrocytic banana-shaped gametocytes Intraerythrocytic tetrad forms Intraneutrophilic gram-positive diplococci Schistocytes
Babesiosis can be diagnosed by visualization of intraerythrocytic parasites in a ring or tetrad form on a blood smear. This patient's blood smear is most likely to show intraerythrocytic tetrad forms. He has a life-threatening illness characterized by fever and hemolysis and has no spleen. Given his residence in the northeastern United States and his outdoor vocation, severe babesiosis is likely. Babesia species are transmitted by the same tick that causes Lyme disease; infection also can be acquired through transfusion of infected erythrocytes outside of the endemic region. Asplenia is a major risk factor for fulminant Babesia infection, as is older age, HIV infection, or other immunocompromising conditions. Clinical manifestations are variable, with most relating to severe hemolytic anemia. Infection can be diagnosed by visualization of intraerythrocytic parasites, which appear as either ring forms or tetrads, with the latter often described as having the Maltese cross appearance shown. Complications, including acute respiratory distress syndrome, disseminated intravascular coagulation, heart failure, kidney failure, and coma, are associated with severe anemia (hematocrit level <30%) and parasitemia (exceeding 10%) and are indications for exchange transfusion. Morulae, basophilic inclusion bodies composed of clusters of bacteria, can be seen in the cytoplasm of monocytes and neutrophils of patients with ehrlichiosis and anaplasmacytosis, respectively. These tick-borne infections cause fever, leukopenia, and thrombocytopenia but are not associated with hemolytic anemia. Both malaria and babesiosis cause a hemolytic anemia and may present with ring-shaped parasitic inclusions inside erythrocytes. However, banana-shaped gametocytes are seen only with Plasmodium falciparum malaria, and the absence of a compatible travel history excludes this diagnosis. Asplenic patients are at risk for overwhelming pneumococcal sepsis with a high bacterial burden; gram-positive diplococci may be visualized inside neutrophils on buffy-coat stain. Although leukopenia and thrombocytopenia are common with fulminant infection, hemolytic anemia is not a feature of pneumococcal sepsis. Schistocytes are a manifestation of microangiopathic hemolytic anemia caused by a thrombotic microangiopathy (TMA), such as hemolytic uremic syndrome or thrombotic thrombocytopenic purpura. Although both these syndromes can cause fever, hemolysis, and thrombocytopenia, the absence of acute kidney injury in this patient excludes a TMA.
A 52-year-old woman is evaluated for recent-onset exertional dyspnea. Medical history is noncontributory, and she takes no medications. On physical examination, vital signs are normal. The central venous pressure is elevated with a prominent a wave. Apical impulse is normal, and a prominent parasternal impulse is noted at the left sternal border. The S1 is normal; the S2 is soft. A grade 4/6 late-peaking systolic murmur is heard at the left sternal border and second left intercostal space. An ejection click is not audible. Echocardiogram demonstrates doming of the pulmonary valve with stenosis, a peak instantaneous systolic gradient of 65 mm Hg, and a mean systolic gradient of 50 mm Hg. Trivial pulmonary regurgitation is present. The right ventricular size and function are normal, but right ventricular hypertrophy is present. The left heart size and function are normal. Which of the following is the most appropriate management of this patient? Endocarditis prophylaxis Exercise stress testing Pulmonary balloon valvuloplasty Pulmonary valve replacement
Balloon valvuloplasty is recommended for patients with symptomatic pulmonary valve stenosis who have appropriate valve morphology, a peak Doppler gradient of greater than 50 mm Hg or a mean gradient greater than 30 mm Hg, and valve characteristics favorable for percutaneous intervention. Balloon valvuloplasty is the most appropriate management of this patient. The palpable, late-peaking systolic murmur located at the second left intercostal space; absence of an ejection click; and features of right ventricular pressure overload suggest severe pulmonary valve stenosis. The echocardiogram confirms the presence of severe pulmonary valve stenosis with a peak systolic gradient of 65 mm Hg and mean systolic gradient of 50 mm Hg. Balloon valvuloplasty is recommended for symptomatic patients with appropriate valve morphology who have a peak Doppler gradient of greater than 50 mm Hg or a mean gradient greater than 30 mm Hg and favorable valve characteristics for percutaneous intervention. Patients with pulmonary valve stenosis without a history of endocarditis or pulmonary valve replacement do not require endocarditis prophylaxis. Exercise stress testing could help determine whether the patient has exercise limitations related to her pulmonary valve stenosis; however, the results would not change the recommendation for intervention. Surgical pulmonary valve replacement should be considered when the valve is dysplastic or when more than moderate coexisting pulmonary valve regurgitation, a small annulus, severe subvalvar or supravalvar pulmonary valve stenosis, or another cardiac lesion that requires operative intervention is present. It is not the preferred treatment for this patient who has appropriate valve morphology for balloon valvuloplasty. Read Related TextNext Question
A 39-year-old woman is evaluated in the emergency department for a 2-day history of bilateral lower extremity weakness and numbness. The patient has no other symptoms and no relevant medical history. She takes no medication. On physical examination, temperature is 36.7 °C (98.1 °F); all other vital signs are normal. Muscle strength testing shows 4/5 weakness in hip flexion bilaterally and in foot dorsiflexion. Temperature sensation is partially reduced below spinal cord level T5. All other physical examination findings are normal. A T2-weighted MRI of the thoracic spine shows a hyperintense lesion at the T4 level. Which of the following is the most appropriate next test to evaluate for multiple sclerosis? Cerebrospinal fluid (CSF) analysis for antibodies to myelin basic protein CSF analysis for oligoclonal bands Erythrocyte sedimentation rate MRI of the brain Visual evoked potentials
Because multiple sclerosis (MS) is the most common cause of transverse myelitis, MRI of the brain should be performed in patients with new-onset transverse myelitis to evaluate for MS lesions. This patient should undergo MRI of the brain. She has signs and symptoms consistent with an episode of inflammatory transverse myelitis. Although idiopathic transverse myelitis can occur, the most common cause of a transverse myelitis event is multiple sclerosis (MS). MRI of the brain should be routinely performed in patients with new-onset transverse myelitis to evaluate for lesions consistent with MS, such as those located in periventricular, juxtacortical (or cortical), and infratentorial white matter. If these lesions are present, MS should be diagnosed if they fulfill the diagnostic criteria for dissemination in space and in time. No MS-specific diagnostic biomarkers are available. Initial studies championed myelin basic protein as a potential marker of MS disease activity, but subsequent evidence found that this biomarker is not associated with MS disease activity and is not useful to confirm a diagnosis of MS. Lumbar puncture to determine the presence of oligoclonal bands is not warranted. Although lumbar puncture sometimes is appropriate in the evaluation of MS, the presence or absence of oligoclonal bands is not part of multiple sclerosis diagnostic criteria because of poor sensitivity and specificity for the disease. Approximately 10% to 15% of patients with MS have normal cerebrospinal fluid on testing; additionally, oligoclonal bands are sometimes elevated in other inflammatory conditions. For these reasons, if MRI confirms the diagnosis of MS, CSF testing is unnecessary. The erythrocyte sedimentation rate may be elevated in patients with inflammatory transverse myelitis. However, the test is nonspecific and would not lead to a diagnosis of the cause of this patient's myelitis, such as MS. Detection of a subclinical lesion in a site remote from the region of clinical dysfunction supports a diagnosis of multifocal MS. Evoked potentials also may help define the anatomic site of the lesion in tracts not easily visualized by imaging (for example, optic nerves, dorsal columns). Evoked potential testing is reserved for situations in which the diagnosis of MS is equivocal. The best initial test remains a brain MRI.
A 22-year-old woman arrives for her annual gynecologic examination, including a Pap smear and refill of her oral contraceptive prescription. She indicates she recently became sexually active with a new partner. Medical history is otherwise unremarkable. Her only medication is her oral contraceptive. On physical examination, vital signs are normal. Pelvic examination is normal. The remainder of the examination is noncontributory. A urine pregnancy test is negative. The Pap smear is normal. Nucleic acid amplification testing is positive for Neisseria gonorrhoeae and negative for Chlamydia trachomatis. The HIV antigen/antibody combination immunoassay is negative. Which of the following is the most appropriate treatment? Cefixime Ceftriaxone Ceftriaxone plus azithromycin Ciprofloxacin
Because of increasing minimum inhibitory concentrations to ceftriaxone among Neisseria gonorrhoeae isolates in the United States, this agent should be combined with a single dose of azithromycin. The most appropriate treatment is ceftriaxone plus azithromycin. This sexually active young woman presented for an annual Pap smear and renewal of oral contraceptives. She was appropriately screened for Chlamydia trachomatis and Neisseria gonorrhoeae infection using nucleic acid amplification testing. The U.S. Preventive Services Task Force recommends this screening for all sexually active women younger than 25 years. If the patient were older than 25 years, she should be tested because of her new sexual partner. Her screening results are positive for N. gonorrhoeae. Even without symptoms or findings on pelvic examination, she should be treated; a significant number of persons with N. gonorrhoeae infection are asymptomatic without evidence of cervicitis. Intramuscular ceftriaxone (250 mg) is the preferred therapy for treatment of N. gonorrhoeae infection, along with a single dose of oral azithromycin (1 g). Although this regimen is given empirically when C. trachomatis and N. gonorrhoeae coinfection is suspected (for example, empiric therapy for cervicitis), because of increasing minimum inhibitory concentrations (MIC) to ceftriaxone among N. gonorrhoeae isolates in the United States, azithromycin is used even when C. trachomatis infection is not present; this combination will increase the success of treatment of the N. gonorrhoeae infection. Oral cefixime does not achieve adequate levels to exceed the MIC of many N. gonorrhoeae isolates, so this option should only be used if ceftriaxone is unavailable. In this circumstance, a single dose of oral azithromycin should be administered with the cefixime. Ciprofloxacin was previously recommended for the treatment of N. gonorrhoeae infections; however, the prevalence of fluoroquinolone resistance among N. gonorrhoeae isolates has increased, and this antibiotic is no longer recommended.
A 53-year-old woman is brought to the office by her husband for follow-up evaluation of behavioral-variant frontotemporal dementia, which was diagnosed 9 months ago. Her clinical symptoms of occasional aphasia, minor memory impairment, behavioral disinhibition, and obsessive-compulsive behaviors have worsened since diagnosis and have become disruptive. She is currently estranged from her son and his family after bringing a large bag filled with plastic bottles collected from his neighbors' trash cans to her granddaughter's birthday party 2 weeks ago. Her husband asks about possible medications to control her symptoms. Which of the following is the most appropriate medication to recommend? Citalopram Donepezil Memantine Methylphenidate Olanzapine
Because there are currently no disease-modifying treatments for behavioral-variant frontotemporal dementia, treatment is symptom based and should target the disease's most troublesome manifestations. This patient should be given the selective serotonin reuptake inhibitor citalopram to control her obsessive-compulsive behaviors. Early changes in social behavior and personality are the defining characteristics of behavioral-variant frontotemporal dementia (FTD). Apathy, diminished interest, loss of empathy, lack of initiative, increased emotionality, disinhibition, euphoria, impulsivity, changes in eating behaviors, hyperorality, and compulsiveness are the most common symptoms reported by families. Other changes include irritability, aggression, verbal abuse, hypomania, and restlessness. The treatment of behavioral-variant FTD is symptom based and should target the most troubling manifestations of the disorder. This patient's obsessive-compulsive tendencies not only have had embarrassing consequences but have resulted in a confrontation with family members. Selective serotonin reuptake inhibitors, such as citalopram, have the potential to alleviate these symptoms. Tricyclic antidepressants may also have this effect. The acetylcholinesterase inhibitor donepezil is approved by the FDA to treat mild to moderate Alzheimer disease. This class of drug has shown modest benefit in improving cognitive performance in patients with this type of dementia without clear improvements in daily functioning. The body of evidence does not support acetylcholinesterase inhibitors as being beneficial in behavioral-variant FTD. Similarly, the N-methyl-D-aspartate receptor antagonist memantine, which is approved for moderate to severe dementia in patients with Alzheimer disease associated with significant functional impairment, has shown no benefit in in patients with behavioral-variant FTD and may, in fact, worsen symptoms. In patients with severe apathy, which can be a common and sometimes debilitating symptom of behavioral-variant FTD, stimulant medications, such as methylphenidate, are sometimes used. This patient has not exhibited apathy. More importantly, referral to a psychiatrist or neurologist is generally recommended before dispensing these medications. Atypical antipsychotic agents (such as olanzapine) can be effective in treating agitation, aggression, delusions, and hallucinations. However, these drugs are not FDA approved for this clinical indication and have an associated black-box warning due to increased cerebrovascular events and mortality rates in patients with dementia. Olanzapine can be considered if there are psychotic symptoms, such as hallucinations or psychotic delusions. This patient has exhibited no psychotic symptoms. In general, antipsychotic agents should be avoided in patients with dementia. Read Related TextNext Question
A 31-year-old man is evaluated in the emergency department for fever and red, painful eyes. He reports a 1-month history of intermittent painful oral and genital ulcers, knee pain, and fever, and 1 week of bilateral eye redness, pain, and photophobia. On physical examination, temperature is 38.3 °C (101.0 °F), and pulse rate is 100/min. Slit lamp examination reveals white cells in the anterior chambers of both eyes. There are aphthous ulcers on the tongue and one aphthous ulcer on the scrotum. Swelling and warmth are noted in both knees. The heart, lung, abdominal, and neurologic examinations are normal. Which of the following is the most likely diagnosis? Behçet syndrome Crohn disease Sarcoidosis Systemic lupus erythematosus
Behçet syndrome is characterized by recurrent painful oral and genital mucosal ulcerations, inflammatory eye disease, and pathergy.
A 31-year-old man is evaluated in the emergency department for fever and red, painful eyes. He reports a 1-month history of intermittent painful oral and genital ulcers, knee pain, and fever, and 1 week of bilateral eye redness, pain, and photophobia. On physical examination, temperature is 38.3 °C (101.0 °F), and pulse rate is 100/min. Slit lamp examination reveals white cells in the anterior chambers of both eyes. There are aphthous ulcers on the tongue and one aphthous ulcer on the scrotum. Swelling and warmth are noted in both knees. The heart, lung, abdominal, and neurologic examinations are normal. Which of the following is the most likely diagnosis? Behçet syndrome Crohn disease Sarcoidosis Systemic lupus erythematosus
Behçet syndrome is characterized by recurrent painful oral and genital mucosal ulcerations, inflammatory eye disease, and pathergy. The most likely diagnosis is Behçet syndrome, a systemic disease associated with inflammatory infiltration of multiple organs. The disease is characterized by recurrent painful oral and genital mucosal ulcerations, inflammatory eye disease (panuveitis, retinal vasculitis), and pathergy (an inflammatory response to skin prick with a sterile needle). Other clinical manifestations include venous thrombosis; central nervous system (CNS) manifestations such as brainstem lesions and aseptic meningitis, with the most common CNS symptoms being headache and diplopia; inflammatory arthritis (usually in the knees); skin lesions; and gastrointestinal inflammation/ulceration indistinguishable from inflammatory bowel disease. Behçet syndrome is most common in countries along the Silk Road and is more prevalent in men. The International Criteria for Behçet Disease are both sensitive and specific for the diagnosis and include the following weighted elements: genital aphthosis (2 points); ocular lesions (uveitis or retinal vasculitis) (2 points); oral aphthosis (1 point); skin lesions (pseudofolliculitis or erythema nodosum) (1 point); vascular lesions (superficial phlebitis, deep vein thrombosis, large vein thrombosis, arterial thrombosis, or aneurysm) (1 point); and pathergy (1 point). Three points are required to establish the diagnosis; this patient has 5 points. Whereas Crohn disease can be associated with aphthous ulcers, uveitis, and peripheral arthritis, the lack of gastrointestinal symptoms make this an unlikely diagnosis. Sarcoidosis can manifest as acute arthritis, bilateral hilar lymphadenopathy, and erythema nodosum, and a more chronic form of arthritis affects 1% to 4% of patients that may involve shoulders, hands, wrists, knees, and ankles. Uveitis is a common manifestation of sarcoidosis. Papular, nodular, and plaque-like skin lesions are common in sarcoidosis, but mucosal or genital ulcerations are not found. Joint involvement occurs in 90% of patients with systemic lupus erythematosus (SLE), most commonly polyarthralgia, with frank arthritis involving both small and large peripheral joints found in 40% of patients. Bilateral knee inflammatory arthritis would be an unusual initial presentation for SLE. Oral ulcers can occur but are classically painless, and genital ulcers typically do not occur. Uveitis is one of the many ocular compilations of SLE but is relatively rare.
A 20-year-old woman comes to the office for follow-up evaluation 2 weeks after being involved in a head-to-head collision in a soccer game. She lost consciousness for several seconds on the field and was removed from play. On the sidelines, she reported a severe occipital headache, dizziness, "brain fog," and nausea. Subsequent neurologic examination and head CT findings were unremarkable. She was started on a 1-week course of ibuprofen and prochlorperazine, which she took regularly. Besides mild cognitive slowing and short-term memory issues, she has been completely asymptomatic without medication for 1 week. On physical examination, blood pressure is 96/66 mm Hg and pulse rate is 62/min. All other physical examination findings, including those from a neurologic examination, are normal. Which of the following is the most appropriate next step in management? Amantadine Brain MRI Neuropsychological testing Return to play
Besides initial screening with a symptom checklist and a neurologic examination, including assessments of cognition and balance, neuropsychological testing should be part of a comprehensive mild traumatic brain injury management strategy for patients with persistent symptoms. The patient should undergo neuropsychological testing. She sustained a mild closed head injury with typical symptoms of a concussion. Loss of consciousness established the presence of mild traumatic brain injury (TBI). Headache, nausea, dizziness, and cognitive dysfunction are all typical symptoms of postconcussion syndrome. Guidelines recommend initial screening with a symptom checklist and a neurologic examination, including assessments of cognition and balance. Guidelines also recommend head CT in those with suspected hemorrhage. Neuropsychological testing provides an additional objective and more sensitive measure of cognitive function and should be part of a comprehensive TBI management strategy for patients with persistent symptoms. Amantadine has been shown to be useful in the management of severe traumatic brain injuries. Data suggest that treatment with amantadine can accelerate functional recovery among patients in vegetative or minimally conscious states after a TBI and can effectively accelerate the pace of recovery of cognitively mediated behaviors compared with placebo. However, there is insufficient evidence to recommend amantadine in cases of mild TBI. A brain MRI is not indicated in this patient, who has shown symptomatic improvement in the 2 weeks since injury and now has normal neurologic examination findings. Further neuroimaging is unnecessary. Return to play before complete recovery may increase the risk of recurrent injury. Concussion symptoms should have resolved completely before the patient resumes exercise. A return-to-play program should then involve a stepwise increase in physical activity, with monitoring for recurrence of symptoms. Guidelines recommend that a patient be asymptomatic without medication both at rest and when subjected to physical exertion. The presence of continued cognitive symptoms in this patient makes return to play inappropriate at this time.
A 60-year-old woman is evaluated for new-onset hypertension. Medical history is significant for metastatic cervical cancer for which she began chemotherapy 2 months ago. She has no history of hypertension. Medications are bevacizumab, cisplatin, paclitaxel, dexamethasone, promethazine, and ranitidine. On physical examination, temperature is normal, blood pressure is 172/106 mm Hg, pulse rate is 82/min, and respiration rate is 18/min. The remainder of the examination is unremarkable. Which of the following medications is the most likely cause of this patient's hypertension? Bevacizumab Cisplatin Dexamethasone Paclitaxel
Bevacizumab is the most likely cause of this patient's hypertension. Bevacizumab is a monoclonal antibody that blocks angiogenesis by inhibiting vascular endothelial growth factor (VEGF) A. It is effective in combination with standard chemotherapy for the treatment of metastatic colon and gynecologic cancers. Inhibition of VEGF appears to cause hypertension through several mechanisms, including altered nitric oxide production, increased endothelin-1 production, and alterations in the pressure-natriuresis relationship. As such, bevacizumab-related hypertension is a mechanism-dependent, on-target toxicity, which results directly from inhibition of the VEGF signaling pathway rather than as a nonspecific drug side effect. Bevacizumab-related hypertension affects a high proportion of treated patients, and incidence is dose dependent. Hypertension is most commonly detected at approximately 60 days after therapy initiation and is reversible upon discontinuation of the medication. Because large increases in both diastolic and systolic blood pressures can occur as early as the first week of therapy, all patients taking an angiogenesis inhibitor (such as bevacizumab) should have their blood pressure carefully monitored during therapy, particularly early on. There is no preferred antihypertensive agent for treatment. Interestingly, the development of hypertension may be a marker of a therapeutic response or improved cancer outcome for patients treated with this class of drugs. Cisplatin has been associated with increased risk for venous thromboembolism, supraventricular tachycardia, myocardial ischemia, and cardiomyopathy. It may be associated with an increased risk for late development of hypertension in long-term cancer survivors; however, it has not been associated with acute onset of hypertension during treatment. Glucocorticoids, such as dexamethasone, may contribute to hypertension through an unclear mechanism. However, the timing and degree of hypertension observed in this patient are typical of a bevacizumab-induced reaction. The most common cardiotoxicities associated with paclitaxel are bradycardia and heart block. Most of these events are asymptomatic, and routine cardiac monitoring is not advocated. Paclitaxel can uncommonly cause hypertension (1% incidence) but is more likely to cause hypotension (up to 12% incidence). When used in combination with doxorubicin, paclitaxel is associated with cardiomyopathy, which occurs at a higher incidence than with doxorubicin alone. Read Related TextNext Question
A 45-year-old man is evaluated for hypertension. He is otherwise healthy and has not had any symptoms. His father died of an acute myocardial infarction at age 49 years. He takes no medications. On physical examination, temperature is normal, blood pressure is 172/80 mm Hg, pulse rate is 80/min, and respiration rate is normal. The lungs are clear. Jugular venous pulse is normal. S1 and S2 are normal. There is a soft ejection click that precedes a grade 2/6 diastolic decrescendo murmur, which is best heard at the right lower sternal border. A transthoracic echocardiogram shows a bicuspid aortic valve with mild aortic regurgitation. The left ventricular ejection fraction is 50%. The ascending aorta is enlarged, with a dimension of 42 mm in the mid-portion. Which of the following is the most appropriate next step in management? Annual transthoracic echocardiography CT angiography of the aorta Infective endocarditis prophylaxis Surgical aortic valve replacement
Bicuspid valve disease is commonly associated with abnormalities of the aorta, including aneurysm, dissection, and coarctation; therefore, all patients with a bicuspid aortic valve should be evaluated for possible aortopathy with CT or cardiac magnetic resonance imaging. The most appropriate next step in management is CT angiography of the aorta. Bicuspid aortic valve is a common congenital heart abnormality, occurring in 1% to 2% of the general population. It is also heritable, and screening of first-degree relatives is recommended. Aortopathy commonly accompanies bicuspid aortic valve disease and may be associated with aortic aneurysm, dissection, and coarctation. Owing to the life-threatening nature of these abnormalities, all patients with a bicuspid aortic valve should be evaluated for possible aortopathy. This patient's severe hypertension suggests the possibility of aortic coarctation. Without a diagnosis and therapeutic intervention, the mean life expectancy of unoperated patients with coarctation of the aorta is only 35 years; major complications consist of early coronary atherosclerosis, stroke, aortic dissection, systemic hypertension, and heart failure. The presence of a radial artery-to-femoral artery pulse delay may be helpful in diagnosis at the bedside, but the definitive study is an imaging evaluation (CT or cardiac magnetic resonance imaging). Patients with a bicuspid aortic valve and severe aortic stenosis or regurgitation require echocardiography every 6 to 12 months; those with mild stenosis or regurgitation should have echocardiography every 3 to 5 years. This patient does not require annual echocardiography; rather, he should be promptly evaluated for abnormalities of the aorta because of the presence of a bicuspid aortic valve. Bicuspid aortic valve carries an increased risk for infective endocarditis, and good dental care is important in these patients. However, guidelines no longer recommend antibiotic prophylaxis for this patient population. Surgical aortic valve replacement is not indicated for mild aortic regurgitation. In general, indications for aortic valve intervention include the presence of symptoms, left ventricular ejection fraction of 50% or less, moderate or severe aortic regurgitation at the time of other cardiac surgery, and significant left ventricular dilatation (end-systolic dimension >50 mm or indexed end-systolic dimension >25 mm/m2).
A 70-year-old man is hospitalized for a 4-week history of dyspnea, orthopnea, and daytime sleepiness. He was diagnosed with amyotrophic lateral sclerosis 6 months ago. His only medication is riluzole. On physical examination, blood pressure is 128/73 mm Hg, pulse rate is 90/min, and respiration rate is 28/min. Oxygen saturation is 87% breathing ambient air. He has right-hand atrophy, decreased mobility, and fasciculations. Lung examination reveals abdominal paradox with breathing, use of accessory breathing muscles, and shallow tachypnea. He is awake, alert, and interactive, but dozes off easily. His speech is clear with no secretions. He is able to move all extremities and shows no cranial nerve abnormality. On arterial blood gas testing, pH is 7.30, PCO2 is 76 mm Hg (10.1 kPa), PO2 is 50 mm Hg (6.65 kPa), and bicarbonate is 36 mEq/L (36 mmol/L) on room air. The calculated alveolar-arterial oxygen gradient is normal. Chest radiograph reveals bilateral basal opacities consistent with atelectasis and shallow inspiration. Which of the following is the most appropriate treatment? Invasive mechanical ventilation Noninvasive ventilation with bilevel positive airway pressure Noninvasive ventilation with continuous positive airway pressure Oxygen administration through nasal cannula
Bilevel positive airway pressure ventilation improves survival and quality of life in patients with neuromuscular disease. The use of bilevel positive airway pressure (BPAP) ventilation is the most appropriate treatment. This patient has features consistent with chronic hypercapnic respiratory failure secondary to neuromuscular disease. He has dyspnea and, more characteristically, orthopnea. The patient has chronic respiratory acidosis with a normal alveolar-arterial (A-a) oxygen gradient. BPAP delivers both inspiratory positive airway pressure and expiratory positive airway pressure and improves survival and quality of life of patients with neuromuscular disease. The settings generate a pressure difference that augments the patient's own respiratory muscle activity, leading to an increase in the size of each breath. The PCO2 level will decrease due to the increase in minute ventilation and efficiency of breathing. Invasive mechanical ventilation (mechanical ventilation with airway intubation or tracheostomy) is a therapeutic option in the setting of acute hypercapnic respiratory failure due to neuromuscular disease. However, the patient is awake, has excellent bulbar control (swallows and gags), and has a chronic disease that responds well to noninvasive ventilation. Continuous positive airway pressure (CPAP) delivers positive airway pressure at a level that remains constant throughout the respiratory cycle preventing upper airway collapse or narrowing during sleep. No additional pressure above the level of CPAP is provided and patients must initiate every breath. CPAP does not increase minute ventilation, and it is not helpful in patients with hypercapnic respiratory failure due to neuromuscular disorders. The patient has a normal A-a oxygen gradient and hypercapnia, which confirm that the hypoxemia is secondary to hypoventilation rather than a ventilation-perfusion mismatch or shunt; therefore, oxygen administration should not be needed once his PCO2 improves. More importantly, the administration of oxygen in the absence of supportive ventilation should be avoided in patients with neuromuscular disease and chronic hypercapnic respiratory failure; it has been associated with acute hypercapnia, in some cases leading to death.
A 47-year-old woman is evaluated in the emergency department for abrupt-onset severe headaches lasting 6 to 8 hours that have occurred three times in the past 4 days. The pain developed spontaneously, reached maximum intensity within 10 seconds, and is holocranial and throbbing. She reports associated nausea and photophobia but no changes in mentation, vision, sensation, or motor function. The first two episodes occurred while showering and the third while at the dinner table 2 hours ago. The patient has anxiety but no personal or family history of headache. She started taking sertraline 1 month ago for anxiety but discontinued it after headache onset. On physical examination, temperature is 37.8 °C (100.0 °F), blood pressure is 130/80 mm Hg, and pulse rate is 72/min. All other physical examination findings, including those from a neurologic examination, are normal. A head CT scan and results of lumbar puncture also are normal. Which of the following is the most appropriate management? Digital subtraction angiography Magnetic resonance angiography of the brain Methylprednisolone sodium succinate Sumatriptan Valproic acid
Brain magnetic resonance angiography or head CT angiography are the diagnostic procedures of choice for reversible cerebral vasoconstriction syndrome, the second most frequent source of thunderclap headache. This patient should undergo magnetic resonance angiography of the brain. Her recent history of thunderclap headache is characteristic of reversible cerebral vasoconstriction syndrome (RCVS). The term thunderclap headache is applied to severe headaches that reach maximum intensity within 1 minute. Although subarachnoid hemorrhage (SAH) is the most frequent cause of thunderclap headache, and some patients may experience an isolated sentinel leak before aneurysmal rupture, three episodes within 4 days would be highly unusual for SAH. Normal results of head CT scans and lumbar puncture (LP) effectively exclude this diagnosis. RCVS is the second most frequent source of thunderclap headaches. Head CT and LP findings are both typically normal. Diagnosis is confirmed by documentation of multifocal constriction of intracranial vessels that normalizes within 3 months of onset. Brain magnetic resonance angiography or head CT angiography are the diagnostic procedures of choice. Predisposing factors include vasoactive drugs (such as sympathomimetic agents, triptans, cocaine, and cannabis) and antidepressants (such as sertraline). The headaches may be triggered by exertion, Valsalva maneuvers, emotion, or showering/bathing. Although some patients develop focal deficits, encephalopathy, or seizures, most have normal findings on clinical examination. Calcium channel blockers, such as verapamil and nimodipine, are the treatments of choice. Digital subtraction angiography in patients with RCVS has been associated with transient neurologic deficits and is typically avoided. Because glucocorticoids are associated with a worse clinical course in RCVS, methylprednisolone sodium succinate has no role in its management. This drug can be appropriate therapy for other headache disorders, such as giant cell arteritis, optic neuritis, cerebral vasculitis, or pituitary apoplexy, but none of these is compatible with the patient's clinical presentation and normal results on a head CT scan and LP. Sumatriptan is an appropriate treatment for acute migraine headache but should be avoided in RCVS. Triptans may predispose patients to RCVS and potentially worsen cerebral vasospasm. Valproic acid has been shown to be effective in the prevention of episodic migraine. The International Classification of Headache Disorders, third edition, (ICHD-3) criteria for all primary headaches, including migraine, include the requirement that headache is "not better accounted for by another ICHD-3 diagnosis." The clinical picture of recurrent thunderclap headache over a span of days or weeks is much more likely to represent RCVS than migraine.
A 64-year-old woman is evaluated in the emergency department 45 minutes after sudden onset of right-sided weakness and the loss of the ability to speak. An emergent noncontrast CT of the head shows no hemorrhage or early signs of infarct. The patient also has hypertension and atrial fibrillation. Medications are hydrochlorothiazide and warfarin. On physical examination, blood pressure is 158/78 mm Hg, and pulse rate is 72/min and irregularly irregular. Global aphasia, left-gaze preference, right hemiparesis, and loss of pain sensation on the right side are noted. Results of laboratory studies show an INR of 1.3. The patient receives intravenous recombinant tissue plasminogen activator (alteplase) 1 hour after symptom onset. Blood pressure is now 168/86 mm Hg, but other vital signs are unchanged, as are results of repeat neurologic examination. Which of the following is the most appropriate next step in management? Aspirin administration CT angiography of the head Intravenous labetalol administration MRI of the brain
CT angiography of the head is the most appropriate test to determine candidacy for endovascular therapy in patients with a cardioembolic stroke who have undergone thrombolysis. This patient should have CT angiography of the head. She has an acute ischemic stroke and was appropriately treated within 3 hours of symptom onset with intravenous recombinant tissue plasminogen activator (alteplase). The neurologic examination was consistent with an acute occlusion of the left intracranial internal carotid artery or middle cerebral artery. The patient's atrial fibrillation and subtherapeutic INR make a cardioembolic stroke subtype likely. Patients with an ischemic stroke and large-vessel occlusion have low recanalization rates with intravenous thrombolysis, and recently completed clinical trials have shown a clinical benefit from the addition of endovascular therapy, such as embolectomy, among carefully selected patients. The first step in patient selection requires the presence of a large vessel occlusion on vessel imaging, which is most quickly seen with CT angiography. Because this patient already has received intravenous alteplase, all antithrombotics need to be held for at least 24 hours after a head CT shows no hemorrhage to prevent hemorrhagic conversion. In patients who do not receive thrombolysis, aspirin can reduce the risk of recurrent stroke within the first 2 weeks when administered within 48 hours of ischemic stroke onset. Intravenous labetalol should not be administered to this patient. Her blood pressure does not meet the postthrombolytic treatment threshold (180/105 mm Hg). If clinical or radiographic evidence of hemorrhage is seen after treatment, lisinopril can be initiated. MRI the area of infarction is not necessary for planning the next treatment steps. Furthermore, obtaining an MRI is time consuming and may lengthen the duration of time before an embolectomy can be performed. In acute ischemic stroke embolectomy trials, the greatest benefit occurred the faster treatment was initiated.
A 56-year-old woman is admitted to the hospital for painful, progressive skin ulceration and scabbing on the thighs of 2 weeks' duration. Medical history is significant for end-stage kidney disease requiring dialysis, type 2 diabetes mellitus, hypertension, and hypercholesterolemia. Medications are insulin, atenolol, erythropoietin, and a multivitamin. On physical examination, she is afebrile. Blood pressure is 136/84 mm Hg, pulse rate is 108/min, respiration rate is 16/min, and oxygen saturation is 98% breathing ambient air. BMI is 32. Pedal pulses are normal. Skin findings are shown. Laboratory studies show calcium level of 10.8 mg/dL (2.7 mmol/L), phosphorus level of 7.6 mg/dL (2.4 mmol/L), and parathyroid hormone level of 650 pg/mL (650 ng/L). Blood cultures are negative. Which of the following is the most likely diagnosis? Calciphylaxis Kyrle disease Nephrogenic systemic fibrosis Progressive systemic sclerosis
Calciphylaxis lesions are intensely painful, angulated, retiform purpuric patches with areas of black necrotic tissue that may form bullae, ulcerate, and leave a hard, firm eschar in patients with end-stage kidney disease. This patient has calciphylaxis. Patients with calciphylaxis present with angulated, lacy, or netlike retiform purpuric patches with areas of central dusky or black necrotic tissue that may form bullae, ulcerate, and leave a hard eschar. For unknown reasons, calcium is deposited in the medial layer of small arteries, which leads to thrombus, infarction of the overlying skin, and significant pain. The thighs and lower abdomen have been reported as the most frequent areas of involvement. Calciphylaxis typically occurs in patients who have underlying end-stage kidney disease; however, only 1% to 2% of patients on dialysis will develop calciphylaxis. Persons with calciphylaxis will usually have a corrected calcium-phosphorus product greater than 60 to 70 mg2/dL2. Therapy is difficult and multifactorial. It includes surgery to debride any areas of potential infection and parathyroidectomy in those patients with hyperparathyroidism. Medical therapies are dialysis, intravenous sodium thiosulfate, decreasing calcium intake, bisphosphonates, and meticulous wound care. Infection leading to sepsis is the leading cause of death. Prevention of infection is critically important. Kyrle disease is most frequently seen in persons with diabetes and end-stage kidney disease. It can present with an individual papule or numerous widespread hyperpigmented papules. The papules have an umbilicated central core. Kyrle disease is caused by collagen extrusion from the dermis into and through the epidermis. Nephrogenic systemic fibrosis is characterized by yellowish, thickened papules and nodules with progressive skin tightening and sclerosis. If biopsied, the skin findings mimic those of scleromyxedema. This condition has been seen in patients undergoing dialysis for end-stage kidney disease who were exposed to gadolinium-containing contrast dyes. Progressive systemic sclerosis presents similarly to nephrogenic systemic fibrosis. It is an autoimmune disease with cutaneous and systemic findings. The cutaneous findings are thickening of the skin, which results in lack of joint mobility, dilated periungual capillary loops, Raynaud phenomenon, digital ulcerations, and digital infarcts. Ulcerations and eschar formation on the upper thighs do not occur in Kyrle disease, nephrogenic systemic sclerosis, or progressive systemic sclerosis.
A 45-year-old woman is evaluated for the recent onset of resistant hypertension. During her last visit, chlorthalidone was added to her medication regimen. She reports no symptoms, and review of the systems is otherwise unremarkable. Current medications are metoprolol, amlodipine, hydralazine, and chlorthalidone. On physical examination, blood pressure is 160/96 mm Hg, and pulse rate is 65/min; other vital signs are normal. BMI is 24. There is no proptosis. The thyroid gland is not enlarged. The remainder of the examination is unremarkable. Laboratory studies: Bicarbonate 34 mEq/L (34 mmol/L) Creatinine 0.8 mg/dL (70.7 µmol/L) Potassium 2.9 mEq/L (2.9 mmol/L) Urine albumin-creatinine ratio 10 mg/g Which of the following is the most appropriate diagnostic test to perform next? Kidney ultrasonography with Doppler Plasma aldosterone concentration/plasma renin activity ratio Plasma fractionated metanephrines Polysomnography
Calculation of the plasma aldosterone concentration/plasma renin activity ratio is used to diagnose primary hyperaldosteronism. The most appropriate diagnostic test to perform next is plasma aldosterone concentration (PAC)/plasma renin activity (PRA) ratio. The patient presents with a triad of resistant hypertension, metabolic alkalosis, and hypokalemia following the addition of a thiazide diuretic, which raises suspicion for primary hyperaldosteronism. Primary hyperaldosteronism, in which aldosterone production cannot be suppressed with sodium loading, is the most common cause of secondary hypertension in middle-aged adults and an important cause of resistant hypertension. Testing for primary hyperaldosteronism is recommended if any of the following are present: resistant hypertension, hypokalemia (spontaneous or substantial, if diuretic induced), incidentally discovered adrenal mass, family history of early-onset hypertension, or stroke at age <40 years. Diuretics should be discontinued prior to testing to assure euvolemia. Calculation of the PAC/PRA ratio is used for screening, with a very high ratio of PAC/PRA suggestive of diagnosis. A positive screening test would reveal a reduced or undetectable PRA or concentration and an inappropriately high (usually >15 ng/dL) PAC, which results in a high PAC/PRA ratio of >20. Confirmatory testing is performed except when initial testing is diagnostic, as in cases of spontaneous hypokalemia with undetectable PRA and PAC (>30 ng/dL [828 pmol/L]). Confirmatory tests include oral and intravenous salt loading and the fludrocortisone suppression and captopril challenge tests. Kidney ultrasonography with Doppler is performed to diagnose renovascular disease, which can be a secondary cause of hypertension. However, no clinical trials have demonstrated that percutaneous intervention results in improvement of hypertension or lessens kidney deterioration; therefore, diagnostic testing is reserved for patients with otherwise strong indications for this study, such as a young woman with possible fibromuscular dysplasia, but not this patient. Plasma fractionated metanephrines are obtained to screen for a pheochromocytoma, which could result in hypertension; however, this is a relatively rare diagnosis that cannot explain the patient's current findings, and there are no symptoms or signs (episodic headaches, palpitations) in this patient to indicate pheochromocytoma. Polysomnography is used to diagnose obstructive sleep apnea, another secondary cause of hypertension. However, clinical suspicion for this diagnosis is low given that it cannot explain the patient's metabolic findings, and the patient does not have obesity and does not present with snoring and daytime sleepiness. Read Related TextNext Question
A 19-year-old man is evaluated in the emergency department for cardiac arrest after he fell through the ice of a frozen lake. He was in the water for less than 10 minutes, but when he was pulled out onto the ice he was unresponsive and no pulse could be felt. Bystander cardiopulmonary resuscitation (CPR) was begun immediately and continued for 25 minutes until emergency medical services arrived. At the scene his rectal temperature was 27 °C (80.6 °F). He was intubated and bag ventilated and continued to receive CPR in the ambulance on the way to the emergency department. On physical examination, temperature is 28 °C (82.4 °F). Oxygen saturation is 97% on mechanical ventilation with 65% oxygen. He is not responsive and shows no spontaneous movement or shivering. His heart rhythm on the monitor is ventricular fibrillation. Which of the following is the most appropriate management? Continue CPR with active external rewarming Continue CPR with active internal (core) rewarming Continue CPR with passive external rewarming Discontinue CPR
Cardiopulmonary resuscitation should be continued in patients with accidental hypothermia accompanied by cardiac arrest until the patient can be rewarmed. Cardiopulmonary resuscitation (CPR) should be continued with active internal (core) rewarming. Conventional treatment of ventricular arrhythmias and asystole is often ineffective until the temperature is raised to greater than 30.0 °C (86.0 °F). Because severe hypothermia may appear clinically similar to death, aggressive rewarming is appropriate in all patients in the absence of obvious irreversible signs of death. A critical first step entails removing wet clothing and covering the patient with insulating material, especially the head and neck. For mildly hypothermic, healthy individuals capable of shivering, this strategy of passive external rewarming alone suffices. Active external rewarming using warm blankets or a forced heated air blanket is commonly used in hemodynamically stable patients with moderate hypothermia. Body cavity lavage with warm fluids is an option for patients with hypothermia that is severe or does not respond to external rewarming. The colon, bladder, and stomach are readily accessible for irrigation but have a small surface area for heat exchange. Rewarming by peritoneal or pleural space irrigation is supported by case reports. Extracorporeal support, including cardiopulmonary bypass, is recommended for patients in cardiac arrest because it maximizes the rewarming rate and can provide hemodynamic support. Although this man has already received nearly an hour of CPR, there are reports of full recovery in patients with cardiac arrest in the setting of accidental hypothermia, sometimes even after CPR has been performed for many hours. Therefore, continued CPR is indicated until the patient can be rewarmed. Discontinuation of CPR is not appropriate because hypothermia prevents reaching a definite conclusion about the futility or possible effectiveness of continued resuscitation.
A 37-year-old woman is evaluated for a 3-day history of recurrent episodes of severe, piercing right maxillary pain lasting several seconds. Attacks have become progressively more frequent, now occurring several times per hour, and can either arise spontaneously or be triggered by washing the face, chewing, or applying facial cosmetics. She has had no associated conjunctival injection, tearing, or nasal congestion or drainage. The patient has multiple sclerosis. Medications are glatiramer acetate and an oral contraceptive. On physical examination, vital signs are normal; BMI is 22. A left afferent pupillary defect is noted, as is unsteadiness of tandem gait. All other physical examination findings are unremarkable, including normal facial sensation bilaterally. A fluid-attenuated inversion recovery MRI reveals periventricular and brainstem hyperintensities that are not seen with contrast enhancement. Which of the following is the most appropriate treatment? Acetazolamide Carbamazepine Indomethacin Lamotrigine
Carbamazepine is the treatment of choice for trigeminal neuralgia; management with this agent requires serum monitoring for hyponatremia and agranulocytosis.
A 37-year-old woman is evaluated for a 3-day history of recurrent episodes of severe, piercing right maxillary pain lasting several seconds. Attacks have become progressively more frequent, now occurring several times per hour, and can either arise spontaneously or be triggered by washing the face, chewing, or applying facial cosmetics. She has had no associated conjunctival injection, tearing, or nasal congestion or drainage. The patient has multiple sclerosis. Medications are glatiramer acetate and an oral contraceptive. On physical examination, vital signs are normal; BMI is 22. A left afferent pupillary defect is noted, as is unsteadiness of tandem gait. All other physical examination findings are unremarkable, including normal facial sensation bilaterally. A fluid-attenuated inversion recovery MRI reveals periventricular and brainstem hyperintensities that are not seen with contrast enhancement. Which of the following is the most appropriate treatment? Acetazolamide Carbamazepine Indomethacin Lamotrigine
Carbamazepine is the treatment of choice for trigeminal neuralgia; management with this agent requires serum monitoring for hyponatremia and agranulocytosis. The patient should receive carbamazepine. She has paroxysmal facial pain typical of trigeminal neuralgia. Episodes of this disorder consist of sharp or shock-like pain and have a duration of less than 1 second to 120 seconds, with variable frequency. The maxillary and mandibular branches of the trigeminal nerve (cranial nerve V) are affected in 95% of affected patients. Attacks may be spontaneous or triggered by innocuous stimuli. Autonomic features are rare. Incidence increases with advanced age. Multiple sclerosis (MS) is present in 5% of patients with trigeminal neuralgia and is a common cause in patients younger than 50 years. Demyelinating plaques in the pontine entry of the trigeminal root may be seen. Carbamazepine is the treatment of choice; management with this agent requires serum monitoring for hyponatremia and agranulocytosis. Acetazolamide is indicated for idiopathic intracranial hypertension (IIH). Headaches associated with this condition are phenotypically similar to tension-type headache or migraine and not to the pain of neuralgia. This patient lacks both significant risk factors for and the papilledema typical of IIH. Her afferent pupillary defect is secondary to MS. Indomethacin is the preferred treatment for chronic paroxysmal hemicrania (CPH). As with the other trigeminal autonomic cephalalgias, CPH pain mainly involves the ophthalmic branch of the trigeminal nerve, resulting in pain localized to the orbital, supraorbital, or temporal regions. Episodes of CPH last 2 to 30 minutes and recur as many as 40 times per day. Ipsilateral cranial autonomic features are present. Attacks may be precipitated by cervical spine rotation or compression, but facial stimulation is not provocative. Response to indomethacin is absolute. Lamotrigine is the treatment of choice for short-lasting unilateral neuralgiform headache attacks with conjunctival injection and tearing (SUNCT), although this type of headache is largely refractory to medical management. Similarly to trigeminal neuralgia, episodes last 1 to 600 seconds and may recur more than 100 times daily. Unlike trigeminal neuralgia, attacks of SUNCT involve the ophthalmic branch of the trigeminal nerve and include the ipsilateral autonomic features of conjunctival injection and tearing.
A 62-year-old man is evaluated for a 1-year history of progressive fatigue, dyspnea on exertion, and lower extremity edema. He has been hospitalized twice in the past year for heart failure. Medical history is otherwise unremarkable. His current medications are furosemide, amlodipine, and spironolactone. He is Black. On physical examination, temperature is normal, blood pressure is 106/68 mm Hg, pulse rate is 68/min, and respiration rate is 14/min. Oxygen saturation breathing ambient air is normal. Jugular venous distention is noted. No crackles are present. A grade 2/6 holosystolic murmur is heard at the left lower sternal border and increases with inspiration. There is no S3 or S4. The liver is enlarged. Pitting edema is present to the mid shin bilaterally. An electrocardiogram reveals sinus rhythm, biatrial enlargement, and low voltage of the QRS complexes. Q waves are present in leads V1 through V3. Echocardiogram reveals severe concentric left ventricular hypertrophy with wall thickness of 16 mm, moderate biatrial dilatation, normal left ventricular cavity size, a left ventricular ejection fraction of 55%, and no regional wall motion abnormalities. Diastolic dysfunction is noted. Right ventricular size and systolic function are normal, and the right ventricular wall thickness is slightly increased. The estimated right ventricular systolic pressure is 68 mm Hg. Tricuspid regurgitation is present. There is no pericardial effusion. Which of the following is the most likely diagnosis? Cardiac amyloidosis Constrictive pericarditis Hypertensive heart disease Hypertrophic cardiomyopathy
Cardiac amyloidosis should be suspected in Black patients older than 50 years who have left ventricular wall thickening that is not explained by loading conditions (for example, hypertension or aortic stenosis) and present with heart failure or features of diastolic dysfunction.
A 62-year-old man is evaluated for a 1-year history of progressive fatigue, dyspnea on exertion, and lower extremity edema. He has been hospitalized twice in the past year for heart failure. Medical history is otherwise unremarkable. His current medications are furosemide, amlodipine, and spironolactone. He is Black. On physical examination, temperature is normal, blood pressure is 106/68 mm Hg, pulse rate is 68/min, and respiration rate is 14/min. Oxygen saturation breathing ambient air is normal. Jugular venous distention is noted. No crackles are present. A grade 2/6 holosystolic murmur is heard at the left lower sternal border and increases with inspiration. There is no S3 or S4. The liver is enlarged. Pitting edema is present to the mid shin bilaterally. An electrocardiogram reveals sinus rhythm, biatrial enlargement, and low voltage of the QRS complexes. Q waves are present in leads V1 through V3. Echocardiogram reveals severe concentric left ventricular hypertrophy with wall thickness of 16 mm, moderate biatrial dilatation, normal left ventricular cavity size, a left ventricular ejection fraction of 55%, and no regional wall motion abnormalities. Diastolic dysfunction is noted. Right ventricular size and systolic function are normal, and the right ventricular wall thickness is slightly increased. The estimated right ventricular systolic pressure is 68 mm Hg. Tricuspid regurgitation is present. There is no pericardial effusion. Which of the following is the most likely diagnosis? Cardiac amyloidosis Constrictive pericarditis Hypertensive heart disease Hypertrophic cardiomyopathy
Cardiac amyloidosis should be suspected in Black patients older than 50 years who have left ventricular wall thickening that is not explained by loading conditions (for example, hypertension or aortic stenosis) and present with heart failure or features of diastolic dysfunction. The most likely diagnosis is cardiac amyloidosis. This patient has heart failure, elevated right heart pressure, and severe concentric wall thickening with preserved systolic function and severe pulmonary hypertension. The electrocardiogram paradoxically demonstrates low QRS voltage and a "pseudoinfarct" pattern: Q waves in the anteroseptal leads without regional wall motion abnormalities on echocardiogram. The most likely diagnosis is cardiac amyloidosis, likely related to a Val122lle mutation in transthyretin (TTR), which is present in 3% to 4% of the Black population. Cardiac amyloidosis should be suspected in Black patients older than 50 years who have left ventricular wall thickening that is not explained by loading conditions (for example, hypertension or aortic stenosis) and present with heart failure or features of diastolic dysfunction. Although a monoclonal gammopathy is present in immunoglobulin light-chain (AL) amyloidosis, this is not a feature of TTR amyloidosis. Diagnosis is established by histopathology; endomyocardial biopsy is more sensitive than abdominal fat pad biopsy. Patients with constrictive pericarditis usually have normal or only mildly increased left ventricular wall thickness. Severe pulmonary hypertension is usually not present. Amyloidosis may be mistaken for hypertensive heart disease; however, the left ventricular wall thickness in this patient is greater than expected for a patient without hypertensive heart disease. Additionally, electrocardiographic features of left ventricular hypertrophy (increased voltage) would be expected. Hypertrophic cardiomyopathy may be symmetric and nonobstructive and may demonstrate a restrictive filling pattern on echocardiogram. However, in most cases, the QRS voltage on electrocardiogram is congruent with the degree of hypertrophy.
A 48-year-old woman is evaluated for exertional substernal chest pain of several weeks' duration. The chest pain consistently subsides with rest. Medical history is significant for episodic migraine. She has no history of hypertension, hyperlipidemia, or other medical problems. She is a nonsmoker. She takes no medications other than naproxen as needed. On physical examination, vital signs are normal. Oxygen saturation is 98% breathing ambient air. The remainder of the examination is unremarkable. An electrocardiogram demonstrates baseline 1.5-mm lateral ST-segment depressions. Nuclear stress testing reveals a mild anterior wall perfusion defect, and a subsequent coronary angiogram demonstrates normal coronary arteries. Which of the following is the most likely diagnosis? Acute coronary syndrome with spontaneous recanalization Cardiac syndrome X Somatic symptom disorder Takotsubo cardiomyopathy
Cardiac syndrome X is characterized by angina and stress testing abnormalities in the absence of angiographically significant coronary artery disease. The most likely diagnosis is cardiac syndrome X. This relatively young patient with no traditional risk factors for coronary artery disease has typical angina, abnormal stress testing results, and angiographically normal coronary arteries. Cardiac syndrome X is a frequent cause of chest pain syndromes in women, and patients often present without traditional risk factors for coronary artery disease. The chest pain is often indistinguishable from classic exertional angina, and stress testing results are frequently abnormal. Many hypotheses have been proposed to explain the pathogenesis of cardiac syndrome X, with one of the most accepted centering on microvascular dysfunction as the cause. Although vasodilators should be tried, symptoms can be difficult to treat. An acute coronary syndrome is unlikely given the lack of risk factors, duration of symptoms, and normal coronary angiogram. Diagnostic criteria for somatic symptom disorder are as follows: at least one somatic symptom causing distress or interference with daily life; excessive thoughts, behaviors, and feelings related to the somatic symptoms; and persistent somatic symptoms for at least 6 months (does not have to be the same symptom for 6 months). This patient does not fulfill the diagnostic criteria for somatic symptom disorder. Takotsubo cardiomyopathy, also known as stress cardiomyopathy or apical ballooning syndrome, is a cause of nonexertional chest pain associated with electrocardiographic changes (often ST-segment elevations) and elevated cardiac enzymes. There is often, but not always, an antecedent psychological or physical stressor. The timing and exertional nature of this patient's symptoms are not consistent with the diagnosis of takotsubo cardiomyopathy. Read Related TextNext Question
A 63-year-old man is evaluated in the emergency department for progressive dyspnea. The patient reports increasing difficulty breathing while lying flat. He has a history of atrial fibrillation and underwent catheter ablation 1 week ago. Medical history is otherwise significant for hypertension. He has no history of heart failure or left ventricular dysfunction. Medications are warfarin, dronedarone, and lisinopril. On physical examination, temperature is normal, blood pressure is 88/72 mm Hg, pulse rate is 112/min, and respiration rate is 16/min. Pulsus paradoxus of 12 mm Hg is present. Oxygen saturation breathing ambient air is 96%. Cardiac examination reveals elevated estimated central venous pressure. Heart sounds are difficult to auscultate. Lung examination reveals no crackles. An electrocardiogram demonstrates sinus tachycardia. Which of the following is most likely responsible for the patient's symptoms? Atrioesophageal fistula Cardiac tamponade Pulmonary vein stenosis Retroperitoneal bleeding
Cardiac tamponade occurs in approximately 1% of patients who undergo catheter ablation procedures for atrial fibrillation; it is the most common serious complication and is likely to result in death if not recognized and treated urgently. The catheter ablation-related complication that is most likely responsible for the patient's symptoms is cardiac tamponade. Although cardiac tamponade occurs in approximately 1% of patients who undergo catheter ablation procedures for atrial fibrillation, it is the most common serious complication and is likely to result in death if not recognized and treated urgently. Cardiac tamponade occurs within a mean of 10 days after the procedure, although it may occur within hours of the procedure or be delayed by weeks. Cardiac tamponade should be suspected when the patient has a compatible history, hypotension, elevated jugular venous pressure, narrow pulse pressure, and pulsus paradoxus. An enlarged cardiac silhouette may be seen on chest radiograph ("water-bottle heart"). The electrocardiogram typically demonstrates sinus tachycardia and electrical alternans. Echocardiography readily detects pericardial effusions and is the primary modality for diagnosing cardiac tamponade. Patients who develop an atrioesophageal fistula most commonly present 1 to 4 weeks after the ablation procedure. Patients may exhibit a sudden onset of neurologic symptoms from esophageal air embolization. Patients may also present with fever, chest pain, seizures, transient ischemic attack after food intake, hematemesis, and endocarditis. MRI and CT are preferred as diagnostic studies. Patients who develop dyspnea months to years after atrial fibrillation ablation may have pulmonary vein stenosis. Other symptoms may include cough, chest pain, and hemoptysis. Pulmonary vein stenosis occurs in approximately 1% to 3% of patients, but intervention to relieve symptoms is required in only 10% of these patients. Guidelines recommend CT or MRI as the preferred diagnostic test in symptomatic patients. Vascular events, including hematomas, pseudoaneurysms at the arterial puncture site, and retroperitoneal hemorrhage from bleeding at the groin access site, are among the most common complications of catheter ablation. Symptoms include hypotension and ipsilateral flank pain. Symptom onset is typically within hours of the procedure. Diagnosis can be established with CT or ultrasonography. Retroperitoneal hemorrhage cannot explain this patient's distant heart sounds, elevated central venous pressure, or pulsus paradoxus.
A 51-year-old woman is evaluated in the hospital after having an ischemic stroke. She has hypertension treated with candesartan and dyslipidemia treated with rosuvastatin. The patient is otherwise in good health. On physical examination, blood pressure is 168/76 mm Hg. A right carotid bruit is heard. Left-sided weakness (including the face) is present. An electrocardiogram shows sinus rhythm. A CT scan of the head shows a hypodensity in the right parietal lobe. A carotid duplex ultrasound shows greater than 80% stenosis in the right internal carotid artery. A magnetic resonance angiogram of the neck without contrast confirms greater than 80% stenosis. Which of the following is the most appropriate next step in management? Carotid endarterectomy Carotid stenting Diagnostic cerebral angiography 30-Day outpatient telemetry
Carotid endarterectomy within 2 weeks of transient ischemic attack or nondisabling stroke is safe and reduces the risk of ischemic stroke in patients with symptomatic carotid artery stenosis. Carotid endarterectomy is the most appropriate next step in management. This patient has an ischemic stroke due to symptomatic extracranial internal carotid artery stenosis, which is associated with a high short-term risk of recurrent stroke despite best medical therapy. Guidelines recommend carotid endarterectomy for patients with symptomatic carotid artery stenosis within 2 weeks of transient ischemic attack or nondisabling stroke. The procedure is safe and reduces the risk of ischemic stroke. The choice of revascularization procedure is largely driven by medical comorbidities and by the expertise of locally available surgeons. However, a consistent finding has been a higher risk of perioperative stroke with stenting. In a clinical trial that enrolled symptomatic patients with carotid stenosis in Europe, endarterectomy was associated with a lower risk of recurrent stroke risk than was stenting. Carotid stenting may be considered in patients with significant cardiac comorbidities that would preclude surgery under general anesthesia because of safety considerations; this patient has no such comorbidities. Patients should be counseled that stenting is associated with a higher risk of perioperative stroke, according to randomized clinical trials of symptomatic patients. Diagnostic angiography is not routinely indicated to confirm the degree of stenosis before carotid revascularization, given the associated risk of vessel injury. Additionally, sufficient information is available from both the magnetic resonance angiogram and the carotid duplex ultrasound. A 30-day period of outpatient telemetry is not required to diagnose the stroke subtype in this patient with established high-grade stenosis in the carotid artery. Evaluation for occult atrial fibrillation is recommended for patients in whom the cause of stroke has not been identified, especially if an embolic cause (suggested by infarcts involving the brain surface or in the distribution of more than one cerebral artery) is suspected.
A 54-year-old woman is evaluated for a 1-year history of generalized abdominal pain that is constant throughout the day, every day. The pain is not triggered by eating and is not relieved by bowel movements. She reports no weight loss and no change in bowel habits. She also has depression. A screening colonoscopy done 8 months earlier was normal. Her only medication is escitalopram. On physical examination, vital signs are normal; BMI is 28. The abdomen is tender to palpation in all quadrants without guarding. The remainder of the examination is normal. Results of laboratory studies, including a complete blood count and liver chemistry tests, are normal. Which of the following is the most appropriate treatment? Alosetron Budesonide Cognitive-behavioral therapy Linaclotide
Centrally mediated abdominal pain syndrome is characterized by near-constant abdominal pain lasting longer than 6 months, involving a large anatomic distribution, and without initiating triggers or alarm features. Cognitive-behavioral therapy is the most appropriate treatment for this patient. She meets the diagnostic criteria for centrally mediated abdominal pain syndrome (CAPS): near-constant abdominal pain lasting longer than 6 months, involving a large anatomic distribution, and without initiating triggers or alarm features. Treatment includes a combination of pharmacologic and/or psychological therapies. She already takes a selective serotonin reuptake inhibitor for depression, but it has not improved her abdominal pain syndrome. Four classes of psychotherapy have shown benefit in patients with CAPS when combined with medical therapy: cognitive-behavioral therapy, psychodynamic-interpersonal therapy, mindfulness- and acceptance-based therapy, and hypnotherapy. Irritable bowel syndrome (IBS) is a symptom complex characterized by abdominal pain and altered bowel habits. The diagnosis is made by the fulfillment of symptom-based criteria, including the presence of recurrent abdominal pain or discomfort at least 3 days per month in the last 3 months that is associated with two or more of the following: relief with defecation, onset associated with a change in frequency of stool, or onset associated with a change in form of stool. IBS is further subtyped based on the predominant stool pattern as IBS with predominant constipation (IBS-C), IBS with predominant diarrhea (IBS-D), mixed IBS, or IBS unclassified. CAPS differs from IBS in that there are no consistent initiating triggers and symptoms are not alleviated with bowel movements. The patient reports no constipation, and her pain is not relieved with bowel movements, so IBS is an unlikely diagnosis. Alosetron is a 5-HT3 antagonist available for women with IBS-D whose symptoms have not responded to conventional therapy; however, owing to an increased risk for severe constipation and ischemic colitis with its use, it is restricted by a mandatory prescribing program. Because this patient does not have IBS, alosetron is not indicated. Budesonide is indicated in patients with inflammatory bowel disease. This patient reports no chronic diarrhea, a hallmark symptom of inflammatory bowel disease, and a colonoscopy earlier this year was normal, showing no inflammation. Linaclotide is a guanylate cyclase-C activator used in the treatment of IBS-C after fiber supplementation or osmotic and stimulatory laxative therapy fails. It has no role in the treatment of CAPS.
A 49-year-old man is evaluated for a 2-day history of posterior neck stiffness and pain that radiates down his left arm and into the fourth and fifth fingers of his left hand. He is left-handed and works as a roofer. The pain worsens when he turns his head to the left and improves when he lies down, although he sometimes has pain when rising from a prone position. He has not had any arm or hand weakness or problems writing. He has no systemic symptoms. On physical examination, vital signs are normal. On palpation, the pain is reproduced when the examiner applies downward pressure with the patient's head bent to the left and extended (Spurling test). Pain is relieved when the patient holds his left arm above the plane of his shoulder. Neck range of motion is limited with both left and right lateral rotation. There is no cervical spine tenderness to palpation. The neurologic examination is normal. Which of the following is the most appropriate management? Cervical collar Cervical MRI Electrodiagnostic testing Gabapentin Neck exercises
Cervical radiculopathy, caused by nerve root compression, usually resolves within 2 to 3 months by using conservative measures; stretching and strengthening exercises of the neck muscles provide the best intermediate-term relief. The most appropriate management of this patient with symptoms consistent with cervical radiculopathy is neck exercises. Stretching and strengthening exercises provide intermediate-term relief of symptoms and should be part of a multimodal approach. Other nonpharmacologic options include acupuncture, early mobilization, and spinal manipulation. Cervical traction appears to be of limited benefit. Patients should also be informed that most patients with neck pain have resolution or near-resolution of symptoms within 2 to 3 months of onset by using conservative measures. The use of a cervical collar in patients with neck pain should be avoided because it can lead to neck muscle atrophy, especially when used for longer than 1 to 2 weeks. Shorter-term use appears to be no more effective for symptom relief than sham interventions. This patient lacks any "red flag" findings that would warrant imaging, whether in the form of plain radiography or MRI. Features that would prompt imaging include constitutional symptoms; personal history of or concern for malignancy; progressive neurologic symptoms; or myelopathic findings, such as difficulty writing, gait disturbance, hypertonia, hyperflexia, or problems with fine manipulation. Electrodiagnostic testing is most helpful to diagnose peripheral nerve entrapment syndromes or peripheral neuropathy as the cause of arm symptoms. Both of these conditions should be considered when arm symptoms are more prominent than neck symptoms. Electrodiagnostic testing can also identify cervical radiculopathy as the cause of neck pain but only when motor axonal injury is present; cervical radicular pain can exist in the absence of axonal injury. Therefore, the best course of action for this patient is conservative treatment without diagnostic testing. NSAIDs are considered first-line pharmacologic therapy for acute neck pain, including acute cervical radiculopathy. Cyclobenzaprine, when used at doses greater than 15 mg/d, has been shown to be effective for treating acute neck pain when muscle spasm is present, although it should be used with caution in older patients. Gabapentin, a neuromodulator, can be used to treat chronic radicular pain; however, it does not have a role in the management of acute radicular symptoms.
Treatment for morbilliform drug rash?
Characteristic findings of morbilliform drug reaction include erythematous papules coalescing into plaques, often with some pruritus, and no accompanying systemic symptoms. This patient is experiencing a classical morbilliform, or exanthematous, drug eruption, which is the most common form of cutaneous adverse drug reaction. Morbilliform eruptions begin 4 to 14 days after the initiation of a new medication. Characteristic skin findings begin as coalescing fine monomorphic papules on the trunk and spread distally and symmetrically. Peripheral eosinophilia or leukocytosis may be seen, but other signs of systemic involvement are not present. The therapy is to stop the medication and treat symptomatically with antihistamines and medium-potency topical glucocorticoids for a brief time while the reaction resolves. Drug reaction with eosinophilia and systemic symptoms (DRESS) is another severe cutaneous adverse reaction. Its onset is notably remote from initiation of the offending agent, often by as much as 2 to 6 weeks. While eosinophilia is a feature of DRESS syndrome, DRESS is associated with other signs of systemic involvement such as liver chemistry abnormalities. DRESS may manifest a morbilliform appearance, but typical features also include skin pain and pronounced facial edema that are absent in morbilliform drug eruption. Stevens-Johnson syndrome results in necrosis and sloughing of the epidermis. Skin pain and involvement of mucous membranes is a key feature. Rare cases of Stevens-Johnson syndrome are precipitated by an infection, but this patient's skin findings are more consistent with morbilliform eruption. Viral exanthem may appear identical in morphology to a morbilliform drug eruption. However, the appearance of this rash should be closer to the onset of symptoms of the infection, as opposed to 3 weeks later. In this case, a common cause of morbilliform reaction and development of rash within 4 days of starting the medication makes morbilliform reaction more likely.
A 67-year-old man is evaluated for headache, fatigue, and weakness for the past several weeks. Medical history is significant for metastatic melanoma that is being treated with ipilimumab, which is his only medication. On physical examination, vital signs are normal. There is a well-healed excisional scar on his posterior right shoulder. The remainder of the physical examination is normal. MRI shows enlarged pituitary with homogeneous enhancement. There is no compression of the optic chiasm. Laboratory studies: Cortisol (8 AM) 3 µg/dL (82.8 nmol/L) Prolactin 12 ng/mL (12 µg/L) Thyroid-stimulating hormone 0.2 µU/mL (0.2 mU/L) Thyroxine (T4), free 0.6 ng/dL (7.7 pmol/L) Which of the following is the most likely cause of this patient's findings? Ipilimumab-induced hypophysitis Lymphocytic hypophysitis Pituitary adenoma Primary hypothyroidism
Checkpoint inhibitors such as nivolumab, ipilimumab, and pembrolizumab have been associated with the development of hypophysitis with most patients presenting with the combination of headache, pituitary enlargement, and hypopituitarism. In this patient, the most likely cause of pituitary enlargement and evidence of hypopituitarism is drug-induced hypophysitis. Ipilimumab is a checkpoint inhibitor that has been repeatedly associated with the development of hypophysitis in up to 17% of treated patients. Most patients present with the combination of headache, pituitary enlargement, and hypopituitarism occurring during the early phase of therapy, typically within 10 weeks. While the pituitary enlargement often resolves spontaneously, the panhypopituitarism appears to be permanent. Lymphocytic hypophysitis is an autoimmune disease most commonly occurring in women during pregnancy and postpartum. It leads to pituitary enlargement, possible mass effect, and often deficiency of adrenocorticotropic hormone (ACTH). While the imaging and laboratory characteristics described could be consistent with lymphocytic hypophysitis, the patient's exposure to a drug known to cause hypophysitis and other aspects of the clinical history are not consistent with this diagnosis. A pituitary adenoma is less likely given the homogeneous enhancement on MRI. Pituitary adenomas are usually evident on MRI after contrast administration. They appear as a relatively nonenhancing lesion within a homogeneously enhancing pituitary gland. This patient's MRI is not consistent with a pituitary adenoma. Untreated primary hypothyroidism can cause pituitary enlargement due to thyrotroph hyperplasia and can mimic a pituitary adenoma. This patient's low thyroid-stimulating hormone and free thyroxine levels do not support a diagnosis of primary hypothyroidism.
A 26-year-old woman is evaluated for a 2-month history of diarrhea characterized by two to three semibloody stools per day associated with cramping lower abdominal pain. She reports no fever, chills, nausea, vomiting, or weight loss. She has not traveled internationally. She takes no medication. On physical examination, vital signs are normal. Abdominal examination shows left and right lower abdominal tenderness to palpation. Rectal examination is remarkable for bright red blood. Stool testing for enteric pathogens is negative. Results of colonoscopy show inflamed mucosa characterized by granularity, erythema, friability, and loss of vascular pattern that starts at the anorectal verge and extends proximally in a continuous and symmetric fashion to the splenic flexure where there is an abrupt transition from affected to normal mucosa. The terminal ileum is normal. Biopsy results for the inflamed mucosa reveal crypt abscesses along with distorted and branching colonic crypts. Which of the following is the most likely diagnosis? Chronic Entamoeba histolytica infection Crohn colitis Cytomegalovirus infection Ulcerative colitis
Chronic bloody diarrhea and abdominal discomfort are typical presenting symptoms of inflammatory bowel disease; endoscopic findings help distinguish ulcerative colitis from Crohn disease. Ulcerative colitis is the most likely diagnosis in this patient. The differential diagnosis of chronic bloody diarrhea includes inflammatory bowel disease (ulcerative colitis or Crohn colitis) and chronic ulcerating infections such as cytomegalovirus or Entamoeba histolytica. This patient's stool studies for enteric pathogens are negative, making Crohn colitis or ulcerative colitis the most likely cause of her symptoms. The mucosal biopsy results showing crypt architectural distortion confirm a chronic colitis such as ulcerative colitis or Crohn disease. The endoscopic description of inflammation beginning at the anorectal verge and extending proximally in a continuous fashion with transition to normal mucosa at splenic flexure is consistent with left-sided ulcerative colitis. Crohn disease characteristically has a patchy progression pattern resulting in "skip lesions" and may spare the rectum, making Crohn colitis less likely in this case. Entamoeba histolytica causes amoebic dysentery. In the United States, amebiasis is usually diagnosed in immigrants or travelers returning from endemic areas, or in institutional settings. Although most infected persons are asymptomatic, others may develop bloody or watery diarrhea, abdominal pain, and fever. Rarely, a chronic syndrome of diarrhea, weight loss, and abdominal pain can mimic inflammatory bowel disease. Stool microscopy for ova and parasites can aid in the diagnosis by detecting the protozoa, but stool antigen testing is more sensitive. This patient's negative travel history and findings on colonoscopy and biopsy are not consistent with amebiasis. Cytomegalovirus infection is an uncommon infection in immunocompetent patients but can occur and cause significant symptoms. The most common presenting symptoms are diarrhea, fever, and abdominal pain; half of patients have grossly bloody stools. The most common finding on endoscopy is well-demarcated ulcerations. On biopsy, histological findings include multinucleated giants cells with eosinophilic inclusions. The rarity of this condition coupled with the patient's colonoscopy and biopsy findings make cytomegalovirus infection an unlikely diagnosis.
A 40-year-old woman is evaluated at a follow-up appointment for hepatitis B virus (HBV) infection, which was diagnosed at age 23 years. The patient also has psoriasis. She is married and is sexually active. Medications are topical clobetasol and an oral contraceptive agent. On physical examination, vital signs are normal. Psoriatic lesions are present on the elbows and knees. No hepatosplenomegaly is noted. Laboratory studies: Alanine aminotransferase 125 U/L Aspartate aminotransferase 112 U/L Hepatitis B surface antigen Positive Hepatitis B e antigen Positive HBV DNA 41,326 IU/mL Which of the following is the most appropriate treatment? Adefovir Lamivudine Pegylated interferon Tenofovir No treatment
Chronic hepatitis B viral infection in the immune-active, hepatitis B e antigen-positive phase should be treated with tenofovir or entecavir to decrease hepatic inflammation and the risk for progression to fibrosis.
A 55-year-old man is referred for evaluation of fatigue, weight gain, decreased libido, and difficulty maintaining an erection. Sexual functioning was normal until 6 months ago, and he has fathered two children. Medical history is significant for polysubstance abuse that is being managed with daily methadone. His medical history is otherwise unremarkable, and his only medication is methadone. On physical examination, vital signs are normal. Neurological, genitalia, and the remainder of the physical examination are normal. Laboratory studies: Follicle-stimulating hormone 5 mU/mL (5 U/L) Luteinizing hormone 4 mU/ml (4 U/L) Prolactin 12 ng/mL (12 µg/L) Testosterone 185 ng/dL (6.4 nmol/L) Thyroid-stimulating hormone 2.4 µU/mL (2.4 mU/L) Thyroxine (T4), free 1.3 ng/dL (16.8 pmol/L) Which of the following is the most likely cause of this patient's hypogonadism? Age-related decline in gonadal function Chronic opioid therapy Pituitary tumor Primary gonadal failure
Chronic opioid use suppresses gonadotroph function, resulting in hypogonadotropic hypogonadism, which is increasingly recognized as a cause of secondary hypogonadism. This patient's opioid use is most likely responsible for his sexual dysfunction. Secondary hypogonadism is typically a result of insufficient gonadotropin-releasing hormone production by the hypothalamus or deficient luteinizing hormone (LH)/follicle-stimulating hormone (FSH) secretion by the anterior pituitary. This patient has symptoms of hypogonadism and a low testosterone level. Low or inappropriately normal FSH and LH levels in the presence of simultaneous low testosterone levels are diagnostic of secondary hypogonadism (hypogonadotropic hypogonadism). Untreated sleep apnea, exogenous testosterone administration, and obesity are common causes of secondary hypogonadism. Other acquired causes include hyperprolactinemia, chronic opioid use, glucocorticoid use, or infiltrative disease (lymphoma or hemochromatosis). Chronic opioid therapy is a well-established cause of hypogonadotropic hypogonadism. The mechanism of opioid-induced hypogonadism is thought to be central hypogonadism, with downregulation of gonadotropin-releasing hormone and subsequently LH and FSH. This, in turn, results in decreased testosterone production. This patient's decline in gonadal function is not a result of his age. Although testosterone levels may decline with age, this patient's testosterone level of 185 ng/dL (6.4 nmol/L) is well below the normal range and below what would be expected for age-related decline in testosterone. Although a pituitary tumor isn't completely ruled out without an MRI, the most likely cause of this patient's hypogonadism is the chronic opioid therapy. Testosterone levels greater than 150 ng/dL (5.2 nmol/L) are less likely to be related to a pituitary tumor. Patients with hypogonadotropic hypogonadism and testosterone levels less than 150 ng/dL (5.2 nmol/L) should have an MRI to evaluate for a pituitary tumor. The normal TSH and prolactin levels also argue against a pituitary or hypothalamic tumor. In primary gonadal failure, there would be elevated FSH and LH levels with a low testosterone level. A low testosterone level with inappropriately normal or low FSH and LH levels is consistent with hypogonadotropic hypogonadism.
A 77-year-old man is evaluated for a 2-month history of worsening fatigue, increasing frequency of urination, nocturia, and anorexia. History is significant for hypertension, hypertriglyceridemia, gastroesophageal reflux disease, and depression. He has been taking low-dose aspirin and valsartan for more than 10 years, omeprazole and St. John's wort for 8 months, and fenofibrate for 2 months. On physical examination, blood pressure is 150/79 mm Hg, and pulse rate is 82/min. The remainder of the examination is unremarkable. Laboratory studies: Creatinine 2.8 mg/dL (247.5 µmol/L); 9 months ago: 1.2 mg/dL (106.1 µmol/L) Urinalysis Specific gravity 1.008; trace blood; 2+ protein; 3-5 erythrocytes/hpf; 5-7 leukocytes/hpf Kidney ultrasound shows 9-cm kidneys without hydronephrosis or calculi bilaterally. Which of the following is the most likely cause of the patient's kidney findings? Aspirin Fenofibrate Omeprazole St. John's wort
Chronic tubulointerstitial nephritis can be caused by proton pump inhibitors, and the median time from drug initiation to diagnosis may exceed 6 to 9 months. The proton pump inhibitor omeprazole is the most likely cause of this patient's kidney findings. Nonspecific symptoms of tubulointerstitial disease include polyuria, malaise, and anorexia, as well as progressive kidney dysfunction. Urinary findings can range from minimal findings to the presence of sterile pyuria, proteinuria, and hematuria. Chronic tubulointerstitial diseases most commonly result from previous injury due to acute interstitial nephritis. Therefore, the history and physical examination should focus on conditions associated with acute tubulointerstitial disease and other potential treatable causes with a careful review of medications. Proton pump inhibitors are the most commonly prescribed drugs worldwide. The occurrence of interstitial nephritis is not dose related, and disease can recur with reexposure. The median time from drug initiation to interstitial nephritis diagnosis is variable but can exceed 6 months to 9 months, with 10 to 11 weeks the most common interval. There is also emerging literature that proton pump inhibitors may contribute to the development and progression of chronic kidney disease. Although large amounts of aspirin and NSAIDs can cause tubulointerstitial nephritis, a daily dose of low-dose aspirin (81 mg) is unlikely to cause interstitial nephritis. It is unlikely that therapeutic doses of aspirin alone cause chronic kidney disease in patients with normal underlying kidney function. Fenofibrate nephrotoxicity is characterized by a rise in serum creatinine typically occurring ≥6 months after initiation of therapy. The sediment is bland, and there is usually no significant proteinuria. The mechanism of increase in creatinine is unknown. This patient has been taking fenofibrate for only 2 months, making this drug an unlikely cause of his interstitial nephritis. St. John's wort (Hypericum perforatum) is most commonly used to treat depression. Adverse effects are uncommon but the following have been reported: gastrointestinal symptoms, dizziness, confusion, tiredness, sedation, photosensitivity, dry mouth, urinary frequency, anorgasmia, and swelling. St. John's wort has not been associated with tubulointerstitial nephritis.
A 24-year-old woman is evaluated for cystitis symptoms of 4 days' duration. She reports no fever, chills, flank pain, or vaginal discharge. She has had similar symptoms three times within the past 10 months. She has been treated each time with trimethoprim-sulfamethoxazole at an urgent care center. The last episode was 5 weeks ago. She has sexual intercourse infrequently. Her only medication is an oral contraceptive. On physical examination, vital signs and other findings are unremarkable. On microscopic urinalysis, leukocytes are too numerous to count, erythrocyte count is 10/hpf, 4+ bacteria are present, and rare squamous epithelial cells are seen. Which of the following is the most appropriate management? Nitrofurantoin Trimethoprim-sulfamethoxazole Urine culture plus ampicillin Urine culture plus cefpodoxime Urine culture plus ciprofloxacin
Ciprofloxacin and levofloxacin are the preferred antimicrobial agents for the treatment of recurrent cystis when trimethoprim-sulfamethoxazole local resistance rates are high or the patient has been treated with an antibiotic for a urinary tract infection within the previous 3 months. The most appropriate management of this patient is urine culture plus ciprofloxacin. This young woman has a classic presentation and typical dipstick urinalysis findings of a lower urinary tract infection (UTI). Urine cultures are not generally necessary to confirm the diagnosis; however, culture and susceptibility testing are indicated when infection is recurrent. Recurrent UTI is defined as three episodes of UTI in the preceding 12 months or two episodes in the preceding 6 months. Recurrent UTI is common in women. A recurrent UTI may be a relapse or reinfection. Relapse is defined as an infection caused by the same strain (by repeat culture) as the initial UTI and occurs within 2 weeks of completing initial therapy. Reinfection is diagnosed if the UTI is caused by a different strain than that causing the initial infection or if a sterile urine culture was documented between episodes. Most recurrences are reinfections. While awaiting results, she should begin empiric treatment with ciprofloxacin twice daily for 7 days. Although fluoroquinolone antibiotics are no longer recommended as first-line agents for the treatment of cystitis because of increasing concerns for potential adverse effects and uropathogen antimicrobial resistance development, ciprofloxacin and levofloxacin are the preferred antimicrobial agents when trimethoprim-sulfamethoxazole local resistance rates are high (>20%) or the patient has been treated with an antibiotic for a UTI within the previous 3 months. Having recently received antibiotics defines this patient's UTI as complicated, warranting 7 to 10 days of treatment with a fluoroquinolone antibiotic. Nitrofurantoin, trimethoprim-sulfamethoxazole, fosfomycin, and oral β-lactams are not recommended as first-line empiric oral therapy in complicated cystitis because of concerns regarding resistance to these agents. In the case of culture-proven sensitivity, these agents can be used in the treatment of complicated UTI. Ampicillin and amoxicillin are no longer acceptable UTI treatment options because more than one third of community-acquired Escherichia coli harbor resistance to this agent.
A 64-year-old man is evaluated for a 5-day history of increased urination and weakness. He was recently diagnosed with non-small cell lung cancer and started on chemotherapy 10 days ago. Medications are cisplatin, gemcitabine, paclitaxel, bevacizumab, and ondansetron. Physical examination reveals a chronically ill-appearing man. Blood pressure is 105/72 mm Hg, and pulse rate is 100/min; other vital signs are normal. The remainder of the examination is unremarkable. Laboratory studies: Complete blood count Normal Calcium 8 mg/dL (2 mmol/L) Creatinine 2.1 mg/dL (185.6 µmol/L); baseline, 0.9 mg/dL (79.6 µmol/L) Electrolytes : Sodium 132 mEq/L (132 mmol/L) Potassium 2.8 mEq/L (2.8 mmol/L) Chloride 105 mEq/L (105 mmol/L) Bicarbonate 18 mEq/L (18 mmol/L) Glucose 90 g/dL (5 mmol/L) Magnesium 1.5 mg/dL (0.62 mmol/L) Phosphorus 1.9 mg/dL (0.61 mmol/L) Fractional excretion of sodium 2% Urinalysis Specific gravity 1.010; pH 6.0; 1+ protein; 1+ glucose; occasional granular casts Which of the following is the most likely cause of this patient's acute kidney injury? Bevacizumab Cisplatin Gemcitabine Paclitaxel
Cisplatin-induced acute kidney injury is characterized by polyuria, tubular injury, hypomagnesemia, and proximal renal tubular acidosis with Fanconi syndrome. The patient has cisplatin-induced acute kidney injury (AKI). AKI due to renal tubular dysfunction can occur following administration of cisplatin and ifosfamide. High-dose methotrexate can also cause renal tubular injury, which can be avoided with aggressive intravenous hydration, forced diuresis, urine alkalization, and administration of leucovorin. Cisplatin induces direct cellular toxicity as a result of their transport through tubular cells, induction of mitochondrial injury, oxidative stress, and activation of apoptotic signaling pathways. Cisplatin-induced AKI develops 7 to 10 days after administration and is characterized by polyuria, tubular injury, hypomagnesemia, and proximal renal tubular acidosis (RTA) with Fanconi syndrome. Cisplatin nephrotoxicity results in volume depletion from urinary excretion of sodium and hypomagnesemia from urinary magnesium loss with ensuing hypokalemia and hypocalcemia. Fanconi syndrome in this patient is supported by the normal anion gap metabolic acidosis, hypophosphatemia, and urinary findings of glycosuria (in the setting of normal blood glucose) and proteinuria. In patients receiving cisplatin, serum electrolyte levels and kidney function must be carefully monitored because early recognition and prompt discontinuation of the drug are essential for renal recovery. Bevacizumab and gemcitabine can cause hypertension and AKI due to thrombotic microangiopathy, which manifests as thrombocytopenia with hemolytic anemia, elevated lactate dehydrogenase, decreased haptoglobin, and schistocytes on peripheral smear. Urinary findings usually demonstrate significant proteinuria and hematuria. No laboratory findings in this patient support this diagnosis, and these drugs do not cause renal tubular injury. Paclitaxel has been associated with subacute diffuse interstitial lung disease and peripheral neuropathy. Paclitaxel does not cause AKI or the electrolyte abnormalities as seen in this patient.
A 19-year-old man is brought to the emergency department after he attended a party with friends. He is anxious and tremulous. He has a history of depression. His only medication is fluoxetine. On physical examination, he is alert and oriented. Temperature is 38.9 °C (102 °F), blood pressure is 136/79 mm Hg, pulse rate is 112/min, and respiration rate is 20/min. Oxygen saturation is 98% breathing ambient air. Physical examination is notable for slow, continuous, horizontal eye movements, tremor of extremities, hyperreflexia, and sustained ankle clonus and spontaneous myoclonus. The physical examination is otherwise normal. Urine toxicology screening is pending. Which of the following is the most likely diagnosis? Anticholinergic toxicity Malignant hyperthermia Neuroleptic malignant syndrome Serotonin syndrome
Classic features of serotonin syndrome include hyperthermia, tremor, hyperreflexia and clonus; treatment is mainly supportive, using benzodiazepines as needed to keep the patient calm and to control blood pressure and heart rate. The most likely diagnosis is serotonin syndrome. The features of hyperthermia, tremor, hyperreflexia, ocular clonus (slow, continuous, horizontal eye movements), other clonus (spontaneous or induced), and anxiety are classic features of this syndrome. Hyperreflexia and clonus help distinguish serotonin syndrome from other hyperthermic syndromes and toxic ingestions. This patient's history supports the diagnosis, which usually occurs after coingestion of several serotonergic medications—for example, fluoxetine and methylenedioxymethamphetamine ("ecstasy"). Treatment is mainly supportive, using benzodiazepines as needed to keep the patient calm and to control blood pressure and heart rate. Physical restraint can lead to agitated exertion and worsen hyperthermia. Autonomic instability is common, so close monitoring is recommended. Only in very severe cases of agitation or hyperthermia do patients need to be deeply sedated, intubated, paralyzed, and sometimes treated with cyproheptadine. Anticholinergic toxicity is unlikely in this patient because he has no signs of mydriasis, dry mucus membranes, or urinary and bowel retention. He does exhibit hyperthermia and agitation, but has clonus and hyperreflexia, which are not associated with anticholinergic toxicity. Malignant hyperthermia would be very unlikely without a history of inhaled anesthesia agents or neuromuscular blockade. Clinical features of malignant hyperthermia usually include higher fever, muscle rigidity, and, occasionally, hemorrhage but not hyperreflexia or clonus. Neuroleptic malignant syndrome would be very unlikely without a history of neuroleptic medications, such as haloperidol. It usually develops subacutely during days or weeks, whereas serotonin syndrome typically develops within hours. Rigidity with hyporeflexia is more common, rather than hyperreflexia and myoclonus in serotonin syndrome. Hyperthermia, altered mental status, and rigidity are features of both syndromes. Neuroleptic malignant syndrome usually takes many days to resolve, whereas serotonin syndrome usually resolves within 24 hours. Read Related TextNext Question
An 18-year-old man is evaluated in the emergency department for abdominal cramping and bloody diarrhea of 6 days' duration. Medical history is unremarkable, and he takes no medications. On physical examination, temperature is 37.0 °C (98.6 °F), blood pressure is 98/60 mm Hg, pulse rate is 100/min, and respiration rate is 16/min. Oxygen saturation is normal breathing ambient air. His abdomen is tender, without guarding or organomegaly. The examination is otherwise unremarkable. Laboratory studies: Haptoglobin Undetectable Hemoglobin 6.1 g/dL (61 g/L) Leukocyte count 6800/µL (6.8 × 109/L) Platelet count 37,000/µL (37 × 109/L) Reticulocyte count 9.8% of erythrocytes Creatinine 3.6 mg/dL (318 µmol/L) Urinalysis 3+ blood; 3+ protein; 0-2 erythrocytes/hpf; 0-2 leukocytes/hpf; several granular casts The peripheral blood smear shows schistocytes and scant platelets without clumps. The direct antiglobulin (Coombs) test is negative. What is the most likely diagnosis? Atypical hemolytic uremic syndrome Hemolytic uremic syndrome Immune hemolytic anemia and thrombocytopenia Rapidly progressive glomerulonephritis
Classic hemolytic uremic syndrome is a diarrhea-associated syndrome of microangiopathic hemolytic anemia, thrombocytopenia, and acute kidney injury caused by Shiga toxin-producing Escherichia coli and, less commonly, Shigella dysenteriae. This patient most likely has hemolytic uremic syndrome (HUS). HUS is a diarrhea-associated syndrome of microangiopathic hemolytic anemia, thrombocytopenia, and acute kidney injury caused by Shiga toxin-producing Escherichia coli, typically with serotypes O157:H7, O104:H4, and, less commonly, Shigella dysenteriae. Shiga toxin binds to endothelial cells, triggering thrombosis and resulting in a thrombotic microangiopathy. It also binds to renal mesangial cells, podocytes, and renal tubular cells, causing direct damage. These actions lead to acute kidney injury. Although it is more commonly seen in children, it can also present in adults. Supportive care is a cornerstone of treatment, and fluid management with volume expansion is critical despite any existing oliguria. Atypical HUS is a congenital syndrome caused by overwhelming complement activation that is not preceded by a diarrheal illness as typical HUS is. The distinction between thrombotic thrombocytopenic purpura and typical and atypical HUS is difficult but important to make because atypical HUS is effectively treated by infusions of eculizumab, a monoclonal antibody directed against the terminal components of the complement cascade. The combination of an immune-mediated hemolytic anemia, usually warm antibody, IgG mediated, and immune thrombocytopenic purpura is known as Evans syndrome. Although this patient has evidence of a hemolytic anemia and thrombocytopenia, the direct antiglobulin test is negative, and schistocytes, reflecting microangiopathic hemolysis, were noted on the peripheral blood smear. Spherocytes would be expected in IgG-mediated autoimmune hemolytic anemia. Patients with rapidly progressive glomerulonephritis (RPGN) typically present with the nephritic syndrome and may sometimes be in advanced kidney failure at the time of presentation. Other symptoms and clinical findings related to an underlying cause may also be present, such as systemic signs of vasculitis (arthritis, epistaxis, hemoptysis) or lung hemorrhage (Goodpasture syndrome). The absence of active urinary sediment and the presence of microangiopathic hemolytic anemia make RPGN an unlikely diagnosis.
A 54-year-old woman is evaluated for a 4-week history of dyspnea on exertion, malaise, fatigue, and anorexia. History is significant for hypertension, gout, and osteoarthritis. Medications are losartan, hydrochlorothiazide, allopurinol, naproxen, and aspirin. On physical examination, blood pressure is 148/84 mm Hg, and pulse rate is 98/min; other vital signs are normal. Conjunctivae are pale. There is 2+ edema of the ankles. Laboratory studies: Hemoglobin 8.0 g/dL (80 g/L) Albumin 3.0 g/dL (30 g/L) Calcium 9.8 mg/dL (2.5 mmol/L) Creatinine 2.2 mg/dL (194.5 µmol/L); 3 weeks ago: 1.2 mg/dL (106.1 µmol/L) Total protein 8.4 g/dL (84 g/L) Urate 7.0 g/dL (0.41 mmol/L) Urinalysis 1+ protein; 2-5 granular casts/hpf; 1-2 erythrocytes/hpf Urine protein-creatinine ratio 6100 mg/g Chest radiograph is normal. Which of the following is the most likely diagnosis? Light chain cast nephropathy NSAID-induced acute tubular injury Renal sarcoidosis Uric acid nephropathy
Clinical clues to the diagnosis of light chain cast nephropathy from multiple myeloma include an elevated urine protein-creatinine ratio with minimal proteinuria by urine dipstick, anemia, and hypercalcemia. The most likely diagnosis is light chain cast nephropathy in this patient with multiple myeloma. In multiple myeloma, acute kidney injury from light chain cast nephropathy is the most common type of kidney disease. Cast nephropathy is characterized by intratubular obstruction with light chain casts that can result in acute tubular injury. A clinical clue to the diagnosis is the presence of an elevated urine protein-creatinine ratio, with minimal proteinuria detected by dipstick urinalysis (dipstick urinalysis detects albumin but not light chains). Other supporting findings are the presence of anemia and hypercalcemia (when calcium measurement is corrected for albumin). Exposure to NSAIDs can cause tubular injury. However, NSAIDs do not cause a discrepancy in proteinuria between urinalysis and urine protein-creatinine ratio. Renal sarcoidosis can result in tubulointerstitial dysfunction. Other kidney manifestations include direct ureteral involvement, retroperitoneal fibrosis, and, more commonly, hypercalcemia, hypercalciuria, nephrolithiasis, and nephrocalcinosis via excessive production of 1,25-dihydroxyvitamin D in granulomas. However, renal sarcoidosis is rare in patients without thoracic sarcoidosis and cannot account for the discrepancy in proteinuria between urinalysis and urine protein-creatinine ratio. Uric acid nephropathy is unlikely given the modest elevation in the serum urate level. Furthermore, an elevated urine protein-creatinine ratio is not consistent with uric acid nephropathy.
A 39-year-old woman is evaluated for a 3-week history of malodorous vaginal discharge. She was treated with antibiotics for a urinary tract infection 3 weeks before the onset of symptoms. She is in a monogamous sexual relationship. Medical history is unremarkable. Her only medication is an oral contraceptive. On physical examination, vital signs are normal. Pelvic examination reveals thin, homogenous, grayish vaginal discharge. There is no adnexal or cervical motion tenderness. The rest of the examination is unremarkable. Laboratory testing reveals a vaginal pH of 5.6; whiff test result is negative. Which of the following is the most appropriate test to confirm the diagnosis? Culture for Gardnerella vaginalis Nucleic acid amplification test for trichomoniasis Potassium hydroxide wet mount study for yeast Saline microscopy for clue cells
Clinical diagnosis of bacterial vaginosis requires three of the following four features: vaginal pH greater than 4.5, thin and homogenous vaginal discharge, positive whiff test result, and clue cells comprising at least 20% of all squamous cells on saline microscopy; culture is not a reasonable test to confirm the diagnosis of bacterial vaginosis and would also be costly and inefficient compared with an office-based diagnosis. Bacterial vaginosis can be diagnosed in this patient with detection of at least 20% clue cells on saline microscopy. Bacterial vaginosis is the most common cause of vaginal discharge and results from an imbalance in the normal vaginal bacterial flora—loss of the normal hydrogen-producing lactobacilli in the vagina and subsequent overgrowth of Gardnerella vaginalis, Mycoplasma species, and other anaerobes. Clinical diagnosis is made when three of the following features are present: vaginal pH greater than 4.5, thin and homogenous vaginal discharge, positive result on a whiff test (application of 10% potassium hydroxide to vaginal secretions resulting in a fishy odor), and clue cells comprising at least 20% of all squamous cells on saline microscopy. Clue cells are vaginal epithelial cells with ill-defined cell borders on microscopy as a result of adherent coccobacilli. The patient has vaginal pH greater than 4.5 and a homogenous thin discharge but a negative whiff test result. A saline wet mount that demonstrates at least 20% clue cells on microscopy will establish the diagnosis of bacterial vaginosis. Treatment is metronidazole or clindamycin. Because bacterial vaginosis represents changes in the vaginal flora, vaginal culture has no role in diagnosis. Although cultures for G. vaginalis will be positive in almost all women with bacterial vaginosis, cultures lack specificity—the organism is found in over 50% of healthy asymptomatic women. Therefore, culture is not a reasonable test to confirm the diagnosis of bacterial vaginosis and would also be costly and inefficient compared with an office-based diagnosis. The nucleic acid amplification test (NAAT) is a highly sensitive test for diagnosis of trichomoniasis and other sexually transmitted infections, such as chlamydia and gonorrhea. Although NAAT can detect G. vaginalis, it is time consuming, expensive, and unnecessary, and simple office-based diagnosis is preferred in straightforward cases such as this one. When office microscopy is not available, NAAT is a reasonable test to perform. Vulvovaginal candidiasis is typically characterized by vaginal itching, irritation, and discharge and may be associated with dysuria and dyspareunia. Examination reveals vulvar edema and excoriation, with thick, white, curdy vaginal discharge. The diagnosis can be made when a saline or 10% potassium hydroxide wet mount of vaginal discharge shows yeast, hyphae, or pseudohyphae. This patient's clinical presentation is not consistent with vaginal candidiasis; a 10% potassium hydroxide wet mount is unnecessary and will not establish the most likely diagnosis.
A 78-year-old man is evaluated in the emergency department. Five days ago he was diagnosed with community-acquired pneumonia at an urgent care center for which he was prescribed clarithromycin. Yesterday he developed generalized weakness and diarrhea. History is also significant for hypertension and gout. He has a history of allopurinol-induced drug rash. Current medications are lisinopril, colchicine, clarithromycin, and febuxostat. On physical examination, temperature is 36.9 °C (98.4 °F), blood pressure is 104/60 mm Hg, pulse rate is 112/min, respiration rate is 18/min, and oxygen saturation is 98% breathing ambient air. There is no rash. The oropharynx appears dry. The chest is clear to auscultation. Tophi are noted at the olecranon processes. There are no swollen or tender joints. Neurologic examination is normal. Laboratory studies: Absolute neutrophil count 1300/µL (1.3 × 109/L) Hemoglobin 9.8 g/dL (98 g/L) Leukocyte count 2700/µL (2.7 × 109/L) Platelet count 96,000/µL (96 × 109/L) Reticulocyte count (corrected) 1.2% Creatine kinase 8433 U/L Creatinine 1.7 mg/dL (150.1 µmol/L) Urinalysis 4+ blood; 0-1 erythrocytes/hpf; 0-2 leukocytes/hpf There are no schistocytes seen on peripheral blood smear. Which of the following is the most appropriate next step in management? Begin high-dose glucocorticoids Begin plasma exchange Stop colchicine and clarithromycin Stop febuxostat
Coadministration of colchicine and CYP3A4 inhibitors (such as clarithromycin and fluconazole) should be avoided because potentially fatal colchicine toxicity with kidney failure, rhabdomyolysis, and bone marrow suppression may occur. Discontinuation of colchicine and clarithromycin is the most appropriate next step in management. The coadministration of colchicine and clarithromycin can result in potentially fatal colchicine toxicity that manifests as rhabdomyolysis, acute kidney injury, and pancytopenia. Colchicine is metabolized in the liver by the CYP3A4 cytochrome and should be avoided in patients taking CYP3A4 inhibitors such as clarithromycin and fluconazole. Coadministration with clarithromycin is particularly concerning, because there have been several case reports of fatal outcomes with the combination (even when taken for a short time). Once the patient recovers, however, colchicine may be reinitiated (as it is the combination of the two drugs that led to the current scenario). High-dose glucocorticoid therapy would be indicated in the event of immune-mediated hemolytic anemia and leukopenia (Evans syndrome). Colchicine toxicity, however, is not immune mediated and would not respond to glucocorticoids. Plasma exchange would be appropriate for thrombotic thrombocytopenic purpura (TTP). Although TTP can cause thrombocytopenia and acute kidney injury, the lack of schistocytes on blood smear argues against microangiopathic hemolytic anemia, and other features of TTP such as fever and mental status changes are also absent. Plasma exchange is not effective for colchicine toxicity. There is no need to discontinue febuxostat; this drug can be continued in patients with chronic kidney disease and would not cause the clinical picture described. In February 2019, the FDA mandated a boxed warning for febuxostat regarding the increased risk for cardiovascular death and all-cause mortality with the drug. The FDA has also limited the approved use of febuxostat for patients who are unresponsive to or cannot tolerate allopurinol.
A 36-year-old man is evaluated for a 3-month history of severely depressed mood; hypersomnia; poor appetite; 6.8-kg (15-lb) weight loss; and loss of interest in family, hobbies, and work. He has not had thoughts of suicide. He has never had similar problems and does not use alcohol, tobacco, or recreational drugs. He wants help but is concerned about the side effects of psychotropic medications. His PHQ-9 score is 13, indicating moderate depression. Laboratory studies reveal a normal serum thyroid-stimulating hormone level. Which of the following is the most appropriate treatment? Amitriptyline Cognitive behavioral therapy Paroxetine Quetiapine
Cognitive behavioral therapy and second-generation antidepressants have proved equally effective for treatment of major depressive disorder, with similar rates of discontinuation; treatment selection should be made after discussion of adverse effect profiles, accessibility, cost, and preferences with the patient. Cognitive behavioral therapy (CBT) is the most appropriate initial therapy for this patient with major depressive disorder. Most patients with mild or moderate depression (PHQ-9 score <15) are treated in the primary care setting. Referral to a psychiatrist is indicated for patients with severe depression, failure of initial therapy, complex psychiatric comorbidities, and high suicide risk. In a 2016 clinical practice guideline, the American College of Physicians recommends that clinicians treat patients with major depressive disorder with CBT or second-generation antidepressants (selective serotonin reuptake inhibitors, serotonin-norepinephrine reuptake inhibitors, serotonin modulators, or atypical antidepressants) after discussing adverse effect profiles, accessibility, cost, and preferences with the patient. On the basis of moderate-quality evidence, CBT and second-generation antidepressants were equally effective treatments for major depressive disorder, with similar rates of discontinuation. For this patient who is opposed to the use of psychotropic medications, CBT is the best initial treatment. Amitriptyline and other tricyclic antidepressants can be used to treat major depressive disorder, but these agents are second-line therapy because of the higher rate of associated side effects. Paroxetine is a second-generation antidepressant that is a reasonable choice for initial management of major depressive disorder. However, there is a particularly high incidence of side effects with paroxetine use. This patient is wary of psychotropic medications, and if one were to be tried, a different second-generation antidepressant with a lower incidence of side effects would be preferred to improve patient adherence. Quetiapine is FDA approved for use in combination with an antidepressant for the treatment of depression that does not respond to initial therapy. First-line treatment has not yet been attempted in this patient, and quetiapine is not appropriate as monotherapy for major depressive disorder.
A 55-year-old man is evaluated for increasing difficulty keeping track of tasks and performing his job adequately. The patient works as an accountant and lately has been making frequent calculation errors. He also has noticed some word-finding difficulties and marked difficulty with short-term memory. Symptoms have progressed over the past 18 months but recently have become more prominent. The patient has a 15-year history of multiple sclerosis and also has depression, which has been in remission. Medications are glatiramer acetate, vitamin D3, and fluoxetine. On physical examination, vital signs are normal. On neurologic examination, the patient can recall only one of three objects at 3 minutes, skips "August" when reciting the months backward, and makes one error when subtracting serial sevens. Which of the following is the most appropriate treatment? Cognitive rehabilitation Increased dosage of fluoxetine Memantine Methylphenidate
Cognitive rehabilitation approaches, such as the development of accommodative strategies and training with challenging cognitive tasks, have been shown to improve symptoms of cognitive deficits in patients with multiple sclerosis. This patient should be referred for cognitive rehabilitation therapy. Cognitive dysfunction occurs in at least 50% of patients with multiple sclerosis (MS). The most common deficits involve short-term memory, processing speed, and executive function. Cognitive disability has a significant effect on the employability of patients with MS and can reduce their overall quality of life. To this point, however, no pharmaceutical agent has been shown to improve these symptoms in patients with MS. In contrast, cognitive rehabilitation approaches, such as the development of accommodative strategies and training with challenging cognitive tasks, have shown this benefit and should be pursued in this patient. Depression also is a common symptom in patients with MS, and the suicide rate is elevated in these patients compared with patients who have depression for other reasons. The depression that occurs in MS is likely multifactorial, involving the emotional response to dealing with a chronic disease, the consequences of demyelinating lesions and inflammatory cytokines, and the adverse effects of treatments (such as the interferon beta formulations). Clinicians should be vigilant for signs of depression and have a low threshold for initiating antidepressants and offering referrals to psychiatry. This patient has not had any severe worsening of depression. Therefore, increasing the dosage of the selective serotonin reuptake inhibitor fluoxetine is unlikely to improve any cognitive deficits. In trials of their effectiveness in improving cognitive deficits in MS, donepezil, memantine, and methylphenidate have shown no benefit. Donepezil has shown modest benefits in cognitive performance in patients with mild to moderate Alzheimer disease, and memantine has shown a similar benefit in patients with moderate to severe Alzheimer disease. Methylphenidate is a central nervous system stimulant used in the treatment of attention deficit hyperactivity disorder and narcolepsy.
A 72-year-old woman is evaluated after being hospitalized for gastrointestinal bleeding. She went to the emergency department after passing a large amount of bright red blood per rectum at home and became lightheaded. She had a second bloody bowel movement in the emergency department. She is otherwise healthy and takes no medication. Findings on physical examination, including vital signs, are normal. Laboratory studies show a hemoglobin level of 9 g/dL (90 g/L). Which of the following is the most appropriate next step in management? Angiography and arterial embolization Colonoscopy within 8 hours with cleansing enemas Colonoscopy within 24 hours with oral bowel preparation Tagged red blood cell scintigraphy Transfusion of packed red blood cells
Colonoscopy is the recommended initial diagnostic test after hemodynamic resuscitation in most patients with significant lower gastrointestinal bleeding. Colonoscopy within 24 hours with adequate bowel preparation is the most appropriate next step in management. Almost 80% of lower gastrointestinal bleeding (LGIB) is due to diverticulosis, colitis, hemorrhoids, or postpolypectomy bleeding. LGIB typically stops within 24 hours. Colonoscopy is the recommended initial diagnostic test after hemodynamic resuscitation in most patients with significant LGIB. LGIB typically occurs in older individuals and presents as acute bright red blood per rectum or red- or maroon-colored stool (hematochezia). Colonoscopy is able to identify the source of bleeding in two thirds of patients. The American College of Gastroenterology's 2016 guidelines for LGIB recommend oral bowel preparation to increase colonoscopy's diagnostic yield. Randomized controlled trials have not shown a benefit in clinical outcomes or cost with rapid bowel preparation and colonoscopy within 8 to 12 hours compared with a standard oral bowel preparation and colonoscopy within 24 hours for patients with LGIB. For patients who continue to bleed and have failed endoscopic hemostasis treatments (for example, electrocoagulation, hemoclips, submucosal epinephrine injection), the next therapeutic step is arterial embolization of the bleeding source. Major complications include contrast dye reactions, acute kidney injury, transient ischemic attack, bowel ischemia, hematoma formation, and femoral artery thrombosis. This patient should be evaluated first with colonoscopy before using a more invasive treatment strategy. Radiographic techniques such as tagged red cell scintigraphy may be useful in evaluating overt gastrointestinal bleeding from an unknown source. Nuclear scans can identify only a general area where bleeding is occurring; they cannot offer accuracy or intervention. Follow-up studies after a positive scan can include repeat endoscopy or angiography; both can offer more accurate localization and therapy. Nuclear scans are often done before angiography. Transfusion strategies specifically for patients with LGIB have not been developed. Data extrapolated from patients with upper gastrointestinal bleeding found that a restrictive transfusion strategy with a transfusion threshold of hemoglobin less than 7 g/dL (70 g/L) improved survival and decreased rebleeding when compared with a threshold of 9 g/dL (90 g/L). Patients with massive bleeding, acute coronary syndrome, symptomatic peripheral vascular disease, or a history of cerebrovascular disease were excluded from these studies and may benefit from a more lenient transfusion strategy.
A 47-year-old woman is evaluated during a follow-up visit for a 2-year history of symptoms diagnosed 4 months ago as fibromyalgia. She reports ongoing widespread pain, fatigue, and difficulty concentrating. She does not have trouble falling asleep, but her sleep is nonrestorative. She was seen 3 months ago, and was given duloxetine and instructed to exercise. The medication produced a modest benefit, but she continues to have difficulty working as a housekeeper. She can only exercise to a limited extent without experiencing disabling pain for the next several days. On a 9-point pain scale, her pain was formerly an 8; after initiation of duloxetine, it decreased to a 5. On physical examination, vital signs are normal. There is tenderness between the shoulder blades and at the occiput, trapezius, elbows, and hips bilaterally. There is no joint swelling. Which of the following is the most appropriate treatment? Add meloxicam Add pregabalin Discontinue duloxetine; start gabapentin Discontinue duloxetine; start sertraline
Combination pharmacologic therapy that takes advantage of complementary mechanisms of action may be helpful in some patients with fibromyalgia.
A 47-year-old woman is evaluated during a follow-up visit for a 2-year history of symptoms diagnosed 4 months ago as fibromyalgia. She reports ongoing widespread pain, fatigue, and difficulty concentrating. She does not have trouble falling asleep, but her sleep is nonrestorative. She was seen 3 months ago, and was given duloxetine and instructed to exercise. The medication produced a modest benefit, but she continues to have difficulty working as a housekeeper. She can only exercise to a limited extent without experiencing disabling pain for the next several days. On a 9-point pain scale, her pain was formerly an 8; after initiation of duloxetine, it decreased to a 5. On physical examination, vital signs are normal. There is tenderness between the shoulder blades and at the occiput, trapezius, elbows, and hips bilaterally. There is no joint swelling. Which of the following is the most appropriate treatment? Add meloxicam Add pregabalin Discontinue duloxetine; start gabapentin Discontinue duloxetine; start sertraline
Combination pharmacologic therapy that takes advantage of complementary mechanisms of action may be helpful in some patients with fibromyalgia. The addition of pregabalin is the most appropriate treatment. This patient's current symptoms and history are consistent with her prior diagnosis of fibromyalgia. Patients with fibromyalgia typically benefit from validation of their symptoms by a physician because many patients have previously had their complaints disregarded by caregivers or family. In addition to psychological support and exercise, pharmacotherapy is a mainstay of fibromyalgia management. Three drugs are FDA approved for fibromyalgia: pregabalin, duloxetine, and milnacipran. Each provides an average benefit of approximately 30% reduction in pain. The latter two agents can also address coexisting mood disorder. This patient has already been appropriately started on duloxetine, and has had significant but insufficient pain improvement. Because patients with fibromyalgia often benefit from combination therapy, the addition of a new treatment, acting through a complementary mechanism, is indicated now for this patient. In contrast to the dual serotonin-norepinephrine reuptake inhibitor duloxetine, which enhances the suppression of pain signals, pregabalin inhibits glutamate release to directly reduce the passage of pain signals from the dorsal root ganglion. The combination is frequently of benefit. Meloxicam is a long-acting NSAID that has better gastrointestinal tolerability than some other NSAIDs. However, NSAIDs have not shown reliable benefit for fibromyalgia pain. Gabapentin shares its mechanism of action with pregabalin; although its pharmacokinetics are less reliable than pregabalin, and it is not FDA approved for this indication, its use in fibromyalgia is accepted; however, the discontinuation of duloxetine would be inadvisable. Although dual serotonin-norepinephrine reuptake inhibitors are beneficial in fibromyalgia, selective serotonin reuptake inhibitors such as sertraline have not shown any benefit when used as single agents.
A 44-year-old man is evaluated during a follow-up visit for treatment of persistently elevated blood pressure. He takes no medications. Physical examination reveals a well-developed muscular man in no apparent distress. Blood pressure is 165/98 mm Hg, and pulse rate is 70/min; other vital signs are normal. BMI is 26. Jugular venous pressure is normal. Cardiac examination is unremarkable. Laboratory studies: Bicarbonate 27 mEq/L (27 mmol/L) Creatinine 1.3 mg/dL (114.9 µmol/L) Potassium 4.5 mEq/L (4.5 mmol/L) Estimated glomerular filtration rate >60 mL/min/1.73 m2 Urine toxicology screen Negative Electrocardiogram reveals normal sinus rhythm; voltage criteria for left ventricular hypertrophy are present. Which of the following is the most appropriate treatment? Amlodipine/benazepril combination once daily Doxazosin and metoprolol, each once daily Hydralazine three times daily Telmisartan and ramipril, each once daily
Combination therapy with amlodipine/benazepril is appropriate treatment for this patient with stage 2 hypertension. The 2017 American College of Cardiology/American Heart Association blood pressure (BP) guideline recommends combination therapy with two first-line antihypertensive drugs of different classes (separately or as a single-dose pill) for adults with stage 2 hypertension and an average BP that is 20/10 mm Hg above their BP target. Stage 2 hypertension is defined as BP ≥140/90 mm Hg. This patient's target BP is <130/80 mm Hg. There are no definitive recommendations for best drug combinations, but some data suggest an ACE inhibitor/calcium channel blocker (CCB) combination may be more efficacious than an ACE inhibitor/thiazide diuretic combination. Using a thiazide diuretic/CCB combination is also an option. It is important to note that in black patients, ACE inhibitors are not as efficacious at reducing BP compared with thiazide diuretics or CCBs. Specific non-recommended initial agents for the treatment of hypertension include β-blockers (due to higher rate of cardiovascular-related events and mortality compared with angiotensin receptor blockers) and α-blockers (due to higher rate of cardiovascular-related events and mortality compared with thiazides), although compelling clinical indications such as atrial fibrillation or benign prostatic hyperplasia may supersede these recommended restrictions. Initiation of once-daily dosing of a combination pill simplifies the medication regimen and is appropriate in this patient to ensure adherence. Therefore, initiation of hydralazine is not correct because two-drug therapy is recommended for this patient, and adherence to a thrice-daily medication will be problematic. Combination therapy with an ACE inhibitor (ramipril) and an angiotensin receptor blocker (telmisartan) is not recommended. Several clinical trials (ONTARGET, NEPHRON-D, ALTITUDE) have revealed more adverse events with these combinations (hyperkalemia, hypotension, acute kidney injury), without additional cardiovascular or renal benefits. Read Related TextNext Question
A 77-year-old man is evaluated for a 6-month history of worsening urinary frequency and nocturia. He takes no medications. On physical examination, vital signs are normal. BMI is 22. Digital rectal examination reveals a firm, enlarged prostate. Laboratory studies reveal a serum prostate-specific antigen level of 18.5 ng/mL (18.5 µg/L). Biopsy of the prostate reveals adenocarcinoma with bilateral gland involvement; the Gleason score is 8. MRI of the pelvis shows evidence of focal extension beyond the prostate capsule. Bone and CT scans are negative for metastatic disease. Which of the following is the most appropriate management? Brachytherapy Gonadotropin-releasing hormone agonist Gonadotropin-releasing hormone agonist plus radiation Radiation
Combination therapy with radiation and a gonadotropin-releasing hormone agonist for up to 2 to 3 years results in an improvement in 10-year disease-free and overall survival and is the standard care for patients with high-risk and very-high-risk prostate cancer. The most appropriate treatment is a gonadotropin-releasing hormone (GnRH) agonist plus radiation. High-risk prostate cancer is defined as a prostate-specific antigen (PSA) level greater than 20 ng/mL (20 µg/L), a Gleason score of 8 to 10, or evidence of extraprostatic extension of the cancer. Imaging studies, such as pelvic CT or MRI, can be used to assess regional lymph node enlargement in selected patients. Imaging is reserved for patients with a PSA level greater 20 ng/mL (20 µg/L), a Gleason score 8 or greater, and other factors. Patients with high-risk prostate cancer have a significant risk of eventually developing metastatic disease, and they all require treatment unless significant medical comorbidity precludes treatment. In this patient population, there is significant risk of both local recurrence and distant metastatic disease. Studies have shown that the addition of androgen deprivation therapy (most commonly with a GnRH agonist, such as leuprolide) to radiation results in an improvement in 10-year disease-free and overall survival compared with radiation alone in men with high-risk prostate cancer. Based on this data, the combination of radiation with a GnRH agonist for up to 2 to 3 years is the standard care for this patient population. Brachytherapy is not indicated in patients with high-risk prostate cancer because no evidence supports the efficacy of brachytherapy in this setting. Brachytherapy is typically only recommended for men with low-risk and limited-volume cancer. Use of a GnRH agonist alone has no role in the treatment of nonmetastatic prostate cancer. In this patient, local treatment (with either radiation or surgery) is definitively indicated, and treatment with a GnRH agonist alone provides no local treatment. Radiation alone is not recommended given the survival advantage associated with combined GnRH agonist and radiation treatment in men with high-risk and very-high-risk prostate cancer.
A 49-year-old man is evaluated for a 1-month history of a painless enlarging left neck mass. His medical history is unremarkable, and he takes no medications. On physical examination, vital signs are normal. BMI is 26. On palpation, there is a 2-cm left anterior cervical lymph node near the angle of the mandible. Laryngoscopy shows a left-sided tonsil cancer. Biopsy of the tonsil reveals moderately differentiated squamous cell carcinoma that is positive for p16 (human papillomavirus). CT scan of the neck reveals a cystic left-sided cervical lymph node; a left tonsil mass is noted. CT scan of the chest is negative. Postoperative pathology reveals a 2.5-cm tonsil cancer with negative margins and 3/26 positive cervical nodes, one with extracapsular extension. Which of the following is the most appropriate treatment? Cisplatin alone Cisplatin plus human papillomavirus vaccine Cisplatin plus radiation Radiation alone
Combined chemotherapy and radiation improves survival in patients with locally advanced head and neck cancer after surgery when there are positive surgical margins or evidence of nodal metastases with extracapsular extension. The most appropriate treatment is cisplatin plus radiation. Treatment of head and neck cancer in the adjuvant setting depends heavily on the results of pathologic analysis of the resection specimen. This should include assessment of human papillomavirus (HPV) status based on tumor staining for p16, a viral protein. HPV has been found to be responsible for most oropharynx cancers (tonsil and base of tongue). Most patients diagnosed with HPV-associated head and neck cancer are younger than expected and do not have traditional risk factors, such as the patient in this case. Positivity for HPV has a significant impact on prognosis but does not yet affect treatment decision making. Options for treatment include no adjuvant therapy, radiation alone, and combined chemotherapy and radiation. No adjuvant therapy is required for early-stage cancers with no high-risk pathologic features, such as poorly differentiated histology and lymphovascular invasion. For early-stage cancers with high-risk features and for some locally advanced cancers (specifically any node-positive cancer), radiation alone is appropriate. Combined chemotherapy and radiation has been shown to improve survival in patients with locally advanced cancer after surgery when there are positive surgical margins or evidence of nodal metastases with extracapsular extension. Extracapsular extension refers to squamous cell carcinoma that breaks through the outer capsule of the lymph node and extends into surrounding tissues. Notably, the number of lymph nodes involved has not been clearly shown to be a predictor of benefit with combined chemotherapy and radiation adjuvant therapy. This patient underwent surgery and had evidence of extracapsular extension; therefore, combined cisplatin and radiation is the most appropriate adjuvant treatment. Chemotherapy alone, either with single-agent cisplatin or cisplatin-based combination chemotherapy, is not indicated. Adjuvant chemotherapy has not been shown to improve survival or other treatment outcomes in this setting. Vaccination of children before sexual activity is important in the primary prevention of HPV infection and the reduction of the incidence of oropharyngeal, cervical, and anal cancers associated with this virus. Immunization plays no role in patients who are already infected with HPV. Radiation alone is not indicated owing to the presence of extracapsular extension.
A 70-year-old woman is evaluated in the emergency department for dyspnea. For the past 5 days, she has had progressive shortness of breath associated with a nonproductive cough. Symptoms are greatest with exertion and possibly when lying down. She has had no fever, chest pain, or increase in her chronic lower extremity edema. Her other medical problems are chronic venous insufficiency, COPD, and hypertension. Medications are chlorthalidone, amlodipine, lisinopril, and a tiotropium inhaler. On physical examination, she is afebrile, blood pressure is 144/88 mm Hg, pulse rate is 90/min, and respiration rate is 22/min. Oxygen saturation is 94% breathing ambient air. BMI is 36. Because her neck is large, it is not possible to estimate central venous pressure. Breath sounds are distant, with occasional end-expiratory wheezing. Heart sounds are distant, and extra sounds or murmurs are not detected. There is 2+ ankle edema with hyperpigmentation localized to the medial aspect of the ankles. B-type natriuretic peptide level is 128 pg/mL (128 ng/L). A chest radiograph shows increased radiolucency of the lung, flat diaphragms, and a narrow heart shadow consistent with COPD. An electrocardiogram shows evidence of left ventricular hypertrophy. This patient's B-type natriuretic peptide level is most consistent with which of the following causes of dyspnea? Acute heart failure Chronic heart failure Noncardiac disease Cannot be determined
Common factors other than ventricular wall stress that influence B-type natriuretic peptide (BNP) levels include kidney failure, older age, and female sex, all of which increase BNP levels; obesity reduces BNP levels. The cause of this patient's dyspnea is indeterminate on the basis of the B-type natriuretic peptide (BNP) level of 128 pg/mL (128 ng/L). In the Breathing Not Properly study of patients who presented to the emergency department with dyspnea, patients with heart failure had a mean BNP level greater than 600 pg/mL (600 ng/L), whereas those with noncardiac causes of dyspnea had levels of approximately 50 pg/mL (50 ng/L). Patients with a history of left ventricular dysfunction but not an acute exacerbation had a BNP level of approximately 200 pg/mL (200 ng/L). In a pooled analysis of 10 studies, BNP values less than 100 pg/mL (100 ng/L) excluded heart failure (90% sensitivity) and greater than 400 pg/mL (400 ng/L) supported the diagnosis of heart failure (74% specificity). BNP levels increase with age and worsening kidney function and are reduced in patients with an elevated BMI. BNP values less than 55 pg/mL (55 ng/L) are likely to rule out heart failure in patients with BMI greater than 35 (sensitivity 90%). In these patients, BNP levels greater than or equal to 170 pg/mL (170 ng/L) are 70% specific for the diagnosis of acute heart failure. This patient's BNP level is not helpful in ruling in or ruling out cardiac disease as the cause of her dyspnea, and further evaluation is required.
A 42-year-old woman is evaluated in the office after care in the emergency department for a new-onset seizure. She has a history of malignant melanoma resected 7 years ago. It had a 3.2-mm depth of invasion by Breslow microstaging. Her only medications are dexamethasone and levetiracetam. On physical examination, vital signs are normal. There is no lymphadenopathy. Results of the neurologic examination are normal. Brain MRI shows a 3.5-cm left frontal lesion consistent with metastatic disease. CT scan of the chest, abdomen, and pelvis is negative. Which of the following is the most appropriate treatment? Ipilimumab and nivolumab Stereotactic radiosurgery to the brain lesion Surgical resection of the brain lesion Vemurafenib plus cobimetinib Whole brain radiation therapy
Melanoma may be associated with solitary or oligometastatic disease, and metastasectomy will prolong survival and can be curative in these patients. Surgical resection of the brain lesion in this patient with previous melanoma would be the most appropriate treatment. Assuming the lesion is surgically accessible, resection would provide a tissue diagnosis confirming the suspicion of metastatic melanoma, provide the best long-term local control, and offer the best potential for cure. Postoperative radiosurgery to the tumor bed may be considered to reduce the risk for recurrence. Melanoma may be associated with solitary or oligometastatic disease, and metastasectomy can be curative in a significant minority of these patients. Metastasectomy with or without systemic medical therapy is associated with increased overall survival compared to systemic medical therapy alone in patients with stage IV melanoma. Based on a cohort analysis of data from a randomized clinical trial, surgery was associated with a median overall survival of 15.8 months compared to 6.9 months with systemic therapy alone. The 4-year overall survival also favored the surgery group, 20.8% versus 7% in the group receiving systemic therapy alone. Immunotherapy with nivolumab, pembrolizumab, or ipilimumab plus nivolumab is used in the treatment of metastatic melanoma but would not be recommended as first-line therapy if there is a resectable isolated metastasis, as in this patient. Gamma knife or other forms of radiosurgery can provide high rates of local control while minimizing radiation exposure to the rest of the brain. With a large mass, however, surgical resection would provide a better probability of long-term control. Treatment with BRAF/MEK inhibitor combination therapy, such as dabrafenib plus trametinib or vemurafenib plus cobimetinib, is effective for BRAF-mutated melanoma and can target disease in the brain, but these agents would not be preferred as initial therapy here because they are less effective than metastasectomy in increasing survival and offering a chance of cure. Whole brain radiation therapy is reserved for patients with multiple metastases that are not amenable to focal therapies (surgery or radiosurgery). Although whole brain radiation therapy does decrease the risk of newer metastases, it is associated with an increased risk of late neurologic dysfunction.
A 32-year-old woman is evaluated for a 10-day history of severe cough with increased sputum production, fever, wheezing, and dyspnea. She has a 12-year history of recurrent abdominal pain with watery stools and poor weight gain, recurrent sinusitis, and chronic cough that is productive of foul sputum. She takes no medications chronically. On physical examination, temperature is 38.2 °C (100.8 °F), blood pressure is 92/64 mm Hg, pulse rate is 101/min, and respiration rate is 24/min. Oxygen saturation is 91% on ambient air. BMI is 18.2. Pulmonary examination reveals wheezes. Additional findings include a scaphoid abdomen and clubbing. Laboratory studies reveal a leukocyte count of 16,000/µL (16 × 109/L), hemoglobin of 10 g/dL (100 g/L), and serum creatinine of 0.7 mg/dL (61.9 µmol/L). Serum immunoglobulin levels are normal. Pulmonary function testing reveals an FVC of 80% of predicted, an FEV1 of 50% of predicted, and an FEV1/FVC ratio of 0.55. Chest radiograph is shown. Which of the following is the most likely underlying diagnosis? Common variable immunodeficiency Complement component deficiency COPD Cystic fibrosis
Conditions suggesting the diagnosis of cystic fibrosis in adults include chronic asthma-like symptoms, chronic sinusitis, nasal polyposis, recurrent pancreatitis, infertility, and bronchiectasis. The most likely underlying diagnosis is cystic fibrosis. This patient has acute symptoms of increased cough, sputum production, fever, chills, wheezing, dyspnea, clubbing, and a chest radiograph (shown) with bilateral upper-lobe predominant bronchiectasis with mucoid impaction (arrows), as well as a history of chronic pulmonary and gastrointestinal disease. This constellation of signs and symptoms is most consistent with an acute exacerbation of bronchiectasis in a patient with cystic fibrosis. Common variable immunodeficiency involves B- and T-cell abnormalities and results in clinically significant immune dysregulation. The primary manifestation is hypogammaglobulinemia, and adults present with recurrent respiratory infections. The gastrointestinal tract is frequently involved with malabsorption or chronic diarrhea. The diagnosis is made by confirming low levels of total IgG and IgA or IgM, as well as by a poor antibody response to vaccines. The patient's normal serum immunoglobulin levels exclude this diagnosis. The most common of the early complement disorders is C2 deficiency; C6 deficiency is the most common of the late complement disorders. Patients lacking one of the early components usually present with a rheumatologic disorder. Those with late complement component deficiencies usually present with recurrent, invasive meningococcal or gonococcal infections. The patient's history and findings are not consistent with either early or late complement component deficiency. Although the patient has obstruction on pulmonary function testing, her age and history of chronic sinus and gastrointestinal disease make a diagnosis of COPD less likely than cystic fibrosis.
A 72-year-old man is evaluated for a 1-year history of bilateral lower extremity edema and abdominal distention. Eight years ago he had esophageal carcinoma treated with radiotherapy. Medical history is otherwise significant for hypertension, type 2 diabetes mellitus, and hypercholesterolemia. Medications are bumetanide, atorvastatin, metformin, and lisinopril. On physical examination, the patient is afebrile, blood pressure is 170/90 mm Hg, pulse rate is 90/min, and respiration rate is normal. Jugular venous distention is present to the angle of the mandible while seated, with prominent pulsations. Cardiac examination reveals an early diastolic sound at the apex. The liver is palpable 5 cm below the costal margin. The abdomen is distended with ascites. There is bilateral pitting edema to the level of the thighs. Pulmonary examination reveals no crackles. Laboratory studies are significant for a B-type natriuretic peptide level of 96 pg/mL (96 ng/L). Chest radiographs are shown. A 12-lead electrocardiogram demonstrates normal sinus rhythm with normal QRS voltage. Echocardiogram demonstrates normal right and left ventricular size and function. Left ventricular wall thickness is normal. Mild tricuspid regurgitation is present. There is respiratory variation in the filling of the right and left ventricles, ventricular septal shift during respiration, and dilation of the inferior vena cava. The estimated right ventricular systolic pressure is 46 mm Hg. There is no pericardial effusion. Which of the following is the most likely diagnosis? Cardiac amyloidosis Constrictive pericarditis Heart failure with preserved ejection fraction Restrictive cardiomyopathy
Constrictive pericarditis is commonly characterized by symptoms of right-sided heart failure, low or normal B-type natriuretic peptide level, and the finding of pericardial thickening or calcification on imaging studies. The most likely diagnosis in this patient with a history of chest radiotherapy is constrictive pericarditis. Constrictive pericarditis is characterized by pericardial thickening, fibrosis, and sometimes calcification that impair diastolic filling and limit total cardiac volume. Most cases are viral or idiopathic in origin; however, cardiac surgery, chest irradiation, autoimmune disease, and tuberculosis or other bacterial infections may be causes. Patients with constrictive pericarditis most commonly present with indolent progression of right-sided heart failure symptoms, including peripheral edema, ascites, and fatigue. In this patient, signs of venous congestion predominate, pericardial calcification is present on the chest radiograph, and the serum B-natriuretic peptide level is low and does not suggest greatly increased wall tension. The echocardiographic findings of respiratory variation in filling of right and left ventricles, ventricular septal shift during respiration, and dilation of the inferior vena cava are characteristic of constrictive pericarditis. Cardiac amyloidosis and restrictive cardiomyopathy are not associated with pericardial calcification, and patients with these conditions more commonly present with moderate to severe pulmonary hypertension rather than the mild pulmonary hypertension seen in this patient. Respiratory variation in filling of the right and left ventricles and ventricular septal shift during respiration are characteristic of pericardial constraint rather than myocardial disease. Patients with heart failure with preserved ejection fraction will present with edema, exertional dyspnea, and fatigue. Normal systolic contraction and abnormal diastolic relaxation are present on echocardiogram and result in restricted filling and high filling pressures. Patients with heart failure typically have an elevated B-type natriuretic peptide level, often greater than 600 pg/mL (600 ng/L), which is not found in this patient.
A 19-year-old man is evaluated during a follow-up evaluation of his type 1 diabetes mellitus. He was diagnosed 4 months ago with symptoms of hyperglycemia. His hemoglobin A1c level at diagnosis was 11.1%, and antibodies to glutamic acid decarboxylase (GAD65) were positive. He was begun on prandial and basal insulin. He now reports progressive improvement in his glycemic control over the last 8 weeks without changes to his diet, activity level, or insulin doses. Data from his glucometer demonstrates an average fasting, preprandial, and bedtime blood glucose level of 80 mg/dL (4.4 mmol/L). He has several postprandial blood glucose values of approximately 60 mg/dL (3.3 mmol/L) associated with hypoglycemic symptoms. His current hemoglobin A1c level is 5.0%. Medications are insulin glargine (8 U) and insulin aspart (2 U before meals). His physical examination is notable for an increase in BMI from 18 at the time of his diabetes diagnosis, to now at 20. Vital signs and the remainder of the physical examination are normal. Which of the following is the most appropriate management of this patient's diabetes? Continue insulin glargine dose, decrease insulin aspart dose Decrease insulin glargine dose, discontinue insulin aspart Discontinue insulin glargine, discontinue insulin aspart Discontinue insulin glargine, discontinue insulin aspart, add sliding-scale insulin regimen
Continuing insulin, even at low doses, is recommended during the "honeymoon phase" of type 1 diabetes mellitus to reduce metabolic stress on functioning beta cells and preserve any residual function for as long as possible. The most appropriate management of this patient's diabetes is to decrease the insulin glargine dose and discontinue insulin aspart. The drastic reduction in endogenous insulin production secondary to pancreatic beta cell destruction in type 1 diabetes creates a glucose toxicity that induces a functional impairment of the remaining beta cells. As exogenous insulin therapy improves glycemic control, the remaining beta cells experience less metabolic stress, resulting in an improvement in the ability to produce insulin. This "honeymoon phase" may occur shortly after the diagnosis of diabetes and may last months to years. It is characterized by drastic improvements in glycemic control and reductions in insulin requirements, as seen in this patient. To prevent rapid return of glucose toxicity and to preserve the remaining beta cells as long as possible, insulin therapy should be continued during the "honeymoon phase" if possible without causing hypoglycemia. It is appropriate to decrease this patient's basal glargine insulin dose to improve his fasting hypoglycemia while also maintaining continuous insulin therapy. Given the symptomatic postprandial hypoglycemia he is experiencing on low doses of prandial insulin, it is appropriate to discontinue it at this time with close monitoring for postprandial hyperglycemia at the end of the "honeymoon phase." Sole use of a sliding-scale insulin regimen is not recommended for glycemic control as it is reactionary in nature to elevated glucose values only. Using this strategy, it would be possible that the patient may not receive daily insulin during the "honeymoon phase," which would accelerate the risk of developing glucose toxicity again.
A 24-year-old man is evaluated for increasing left buttock pain that worsens over the course of the day. It is improved by exercise and does not improve with rest. The pain had a gradual onset 1 year ago without a clear precipitant. At that time, radiographs were unremarkable, and physical therapy provided no benefit. He has stopped exercising because of the increasing discomfort. Family history includes ankylosing spondylitis in his father. The patient takes no medications. On physical examination, vital signs are normal. Normal range of motion of the lumbar spine is noted. There is no reduction in lateral bending and no limitation of cervical spine motion in any plane. There are no warm, erythematous, or swollen joints. Laboratory studies show the presence of HLA-B27; normal complete blood count and C-reactive protein; and negative rheumatoid factor and anti-cyclic citrullinated peptide antibodies. Radiographs of the lumbar spine are normal. Which of the following is the most appropriate treatment? Diclofenac Etanercept Methotrexate Sulfasalazine
Continuous full-dose NSAIDs are first-line therapy for ankylosing spondylitis. The most appropriate treatment is an NSAID such as diclofenac for this patient with ankylosing spondylitis, a chronic inflammatory disease affecting the axial skeleton (including sacroiliac joints), entheses, and peripheral joints. It has a strong familial predilection, the strongest association with HLA-B27 among the forms of spondyloarthritis, and a male predominance. This man's history is suggestive of ankylosing spondylitis, with symptoms of inflammatory back pain, the presence of HLA-B27, a family history of ankylosing spondylitis, and a change in his exercise routine. He is early in the course of his disease and has not developed plain radiographic or laboratory abnormalities. Continuous full-dose NSAIDs are first-line therapy and can help relieve pain and stiffness. Studies of continuous full-dose NSAIDs demonstrate symptomatic relief as well as reduced sacroiliac and spine inflammation as seen on MRI in some patients. In a young man with no medical comorbidities, they are associated with a low incidence of side effects. Patients with ankylosing spondylitis are more likely to respond to NSAIDs and do so more rapidly and completely than patients with chronic low back pain from other causes. Results from most randomized controlled trials of methotrexate use in ankylosing spondylitis have shown no benefit, and it is therefore not appropriate in this patient. A considerable amount of more recent data suggests that tumor necrosis factor (TNF)-α inhibitors can be particularly helpful in the management of symptoms of ankylosing spondylitis. However, NSAIDs remain first-line therapy. There is variability in the expression of disease, and some patients can be managed with complete symptom relief without immunosuppression. The efficacy of TNF-α inhibitors for symptom relief needs to be weighed against their cost, potential side effects, and the fact that these agents have not been definitively established to be disease modifying. Sulfasalazine has historically been used in the treatment of ankylosing spondylitis, but its use as second-line therapy has declined as more effective agents, such as TNF-α inhibitors, have emerged. This patient's primary site of involvement is the spine, and many experts now relegate the use of sulfasalazine for treatment of the peripheral arthritis associated with ankylosing spondylitis and only if NSAIDs are ineffective and a TNF-α inhibitor is not available or affordable.
A 27-year-old woman is evaluated in the emergency department after a sudden, first-time episode of loss of consciousness while standing in line to board a tour bus. She had warning symptoms of tunnel vision and palpitations, after which she lost consciousness and fell. According to her father, who witnessed the episode, she was limp and unconscious for approximately 20 to 30 seconds, during which time she displayed intermittent twitching of all four limbs, with the limbs shaking independently at separate times. After the patient regained consciousness, she was confused about why she was on the ground but answered questions appropriately and was oriented to self and place. All physical examination findings are normal. Which of the following is the most likely diagnosis? Atonic seizure Convulsive syncope Generalized tonic-clonic seizure Myoclonic seizure Tonic seizure
Convulsive syncope is a seizure type typically associated with tunnel vision, palpitations, short duration of loss of consciousness (<1 minute), movements and shaking, and immediate and complete neurologic recovery. The patient most likely had an episode of convulsive syncope. Syncope is nontraumatic, complete transient loss of consciousness and loss of postural tone. Onset is abrupt and recovery is spontaneous, rapid, and complete. Neurally mediated syncope, the most common type of syncope, generally occurs with standing and is associated with a prodrome of nausea, lightheadedness, and warmth. It may follow cough, urination, defecation, pain, or laughing. This patient's tunnel vision, palpitations, short duration of loss of consciousness (<1 minute), and immediate and complete neurologic recovery is typical of syncope. Movements and shaking (in this instance, nonepileptic myoclonus) are common with syncope. In fact, syncope without any movements is the exception rather than the rule. Atonic seizures can be associated with falling and brief loss of consciousness with decreased tone, but their duration is much briefer (a few seconds only) than that of syncope, and they usually do not have warning symptoms, such as tunnel vision or palpitations. Generalized tonic-clonic seizures have increased tone at the onset (tonic phase), followed by rhythmic, synchronous jerking of all limbs (clonic phase), typically for more than 1 minute. These seizures are followed by confusion, lethargy, and (sometimes) combativeness. This patient's seizure, especially her normal neurologic state after regaining consciousness, does not match this description. Myoclonic seizures are very brief, typically lasting less than 1 second, and usually involve synchronous limb jerking (all limbs shaking together at the same time). The duration of this patient's seizure and her asynchronous limb jerking are not consistent with myoclonic seizure. Tonic seizures may involve brief loss of consciousness and falling, but there is no associated aura (prodrome or warning symptoms). Additionally, patients generally have increased tone that leads to the fall.
A 60-year-old man is evaluated in the hospital for a 2-day history of intermittent chest pain and dyspnea on exertion. Medical history is significant for type 2 diabetes mellitus, hypertension, hyperlipidemia, COPD, and peripheral neuropathy. His ability to exercise is limited by his COPD. Medications are metformin, simvastatin, low-dose aspirin, lisinopril, amlodipine, and an albuterol-ipratropium inhaler. On physical examination, temperature is normal, blood pressure is 128/78 mm Hg, pulse rate is 80/min, and respiration rate is 16/min. Oxygen saturation is 94% breathing ambient air. Pulmonary examination reveals expiratory wheezing bilaterally. Heart sounds are distant. No edema is present. Serial serum troponin I measurements are negative. An electrocardiogram demonstrates left ventricular hypertrophy with repolarization abnormalities. Which of the following is the most appropriate diagnostic test to perform next? Adenosine single-photon emission CT Coronary angiography Coronary CT angiography Exercise electrocardiography
Coronary CT angiography (CTA) is the most appropriate diagnostic test to perform next. This patient with an intermediate pretest probability of coronary artery disease (CAD) has chest pain without evidence of acute coronary syndrome, and he should undergo risk stratification. Because his baseline electrocardiogram (ECG) has evidence of left ventricular hypertrophy with repolarization abnormalities, which limits the ability to interpret exercise ECG findings, stress testing with adjunctive imaging or anatomic assessment for CAD is indicated. Coronary CTA is a noninvasive anatomic imaging study to evaluate for obstructive CAD. In the PROMISE trial, which compared coronary CTA with functional stress testing among patients with an intermediate pretest probability for CAD, the 2-year composite outcome (death, myocardial infarction, hospitalization for unstable angina, or major procedural complication) was similarly low with both types of diagnostic testing. Vasodilators, such as regadenoson or adenosine, produce hyperemia and a flow disparity between myocardium supplied by unobstructed vessels and myocardium supplied by the stenotic vessel (in which the distal vasculature is already maximally dilated). In addition to identifying the presence of disease, perfusion imaging can define the location and extent of reduced perfusion and provide additional prognostic information. Single-photon emission CT with the vasodilator adenosine can be performed in patients with underlying reactive airways disease but should be avoided in patients who are actively wheezing, such as this one. The newer selective adenosine A2A receptor agonists are associated with less bronchospasm; however, bronchospasm may still occur as a result of some activation of the A2B and A3 receptors. Coronary angiography is an invasive test that carries significant risks. It should be considered in patients who have a high pretest probability for obstructive CAD, including symptomatic patients with abnormal findings on noninvasive testing for CAD or patients with an acute coronary syndrome.
How to diagnose cough variant asthma?
Cough-variant asthma refers to asthma in which the predominant manifestation is cough, and without other typical asthma symptoms; the diagnosis is supported by abnormal spirometry or methacholine challenge testing if spirometry is normal. The most appropriate management is to perform methacholine challenge testing. This patient has a chronic cough with no cause identified by history or physical examination, and a normal chest radiograph. In such patients who are not taking ACE inhibitors and are not exposed to environmental irritants or tobacco smoke, the most common causes are asthma, gastroesophageal reflux disease (GERD), and rhinosinusitis. Cough-variant asthma refers to asthma in which the predominant manifestation is cough, without other typical asthma symptoms such as wheezing, breathlessness, and chest tightness. Although most patients with asthma have obstructive physiology on pulmonary function testing, in those patients with normal spirometry, methacholine challenge testing is indicated to evaluate for bronchial hyperreactivity, which supports a diagnosis of asthma. Bronchial challenge testing uses a controlled inhaled stimulus to induce bronchospasm in association with spirometry; a positive test is indicated by a drop in the measured FEV1. Methacholine is a commonly used agent that induces cholinergic bronchospasm at low concentrations in patients with asthma; levels of exhaled nitric oxide may also be elevated. Positive methacholine testing is not specific enough to diagnose asthma; therefore, patients with cough and a positive methacholine challenge must also respond clinically to treatment with asthma therapies to be considered to have cough-variant asthma.
A 47-year-old woman is evaluated in the hospital for a 4-month history of increasingly frequent falls and episodes of confusion. She was fired from her job 3 months ago because of poor performance. Since that time, she also has had occasional anxiety, problems sleeping, and visual hallucinations. For the past 2 weeks, she has had repeated falls, exhibited erratic behavior, and been unable to prepare even a cup of coffee. On physical examination, vital signs are normal. The patient is agitated and disoriented to place and time. She exhibits word-finding difficulty and intermittent myoclonic movements of the arms and legs. She scores 8/30 (normal, ≥24) on the Mini-Mental State Examination. Results of laboratory studies are normal, including cerebrospinal fluid findings of a normal leukocyte count and normal glucose and protein levels. An electroencephalogram shows periodic sharp wave complexes. Diffusion-weighted brain MRIs show high-signal changes in the basal ganglia and cerebral cortex. Which of the following is the most likely diagnosis? Creutzfeldt-Jakob disease HIV encephalopathy Lyme disease Wernicke encephalopathy
Creutzfeldt-Jakob disease is a transmissible prion-related disorder characterized by rapidly progressive dementia, myoclonus, sleep problems, and other psychiatric symptoms; typical imaging findings include an increased signal in the cortex and subcortical structures on diffusion-weighted MRI sequences. This patient most likely has Creutzfeldt-Jakob disease (CJD). CJD is the most common form of prion disease in humans, with most cases being sporadic. Iatrogenic transmission of CJD is possible and has resulted mainly from receipt of growth hormone prepared from cadaveric pituitaries and contaminated cadaveric dura mater allografts. Contaminated surgical instruments have also been documented to transmit CJD in rare instances. Sporadic CJD does not appear to be transmissible by blood. Brain MRIs typically show a pattern of increased intensity in the diffusion-weighted sequence in the basal ganglia and various cortical regions. Periodic sharp wave complexes are often seen on electroencephalography. Elevated levels of tau and 14-3-3 proteins in the cerebrospinal fluid (CSF) also are typical but not necessary for diagnosis. Although rapidly progressive dementia has a broad differential diagnosis, CJD is very likely if it is accompanied by myoclonus, sleep problems, and other psychiatric symptoms. Progressive neurologic decline resulting in death occurs rapidly, typically within 6 to 12 months. HIV encephalopathy is characterized by a subacute syndrome of cognitive and motor dysfunction. Although HIV screening is standard for rapid-onset dementia, the electroencephalographic and MRI findings are inconsistent with a diagnosis of acute HIV encephalopathy. Lyme disease can be responsible for a seldom-reported inflammatory encephalomyelitis. The disease often mimics multiple sclerosis, with MRI abnormalities of the brain or spinal cord and abnormal CSF findings, including increased total immunoglobulin and oligoclonal bands. The patient's rapidly progressive dementia, myoclonus, psychiatric symptoms, and normal CSF findings are not compatible with this diagnosis. Wernicke encephalopathy is an acute neuropsychiatric syndrome characterized by the triad of nystagmus and ophthalmoplegia, mental status changes, and ataxia. It is caused by thiamine deficiency, most often in patients who abuse alcohol, but can be seen in patients being treated for cancer, those with chronic vomiting or malnutrition, or those who have had gastrointestinal surgery, in particular bariatric surgery. The patient's findings are not compatible with Wernicke encephalopathy.
What's in blood products: -Cryoprecipitate -K-centra -Fresh frozen plasma
Cryo- fibrinogen (factor I), factor VIII, factor XIII, von Willebrand factor, and fibronectin. K-centra- all 4 vitamin-K dependent coagulation factors (II, VII, IX and X), and the antithrombotic Proteins C and S Fresh frozen plasma All of the clotting factors, fibrinogen (400 to 900 mg/unit), plasma proteins (particularly albumin), electrolytes, physiological anticoagulants (protein C, protein S, antithrombin, tissue factor pathway inhibitor) and added anticoagulants
A 62-year-old woman is evaluated during a follow-up visit for recently diagnosed stage IIIA high-grade serous ovarian cancer. She underwent total abdominal hysterectomy and bilateral salpingo-oophorectomy and completed six cycles of chemotherapy with cisplatin and paclitaxel. The patient's paternal aunt was diagnosed with breast cancer at age 52 years. There is no personal or family history of any other cancers. She takes no medications. On physical examination, vital signs and the remainder of the examination are normal. Laboratory studies reveal a serum CA-125 value of 14 U/mL (14 kU/L) after chemotherapy (383 U/mL [383 kU/L] at diagnosis). Which of the following is the most appropriate test to perform next? Chest radiography annually CT of the abdomen and pelvis in 3 months Genetic testing for BRCA1 and BRCA2 mutations Genetic testing for Lynch mutations
Current guidelines recommend BRCA1 and BRCA2 genetic testing for all women with epithelial ovarian cancer, regardless of age of onset, family history, or ancestry. This patient should be referred to a genetic counselor for genetic testing for BRCA1 and BRCA2 mutations. Current guidelines recommend BRCA1 and BRCA2 genetic testing for all women with epithelial ovarian cancer, regardless of age of onset, family history, or ancestry. Of women with ovarian cancer, 10% to 15% carry a mutation in one of these genes. Approximately 75% of epithelial ovarian cancers are serous histology—the type most likely to be associated with a BRCA1 or BRCA2 mutation. This patient has a family history of breast cancer in a paternal aunt who was age 52 years, but even with no family history of breast or ovarian cancer, BRCA1 and BRCA2 genetic testing would still be recommended. Although she had her ovaries removed, positive test results for a BRCA1 or BRCA2 mutation would have implications for breast cancer risk and screening, with annual mammography and breast MRI recommended. She may also consider prophylactic mastectomies. If she carries a BRCA1 or BRCA2 mutation, her siblings and children have a 50% chance of inheriting the same mutation. Genetic testing is best performed by a genetic counselor, with appropriate pre- and post-test counseling. Routine surveillance imaging studies are not recommended for monitoring patients in clinical remission after initial treatment; therefore, neither annual chest radiography nor an abdominal-pelvic CT scan in 3 months would be recommended unless the patient develops signs or symptoms suggestive of recurrent disease. Hereditary nonpolyposis colon cancer (HNPCC), also known as Lynch syndrome, is an autosomal dominant disorder caused by a germline mutation in one of the DNA mismatch repair genes. Patients with Lynch syndrome have a 3% to 14% lifetime risk of ovarian cancer, and the possibility of HNPCC mutation testing should be considered in patients with ovarian cancer. However, testing for HNPCC mutations is usually only offered to patients who have a personal or family history, or both, of additional HNPCC-related cancers, particularly colorectal, small bowel or endometrial cancers, or transitional cell cancers of the renal pelvis or ureters. This patient has no personal or family history of these cancers and would not usually be offered HNPCC testing based on her history of ovarian cancer alone. Models such as PREMM1,2,6 can be used to determine if a patient is eligible for HNPCC testing.
A 50-year-old woman is hospitalized for right arm weakness and altered mental status. Over the past 4 months, she has had gradual onset of headaches that have progressively worsened, and her family reports that the patient has had cognitive problems over the past 1 to 2 weeks. Today she developed right arm weakness. She has no other pertinent history and takes no medications. On physical examination, the patient is not oriented to place or date. Vital signs are normal. The patient has 4+/5 strength of the right upper extremity and achieves 16 of 30 points on the Mini-Mental State Examination; the remainder of the neurologic examination is normal. Laboratory studies show a normal erythrocyte sedimentation rate, complete blood count with differential, and comprehensive metabolic panel; a negative ANCA; and a negative urinalysis. Cerebrospinal fluid analysis shows a leukocyte count of 12/µL (12 × 106/L; all lymphocytes) and a protein level of 70 mg/dL (700 mg/L); Gram stain and cultures are negative. MRI of the brain shows diffuse nonspecific white matter changes and a low attenuation area in the right frontal lobe. Cerebral angiogram demonstrates multiple areas of vessel dilation and stenosis. Brain biopsy shows granulomatous vessel inflammation. In addition to high-dose glucocorticoids, which of the following is the most appropriate management? Adalimumab Cyclophosphamide Methotrexate Rituximab
Cyclophosphamide with high-dose glucocorticoids is appropriate treatment for primary angiitis of the central nervous system. In addition to high-dose glucocorticoids, cyclophosphamide is the most appropriate treatment for this patient who has primary angiitis of the central nervous system. Patients typically present with gradual progressive neurologic symptoms such as headaches, cognitive impairment, and other neurologic deficits such as strokes. Laboratory studies, including erythrocyte sedimentation rate, are typically normal, but 90% of patients have abnormal cerebrospinal fluid with lymphocytic pleocytosis and elevated total protein. MRI, MR angiogram, or CT angiogram often demonstrates nonspecific findings; cerebral angiogram sometimes reveals beading, or alternating stenosis and dilation of vessels consistent with vasculitis. Brain biopsy is falsely negative in 50% of patients due to patchy distribution of abnormalities, but when positive will show granulomatous vasculitis. Due to the rarity of this vasculitis, treatment is based on expert opinion and retrospective studies. High-dose glucocorticoids and cyclophosphamide have been reported to be effective in inducing remission and, given the severity of this patient's presentation, is the most appropriate choice. Glucocorticoids could be weaned over 3 to 6 months or longer, depending upon patient response to treatment. Cyclophosphamide could be discontinued after 3 to 6 months, and a maintenance drug such as azathioprine or mycophenolate mofetil could be started. Anecdotal reports indicate that rituximab and tumor necrosis factor α inhibitors (such as adalimumab) may be effective; however, until more reports and studies are available, these would not be the first choice of treatment. Methotrexate is not indicated for this patient because it does not penetrate the central nervous system well and would not be effective.
A 45-year-old man is evaluated for itching with dry scaling skin of 1 month's duration. His medical history is noncontributory, and he takes no medications. On physical examination, vital signs are normal. Skin findings are shown. The remainder of the examination is unremarkable. Which of the following is the most appropriate diagnostic test to perform next? Potassium hydroxide examination Scabies preparation Tzanck preparation Wood lamp examination
Diagnosis of dermatophyte infection can be performed by examination of the scale with potassium hydroxide; the presence of branching hyphae is diagnostic. The most appropriate diagnostic test to perform on this patient is the potassium hydroxide (KOH) examination. This patient's skin lesions are erythematous annular patches with noticeable surface scale. When confronted with an annular scaly patch, the most common diagnosis is tinea from a dermatophyte infection. Direct microscopic examination of KOH-prepared specimens is the simplest, cheapest method used for the diagnosis of dermatophyte infections of the skin. After scraping the leading edge of scale with a number 15 blade or the edge of the glass slide, apply 2 to 3 drops of KOH on the debris and then apply a coverslip. Evaluate the specimen initially with 10 power magnification. Tinea is confirmed by the presence of septated branching hyphae. A scabies preparation is used when the patient has burrows. Burrows are pathognomonic for a scabies infection and are most frequently found along the wrists, fingers, and palms. To obtain the specimen, place a drop of mineral oil on the burrow and scrape the area with a number 15 blade. After placing the specimen on a glass slide, apply the coverslip, and examine with 10 power magnification to identify mites, eggs, and/or feces. A Tzanck preparation should be performed on patients when the skin findings are concerning for a herpes virus infection. Grouped vesicles on an erythematous base or tender vesicles in a dermatomal distribution are both primary skin findings in which a Tzanck preparation can be performed. This is done by deroofing the blister and scraping the base of the blister. The scraping is smeared on a glass microscope slide and stained with an appropriate stain (Tzanck or Giemsa). The presence of multinucleated giant cells is diagnostic for herpes viral infections. This finding, however, cannot distinguish the responsible virus. Viral culture or direct immunofluorescence can be used to identify the specific virus, if needed. A Wood lamp is an ultraviolet light source that can be used to evaluate hypo- and depigmentation lesions as seen in vitiligo. Vitiligo will appear bright white and sharply delineated when examined with a Wood light and is most helpful in fair-skinned persons where depigmentation may not otherwise be visible. A Wood lamp can also detect skin fluorescence associated with erythrasma and the urine fluorescence in patients with porphyria cutanea tarda.
A 45-year-old man is evaluated for watery diarrhea accompanied by nausea and bloating. Symptoms began 4 weeks earlier with abdominal cramping and explosive watery stools. Now, he reports up to five loose bowel movements per day, with no blood and no nocturnal symptoms. He also describes generalized abdominal discomfort that is not relieved after a bowel movement. He works at a child care center. He is otherwise healthy and takes no medication. On physical examination, vital signs are normal. Abdominal examination shows periumbilical tenderness with no rebound or guarding. The remainder of the examination is unremarkable. Results of laboratory studies, including hemoglobin level and a comprehensive metabolic panel, are within normal limits. Which of the following is the most appropriate next step in management? Colonoscopy CT scan of the abdomen and pelvis 24-Hour urine 5-hydroxyindoleacetic acid measurement Stool testing for Giardia lamblia
Giardia lamblia infection is a common parasitic infection that occurs most often among children, child care workers, and backpackers or campers who drink untreated water from lakes, rivers, or wells. Stool testing for Giardia infection is the most appropriate next step in the management of this patient. The patient presents with persistent diarrhea, defined as lasting between 2 and 4 weeks, and because of his occupational exposure, the most likely cause is Giardia lamblia (also known as Giardia intestinalis). Transmission is waterborne or through the fecal-oral route. It occurs most commonly among children (especially in developing countries), child care workers, and backpackers or campers who drink untreated water from lakes, rivers, or wells. Giardia is a noninvasive organism that infects the small intestine and causes diarrhea associated with nausea, bloating, and foul-smelling gas due to malabsorption. Because Giardia cysts are intermittently shed, stool microscopy is less sensitive than stool antigen testing, and the recommended test is a stool enzyme-linked immunosorbent assay. Treatment options include tinidazole, metronidazole, and nitazoxanide. Treatment of giardiasis with metronidazole is curative in more than 85% of patients. A colonoscopy is not indicated because this patient's diarrhea is not associated with weight loss, abdominal pain, or blood in the stool, making inflammatory bowel disease an unlikely diagnosis. Microscopic colitis would be an unlikely diagnosis in a 45-year-old man who is not taking any medications known to be associated with microscopic colitis, such as NSAIDs. A CT scan of the abdomen and pelvis would be unlikely to suggest a cause for watery diarrhea of 4 weeks' duration. The patient's symptoms and occupation suggest a parasitic cause of diarrhea that would not be diagnosed with an imaging study. A 24-hour 5-hydroxyindoleacetic acid measurement is used to evaluate for carcinoid tumors. Up to 85% of patients with gastrointestinal carcinoid syndrome experience intermittent flushing. In addition to flushing, diarrhea is prominent in most patients and is related to rapid intestinal transit time. Approximately 40% of patients also have right-sided valvular heart disease. This patient has no flushing or evidence of a heart murmur, making carcinoid syndrome unlikely and measurement of 5-hydroxyindoleacetic unnecessary.
A 42-year-old woman is hospitalized with dyspnea, dry cough, and fever that has been increasing over the past few days. She underwent allogeneic hematopoietic stem cell transplantation (HSCT) 7 months ago, with a recent occurrence of graft-versus-host disease of the skin and gastrointestinal tract. Donor and recipient were both cytomegalovirus seropositive before transplantation, the patient received monitoring for cytomegalovirus reactivation for 6 months after transplantation, and the cytomegalovirus nucleic acid amplification test was negative 1 month ago. Medications are prednisone, tacrolimus, trimethoprim-sulfamethoxazole, and acyclovir. On physical examination, temperature is 38.1 °C (100.6 °F), blood pressure is 125/78 mm Hg, pulse rate is 104/min, and respiration rate is 24/min. Oxygen saturation is 88% breathing ambient air. Crackles are audible bilaterally on pulmonary examination. No lymphadenopathy is noted. The remainder of the examination is noncontributory. Laboratory studies show a leukocyte count of 3500/µL (3.5 × 109/L), with 85% polymorphonuclear cells, 8% lymphocytes, 5% monocytes, and 2% eosinophils. A chest radiograph shows bilateral diffuse infiltrates. Chest CT scan shows bilateral diffuse ground-glass opacities without pleural effusions. In addition to starting broad-spectrum antibacterial therapy, which of the following is the most appropriate initial treatment? Atovaquone Ganciclovir Liposomal amphotericin No additional therapy
Cytomegalovirus disease after transplantation may manifest as a nonspecific febrile illness; may cause leukopenia and thrombocytopenia; or may cause organ-specific disease, most often pneumonitis, colitis, esophagitis, or hepatitis; ganciclovir therapy should be initiated empirically in patients who present with signs and symptoms of cytomegalovirus infection.
A 42-year-old woman is hospitalized with dyspnea, dry cough, and fever that has been increasing over the past few days. She underwent allogeneic hematopoietic stem cell transplantation (HSCT) 7 months ago, with a recent occurrence of graft-versus-host disease of the skin and gastrointestinal tract. Donor and recipient were both cytomegalovirus seropositive before transplantation, the patient received monitoring for cytomegalovirus reactivation for 6 months after transplantation, and the cytomegalovirus nucleic acid amplification test was negative 1 month ago. Medications are prednisone, tacrolimus, trimethoprim-sulfamethoxazole, and acyclovir. On physical examination, temperature is 38.1 °C (100.6 °F), blood pressure is 125/78 mm Hg, pulse rate is 104/min, and respiration rate is 24/min. Oxygen saturation is 88% breathing ambient air. Crackles are audible bilaterally on pulmonary examination. No lymphadenopathy is noted. The remainder of the examination is noncontributory. Laboratory studies show a leukocyte count of 3500/µL (3.5 × 109/L), with 85% polymorphonuclear cells, 8% lymphocytes, 5% monocytes, and 2% eosinophils. A chest radiograph shows bilateral diffuse infiltrates. Chest CT scan shows bilateral diffuse ground-glass opacities without pleural effusions. In addition to starting broad-spectrum antibacterial therapy, which of the following is the most appropriate initial treatment? Atovaquone Ganciclovir Liposomal amphotericin No additional therapy
Cytomegalovirus disease after transplantation may manifest as a nonspecific febrile illness; may cause leukopenia and thrombocytopenia; or may cause organ-specific disease, most often pneumonitis, colitis, esophagitis, or hepatitis; ganciclovir therapy should be initiated empirically in patients who present with signs and symptoms of cytomegalovirus infection. Ganciclovir for cytomegalovirus is the most appropriate empiric treatment in this patient at this time. The risk of cytomegalovirus reactivation is related to serologic status of the donor and recipient and is unlikely when donor and recipient are both negative. Cytomegalovirus disease after transplantation may manifest as a nonspecific febrile illness; may cause leukopenia and thrombocytopenia; or may cause organ-specific disease, most often pneumonitis, colitis, esophagitis, or hepatitis. This patient is at significant risk for cytomegalovirus infection and has a clinical presentation consistent with cytomegalovirus pneumonia, specifically her presenting symptoms and radiographic findings of diffuse bilateral lung involvement. Although it has been more than 6 months from her hematopoietic stem cell transplantation (HSCT), her recent graft-versus-host disease increases infection risk. She is not receiving cytomegalovirus prophylaxis at this time; her acyclovir therapy is effective prophylaxis for herpes simplex virus and varicella-zoster virus but not for cytomegalovirus. Cytomegalovirus pneumonia after HSCT is associated with poor prognosis and significant mortality. Treatment with ganciclovir should be started while proceeding with the diagnostic evaluation, including nucleic acid amplification testing and bronchoscopy. Atovaquone is an appropriate treatment for Pneumocystis jirovecii pneumonia. Although this patient is at risk for P. jirovecii and her presentation is consistent with it, she is taking trimethoprim-sulfamethoxazole, which is effective as prophylaxis, making this infection unlikely. Liposomal amphotericin would provide broad-spectrum antifungal coverage in this patient who is at risk for fungal infection. However, her presentation and radiographic findings are not consistent with invasive fungal infection, which is more likely to demonstrate nodules on CT. Moreover, the toxicity of amphotericin would preclude its use empirically in this setting.
A 40-year-old man is evaluated during a follow-up visit for an extremely pruritic skin eruption for several months that was recently diagnosed as dermatitis herpetiformis. He denies gastrointestinal symptoms and is otherwise healthy and takes no medications. On physical examination, vital signs are normal. There are numerous excoriations on elbows, knees, and buttocks with rare intact vesicles. In addition to a gluten-free diet, which of the following is the most appropriate treatment for this patient? Betamethasone valerate Dapsone Diphenhydramine Prednisone Sulfasalazine
Dapsone should be used in conjunction with a gluten-free diet as first-line treatment of dermatitis herpetiformis.
A 40-year-old man is evaluated during a follow-up visit for an extremely pruritic skin eruption for several months that was recently diagnosed as dermatitis herpetiformis. He denies gastrointestinal symptoms and is otherwise healthy and takes no medications. On physical examination, vital signs are normal. There are numerous excoriations on elbows, knees, and buttocks with rare intact vesicles. In addition to a gluten-free diet, which of the following is the most appropriate treatment for this patient? Betamethasone valerate Dapsone Diphenhydramine Prednisone Sulfasalazine
Dapsone should be used in conjunction with a gluten-free diet as first-line treatment of dermatitis herpetiformis. This patient has dermatitis herpetiformis (DH), and the most appropriate treatment is a gluten-free diet and dapsone. DH is a subepidermal autoimmune bullous disorder that is extremely pruritic. There are small tense vesicles and papules, which are rarely intact, so the usual presentation is excoriations on the elbows, knees, and buttocks. The diagnosis can be confirmed with skin biopsy for routine and direct immunofluorescence testing. Deposition of granular IgA in the dermal papillary tips is pathognomonic of dermatitis herpetiformis. While dapsone is effective in inducing a clinical remission of DH, a gluten-free diet is the preferred long-term therapy. DH has a strong association with celiac disease. Dapsone will successfully treat the skin symptoms of dermatitis herpetiformis, but does not treat the associated gastrointestinal disease. A gluten-free diet treats both the cause and the symptoms of both diseases. In patients with celiac disease, failure to maintain a gluten-free diet increases the risk for small bowel lymphoma. Prior to initiating therapy with dapsone, patients should be checked for glucose-6-phosphate dehydrogenase deficiency because these patients have a high risk of hemolysis on dapsone therapy. Potent topical glucocorticoids, such as betamethasone valerate, may help alleviate pruritus associated with DH but are not adequate monotherapy and should always be used in combination with dapsone and a gluten-free diet. Long-term side effects of topical glucocorticoids include cutaneous atrophy, striae, and hypopigmentation. Numerous oral antihistamines are available for the treatment of pruritus, and anecdotal experience suggests that the more sedating antihistamines such as diphenhydramine may have a better antipruritic effect than the less-sedating products. However, DH is not a histamine-mediated disease, and the use of antihistamines for this condition is not very effective and may result in intolerable sedation in some patients. In addition to the significant complications associated with the long-term use of prednisone, it is not effective in inducing or maintaining clinical remission in patients with DH and should not be used. For patients who cannot tolerate dapsone, other sulfonamide drugs are a potential treatment option for DH. Case reports suggest that sulfasalazine is effective. While sulfasalazine does not cause hemolysis, there is an increased risk for agranulocytosis and hypersensitivity reactions. As with dapsone, periodic laboratory monitoring for significant side effects is recommended.
A 50-year-old woman is evaluated for a 4-month history of left flank pain, worsening despite NSAIDs and now requiring opioid analgesics, and 4-kg (8.8 lb) weight loss. Her medical history is otherwise unremarkable. On physical examination, the skin and mucous membranes are pale. Vital signs are normal. There is left flank tenderness on palpation. The remainder of the physical examination is unremarkable. Laboratory studies reveal a hemoglobin level of 10.1 g/dL (101 g/L). CT scan of the chest, abdomen, and pelvis reveals a 10-cm left kidney mass with radiographic features suggesting renal cell carcinoma and a 1.2-cm single pulmonary nodule. Which of the following is the most appropriate management? Left radical nephrectomy Nivolumab PET/CT scan Radiation to the kidney mass
Debulking nephrectomy improves survival in patients diagnosed with metastatic kidney cancer, particularly in those with limited metastatic disease. The most appropriate management is left radical nephrectomy. Renal cell carcinomas arise in the renal cortex and are the most common type of tumors affecting the kidney. Patients with renal cell carcinoma are often asymptomatic until they have advanced disease, but possible symptoms include hematuria, an abdominal mass, abdominal pain, and unexplained weight loss. However, the classic triad of flank pain, hematuria, and a palpable abdominal mass occurs in only approximately 9% of patients. This patient most likely has metastatic kidney cancer based on identification of a large kidney mass associated with a suspicious pulmonary nodule and anemia. However, there is no histologic diagnosis of cancer at this point. Debulking nephrectomy has been shown to improve survival in patients diagnosed with metastatic kidney cancer, particularly in those with limited metastatic disease. Patients with lung-only metastasis and good performance status benefit most. In this patient, therefore, nephrectomy would serve two purposes. It would confirm the diagnosis and also serve as treatment for her metastatic kidney cancer. Following nephrectomy it would be appropriate to consider metastasectomy for removal of the isolated pulmonary lesion. Nivolumab, a monoclonal antibody directed against programmed death receptor 1, is currently FDA approved for treatment of advanced clear cell carcinoma of the kidney. However, nivolumab is not recommended as first-line treatment and should not be started before establishing a histologic diagnosis of clear cell carcinoma. Clear cell carcinoma has not been confirmed in this patient, so the use of nivolumab would be premature. PET/CT scan plays no role in the evaluation of a patient suspected of having kidney cancer. The presence or absence of fluorodeoxyglucose avidity would not influence management. External beam radiation does not play a role in the treatment of metastatic kidney cancer except for palliative treatment of metastatic sites, such as painful osseous metastatic lesions. In this patient with a large kidney mass and one site of metastatic disease, surgery is indicated. Radiation has not been shown to improve outcomes in this setting and is not recommended.
A 59-year-old woman comes to the office for a follow-up evaluation of Parkinson disease, which was diagnosed 8 years ago and has been treated with carbidopa-levodopa, amantadine, and rasagiline. Primary symptoms are tremor, slowness, shuffling of the feet, and intermittent freezing of gait, which has caused her to fall several times; the tremor and slowness substantially improve after administration of carbidopa-levodopa, but she exhibits prominent involuntary, nonpatterned, dance-like movements for approximately 2 hours after a dose. She has tried increasing the carbidopa-levodopa dosage but could not tolerate the resultant nausea and orthostatic hypotension. On initial physical examination, which takes place before she takes her scheduled dose of carbidopa-levodopa, a prominent tremor is noted. The patient's spontaneous and repetitive movements are very slow, and she has difficulty taking more than a few steps before needing assistance to maintain balance. A repeat examination performed 1 hour after she takes the medication reveals suppression of the tremor, remarkable improvement of speed and gait, and the presence of diffuse high-amplitude flowing involuntary movements in all extremities. An MRI of the brain is unremarkable. Which of the following is the most appropriate next step in her treatment? Apomorphine Deep brain stimulation Droxidopa Rotigotine patch
Deep brain stimulation is appropriate for patients with Parkinson disease who derive a continued benefit from carbidopa-levodopa but experience medication-related complications. This patient should have deep brain stimulation, which is indicated for patients with advanced Parkinson disease who derive a continued benefit from carbidopa-levodopa but experience medication-related complications. The patient has two common levodopa-related complications: motor fluctuations (caused by the wearing off of the beneficial effects of the medication before the next dose is administered) and medication-induced dyskinesia (involuntary choreic movements). Attempting to increase the dose of levodopa to mitigate the "wearing-off" phenomenon resulted in dyskinesia, nausea, and hypotension. Deep brain stimulation enables a persistent motor benefit and reduction of the total levodopa dosage, which would diminish the levodopa-induced adverse effects, including dyskinesia, nausea, and orthostatic hypotension. Apomorphine is a fast-acting subcutaneous dopamine agonist that is indicated for rapid relief of symptoms caused by sudden wearing off of a Parkinson medication. It is inappropriate for this patient, who experiences predictable episodes of wearing off of the carbidopa-levodopa benefit before she takes the next dose. Apomorphine also does not address the dyskinesia or nausea. Droxidopa has been approved for management of neurogenic orthostatic hypotension in Parkinson disease. This treatment, however, would not improve the wearing off of the dopaminergic medication before the next dose or the patient's other symptoms of gait freezing, dyskinesia, and medication-induced nausea. Rotigotine, a dopamine agonist, can worsen both nausea and orthostatic hypotension. Additionally, an overall increase in dopaminergic medication dosing is likely to aggravate dyskinesia. Read Related TextNext Question
A 59-year-old woman is evaluated for a 5-year history of bilateral hand pain and stiffness of several hand joints, with enlargement and crookedness developing at some of the hand joints over the past few years. The stiffness is present in the morning and lasts about 30 minutes. One area of enlargement, at the right fourth distal interphalangeal (DIP) joint, has occasionally become abruptly swollen, red, and tender, and is currently inflamed. Family history is significant for deformed and painful, arthritic hand joints in her mother. On physical examination, vital signs are normal. There are no rashes or nail changes. Grossly deformed digits on both hands with bony enlargement and deviation at the right second, left second and third, and bilateral fourth DIP joints are noted. A tender, erythematous bony nodule is present over the right fourth DIP joint. Significant squaring of the right carpometacarpal joint is also present. The remainder of the musculoskeletal examination is normal. Laboratory studies show normal erythrocyte sedimentation rate, C-reactive protein, serum urate, rheumatoid factor, and anti-cyclic citrullinated peptide antibodies. A hand radiograph is shown. Which of the following is the most likely diagnosis? Erosive osteoarthritis Gout Psoriatic arthritis Rheumatoid arthritis
Diagnosis of erosive hand osteoarthritis is based essentially on radiographic findings of articular surface erosions at the central portion of the joint; erosions are often symmetric and occur mainly in the distal interphalangeal joints. Erosive osteoarthritis (OA) is the most likely diagnosis. Erosive OA is thought to be an aggressive subset of hand osteoarthritis that has periods of prominent inflammatory symptoms and characteristic erosive changes on imaging. Patients often experience pain, swelling, and joint deformities. Inflammatory damage to soft-tissue structures can result in deviation of the digits. Diagnosis is mostly based on the radiographic findings of articular surface erosions at the central portion of the joint. Erosions are often symmetric and occur mainly in the distal interphalangeal (DIP) and, to a lesser extent, proximal interphalangeal (PIP) joints. Radiographs reveal a "gull-wing" deformity, resulting from marginal sclerosis and osteophytes on the distal side of the joints, whereas the proximal side is centrally eroded or collapsed and thinned. Women are affected more frequently than men. Erythrocyte sedimentation rate (ESR), C-reactive protein (CRP), rheumatoid factor, and anti-cyclic citrullinated peptide antibodies are usually normal, as seen in this patient. Heberden nodes and squaring at the carpometacarpal joint also support a diagnosis of erosive OA in this patient. Although gout flares may be superimposed upon sites of nodal OA, the inflamed DIP joints in this patient are less likely to represent a gouty flare given the normal ESR, CRP, and urate levels. Moreover, radiographs of patients with tophaceous gout often show marginal and juxta-articular punched-out erosions, which are not seen in this patient. Psoriatic arthritis may be symmetric and affect the hands, including the DIP joints, but the patient reports no personal history of psoriasis, does not indicate significant morning stiffness, has no rashes or nail pitting, and does not have the marginal erosions on radiograph that would support this diagnosis. Although the patient has symmetric hand arthritis and fits the typical demographics of a patient with rheumatoid arthritis (RA), the predominance of DIP symptoms, the lack of prolonged morning stiffness, the laboratory studies negative for inflammatory markers and/or RA autoantibodies, and the typical "gull-wing" deformity on radiograph (rather than marginal erosions) do not support RA as the diagnosis.
A 26-year-old woman is evaluated for palpitations and fatigue of several months' duration. Her symptoms are present at rest and aggravated during light activity, including climbing stairs and walking quickly. She reports no dyspnea, chest pain, orthopnea, lower extremity edema, or syncope. Medical history is otherwise unremarkable, and she takes no medications. On physical examination, temperature is normal, blood pressure is 110/65 mm Hg, and pulse rate is 115/min. Orthostatic vital signs do not demonstrate any significant changes with posture. Precordial examination reveals mild tachycardia but regular rhythm with no gallops. No thyroid fullness or tremors are noted. The remainder of the examination is normal. Laboratory studies are significant for a normal complete blood count, serum thyroid-stimulating hormone level, and 24-hour excretion of urine catecholamines. A 12-lead electrocardiogram is notable for sinus tachycardia with a normal P-wave axis. A 24-hour ambulatory electrocardiographic monitor shows sinus tachycardia throughout the waking hours, with a decline in heart rate to less than 90/min during the night. The average heart rate is 110/min. An echocardiogram reveals a structurally normal heart with normal ventricular function. Which of the following is the most likely diagnosis? Generalized anxiety disorder Inappropriate sinus tachycardia Somatic symptom disorder Subclinical hyperthyroidism
Diagnosis of inappropriate sinus tachycardia is based on symptoms and findings on ambulatory electrocardiographic monitoring after the exclusion of other causes of tachycardia, such as hyperthyroidism, anemia, pheochromocytoma, and structural heart disease. This patient with sinus tachycardia and a structurally normal heart has symptoms, electrocardiographic features, and ambulatory electrocardiographic findings consistent with the diagnosis of inappropriate sinus tachycardia (IST). IST is characterized by elevated resting heart rate, with exaggerated increases in heart rate with light activity. The sinus rates typically decrease during sleep, as identified on this patient's ambulatory electrocardiographic monitor. IST frequently presents in women in their second to fourth decades and appears to be more common in health care professionals. Symptoms vary and can include palpitations, lightheadedness, syncope or near-syncope, dyspnea, and fatigue. Emotional manifestations, such as anxiety or depression, may also be present. Diagnosis of IST is based on the exclusion of other causes of tachycardia, such as hyperthyroidism, anemia, pheochromocytoma, and structural heart disease. First-line therapy is removal of aggravating factors and exercise therapy. In patients with bothersome and persistent symptoms, pharmacologic therapy with β-blockers or calcium channel blockers can be considered. Ivabradine is a relatively new drug that impairs the If or "I-funny" current and may play a role in the treatment of IST refractory to standard therapy. The diagnostic criteria for generalized anxiety disorder are as follows: (1) excessive anxiety or worry about a number of events or activities occurring for 6 months or longer; (2) the patient recognizes it is difficult to control the worry; (3) the anxiety or worry is associated with three or more of the following symptoms: restlessness, easy fatigability, difficulty concentrating, irritability, muscle tension, and sleep disturbance; (4) the anxiety, worry, or symptoms cause impairment at school, work, or other settings and cannot be attributable to medical or other psychiatric conditions, medications, or substance use. Diagnostic criteria for somatic symptom disorder include at least one somatic symptom causing distress or interference with daily life; excessive thoughts, behaviors, and feelings related to the somatic symptom(s); and persistent somatic symptoms for at least 6 months. This patient does not fulfill the diagnostic criteria for generalized anxiety disorder or somatic symptom disorder. Subclinical hyperthyroidism is a laboratory-based diagnosis, defined as the presence of a suppressed thyroid-stimulating hormone level with normal triiodothyronine and thyroxine levels. Symptoms of thyrotoxicosis are typically mild; most patients are asymptomatic. This patient's normal thyroid-stimulating hormone level argues against this diagnosis.
A 24-year-old man with progressive fatigue and intermittent dark urine over several months is evaluated in the emergency department for exertional dyspnea, abdominal pain, and red urine. On physical examination, he is pale. Temperature is 37.0 °C (98.6 °F), blood pressure is 110/70 mm Hg, pulse rate is 112/min, and respiration rate is 16/min. Oxygen saturation is 98% breathing ambient air. Scattered petechiae are visible on the skin. The abdomen is not distended and is diffusely tender to palpation without guarding. Bowel sounds are normal. The remainder of the examination is normal. Laboratory studies: Haptoglobin Undetectable Hemoglobin 7.2 g/dL (72 g/L) Leukocyte count 1200/µL (1.2 × 109/L) with 70% neutrophils and 30% lymphocytes Mean corpuscular volume 84 fL Platelet count 23,000/µL (23 × 109/L) Reticulocyte count 8% of erythrocytes Lactate dehydrogenase 500 U/L Urinalysis 4+ blood; 0-1 erythrocytes/hpf; 0 leukocytes/hpf The peripheral blood smear shows normal-appearing erythrocytes without spherocytes, schistocytes, agglutinated erythrocytes, or immature-appearing leukocytes. Which of the following is the most appropriate next test? Direct antiglobulin test Flow cytometry Methylmalonic acid measurement Quantitative and functional levels of C1 esterase inhibitor
Diagnosis of paroxysmal nocturnal hemoglobinuria is based on flow cytometry results, which can detect CD55 and CD59 deficiency on the surface of peripheral erythrocytes or leukocytes. Flow cytometry is the most appropriate test to perform next. This patient has pancytopenia with significant intravascular hemolysis and hemoglobinuria, most likely caused by paroxysmal nocturnal hemoglobinuria (PNH). Patients with PNH commonly note fatigue and nonspecific abdominal pain, which worsens during times of increased hemolysis. These patients have a mutation in the gene that regulates production of glycophosphatidylinositol-anchoring proteins. Flow cytometry will demonstrate the absence of glycophosphatidylinositol-linked proteins, such as CD55 and CD59, on leukocytes; these proteins normally prevent complement activation on the erythrocyte surface. Bone marrow aplasia and corresponding pancytopenia in these patients are common, and they have an increased risk of evolving to a myelodysplastic syndrome or acute leukemia. Patients with PNH also have an increased risk for thrombotic complications, including thrombi in unusual sites, such as the portal venous system or sagittal veins in the central nervous system. Hereditary angioedema is caused by the unregulated activation of the complement cascade owing to lack of or ineffective function of C1 esterase inhibitor, leading to increased vascular permeability. C1 esterase inhibitor deficiency has no role in the pathogenesis of PNH. While complement-mediated hemolysis is a core feature of PNH, the complement system functions normally. Neither excess of complement nor IgG antibodies are noted on the erythrocyte surface that would be revealed by a direct antiglobulin test. Vitamin B12 deficiency can cause pancytopenia and is commonly associated with some features of intramedullary hemolysis resulting from ineffective erythropoiesis. However, vitamin B12 deficiency is associated with macrocytic anemia, which is not present in this patient. Hemoglobinuria is present in PNH but not seen in vitamin B12 deficiency. The first step in diagnosing vitamin B12 deficiency is a cobalamin measurement. Methylmalonic acid and homocysteine measurements may be helpful in cases of diagnostic uncertainty because they are elevated in 98% of patients with cobalamin deficiency. Methylmalonic acid measurement is unnecessary in this patient.
A 74-year-old man is evaluated for pain and stiffness in the mid and lower spine that has progressively worsened over the past 10 years. The pain is worse with physical activity. He notes recent difficulty when bending to pick something up from the floor. He takes no medications. On physical examination, vital signs are normal. No rash or nail changes are seen. Limited range of motion and pain on motion of thoracic and lumbar spine are noted. There is no peripheral joint swelling or tenderness and no sacroiliac tenderness. The FABER test of the hip is normal. Laboratory studies show a normal erythrocyte sedimentation rate. Thoracolumbar spine radiographs reveal bridging ossification on the right side along the anterolateral aspects of the vertebral bodies of T9-L2. Radiographs of the sacroiliac joints are normal. Which of the following is the most likely diagnosis? Ankylosing spondylitis Calcium pyrophosphate deposition disease Diffuse idiopathic skeletal hyperostosis Psoriatic arthritis
Diffuse idiopathic skeletal hyperostosis is a noninflammatory condition that involves ossification of spinal ligaments and entheses and usually presents as back pain and stiffness; characteristic radiographic changes include confluent ossification of at least four contiguous vertebral levels, usually on the right side of the spine. Diffuse idiopathic skeletal hyperostosis (DISH) is the most likely diagnosis. DISH is a noninflammatory condition involving ossification of spinal ligaments and entheses. DISH can be asymptomatic or present in various ways, including as pain and stiffness in the spine, with the thoracic spine being most often involved. Although features such as ligamentous ossification are shared with ankylosing spondylitis, the sacroiliac joints are not involved in DISH. The most characteristic radiographic change of DISH is confluent ossification of at least four contiguous vertebral levels, usually on the right side of the spine. The left-sided sparing is possibly related to mechanical pressure from the aorta, and its pulsations, as a barrier toward production of bony hyperostosis on the left side. Ankylosing spondylitis is characterized by bilateral sacroiliac joint abnormalities on spinal radiographs. Other spinal sections may also be involved, but sacroiliac joint involvement is cardinal for diagnosis. Inflammatory back pain is the most common symptom and is usually comprised of four of five of the following features: onset of back discomfort before the age of 40 years; insidious onset; improvement with exercise; no improvement with rest; and pain at night (with improvement upon arising). Calcium pyrophosphate deposition may affect the axial spine but more often affects the peripheral skeleton. It is often asymptomatic, is not known to have a predilection for a particular segment of the spine, and appears as punctate and linear radiodensities of calcification within the cartilage on radiographs. It does not result in ossification of spinal ligaments and entheses. Psoriatic arthritis can affect the spine, with inflammatory back pain symptoms similar to those listed with ankylosing spondylitis. Differences between psoriatic arthritis and DISH include that sacroiliac joints may be asymmetrically involved in psoriatic arthritis but are not involved in DISH, and the spondylitis is random and not necessarily continuous in psoriatic arthritis, whereas a defining characteristic of DISH is involvement of four or more vertebral bodies. DISH syndesmophytes (ligamentous ossification) are located on the anterior and lateral margins of vertebrae, but they tend to be nonmarginal in psoriatic arthritis.
A 75-year-old man comes to the office for a follow-up evaluation of Parkinson disease, which was diagnosed 12 years ago. Worsening motor symptoms have been controlled with a combination of carbidopa-levodopa and pramipexole for the past 3 years. According to family members, the patient becomes rigid and slow before taking his next dose of Parkinson medications, but these symptoms resolve after he has taken them. He reports recent visual hallucinations that have been distressing, occasionally seeing small animals and unfamiliar people mostly at night, which sometimes frightens him. He has had no recent illness, fever, anhedonia, or depressive symptoms. On physical examination, vital signs are normal. The patient performs well on screening cognitive testing. He reports no active hallucinations. Masked facies, mild dysarthria, resting tremor, bradykinesia, and cogwheel rigidity are noted. Gait is narrow based and shuffling. Other physical examination findings are unremarkable. Results of laboratory studies, including a complete blood count, comprehensive metabolic panel, serum creatine kinase measurement, and urinalysis, are normal. Which of the following is the most appropriate next step in treatment? Add aripiprazole Add pimavanserin Add risperidone Discontinue pramipexole
Dopamine agonists are associated with higher risk of specific adverse effects, including psychosis, impulse control disorder, punding, sleep attacks, ankle edema, and confusion, that may limit their use, especially in older patients with Parkinson disease. The dopamine agonist pramipexole should be discontinued in this patient with Parkinson disease psychosis. Dopamine agonists, such as pramipexole, can be used as adjuvant therapy to limit the total dose of levodopa. Dopamine agonists are associated with higher risk of specific adverse effects, including impulse control disorder, punding, sleep attacks, ankle edema, and confusion, that may limit their use, especially in older patients. Impulse control disorder is the increased tendency for compulsive behaviors, such as excessive gambling, shopping, or hypersexuality. Punding is a complex prolonged, purposeless, and stereotyped behavior (such as collecting, sorting, cataloguing, or assembling and disassembling common objects). Sleep attacks involve falling asleep without warning. Medication-induced psychosis, especially visual hallucinations, triggered by dopaminergic medications also is a potential complication of Parkinson disease. All patients with Parkinson disease and psychosis should be evaluated for systemic triggers (such as infection and metabolic disturbance) and major depression. Reducing the dose of levodopa also can be considered after removal of the dopamine agonist, but this step may not be possible because of worsening of motor symptoms (as may occur in this patient given the recurrence of rigidity and slowness before each dose of medication). The decision to treat psychosis with medication adjustments or discontinuation should be based on the presence of distress to patient and caregivers, lack of insight in the patient, and progressive frequency of psychosis. Pimavanserin, a nondopaminergic atypical antipsychotic agent and selective serotonin 5-hydroxytryptamine receptor 2A inverse agonist, is the only FDA-approved medication for Parkinson psychosis. Quetiapine and clozapine also can be considered in this setting, but most other atypical antipsychotic agents (including aripiprazole, risperidone, and olanzapine) and all typical (first-generation) antipsychotic agents should be avoided. Atypical antipsychotic agents should be used only after discontinuation of any non-levodopa Parkinson medication and resolution of systemic triggers.
A 73-year-old woman is evaluated for a 10-year history of osteoarthritis affecting multiple joints over the years, including the distal joints of her fingers, bases of the thumbs, knees, and cervical and lower lumbar spines. She has chronic daily pain in at least one joint. She has tried nonpharmacologic measures, and she had minimal benefit from intra-articular glucocorticoid and hyaluronic acid injections to her knees. She was recently diagnosed with peptic ulcer disease. History is also significant for coronary artery disease, diabetes mellitus, and hypertension. Medications are enalapril, carvedilol, metformin, atorvastatin, pantoprazole, and low-dose aspirin. On physical examination, vital signs are normal. Heberden nodes and squaring of the bilateral carpometacarpal joints are present. Crepitus and limited extension of the cervical spine are noted. Bilateral knee varus deformity and bony enlargement are present, with crepitus on range of motion. Which of the following is the most appropriate treatment? Duloxetine Gabapentin Ibuprofen Topical capsaicin
Duloxetine is FDA approved for chronic musculoskeletal pain and has been shown to have analgesic efficacy for chronic low back pain and knee osteoarthritis pain. Duloxetine, a serotonin and norepinephrine reuptake inhibitor with central nervous system effects, is a good treatment option for this patient with generalized osteoarthritis (OA). Duloxetine is FDA approved for chronic musculoskeletal pain and has been shown to have analgesic efficacy for chronic low back pain and knee OA pain, implicating the role of central sensitization in OA pain modulation. This patient has already tried multiple nonpharmacologic measures, as well as intra-articular glucocorticoid and hyaluronic injections, all with insufficient symptomatic relief. Duloxetine is a reasonable choice given the patient's comorbidities and generalized musculoskeletal pain. Gabapentin and pregabalin are more effective than placebo in the treatment of neuropathic pain conditions such as postherpetic neuralgia and diabetic neuropathy. These drugs are expensive and both are associated with dose-dependent dizziness and sedation. There is no evidence of their effectiveness specifically for chronic OA pain. NSAIDs inhibit cyclooxygenase (COX) enzymes, blocking the generation of the lipid prostaglandin E2 (PGE2). PGE2 stimulates inflammation, vasodilation, smooth muscle contraction, pain, and fever. However, PGE2 also maintains gastric mucosa and promotes kidney sodium excretion and glomerular filtration. Other COX products include thromboxane A2, a prothrombotic regulator of platelets, and prostacyclin, an antithrombotic and vasodilatory lipid. Because NSAIDs inhibit all of these, the consequences of COX inhibition are complex and accompanied by multiple potential side effects. Side-effect risk is increased in older patients and those with preexisting comorbidities. Therefore, ibuprofen is not an advisable choice in an elderly patient with peptic ulcer disease, hypertension, and heart disease. Topical capsaicin may benefit localized OA (for example, knee only) but is impractical in this case given the multiple areas of involvement. Furthermore, duloxetine is more likely to be efficacious given how many treatments this patient has tried and failed.
A 62-year-old man was admitted to the hospital following a motor vehicle accident. He underwent a craniotomy for treatment of intracranial hemorrhage following which he was started on carbamazepine. On postoperative day 19, the patient was noted to have a rash on his chest, arms, proximal legs, and buttocks, as well as redness on his face. The next day he had diffuse papules and macules on his trunk and proximal extremities, tachycardia, and hypotension. Hypotension and tachycardia responded to fluid resuscitation. On physical examination, vital signs are normal. The patient is lethargic and confused. Skin findings are shown. He has prominent swelling in his face. There is minimal skin erosion and no sloughing when shearing pressure is applied with the examiner's finger. There are a few erosions in the oropharynx, but no eye or genital involvement. Generalized lymphadenopathy is present. Which of the following is the most likely cause of this patient's clinical presentation? Drug reaction with eosinophilia and systemic symptoms (DRESS) Retiform purpura Stevens-Johnson syndrome Toxic epidermal necrolysis
Drug reaction with eosinophilia and systemic symptoms (DRESS) is a systemic drug hypersensitivity reaction that presents with rash, prominent facial edema, lymphadenopathy, and fever 2 to 6 weeks after the initiation of the causative drug.
A 62-year-old man was admitted to the hospital following a motor vehicle accident. He underwent a craniotomy for treatment of intracranial hemorrhage following which he was started on carbamazepine. On postoperative day 19, the patient was noted to have a rash on his chest, arms, proximal legs, and buttocks, as well as redness on his face. The next day he had diffuse papules and macules on his trunk and proximal extremities, tachycardia, and hypotension. Hypotension and tachycardia responded to fluid resuscitation. On physical examination, vital signs are normal. The patient is lethargic and confused. Skin findings are shown. He has prominent swelling in his face. There is minimal skin erosion and no sloughing when shearing pressure is applied with the examiner's finger. There are a few erosions in the oropharynx, but no eye or genital involvement. Generalized lymphadenopathy is present. Which of the following is the most likely cause of this patient's clinical presentation? Drug reaction with eosinophilia and systemic symptoms (DRESS) Retiform purpura Stevens-Johnson syndrome Toxic epidermal necrolysis
Drug reaction with eosinophilia and systemic symptoms (DRESS) is a systemic drug hypersensitivity reaction that presents with rash, prominent facial edema, lymphadenopathy, and fever 2 to 6 weeks after the initiation of the causative drug. This patient's symptoms are characteristic of drug reaction with eosinophilia and systemic symptoms (DRESS) or drug hypersensitivity syndrome. DRESS is a severe, life-threatening, idiosyncratic medication reaction. The most common culprit medications include sulfonamide antibiotics, allopurinol, and anticonvulsants, but many more have been implicated. DRESS is unique in that the onset is usually 2 to 6 weeks after exposure to a causative medication. Because of this delayed onset, DRESS is often underrecognized or misdiagnosed. The eruption is usually an exuberant morbilliform eruption with prominent facial edema, lymphadenopathy, fever, and, in severe cases, hypotension. Typical laboratory findings include leukocytosis with eosinophilia and increased serum alanine aminotransferase levels. Cessation of the causative agent and systemic glucocorticoids with a long taper are the best treatment. DRESS reactions with carbamazepine may cross-react with phenytoin and phenobarbital, but not levetiracetam or valproic acid. The term retiform purpura describes the angulated or netlike configuration that reflects the underlying vascular structure in the skin. Various conditions can cause retiform purpura, many of which disrupt arterial blood flow such as thrombosis and embolization. Stevens-Johnson syndrome (SJS) and toxic epidermal necrolysis (TEN) are both severe drug reactions, but these reactions happen closer to the start date of the offending agent than DRESS syndrome. SJS/TEN lack the facial edema of DRESS syndrome, and DRESS syndrome lacks the epidermal necrosis of SJS/TEN. DRESS can cause mucositis in the minority of cases. Both SJS/TEN and DRESS can result in liver or kidney function abnormalities, but eosinophilia is strongly indicative of DRESS.
An 81-year-old woman is evaluated following drug-eluting stent placement for stable but disabling angina 1 month ago. After stent placement, she was started on aspirin and clopidogrel. She is currently asymptomatic, and she is interested in minimizing the duration of dual antiplatelet therapy because of her age and comorbid illnesses. Medical history is significant for hyperlipidemia, hypertension, stage 3 chronic kidney disease, and previous peptic ulcer disease. Other medications are lisinopril, metoprolol, atorvastatin, and pantoprazole. On physical examination, vital signs are normal. Oxygen saturation is 98% breathing ambient air. The remainder of the examination is unremarkable. Which of the following is the most appropriate management of this patient's antiplatelet therapy? Discontinue clopidogrel now Discontinue clopidogrel in 5 months Discontinue clopidogrel in 11 months Discontinue clopidogrel and aspirin in 11 months Discontinue clopidogrel in 29 months
Dual antiplatelet therapy is recommended for at least 6 months after drug-eluting stent placement for treatment of stable angina. This patient should receive dual antiplatelet therapy (DAPT) for 5 additional months, after which clopidogrel can be discontinued. In patients who undergo percutaneous coronary intervention (PCI), DAPT with aspirin plus a P2Y12 inhibitor is indicated to prevent stent thrombosis and future cardiovascular events. However, recently, there has been considerable movement regarding the optimal duration of DAPT following drug-eluting stent placement. It is important to find the correct balance between blocking platelet activity during the process of reendothelialization of the stent and exposure to the attendant bleeding risk. Current guidelines suggest that the minimum duration for DAPT following drug-eluting stent placement for patients without an acute coronary syndrome is at least 6 months, with the option to continue therapy for a longer duration in those with a high risk for thrombosis-related complications and a favorable bleeding profile. Ultimately, the decision may be individualized at the patient level, such as with this patient, who is at relatively high bleeding risk given her age and comorbid conditions. In patients who undergo PCI with bare metal stent placement, DAPT may be discontinued after 1 month; however, this patient with a drug-eluting stent requires longer therapy. Patients with acute coronary syndrome treated with PCI (bare metal or drug-eluting stent placement) should be optimally treated with DAPT for at least 12 months. Aspirin should be continued indefinitely in patients with coronary artery disease, unless contraindicated. No data support discontinuing both antiplatelet agents following PCI. A study of DAPT comparing 12 months of therapy with 30 months of therapy suggested a benefit to the longer duration in some patients. Extending DAPT for a longer duration should be individualized on the basis of predicted risk for additional cardiovascular events and risk for bleeding.
A 76-year-old woman is evaluated before discharge. She was diagnosed with a non-ST-elevation myocardial infarction 3 days ago. She declined angiography, and nuclear stress testing revealed a small lateral perfusion defect and normal left ventricular ejection fraction. She has had no further discomfort since admission. Medical history is significant for hyperlipidemia, hypertension, and transient ischemic attack. Medications are low-dose aspirin, ramipril, metoprolol, and atorvastatin. On physical examination, vital signs and the remainder of the examination are unremarkable. In addition to low-dose aspirin, which of the following is the optimal antithrombotic regimen for this patient? Prasugrel Ticagrelor Warfarin No additional antithrombotic therapy
Dual antiplatelet therapy, composed of aspirin and a P2Y12 inhibitor, is indicated in all patients following acute coronary syndrome for the prevention of vascular ischemic events. This patient with a non-ST-elevation myocardial infarction (NSTEMI) should receive low-dose aspirin indefinitely and ticagrelor for 1 year. Dual antiplatelet therapy, composed of aspirin and a P2Y12 inhibitor, is indicated in all patients following acute coronary syndrome (ACS) for the prevention of vascular ischemic events. Aspirin should be continued indefinitely, and in patients with ACS treated with medical therapy alone, the optimal duration for P2Y12 inhibitor therapy (clopidogrel or ticagrelor) is at least 12 months. Although clopidogrel is an option in this patient, in the PLATO trial, the combination of low-dose aspirin and ticagrelor was superior to aspirin and clopidogrel in reducing the incidence of cardiovascular death, myocardial infarction, and stroke following ACS; the effect size was identical in medically treated patients and those who underwent early invasive treatment. The American College of Cardiology and American Heart Association prefer ticagrelor to clopidogrel (class IIa, level B recommendation) but caution that additional studies with focused objectives are needed. Prasugrel, another P2Y12 inhibitor, is recommended for use only in patients with ACS who undergo percutaneous coronary intervention. Prasugrel is no more effective than clopidogrel in comparison studies of medically treated patients with ACS, and it carries a higher risk for bleeding than does clopidogrel. Anticoagulation with warfarin is not routinely recommended following ACS unless there is another indication for its use, such as venous thromboembolic disease or atrial fibrillation. Aspirin therapy alone would not offer adequate protection against cardiovascular events in this patient with NSTEMI.
A 73-year-old woman is evaluated for a 10-year history of osteoarthritis affecting multiple joints over the years, including the distal joints of her fingers, bases of the thumbs, knees, and cervical and lower lumbar spines. She has chronic daily pain in at least one joint. She has tried nonpharmacologic measures, and she had minimal benefit from intra-articular glucocorticoid and hyaluronic acid injections to her knees. She was recently diagnosed with peptic ulcer disease. History is also significant for coronary artery disease, diabetes mellitus, and hypertension. Medications are enalapril, carvedilol, metformin, atorvastatin, pantoprazole, and low-dose aspirin. On physical examination, vital signs are normal. Heberden nodes and squaring of the bilateral carpometacarpal joints are present. Crepitus and limited extension of the cervical spine are noted. Bilateral knee varus deformity and bony enlargement are present, with crepitus on range of motion. Which of the following is the most appropriate treatment? Duloxetine Gabapentin Ibuprofen Topical capsaicin
Duloxetine is FDA approved for chronic musculoskeletal pain and has been shown to have analgesic efficacy for chronic low back pain and knee osteoarthritis pain.
A 44-year-old woman is evaluated for a breast lump she noticed 1 week ago. There is no nipple discharge. She birthed two children, the first at age 25 years, and her menstrual cycles are regular. She has no family members with breast or ovarian cancer. She has no other medical problems and takes no medications. On physical examination, vital signs are normal. BMI is 28. Examination of the breasts reveals dense tissue bilaterally and no skin changes. A mass is noted in the upper inner area of the right breast, measuring 1.8 cm; it is firm, mobile, and nontender, with ill-defined margins. There is no evidence of axillary, cervical, or supraclavicular lymphadenopathy. The remainder of the examination is unremarkable. Which of the following is the most appropriate diagnostic test to perform next? Breast MRI Core-needle biopsy Diagnostic mammography and ultrasonography Ultrasonography
For evaluation of palpable breast abnormalities in women aged 40 years or older, mammography, followed in most cases by ultrasonography, is recommended. This patient's breast mass should be evaluated with both diagnostic mammography and ultrasonography. Any dominant mass in the breast warrants diagnostic imaging to determine the nature of the mass and the appropriate management. A clinical examination alone cannot differentiate between a cyst and a solid mass. In a woman aged 30 years or older, diagnostic mammography is recommended for evaluation of a palpable mass, to assess for a spiculated density or associated pleomorphic calcifications that may indicate malignancy. Diagnostic mammography may also include magnification views of the focal area of concern. In addition, targeted ultrasonography is needed to determine whether the mass is cystic or solid; however, ultrasonography is unnecessary in cases in which mammography shows a clearly benign correlate or a normal, fatty area of breast tissue in the location of the palpable finding. This diagnostic evaluation can help determine a benign finding from an indeterminate or suspicious finding requiring needle biopsy. For women aged 40 years or older, mammography, followed in most cases by ultrasonography, is recommended. For women aged 30 to 39 years old, ultrasonography or mammography may be performed first at the discretion of the radiologist or referring clinician. Breast MRI and other advanced imaging have little to no role in the routine diagnostic evaluation of palpable breast abnormalities. Breast MRI is not an appropriate imaging technique for evaluation of palpable symptoms because it provides little added information to careful evaluation with mammography and ultrasonography. Definitive diagnosis of a breast mass is obtained by tissue sampling using fine-needle aspiration, core-needle biopsy with or without stereotactic or ultrasound guidance, or excisional biopsy. Fine-needle aspiration is generally reserved for ultrasound-confirmed cystic lesions. Core-needle biopsy is the test of choice for most solid lesions, as it provides more tissue for histology and tissue markers. Excisional biopsy is used when core-needle biopsy findings are nondiagnostic or when biopsy and imaging studies do not concur. Further management of abnormal pathologic findings requires consultation with a breast surgeon and oncologist. Needle aspiration, core-needle biopsy, and excision are all premature until the imaging evaluation is completed. Ultrasonography is often preferred in women younger than age 30 years because the increased density of breast tissue in younger women limits the usefulness of mammography. Ultrasonography may also be a better choice for pregnant patients in order to avoid radiation exposure.
A 68-year-old man develops abrupt, pleuritic, right-sided chest pain, and dyspnea 90 minutes into a flight from Nashville to Phoenix. He is on the second week of a prednisone taper for a recent COPD exacerbation in the setting of bullous emphysema. He is on supplemental oxygen at home at 2 L/min, which was augmented to 4 L/min for the flight. Current medications are a long-acting β2-agonist, a long-acting muscarinic agent, and prednisone. On physical examination, blood pressure is 138/78 mm Hg, pulse rate is 114/min, and respiration rate is 24/min. He appears moderately distressed. Lung examination reveals diminished breath sounds bilaterally, tympany to percussion bilaterally, and end-expiratory wheezing. He has strong symmetric peripheral pulses but no neck vein distention. Cardiac examination is unremarkable. Which of the following is the most likely diagnosis? Descending aortic dissection Pneumocystis jirovecii pneumonia Pneumothorax Pulmonary embolism
During air travel, pneumothorax is most likely to occur at cruising altitude in patients with bullous lung disease, particularly those with a recent exacerbation of airways disease who are, therefore, more prone to air trapping.
A 68-year-old man develops abrupt, pleuritic, right-sided chest pain, and dyspnea 90 minutes into a flight from Nashville to Phoenix. He is on the second week of a prednisone taper for a recent COPD exacerbation in the setting of bullous emphysema. He is on supplemental oxygen at home at 2 L/min, which was augmented to 4 L/min for the flight. Current medications are a long-acting β2-agonist, a long-acting muscarinic agent, and prednisone. On physical examination, blood pressure is 138/78 mm Hg, pulse rate is 114/min, and respiration rate is 24/min. He appears moderately distressed. Lung examination reveals diminished breath sounds bilaterally, tympany to percussion bilaterally, and end-expiratory wheezing. He has strong symmetric peripheral pulses but no neck vein distention. Cardiac examination is unremarkable. Which of the following is the most likely diagnosis? Descending aortic dissection Pneumocystis jirovecii pneumonia Pneumothorax Pulmonary embolism
During air travel, pneumothorax is most likely to occur at cruising altitude in patients with bullous lung disease, particularly those with a recent exacerbation of airways disease who are, therefore, more prone to air trapping. The most likely diagnosis is pneumothorax. An estimated 12% of in-flight medical emergencies involve a respiratory complaint. Commercial airline cabins are partially pressurized, typically to the equivalent of approximately 1400 to 2500 meters (4000 to 8000 feet) above sea level, limiting exposure to extreme hypobaric conditions. The risk of pneumothorax is therefore mitigated but it is most likely to occur at cruising altitude in patients with bullous lung disease, particularly those with a recent exacerbation of airways disease who are, therefore, more prone to air trapping. Pain is typically pleuritic; dyspnea may be present, depending upon the volume of trapped air. If tension physiology develops (hypotension, shock, altered mental status), needle thoracostomy using the on-board equipment can be life-saving. Descending to a lower altitude may also be beneficial, because cabin pressure is inversely related to the altitude of the aircraft. Descending aortic dissection typically presents with acute, severe back pain. Although chest pain can occur, it is less common. Radiation of pain to the abdomen can occur with disruption of blood flow to the abdominal viscera. Hypertension is the most important risk factor and is present in more than half of patients with descending aortic dissection. Pulse deficits are common. However, this patient does not have a history of hypertension and has normal peripheral pulses, making a diagnosis of descending aortic dissection unlikely. Pneumocystis jirovecii pneumonia is unlikely to present in this manner. Although systemic glucocorticoids increase the risk for opportunistic lung infections such as P. jirovecii pneumonia, the risk is minimal during the course of a typical prednisone burst and taper used for a COPD exacerbation. In general, those taking prednisone dosage equivalents of at least 20 mg/day for more than 3 weeks should receive P. jirovecii pneumonia prophylaxis. Air travel increases the risk of venous thromboembolism, though the risk is higher during relative immobilization on long flights and in those with other risk factors, such as cancer. The risk of pulmonary embolism does not really increase until the flight distance becomes greater than 5000 km (3000 miles). It is unlikely that a flight duration of only 90 minutes would heighten thrombotic risk in this patient.
A 52-year-old woman is evaluated during a follow-up visit. She was discharged from the hospital 3 weeks ago following a small non-ST-elevation myocardial infarction treated with drug-eluting stent placement in the right coronary artery. An echocardiogram obtained during hospitalization showed normal left ventricular function and normal valvular function. Her hospital course was uncomplicated. Since discharge, she has had shortness of breath. Medical history is significant for hyperlipidemia. Medications are aspirin, ticagrelor, lisinopril, metoprolol, and atorvastatin. On physical examination, vital signs are normal. Oxygen saturation is 99% breathing ambient air. The estimated central venous pressure is normal. Cardiac examination reveals no S3 or murmurs. The lungs are clear to auscultation. A chest radiograph is normal. An electrocardiogram is unchanged from those obtained in the hospital. Which of the following is the most likely cause of this patient's dyspnea? Heart failure In-stent restenosis Stent thrombosis Ticagrelor-mediated side effect Ventricular septal rupture
Dyspnea is a well-recognized and often self-limited side effect of ticagrelor therapy. Ticagrelor is the most likely cause of dyspnea in this patient with a normal physical examination, no electrocardiographic changes, and normal findings on imaging studies. Ticagrelor is a P2Y12 inhibitor that may be used as a component of dual antiplatelet therapy in select patients with coronary artery disease, including those treated with percutaneous coronary intervention. Dyspnea is a well-recognized side effect of ticagrelor therapy. In clinical trials, 15% to 20% of patients taking ticagrelor experienced dyspnea, although only 5% to 7% required cessation of the drug. In most cases, ticagrelor-mediated dyspnea is self-limited, but it often results in additional testing. Heart failure can complicate myocardial infarction. In this case, however, the normal findings on physical examination, chest radiograph, and echocardiogram make heart failure an unlikely cause of the patient's symptoms. In patients with shortness of breath after myocardial infarction, it is important to rule out complications of recent percutaneous coronary intervention, such as in-stent stenosis and stent thrombosis. Patients with in-stent restenosis will exhibit recurrent signs and symptoms of ischemia, including chest pain and dyspnea; however, in-stent restenosis develops months to years after stent implantation, not weeks. In contrast to in-stent restenosis, stent thrombosis is usually a fulminant event, commonly manifesting as acute myocardial infarction or death. The timing and persistent nature of this patient's symptoms, coupled with the unchanged electrocardiographic findings, rule out stent thrombosis as a cause of this patient's symptoms. Ventricular septal defect (VSD) resulting from rupture of the intraventricular septum is a rare complication of transmural infarction involving the right coronary artery (in which the VSD tends to affect the basal inferior septum) or the left anterior descending artery (in which the VSD is usually located within the apical septum). VSDs typically occur within 3 to 5 days of STEMI presentation. Patients present with worsening heart failure and shock, and a harsh holosystolic murmur may be heard at the left lower sternal border. A small NSTEMI would not result in a VSD, and the laboratory and physical examination findings rule out this diagnosis. Read Related TextNext Question
A 41-year-old woman is evaluated for a 2-year history of tremor in the dominant right hand. She says that the tremor has begun interfering with her work as a hairdresser, especially when she uses scissors. She also reports tightness in the forearm. The patient is able to eat, write, and type without difficulty and has had no trauma, imbalance, slowness of movement, or change in gait speed. Alcohol has no effect on the tremor. There is no family history of tremor. On physical examination, vital signs are normal. A right upper extremity tremor is noted, as are rhythmic flexion of the wrist, involuntary flexion of the fingers, and pronation of the forearm. The tremor is present both at rest and during action and resolves by changing the position of an outstretched arm. No dysmetria, dysdiadochokinesia, bradykinesia, rigidity, shuffling gait, or reduced arm swing is noted. Her handwriting is neither tremulous nor micrographic. An MRI of the brain is unremarkable. Which of the following is the most likely diagnosis? Cerebellar tremor Dystonic tremor Essential tremor Parkinson disease Rubral tremor
Dystonic tremor occurs both at rest and with action and is characterized by associated dystonic posturing and the presence of a null point at which a change in the position of the affected limb resolves the tremor. This patient has a dystonic tremor, which occurs both at rest and with action and is characterized by associated dystonic posturing and the presence of a null point at which change in the position of the affected limb resolves the tremor. The null point is the position at which the trajectories of the forces caused by dystonic coactivation of agonist and antagonist muscles neutralize each other, which leads to resolution of the tremor. In addition, the action component of tremor has task specificity in that it is worse with use of scissors but spares the handwriting. Cerebellar tremor is characterized by increasing tremor amplitude as the limb approaches the target (terminal intention tremor) and the presence of associated cerebellar symptoms. These features are absent in this patient. Essential tremor is the most common movement disorder and often presents with a bilateral upper extremity postural and action tremor. It is not associated with dystonic features. Additional features include bilateral involvement and a positive family history. Amelioration of the tremor by ethanol is typical and did not occur in this patient. Parkinsonian tremor is prominent at rest and can reemerge after a brief delay when the arms are held in an outstretched position. Although dystonia can be seen secondary to Parkinson disease, the absence of other associated features, especially the bradykinesia required for a diagnosis of Parkinson disease, excludes this diagnosis. Rubral tremor is caused by focal injury to cerebellar outflow pathways and is characterized by a coarse tremor that is present at rest but most severe during action. This type of tremor has a prominent proximal component and interferes with various actions, such as feeding, typing, and writing, in a nonselective way. Also, MRIs of the brain reveal a focal causative lesion that is not present in this patient.
A 68-year-old man is evaluated for 3 weeks of painful "blood blisters" on his legs and feet. He has noticed burning and tingling of his right foot and three toes over the last 2 weeks. Medical history is significant for rheumatoid arthritis for 22 years. He is a 60-pack-year cigarette smoker. He occasionally uses cocaine. He denies fever, chills, and night sweats, anorexia, or weight loss. Medications are hydroxychloroquine and methotrexate. On physical examination, temperature is 36.6 °C (97.8 °F), blood pressure is 134/79 mm Hg, and pulse rate is 82/min. Changes typical of destructive rheumatoid arthritis are apparent in the hands, wrists, elbows, and feet. Palpable purpura is present over the legs and ankles. Periungual purpura is present on three fingers. There is reduced sensitivity to pinprick and fine touch over the lateral right foot and dorsal lateral three toes. There is no murmur on his cardiac examination. Laboratory studies show substantially elevated rheumatoid factor titer. Antinuclear antibody titer is less than 1:40. ANCA screen is negative. Complement levels are normal. Results of kidney and liver chemistry tests are normal. Which of the following is the most likely cause of the patient's skin findings? Actinic purpura Illicit drug use Infective endocarditis Rheumatoid vasculitis
Elderly smokers with long-standing rheumatoid arthritis and high rheumatoid factor titers are at risk for rheumatoid vasculitis. Rheumatoid vasculitis commonly affects small to medium-sized vessels of the skin, digits, peripheral nerves, eyes, and heart. Small-vessel vasculitis (leukocytoclastic vasculitis) appears as palpable purpura. In medium-vessel disease, nodules, ulcerations, livedo reticularis, and digital infarcts can occur. Periungual purpura (Bywaters lesions) is the result of nailfold thrombosis and appears as purpuric papules on the digital pulp of a few digits and in the nail fold area. Actinic purpura is caused by age- and sun damage-related capillary fragility and bleeding under atrophic skin. It is recognized as a flat, noninflamed ecchymosis typically in sun damaged skin on the forearms and dorsal hands. Illicit drug use can result in leukocytoclastic vasculitis, most notably from cocaine adulterated with levamisole. These patients generally have palpable purpura, often with involvement of the ears. Laboratory studies typically show an elevated ANCA titer, with a mixed perinuclear and cytoplasmic pattern. These findings are absent in this patient. Infective endocarditis can result in localized immune-mediated vasculitis secondary to vascular occlusion by infected microthrombi. Infective endocarditis is an unlikely diagnosis in this patient in the absence of heart murmur, fever (present in 90% of patients), or constitutional symptoms such as anorexia and weight loss. Read Related TextNext Question
A 35-year-old woman is evaluated for exertional dyspnea of 6 months' duration. She reports no other symptoms. Medical history is unremarkable, and she takes no medications. On physical examination, vital signs are normal. Oxygen saturation is 96% breathing ambient air. The estimated central venous pressure is elevated. Apical impulse is normal; a parasternal impulse is noted at the left sternal border. A soft systolic murmur is heard at the second left intercostal space, and a diastolic flow rumble is heard at the left sternal border. Fixed splitting of the S2 is noted throughout the cardiac cycle. The remainder of the physical examination is normal. An electrocardiogram demonstrates right axis deviation and incomplete right bundle branch block. Which of the following is the most likely diagnosis? Atrial septal defect Bicuspid aortic valve with aortic stenosis Congenital pulmonary stenosis Mitral stenosis
Elevated central venous pressure, fixed splitting of the S2, a right ventricular heave, and right-axis deviation and incomplete right bundle branch block on electrocardiogram are characteristic findings in patients with ostium secundum atrial septal defect. An ostium secundum atrial septal defect (ASD) is the most likely diagnosis in this patient. Adults with an ASD most often present with dyspnea or atrial arrhythmias; elevated central venous pressure, fixed splitting of the S2, and a right ventricular heave are characteristic findings. The fixed splitting of the S2 in patients with an ostium secundum ASD results from prolongation of right ventricular systole and lack of respiratory change in the right ventricular stroke volume. With expiration, the decrease in venous return is counteracted by an increase in left-to-right shunting, resulting in a fixed right ventricular preload. A large left-to-right shunt causes a pulmonary midsystolic flow murmur and a tricuspid diastolic flow rumble owing to increased flow. In ostium secundum ASD, the electrocardiogram (ECG) demonstrates right-axis deviation and incomplete right bundle branch block. Aortic stenosis due to a bicuspid aortic valve causes a systolic murmur at the second right intercostal space. The central venous pressure is normal in aortic stenosis, and a right ventricular impulse would not be expected. A systolic ejection click is often heard in patients with bicuspid aortic valve, but fixed splitting of the S2 is not heard. The ECG typically demonstrates a normal axis and features of left ventricular hypertrophy. Pulmonary stenosis is usually congenital, and severe obstruction can cause exertional dyspnea. Physical examination features depend on the severity of obstruction and associated elevation of right heart pressure; these include central venous pressure elevation with a prominent a wave and a parasternal impulse. The systolic murmur of pulmonary stenosis is generally heard at the second left intercostal space, and the timing of the murmur is related to stenosis severity. An ejection click is often heard; the proximity of the click to the S2 varies depending on the severity of stenosis. S2 becomes fixed in severe pulmonary stenosis. The ECG demonstrates right axis deviation and features of right ventricular hypertrophy. Patients with mitral stenosis might present with symptoms of dyspnea. A right ventricular impulse can occur, and the central venous pressure might be elevated with associated pulmonary hypertension or tricuspid regurgitation. The murmur of mitral stenosis is generally best heard at the apex. An opening snap might be heard, followed by a diastolic murmur. Fixed splitting of the S2 is not heard. The ECG typically demonstrates left atrial enlargement. Read Related TextNext Question
A 60-year-old man is evaluated during a routine office visit. He was diagnosed with type 2 diabetes mellitus 6 years ago. Medical history is significant for coronary artery disease, hypertension, hyperlipidemia, and biliary pancreatitis. Medications are lisinopril, metoprolol, metformin, aspirin, and atorvastatin. On physical examination, other than a blood pressure of 152/91 mm Hg, the vital signs are normal. BMI is 27. The remainder of the examination is normal. Laboratory studies show a hemoglobin A1c level of 8.2%. Which of the following is the most appropriate treatment for this patient? Empagliflozin Glipizide Liraglutide Sitagliptin
Empagliflozin has been shown to reduce cardiovascular-related events and all-cause mortality in patients with type 2 diabetes mellitus and cardiovascular disease. The patient has uncontrolled diabetes in the setting of coronary artery disease, and empagliflozin is the most appropriate treatment. Empagliflozin increases excretion of glucose by the kidneys through inhibition of the sodium-glucose transporter-2 (SGLT2) receptors. Empagliflozin received approval from the FDA for patients with type 2 diabetes and established cardiovascular disease based upon the results of the Empagliflozin Cardiovascular Outcome Event Trial in Type 2 Diabetes Mellitus Patients (EMPA-REG OUTCOME). This study demonstrated a reduction in the primary composite outcome (cardiovascular-related death, nonfatal myocardial infarction, nonfatal stroke) and all-cause mortality when empagliflozin was added to standard care versus placebo. Empagliflozin has the additional potential benefit of inducing weight loss and blood pressure lowering in this patient with overweight and uncontrolled hypertension. Although the sulfonylurea, glipizide, could improve the patient's glycemic control, it has the potential side effect of weight gain; the combination of metformin plus an SGLT2 inhibitor is superior to metformin plus a sulfonylurea (mean between-group difference, 4.7 kg [CI, 4.4 to 5.0 kg]). The combination of metformin and an SGLT2 inhibitor reduces systolic blood pressure more than that of metformin and a sulfonylurea (between-group difference, 5.1 mm Hg [CI, 4.2 to 6.0 mm Hg]). In patients with type 2 diabetes and cardiovascular risk factors, a significant reduction in the primary composite outcome (cardiovascular death, nonfatal myocardial infarction, or nonfatal stroke) and rates of cardiovascular death and all-cause mortality has also been associated with liraglutide. There have been postmarketing reports of fatal and nonfatal acute pancreatitis associated with liraglutide. While it is not known if liraglutide increases risk for development of pancreatitis in patients with a history of pancreatitis, many experts avoid its use in this patient population. The dipeptidyl peptidase-4 (DPP-4) inhibitor, sitagliptin, could improve the patient's glycemic control; however, the combination of metformin and an SGLT2 inhibitor reduces systolic blood pressure more than metformin and a DPP-4 inhibitor (pooled between-group difference, 4.1 mm Hg [CI, 3.6 to 4.6 mm Hg]) and SGLT2 inhibitors reduced weight more than DPP-4 inhibitors (between group difference, 2.5 to 2.7 kg). There have been postmarketing reports of fatal and nonfatal acute pancreatitis associated with sitagliptin. While no causal relationship has been established, FDA labeling guidelines recommend that sitagliptin be used with caution in patients with a history of pancreatitis and some experts recommend against its use entirely in this population.
A 34-year-old woman is evaluated 4 days after being hospitalized for severe epigastric abdominal pain associated with nausea and vomiting. At the time of hospitalization, her serum lipase level was 4650 U/L and an abdominal ultrasound examination showed cholelithiasis. She has taken nothing by mouth (NPO) since hospitalization and reports having no appetite. She is being treated with intravenous fluids and opioid pain medication. A CT scan of the abdomen and pelvis with contrast showed changes consistent with pancreatic necrosis. Her medical history is unremarkable and she took no medication before hospitalization. On physical examination, temperature is 38 °C (100.4 °F), blood pressure is 130/70 mm Hg, pulse rate is 90/min, and respiration rate is 15/min. Abdominal examination shows tenderness in the epigastrium with no abdominal distention. Which of the following is the most appropriate next step in management of her nutrition? Initiate enteral feeding Initiate total parenteral nutrition Maintain NPO status until cholecystectomy is performed Maintain NPO status until the lipase level normalizes
Enteral nutrition is preferred in patients with acute pancreatitis because of the benefit of maintaining a healthy gut mucosal barrier to prevent translocation of bacteria. Placement of a nasoenteric tube for enteral feeding is the most appropriate next step in this patient's management. The patient has taken nothing by mouth (NPO) since hospitalization 4 days earlier and requires nutritional support. She continues to have abdominal pain requiring intravenous opioid pain medication, as well as nausea and no appetite. Enteral nutrition is preferred in patients with acute pancreatitis because of the benefit of maintaining a healthy mucosal barrier in the gut to prevent translocation of bacteria. Both nasogastric and nasojejunal enteral feeding are safe and have comparable effectiveness. Studies show nasogastric tube feeding is well tolerated in these patients, and placement of a nasogastric tube may be easier, more cost effective, and faster than placement of a nasojejunal tube. Total parenteral nutrition (TPN) is discouraged in patients with acute pancreatitis because the mucosal barrier is not maintained when patients are NPO for prolonged periods, which may lead to higher rates of bacterial translocation into necrotic pancreatic tissue. TPN also increases the risk for bacterial and fungal bloodstream infections, as well as venous thrombosis associated with more proximal indwelling intravenous catheters required for TPN administration. Cholecystectomy will not be an option for this patient in the near future (likely not during this hospitalization) because she has severe necrotic acute pancreatitis. Maintaining NPO status for a prolonged period of time before surgery is not appropriate clinical management of her nutritional needs. There are no data to support withholding feeding in patients with acute pancreatitis based on enzyme levels. Lipase levels three to five times the upper level of normal are used as diagnostic criteria for acute pancreatitis but have no impact on clinical management decisions.
A 45-year-old man is evaluated for a 3-month history of swollen and painful hands and toes of both feet, without trauma or injury. He also has morning stiffness of the small joints of the hands and feet lasting 2 hours. He takes no medications. On physical examination, vital signs are normal. Diffuse tenderness and swelling are noted in the third and fourth toes of the right foot, fourth toe of the left foot, and third and fourth digits of the left hand. The remainder of the joint examination is normal. The appearance of the hand digits is shown. Plain radiographs of the feet show joint-space narrowing and soft-tissue swelling of the digits without erosions. Which of the following disorders is most likely to be associated with this patient's presentation? Infectious arthritis Rheumatoid arthritis Spondyloarthritis Systemic lupus erythematosus
Enthesitis is highly suggestive of spondyloarthritis; when particularly severe, the inflammation may extend along the associated tendon and local ligaments, resulting in dactylitis ("sausage digits"). Spondyloarthritis is the most likely diagnosis in this patient with enthesitis. Spondyloarthritis refers to a group of disorders that share an overlapping set of features, including inflammation of the axial skeleton, tendons, and entheses (insertion of tendon to bone); tendon and enthesis calcification; an association with HLA-B27; and mucocutaneous, gastrointestinal, and ocular inflammation. The four disorders of spondyloarthritis are ankylosing spondylitis, psoriatic arthritis, inflammatory bowel disease-associated arthritis, and reactive arthritis. The enthesis is a complex structure at the site of insertion of a tendon or ligament onto the bone. Inflammation of the enthesis (enthesitis) is highly suggestive of spondyloarthritis. When enthesitis is particularly severe, the inflammation may extend along the associated tendon and local ligaments, resulting in dactylitis ("sausage digits"). This patient has dactylitis, with diffuse inflammatory swelling of multiple digits, making spondyloarthritis the likely cause of this patient's presentation. Infectious arthritis typically presents with pain, swelling, warmth, and erythema of a single joint, accompanied by fever and constitutional symptoms, and is not associated with enthesitis/dactylitis. Rheumatoid arthritis is characterized by a chronic inflammatory polyarthritis affecting large and small joints with a predilection for the small joints of the hands and feet and is not associated with enthesitis/dactylitis. System lupus erythematosus (SLE) should be considered in any patient who presents with unexplained multisystem disease. The most common early SLE manifestations include constitutional symptoms (fever, weight loss, or severe fatigue), arthralgia/arthritis, and skin disease. Joint involvement occurs in 90% of patients with SLE, with inflammatory polyarthralgia the most common presentation. SLE is unlikely due to the lack of rash or other clinical features; furthermore, it is not associated with enthesitis/dactylitis.
A 25-year-old man is evaluated for a sensation of solid food "sticking" several times per week. He reports that he sometimes forces himself to vomit when he feels food "stuck" in the esophagus, but he has never gone to the emergency department. He takes a multivitamin and is generally healthy. On physical examination, vital signs and other findings, including those of an abdominal examination, are unremarkable. Upper endoscopy findings are shown. Biopsies of the esophagus show more than 18 eosinophils/hpf. Which of the following is the most likely diagnosis? Achalasia Candida esophagitis Eosinophilic esophagitis Pill-induced esophagitis
Eosinophilic esophagitis typically presents in young men with symptoms of dysphagia and in patients with a history of food allergies, eczema, and asthma. Eosinophilic esophagitis is the most likely diagnosis in this patient. Defined as esophageal squamous mucosal inflammation caused by eosinophilic infiltration, eosinophilic esophagitis is commonly seen in young men presenting with symptoms of dysphagia. It often co-occurs in patients with food allergies, asthma, and eczema. Rings and furrows in the esophagus are common findings on upper endoscopy. The diagnosis is confirmed with biopsies of the esophagus showing more than 15 eosinophils/hpf in the absence of other known causes of esophageal eosinophilia. For patients with confirmed eosinophilic esophagitis, treatment includes restriction of dietary elements (elemental diet or targeted elimination diet) and swallowed aerosolized topical glucocorticoids or a proton pump inhibitor. Patients should also be counseled that the condition may recur. This patient's endoscopic findings are not consistent with a diagnosis of achalasia. In patients with achalasia, endoscopy often shows a dilated esophageal body with resistance at the gastroesophageal junction. A manometry test confirms the diagnosis of achalasia. The most effective treatment options include pneumatic dilation and surgical myotomy. Candida esophagitis rarely occurs in immunocompetent patients but is seen more often in patients with HIV and other immunocompromised patients. The most common symptom is odynophagia rather than dysphagia. Oral thrush may also be present, but its absence does not exclude esophageal involvement. Endoscopic findings in patients with Candida esophagitis are small, raised, white plaques. It is treated with oral fluconazole and management of the immunocompromised state. Because this patient takes only a multivitamin and has no ulcers seen on endoscopy, pill-induced esophagitis is unlikely. Pill-induced esophagitis has been observed with medications including alendronate, quinidine, tetracycline, doxycycline, potassium chloride, ferrous sulfate, and mexiletine. Pills typically cause local injury at sites of anatomic narrowing of the esophagus. Clinical symptoms include chest pain, dysphagia, and odynophagia. Symptoms may begin hours to days after starting therapy, and stopping the medication often leads to symptom relief.
A 23-year-old woman is evaluated in the emergency department. Four days ago she developed a cold sore on her lower lip. This was followed by the development of painful sores throughout her mouth. The next day she developed a rash on her hands, arms, legs, and face, and eventually on her palms and soles. She takes no medications. On physical examination, vital signs are normal. The patient has pain in her mouth, but is in no acute distress. Erosions are present in the oropharynx. There are five distinct monomorphic clustered 2- to 3-mm erosions on the left lower lip. Skin findings on the palm are shown. The arms and face have lesions similar to those on the palm. There is no sloughing of the skin when shearing force is applied. The lesions affect about 7% body surface area. There is no corneal injection or tearing and no genital erosion or ulceration. Which of the following is the most likely diagnosis? Disseminated gonococcemia Erythema multiforme major Stevens-Johnson syndrome Toxic epidermal necrolysis
Erythema multiforme (EM) features the development of characteristic tricolored targetoid plaques, as this patient demonstrates on the extremities and face. In EM major, these targetoid plaques are accompanied by mucous membrane involvement. The most common cause of EM is infection, and herpes simplex virus 1 and 2 are the most commonly recognized, followed by Mycoplasma pneumoniae. The treatment of EM major is supportive, and most cases resolve in a week. Suppressive therapy may be necessary for patients who have relapses of EM with herpes outbreaks. Patients with disseminated gonococcal infection and bacteremia present with vesiculopustular or hemorrhagic macular skin lesions, fever, chills, and polyarthralgia. Knees, elbows, and distal joints are typical sites of involvement. Tenosynovitis of the dorsa of the hands and/or feet is a characteristic feature. Gonococcemia does not cause targetoid plaques or mucositis. Stevens-Johnson syndrome (SJS) and toxic epidermal necrolysis (TEN) are typically medication reactions. Mucous membrane involvement and targetoid lesions occur as well. Unlike EM, however, the targetoid lesions in SJS and TEN lack the true tricolor appearance and are thus labeled atypical targetoid plaques. Like EM, SJS can sometimes be triggered by infection, but unlike EM, SJS is characterized by necrosis and sloughing of the skin. SJS and TEN have systemic manifestations such as acute kidney injury and elevated liver chemistry tests, whereas EM does not.
A 72-year-old woman is evaluated in the emergency department 5 hours after developing difficulty speaking and facial weakness on the right. She takes no medication. On physical examination, vital signs are normal. The patient is awake and attentive. Spontaneous speech is slow. Right-sided facial weakness and dysarthria are noted. Hemoglobin level, platelet count, and coagulation profile are within normal limits. An electrocardiogram is normal. A CT scan of the head shows an acute left frontal ischemic stroke. A carotid duplex ultrasound reveals less than 40% stenosis in both internal carotid arteries; a transcranial Doppler ultrasound is normal. A transthoracic echocardiogram shows an ejection fraction of 50% but is otherwise unremarkable. Aspirin and rosuvastatin are initiated, and the patient is admitted to the telemetry unit for 3 days, during which time she remains in sinus rhythm. Which of the following is the most appropriate next step in management? Addition of clopidogrel Outpatient cardiac telemetry Substitution of apixaban for aspirin Transesophageal echocardiography
Evaluation of cryptogenic stroke with prolonged outpatient rhythm monitoring may yield a new diagnosis of atrial fibrillation in almost one third of patients. This patient should have outpatient cardiac telemetry. She has a cryptogenic infarct with no clear source identified on arterial imaging, no evidence of atrial fibrillation (AF) or other high-risk embolic cause, and a stroke location that is not typical for lacunar infarcts. Accumulating data on patients with cryptogenic stroke indicate that an evaluation for AF with an outpatient rhythm monitor may yield a new diagnosis of AF in almost one third of patients. Given the high risk of recurrent stroke associated with AF, additional outpatient evaluation is warranted. The various options available for monitoring include mobile outpatient cardiac telemetry, 24-hour electrocardiographic monitoring, transtelephonic and event monitors, and implantable subcutaneous devices. Longer monitoring results in a higher diagnostic yield. In one study comparing 30-day monitoring to 24-hour monitoring, atrial fibrillation lasting for at least 30 seconds was found in 16.1% of patients monitored for 30 days vs. 3.2% of patients monitored for 24 hours, and AF lasting for at least 2.5 minutes was found in 9.9% of patients with prolonged monitoring vs. 2.5% of patients with 24-hour monitoring. The combination of clopidogrel and aspirin is associated in the long term with a higher risk of hemorrhagic complications compared with a single antiplatelet agent only. A 2018 systematic review and meta-analysis showed that dual antiplatelet therapy with clopidogrel and aspirin given within 24 hours after high-risk TIA or minor ischemic stroke and continued for 10 to 21 days reduced subsequent nonfatal stroke risk but had no effect on all-cause mortality. It can be considered in patients who do not have a known cardiac source that requires anticoagulation. Aspirin should be continued following dual antiplatelet therapy for long-term secondary prevention of stroke. Apixaban and similar anticoagulants have not been shown to be effective for the routine prevention of cryptogenic stroke. Warfarin and aspirin have similar rates of recurrent stroke in patients with noncardioembolic stroke; warfarin is not indicated as first-line therapy for stroke prevention in cryptogenic stroke. Transesophageal echocardiography is unnecessary because the patient is in sinus rhythm and already has had structural imaging of the valves and chamber sizes, which makes a transesophageal echocardiogram likely to be of low yield. This diagnostic test also is invasive and costly and is of higher yield when used to evaluate for unusual causes of cardiac emboli, such as valvular endocarditis or intracardiac tumors. Read Related TextNext Question
A physician has noticed that a 67-year-old colleague in the medical practice has been increasingly forgetful in recent months. The colleague has had more difficulty remembering the names of her new patients and the medical students who are doing rotations in the practice, and she has also frequently missed meetings because of scheduling mix-ups. She admits that she cannot keep track of all of the new medications for diabetes mellitus without using a reference. The physician is unaware of the colleague's personal medical history but is concerned about her memory and cognitive status. Which of the following is the most appropriate management? Continue to monitor the colleague Directly approach the colleague and help her plan for confidential evaluation Offer to confidentially evaluate the colleague Report the colleague to the state medical board
Every physician is responsible for protecting patients from an impaired colleague and for assisting an impaired colleague by identifying appropriate sources of help. The most appropriate management is to directly approach the colleague with the concerns of impairment and guide her through a plan to determine whether impairment exists and, if so, how to manage it. Impairment may be caused by medical or psychiatric illness or the use of psychoactive substances; however, the presence of these conditions does not necessarily signify impairment. According to the American College of Physicians Ethics Manual, every physician is responsible for protecting patients from an impaired colleague and for assisting an impaired colleague by identifying appropriate sources of help. These responsibilities should not be hampered by personal relationships, shame, or fear of harming a colleague. This physician's colleague, who may be demonstrating signs of cognitive impairment, would benefit from assistance. When signs of physician impairment are present, it is imperative to intervene before patient harm occurs, if possible. Therefore, continuing to monitor the colleague is not the best option. Although the physician may be tempted to offer to perform the colleague's medical evaluation, several risks are associated with caring for patients with whom a previous relationship exists, including impaired objectivity, insufficient history taking (for example, sexual history), incomplete examination, and incomplete or biased assessment. The colleague should undergo a confidential and complete medical assessment with another physician to search for reversible causes of impairment. There is a clear ethical responsibility to report a physician who appears to be impaired to an appropriate authority, which may include a chief of staff, chief of service, or, if the impairment is serious, a state medical board. The legal requirements and thresholds for reporting impaired physicians vary. Most state medical societies or medical boards have physician health programs for physicians with potentially impairing illnesses that can be accessed through a voluntary or mandatory track; these physician health programs provide confidential assessment and treatment, with the goal of restoration of function. At this point, there is no objective evidence of impairment nor does patient harm seem likely, and a confidential evaluation would be the most appropriate first step
A 40-year-old man is evaluated for a firm, flesh-colored lesion that has been growing on the back for several months. Foul-smelling material can be expressed from the lesion. The patient is bothered by the periodic drainage. He is otherwise healthy and takes no medications. On physical examination, vital signs are normal. There is a 3-cm subepidermal nodule with a central punctum on the upper back. There is no erythema or warmth of surrounding skin. Which of the following is the most appropriate treatment for this patient? Excision Incision Incision and drainage Oral antibiotics
Excision is the most appropriate treatment for bothersome epidermal inclusion cysts since the epithelial lining must be removed for complete treatment. This patient has an epidermal inclusion cyst (sometimes called epidermoid cyst), and the most appropriate treatment is excision. The diagnosis of epidermal inclusion cysts is clinical. Epidermal inclusion cysts range from a few millimeters to several centimeters in size and are subepidermal, freely movable nodules with an epithelial lining and a core of accumulated keratin debris. They are most commonly found on the trunk. Epidermal inclusion cysts are often incorrectly referred to as "sebaceous cysts," a term that is erroneous because they possess keratin rather than sebum. Often they have a central punctum by which foul-smelling material can be expressed; the odor is derived from the presence of anaerobic bacteria. Epidermal inclusion cysts are benign but are frequently removed if they become bothersome to the patient. In this case, treatment is by excision with removal of the entire cyst wall. When epidermal inclusion cysts rupture, they may become tender, inflamed, and occasionally infected. Ruptured cysts are often mistaken for furuncles or infectious abscesses. Ruptured cysts are treated with incision and drainage, and intralesional glucocorticoid injections may be used to reduce the inflammation. Incision, as well as incision with drainage, can lead to temporary improvement of an inflamed epidermal cyst; however, recurrences are common as the epithelial lining must be removed for complete treatment. Therefore, these are not the preferred treatment. Antibiotics are rarely required; they should only be used when there is concern for secondary infection.
A 65-year-old woman is evaluated during a follow-up visit for breast cancer management. She was diagnosed 6 years ago with stage IIB left breast cancer. Pathologic findings showed a 2.5-cm, grade 3 infiltrating ductal cancer that was estrogen receptor positive, progesterone receptor positive, and HER2 negative. Three axillary lymph nodes were positive. She received adjuvant chemotherapy followed by 5 years of anastrozole. She takes no other medications. She has had no side effects from the anastrozole, and her bone density has remained normal. On physical examination, vital signs are normal. There are well-healed incisions of the left breast and axilla. There are no breast masses. Which of the following is the most appropriate management? Continue anastrozole; add tamoxifen for an additional 5 years Continue anastrozole alone for an additional 5 years Stop anastrozole Stop anastrozole; begin tamoxifen for an additional 5 years
Extending aromatase-inhibitor therapy beyond 5 years to 10 years in postmenopausal women with hormone receptor-positive early breast cancer is associated with an increase in the 5-year disease-free survival rate but not overall survival. The most appropriate management for this patient with early-stage breast cancer is to continue anastrozole for 5 years. In the North America Breast Cancer Group MA.17R trial, 1918 postmenopausal women with hormone receptor-positive early breast cancer who had completed 5 years of an aromatase inhibitor were randomly assigned to 5 years of letrozole or placebo. Most of the women had also received 5 years of tamoxifen before starting aromatase inhibitor therapy. The 5-year disease-free survival rate was 95% with letrozole versus 91% with placebo. Overall survival was the same in both groups. Patients on letrozole had a higher incidence of bone pain, bone fractures, and new-onset osteoporosis. This patient has tolerated anastrozole therapy well, has preserved bone density, and remains at higher risk (tumor size greater than 2 cm and pathologically involved lymph nodes). Extending aromatase inhibitor therapy to 10 years is reasonable for this patient. Tamoxifen can be used sequentially with an aromatase inhibitor during the first 5 years of hormonal therapy. Studies have demonstrated equivalent results between sequential therapy (2 years of either tamoxifen or an aromatase inhibitor followed by 3 years of the other agent) compared with 5 years of an aromatase inhibitor. However, no data show benefit of tamoxifen, either as a single agent or in combination with an aromatase inhibitor, after completing 5 years of an aromatase inhibitor.
A 29-year-old man is evaluated in the emergency department for a 3-week history of headaches. He reports a painful burning sensation in his toes and feet for the past few years, particularly after he exercises at the gym, and states that he does not sweat as much after exercise compared with his peers. He takes no medications. Family history is notable for the following: His maternal grandfather and maternal granduncle had similar burning sensations in their feet for years and died from strokes in their early 40s; and his mother has occasional burning sensations in her feet as well as corneal dystrophy. On physical examination, blood pressure is 160/95 mm Hg; other vital signs are normal. Numerous angiokeratomas over the sternal area are present. Reduced pain and temperature sensation in the lower extremities bilaterally is noted. Laboratory studies show a blood urea nitrogen level of 60 mg/dL (21.4 mmol/L) and a serum creatinine level of 4.1 mg/dL (362.4 µmol/L); urinalysis shows 2+ blood and 3+ protein. Kidney ultrasound shows increased echogenicity in bilateral kidneys. Which of the following is the most likely diagnosis? Fabry disease Hereditary nephritis Medullary cystic kidney disease Tuberous sclerosis complex
Fabry disease should be considered as a cause of chronic kidney disease of unknown etiology in young adulthood. The most likely disease is Fabry disease, an X-linked recessive inborn error of glycosphingolipid metabolism caused by deficiency of α-galactosidase A. The enzyme deficiency leads to defective storage of sphingolipid and progressive endothelial accumulation, causing abnormalities in the skin, eye, kidney, heart, brain, and peripheral nervous system. Typically, the disease begins in childhood with episodes of pain and burning sensations in the hands and feet. These painful episodes can be brought on by exercise, fever, fatigue, or other stressors. In addition, young patients often develop angiokeratomas (violaceous papules with overlying scale), decreased perspiration, and corneal and lens opacities of the eyes. The disease is progressive, and symptoms of kidney, heart, and/or neurologic involvement usually occur between the ages of 30 and 45 years. As an X-linked disorder, males who inherit a mutation in the gene responsible for α-galactosidase always display the disease phenotype. Females, on the other hand, who inherit only one copy of a disease-causing mutation, can show a wide range of clinical manifestations, ranging from asymptomatic carriers to severe heart and kidney failure no different from a hemizygous male's phenotype. This variability is likely due to varying degrees of random inactivation of one copy of the X chromosome in each cell (lyonization). The most common finding of Fabry disease seen in heterozygous females is corneal dystrophy, which occurs in more than half of females. Fabry disease should be considered as a cause of chronic kidney disease of unknown etiology in young adulthood, especially when there is a family history of early end-stage kidney disease or cardiovascular-related death (via myocardial infarction or cerebrovascular accident). Diagnosis can be made via kidney biopsy but also noninvasively with measurement of leukocyte enzymatic activity and subsequent genetic confirmation. Screening for the disease is recommended for family members of affected patients. Enzyme replacement therapy with recombinant human α-galactosidase A is available. Hereditary nephritis (Alport syndrome), like Fabry disease, can present with kidney failure if diagnosed late but is not associated with abnormalities in the skin or peripheral nervous system. Medullary cystic kidney disease is a familial form of kidney disease with no known extrarenal symptomatology and usually presents with bland urinary findings. This degree of kidney failure in a patient with tuberous sclerosis complex would be associated with abnormal kidney imaging, such as angiomyolipomas and cysts, and the classic skin lesions in this condition are hamartoma formations, not angiokeratomas. Read Related TextNext Question
A 78-year-old woman is evaluated for frequent rectal urgency with the passage of explosive, loose to watery stool. She says she is often unable to get to the bathroom in time. Episodes have occurred after meals and in the early morning over the past 3 months. She has Alzheimer dementia and lives with her daughter. She also has hypertension and chronic back pain. Her history includes occasional constipation that resolves when treated with docusate. Her last screening colonoscopy at age 70 years showed diffuse diverticulosis. Her medications are lisinopril, acetaminophen, memantine, docusate as needed, and a calcium supplement with vitamin D. On physical examination, vital signs are normal. Fecal soiling is noted. Rectal examination shows normal anal tone and brown stool in the rectal vault. A complete blood count and thyroid-stimulating hormone level are normal. Which of the following is the most appropriate next step in management? Abdominal radiograph Anorectal manometry Loperamide Psyllium
Fecal loading (excess stool in the colon) with resultant overflow diarrhea is a common cause of fecal incontinence in elderly patients, particularly those who are hospitalized or have degenerative neurologic disorders.
A 78-year-old woman is evaluated for frequent rectal urgency with the passage of explosive, loose to watery stool. She says she is often unable to get to the bathroom in time. Episodes have occurred after meals and in the early morning over the past 3 months. She has Alzheimer dementia and lives with her daughter. She also has hypertension and chronic back pain. Her history includes occasional constipation that resolves when treated with docusate. Her last screening colonoscopy at age 70 years showed diffuse diverticulosis. Her medications are lisinopril, acetaminophen, memantine, docusate as needed, and a calcium supplement with vitamin D. On physical examination, vital signs are normal. Fecal soiling is noted. Rectal examination shows normal anal tone and brown stool in the rectal vault. A complete blood count and thyroid-stimulating hormone level are normal. Which of the following is the most appropriate next step in management? Abdominal radiograph Anorectal manometry Loperamide Psyllium
Fecal loading (excess stool in the colon) with resultant overflow diarrhea is a common cause of fecal incontinence in elderly patients, particularly those who are hospitalized or have degenerative neurologic disorders. An abdominal radiograph is the most appropriate next step in management of this patient. Fecal incontinence is the involuntary loss of stool. Prevalence increases with age and is 16% in adults older than 70 years. Urge fecal incontinence is the inability to postpone defecation. This patient's explosive diarrhea is the likely source of her urge fecal incontinence. Before treating the diarrhea, it is essential to determine whether the diarrhea is due to overflow from fecal loading (excess stool in the colon). An abdominal radiograph is a simple, safe, and inexpensive diagnostic test. Fecal loading with resultant overflow diarrhea is a common cause of fecal incontinence in elderly patients, particularly those who are hospitalized or have degenerative neurologic disorders. This patient has several risk factors for developing fecal loading, including Alzheimer dementia, history of constipation, and use of constipating medications (memantine and calcium). Anorectal manometry is useful in the diagnostic evaluation of fecal incontinence to help assess for anal sphincter weakness, rectal hypersensitivity, and/or dyssynergic defecation. Due to its limited availability, cost, and logistical considerations for the patient, its use is reserved for patients with rectal findings suggestive of anal sphincter weakness or dyssynergia, or patients with a lack of response to initial treatment measures. A trial of loperamide may alleviate this patient's diarrhea in the short term; however, loperamide use will add to the underlying fecal loading, causing her diarrhea and fecal incontinence to worsen over time. Use of psyllium is unlikely to alleviate the patient's symptoms and would add stool bulk to an already overloaded colon, which would likely worsen her urgency, explosive diarrhea, and fecal incontinence. Furthermore, the patient is likely to experience bloating and abdominal distention caused by the fermentation of the psyllium in the gastrointestinal tract.
Complications of Felty Syndrome?
Felty syndrome consists of the triad of long-standing aggressive rheumatoid arthritis, neutropenia, and splenomegaly and is associated with the risk for serious infections, lower extremity ulcers, lymphoma, and vasculitis.
A 35-year-old woman is evaluated in the emergency department for a 3-week history of progressive dyspnea, fatigue, chest "fullness," and bilateral peripheral edema. Her medical history is notable for systemic lupus erythematosus manifested by malar rash, polyarticular arthritis, and previous pleural effusion. Her medications are hydroxychloroquine, ibuprofen, and prednisone. On physical examination, temperature is 38.1 °C (100.6 °F), blood pressure is 108/68 mm Hg with pulsus paradoxus of 18 mm Hg, pulse rate is 160/min, and respiration rate is 18/min. Oxygen saturation is 97% breathing ambient air. Jugular venous distention to the angle of the mandible is present. Heart sounds are soft. No rubs, murmurs, or diastolic sounds are present. Pitting edema is noted bilaterally to above the ankle. A chest radiograph and electrocardiogram are shown. (CXR- cardiomegaly; EKG- electrical alternans) Which of the following is the most likely diagnosis? Cardiac tamponade Constrictive pericarditis Pulmonary embolism Pulmonary hypertension
Findings of congestion, hypotension, pulsus paradoxus, enlarged cardiac silhouette on chest radiograph, and electrical alternans on electrocardiogram support the diagnosis of cardiac tamponade. The most likely diagnosis is cardiac tamponade. Pericarditis is a common cardiac manifestation of systemic lupus erythematosus (SLE), occurring in up to 40% of patients with SLE. Pericarditis may be associated with a neutrophilic pericardial effusion that can rarely lead to tamponade. In this patient, progressive dyspnea, fatigue, chest fullness, and peripheral edema are symptoms consistent with significant cardiac tamponade, and the physical examination findings of jugular venous distention and pulsus paradoxus (fall in systolic pressure of >10 mm Hg with inspiration) are supportive. The chest radiograph demonstrates an enlarged cardiac silhouette ("water-bottle heart"), and the electrocardiogram demonstrates electrical alternans, which may represent swinging of the heart within a large pericardial effusion. This patient also has supraventricular tachycardia. The pericardium in constrictive pericarditis is rigid and noncompliant, resulting in a total cardiac volume that is largely fixed. Ventricular filling occurs rapidly in early diastole and terminates abruptly near mid-diastole owing to the pericardial restraint. The jugular venous pressure is elevated in nearly all patients, with prominent x and y descents. Physical findings that also may be present include a pericardial knock, pulsus paradoxus, pleural effusion, congestive hepatomegaly, and peripheral edema or ascites. In patients with long-standing constrictive pericarditis, hepatic failure and cirrhosis may be present. Pericardial constriction would not lead to a large cardiac silhouette or electrical alternans, as seen in this patient. Patients with SLE have increased risk for venous thromboembolic disease. The electrocardiogram in acute pulmonary embolism most commonly demonstrates sinus tachycardia but may also show a new complete or incomplete right bundle branch block or an S1Q3T3 pattern (prominent S wave in lead I, Q wave in lead III, and inverted T wave in lead III). An enlarged globular heart on chest radiograph and electrical alternans on electrocardiogram are not features. Patients with SLE may have pulmonary thromboembolic disease, pulmonary veno-occlusive disease, or advanced interstitial lung disease with hypoxemia leading to pulmonary hypertension; however, severe symptomatic pulmonary hypertension is more commonly associated with scleroderma or mixed connective tissue disease. An enlarged globular heart on chest radiograph would be unlikely, and the electrocardiogram is likely to demonstrate a right axis deviation and right ventricular hypertrophy but not electrical alternans.
Tx for glucocorticoid-dependent minimal change glomerulopathy?
For frequently relapsing or glucocorticoid-dependent disease, treatment options include cyclophosphamide, calcineurin inhibitors (tacrolimus or cyclosporine), mycophenolate mofetil, and rituximab. In addition to immunosuppression, patients should receive standard therapy for the nephrotic syndrome, including an ACE inhibitor or angiotensin receptor blocker (this patient is already taking lisinopril, with well-controlled blood pressure), diuretics for edema management, and cholesterol-lowering medication if total cholesterol >200 mg/dL (5.1 mmol/L). In rare cases of MCG secondary to malignancies (Hodgkin lymphoma, non-Hodgkin lymphoma, thymoma), medications (NSAIDs, lithium), infections (strongyloides, syphilis, mycoplasma, ehrlichiosis), and atopy (pollen, dairy products), treatment of the underlying condition without immunosuppression may be sufficient.
An 18-year-old woman is evaluated for a 2-year history of repetitive fast movements of the neck that tilt the head to the right side. She reports that these movements are preceded by a feeling of discomfort in the right shoulder and that if she concentrates and taps the right foot immediately after this sensory cue, she usually can avoid the neck movements. She is a senior in high school and says she has not been able to use this method successfully in the classroom, especially during examinations, when she often experiences a cluster of movements that exhaust and distract her. Her mother notes that before onset of the neck symptoms, the patient used to roll her eyes and clear her throat frequently. She has obsessive-compulsive disorder treated with sertraline. On physical examination, vital signs are normal. During the examination, the patient displays rapid tilting movements of the head followed by rolling of the head and shoulder; these movements repeat several times in a stereotyped manner. The patient can suppress these movements on request. Which of the following is the most appropriate treatment? Botulinum toxin injection Clonidine administration Cognitive behavioral therapy Haloperidol administration
First-line agents used to treat Tourette syndrome when the associated tics interfere with education, daily function, or work are clonidine, guanfacine, topiramate, and tetrabenazine. This patient should be treated with clonidine. The suppressible stereotyped neck movements and premonitory sensory cues are consistent with motor tics, and the history of other motor (eye rolling) and vocal (throat clearing) tics, childhood onset, duration of greater than 1 year, and comorbidity of obsessive-compulsive disorder (OCD) are consistent with the diagnosis of Tourette syndrome. Clonidine is a first-line agent used to treat Tourette syndrome. Anti-tic medications should be considered when tics interfere with education, daily function, or work. Other first-line medications include guanfacine, topiramate, and the dopamine-depleter agent tetrabenazine. Second-line treatments include antipsychotic agents (such as haloperidol), but their benefit should be weighed against risk of tardive dyskinesia. Although botulinum toxin can be considered as an off-label option in the treatment of severe refractory cervical tics, this patient has not yet tried medication to control her tics. Botulinum toxin also is indicated in the treatment of cervical dystonia, but this disorder is not consistent with her clinical findings of suppressibility and the absence of sustained motor activity. Treatment of mild tics includes reassurance, treatment of psychiatric comorbidities (such as the sertraline she already takes for OCD), and cognitive behavioral therapy to teach patients about tic diversion techniques (such as the foot taps she initiates after sensory premonitory cues). Given that this patient's symptoms have persisted, pharmacologic therapy for Tourette syndrome is appropriate. Haloperidol should not be considered before a trial of first-line anti-tic medications is attempted. Clonidine appears to be better tolerated than antidopaminergic drugs, such as haloperidol or risperidone. Additionally, the tardive dyskinesia that can result from neuroleptic agents may take months to years to resolve after discontinuation of the culprit drug.
A 47-year-old man is evaluated for gradually progressive exertional dyspnea and two episodes of self-limited atrial fibrillation during the past 6 months. The atrial fibrillation was attributed to alcohol consumption, and no additional testing was performed. Medical history is otherwise noncontributory. He takes no medications. On physical examination, vital signs are normal. The estimated central venous pressure is elevated. A parasternal impulse is noted at the left sternal border. Persistent splitting of the S2 is present. A grade 2/6 systolic murmur is heard at the second left intercostal space, with a separate grade 3/6 holosystolic murmur at the apex. The remainder of the examination is normal. An electrocardiogram is shown. Which of the following is the most likely diagnosis? Coronary sinus atrial septal defect (ASD) Ostium primum ASD Ostium secundum ASD Sinus venosus ASD
Fixed splitting of the S2, a mitral regurgitation murmur, and left-axis deviation on electrocardiogram are consistent with an ostium primum atrial septal defect. This patient's examination findings and electrocardiogram (ECG) are consistent with a diagnosis of ostium primum atrial septal defect (ASD). The patient has clinical features of ASD, presenting with dyspnea, paroxysmal atrial fibrillation, and features of right heart volume overload with elevation of the central venous pressure and a right ventricular lift. A pulmonary midsystolic flow murmur is caused by increased flow from a large left-to-right shunt. Fixed splitting of the S2 throughout the cardiac cycle is a characteristic clinical feature of ASD. The apical systolic murmur of mitral regurgitation is related to the mitral valve cleft. This combination of fixed splitting of the S2, mitral regurgitation murmur, and left-axis deviation on ECG are most consistent with an ostium primum ASD. Patients with a coronary sinus ASD have features of right heart volume overload but do not have a mitral regurgitation murmur because no mitral valve disease is present. The ECG may be normal or demonstrate first-degree atrioventricular block and incomplete right bundle branch block. Patients with ostium secundum ASD have right heart volume overload but do not generally have mitral regurgitation. The ECG may demonstrate first-degree atrioventricular block and incomplete right bundle branch block, or it may be normal. This patient's ECG finding of left axis deviation is not seen in patients with ostium secundum ASD. Patients with sinus venosus ASD have features of right heart volume overload but do not have a mitral regurgitation murmur because no mitral valve disease is present. The ECG may be normal or demonstrate first-degree atrioventricular block, left P-wave axis or abnormal P-wave axis, and incomplete right bundle branch block, but left-axis deviation is not expected.
An 82-year-old woman is evaluated for persistent swellings in her neck, armpit, and groin. She describes no additional symptoms. She has long-standing systolic hypertension for which she takes chlorthalidone. On physical examination, blood pressure is 154/82 mm Hg. The other vital signs are normal. There are bilateral enlargements of cervical, supraclavicular, axillary, and inguinal lymph nodes. The remainder of the examination is unremarkable. Laboratory studies: Hemoglobin 12.5 g/dL (125 g/L) Leukocyte count 14,000/µL (14 × 109/L) with 66% lymphocytes, 23% segmented neutrophils, 6% monocytes, 1% eosinophils, and 4% bands Platelet count 174,000/µL (174 × 109/L) Smudge cells are noted on a peripheral blood smear. Which of the following is the most appropriate diagnostic test to perform next? Bone marrow aspiration and biopsy CT scan of the chest, abdomen, and pelvis Cytogenetic studies on peripheral blood Flow cytometry on peripheral blood
Flow cytometry on peripheral blood is the most appropriate test to make an initial diagnosis of chronic lymphocytic leukemia.
An 82-year-old woman is evaluated for persistent swellings in her neck, armpit, and groin. She describes no additional symptoms. She has long-standing systolic hypertension for which she takes chlorthalidone. On physical examination, blood pressure is 154/82 mm Hg. The other vital signs are normal. There are bilateral enlargements of cervical, supraclavicular, axillary, and inguinal lymph nodes. The remainder of the examination is unremarkable. Laboratory studies: Hemoglobin 12.5 g/dL (125 g/L) Leukocyte count 14,000/µL (14 × 109/L) with 66% lymphocytes, 23% segmented neutrophils, 6% monocytes, 1% eosinophils, and 4% bands Platelet count 174,000/µL (174 × 109/L) Smudge cells are noted on a peripheral blood smear. Which of the following is the most appropriate diagnostic test to perform next? Bone marrow aspiration and biopsy CT scan of the chest, abdomen, and pelvis Cytogenetic studies on peripheral blood Flow cytometry on peripheral blood
Flow cytometry on peripheral blood is the most appropriate test to make an initial diagnosis of chronic lymphocytic leukemia. This patient's elevated lymphocyte count and the identification of smudge cells on the peripheral blood smear, together with the presence of lymphadenopathy on physical examination, point to a possible diagnosis of chronic lymphocytic leukemia (CLL). B-cell CLL is the most common form of adult leukemia, accounting for 10% of hematologic malignancies. Patients are usually asymptomatic at presentation, with CLL identified by a lymphocytosis on a routine complete blood count. Flow cytometry is the most appropriate test to make an initial diagnosis of CLL. It can be easily performed on peripheral blood and will show B-cell antigens (CD19, CD20, and CD23), coexpression of CD5 (usually a T-cell marker), and low levels of a monoclonal surface immunoglobulin. In most cases, CLL can be suspected based on the complete blood count and review of a peripheral blood smear. However, other conditions may present with lymphocytosis, which flow cytometry can help distinguish from typical CLL. Other conditions include reactive lymphocytosis, T-cell leukemias, and other B-cell lymphoproliferative disorders involving the blood, such as mantle cell and marginal zone lymphomas and hairy cell leukemia. A bone marrow aspiration and biopsy can be used to diagnose CLL and will provide prognostic information based on the extent of marrow involvement. However, flow cytometry has replaced bone marrow examination, which is not routinely needed as part of diagnosing or staging the disease, although it may still be useful in evaluating atypical cases or the mechanisms of cytopenia in patients with CLL. A CT scan may show lymphadenopathy or nonpalpable splenomegaly in patients with CLL; however, it does not provide additional information that is helpful in the management of disease in asymptomatic patients. Prognosis for patients with CLL is determined by gene mutation status (immune globulin variable heavy-chain mutation) and fluorescence in situ hybridization or array-based karyotyping. In patients with CLL, a deletion of chromosome 17p is concerning because it is associated with a short median survival (less than 3 years). These cytogenetic and molecular genetic studies may have implications in choice of therapies. However, these studies would not typically be ordered until a definitive diagnosis of CLL is obtained.
A 19-year-old woman is evaluated for a 6-month history of recurrent episodes of confusion that occur approximately once monthly. Her boyfriend says she has periods of wide-eyed staring, chewing motions, and repetitive grabbing of her clothes with the right hand. The patient sometimes experiences a strange but familiar feeling before the episodes but does not remember the episodes themselves, which last approximately 45 to 60 seconds and are followed by exhaustion and sleepiness for 20 minutes. All physical examination findings are normal. Which of the following is the most likely diagnosis? Absence seizures Atonic seizures Focal seizures Myoclonic seizures
Focal seizures with altered awareness typically are infrequent, are associated with warning symptoms (aura), last more than 30 seconds, have associated mouth or limb automatisms (semipurposeful repetitive movements), and are followed by confusion and/or exhaustion. The patient is experiencing focal seizures with altered awareness (formerly known as complex partial seizures or focal dyscognitive seizures). These seizures typically are infrequent, are associated with "warning" symptoms (aura, which may consist of an epigastric rising sensation or a feeling of déjà vu), last more than 30 seconds, have associated mouth or limb automatisms (semipurposeful repetitive movements), and are followed by confusion and/or exhaustion. Patients who experience this type of seizure often have no memory of the episode itself. Absence seizures may also present with staring and confusion but typically are more frequent (occurring multiple times per day), last less than 15 seconds, and are associated with immediate recovery, to the point that patients and witnesses may not realize a seizure has occurred. Absence seizures are most characteristic of childhood absence epilepsy, which typically resolves by puberty but also can occur in adults with idiopathic generalized epilepsy syndromes. Atonic seizures involve the abrupt loss of muscle tone and typically are associated with falling down and a brief loss of consciousness lasting only a few seconds. Atonic seizures are one cause of "drop attacks" in which the patient will suddenly, and without warning symptoms, drop to the ground. Individuals with drop attacks can get themselves up and typically will deny loss of consciousness. Other causes of drop attacks include cataplexy, vertebrobasilar transient ischemic attack, and vestibular pathologies. The patient's seizures do not match the description of atonic seizures. Myoclonic seizures generally consist of a single jerk of the entire body, usually last less than 1 second, and are associated with retained awareness and no postictal confusion. In contrast, this patient's seizures last at least 45 seconds and are characterized by staring, chewing motions, repetitive grabbing of her clothes, and a 20-minute postseizure period of exhaustion and sleepiness.
A 56-year-old man is brought to the emergency department after being found lying unresponsive in the local train station. Medical history is significant for chronic alcohol dependence. He is homeless. The patient is a frequent visitor to the emergency department for minor trauma and ailments; his last visit was 6 months ago. Until that time, the patient lived in various shelters and received at least one nutritious meal per day. His whereabouts and living circumstances since that time are unknown. His medical history is otherwise not significant, and at his last visit to the emergency department, he was taking no medications. On physical examination, vital signs are normal. The patient is disheveled, cachectic, and malodorous. He moans in response to painful stimuli and moves all extremities. He has poor dentition. Hepatomegaly is noted. Laboratory studies: Hemoglobin 7.4 g/dL (74 g/L) Leukocyte count 4200/µL (4.2 × 109/L) Mean corpuscular volume 110 fL Platelet count 97,000/µL (97 × 109/L) Reticulocyte count 1% of erythrocytes Blood alcohol level 500 mg/dL (108 mmol/L) Hypersegmented neutrophils are seen on the peripheral blood smear. Which of the following is the most likely cause of this patient's anemia? Cobalamin deficiency Folate deficiency Inflammatory anemia Iron deficiency
Folate deficiency should be suspected in patients with macrocytic anemia, malnutrition, and alcohol dependence. The most likely diagnosis is folate deficiency. The patient has a macrocytic anemia associated with malnutrition and presumed decreased consumption of folate-rich foods and chronic alcoholism. A major source of folate is grains and green leafy vegetables, but folate absorption is decreased in the presence of chronic alcoholism. This patient is at risk for folate deficiency based on his homelessness, cachetic appearance suggesting malnutrition, and chronic alcoholism. Folate is not efficiently stored, and folate deficiency can develop within weeks of inadequate intake. Measuring the folate level is often of limited utility because of the ease and safety of initiating folate replacement. This patient should receive supplemental folate in addition to mental health and social service support. Cobalamin deficiency will also produce a macrocytic anemia with hematologic features identical to that seen in folate deficiency. Unlike folate, cobalamin is effectively stored, and dietary inadequacy is not a common cause of deficiency. Alcohol dependence is not a risk factor for cobalamin deficiency. Although cobalamin deficiency is a conceivable cause of anemia in this patient, folate deficiency is much more likely. Inflammatory anemia is caused by underlying proinflammatory states, such as infection, cancer, and autoimmune disease, and other conditions not traditionally characterized as proinflammatory, including chronic heart failure and diabetes mellitus. It is associated with a normocytic or microcytic anemia, not with macrocytosis or hypersegmented neutrophils as seen in this patient. The hallmark of iron deficiency is a microcytic hypochromic anemia. Macrocytosis and hypersegmented neutrophils are not seen, making iron deficiency an unlikely diagnosis in this patient. Read Related TextNext Question
A 58-year-old woman was evaluated for dark brown pigmentation on her thumb nail. She noticed it was getting darker and wider. She is otherwise healthy and takes no medications. On physical examination, vital signs are normal. The other nails are uninvolved. Nail findings are shown. The remainder of the examination is normal. Which of the following is the most likely diagnosis? Lichen planus Melanonychia Normal aging Onychomycosis
Melanonychia is a longitudinal brown pigmentation of the nail plate; it can be a normal variant in persons with darker skin types, but it may also occur as a result of systemic disease, medication, infection, or an underlying melanocytic lesion. This patient has melanonychia, a brown longitudinal pigmentation of the nail plate. It can be a normal variant in persons with darker skin types, but it may also occur as a result of systemic disease, medication, infection, or an underlying melanocytic lesion such as malignant melanoma. It is extremely difficult to clinically differentiate benign melanonychia from acral melanoma involving the nail. Certain findings may suggest a stronger likelihood of melanoma and the need for biopsy. These include: melanonychia that first appears in adulthood; involvement of a single digit (especially thumb or great toe); rapid growth; width greater than 3 to 4 mm; proximal width greater than distal width; variation in pigmentation; secondary nail changes such as splitting or dystrophy; pigmentation that extends beyond the nail fold; or personal history of melanoma. This patient has longitudinal melanonychia limited to the thumb that is changing in width. Furthermore, in this photo, there is a pigmented macule on the lateral nailfold that is called a Hutchinson sign. This is an important clinical clue for subungual melanoma. A biopsy should be performed. Lichen planus can develop in the nails. The nail plate will become thinner or be completely destroyed, and the cuticle may attach to the nail plate (pterygium). Other findings may include longitudinal ridging, nail plate thinning, and longitudinal fissuring. Red streaking of the nail may also occur. Patients may have other cutaneous or mucosal findings of lichen planus. This patient lacks the dystrophic changes commonly associated with lichen planus. With aging, the thickness, curvature, surface, and color of the nail plate can change. The underlying mechanisms are unknown. The nail plate may become thicker or thinner. The normal texture of the nail can become rougher and more friable, resulting in striations and splitting. The color of the nail may become paler or yellow. These changes may predispose nails to onychomycosis, pain, and subungual hemorrhage. Normal aging is not associated with the occurrence of a growing, longitudinal stripe in one nail. Onychomycosis is a fungal infection of the nail. The most common pattern is characterized by the distal corner of the nail becoming yellow and lifted, with the development of subungual debris. Eventually this process can spread proximally and laterally to involve the entire nail plate. Onychomycosis does not result in a well-defined dark brown longitudinal stripe as seen in this patient. Read Related TextNext Question
A 25-year-old woman is evaluated for a 2-month history of joint pain and swelling involving the hands, knees, and feet. On physical examination, vital signs are normal. Swelling is noted at the second and third metacarpophalangeal joints of both hands, the left wrist, both knees, and the second through fifth metatarsophalangeal joints of both feet. Laboratory studies are positive for rheumatoid factor and anti-cyclic citrullinated peptide antibodies. Radiographs of the hands and feet are normal. In addition to methotrexate and short-term prednisone, which of the following should be initiated? Folic acid Niacin Thiamine Vitamin C
Folic acid supplementation should be initiated in patients beginning therapy with methotrexate to reduce the risk of side effects and discontinuation of methotrexate. In addition to methotrexate, folic acid, 1 mg/d, should be initiated in this patient with rheumatoid arthritis (RA). RA is characterized by a chronic inflammatory polyarthritis affecting large and small joints with a predilection for the small joints of the hands and feet. Patients with RA typically report joint pain and inflammatory symptoms, including swelling and morning stiffness often lasting several hours. Joint swelling (softness or bogginess of the affected joint) is palpable on joint examination. Glucocorticoids, such as prednisone, act rapidly to control inflammation and joint symptoms, and can be useful until slower-acting disease-modifying antirheumatic drugs (DMARDs) achieve full effect. DMARD treatment typically begins with weekly administration of methotrexate, which is the anchor drug in RA and is used in both monotherapy and combination therapy. Daily folic acid supplementation with 1 mg (or weekly folinic acid supplementation) has been found to reduce the mucosal, hematologic, hepatic, and gastrointestinal side effects of methotrexate. Folic acid supplementation also reduces discontinuation of methotrexate for any reason. Methotrexate blocks the cellular utilization of folic acid, and folate depletion is considered to be the cause of most of the side effects associated with methotrexate therapy. Supplementation with folic acid reduces the incidence of side effects without any loss of methotrexate efficacy in treating RA. Folic acid can be taken on the same day as methotrexate because folic acid and methotrexate enter the cell via different pathways. Folinic acid is the reduced form of folate and is typically reserved for patients who have not had a satisfactory response to folic acid. Folinic acid is considerably more expensive than folic acid, and proper timing and administration of folinic acid is complex. Niacin, thiamine, and vitamin C have no known utility in treating RA or preventing methotrexate side effects.
A 32-year-old woman is evaluated for bothersome vaginal discharge of 2 weeks' duration. Her last Pap smear was obtained 15 months ago and was normal. Nucleic acid amplification test results confirm Trichomonas vaginalis infection. Treatment of the patient is initiated. Which of the following is the most appropriate additional management? Pap testing Test sexual partner for trichomoniasis Treat sexual partner for trichomoniasis No further testing or intervention is necessary
Following diagnosis and treatment of a woman with Trichomonas vaginalis infection, the sexual partner should be treated and both individuals should be screened for other sexually transmitted infections; retesting of women for T. vaginalis infection within 3 months of treatment is also recommended. The most appropriate additional management is to treat the patient's sexual partner for trichomoniasis. Trichomoniasis, which is caused by Trichomonas vaginalis, is the most common nonviral sexually transmitted infection (STI) worldwide. Unlike other STIs that predominate in adolescents and younger adults, rates of trichomoniasis are evenly distributed among women of all age groups. It is caused by motile flagellated protozoa that infect the urogenital tract, causing inflammatory vaginitis and urethritis. Treatment with a single 2-g dose of metronidazole is associated with a high rate of cure and should be offered to all symptomatic women, including pregnant women. Because of the high rate of reinfection among women treated for trichomoniasis (17% within 3 months in one study), retesting for T. vaginalis is recommended by the Centers for Disease Control and Prevention (CDC) for all sexually active women within 3 months after initial treatment, regardless of whether they believe their sexual partners were treated. Testing by nucleic acid amplification can be conducted as soon as 2 weeks after treatment. It is important that sexual partners also be treated, even if they are asymptomatic; documentation of infection is not required before treatment in any partners. Data are insufficient to support retesting men after treatment for trichomoniasis. T. vaginalis infection is associated with a two- to threefold increased risk for HIV acquisition. Therefore, the CDC recommends that testing for other STIs, including HIV, be performed in persons infected with T. vaginalis. Pap testing is not needed for management of trichomoniasis because the infection is not associated with cervical malignancy. Because of the high rate of partner infection with T. vaginalis and its association with other STIs, it would be inappropriate to not provide further testing or intervention after a primary T. vaginalis diagnosis.
A 65-year-old man is evaluated after a positive stool antigen test for Helicobacter pylori infection obtained to confirm eradication after therapy. H. pylori gastritis was diagnosed in the setting of a duodenal ulcer. Four weeks ago, he completed a 10-day course of eradication therapy consisting of amoxicillin, clarithromycin, and omeprazole. He reports taking all medications as prescribed during treatment and reports no upper gastrointestinal symptoms or melena. The patient does not smoke cigarettes or drink alcohol. He has no known drug allergies. Which of the following is the most appropriate 14-day treatment regimen? Amoxicillin, clarithromycin, and omeprazole Amoxicillin, metronidazole, and omeprazole Bismuth, metronidazole, omeprazole, and tetracycline Clarithromycin, metronidazole, and omeprazole
For Helicobacter pylori infection that persists after eradication therapy, the salvage therapy regimen should consist of different antibiotics from those used in the initial, unsuccessful regimen.
A 65-year-old man is evaluated after a positive stool antigen test for Helicobacter pylori infection obtained to confirm eradication after therapy. H. pylori gastritis was diagnosed in the setting of a duodenal ulcer. Four weeks ago, he completed a 10-day course of eradication therapy consisting of amoxicillin, clarithromycin, and omeprazole. He reports taking all medications as prescribed during treatment and reports no upper gastrointestinal symptoms or melena. The patient does not smoke cigarettes or drink alcohol. He has no known drug allergies. Which of the following is the most appropriate 14-day treatment regimen? Amoxicillin, clarithromycin, and omeprazole Amoxicillin, metronidazole, and omeprazole Bismuth, metronidazole, omeprazole, and tetracycline Clarithromycin, metronidazole, and omeprazole
For Helicobacter pylori infection that persists after eradication therapy, the salvage therapy regimen should consist of different antibiotics from those used in the initial, unsuccessful regimen. Bismuth, metronidazole, omeprazole, and tetracycline for 14 days is the most appropriate treatment regimen for this patient. The salvage therapy regimen should consist of different antibiotics from those used in the initial, unsuccessful regimen. This strategy reduces the likelihood of antibiotic resistance, the major reason for treatment failure. Additional factors in choosing salvage therapy after initial treatment failure include a history of quinolone antibiotic use and penicillin allergy. If the initial unsuccessful treatment was clarithromycin triple therapy and the patient is allergic to penicillin, the best salvage therapy is a bismuth quadruple therapy (bismuth, metronidazole, omeprazole, and tetracycline). In patients with no penicillin allergy, but whose history includes quinolone antibiotic use for any reason, salvage therapies could include bismuth quadruple therapy, rifabutin triple therapy (rifabutin, penicillin, and a proton pump inhibitor [PPI] such as omeprazole) or high-dose dual therapy (amoxicillin and a PPI). Disruption of therapy can cause treatment failure, and the likelihood of treatment success diminishes with each successive treatment attempt. It is essential to counsel the patient carefully on the importance of treatment adherence and potential side effects of the therapy. Treatment with the same regimen (amoxicillin, clarithromycin, and omeprazole) for a longer period of time will be ineffective because the patient's Helicobacter pylori infection is likely resistant to clarithromycin and/or penicillin. The combination of amoxicillin, metronidazole, and omeprazole is not a recognized treatment regimen for H. pylori in any setting. Metronidazole is typically used in patients with penicillin allergy. Using metronidazole rather than amoxicillin is unlikely to result in H. pylori eradication because resistance to clarithromycin is also a likely cause of the initial treatment's failure. Furthermore, the regimen of clarithromycin, metronidazole, and omeprazole could lead to the development of metronidazole resistance in the likely event of treatment failure with this regimen.
A 60-year-old man is evaluated in the emergency department for headache, nausea, vomiting, and confusion lasting 4 hours. He ran out of his hypertensive medications a few days ago. Current medications are lisinopril, metoprolol succinate, hydrochlorothiazide, and aspirin. On physical examination, blood pressure is 230/140 mm Hg and pulse rate is 100/min. All other vital signs are normal. He is too uncooperative to perform a mental status examination or funduscopic examination. The cardiovascular examination is positive for an S4 but otherwise normal. Laboratory studies reveal normal electrolytes ; serum creatinine is 1.6 mg/dL (141.4 µmol/L). It was 1.2 mg/dL (106 µmol/L) at his last outpatient appointment. Electrocardiogram shows left ventricular hypertrophy and sinus tachycardia. Chest radiograph is normal. CT scan of the brain shows no acute findings. Which of the following is the most appropriate treatment? Intravenous hypertensive therapy to lower systolic blood pressure (SBP) to 160 mm Hg within the first 6 hours Intravenous hypertensive therapy to lower SBP to 120 mm Hg within the first hour Intravenous hypertensive therapy to lower SBP to 160 mm Hg within the first 48 hours Resume usual oral antihypertensive regimen and observe
For adults with a hypertensive emergency and without a compelling condition (such as aortic dissection) systolic blood pressure should be reduced by no more than 25% within the first hour; then, if stable, to 160 mm Hg within the next 2 to 6 hours; and then cautiously to normal during the following 24 to 48 hours. The appropriate treatment is intravenous hypertensive therapy to lower the systolic blood pressure (SBP) to 160 mm Hg within the first 6 hours. Appropriate intravenous agents could include fenoldopam, nicardipine, or nitroprusside. Hypertensive emergency refers to elevation of SBP greater than 180 mm Hg, diastolic blood pressure (DBP) greater than 120 mm Hg, or both, that is associated with end-organ damage. Patients with hypertensive emergency require rapid, tightly controlled reductions in blood pressure that avoid overcorrection. Management typically occurs in an ICU with continuous arterial blood pressure monitoring and continuous infusion of antihypertensive agents. According to the 2017 American College of Cardiology/American Heart Association hypertension guidelines, for adults with a compelling condition (aortic dissection, severe preeclampsia or eclampsia, or pheochromocytoma crisis), SBP should be reduced to less than 140 mm Hg during the first hour and to less than 120 mm Hg in aortic dissection. For adults without a compelling condition, such as this patient, SBP should be reduced by no more than 25% within the first hour; then, if stable, to 160 mm Hg within the next 2 to 6 hours; and then cautiously to normal during the following 24 to 48 hours. Because autoregulation of tissue perfusion is disturbed in hypertensive emergencies, reducing blood pressure too rapidly can result in ischemic organ damage. Therefore, targeting a blood pressure of 120/80 mm Hg during the first hour of treatment is inappropriate because it could result in further worsening of kidney injury, encephalopathy, or both. Conversely, lowering the SBP to 160 mm Hg during 48 hours is likely too slow and not in keeping with current guidelines. This patient's normal medication combined with observation is not aggressive enough. Eventually he will need a stable outpatient hypertension regimen with education on the importance of adherence to that regimen, but not in this acute setting.
A 24-year-old woman is evaluated for a rash on her face for several months' duration. It is asymptomatic but has gotten worse over the summer. She washes her face with soap and water twice daily and has recently changed soaps. Previously her skin was normal. She is otherwise healthy. Her only medication is an oral contraceptive pill, which she started 3 months ago. On physical examination, vital signs are normal. Skin findings are shown. Which of the following is the most likely diagnosis? Irritant contact dermatitis Melasma Morbilliform drug eruption Postinflammatory hyperpigmentation
Melasma is an acquired hypermelanotic condition most commonly affecting women of childbearing age; it is characterized by tan-brown reticulated patches in the centrofacial, malar, and mandible areas. This patient has melasma, an acquired hypermelanotic condition most commonly affecting women of childbearing age. It presents as tan-brown reticulated patches in the centrofacial, malar, and mandible areas. Pigmentation can develop rapidly, often over weeks. Melasma is more apparent or more frequent in patients with darker skin and who live in sunny areas. The pathogenesis is unknown. Common causes are hormonal factors such as pregnancy and oral contraceptives, ultraviolet light, and genetic predisposition. While melasma has no medical significance, it can be very distressing to patients. Treatment is challenging and involves strict sun avoidance and topical depigmenting agents and chemical peels. Irritant contact dermatitis is caused by a direct toxic effect on the epidermis from exposure to a chemical such as a cleaning agent, other caustic substances, or repeated wetting and drying. For example, excessive washing with harsh soap will often lead to dry irritated skin, which is not immune mediated. This patient's skin findings include primarily hyperpigmentation without evidence of dry irritated skin, making irritant contact dermatitis unlikely. Morbilliform (meaning measles-like) drug eruption is characterized by a sudden generalized symmetrical appearance of bright red macules and papules most prominently on the trunk and extremities. Systemic symptoms include pruritus and sometimes low-grade fever. Eruptions typically start within 1 week of starting a new medication. This patient's brown pigmentation on her face without systemic symptoms is not compatible with a morbilliform drug reaction. Postinflammatory hyperpigmentation is a darkening of the skin resulting from cutaneous inflammation. Common causes include acne vulgaris and eczematous dermatoses. The duration of postinflammatory pigmentary changes varies, depending on the location and degree of inflammation. Hyperpigmentation on the lower legs can take several years to fade. Some postinflammatory pigment changes are permanent. Treatment of postinflammatory pigment changes includes treatment of any underlying skin inflammation, sun avoidance or sun protection, and consideration of a bleaching cream. The patient had no history of a previous inflammatory skin condition making postinflammatory hyperpigmentation an unlikely diagnosis. Read Related TextNext Question
A 75-year-old man is evaluated for a 3-month history of worsening urinary hesitancy, decreased stream, and a sense of incompletely emptying his bladder. His medical history is significant for COPD, hypertension, and stroke. He has dyspnea with minimal exertion. His medications are ipratropium bromide, salmeterol, fluticasone, amlodipine, atorvastatin, and aspirin. On physical examination, there is evidence of mild muscle wasting. His respiration rate is 18/min; other vital signs are normal. Oxygen saturation is 92% breathing ambient air. Pulmonary examination reveals decreased breath sounds throughout both lung fields. Digital rectal examination reveals an enlarged prostate without any palpable nodules. Laboratory studies reveal a serum prostate-specific antigen level of 6 ng/mL (6 µg/L). Biopsy of the prostate confirms prostate cancer with a Gleason score of 6. The patient begins therapy with tamsulosin, and his voiding symptoms improve. Which of the following is the most appropriate management of his prostate cancer? Active surveillance Androgen deprivation therapy Bone scan Radiation Observation
For men with early-stage prostate cancer who have limited life expectancy or significant medical comorbidities, observation is the most appropriate management. The most appropriate management in this patient with early-stage prostate cancer and significant medical comorbidities is observation. His medical history is consistent with a predicted life expectancy of less than 5 years, his voiding symptoms are better after treatment with an α-blocker, and he has no other symptoms attributable to his cancer. Based on the typically indolent course of a low-grade localized prostate cancer, this patient will most likely die as a result of one of his other medical conditions rather than his prostate cancer. He is also at significant risk for complications associated with treatment. These issues should all be discussed with the patient so he can understand the competing issues involved and thereby permit him to be an active participant in decision making. Observation is distinct from active surveillance. Observation consists of periodic follow-up to assess symptoms related to cancer progression that require palliation. Active surveillance, however, is a plan to defer definitive prostate cancer therapy until there is evidence of cancer progression based on evidence from systematic monitoring, including changes in prostate-specific antigen level and findings on digital rectal examination, biopsy results, or both. Because this patient's early-stage cancer is not appropriate for treatment, he is not a candidate for active surveillance. Androgen deprivation therapy is not a recommended treatment for early-stage prostate cancer and will expose this patient to unnecessary side effects, including decreased libido and erectile dysfunction, osteoporosis, fatigue, gynecomastia, and hot flushes. Bone scan is very unlikely to reveal bone metastases in asymptomatic men who have low-risk prostate cancer and, therefore, is not indicated. Radiation is not appropriate for this patient because of his short predicted life expectancy, the indolent course of early-stage prostate cancer, and prostate radiation side effects, including bladder, bowel, and rectum injury along with erectile dysfunction.
A 75-year-old woman is evaluated during a routine visit. Medical history is significant for hypertension and coronary artery disease with placement of a stent in the mid right coronary artery 5 years ago. She is symptom free. Medications are metoprolol succinate, lisinopril, aspirin, and atorvastatin. On physical examination, temperature is normal, blood pressure is 130/80 mm Hg, pulse rate is 72/min, and respiration rate is 16/min. BMI is 23. The precordial cadence is irregularly irregular. The remainder of the examination is unremarkable. An electrocardiogram shows atrial fibrillation. Which of the following is the most appropriate treatment? Add clopidogrel Add oral anticoagulation Discontinue aspirin and begin clopidogrel and oral anticoagulation Discontinue aspirin and begin oral anticoagulation No change in therapy
For most patients with high-risk atrial fibrillation and stable coronary artery disease, oral anticoagulation therapy without an antiplatelet agent is sufficient for prevention of both acute coronary syndromes and thromboembolism. This patient with coronary artery disease (CAD) and atrial fibrillation should be treated with oral anticoagulation alone. Thromboembolism (stroke or systemic embolism) is the most feared and devastating complication of atrial fibrillation. Patients with atrial fibrillation and a moderate to high risk for stroke (CHA2DS2-VASc score ≥2 in men and ≥3 in women) should be treated with oral anticoagulation for stroke prevention. This patient has a CHA2DS2-VASc score of 5 (1 point each for female sex, hypertension, and vascular disease; 2 points for age ≥75 years) and should receive oral anticoagulation therapy. Dose-adjusted warfarin is an effective low-cost option for stroke prevention in patients with atrial fibrillation. The chief limitations of warfarin are the need for frequent INR monitoring and adjustment as well as the numerous food and drug interactions associated with its use. The FDA has approved several non-vitamin K antagonist oral anticoagulants for the prevention of stroke in patients with nonvalvular atrial fibrillation, including dabigatran, rivaroxaban, edoxaban, and apixaban. These agents are preferred to warfarin in the prevention of stroke in patients with nonvalvular atrial fibrillation and do not necessitate routine coagulation monitoring. Warfarin remains the agent of choice in patients with valvular atrial fibrillation (moderate to severe rheumatic mitral stenosis). Antiplatelet therapy with aspirin is typically indicated in patients with CAD; however, aspirin alone or dual antiplatelet therapy (aspirin plus clopidogrel) is insufficient for stroke prevention in patients at high risk, such as this one. Combination therapy with oral anticoagulation and antiplatelet therapy, such as aspirin or clopidogrel, is associated with significantly increased risk for bleeding and no apparent incremental benefit. This patient already has several risk factors for bleeding, including a low BMI, hypertension, and female sex, and it is important to minimize the risk for bleeding when oral anticoagulation is prescribed. In patients with atrial fibrillation and CAD, it is recommended that antiplatelet therapy be discontinued unless the patient has recent active CAD, defined as acute coronary syndrome or revascularization in the past 12 months. Because this patient has a history of percutaneous coronary intervention performed 5 years ago, there is no indication for concomitant antiplatelet therapy with oral anticoagulation. Read Related TextNext Question
A 60-year-old man is evaluated for a 6-month history of worsening urinary frequency, urgency, hesitancy, incomplete emptying, nocturia, and weakened stream. He reports no dysuria, incontinence, or acute urinary retention. Medical history is also significant for erectile dysfunction. He takes no medications. On physical examination, vital signs are normal. Rectal examination reveals a diffusely enlarged prostate that is nontender to palpation, with no masses or nodules noted. Testicular size is normal. A comprehensive metabolic profile and urinalysis are normal; an 8:00 AM total testosterone level is also normal. Which of the following is the most appropriate treatment? Finasteride Oxybutynin Tadalafil Tamsulosin
For patients with concomitant benign prostatic hyperplasia and erectile dysfunction, a trial of tadalafil (a phosphodiesterase-5 inhibitor) has been shown to be effective and is the only FDA-approved option to treat both conditions.
A 60-year-old man is evaluated for a 6-month history of worsening urinary frequency, urgency, hesitancy, incomplete emptying, nocturia, and weakened stream. He reports no dysuria, incontinence, or acute urinary retention. Medical history is also significant for erectile dysfunction. He takes no medications. On physical examination, vital signs are normal. Rectal examination reveals a diffusely enlarged prostate that is nontender to palpation, with no masses or nodules noted. Testicular size is normal. A comprehensive metabolic profile and urinalysis are normal; an 8:00 AM total testosterone level is also normal. Which of the following is the most appropriate treatment? Finasteride Oxybutynin Tadalafil Tamsulosin
For patients with concomitant benign prostatic hyperplasia and erectile dysfunction, a trial of tadalafil (a phosphodiesterase-5 inhibitor) has been shown to be effective and is the only FDA-approved option to treat both conditions. The most appropriate treatment for this patient with benign prostatic hyperplasia and erectile dysfunction is tadalafil. This patient has lower urinary tract symptoms (LUTS) due to benign prostatic hyperplasia (BPH), with both obstructive symptoms (decreased stream, incomplete emptying, hesitancy) and irritative symptoms (nocturia, frequency, urgency). Diagnosing BPH can be challenging because of the many causes of LUTS; furthermore, there is poor correlation between prostate size on examination and urinary symptoms. Nonetheless, a careful history and examination can usually render the diagnosis. Men older than 50 years are likely to have BPH as a cause of LUTS, whereas men younger than 40 years are likely to have other causes of LUTS. Furthermore, the patient has a history of untreated erectile dysfunction; in cases of BPH and erectile dysfunction, a trial of tadalafil, a phosphodiesterase-5 inhibitor, is recommended. In this setting, tadalafil has been shown to be clinically and symptomatically effective and is the only FDA-approved option to treat the symptoms of both conditions. Finasteride and other 5α-reductase inhibitors are considered second-line medical therapy for BPH. The American Urological Association recommends the addition of 5α-reductase inhibitors in cases of BPH refractory to α-blocker monotherapy, not as a first-line monotherapy choice. Furthermore, this medication class has been known to lead to erectile and ejaculatory dysfunction. Oxybutynin and other anticholinergic agents are effective in treating irritative LUTS of BPH. However, given their mechanism of action, anticholinergic agents can lead to worsening obstructive symptoms and could also lead to worsening erectile dysfunction. Tamsulosin and other α-blocking agents are first-line medical therapy for symptomatic BPH. However, α-blockers have numerous side effects, including hypotension, orthostasis, and sexual dysfunction. Tamsulosin could worsen this patient's erectile dysfunction and thus would not be the most appropriate treatment choice.
An 83-year-old woman is evaluated for a 15-year history of gout. Attacks were initially limited to the first metatarsophalangeal joints and occurred once or twice a year. She now has recurrent attacks, occurring every 3 months. Her feet, ankles, knees, wrists, elbows, and hands have been affected. Attacks last for 1 to 2 weeks and have been treated with NSAIDs. History is also significant for recurrent nephrolithiasis, stage 3 chronic kidney disease, and hypertension. Medications are allopurinol, 400 mg/d; colchicine, 0.6 mg/d; and lisinopril. She tolerates her medications well. On physical examination, vital signs are normal. There are palpable masses at the olecranon processes and white nodules on a few distal and proximal interphalangeal joints of both hands. Laboratory studies show a serum creatinine level of 1.0 mg/dL (88.4 µmol/L) and a serum urate level of 5.8 mg/dL (0.34 mmol/L). Which of the following is the most appropriate next step in management? Add probenecid Discontinue allopurinol; begin pegloticase infusions Increase allopurinol Increase colchicine
For patients with inadequately treated tophaceous gout who tolerate allopurinol, the dose can be titrated to a maximum of 800 mg/d in 100-mg increments to alleviate symptoms. Increasing allopurinol to 500 mg/d is the most appropriate next step in management for this patient with symptomatic tophaceous gout. Although she has stage 3 chronic kidney disease (with an estimated glomerular filtration rate <60 mL/min/1.73 m2), she is tolerating allopurinol, 400 mg/d, without issue and does not take a diuretic, so she is not at high risk for allopurinol hypersensitivity. Allopurinol can be titrated to a maximum of 800 mg/d in 100-mg increments. It is important to note that two recently published guidelines differ regarding the role of pharmacologic urate-lowering therapy in patients with gout. The 2016 American College of Physicians guideline notes a lack of evidence supporting a specific target level for urate lowering; this guideline stresses discussing the risks and benefits of urate-lowering therapy with patients and suggests a "treat to avoid symptoms" approach without specifically considering the serum urate levels. The 2016 European League Against Rheumatism (EULAR) recommendations support a "treat-to-target" approach (consistent with the 2012 American College of Rheumatology gout guidelines), reducing the serum urate level to less than 6.0 mg/dL (0.35 mmol/L) in patients without tophi and less than 5.0 mg/dL (0.30 mmol/L) in patients with tophi. Based on both the ACP and EULAR guidelines, this patient meets criteria for more aggressive therapy. Adding probenecid might be reasonable in some cases but is contraindicated in this patient because of her estimated glomerular filtration rate of less than 60 mL/min/1.73 m2, tophi, and recurrent nephrolithiasis. Pegloticase is a costly drug that should be considered only when other options have been exhausted. Increasing colchicine to twice daily for prophylaxis of acute attacks offers no clear benefit over once daily dosing. More importantly, this would not affect the serum urate level or cause resorption of the tophi.
Indications for cardiac surgery in infective endocarditis?
For patients with infective endocarditis, cardiac surgery is indicated for 1. persistent infection lasting longer than 5 to 7 days while on appropriate antimicrobial therapy; 2. symptomatic heart failure; left-sided involvement with Staphylococcus aureus, fungal infections, or highly resistant organisms; 3. complications such as heart block, annular or aortic abscess, or destructive penetrating lesions 4. prosthetic valve infective endocarditis and relapsing infection.
A 62-year-old man is evaluated for a 2-month history of nonproductive cough, progressive dyspnea, and fatigue. He reports a 6.8-kg (15-lb) weight loss during this time. He has a 30-pack-year history of smoking. He worked in a navy ship yard 32 years ago. His history is otherwise unremarkable. On physical examination, vital signs are normal. Oxygen saturation is 94% breathing ambient air. Lung examination findings are consistent with a right pleural effusion. The remainder of the examination is normal. A complete blood count and metabolic profile are normal. Chest radiograph shows a large right pleural effusion. Thoracentesis is performed and removes 1200 mL of serosanguineous fluid. Pleural fluid analysis: Cytology Negative Glucose 89 mg/dL (4.9 mmol/L) Lactate dehydrogenase 200 U/L pH 7.36 Total protein 3.8 g/dL (38 g/L) Laboratory studies: Serum lactate dehydrogenase 235 U/L Serum total protein 6.2 g/dL (62 g/L) The patient returns 2 weeks later with a recurrent pleural effusion. CT scan of his chest demonstrates a moderate right pleural effusion with no parenchymal or pleural abnormalities noted. Thoracentesis is repeated; pleural fluid analysis is similar to the initial analysis, and cytology is again negative. Which of the following is the most appropriate next step? Closed pleural biopsy Measure pleural fluid triglycerides Resend a pleural fluid specimen for cytology Thoracoscopy and pleural biopsy
For patients with negative cytology in whom malignancy is suspected, thoracoscopy with pleural biopsy allows for direct visualization of the pleural surface and has a diagnostic sensitivity for malignant disease of greater than 90%. This patient should be referred for thoracoscopy and pleural biopsy. He has a recurrent exudative pleural effusion. The characterization of pleural fluid as a transudate or exudate helps narrow the differential diagnosis and direct subsequent investigations. An effusion is considered an exudate if any of the following criteria are met: pleural fluid total protein/serum total protein greater than 0.5; pleural fluid lactate dehydrogenase (LDH)/serum LDH greater than 0.6; pleural fluid LDH greater than 2/3 the upper limit of normal for serum LDH. This patient has an exudate. Despite the negative chest radiograph and CT scan, this exudate is concerning for malignancy considering his age, smoking history, and work in a shipyard with potential exposure to asbestos. The cytology of the pleural fluid was negative, but cytology is only 60% sensitive for malignancy. Closed pleural biopsy is less sensitive than cytology and should not be performed. A chylous effusion can be suspected by its milky appearance (seen in 50% of patients) and is associated with traumatic and nontraumatic etiologies. Nontraumatic chylous effusion is most commonly due to malignancy (lymphoma, chronic lymphocytic leukemia, metastatic cancer). Traumatic chylous effusions are most commonly associated with thoracic surgical procedures. A pleural fluid triglyceride level greater than 110 mg/dL (1.24 mmol/L) is characteristic of a chylothorax. There is no reason to suspect a chylothorax at this point; thoracoscopic pleural biopsy will be of higher diagnostic yield. The yield of sending more than two cytology specimens taken on different occasions is low. If cytology is negative and malignancy is still suspected, thoracoscopy with pleural biopsy allows for direct visualization of the pleural surface and has greater than 90% sensitivity for the diagnosis of malignancy.
A 63-year-old man is evaluated for a 2-month history of progressive lower extremity weakness. He reports increased difficulty arising from a chair, climbing stairs, and reaching overhead. He was diagnosed with polymyositis 6 months ago; prednisone and azathioprine were initiated at that time, with clinical improvement. He currently takes prednisone, 20 mg/d, and azathioprine, 100 mg/d. On physical examination, vital signs are normal. The patient cannot arise from the chair without using his arms to push off and can squat only by using his hands to hold on to the chair. He has cushingoid features and scattered ecchymoses on the forearms. There is no rash. Laboratory studies show a serum creatine kinase level of 160 U/L. Which of the following is the most appropriate management? Order electromyography Order muscle biopsy Reduce azathioprine dose Reduce prednisone dose
Glucocorticoid myopathy should be suspected in a patient with recent initiation of high-dose glucocorticoids, cushingoid features, initial clinical improvement, and reduction in serum creatine kinase levels who has an increase in weakness with reduction in the glucocorticoid dose; treatment is appropriate dose reduction or discontinuation if possible. The most appropriate management is reduction of the prednisone dose in this patient with drug-induced myopathy caused by glucocorticoids. Glucocorticoid myopathy is a common consequence of prolonged treatment with high-dose glucocorticoids, as seen in this patient treated for polymyositis. Distinguishing between glucocorticoid-induced myopathy and active inflammatory myopathy is important, particularly in older patients for whom long-term glucocorticoid side effects can be particularly debilitating. The history is suggestive of glucocorticoid myopathy when a patient has been started on high-dose glucocorticoids within a few months, has cushingoid features, and has normal serum creatine kinase levels. No single feature is diagnostic on electromyogram (EMG) or muscle biopsy. Weakness can often be addressed by reducing the glucocorticoid dose and following the patient over the next 3 to 4 weeks to assess for gains in strength. Colchicine and hydroxychloroquine are also implicated as a cause of drug-induced myopathy. Statins are a frequent cause of myalgia and/or asymptomatic serum creatine kinase elevations and, rarely, statin use can result in an immune-mediated necrotizing myopathy associated with antibodies to the enzyme HMG-CoA reductase. An EMG could potentially reveal findings of low amplitude motor unit potentials or be normal in glucocorticoid myopathy. However, the initial clinical improvement and reduction in the muscle enzymes with reduction in glucocorticoid dose most strongly suggest drug-induced myopathy, and an EMG in this situation is unlikely to provide helpful results. In glucocorticoid-induced myopathy, nonspecific atrophy of type IIb fibers without necrosis or inflammation can be seen that would help substantiate the diagnosis. However, muscle biopsy is an invasive procedure that is not indicated in a patient in whom initial clinical improvement and normalization of muscle enzyme levels has occurred, suggesting the diagnosis of glucocorticoid-induced myopathy. Immunosuppressive therapy with azathioprine is used for glucocorticoid-resistant disease or glucocorticoid sparing. Reducing the azathioprine dose is not likely to be of diagnostic or therapeutic benefit because proximal muscle weakness is not an adverse effect of azathioprine.
A 73-year-old man is evaluated during a routine examination. He is physically active and participates regularly in charitable running events. He has no cardiac symptoms, and his medical history is unremarkable. On physical examination, vital signs are normal. The lungs are clear. Jugular venous pulse is normal. Carotid upstrokes are normal. The apical impulse is palpable and not displaced or sustained. There is a grade 3/6 diastolic decrescendo murmur best heard at the left lower sternal border. The remainder of the examination is normal. A transthoracic echocardiogram shows severe aortic regurgitation. The left ventricular ejection fraction is 65%. The left ventricle is minimally dilated with an end-systolic dimension of 40 mm. Which of the following is the most appropriate next step in management? Carvedilol Lisinopril Surgical aortic valve replacement Clinical and echocardiographic follow-up in 6 to 12 months
For patients with severe aortic regurgitation, indications for surgery are the presence of attributable symptoms, left ventricular ejection fraction less than 50%, or significant left ventricular dilatation; in the absence of these findings, surveillance echocardiography every 6 to 12 months is recommended. The most appropriate next step in management is clinical and echocardiographic follow-up in 6 to 12 months. This asymptomatic patient with severe aortic regurgitation does not currently meet the criteria for surgical aortic valve replacement. For patients with severe aortic regurgitation, surgical aortic valve replacement is recommended in the presence of symptoms attributable to regurgitation, left ventricular ejection fraction less than 50%, or another indication for cardiac surgery. In addition, surgical aortic valve replacement can be beneficial in asymptomatic patients with significant left ventricular dilatation (end-systolic dimension >50 mm or indexed end-systolic dimension >25 mm/m2). In the absence of these findings, clinical evaluation and surveillance echocardiography every 6 to 12 months is recommended. The clinical impact of β-blockers, such as carvedilol, in patients with chronic aortic regurgitation is uncertain. Carvedilol is useful in patients with severe aortic regurgitation and heart failure when combined with other standard therapies, such as diuretics, ACE inhibitors, and aldosterone antagonists; however, this patient has no indication for β-blocker therapy. ACE inhibitors or angiotensin receptor blockers may be used in patients with chronic severe aortic regurgitation and concomitant hypertension, although these agents, as well as dihydropyridine calcium channel blockers, have not been shown to delay the need for surgery in asymptomatic patients without hypertension. Consequently, they are not indicated in this patient in the absence of hypertension.
A 38-year-old woman is evaluated. She has just been diagnosed with hormone receptor-negative, HER2-positive stage II infiltrating ductal breast cancer. She takes no medications. Neoadjuvant chemotherapy with docetaxel, trastuzumab, and pertuzumab is planned; a port has been placed, and treatment is to start next week. The patient has a 2-year-old daughter and has very recently decided that she wishes to conceive again. On physical examination, vital signs are normal. A 2.5-cm left breast mass is noted, with no regional lymphadenopathy. Which of the following is the most appropriate management? Advise against any further pregnancies Cancel chemotherapy and proceed with mastectomy and radiation therapy Delay chemotherapy until counseled by a fertility specialist Proceed with chemotherapy
For women who desire future childbearing who are medically stable, initiation of chemotherapy can be delayed for a brief time to allow assessment by a fertility specialist. The most appropriate option for this patient is to meet with a fertility specialist. Young women with cancer who may wish to bear children after cancer treatment can be seen emergently and counseled about their options. An established fertility preservation option for a woman with a partner is in vitro fertilization with embryo freezing. Newer options include freezing of unfertilized eggs and ovarian cryopreservation with future reimplantation. Delaying initiation of chemotherapy for a brief time in a stable patient so she can be assessed and eggs or embryos harvested if appropriate is acceptable. Premenopausal women with breast cancer who do not wish to become pregnant should use a nonhormonal method of birth control. Although a hormone receptor-negative, HER2-positive cancer is potentially aggressive and initiation of her neoadjuvant treatment should not be unnecessarily delayed, it is not an emergency and there is time for her to meet with a fertility specialist. With the use of chemotherapy and anti-HER2 therapy in combination with appropriate local therapy, she should have a reasonably high likelihood of cure. Restricting her therapy to only local therapy with surgery and radiation decreases her chance for cure and is not the best option. There is no evidence that women who become pregnant after a diagnosis of breast cancer have a higher rate of recurrence than similarly staged patients who do not become pregnant. Therefore, there is no reason to counsel this patient against future pregnancies.
A 33-year-old woman is evaluated following an incidentally discovered hepatic adenoma. One week earlier, she was evaluated for nephrolithiasis with CT urogram. A 4-cm right-lobe liver lesion was also seen. Follow-up MRI identified findings consistent with hepatic adenoma. She is experiencing no symptoms now but has noted a sense of right-upper-quadrant abdominal discomfort in the past. Her only medication is an oral contraceptive agent. All physical examination findings are normal. Which of the following is the most appropriate management? Discontinue the oral contraceptive agent Radiofrequency ablation Surgical resection Observation
For women with asymptomatic hepatic adenomas smaller than 5 cm in size, estrogen-containing oral contraceptive agents should be discontinued, and follow-up liver imaging is recommended every 6 months for at least 2 years. The most appropriate management is to discontinue the oral contraceptive agent. Hepatocellular adenomas are generally seen in women because they can be dependent on estrogen for growth. They may also occur in men, and these adenomas carry a higher risk for malignant transformation. Hepatic adenomas may also be associated with anabolic-steroid use, obesity, and the metabolic syndrome. They are often solitary, ranging in size from 1 to 30 cm. Most patients are asymptomatic, and the adenoma is discovered incidentally during imaging for unrelated problems or because of right-upper-quadrant abdominal pain. The diagnosis is often based on the characteristic imaging findings in a woman of reproductive age. Histology can confirm the diagnosis, but biopsy is not always needed in cases where imaging findings are characteristic; needle biopsy is avoided because it is associated with bleeding. Hepatic adenomas may increase in size during pregnancy. Women with adenomas greater than 2 cm who are contemplating pregnancy, or who are pregnant, can be treated with radiofrequency ablation but the decision to treat and the treatment modality must be individualized. The natural history and prognosis of hepatic adenomas is ill-defined. The risk for malignant transformation of hepatic adenomas is approximately 10%. Adenomas larger than 5 cm in size have an increased risk for bleeding that can occasionally cause hemodynamic compromise. In general terms, surgical resection is typically recommended for any symptomatic hepatic adenoma, adenomas 5 cm or larger, or an adenoma found in a male. Asymptomatic women who are taking estrogen-containing oral contraceptive agents and who have adenomas smaller than 5 cm in size can be managed conservatively, as the risk for hemorrhage or malignant transformation is relatively low. However, liver imaging should be conducted every 6 months for at least 2 years to ensure that the lesion is not growing. Most adenomas tend to regress or even disappear when oral contraceptive agents are discontinued. Failure to regress, continued growth, or an elevation of the α-fetoprotein level may be an indication for surgical resection.
A 25-year-old woman comes to the office to establish care. She is 5 weeks pregnant. She has hypothyroidism due to Hashimoto thyroiditis that was diagnosed 4 years ago. She is asymptomatic. Her only medications are levothyroxine and folic acid. On physical examination, vital signs are normal. The patient's thyroid gland is nontender and diffusely enlarged without nodules. Pregnancy test is positive. Serum thyroid-stimulating hormone level measured 4 months ago was 2.2 µU/mL (2.2 mU/L). Which of the following is the most appropriate management of this patient's hypothyroidism? Check serum thyroid-stimulating hormone in 2 months Decrease levothyroxine dose by 30% Increase levothyroxine dose by 30% Stop levothyroxine and start liothyronine
For women with hypothyroidism adequately treated with levothyroxine before pregnancy, dosing can be empirically increased by 30% when pregnancy is confirmed. Levothyroxine is the treatment of choice for the management of hypothyroidism during pregnancy. Patients can be counseled to start taking an additional two tablets of their prepregnancy levothyroxine dose per week, which is roughly equivalent to a 30% increase. In euthyroid women without thyroid disease, the total body thyroxine (T4) pool increases by 40% to 50% during pregnancy. This is mediated by the stimulatory effects of thyroid-stimulating hormone (TSH) and placental human chorionic gonadotropin. Pregnant women with hypothyroidism are unable to augment thyroidal production of T4 and triiodothyronine (T3). The levothyroxine dose of hypothyroid women must therefore be adjusted to maintain a euthyroid state. Because T4 requirements may begin to increase as early as 4 to 6 weeks of pregnancy, women with hypothyroidism should increase their levothyroxine dose or serum TSH should be measured as soon as pregnancy is confirmed. TSH should be measured every 4 weeks for the first half of pregnancy and around 30 weeks of gestation in all women with hypothyroidism. A TSH value in the lower half of the reference range should be targeted (equivalent to a TSH level below 2.5 µU/mL [2.5 mU/L]) both in preconception planning and during pregnancy. Continuing the patient on her current dose of levothyroxine and checking serum TSH in 2 months or decreasing the patient's levothyroxine dose are both inappropriate management and could precipitate maternal or fetal hypothyroidism. The use of liothyronine or T3-containing preparations including desiccated thyroid is contraindicated in pregnancy because the fetal central nervous system is relatively impermeable to T3. Thyroid hormone is essential for normal fetal development and is especially critical for the fetal brain.
A 56-year-old man is evaluated for worsening memory and increasing behavioral problems. He was fired from his job as an accountant 6 months ago after forgetting to file necessary forms, making repeated calculation errors, and sending inappropriate emails to coworkers. He developed compulsive spending habits 3 years ago and completely depleted his retirement savings account; he has shown little concern about his financial losses. The patient was previously very physically active but now spends most of the day obsessively watching television. His wife says he eats high-calorie, nonnutritional food constantly and has gained 27.3 kg (60.1 lb) in the past 2 years. No other symptoms, including visual hallucinations, are reported. On physical examination, the patient has a blunted affect and provides little information spontaneously. Vital signs are normal. He scores 18/30 (normal, ≥26) on the Montreal Cognitive Assessment and has significant difficulty with letter and number sequencing, word fluency, and memory, recalling only two of five items. Remaining physical examination findings are normal. Which of the following is the most likely diagnosis? Alzheimer disease Dementia with Lewy bodies Frontotemporal dementia Parkinson disease
Frontotemporal dementia typically involves an alteration in personality and behavior that develops years before the onset of cognitive impairment. This patient's age, gender, history, and cognitive evaluation findings all support a diagnosis of behavioral-variant frontotemporal dementia (FTD). FTD typically involves an alteration in personality and behavior that develops years before the onset of cognitive impairment; altered behaviors typically manifest as obsessive-compulsive tendencies, impulsivity, apathy, impaired judgment, emotional coldness, disinhibition, excessive spending, and excessive eating, particularly of high-calorie, nonnutritional foods. Amyotrophic lateral sclerosis, the major form of motor neuron disease, is found in 20% to 30% of patients diagnosed with behavioral-variant FTD. The presence of motor neuron disease generally portends a much more rapid progression of FTD but is exceptionally rare in other neurodegenerative disorders. Classically, Alzheimer disease presents with the insidious development of recent memory loss. Forgetfulness of the details of recent events predominates early in the disease course. Aphasia is frequently seen early and is initially characterized by word-finding difficulties. Visuospatial dysfunction also is common and often presents as episodes of becoming lost in familiar environments or problems in assembling objects (constructional apraxia). Executive dysfunction may manifest as impairment of problem-solving abilities, judgment, and multitasking. Obsessive-compulsive tendencies, impulsivity, apathy, impaired judgment, emotional coldness, disinhibition, and excessive eating are not typical of Alzheimer disease. Core features of dementia with Lewy bodies are fluctuating cognition, recurrent visual hallucinations, and parkinsonism. Cognitive fluctuations include variations in both attention and level of arousal and can vary hour to hour, day to day, or week to week. These features are not present in this patient. Cognitive impairments in Parkinson disease are subcortical, which implies that they primarily involve slow processing speed, impaired short-term memory, and attention deficits with relative sparing of cortical functions, such as language and declarative memory. Dopamine agonists are associated with higher risk of specific adverse effects, including impulse control disorder, punding (repetition of complex motor behaviors), sleep attacks, hallucinations, and confusion, that are not seen in untreated Parkinson disease.
A 33-year-old woman is evaluated 4 days after removing an embedded tick from her left arm. She reports that the tick was attached for less than 12 hours. She noticed itching at the site of tick attachment 2 days ago but otherwise has been asymptomatic. She preserved the tick in a bottle, and it is confirmed visually to be an adult black-legged deer tick (Ixodes scapularis). On physical examination, vital signs are normal. A small area of induration is noted on the left arm, with no erythema, tenderness, or warmth. All other physical examination findings are unremarkable. Which of the following is the most appropriate initial management? Borrelia burgdorferi polymerase chain reaction testing of the tick Borrelia burgdorferi serologies Doxycycline Reassurance that the risk of Lyme disease is low
In patients at high risk, doxycycline prophylaxis has been shown to decrease the risk of Lyme disease if started within 72 hours of tick removal, assuming that the tick has been attached for at least 36 hours. The most appropriate initial management for this patient is reassurance that the risk of Lyme disease is low. In selected patients at high risk, doxycycline prophylaxis has been shown to decrease the risk of Lyme disease if given within 72 hours of tick removal, assuming that the tick has been attached for greater than 36 hours. Knowledge of the epidemiology of Lyme disease in the area of practice is important for making informed decisions about prophylaxis. Even in endemic regions, such as the northeastern and upper midwestern United States, few vector ticks are infected with Borrelia burgdorferi. Most patients in highly endemic areas do not develop symptomatic infection after a tick bite because bacterial transmission rarely occurs unless the vector tick has fed for at least 36 hours. In the subset of patients from highly endemic areas seen within 72 hours of tick removal in whom attachment with an Ixodes species tick for greater than 36 hours can be substantiated, a single dose of doxycycline (200 mg) has been shown to decrease the risk of developing infection. This patient was exposed to the tick that transmits Lyme disease. However, given the short duration of attachment, her risk for infection is negligible. Furthermore, she is beyond the 72-hour window during which prophylaxis has been shown to be beneficial. The most appropriate management is reassurance, with the caveat that she return for evaluation if she develops a rash, fever, or other suggestive symptoms within a month of the exposure. Diagnostic testing with polymerase chain reaction to confirm tick infection is not routinely available and not informative because of the low risk of transmission, even if B. burgdorferi is identified in the vector. Checking B. burgdorferi titers similarly has low yield because antibodies may not be present during the early stages of infection, and paired serologic testing with a subsequent convalescent specimen in the absence of symptoms is not cost effective.
A 55-year-old woman is evaluated for increasing serum creatinine and oliguria; she has cirrhosis. She was hospitalized 3 days ago for worsening ascites, confusion, and an elevated serum creatinine level of 1.5 mg/dL ([132.6 µmol/L]; baseline, 1.1 mg/dL [97.2 µmol/L]). Her diuretics were held, and lactulose was continued. An abdominal paracentesis was negative for spontaneous bacterial peritonitis. Intravenous albumin was administered at 1 g/kg/d for 2 days, and today her serum creatinine level is 3.0 mg/dL (265.2 µmol/L). Urine output for the previous 24 hours was 300 mL. History is significant for cirrhosis secondary to nonalcoholic steatohepatitis. Outpatient medications are lactulose, spironolactone, furosemide, and propranolol. On physical examination, the patient is confused. She is afebrile, blood pressure is 100/70 mm Hg (stable since admission), pulse rate is 84/min, and respiration rate is 16/min. Asterixis is noted. The skin and sclera are icteric. The jugular venous pressure is normal. Ascites is present. There is 3+ lower extremity edema. The remainder of the examination is normal. Current laboratory studies: Bicarbonate 18 mEq/L (18 mmol/L) Creatinine 3.0 mg/dL (265.2 µmol/L) Potassium 5 mEq/L (5 mmol/L) Sodium 129 mEq/L (129 mmol/L) Urine sodium <10 mEq/L (10 mmol/L) Urinalysis Specific gravity 1.025; pH 5.0; trace protein; 2-4 erythrocytes/hpf; 1-3 pigmented granular casts/hpf Abdominal ultrasound demonstrates ascites, and normal-sized kidneys with no hydronephrosis. Which of the following is the most appropriate treatment? Hemodialysis Isotonic crystalloid Octreotide and oral midodrine Transjugular intrahepatic portosystemic shunt
General management of type 1 hepatorenal syndrome includes discontinuing diuretics, volume replacement with albumin, and use of vasoconstrictors. Vasoconstrictor therapy with octreotide and oral midodrine is appropriate for this patient with type 1 hepatorenal syndrome (HRS). Type 1 HRS is a clinical diagnosis made after exclusion of other causes of kidney dysfunction. It is characterized by a rise in serum creatinine of at least 0.3 mg/dL (26.5 µmol/L) and/or ≥50% from baseline within 48 hours, bland urinalysis, and normal findings on kidney ultrasound. It is also supported by a lack of improvement in kidney function after withdrawal of diuretics and 2 days of volume expansion with intravenous albumin. Often, patients also have low urine sodium, low fractional excretion of sodium, and oliguria. Type 2 HRS is defined as a more gradual decline in kidney function associated with refractory ascites. General management of type 1 HRS includes discontinuing diuretics, restricting sodium, restricting water in hyponatremic patients, and searching for precipitating factors. Initial therapeutic interventions include treatment with vasoconstrictors in conjunction with intravenous albumin. Dialysis should be initiated only if the patient does not respond to HRS medical therapy with midodrine and octreotide and/or if indications for dialysis develop. Absolute indications for dialysis include hyperkalemia, metabolic acidosis, and pulmonary edema refractory to medical therapy; uremic symptoms; uremic pericarditis; and certain drug intoxications. Currently, this patient has no acute indications for dialysis. The patient was volume expanded with intravenous albumin, which should have corrected hypovolemia. Furthermore, she is not volume depleted on physical examination, has edema and ascites, and has stable blood pressure without tachycardia. Therefore, intravenous fluids are not indicated. The transjugular intrahepatic portosystemic shunt is primarily used to treat variceal hemorrhage and ascites. It has been used as a last resort in the treatment of refractory ascites in highly selected patients with HRS who do not respond to medical therapy and who are awaiting liver transplantation. Complications include an increase in the rate of hepatic encephalopathy and risk of kidney injury associated with intravenous contrast. This procedure is not indicated at this time. Read Related TextNext Question
A 50-year-old man was diagnosed with hypertrophic cardiomyopathy following an episode of syncope. An implantable cardioverter-defibrillator was placed, and metoprolol was initiated. Genetic testing revealed a mutation of the β-myosin heavy-chain gene associated with hypertrophic cardiomyopathy. Medical history is otherwise unremarkable. The patient has a 16-year-old daughter who is asymptomatic. Which of the following is the most appropriate management of this patient's daughter? Electrocardiography and echocardiography screening at age 21 years Electrocardiography and echocardiography screening at age 40 years Electrocardiography and echocardiography screening if symptoms develop Genetic counseling and testing
Genetic counseling and testing are recommended for patients with hypertrophic cardiomyopathy (HCM) and for all first-degree family members of patients with HCM who have an identified genetic mutation, regardless of the presence or absence of symptoms. The most appropriate management of the patient's 16-year-old daughter is genetic counseling and testing. Hypertrophic cardiomyopathy (HCM) is an autosomal dominant heritable disorder characterized by the presence of increased left ventricular wall thickness in the absence of loading conditions or other underlying causes. In patients with HCM and a known pathogenic mutation, genetic counseling and testing is recommended for all first-degree family members, regardless of the presence or absence of symptoms. In this case, genetic testing of the index patient has revealed a mutation known to be associated with HCM. Mutation-negative family members do not require further evaluation (such as with electrocardiography or echocardiography) and have no increased risk for future development of HCM. Family members who test positive for an HCM-related genetic mutation should be further evaluated with physical examination, 12-lead electrocardiography, and echocardiography (or cardiac magnetic resonance imaging). After genetic and clinical testing, some patients will be identified as carriers of only the genetic mutation without left ventricular hypertrophy (termed genotype-positive but phenotype-negative). Genetic counseling is an important facet of care for patients with HCM, regardless of whether genetic testing is performed. Counseling enables informed decision making about the risks and benefits of testing and facilitates interpretation of the results. The development of HCM is a dynamic process, and phenotypic expression may change as a patient ages. Because of this, if the daughter tests positive for the mutated gene but the electrocardiogram and echocardiogram are normal, she may still require repeated screening because of the possibility of phenotypic disease expression at any age. In that case, screening with physical examination, 12-lead electrocardiography, and echocardiography is recommended every 1 to 2 years in children and adolescents and every 3 to 5 years in adults as well as if there is a change in clinical status. Mutation carriers with phenotypic expression of HCM should undergo further risk stratification for sudden cardiac death.
A 56-year-old woman is evaluated for severe vaginal itching and discomfort. Her symptoms have progressively worsened over the last 4 months. There is no associated vaginal discharge or vaginal odor. She is experiencing significant vaginal dryness, and intercourse has become painful despite the use of lubricants. She has been menopausal since age 53 years. She takes no medications. On physical examination, vital signs are normal. Physical examination reveals dry vaginal epithelium that is smooth and shiny. Blood vessels are visible beneath the pale vaginal mucosa, and increased friability is evident. Vaginal pH is 6.0. Wet mount shows occasional leukocytes. Whiff test result is negative. There are no clue cells and no hyphae on potassium hydroxide preparation. Which of the following is the most likely diagnosis? Acute allergic contact dermatitis Genitourinary syndrome of menopause Vulvar lichen planus Vulvar lichen sclerosus
Genitourinary syndrome of menopause is a clinical diagnosis characterized by vulvar itching, vaginal dryness, and dyspareunia; pelvic examination findings include pale, shiny vaginal walls; decreased rugae; and petechiae. The most likely diagnosis is genitourinary syndrome of menopause (vaginal atrophy). The clinical history and physical examination are most helpful for diagnosing genitourinary syndrome of menopause. Approximately 10% to 40% of menopausal women experience symptoms related to vaginal atrophy, including vulvar itching, vaginal dryness, and dyspareunia. On physical examination, pale and shiny vaginal walls, decreased rugae, and petechiae are characteristic findings. In contrast to menopausal vasomotor symptoms, which may last for a few years and resolve spontaneously, genitourinary syndrome of menopause is frequently progressive and often requires treatment. Mild to moderate symptoms can be treated with vaginal moisturizers and lubricants, but more severe symptoms, as experienced by this patient, are best treated with vaginal estrogen. Like genitourinary syndrome of menopause, acute allergic contact dermatitis presents with intense pruritus, often worse at night, as well as burning and stinging. Defining characteristics on physical examination include a discrete, well-demarcated area of erythema and edema. Fissures may be present along the labial folds. Excoriations are common and may become secondarily infected. The course of acute contact dermatitis progresses over days, not months as experienced by this patient. The most commonly implicated culprits are fragrances, medications, and preservatives in medications, such as glucocorticoids. Diagnosis is typically made on clinical grounds; biopsy is rarely necessary. Treatment generally consists of allergen avoidance and soaking in warm water (bathtub or sitz bath), followed by application of an emollient, such as petrolatum or a low-potency glucocorticoid. Lichen planus is an inflammatory condition that can affect the skin, nails, or mucosa. Clinical presentation includes white lines and patches (Wickham striae) or painful erythema and erosions (erosive variant). Therapies are glucocorticoids (systemic and topical) and immunosuppressive agents in severe disease. Lichen sclerosus is an inflammatory condition that often presents as white, atrophic patches on the genital and perianal skin. It differs from lichen planus in its clinical presentation of white patches that circumferentially involve the vaginal introitus and perianal area ("figure 8" appearance). Prepubertal girls and postmenopausal women are at highest risk. Biopsy establishes the diagnosis and can differentiate it from other inflammatory disorders. Treatment is with potent topical glucocorticoids.
A 45-year-old man is evaluated for watery diarrhea accompanied by nausea and bloating. Symptoms began 4 weeks earlier with abdominal cramping and explosive watery stools. Now, he reports up to five loose bowel movements per day, with no blood and no nocturnal symptoms. He also describes generalized abdominal discomfort that is not relieved after a bowel movement. He works at a child care center. He is otherwise healthy and takes no medication. On physical examination, vital signs are normal. Abdominal examination shows periumbilical tenderness with no rebound or guarding. The remainder of the examination is unremarkable. Results of laboratory studies, including hemoglobin level and a comprehensive metabolic panel, are within normal limits. Which of the following is the most appropriate next step in management? Colonoscopy CT scan of the abdomen and pelvis 24-Hour urine 5-hydroxyindoleacetic acid measurement Stool testing for Giardia lamblia
Giardia lamblia infection is a common parasitic infection that occurs most often among children, child care workers, and backpackers or campers who drink untreated water from lakes, rivers, or wells.
A 26-year-old woman is hospitalized with a temperature of 38 °C (101.5 °F) 10 days after her first cycle of chemotherapy with rituximab, cyclophosphamide, doxorubicin, vincristine and prednisone (R-CHOP) for diffuse large B-cell lymphoma. Other than fever, she has no symptoms of infection. On physical examination, temperature is 38.6 °C (101.5 °F). The remainder of the vital signs and physical examination are normal. Pertinent laboratory results are a leukocyte count of 1100/µL (1.1 × 109/L) with 10% neutrophils, platelet count of 144,000/µL (144 × 109/L), and hemoglobin level of 11.2 g/dL (112 g/L). Blood and urine culture results are pending. Chest radiographic findings are negative. Empiric broad-spectrum antibiotics are initiated. Which of the following is the most appropriate additional management? Reduce the doses of cyclophosphamide and doxorubicin Start granulocyte colony-stimulating factor (G-CSF) now Start G-CSF on day 2 of her subsequent cycles of therapy Start levofloxacin prophylaxis before next cycles of therapy
Granulocyte colony-stimulating factor is effective for prophylaxis and secondary prophylaxis of neutropenia in patients undergoing chemotherapy but has little benefit in patients who are currently neutropenic. This patient has fever and neutropenia caused by her chemotherapy and should start granulocyte colony-stimulating factor (G-CSF) with her next cycle of chemotherapy. Febrile neutropenia is defined as a single fever (101 °F [38.3 °C]) or sustained elevated temperature (38 °C [100.4 °F]) in a patient with a current or anticipated absolute neutrophil count less than 500 cells/µL (0.5 × 109/L). Most infections are believed to arise from the patient's endogenous flora. Neutropenia typically occurs 5 to 15 days after administration of chemotherapy. G-CSF and granulocyte-macrophage colony-stimulating factor can be given prophylactically to patients receiving chemotherapeutic regimens that carry a high risk of neutropenia and as secondary prophylaxis in patients such as this one, who have had a previous episode. In this patient, initiating G-CSF or pegylated G-CSF on day 2 of her next cycle of chemotherapy would reduce the risk of another bout of febrile neutropenia and help maintain the dose intensity of her chemotherapy. Reducing this patient's dose of chemotherapy for the next cycle would not be an appropriate strategy to avoid neutropenia because she is a young patient being treated with curative intent, and reducing her chemotherapy dosages might compromise her outcome. The prophylactic use of G-CSF is a better strategy. There is little benefit to starting G-CSF once a patient already has fever and neutropenia. Although there may be a slightly more rapid return of neutrophils, the benefit is minimal, and there is no demonstrated reduction in mortality. Fluoroquinolone prophylaxis is typically reserved for high-risk patients with ongoing neutropenia. High-risk patients typically include those undergoing allogeneic hematopoietic cell transplantation or receiving induction chemotherapy for acute leukemia. Neither of these applies to this patient.
A 72-year-old woman is evaluated during a follow-up visit. She has a 3-year history of heart failure with a left ventricular ejection fraction of 25% and New York Heart Association functional class III symptoms. She has an implantable cardioverter-defibrillator. She reports manageable lightheadedness with standing. Medications are lisinopril, carvedilol, and spironolactone at maximally tolerated doses. On physical examination, the patient is afebrile, blood pressure is 98/64 mm Hg, and pulse rate is 68/min. The estimated central venous pressure is 6 cm H2O. An S3 is present. The lungs are clear to auscultation. There is no lower extremity edema. Which of the following is the most appropriate management? Add ivabradine Add valsartan-sacubitril Discontinue carvedilol and start ivabradine Discontinue lisinopril and start valsartan-sacubitril Continue current medications
Guideline-directed medical therapy for symptomatic heart failure with reduced ejection fraction includes an ACE inhibitor, β-blocker (specifically, metoprolol succinate, carvedilol, or bisoprolol), and aldosterone antagonist. The most appropriate management of this patient with heart failure with reduced ejection fraction (HFrEF) is to continue the current medication regimen. Guideline-directed medical therapy for symptomatic heart failure includes treatment with an ACE inhibitor, β-blocker (specifically, metoprolol succinate, carvedilol, or bisoprolol), and an aldosterone antagonist. All of these agents reduce mortality in patients with HFrEF. This patient is already taking guideline-directed medical therapy at maximally tolerated doses, as evidenced by her symptomatic orthostatic hypotension; therefore, no changes need to be made to her medication regimen. Ivabradine inhibits the If or "I-funny" channel of the sinoatrial node, resulting in a reduction in heart rate. In patients with HFrEF (left ventricular ejection fraction ≤35%) and New York Heart Association functional class II to IV symptoms who are in sinus rhythm with a heart rate of 70/min or higher and taking guideline-directed medical therapy, treatment with ivabradine has been associated with a reduction in heart failure hospitalizations. Before the resting heart rate is assessed for potential initiation of ivabradine, β-blockers should be titrated to maximally tolerated doses. Because this patient has a heart rate of 68/min and is already taking a maximally tolerated dose of carvedilol, she is not a candidate for ivabradine. It is important to note that ivabradine should be given in addition to β-blocker therapy, not in its place. Valsartan-sacubitril is an angiotensin receptor-neprilysin inhibitor, a new drug class that combines an angiotensin receptor blocker with a neprilysin inhibitor. The neprilysin inhibitor prevents the breakdown of B-type natriuretic peptide, leading to enhanced diuresis, natriuresis, and myocardial relaxation. Compared with ACE inhibitor therapy, this drug combination has been shown to reduce the composite endpoint of cardiovascular death or heart failure hospitalization by 20% in symptomatic patients with HFrEF. However, in a clinical trial assessing response to valsartan-sacubitril, the major cause of withdrawal was hypotension. In this patient who is taking an ACE inhibitor and β-blocker and is experiencing symptomatic orthostatic hypotension, the substitution of valsartan-sacubitril for lisinopril is not indicated. Additionally, adding valsartan-sacubitril to her current medication regimen, which includes lisinopril, would greatly increase her risk for angioedema and hypotension. ACE inhibitors should be discontinued at least 36 hours before initiating valsartan-sacubitril. If she did not have symptomatic hypotension, switching lisinopril to valsartan-sacubitril would be an appropriate option. Read Related TextNext Question
A 62-year-old man is evaluated in the emergency department after being struck by a vehicle while crossing the street 3 hours earlier and hitting his head on the pavement. He did not lose consciousness. The patient now reports a moderate headache. He describes the headache as global, dull, and aching without photophobia or phonophobia. He has had no nausea, visual changes, or neurologic symptoms since the incident. On physical examination, vital signs are normal, as are findings from a neurologic examination. Which of the following is the most appropriate initial step in management? Head CT with contrast Head CT without contrast Hospital observation Prochlorperazine administration
Guidelines recommend imaging with noncontrast CT for patients with mild traumatic brain injury (TBI) who are exposed to dangerous mechanisms of injury, such as falls from greater than 3 feet or 5 stairs, vehicular ejection, or vehicle-pedestrian motor vehicle collisions; other indications for head CT in mild TBI include age older than 60 years, severe headache, vomiting, Glasgow Coma Scale score of less than 15, focal deficit(s), posttraumatic seizure, coagulopathy, or persistent drowsiness or amnesia. This patient should have CT of the head without contrast. He sustained a mild traumatic brain injury (TBI), defined as head injury due to contact and/or acceleration or deceleration forces, and now reports headache. Guidelines recommend imaging with noncontrast CT for patients exposed to mechanisms of injury perceived to be dangerous. These include falls from greater than 3 feet or 5 stairs, vehicular ejection, or vehicle-pedestrian motor vehicle collisions. Age greater than 60 years also is considered an indication for imaging, as are severe headache, vomiting, Glasgow Coma Scale score less than 15, focal deficit(s), posttraumatic seizure, coagulopathy, or persistent drowsiness or amnesia. Most patients with mild TBI will not require brain imaging. Contrast administration with CT is helpful in the evaluation of malignant, vascular, and some inflammatory disorders of the nervous system. However, the addition of contrast to head CT adds nothing to the evaluation of acute head injury. Visualization of skull fracture and acute intracranial hemorrhage is best accomplished through noncontrasted head CT. Hospital observation of patients with mild TBI may be indicated for those with intractable headache or vomiting, focal neurologic deficits, or persistent impairment of memory or consciousness but not for this patient with less severe symptoms. In all of these settings, however, patients should first undergo head CT to exclude intracranial hemorrhage. Hospital admission without imaging would be inappropriate for patients with TBI. Intravenous prochlorperazine is indicated in the management of acute migraine in the emergency department. This patient describes neither nausea nor headache compatible with migraine.
A 52-year-old man is evaluated for gradual-onset breast tenderness and enlargement over the past 6 months. Medical history is significant for hypertension and symptomatic heart failure with reduced ejection fraction diagnosed 1 year ago. His medications are metoprolol, lisinopril, and spironolactone. He has noticed no change in his sexual function over the past year and reports the presence of morning erections. On physical examination, vital signs are normal. BMI is 31. There is notable bilateral breast tenderness with rubbery concentric masses noted bilaterally at the areolae. There is no thyroid enlargement, hepatomegaly, or testicular abnormalities including atrophy or mass. Which of the following is the most likely diagnosis? Breast cancer Germ cell tumor Hypogonadal-associated gynecomastia Pseudogynecomastia Spironolactone-induced gynecomastia
Gynecomastia can be an adverse effect of medications; spironolactone causes an imbalance between free estrogen and free androgen resulting in glandular breast tissue enlargement. This patient most likely has spironolactone-induced gynecomastia. Gynecomastia occurs due to an imbalance between free estrogen and free androgen actions in breast tissue. Spironolactone is a known cause of gynecomastia. Spironolactone can increase the aromatization of testosterone to estradiol, decrease the testosterone production by the testes, and displace testosterone from sex hormone-binding globulin, thereby increasing its metabolic clearance rate. Additionally, spironolactone also acts as an antiandrogen by binding to androgen receptors and displacing binding of testosterone and dihydrotestosterone to their receptors. Other recognized drug-related causes of gynecomastia include marijuana, alcohol, 5α-reductase inhibitors, H2-receptor antagonists, digoxin, ketoconazole, calcium channel blockers, ACE inhibitors, antiretroviral agents, tricyclic antidepressants, and selective serotonin reuptake inhibitors. Breast cancers are typically unilateral and nontender with a discrete fixed mass displaced from the nipple-areolar complex, whereas gynecomastia presents as a rubbery, concentric, subareolar mass. Gynecomastia is typically bilateral and often associated with breast tenderness. Germ cell tumors account for 95% of testicular neoplasms with 6% of patients presenting with gynecomastia at time of diagnosis. The temporal association of gynecomastia with the initiation of a medication known to cause gynecomastia and a normal testicular examination makes germ cell tumor a much less likely diagnosis. Hypogonadism, primary more so than secondary, is associated with gynecomastia due to an increase in estradiol relative to testosterone secretion. With primary hypogonadism, a rise in luteinizing hormone results in increased aromatization of testosterone to estradiol; this elevated luteinizing hormone is absent in secondary hypogonadism thus making gynecomastia a less prominent sign. This patient has unchanged sexual functioning, morning erections, and no evidence of testicular atrophy making hypogonadism an unlikely diagnosis. Pseudogynecomastia is often seen in men with obesity. It occurs due to an increase in breast fat without any proliferation of glandular tissue. Gynecomastia and pseudogynecomastia are differentiated by examination. Pseudogynecomastia is characterized by the presence of subareolar adipose tissue, without glandular proliferation. True gynecomastia typically distorts the normally flat contour of the male nipple, causing it to protrude owing to the mass of glandular tissue beneath it. In pseudogynecomastia, the nipple is typically still flat but soft, and nondescript subcutaneous fat tissue is present in the breast area.
A 31-year-old man undergoes pretravel consultation. He plans to leave in 8 days for a safari trip to Kenya. He received yellow fever and typhoid vaccinations 18 months ago, and he is undergoing a work-related three-dose hepatitis B vaccination series. He also has a prescription for prophylactic antimalarial medication. No serum IgG antibodies to hepatitis A were detected in a recent blood test. He smokes cigarettes and occasionally drinks an alcoholic beverage. On physical examination, vital signs are normal, and other findings are unremarkable. Which of the following is the most appropriate pretravel management for this patient? First dose of hepatitis A vaccine with a second dose in 7 days Immune globulin Single dose of hepatitis A vaccine Single dose of hepatitis A vaccine plus immune globulin No intervention
Hepatitis A vaccination should ideally occur 2 to 4 weeks before travel to an endemic region; however, a single dose of the vaccine given any time before travel provides adequate protection to otherwise healthy persons. This patient should be given a single dose of hepatitis A vaccine. Worldwide, hepatitis A is the most common cause of acute viral hepatitis. Infection with this human-only RNA picornavirus is spread primarily through the fecal-oral route. Rates of endemicity are highest in developing nations, where hygiene and sanitary measures are less than optimal. Since the availability of effective hepatitis A vaccines in the mid-1990s, this enteric virus has been designated the leading cause of infection and death among vaccine-preventable diseases. Infected adolescents and adults usually present with fever, malaise, nausea, and anorexia, with more than 70% exhibiting jaundice. Most infection is benign and resolves uneventfully. Rarely, patients may experience fulminant hepatitis and acute liver failure. However, hepatitis A does not become a chronic disease. Either of the two inactivated cell culture-produced vaccines that have become part of the recommended routine childhood vaccine schedule can be used. In addition to avoiding potentially contaminated food and water, vaccination is also strongly advised for those planning to travel to areas that pose significant risk of infection. Ideally, vaccination should occur at least 2 to 4 weeks before departure. Nevertheless, a single dose of vaccine given any time before travel provides adequate protection in otherwise healthy persons. Depending on the particular vaccine product used, a second dose is administered between 6 to 18 months later. Another dose of vaccine administered 7 days after the first would have no immunologic boosting effect. Passive immunization with intramuscular immune globulin adds no benefit when administered alone or in combination with vaccination for most healthy patients when time before travel is short. However, to optimally guard against infection, immune globulin is recommended with vaccination in persons with chronic liver disease or other chronic medical conditions, immunocompromised states, and older adults if travel is scheduled in less than 2 weeks. Immune globulin alone would also be warranted in persons who are allergic to or decline vaccination and in children younger than 12 months for whom the vaccine is not approved. Providing no intervention would place this traveler at undue risk of acquiring hepatitis A infection.
A 36-year-old woman is evaluated for a 6-year history of tender, foul-smelling, draining nodules in the inguinal folds. She has had occasional nodules in the axillae. She has taken multiple courses of oral clindamycin with only temporary improvement. She has a 14-pack-year smoking history. Medical history is otherwise unremarkable. On physical examination, vital signs are normal. BMI is 34. There are tender nodules, draining sinus tracts, and comedones in the inguinal folds. Scarring is present in right and left axillae. Additional inguinal skin findings are shown. Which of the following is the most likely diagnosis? Carbuncles Chancroid Epidermal inclusion cysts Hidradenitis suppurativa
Hidradenitis suppurativa is characterized by inflammatory abscesses, sinus tracts, and scarring in intertriginous areas; it is associated with smoking, obesity, and the metabolic syndrome. This patient has hidradenitis suppurativa, a chronic inflammatory disease that predominantly affects the apocrine-gland-bearing areas of the skin. The common sites are the axillae, breasts and inframammary creases, inguinal folds, and gluteal cleft. It is recognized by its characteristic inflammatory abscesses, sinus tracts (with foul-smelling drainage), and scarring in intertriginous areas. The pathogenesis of hidradenitis begins with follicular occlusion but not infection or inflammation of the apocrine glands. Following occlusion, secretions build up in the follicular duct and result in rupture and a subsequent inflammatory reaction that resembles a bacterial abscess. Following this, an acute inflammatory reaction is triggered in the surrounding tissue. The role of bacteria is controversial and is likely a secondary colonization since lesions are initially sterile, and antibiotics are not entirely effective in preventing new lesions. Hidradenitis is associated with smoking, obesity, and metabolic syndrome. The chronic and recurrent nature of hidradenitis helps distinguish it from other infectious causes. Treatment is difficult. Clindamycin-rifampin combination antibiotics, infliximab, and surgical excision have the greatest evidence of effectiveness. It is important to recognize hidradenitis suppurativa early in order to initiate management to reduce scarring and progression of the disease. Carbuncles can present as tender nodules with purulent drainage; however, these typically respond well to antibiotics and are not chronic in nature. Recurrent carbuncles or abscesses should raise suspicion for possible hidradenitis. Sinus tracts are not typically involved with carbuncles. Chancroid is a sexually transmitted disease caused by Haemophilus ducreyi. It presents with one or more painful ulcerations, typically on the penis or labia. Inguinal lymphadenopathy may be present, but sinus tracts and inflammatory abscesses are not seen. Epidermal inclusion cysts are benign nodules with a central punctum and a chamber containing keratinaceous material. When punctured, they often eject a copious amount of foul-smelling keratinaceous material. The cyst wall may occasionally rupture, leading to the formation of a tender red swollen nodule that resembles a furuncle. Epidermal inclusion cysts are typically solitary lesions with no sinus tract involvement and minimal scarring.
A 50-year-old woman is seen following screening digital mammography. She is asymptomatic and has no medical problems. Other than her age and sex, she has no additional risk factors for breast cancer and no family history of breast cancer. The mammogram report notes no suspicious lesions but indicates that the breasts are heterogeneously dense (density category C). On previous physical examination, vital signs and breast examination were normal. In addition to education on breast density, which of the following is the most appropriate adjunctive breast cancer screening? Breast MRI Digital breast tomosynthesis Repeat digital mammography in 6 months No further testing
High breast density alone does not necessitate adjunctive breast imaging other than routine screening mammography.
A 50-year-old woman is seen following screening digital mammography. She is asymptomatic and has no medical problems. Other than her age and sex, she has no additional risk factors for breast cancer and no family history of breast cancer. The mammogram report notes no suspicious lesions but indicates that the breasts are heterogeneously dense (density category C). On previous physical examination, vital signs and breast examination were normal. In addition to education on breast density, which of the following is the most appropriate adjunctive breast cancer screening? Breast MRI Digital breast tomosynthesis Repeat digital mammography in 6 months No further testing
High breast density alone does not necessitate adjunctive breast imaging other than routine screening mammography. This patient should not undergo adjunctive screening. Breast density is an increasingly recognized risk factor for breast cancer. It is categorized by the Breast Imaging Reporting and Data System (BI-RADS) as (a) almost entirely fatty, (b) scattered areas of fibroglandular tissue, (c) heterogeneously dense, or (d) extremely dense. Women with dense breasts should be informed that high breast density is common (present in up to 50% of women) and increases breast cancer risk but not breast cancer-related mortality. Dense breasts also decrease the sensitivity of mammography. However, women with high breast density without additional risk factors may experience more harms than benefits from supplemental breast imaging. Although supplemental screening with ultrasonography or MRI may increase the cancer detection rate, the impact on important clinical outcomes is unknown. Up to 90% of positive results on supplemental ultrasonography and 66% to 97% of positive results on supplemental MRI are false-positive findings, which may result in additional testing or invasive procedures. Despite this, nearly a quarter of state legislatures mandate that patients be notified of breast density findings, and some specifically recommend adjunctive screening with ultrasonography. Women with dense breasts and other risk factors that impart a lifetime risk for breast cancer of 20% to 25% or higher, as calculated by models largely dependent on family history, should also undergo breast MRI in addition to screening mammography. The use of supplemental breast MRI in women who have less than a 20% lifetime risk for breast cancer is not currently supported by guidelines. Digital breast tomosynthesis creates a three-dimensional image of the breast. Clinical studies suggest that tomosynthesis may have a sensitivity equal to or exceeding the sensitivity of digital mammography in the detection of breast cancer in women with dense breasts and that tomosynthesis may decrease the recall rate from screening mammography. Despite these promising findings that may improve screening accuracy, there are no randomized trials or long-term follow-up data. Most major breast cancer screening guidelines promote biennial screening from age 50 to 75 years. No guideline recommends biannual digital mammography for patients whose only risk factor is high breast density.
A 51-year-old woman is evaluated for increasingly poor ambulation ever since having a relapse of multiple sclerosis (MS) 20 months ago. She was diagnosed with MS 4 years ago and has been taking natalizumab, baclofen, and a vitamin D supplement since that time. On physical examination, vital signs are normal. Muscle strength is 4/5 in the left hip flexor, knee flexor, and ankle dorsiflexor. Gait is slow and deliberate with the assistance of a cane; left leg swing is noted. All other physical examination findings are unremarkable. Which of the following medications is the most appropriate treatment? Dalfampridine Dextromethorphan-quinidine Gabapentin Memantine Modafinil
In patients with multiple sclerosis and impaired mobility, dalfampridine is the most appropriate medication. This patient should be given dalfampridine. She has multiple sclerosis (MS) and had a relapse more than 1 year ago that resulted in poor recovery of left leg function, which is likely now permanent. Maintenance of mobility in patients with MS is essential for maintaining overall quality of life, and maintenance of an active, healthy lifestyle can help stave off future disability. Physical and occupational therapy is useful to ensure gait safety and improve walking ability and endurance. Assistive aids, such as braces, canes, walkers, and electrostimulatory walk-assist devices, can be useful for many patients. Two recent phase III studies found that dalfampridine, a voltage-gated potassium channel antagonist, significantly improved timed 25-foot walking speeds in patients with multiple sclerosis and gait impairment. This medication theoretically helps boost conductance through demyelinated axonal pathways, which may be of most benefit for long axons, such as those providing motor signals to the legs. As a consequence of this method of action, however, dalfampridine has the rare adverse effect of seizures and should not be used in patients with kidney impairment, given the reduced clearance of the drug and resultant potentially higher rate of seizures. Pseudobulbar affect, a less common symptomatic manifestation of MS, can act as a significant impediment to social interaction in patients with MS. This symptom manifests as uncontrolled fits of laughter or crying that occur without distinct or appropriate triggers. A successful trial of the combination agent dextromethorphan-quinidine has led to FDA approval of the use of this pharmacotherapy for pseudobulbar affect in MS. It is not effective for improving or maintaining ambulation. Gabapentin's most frequent use in MS is to treat neuropathic pain, which this patient currently is not experiencing. There is no evidence that this medication benefits impaired mobility. Memantine frequently is used in mild to severe Alzheimer disease to boost cognition but has not been shown to benefit cognitive dysfunction or to increase mobility in MS patients. In fact, one small study found transient worsening of MS symptoms in those taking memantine. Modafinil also is inappropriate. This drug is frequently used to treat MS-related fatigue but has no impact on muscle strength or walking ability.
A 68-year-old man notes 3 days of melena and the recent onset of epistaxis and easy bruising. He had no bleeding problems until the past week. He has advanced ischemic cardiomyopathy and had a left ventricular assist device (LVAD) placed 3 months ago. He had no bleeding history before LVAD implantation surgery, and his preoperative coagulation studies were normal. Medications are atorvastatin, carvedilol, lisinopril, spironolactone, and warfarin initiated after LVAD placement. On physical examination, other than a pulse rate of 112/min, vital signs are normal. Oxygen saturation is 94% breathing ambient air. He has crusted blood in the left nares, scattered ecchymoses, and multiple petechiae. The surgical scar on the anterior chest appears well healed. Stool for fecal occult blood is strongly positive. The remainder of the examination is normal. Laboratory studies: Activated partial thromboplastin time 40 s Hemoglobin 8.0 mg/dL (80 g/L) Platelet count 130,000/µL (130 × 109/L) Prothrombin time 19 s INR 2.0 Platelet Function Analyzer-100 Prolonged Aminotransferases Normal Fibrinogen 350 mg/dL (3.5 g/L) Which of the following is the most likely cause of this patient's new bleeding symptoms? Acquired von Willebrand disease Coagulopathy of liver disease Dysfibrinogenemia Immune thrombocytopenic purpura
High sheer force seen in some patients with prosthetic heart valves, abnormal native valves, and left ventricular assist device placement may cause an acquired von Willebrand disease with clinical bleeding. Acquired von Willebrand disease (vWD) should be suspected in a patient who has previously been well without bleeding symptoms but now has bleeding manifestations and laboratory features consistent with vWD. Acquired vWD typically occurs in patients who develop an autoantibody to von Willebrand factor, analogous to that seen with acquired factor VIII deficiency, and in conditions of high sheer pressure within the vasculature, traditionally in patients with a prosthetic heart valve that has become dislodged but also in patients with prosthetic heart valves that appear normal in function and in those with valvular heart disease who have not had surgery. More recently, this disorder has been linked to left ventricular assist devices (LVADs). This patient has the expected INR elevation associated with therapeutic-dose warfarin routinely prescribed after LVAD implantation. But the relatively abrupt onset of bleeding manifestations is associated with prolongation of the Platelet Function Analyzer-100 test, suggesting a new qualitative platelet defect. Acquired vWD can be confirmed by low levels of von Willebrand antigen or ristocetin cofactor activity. The liver is responsible for synthesizing all clotting factors as well as anticoagulant and antifibrinolytic factors. Because of this fact, patients with severe liver disease will have significantly prolonged prothrombin and activated partial thromboplastin times (PT and aPTT) resulting from decreased levels of coagulation factors. This patient only has slightly prolonged aPTT and PT and no history or evidence of severe liver disease, making the coagulopathy of liver disease unlikely. Acquired fibrinogen abnormalities are relatively common, whereas inherited fibrinogen disorders are rare. The most common causes include liver disease (dysfibrinogenemia), disseminated intravascular coagulation (hypofibrinogenemia), and certain medications. This patient has no evidence of liver disease, and his fibrinogen level is normal, which together exclude disseminated intravascular coagulation. Immune thrombocytopenic purpura is associated with a low platelet count and not with a qualitative platelet abnormality.
A 19-year-old man is evaluated for a 2-week history of shaking episodes sometimes associated with falling that are followed by a period of unresponsiveness. He reports having a 30-minute episode just before getting in the car to go to this appointment. Episodes have been occurring daily, sometimes as often as four times per day, and last 20 to 45 minutes. According to the patient's mother, the episodes involve limb shaking with the eyes closed, intermittent cessation of breathing, and a reddening of the face. The patient remains standing for a few seconds after the shaking starts but on occasion becomes limp and falls. He does not respond when his name is called or his arms or legs are lightly touched. The shaking, which gradually increases and decreases in intensity, typically involves both arms and sometimes the legs. He is exhausted but oriented and responsive afterward and has rapid breathing. All physical examination findings are normal. Which of the following is the most likely diagnosis? Convulsive status epilepticus Generalized tonic clonic seizure Myoclonic seizure Psychogenic nonepileptic spell/event
Historical features that suggest the diagnosis of psychogenic nonepileptic spells/events include variability of symptoms, closing of eyes, long duration of shaking that waxes and wanes, and lack of postictal confusion; confirmation with video electroencephalographic monitoring usually is required. This patient most likely is experiencing psychogenic nonepileptic spells/events (PNES). Although event capture during video electroencephalographic monitoring usually is required to confirm the diagnosis, the history is strongly suggestive of PNES because of the variability of symptoms (events not entirely stereotyped), eyes being closed, facial redness due to intermittent breath holding, shaking that starts with retained posture, long duration of shaking that waxes and wanes, limpness after falling, variable limb involvement, and rapid breathing without postictal confusion afterward. Convulsive status epilepticus (CSE) is unlikely in this patient. The (relatively) long duration of symptoms without subsequent confusion or altered mental status places doubt on the diagnosis of CSE, which is not associated with a return to baseline mental status after seizures cease. Unlike this patient's episodes, a generalized tonic-clonic seizure is characterized by stereotyped movements (essentially the same from one event to the next), eyes being open, cyanosis due to cessation of breathing, a duration of 2 to 5 minutes, stiffness and falling before symptoms of shaking, shaking that initially is fast but gradually slows and stops, and postictal confusion. Myoclonic seizures are typically very brief (<1 second) and characterized by synchronous jerking or shaking of the limbs with retained awareness and no postictal confusion afterward. Given the duration and variable nature of this patient's seizures, they are not myoclonic.
A 67-year-old woman with multiple myeloma is evaluated for back pain. The pain began several months ago but has dramatically worsened in the past 2 weeks. It is located in the lumbar and thoracic spine with associated paraspinal muscle spasms. The pain does not radiate into the buttocks or legs, and there has been no change in gait or bowel or bladder function. She rates the pain as an 8 on a 10-point scale at its worst. Medical history is significant for multiple myeloma and end-stage kidney disease on hemodialysis. Medications are acetaminophen, amlodipine, aspirin, metoprolol, sertraline, bortezomib, dexamethasone, and lenalidomide. On physical examination, vital signs are normal. Palpation elicits tenderness over the thoracic and lumbar spine. The abdomen is not distended, and there are no palpable masses. Neurologic examination is normal. Restaging CT scans from 2 months ago reveal lytic lesions in the lumbar spine and left iliac crest. Spine MRI is scheduled. Which of the following is the most appropriate treatment of this patient's pain? Fentanyl patch Gabapentin Hydromorphone Morphine Tramadol
Hydromorphone is the preferred opioid to treat cancer-related pain in patients with chronic kidney disease. The most appropriate treatment of this patient's pain is hydromorphone. This patient's back pain is caused by progressive myeloma, and she requires rapid treatment of her pain with oral opioids initially. Given the concern for worsening back pain in the setting of malignancy, she requires an urgent MRI of her spine to rule out impending malignant spinal cord compression, and aggressive pain treatment while pursuing a diagnostic strategy is critical. Hydromorphone is a potent opioid agonist that is thought to be safer in patients with severe kidney impairment, such as this patient on hemodialysis. A transdermal fentanyl patch is an effective analgesic for opioid-tolerant patients. It does not have clinically relevant active metabolites that would accumulate in the setting of end-stage kidney disease; however, it should be used only in opioid-tolerant patients. This patient is opioid naïve, and she should not be started on a long-acting agent until her total daily opioid needs are identified and an appropriate equianalgesic dose of fentanyl is calculated. Gabapentin binds to the α2δ subunit of voltage-gated calcium channels; it can be an effective nonopioid adjuvant in the treatment of various pain types, including neuropathic pain. Titration of gabapentin can be prolonged to avoid adverse effects; therefore, it would not be helpful in this patient in need of more rapid analgesia. Morphine is a prototypical opioid agonist, but its active metabolites accumulate in the setting of kidney failure and increase the risk for adverse neuroexcitatory effects with aggressive titration. Morphine, codeine, and meperidine are all contraindicated in patients with kidney failure (glomerular filtration rate <30 mL/min/1.73 m2). Tramadol is a weak opioid agonist whose analgesic activity is influenced by its inhibition of serotonin and norepinephrine reuptake. It is a poor analgesic in the setting of cancer-related pain and should not be used in patients with kidney failure due to accumulation of active metabolites. In addition, tramadol has the potential for significant drug interactions.
A 30-year-old woman is evaluated in the emergency department after she was rescued from her home where her vinyl sofa caught fire. She is intubated and unconscious. On physical examination, blood pressure is 108/78 mm Hg, pulse rate is 100/min, and respiration rate is 24/min. Oxygen saturation by pulse oximetry is 100% on mechanical ventilation using 50% oxygen. She is unresponsive. She has no visible burns on her skin, and her airway secretions are clear. Brainstem reflexes are all intact. Laboratory studies: Serum electrolytes : Sodium 140 mEq/L (140 mmol/L) Potassium 4.4 mEq/L (4.4 mmol/L) Chloride 99 mEq/L (99 mmol/L) Bicarbonate 13.1 mEq/L (13.1 mmol/L) Arterial blood gas studies: pH 7.29 Pco2 28 mm Hg (3.7 kPa) PO2 233 mm Hg (31 kPa) Carboxyhemoglobin 5% Methemoglobin 2% Lactate 11 mEq/L (11 mmol/L) The oxygen is increased to 100%. Which of the following is the most appropriate treatment? Hydroxocobalamin Hyperbaric oxygen therapy Methylene blue Sodium nitrite
Hydroxocobalamin effectively removes cyanide from the mitochondrial respiration system and is the preferred antidote for cyanide poisoning.
A 30-year-old woman is evaluated in the emergency department after she was rescued from her home where her vinyl sofa caught fire. She is intubated and unconscious. On physical examination, blood pressure is 108/78 mm Hg, pulse rate is 100/min, and respiration rate is 24/min. Oxygen saturation by pulse oximetry is 100% on mechanical ventilation using 50% oxygen. She is unresponsive. She has no visible burns on her skin, and her airway secretions are clear. Brainstem reflexes are all intact. Laboratory studies: Serum electrolytes : Sodium 140 mEq/L (140 mmol/L) Potassium 4.4 mEq/L (4.4 mmol/L) Chloride 99 mEq/L (99 mmol/L) Bicarbonate 13.1 mEq/L (13.1 mmol/L) Arterial blood gas studies: pH 7.29 Pco2 28 mm Hg (3.7 kPa) PO2 233 mm Hg (31 kPa) Carboxyhemoglobin 5% Methemoglobin 2% Lactate 11 mEq/L (11 mmol/L) The oxygen is increased to 100%. Which of the following is the most appropriate treatment? Hydroxocobalamin Hyperbaric oxygen therapy Methylene blue Sodium nitrite
Hydroxocobalamin effectively removes cyanide from the mitochondrial respiration system and is the preferred antidote for cyanide poisoning. This patient should be treated with intravenous hydroxocobalamin, which is the preferred antidote for cyanide poisoning. Cyanide toxicity is common in victims of house fires, with up to 90% of rescued victims having elevated cyanide levels and 35% having significantly elevated levels, which is higher than the rate of carbon monoxide poisoning among such victims. Cyanide disrupts oxidative phosphorylation, forcing cells to convert to anaerobic metabolism despite adequate oxygen supply. The result in severe cases is multiorgan failure with coma, seizures, and cardiovascular symptoms, including hypotension, bradycardia, heart block, and ventricular arrhythmias. Early manifestations are nonspecific. Diagnostic clues include lactic acidosis and inappropriately elevated central venous oxyhemoglobin saturation, which manifests as bright red venous blood. Cyanide levels are not readily available and because toxicity is rapidly fatal, prompt empiric treatment is imperative in suspected cases. Hydroxocobalamin avidly binds to cyanide to produce cyanocobalamin, which is soluble, nontoxic, and readily excreted. In addition, ongoing exposure, such as contaminated clothing, should also be eliminated. Hydroxocobalamin can affect accuracy of lab results for methemoglobin, lactate, and other tests, so it is important to obtain blood for these tests before administering the antidote, if possible. Carbon monoxide is removed by competitive binding of oxygen to hemoglobin. The initial treatment is administration of 100% oxygen, which reduces the half-life of carboxyhemoglobin from 5 hours to 90 minutes. Hyperbaric oxygen therapy yields an even higher alveolar PO2, thereby reducing the half-life to 30 minutes while substantially increasing the amount of oxygen directly dissolved in blood. However, hyperbaric oxygen therapy would not be appropriate because this patient's carboxyhemoglobin level is not high enough to suggest severe carbon monoxide poisoning. Hyperbaric oxygen is usually recommended for levels of 25% to 40% or higher, or for victims with lower levels who are pregnant. Methylene blue would be recommended for toxic levels of methemoglobin, usually 20% to 30% or higher, but it is not indicated for this patient. Although sodium nitrite is an antidote for cyanide poisoning, it is contraindicated in victims of smoke inhalation because it works by inducing methemoglobinemia, which would further impair oxygen delivery by additive or synergistic effects on oxygen binding and delivery in cases of carbon monoxide toxicity. Sodium thiosulfate is also used as an antidote for cyanide toxicity and is safer than sodium nitrite, but has a slower onset of action. It is considered second-line therapy after hydroxocobalamin, but the two agents can be given to the same patient, possibly with synergistic effect. However, they should not be administered simultaneously or through the same intravenous catheter.
A 56-year-old woman is evaluated for hypernatremia. She was admitted to the ICU 6 days ago for pyelonephritis and septic shock requiring intubation, administration of fluids, norepinephrine, and cefepime. She developed nonoliguric acute kidney injury. She has been weaned off the norepinephrine, and she has been extubated. Her serum sodium level has increased from 142 mEq/L (142 mmol/L) to 148 mEq/L (148 mmol/L) over the past 72 hours. Physical examination and vital signs are normal. Urine output is 2.5 L over the past 24 hours. Laboratory studies: Blood urea nitrogen 74 mg/dL (26.4 mmol/L) Creatinine 2.8 mg/dL (247.5 µmol/L) Electrolytes : Sodium 148 mEq/L (148 mmol/L) Potassium 3.7 mEq/L (3.7 mmol/L) Chloride 112 mEq/L (112 mmol/L) Bicarbonate 26 mEq/L (26 mmol/L) Glucose 136 mg/dL (7.5 mmol/L) Urine osmolality 420 mOsm/kg H2O Which of the following is the most likely cause of this patient's hypernatremia? Adrenal insufficiency Central diabetes insipidus Glycosuria Osmotic diuresis
Hypernatremia may be caused by osmotic diuresis, in which the urine osmolality is usually between 300 and 600 mOsm/kg H2O. Urea osmotic diuresis is the most likely cause of this patient's hypernatremia. This patient is recovering from acute kidney injury and is having a urea diuresis, with an elevated urine osmolality of 420 mOsm/kg H2O. In osmotic diuresis, urine osmolality is usually between 300 and 600 mOsm/kg H2O. The majority of the osmolality of the urine is made up of nonelectrolytes. This loss of electrolyte-free water is causing her serum sodium level to increase. The two major nonelectrolytes found in urine are urea and glucose. Her glucose is only 136 mg/dL (7.5 mmol/L), below the threshold for glucose appearing in the urine; therefore, the likely cause is excretion of urea. This can be confirmed by measuring urea in the urine. Hyponatremia is found in 70% to 80% of patients with adrenal insufficiency and is a consequence of sodium loss and volume depletion caused by mineralocorticoid deficiency and increased vasopressin secretion caused by cortisol deficiency. In addition, hyperkalemia and hyperchloremic acidosis is found in approximately 50% of patients with adrenal insufficiency. Hypernatremia and hypokalemia would be unusual manifestations of adrenal insufficiency. In the absence of antidiuretic hormone, excessive water is excreted by the kidneys, and the urine osmolality is low. The patient's urine osmolality is 420 mOsm/kg H2O, making diabetes insipidus unlikely. In addition, there is no reason to suspect diabetes insipidus in this woman. Nevertheless, it would not be possible to rule out partial nephrogenic diabetes insipidus until her urea normalized. Glycosuria is not the cause of this patient's hypernatremia because her glucose level is below the tubular threshold for reabsorption, approximately 180 mg/dL (10 mmol/L). A urine dipstick would verify the lack of glycosuria.
A 40-year-old woman is evaluated after having two relapses of multiple sclerosis, which was diagnosed 3 years ago. She works as a human resource manager and has no other medical issues. The patient has been taking once-weekly interferon beta-1a by intramuscular injection and a vitamin D supplement since diagnosis and has been adherent to the drug regimen. After reviewing her options, she elects to change her disease-modifying therapy to the oral medication fingolimod. All physical examination findings, including vital signs, are normal. Which of the following should now be a part of her drug monitoring program? JC virus antibody testing Ophthalmologic examinations Serum creatinine measurement Tuberculin skin testing
In patients with multiple sclerosis who take fingolimod as a disease-modifying therapy, regular ophthalmic examinations are necessary because of the increased risk of macular edema with this medication. This patient should now have regular ophthalmologic examinations. Fingolimod is a once-daily pill that results in sequestration of activated lymphocytes in lymph nodes. Fingolimod reduces relapse rates by approximately half over 2 years compared with placebo and also reduces the risk of disability progression and accumulation of new MRI lesions. In clinical trials of fingolimod as treatment of multiple sclerosis (MS), the medication was found to confer a 0.5% risk of macular edema. This adverse effect is reversible and typically resolves after discontinuation of the drug. If unrecognized, however, macular edema can result in permanent visual deficits. Therefore, regular macular examinations (that is, visual evaluation or measurement through optical coherence tomography) are recommended for patients taking this medication. Fingolimod also requires first-dose monitoring because of the first-dose bradycardia that occurs in most patients and should not be used in patients with heart block. There have been rare reports of patients who take fingolimod as a disease-modifying therapy for MS developing progressive multifocal leukoencephalopathy (PML). However, no evidence suggests that serum antibodies to the JC virus are a useful tool for predicting this adverse effect of the drug. Extensive data do support serum testing to predict the risk of PML in patients with MS who take natalizumab, in whom PML is much more common. The overall risk is approximately 1 in 1000 but is significantly higher in patients who have previous exposure to immunosuppressant agents or chemotherapy and those who have elevated serum titers of antibodies against the JC virus. Stratification of risk by treatment history and JC virus antibody testing is thus an important step before initiating natalizumab. Kidney complications have not been associated with fingolimod use, which makes serum creatine measurement every 6 months unnecessary. Tuberculosis has not been reported as an adverse effect of fingolimod. Although the risk of opportunistic infections may be increased with use of this drug, tuberculin skin testing is not recommended specifically, and given this patient's office-based occupation, she has no specific expected exposures that would require such testing.
A 50-year-old man is evaluated during a follow-up visit for hypertension. Losartan was started 3 months ago. He is asymptomatic. Family history is notable for his father who was on chronic hemodialysis and died of a ruptured brain aneurysm at the age of 62 years. On physical examination, blood pressure is 152/96 mm Hg, and pulse rate is 75/min; other vital signs are normal. BMI is 20. The thyroid gland is not enlarged. Abdominal examination reveals bilateral flank fullness and tenderness on deep palpation; the abdomen is otherwise soft and without bruits. The remainder of the examination is unremarkable. Laboratory studies: Bicarbonate 26 mEq/L (26 mmol/L) Creatinine 1.0 mg/dL (88.4 µmol/L) Potassium 4.7 mEq/L (4.7 mmol/L) Urinalysis Trace protein; 10-20 erythrocytes; 0-5 leukocytes Which of the following is the most appropriate diagnostic test to perform next? Kidney ultrasonography Plasma aldosterone concentration/plasma renin activity ratio Plasma fractionated metanephrines Thyroid-stimulating hormone
Hypertension is common in patients with autosomal dominant polycystic kidney disease (ADPKD), often preceding chronic kidney disease; kidney ultrasonography is the most common and least costly screening and diagnostic method for ADPKD.
A 50-year-old man is evaluated during a follow-up visit for hypertension. Losartan was started 3 months ago. He is asymptomatic. Family history is notable for his father who was on chronic hemodialysis and died of a ruptured brain aneurysm at the age of 62 years. On physical examination, blood pressure is 152/96 mm Hg, and pulse rate is 75/min; other vital signs are normal. BMI is 20. The thyroid gland is not enlarged. Abdominal examination reveals bilateral flank fullness and tenderness on deep palpation; the abdomen is otherwise soft and without bruits. The remainder of the examination is unremarkable. Laboratory studies: Bicarbonate 26 mEq/L (26 mmol/L) Creatinine 1.0 mg/dL (88.4 µmol/L) Potassium 4.7 mEq/L (4.7 mmol/L) Urinalysis Trace protein; 10-20 erythrocytes; 0-5 leukocytes Which of the following is the most appropriate diagnostic test to perform next? Kidney ultrasonography Plasma aldosterone concentration/plasma renin activity ratio Plasma fractionated metanephrines Thyroid-stimulating hormone
Hypertension is common in patients with autosomal dominant polycystic kidney disease (ADPKD), often preceding chronic kidney disease; kidney ultrasonography is the most common and least costly screening and diagnostic method for ADPKD. Kidney ultrasonography is the most appropriate diagnostic test to perform in this patient. The presence of hypertension, microhematuria, and a positive family history of chronic kidney disease requiring dialysis, as well as a brain aneurysm, raises clinical suspicion for autosomal dominant polycystic kidney disease (ADPKD). Clinical manifestations of ADPKD include gradual kidney enlargement, which may cause persistent abdominal pain and/or early satiety. Hypertension is common in patients with ADPKD, often preceding chronic kidney disease. Kidney cyst enlargement leads to stimulation of the intrarenal and circulating renin-angiotensin-aldosterone system. More than 50% of patients with ADPKD develop recurrent flank or back pain; causes include kidney stones, cyst rupture or hemorrhage, or infection. Nephrolithiasis occurs in approximately 20% of patients. A ruptured intracranial cerebral aneurysm resulting in a subarachnoid or intracerebral hemorrhage is the most serious extrarenal complication of ADPKD. Kidney ultrasonography is the most common and least costly screening and diagnostic method for ADPKD; it would reveal bilaterally enlarged kidneys with multiple cysts. Calculation of the plasma aldosterone concentration/plasma renin activity ratio is used to screen for primary hyperaldosteronism, characterized by a triad of resistant hypertension, metabolic alkalosis, and hypokalemia. Although <50% of patients with primary hyperaldosteronism manifest hypokalemia, this diagnosis cannot account for the patient's flank fullness on palpation and hematuria or his family history of kidney failure. Plasma fractionated metanephrines are obtained to screen for a pheochromocytoma, which could result in hypertension, but this patient has no symptoms or signs (for example, resistant hypertension, headaches, sweating) to indicate the presence of this tumor, and this diagnosis cannot account for the patient's flank fullness and hematuria. Although hyperthyroidism can be a secondary cause of hypertension, the patient does not manifest any symptoms or signs (heat intolerance, resting tachycardia, thyromegaly) to suggest this, making measurement of thyroid-stimulating hormone unnecessary.
A 52-year-old woman is evaluated in the emergency department for a 2-day history of lower extremity weakness and nausea. She reports no diarrhea. History is significant for hypertension treated with amlodipine. She has a history of alcohol abuse. On physical examination, vital signs are normal. On neurologic examination, lower extremity strength is 4/5. The remainder of the examination is unremarkable. Laboratory studies: Albumin 3.0 g/dL (30 g/L) Calcium 8.4 mg/dL (2.1 mmol/L) Creatinine 0.7 mg/dL (61.9 µmol/L) Electrolytes : Sodium 136 mEq/L (136 mmol/L) Potassium 2.1 mEq/L (2.1 mmol/L) Chloride 104 mEq/L (104 mmol/L) Bicarbonate 26 mEq/L (26 mmol/L) Magnesium 1.4 mg/dL (0.58 mmol/L) Urine potassium 30 mEq/L (30 mmol/L) Which of the following is the most likely cause of this patient's hypokalemia? Hypoalbuminemia Hypocalcemia Hypomagnesemia Poor nutrition
Hypomagnesemia can cause symptomatic hypokalemia via renal losses of potassium; importantly, the hypokalemia will be refractory to therapy until magnesium is repleted. The most likely cause of this patient's hypokalemia is hypomagnesemia. Symptoms of hypokalemia include weakness or paralysis; decreased gastrointestinal motility or ileus with nausea; and cardiac arrhythmias. This patient has symptomatic hypokalemia, presenting with lower extremity weakness and nausea. Hypokalemia can be caused by renal losses or nonrenal losses. In this case, the urine potassium is inappropriately high, pointing toward renal potassium wasting. Renal losses of potassium can occur with renal tubular acidosis; renal excretion of non-reabsorbable anions such as bicarbonate, hippurate, and ketones; drugs such as aminoglycosides or cisplatinum; Gitelman and Bartter syndromes; or hypomagnesemia. Hypomagnesemia is common, occurring in up to 12% of hospitalized patients. Chronic alcohol abuse results in excessive urinary excretion of magnesium and appears to reflect a reversible alcohol-induced tubular dysfunction. Intracellular magnesium is necessary to modulate the excretion of potassium through the potassium channel in the cortical collecting tubule. Hypomagnesemia results in loss of potassium through this channel. Importantly, the hypokalemia will be refractory to therapy until magnesium is repleted. Target levels for magnesium replacement are at least 2 mg/dL (0.83 mmol/L). Unlike calcium and magnesium, which have substantial binding to albumin, potassium levels are not affected by the concentration of albumin. Low levels of magnesium (due to alcohol abuse or malnutrition) activate G-proteins that stimulate calcium-sensing receptors and decrease parathyroid hormone (PTH) secretion and are a cause of hypocalcemia. Low magnesium levels are also associated with resistance to PTH activity at the level of bone, further contributing to hypocalcemia. Hypocalcemia and hypokalemia in this patient are a direct result of chronic magnesium loss due to the patient's alcoholism. Hypocalcemia has no significant effect on potassium concentration. Because the kidneys can reduce urine potassium excretion to <20 mEq/24 h (20 mmol/24 h), hypokalemia from inadequate intake is uncommon. Urinary or gastrointestinal losses of potassium are most common. Assessment of urine potassium excretion is critical to establish renal potassium wasting. Urine potassium loss >20 mEq/24 h (20 mmol/24 h), a spot urine potassium >20 mEq/L (20 mmol/L), or a spot urine potassium-creatinine ratio >13 mEq/g (1.5 mEq/mmol) suggests excessive urinary losses. Conversely, urine potassium loss <20 mEq/24 h (20 mmol/24 h) suggests cellular shift, decreased intake, or extrarenal losses of potassium. This patient's high urinary potassium suggests urinary loss rather than poor nutrition as the cause of hypokalemia.
A 42-year-old man comes to the office for a follow-up evaluation 4 weeks after undergoing laparoscopic abdominal surgery. Two days after the operation, he developed severe, unrelenting pain of the right shoulder and arm lasting 1 week followed by progressive weakness of the right shoulder and proximal arm. He currently has no pain but cannot maintain the right arm above his head. The patient reports numbness affecting the right arm and says that his grip has become weak. He has had no problem with speech, swallowing, or moving the lower extremities. He has hyperlipidemia treated with atorvastatin. On physical examination, vital signs are normal. Motor examination reveals moderate weakness of arm abduction, forearm extension, and hand grip on the right side. Winging of the right scapula is present. Trapezius and deltoid muscle bulk is reduced on the right side compared with the left. Right biceps and brachioradialis reflexes are absent, but other deep tendon reflexes are intact. Patchy areas of sensory loss are noted over the right shoulder, forearm, and palm. The cranial nerves are intact, the lower extremities have normal strength bilaterally, and gait and balance are both normal. Results of laboratory studies show an erythrocyte sedimentation rate of 18 mm/h, a serum creatine kinase level of 150 U/L, and a hemoglobin A1c value of 5.7%. MRI of the cervical spine is unremarkable. Which of the following is the most likely diagnosis? Compressive polyradiculopathy Idiopathic brachial plexopathy Lambert-Eaton myasthenic syndrome Statin myopathy
Idiopathic brachial plexopathy is characterized by subacute severe pain followed by resolution of pain and progressive weakness and atrophy involving the shoulder girdle and upper extremity muscles; this syndrome often is triggered by a preceding event, such as an infection or surgery. Idiopathic brachial plexopathy (also known as neuralgic amyotrophy and Parsonage-Turner syndrome) is characterized by subacute severe pain followed by resolution of pain and progressive weakness and atrophy involving the shoulder girdle and upper extremity muscles. This syndrome often is triggered by a preceding event, such an infection or surgery. In this patient, motor and sensory involvement in the territory of multiple nerves and roots is consistent with a process involving the brachial plexus. Imaging rules out a pathologic process at the level of the cervical spine. An iatrogenic traumatic plexopathy caused by surgery is unlikely, given the absence of weakness immediately after the procedure and the abdominal location of the procedure. Although compressive polyradiculopathy secondary to degenerative spinal disease can simulate plexopathy, with a presentation consisting of mixed motor, sensory, and pain symptoms in multiple nerve root distributions, negative cervical imaging rules out this entity in this patient. Lambert-Eaton syndrome is associated with neuromuscular junction dysfunction. Sensory deficits, prominent pain, and focal weakness are atypical. Statins can cause a toxic myopathy, but sensory deficits are not expected findings in myopathies, and asymmetry also is atypical. Moreover, in this patient, rapid weakness appeared after resolution of pain, whereas in statin myopathy, pain often persists along with muscle necrosis-induced weakness.
A 58-year-old man is admitted to the hospital for a 2-week history of worsening constipation. He has end-stage heart failure (New York Heart Association functional class IV) and stage 3 chronic kidney disease. Medications include bisoprolol, furosemide, losartan, spironolactone, hydromorphone (for dyspnea palliation), bisacodyl, lactulose, senna, docusate, and tap water enema. On physical examination, respiration rate is 20/min; other vital signs are normal. Oxygen saturation is 92% breathing ambient air. Cardiac examination reveals an S3, jugular venous distention, and peripheral edema. Crackles are auscultated at the lung bases. Moderate abdominal distention is noted, with tenderness to palpation. Which of the following is the most appropriate treatment of this patient's constipation? Lubiprostone Methylnaltrexone Polyethylene glycol Sodium phosphate enema
If maximal medical therapy has failed to achieve laxation in patients taking opioids, peripheral opioid antagonists, such as methylnaltrexone, can be considered; methylnaltrexone does not reverse the analgesic or antidyspneic effects of systemically administered opioids.
A 58-year-old man is admitted to the hospital for a 2-week history of worsening constipation. He has end-stage heart failure (New York Heart Association functional class IV) and stage 3 chronic kidney disease. Medications include bisoprolol, furosemide, losartan, spironolactone, hydromorphone (for dyspnea palliation), bisacodyl, lactulose, senna, docusate, and tap water enema. On physical examination, respiration rate is 20/min; other vital signs are normal. Oxygen saturation is 92% breathing ambient air. Cardiac examination reveals an S3, jugular venous distention, and peripheral edema. Crackles are auscultated at the lung bases. Moderate abdominal distention is noted, with tenderness to palpation. Which of the following is the most appropriate treatment of this patient's constipation? Lubiprostone Methylnaltrexone Polyethylene glycol Sodium phosphate enema
If maximal medical therapy has failed to achieve laxation in patients taking opioids, peripheral opioid antagonists, such as methylnaltrexone, can be considered; methylnaltrexone does not reverse the analgesic or antidyspneic effects of systemically administered opioids. The most appropriate treatment of this patient's constipation is methylnaltrexone. This patient presents with significant constipation refractory to enema therapy, osmotic laxatives (lactulose), and stimulants (bisacodyl, senna). There are several causes of this patient's constipation, but special attention must be paid to his opioid use for dyspnea palliation. In patients with laxative-refractory opioid-induced constipation, the American Gastroenterological Association (AGA) Institute recommends two peripherally acting µ-opioid receptor antagonists, oral naldemedine or subcutaneous methylnaltrexone, and naloxegol, a pegylated form of naloxone. By reversing μ-opioid receptor activation in the gut, methylnaltrexone can cause laxation in less than 60 minutes and does not reverse analgesic or antidyspneic effects of systemically administered opioids. It is contraindicated in patients with bowel obstruction. When constipation symptoms do not respond to osmotic and stimulant laxative therapy, the chloride channel activator lubiprostone can be considered. Lubiprostone is FDA approved for the treatment of opioid-induced constipation; however, the AGA Institute concluded there was insufficient evidence to recommend lubiprostone. Regardless, lubiprostone can cause shortness of breath and is likely a poor choice for a patient taking opioids to palliate dyspnea. Polyethylene glycol is an osmotic laxative that increases the water content of stools to improve bowel motility. This patient is already taking an osmotic laxative (lactulose), and the addition of polyethylene glycol is unlikely to have any effect in the setting of opioid-induced bowel dysmotility refractory to stimulants and enema therapy. Sodium phosphate enemas are contraindicated in older adult patients and in patients with kidney failure or heart failure because of the risks for dangerous electrolyte shifts and renal toxicity.
A 31-year-old man is evaluated during a follow-up visit for IgA nephropathy found on kidney biopsy 3 months ago, at which time lisinopril was initiated. He is asymptomatic. Physical examination and vital signs are unremarkable. Laboratory studies: Current 3 Months Ago Creatinine 1.1 mg/dL (97. 2 µmol/L) 1.0 mg/dL (88.4 µmol/L) Potassium 4.8 mEq/L (4.8 mmol/L) 4.4 mEq/L (4.4 mmol/L) Urinalysis 3+ blood; 2+ protein 3+ blood; 3+ protein Urine protein-creatinine ratio 700 mg/g 1200 mg/g Which of the following is the most appropriate next step in management? Add losartan Add oral glucocorticoid therapy Start alternating courses of intravenous and oral glucocorticoid therapy Make no changes to the current medication regimen
No changes need to be made to this patient's current medication regimen. He was diagnosed with IgA nephropathy 3 months ago, at which time the ACE inhibitor lisinopril was initiated. Antiproteinuric therapy using an ACE inhibitor or angiotensin receptor blocker (ARB) is the hallmark for treating IgA nephropathy and remains the most proven therapy in slowing progression of the disease. This patient currently has preserved kidney function and proteinuria <1000 mg/24 h; therefore, continuing conservative therapy with the ACE inhibitor lisinopril is appropriate. Combined use of any of the three renin-angiotensin system drug classes (ACE inhibitor, ARB, and direct renin inhibitors) is not recommended given the results of several clinical trials that revealed more adverse events with these combinations (hyperkalemia, hypotension, acute kidney injury), without additional cardiovascular or renal benefits. Therefore, adding the ARB losartan to this patient's medication regimen is not recommended. The risk for disease progression appears to be significantly increased when patients have proteinuria >1000 mg/24 h, particularly in the setting of reduced kidney function. Studies on using immunosuppression (particularly glucocorticoids) have used this 1000 mg/24 h proteinuria threshold for enrollment and have demonstrated conflicting results. Although earlier small studies have shown a clear benefit in adding glucocorticoids (oral or a combination of intravenous and oral) to ACE inhibitors or ARBs when proteinuria is >1000 mg/24 h, more recent larger studies (STOP-IgAN and TESTING studies) have raised concerns about the toxicity of this treatment strategy outweighing any potential benefits. Therefore, the use of immunosuppression in IgA nephropathy remains controversial.
A 52-year-old man fails to attend a scheduled appointment. He was initially evaluated for bilateral knee osteoarthritis 1 year ago, and treatment with weight loss, NSAIDs, and physical therapy was recommended. Over the past year, the patient missed three scheduled appointments, did not attend physical therapy, arrived for urgent care assessment twice with requests for stronger pain medications, and did not complete sufficient trials of oral nonopioid pharmacologic agents. Attempts to reach the patient by phone to discuss adherence to his care plan have not been successful. The visit today was scheduled to discuss the difficulties in his treatment and assess his barriers to care. Medical history is significant for bipolar disorder. In past visits, he has not appeared manic or suicidal. Which of the following is the most appropriate management? Refer the patient to a psychiatrist Report the patient to the local mental health crisis team Send the patient a letter warning that the relationship may be terminated Terminate the patient relationship immediately
If the physician-patient relationship becomes irreparably compromised because of lack of trust, lack of mutual goals, or failure to maintain an effective working relationship despite efforts to resolve differences, the relationship can be terminated The most appropriate management is to send the patient a formal, written warning informing him that the patient-physician relationship may be terminated unless he is able to meaningfully participate in the plan of care. Physician-patient relationships are formed on the basis of mutual agreement. Rarely, the relationship fails to reach mutual goals and becomes unproductive. In some cases, the patient may not adhere to recommended therapies or may demonstrate inappropriate behavior with the physician or staff members, and it may be appropriate for the physician to terminate the relationship. After reasonable attempts to resolve differences have failed, the patient should be notified in writing that the relationship has been terminated and that care should be obtained from a different provider, usually with a several-week time frame for the patient to continue receiving urgent care. Terminations should occur only if the patient is medically stable and when alternative care is available. If a patient threatens a physician or staff member, the termination may be immediate. Although a psychiatrist might provide interventions to help this patient better adhere to care recommendations, such a referral is unnecessary in making a decision to terminate an ineffective physician-patient relationship. This patient has not demonstrated signs of an unstable mental health condition that warrants intervention by a crisis team. Patient abandonment is unethical and may be a cause for legal action. In this case, the patient has not yet received a formal warning that his failure to adhere to treatment goals may result in his termination from the practice. Therefore, he should not be released from the practice immediately.
A 28-year-old woman undergoes follow-up evaluation after a recent hospitalization for meningococcal bacteremia. Lumbar puncture results during hospitalization were negative for meningitis. She completed a course of ceftriaxone 1 week ago and reports feeling well since stopping antibiotics. Medical history is notable for gonococcal arthritis of her knee 2 years ago. She has a sister with a history of meningococcal meningitis. She takes no medications. On physical examination, vital signs are normal. The remainder of the examination is unremarkable. Which of the following is the most appropriate preventive measure? Intravenous immune globulin Plasma infusion Prophylactic ciprofloxacin Quadrivalent meningococcal conjugate vaccine
Immunization with the quadrivalent meningococcal conjugate vaccine is the mainstay of infection prevention in patients with terminal complement deficiency. The most appropriate preventive measure for this patient is immunization with the quadrivalent meningococcal conjugate vaccine. A personal history of recurrent Neisseria infection or history of infection in multiple family members suggests a deficiency in one of the terminal complement components that make up the membrane attack complex (MAC). The MAC comprises C5 to C9, and a deficiency in any of these constituents leads to an impaired ability to combat Neisseria infections, particularly N. meningitidis. Patients are at risk for recurrent meningococcal infection, often caused by unusual serogroups. For unclear reasons, infection in this population is often uncharacteristically mild. Evaluation for terminal complement deficiency is performed by quantitation of total hemolytic complement (CH50). If the CH50 is low, more specific testing for individual components of the MAC may be performed. Immunization is the mainstay of infection prevention in patients with defects in terminal complement. The Advisory Committee on Immunization Practices recommends use of a conjugate quadrivalent meningococcal vaccine as well as a vaccine active against serogroup B meningococcal infection, with a booster of the conjugate quadrivalent meningococcal vaccine given every 5 years for adults with terminal complement deficiency. These patients should also receive both pneumococcal (polysaccharide and conjugate) and Haemophilus influenzae type B vaccines. Intravenous immune globulin does not have appreciable levels of complement and would not be useful for infection prevention in a patient with terminal complement deficiency. Although plasma is rich in complement, plasma infusion is not a feasible long-term approach to repletion of complement levels. Additionally, such treatment would be associated with an increased risk of bloodborne diseases and the potential development of antibody against the missing component. Prophylactic ciprofloxacin has a role in reducing the risk of meningococcal disease after close exposure to an infected person, but no data support using chronic prophylactic antibiotics in patients with terminal complement deficiencies. Instead, patients should be counseled to be vigilant for development of fever, rash, headache, or other symptoms concerning for Neisseria infection.
A 49-year-old-man is evaluated 1 day after having an episode of right arm weakness without pain that lasted 5 minutes. He is now asymptomatic. The patient has type 2 diabetes mellitus and dyslipidemia. Medications are aspirin, metformin, and atorvastatin. On physical examination, blood pressure is 126/68 mm Hg, pulse rate is 86/min and regular, and respiration rate is 12/min. No carotid bruits or cardiac murmurs are heard on cardiac auscultation. All other physical examination findings are normal. An electrocardiogram shows normal sinus rhythm with no ST-segment or T-wave changes. Which of the following is the most appropriate initial imaging test? Carotid duplex ultrasonography CT angiography of the neck MRI of the brain Transesophageal echocardiography
In a patient with a transient ischemic attack, carotid duplex ultrasonography is an inexpensive, readily available, and noninvasive imaging modality for identifying high-grade stenosis and the possible need for surgery. This patient should have carotid duplex ultrasonography. He most likely has had a transient ischemic attack (TIA) referable to the left hemisphere and is at risk of ischemic stroke within the next 90 days, with the highest risk occurring within the first 2 days. Although his ABCD2 score (based on patient Age, Blood pressure, Clinical presentation, Duration of symptoms, and presence of Diabetes mellitus) of 3 indicates a predicted stroke risk of 1.3% and the need for hospitalization and rapid evaluation, these scores are neither sensitive nor specific enough to identify patients at highest risk of stroke. Several studies have identified extracranial symptomatic internal carotid artery stenosis as an indicator of stroke risk of greater than 70% after TIA; this risk can be modified with carotid revascularization. Several modalities are available for imaging the internal carotid artery and identifying extracranial symptomatic stenosis, including duplex ultrasonography, CT angiography, and magnetic resonance angiography (MRA). Duplex ultrasonography has the benefit of being inexpensive, readily available, low risk, and noninvasive and thus is an appropriate early study. If the carotid duplex ultrasound shows high-grade stenosis, confirmation of this finding by CT angiography or MRA is required before surgical intervention. If the carotid duplex ultrasound is unrevealing, then additional vessel imaging of the neck is unnecessary. CT angiography of the neck is more costly and less widely available than carotid duplex ultrasonography. Additionally, the radiation exposure makes CT angiography less desirable as an initial imaging test. Brain MRI is not the initial test of choice because the presence or absence of a cerebral infarct will not immediately change medical management or affect the patient's stroke risk. Cerebral imaging with either CT or MRI eventually may be required in this patient but should occur after an evaluation of the internal carotid artery that may lead to hospitalization and surgical intervention. In patients with risk factors for stroke after a TIA, transesophageal echocardiography is (TEE) is unlikely to immediately change management, is invasive, and has low yield for finding an embolic source of stroke in patients who are in sinus rhythm. TEE can be considered in certain patients with stroke if they are young and have no apparent risk factors for stroke or if unusual causes are suspected, such as cardiac tumor, patent foramen ovale, aortic arch atherosclerosis, or endocarditis.
A 59-year-old woman comes to the office for a follow-up evaluation of a 2-year history of worsening urinary urgency and frequency. Episodes of urinary incontinence have occurred more often in the past 6 months because she cannot walk fast enough to get to the bathroom in time. Recently, she also has experienced intermittent urinary hesitancy and a frequent feeling of incomplete emptying, sometimes requiring changing body position or manual pelvic pressure. The patient has a 5-year history of secondary progressive multiple sclerosis. She takes no disease-modifying therapy. Her only medication is a vitamin D supplement. Physical examination findings are unremarkable. Urinalysis results are normal. Which of the following is the most appropriate next step in management? Dalfampridine administration Oxybutynin administration Solifenacin administration Urodynamic testing
In a patient with multiple sclerosis and mixed urinary symptoms that suggest both a hypertonic and a hypotonic bladder, urodynamic testing to guide medication choices is appropriate, as are the potential use of urinary catheterization and monitoring of postvoid residuals while the patient is treated. This patient should undergo urodynamic testing. Her symptoms and history of multiple sclerosis (MS) suggest the presence of a neurogenic bladder. Given the mix of symptoms of urgency and frequency (symptoms of an overactive, spastic bladder) plus hesitancy and retention (symptoms of a hypotonic bladder or overactive sphincter), the patient has aspects of both hypertonic and hypotonic bladder. She also may have bladder-sphincter dyssynergia, in which the contraction of the bladder wall is not properly timed with relaxation of the urinary sphincter. Management of complicated forms of neurogenic bladder involves proper diagnosis through urodynamic testing (which will guide medication choices), potential use of urinary catheterization, and monitoring of postvoid residuals while the patient is treated to avoid urinary retention. Involvement of a urologist is often required. Dalfampridine has been shown to improve walking speed in patients with MS. Although the patient says that incontinence occurs when she is unable to get to the bathroom fast enough, use of this medication would not address the underlying cause of her urinary symptoms. Further, frequent urinary tract infections are a common adverse effect of this medication, and thus using it without first addressing her current urinary symptoms would be counter-productive. Urinary frequency and urgency are more readily managed than other patterns of bladder dysfunction and are often amenable to abstinence from caffeine, timed voids, and anticholinergic medications, such as oxybutynin or tolterodine. Patients with urinary hesitancy or retention, such as this patient, should not be treated with anticholinergic agents because these medications can worsen retention and lead to predisposition to urinary tract infections. This patient with mixed bladder symptoms should be evaluated with urodynamic testing. Solifenacin has been shown to reduce symptoms of overactive bladder in patients with MS. However, this patient's mixed bladder symptoms suggest that she is prone to urinary retention. Use of this medication without concurrent use of catheterization and monitoring of postvoid residuals would result in worsened urinary retention and predispose the patient to more frequent urinary tract infections and, possibly, bladder cancer. Read Related TextNext Question
A 19-year-old man is evaluated in the hospital after admission for subacute onset of bilateral lower extremity paraplegia, urinary incontinence, and sensory deficits. His initial treatment was a 5-day course of high-dose intravenous methylprednisolone. Four days after completion of the infusions, no clinical improvement has occurred. The patient had flu-like symptoms for several days before onset of neurologic symptoms but otherwise has been healthy. On physical examination, vital signs are normal. Muscle strength is 0/5 in both legs and 5/5 in both arms. Reflexes are absent in the lower extremities and normal in the upper extremities. Moderate sensory loss is noted below T3 bilaterally. Cerebrospinal fluid analysis: Erythrocyte count 2/µL (2 × 106/L) Leukocyte count 38/µL (38 × 106/L), with a predominance of lymphocytes Glucose Normal Protein 62 mg/dL (620 mg/L) A T2-weighted MRI of the thoracic spine shows a hyperintense lesion in the thoracic cord at T2 with peripheral contrast enhancement. An MRI of the brain is normal. Which of the following is the most appropriate next step in treatment? Antituberculosis drug regimen Inpatient rehabilitation only Intravenous immunoglobulin therapy Plasma exchange therapy
In a patient with signs and symptoms of idiopathic transverse myelitis in whom first-line treatment with a high-dose intravenous glucocorticoid has been ineffective, plasma exchange therapy is the most appropriate next step in treatment. This patient is exhibiting signs and symptoms consistent with idiopathic transverse myelitis (TM). Idiopathic TM is a monophasic inflammatory and demyelinating myelopathy affecting a portion of the spinal cord. Affected patients frequently experience a subacute onset of weakness, sensory changes, and bowel or bladder dysfunction, which is sometimes preceded by back pain or a thoracic banding sensation. Diagnostic criteria for idiopathic TM require the presence of clinical features of the syndrome, evidence of inflammation (either leukocytosis in the cerebrospinal fluid or contrast enhancement on MRI), and exclusion of other potential causes. First-line treatment for this disorder is administration of high-dose intravenous glucocorticoids. A 5-day course of high-dose methylprednisolone, however, had no beneficial effect on this patient's symptoms. The most appropriate next step is plasma exchange therapy, which has been shown to improve outcomes in patients with idiopathic transverse myelitis that is refractory to glucocorticoids. The symptoms experienced by this patient are unlikely to be the result of tuberculosis. Tuberculosis can cause central nervous system infection and inflammation, but when it does, it most often involves the meninges, which are not affected in this patient, and spinal fluid analysis frequently shows a low glucose level and a mononuclear pleocytosis. The acute onset of symptoms also is highly unusual for tuberculosis. Although this patient will benefit from inpatient rehabilitation, it should be delayed until all of the other treatment options have been exhausted. Transfer to rehabilitation without initiation of plasma exchange would not provide the patient with the best chance for recovery. Intravenous immune globulin therapy has not been shown to provide benefit in glucocorticoid-refractory transverse myelitis and thus would be inappropriate treatment for this patient.
A 19-year-old woman is evaluated in the emergency department for a 6-month history of headaches that initially were intermittent but have been occurring daily for the past 6 weeks. She describes the pain as bilateral, frontotemporal, and a steady pressure. Neck stiffness has been present for 8 weeks. Vision intermittently darkens for seconds. For the past 3 nights, she has noted pulsatile tinnitus. The patient has inflammatory acne treated with minocycline and topical benzoyl peroxide gel. On physical examination, vital signs are normal; BMI is 22. Papilledema and left abducens nerve (cranial nerve VI) palsy are noted, but all other findings are normal. Results of brain MRI and magnetic resonance venography are normal. Which of the following is the most appropriate management? Amitriptyline Indomethacin Lumbar puncture Magnetic resonance angiography Temporal artery biopsy
In a patient with suspected idiopathic intracranial hypertension, lumbar puncture is required to confirm the diagnosis. This patient should have a lumbar puncture. The clinical picture is compatible with idiopathic intracranial hypertension (IIH), and lumbar puncture is required to confirm the diagnosis. Although most presentations of IIH occur in the context of obesity in women of child-bearing age, several other risk factors have been identified. Exposure to minocycline or other members of the tetracycline class of antibiotics has been linked to the development of IIH. Retinoic acid, estrogen and progesterone supplements, and glucocorticoids are other drugs implicated as causes of this disorder. Headaches, vision changes, and intracranial noises are the most common presenting symptoms. Visual blurring, diplopia, and brief episodic dimming (obscurations) also are common, and papilledema is characteristic; abducens nerve (cranial nerve VI) palsy occurs occasionally. Cerebrospinal fluid (CSF) analysis typically shows an elevated opening pressure (>250 mm H2O) with normal CSF composition. The treatment of choice is acetazolamide. Amitriptyline is the preventive medication of choice for chronic tension-type headache (CTTH). Although the headache described in this patient is similar to that of CTTH, the presence of vision changes and intracranial noises would be atypical, and abducens nerve palsy is incompatible with that diagnosis. Of the indomethacin-responsive headache syndromes, only hemicrania continua is possible in a patient with daily headache. However, the bilateral location of the headache pain, the absence of cranial autonomic features, and the presence of papilledema and abducens nerve palsy are incompatible with that diagnosis. Magnetic resonance angiography is indicated in cases of suspected intracranial or cervical arterial disease. Although oculomotor nerve (cranial nerve III) palsy is common with an aneurysm of the posterior communicating artery, isolated abducens nerve palsy from vascular lesions is unusual. Carotid or vertebral dissections may result in Horner syndrome or, less commonly, ophthalmoplegia but not papilledema. Temporal artery biopsy would be indicated in cases of suspected giant cell arteritis, which most commonly presents after age 50 years. Despite classic involvement of the temporal artery, pain may occur in various head locations. Amaurosis and jaw claudication are common associated features. Although cranial nerve palsies occasionally are noted, papilledema is not.
A 79-year-old woman is evaluated for persistent depression, declining function, and memory changes. According to family members, she has been depressed ever since her husband died 3 years ago. They further report that the patient's memory has gotten much worse in the past year and that she no longer cooks or cleans her home, despite previously preparing lavish weekly family meals and keeping a meticulous house. The patient says she feels healthy but "just no longer cares" or sometimes "forgets." She started taking paroxetine at the time of her husband's death and switched to venlafaxine 1 year later because of no improvement in her depression; although symptoms are now slightly less severe, they have not abated. On physical examination, vital signs are normal. The two-question depression screening test is positive, and her score on the Patient Health Questionnaire (PHQ)-9 is 11/30 (moderate depression). The patient scores 23/30 (normal, ≥24) on a Mini-Mental State Examination; an evaluation for reversible causes of memory impairment is unrevealing. An MRI obtained 1 year ago was normal. Which of the following is the most appropriate next step? Addition of mirtazapine Cerebrospinal fluid analysis Neuropsychological testing Repeat MRI of the brain
In a patient with treatment-refractory depression who also has cognitive and functional decline, neuropsychological testing can be useful in identifying a pattern or degree of cognitive impairment that is not typical of depression-related cognitive impairment. This patient should undergo neuropsychological testing. She has treatment-refractory depression but also cognitive impairment and some functional deficits. Given the progression of the cognitive symptoms, additional diagnostic studies should be pursued to evaluate for cognitive impairment independent of that associated with depression. More than half of patients with late-life major depression exhibit clinically significant cognitive impairment, most frequently affecting processing speed, executive function, and visuospatial ability. Notably, late-life depression may be a prodromal feature of Alzheimer disease. A history should be obtained from the patient and someone well acquainted with the patient who can provide information on current daily functioning relative to premorbid functioning. A tool for identifying and assessing the severity of depression is the Patient Health Questionnaire (PHQ)-9. A score of 5 to 9 indicates mild depression, 10 to 14 moderate depression, 15 to 19 moderately severe depression, and 20 or higher severe depression. In a patient with possible depression-related cognitive and functional decline, a trial of antidepressant therapy is an appropriate first step. Psychotherapy (cognitive-behavioral therapy, psychodynamic therapy, and interpersonal therapy) and psychopharmacology, alone or in combination, are mainstays of treatment and can prove synergistic. Patients who do not respond to full-dose antidepressant monotherapy for 6 weeks may respond to a different antidepressant drug, either from the same or a different class, or the addition of a second antidepressant drug. If the patient does not respond to an adequate trial of therapy, other diagnoses should be considered. Because this patient has not responded to two different antidepressants, the addition of mirtazapine is unlikely to be helpful. Neuropsychological testing can be useful in identifying a pattern or degree of cognitive impairment that is not typical of depression-related cognitive impairment. When results of neuropsychological testing are more consistent with Alzheimer disease than depression, additional diagnostic studies, such as 18F-fluorodeoxyglucose PET, could be pursued, as could a trial of cognitive enhancers (such as acetylcholinesterase inhibitors). Although some Alzheimer disease-specific biomarkers can be detected by cerebrospinal fluid (CSF) analysis, abnormal Aβ42 peptide levels are often seen in an older population (30%-50%) in the absence of cognitive impairment. These CSF studies are most helpful in younger patients to discriminate normal from abnormal findings. A normal head CT scan and normal MRI of the brain do not rule out an underlying neurodegenerative process, particularly if a disease is in its mild stages, and are thus inappropriate in a patient with probable depression as the cause of her symptoms.
A 29-year-old woman is evaluated before attempting pregnancy. Juvenile myoclonic epilepsy was diagnosed 11 years ago, at which time she started taking valproic acid; she has had no symptoms for 10.5 years. Her only other medication is an oral contraceptive agent. She is concerned about taking her medications if she becomes pregnant. All physical examination findings are normal, as was her most recent electroencephalogram. A plan is made to discontinue the oral contraceptive, start folic acid, and then taper the valproic acid. Which of the following is the most appropriate additional step in treatment? Gabapentin Levetiracetam Oxcarbazepine Topiramate No additional treatment is necessary
In a woman with childbearing potential, levetiracetam and lamotrigine are the most appropriate treatment options because of their relatively low risk of teratogenicity. Besides discontinuing the oral contraceptive, starting folic acid, and then tapering off valproic acid, this patient should begin taking levetiracetam for her juvenile myoclonic epilepsy (JME). This type of generalized epilepsy requires a history of myoclonic seizures for diagnosis but is also usually associated with generalized tonic-clonic seizures. In a woman with childbearing potential, levetiracetam and lamotrigine are the most appropriate treatment options because of their relatively low risk of teratogenicity. Levetiracetam is often preferred because lamotrigine can worsen myoclonus in some patients. Valproic acid is strongly associated with neural tube defects and lower IQ in offspring and should be avoided in women with childbearing potential, unless absolutely necessary. In this patient, who has been seizure free for 10.5 years and has never taken another antiepileptic drug, replacing valproic acid with another medication should be attempted. In patients whose epilepsy is difficult to control with other medications, valproic acid may be continued during pregnancy. Gabapentin and oxcarbazepine also have a relatively favorable profile in terms of lower fetal risk, but both are known to worsen generalized epilepsy and thus should be avoided in patients with JME. Maternal topiramate use is associated with cleft lip/palate in offspring. This teratogenic drug should be avoided by women with childbearing potential. JME most often requires lifelong treatment, even if patients have normal results on testing and are seizure free for many years. Therefore, providing this patient no additional treatment is inappropriate. Read Related TextNext Question
A 38-year-old man is evaluated after passing his second kidney stone. History is significant for chronic pancreatitis secondary to a past history of alcohol abuse. He has three to four loose bowel movements each day. He reports no fever, flank pain, or dysuria. There is no family history of kidney disease, hyperparathyroidism, or nephrolithiasis. Current medications are pancreatic enzymes and multivitamins. Physical examination reveals a thin man. Vital signs and the remainder of the examination are unremarkable. Laboratory studies: Calcium 8.5 mg/dL (2.1 mmol/L) Creatinine 0.7 mg/dL (61.9 µmol/L) Electrolytes : Sodium 137 mEq/L (137 mmol/L) Potassium 3.5 mEq/L (3.5 mmol/L) Chloride 104 mEq/L (104 mmol/L) Bicarbonate 21 mEq/L (21 mmol/L) Urinalysis Specific gravity; pH 5.0; negative dipstick; positive for calcium oxalate crystals In addition to increasing fluid intake, which of the following is the most appropriate management? Add allopurinol Add potassium citrate Add vitamin C Decrease calcium intake Increase protein intake
In addition to increasing fluid intake, potassium citrate is appropriate to prevent future calcium oxalate stones in this patient. Patients with chronic diarrhea and malabsorption are at increased risk for forming calcium oxalate stones for three reasons. First, because of the diarrhea and concomitant metabolic acidosis, urine citrate, an inhibitor of crystallization, is often reduced. In addition, volume depletion from the diarrhea decreases urine volume and thus increases the concentration of calcium and oxalate in the urine. Finally, in malabsorption, especially fat malabsorption as occurs in chronic pancreatitis, enteric calcium binds to fat as opposed to oxalate, leaving oxalate free to be absorbed and excreted in the urine. Although treatment should be based on the metabolic evaluation in this patient, his low urine pH and low serum bicarbonate level suggest that he has metabolic acidosis. Decreased systemic pH lowers urine citrate excretion. Supplementation with citrate as a base equivalent will help correct the acidosis and increase urine citrate, bind urinary calcium, and decrease the formation of calcium oxalate stones. If the 24-hour urine metabolic evaluation showed elevated urine uric acid or if stone analysis revealed a uric acid nidus, allopurinol could be considered; however, in the absence of this information, allopurinol should not be prescribed. Vitamin C increases urine oxalate excretion and would not have the desired effect of decreasing calcium oxalate stone formation. Restricting dietary calcium intake in patients with hypercalciuria may paradoxically increase the risk of kidney stone formation by causing decreased binding of calcium with oxalate in the gut with increased absorption and urinary excretion of oxalate; therefore, dietary calcium should not be limited. Increased protein intake increases glomerular filtration, and therefore the excretion of calcium, and would not contribute to decreased kidney stone formation. In addition, high protein diets may exacerbate hypocitraturia.
A 46-year-old man is evaluated for increased urination and thirst of 3 days' duration. History is significant for pulmonary sarcoidosis diagnosed 4 months ago; prednisone, 40 mg/d, was initiated and subsequently tapered to 10 mg/d. Initial symptoms were cough and dyspnea on exertion, which had improved with treatment. He is taking no other medications. Physical examination and vital signs are normal. Laboratory studies: Blood urea nitrogen 16 mg/dL (5.7 mmol/L) Calcium 9.9 mg/dL (2.5 mmol/L) Creatinine 1.1 mg/dL (97.2 µmol/L) Electrolytes : Sodium 146 mEq/L (146 mmol/L) Chloride 110 mEq/L (110 mmol/L) Potassium 3.8 mEq/L (3.8 mmol/L) Bicarbonate 26 mEq/L (26 mmol/L) Urine sodium 20 mEq/L (20 mmol/L) Urine osmolality 115 mOsm/kg H2O In addition to increasing the prednisone, which of the following is the most appropriate treatment? Desmopressin acetate Hydrochlorothiazide Intravenous 5% dextrose Tolvaptan
In addition to increasing the prednisone, the antidiuretic hormone (ADH) analogue desmopressin acetate is the most appropriate treatment. This patient most likely has central nervous system sarcoidosis and central diabetes insipidus (DI). Nearly half of hypothalamic-pituitary sarcoidosis cases occur in the course of previously treated sarcoidosis. Central DI results from inadequate release of ADH from the posterior pituitary gland. In the presence of ADH, aquaporin water channels are inserted in the collecting tubules and allow water to be reabsorbed. In the absence of ADH, excessive water is excreted by the kidneys. Frank hypernatremia is unusual because patients develop extreme thirst and polydipsia, and with free access to water, can maintain serum sodium in the high normal range. When patients do not drink enough to replace the water lost in the urine, due to poor or absent thirst drive or lack of free access to water, they develop hypernatremia. DI is diagnosed with simultaneous laboratory evidence of inability to concentrate urine (urine osmolality <300 mOsm/kg H2O) in the face of elevated serum sodium and osmolality. If necessary, a water deprivation test can confirm the diagnosis. A response to exogenous ADH would support a diagnosis of central DI, whereas a lack of response is seen in nephrogenic DI. Although volume depletion induced by hydrochlorothiazide will help decrease urine output and conserve water, this treatment is reserved for patients with nephrogenic DI and is not necessary in central DI. The hypernatremia in this case is mild, without symptoms, and can be rapidly treated with the administration of desmopressin acetate; therefore, it is not necessary to administer intravenous 5% dextrose. Tolvaptan is a vasopressin receptor antagonist sometimes used for the syndrome of inappropriate antidiuretic hormone secretion, which is characterized by normal volume status, hyponatremia, and inappropriately elevated urine osmolality (not present in this patient).
A 71-year-old man is evaluated in the hospital for an elevated serum creatinine level. He was hospitalized 2 days ago for a 4-day history of progressive right lower leg cellulitis. History is also significant for type 2 diabetes mellitus with prior episodes of cellulitis. Medications are basal and prandial insulin. On physical examination, temperature is 38.9 °C (102.0 °F), blood pressure is 150/100 mm Hg, pulse rate is 100/min, and respiration rate is 20/min. A well-defined area of tender erythema and edema is present over the right foot and leg to just below the knee. The remainder of the examination is unremarkable. Laboratory studies: Leukocyte count 13,500/µL (13.5 × 109/L) C3 50 mg/dL (500 mg/L) C4 12 mg/dL (120 mg/L) Creatinine On admission: 2.4 mg/dL (212.2 µmol/L); baseline: 1.1 mg/dL (97.2 µmol/L) Urinalysis 3+ blood; 3+ protein Urine protein-creatinine ratio 4100 mg/g Kidney biopsy shows endocapillary proliferation on light microscopy, co-dominant granular staining for C3 and IgA on immunofluorescence microscopy, and subepithelial hump-like deposits on electron microscopy. Which of the following is the most likely cause of this patient's kidney disease? Staphylococcus aureus Streptococcus agalactiae Streptococcus pneumoniae Streptococcus pyogenes
In adults, most cases of infection-related glomerulonephritis are no longer poststreptococcal, and the glomerulonephritis often coexists with the triggering infection. Staphylococcus aureus is the most likely cause of this patient's infection-related glomerulonephritis (IRGN). This patient has acute kidney injury in the setting of cellulitis, with active urine sediment and low serum complement levels. The biopsy shows a proliferative glomerulonephritis on light microscopy with immunofluorescence of C3 and IgA and subepithelial hump-like deposits on electron microscopy, confirming a diagnosis of IRGN. In the developed world, the epidemiology of IRGN has drastically shifted over the past few decades, moving away from streptococcal-associated glomerulonephritides to infections caused primarily by S. aureus and, at a significantly lower rate, gram-negative bacteria. In this patient with cellulitis and IRGN occurring at the time of infection, S. aureus is the most likely culprit pathogen. In patients with poststreptococcal glomerulonephritis (group A Streptococcus, or Streptococcus pyogenes), there is a latent period between the resolution of the streptococcal infection and the acute onset of the nephritic syndrome, usually 7 to 10 days after oropharyngeal infections and 2 to 4 weeks after skin infections. In adults with non-poststreptococcal IRGN, the glomerulonephritis often coexists with the triggering infection. Sites of infection can include the upper and lower respiratory tract, skin/soft tissue, bone, teeth/oral mucosa, heart, deep abscesses, shunts, and indwelling catheters. Notably, this patient's kidney failure has occurred at the same time as the cellulitis, consistent with a staphylococcal-mediated form of IRGN. Streptococcus pneumoniae is an uncommon cause of cellulitis, and Streptococcus agalactiae (group B Streptococcus) is capable of causing cellulitis in nonpregnant adults in special circumstances (lymphedema, vascular insufficiency, chronic dermatitis, or radiation-induced cutaneous injury). S. pyogenes and S. aureus are much more common causes of cellulitis, and the co-occurrence of the infection and IRGN points to S. aureus as the most likely culprit.
A 71-year-old man is evaluated for a 10-day history of malaise and fatigue. Two years ago, he underwent dual-chamber pacemaker implantation for third-degree atrioventricular block. He reports discomfort at the site of his pacemaker. He has had no fever, chills, or weight loss. Medical history is otherwise significant for well-controlled hypertension and hyperlipidemia treated with chlorthalidone and atorvastatin. On physical examination, vital signs are normal. The skin overlying the patient's pacemaker pocket is slightly erythematous and is warm and tender to palpation. The remainder of the physical examination is noncontributory. A complete blood count and erythrocyte sedimentation rate are ordered. Which of the following is the most appropriate management? Blood cultures Empiric therapy with cephalexin Pacemaker pocket aspiration PET/CT scanning
In all patients suspected of having infection of a cardiac implanted electronic device, a minimum of two blood cultures should be drawn from separate sites. The most appropriate management of this patient is to obtain a complete blood count, erythrocyte sedimentation rate, and two sets of blood cultures. Patients with a cardiac implanted electronic device can develop a localized tissue infection at the implant site (pocket infection) or a systemic infection with bacteremia (such as endocarditis). These infections can occur after initial implantation, late after implantation, or after a battery replacement or revision. The most common pathogens are skin colonizers, such as the coagulase-negative Staphylococcus spp. and S. aureus. Symptoms of cardiac device infection include fever, chills, malaise, lassitude, or failure to thrive, especially in the elderly. There may also be local findings suggestive of infection, such as redness or warmth at the pacemaker pocket. These patients should undergo laboratory evaluation for signs of infection. An elevated erythrocyte sedimentation rate, leukocytosis with a left shift, and anemia all suggest infection of a cardiac implanted electronic device. All patients suspected of having infection (with or without fever) should have a minimum of two blood cultures drawn from separate sites. Once there is suspicion for infection of a cardiac implanted electronic device, referral to an electrophysiologist or an infectious disease specialist is mandatory. Empiric antibiotic therapy without first identifying the presence and cause of the infection is not best medical practice. In addition, antibiotic therapy for a confirmed infection without removal of the device is not curative and is associated with a high fatality rate. Pacemaker pocket aspiration should never be performed because it can seed a sterile pocket and lead to infection, especially if there is superficial cellulitis without deeper tissue involvement. In patients with high suspicion of infection despite equivocal laboratory findings, PET/CT can help determine whether there is evidence of local tissue inflammation consistent with a device infection. However, the basic evaluation for infection should be completed and analyzed first.
A 72-year-old man is hospitalized for a 1-week history of worsening shortness of breath; he also has worsening lower extremity edema despite an increase in his furosemide dose 2 days ago. History is significant for hypertension, stage G3a chronic kidney disease, and heart failure with a preserved ejection fraction. Outpatient medications are amlodipine, lisinopril, furosemide, and low-dose aspirin. On physical examination, blood pressure is 112/60 mm Hg, and pulse rate is 97/min. BMI is 28. Cardiac examination reveals an elevated jugular venous pressure and an S4. Breath sounds are diminished at the lung bases. There is 2+ pitting edema of the lower legs. Laboratory studies: Blood urea nitrogen 64 mg/dL (22.8 mmol/L); 2 weeks ago, 40 mg/dL (14.3 mmol/L) Creatinine 2.3 mg/dL (203.3 µmol/L); 2 weeks ago, 1.9 mg/dL (168 µmol/L) Sodium 130 mEq/L (130 mmol/L); 2 weeks ago, 133 mEq/L (133 mmol/L) Urinalysis Specific gravity 1.009; 1+ protein; few hyaline casts Chest radiograph shows bibasilar effusions and vascular congestion. Which of the following is the most appropriate treatment? Add conivaptan Add dobutamine infusion Increase furosemide Start ultrafiltration
In cardiorenal syndrome type 1, loop diuretics are first-line therapy for managing volume overload in patients with decompensated heart failure with evidence of peripheral and/or pulmonary edema. Increasing the furosemide dose is the most appropriate treatment in this patient with cardiorenal syndrome type 1 (CRS1). CRS is a disorder of the heart and kidneys in which acute or long-term dysfunction in one organ induces acute or long-term dysfunction in the other. CRS is characterized by the triad of concomitant decreased kidney function, diuretic-resistant heart failure with congestion, and worsening kidney function during heart failure therapy. CRS1 is defined as a worsening kidney function in patients with acute worsening of cardiac function (decompensated heart failure, acute coronary syndrome, cardiogenic shock). Management is challenging because treatment directed toward improving cardiac function can worsen kidney function. For this patient, an increase to his loop diuretic dose to a sufficient dose to induce a diuresis is appropriate. Among patients with decompensated heart failure, the best outcomes may occur with aggressive fluid removal even if associated with mild to moderate worsening of kidney function. An elevated blood urea nitrogen (BUN)-creatinine ratio should not discourage the use of diuretic therapy in patients with evidence of congestion. The decline in his kidney function with relative increase in BUN-creatinine ratio reflects the CRS1 physiology, rather than volume depletion due to diuresis. Vasopressin receptor antagonists such as conivaptan can be used for the treatment of patients with hypervolemic or euvolemic hyponatremia. However, there is no evidence that treatment of hyponatremia improves clinical outcomes in patients with severe chronic heart failure. Dobutamine, as well as milrinone, is used in the management of cardiogenic shock or decompensated acute heart failure with severe impaired left ventricular function and would not be indicated for a patient with preserved ejection fraction. Ultrafiltration therapy is reserved for patients with severe volume overload refractory to medical management. In the Cardiorenal Rescue Study in Acute Decompensated Heart Failure (Caress-HF) study, the use of a stepwise pharmacologic therapy algorithm was superior to ultrafiltration for the preservation of kidney function at 96 hours, with a similar weight loss between the two strategies. Read Related TextNext Question
A 42-year-old man is admitted to the ICU for nonresponsive pneumonia with a 7-day history of shortness of breath and cough. He was diagnosed with pneumonia and prescribed levofloxacin 3 days ago and has been adherent to his medication; however, his shortness of breath has worsened to the point he is unable to climb one flight of stairs. Medical history is remarkable for hospitalization 5 years ago for alcohol withdrawal and delirium tremens. He drinks a six-pack of beer every weekday and a case of beer on the weekends. His only medication is levofloxacin. On physical examination, he is in mild respiratory distress but alert and oriented. Temperature is 38.7 °C (101.7 °F), blood pressure is normal, pulse rate is 122/min, and respiration rate is 24/min. Oxygen saturation is 89% breathing ambient air. Pulmonary auscultation reveals decreased breath sounds at the right lung base. Laboratory studies show a leukocyte count of 18,700/µL (18.7 × 109/L) and a serum creatinine level of 2.3 mg/dL (203 µmol/L). A chest radiograph shows a right lower lobe infiltrate. Levofloxacin is discontinued. In addition to initiating azithromycin, which of the following is the most appropriate antibiotic treatment? Ceftriaxone Clindamycin Metronidazole Piperacillin-tazobactam
In community-dwelling patients with aspiration pneumonia, the most common organisms are anaerobic bacteria, such as microaerophilic streptococci, Fusobacterium, Peptostreptococcus, and Prevotella species as well as Enterobacteriaceae.
A 42-year-old man is admitted to the ICU for nonresponsive pneumonia with a 7-day history of shortness of breath and cough. He was diagnosed with pneumonia and prescribed levofloxacin 3 days ago and has been adherent to his medication; however, his shortness of breath has worsened to the point he is unable to climb one flight of stairs. Medical history is remarkable for hospitalization 5 years ago for alcohol withdrawal and delirium tremens. He drinks a six-pack of beer every weekday and a case of beer on the weekends. His only medication is levofloxacin. On physical examination, he is in mild respiratory distress but alert and oriented. Temperature is 38.7 °C (101.7 °F), blood pressure is normal, pulse rate is 122/min, and respiration rate is 24/min. Oxygen saturation is 89% breathing ambient air. Pulmonary auscultation reveals decreased breath sounds at the right lung base. Laboratory studies show a leukocyte count of 18,700/µL (18.7 × 109/L) and a serum creatinine level of 2.3 mg/dL (203 µmol/L). A chest radiograph shows a right lower lobe infiltrate. Levofloxacin is discontinued. In addition to initiating azithromycin, which of the following is the most appropriate antibiotic treatment? Ceftriaxone Clindamycin Metronidazole Piperacillin-tazobactam
In community-dwelling patients with aspiration pneumonia, the most common organisms are anaerobic bacteria, such as microaerophilic streptococci, Fusobacterium, Peptostreptococcus, and Prevotella species as well as Enterobacteriaceae. In addition to starting azithromycin for empiric coverage of atypical organisms, piperacillin-tazobactam is the most appropriate antibiotic treatment for this patient. He requires ICU admission for severe, progressive community-acquired pneumonia (CAP) after outpatient therapy with levofloxacin has failed to improve his infection. Levofloxacin was an appropriate empiric treatment choice for CAP in this patient, considering his history of heavy alcohol use, which increases the risk of infection with Enterobacteriaceae, including Klebsiella species. Antibiotic treatment failure can result from poor adherence, a noninfectious cause of symptoms (such as pulmonary embolism), a nonbacterial infection (such as histoplasmosis), lack of source control (as with empyema), or infection with a resistant organism. The patient's history of heavy alcohol use suggests the possibility of aspiration pneumonia, whereby normal oropharyngeal organisms gain access to the lower airways and cause infection. Other risk factors for aspiration pneumonia include poor dentition, gastroesophageal reflux, dysphagia, vomiting, and reduced consciousness, as can be seen in patients with alcoholism, illicit drug use, or seizures. The localization to the right lower lobe is also consistent with aspiration because this area is dependent when patients are lying supine. In community-dwelling patients with aspiration pneumonia, the most common organisms are anaerobic bacteria, such as microaerophilic streptococci, Fusobacterium, Peptostreptococcus, and Prevotella species; however, Enterobacteriaceae may also be present. Therefore, an empiric agent active against anaerobic organisms and gram-negative bacteria is indicated; piperacillin-tazobactam meets these criteria. Ceftriaxone has adequate coverage against Enterobacteriaceae (as well as other more common flora such as Streptococcus pneumoniae) but has limited activity against anaerobic organisms. Therefore, this agent would not be ideal when aspiration pneumonia is a concern. Clindamycin is active against gram-positive anaerobic organisms but not against gram-negative agents, such as Fusobacterium or Prevotella. It also lacks activity against other aerobic gram-negative bacilli, such as Haemophilus influenzae. Metronidazole has excellent activity against anaerobic gram-negative rods but limited utility against anaerobic gram-positive cocci and streptococcal species, so it would not be appropriate as empiric therapy for aspiration pneumonia.
A 43-year-old man is evaluated in the emergency department for a 1-week history of cough, shortness of breath, chest pain, and night sweats. He has a 25-pack-year smoking history. On physical examination, temperature is 38.8 °C (102 °F), blood pressure is 134/82 mm Hg, pulse rate is 142/min, and respiration rate is 30/min. Oxygen saturation is 88% breathing ambient air. There are decreased breath sounds at the right base and dullness to percussion. The remainder of the examination is noncontributory. Laboratory studies reveal a leukocyte count of 29,000/µL (29.0 × 109/L). Chest radiograph shows a large right pleural effusion with associated compressive atelectasis or consolidation, and consolidation in the right upper lobe. The patient is prescribed broad-spectrum antibiotics and a diagnostic thoracentesis is performed that removes 100 mL of serous pleural fluid. Pleural fluid studies: pH 7.0 Lactate dehydrogenase 2310 U/L Total protein 5.2 g/dL (52 g/L) Glucose 42 mg/dL (2.3 mmol/L) Gram stain Negative Which of the following is the most likely diagnosis? Complicated parapneumonic effusion Empyema Malignant effusion Uncomplicated parapneumonic effusion
In general, parapneumonic effusions associated with a pH less than 7.2 or pleural fluid glucose level less than 60 mg/dL (3.3 mmol/L) require thoracostomy drainage in addition to antibiotics. This patient has community-acquired pneumonia and complicated parapneumonic effusion. A complicated parapneumonic effusion is defined as an effusion associated with a pneumonia that has a pH less than 7.2 and glucose less than 60 mg/dL (3.3 mmol/L). Complicated parapneumonic effusions occur when bacteria invade the pleural space. However, because bacteria may be cleared rapidly from the pleural space, the Gram stain is typically negative and cultures are usually sterile. Complicated parapneumonic effusions have a variable response to antibiotics alone. Pleural effusions greater than 10 mm in depth on chest radiograph and associated with a pneumonic illness should be sampled. In general, these require thoracostomy tube drainage when the pH is less than 7.2 or the pleural fluid glucose level is less than 60 mg/dL (3.3 mmol/L). The American College of Chest Physicians consensus guidelines concur as thoracostomy drainage speeds clinical recovery and hospital discharge. An empyema is defined as a bacterial infection of the pleural space that results in frank pus on visual inspection of the pleural fluid or a positive Gram stain. A positive pleural fluid culture is not required for diagnosis as cultures are less sensitive than Gram stain in the detection of bacteria. The management of empyema includes early thoracic surgical consultation because thoracoscopic or open debridement and drainage is often required to successfully manage this condition. However, this patient's pleural fluid was described as serous and the Gram stain was negative making empyema an unlikely diagnosis. Pleural fluid acidosis (pH less than 7.3) is seen in complicated parapneumonic effusions, tuberculous pleuritis, rheumatoid and lupus pleuritis, esophageal rupture, and malignancy. A low pleural fluid glucose level results from either increased utilization within the pleural space (bacteria, malignant cells) or decreased transport into the pleural space (rheumatoid pleurisy), and a concentration less than 60 mg/dL (3.3 mmol/L) narrows the differential diagnosis significantly. Although a malignant effusion can have a low pH and glucose, this patient's presentation is more consistent with a parapneumonic effusion. An uncomplicated parapneumonic effusion is characterized by a pH greater than 7.2 and glucose greater than 60 mg/dL (3.3 mmol/L). These effusions do not require drainage and typically resolve with antibiotic therapy alone.
A 31-year-old woman seeks preconception counseling. She has end-stage kidney disease secondary to focal segmental glomerulosclerosis. She received a haploidentical kidney transplant from her brother 2 years ago. She had an acute rejection episode 18 months ago that was successfully treated with glucocorticoids, and kidney function has been stable since that time. She currently feels well and reports no symptoms. Current medications are mycophenolate mofetil, tacrolimus, and prednisone. On physical examination, vital signs are normal. The allograft is palpable in the right lower quadrant and is nontender. Laboratory studies show a serum creatinine level of 1.3 mg/dL (114.9 µmol/L). Which of the following is the most appropriate management? Advise against pregnancy Discontinue mycophenolate mofetil; begin azathioprine Discontinue tacrolimus; begin cyclosporine Proceed with pregnancy without further intervention
In kidney transplant recipients who are planning pregnancy, mycophenolate mofetil, sirolimus, and everolimus must be discontinued 3 to 6 months prior to conception and replaced with azathioprine, which is generally safer and well tolerated in pregnancy. The most appropriate management for this kidney transplant recipient who is planning pregnancy is to discontinue mycophenolate mofetil and begin azathioprine. Fertility increases after kidney transplantation, although fertility rates remain lower and pregnancy complications are higher compared with the general population. Pregnancy planning for a kidney transplant recipient is essential to improve outcomes and includes adjusting medications and optimizing clinical status. Kidney transplant recipients should wait 1 to 2 years with a stable allograft before attempting conception. Other comorbid conditions (such as systemic lupus erythematosus) should also be stable prior to conception. Outcomes are improved with better allograft function (serum creatinine <1.5 mg/dL [132.6 µmol/L]) and stable immunosuppression. Mycophenolate mofetil (as well as sirolimus and everolimus) is teratogenic and needs to be replaced 3 to 6 months prior to conception with azathioprine, which is generally safer and well tolerated in pregnancy. Calcineurin inhibitors (both tacrolimus and cyclosporine) have been used safely in pregnancy. Therefore, tacrolimus does not need to be discontinued and replaced with cyclosporine in this patient. Although this would constitute a high-risk pregnancy, the success rate of pregnancies in patients with kidney transplants is high; with stable allograft function, a normotensive patient and her fetus have favorable prognoses, and therefore the patient could proceed with pregnancy following modification of her maintenance immunosuppression regimen.
A 65-year-old man is evaluated after a recent diagnosis of prostate cancer with multiple painful metastases to the lumbar spine. He was treated initially with both bicalutamide and leuprolide for 4 weeks, after which he continued on leuprolide alone with excellent symptomatic relief of bone pain. He started treatment 2 months ago. On physical examination, vital signs and the remainder of the examination are normal. Laboratory studies reveal a serum prostate-specific antigen level of 20 ng/mL (20 µg/L), down from 122 ng/mL (122 µg/L) at the time of diagnosis 2 months ago. Complete blood count and liver chemistry test results are normal. Which of the following is the most appropriate management? Continue leuprolide alone Leuprolide plus docetaxel Radiation to the lumbar spine Stop leuprolide and start docetaxel
In men with clinical metastatic prostate cancer that is castrate sensitive, docetaxel along with continuation of androgen deprivation therapy has been shown to improve survival and is accepted as standard care. The most appropriate management is leuprolide plus docetaxel. Standard therapy for metastatic prostate cancer or evidence of disseminated disease based on an elevated serum prostate-specific antigen (PSA) is androgen deprivation therapy (ADT). ADT consists of inhibiting androgen synthesis by using a gonadotropin-releasing hormone (GnRH) agonist (such as leuprolide) or blocking the androgen receptor with an antiandrogen agent (such as flutamide or bicalutamide). Bilateral orchiectomy is a reasonable alternative to GnRH agonist therapy, particularly in the very elderly. ADT is typically combined with docetaxel chemotherapy in patients with extensive metastatic disease. This patient has castrate-sensitive metastatic prostate cancer. He presented with clinical metastatic disease and had a significant response to treatment with leuprolide, based on both a reduction in his PSA level and improvement in pain related to his metastatic lesion to the lumbar spine. Two randomized trials have confirmed a survival benefit associated with combined ADT and docetaxel chemotherapy for men with castrate-sensitive metastatic prostate cancer. Based on the observed improvement in survival, use of docetaxel in this setting is now accepted standard care and should be recommended unless a contraindication to chemotherapy exists. Myelosuppression is the most common side effect of docetaxel. Hepatic dysfunction or compromised bone marrow function are contraindications to its use. This patient's medical history does not indicate any contraindication to treatment with docetaxel. Continuation of leuprolide alone would not be recommended in a patient who is a candidate for docetaxel chemotherapy based on the demonstrated improvement in survival in men treated with six cycles of docetaxel. Although radiation to the lumbar spine would be indicated for pain control or for treatment of neurologic compromise due to nerve compression by metastatic disease, this patient has no evidence of neurologic compromise, and his pain improved with leuprolide treatment. Therefore, radiation is not indicated at this time.
A 47-year-old man is evaluated during a follow-up visit to manage fatigue and decreased libido. Medical history is significant for hypertension and dyslipidemia. Medications are hydrochlorothiazide and atorvastatin. On physical examination, vital signs are normal. He has normal hair distribution, no gynecomastia, and normal testicular examination. Laboratory studies obtained at 3 PM revealed a total testosterone level of 275 ng/dL (9.5 nmol/L) and a luteinizing hormone level of 5 mU/mL (5 U/L). Which of the following is the most appropriate management? Initiate testosterone replacement therapy Measure serum iron and total iron binding capacity Measure testosterone at 8 AM Obtain pituitary MRI
In men with specific signs and symptoms of hypogonadism, measuring an 8 AM total testosterone level is indicated; if the testosterone level is low, a second 8 AM confirmatory testosterone level is measured. The most appropriate management is to repeat testosterone measurement at 8 AM. The clinical diagnosis of hypogonadism is made on the basis of signs and symptoms consistent with androgen deficiency with the finding of low morning testosterone concentrations on at least two occasions. Timing of initial laboratory assessment is important due to the diurnal variation of testosterone. Additionally, assessment of hypogonadism should not be undertaken during acute illness. The next step in management of this patient would be to obtain a morning testosterone concentration; if low, testing should be confirmed with a repeat morning measurement. After the initial diagnosis, determination of primary or secondary hypogonadism is established by measurement of luteinizing hormone (LH) and follicle-stimulating hormone (FSH) levels. Elevated gonadotropin levels are seen in primary hypogonadism with LH and FSH levels low or inappropriately normal in secondary hypogonadism. Based on Endocrine Society guidelines, men with hypogonadism should be treated with exogenous testosterone when they have consistent signs and symptoms of hypogonadism and low serum testosterone levels. Patients requiring testosterone replacement therapy should have testosterone levels monitored at 3 and 6 months after initiation and annually thereafter; the goal total testosterone level should be in the mid-normal range. Monitoring of the prostate specific antigen and hematocrit level should follow Endocrine Society guidelines. Before initiating testosterone therapy in this patient, the diagnosis of hypogonadism needs to be confirmed with two appropriately timed testosterone measurements. The most sensitive and cost-effective initial diagnostic study in patients with suspected hemochromatosis is measurement of the fasting serum transferrin saturation (calculated as [serum iron/total iron binding capacity] ×100). Serum ferritin level measurement is indicated in patients with elevated transferrin saturation. Evaluation for hemochromatosis is only appropriate after the diagnosis of hypogonadism is established. The first diagnostic step for this patient is the collection of two properly collected serum testosterone levels to establish the diagnosis of hypogonadism. If subsequent evaluation of the patient confirms the presence of hypogonadotrophic hypogonadism (low testosterone and low LH and FSH levels), MRI of the pituitary to detect a pituitary adenoma or other mass would be appropriate. A pituitary MRI would be premature at this point.
A 58-year-old man is evaluated in an urgent care center for a 3-day history of right-sided scrotal swelling, pain, and dysuria. He reports no antecedent trauma, nausea, or vomiting. He is sexually active with both men and women and uses condoms intermittently. He does not take any medications. On physical examination, temperature is 38.5 °C (101.3 °F), pulse rate is 101/min, and other vital signs are normal. The right hemiscrotum is edematous, with tenderness to palpation of the superolateral aspect. The scrotal pain lessens with elevation of the scrotum. There is no penile discharge. Results of nucleic acid amplification testing for chlamydia and gonorrhea are pending. Which of the following is the most appropriate treatment? Ceftriaxone Ceftriaxone and doxycycline Ceftriaxone and levofloxacin Ibuprofen and scrotal support
In older men and persons who practice insertive anal intercourse, infectious epididymitis should be treated with ceftriaxone and a fluoroquinolone, such as levofloxacin. The most appropriate treatment is ceftriaxone and levofloxacin. This patient's history and physical examination findings (fever, erythema, and swelling of the hemiscrotum; tenderness to palpation near the epididymis; and urinary symptoms) are concerning for a diagnosis of acute epididymitis. Prehn sign, which is alleviation of pain with elevation of the testicle or scrotum, can clinically support this diagnosis. Infectious epididymitis has a bimodal distribution: men younger than 35 years and older than 55 years. In younger patients, sexually transmitted infections (chlamydia and gonorrhea) are the most likely cause. In older patients and those who practice insertive anal intercourse, Escherichia coli, Enterobacteriaceae, and Pseudomonas species should be considered. In older men and persons who practice insertive anal intercourse, infectious epididymitis should be treated with ceftriaxone and a fluoroquinolone, such as levofloxacin, to provide antimicrobial activity against cephalosporin-resistant organisms, including Pseudomonas species. In younger patients (age <35 years), the most common infectious etiologies of acute epididymitis include Chlamydia trachomatis and Neisseria gonorrhoeae. Ceftriaxone is adequate coverage for N. gonorrhoeae but not C. trachomatis infection, and would not be an appropriate choice in a younger patient. In these men, and in the absence of risk factors for gram-negative infection (anal intercourse, urologic instrumentation), empirically treating with ceftriaxone and doxycycline (or azithromycin, if the patient is intolerant to doxycycline) would be appropriate. Epididymitis can also have noninfectious causes (for example, trauma, autoimmune disease, or vasculitis). Treatment includes scrotal support, ice, and NSAIDs. Analgesic agents and scrotal support are supportive measures that can be offered to all patients presenting with acute epididymitis. However, the most appropriate treatment for this patient with risk factors for bacterial epididymitis and no history of trauma or findings supporting autoimmune disease or vasculitis would be initiation of an antimicrobial regimen rather than supportive therapies alone.
A 56-year-old man is evaluated for palpitations and difficulty sleeping over the past month. His past medical history is significant for hypothyroidism following subtotal thyroidectomy 6 months ago for management of compressive symptoms from a multinodular goiter. Two months ago, he was diagnosed with hypogonadism and was prescribed intramuscular testosterone. He takes his levothyroxine on an empty stomach with a cup of coffee every morning. His medications are levothyroxine, testosterone enanthate, calcium carbonate, and omeprazole for gastroesophageal reflux disease. On physical examination, his vital signs are normal. He has a well-healed anterior neck scar. There is a fine tremor of his outstretched hands. The remainder of the examination is normal. Laboratory studies: 2 Months Ago Today Serum thyroid-stimulating hormone 1.5 µU/mL (1.5 mU/L) 0.08 µU/mL (0.08 mU/L) Thyroxine (T4), free 1.1 ng/dL (14.2 pmol/L) 1.4 ng/dL (18.1 pmol/L) Which of the following is the most likely explanation for the thyroid function test results? Calcium carbonate Levothyroxine with coffee Omeprazole Testosterone
In patients receiving thyroxine replacement therapy, initiation of estrogen or raloxifene increases thyroxine-binding globulin levels whereas testosterone reduces thyroxine-binding globulin levels; in either situation a change in thyroxine dosage may be required. The most likely explanation for the observed change in this patient's thyroid function test results is the initiation of testosterone. The administration of androgens and anabolic steroids leads to a reduction in thyroxine-binding globulin, which consequently increases the proportion of metabolically active free thyroxine that is available. Consequently, a reduction in levothyroxine dosing may be needed to prevent iatrogenic thyrotoxicosis. Conversely, higher levothyroxine doses are often required after the initiation of estrogen or selective estrogen receptor modulating therapies (tamoxifen and raloxifene) due to an increase in serum thyroxine-binding globulin concentrations. Administering levothyroxine with coffee or calcium carbonate and treatment with omeprazole all decrease the absorption of thyroxine, resulting in reduced levels of free thyroxine (T4) and elevated levels of thyroid-stimulating hormone. Calcium carbonate reduces the maximum absorption of levothyroxine by 25%. Because calcium and ferrous sulfate can bind levothyroxine, the ingestion of either medication, even as a component of a multivitamin, should be separated from levothyroxine by 4 hours. Administering levothyroxine with coffee, whether caffeinated or not, has been shown to reduce the absorption of levothyroxine. Ideally, levothyroxine should be taken 60 minutes before food or coffee is consumed. Oral levothyroxine is absorbed in the jejunum and ileum. The absorption of an orally administered dose is 70% to 80% under optimum fasting conditions, and an acidic gastric pH is important. Chronic therapy with omeprazole and other proton pump inhibitors impairs levothyroxine absorption by increasing gastric pH.
A 61-year-old man is seen for medical evaluation before a pancreaticoduodenectomy for suspected pancreatic cancer scheduled in 7 days. He reports no recent chest pain or bleeding complications after undergoing drug-eluting stent placement to the left anterior descending artery for an ST-elevation myocardial infarction 5 months ago. He has been riding his bike 10 miles daily since recovering from the myocardial infarction. Medications are aspirin, clopidogrel, losartan, atorvastatin, and atenolol. On physical examination, vital signs are normal. Scleral icterus and jaundice are noted. Cardiac examination is normal, the lungs are clear, and the abdomen is nontender. There is no lower extremity edema. Which of the following is the most appropriate perioperative management of this patient's antiplatelet therapy? Continue clopidogrel and aspirin Withhold aspirin and clopidogrel now Withhold aspirin now; continue clopidogrel Withhold clopidogrel now; continue aspirin
In patients taking dual antiplatelet therapy, if the risk of surgical delay exceeds the risk for stent thrombosis, discontinuation of the P2Y12 inhibitor can be considered after a minimum of 30 days in the case of bare metal stent placement or 3 months after drug-eluting stent placement. The most appropriate management of this patient's antiplatelet therapy is to withhold clopidogrel 5 to 7 days before surgery and continue aspirin. According to the 2016 American College of Cardiology/American Heart Association focused update on the duration of dual antiplatelet therapy (DAPT), perioperative management is based on surgical bleeding risk balanced with the risk for stent thrombosis. The risk for stent thrombosis is contingent on both the indication for coronary stent placement (stable ischemic heart disease [SIHD] or acute coronary syndrome [ACS]) and the amount of time that has passed from the time of stent placement. For patients who have a stent placed for SIHD, DAPT (aspirin plus a P2Y12 inhibitor) should be continued for at least 30 days after bare metal stent placement and a minimum of 6 months after drug-eluting stent placement. DAPT is recommended for 1 year in patients with ACS, regardless of medical management or cardiac stent placement, with the understanding that discontinuation may be reasonable if high bleeding risk is identified or the patient has overt bleeding on DAPT. However, in the perioperative setting, and especially if the risk of surgical delay exceeds the risk for stent thrombosis, discontinuation of the P2Y12 inhibitor can be considered after a minimum of 30 days in the case of bare metal stent placement or 3 months after drug-eluting stent placement. It is optimal to continue DAPT for 6 months after drug-eluting stent placement, especially in the case of ACS, if the risk for surgical delay does not exceed the risk for stent thrombosis. Aspirin should be continued, if at all possible, along with restarting of DAPT as soon as bleeding risk has sufficiently diminished. If surgery must be performed within those periods after percutaneous coronary intervention, DAPT should be maintained perioperatively unless the risk for major bleeding exceeds the risk for stent thrombosis. Communication with the surgical team and the patient regarding risks and benefits of withholding DAPT within these time frames is advised.
A 55-year-old man is hospitalized after he was injured at a construction site structure collapse. He has a fractured pelvis, a shoulder dislocation, a mild concussion, and multiple abrasions. He is scheduled to undergo surgery for the fractured pelvis tomorrow morning. History is significant for hyperlipidemia, hypertension, coronary artery disease, and a non-ST-elevation myocardial infarction 2 years ago treated with drug-eluting stent placement. Current medications are atorvastatin, metoprolol, amlodipine, and aspirin. On physical examination, temperature is normal, blood pressure is 170/90 mm Hg, pulse rate is 102/min, and respiration rate is 20/min. Oxygen saturation is 96% breathing ambient air. Multiple abrasions and ecchymosis are noted. Cardiac examination reveals tachycardia but is otherwise normal. Which of the following is the most appropriate management of this patient's medications before surgery? Continue amlodipine, aspirin, and metoprolol; withhold atorvastatin Continue amlodipine, atorvastatin, and metoprolol; withhold aspirin Continue atorvastatin, metoprolol, and aspirin; withhold amlodipine Continue all medications
In patients undergoing noncardiac surgery, β-blockers and statins should be continued in those who have been taking the drugs long term, and aspirin generally should be continued in patients with coronary stents unless the bleeding risk is prohibitively high. This patient should continue all medications the morning of surgery. In general, many medications are well tolerated throughout the perioperative period. The 2014 American College of Cardiology/American Heart Association guideline on perioperative cardiovascular evaluation and management of patients undergoing noncardiac surgery includes recommendations for the management of statins, β-blockers, and antiplatelet agents. Statins should be continued in patients who have been taking statins long term. In patients undergoing vascular surgery, initiation of statins is reasonable, and perioperative statin initiation can also be considered in patients who would otherwise qualify for statin therapy based on current guideline-directed medical therapy. Likewise, β-blockers should be continued in patients who have been taking β-blockers long term, especially those with coronary artery disease. Preoperative initiation of β-blockers may be indicated for some patients undergoing nonurgent surgery, including those with intermediate- or high-risk myocardial ischemia on preoperative stress testing, those with three or more Revised Cardiac Risk Index risk factors (diabetes mellitus, heart failure, coronary artery disease, chronic kidney disease, cerebrovascular disease), or those who otherwise have a compelling indication for β-blockade. However, β-blockers should never be started on the day of surgery, but rather with enough time preoperatively to assess safety and tolerability. There are very few data available regarding the benefits versus risks of continuing calcium channel blockers such as amlodipine. There are no known interactions with anesthetic agents, and continuing calcium channel blockers is not associated with hemodynamic instability. Most experts, in the absence of data, recommend continuing this class of medication for patients already taking them. In general, patients with coronary stents should be continued on aspirin throughout the perioperative period unless the bleeding risk is prohibitively high, as is the case with many neurosurgical procedures. It is important to discuss perioperative management of aspirin and P2Y12 inhibitors with the surgical team to reach a consensus recommendation.
A 30-year-old woman undergoes predischarge evaluation 1 month after allogeneic hematopoietic stem cell transplantation for acute myeloid leukemia. The posttransplant course was complicated by prolonged neutropenia and Escherichia coli bacteremia. The infection and neutropenia have resolved with broad-spectrum antibiotic therapy. She received all indicated pretransplant immunizations. Medications are trimethoprim-sulfamethoxazole for pneumocystis pneumonia prophylaxis and tacrolimus. On physical examination, vital signs are normal. The indwelling central line site appears normal without erythema, swelling, or tenderness. Lungs are clear throughout. No skin lesions are present. Laboratory studies show a leukocyte count of 2700/µL (2.7 × 109/L) (differential of 57% polymorphonuclear cells, 30% lymphocytes, 10% monocytes, and 3% eosinophils) and a serum creatinine level of 0.7 mg/dL (61.9 µmol/L). Pretransplant serologies were positive for cytomegalovirus, Epstein-Barr virus, varicella-zoster virus, and Toxoplasma. In addition to continuing the trimethoprim-sulfamethoxazole, which of the following is the most appropriate outpatient prophylaxis for this patient? Acyclovir Ciprofloxacin Posaconazole Valganciclovir
In patients who have undergone hematopoietic stem cell transplantation, the risk of invasive fungal infection remains elevated for the first few months, even after recovery of neutrophil counts, so antifungal prophylaxis should be continued during this time. Posaconazole is the most appropriate prophylactic therapy for this patient after hospital discharge. Patients who undergo allogeneic hematopoietic stem cell transplantation (HSCT) usually experience a period of prolonged neutropenia after the pretransplant myeloablative conditioning regimen. This prolonged, severe neutropenia is a significant risk factor for invasive bacterial and fungal infections, and prophylaxis for both is indicated. The risk of invasive fungal infection remains elevated, however, even after recovery of neutrophils for the first few months after allogeneic HSCT and even later in the setting of graft-versus-host disease. Therefore, although this patient is doing well and her neutrophil count has recovered, antifungal prophylaxis should be provided in addition to continuing trimethoprim-sulfamethoxazole therapy. Posaconazole provides activity against Candida and Aspergillus and other moulds that are the cause of an increasing proportion of invasive fungal infections after HSCT. Acyclovir is used to reduce reactivation of herpes simplex virus during periods of neutropenia, but its administration is no longer needed. Additionally, this patient is seropositive for varicella-zoster virus because of immunization, so varicella-zoster prophylaxis is unnecessary. Finally, acyclovir has no role in preventing reactivation of Epstein-Barr virus or cytomegalovirus, two other infections for which this patient is at risk. Ciprofloxacin is a preferred agent for antibacterial prophylaxis during neutropenia because it has activity against most of the gram-negative bacteria of greatest concern. But with resolution of neutropenia and completion of therapy for known infection, continued antibacterial therapy should be avoided to lessen toxicities and complications such as Clostridium difficile colitis. Valganciclovir is used to prevent or treat cytomegalovirus infection. This patient is at risk for reactivation cytomegalovirus because of her positive serology. However, because of the leukopenia that is a common toxicity of valganciclovir, a strategy of monitoring for cytomegalovirus and starting pre-emptive therapy only when indicated is preferred over prophylaxis after HSCT.
A 70-year-old man is evaluated in the emergency department for headache, confusion, and weakness. Medical history is significant for ischemic cardiomyopathy with heart failure and mural thrombus. Medications are lisinopril, carvedilol, spironolactone, furosemide, and warfarin. On physical examination, the patient is disoriented. Temperature and respiration rate are normal, blood pressure is 160/90 mm Hg, and pulse rate is 88/min. The examination is otherwise unremarkable. Laboratory studies show an INR of 7.5. CT scan of the head shows left intraparenchymal hemorrhage. In addition to vitamin K administration, which of the following is the most appropriate treatment? Fresh frozen plasma Idarucizumab Prothrombin complex concentrate Recombinant activated factor VII
In patients with INR elevation and bleeding associated with warfarin administration, urgent reversal of anticoagulation should be accomplished using vitamin K and prothrombin complex concentrates. The most appropriate treatment for this patient is vitamin K and prothrombin complex concentrates (PCCs), specifically, four-factor PCC. Four-factor PCC is a combination of inactivated factors II, VII, IX, and X. Treatment recommendations for over-anticoagulation with warfarin are based on the degree of over-anticoagulation as measured by the INR and clinical manifestations of bleeding. The first step in all cases of over-anticoagulation is withholding warfarin. For patients with an INR of 4.5 to 10 and with no evidence of bleeding, routine use of vitamin K is not recommended. For patients with an INR greater than 10 and with no evidence of bleeding, oral vitamin K is recommended. For patients with major bleeding, such as this patient with an intracerebral hemorrhage, rapid reversal of anticoagulation is recommended with four-factor PCC rather than fresh frozen plasma (FFP). These patients are also likely to benefit from the addition of vitamin K by slow intravenous injection rather than using coagulation factors alone. Clinical trials have shown four-factor PCC to be noninferior to FFP in reversing warfarin anticoagulation. However, four-factor PCC is preferred because of its rapid reversal of the INR, rapid infusion and administration, and no risk of volume overload as is sometimes seen with plasma infusions. Additionally, four-factor PCC does not require thawing or typing. Although FFP contains the appropriate clotting factors, it requires thawing and large volumes to correct the INR. This patient also has heart failure, making infusion of FFP problematic because of the volume needed. Idarucizumab is a monoclonal antibody approved for reversal of the anticoagulant effects of dabigatran. It is not used to reverse the effects of vitamin K-antagonist anticoagulants. Recombinant activated factor VII has been found to reduce the INR in warfarin-associated bleeding, but its overall effect on outcome has been unclear, with a varied rate of thromboembolic events. It is not recommended for acute warfarin reversal. Read Related TextNext Question
A 65-year-old woman is evaluated in the hospital following primary percutaneous coronary intervention with stent placement in the mid left anterior descending artery. She presented 4 hours ago with findings of an anterior myocardial infarction complicated by heart failure. Medical history is significant for hyperlipidemia, hypertension, type 2 diabetes mellitus, and previous ACE inhibitor-induced cough. Medications are atorvastatin, aspirin, clopidogrel, metformin, and furosemide. On physical examination, vital signs are normal. Oxygen saturation is 98% on 3 L oxygen by nasal cannula. The estimated central venous pressure is elevated. An S3 is present. Pulmonary examination reveals bibasilar crackles. Laboratory studies are notable for normal serum creatinine and serum potassium levels. A chest radiograph shows pulmonary edema. An echocardiogram demonstrates a left ventricular ejection fraction of 40%. Which of the following is the most appropriate treatment? Carvedilol Diltiazem Hydralazine-isosorbide dinitrate Valsartan
In patients with ST-elevation myocardial infarction, an ACE inhibitor should be initiated within 24 hours of presentation; an angiotensin receptor blocker may be used if the patient is intolerant of ACE inhibitors.
A 65-year-old woman is evaluated in the hospital following primary percutaneous coronary intervention with stent placement in the mid left anterior descending artery. She presented 4 hours ago with findings of an anterior myocardial infarction complicated by heart failure. Medical history is significant for hyperlipidemia, hypertension, type 2 diabetes mellitus, and previous ACE inhibitor-induced cough. Medications are atorvastatin, aspirin, clopidogrel, metformin, and furosemide. On physical examination, vital signs are normal. Oxygen saturation is 98% on 3 L oxygen by nasal cannula. The estimated central venous pressure is elevated. An S3 is present. Pulmonary examination reveals bibasilar crackles. Laboratory studies are notable for normal serum creatinine and serum potassium levels. A chest radiograph shows pulmonary edema. An echocardiogram demonstrates a left ventricular ejection fraction of 40%. Which of the following is the most appropriate treatment? Carvedilol Diltiazem Hydralazine-isosorbide dinitrate Valsartan
In patients with ST-elevation myocardial infarction, an ACE inhibitor should be initiated within 24 hours of presentation; an angiotensin receptor blocker may be used if the patient is intolerant of ACE inhibitors. The most appropriate treatment of this patient with anterior ST-elevation myocardial infarction (STEMI) complicated by heart failure is valsartan. This patient is at significant risk for short-term and long-term morbidity, and early institution of guideline-directed medical therapy (within 24 hours of presentation) is crucial to improve survival and keep the patient free of symptoms. Although ACE inhibitors are indicated in most patients with STEMI and particularly in patients with impaired left ventricular function and heart failure, angiotensin receptor blockers (such as valsartan) are a suitable alternative and offer similar morbidity and mortality benefits. Additionally, angiotensin receptor blockers are associated with a significantly lower incidence of cough and angioedema than ACE inhibitors. Because this patient has a history of ACE inhibitor-induced cough, valsartan is the most appropriate choice. In patients with STEMI, β-blockers (such as carvedilol) decrease myocardial oxygen demand, reduce the incidence of ventricular arrhythmias, and improve long-term survival. Whenever possible, β-blockers should be introduced within the first 24 hours of STEMI presentation. The COMMIT/CCS-2 trial demonstrated that early β-blocker initiation was associated with lower risk for reinfarction and lethal ventricular arrhythmias, but it also increased risk for cardiogenic shock. In this case, the patient has evidence of pulmonary edema, and early institution of β-blockade may worsen heart failure and increase the risk for cardiogenic shock. A β-blocker should be initiated once the patient is stabilized. Like β-blockers, calcium channel blockers (such as diltiazem) are negative inotropic and chronotropic agents. However, unlike β-blockers, calcium channel blockers are generally avoided in the context of STEMI and are contraindicated with left ventricular dysfunction. Hydralazine-isosorbide dinitrate can be used as a vasodilator, particularly as an add-on therapy in the setting of chronic heart failure, although it has been shown to be inferior to ACE inhibitors in a head-to-head study. Hydralazine-isosorbide dinitrate would not be the preferred agent in this setting, unless the patient had a significantly elevated serum creatinine level or a history of ACE inhibitor-induced angioedema. A 2018 systematic review found that supplemental oxygen in the setting of normal peripheral oxygen saturation increases mortality in patients with acute myocardial infarction. A subsequent international guideline strongly recommends that oxygen therapy not be initiated for patients with an acute myocardial infarction and oxygen saturation as measured by pulse oximetry (SpO2) of 93% or higher. If supplemental oxygen is initiated, the guideline recommends not exceeding an SpO2 of 96%. This patient's oxygen saturation is 98%, and her supplemental oxygen should be reduced.
A 66-year-old man has just received an aortic valve replacement with a mechanical prosthesis. He is otherwise healthy and takes no medications. On physical examination, vital signs are normal. There is a regular rhythm with a normal S1, a mechanical S2, and no murmurs. The remainder of the physical examination is normal. Which of the following is the most appropriate antithrombotic therapy? Apixaban Dabigatran Warfarin Warfarin and aspirin No anticoagulation required
In patients with a mechanical prosthetic valve, low-dose aspirin is recommended in addition to warfarin therapy to reduce the risk for ischemic events. The most appropriate antithrombotic therapy is the combination of warfarin and aspirin. Lifelong oral anticoagulation with warfarin is recommended for all patients with a mechanical prosthesis and those with bioprostheses with other indications for anticoagulation. The target INR is based on prosthesis location, with a target of 2.5 for patients with a mechanical aortic prosthetic valve and 3.0 for patients with a mechanical mitral prosthetic valve and those with a mechanical aortic prosthetic valve and additional risk factors for thromboembolism (atrial fibrillation, left ventricular dysfunction, previous thromboembolism, hypercoagulable condition, or older-generation mechanical aortic valve replacement). The addition of aspirin (75-100 mg/d) is also recommended to reduce the risk for ischemic events. Adding aspirin to warfarin therapy reduces mortality, particularly mortality from vascular causes, and major systemic embolism (1.9% per year versus 8.5% per year). Although there is some increase in bleeding, the risk of the combined treatment was more than offset by the considerable benefit. The need for anticoagulation among patients with bioprosthetic valves is less clear. Oral anticoagulation with warfarin should be considered for at least 3 months and as long as 6 months after implantation of a mitral or aortic bioprosthesis. An INR of 2.5 should be targeted in these patients. Long-term aspirin use is reasonable for all patients with bioprosthetic valves. Apixaban, a direct factor Xa inhibitor, has not been studied for the prevention of valve thrombosis and thromboembolic events in patients with a mechanical valve prosthesis; therefore, it would be inappropriate to initiate apixaban in this patient. Direct thrombin inhibitors, such as dabigatran, are contraindicated in patients with mechanical valves, owing to the excessive thrombotic complications observed in clinical trials with these agents. Foregoing anticoagulation would be inappropriate for this patient because lifelong warfarin is indicated in all patients with a mechanical valve prosthesis to decrease the incidence of thromboembolism and the associated morbidity, such as ischemic stroke, cerebrovascular accident, and peripheral systemic embolism.
A 60-year-old man is evaluated for a 5-month history of exertional chest discomfort that improves with rest. His symptoms have progressively worsened such that he has reduced his activity to a minimum. Medical history is significant for hypertension and hyperlipidemia. Medications are low-dose aspirin, losartan, hydrochlorothiazide, and atorvastatin. On physical examination, the patient is afebrile, blood pressure is 122/71 mm Hg, and pulse rate is 74/min. Cardiac examination shows a normal S1 and S2. A grade 2/6 crescendo-decrescendo systolic murmur is heard best at the upper sternal border with no radiation. Lung examination is normal. A stress echocardiogram shows 2-mm ST-segment depression at peak stress, normal left ventricular function at rest, normal valvular function, and anterior hypokinesis at peak stress (normal at rest). A 5.4-cm ascending thoracic aortic aneurysm is noted at the level of the sinuses of Valsalva. Coronary angiogram reveals 80% stenosis of the left main coronary artery bifurcation with no significant disease of the left anterior descending, left circumflex, or right coronary arteries. Which of the following is the most appropriate next step in the patient's management? Coronary artery bypass graft surgery Metoprolol and isosorbide mononitrate Percutaneous coronary intervention Simultaneous coronary artery bypass graft surgery and aortic repair
In patients with a thoracic aortic aneurysm greater than 4.5 cm in diameter who require coronary artery bypass graft surgery or surgery to repair valve pathology, aortic repair should be performed at the time of cardiac surgery. The most appropriate management of this patient is simultaneous coronary artery bypass graft (CABG) surgery and aortic repair. On the basis of his angiographic findings (80% stenosis of the left main coronary artery bifurcation), he should undergo revascularization with CABG surgery. In patients with an ascending aorta or aortic root greater than 4.5 cm in diameter who require CABG surgery or surgery to repair valve pathology, aortic repair should be performed at the time of cardiac surgery. Anatomic imaging, such as CT angiography or magnetic resonance angiography, is recommended to plan for open aortic repair before the surgical procedure. Patients with left main coronary artery disease have traditionally been treated with CABG surgery; however, because this patient has concomitant thoracic aortic aneurysmal disease, CABG surgery without aortic repair is not the best management option. Patients with established coronary artery disease benefit from optimal medical therapy, including β-blockers and long-acting nitrates. However, given this patient's thoracic aortic aneurysm and severe coronary artery disease, optimal medical therapy in the absence of revascularization and aortic repair is inappropriate. Percutaneous coronary intervention is not encouraged for patients with complex disease of the left main coronary artery, especially in the presence of a thoracic aortic aneurysm. Read Related TextNext Question
A 64-year-old man is evaluated in the emergency department for acute right lower leg pain that began 2 days ago. The pain is now severe at rest, and he reports coolness of the right foot. He has a 3-year history of intermittent claudication. He underwent right femoral-popliteal bypass graft surgery for life-limiting claudication 1 year ago. Medical history is otherwise significant for hypertension, hyperlipidemia, and type 2 diabetes mellitus. He quit smoking 4 years ago. Medications are low-dose aspirin, ramipril, hydrochlorothiazide, rosuvastatin, and metformin. On physical examination, vital signs are normal. The right foot is cool and pale, sensation is intact, and muscle strength is normal. The pedal pulses are not palpable in the right leg. Arterial Doppler ultrasound signals are not detectable over the right dorsalis pedis and right posterior tibial arteries. Intravenous anticoagulation with heparin is initiated. Which of the following is the most appropriate management? Arterial duplex ultrasonography CT angiography Emergent right leg amputation Intravenous recombinant tissue plasminogen activator Urgent invasive angiography
In patients with acute limb ischemia, invasive angiography should be performed immediately to define the anatomic level of occlusion and plan for revascularization. The most appropriate next step in the management of this patient with acute limb ischemia (ALI) is urgent invasive angiography to determine the extent of disease and to plan treatment. ALI is defined by a sudden or rapid decrease in limb perfusion, often manifested as a new pulse deficit, rest pain, pallor, and/or paralysis. The most common causes of ALI include in situ thrombosis of a lower extremity bypass graft or endovascular stent and thromboembolism. Patients with ALI and a viable extremity are most commonly treated with surgical embolectomy or catheter-directed thrombolysis. This patient is demonstrating several classic signs of ALI, including pulselessness, pallor, and pain, and he is at heightened risk for ALI owing to his history of femoral-popliteal bypass graft surgery. The most appropriate next step is to perform invasive angiography to define the level of occlusion and to determine whether surgical embolectomy or catheter-directed thrombolysis is an option. Delay in treatment may lead to worsening limb perfusion, limb necrosis, or the need for lower extremity amputation. Noninvasive imaging studies, such as arterial duplex ultrasonography and CT angiography, may be useful to detect the level of stenosis and extent of disease, but these tests will substantially delay the treatment of ALI in this patient. Emergent lower extremity amputation is necessary in patients with ALI who have a nonviable extremity (gangrene, paralysis); however, this patient exhibits signs of viability (intact sensation and muscle strength) and should not undergo immediate amputation. In contrast to the treatment of acute ischemic stroke or acute ST-elevation myocardial infarction, systemic thrombolytic therapy (such as with recombinant tissue plasminogen activator) has not been proved beneficial in the treatment of ALI and should not be used in this patient. Read Related TextNext Question
A 72-year-old man is evaluated for exertional left calf and foot pain. Three weeks ago, the patient developed an ulcer on the medial aspect of the left great toe. His medical history is significant for coronary artery disease, type 2 diabetes mellitus, hypertension, and hyperlipidemia. Medications are low-dose aspirin, lisinopril, metoprolol, metformin, and atorvastatin. On physical examination, blood pressure is 155/84 mm; other vital signs are normal. There are no palpable pulses in the left leg. Right femoral, popliteal, and pedal pulses are faint. Ankle-brachial index testing: Right systolic brachial pressure 155 mm Hg Left systolic brachial pressure 145 mm Hg Left posterior tibialis pressure 255 mm Hg Left dorsalis pedis pressure 255 mm Hg Which of the following is the most appropriate diagnostic test to perform next? Exercise ankle-brachial index Lower extremity CT angiography Toe-brachial index Venous duplex ultrasonography
In patients with an ankle-brachial index greater than 1.40, a toe-brachial index may be used to diagnose peripheral artery disease. The most appropriate diagnostic test to perform next in this patient is a toe-brachial index. The diagnostic tool that is most frequently used to identify peripheral artery disease (PAD) is the ankle-brachial index (ABI). To calculate the ABI, the higher ankle pressure in each leg is divided by the higher brachial pressure. An ABI of 0.90 or lower establishes a diagnosis of PAD, whereas an ABI greater than 1.40 indicates the presence of calcified, noncompressible arteries in the lower extremities and is considered uninterpretable. In patients with an ABI greater than 1.40, an appropriate next step is to measure toe pressure or calculate a toe-brachial index (systolic great toe pressure divided by systolic brachial pressure). A great toe systolic pressure below 40 mm Hg or a toe-brachial index below 0.70 is consistent with PAD. Because this patient has a left ABI of 1.65, a toe-brachial index is indicated. Exercise ABI testing is useful when patients have a borderline ABI (0.91-1.00) or normal ABI (1.00-1.40) and a high likelihood of PAD. The American Heart Association has proposed a postexercise ankle pressure decrease of more than 30 mm Hg or a postexercise ABI decrease of more than 20% as a diagnostic criterion for PAD. Other organizations have proposed a postexercise ABI of less than 0.90 and/or a 30-mm Hg drop in ankle pressure after exercise. This patient's resting ABI value is greater than 1.40; therefore, exercise ABI is not indicated to diagnose PAD. CT angiography and magnetic resonance angiography are often reserved for planning endovascular or surgical revascularization rather than for diagnosis of PAD. In patients with venous leg ulcers, venous duplex ultrasonography is indicated to evaluate for chronic venous insufficiency and deep vein thrombosis (acute or chronic). The location of the ulceration and the lack of other findings consistent with venous disease make that diagnosis unlikely, and venous duplex ultrasonography is therefore unnecessary.
A 69-year-old man is evaluated during a routine examination. He is asymptomatic. Medical history is significant for hypertension. He has a 50-pack-year smoking history but quit smoking 7 years ago. Medications are aspirin, lisinopril, and amlodipine. On physical examination, vital signs are normal. A bruit is heard over the abdomen, and a pulsatile mass is present in the epigastrium. The remainder of the examination is unremarkable. A Duplex ultrasound of the abdomen shows an abdominal aortic aneurysm with transverse diameter of 6.2 cm. Which of the following is the most appropriate next step in management? CT angiography of the abdominal aorta and iliac vessels Endovascular repair Open surgical repair Switch amlodipine to metoprolol
In patients with an indication for abdominal aortic aneurysm repair, the choice between open surgical repair and endovascular aneurysm repair is driven in part by the location of the aneurysm and involvement of the renal and mesenteric arteries. The most appropriate next step in management of this patient with an abdominal aortic aneurysm (AAA) is CT angiography of the abdominal aorta and iliac vessels to plan for aortic repair. The strongest risk factor for rupture of an AAA is maximal aortic diameter; this measurement is the dominant indication for repair. Aortic repair should be performed in suitable patients with an AAA diameter of 5.5 cm or larger, in patients with rapid expansion in AAA size (>0.5 cm/year), and in patients presenting with symptoms resulting from AAA (abdominal or back pain/tenderness). In patients with an indication for aortic repair, the choice is between open surgical repair and endovascular aneurysm repair (EVAR). Open surgical repair involves abdominal flank incision and opening the aneurysm sac with interposition of a synthetic graft. EVAR is a less invasive method involving intraluminal introduction of a covered stent through the aneurysm sac, with the stent acting as a sleeve. The choice of procedure is driven by several considerations, including the patient's operative risk, expected lifespan, and ability to adhere to the monitoring requirements of EVAR; the location of the AAA; and involvement of the renal and mesenteric arteries. Suprarenal and juxtarenal aneurysms most often necessitate open surgical repair, whereas infrarenal aneurysms may be treated with open surgery or EVAR. In this patient, CT angiography or magnetic resonance angiography should be performed to determine the location and extent of the AAA. Abdominal duplex ultrasonography is insufficient to determine AAA location. In addition, CT measurements exceed ultrasound measurements in 95% of cases. Although controlling risk factors for cardiovascular disease is essential in patients with AAA, there is little compelling evidence for treating hypertension in these patients with a specific agent, including β-blockers, to prevent aneurysm expansion. Because this patient's blood pressure is well controlled, no change in antihypertensive therapy is indicated. Importantly, an AAA with a diameter of 6.2 cm has a 10% to 20% annual risk for rupture, and definitive surgical therapy is indicated. Read Related TextNext Question
A 62-year-old man is evaluated for painful ulcerations on the right lower leg at the site of previous minor trauma. Medical history is significant for hypertension, type 2 diabetes mellitus, and hyperlipidemia. He is a 50-pack-year smoker. Medications are hydrochlorothiazide, lisinopril, metformin, and atorvastatin. On physical examination, vital signs are normal. BMI is 28. The leg ulcers are shown. He has increased pain with elevation of the right leg. Pedal pulses are weak on left leg and absent on right leg. The right lower leg is cool to touch. Resting ankle-brachial index is 0.3 on the right leg and 0.7 on the left leg. Which of the following is the most appropriate management of the ulcer? Compression stockings Oral cilostazol Skin grafting Surgical revascularization
In patients with arterial insufficiency ulcers, surgical revascularization to improve the lower extremity circulation is often necessary to facilitate wound healing. This patient has an arterial insufficiency ulcer characterized by a well-demarcated painful ulcer with a dry wound bed; surgical revascularization will improve lower extremity circulation and facilitate wound healing. Arterial ulcers are most commonly found on the tips of and between the digits, but they can also form at sites of increased pressure, such as the lateral malleolus and metatarsal heads, or at sites of previous trauma. Arterial ulcers tend to be painful; discomfort may increase with elevation of the legs, compression, or any other maneuver that further limits circulation. Unlike venous stasis ulcers, which often have somewhat irregular borders and a significant amount of surrounding post-inflammatory hyperpigmentation, arterial insufficiency ulcers have a "punched out" appearance with surrounding erythema. The extremities as a whole often appear pale, and the skin is thin, taut, and shiny, and lacks hair. Pedal pulses are often difficult to appreciate; the toes are cold, and capillary refill is poor. If arterial insufficiency is suspected, diagnosis can be confirmed by measuring the ankle-brachial index (ABI). An ABI less than 0.9 is indicative of peripheral artery disease. Basic care of arterial ulcers includes debridement of necrotic tissue and proper dressings, but the most beneficial treatment is restoration of adequate circulation to the extremity. Surgical revascularization, percutaneous angioplasty, or stent placement are all options to restore blood flow to the affected area. Compression stockings are a preferred treatment for venous stasis ulcerations. They should not be applied to a patient with arterial insufficiency ulcerations because of the risk of further ischemia. Oral cilostazol is a phosphodiesterase inhibitor with antiplatelet activity and vasodilatory properties. When combined with a structured exercise program, cilostazol can improve maximum and pain-free treadmill-walking distance and quality-of-life measures in patients with peripheral artery disease. There is limited evidence of its benefit in arterial insufficiency ulcers. Skin grafting is a procedure often used for the treatment of recalcitrant venous ulcers that fail to respond to compression and usual wound care. Revascularization, not skin grafting, is the primary treatment of arterial ulcers.
A 55-year-old woman is evaluated during a routine examination. Her medical history is significant for mitral regurgitation. She has no history of cardiovascular symptoms. She is moderately active, plays doubles tennis, and occasionally participates in road cycling. She has no other medical problems and takes no medications. On physical examination, vital signs are normal. Cardiac auscultation reveals a high-pitched midsystolic click and a grade 3/6 late systolic murmur that is loudest at the apex. Standing results in the click-murmur complex increasing in intensity and moving closer to S1. An echocardiogram shows severe mitral regurgitation and myxomatous degeneration of the mitral valve. The left ventricular ejection fraction is 65% with normal chamber size. After discussion regarding early surgical repair versus surveillance, she chooses to defer surgery. In the absence of clinical symptoms, which of the following is the most appropriate timing for the next clinical evaluation in this patient? 3 months 6 to 12 months 1 to 2 years 3 to 5 years
In patients with asymptomatic severe mitral regurgitation with preserved left ventricular function who do not have an indication for surgery, clinical and echocardiographic surveillance every 6 to 12 months is recommended. In the absence of symptoms, the most appropriate timing for this patient's next clinical evaluation is in 6 to 12 months. Abnormalities in any of the structures of the mitral valve apparatus, including anterior and posterior mitral leaflets, the annulus, the papillary muscles, and the chordae tendineae, can result in mitral regurgitation. Organic, or primary, mitral regurgitation refers to processes involving the leaflets, such as mitral valve prolapse, myxomatous degeneration (the abnormal accumulation of proteoglycans), collagen vascular disease, and infective endocarditis. Processes that affect the support structures, such as coronary artery disease and left ventricular remodeling in the setting of left ventricular dysfunction, result in functional, or secondary, mitral regurgitation. This patient has asymptomatic severe mitral regurgitation resulting from myxomatous degeneration of the mitral valve with preserved left ventricular function. Surgical treatment with repair of the mitral valve is indicated for chronic severe primary mitral regurgitation in (1) symptomatic patients with left ventricular ejection fraction greater than 30%, (2) asymptomatic patients with left ventricular dysfunction (left ventricular ejection fraction of 30%-60% and/or left ventricular end-systolic diameter ≥40 mm), and (3) patients undergoing another cardiac surgical procedure. Mitral valve repair should also be considered in asymptomatic patients with chronic severe primary mitral regurgitation who have new-onset atrial fibrillation or pulmonary hypertension (pulmonary artery systolic pressure >50 mm Hg). Notably, recent studies have suggested a potential mortality benefit of early surgery for patients with asymptomatic severe mitral regurgitation with preserved left ventricular function when the operative risk is low. Therefore, consideration of early surgery, even in the absence of symptoms or an abnormal left ventricle, is appropriate in select candidates and should involve shared decision making with the patient. In this patient who has chosen to defer early surgery, clinical and echocardiographic surveillance is recommended, with follow-up every 6 to 12 months. Read Related TextNext Question
A 56-year-old man is evaluated for cardiovascular risk assessment. At a recent employee health screening, he was informed that he has elevated cholesterol levels. He feels well and exercises three to four times per week without any symptoms. He does not use tobacco. Medical history is unremarkable. Family history is notable for a myocardial infarction in his father at age 54 years. He takes no medications. On physical examination, the patient is afebrile, blood pressure is 124/82 mm Hg, and pulse rate is 76/min. BMI is 28. Cardiovascular examination is normal. Laboratory studies: Total cholesterol 194 mg/dL (5.02 mmol/L) HDL cholesterol 38 mg/dL (0.98 mmol/L) LDL cholesterol 122 mg/dL (3.16 mmol/L) Triglycerides 170 mg/dL (1.92 mmol/L) His 10-year risk for atherosclerotic cardiovascular disease using the Pooled Cohort Equations is 7%. Which of the following is the most reasonable next step in management? Adenosine cardiac magnetic resonance imaging Coronary artery calcium scoring Exercise stress echocardiography Lipoprotein(a) measurement Pharmacologic nuclear stress testing
In patients with borderline or intermediate 10-year risk for atherosclerotic cardiovascular disease, coronary artery calcium scoring may be used to further risk-stratify patients to guide primary prevention therapy. This asymptomatic patient at borderline risk for coronary artery disease can undergo coronary artery calcium (CAC) scoring for further risk stratification. Using the Pooled Cohort Equations, the 10-year atherosclerotic cardiovascular disease (ASCVD) risk for fatal and nonfatal myocardial infarction or stroke can be categorized as low (<5%), borderline (5% to <7.5%), intermediate (≥7.5% to <20%), and high (≥20%). In adults at borderline or intermediate risk, the presence of risk-enhancing factors may justify initiation of moderate-intensity statin therapy. Risk enhancers include family history of premature ASCVD, LDL cholesterol level of 160 mg/dL (4.14 mmol/L) or higher, metabolic syndrome, chronic kidney disease, chronic inflammatory conditions, history of premature menopause, history of preeclampsia, South Asian ancestry, or triglyceride level of 175 mg/dL (1.98 mmol/L) or higher. If the decision about initiating statin therapy remains uncertain, it is reasonable to obtain a CAC score to guide therapeutic decisions. If the CAC score is 0, it is appropriate to withhold statin therapy and reassess in 5 to 10 years as long as higher-risk conditions (such as family history of premature ASCVD and cigarette smoking) are absent. If the CAC score is 1 to 99, it is reasonable to initiate statin therapy for most patients aged 55 years or older. In patients with a CAC score of 100 or higher or at the 75th percentile or higher, it is reasonable to initiate statin therapy regardless of age. Physiologic tests of coronary perfusion are not indicated because this patient is asymptomatic. Furthermore, adenosine cardiac magnetic resonance imaging, exercise stress echocardiography, and pharmacologic nuclear stress testing are not used to reclassify risk among asymptomatic patients. Elevated levels of lipoprotein(a), a subtype of LDL cholesterol, have been associated with increased risk for cardiovascular disease, especially coronary artery disease. However, this marker is not routinely used for purposes of reclassification of asymptomatic patients. A relative indication for its measurement is family history of premature ASCVD. In this patient at borderline risk, a CAC score is more likely to provide actionable guidance.
An 82-year-old man is admitted to the ICU with a 7-day history of fever and cough productive of green sputum. He is unable to climb the stairs to his bedroom without becoming short of breath. Medical history is remarkable for bronchiectasis and polymyalgia rheumatica. His only medication is prednisone, 10 mg/d. On physical examination, temperature is 38.8 °C (101.8 °F), blood pressure is normal, pulse rate is 115/min, and respiration rate is 25/min. Oxygen saturation is 88% breathing ambient air. Crackles are heard in the right lung base. A sputum Gram stain shows 3+ polymorphonuclear cells and 2+ gram-negative organisms. A chest radiograph is shown. Which of the following is the most appropriate treatment? Ampicillin-sulbactam and levofloxacin Aztreonam and ciprofloxacin Cefepime and ciprofloxacin Ceftriaxone and azithromycin Ceftriaxone and levofloxacin
In patients with community-acquired pneumonia and risk factors for Pseudomonas aeruginosa infection, the use of dual therapy with antipseudomonal, antipneumococcal β-lactam, or an antipseudomonal carbapenem, and antipseudomonal quinolone agents is recommended for initial empiric therapy. The most appropriate empiric treatment regimen for this patient is cefepime and ciprofloxacin. Pseudomonas aeruginosa is an uncommon cause of community-acquired pneumonia (CAP), reported in 1% to 8% of patients in various case series. However, it is important to recognize risk factors predisposing patients to this organism because standard treatment regimens for CAP require modification when Pseudomonas is suspected. Risk factors for Pseudomonas infection include structural lung disease (such as bronchiectasis, COPD, and cystic fibrosis) and frequent COPD exacerbations requiring repeated courses of antibiotics or glucocorticoids. This patient has bronchiectasis, glucocorticoid use, and the presence of gram-negative bacilli on sputum Gram stain as risk factors for Pseudomonas infection. Initial treatment with an appropriate antibiotic regimen has been shown to decrease mortality in patients with CAP caused by Pseudomonas. For this reason, the use of two agents with antipseudomonal activity is advocated for empiric therapy, with de-escalation when culture and sensitivity results become available. Even in patients with risk factors for Pseudomonas infection, it is important to continue treatment with a combination that is also active against Streptococcus pneumoniae and Legionella species until culture results are available. Recommended regimens include dual therapy with an antipseudomonal, antipneumococcal β-lactam (such as cefepime or piperacillin-tazobactam) or an antipseudomonal carbapenem (such as imipenem or meropenem) and an antipseudomonal quinolone (ciprofloxacin or levofloxacin). The combination of cefepime and ciprofloxacin meets these requirements. Neither ceftriaxone nor ampicillin-sulbactam is active against Pseudomonas and should not be used when this organism is a concern. Although levofloxacin is effective against quinolone-susceptible Pseudomonas, administration of a single antipseudomonal agent is not recommended because of the high rates of antibiotic resistance. The combination of aztreonam and ciprofloxacin provides double coverage against Pseudomonas; however, it is inactive against S. pneumoniae and therefore is inappropriate for initial empiric therapy.
A 47-year-old woman is evaluated for recent fatigue and dyspnea. She underwent an abdominal hysterectomy for recurrent severe menorrhagia 3 weeks ago and has had persistent fatigue and exertional dyspnea since the procedure. She has no other symptoms. Medical history is significant for Eisenmenger syndrome related to a ventricular septal defect. Medications are sildenafil and bosentan. On physical examination, temperature is normal, blood pressure is 98/60 mm Hg, pulse rate is 80/min and regular, and respiration rate is 20/min. The estimated central venous pressure is elevated with a prominent a wave. The apical impulse is normal, and a prominent parasternal impulse is noted at the left sternal border. The S1 is normal, and the S2 is loud. Digital clubbing and central cyanosis are noted. The abdominal wound is healing well. Laboratory testing reveals a hemoglobin level of 10.2 g/dL (102 g/L). The hemoglobin level was 15 g/dL (150 g/L) before hysterectomy. One year ago, the hemoglobin level was 18.5 g/dL (185 g/L). Which of the following is the most appropriate management? Erythropoietin Heart-lung transplantation Intravenous epoprostenol Short-course iron therapy
In patients with cyanotic conditions, such as Eisenmenger syndrome, iron deficiency is common, and short-term iron therapy will improve exercise capacity and quality of life. The most appropriate management is initiation of a short course of iron therapy for this patient with Eisenmenger syndrome and symptomatic postoperative anemia. Most patients with cyanosis have compensated erythrocytosis with stable hemoglobin levels. Iron deficiency and resultant microcytosis in these patients is often caused by inappropriate phlebotomy or blood loss. Correction of iron deficiency is associated with increased exercise capacity and improved quality of life. This patient's baseline hemoglobin level was greater than 18 g/dL (180 g/L). She developed preoperative anemia and subsequent perioperative anemia that has persisted and caused dyspnea and fatigue. Oral iron therapy often causes a rapid increase in erythrocyte mass and requires careful monitoring. Administration of ferrous sulfate or ferrous gluconate is recommended with repeat hemoglobin assessment in 7 to 10 days. Once the serum ferritin level or transferrin saturation is within the normal range, iron supplementation may be discontinued. It is important not to exceed the acceptable serum ferritin level. In the absence of dehydration, a hemoglobin level greater than 20 g/dL (200 g/L) and a hematocrit level greater than 65% are associated with hyperviscosity symptoms, such as headache, difficulty concentrating, weakness, and fatigue. Erythropoietin is used primarily in patients with anemia related to kidney disease. Iron therapy, not erythropoietin, is recommended for patients with cyanotic cardiac disease with iron deficiency anemia. This patient with cyanotic congenital heart disease and relative anemia is likely to have an elevated erythropoietin level; erythropoietin therapy for this patient will be expensive, likely ineffective, and potentially dangerous. Heart-lung transplantation is an option for end-stage cardiopulmonary disease in patients with Eisenmenger syndrome; however, it is not indicated without a trial of standard medical therapy with iron. Limited data are available regarding the efficacy of epoprostenol in patients with Eisenmenger syndrome and pulmonary hypertension. Intravenous epoprostenol is typically avoided because of the risk for paradoxical embolism with continuous intravenous therapy in patients with an intracardiac right-to-left shunt. Oral, subcutaneous, and inhaled therapies are preferred. However, this patient's symptoms are likely the result of iron deficiency, and a trial of iron replacement is first-line therapy. Read Related TextNext Question
A 22-year-old man was admitted to the intensive care unit 24 hours ago with nausea, vomiting, and lethargy of 2 days' duration. Medical history is unremarkable, and he takes no medications. His admission laboratory values were consistent with diabetic ketoacidosis, and he was initiated on intravenous fluids and intravenous insulin therapy. After 24 hours, he is currently receiving 0.45% normal saline at 250 mL/h with 20 mEq (20 mmol) of potassium chloride per liter and an insulin drip at 5 U/h. His nausea continues, but his vomiting has ceased. He is unable to eat. On physical examination, vital signs are normal. He is alert and oriented, and the remainder of his physical examination is unremarkable. Laboratory studies: Electrolytes Sodium 141 mEq/L (141 mmol/L) Potassium 4.0 mEq/L (4.0 mmol/L) Chloride 104 mEq/L (104 mmol/L) Bicarbonate 20 mEq/L (20 mmol/L) Glucose 180 mg/dL (10 mmol/L) pH 7.32 Which of the following is the most appropriate management? Continue current insulin drip rate Decrease insulin drip rate and add intravenous dextrose Discontinue intravenous potassium Transition insulin drip to subcutaneous insulin regimen
In patients with diabetic ketoacidosis, intravenous insulin therapy should be continued until complete resolution of the anion gap acidosis; as acidosis improves, it may be necessary to reduce the insulin infusion rate and add intravenous dextrose to prevent hypoglycemia. The most appropriate management for this patient is to decrease the insulin drip rate and add intravenous (IV) dextrose. This patient continues to have anion gap acidosis and should remain on IV insulin therapy to suppress ketogenesis until this resolves. His current blood glucose level below 200 to 250 mg/dL (11.1-13.9 mmol/L) increases his risk for hypoglycemia with continued IV insulin therapy. This risk can be mitigated by transitioning his IV fluids to 5% dextrose with 0.45% normal saline at 150 to 250 mL/h to maintain his glucose between 150 and 200 mg/dL (8.3-11.1 mmol/L) until his diabetic ketoacidosis resolves. Reducing the insulin drip rate to maintain his blood glucose between 150 and 200 mg/dL (8.3-11.1 mmol/L) will also decrease the risk of hypoglycemia while still suppressing ketogenesis. If he continues at the current insulin drip rate or if he does not have the addition of IV dextrose, he has an increased risk of developing hypoglycemia now that his blood glucose is less than 200 mg/dL (11.1 mmol/L). Insulin deficiency and urinary potassium losses induce shifts in potassium from the intracellular to extracellular compartments. This can result in low or normal serum potassium levels in the setting of depleted potassium stores. Potassium shifts back from the extracellular to intracellular compartments with resultant hypokalemia as insulin administration corrects the hyperglycemia, anion gap acidosis, and hyperosmolality associated with diabetic ketoacidosis. To avoid cardiac complications, serum potassium must be greater than 3.3 mEq/L (3.3. mmol/L) before initiating IV insulin. Once potassium is 3.3 mEq/L (3.3 mmol/L) or higher and IV insulin has been initiated, 20 to 30 mEq (20-30 mmol/L) of potassium chloride will usually need to be added to each liter of IV fluid to maintain a serum potassium level of 4.0 to 5.0 mEq/L (4.0-5.0 mmol/L). He is at risk of developing hypokalemia if his potassium supplementation is discontinued at this time. Although subcutaneous insulin would provide insulin to suppress ketogenesis, the IV insulin drip provides greater flexibility in dose adjustments than subcutaneous insulin. Once the patient's anion gap acidosis has resolved, transitioning from the insulin drip to a subcutaneous insulin regimen should occur.
An 80-year-old woman is evaluated for a 6-month history of worsening exertional dyspnea. Two nights ago, she awoke with sudden-onset dyspnea that was relieved with ambulation. She has not had chest pain. Medical history is significant for myocardial infarction 8 years ago. She also has a history of left ventricular dysfunction but has been previously well compensated. Her medications are lisinopril, aspirin, metoprolol, and rosuvastatin. On physical examination, temperature is normal, blood pressure is 95/60 mm Hg, pulse rate is 56/min, and respiration rate is 18/min. The lungs are clear. The carotid upstroke is low in volume. The apical impulse is laterally displaced and enlarged. S1 is soft; the aortic component of S2 is diminished. There is no S3 or S4. A grade 2/6 mid-peaking systolic murmur is heard throughout the precordium. The remainder of the examination is normal. An echocardiogram demonstrates a left ventricular ejection fraction of 32%. The aortic valve is slightly calcified. The stroke volume is markedly decreased (23 mL/m2). The mean aortic gradient is 20 mm Hg (consistent with mild to moderate stenosis), and the aortic valve area is calculated to be 0.7 cm2 (consistent with severe stenosis). Which of the following is the most appropriate next step in management? Aortic valve replacement Coronary angiography Dobutamine echocardiography Switch lisinopril to valsartan-sacubitril
In patients with findings of low-flow, low-gradient aortic stenosis, the primary abnormality may be either severe ventricular dysfunction with pseudostenosis or critical aortic stenosis; dobutamine echocardiography is needed to distinguish between the two entities. The most appropriate next step in management is dobutamine echocardiography. This patient has low-flow, low-gradient aortic stenosis. The primary abnormality may be either severe ventricular dysfunction with pseudostenosis or critical aortic stenosis. Severe ventricular dysfunction with diminished cardiac output leads to decreased opening forces on the aortic valve, and the valve area may be calculated to be low even though the valve is not anatomically severe (pseudostenosis). Dobutamine, an inotropic agent, can be used to increase cardiac output and differentiate between these two entities. Dobutamine will cause an increase in the aortic valve area in pseudostenosis, whereas in anatomically severe aortic stenosis, the aortic valve area will not significantly increase. Aortic valve replacement should be considered for patients with anatomically severe aortic stenosis. This procedure is currently inappropriate for a patient in whom the diagnosis of severe aortic stenosis has not been established. Coronary angiography is recommended before valve surgery in patients with severe valvular heart disease and a history of coronary artery disease, suspected myocardial ischemia, or left ventricular systolic dysfunction. These patients may require concomitant revascularization at the time of surgery. However, coronary angiography is indicated only in patients for whom valve surgery has been determined to be appropriate. The 2016 American College of Cardiology/American Heart Association/Heart Failure Society of America heart failure focused update recommends that valsartan-sacubitril be substituted for an ACE inhibitor or an angiotensin receptor blocker in patients with chronic symptomatic heart failure with reduced ejection fraction (New York Heart Association functional class II or III) who have tolerated ACE inhibitor or angiotensin receptor blocker therapy well. In this patient, differentiation of pseudostenosis from true aortic stenosis is most appropriate before initiating an increase in medical therapy. Read Related TextNext Question
A 69-year-old man is evaluated after a recent hospitalization for heart failure. On hospital admission, he was appropriately treated, and an echocardiogram showed a left ventricular ejection fraction of 30% and left ventricular hypertrophy. He is currently asymptomatic. Medical history is otherwise significant for hypertension and hyperlipidemia. Medications are enalapril, furosemide, low-dose aspirin, and atorvastatin. On physical examination, vital signs are normal. There is no jugular venous distention. Cardiac examination is normal. No edema is noted. Which of the following is the most appropriate treatment? Add bisoprolol Add diltiazem Add spironolactone Discontinue enalapril and add losartan No additional therapy
In patients with heart failure with reduced ejection fraction, β-blockers improve remodeling, increase ejection fraction, and reduce hospitalization and mortality when added to ACE inhibitor and diuretic therapy. This patient with heart failure with reduced ejection fraction (HFrEF) should be started on a β-blocker, such as bisoprolol. In patients with heart failure, each follow-up visit should include evaluation for progression of heart failure symptoms, assessment of volume status, and a review of the patient's medications to ensure guideline-directed treatment. Optimal medical therapy includes an ACE inhibitor, β-blocker (specifically, metoprolol succinate, carvedilol, or bisoprolol), and an aldosterone antagonist (in symptomatic patients). β-Blockers improve remodeling, increase ejection fraction, and reduce hospitalization and mortality when added to ACE inhibitor and diuretic therapy. β-Blocker and ACE inhibitor therapies are indicated for all classes of heart failure, including New York Heart Association functional class I. This patient was discharged from the hospital before initiation of β-blocker therapy, and he should be started on bisoprolol now. Diltiazem and verapamil are associated with worse outcomes in patients with HFrEF, and these agents should not be used in this population. If calcium channel blockers are necessary for hypertension treatment, amlodipine or felodipine have been shown to be safe in patients with heart failure. Aldosterone antagonists (spironolactone, eplerenone) have been studied in patients with heart failure and New York Heart Association functional class II to IV symptoms and have been shown to reduce mortality and morbidity. Because of the risk for kidney dysfunction and hyperkalemia, these drugs should be used only in patients with a serum creatinine level below 2.5 mg/dL (221 µmol/L) in men or below 2.0 mg/dL (176.8 µmol/L) in women, and with a serum potassium level below 5.0 mEq/L (5.0 mmol/L). These drugs are not as effective as diuretics at the doses used in heart failure therapy (12.5-25 mg/d for spironolactone, 25-50 mg/d for eplerenone). The patient is asymptomatic now, and spironolactone is not indicated. If it were indicated, it would not be added until after uptitration of the β-blocker and ACE inhibitor doses. Angiotensin receptor blockers (ARBs), such as losartan, are a suitable alternative to ACE inhibitors for the treatment of HFrEF and offer similar morbidity and mortality benefits. In patients taking an ACE inhibitor, the primary reason to switch to an ARB is ACE inhibitor-induced cough or angioedema. Caution should be exercised when switching patients to an ARB as a result of angioedema because this side effect has been reported with ARB therapy as well. This patient does not have side effects that would prompt switching an ACE inhibitor to an ARB.
A 64-year-old woman is evaluated during a posthospital visit for severe COPD with an FEV1 of 30% of predicted. She has been admitted three times during the last year with acute exacerbations characterized by cough, increased purulent sputum production, and dyspnea. She is now at baseline of her exertional dyspnea and has no cough. She has already participated in a pulmonary rehabilitation program. She currently takes tiotropium, budesonide/formoterol, and albuterol. On physical examination, vital signs are normal. Oxygen saturation is 90% on 3 L/min of supplemental oxygen at rest and with exertion. Pulmonary examination reveals decreased breath sounds throughout. The remainder of the examination is noncontributory. Which of the following is the most appropriate treatment to reduce this patient's COPD exacerbations? Chronic low-dose oral glucocorticoid Chronic macrolide therapy Increase supplemental oxygen Nebulized hypertonic saline
In patients with severe COPD and frequent exacerbations, chronic macrolide therapy has been shown to decrease COPD exacerbations. The most appropriate guideline-recommended (grade 2A) treatment of this patient with severe COPD and frequent exacerbations is chronic macrolide therapy. Macrolide antibiotics have inflammatory and antimicrobial effects and may reduce the frequency of exacerbations when used long-term by patients with severe COPD. Several clinical trials to assess prophylactic use and benefit have demonstrated a reduction in the rate of exacerbation in patients with moderate to severe COPD with one or more moderate or severe exacerbations in the previous year despite optimal maintenance inhaler therapy. The duration and exact dosage of macrolide therapy are unknown. The primary concerns with long-term macrolide therapy are development of antibiotic resistance, including macrolide-resistant strains of nontuberculous mycobacteria. In addition, hearing loss and potentially fatal arrhythmias due to prolongation of the QT interval have occurred in association with azithromycin. The long-term use of systemic glucocorticoids is avoided in the chronic management of COPD due to lack of demonstrated benefit and recognized increased risk of significant side effects such as diabetes, hypertension, muscle weakness, and decreased functional status. The use of supplemental oxygen in patients with COPD and hypoxemia has been shown to improve quality of life and mortality in patients who have resting hypoxemia with an arterial PO2 of 55 mm Hg (7.31 kPa) or lower, or oxygen saturation on pulse oximetry (Spo2) of 88% or lower. This patient's Spo2 is above 88% at rest and on exertion on her current level of supplemental oxygen. Increasing this further has not been shown to decrease the rate of acute exacerbations. Mucolytics such as nebulized hypertonic saline and airway clearance maneuvers may provide some symptomatic relief in patients with significant sputum production, but this patient does not have cough or sputum production. Moreover, these interventions have not been shown to decrease the rate of acute exacerbations in patients with COPD.
A 67-year-old man is evaluated during a follow-up visit. Medical history is significant for a 7-year history of heart failure with placement of an implantable cardioverter-defibrillator 6 years ago. He has New York Heart Association functional class II symptoms and is currently stable. Since his last visit 6 months ago, he has had no changes in medications, symptoms, or other medical issues. Medications are valsartan-sacubitril, carvedilol, furosemide, and spironolactone. On physical examination, the patient is afebrile, blood pressure is 108/74 mm Hg, and pulse rate is 64/min. He has no jugular venous distention or S3. No edema is noted. An echocardiogram obtained 1 year ago demonstrated a left ventricular ejection fraction of 25% and left ventricular end-diastolic diameter of 6.7 cm; these findings are unchanged from 2 years ago. Heart failure education and the need for diet and medication adherence are reinforced. Which of the following is the most appropriate testing to perform at this visit? Echocardiography 24-Hour ambulatory electrocardiographic monitoring Serum B-type natriuretic peptide level measurement Serum electrolyte measurement and kidney function studies
In patients with heart failure, each follow-up visit should include evaluation of current symptoms and functional capacity; assessment of volume status, electrolytes, and kidney function; and review of the patient's medication regimen for adequacy. Serum electrolyte measurement and kidney function studies should be performed at this visit. In patients with heart failure, each follow-up visit should include evaluation of current symptoms and functional capacity; assessment of volume status, electrolytes, and kidney function; and review of the patient's medication regimen for adequacy. It is also important to reinforce heart failure education and the need for diet and medication adherence. This patient with stable chronic heart failure is taking appropriate medications, has had no changes in his symptoms or exercise tolerance, and has no evidence of volume overload. Because he is taking a diuretic, an angiotensin receptor blocker, and spironolactone, assessing his electrolyte levels and kidney function is appropriate. Routine repeat echocardiography is not recommended in a stable patient with heart failure. According to current guidelines, repeat echocardiography should be performed if the patient has had a change in clinical status, received a new medication that might significantly improve ejection fraction (such as a β-blocker), or is being considered for device therapy after medical optimization. Twenty-four hour ambulatory electrocardiographic monitoring is not indicated to routinely screen for arrhythmias in patients with heart failure in the absence of symptoms or signs worrisome for an arrhythmia. The B-type natriuretic peptide (BNP) level has prognostic value in patients with heart failure and can be used to determine whether heart failure is progressing. However, determining a prognosis according to an absolute value is difficult, and models that have used BNP levels have different BNP cutpoints and lengths of follow-up. At this time, a BNP level is not routinely obtained in an otherwise stable patient. Other biomarkers of myocardial fibrosis, such as soluble ST2 receptor and galectin-3, have been shown to be predictive of death and hospitalization in small trials, but their role in the management of patients with heart failure is not yet established.
A 62-year-old woman with alcoholic cirrhosis is evaluated for hypovolemic shock. She was hospitalized 24 hours ago with upper gastrointestinal bleeding. She underwent upper endoscopy, and a bleeding distal esophageal varix was controlled with epinephrine injection and banding. Gastric varices were also noted. Today she vomited 300 mL of bright red blood. Current medications are lactulose, rifaximin, pantoprazole, norfloxacin, and octreotide. On physical examination, blood pressure is 79/54 mm Hg, pulse rate is 95/min, and respiration rate is 21/min. Oxygen saturation is 94% on 2 L/min of oxygen through nasal cannula. Ascites and splenomegaly are present. Laboratory studies this morning reveal a hemoglobin level of 8 g/dL (80 g/L), platelet count of 74,000/µL (74 × 109/L), and an INR of 1.4. Which of the following is the most appropriate immediate management? Packed red blood cell transfusion Recombinant factor VII infusion Transjugular intrahepatic portosystemic shunting Upper endoscopy
In patients with hemorrhagic shock, initial management includes volume resuscitation with blood products to stabilize the patient. The most appropriate management for this patient is a transfusion of packed red blood cells. Clinicians should base decisions on blood transfusion on the full clinical picture, recognizing that overtransfusion may be as damaging as undertransfusion. A restrictive transfusion policy aiming for a hemoglobin level of 7 to 8 g/dL (70 to 80 g/L) is suggested in hemodynamically stable patients. More liberal blood transfusion thresholds lead to increased portal pressures and risk of further bleeding. This patient most likely is experiencing hemorrhagic shock from a recurrent variceal bleeding episode. Because the patient is hemodynamically unstable, initial guideline-based management includes volume resuscitation, and transfusion of blood products is the best method for achieving this. In patients who are bleeding and have a platelet count less than 50,000/μL (50 × 109/L) or who have an INR greater than 1.5, transfusion of platelets or fresh frozen plasma, respectively, is indicated. Historically, uncontrolled variceal bleeding offered a compelling rationale for use of recombinant factor VII, but a 2014 meta-analysis of almost 500 patients from two randomized clinical trials evaluated the role of factor VII following variceal bleed. In both trials, there was no indication that factor VII improved outcomes. Therefore, it would not be appropriate for this patient. Emergent upper endoscopy and consideration of placing a transjugular intrahepatic portosystemic shunt are indicated in this patient, but are pursued after initial resuscitation efforts are completed. Because this patient is hemodynamically unstable and these procedures take time to coordinate, they would not supersede initial resuscitative efforts with blood products.
A 64-year-old woman is evaluated for worsening leg weakness. She was evaluated 2 weeks ago for a 3-week history of lower extremity weakness; laboratory studies showed a serum creatine kinase level of 23,000 U/L. History is notable for hypercholesterolemia treated with lovastatin, which was discontinued at the initial visit. On physical examination, vital signs are normal. There is no rash. Examination shows 3/5 strength of the quadriceps and hamstrings (4/5 strength 2 weeks ago); 4/5 strength in the deltoids, biceps, and triceps; and 5/5 strength of the neck flexors, neck extensors, and distal upper and lower extremities. The remainder of the examination is normal. Current laboratory studies show an erythrocyte sedimentation rate of 25 mm/h and a serum creatine kinase level of 20,876 U/L. In addition to a muscle biopsy, which of the following is most likely to establish the diagnosis? Anti-cyclic citrullinated peptide antibodies Anti-cytosolic 5′-nucleotidase 1A antibodies Anti-histidyl tRNA synthetase antibodies Anti-HMG Co-A reductase antibodies
In patients with immune-mediated necrotizing myopathy, myositis may persist despite statin discontinuation and is often associated with the production of anti-HMG Co-A reductase antibodies. In addition to a muscle biopsy, anti-HMG Co-A reductase antibody testing is most likely to establish the diagnosis. A small number of patients taking statins develop immune-mediated necrotizing myopathy (IMNM), which can be easily confused with polymyositis. IMNM is thought to occur as a result of an immune response to HMGCR, the pharmacologic target of statins. IMNM can be a paraneoplastic phenomenon related to connective tissue disease, or drug related, most commonly statins. Statin use may initiate or worsen the syndrome. In patients taking statins, myositis may rapidly progress and persist despite drug discontinuation due to this abnormal immune response and is often associated with the production of anti-HMG Co-A reductase antibodies. The histologic finding of necrotic muscle fibers with only minimal inflammatory cells that do not invade the muscle fibers is characteristic of IMNM, and, given the patient's history of exposure to statins, a muscle biopsy and determination of anti-HMG CO-A reductase antibodies will most likely establish the diagnosis. The condition may respond to immunosuppressive therapy. Anti-cyclic citrullinated peptide antibodies are not seen in inflammatory myopathies; they have a relatively high specificity (>90%) for rheumatoid arthritis and may be useful in the differential diagnosis of early polyarticular inflammatory arthritis. Anti-cytosolic 5′-nucleotidase 1A antibodies have been identified in about half of patients with inclusion body myositis. Strong reactivity for these antibodies has relatively high specificity for inclusion body myositis compared with other muscle diseases. Anti-histidyl-tRNA synthetase antibodies, particularly the subset anti-Jo-1 antibodies, are associated with the presence of interstitial lung disease in patients with inflammatory myopathies and, in particular, with antisynthetase syndrome of interstitial lung disease, Raynaud phenomenon, inflammatory arthritis, and mechanic's hands.
A 68-year-old man is evaluated for a 2-month history of exertional dyspnea. Medical history is significant for diabetes mellitus, hypertension, and hyperlipidemia. Medications are lisinopril, hydrochlorothiazide, metformin, and atorvastatin. On physical examination, vital signs are normal. Oxygen saturation is 99% breathing ambient air. Cardiopulmonary examination shows a regular rhythm and a paradoxically split S2. There is no peripheral edema. An electrocardiogram is shown. (LBBB) Which of the following is the most appropriate diagnostic test to perform next? Adenosine single-photon emission CT Coronary artery calcium scoring Exercise single-photon emission CT Exercise electrocardiography
In patients with left bundle branch block, the preferred diagnostic test for coronary artery disease is a vasodilator stress test because myocardial perfusion imaging with exercise or dobutamine stress may result in a false-positive perfusion defect in the basilar septum. Adenosine single-photon emission CT is the most appropriate next diagnostic test. This patient with several cardiovascular risk factors has exertional dyspnea, which may be an angina equivalent, and he should undergo stress testing to evaluate for coronary artery disease (CAD). Because his electrocardiogram (ECG) shows left bundle branch block, ST-segment changes with exercise cannot be used to evaluate for the presence of obstructive CAD. He must undergo stress testing with additional imaging, such as nuclear perfusion imaging or stress echocardiography. Exercise is typically the preferred mode of stress because of the additional functional information it provides. However, in the case of left bundle branch block, myocardial perfusion imaging with exercise or dobutamine stress may result in a false-positive perfusion defect in the basilar septum, and these stressors should be avoided. Instead, vasodilator stress testing should be used. Vasodilators, such as dipyridamole, regadenoson, and adenosine, produce hyperemia and a flow disparity between myocardium supplied by unobstructed vessels and myocardium supplied by the stenotic vessel (due to the inability of the distal vasculature to dilate). Coronary artery calcium scoring, which quantifies the amount of calcium in the walls of the coronary arteries, would document the presence of atherosclerotic disease in this symptomatic patient with risk factors, but it would not determine whether there is obstructive CAD. Although the absence of any coronary artery calcification has been shown to have a high specificity for the absence of obstructive CAD, trials evaluating coronary artery calcium scoring have typically focused on primary prevention in asymptomatic patients.
A 45-year-old woman is evaluated for a 12-month history of exertional dyspnea. She experiences shortness of breath during mild exertion, such as house chores and walking on flat surfaces. She describes her symptoms as debilitating, as they have interfered with her activities of daily living. She has not had symptoms at rest, and she has had no palpitations. She takes hydrochlorothiazide for hypertension. On physical examination, temperature is normal, blood pressure is 112/72 mm Hg, pulse rate is 76/min, and respiration rate is normal. The apical impulse is slightly sustained but not displaced. S1 is increased. There is an early diastolic sound followed by a soft rumble heard best at the apex. S2 is normal. An echocardiogram shows findings consistent with moderate rheumatic mitral stenosis and minimal mitral regurgitation. The mean gradient across the mitral valve is 8 mm Hg, and the mitral valve area is calculated to be 1.8 cm2. The mitral valve is pliable. Moderate pulmonary hypertension is present, with an estimated pulmonary artery systolic pressure of 45 mm Hg. Which of the following is the most appropriate next step in management? Exercise echocardiography Medical therapy Percutaneous balloon mitral valvuloplasty Surgical mitral valve replacement
In patients with mitral stenosis who have a discrepancy between the clinical findings and the echocardiographic findings, exercise echocardiography should be pursued to assess the response of the mitral gradient and pulmonary pressures. The most appropriate next step in management is exercise echocardiography. This patient has symptoms typical of severe mitral stenosis (progressive exertional dyspnea) but echocardiographic findings consistent with only moderate mitral stenosis. Symptoms of mitral stenosis can be indolent, with patients remaining asymptomatic for years and then presenting with a gradual decrease in activity. Other symptoms include dyspnea, orthopnea, fatigue, and, less commonly, hemoptysis or systemic embolization. Symptoms typically are not present until the mitral valve area is less than 1.5 cm2, although tachycardia may precipitate symptoms at larger valve areas. In patients with a discrepancy between the clinical findings (debilitating symptoms) and the echocardiographic findings (moderate stenosis), additional testing is necessary to ensure that the patient's symptoms can be attributed to mitral stenosis. In this case, stress echocardiography should be pursued to assess the response of the mitral gradient and pulmonary pressures. Both exercise and dobutamine echocardiography are viable options, but exercise is generally preferred as the more physiologic test. For patients with mitral stenosis, the valve gradient is heavily flow dependent and may become severely elevated only with exercise. Exercise and the accompanying increases in heart rate augment cardiac output and transvalvular flow and shorten the diastolic filling time in the left ventricle. These conditions can unmask mitral stenosis that does not appear to be functionally severe at rest. With an increase in left atrial pressures during exercise, significant pulmonary hypertension also may become evident. Medical therapy for mitral stenosis consists of diuretics or long-acting nitrates, which may help alleviate symptoms such as dyspnea. In addition, β-blockers or nondihydropyridine calcium channel blockers can lower heart rate and improve left ventricular diastolic filling time. Medical therapy would be appropriate if the mitral stenosis were not hemodynamically significant or if she were not a surgical candidate. After hemodynamically significant mitral stenosis is confirmed, percutaneous or surgical therapy can be considered. However, these interventions are not appropriate before confirmation with exercise echocardiography.
A 40-year-old woman is evaluated for a 12-year history of inflammatory arthritis, puffy fingers, Raynaud phenomenon, serositis, and gastroesophageal reflux disease. On physical examination, vital signs are normal. The fingers are puffy, and cyanosis of the fingertips is noted. Cardiac examination is normal. The lungs are clear. There is no friction rub heard on examination. Laboratory studies show positive anti-U1-ribonucleoprotein antibodies. Which of the following complications of this patient's disease is a major cause of early mortality? Central nervous system vasculitis Pericarditis Pulmonary arterial hypertension Rapidly progressive glomerulonephritis
In patients with mixed connective tissue disease, pulmonary arterial hypertension and interstitial lung disease are associated with premature mortality; therefore, regular follow-up and monitoring for these conditions is warranted. Pulmonary arterial hypertension (PAH) and interstitial lung disease are the two conditions most commonly associated with premature mortality in patients like this with mixed connective tissue disease (MCTD). MCTD is rare (1:1,000,000). Age at onset is between 30 and 50 years, with a 9:1 female predominance. Most patients have no known risk factors. MCTD is an overlap syndrome that includes features of systemic lupus erythematosus (SLE), systemic sclerosis, and/or polymyositis in the presence of anti-U1-ribonucleoprotein (RNP) antibodies. More than 50% of patients with MCTD have hand edema and synovitis at disease onset. About one third develop myositis, and nearly half develop decreased esophageal motility and fibrosing alveolitis. PAH occurs in 20%, with fatigue often the initial symptom. PAH (and interstitial lung disease) is associated with poor prognosis and is a major cause of mortality in patients with MCTD. The course of MCTD is variable; the likelihood of developing PAH, interstitial lung disease, and/or cardiovascular disease increases with disease duration. Regular follow-up and monitoring for these conditions is warranted. Patients with MCTD should undergo high-resolution CT of the chest, echocardiography, and pulmonary function testing. Central nervous system (CNS) disease is common in MCTD, with trigeminal neuralgia occurring in up to 25% of patients. CNS vasculitis is extremely rare, and CNS disease is not typically a cause of mortality in patients with MCTD. Pericarditis is likely the most common cardiac manifestation of MCTD and while contributing to excess morbidity, it does not result in premature death. As with other chronic inflammatory diseases, premature atherosclerosis is common in patients with MCTD and can contribute to both morbidity and early mortality. Kidney disease is found in approximately 25% of patients with MCTD and most often presents as a membranous nephropathy that typically does not lead to premature death. Glomerulonephritis is rarely observed. Renovascular hypertension as seen in systemic sclerosis is a rare cause of mortality.
A 74-year-old woman is evaluated in the emergency department for decreased responsiveness. She has become progressively confused and lethargic. She lives alone and stopped taking her medications at some time unknown. She was brought to the emergency department by a family member. The patient had a near-total thyroidectomy for multinodular goiter 3 years ago. On physical examination, temperature is 36.1 °C (97.0 °F), blood pressure is 80/45 mm Hg, pulse rate is 46/min, respiration rate is 10/min, and oxygen saturation is 92% breathing ambient air. BMI is 28. The patient is arousable with painful stimuli. She has a well-healed anterior neck scar. The patient's skin is cool and dry. She has periorbital edema and bipedal edema. Other than bradycardia, the cardiac examination is normal. The relaxation phase of her deep tendon reflexes is delayed. Laboratory results show a sodium level of 129 mEq/L (129 mmol/L). Intravenous fluids are initiated. Which of the following is the essential initial step in the management of this patient? Administer intravenous hydrocortisone Administer intravenous levothyroxine and liothyronine Administer norepinephrine Administer oral levothyroxine
In patients with myxedema coma, intravenous hydrocortisone should be administered before thyroid hormones to treat possible adrenal insufficiency. The most appropriate next step in the management of this patient is to administer intravenous hydrocortisone. This patient has myxedema coma, which has a very high mortality rate if there is a delay in treatment. Myxedema coma is more common in elderly women; it may occur in those with a history of hypothyroidism or no antecedent illness. Myocardial infarction, infection, stroke, trauma, and gastrointestinal bleeding are common precipitating events. Mental status changes and hypothermia are the most common clinical manifestations. Ventilatory drive is decreased, resulting in hypoxemia and hypercapnia. Additional signs include bradycardia, hypoglycemia, hyponatremia, and/or hypotension. This patient's condition was likely precipitated by recent nonadherence with her prescribed medications. The serum cortisol level should be checked as soon as possible to evaluate for concomitant adrenal insufficiency prior to initiation of thyroid hormone replacement. While awaiting the results of the serum cortisol measurement, it is advisable to empirically initiate high-dose hydrocortisone. This therapy may be discontinued if the serum cortisol level is found to be normal or high. In patients with adrenal insufficiency, administering thyroid hormone prior to glucocorticoids could precipitate an adrenal crisis by augmenting cortisol metabolism. Following the administration of glucocorticoids, intravenous thyroid hormone replacement should be initiated. Treatment with levothyroxine is universally recommended. Although controversial, some experts suggest administering liothyronine concomitantly. Once clinically improved, the patient can be transitioned to oral levothyroxine. Hypotension can generally be resolved over a matter of hours with the administration of fluids and treatment of the hypothyroidism. Persistent hypotension can be treated with a vasopressor drug. The initiation of norepinephrine is premature in this patient.
A 68-year-old man is evaluated for increasing cognitive deficits, walking difficulties with occasional falls, and depression. He has become more forgetful over the past 5 years, has a 1-year history of shuffling his feet when walking, awakens three to four times each night to urinate (sometimes becoming incontinent), and generally has become more withdrawn and irritable. He also has difficulty remembering names and appointments, and his wife now manages their finances and his medications, which consist of hydrochlorothiazide for hypertension and metformin for type 2 diabetes mellitus. On physical examination, vital signs are normal. The patient is slow to rise from a seated position. Gait examination reveals slow, short steps and difficulty turning. He scores 22/30 (normal, ≥24) on the Mini-Mental State Examination, losing four points in the orientation sections, one point in the registration section, and three points in the attention/calculation section. An MRI of the brain shows moderate generalized atrophy and prominent cerebral ventricles. Which of the following is most likely to be diagnostic in this patient? Amyloid PET scan Fluorodeoxyglucose PET scan Lumbar puncture Neuropsychological testing
In patients with normal pressure hydrocephalus (NPH), lumbar puncture with removal of a large volume of cerebrospinal fluid (CSF) and subsequent measurement of CSF opening pressure can help confirm the diagnosis and determine if the NPH is reversible. This patient most likely has normal pressure hydrocephalus (NPH) and should undergo lumbar puncture with removal of a large volume of cerebrospinal fluid (CSF) and subsequent measurement of CSF opening pressure. Because NPH is reversible, detecting it early is essential. The triad of gait changes, urinary incontinence, and cognitive impairment is most characteristic of NPH, with gait impairment being the most prominent feature. The MRI shows ventricular dilation typical of normal pressure hydrocephalus. The diagnosis is suggested by an expansion of the entire ventricular system in the absence of sulcal atrophy or an obstruction. A timed gait evaluation performed just before CSF removal and again before 60 minutes have passed since removal is appropriate to confirm the diagnosis. If gait improves after CSF removal, then NPH likely is present and reversal is possible. Amyloid PET scanning is approved for clinical use in patients with suspected Alzheimer disease. The most prominent early symptom of Alzheimer disease is memory disturbance, not gait disturbance or urinary incontinence. More importantly, however, this imaging test would delay the diagnosis of NPH, a potentially reversible cause of his symptoms. Although a fluorodeoxyglucose PET scan can be used for pattern recognition of certain dementias, it is a nonspecific test and is unlikely to definitively establish the diagnosis in this patient. Neuropsychological testing is most helpful in patients with mild symptoms of cognitive impairment, especially in localizing these symptoms. It also can help establish or exclude malingering as the reason for the symptoms. It likely would have limited utility for this patient who had good results on the memory portion of the Mini-Mental State Examination and whose other symptoms are gait impairment and urinary incontinence.
A 59-year-old woman is evaluated during a follow-up visit for mouth pain of 2 years' duration. Her mouth pain was previously localized to the tongue and floor of the mouth; however, it is now diffuse, burning, and constant. The pain has also worsened over the past year despite continued dosage escalation of a transdermal fentanyl patch and oral hydromorphone. She has no pain with swallowing. Medical history is significant for squamous cell carcinoma of the tongue treated with chemotherapy and radiation therapy; her treatment course was complicated by severe mucositis. She has been without evidence of disease for the past 2 years. On physical examination, vital signs are normal. The patient is alert and oriented but inattentive and slow to answer questions. Reflexes are exaggerated in the upper and lower extremities, and myoclonic jerking of the legs and clonus at the ankles are noted. Strength is normal. Mouth examination reveals erythematous oral mucosa with no focal areas of erosion or masses. A CT scan of the head and neck shows no evidence of recurrent disease. Which of the following is the most likely diagnosis? Malingering Opioid-induced hyperalgesia Opioid withdrawal Pseudoaddiction
In patients with opioid-induced hyperalgesia, the character of their pain may change during the course of opioid therapy, pain may worsen with increased opioid dosages, and pain may decrease when opioid dosages are reduced. The most likely diagnosis is opioid-induced hyperalgesia. Opioid-induced hyperalgesia is thought to result from repeated exposure to systemic opioids. In patients with this pain syndrome, the character of their pain may change during the course of opioid therapy, pain may worsen with increased opioid dosages, and pain may decrease when opioid dosages are reduced. In this case, the patient has chronic pain as a sequela of her cancer diagnosis and treatment, and she has required significant dosage escalation in the absence of progressive disease. She also has evidence of cognitive slowing, hyperreflexia, and myoclonus as a result of the neuroexcitatory effects of long-term opioid use; these signs are concerning for opioid-induced toxicity. Appropriate management of opioid-induced hyperalgesia includes opioid dosage reduction in a monitored setting with close follow-up. Malingering is characterized by the development of symptoms that lack a pathologic basis. The key feature is faking or exaggerating symptoms for an obvious external benefit, such as money, drugs, or escaping criminal prosecution. Malingering should be suspected in patients with inconsistencies in the history and physical examination; nonadherence to recommended diagnostic testing or treatments; known or suspected personality disorder; and legal difficulties. There is no indication of these traits to suggest malingering as the cause of this patient's symptoms. Opioid withdrawal is unlikely in this patient given her constant symptoms and escalation in the dosages of hydromorphone and transdermal fentanyl. Pseudoaddiction is a phenomenon in which patients exhibit behaviors concerning for substance use disorder or addiction that are driven by inadequate pain control in the setting of a documented progressive disease. Although this patient has a sequela of cancer therapy, her clinical status is unchanged, and there is no evidence of progressive disease on physical examination or CT scan. Read Related TextNext Question
A 43-year-old man is evaluated for a change in his usual migraine headache. The headaches are now more frequent and respond poorly to his previously effective migraine headache medications. His medical history is otherwise unremarkable. Medications include ibuprofen and sumatriptan. On physical examination, vital signs are normal. Physical examination including funduscopic examination is normal. MRI reveals a 1-cm pituitary adenoma with suprasellar extension with no compression or abutment of the optic chiasm or optic nerves. Laboratory studies show a cortisol level of 17 µg/dL (469.2 nmol/L), thyroid-stimulating hormone level of 2.6 µU/mL (2.6 mU/L), and thyroxine (T4) level of 1.2 ng/dL (15.5 pmol/L). Which of the following is the most appropriate diagnostic test to perform next? Measurement of prolactin and insulin-like growth factor 1 (IGF-1) Measurement of urine free cortisol Visual field examination No further evaluation is needed
In patients with pituitary tumors, pituitary hypersecretion should be ruled out by biochemical testing. Measurement of prolactin and insulin-like growth factor 1 (IGF-1) is recommended for the evaluation of possible hypersecretion of an incidentally discovered pituitary tumor. When a pituitary tumor is incidentally noted, investigation must determine (1) whether it is causing a mass effect, (2) whether it is secreting excess hormones, and (3) whether it has a propensity to grow and cause problems in the future. After a thorough history and physical examination, biochemical testing can be undertaken in a targeted fashion based on the patient's clinical signs and symptoms. Although not generally useful in the differential diagnosis of a pituitary mass, all patients should be evaluated for hormone hyposecretion in order to identify and replace hormone deficiencies. Initial tests to evaluate for hormone deficiency should include measurement of 8 AM cortisol, thyroid-stimulating hormone (TSH), free (or total) thyroxine (T4), follicle stimulating hormone (FSH), testosterone in men and menstrual history in women (normal menstrual cycles eliminates the need to measure hormone levels). Prolactin and IGF-1 are measured to rule out pituitary hormone hypersecretion. Screening for growth hormone excess in pituitary incidentalomas may allow early detection of a growth hormone secreting tumor therefore increasing the chance of a surgical cure. If the tumor is not causing mass effect and there is no evidence of hormone excess, a pituitary MRI should be repeated in 6 months for a macroadenoma (≥1 cm) and 12 months for a microadenoma (<1 cm) to assess for growth. If no growth occurs, MRIs should be repeated every 1 to 2 years for the next 3 years and then intermittently thereafter. Measurement of urine free cortisol is not necessary in every patient with a pituitary tumor. This should be measured only when Cushing disease is suspected based on clinical history and physical examination findings. This patient has no physical features suspicious for Cushing disease nor does he have diseases associated with Cushing disease such as diabetes or hypertension. Visual field examination is absolutely necessary in all patients with a macroadenoma that abuts or compresses the optic chiasm, but it is not necessary in patients with no evidence of involvement of the optic chiasm on MRI. This patient's tumor does not abut or compress the optic chiasm or optic nerves. No further evaluation is inappropriate as pituitary hypersecretion should be evaluated in all pituitary tumors. Read Related TextNext Question
A 32-year-old woman arrives to establish primary care. She is sexually active with a new male partner and reports consistent condom use. Medical history is significant for chlamydia cervicitis 5 years ago and treatment for syphilis 6 years ago. Her only medication is an oral contraceptive. On physical examination, vital signs and the remainder of the examination are unremarkable. A syphilis enzyme immunoassay is positive; rapid plasma reagin testing is negative. A fluorescent treponemal antibody test is positive. Nucleic acid amplification testing is negative for Neisseria gonorrhoeae and Chlamydia trachomatis. HIV antigen/antibody combination testing is negative. Which of the following is the most appropriate management? Intramuscular benzathine penicillin, single dose Intramuscular benzathine penicillin, weekly for 3 weeks Repeat serology with Treponema pallidum particle agglutination assay No further testing or treatment
In patients with previously treated syphilis, treponemal serology results will remain positive, but nontreponemal tests will be negative; these patients require no further testing or treatment. This woman's serologic results are consistent with successfully treated syphilis, and no additional testing or treatment is indicated. Many laboratories use a "reverse" screening strategy, whereby a treponemal test, such as an enzyme immunoassay (EIA), is performed first and, if positive, is followed by a nontreponemal test (rapid plasma regain [RPR] or Venereal Disease Research Laboratory [VDRL] test). If the EIA is positive and the RPR or VDRL is negative, the positive result should be confirmed by a second treponemal test (such as the fluorescent treponemal antibody test performed in this patient). A positive EIA (confirmed by a second test) with a negative RPR is the expected serologic result in a patient who has been treated for syphilis. The treponemal test (EIA) remains positive indefinitely, but the nontreponemal test (RPR or VDRL) should remain negative. If the nontreponemal test became positive again, it would indicate a new infection, and treatment, based on disease stage, would be indicated. If this patient had reported no history of treatment for syphilis, she should be treated for syphilis of unknown duration, which consists of 3 weekly doses of intramuscular benzathine penicillin. Single-dose benzathine penicillin is indicated for the treatment of primary, secondary, and early latent syphilis. Because the positive EIA result was already confirmed by a second treponemal test, the fluorescent treponemal antibody, additional testing with another treponemal test, such as the Treponema pallidum particle agglutination assay, is not necessary.
A 57-year-old woman is evaluated during hospitalization following surgical fixation of a right femur neck pathologic fracture. Pathology of the femur shows a neoplasm containing numerous giant cells consistent with brown tumor. On physical examination, vital signs are normal. There is a palpable mass on the lower left side of the right neck. There is an incision with surgical staples on the right hip. The remainder of the examination is unremarkable. Laboratory studies: Alkaline phosphatase 260 U/L Calcium 13.2 mg/dL (3.3 mmol/L) Creatinine 1.6 mg/dL (141.4 µmol/L) Phosphorus 1.9 mg/dL (0.6 mmol/L) Parathyroid hormone 1142 pg/mL (1142 ng/L) Neck ultrasound shows a solid hypervascular mass (6 × 2.9 × 3 cm) posterior to the left lobe of the thyroid, with compression and displacement of the trachea. A parathyroidectomy is planned. Which of the following is the most appropriate test to perform next? 1,25-Dihydroxyvitamin D level 24-Hour urine calcium level 25-Hydroxyvitamin D level Ionized calcium level
In patients with primary hyperparathyroidism who are undergoing parathyroidectomy surgery, identifying and correcting vitamin D deficiency is important to avoid postoperative hypocalcemia, which occurs due to rapid flux of serum calcium into bone (hungry bone syndrome). The most appropriate test to perform next for this patient is measurement of her 25-hydroxyvitamin D level. Vitamin D deficiency is common in patients with primary hyperparathyroidism (HPT) due to increased conversion of 25-hydroxyvitamin D to 1,25-dihydroxyvitamin D. Supplementation of vitamin D in patients with HPT has been shown to reduce parathyroid hormone levels, decrease bone turnover, and improve bone mineral density. Identifying and treating vitamin D deficiency perioperatively helps manage transient hypocalcemia, which routinely occurs after parathyroidectomy and especially in severe HPT where high bone turnover (as evidence by an elevated alkaline phosphatase) portends hungry bone syndrome. Due to increased conversion, 1,25-dihydroxyvitamin D levels are frankly elevated in most instances of hyperparathyroidism and even in patients who are vitamin D deficient. It is not a useful test to identify and manage vitamin D deficiency in any circumstance and could be falsely reassuring in patients with HPT. Given kidney excretion of calcium is the dominant mechanism by which hypercalcemia is corrected, urine calcium excretion can be assumed to be high in severe hypercalcemia with the exception of patients with severe acute kidney injury. Although guidelines suggest routine assessment of 24-hour urine calcium in the evaluation of HPT to exclude familial hypocalciuric hypercalcemia, the need for such screening primarily occurs when parathyroid hormone and calcium levels are mildly elevated. Additionally, a clinical diagnosis has already been established for this patient making 24-hour urine calcium measurement unnecessary. Ionized calcium is the best test to assess the state of calcium homeostasis and is recommended by some experts when managing HPT. However, it is primarily of use when evaluating and managing hypocalcemia, especially when assumptions cannot be made regarding serum protein concentrations and blood pH.
A 38-year-old man comes to the office to discuss treatment of his recently diagnosed multiple sclerosis. The patient also has diabetes mellitus and nonalcoholic steatohepatitis. Medications are metformin and atorvastatin. On physical examination, vital signs are normal; BMI is 30. All remaining physical examination findings are unremarkable. A T2-weighted MRI of the brain obtained just before diagnosis shows three hyperintense lesions in the periventricular white matter, a hyperintense lesion in the pons, and three juxtacortical hyperintense lesions. The pontine lesion was enhanced with administration of intravenous contrast. Which of the following is the most appropriate treatment? Fingolimod Glatiramer acetate Interferon beta Natalizumab
Interferon beta preparations or glatiramer acetate are considered first-line agents for relapsing-remitting multiple sclerosis, given their favorable risk profiles; glatiramer acetate is preferred in patients with liver disease. This patient should receive glatiramer acetate. He has newly diagnosed multiple sclerosis (MS) and should begin a disease-modifying therapy. Definitive treatment of MS relies less on addressing relapses as they occur than on preventing relapse occurrence (and the associated accrual of disability) in the first place. This prevention is achieved with MS disease-modifying therapies, a series of immunomodulatory or immunosuppressive medications that have been shown to reduce the risk of relapse, disability progression, and new lesion formation on MRI. Several disease-modifying therapies have been approved by the FDA for use in relapsing-remitting MS, each differing in their route of administration, mechanism of action, and potential adverse effects. Generally, most physicians recommend self-injection medications (one of the interferon beta preparations or glatiramer acetate) as first-line agents, given their favorable risk profiles. Glatiramer acetate, a copolymer of four amino acids, is administered daily or several times weekly by subcutaneous injection. Glatiramer acetate and high-dose interferon beta formulations exhibit similar reductions in relapse rates compared with placebo and are equivalent in head-to-head studies. In choosing which therapy is most appropriate to recommend, the clinician should consider all comorbid conditions. Given the patient's history of liver disease, the most appropriate treatment is glatiramer acetate. This medication has no known adverse effects on liver function. Liver dysfunction is a potential adverse effect of fingolimod, the interferon beta preparations, and natalizumab. Therefore, none of these drugs would be the best choice for this patient. Additionally, natalizumab is recommended for patients who have not responded to previous disease-modifying therapy. Because this patient has not yet been treated for his MS, use of natalizumab as a first-line agent would be inappropriate.
A 22-year-old man is evaluated in the hospital after sustaining a traumatic brain injury in a motorcycle accident 4 hours earlier. The patient lost consciousness briefly at the scene but was wearing a helmet. He is now awake and reports a severe headache, nausea, vertigo, left-ear tinnitus and deafness, and rhinorrhea when he sits upright. On physical examination, temperature is 38.1 °C (100.6 °F), blood pressure is 110/60 mm Hg, pulse rate is 104/min, and respiration rate is 20/min. Oxygen saturation is 95% with the patient breathing ambient air. Right hemotympanum and deafness are noted. Bruising is shown. Results of laboratory studies show a normal complete blood count, comprehensive metabolic profile, and urinalysis; a urine drug screen is positive for opiates. A noncontrast head CT scan shows a small right temporal bone fracture but no intracranial hemorrhage. Chest radiographs and a cervical spine CT scan are normal. Which of the following is the most appropriate initial treatment? Acetaminophen Ceftriaxone Dexamethasone Naloxone Norepinephrine
In patients with severe head injury, fever must be controlled aggressively with an agent such as acetaminophen. This patient should be treated with acetaminophen. The patient has symptoms and signs of a severe head injury. Severe headache combined with nausea and focal neurologic symptoms, such as tinnitus and hearing loss, raises concerns of a major cranial injury. Bilateral periorbital bruising ("raccoon eyes"), mastoid bruising ("Battle sign"), and hemotympanum suggest the presence of a basilar skull fracture. Cerebrospinal fluid (CSF) rhinorrhea, epistaxis, and cranial nerve palsies also can be noted with these fractures, and fractures of the temporal bone occur in 75% of affected patients. Maintenance of an arterial PO2 level of greater than 60 mm Hg (8.0 kPa) and a systolic blood pressure of greater than 90 mm Hg have been shown to improve outcomes in these patients. Fever worsens outcomes after stroke and possibly after severe head injury, most likely by promoting secondary brain injury, and acetaminophen is the most appropriate treatment at this time. Prophylactic antibiotics, such as ceftriaxone, are inappropriate for this patient. Fever is a common complication of severe head injury and should be managed aggressively with antipyretic agents. There is no evidence of focal infection or sepsis. Although rhinorrhea in this patient most likely represents CSF drainage, no evidence supports the use of antibiotics without evidence of meningitis. Dexamethasone is contraindicated in severe head injury because glucocorticoids have been shown to worsen its prognosis. Naloxone is inappropriate for this patient. Although the urine drug screen was positive for opiates, there were no signs of central nervous system or respiratory depression. Naloxone also can worsen nausea and dizziness and occasionally cause sedation. Norepinephrine is not required in this patient. Although blood pressure normalization in the presence of significant systolic hypotension (<90 mm Hg) has been shown to improve outcomes in severe closed head injury, this patient's systolic blood pressure does not require vasopressor support.
A 74-year-old man is evaluated for severe chronic shortness of breath. Medical history is significant for New York Heart Association functional class III heart failure and severe COPD. He was hospitalized 3 weeks ago for an exacerbation of his COPD. He has returned to his baseline oxygen requirements, but his continued shortness of breath is a significant impediment to his quality of life. The patient's goal is comfort. He does not desire any additional interventions for his heart failure or COPD. Medications are tiotropium, fluticasone propionate/salmeterol, albuterol, amlodipine, lisinopril, hydrochlorothiazide, and oxygen by nasal cannula. On physical examination, the patient is comfortable at rest but develops dyspnea while ambulating, with associated anxiety. Temperature is 36.9 °C (98.4 °F), blood pressure is 124/68 mm Hg, pulse rate is 98/min, and respiration rate is 32/min. Oxygen saturation is 93% breathing 4 L/min of oxygen by nasal cannula. Pulmonary examination reveals distant breath sounds and a prolonged expiratory phase; the lungs are otherwise clear to auscultation. The estimated central venous pressure is 6 cm H2O. Cardiac examination reveals an S4 but is otherwise normal. There is no peripheral edema. Chest radiograph shows evidence of hyperinflation but no signs of heart failure, pneumonia, or pneumothorax. Which of the following is the most appropriate treatment of this patient's dyspnea? Oral furosemide Oral hydromorphone Oral lorazepam Nebulized morphine
In patients with severe refractory dyspnea, appropriately dosed oral opioids are first-line therapy for symptomatic relief. The most appropriate treatment is oral hydromorphone. This patient has severe chronic dyspnea that is refractory to maximal therapy for his underlying heart failure and COPD. Oral opioids, dosed appropriately, have been found to be both safe and efficacious in the treatment of dyspnea. Treatment efficacy is thought to be related to modulation of shared neural structures that are involved in the pathogenesis of both pain and dyspnea, as there are numerous μ-opioid receptors throughout the respiratory centers in the central nervous system. No evidence suggests that one opioid is superior to another in the treatment of dyspnea, and agent selection should be based on individual patient considerations, such as avoidance of morphine products in patients with reduced kidney function. Furosemide would be a reasonable selection in a patient with volume overload as the driver of dyspnea; however, this patient has no evidence to suggest this cause (such as an elevated central venous pressure, an S3, crackles, or peripheral edema). Benzodiazepines, such as lorazepam, are effective for short-term control of anxiety symptoms. However, this patient's anxiety is a result of his shortness of breath, and as such, treatment should focus on relieving the dyspnea, not on treating the associated anxiety. For this reason, benzodiazepines should be considered only when adjunctive therapy is indicated. Although systemic opioids have shown clear benefit in treating refractory dyspnea, inhaled opioids, such as nebulized morphine, have not shown significant efficacy in several placebo-controlled trials.
A 69-year-old man is evaluated during a follow-up visit. He initially presented with a 3-month history of chest pressure and dyspnea that occurred primarily with exertion. Despite maximal medical therapy, his symptoms have not abated and adversely affect his quality of life. Medical history is significant for type 2 diabetes mellitus, hypertension, and hyperlipidemia. Medications are low-dose aspirin; metformin; long-acting nitroglycerin; and optimal doses of metoprolol, lisinopril, ranolazine, and simvastatin. On physical examination, blood pressure is 128/73 mm Hg, and pulse rate is 60/min. Cardiac examination reveals a normal S1 and S2 without an S3 or S4. There is no lower extremity edema. Coronary angiogram is significant for a normal left main coronary artery and a 90% stenosis in the proximal left anterior descending artery resulting from ulcerated plaque. Which of the following is the most appropriate management? β-Carotene Cardiac rehabilitation Percutaneous coronary intervention Ticagrelor
In patients with stable angina, diagnostic angiography and percutaneous coronary intervention are reserved for patients with refractory angina symptoms while receiving optimal medical therapy, those who are unable to tolerate optimal medical therapy owing to side effects, or those with high-risk features on noninvasive exercise and imaging tests. This patient with diabetes mellitus and single-vessel coronary artery disease should undergo revascularization with percutaneous coronary intervention (PCI) or coronary artery bypass grafting. PCI has not been shown to be superior to optimal medical therapy in patients with stable angina for reduction of cardiovascular endpoints, such as mortality and myocardial infarction. However, PCI has been associated with improvement in quality of life by reducing the severity and frequency of angina. Current guidelines recommend that diagnostic angiography and PCI be reserved for patients with refractory symptoms while receiving optimal medical therapy, those who are unable to tolerate optimal medical therapy owing to side effects, or those with high-risk features on noninvasive exercise and imaging tests. This patient has 90% stenosis of the proximal left anterior descending artery and refractory symptoms; therefore, PCI is a reasonable therapeutic option. The use of β-carotene, selenium, chromium, vitamin C, vitamin E, and estrogen has not been associated with improved cardiovascular outcomes or relief of symptoms and is not recommended in patients with ischemic heart disease. Cardiac rehabilitation may be appropriate after revascularization occurs; however, this patient should first undergo revascularization to treat his crescendo angina symptoms. Aspirin is associated with a decreased risk for myocardial infarction, stroke, and cardiovascular death in patients with coronary artery disease. Aspirin doses of 81 mg to 162 mg daily are recommended in all patients with established coronary artery disease unless contraindicated. In patients allergic to aspirin, clopidogrel is recommended as an alternative. The use of newer antiplatelet agents (prasugrel, ticagrelor) as monotherapy has not been tested in patients with stable angina. Dual antiplatelet therapy for chronic angina in the absence of stent placement is not recommended.
A 78-year-old man is evaluated for exertional dyspnea. He was previously asymptomatic, but 4 months ago he began having shortness of breath during moderate levels of activity. The dyspnea dissipates with rest. He is otherwise healthy and takes no medications. On physical examination, temperature is normal, supine blood pressure is 132/80 mm Hg, pulse rate is 80/min, and respiration rate is 22/min. The lungs are clear to auscultation. The carotid upstroke is delayed. There is a grade 3/6 late-peaking systolic murmur best heard at the base of the heart with radiation to both carotid arteries. S1 is normal; the aortic component of S2 is diminished. The remainder of the examination is unremarkable. An echocardiogram demonstrates a left ventricular ejection fraction of 65%. There is moderate aortic stenosis, with a mean gradient of 28 mm Hg and an aortic valve area of 1.5 cm2. Which of the following is the most appropriate next step in management? Cardiac catheterization Surgical aortic valve replacement Transcatheter aortic valve replacement Continued clinical observation
In patients with symptoms of aortic stenosis and discrepancies between the physical examination and echocardiographic findings, the severity of stenosis should be established with cardiac catheterization before aortic valve replacement is performed. The most appropriate next step in management is cardiac catheterization. In patients with symptomatic severe aortic stenosis, surgery with aortic valve replacement is indicated as a life-saving therapy. However, accurate measures of the severity of stenosis are needed to ensure that symptoms are caused by valve obstruction, rather than concurrent coronary, pulmonary, or other systemic disease. The severity of aortic stenosis is determined by Doppler echocardiography and sometimes cardiac catheterization. In some patients, the Doppler echocardiogram may underestimate severity because of the angle-dependent nature of the study. An invasive hemodynamic study is indicated in symptomatic patients being considered for surgery when there are discrepancies between the findings on physical examination and the echocardiographic results. This patient's exertional dyspnea and physical examination findings (carotid tardus, diminished aortic component of S2, late-peaking systolic murmur) are consistent with severe aortic stenosis; however, the echocardiogram reveals findings of only moderate aortic stenosis. Therefore, further evaluation with cardiac catheterization is needed. Surgical or transcatheter aortic valve replacement is not indicated in this patient until there is certainty that the aortic valve lesion is severe and can account for the patient's symptoms. Identification of symptomatic aortic stenosis is crucial; in the absence of symptoms, patients with aortic stenosis—even severe aortic stenosis—have a low risk for mortality, with an estimated risk for sudden death of less than 1%. Although variable, symptoms of heart failure, angina, or syncope generally begin once the valve area is below 1.0 cm2. Once symptoms develop, prognosis is poor without valve replacement, with an average survival of less than 10% over the next 2 to 3 years. Therefore, continued clinical observation is not the best option for this patient.
A 32-year-old man is evaluated for a 1-week history of jaundice and pruritus. One month earlier, he completed treatment for sinusitis with amoxicillin-clavulanate. His sinusitis symptoms resolved with therapy. On physical examination, vital signs are normal. BMI is 26. Jaundice is noted. Excoriations are seen on the extremities. The abdominal examination is normal. Laboratory studies: INR 1.0 Albumin 3.8 g/dL (38 g/L) Alkaline phosphatase 580 U/L Alanine aminotransferase 42 U/L Aspartate aminotransferase 38 U/L Total bilirubin 8.6 mg/dL (147.1 µmol/L) Hepatitis A, B, and C virus and Epstein-Barr virus studies show no evidence of current or previous infection. Testing for antinuclear antibody and anti-smooth muscle antibody is negative. The ceruloplasmin level is normal. Results of abdominal ultrasonography are normal, with no evidence of biliary dilation, gallstones, or hepatic parenchymal abnormalities. Which of the following is the most appropriate next step in management? Endoscopic retrograde cholangiopancreatography Liver biopsy Prednisone administration Continued observation
In patients with well-preserved liver function, drug-induced liver injury should be managed with discontinuation of the offending medication and observation until resolution of symptoms occurs. Continued observation is the most appropriate management of this patient. Drug-induced liver injury is a rare adverse reaction to medication that can result in jaundice, liver failure, and, potentially, death. This patient meets the criteria for diagnosis, which requires a history of drug or supplement exposure within 6 to 12 months, a biochemical pattern that fits the hepatotoxicity profile of the causative agent (in this case, cholestatic for amoxicillin-clavulanate), improvement after drug removal (dechallenge), and the absence of underlying liver or biliary diseases. The most common medications associated with drug-induced liver injury are antibacterial agents (especially amoxicillin-clavulanate) as well as herbal and dietary supplements. In this patient with drug-induced liver injury and well-preserved liver function, the offending medication has already been discontinued, and observation until resolution of symptoms occurs is the best course of action. The prognosis of drug-induced liver injury is generally good, with 70% of patients recovering without needing hospitalization and 90% recovering without developing acute liver failure. Endoscopic retrograde cholangiography (ERCP) is used to treat biliary obstruction. In this setting, ultrasound has confirmed the absence of biliary dilation, and therefore, ERCP would not be expected to provide benefit. A liver biopsy is rarely necessary in patients with drug-induced liver injury but is helpful in cases of uncertainty or suspected drug-induced autoimmune hepatitis. Zone 3 necrosis and eosinophilia are classic histologic findings in drug-induced liver injury, but other findings may include hepatitis, cholestasis, steatosis, or granulomas. This patient has a classic presentation, and a cholestatic biochemical profile and negative antibody tests make autoimmune hepatitis unlikely. No specific antidotes are available for idiosyncratic drug-induced liver injury. The administration of glucocorticoids like prednisone is not indicated unless there is suspicion for drug hypersensitivity reaction. The absence of a morbilliform rash, fever, facial swelling, lymphadenopathy, and substantially elevated aminotransferase levels make a hypersensitivity reaction unlikely and prednisone unnecessary in this patient.
A 38-year-old man is evaluated during a follow-up visit for elevated blood pressure found for the first time during his last visit. He reports back pain of several weeks' duration after an episode of heavy lifting at work. History is also notable for seasonal allergies. He currently takes ibuprofen daily for the back pain and loratadine as needed for allergies. On physical examination, the patient is well developed and muscular, and in no apparent distress. The average of three blood pressure measurements is 139/84 mm Hg, and pulse rate is 52/min; other vital signs are normal. BMI is 26. The remainder of the examination is normal. Office electrocardiogram is normal. In addition to follow-up in 1 month, which of the following is the most appropriate management? Begin amlodipine Begin hydrochlorothiazide Discontinue ibuprofen Discontinue loratadine
Many medications, such as NSAIDS, can result in reversible elevations in blood pressure; discontinuation of the drug and a reassessment of blood pressure 1 month later are necessary to confirm a return to normal blood pressure measurement. The most appropriate management is to discontinue ibuprofen and measure the blood pressure (BP) in 1 month. The American College of Cardiology/American Heart Association BP guideline defines stage 1 hypertension as a systolic BP of 130 to 139 mm Hg or diastolic BP of 80 to 89 mm Hg. This change is based on epidemiologic studies that indicate systolic BP >115 mm Hg and diastolic BP >75 mm Hg are associated in a linear fashion with cardiovascular events. In the initial assessment of a patient with elevated BP, before a diagnosis of hypertension is made, it is imperative that a complete history with a list of prescription, nonprescription (including complementary and alternative medications such as herbals), and illicit drugs is elicited. NSAIDs are one of the medication classes that can result in reversible elevations in BP, the mechanism for which is increased sodium retention. Therefore, ibuprofen, an NSAID, should be discontinued and BP remeasured. If the patient has ongoing pain, an alternative pain management strategy that does not result in BP elevation, such as nondrug interventions, topical analgesics (if appropriate), or acetaminophen, should be prescribed because pain can also result in BP elevation. This patient's elevated BP would need to be confirmed 1 month after discontinuation of ibuprofen before antihypertensive medications such as amlodipine or hydrochlorothiazide are started in this otherwise healthy young individual. Antihistamines such as loratadine generally do not result in BP elevation; therefore, loratadine does not need to be discontinued in this patient.
A 54-year-old man undergoes follow-up evaluation for deep venous thrombosis after a left hip replacement 2 months ago. He is feeling well, his swelling has improved, and he reports no pain. His only medication is rivaroxaban. On physical examination, vital signs are normal. The examination is otherwise unremarkable. Which of the following is the most appropriate management? Completion of 3-month course of anticoagulation D-dimer measurement Extended anticoagulation Thrombophilia evaluation
In provoked deep venous thrombosis with reversible risk factors, 3 to 6 months of anticoagulation is recommended. The most appropriate management is to continue anticoagulation for a 3-month course. Decisions on anticoagulation therapy duration depend on the recurrent venous thromboembolism (VTE) risk without anticoagulation, the bleeding risk with anticoagulants, and the patient's preference. In patients with proximal leg deep venous thrombosis (DVT) or pulmonary embolism, the cumulative VTE recurrence risk after discontinuation of anticoagulation is (1) 1% after 1 year and 3% after 5 years for VTE provoked by major surgery; (2) 5% after 1 year and 15% after 5 years for VTE provoked by a nonsurgical reversible risk factor; and (3) 10% after 1 year and 30% after 5 years for unprovoked VTE. This patient experienced a provoked DVT after major surgery. In provoked thrombosis with reversible risk factors, guidelines recommend 3 to 6 months of anticoagulation. Because of the high risk of recurrence with discontinuation of anticoagulant therapy, an unprovoked VTE is a consideration for extended anticoagulant therapy. When extended (long-term) anticoagulation therapy is chosen, the risks, benefits, and burden of long-term therapy must be re-evaluated periodically. This patient's DVT was provoked by surgery; therefore, extended anticoagulation is not a reasonable option. Identifying an inherited thrombophilia often does not change treatment decisions in a patient with VTE. Recent guidelines recommend against routine thrombophilia testing because identification of inherited abnormalities does not alter the duration of recommended anticoagulation or reliably predict risk of VTE recurrence. Finally, the American Society of Hematology's Choosing Wisely Campaign specifically recommends against thrombophilia testing in adults with VTE that occurs in the setting of a major transient risk factor, such as surgery, prolonged immobility, or trauma. In patients with unprovoked DVT, a common strategy has been to measure D-dimer levels 2 weeks after discontinuation of a 3-month course of anticoagulant therapy. Patients with elevated D-dimer levels were considered for further anticoagulation. However, although elevated levels of D-dimer can identify persons at risk, this test has low specificity, and a normal D-dimer level cannot safely rule out the possibility of future recurrence, thus limiting the utility of this testing strategy. Furthermore, this patient had a provoked DVT, which is another reason not to perform follow-up D-dimer testing.
A 45-year-old woman is evaluated for increased pain and swelling in her left knee and right ankle. She has a 5-year history of psoriatic arthritis and psoriasis treated with methotrexate, which previously improved her joint and skin symptoms. She also takes folic acid. On physical examination, vital signs are normal. There is an erythematous, flaky patch on the left elbow. Nail pitting is present. The left knee has a moderate effusion and is warm without erythema, with discomfort on range of motion. The right ankle is swollen anteriorly, with discomfort on range of motion in flexion and extension. Laboratory studies show an erythrocyte sedimentation rate of 30 mm/h. Plain radiographs of the left knee show mild medial and lateral joint space narrowing. Aspiration of the left knee shows a leukocyte count of 7500/µL (7.5 × 109/L), with 50% neutrophils. Which of the following is the most efficacious medication to add to this patient's treatment regimen? Abatacept Hydroxychloroquine Infliximab Rituximab
In recalcitrant psoriatic arthritis, the combination of methotrexate and a tumor necrosis factor α inhibitor has shown efficacy in managing joint symptoms and slowing the progression of radiographic damage, including joint space narrowing and erosions. The most appropriate medication to add to this patient's treatment regimen is a tumor necrosis factor (TNF)-α inhibitor such as infliximab. In patients with psoriatic arthritis who have uncontrolled disease while taking methotrexate at a dose of 25 mg weekly, the addition of a TNF-α inhibitor is indicated. Randomized controlled trials have established the efficacy of the combination of methotrexate and TNF-α inhibitors for reducing symptoms, restoring function, and limiting joint damage. Other biologics shown to be effective include ustekinumab and secukinumab. Abatacept is a biologic disease-modifying agent that targets the T-cell costimulation pathway and is used in the treatment of rheumatoid arthritis. A single randomized controlled trial has shown that abatacept could be efficacious in psoriatic arthritis. However, in a patient with psoriatic arthritis in whom a TNF-α inhibitor has not been used, abatacept is not the appropriate next choice in management due to limited experience in these patients. Hydroxychloroquine is a long-acting anti-inflammatory medication that interferes with the innate immune system inflammatory response and is used to treat rheumatoid arthritis, systemic lupus erythematosus, and dermatomyositis. It has been reported to be associated with an increased incidence of severe cutaneous reactions in psoriasis and is not the best choice. Rituximab is a monoclonal antibody that targets CD20+ B cells used in the treatment of rheumatoid arthritis, vasculitis, and various malignancies. It has not been used in the treatment of psoriatic arthritis.
A 49-year-old woman is evaluated in the emergency department for a 12-hour history of new-onset right-sided visual loss. The patient has type 2 diabetes mellitus treated with metformin. On physical examination, blood pressure is 138/72 mm Hg, and pulse rate is 102/min and irregularly irregular. Cardiac auscultation reveals no carotid bruits or cardiac murmurs. A visual field deficit is present on the right side of both eyes. No weakness or sensory loss is noted. A CT scan of the head shows a hypodensity in the left occipital lobe, and a carotid duplex ultrasound shows less than 60% stenosis of both internal carotid arteries with normal vertebral artery flow. An electrocardiogram shows atrial fibrillation. Which of the following medications should be administered now? Apixaban Aspirin Clopidogrel Intravenous heparin
In the absence of thrombolytic therapy for cardioembolic stroke, short-term therapy with aspirin reduces the risk of stroke at 2 weeks. This patient should be treated with aspirin. She most likely has a cardioembolic stroke related to atrial fibrillation. She is beyond the treatment window for intravenous thrombolysis and is not a candidate for intra-arterial therapy because of the location of the lesion in the posterior circulation and already demonstrated infarction. Therefore, medical therapy is indicated for preventing recurrent stroke. Only aspirin and heparinoids have been studied for their effectiveness in the acute stroke setting, and only aspirin has been shown to reduce the risk of stroke at 2 weeks. Therefore, aspirin monotherapy is a reasonable first-line antiplatelet regimen for secondary stroke prevention in the acute setting. For stable patients with small strokes, long-term warfarin therapy is typically administered 24 hours after hospitalization to reduce the risk of hemorrhagic transformation. Hemorrhagic transformation is more likely to occur in larger infarcts (those involving more than one third of hemispheric volume) and in infarcts with a cardioembolic cause. Warfarin is often withheld for 2 weeks in the setting of large strokes, strokes that have undergone hemorrhagic transformation, or uncontrolled hypertension. The use of novel oral anticoagulants, including apixaban, has not been established as effective in the acute setting after a stroke. Apixaban may be a reasonable long-term option for stroke prevention at hospital discharge. Clopidogrel has not been studied as monotherapy for acute ischemic stroke and is currently not recommended. Intravenous heparin is not effective in reducing the risk of recurrent stroke within the acute hospitalization period in patients with all stroke subtypes, including ischemic strokes with a cardioembolic cause. Intravenous heparin can be considered in patients with a high short-term risk of recurrent ischemic stroke, such as those with mechanical heart valves, but the benefit of this approach must be weighed against the risk of hemorrhagic conversion.
A 55-year-old woman is evaluated after a recent hospital admission for ischemic stroke. She presented with mild right-sided hemiparesis, but it completely resolved after the second hospital day. An electrocardiogram, transthoracic echocardiogram, and carotid artery ultrasound were normal. A subsequent transesophageal echocardiogram was also normal. She has a 10-year history of hypertension treated with hydrochlorothiazide. On physical examination, vital signs are normal. Cardiac examination reveals a regular rate and rhythm. Neurologic examination is nonfocal. Hand grip strength, arm flexion, and arm extension are 5/5 on the right and left sides. There is no edema. A 12-lead electrocardiogram demonstrates normal sinus rhythm. Which of the following is the most appropriate diagnostic testing option? Antiphospholipid antibody testing Cardiac catheterization30-Day event-triggered loop recording 48-Hour ambulatory electrocardiographic monitoring
In the evaluation of cryptogenic stroke, noninvasive ambulatory electrocardiographic (ECG) monitoring for 30 days improves the detection of atrial fibrillation by fivefold compared with short-term ECG monitoring. The most appropriate diagnostic testing option is 30-day event-triggered loop recording. This patient had a cryptogenic stroke given the absence of an apparent vascular, structural, or thromboembolic cause for her ischemic event. Among patients aged 55 years or older who have a cryptogenic ischemic neurologic event, such as a stroke or transient ischemic attack, occult intermittent atrial fibrillation is thought to be present in up to 25% of cases. These patients have a high risk for recurrent stroke, and efforts should be made to detect and treat atrial fibrillation for stroke prevention. Noninvasive ambulatory electrocardiographic (ECG) monitoring for 30 days improves the detection of atrial fibrillation by fivefold compared with short-term ECG monitoring. Patients who have premature atrial contractions and other findings of ectopy on short-term telemetry may be more likely to have paroxysmal atrial fibrillation. If noninvasive ambulatory ECG monitoring for 30 days is inconclusive, implantation of a cardiac monitor (loop recorder) is reasonable to optimize detection of silent atrial fibrillation. Patients who undergo longer-term ECG monitoring and are subsequently diagnosed with atrial fibrillation are more likely to receive appropriate secondary prevention stroke therapy with oral anticoagulants. Antiphospholipid antibodies are acquired autoantibodies against phospholipids and phospholipid-binding proteins, such as cardiolipin and β2-glycoprotein I. In vitro, they can prolong clotting tests, but in vivo, they increase the risk for venous and arterial thrombosis and are associated with increased fetal loss. Testing for autoimmune and hypercoagulable disorders, such as antiphospholipid antibody syndrome, can be considered in young patients with otherwise unexplained stroke. Aside from stroke, this patient does not have any features concerning for a prothrombotic state or autoimmune disease. Therefore, testing for antiphospholipid antibodies is not appropriate. Cardiac catheterization is not indicated in this patient. Because she had a normal transesophageal echocardiogram, there is no concern for a structural abnormality, such as an intracardiac shunt. Short-term ECG monitoring (24-48 hours) with an ambulatory ECG monitor is inferior to extended ECG monitoring in the detection of paroxysmal atrial fibrillation.
A 69-year-old man is evaluated during a routine visit. He is asymptomatic. Medical history is remarkable for hypertension, atrial fibrillation, and type 2 diabetes mellitus. He has a 25-pack-year smoking history but quit smoking 3 years ago. Medications are amlodipine, metoprolol, rivaroxaban, and atorvastatin. Physical examination, including vital signs, is normal. His estimated 10-year risk for atherosclerotic cardiovascular disease (ASCVD) according to the Pooled Cohort Equations is 14.6%. Which of the following is most appropriate for primary prevention of ASCVD in this patient? Add aspirin Add fish oil Switch rivaroxaban to aspirin No further intervention
In the primary and secondary prevention of cardiovascular events, the addition of aspirin to long-term anticoagulation is associated with significantly increased bleeding events and is not routinely recommended.
A 69-year-old man is evaluated during a routine visit. He is asymptomatic. Medical history is remarkable for hypertension, atrial fibrillation, and type 2 diabetes mellitus. He has a 25-pack-year smoking history but quit smoking 3 years ago. Medications are amlodipine, metoprolol, rivaroxaban, and atorvastatin. Physical examination, including vital signs, is normal. His estimated 10-year risk for atherosclerotic cardiovascular disease (ASCVD) according to the Pooled Cohort Equations is 14.6%. Which of the following is most appropriate for primary prevention of ASCVD in this patient? Add aspirin Add fish oil Switch rivaroxaban to aspirin No further intervention
In the primary and secondary prevention of cardiovascular events, the addition of aspirin to long-term anticoagulation is associated with significantly increased bleeding events and is not routinely recommended. No further intervention is the most appropriate management of this patient. Although he has a 10-year risk for atherosclerotic cardiovascular disease (ASCVD) of 14.6%, he is at high risk for bleeding with the addition of aspirin therapy. The U.S. Preventive Services Task Force recommends low-dose aspirin for the primary prevention of ASCVD and colorectal cancer in adults aged 50 to 59 years with a 10-year ASCVD risk of 10% or higher who do not have an increased risk for bleeding, have a life expectancy of at least 10 years, and are willing to take low-dose aspirin daily for at least 10 years. Patients aged 60 to 69 years may also benefit from aspirin; however, the net benefit is smaller because of the increased risk for bleeding in this population. In addition to increasing age, male sex and use of anticoagulants or NSAIDs increase the risk for bleeding. Many patients with cardiovascular disease who are eligible for secondary prevention with aspirin therapy also require long-term oral anticoagulant therapy for atrial fibrillation. In these patients, the addition of aspirin to anticoagulant therapy provides some additional protection against cardiovascular events, but the risk for major bleeding is significantly increased. Aspirin is not generally recommended in these patients, and the American College of Cardiology/American Heart Association specifically recommend against the use of low-dose aspirin for primary prevention in patients at increased risk for bleeding, including those on anticoagulant therapy. Although this patient's risk for ASCVD is higher than 10%, he is already receiving anticoagulant therapy with rivaroxaban; therefore, the potential benefits of aspirin therapy for primary prevention of ASCVD are likely outweighed by the increased risk for bleeding. High doses of fish oil increase bleeding time in vitro by suppressing platelet-activating factor, but this mechanism has not been associated with higher rates of clinical bleeding, even when the supplement is combined with aspirin or warfarin. However, a 2018 systematic review concluded that omega-3 fatty acid supplementation does not reduce heart disease, stroke, or death; therefore, fish oil supplementation cannot be recommended. Switching rivaroxaban to low-dose aspirin is not recommended because this patient is at high risk for atrial fibrillation-related stroke. For this patient, anticoagulant therapy is significantly superior to aspirin in reducing his risk for stroke.
A 42-year-old man is admitted to the hospital with an acute change in mental status and fever of 2 days' duration. Medical history is noncontributory, and he takes no medications. On physical examination, temperature is 38.2 °C (100.8 °F), blood pressure is 108/70 mm Hg, pulse rate is 104/min, and respiration rate is 18/min. Oxygen saturation is 96% breathing ambient air. He is agitated and disoriented to place and time. Petechiae are noted on his shins. The remainder of the examination is normal. Laboratory studies: Haptoglobin 20 mg/dL (200 mg/L) Hemoglobin 10.2 g/dL (102 g/L) Leukocyte count 9800/µL (9.8 × 109/L) Platelet count 44,000/µL (44 × 109/L) Reticulocyte count 6.8% of erythrocytes Creatinine 1.4 mg/dL (123.8 µmol/L) Lactate dehydrogenase 1600 U/L The direct antiglobulin (Coombs) test is negative. Therapy should be immediately initiated pending results of which of the following studies? ADAMTS13 activity Coagulation studies Peripheral blood smear Stool culture and testing for Shiga toxin
In the proper clinical setting, a peripheral blood smear showing schistocytes in a patient with thrombocytopenia and hemolytic anemia establishes a presumptive diagnosis of thrombotic thrombocytopenic purpura and confirms the need to initiate early, life-saving therapy. The finding of schistocytes on the peripheral blood smear is all that is needed to initiate therapy for presumed thrombotic thrombocytopenic purpura (TTP). This patient's clinical features of fever, change in mental status, thrombocytopenia, and features of hemolytic anemia (low haptoglobin level, elevated lactate dehydrogenase level, elevated reticulocyte count) suggest TTP. The presence of schistocytes on the peripheral blood smear will confirm the diagnosis of a microangiopathic hemolytic anemia (MAHA) and a presumptive diagnosis of TTP and allows for early treatment. Prompt diagnosis is critical because TTP is fatal in 90% of patients without therapy. Patients require emergent treatment with plasma exchange. The most common cause of TTP results from a deficiency in the protease ADAMTS13, which cleaves the high-molecular-weight multimers of von Willebrand factor (vWF). The decrease in ADAMTS13 activity leads to accumulation of clumps of ultra-large-molecular-weight vWF multimers, which bind to masses of platelets leading to microvascular occlusion and thrombocytopenia. If the peripheral blood smear supports the diagnosis of TTP, testing for ADAMTS13 activity is important to confirm the diagnosis and establish the autoimmune cause. However, the turn-around time for this test result is long, and therapy should not be delayed while awaiting results. This patient's clinical history, laboratory findings, and schistocytes on the peripheral blood smear are sufficient to initiate life-saving treatment. Coagulation studies are almost always normal in patients with TTP. Tissue ischemia may result in laboratory findings consistent with disseminated intravascular coagulation, but neither normal nor abnormal coagulation studies affect the decision to initiate early therapy. In a patient with thrombocytopenia, MAHA, and bloody diarrhea, stool cultures and toxin testing for Shiga toxin or Shiga toxin-producing organisms are indicated to differentiate TTP from hemolytic uremic syndrome. Because this patient does not have diarrhea, stool studies are not indicated, and results do not dictate when to initiate therapy.
A 54-year-old man is evaluated for a 5-month history of mid lower back pain. The pain radiates from the center of his back to his flanks and abdomen. He has intermittent numbness in the areas of the pain. He also reports a 2.3-kg (5-lb) weight loss over the past 3 months. He emigrated from India to the United States 20 years ago and travels to India yearly. He has tried ibuprofen for the pain without relief and takes no other medications. On physical examination, temperature is 38.2 °C (100.8 °F); other vital signs are normal. Examination of the spine reveals kyphosis, tenderness to palpation, and paraspinal muscle tightness around T10. Decreased sensation to light touch along the T10 dermatome is noted. Chest radiograph shows calcified granuloma in the right upper lobe. Thoracic spine radiograph shows marked kyphosis and erosive changes and collapse of the anterior portions of the T10 and T11 vertebral bodies, destruction of the T10-11 disk space, and surrounding fusiform paravertebral swelling. Which of the following is the most likely diagnosis? Herniated intervertebral disk Multiple myeloma Osteoporosis Tuberculous vertebral osteomyelitis
In tuberculous vertebral osteomyelitis, the lower thoracic spine is the most frequently involved segment; common symptoms are back pain, fever, weight loss, and neurologic abnormalities.
A 54-year-old man is evaluated for a 5-month history of mid lower back pain. The pain radiates from the center of his back to his flanks and abdomen. He has intermittent numbness in the areas of the pain. He also reports a 2.3-kg (5-lb) weight loss over the past 3 months. He emigrated from India to the United States 20 years ago and travels to India yearly. He has tried ibuprofen for the pain without relief and takes no other medications. On physical examination, temperature is 38.2 °C (100.8 °F); other vital signs are normal. Examination of the spine reveals kyphosis, tenderness to palpation, and paraspinal muscle tightness around T10. Decreased sensation to light touch along the T10 dermatome is noted. Chest radiograph shows calcified granuloma in the right upper lobe. Thoracic spine radiograph shows marked kyphosis and erosive changes and collapse of the anterior portions of the T10 and T11 vertebral bodies, destruction of the T10-11 disk space, and surrounding fusiform paravertebral swelling. Which of the following is the most likely diagnosis? Herniated intervertebral disk Multiple myeloma Osteoporosis Tuberculous vertebral osteomyelitis
In tuberculous vertebral osteomyelitis, the lower thoracic spine is the most frequently involved segment; common symptoms are back pain, fever, weight loss, and neurologic abnormalities. The most likely diagnosis is tuberculous vertebral osteomyelitis (Pott's disease), the most common form of skeletal tuberculosis. Skeletal tuberculosis accounts for 10% to 35% of cases of extrapulmonary tuberculosis. Spinal involvement usually results from the hematogenous spread of Mycobacterium tuberculosis into the cancellous bone tissue of the vertebral bodies, from a primary pulmonary focus or extrapulmonary foci such as the lymph nodes. Predisposing factors for skeletal tuberculosis include previous tuberculosis infection, malnutrition, alcoholism, diabetes mellitus, and HIV infection. Delays in diagnosis are common because the onset of symptoms is insidious, and disease progression is slow. The duration of symptoms prior to diagnosis may range from weeks to years. The lower thoracic spine is the most frequently involved segment; common symptoms are back pain, fever, weight loss, and neurologic abnormalities. Paraplegia is the most devastating complication. Diagnosis is suggested by typical clinical presentation, along with evidence of past exposure to tuberculosis or concomitant visceral tuberculosis, and findings on neuroimaging modalities. Bone tissue or abscess samples from needle or surgical biopsy stained for acid-fast bacilli and/or positive cultures are most helpful in diagnosis. Herniated intervertebral disk usually occurs in the lower lumbar spine (at L4-5 or L5-S1 levels) and typically causes pain and sometimes sensory symptoms radiating down the leg(s). This patient's constitutional symptoms of chills, weight loss, and low-grade fever, as well as image findings of erosion in the vertebral body and paravertebral swelling, are not consistent with herniated intervertebral disk. Multiple myeloma is the most common primary malignant neoplasm of the skeletal system, and the vertebral column is a commonly affected site. The classic radiographic appearance includes multiple, small, well-circumscribed, lytic, punched-out, round lesions that can be found in the skull, pelvis, and spine on skeletal survey, not erosive changes as seen in this patient. Osteoporosis with vertebral compression fracture may cause severe back pain and the thoracolumbar area is a common site, but patients do not typically experience neurologic or constitutional symptoms. Erosive changes with surrounding fusiform paravertebral swelling suggest another diagnosis.
A 48-year-old man is hospitalized with a fever and rigors. He has primary sclerosing cholangitis and ulcerative colitis. His only medication is mesalamine. On physical examination, temperature is 38.6 °C (101.5 °F), blood pressure is 118/75 mm Hg, pulse rate is 95/min, and respiration rate is 18/min. Abdominal examination is notable for right-upper-quadrant tenderness to palpation. The remainder of the examination is normal. Laboratory studies show a leukocyte count of 14,000/µL (14 × 109/L). Blood cultures are pending. A 1-L bolus of intravenous normal saline is administered, and intravenous piperacillin-tazobactam is given. Which of the following is the most appropriate test to perform next? CA 19-9 measurement Endoscopic retrograde cholangiopancreatography IgG4 measurement Percutaneous transhepatic biliary tube placement PET scan
Indications for endoscopic retrograde cholangiopancreatography in patients with primary sclerosing cholangitis are bacterial cholangitis, increasing jaundice, increasing pruritus, or a dominant stricture on imaging. Endoscopic retrograde cholangiopancreatography (ERCP) is the most appropriate next test for this patient. This patient with primary sclerosing cholangitis (PSC) presents with fever, rigors, right-upper-quadrant pain, and leukocytosis, all of which are consistent with bacterial cholangitis. Indications for ERCP in patients with PSC are bacterial cholangitis (as in this patient), increasing jaundice, increasing pruritus, or a dominant stricture seen on imaging. Symptoms of bacterial cholangitis, increasing jaundice, and pruritus can signify strictures that may improve with dilation or stenting, or, alternatively, removing sludge or stone debris in the bile ducts via ERCP. A dominant stricture in a patient with PSC must be evaluated for cholangiocarcinoma by obtaining biliary brushings for cytologic examination, and, if available, fluorescent in situ hybridization to evaluate chromosomal abnormalities. The CA 19-9 level will be elevated in the setting of bacterial cholangitis. The risk for false-positive results makes CA 19-9 measurement inappropriate in this context. CA 19-9 levels can be used as an adjunctive tool in the diagnosis of cholangiocarcinoma, but the diagnosis cannot be made based only on this marker. IgG4 levels should be checked in patients with a new diagnosis of presumed PSC because IgG4 cholangitis is a steroid-responsive condition, whereas PSC is not. Testing for IgG4 does not assist in the management of this patient's cholangitis. A percutaneous transhepatic biliary tube can be employed when ERCP is unsuccessful at traversing a biliary stricture, but because of its invasiveness and inconvenience, it would not be a first-line tool for assessing and treating a patient with bacterial cholangitis. The role of PET in the evaluation and management of cholangiocarcinoma is evolving. However, this is not the test of choice in a patient with bacterial cholangitis because it does not allow for biliary intervention. Furthermore, PET scans can be associated with false-positive results for malignancy in the setting of bacterial cholangitis, and also with false-negative results due to the desmoplastic reaction of cholangiocarcinoma tumors.
A 22-year-old man returns for follow-up evaluation; he was recently diagnosed with HIV infection, and he began antiretroviral therapy 2 weeks ago. He also received influenza vaccination and the 13-valent pneumococcal conjugate vaccine at that time. He reports that he has sex with men. Medical history is notable for previous chlamydia infection and genital warts. Medications are tenofovir, emtricitabine, and dolutegravir. On physical examination, vital signs are normal. A few small lesions consistent with warts are noted on the penis. The examination is otherwise unremarkable. Laboratory studies: CD4 cell count 527/µL Hepatitis A IgG Positive Hepatitis A IgM Negative Hepatitis B surface antibody Negative Hepatitis B surface antigen Positive Which of the following is the most appropriate immunization to be given today? 23-Valent pneumococcal polysaccharide vaccine Hepatitis A vaccine Hepatitis B vaccine Human papillomavirus vaccine
Indications for influenza, tetanus-diphtheria-pertussis, hepatitis A virus, and human papillomavirus vaccines are the same for patients with HIV infection as for the general population. Human papillomavirus (HPV) is the most appropriate immunization for this patient at this time. He was recently diagnosed with HIV infection and has begun antiretroviral therapy. At baseline, his CD4 cell count is normal. He is in an age group for which HPV immunization is recommended, and that recommendation is the same regardless of HIV status. The presence of genital warts does not change the indication for HPV vaccination. He should begin the HPV vaccine series with the first injection today. Indications for influenza, tetanus-diphtheria-pertussis, and hepatitis A virus (HAV) vaccines are also the same for patients with HIV infection as for the general population. Pneumococcal vaccination is important for all persons with HIV infection, regardless of CD4 cell count. As with other immunocompromised persons, patients with HIV should receive the 13-valent conjugate and 23-valent polysaccharide vaccines, in that order. This patient has already received the pneumococcal conjugate vaccine and needs the polysaccharide vaccine, but at least 8 weeks must elapse between these two vaccines to allow for better immune response in this prime-boost strategy. Therefore, giving him the pneumococcal polysaccharide vaccine at this visit would be premature. Serum IgM antibodies to HAV are detectable at the time of symptom onset and remain detectable for approximately 3 to 6 months. Serum IgG antibodies appear in convalescence and remain detectable for decades. The presence of anti-HAV IgG in the absence of anti-HAV IgM indicates past infection or vaccination. This patient does not need vaccination against HAV. Hepatitis B virus (HBV) surface antibody testing is negative, indicating a lack of immunity to HBV; he also has risk factors for HBV that would warrant HBV vaccination. However, the patient tested positive for hepatitis B surface antigen, indicating he already has HBV infection and would not benefit from immunization with the HBV vaccine.
A 55-year-old man is evaluated for an increase in his serum creatinine level. History is significant for hypertension treated with lisinopril for 3 years. He reports no changes or additions to his medication regimen during the past year. On physical examination, the patient is afebrile, blood pressure is 145/92 mm Hg, and pulse rate is 84/min. There is no rash, alopecia, or joint abnormalities. The remainder of the examination is unremarkable. Laboratory studies: Complete blood count Normal Creatinine First specimen, 1.3 mg/dL (114.9 µmol/L); repeat specimen, 1.5 mg/dL (132.6 µmol/L); baseline 6 months ago, 0.9 mg/dL (79.6 µmol/L) Estimated glomerular filtration rate (eGFR) , using the Chronic Kidney Disease Epidemiology (CKD-EPI) Collaboration creatinine formula >60 mL/min/1.73 m2 Urinalysis Trace protein Ultrasound reveals normal-sized kidneys with increased echogenicity, and no hydronephrosis or abnormalities of the collecting system are seen. Which of the following is the most appropriate management? Discontinue lisinopril Obtain a 24-hour creatinine clearance Recalculate the eGFR using the CKD-EPI cystatin C formula Schedule a kidney biopsy
Indications for kidney biopsy include glomerular hematuria, severely increased albuminuria, acute or chronic kidney disease of unclear cause, and kidney transplant dysfunction or monitoring. A kidney biopsy is appropriate for this patient with a decline in kidney function. Clinical and laboratory features are often insufficient for definitive diagnosis of kidney disease. Kidney biopsy may therefore be essential for diagnosis and management. Indications include glomerular hematuria, severely increased albuminuria, acute or chronic kidney disease of unclear cause, and kidney transplant dysfunction or monitoring. In this case, the patient has an elevated serum creatinine without clear cause. Although further serologic testing may be done to guide diagnosis, a kidney biopsy will provide definitive diagnosis. Due to preferential dilation of the efferent arteriole, the ACE inhibitor lisinopril may reduce glomerular filtration rate (GFR) and hence increase serum creatinine; however, this change in serum creatinine will occur within days of drug initiation and then stabilize. In this case, the serum creatinine increased without a change in dose, suggesting an unrelated cause. A 24-hour creatinine clearance may be helpful in estimating kidney function but will not change the evaluation at this time. Most studies show that the Chronic Kidney Disease Epidemiology (CKD-EPI) Collaboration equation or the Modification of Diet in Renal Disease (MDRD) study equation provides a more accurate estimated GFR (eGFR) compared with creatinine clearance. Although this patient's CKD-EPI eGFR is reported as >60 mL/min/1.73 m2, his increasing serum creatinine is consistent with a significantly declining eGFR. Although cystatin C will potentially add to the accuracy of the CKD-EPI formula, it will not change the evaluation, as GFR is declining as documented by the serum creatinine elevation (and no baseline cystatin C is available to show otherwise). Read Related TextNext Question
A 38-year-old man is evaluated during a new-patient appointment. The patient reports no rectal bleeding or other gastrointestinal symptoms. His family history includes colon cancer diagnosed in his father at age 52 years. His personal medical history is unremarkable and he takes no medication. All physical examination findings, including vital signs, are normal. When should this patient undergo his first screening colonoscopy? Now Age 40 years Age 42 years Age 50 years
Individuals with a first-degree relative with colon cancer or an advanced adenoma diagnosed at an age younger than 60 years, or two or more first-degree relatives with colon cancer or advanced adenoma diagnosed at any age, should begin colon cancer screening at age 40 years (or 10 years earlier than the youngest age at which colon cancer was diagnosed in a first-degree relative, whichever is first). This patient should undergo his first screening colonoscopy at age 40 years. Based on the patient's family history, he is at increased risk for colon cancer. Individuals with a first-degree relative with colon cancer or an advanced adenoma diagnosed at an age younger than 60 years, or two or more first-degree relatives with colon cancer or advanced adenoma diagnosed at any age, should begin colon cancer screening at age 40 years (or 10 years earlier than the youngest age at which colon cancer was diagnosed in a first-degree relative, whichever is first). Colonoscopy, rather than imaging or stool-based testing, is the recommended screening modality. If colonoscopy is performed and findings are normal, the recommended interval for repeat screening is 5 years if the first-degree relative was younger than age 60 years at the time of diagnosis and 10 years if the first-degree relative was age 60 years or older at the time of diagnosis. Although age 42 years is 10 years earlier than the age at which his father was diagnosed with colon cancer, guidelines recommend starting colon cancer screening at age 40 years or 10 years earlier than the family member's diagnosis, whichever comes first. Therefore, age 40 years is the appropriate age at which to begin screening in this patient. In average-risk patients, 50 years is the recommended age to begin screening for colon cancer. Because of this patient's family history, waiting until age 50 years to begin screening for colon cancer is not appropriate.
A 60-year-old woman is evaluated in the emergency department for a 4-week history of worsening midthoracic back pain and a 1-week history of leg weakness and numbness in her lower abdomen and legs. She does not have any bowel or bladder incontinence. She has hormone receptor-positive, HER2-negative breast cancer that has metastasized to the spine, ribs, and pelvis. Current medications are letrozole and palbociclib. Until this recent deterioration, she has had excellent functional status. On physical examination, vital signs are normal. She has tenderness to percussion on the T6 vertebra. There is a sensory level at the T8 vertebral body; bilateral, proximal, and distal lower-extremity weakness; hyperactive knee and ankle reflexes; and bilateral plantar extension responses. The remainder of the physical examination is normal. MRI scans show a large lytic lesion in the T6 vertebral body, with an epidural mass causing compression of the spinal canal. In addition to intravenous dexamethasone, which of the following is the most appropriate treatment? Elective radiation therapy Immediate decompressive surgery Palliative care only Urgent radiation therapy
Initial administration of dexamethasone followed by decompressive surgery for epidural spinal cord compression is recommended in patients who are acceptable surgical candidates and who have an expected survival of at least 3 months. This patient should receive immediate decompressive surgery. Once spinal cord compression is diagnosed, treatment must be started urgently to prevent irreversible loss of neurologic function. Glucocorticoids should be given immediately and promptly followed by surgery, external beam radiation, or stereotactic body radiation. A study of 101 patients with epidural spinal cord compression showed that patients who received surgery followed by radiotherapy had better outcomes and lower morbidity and mortality than those who received radiotherapy alone. Candidates for surgery include those with a life expectancy of at least 3 months and the absence of comorbidities that would otherwise be a contraindication to surgery. Radiation therapy would never be done electively in a patient with epidural cord compression because delays in treatment lead to further neurologic deterioration. Palliative care and supportive care alone might be appropriate in a patient with a very poor prognosis who does not want any further active antineoplastic therapy. Patients in this situation must be aware of the high likelihood of neurologic deterioration, including loss of ambulation and loss of control of bowel and bladder function. In most patients with metastatic cancer, treatment to avoid these complications is recommended. Urgent radiation would be appropriate in patients who are not surgical candidates because of comorbidities, who have an expected survival of less than 3 months, or who refuse surgery. This patient has a life expectancy of more than 3 months and has no medical contraindications to surgery.
A 51-year-old woman underwent CT scan of the abdomen following a motor vehicle accident. This revealed a 2.5-cm right adrenal mass with a density of 6 Hounsfield units (compatible with adrenal adenoma). Medical history is significant for hypertension diagnosed 2 years ago. She is perimenopausal and has been experiencing sweating and hot flushes. Medications are hydrochlorothiazide and doxazosin. On physical examination, blood pressure is 142/90 mm Hg, pulse rate is 90/min. Other vital signs are normal. BMI is 33. There are no supraclavicular fats pads or abdominal striae. Laboratory studies show a cortisol level following a 1-mg overnight dexamethasone suppression test of 8 µg/dL (220.8 nmol/L), plasma aldosterone-plasma renin ratio (ARR) of 13, and plasma fractionated free metanephrines of 32 pg/mL (0.17 nmol/L). Which of the following is the most likely diagnosis? Non-hormone-secreting adrenal adenoma Pheochromocytoma Primary aldosteronism Subclinical Cushing syndrome
Initial testing for subclinical Cushing syndrome is a 1-mg overnight dexamethasone suppression test; a cortisol level greater than 5 µg/dL (138 nmol/L) is considered a positive test. The most likely diagnosis in this patient with an incidentally discovered adrenal mass is subclinical Cushing syndrome. All patients with an incidental adrenal mass should be screened for subclinical Cushing syndrome (SCS), a condition characterized by adrenocorticotropic hormone (ACTH)-independent cortisol secretion that may result in metabolic (hyperglycemia and hypertension) and bone (osteoporosis) effects of hypercortisolism, but not the more specific features of Cushing syndrome (centripetal obesity, facial plethora, abnormal fat deposition in the supraclavicular or dorsocervical areas, and wide violaceous striae). The preferred diagnostic test for SCS is a 1-mg overnight dexamethasone suppression test, with a morning cortisol level greater than 5 µg/dL (138 nmol/L) considered positive. Following a positive result for SCS, measurement of ACTH, dehydroepiandrosterone sulfate (DHEAS), urine free cortisol, and an 8-mg overnight dexamethasone suppression test are often required to confirm autonomous cortisol secretion. If SCS is confirmed, the risks and benefits of surgery need to be considered. Surgery for SCS has been associated with improvements in bone density and glucose, lipid, and blood pressure control. This patient's serum cortisol level did not suppress to less than 5 µg/dL (138 nmol/L) following dexamethasone administration. Under normal conditions, exposure to dexamethasone results in suppression of ACTH secretion and hence suppression of adrenal cortisol secretion. In the setting of autonomous cortisol secretion from an adrenal mass, cortisol suppression following dexamethasone administration would not occur, as tumor production of cortisol is not under normal physiologic feedback control. The patient's plasma free metanephrine levels were within the normal range. This test has excellent sensitivity, and a normal result rules out pheochromocytoma. Primary aldosteronism is also unlikely based on a calculated plasma aldosterone-plasma renin ratio (ARR) of less than 20. Her blood pressure medications, hydrochlorothiazide and doxazosin, have minimal or mild effects on the ARR, and therefore the result can be reliably interpreted.
A 67-year-old man is evaluated for fatigue and anemia of approximately 2 month's duration. He has no other symptoms and takes no medications. On physical examination, vital signs are normal. Petechiae are noted on the lower extremities, and there is conjunctival pallor. The spleen is palpable 4 cm below the costal margin. There is no lymphadenopathy and no hepatomegaly. Laboratory studies: Hemoglobin 10.4 g/dL (104 g/L) Leukocyte count 2200/µL (2.2 × 109/L) with 63% lymphocytes, 34% segmented neutrophils, 3% monocytes Platelet count 73,000/µL (73 × 109/L) Flow cytometry of peripheral blood confirms kappa-restricted CD20+ B cells expressing CD11c, CD25, and CD103. Peripheral blood smear is shown. Which of the following is the most appropriate treatment? Cladribine Interferon alfa Rituximab Splenectomy Observation
Initial therapy for hairy cell leukemia is a purine nucleoside analogue, either cladribine or pentostatin. This patient has hairy cell leukemia, a low-grade B-cell disorder characterized by cytopenia and splenomegaly without lymphadenopathy. Hairy cells (classic appearance of thread-like projections emanating from the cell surface) are typically seen in the peripheral blood smear, and findings on flow cytometry demonstrate positivity for surface markers CD20, CD11c, CD25, and CD103. Cladribine is an appropriate initial therapy for hairy cell leukemia. Cladribine, as well as another purine nucleoside analogue, pentostatin, is associated with complete response rates exceeding 80% to 90%. A single 5- or 7-day course of cladribine is often all that is required. Many patients have durable remissions, although a significant minority do relapse and require retreatment with the same or alternate agents. Interferon-alfa therapy is active in the treatment of hairy cell leukemia but is rarely used anymore given its toxicity and inferior response rate compared to other agents. Hairy cell leukemia is typically CD20 positive and should respond to rituximab. However, clinical trials of rituximab have shown mixed results. Many experts reserve rituximab for patients failing two cycles of a purine nucleoside analogue. Rituximab is not given as initial therapy. Hypersplenism in hairy cell leukemia may play a larger role in the mechanism of cytopenias compared to other lymphoproliferative disorders. Splenectomy can be an effective treatment for relieving symptoms and ameliorating cytopenias associated with hairy cell leukemia. Before effective systemic therapies were available, splenectomy was the initial treatment of choice. However, splenectomy does not treat the underlying marrow infiltration and is no longer considered appropriate initial therapy. Some patients with hairy cell leukemia are asymptomatic and do not require immediate therapy. However, this patient has fatigue and anemia, so observation would not be appropriate.
An 81-year-old man is evaluated for a 2-week history of gross hematuria. He is a former cigarette smoker, having quit 15 years ago. He is otherwise healthy and takes no medications. On physical examination, vital signs and other findings are normal. An ultrasound of the kidneys, ureters, and bladder is unremarkable. Cystoscopy reveals a lesion along the bladder wall. Transurethral resection of the bladder tumor reveals high-grade, stage T1 transitional cell carcinoma. Which of the following is the most appropriate treatment? Cisplatin-based systemic chemotherapy Intravesicular bacillus Calmette-Guérin Intravesicular bacillus Calmette-Guérin followed by partial cystectomy Partial cystectomy
Initial treatment for high-grade or recurrent low-grade bladder cancer is transurethral resection of the bladder tumor followed by intravesical chemotherapy and periodic cystoscopy. The most appropriate treatment is intravesicular bacillus Calmette-Guérin (BCG) for six treatments followed by periodic cystoscopy. Bladder cancer is the most commonly diagnosed cancer of the urinary tract. In the United States, almost all bladder cancer is transitional cell carcinoma. Risk factors include advanced age, white ethnicity, various occupational exposures, and cigarette smoking. Smoking is the most important risk factor and encompasses current and former smokers and persons exposed to secondhand smoke. Persons at occupational risk include metal workers, painters, miners, textile workers, and leather workers. The most common presenting symptom of bladder cancer is gross hematuria. Most patients diagnosed with bladder cancer will be found to have non-muscle invasive disease, which can be purely exophytic or can involve limited bladder wall invasion. For patients with non−muscle invasive cancer, transurethral resection of the bladder tumor (TURBT) is the standard initial management. For those with high-grade or recurrent low-grade cancer, standard care is to follow TURBT with six rounds of intravesical chemotherapy, most commonly BCG or mitomycin given directly into the bladder. This procedure is done both for treatment of the identified lesion and to help prevent additional bladder tumors from developing. The risk of either recurrent cancer or a new bladder cancer arising elsewhere is significant. Therefore, after BCG or mitomycin treatment, it is essential to perform cystoscopy 3 months after treatment and subsequently at 3- to 6-month intervals to look for new cancers. Patients with noninvasive bladder cancer have an excellent prognosis, and cancer-related deaths rarely occur. Patients who develop recurrent disease with favorable tumor characteristics may be retreated with intravesicular therapy but, if higher-risk tumors are found, may require cystectomy. Cisplatin-based chemotherapy has no role in the treatment of non−muscle invasive cancer. It is used most often as a neoadjuvant treatment before surgery in patients with muscle-invasive cancer. The role of adjuvant chemotherapy in muscle-invasive cancer is much less clear. Partial cystectomy is not needed for the treatment of early-stage, non-muscle invasive bladder cancer. However, it has a role in selected patients with muscle-invasive cancer.
A 53-year-old man is evaluated in the emergency department after 4 days of cough, fever, chills, myalgia, and poor appetite. He currently has increased dyspnea and lightheadedness. His child was diagnosed with influenza 2 weeks ago. On physical examination, temperature is 38.8 °C (101.8 °F), blood pressure is 82/40 mm Hg, pulse rate is 128/min, and respiration rate is 17/min. Oxygen saturation is 92% on ambient air. The cardiac examination reveals regular rhythm and tachycardia without an S3 or jugular venous distention. Lungs are clear on auscultation and extremities are warm. The remainder of the examination is normal. Laboratory studies: Hemoglobin 10 g/dL (100 g/L) Lactate 4.6 mEq/L (4.6 mmol/L) Leukocyte count 18,000/µL (18 × 109/L) Arterial blood gases : pH 7.32 PCO2 32 mm Hg (4.3 kPa) PO2 70 mm Hg (9.3 kPa) Bicarbonate 16 mEq/L (16 mmol/L) A chest radiograph shows basilar ground-glass opacities on the right. Electrocardiogram reveals sinus tachycardia but is otherwise normal. Which of the following is the most appropriate initial treatment? 0.9% saline bolus Intravenous furosemide Norepinephrine Packed red blood cells
Initial treatment of septic or distributive shock should focus on aggressive fluid resuscitation with crystalloids within the first 3 hours of presentation. This patient should receive a 0.9% saline bolus. He has signs of septic shock from influenza (fever, tachycardia, hypotension, elevated leukocyte count, and exposure to influenza). In patients with septic or distributive shock, initial resuscitation efforts should be aimed at giving crystalloid fluids (0.9% saline, Ringer's lactate). The 2016 Surviving Sepsis guidelines recommend giving 30 mL/kg crystalloid solution within 3 hours of presentation in patients who demonstrate signs of tissue hypoperfusion. Judicious fluid administration is warranted thereafter, as intravascular volume overload can contribute to pulmonary edema and pleural effusions. This patient has several features of hypoperfusion, including hypotension, tachycardia, metabolic acidosis, and an elevated blood lactate. Furosemide would be appropriate if the patient were in cardiogenic shock and presenting with signs of volume overload. The history, examination, and chest radiograph are not consistent with cardiogenic shock or volume overload. Therefore, furosemide is not recommended and would only promote further hypotension. If hypotension does not rapidly correct with fluids, vasopressors should be titrated to maintain a mean arterial pressure of 65 mm Hg or greater. Norepinephrine is considered first-line therapy. This patient has not received crystalloid solution yet so initial therapy with norepinephrine is incorrect. However, in patients who are refractory to volume loading, vasopressor therapy is recommended to help improve hemodynamic stability. In the absence of extenuating circumstances (myocardial ischemia, severe hypoxemia, or active hemorrhage), the Surviving Sepsis guidelines recommend that red blood cell transfusion only be given if hemoglobin is less than 7 g/dL (70 g/L). Because this patient has a hemoglobin level above 7 g/dL, there is no direct role for packed red blood cell transfusion.
A 76-year-old man is evaluated in the emergency department for confusion, an unsteady gait, tinnitus, nausea, and vomiting. His family has noticed progressive functional decline over the past 2 weeks. History is significant for osteoarthritis. His only medication is aspirin. On physical examination, the patient is tachypneic and obtunded. Temperature is normal, blood pressure is 140/68 mm Hg, pulse rate is 96/min, respiration rate is 24/min, and oxygen saturation is 99% breathing ambient air. The neurologic examination is nonfocal. Laboratory studies: Electrolytes : Sodium 142 mEq/L (142 mmol/L) Potassium 3.2 mEq/L (3.2 mmol/L) Chloride 100 mEq/L (100 mmol/L) Bicarbonate 20 mEq/L (20 mmol/L) Arterial blood gases : pH 7.56 PCO2 22 mm Hg (2.9 kPa) Which of the following is the most likely acid-base diagnosis? Respiratory alkalosis with chronic compensation Respiratory alkalosis and increased anion gap metabolic acidosis Respiratory alkalosis and metabolic alkalosis Respiratory alkalosis, increased anion gap metabolic acidosis, and metabolic alkalosis
Interpretation of acid-base disorders requires the identification of the likely dominant acid-base disorder, followed by an assessment of the compensatory response; when measured values fall outside the range of the predicted compensatory response, a mixed acid-base disorder is considered present. The most likely diagnosis is a complex mixed acid-base disorder consisting of respiratory alkalosis, increased anion gap metabolic acidosis, and metabolic alkalosis. Interpretation of acid-base disorders requires the identification of the likely dominant acid-base disorder, followed by an assessment of the compensatory response. When measured values fall outside the expected compensatory range, a mixed acid-base disorder is considered present. Multiple acid-base disturbances may coexist, as seen in this patient. Because the blood pH is 7.56, the patient's dominant acid-base disorder is an alkalosis. The low PCO2 indicates a respiratory component to the alkalosis. The expected metabolic compensation for chronic respiratory alkalosis is a reduction in the serum bicarbonate of 4 to 5 mEq/L (4-5 mmol/L) for each 10 mm Hg (1.3 kPa) decrease in the PCO2(in this case, the decrease in PCO2 is 20 mm Hg [2.7 kPa]). The expected serum bicarbonate concentration in this patient is calculated as follows: Normal Bicarbonate − Expected Compensation 24 − (8-10) mEq/L (mmol/L) = 14-16 mEq/L (14-16 mmol/L) Because the measured bicarbonate of 20 mEq/L (20 mmol/L) is higher than expected, this suggests coexistence of a metabolic alkalosis. An elevated anion gap is also present, indicating the presence of an increased anion gap metabolic acidosis. Assessing the ratio of the change in the anion gap (Δ anion gap) to the change in bicarbonate level (Δ bicarbonate), or the "delta-delta (Δ-Δ) ratio," may indicate the presence of a coexistent acid-base disturbance. A ratio of <0.5 to 1 may reflect the presence of concurrent normal anion gap metabolic acidosis, whereas a ratio >2 may indicate the presence of metabolic alkalosis. This patient's Δ-Δ ratio is 2.5 [Δ anion gap/Δ bicarbonate = (22 − 12)/(24 − 20) = 2.5], confirming the coexistence of the metabolic alkalosis. The clinical situation most likely to present with this acid-base disorder is salicylate toxicity. Central hyperventilation from salicylate will cause the respiratory alkalosis; salicylate itself will cause the anion gap metabolic acidosis; and vomiting will cause the metabolic alkalosis.
A 55-year-old woman is evaluated to discuss additional cancer therapy. She was recently diagnosed with stage IIIB ovarian high-grade serous carcinoma. She underwent surgical staging, total abdominal hysterectomy, salpingo-oophorectomy, and debulking, with residual tumor size less than 1 cm. Genetic testing was negative for BRCA1 and BRCA2 mutations. She takes no medications. On physical examination, vital signs are normal. There is a healing midline abdominal incision. The remainder of the examination is normal. Her performance status is excellent (Eastern Cooperative Oncology Group performance status = 0. Which of the following is the most appropriate treatment? High-dose chemotherapy with hematopoietic stem cell transplantation Intravenous and intraperitoneal cisplatin and paclitaxel chemotherapy Intravenous cisplatin and paclitaxel chemotherapy Olaparib
Intravenous and intraperitoneal cisplatin and paclitaxel chemotherapy is a treatment option for patients with advanced ovarian cancer. The most appropriate initial treatment for this patient with advanced ovarian cancer is intravenous (IV) and intraperitoneal (IP) cisplatin and paclitaxel chemotherapy. This treatment is associated with a 16-month improvement in median overall survival in women with stage III ovarian cancer and no residual mass greater than 1 cm after surgical debulking compared with intravenous chemotherapy alone. Patients receiving IP/IV chemotherapy have a higher incidence of toxicities, including leukopenia, thrombocytopenia, fever, infection, fatigue, pain, and gastrointestinal events. Less than half of patients starting IP/IV therapy can complete all six cycles of therapy. However, the National Cancer Institute encourages the use of IP/IV chemotherapy for optimally debulked stage III ovarian cancer. Obstacles to the widespread adoption of IP/IV chemotherapy include not only the higher toxicities but also placement of an IP catheter and training of nursing staff in IP chemotherapy administration. The original IP/IV regimen required hospitalization for the first 3 days of each cycle. Modified regimens that allow all treatment to be given on an outpatient basis are now accepted options. High-dose chemotherapy with hematopoietic stem cell transplantation is not recommended for the treatment of ovarian cancer, neither as initial treatment nor after recurrence. Patients with left colon or rectosigmoid resections had a higher risk of discontinuing IP therapy, so this approach is no longer recommended in patients who undergo bowel resection. The types of catheters used have improved, with fewer catheter-related problems. Olaparib is approved for use to treat relapsed ovarian cancer in women with BRCA1 or BRCA2 germline mutations. This oral poly(ADP-ribose) polymerase (PARP) inhibitor is FDA approved as monotherapy for patients with germline BRCA-mutated advanced ovarian cancer previously treated with three or more lines of chemotherapy. Patients with germline BRCA1- or BRCA2-associated cancers show responses in ovarian, breast, pancreatic, and prostate cancers, with a 31% response rate and median duration of response of 7 months in platinum-resistant ovarian cancers. Olaparib is also approved for maintenance therapy for patients with BRCA1 or BRCA2 mutations with a complete or partial remission after first-line platinum-based chemotherapy. However, it is not used as initial therapy in ovarian cancer.
A 69-year-old man with previously untreated chronic lymphocytic leukemia returns to the office after his third hospitalization this year for pneumonia. He received the pneumococcal conjugate vaccine at age 65 years and the polysaccharide vaccine at age 66 years. He is taking no medications. On physical examination, vital signs are normal. He has diffuse lymphadenopathy with nodes up to 2.5 cm in size and a palpable spleen below his left costal margin. The chest is clear to auscultation. Laboratory studies: Leukocyte count 34,000/µL (34 × 109/L) with 91% lymphocytes, 5% neutrophils, 3% monocytes, and 1% bands Platelet count 118,000/µL (118 × 109/L) Hemoglobin 12.2 g/dL (122 g/L) Which of the following is the most appropriate management? Assess immunoglobulin levels Start bendamustine and rituximab Start granulocyte colony-stimulating factor (G-CSF) injections Start ibrutinib
Intravenous immunoglobulin G replacement therapy reduces the risk of infections in patients with chronic lymphocytic leukemia with documented hypogammaglobulinemia. This patient with chronic lymphocytic leukemia (CLL) has had three hospitalizations for pneumonia in the past year, and assessing his immunoglobulin levels would be appropriate. Patients with CLL frequently are hypogammaglobulinemic and are prone to sinus and pulmonary infection. Intravenous immunoglobulin G replacement therapy has been shown to reduce the risk of infections in patients with CLL with documented hypogammaglobulinemia. Surveillance for viral infections and early initiation of antimicrobial agents for presumed bacterial infections are also essential. Although this patient has lymphocytosis, lymphadenopathy, and splenomegaly, their presence alone is not an indication to treat his underlying CLL in the absence of symptoms or more significant cytopenias. Although the patient has only 5% neutrophils and 1% bands, his total leukocyte count is 34,000/µL (34 × 109/L), and thus he is not significantly neutropenic. Even if he were neutropenic, long-term granulocyte colony-stimulating factor (G-CSF) injections would not be appropriate. Ibrutinib is an appropriate therapy for CLL that requires treatment because it is taken orally, is relatively easy for most patients to tolerate, and is associated with high response rates in both untreated and previously treated patients. However, there is no clear benefit to immediate initiation of ibrutinib therapy in asymptomatic patients.
A 42-year-old man was hospitalized 2 days ago with necrotizing fasciitis of the leg. He underwent extensive debridement and fasciotomy 2 hours earlier. He currently has severe pain in his leg wounds. He was well before this illness. Current medications are imipenem, clindamycin, vancomycin, and a morphine infusion. On physical examination, blood pressure is 150/99 mm Hg, pulse rate is 110/min, and respiration rate is 24/min. He is awake, anxious, grimacing, and wriggling in bed. The remainder of the examination is noncontributory. Which of the following is the most appropriate treatment? Add gabapentin Epidural for regional analgesia Intravenous bolus of morphine Replace morphine with intravenous fentanyl
Intravenous opioids are the first-line drug class of choice to treat nonneuropathic pain in critically ill patients; all intravenously administered opioids have equi-analgesic efficacy and are associated with similar clinical outcomes when titrated to similar pain intensity end points. This patient should receive an intravenous push of morphine. He has acute postoperative pain of the leg and is experiencing severe pain despite a morphine infusion. Analgesia should be titrated to a specific pain management goal while preventing and monitoring for side effects. A bolus dose of morphine will reach therapeutic levels faster than changing rates of infusion. The bolus dose should be repeated until the patient achieves the therapeutic goal, a protocol limit is reached, or side effects occur. He should be monitored closely while receiving acute pain treatment. Common side effects include somnolence, depression of respiratory drive, urinary retention, and nausea and vomiting. Adding gabapentin is a good strategy for patients with neuropathic pain. Enteral gabapentin added to parenteral opioids can reduce the doses of opiates needed and improve pain control in mechanically ventilated patients. However, this patient is experiencing acute nonneuropathic pain and an intravenous opioid is the drug class of choice. Epidural analgesia is an effective means to control pain in critically ill patients. This has been demonstrated in patients with cardiac or thoracic surgery or in the setting of rib fractures. It can lead to a reduction of opiate dosing and improved pain control during dressing changes in patients such as this one; however, it takes time to implement and the patient should receive acute pain control before consideration of alternatives to improve his baseline pain control. Changing the medication to fentanyl is not appropriate as all intravenously administered opioids have equi-analgesic efficacy and are associated with similar clinical outcomes when titrated to similar pain intensity end points. The choice of opiate should be based on pharmacological properties. Fentanyl has a shorter half-life than morphine, and this patient was not having any adverse effect or contraindication to morphine.
A 45-year-old man is evaluated for a 6-month history of pruritic rash in the inguinal and gluteal folds. He has used topical ketoconazole for 3 weeks with no improvement. Medical history is significant for type 2 diabetes mellitus. Medications are metformin and ketoconazole cream. On physical examination, vital signs are normal. BMI is 32. Occipital scalp has erythematous plaque with scale. Skin findings are shown. Similar findings are noted in the gluteal cleft. Elbows and knees are clear. Nail pitting is not present. Which of the following is the most likely diagnosis? Allergic contact dermatitis Candidiasis Inverse psoriasis Seborrheic dermatitis Tinea cruris
Inverse psoriasis is characterized by red, thin plaques with variable amounts of scale in the axillae, intergluteal cleft, and perineum, and under the breasts and pannus.
A 45-year-old man is evaluated for a 6-month history of pruritic rash in the inguinal and gluteal folds. He has used topical ketoconazole for 3 weeks with no improvement. Medical history is significant for type 2 diabetes mellitus. Medications are metformin and ketoconazole cream. On physical examination, vital signs are normal. BMI is 32. Occipital scalp has erythematous plaque with scale. Skin findings are shown. Similar findings are noted in the gluteal cleft. Elbows and knees are clear. Nail pitting is not present. Which of the following is the most likely diagnosis? Allergic contact dermatitis Candidiasis Inverse psoriasis Seborrheic dermatitis Tinea cruris
Inverse psoriasis is characterized by red, thin plaques with variable amounts of scale in the axillae, intergluteal cleft, and perineum, and under the breasts and pannus. Based on the finding of the well-demarcated, erythematous plaques in intertriginous areas, this patient has inverse psoriasis. Psoriasis is a chronic inflammatory dermatosis that manifests with scaling, variably pruritic plaques that may be recalcitrant to topical therapy. There are many different patterns of psoriasis including classic psoriasis vulgaris (erythematous patches with a thick, adherent scale), inverse psoriasis (red, thin plaques with variable amount of scale in the axillae, under the breasts or pannus, intergluteal cleft, and perineum), sebopsoriasis (red, thin plaques in the scalp, eyebrows, nasolabial folds, central chest, and pubic area), and guttate psoriasis (0.5- to 2-cm red plaques that erupt suddenly on the trunk often after a group A streptococcal infection). Psoriasis can also involve the nails presenting as pit-like indentations and "oil spots" often involving multiple nails. Inverse psoriasis can be difficult to diagnosis because it often lacks the classic silvery scale. It also resembles other common dermatologic conditions such as tinea, intertrigo, and allergic contact dermatitis. Allergic contact dermatitis, while possible in these locations, would be less likely without history of exposure to an allergen. Allergic contact dermatitis does not explain the patient's erythematous plaques with scale on the scalp. Candida is frequently found in the flexures of patients with obesity, but typically presents with a bright red plaque with satellite papules and pustules. Candida would also respond to topical antifungal treatment. Seborrheic dermatitis can be found in the scalp, face, chest, and groin. It is more prevalent in those with HIV/AIDS or neurologic diseases such as Parkinson disease. This rash usually has a greasy, sometimes yellow scale and improves with antifungal medications. Tinea cruris is also commonly found in the inguinal folds. It typically presents as an annular plaque with an active, scaly border. The lack of scale and lack of response to antifungal therapy makes tinea cruris an unlikely diagnosis.
A 35-year-old woman is seen in follow-up evaluation for her type 2 diabetes mellitus. She was diagnosed 3 years ago. She checks her fasting and 2-hour postprandial blood glucose values several times per week. Her fasting blood glucose levels range from 100 to 110 mg/dL (5.5-6.1 mmol/L) and her 2-hour postprandial values are 120 to 165 mg/dL (6.7-9.1 mmol/L). Her review of symptoms is positive for chronic heavy menses. Medications are metformin and liraglutide. On physical examination, blood pressure is 123/74 mm Hg and pulse rate is 76/min. BMI is 31.2. The examination is otherwise unremarkable. Laboratory studies: Hematocrit 33% Iron studies Ferritin 11 ng/mL (11 µg/L) Iron 40 µg/dL (7.2 µmol/L) Iron-binding capacity , total 600 µg/dL (107.4 µmol/L) Hemoglobin A1c 7.3% Which of the following is the most appropriate management of the elevated hemoglobin A1c level? Basal insulin Empagliflozin Ferrous sulfate Hemoglobin electrophoresis
Iron-deficiency anemia can erroneously increase the hemoglobin A1c level due to an increase in the proportion of older erythrocytes. The most appropriate management for this patient is to initiate ferrous sulfate. This patient has iron-deficiency anemia, a hypoproliferative anemia, which has been shown to erroneously increase the hemoglobin A1c level due to an increase in the proportion of older erythrocytes. Hemoglobin A1c testing measures hemoglobin glycation as a consequence of glucose exposure over the preceding 8 to 12 weeks. Given the patient's age and relatively few comorbidities, her goal hemoglobin A1c level should be less than 6.5% to 7%. Her hemoglobin A1c level is above this goal, but her fingerstick blood glucose data are within her fasting goal of 80 to 130 mg/dL (4.4-7.2 mmol/L) and within her 2-hour postprandial goal of less than 180 mg/dL (10 mmol/L) per the American Diabetes Association (ADA) guidelines. Initiating iron supplementation to correct her iron deficiency anemia will increase erythrocyte turnover and shift the proportion toward younger cells, thus allowing a more accurate measurement of glycemic exposure by the hemoglobin A1c to guide therapeutic decisions. The patient's fasting blood glucose values are within her goal range of 80 to 130 mg/dL (4.4-7.2 mmol/L) per the ADA guidelines. Initiating basal insulin based solely on the elevated hemoglobin A1c value with her current fasting blood sugars will increase her risk of hypoglycemia. Similarly, the risk of hypoglycemia is increased by initiating empagliflozin. Increasing the accuracy of the hemoglobin A1c measurement by correcting her iron deficiency anemia should be addressed before considering other drug therapy. Hemoglobin A1c measurements may be unreliable not only in the setting of anemia, but also in the presence of certain hemoglobinopathies or kidney or liver disease. For example, hemoglobin A1c values may be falsely elevated in patients with hemoglobin F or low with hemoglobin S. However, this patient has an explanation for the discordant hemoglobin A1c and blood glucose results, and the iron deficiency anemia should be the focus of management. Additionally, newer methods of measuring A1c are not altered by the presence of the most common hemoglobinopathies. Therefore, a hemoglobin electrophoresis is not indicated at this time.
A 60-year-old woman is admitted to the hospital with sudden-onset, cramping abdominal pain of moderate severity in the right lower quadrant, followed several hours later by a bloody bowel movement. She has coronary artery disease; medications are atorvastatin, metoprolol, sublingual nitroglycerin, and low-dose aspirin. On physical examination, the patient appears comfortable. Pulse rate is 110/min; BMI is 35. Other vital signs are normal. The abdomen is nondistended with normal bowel sounds. Deep palpation elicits tenderness in the right lower quadrant with no rebound or guarding. A CT scan without contrast shows thickening of the ascending colon. Colonoscopy results show a segment of subepithelial hemorrhage, edema, and erythema from the cecum to the hepatic flexure. Which of the following is the most appropriate test to perform next? CT angiography Doppler ultrasonography of mesenteric vessels MR angiography Selective catheter angiography
Isolated right-colon ischemia may be a warning sign of acute mesenteric ischemia caused by embolism or thrombosis of the superior mesenteric artery and should be evaluated using CT angiography. CT angiography is the best next test for this patient, whose clinical presentation with the sudden onset of right-sided, cramping abdominal pain followed by a bloody bowel movement is typical of isolated right-colon ischemia. A CT scan showing thickening of the ascending colon and the colonoscopy features are helpful in confirming this diagnosis. The most common cause of colon ischemia is a nonocclusive low-flow state in the colonic microvasculature. Most cases of colonic ischemia involve the left colon, which is supplied by the inferior mesenteric artery; as with ischemia involving the right colon, the diagnosis is clinical and supported by CT and colonoscopy. Patients with left-sided colonic ischemia tend to heal well with conservative therapy alone, whereas isolated right-colon ischemia can be the harbinger of acute mesenteric ischemia caused by a focal thrombus or embolus of the superior mesenteric artery. This artery supplies both the small intestine and right colon, and the consequences of acute mesenteric ischemia involving the small bowel are severe, with mortality rates that can approach 60%. For this reason, patients with isolated right-colon ischemia require urgent, noninvasive imaging of the mesenteric vasculature to assess the extent of ischemia and nature of the intervention. CT angiography is the recommended method of imaging for diagnosing acute mesenteric ischemia because it can be obtained rapidly. CT angiography visualizes the origins and length of the vessels, characterizes the extent of occlusion, and aids in planning revascularization. Doppler ultrasonography of the mesenteric vessels is an effective, low-cost tool that can assess the proximal visceral vessels but has limited ability to visualize distal vessels. It is best reserved for the evaluation of patients with chronic mesenteric ischemia, which typically presents with postprandial abdominal pain, sitophobia, and weight loss. MR angiography provides information about mesenteric arterial flow and avoids the potential harms of radiation and use of contrast that are associated with CT angiography; however, MR angiography takes longer to perform, lacks the required resolution to identify arterial occlusion, and can overestimate the severity of stenosis. Selective catheter angiography was the standard method for diagnosing mesenteric ischemia; however, it is now used after a revascularization plan has been chosen because CT angiography can be obtained rapidly and is noninvasive.
A 47-year-old woman is evaluated for an onset over 2 to 3 weeks of low-grade, intermittent fever; blanching of the second and third right fingertips in response to cold exposure; cracking and peeling of the skin on the sides of the second digits and palms; and pain and swelling of the second and fourth proximal interphalangeal joints bilaterally. She does not take any medications. On physical examination, temperature is 37.6 °C (99.7 °F), pulse rate is 95/min, and respiration rate is 20/min; blood pressure and oxygen saturation are normal. The tips of the digits are cool without discoloration. Erythema of the malar area, forehead, and chin is present. Pulmonary examination reveals crackles at the lung bases. There is no weakness. Skin findings are shown. Laboratory studies: Creatine kinase 115 U/L Antinuclear antibodies Titer: 1:1280 Anti-double-stranded DNA antibodies Negative Anti-Jo-1 antibodies Positive Anti-Smith antibodies Negative Which of the following is the most likely diagnosis? Antisynthetase syndrome Sjögren syndrome Systemic lupus erythematosus Systemic sclerosis
Key Point Antisynthetase syndrome is characterized by interstitial lung disease, myositis, Raynaud phenomenon, nonerosive inflammatory arthritis, constitutional findings such as low-grade fever, and mechanic's hands; anti-aminoacyl-tRNA synthetases antibodies, such as anti-Jo-1, are highly suggestive of the diagnosis. The most likely diagnosis is antisynthetase syndrome, which is characterized by interstitial lung disease, myositis, Raynaud phenomenon, nonerosive inflammatory arthritis, constitutional findings such as low-grade fever, and mechanic's hands. Mechanic's hands is a dermatologic manifestation unique to antisynthetase syndrome and is characterized by hyperkeratotic skin along the ulnar aspect of the thumb and radial aspects of the digits, most commonly on the index and middle fingers, and involvement of the palms. Unlike eczema, there is no pruritus, vesicles, or hand dominance. Antisynthetase syndrome can occur in patients with dermatomyositis or polymyositis; about one third of patients with dermatomyositis belong to this subset characterized by a relatively abrupt onset and features. Antinuclear antibodies (ANA) are often positive in patients with antisynthetase syndrome, but anti-aminoacyl-tRNA synthetases antibodies, including the subset anti-Jo-1 antibodies, are more specific for the diagnosis. In patients with Sjögren syndrome, constitutional findings and inflammatory arthritis are common, but Raynaud phenomenon is less so. Rash and mechanic's hands are not characteristics. Furthermore, dry eyes and dry mouth are the most common presenting symptoms, along with positive anti-Ro/SSA and anti-La/SSB antibodies. Fever, Raynaud phenomenon, rash, inflammatory arthritis, and positive ANA are seen in systemic lupus erythematosus. However, an abrupt onset rarely occurs, mechanic's hands are not a feature, and anti-Jo-1 antibodies are not characteristic. In patients with systemic sclerosis, fever and inflammatory arthritis are far less likely. Furthermore, although Raynaud phenomenon is very common in systemic sclerosis, the findings of either localized or diffuse skin tightening and thickening, including sclerodactyly, are absent in this patient. Mechanic's hands is not a feature of systemic sclerosis, and anti-Jo-1 antibodies are not characteristic.
A 51-year-old man is evaluated in the hospital for acute kidney injury. He was admitted 14 days ago with sepsis and community-acquired pneumonia requiring mechanical ventilation. He was treated empirically with ceftriaxone and azithromycin; additional medications included omeprazole, insulin, and subcutaneous heparin. On hospital day 3, blood cultures were positive for Streptococcus pneumoniae, and ceftriaxone was continued. Hospital course was complicated by the ICU stay, with mechanical ventilation and pneumothorax requiring thoracostomy tube placement. On admission, his serum creatinine level was 1.0 mg/dL (88.4 µmol/L), increasing to 1.9 mg/dL (168 µmol/L) on hospital day 10; omeprazole and ceftriaxone were discontinued, and he was transferred to the floor. Today, hospital day 14, he is oliguric. History is significant for hypertension and diabetes mellitus. Outpatient medications are losartan and metformin. On hospital day 14, the patient is on 4-L oxygen by nasal cannula. A right-sided thoracostomy tube is in place. Temperature is 36.1 °C (97.0 °F), blood pressure is 145/78 mm Hg, pulse rate is 92/min, and respiration rate is 12/min. There are coarse rhonchi in the left lower lung field. The remainder of the examination is normal. Current (hospital day 14) laboratory studies: Creatinine 2.7 mg/dL (238.7 µmol/L) Fractional excretion of sodium 3% Urinalysis 2+ blood; 2+ protein; 5-10 erythrocytes/hpf; 5-10 leukocytes/hpf; 3-5 granular casts/hpf Urine protein-creatinine ratio 1100 mg/g Kidney ultrasound is normal and without hydronephrosis. Which of the following is the most appropriate next step in management? Empiric glucocorticoids Kidney biopsy Normal saline fluid bolus Urinary catheter placement
Kidney biopsy should be considered in patients with acute kidney injury from no apparent or unclear cause, suspected glomerulonephritis, or unexplained systemic disease.
A 51-year-old man is evaluated in the hospital for acute kidney injury. He was admitted 14 days ago with sepsis and community-acquired pneumonia requiring mechanical ventilation. He was treated empirically with ceftriaxone and azithromycin; additional medications included omeprazole, insulin, and subcutaneous heparin. On hospital day 3, blood cultures were positive for Streptococcus pneumoniae, and ceftriaxone was continued. Hospital course was complicated by the ICU stay, with mechanical ventilation and pneumothorax requiring thoracostomy tube placement. On admission, his serum creatinine level was 1.0 mg/dL (88.4 µmol/L), increasing to 1.9 mg/dL (168 µmol/L) on hospital day 10; omeprazole and ceftriaxone were discontinued, and he was transferred to the floor. Today, hospital day 14, he is oliguric. History is significant for hypertension and diabetes mellitus. Outpatient medications are losartan and metformin. On hospital day 14, the patient is on 4-L oxygen by nasal cannula. A right-sided thoracostomy tube is in place. Temperature is 36.1 °C (97.0 °F), blood pressure is 145/78 mm Hg, pulse rate is 92/min, and respiration rate is 12/min. There are coarse rhonchi in the left lower lung field. The remainder of the examination is normal. Current (hospital day 14) laboratory studies: Creatinine 2.7 mg/dL (238.7 µmol/L) Fractional excretion of sodium 3% Urinalysis 2+ blood; 2+ protein; 5-10 erythrocytes/hpf; 5-10 leukocytes/hpf; 3-5 granular casts/hpf Urine protein-creatinine ratio 1100 mg/g Kidney ultrasound is normal and without hydronephrosis. Which of the following is the most appropriate next step in management? Empiric glucocorticoids Kidney biopsy Normal saline fluid bolus Urinary catheter placement
Kidney biopsy should be considered in patients with acute kidney injury from no apparent or unclear cause, suspected glomerulonephritis, or unexplained systemic disease. The most appropriate next step in the evaluation of this patient's acute kidney injury (AKI) is a kidney biopsy. This patient with sepsis has developed AKI that has continued to worsen despite discontinuation of antibiotics 4 days ago. His urinalysis is notable for hematuria, pyuria, and proteinuria. The differential diagnosis includes ischemic or toxic acute tubular necrosis in the setting of hypotension, acute interstitial nephritis (AIN) from antibiotics or a proton pump inhibitor, and infection-associated glomerulonephritis. Given multiple potential contributing factors, a kidney biopsy is necessary to differentiate. Glucocorticoids would be indicated if AIN is identified on biopsy, but empiric therapy is not justified because other causes may be contributing to his AKI. Moreover, glucocorticoids can cause significant adverse side effects, including worsening glycemic control in a patient with diabetes mellitus. If AIN is confirmed on kidney biopsy, a trial of glucocorticoids can be considered. Prerenal AKI results from decreased renal perfusion and will typically respond to restoration of effective arterial circulation. However, the patient has no findings of hypoperfusion, and the elevated fractional excretion of sodium argues against this diagnosis (typically <1%) and the need for a fluid challenge. Such an intervention may be deleterious in a patient with oliguric AKI. Postrenal AKI results from urinary tract obstruction. Bladder outlet obstruction should be suspected in patients with prostate enlargement or in the setting of diabetes mellitus (neurogenic bladder), pain medications, or anticholinergic medications. This patient's kidney ultrasound does not show obstruction; therefore, placement of a urinary catheter is not appropriate and would increase his risk of catheter-associated urinary tract infection.
Tx for gestational HTN?
Labetalol and methyldopa are the agents of choice in treating pregnant patients with hypertension. In this pregnant patient, labetalol is the most appropriate treatment to achieve blood pressure control. Hypertension diagnosed before the 20th week of gestation is most consistent with a new diagnosis of chronic hypertension rather than preeclampsia. Pharmacologic treatment of chronic hypertension in pregnancy is recommended to achieve the goal of limiting maternal end-organ damage; there is no evidence that blood pressure control during pregnancy will prevent preeclampsia. The definitive threshold for treatment of chronic hypertension in pregnancy remains controversial. A 2013 guideline published by the American College of Obstetricians and Gynecologists made a strong recommendation for initiation of pharmacologic therapy for pregnant women with persistent chronic hypertension at a systolic blood pressure of 160 mm Hg or higher or a diastolic blood pressure of 105 mm Hg or higher. However, other guidelines, including one from the European Society of Cardiology, continue to recommend initiation of pharmacologic therapy at a systolic blood pressure of 150 mm Hg or higher or a diastolic blood pressure of 95 mm Hg or higher in women without symptoms or evidence of end-organ damage due to hypertension. Although all antihypertensive agents cross the placenta, methyldopa and labetalol appear to be the safest antihypertensive agents in pregnant women. Diet and weight loss should be used in conjunction with pharmacologic therapy in this patient; however, nonpharmacologic strategies alone are insufficient in a patient with hypertension of this magnitude. Foregoing pharmacologic treatment of chronic hypertension may result in maternal end-organ damage. ACE inhibitors (such as lisinopril), angiotensin receptor blockers (such as losartan), spironolactone, and direct renin inhibitors (aliskiren) are teratogenic and are therefore contraindicated during pregnancy. During the first trimester, these agents can cause central nervous system and cardiovascular malformations in the fetus. Second-trimester exposure can cause developmental malformations of the kidneys and genitourinary system.
A 48-year-old man is evaluated in the emergency department for skin trauma sustained in a freshwater lake 2 days ago, with abrasions and tiny lacerations over the right forearm; he developed a fever and pain at the site of trauma 1 day ago. Medical history is remarkable for cirrhosis secondary to alcohol use. He takes no medications. On physical examination, temperature is 39.1 °C (102.4 °F), blood pressure is 100/70 mm Hg, pulse rate is 120/min, and respiration rate is 25/min. The right forearm is tender and warm, with several hemorrhagic bullae noted. The remainder of the examination is unremarkable. A plain radiograph of the right forearm shows no evidence of gas or a foreign body. Surgical exploration and debridement is performed, confirming a diagnosis of necrotizing fasciitis. Gram stain of intraoperative tissue specimens reveals gram-negative bacilli. Empiric antibiotic treatment with vancomycin plus piperacillin-tazobactam is initiated. Twenty-four hours later, the tissue culture grows Aeromonas hydrophila. Which of the following is the most appropriate treatment? Ciprofloxacin plus doxycycline Linezolid plus metronidazole Nafcillin plus rifampin Vancomycin plus clindamycin
Lacerations and puncture wounds sustained in fresh and brackish water environments can result in necrotizing infection with Aeromonas hydrophila; this infection should be treated with surgery, supportive care, and antibiotics with gram-negative coverage, such as doxycycline plus ciprofloxacin. The most appropriate treatment for this patient is ciprofloxacin plus doxycycline. This patient has Aeromonas hydrophila-associated skin and soft tissue infection. Aeromonas species are found in aquatic environments, including fresh water and brackish water, and grow best during warmer months. Lacerations and puncture wounds sustained in these environments can result in wound infection. Aeromonas infections of the skin and soft tissue and of the bloodstream are more likely to occur in patients with underlying immunocompromising conditions, such as cirrhosis and cancer, and are more common in men. Necrotizing fasciitis caused by this gram-negative bacillus requires surgery, supportive care, and antibiotics. Pending culture data, empiric therapy for necrotizing skin infections typically consists of broad-spectrum antibiotics such as vancomycin plus piperacillin-tazobactam. When the diagnosis of A. hydrophila infection is established, doxycycline plus ciprofloxacin or ceftriaxone is recommended. The combination of linezolid and metronidazole, although effective against aerobic gram-positive organisms, such as staphylococci and enterococci as well as many anaerobes, is not active against Aeromonas. Nafcillin and rifampin would be active against methicillin-susceptible S. aureus, particularly prosthetic joint infections, but is not active against A. hydrophila. Vancomycin plus clindamycin would be effective in necrotizing fasciitis with associated toxic shock caused by methicillin-resistant S. aureus but would not be active against A. hydrophila.
A 35-year-old man is evaluated for two breakthrough focal seizures in the past week. He started having focal and generalized tonic-clonic seizures 20 years ago. The seizures previously had been well controlled by phenytoin, which he has taken since age 16 years, but he missed three doses of medication this week. He has had no recent illnesses or fevers. On physical examination, all findings, including vital signs, are normal. Results of bone densitometry show osteoporosis (T-score of −2.8). In addition to starting the patient on alendronate, which of the following drugs should be substituted for the phenytoin? Carbamazepine Lamotrigine Phenobarbital Valproic acid
Lamotrigine does not have the potential to cause or worsen osteoporosis in patients with epilepsy. Lamotrigine is the most appropriate antiepileptic drug (AED) to treat this patient's focal seizures. Carbamazepine, phenytoin, and phenobarbital are all inducers of the cytochrome p450 system; these drugs increase breakdown of vitamin D, which results in increased parathyroid hormone levels, and thus cause bone loss and osteoporosis. Valproic acid also is associated with bone loss, although the mechanism of this effect is unclear. Lamotrigine and other AEDs that do not induce the P450 system, such as levetiracetam, have no recognized effect on bone turnover; therefore, of the choices listed, lamotrigine is the most appropriate AED to use in a patient with established osteoporosis. All patients on chronic AED therapy with phenytoin, carbamazepine, phenobarbital or valproic acid should undergo initial bone densitometry testing after 5 years of therapy, regardless of age, sex, or menopausal status. Those especially at risk are nonambulatory and physically inactive patients with epilepsy who have a non-weight-bearing status. Screening should be repeated, depending on the results, but not more frequently than every 2 years. Patients with AED-associated osteoporosis should be assessed for adequate intake of calcium and vitamin D, with initiation of supplementation if intake is inadequate or if low levels of 25-hydroxyvitamin D are noted. Otherwise, treatment for osteoporosis associated with AEDs is the same as for osteoporosis in general and usually involves administration of a bisphosphonate, such as alendronate.
A 45-year-old man is hospitalized with a 5-day history of fever, bloody diarrhea, and abdominal pain. Medical history is significant for end-stage kidney disease, for which he underwent kidney transplantation 1 year ago. Medications are prednisone, mycophenolate, and tacrolimus. On physical examination, vital signs are normal. Conjunctival pallor is present. Abdominal palpation elicits diffuse abdominal pain. The remainder of the examination is unremarkable. Laboratory studies: Hemoglobin 9.5 mg/dL (95 g/L) Leukocyte count 3400/µL (3.4 × 109/L) Platelet count 98,000/µL (98 × 109/L) Alanine aminotransferase 99 U/L Aspartate aminotransferase 88 U/L Creatinine 1.5 mg/dL (133 µmol/L) Which of the following is the most likely diagnosis? Cytomegalovirus infection Entamoeba histolytica infection Salmonella enteritidis infection Strongyloides stercoralis infection
Latent cytomegalovirus infection is present in 60% to 90% of adults, and patients who are immunosuppressed may experience disease reactivation with retinitis, pneumonitis, hepatitis, bone marrow suppression, colitis with bloody diarrhea, esophagitis, or adrenalitis. The most likely diagnosis in this patient is cytomegalovirus infection. Approximately 60% to 90% of adults have latent cytomegalovirus infection, with reactivation of disease common in persons who are immunosuppressed (patients with AIDS, transplant recipients, patients taking glucocorticoids). Cytomegalovirus is an important pathogen in kidney transplant recipients, and the risk of cytomegalovirus infection depends on the serologic status of the kidney donor and recipient at the time of transplantation. The highest risk occurs when a seronegative recipient (one who has never had a cytomegalovirus infection) receives a kidney from a seropositive donor. Cytomegalovirus can cause retinitis (especially in persons with AIDS), pneumonitis, hepatitis, bone marrow suppression, colitis with bloody diarrhea, esophagitis, and adrenalitis. This patient recently received a kidney transplant; he has bone marrow suppression (leukopenia and thrombocytopenia), hepatitis (elevated aminotransferase levels), and bloody diarrhea consistent with cytomegalovirus reactivation. Diagnosis relies on isolation of the virus from bodily fluids, such as urine; detection of cytomegalovirus pp65 antigen in leukocytes; cytopathic demonstration of "owl's eye" intracellular inclusions from tissue biopsy (colon in this case) (shown); polymerase chain reaction; and serologic assays. Antiviral treatment is typically indicated in cases of disease reactivation in immunocompromised patients and occasionally in immunocompetent hosts with severe disease. Valganciclovir is the first-line agent and is also used as prophylaxis or pre-emptive therapy in certain transplant patients. Entamoeba histolytica and Salmonella enteritidis can cause bloody diarrhea, but neither presents with pancytopenia. Therefore, they are unlikely possibilities in a solid organ transplant recipient. Furthermore, E. histolytica would not cause elevated aminotransferase levels in the absence of a liver abscess. Strongyloides stercoralis is the only parasite that has an autoinfection route (ability to complete its life cycle entirely within the human host) resulting in an increasing burden of parasites that can survive for decades in patients. Disseminated strongyloidiasis after solid organ transplantation can present with abdominal pain and diarrhea, but it is usually nonbloody. Furthermore, disseminated Strongyloides infection may present with sepsis with colonic bacteria, serpiginous rash, meningitis, or eosinophilic pneumonia. All patients scheduled for solid organ transplantation are now screened with a Strongyloides antibody and treated with ivermectin to decrease the incidence of this infec
A 32-year-old woman is evaluated for a recent episode of transient left monocular blindness. She noted dimness of vision in the left eye that came on suddenly and persisted for 15 minutes and then resolved completely. She had no accompanying headache or other symptoms. She has an 8-year history of systemic lupus erythematosus, which initially manifested as photosensitivity, discoid rash, and arthritis. She has responded well to treatment and has been doing well without active disease for the past year. History is significant for recurrent first trimester pregnancy loss attributed to positive antiphospholipid and anticardiolipin antibodies. Medications are hydroxychloroquine, prednisone, aspirin, and a daily multivitamin. On physical examination, vital signs are normal. Cardiac rhythm is normal. A 2/6 holosystolic murmur is heard at the apex with radiation toward the axilla. Temporal and carotid artery pulsations are normal; there is no scalp tenderness or vascular bruits. The remainder of the physical examination, including ophthalmologic examination, is normal. Which of the following is the most likely cause of her visual symptom? Bacterial endocarditis Carotid artery stenosis Giant cell arteritis Libman-Sacks endocarditis
Libman-Sacks endocarditis (nonbacterial thrombotic endocarditis) is the most likely cause of this patient's transient monocular blindness. She has an 8-year history of systemic lupus erythematosus (SLE) and positive antiphospholipid antibodies (anticardiolipin antibodies plus lupus anticoagulant) with recurrent pregnancy loss. She meets the criteria for antiphospholipid syndrome. Patients who have SLE with positive antiphospholipid antibodies are at a high risk for developing valvular dysfunction/thickening, and in some cases manifesting as Libman-Sacks endocarditis. A recent study confirmed this significant association between valvular heart disease and antiphospholipid antibody positivity. It was also found that the highest risk was seen in double-positive antiphospholipid antibodies/lupus anticoagulant patients, as is the case in this patient. Libman-Sacks endocarditis may affect 11% or more of patients with SLE and has no relationship to disease activity. The condition is associated with large verrucous lesions near the edge of the valve, most often the mitral valve. Typical lesions consist of immune complexes, mononuclear cells, and fibrin and platelet thrombi. Libman-Sacks endocarditis is usually asymptomatic but can be responsible for numerous complications, including embolic stroke, or in this case a transient ischemic attack in the territory of the ophthalmic artery, peripheral emboli, and infective endocarditis. Bacterial endocarditis can be a source of cardiogenic emboli, but 90% of patients with infective endocarditis have fever, and often other constitutional symptoms, which are absent in this patient. Patients with SLE are prone to develop premature atherosclerosis (and can be a cause of stroke or transient ischemia attack), but this patient has no findings of atherosclerosis involving the carotid artery; furthermore, this diagnosis cannot account for the patient's heart murmur. Transient monocular blindness can be a manifestation of giant cell arteritis, but this diagnosis is extremely rare in individuals less than 40 years old.
A 35-year-old woman is evaluated for constipation. She reports passage of hard stool every 3 to 4 days and associated bloating. Her symptoms have been present for more than 10 years and have progressed gradually. Trials of over-the-counter fiber supplementation and polyethylene glycol worsened the bloating, prompting discontinuation. Senna tea was ineffective. Bisacodyl caused abdominal cramping. She is otherwise healthy and currently takes no medication. On physical examination, vital signs and other findings are normal. Which of the following is the most appropriate treatment? Lactulose Linaclotide Methylnaltrexone Rifaximin
Linaclotide is a peripherally acting guanylate cyclase-C receptor agonist that is FDA approved for the treatment of chronic idiopathic constipation in adults with symptoms refractory to first-line therapies. Linaclotide is the most appropriate treatment for this patient. Linaclotide is a peripherally acting guanylate cyclase-C receptor agonist that is FDA approved for the treatment of chronic idiopathic constipation in adults. Linaclotide increases intracellular and extracellular cyclic guanosine monophosphate, which results in chloride and bicarbonate secretion into intestinal lumen, increasing intestinal fluid content and accelerated transit time. Its superiority to placebo in the treatment of constipation was demonstrated in two 12-week, high-quality randomized controlled trials. Diarrhea occurred in 16% of patients receiving linaclotide (compared to 4.7% in those receiving placebo) in the 12-week clinical trials. The potential for diarrhea can be minimized by taking linaclotide on an empty stomach, ideally 30 minutes before the first meal of the day. Plecanatide is the second guanylate cyclase-C receptor agonist to receive FDA approval for the treatment of chronic idiopathic constipation. Diarrhea was also the most commonly reported side effect of plecanatide therapy in the 12-week clinical trials (5% versus 1% in the placebo group). Lubiprostone, a chloride channel agonist, is a third agent with FDA approval for the treatment of chronic idiopathic constipation. The most common side effects reported in clinical trials were nausea, reported by 29% of patients taking lubiprostone, and diarrhea, reported by 12%. The efficacy and safety of linaclotide, plecanatide, and lubiprostone have not been established in patients aged younger than 18 years nor in pregnant patients. Because of cost effectiveness, over-the-counter laxatives, such as fiber supplements, polyethylene glycol, and bisacodyl, should be pursued first in patients with constipation, but they were ineffective in this patient. Osmotic laxatives include magnesium hydroxide, lactulose, sorbitol, and polyethylene glycol (PEG); clinical trials have demonstrated the superiority and safety of PEG. The patient tried over-the-counter PEG, and it was poorly tolerated due to bloating. Substituting another, less effective, osmotic agent (for example, lactulose) will lead to similar adverse effects without relieving constipation. Methylnaltrexone, an injectable peripheral opioid antagonist that does not cross the blood-brain barrier, is very effective in treating opioid-induced constipation without adversely affecting analgesia. Bowel obstruction is an absolute contraindication to methylnaltrexone. Because this patient does not have opioid-induced constipation, methylnaltrexone is an inappropriate choice. Rifaximin is a nonabsorbable antibiotic used in the treatment of irritable bowel syndrome predominated by diarrhea, hepatic encephalopathy, and travelers' diarrhea. It has no proven efficacy in chronic idiopathic constipation and is therefore not indicated in this patient.
A 69-year-old woman is seen in the office following a left thyroid lobectomy and isthmusectomy 1 week ago for management of compressive symptoms related to a large left thyroid nodule. The preoperative thyroid/neck ultrasound showed the nodule without suspicious features and no abnormal cervical lymph nodes. The pathology report describes a 4.5-cm left adenomatous nodule in a background of multinodular hyperplasia. There is a single focus of papillary thyroid carcinoma measuring 0.5 cm in the greatest dimension. No lymphovascular or extrathyroidal invasion is noted. Surgical margins are negative. The patient is currently feeling well and reports complete resolution of her prior symptoms. Her medical history is otherwise unremarkable, and she takes no medications. On physical examination, vital signs are normal. There is a well-healed anterior neck scar. Laboratory studies show a serum thyroid-stimulating hormone (TSH) level of 1.8 µU/mL (1.8 mU/L). Which of the following is the most appropriate treatment? Levothyroxine to suppress serum TSH Radioactive iodine (131I) therapy Resection of the remaining thyroid lobe No additional treatment
Lobectomy is the treatment of choice for low-risk papillary thyroid cancer that is confined to the thyroid gland, completely resected at surgery, does not demonstrate aggressive pathologic features (lymphovascular invasion or tall cell variant), and has not metastasized. No additional treatment is needed. The incidence of thyroid cancer has increased over the last four decades with much of this change attributable to a rise in the diagnosis of small noninvasive papillary thyroid carcinomas. The incidence of papillary microcarcinoma (<1 cm) is 5% to 15% in the United States based on autopsy series. Low-risk papillary thyroid cancer is that which is confined to the thyroid gland, completely resected at surgery, does not demonstrate aggressive pathologic features (lymphovascular invasion or tall cell variant), and has not metastasized. The risk of disease-related death is less than 1%, and the risk of structural disease recurrence is 1% to 2% for low-risk unifocal papillary microcarcinomas. Patients receiving either lobectomy or thyroidectomy have similarly excellent outcomes. Therefore, resection of the remaining thyroid lobe would not be required for this patient. Suppression of thyroid-stimulating hormone (TSH) with levothyroxine therapy may also be used to improve morbidity and reduce mortality, particularly in patients with persistent disease or distant metastases. The necessary degree of TSH suppression varies according to the risk of cancer progression and comorbidities of the patient. Patients with persistent disease typically require lowering of their TSH level to less than 0.1 μU/mL (0.1 mU/L), whereas patients who are disease-free with a low risk of recurrence should maintain a TSH level of 0.3 to 2.0 μU/mL (0.3-2.0 mU/L). This is typically accomplished without the use of thyroxine suppressive therapy. Patients with distant metastases have improved survival with successful radioiodine therapy, whereas administration of radioactive iodine may decrease the likelihood of recurrent disease in those patients with nodal metastases. Radioactive iodine (131I) therapy offers this patient no appreciable benefit since her prognosis is already excellent and such therapy would result in substantial unnecessary costs and radiation exposure.
A 47-year-old woman is evaluated for her skin "turning to leather." She notes that she had originally developed an itchy pink rash on the skin over her forearms, elbows, and upper arms. Eventually the rash turned to thick hard skin. She has similar skin findings in the area of her waistband. She denies symptoms of Raynaud phenomenon. On physical examination, vital signs are normal. Skin findings are shown. There are no changes over the face or lips. Which of the following is the most likely diagnosis? Diffuse cutaneous systemic sclerosis Limited cutaneous systemic sclerosis (CREST) Localized scleroderma (morphea) Scleredema Scleromyxedema
Localized scleroderma (morphea) is characterized as isolated sclerotic circumscribed plaques; it is not associated with Raynaud phenomenon or systemic disease and does not include sclerodactyly. This patient has localized scleroderma, or morphea. Localized scleroderma is one of the scleroderma spectrum disorders, a group of diseases sharing the feature of skin hardening. The scleroderma spectrum disorders include systemic sclerosis (SSc), localized scleroderma, and scleroderma-like conditions. SSc is further characterized as two distinct subsets based on skin involvement: limited SSc and diffuse SSC. In limited SSc, scleroderma is restricted to the fingers and hands and, to a lesser extent, the face and neck. Scleroderma involving the fingers and hands, chest, abdomen, forearms, upper arms, and shoulders indicates diffuse SSc. Morphea is a localized form of scleroderma limited to one or more indurated plaques confined to the torso and proximal extremities. Although histologic findings of morphea are identical to those seen in patients with SSc, visceral manifestations and Raynaud phenomenon are absent. This patient has a generalized variant of morphea called pansclerotic morphea, which often appears in areas of friction and pressure, such as the waistband area or inframammary regions. Diffuse and limited SSc involve the fingers first, and sclerosis proceeds proximally, sometimes very rapidly. Scleredema has a variable clinical presentation but is most easily recognizable as large, noninflammatory woody-indurated plaques over the shoulder girdle, neck, and upper extremities. In adults, scleredema occurs most commonly as a complication of diabetes mellitus but may be seen following a streptococcal infection in children or as a complication of monoclonal gammopathy. Scleromyxedema is characterized by deposition of mucin with large numbers of stellate fibroblasts in the dermis, presenting as waxy yellow-red papules overlying thickened skin. It affects the face, upper torso, and upper extremities. The condition is rare but is most frequently associated with paraproteinemia and may therefore occur in the setting of multiple myeloma or AL amyloidosis. In a few patients, a mild inflammatory myopathy may accompany the disorder.
A 49-year-old woman is evaluated for recently worsening joint symptoms. She has a 13-year history of Crohn disease characterized by four to six stools daily and mild crampy abdominal pain. She also has a 1-year history of arthritis. She currently has pain in the left knee, right ankle, and two joints of the right foot; diffuse swelling involving the left third toe; and 30 minutes of morning stiffness. She has been treated with various NSAIDs, which seem to worsen her bowel disease. She has tried, in succession, azathioprine, mesalamine, and methotrexate, without notable improvement in her symptoms. She currently is taking methotrexate. On physical examination, vital signs are normal. The left knee has a small effusion. Dactylitis of the left third toe is present. The right ankle and the right second and third metatarsophalangeal joints are tender to palpation. Which of the following is the most appropriate long-term treatment? Adalimumab Intra-articular glucocorticoid injections Prednisone Rituximab
Long-term treatment options for bowel and joint symptoms associated with inflammatory bowel disease include sulfasalazine, azathioprine, 6-mercaptopurine, methotrexate, and the tumor necrosis factor α inhibitors adalimumab, certolizumab pegol, golimumab, and infliximab. Adalimumab is the most appropriate treatment for this patient with inflammatory bowel disease-associated arthritis. Various pharmacologic agents may be useful in the treatment of both intestinal manifestations and peripheral arthritis related to Crohn disease and ulcerative colitis, including sulfasalazine, azathioprine, 6-mercaptopurine, methotrexate, glucocorticoids, and certain tumor necrosis factor (TNF)-α inhibitors. Adalimumab, certolizumab pegol, golimumab, and infliximab are more effective than other TNF-α inhibitors in treating the combination of bowel and joint manifestations. This patient with Crohn disease has developed peripheral oligoarthritis and dactylitis ("sausage digit"). She also has symptomatic bowel disease. She is most likely to improve her bowel and joint disease by the addition of a TNF-α inhibitor. Intra-articular glucocorticoid injections can be used to treat inflammatory arthritis. However, the duration of symptom relief can be short term, practical use is limited by the size and number of joints involved, and this therapy will have no effect on the patient's dactylitis or inflammatory bowel disease. Glucocorticoids are effective for inducing but not maintaining remission in Crohn disease. Although prednisone could be used to improve the patient's joint and bowel symptoms on a short-term basis, it is unlikely to control her bowel and joint symptoms in the long term and is increasingly likely to be associated with significant side effects the longer it is used. Rituximab depletes B cells and is used in combination with methotrexate to treat rheumatoid arthritis in patients who have not adequately responded to a TNF-α inhibitor. Rituximab is not indicated for this patient's arthritis or bowel disease and is not recommended.
A 45-year-old man is seen for a routine evaluation of his blood pressure. He has gained 1.5 kg (3.3 lb) since his last visit 3 weeks ago. History is significant for stage G4 chronic kidney disease, hypertension, type 2 diabetes mellitus, and coronary artery disease. Medications are amlodipine, lisinopril, carvedilol, chlorthalidone, basal and prandial insulin, atorvastatin, and low-dose aspirin. On physical examination, blood pressure is 165/100 mm Hg, pulse rate is 58/min, and respiration rate is 16/min. There is 1+ bilateral leg edema. The remainder of the physical examination is noncontributory. Laboratory studies: Blood urea nitrogen 44 mg/dL (15.7 mmol/L) Creatinine 2.8 mg/dL (247.5 µmol/L) Potassium 5.4 mEq/L (5.4 mmol/L) Estimated glomerular filtration rate 26 mL/min/1.73 m2 Urinalysis Normal In addition to maintaining a low sodium diet, which of the following is the most appropriate treatment of this patient's blood pressure? Add hydralazine Add losartan Stop amlodipine; begin spironolactone Stop chlorthalidone; begin furosemide
Loop diuretics are a cornerstone of blood pressure management in patients with advanced chronic kidney disease. Stopping chlorthalidone and beginning furosemide is the most appropriate treatment for this patient's hypertension. Hypertension is a risk factor for cardiovascular morbidity and mortality. The most common cause of death in patients with chronic kidney disease (CKD) is cardiovascular disease. Therefore, blood pressure control is critical for preventing disease and death. Diuretics are central to the management of hypertension. Although thiazide diuretics (such as chlorthalidone) are recommended first-line agents for hypertension, efficacy decreases with advanced stages of CKD. Therefore, loop diuretics (furosemide, bumetanide, torsemide) are a cornerstone of blood pressure management in patients with advanced CKD. Sodium retention and impaired natriuresis lead to volume expansion and an increase in blood pressure. Loop diuretics are effective natriuretics and retain their activity even at low glomerular filtration rate (GFR); however, doses need to be increased as GFR declines to maintain appropriate urine output and negative fluid balance. Loop diuretics are equally effective if given in equipotent doses. The loop diuretic furosemide would effectively treat this patient's volume expansion and therefore reduce blood pressure. Additional benefit would include increased urine potassium and acid excretion to treat the mild hyperkalemia and metabolic acidosis associated with CKD. The most common side effects of loop diuretics are electrolyte and fluid abnormalities, hypersensitivity, and ototoxicity. Hydralazine is not indicated for this patient who has volume expansion and needs diuresis; this drug is also relatively contraindicated in coronary artery disease. Losartan is an angiotensin receptor blocker, which should not be combined with an ACE inhibitor due to the risk of hyperkalemia and acute kidney injury. Although spironolactone can help reduce blood pressure and edema through its anti-aldosterone effects and blockade of sodium reabsorption in the cortical collecting duct, it would be contraindicated in this patient with advanced CKD and hyperkalemia.
A 27-year-old woman is evaluated for itchy bumps on her abdomen. The patient is pregnant and in her third trimester. She was diagnosed with pruritic urticarial papules and plaques of pregnancy (PUPPP). On physical examination, vital signs are normal. Skin findings are shown. Which of the following is the most appropriate treatment? Early delivery Topical glucocorticoids Topical imiquimod Ursodeoxycholic acid
Low- to medium-potency topical glucocorticoids are first-line therapy for pruritic urticarial papules and plaques of pregnancy (PUPPP). The patient has pruritic urticarial papules and plaques of pregnancy (PUPPP), and the most appropriate treatment is topical glucocorticoids. PUPPP is a self-limiting condition that typically occurs in primiparous women at the end of the third trimester or immediately postpartum. Lesions are characterized as extremely pruritic, erythematous papules within striae, most often on the abdomen with sparing of the umbilicus. The lesions can be confined to the abdomen or spread to the extremities and torso. If the lesions coalesce, urticarial plaques within striae can be an additional finding. The symptoms typically last 4 to 6 weeks and usually abate by the second postpartum week. Topical glucocorticoids can help relieve some of the symptoms. Glucocorticoids are pregnancy category C, but are generally thought to be safe, especially topical formulations. Early delivery is rarely required to manage PUPPP. Most patients can be managed successfully with topical glucocorticoids and oral antihistamines. On rare occasions a burst of oral glucocorticoids with a quick taper can be used if the symptoms are particularly severe, and this approach is likely preferable to early delivery. Topical imiquimod is an immunomodulator typically used for actinic keratoses, superficial basal cell carcinoma, and genital warts. It is not appropriate treatment for PUPPP. Ursodeoxycholic acid is used in intrahepatic cholestasis of pregnancy (ICP). Symptoms include pruritus in most patients and jaundice in 10% to 25% of patients. Elevated serum bile acids are diagnostic of ICP. Ursodeoxycholic acid is effective for ICP. Fetal complications such as preterm labor, fetal distress, and intrauterine death are increased in ICP. Intrauterine deaths can occur late in gestation, and women with proven ICP typically are induced at 36 to 38 weeks of gestation. The patient's pruritic urticarial papules located on the abdomen are not compatible with the diagnosis of intrahepatic cholestasis of pregnancy, and ursodeoxycholic acid is not indicated.
A 58-year-old man with a history of severe COPD is evaluated for his chronic exertional dyspnea. He is taking his inhalers and medications as prescribed with excellent technique and has completed a pulmonary rehabilitation program within the last 6 months and is continuing his exercise program. He is using supplemental oxygen but despite adequate oxygenation still has significant exertional dyspnea. Current medications are umeclidinium/vilanterol, mometasone, and albuterol. He feels that his current quality of life is poor and would like other treatment options. On physical examination, vital signs are normal. Oxygen saturation is 90% on 2 L/min of oxygen. BMI is normal. Pulmonary examination reveals decreased breath sounds throughout with a prolonged expiratory phase but no wheezing. Spirometry demonstrates severe airflow obstruction. A recent chest CT shows heterogeneous emphysema without any nodules. A recent echocardiogram shows diastolic dysfunction but no evidence of pulmonary hypertension. Which of the following is the most appropriate management? Add daily roflumilast Evaluation for lung volume reduction surgery Obtain a right heart catheterization Repeat pulmonary rehabilitation program
Lung volume reduction surgery improves quality of life and survival for patients with upper-lobe predominant emphysema and significant exercise limitations.
A 58-year-old man with a history of severe COPD is evaluated for his chronic exertional dyspnea. He is taking his inhalers and medications as prescribed with excellent technique and has completed a pulmonary rehabilitation program within the last 6 months and is continuing his exercise program. He is using supplemental oxygen but despite adequate oxygenation still has significant exertional dyspnea. Current medications are umeclidinium/vilanterol, mometasone, and albuterol. He feels that his current quality of life is poor and would like other treatment options. On physical examination, vital signs are normal. Oxygen saturation is 90% on 2 L/min of oxygen. BMI is normal. Pulmonary examination reveals decreased breath sounds throughout with a prolonged expiratory phase but no wheezing. Spirometry demonstrates severe airflow obstruction. A recent chest CT shows heterogeneous emphysema without any nodules. A recent echocardiogram shows diastolic dysfunction but no evidence of pulmonary hypertension. Which of the following is the most appropriate management? Add daily roflumilast Evaluation for lung volume reduction surgery Obtain a right heart catheterization Repeat pulmonary rehabilitation program
Lung volume reduction surgery improves quality of life and survival for patients with upper-lobe predominant emphysema and significant exercise limitations. Evaluation for lung volume reduction surgery is the most appropriate management for this patient with heterogeneous emphysema, significant exercise limitation, and poor quality of life. Lung volume reduction surgery excises areas of emphysematous lung, improves the mechanical efficiency of respiratory muscles, and increases the elastic recoil of the lungs to improve expiratory flow and reduce exacerbations. The National Emphysema Treatment Trial (NETT) demonstrated that overall, lung-volume-reduction surgery increases the chance of improved exercise capacity but does not confer a survival advantage over medical therapy. It does yield a survival advantage for patients with both predominantly upper-lobe emphysema and low base-line exercise capacity. Symptomatic improvement with the surgery appears to be durable. It is reasonable to evaluate this patient with heterogenous emphysema, poor exercise capacity, and reduced quality of life for lung volume reduction surgery. Roflumilast is an oral selective phosphodiesterase-4 inhibitor. It is used primarily as add-on therapy in severe COPD associated with chronic bronchitis and a history of recurrent exacerbations despite other therapies; it has been shown to relieve symptoms and reduce risk and frequency of exacerbations in these individuals. However, it is not a bronchodilator, is expensive, and has not been shown to be effective in other groups of patients with COPD. It is not indicated in the treatment of primary emphysema and has not been shown to decrease exertional dyspnea and would not benefit this patient. Although patients with chronic lung conditions can develop pulmonary hypertension, this patient had an unremarkable echocardiogram and has no evidence of pulmonary hypertension on examination. Therefore, proceeding with a right heart catheterization is unnecessary and unlikely to change management at this time. This patient has already participated in a pulmonary rehabilitation program. Although it is helpful for patients to continue exercise, restarting a formal pulmonary rehabilitation program is unlikely to provide him with any more significant symptomatic improvement.
A 67-year-old man is evaluated in follow-up for urinary incontinence. Six months ago, he began tamsulosin for occasional nocturia, frequency, and urgency related to benign prostatic hyperplasia. Tamsulosin decreased the frequency of nocturia, but he continued to have daytime urinary urgency with a few occasions of urine leakage. He attempted to control his symptoms with behavioral modification, including bladder training and scheduled voiding, but he still has episodes of urgency and leakage. He prefers not to undergo any surgical intervention. Medical history is otherwise significant for heart failure with preserved ejection fraction. Medications include benazepril, carvedilol, furosemide, spironolactone, tamsulosin, and aspirin. On physical examination, blood pressure is 102/60 mm Hg, and pulse rate is 72/min. Other vital signs and the remainder of the physical examination are normal. Bladder ultrasonography shows a postvoid residual urine volume of 30 mL. Which of the following is the most appropriate treatment? Dutasteride Intermittent bladder catheterization Mirabegron Sacral nerve root neurostimulation
Male patients with urgency urinary incontinence who have not achieved satisfactory relief of symptoms with behavioral therapy may benefit from the use of anticholinergic agents or mirabegron
A 67-year-old man is evaluated in follow-up for urinary incontinence. Six months ago, he began tamsulosin for occasional nocturia, frequency, and urgency related to benign prostatic hyperplasia. Tamsulosin decreased the frequency of nocturia, but he continued to have daytime urinary urgency with a few occasions of urine leakage. He attempted to control his symptoms with behavioral modification, including bladder training and scheduled voiding, but he still has episodes of urgency and leakage. He prefers not to undergo any surgical intervention. Medical history is otherwise significant for heart failure with preserved ejection fraction. Medications include benazepril, carvedilol, furosemide, spironolactone, tamsulosin, and aspirin. On physical examination, blood pressure is 102/60 mm Hg, and pulse rate is 72/min. Other vital signs and the remainder of the physical examination are normal. Bladder ultrasonography shows a postvoid residual urine volume of 30 mL. Which of the following is the most appropriate treatment? Dutasteride Intermittent bladder catheterization Mirabegron Sacral nerve root neurostimulation
Male patients with urgency urinary incontinence who have not achieved satisfactory relief of symptoms with behavioral therapy may benefit from the use of anticholinergic agents or mirabegron This patient reports symptoms consistent with urgency urinary incontinence, which can be best addressed with behavioral training and the use of anticholinergic agents or mirabegron. Urgency incontinence is characterized by loss of urine accompanied by a sense of urgency. The treatment of urinary incontinence generally progresses in a stepwise manner. Lifestyle changes and behavioral therapy should be initiated first, followed by pharmacologic therapy and devices, and finally surgery if all other therapies have failed. The patient is already appropriately using behavioral therapy in the form of bladder training and scheduled voiding. The addition of pharmacologic therapy is now appropriate. Anticholinergic drugs (darifenacin, fesoterodine, oxybutynin, solifenacin, tolterodine, trospium) reduce involuntary bladder contractions by blocking the muscarinic cholinergic receptors. Anticholinergic medications are appropriate for both men and women with urgency urinary incontinence, but caution should be exercised when initiating them in men with benign prostatic hyperplasia due to risk for urinary retention. The β-agonist mirabegron, another pharmacologic option for treatment of urgency urinary incontinence, enhances the inhibitory adrenergic signals to the detrusor muscle. Clinicians should base the choice of pharmacologic agents on tolerability, adverse effect profile, ease of use, and cost of medication. Dutasteride is a 5α-reductase inhibitor used to treat benign prostatic hyperplasia. In this patient who is already being treated with tamsulosin and in whom postvoid residual bladder volume suggests that bladder outlet obstruction has been adequately addressed, there is no additional benefit from adding another therapy for benign prostatic hyperplasia; this therapy will not address the urgency and incontinence problems. Intermittent self-catheterization might be a useful strategy for a patient with overflow incontinence due to bladder outlet obstruction. However, that is not the case, as demonstrated by this patient's bladder ultrasound, which shows a postvoid residual urine volume of only 30 mL. Sacral nerve root stimulation is an acceptable treatment for urgency urinary incontinence in patients in whom behavioral and pharmacologic therapies fail. Placement of a sacral nerve root stimulator typically involves conscious sedation and may require general anesthesia.
A 29-year-old man is evaluated during a new patient visit. He is concerned about gradual weight gain over the last several years. He does not pay much attention to his diet and describes his lifestyle as generally sedentary. He does not use tobacco products and does not drink more than one to two alcoholic drinks per week. He reports no problems with nonrestorative sleep or daytime hypersomnolence. He takes no medications. On physical examination, blood pressure is 149/90 mm Hg, and pulse rate is 87/min. BMI is 35. His waist circumference is 107 cm (42 in). The remainder of the physical examination is normal. Which of the following is the most appropriate test to perform next? Exercise stress test Hepatic ultrasonography Overnight polysomnography Serum lipid panel Thyroid function studies
Management guidelines for patients with overweight or obesity recommend measurement of height and weight; calculation of BMI; measurement of waist circumference; and measurement of cardiovascular disease risk factors, including blood pressure, fasting blood glucose (or hemoglobin A1c), and serum lipid levels. This patient with obesity should undergo serum lipid screening. The American Heart Association, American College of Cardiology, and The Obesity Society guideline for the management of overweight and obesity recommends measurement of height and weight; calculation of BMI; measurement of waist circumference; and measurement of cardiovascular disease risk factors, including blood pressure, fasting blood glucose (or hemoglobin A1c), and serum lipid levels. Obesity is associated with insulin resistance and dyslipidemia characterized by elevations of total cholesterol, LDL cholesterol, VLDL cholesterol, and triglycerides and a reduction in HDL cholesterol. The history should address symptoms of obesity-related comorbid conditions, but no additional screening is recommended in the absence of suggestive symptoms or findings. Exercise stress testing is recommended for patients with symptoms of cardiac ischemia. Patients with overweight or obesity are at increased risk for cardiovascular disease and should be questioned about symptoms and referred for testing if symptoms are present. Routine screening of asymptomatic patients is not recommended. Hepatic ultrasonography is not necessary for this patient. The risk for nonalcoholic steatohepatitis is increased in patients with overweight or obesity. A presumptive diagnosis can be made in a patient with mild abnormalities in aminotransferase levels, risk factors for nonalcoholic fatty liver disease (such as diabetes mellitus, obesity, and hyperlipidemia), and ultrasonographic features consistent with hepatic steatosis. The recommended treatment of nonalcoholic steatohepatitis is weight loss, but no screening for this condition is recommended. Risk for obstructive sleep apnea is increased among patients with overweight or obesity, particularly those with neck circumference greater than 38 cm (15 in) in women and greater than 43 cm (17 in) in men. The patient should be referred for overnight polysomnography only if symptoms of the disorder are present. He does not report nonrestorative sleep or daytime hypersomnolence, which would suggest obstructive sleep apnea. The prevalence of an endocrine cause of obesity is very low. Thyroid function testing should be reserved for patients with symptoms and findings indicating thyroid disease.
A 30-year-old woman is evaluated for an acne flare on her face. She is 12 weeks pregnant. When she was younger, she was on a course of isotretinoin, and it cleared her acne. She is otherwise in good health and has not been on any medications other than a prenatal vitamin since she became pregnant. On physical examination, her vital signs are normal. Skin findings are shown. The remainder of the examination is normal. Which of the following is the most appropriate treatment for the patient? Oral doxycycline Oral isotretinoin Oral spironolactone Topical erythromycin Topical tazarotene
Management of dermatologic conditions during pregnancy with topical agents should be considered before prescribing systemic medications because they are lower risk, with the exception of tazarotene (category X). The patient has some mild inflammatory and comedonal acne that would be best treated with topical erythromycin. There are two main types of acne lesions: inflammatory and noninflammatory. The prototypical lesion of noninflammatory acne is the comedone, which results from follicular plugging by keratin. Comedones can be open ("blackheads") or closed ("whiteheads"). Acne typically starts with closed comedones that are visible as subtle small 1- to 2-mm flesh-colored papules. Inflammatory acne consists of erythematous pustules, nodules, or cysts (deeper and often painful) that occur later and are the result of inflammation in and around the hair follicle. Lesion morphology should guide treatment. It is also important to consider the safety of medications used for acne treatment during pregnancy. Topical erythromycin is pregnancy category B and can be used safely during pregnancy. To reduce the incidence of bacterial resistance, topical erythromycin can be combined with topical benzoyl peroxide, also safe in pregnancy. Doxycycline is pregnancy category D and can cause dental staining and enamel hypoplasia when used in the second and third trimesters; it is not to be used during pregnancy. Isotretinoin is a teratogen (pregnancy category X) and can cause fetal loss, especially when used in the first trimester. It can also cause severe head and cardiac defects. Oral isotretinoin is reserved for the treatment of severe nodulocystic acne and gram-negative folliculitis in nonpregnant patients. Patients must be monitored carefully once therapy with isotretinoin is initiated and participation in the risk prevention program iPLEDGE is mandated for all patients, providers, and dispensing pharmacies. Mandatory office visits are required every 30 days with regular monitoring for pregnancy and medication side effects. Acne in women, particularly those whose acne flares around menses, may respond to spironolactone. Spironolactone is pregnancy category C, and in rat studies has caused delayed sexual maturation in female rats and feminization of male rat fetuses. Topical retinoids are appropriate for comedonal and mild inflammatory acne. Tazarotene is a topical retinoid, but is rated pregnancy category X and should not be used in this patient.
A 48-year-old woman is evaluated in the emergency department for lower extremity weakness, nausea, and increased somnolence occurring during the past 24 hours. She had constipation for 3 days, for which she drank one bottle of milk of magnesia each night. History is significant for hypertension as well as stage G4 chronic kidney disease secondary to autosomal dominant polycystic kidney disease. Her only medication is lisinopril. On physical examination, temperature is 36.6 °C (97.9 °F), blood pressure is 94/54 mm Hg, pulse rate is 58/min, respiration rate is 16/min, and oxygen saturation is 92% breathing ambient air. Bilateral flank fullness is present. Deep tendon reflexes are diminished diffusely. Strength in the lower extremities is 3/5. Laboratory studies: Calcium 8.0 mg/dL (2 mmol/L) Creatinine 3.9 mg/dL (344.8 µmol/L) Electrolytes : Sodium 138 mEq/L (138 mmol/L) Potassium 3.7 mEq/L (3.7 mmol/L) Chloride 104 mEq/L (104 mmol/L) Bicarbonate 22 mEq/L (22 mmol/L) Magnesium 8.1 mg/dL (3.3 mmol/L) Phosphorus 4.4 mg/dL (1.4 mmol/L) In addition to administration of 0.9% saline and furosemide, which of the following is the most appropriate treatment? Hemodialysis Intravenous calcium Intravenous potassium Intravenous sodium bicarbonate Oral sodium polystyrene sulfonate
Management of hypermagnesemia includes discontinuation of magnesium-containing medications, administration of saline diuresis to enhance magnesium excretion, and administration of intravenous calcium to treat severe symptoms. In addition to administration of 0.9% saline and furosemide, intravenous calcium is appropriate treatment for this patient with hypermagnesemia. Because the kidney can efficiently eliminate magnesium, hypermagnesemia occurs infrequently and most commonly results from excessive intake in the setting of decreased kidney function. Numerous medications such as antacids and laxatives contain magnesium, and magnesium sulfate is the treatment of choice for prevention of eclampsia. Hypermagnesemia is usually not associated with significant symptoms until the level is >7.2 mg/dL (2.9 mmol/L). Symptoms include somnolence, headache, loss of deep tendon reflexes, bradycardia, hypotension, and hypocalcemia. At levels >12 mg/dL (4.9 mmol/L), flaccid paralysis, respiratory failure, and complete heart block can occur. Hypermagnesemia is usually self-limited. Magnesium-containing medications should be discontinued, and magnesium excretion can be enhanced with saline diuresis. In patients with hypotension and significant neuromuscular deficits, treatment is aimed at direct antagonism of the effects of hypermagnesemia, which is accomplished by intravenous administration of calcium. After this, efforts to lower the serum level should be instituted. If kidney function is adequate, a trial of 0.9% saline and furosemide will increase kidney excretion. Magnesium-containing agents should be limited or avoided in individuals with kidney disease. If kidney failure is advanced, hemodialysis will effectively lower magnesium levels. However, this would require access to the central venous system and mobilization of the dialysis team, which can take several hours, and in the interim patients with symptomatic hypermagnesemia should be given intravenous calcium. There is no role for either intravenous potassium or sodium bicarbonate other than correcting preexisting metabolic abnormalities. Although sodium polystyrene sulfonate will bind a small amount of magnesium, this effect is minimal and not advocated as a treatment.
A 38-year-old man is evaluated during a follow-up visit for elevated blood pressure found for the first time during his last visit. He reports back pain of several weeks' duration after an episode of heavy lifting at work. History is also notable for seasonal allergies. He currently takes ibuprofen daily for the back pain and loratadine as needed for allergies. On physical examination, the patient is well developed and muscular, and in no apparent distress. The average of three blood pressure measurements is 139/84 mm Hg, and pulse rate is 52/min; other vital signs are normal. BMI is 26. The remainder of the examination is normal. Office electrocardiogram is normal. In addition to follow-up in 1 month, which of the following is the most appropriate management? Begin amlodipine Begin hydrochlorothiazide Discontinue ibuprofen Discontinue loratadine
Many medications, such as NSAIDS, can result in reversible elevations in blood pressure; discontinuation of the drug and a reassessment of blood pressure 1 month later are necessary to confirm a return to normal blood pressure measurement.
A 61-year-old woman is evaluated for follow-up of stage IVA hypopharyngeal squamous cell carcinoma, which was treated with combined cisplatin and radiation therapy 1 year ago. She has xerostomia and some dysphagia since completing therapy. She has a 30-pack-year smoking history and quit smoking 1 year ago. She takes no medications. On physical examination, vital signs are normal. BMI is 21. There is chronic induration of the right neck and limitation in neck range of motion, findings that have been noted on previous examinations. There is no palpable lymphadenopathy. Laboratory studies reveal a normal thyroid-stimulating hormone level. Recent laryngoscopy identified no evidence of recurrent cancer or new primary cancer. Which of the following is the most appropriate screening or surveillance test to perform next? Head and neck CT scan Low-dose chest CT scan PET/CT scan Thyroid ultrasonography
Many patients with head and neck cancer have significant smoking histories and should be offered lung cancer screening if they meet high-risk criteria.
A 61-year-old woman is evaluated for follow-up of stage IVA hypopharyngeal squamous cell carcinoma, which was treated with combined cisplatin and radiation therapy 1 year ago. She has xerostomia and some dysphagia since completing therapy. She has a 30-pack-year smoking history and quit smoking 1 year ago. She takes no medications. On physical examination, vital signs are normal. BMI is 21. There is chronic induration of the right neck and limitation in neck range of motion, findings that have been noted on previous examinations. There is no palpable lymphadenopathy. Laboratory studies reveal a normal thyroid-stimulating hormone level. Recent laryngoscopy identified no evidence of recurrent cancer or new primary cancer. Which of the following is the most appropriate screening or surveillance test to perform next? Head and neck CT scan Low-dose chest CT scan PET/CT scan Thyroid ultrasonography
Many patients with head and neck cancer have significant smoking histories and should be offered lung cancer screening if they meet high-risk criteria. The most appropriate diagnostic test to perform next is low-dose noncontrast CT scan of the chest. Following definitive treatment for head and neck cancer, patients require ongoing surveillance to look for evidence of recurrence as well as evidence of second primary head and neck cancer. Especially in patients with smoking-related cancers, the risk of a second primary cancer is substantial. Surveillance consists of history, physical examination, and laryngoscopy to examine the entire upper aerodigestive tract. This patient meets the criteria for screening for lung cancer, independent of the previous head and neck cancer. The U.S. Preventive Services Task Force (USPSTF) recommends annual low-dose CT imaging of the thorax for all patients 50 to 80 years of age who have a 20-pack-year smoking history and currently smoke or who have quit within the past 15 years. The history of head and neck cancer further increases the risk for subsequent lung cancer in this patient and heightens the importance of such screening. There is currently no established role for imaging studies of the head and neck or PET/CT scan in the absence of signs or symptoms indicative of metastatic disease. Imaging is indicated, however, for evaluation of concerning signs or symptoms suggestive of cancer recurrence or a new primary cancer. The most clearly defined risk factor for thyroid cancer is radiation exposure of the thyroid during childhood. Radiation treatment of childhood cancers is the most likely source of exposure, but other sources include fallout from atomic weapons and nuclear power plant accidents. There are no recommendations for thyroid cancer screening after radiation exposure in adulthood. Such patients should be screened periodically for hypothyroidism with a serum thyroid-stimulating hormone level, but routine thyroid ultrasonography is not recommended.
A 49-year-old man is evaluated for a 4-month history of cough, chest pressure, and double vision. He has no fever, night sweats, or weight loss. He has never smoked. On physical examination, vital signs are normal. There is ptosis bilaterally. The remainder of the physical examination is normal. A CT scan of the chest is shown. Which of the following is the most appropriate test to perform next? Acetylcholine receptor antibody α-Fetoprotein β-Human chorionic gonadotropin Lactate dehydrogenase
Masses in the anterior mediastinum are usually recalled as the "terrible T's": thymoma, teratoma/germ cell tumor, "terrible" lymphoma, and thyroid. An acetylcholine receptor (AChR) antibody test should be ordered. The mediastinum can be divided into three separate compartments (anterior, middle, and posterior), which can help narrow the differential diagnosis of a mediastinal mass. Each compartment normally contains separate and distinct anatomic structures that can lead to development of a mass. Patients may be asymptomatic and are often diagnosed after obtaining a chest radiograph for another reason, whereas others present with symptoms related to compression of adjacent structures. For example, they may present with dyspnea if the airway is compressed from a nearby mass or with upper extremity edema if vascular structures are compressed. This patient has an anterior mediastinal mass (arrow) and neurologic symptoms. Masses in this location are usually remembered as the "terrible T's": thymoma, teratoma/germ cell tumor, "terrible" lymphoma, and thyroid. Additional considerations include thoracic aneurysm. Thymomas are the most common cause of an anterior mediastinal mass. Patients usually present as middle-age adults and may develop paraneoplastic syndromes. For example, myasthenia gravis can develop in 30% to 50% of patients with a thymoma. In comparison, only 10% to 15% of patients with myasthenia gravis have a thymoma. The second most common cause is lymphoma; these patients are typically younger at the time of presentation. Other less common paraneoplastic syndromes include pure red blood cell aplasia, nonthymic cancers, and acquired hypogammaglobulinemia. Myasthenia gravis is an autoimmune disorder of the neuromuscular junction that is characterized by fatigable (or fluctuating) muscular weakness. Common symptoms are ptosis and diplopia, which this patient has. Diagnosis of myasthenia gravis is based on clinical findings, detection of disease-specific antibodies (acetylcholine receptor antibodies in 90% of patients and anti-muscle-specific kinase [MuSK] antibodies in another 5% [with 5% of patients remaining antibody negative]), and electromyography findings (such as a characteristic decremental response to repetitive stimulation). Thymectomy should be performed in all patients with thymoma. α-Fetoprotein and β-human chorionic gonadotropin are commonly elevated in germ cell tumors, which are also part of the differential diagnosis of an anterior mediastinal mass. However, this patient also has symptoms of myasthenia gravis, which is not associated with germ cell tumors. Lactate dehydrogenase is commonly elevated in lymphoma and seminomas but is not as specific as other tumor markers and will not help determine the cause of this patient's symptoms.
A 36-year-old man is evaluated for severe muscle cramping. The patient reports occasional painful muscle cramps, sometimes associated with brownish urine discoloration, after intense physical activity since he was a teenager and intermittent proximal muscle pain; he has had no stiffness, rigidity, or other muscle weakness or pain. He says that if he briefly interrupts his exercise routine after onset of cramping, he can continue further exercise without difficulty; eating carbohydrate-rich food also increases his exercise tolerance. On physical examination, vital signs are normal. "Mild" muscle weakness (inability to hold a limb against full resistance) and pain is present in the shoulder and limb girdle muscles. Muscle tone and bulk are normal, as are all reflexes. Results of sensory examination are unremarkable. Normal relaxation after hand grip is noted. Results of laboratory studies show a serum creatine kinase level of 500 U/L and a serum creatinine level of 1.8 mg/dL (159 µmol/L); urinalysis results are normal. Which of the following is the most likely diagnosis? Acute intermittent porphyria Limb-girdle muscular dystrophy McArdle disease (metabolic myopathy) Myotonic dystrophy Polymyositis
McArdle disease is a metabolic myopathy associated with exercise intolerance, cramping, myalgia, mild weakness, "second-wind" phenomenon, and myoglobinuria. This patient has metabolic myopathy consistent with McArdle disease (glycogen storage disease V). This diagnosis is supported by a history of exercise intolerance associated with myalgia, "mild" weakness, and myoglobinuria and by laboratory evidence of chronic kidney disease (elevated serum creatinine level secondary to myoglobinuria). McArdle disease is caused by myophosphorylase deficiency and may be underdiagnosed because of the variability and intermittent nature of its symptoms. Patients with McArdle disease often do not have any muscle weakness or atrophy at baseline evaluation but may develop transient "mild" weakness after exertion. They can protect against severe exercise intolerance by eating a carbohydrate-rich diet before exercise; they also often describe a characteristic "second-wind" phenomenon by which a brief break taken after onset of cramping and fatigue allows for a higher level of exercise tolerance with much less difficulty. Acute intermittent porphyria (AIP) is characterized by acute attacks of abdominal pain and vomiting and by central, peripheral, sensory, motor, autonomic, and enteric nervous system abnormalities. Patients may have episodes of reddish-brown urine (porphobilin and porphyrins) during an acute attack. AIP does not have a myopathic presentation and is not associated with intermittent exercise intolerance or "second-wind" phenomenon. Limb-girdle muscular dystrophies are associated with baseline muscle atrophy and weakness and not with crises of exercise intolerance. Myotonic dystrophy is unlikely in the absence of myotonia (delayed muscle relaxation), fixed muscle weakness, stiffness, and multiorgan involvement. Crises of exercise intolerance and myoglobinuria are not typical in myotonic dystrophy. Symmetric painless proximal weakness of the arms and legs is the classic feature of polymyositis. Onset is acute or subacute, with disease progressing over weeks. A prolonged clinical course without baseline weakness and a history of exercise intolerance is not consistent with polymyositis.
A 26-year-old man is evaluated during a follow-up visit after presenting to an urgent care clinic for back pain 1 week ago. Laboratory studies at that time were significant for a serum creatinine level of 1.4 mg/dL (123.8 µmol/L); other laboratory studies, including urinalysis, were normal. A urine albumin-creatinine ratio obtained in preparation for this visit is 10 mg/g. He is a personal trainer, and his daily exercise regimen includes weightlifting. He states that his back pain has resolved. He occasionally takes ibuprofen; the last use was 1 week ago. He takes no over-the-counter supplements. On physical examination today, vital signs are normal. BMI is 29. The patient is muscular, without signs of obesity. There is no muscle tenderness. Which of the following is the most appropriate management? Avoid all NSAID medications Measure the serum creatine kinase level Measure the serum cystatin C level Schedule a kidney biopsy
Measurement of the serum cystatin C level is appropriate for this patient. Cystatin C may be preferable to creatinine to assess kidney function in individuals with higher muscle mass. An increase in muscle mass would be expected to result in an increase in serum creatinine level in the absence of change in kidney function. This muscular man with a BMI of 29 has increase in muscle mass. Because serum creatinine is derived from the metabolism of creatinine produced by muscle, a significant increase in muscle mass would be expected to increase serum creatinine. An elevation in serum creatinine could also occur with creatine supplements, which he is not taking. This patient has a normal urinalysis and no proteinuria, all of which indicate no evidence of underlying kidney disease. Cystatin C, which is cleared by the kidney, is produced by all nucleated cells; therefore, levels are less dependent on muscle mass. Cystatin C can also be used for more accurate glomerular filtration rate estimation in these patients as a component of the Chronic Kidney Disease Epidemiology Collaboration equation. Although NSAIDs can cause acute kidney injury, the remote and infrequent use by this patient is unlikely to have any effect on serum creatinine. The hemodynamic effects of NSAIDs will disappear within 24 hours of stopping the medication, and interstitial nephritis from NSAIDs is unlikely to present with occasional dosing and is usually associated with proteinuria. The adverse effects of renal fibrosis associated with NSAIDs are only seen with extensive and long-term use. Creatine kinase levels can be measured to evaluate for the presence of rhabdomyolysis. Rhabdomyolysis significant enough to cause kidney injury would be expected to result in myoglobinuria reflected by heme-positive urine in the absence of red cells. No blood was seen on urinalysis. In the absence of other changes suggesting glomerular or interstitial disease, a kidney biopsy is not necessary. Read Related TextNext Question
A 49-year-old woman is evaluated for a 3-year history of pelvic pain. An extensive evaluation has not found a clearly defined pathophysiologic or anatomic cause, and therapy has been targeted to general pain management lately. She has been a willing and cooperative participant in biofeedback, cognitive behavioral therapy, physical therapy, hypnosis, acupuncture, meditation, and stress-reduction techniques, without significant pain relief. Her pain has been unresponsive to multiple trials of nonopioid analgesics and antidepressants, and she has tried oral tapentadol and tramadol, which were also ineffective. She currently takes acetaminophen and gabapentin. Medical history is significant for end-stage kidney disease, for which she receives hemodialysis, and hypertension. Other medications are metoprolol succinate, amlodipine, intravenous iron, and an erythropoiesis-stimulating agent. Which of the following is the most reasonable treatment option for this patient's chronic pain? Oral immediate-release morphine sulfate Oral medical cannabis oil Oral methadone Topical lidocaine
Medical cannabis has demonstrated some efficacy in the treatment of chronic noncancer pain. A trial of oral medical cannabis oil would be a reasonable treatment in the management of this patient's chronic pain. Medical cannabis, although classified as a scheduled agent by the U.S. Drug Enforcement Administration on a federal level, has been approved by many states as a treatment for chronic pain. Current data on the effectiveness of medical cannabis for chronic pain are characterized by significant heterogeneity in both patient populations and cannabis preparations, although recent systematic reviews have demonstrated that cannabis has some efficacy in the treatment of chronic noncancer pain. Only two cannabinoid drugs (dronabinol and nabilone) are licensed for sale in the United States, and both drugs are available only in oral form. The pharmacokinetics of oral cannabis differ greatly from those of smoked cannabis, which has varying implications. Oral cannabis is slow in onset of action but produces more pronounced, and often unfavorable, psychoactive effects that last much longer than those experienced with smoking. On the other hand, smoked cannabis is quickly absorbed into the blood, and effects are immediate. However, examining the effects of smoked marijuana can be difficult because the absorption and efficacy of cannabis on symptom relief depend on subject familiarity with smoking and inhaling. This patient with end-stage kidney disease has a complex chronic pain syndrome that is unresponsive to multiple trials of nonpharmacologic and nonopioid analgesic therapies. If she resides in a state in which medical cannabis is available, oral medical cannabis oil would be a reasonable treatment option. Oral immediate-release morphine sulfate should be avoided in this patient with end-stage kidney disease who is receiving dialysis because it could cause opioid-induced neurotoxicity with repeated use. Oral methadone is a potent opioid agonist and N-methyl-D-aspartate receptor antagonist. Its complex pharmacokinetics and variable half-life restrict its general use, and it should not be prescribed by clinicians who lack experience in its management. Topical lidocaine does not penetrate into the deep myofascial tissues and would not be an effective agent for this patient with pelvic pain.
A 55-year-old man is evaluated for ascites. He recently went to the emergency department, where paracentesis was performed. He was then discharged for outpatient follow-up. He has a history of cirrhosis due to nonalcoholic steatohepatitis and also has hypertension. Endoscopy 3 months earlier showed small varices without stigmata, making prophylaxis for esophageal variceal bleeding unnecessary. His only medication is lisinopril. On physical examination, vital signs are normal; BMI is 28. Abdominal examination shows abdominal distention without tenderness. Laboratory studies of the ascitic fluid show a leukocyte count of 80/µL with 20% neutrophils and protein level of 1.6 g/dL (16 g/L). Serum studies show a creatinine level of 1.3 mg/dL (114.9 µmol/L) and sodium level of 134 mEq/L (134 mmol/L). An abdominal ultrasound from the emergency department shows changes consistent with cirrhosis. The portal vein and hepatic veins are patent with normal flow direction. A moderate amount of free-flowing ascites is seen. In addition to initiating a sodium-restricted diet, which of the following is the most appropriate next step in management? Discontinue lisinopril Initiate free-water restriction Initiate propranolol Insert an indwelling drain into the peritoneal cavity
Medications that decrease kidney perfusion, including NSAIDs, ACE inhibitors, and angiotensin receptor blockers, should be discontinued in patients with ascites. Discontinuing lisinopril is the most appropriate next step in the management of this patient with ascites. Blood pressure falls with worsening cirrhosis, resulting in reduced renal blood flow and glomerular filtration. A compensatory upregulation of the renin-angiotensin system results in increased levels of vasoconstrictors, including vasopressin, angiotensin, and aldosterone, which support systemic blood pressure and kidney function. ACE inhibitors and angiotensin receptor blockers impair the compensatory response to cirrhosis-related hypotension and thereby impair the ability to excrete excess sodium and water and may also affect survival. Medications that decrease kidney perfusion, including NSAIDs, ACE inhibitors such as lisinopril, and angiotensin receptor blockers, should be discontinued because their use often worsens ascites due to portal hypertension. The mainstay of therapy of ascites is to initiate dietary changes, restricting sodium intake to less than 2000 mg (87 mEq) daily. If sodium restriction does not result in significant improvement of ascites, the initiation of diuretic therapy with spironolactone with or without furosemide can be effective in increasing urinary sodium excretion. Free-water restriction can be useful for the management of dilutional hyponatremia that is sometimes seen in patients with advanced liver dysfunction. This patient has a normal serum sodium concentration, so free-water restriction is not indicated. Propranolol and other nonselective β-blockers are often used prophylactically for the prevention of variceal hemorrhage, but they do not have a role in the management of ascites. Furthermore, in some patients with ascites that is refractory to medical management, β-blockers may worsen clinical outcomes, including survival. Indwelling drains for ascites have been used for patients with malignant ascites, but in the setting of portal hypertensive ascites, such as seen in this patient, indwelling drains are associated with a high risk for infection and their use is contraindicated.
A 62-year-old woman is evaluated for management of her type 2 diabetes mellitus. She was diagnosed 2 years ago, and treatment was advanced to include lifestyle modifications, metformin, liraglutide, and empagliflozin, but it was not successful in reaching her hemoglobin A1c goal. Her current regimen includes metformin and basal and prandial insulins. Her fasting and preprandial blood glucose values range from 150 to 200 mg/dL (8.3-11.1 mmol/L) with intermittent episodes of hypoglycemia. She has had a weight gain of 2.3 kg (5 lb) over the last 3 months. Medical history is also significant for hypertension, hyperlipidemia, and osteoarthritis. Medications are detemir insulin, lispro insulin, metformin, lisinopril, and atorvastatin. On physical examination, blood pressure is 142/90 mm Hg and pulse rate is 63/min. BMI is 36. The remainder of the physical examination is unremarkable. Laboratory studies show a hemoglobin A1c level of 8.8%. Which of the following is the most appropriate management of this patient's diabetes? Add sitagliptin Add pioglitazone Increase insulin Metabolic surgery referral
Metabolic surgery demonstrates greater improvements in glycemic control and cardiovascular risk factors compared with optimized medical therapy and lifestyle modifications. This patient should be referred for metabolic surgery. The gastrointestinal tract plays an important role in glucose homeostasis and serves as an important physiologic target for improving glycemic control. Short- and mid-term data from randomized controlled trials of metabolic surgery demonstrated greater improvements in glycemic control and cardiovascular risk factors compared with optimized medical therapy and lifestyle modifications. Retrospective cohort studies and prospective observational studies suggest a reduction in cardiovascular deaths and lower incidence of cardiovascular events in patients undergoing metabolic surgery. Metabolic surgery should be recommended to patients with type 2 diabetes with class III obesity (BMI ≥40 [≥37.5 in Asian Americans]) independent of glycemic control and diabetes treatment regimen and to patients with type 2 diabetes with class II obesity (BMI 35.0-39.9 [32.5-37.4 in Asian Americans]) who fail to meet their glycemic goals despite optimizing medical therapies and lifestyle modifications. In addition, patients with class I obesity (BMI 30.0-34.9 [27.5-32.4 in Asian Americans]) who do not meet their glycemic goals despite optimizing medical therapy should be considered for metabolic surgery. This patient with class II obesity has an inability to meet her hemoglobin A1c goal of less than 7% on her current medical regimen. Further modifications to her current regimen may either exacerbate hypoglycemia and accelerate weight gain or not reach her glycemic goal. Although sitagliptin, a dipeptidyl peptidase (DPP)-4-inhibitor, is a weight neutral oral agent that could potentially improve her hemoglobin A1c level by 0.95% to 1.1%, it would not achieve her target hemoglobin A1c of less than 7%. Pioglitazone, a thiazolidinedione (TZD), improves insulin sensitivity and hemoglobin A1c by 0.9% to 1.1%. The addition of pioglitazone in this patient would likely not achieve her target hemoglobin A1c goal and could potentially induce additional weight gain, a known side effect of this drug class. Increasing the patient's insulin doses may improve her hemoglobin A1c to goal, but it will exacerbate her hypoglycemia and promote additional weight gain.
A 60-year-old man is evaluated for management of type 2 diabetes mellitus. Three months ago he was diagnosed with type 2 diabetes mellitus and chronic kidney disease. At that time, he received diabetes education and began lifestyle modifications. Medical history is significant for chronic kidney disease, hypertension, and hyperlipidemia. His medications are lisinopril and atorvastatin. On physical examination, his blood pressure is 146/88 mm Hg. BMI is 28.5. The remainder of his vital signs and physical examination are normal. Laboratory studies show a hemoglobin A1c level of 8.3%, serum creatinine level of 1.5 mg/dL (132.6 µmol/L), and estimated glomerular filtration rate of 48 mL/min/1.73 m2. Which of the following is the most appropriate treatment of this patient's diabetes? Empagliflozin Glipizide Metformin Saxagliptin
Metformin is considered to be safe in those with an estimate glomerular filtration rate (eGFR) greater than 45 mL/minute/1.73 m2 and is contraindicated in those with an eGFR less than 30 mL/min/1.73 m2. The most appropriate treatment of this patient's diabetes mellitus is to initiate metformin. This patient's hemoglobin A1c is above goal despite lifestyle modifications, and his treatment plan should be intensified. The first-line therapy recommended by the American Diabetes Association in conjunction with lifestyle modifications for treatment of type 2 diabetes is metformin. Previously, metformin use was contraindicated at serum creatinine levels of 1.4 mg/dL (123.8 µmol/L) or higher in women and 1.5 mg/dL (132.6 µmol/L) or higher in men. However, in a recent update the FDA concluded that metformin is considered to be safe in those with an estimated glomerular filtration rate (eGFR) greater than 45 mL/min/1.73 m2 and is contraindicated in those with an eGFR less than 30 mL/min/1.73 m2. The FDA recommends not initiating metformin for patients with eGFR greater than 30 mL/min/1.73 m2 to less than 45 mL/min/1.73 m2, or alternately, initiating metformin at a reduced dose (50%) with frequent monitoring of kidney function (every 3 months). The patient qualifies metformin therapy and frequent monitoring with his current kidney function. Empagliflozin, a sodium-glucose transporter-2 (SGLT2) inhibitor, is a second-line agent after metformin initiation. In addition, SGLT2 inhibitors may cause hypotension due to intravascular volume depletion, especially in patients with kidney impairment, and acute kidney injury has been reported with its use. The FDA advises that risk factors for acute kidney injury (hypovolemia, chronic kidney disease, heart failure, and use of diuretics, ACE inhibitors, angiotensin receptor blockers, or NSAIDs) be considered before initiating this class of drugs. The sulfonylurea, glipizide, is a second-line agent after metformin initiation. Glipizide could be considered as part of dual therapy if the glycemic goal is not reached with metformin. While sulfonylureas are the least expensive oral agent to add to metformin, they are associated with an increased risk for hypoglycemia as well as weight gain as compared with other potential combination therapies. Saxagliptin, a dipeptidyl peptidase-4 (DPP-4) inhibitor, is a second-line agent after metformin initiation. Saxagliptin could be considered as part of dual therapy if the glycemic goal is not reached with metformin. However, the FDA has warned that the DPP-4 inhibitors saxagliptin and alogliptin may increase the risk for heart failure, especially in patients who already have heart or kidney disease.
A 54-year-old man is evaluated at a follow-up visit after being diagnosed with type 2 diabetes mellitus 3 months ago. His initial hemoglobin A1c level was 8.5%. He opted for lifestyle modifications initially. He has lost 4.5 kg (10 lb) in the interim after making significant changes to his diet and increasing his activity level. His average blood glucose level currently is 180 mg/dL (10 mmol/L). Medical history is otherwise unremarkable. On physical examination, blood pressure is 130/74 mm Hg and pulse is 70/min. BMI is 27. The examination is otherwise unremarkable. His repeat hemoglobin A1c level today is 7.9%, but he would like it lower. Results of other laboratory studies are within normal ranges. Which of the following is the most appropriate management? Continue current management Initiate empagliflozin Initiate liraglutide Initiate metformin
Metformin is first-line therapy for all patients with type 2 diabetes without contraindications. The American Diabetes Association (ADA) recommends a hemoglobin A1c goal of less than 6.5% to 7% in patients who are early in the disease course and with few comorbidities. This patient meets these criteria and should aim for tighter glycemic control than he is currently achieving with 3 months of lifestyle modifications alone. Pharmacologic therapy should now be added to his lifestyle modifications. The American College of Physicians recommends a hemoglobin A1c level between 7% and 8% in most patients with type 2 diabetes. More stringent targets may be appropriate for patients who have a long life expectancy (>15 years) and are interested in more intensive glycemic control with pharmacologic therapy despite the risk for harms, including but not limited to hypoglycemia, patient burden, and pharmacologic costs. The ADA and the American College of Physicians (ACP) recommend metformin as first-line therapy for all patients with type 2 diabetes without contraindications. This recommendation is based on data from multiple studies demonstrating the effectiveness and safety of metformin. In addition, metformin is inexpensive. The patient has had a reduction in his weight and hemoglobin A1c with lifestyle modifications alone over a 3-month period. Despite this, he remains above his hemoglobin A1c goal of less than 6.5% to 7%. Therefore, continuing his current management protocol would not be appropriate. Achieving glucose goals early in the disease course can reduce the risk of developing microvascular and possibly macrovascular complications in the future. He will require the addition of pharmacologic agents at this time to reach his glycemic target. Empagliflozin has the ability to improve glycemic control while also reducing blood pressure and weight. Empagliflozin is considered second-line therapy after metformin by ADA and ACP. Liraglutide also has the ability to improve glycemic control while also reducing weight. There are studies also demonstrating a reduction in blood pressure with liraglutide use. Liraglutide is considered second-line therapy after metformin by the ADA and ACP. If the patient does not achieve his hemoglobin A1c target after 3 months of lifestyle modifications and metformin, dual therapy with metformin and liraglutide or metformin and empagliflozin could be considered.
A 67-year-old woman is evaluated for management of her type 2 diabetes mellitus. Medical history is significant for type 2 diabetes diagnosed 15 years ago, hypertension, hyperlipidemia, and obesity. She also has diabetic complications including nephropathy, retinopathy, and neuropathy. She does not have hypoglycemia. Medications are enalapril, atorvastatin, insulin glargine, insulin aspart, and metformin. On physical examination, vital signs are normal. BMI is 31. Ophthalmoscopic examination reveals nonproliferative diabetic retinopathy. A foot examination reveals an insensate foot with intact skin. Vibratory sense is absent in the toes and ankle. The remainder of the physical examination is unremarkable. Laboratory studies show a hemoglobin A1c level of 7.7% and serum creatinine level of 1.4 mg/dL (123.8 µmol/L). She has had a gradual decline in her estimated glomerular filtration rate (eGFR) from 50 to 39 mL/min/1.73 m2 over the last 5 years. Which of the following is the most appropriate management? Continue current regimen Discontinue metformin dose Increase insulin glargine dose Increase glipizide dose
Metformin may be continued in patients with an estimated glomerular filtration rate to 30 to 45 mL/min/1.73 m2 after consideration of risks and benefits; if metformin is continued, frequent monitoring of kidney function (every 3 months) is recommended. The most appropriate management of this patient's diabetes mellitus is to continue the current regimen. This is an older patient with multiple comorbidities. She is at her goal hemoglobin A1c level of less than 8% per the American Diabetes Association guidelines with her current regimen. In contrast, the American College of Physicians recommends that clinicians should avoid targeting an hemoglobin A1c level in patients with a life expectancy less than 10 years due to advanced age (80 years or older), residence in a nursing home, or chronic conditions (such as end-stage kidney disease) because the harms outweigh the benefits in this population. The FDA previously considered serum creatinine levels of 1.4 mg/dL (123.8 µmol/L) or higher in women and 1.5 mg/dL (132.6 µmol/L) or higher in men a contraindication to metformin use due to concerns for development of lactic acidosis. After further review of safety data from multiple studies, the criteria for continued safe use of metformin have been revised by the FDA. The use of serum creatinine for determining safe use of metformin was replaced by estimated glomerular filtration rate (eGFR) to better estimate kidney function. Patients who have a decrease in eGFR to 30 to 45 mL/min/1.73 m2 while treated with metformin may continue use after consideration of risks and benefits. If metformin is continued, frequent monitoring of kidney function (every 3 months) is recommended. Metformin should be discontinued if the eGFR falls below 30 mL/min/1.73 m2. Because she is at her goal hemoglobin A1c level of less than 8%, intensifying her therapy with either insulin glargine or glipizide is unnecessary, In addition, increasing the insulin glargine or glipizide dose in the setting of worsening kidney function could increase the risk for hypoglycemia. Additional weight gain may also occur with an increased insulin glargine or glipizide dose.
A 24-year-old woman has a 4-year history of monthly headaches. The pain is bitemporal, throbbing, and worsened by bending or ascending stairs. Approximately half of the headaches have been severe with associated nausea. She has no precursor symptoms, no photophobia or phonophobia, and no visual or neurologic symptoms. Over the past year, the headaches have not changed in character but have gradually become more frequent, increasing from 4 to 10 days per month. Episodes last as long as 8 hours and typically respond to ibuprofen within 1 hour. She takes no other medication. All physical examination findings, including vital signs and those from a neurologic examination, are unremarkable. Which of the following is the most appropriate treatment? Citalopram Gabapentin Metoprolol Onabotulinum toxin A Rizatriptan
Metoprolol, propranolol, timolol, divalproex sodium, and topiramate all have Level A evidence for prevention of episodic migraine and should be considered for migraine occurring at a frequency of at least 5 days per month. This patient should be treated with metoprolol. Her headaches meet the criteria for episodic migraine without aura. Pharmacologic prevention should be considered for migraine occurring at a frequency of at least 5 days per month. According to guidelines, metoprolol, propranolol, timolol, divalproex sodium, and topiramate all have Level A evidence for prevention of episodic migraine, with atenolol, amitriptyline, and venlafaxine having level B evidence. Approximately a third of patients with episodic migraine meet criteria for pharmacologic prevention, yet only 3% to 13% receive this treatment. The goal of prevention is an at least 50% reduction in headache frequency; reductions in migraine intensity, disability, and cost are other established outcomes. Drug tapering or elimination should be considered after a period of 6 to 12 months of adequate control. No data support the use of citalopram or other selective serotonin reuptake inhibitor antidepressants in migraine prevention. Insufficient evidence about the effectiveness of gabapentin in migraine prevention is available; therefore, its use for this purpose is inappropriate. Onabotulinum toxin A has an FDA indication for the prevention of chronic migraine but is not approved and has no established efficacy in the prevention of episodic migraine. Chronic migraine is defined as headache occurring 15 or more days per month with at least 8 days meeting full criteria for migraine or responding to migraine-specific medication. Rizatriptan is an effective treatment option for acute migraine. However, this patient is responding well to an NSAID, and no adjustment in acute therapy is warranted. Escalation in headache frequency requires migraine-preventive measures.
A 33-year-old woman is evaluated for a 4-week history of recurrent worsening joint pain and a new rash on the chest and upper back. She reports sun exposure from spending time at the beach. She has no other symptoms. She has no other medical problems and takes no medications. On physical examination, vital signs are normal. Diffuse tenderness of multiple small joints of the hands is noted. The remainder of the examination is normal. The appearance of the rash is shown. Which of the following is the most likely diagnosis for the rash? Acute cutaneous lupus erythematosus Dermatomyositis Discoid lupus erythematous Subacute cutaneous lupus erythematous
Subacute cutaneous lupus erythematosus (SCLE) can present as annular with central clearing or papulosquamous with patchy erythematous plaques and papules, and both forms can be seen in the same patient; about 50% of patients with SCLE also have systemic lupus erythematosus. This patient has subacute cutaneous lupus erythematosus (SCLE) and likely has systemic lupus erythematosus (SLE). She has a new rash on the torso with annular and patchy papular areas. This rash is most likely SCLE, a photosensitive rash occurring especially on the arms, neck, and upper trunk, usually sparing the central face. The rash can be annular with central clearing or papulosquamous with patchy erythematous plaques and papules. In some patients, the rash may have both of the characteristics, as seen in this patient. SCLE often has a fine scale that may leave postinflammatory hypo- or hyperpigmentation. SCLE is associated with anti-Ro/SSA antibodies, with a prevalence of 75%. About 50% of patients with SCLE also have SLE. These patients more typically have mild systemic symptoms, most commonly arthritis and myalgias; patients with lupus vasculitis, central nervous system lupus, and nephritis are found in less than 10% of patients with SCLE. Psoriasis can present with similar lesions but are usually photoresponsive (improve with sun exposure), unlike this patient. Essentially 100% of patients with acute cutaneous lupus erythematosus (ACLE) have SLE. ACLE may present in multiple forms, with the most commonly recognized being a characteristic localized malar (butterfly) eruption. Less commonly it can appear as a generalized eruption, which typically appears as an erythematous maculopapular eruption of sun-exposed skin such as the extensor surfaces of the arms and hands. Patients with dermatomyositis frequently have areas of poikiloderma with ill-defined patchy erythema and "salt-and-pepper" dyspigmentation accompanied by telangiectasias on the chest or upper back. There may be slightly violaceous erythema over the "V" of the neck, chest, and the upper back (the "Shawl sign") or on the lateral hips (the "holster sign"). Discoid lupus erythematosus (DLE) occurs in 20% of patients with SLE but more commonly occurs as an isolated, nonsystemic finding; patients with isolated DLE usually do not go on to develop SLE. DLE usually affects the scalp and face and presents as hypo- and/or hyperpigmented, possibly erythematous, patches or thin plaques that may be variably atrophic or hyperkeratotic.
A 72-year-old woman is evaluated for a 12-month history of increasing forgetfulness. The patient is retired from her position as a professor of economics at a local university. She takes no medication and drinks no alcoholic beverages. Her husband says that his wife's occasional forgetfulness has caused no major problems and that she is able to function normally. On physical examination, vital signs are normal. Her Geriatric Depression Scale score is normal and Montreal Cognitive Assessment score is 25/30 (normal, ≥26), with a score of 1/5 on the delayed recall section and 5/6 on the orientation section. All other physical and neurologic examination findings are normal. Results of laboratory studies show normal vitamin B12, thyroid-stimulating hormone, and 25-hydroxyvitamin D levels. Complete neuropsychological testing findings indicate normal global cognition, language function, and attention, but verbal learning and memory performance are 2 SDs below that of an age- and education-matched control population. An MRI of the brain shows a slight loss of hippocampal volume that is greater on the right than the left but is otherwise normal. Which of the following is the most likely diagnosis? Alzheimer disease Depression Mild cognitive impairment Normal aging
Mild cognitive impairment is a cognitive state between normal aging and dementia characterized by a decline in cognitive functioning that is greater than what is expected with normal aging but has not resulted in significant functional disability. The patient's symptoms of subjective cognitive problems, near-normal function, and objective cognitive impairment (score of 1/5 on the delayed recall section of the Montreal Cognitive Assessment) are most consistent with a diagnosis of mild cognitive impairment (MCI), amnestic type. MCI is a cognitive state between normal aging and dementia characterized by a decline in cognitive functioning that is greater than what is expected with normal aging but has not resulted in significant functional disability. The suggestive MRI evidence of minimal hippocampal atrophy is a biomarker conferring a risk of symptom progression. Given her objective performance on cognitive tests and the noted MRI changes, the patient has an annual risk of developing Alzheimer disease of 5% to 15%. In contrast to MCI, dementia is a progressive deterioration of cognitive function severe enough to impair occupational or social functioning. Alzheimer disease is the most common type of dementia (60%-80%). Whereas typical Alzheimer disease presents with the insidious development of recent memory loss, as this patient has experienced, she lacks the functional impairment that is a hallmark of dementia. Multiple relationships between depression and cognitive impairment have been reported in the literature: late-life depression may lead to prodromal Alzheimer disease; depression may be a risk factor for future development of cognitive impairment, dementia, and Alzheimer disease; cerebrovascular disease can precipitate late-life depression; and cognitive impairment itself can lead to depression. More than half of patients with late-life major depression exhibit clinically significant cognitive impairment, most frequently affecting processing speed, executive function, and visuospatial ability. However, this patient's depression screening was negative, and thus depression is unlikely to account for her symptoms. Normal aging can be associated with memory problems, but cognitive testing shows functioning within the normal range. In contrast, this patient's verbal learning and memory performance are 2 SDs below that of an age- and education-matched control population and are not consistent with normal aging.
A 43-year-old man is evaluated for 4 days of fever and arthralgia, as well as a raised purple rash over his lower extremities of 1 week's duration. He is a current intravenous drug user. He has no other medical problems and takes no medications. On physical examination, temperature is 37.8 °C (100 °F) and blood pressure is 132/85 mm Hg; other vital signs are normal. The large and small joints are tender to palpation without evidence of synovitis. The abdomen is not tender to palpation. The liver edge is palpable below the right costal margin. The skin findings are shown. Laboratory studies: Alanine aminotransferase 143 U/L Aspartate aminotransferase 130 U/L Total bilirubin 2.2 mg/dL (37.6 µmol/L) Creatinine 1.0 mg/dL (88.4 µmol/L) Hepatitis B surface antigen Positive IgM antibody to hepatitis B core antigen Positive Hepatitis B e antigen Positive HBV DNA 10,000,000 IU/mL Hepatitis C antibody Negative HIV-1/HIV-2 antigen and antibody combination immunoassay Negative Cryoglobulin Normal Urinalysis Normal Which of the following is the most appropriate treatment? Entecavir Plasma exchange Prednisone and cyclophosphamide Sofosbuvir and ledipasvir Tenofovir, emtricitabine, and raltegravir
Mild hepatitis B virus-related polyarteritis nodosa is treated with antiviral agents. The most appropriate treatment is entecavir. Hepatitis B virus (HBV)-related polyarteritis nodosa (PAN) is the most likely diagnosis in this patient. The patient has fever, arthralgia, and evidence of cutaneous vasculitis (rash). There is also evidence of hepatitis (elevated bilirubin and aminotransferase levels) likely related to his use of intravenous drugs. Testing for hepatitis B surface antigen and hepatitis B e antigen is positive, as is IgM antibody to hepatitis B core antigen, with a significant HBV DNA viral load; all of these results are consistent with a recent HBV infection. PAN and mixed cryoglobulinemia are rare HBV-associated vasculitides. This patient has undetectable serum cryoglobulins, excluding the diagnosis of mixed cryoglobulinemia. There are no data from randomized trials to guide therapy in patients with HBV-related PAN. Patients with HBV infection and mild manifestations of PAN are usually treated with antiviral agents such as entecavir. Patients (such as this one) with mild PAN have constitutional symptoms, arthritis or arthralgia, anemia, and skin lesions. More severe disease is characterized by organ dysfunction (for example, myocarditis, kidney insufficiency and hypertension, mononeuritis multiplex) or life-threatening systemic manifestations. Patients with severe manifestations of PAN are often treated with an antiviral agent and a short course of prednisone and plasma exchange. Prednisone and cyclophosphamide are agents typically used to treat mild and severe manifestations of idiopathic PAN, the most commonly recognized form of PAN. Because this patient has HBV-related PAN, treating the causative viral infection with an antiviral agent is the preferred management. Sofosbuvir and ledipasvir are direct-acting antiviral agents used to treat hepatitis C virus (HCV) infection and would be an appropriate choice for mild HCV-related PAN. However, this patient's hepatitis C antibody is negative and these agents are not indicated. Tenofovir, emtricitabine, and raltegravir are used to treat HIV infection. This drug combination would be an appropriate choice for HIV-related PAN, but the patient's HIV test is negative and these agents are not indicated. Tenofovir alone would be a reasonable treatment for this patient.
A 67-year-old woman arrives at the emergency department with hematuria of 3 days' duration. She has no other sites of bleeding. Medical history is significant for recently diagnosed nonvalvular paroxysmal atrial fibrillation and hypertension. Medications are hydrochlorothiazide and dabigatran. On physical examination, vital signs are normal and other examination findings are unremarkable. A urine sample is grossly bloody. Laboratory studies: Activated partial thromboplastin time 58 s Hemoglobin 12.8 g/dL (128 g/L) Prothrombin time 12 s Creatinine 1.1 mg/dL (97.2 µmol/L) Urinalysis pH 5.0; 4+ blood; no protein. On microscopic examination, erythrocytes were too numerous to count. No casts or dysmorphic erythrocytes observed. In addition to discontinuing dabigatran, which of the following is the most appropriate immediate management? Four-factor prothrombin complex concentrate Fresh frozen plasma Idarucizumab Vitamin K Observation
Minor bleeding in patients taking a non-vitamin K antagonist oral anticoagulant can be managed by discontinuation of the anticoagulant alone without additional therapy. Observation is most appropriate in this patient at this time. She has no major or clinically relevant bleeding; therefore, a conservative treatment approach is reasonable. Her hemoglobin level is normal, she is hemodynamically stable, and she has no other signs of bleeding. The elimination half-life of dabigatran is 12 hours with anticoagulant effect for 24 hours in patients with normal kidney function. Dabigatran should be withheld for 24 hours or longer in this patient pending evaluation of her urinary tract. She has nonglomerular hematuria (no proteinuria, casts, or dysmorphic erythrocytes) and requires an evaluation of the upper and lower urinary tract for a cause of the hematuria. The necessity to fully evaluate hematuria is not influenced by the use of anticoagulant therapy. Fresh frozen plasma has been used in patients experiencing major bleeding while taking a non-vitamin K antagonist oral anticoagulant, but it is not typically adequate monotherapy in patients experiencing severe bleeding; it is often combined with four-factor prothrombin complex concentrate (4f-PCC). However, neither fresh frozen plasma nor 4f-PCC would be necessary in this patient because she is hemodynamically stable, the bleeding is not life threatening, and the anticoagulant effect of dabigatran will dissipate in 24 hours. Idarucizumab is a humanized monoclonal antibody fragment with a very high affinity for dabigatran. It is approved for dabigatran reversal for emergency surgery, for urgent procedures, or in patients experiencing life-threatening or uncontrolled bleeding. Severe adverse effects may include thromboembolic events and possible hypersensitivity reactions, such as bronchospasm and pruritus. Because this patient does not have life-threatening bleeding, idarucizumab is not indicated. Vitamin K administration has no effect on bleeding caused by non-vitamin K antagonist oral anticoagulants such as dabigatran. Vitamin K can be used to reverse over-anticoagulation with warfarin.
A 65-year-old man was hospitalized 24 hours ago with findings of a large anterior myocardial infarction. He underwent primary percutaneous coronary intervention with stent placement in the proximal left anterior descending artery. He is currently asymptomatic. Medical history is significant for hyperlipidemia and hypertension. Medications are atorvastatin, aspirin, prasugrel, captopril, and metoprolol. On physical examination, temperature is normal, blood pressure is 110/65 mm Hg, pulse rate is 65/min, and respiration rate is 18/min. Oxygen saturation is 98% breathing ambient air. The remainder of the examination is unremarkable. An electrocardiogram obtained in the coronary care unit is shown (Mobitz Type 2). Which of the following is the most appropriate treatment? Atropine Discontinue metoprolol and observe Emergent coronary angiography Emergent pacing
Mobitz type 2 second-degree atrioventricular block is an uncommon but potentially life-threatening electrical complication of anterior myocardial infarction; temporary or permanent pacing is indicated in this setting. This patient with Mobitz type 2 second-degree atrioventricular (AV) block following a large anterior myocardial infarction requires temporary pacing. In the peri-infarct setting, arrhythmias are common, and benign forms of vagally mediated heart block must be differentiated from more serious conduction abnormalities. Second-degree AV block is frequently seen with acute myocardial infarction. When progressive PR prolongation is observed before a blocked beat, second-degree Mobitz type 1 (Wenckebach) block is present. When the PR interval is constant before nonconducted P waves, the second-degree block is termed Mobitz type 2 block. When 2:1 block is present, Mobitz type 1 versus type 2 block may not be distinguishable with surface electrocardiography. Mobitz type 2 second-degree AV block is an uncommon but potentially life-threatening electrical complication of an anterior myocardial infarction. It usually represents a block lower in the conduction system and suggests significant conduction disease, likely resulting from ischemia and necrosis of the septum (along which the His-Purkinje system is located). Because of the high risk for progression to complete heart block, prompt implantation of a temporary or permanent pacemaker is indicated. Atropine improves AV nodal conduction and quickens the sinus rate. In patients with Mobitz type 2 block, atropine may not alter conduction, and the degree of heart block may paradoxically worsen. Furthermore, this agent has a relatively short-lived effect and will not address the underlying pathology. Cessation of β-blocker therapy alone will not improve conduction in this patient, as the pathology of Mobitz type 2 block is beyond the AV node. In contrast, Mobitz type 1 second-degree AV block is frequently seen with acute inferior myocardial infarction and may be associated with periods of complete heart block. Observation and withholding of AV nodal blocking agents is appropriate in that context, as the need for permanent pacing is extremely low with inferior myocardial infarction. In the absence of chest pain or ST-segment elevations on electrocardiogram, urgent angiography is not indicated.
A 25-year-old woman is evaluated for pruritic lesions on the legs and in the groin area that first appeared several months ago. The patient is otherwise in good health and takes no medications. On physical examination, vital signs are normal. Skin findings are shown. Which of the following is the most likely diagnosis? Condylomata acuminata Herpes simplex virus infection Molluscum contagiosum infection Seborrheic keratoses
Molluscum contagiosum is a common cutaneous viral infection caused by the poxvirus molluscum contagiosum. It occurs in three settings: children, adults as a sexually transmitted disease, and AIDS patients. The lesions are flesh-colored to yellow smooth papules with a shiny surface and an umbilicated center. They can be located anywhere on the skin, but in adults they typically involve the genital area; in AIDS patients, they often involve the face. Diagnosis can be made by the clinical appearance. In immunocompromised patients, cryptococcosis, histoplasmosis, or Penicillium marneffei infections may resemble molluscum lesions and should be differentiated by skin biopsy. Although lesions can self-resolve, this may take months to years. Therapy includes destructive techniques including cryotherapy, salicylic acid, cantharidin, or physical removal with curettage. Anogenital warts (condylomata acuminata) are the most common sexually transmitted infection and are most often caused by human papillomavirus types 6 and 11. They present as single or multiple papules on the penis, vulva, or perianal area and may be variably sized flat-topped or cauliflower-like papules. They do not have a shiny surface nor are they umbilicated. Lesions are diagnosed based on clinical appearance. Although warts may resolve spontaneously, treatment often is required. Treatment of anogenital warts may include chemical or physical destruction, immunologic therapy, or excision. Herpes simplex virus (HSV) infection presents as localized, grouped pink macules and papules that progress to vesicles on an erythematous base and then evolve into pustules with subsequent ulcers and erosions. HSV1 traditionally causes orofacial lesions, and HSV2 most often causes genital lesions; however, both viruses can lead to either oral or genital lesions. Diagnosis of HSV can be made clinically, but several rapid tests are widely available, such as direct-fluorescent antibody and polymerase chain reaction. Oral antiviral agents including acyclovir, valacyclovir, and famciclovir are considered first-line therapies. Seborrheic keratosis is a benign neoplasm of the skin that has a "stuck-on" appearance. They have a wide range of shades from tan to black, have different morphologies, and vary in size from a few millimeters to several centimeters. It is more common with advancing age and can be seen anywhere on the body, but they characteristically spare the palms and soles. This young patient's shiny papules are not compatible with the diagnosis of seborrheic keratoses. Read Related TextNext Question
A 52-year-old man is evaluated during a visit to establish care. He is asymptomatic, but he is seeking advice on how to modify his risk for cardiovascular disease. He drinks one glass of wine with dinner most nights, and he quit smoking 12 years ago. Family history is significant for a myocardial infarction in his father at age 61 years. He takes no medications. The patient is Hispanic. On physical examination, temperature is normal, blood pressure is 128/76 mm Hg, and pulse rate is 74/min. BMI is 28. The remainder of the physical examination is unremarkable. Laboratory studies: Total cholesterol 200 mg/dL (5.18 mmol/L) HDL cholesterol 30 mg/dL (0.78 mmol/L) LDL cholesterol 130 mg/dL (3.37 mmol/L) Triglycerides 200 mg/dL (2.26 mmol/L) Which of the following risk factors most increases this patient's risk for cardiovascular disease? Alcohol use Ethnicity Family history Hyperlipidemia Smoking history
Most cardiovascular risk can be attributed to modifiable risk factors; among them, elevated lipid levels impart the highest risk for cardiovascular disease. This patient's hyperlipidemia most increases his risk for cardiovascular disease (CVD). Most cardiovascular risk can be attributed to modifiable risk factors, and among them, elevated cholesterol levels impart the highest risk for CVD. Reductions in lipid levels can decrease overall risk, and current cholesterol treatment guidelines focus on risk for future cardiovascular events rather than absolute lipid levels. This patient's 10-year cardiovascular risk based on the American College of Cardiology (ACC)/American Heart Association (AHA) Pooled Cohort Equations risk calculator is 6.9%, and he should be counseled regarding therapeutic lifestyle changes, including dietary modification, regular physical activity, weight loss, and continued smoking cessation. According to the 2018 AHA/ACC Guideline on the Management of Blood Cholesterol, this patient's cardiovascular risk is categorized as borderline (5% to <7.5%). In the presence of risk-enhancing factors, statin therapy is a reasonable consideration. Risk enhancers include family history of premature atherosclerotic CVD (men aged ≤55 years, women aged ≤65 years), LDL cholesterol level of 160 mg/dL (4.14 mmol/L) or higher, metabolic syndrome, chronic kidney disease, chronic inflammatory conditions, history of premature menopause or previous history of preeclampsia, South Asian ancestry, or triglyceride level of 175 mg/dL (1.98 mmol/L) or higher. Moderate alcohol intake (one to two drinks daily for men, one drink daily for women) has been linked with decreased incidence of CVD; however, heavy alcohol consumption has been shown to increase cardiovascular risk. The deleterious effects of alcohol consumption must be weighed against the potential benefits when patients are counseled about risk modification. The risk for CVD varies considerably among different ethnic groups. The prevalence of heart disease is lower among persons of Hispanic ethnicity than in most other ethnic groups, and Hispanics tend to have lower rates of traditional modifiable risk factors. Family history of premature CVD is an independent cardiovascular risk factor but typically adds little precision when included in multivariate risk models. In most studies, premature CVD is defined as CVD in a first-degree male relative younger than 55 years or a first-degree female relative younger than 65 years. A family history of premature CVD doubles the risk for myocardial infarction in men and increases the risk by 70% in women. This patient's father died at age 61 years; therefore, family history is not the most important risk factor for this patient. Although active smoking remains a strong risk factor for CVD, smoking cessation substantially reduces cardiovascular risk within 2 years, with risk returning to the level of a nonsmoker at approximately 10 years. Smoking status should be addressed at every visit, and cessation counseling should be offered to patients who are actively smoking. Read Related TextNext Question
A 52-year-old man is evaluated in the ICU for dyspnea that developed after aspiration of gastric contents during an upper endoscopy. The endoscopy was performed for evaluation of upper gastric bleeding due to peptic ulcer disease. His only medication is pantoprazole. On physical examination, temperature is 37.3 °C (99.1 °F), blood pressure is 150/99 mm Hg, pulse rate is 110/min, and respiration rate is 28/min. Oxygen saturation is 90% on a 100% oxygen nonrebreather mask. He is awake, diaphoretic, and anxious. Lung examination reveals scant bilateral crackles and rhonchi as well as use of accessory muscles. The remainder of the physical examination is normal. Arterial blood gas studies on a 100% oxygen nonrebreather mask show a pH of 7.35, a PCO2 of 46 mm Hg (6.1 kPa), and a PO2 of 55 mm Hg (7.3 kPa). Chest radiograph reveals new bilateral opacities. Which of the following is the most appropriate treatment? Continue current therapy High-flow humidified nasal cannula Intubation and mechanical ventilation Noninvasive mechanical ventilation
Most patients with acute respiratory distress syndrome require intubation and mechanical ventilation to ensure adequate delivery of high levels of inspired oxygen and positive end-expiratory pressure to stabilize alveoli. The most appropriate treatment is intubation and mechanical ventilation. This patient has dyspnea, severe hypoxemia, bilateral opacities on chest radiograph, and acute hypoxemic respiratory failure after aspiration, all of which are diagnostic for acute respiratory distress syndrome (ARDS). ARDS is associated with heterogeneous but often widespread damage to the alveolar epithelium and vascular endothelium, as well as surfactant dysfunction leading to alveolar instability and collapse. The changes can severely reduce lung compliance, making adequate ventilation difficult and further worsening hypoxia. Most patients with ARDS require intubation and mechanical ventilation to ensure adequate delivery of high levels of inspired oxygen and positive end-expiratory pressure to stabilize alveoli. Continuing oxygen through a face mask would not be appropriate. Although the patient is maintaining appropriate oxygen saturation, the presence of anxiety, diaphoresis, tachycardia, tachypnea, hypercapnia, and hypoxemia signal impending respiratory failure that should be treated with intubation and mechanical ventilation. High-flow humidified nasal cannula devices decrease the work of breathing, provide heated and humidified air, provide a reliable FIO2, and decrease dead space. Their initial application should occur in the critical care setting with close monitoring for tolerance and effectiveness. A recent trial demonstrated that in patients with acute hypoxemic respiratory failure, the use of high flow nasal cannula led to decreased mortality compared to continuing face mask oxygen or noninvasive face mask ventilation; however, the study excluded patients with hypercapnia, such as this patient. This patient shows features of respiratory muscle fatigue, including rapid breathing, diaphoresis, use of accessory muscles of respiration, and an elevated arterial PCO2, all indications for mechanical ventilation. Noninvasive positive pressure ventilation (NIPPV) is the delivery of positive airway pressure using a cushioned face mask or helmet or without the use of an invasive connection directly in a patient's airway. In patients with hypoxemic respiratory failure, the use of NIPPV is controversial. Select patients may benefit from short-duration NIPPV to avoid intubation and associated complications but some studies have demonstrated increased mortality, likely due to delay in the implementation of appropriate invasive mechanical ventilation. This patient with deteriorating ventilation should be intubated and mechanically ventilated.
A 52-year-old man is evaluated for difficult-to-control hypertension. Biochemical evaluation confirms a diagnosis of primary aldosteronism. Medications are amlodipine, losartan, and metoprolol. On physical examination, blood pressure is 149/98 mm Hg and pulse rate is 75/min. The remainder of the vital signs and physical examination are unremarkable. CT scan shows a 0.8-cm right adrenal mass with a density of 13 Hounsfield units. Which of the following is the most appropriate management? Adrenal vein sampling Increase metoprolol Increase losartan Right adrenalectomy
Most patients with biochemically confirmed primary aldosteronism should undergo adrenal vein sampling to confirm the source of the hyperaldosteronism. The next step in this patient's management is adrenal vein sampling. Once the diagnosis of primary aldosteronism has been confirmed biochemically, radiographic localization with abdominal CT is indicated. CT is recommended over MRI in most cases due to similar efficacy and lower cost. Adrenal hyperplasia and adenomas secreting excess aldosterone, however, may not always be visualized. Adrenal vein sampling is, therefore, needed in most patients to determine the source of aldosterone secretion when imaging is unrevealing and to confirm lateralization when imaging demonstrates an adrenal adenoma, such as in this case. Adrenal vein sampling is especially important in older patients because of a higher frequency of nonfunctioning adrenal incidentalomas. Patients with an aldosterone-secreting adenoma are usually offered adrenalectomy, whereas those with primary aldosteronism due to bilateral adrenal hyperplasia are treated medically. Omission of adrenal vein sampling can lead to misdiagnosis in approximately 25% of cases, and subsequent unnecessary adrenalectomy, or medical therapy when adrenalectomy could be offered. Right adrenalectomy should not be performed in this patient without further confirmation of the source of primary aldosteronism. Aldosterone likely exerts direct toxic effects on cardiac cells as evidenced by a higher prevalence of left ventricular hypertrophy and decreased left ventricular function when compared with matched control patients with similar levels of hypertension. These deleterious effects are likely mediated by mineralocorticoid receptors in the heart, coronary arteries, aorta, and other blood vessels. Increasing this patient's losartan or metoprolol dose may lead to better control of his hypertension, but neither of these medications blocks the aldosterone receptor. Therefore, the patient would still be subject to the deleterious effects of excess stimulation of aldosterone receptors, which may lead to cardiac disease.
A 68-year-old woman is evaluated for a 1-year history of neck stiffness and dull, achy neck pain. She also notes intermittent difficulty with dexterity while performing fine motor tasks at the hair salon where she works. Medications are ibuprofen as needed. On physical examination, vital signs are normal. Range of motion of the neck is limited because of pain and stiffness. Fine finger movements exhibit subtle slowness. Reflex examination findings are normal, including a plantar flexor response. Muscle strength is 5/5 throughout. Gait is normal. An MRI of the cervical spine shows multilevel cervical stenosis that is worst at C4/5 and C6/7. There is moderate deformation of the cord, but no signal change in the cord is noted. Which of the following is the most appropriate next step in management? Gabapentin Neck immobilization in a hard cervical collar Neurosurgical intervention Physical therapy
Most patients with chronic cervical and lumbar stenosis respond well to conservative measures, such as physical therapy and pain control. This patient should have physical therapy. Spinal cord compression can result from acute or chronic causes. Evaluation and confirmation of suspected acute spinal cord compression with appropriate neuroimaging studies should occur in an urgent manner. Immediate treatment may be necessary to prevent severe and irreversible neurologic injury. Patients with chronic spinal stenosis due to osteoarthritic degenerative spinal disease frequently have chronic myelopathic symptoms, most often involving the cervical and lumbar spines. This patient has chronic symptoms of cervical stenosis due to multilevel disc disease. Most patients with chronic cervical and lumbar stenosis respond well to conservative measures, such as physical therapy and pain control. However, those with symptoms of more moderate to severe disease who also have signs of myelopathy on examination, such as progressive leg weakness, spasticity, distal numbness, and bladder impairment, may require surgical intervention. The lack of these findings in this patient supports the use of conservative measures, such as physical therapy. Gabapentin can be useful in patients with neuropathic pain. This patient, however, has none of the typical symptoms of neuropathic pain, such as burning, electrical, or frostbite-like sensations. Immobilization of the neck in hard (or soft) cervical collar would be an unnecessary and excessive restriction for someone with chronic cervical stenosis, especially in light of potential adverse effects. A hard cervical collar most commonly is used for cervical spine stabilization after trauma, surgery, and fractures or dislocations. It would be excessive in this patient whose clinical examination and imaging findings show no true cord compression or spinal instability. Although some patients with spinal stenosis will eventually require neurosurgical intervention, outcomes for multilevel disc disease are generally poor, and this approach should be used only after conservative measures have been exhausted. The finding of signal abnormality within the cord may correlate with the severity of compressive myelopathy. Multiple levels of signal abnormality may be a more significant finding than single-level changes. The lack of clear cord compression on examination or signal-intensity changes on MRI indicates that this patient does not need emergent surgery.
A 36-year-old woman is hospitalized with a 2-week history of gradually worsening dyspnea on exertion, facial swelling, and a sensation of head fullness. She has had fatigue, night sweats, and a 4.5-kg (10-lb) weight loss over the past 2 months. Her medical history is otherwise unremarkable, and she takes no medications. On physical examination, vital signs are normal. She is not in respiratory distress and has no stridor. She has facial and neck edema, and her face appears flushed. There is no lymphadenopathy. Lungs are clear on auscultation. Cardiac and abdominal examinations are normal. A CT scan of the chest done in the emergency department shows an 8.5-cm right mediastinal mass with compression of the superior vena cava. There is no evidence of thrombosis. There are no other areas of lymphadenopathy and no lung masses. Which of the following is the most appropriate next step in management? Dexamethasone Mediastinoscopy Percutaneous intravascular stent placement Urgent radiation therapy
Most patients with superior vena cava syndrome do not require emergency intervention, and a tissue biopsy should be obtained. This patient has symptoms, signs, and CT scan findings consistent with superior vena cava (SVC) syndrome due to a mediastinal malignancy, with symptoms developing over 2 weeks, and should receive a mediastinoscopy. It is important to establish a tissue diagnosis before treatment. There are no peripheral sites to biopsy in this case: the only abnormal result found on imaging and examination is the mediastinal mass. Although some patients may be unable to have surgery if they are in severe respiratory distress, are hemodynamically unstable, or have evidence of cerebral edema, most patients who present with SVC syndrome can tolerate a surgical biopsy. Mediastinoscopy with biopsy has a diagnostic yield of over 90% and is more likely than core biopsies to obtain specimens adequate for determining nodal architecture and histologic subtype if a lymphoma is present. SVC syndrome is caused by thrombosis or other nonmalignant obstructions in about 35% of cases. Of malignant causes, approximately 75% are due to lung cancers, with non-small cell lung cancers most common. Lymphoma and metastatic cancer each cause about 10% of cases; thymoma, germ cell cancer, and mesothelioma are each present in 1% to 3% of cases. Dexamethasone is often used to treat SVC syndrome and, particularly in lymphomas, may decrease symptoms. However, dexamethasone may obscure histologic findings in lymphomas, and a biopsy should be done before starting the patient on glucocorticoids. Percutaneous intravascular stents can be placed in patients with respiratory distress before tissue biopsy, but this patient is not having respiratory distress and does not require stent placement. Radiation requires an initial tissue diagnosis and is used as initial treatment in some but not all malignancies that cause SVC syndrome. If this patient has a lymphoma, as is clinically most likely, she would be treated with initial chemotherapy.
A 71-year-old woman is evaluated for difficulty holding her head upright. She notes that her head feels heavy and reports intermittent difficulty with swallowing and speech that is worse in the evening. She has had no pain, sensory changes, weakness in the extremities, or cognitive or visual symptoms. She has no other medical problems and takes no medication. On physical examination, vital signs are normal. Speech is mildly dysarthric. Cervical extension is weak. No ptosis, ophthalmoplegia, sensory deficit, or weakness in the extremities is noted. Results of laboratory studies show a normal serum creatine kinase level; no acetylcholine receptor antibodies are detected. Findings from routine nerve conduction and needle electromyography studies of the limbs are unremarkable, but a repetitive stimulation protocol reveals a decremental response. An MRI of the brain is normal. Which of the following is the most likely diagnosis? Bulbar amyotrophic lateral sclerosis Inclusion body myositis Multiple sclerosis Myasthenia gravis Polymyositis
Myasthenia gravis can present with pronounced weakness of cervical or bulbar muscles. Fluctuation in weakness, with a fatigable pattern that worsens later in the day, suggests this neuromuscular junction disorder. In the setting of probable myasthenia gravis, seronegative status for acetylcholine receptor antibodies should trigger testing for muscle-specific kinase (MuSK) antibodies, which are positive in half of seronegative patients. Weakness of cervical extension, such as this patient exhibits, is a hallmark of MuSK antibody-positive myasthenia gravis. Although nerve conduction studies using a special protocol for repetitive stimulation can reveal a pattern of decrementing weakness in neuromuscular junction disorders, this protocol is not part of routine electromyography. Therefore, if suspicion for myasthenia gravis is high, a repetitive stimulation protocol should be specifically requested. Although amyotrophic lateral sclerosis can present with prominent bulbar and cervical weakness, the fluctuating nature of the weakness, the absence of tongue weakness or fasciculations and of upper motor neuron signs, and the negative electromyography (EMG) findings make this an unlikely diagnosis. Inclusion body myositis is a progressive inflammatory myopathy that can present with bulbar, forearm flexor, and quadriceps weakness. The absence of additional muscle involvement, the normal creatine kinase level, and the normal limb findings on needle EMG rule out this entity. Multiple sclerosis (MS) can cause bulbar weakness secondary to central demyelination. Focal neck extensor weakness is atypical. The patient's history and normal MRI exclude MS. Polymyositis also can present with head drop, but often proximal limb muscles are involved. In addition, a normal creatine kinase level and normal results on routine limb EMG rule out this entity. Read Related TextNext Question
A 25-year-old woman is hospitalized for a 4-week history of swelling of the legs, weight gain, and shortness of breath on exertion. She was diagnosed with systemic lupus erythematosus 1 year ago when she presented with polyarthritis, rash, and alopecia. She was initially treated with hydroxychloroquine and prednisone with a good response. On physical examination, blood pressure is 142/96 mm Hg; other vital signs are normal. There is pitting edema of the lower extremities extending to the knees. The remainder of the physical examination is normal. Laboratory studies: Erythrocyte sedimentation rate 68 mm/h Hematocrit 38% Complements (C3 and C4) Low Creatinine 1.0 mg/dL (88.4 µmol/L) Anti-Smith antibodies Positive Anti-double-stranded DNA antibodies Positive Urinalysis 3+ protein; no erythrocytes; no leukocytes; no casts Urine protein 6000 mg/24 h The patient is started on prednisone, along with diuretics and an ACE inhibitor. Kidney biopsy results show class V (membranous) lupus nephritis with absent chronicity and mild activity. Which of the following is the most appropriate treatment of the kidney disease? Adalimumab Belimumab Cyclophosphamide Methotrexate Mycophenolate mofetil
Mycophenolate mofetil is the most appropriate initial immunosuppressive therapy in the treatment of isolated class V lupus nephritis, especially without kidney dysfunction.
A 54-year-old man is evaluated during a routine examination. He is asymptomatic, and he exercises regularly without any limitations. Medical history is significant for hypertension and a bicuspid aortic valve with an enlarged aortic root (measuring 5.1 cm 6 months ago). Family history is unremarkable. His only medication is losartan. On physical examination, blood pressure is 122/74 mm Hg. Cardiac examination reveals a midsystolic ejection click and a grade 2/6 crescendo-decrescendo systolic murmur at the second right intercostal space. The remainder of the examination is unremarkable. Transthoracic echocardiogram shows normal left ventricular function and a bicuspid aortic valve. The mean gradient across the aortic valve is 20 mm Hg, and the aortic valve area is 1.6 cm2. The ascending aorta is 5.1 cm; the descending thoracic aorta is incompletely visualized. Which of the following is the most appropriate management? Aortic valve replacement and ascending aortic repair Ascending aortic repair Dobutamine stress echocardiography Repeat echocardiography in 6 months
Patients with a bicuspid aortic valve and a thoracic aortic aneurysm should undergo echocardiography every 6 months if the aortic diameter is larger than 4.5 cm or the rate of enlargement exceeds 0.5 cm/year. The most appropriate management of this patient with an ascending thoracic aortic aneurysm and a bicuspid aortic valve is repeat echocardiography in 6 months. Patients with a bicuspid aortic valve are prone to enlargement of the ascending aorta, and patients with both a bicuspid aortic valve and enlarged aortic dimensions are at higher risk for aortic dissection. Surveillance echocardiography should be performed in these patients to monitor aortic growth. Patients with a bicuspid aortic valve and a thoracic aortic aneurysm should undergo annual imaging if the aortic diameter has been stable and smaller than 4.5 cm. If the aortic diameter is 4.5 cm or larger or the rate of enlargement exceeds 0.5 cm/year, imaging should be performed every 6 months. Operative aortic valve repair or replacement is indicated in asymptomatic patients with a bicuspid aortic valve and a thoracic aortic aneurysm diameter of 5.5 cm or larger, according to American College of Cardiology/American Heart Association guidelines. However, surgical intervention can be beneficial in those patients with a bicuspid aortic valve, thoracic aortic aneurysm diameter of 5.0 cm or larger, and an additional risk factor for dissection/rupture (such as family history of aortic dissection or aortic growth rate ≥0.5 cm/year). In this asymptomatic patient with an ascending aortic diameter of 5.1 cm without additional risk factors, it is premature to repair or replace the aorta. Some patients with reduced left ventricular (LV) function and calcific aortic stenosis have severe aortic stenosis based on valve area but a gradient that is less than 30 mm Hg. Whether symptoms in this "low-flow/low-gradient" aortic stenosis are caused primarily by aortic valve disease with resultant LV dysfunction or the effective valve area is reduced owing to poor leaflet excursion can be best determined with dobutamine stress echocardiography. In this patient, there is no indication to perform dobutamine stress echocardiography, as he has normal LV function.
A 25-year-old woman is hospitalized for a 4-week history of swelling of the legs, weight gain, and shortness of breath on exertion. She was diagnosed with systemic lupus erythematosus 1 year ago when she presented with polyarthritis, rash, and alopecia. She was initially treated with hydroxychloroquine and prednisone with a good response. On physical examination, blood pressure is 142/96 mm Hg; other vital signs are normal. There is pitting edema of the lower extremities extending to the knees. The remainder of the physical examination is normal. Laboratory studies: Erythrocyte sedimentation rate 68 mm/h Hematocrit 38% Complements (C3 and C4) Low Creatinine 1.0 mg/dL (88.4 µmol/L) Anti-Smith antibodies Positive Anti-double-stranded DNA antibodies Positive Urinalysis 3+ protein; no erythrocytes; no leukocytes; no casts Urine protein 6000 mg/24 h The patient is started on prednisone, along with diuretics and an ACE inhibitor. Kidney biopsy results show class V (membranous) lupus nephritis with absent chronicity and mild activity. Which of the following is the most appropriate treatment of the kidney disease? Adalimumab Belimumab Cyclophosphamide Methotrexate Mycophenolate mofetil
Mycophenolate mofetil is the most appropriate initial immunosuppressive therapy in the treatment of isolated class V lupus nephritis, especially without kidney dysfunction. Mycophenolate mofetil is the most appropriate treatment of this patient's kidney disease. Classification of lupus nephritis is based on findings by light microscopy, electron microscopy, and immunofluorescence. This patient with recently diagnosed systemic lupus erythematosus (SLE) now presents with proteinuria likely due to lupus nephritis (likely class V, membranous). Guidelines recommend aggressive therapy with immunosuppressives for significant kidney involvement. A number of immunosuppressive therapies are beneficial in the treatment of SLE nephritis, including mycophenolate mofetil, cyclophosphamide, azathioprine, and rituximab. In the treatment of isolated class V lupus nephritis, especially without kidney dysfunction, mycophenolate mofetil is the most appropriate initial immunosuppressive therapy based on the guideline recommendations. Importantly, mycophenolate mofetil is teratogenic and has been associated with fetal harm and death; it must be stopped 3 months before a planned pregnancy. Adalimumab has not been shown to be effective in lupus nephritis and may potentially worsen the disease based on animal data. Belimumab may be considered in patients with continued SLE activity after standard therapy has been tried and found to be ineffective. Its role in the treatment of lupus nephritis continues to evolve but is currently not well defined. Its use in this patient should not be considered before having tried standard therapy. Cyclophosphamide may be considered in this patient and used appropriately in patients with lupus nephritis but is not an appropriate first choice due to a higher rate of side effects compared with mycophenolate mofetil, as well as its effect on reducing fertility and premature menopause. Cyclophosphamide is typically reserved for severe active nephritis to induce remission, followed by mycophenolate mofetil or possibly azathioprine as maintenance therapy. Methotrexate is not effective in lupus nephritis and may be associated with toxicity in a patient with kidney disease.
A 52-year-old woman is evaluated for midabdominal pain and nausea. These symptoms worsen after eating large meals. The pain does not alleviate after bowel movements, and she reports frequent constipation. She has gained 2.3 kg (5.1 lb) over the past year. The pain began 1 year earlier, after she was hospitalized with an episode of gallstone pancreatitis for which she underwent laparoscopic cholecystectomy. After cholecystectomy, she had persistent abdominal pain. Over the next several weeks she had upper and lower endoscopies, a CT scan of the abdomen and pelvis, and multiple laboratory tests; all were normal. She states that the only thing that helps the pain is hydrocodone, but as the dose of hydrocodone has increased, so has her pain intensity. On physical examination, vital signs are normal; BMI is 26. Tenderness to palpation in all quadrants is noted on abdominal examination. Other findings are unremarkable. Which of the following is the most likely diagnosis? Chronic pancreatitis Distal intestinal obstruction syndrome Irritable bowel syndrome Narcotic bowel syndrome
Narcotic bowel syndrome, also known as opiate-induced gastrointestinal hyperalgesia, is a centrally mediated disorder of gastrointestinal pain characterized by a paradoxical increase in abdominal pain with increasing doses of opioids. Narcotic bowel syndrome is the most likely diagnosis in this patient. Narcotic bowel syndrome, also known as opiate-induced gastrointestinal hyperalgesia, is a centrally mediated disorder of gastrointestinal pain characterized by a paradoxical increase in abdominal pain with increasing doses of narcotics. Many patients are prescribed opioid pain medication for an unrelated medical condition but have increasing pain over time despite clinical evidence of improvement or resolution of the initial condition. Patients often fear tapering off opioids and believe the opioids are the only treatment that alleviates their pain. Chronic pancreatitis may result from a severe episode of acute pancreatitis or recurrent episodes of acute pancreatitis. It may require opioid pain medication for flares of pain or for constant, daily pain, but the pain is usually relieved with opioids. This patient only had one episode of uncomplicated pancreatitis and has no other risk factors for chronic pancreatitis and no evidence of chronic pancreatitis (pancreatic calcifications) on abdominal imaging. Distal intestinal obstruction syndrome is caused by thickened intestinal contents that completely or partially block the small intestinal lumen in patients with cystic fibrosis. It is characterized by progressive, crampy abdominal pain in the right lower quadrant, sometimes progressing to vomiting. This patient has no history of cystic fibrosis and no symptoms of a small-bowel obstruction making this an unlikely diagnosis. The diagnosis of irritable bowel syndrome is typically made in the presence of recurrent abdominal pain or discomfort at least 3 days per month in the last 3 months that is associated with two or more of the following: relief with defecation, onset associated with a change in frequency of stool, or onset associated with a change in form (appearance) of stool. While this patient has abdominal pain and constipation, her symptoms are worsened with eating, not relieved with defecation, and the onset of pain is clearly tied to an acute abdominal diagnosis and treatment with an opioid, making IBS an unlikely diagnosis.
A 64-year-old man is evaluated for treatment of extensive psoriasis on 20% body surface area, covering the knees, elbows, scalp, and trunk. Medical history is otherwise unremarkable, and he takes no medications. On physical examination, vital signs are normal. He has no fingernail changes and no arthritis. Skin findings are shown. Which of the following forms of phototherapy is the most appropriate initial treatment? Narrowband ultraviolet B (UVB) phototherapy Photodynamic therapy Psoralen plus ultraviolet A (PUVA) photochemotherapy Retinoid plus PUVA (RePUVA) photochemotherapy Ultraviolet A (UVA) phototherapy
Narrowband ultraviolet B (UVB) is the standard form of phototherapy used in the treatment of extensive psoriasis Narrowband ultraviolet B (UVB) therapy has become the standard form of phototherapy used in the initial treatment of psoriasis. The light source emits a narrow band of UVB radiation, in this case 311 nm of ultraviolet light. This wavelength was chosen for its efficacy and the fact that this wavelength of light is not absorbed by DNA. The theory is that if the light is not absorbed by DNA, less skin damage will occur, and there will be far fewer skin cancers as a side effect of the therapy. Broadband UVB was the mainstay of therapy until the advent of narrowband UVB. Broadband is still used in some centers today. Broadband UVB emits light in the 280 to 320 nm range; 260 nm is the wavelength of light with the maximum absorption by DNA. DNA absorption of UVB drops off dramatically after 290 nm. Broadband UVB contains wavelengths of light capable of causing DNA damage, leading to ultraviolet skin damage and ultimately skin cancer. Photodynamic therapy is not used for the treatment of psoriasis, but rather for the treatment of actinic keratoses. Aminolevulinic acid, a chemical precursor of the heme synthesis pathway, is applied to the skin before its exposure to visible blue light. Aminolevulinic acid is used as a photosensitizer and is absorbed by the actinically damaged skin cells; upon exposure to light it causes necrosis of the actinically damaged skin by producing reactive oxygen species. It can be extremely painful for patients to undergo photodynamic therapy. Psoralen plus ultraviolet A (PUVA) therapy is now used almost exclusively for the treatment of cutaneous T-cell lymphoma. It is effective for treating psoriasis, but it has been shown to cause an increase in the risk of nonmelanoma and melanoma skin cancer. For these reasons, it is not used as first-line therapy for psoriasis. Oral retinoid (acitretin) plus PUVA therapy, also called RePUVA, has been used in the past for treating psoriasis and cutaneous T-cell lymphoma. It is highly efficacious, but has the highest side effect profile of the various forms of phototherapy and is not used frequently today. Short-term complications of RePUVA include profound nausea in some and risk of treatment-related burns. Long-term complications include an 11-fold increase in cutaneous squamous cell carcinoma and slight increase in the risk for basal cell carcinoma, melanocytic atypia, and melanoma. Ultraviolet A (UVA) alone is rarely used in the dermatology clinic. Morphea (localized scleroderma) has been shown to benefit from exposure to UVA phototherapy.
A 35-year-old man is evaluated for chronic cough productive of foul-smelling sputum. He has been treated with four courses of antibiotics in the last 12 months; several infections have been associated with Pseudomonas species. His symptoms have been present for many years and are also associated with chronic sinusitis. He was recently diagnosed with infertility. On physical examination, all vital signs are normal. Oxygen saturation is 98% on ambient air. Clubbing is present. Pulmonary examination reveals rhonchi and wheezes in the upper lobes bilaterally. Chest radiograph reveals bilateral upper-lobe bronchiectasis. Laboratory studies reveal a negative sweat chloride test of 39 mEq/L (39 mmol/L). Which of the following is the most appropriate management? Begin chronic ciprofloxacin therapy Begin tiotropium Repeat sweat chloride testing Test for α1-antitrypsin deficiency
Negative sweat chloride testing does not exclude the diagnosis of cystic fibrosis in patients with high pretest probability of disease. The most appropriate next step is to repeat the sweat chloride testing. Diagnosis of cystic fibrosis (CF) is based on a combination of CF-compatible clinical findings in conjunction with either biochemical (sweat testing, nasal potential difference) or genetic (CFTR mutations) techniques. Genetic counseling should always occur before any genetic test is performed. The essential components of counseling include informing the patient of the test purpose, implications of diagnosis, and alternative testing options. Ultimately, the decision of whether or not to be tested rests with the patient. Use of the sweat test has been the mainstay of laboratory confirmation. Negative sweat chloride testing does not exclude the diagnosis of CF. This patient has many findings suggestive of CF upper-lobe predominant bronchiectasis, chronic sinus disease, colonization with Pseudomonas, clubbing, and infertility. Therefore, the index of suspicion for CF, as well as ciliary dyskinesia disorders, is extremely high for this patient. Because sweat chloride testing can give variable results, it is appropriate to repeat the testing and, if still negative, refer the patient to a center with expertise in CF or genetic testing. Chronic oral macrolide antibiotics and inhaled antibiotic therapy are beneficial in patients with a confirmed diagnosis of CF. However, individuals with bronchiectasis are often exposed to several courses of antibiotics, and resistance to quinolones is common. Therefore, initiation of chronic ciprofloxacin therapy is not indicated. In a recent phase 3 trial, tiotropium was well tolerated in patients with CF, but lung function improvements compared with placebo were not statistically significant and such treatment is not generally recommended. Patients with α1-antitrypsin deficiency can present with lung disease and liver disease. A characteristic radiographic finding of the emphysema associated with α1-antitrypsin deficiency is bullous changes most prominent at the bases, which are not present in this patient. Additionally, α1-antitrypsin deficiency cannot account for the patient's sinus disease or infertility. Therefore, α1-antitrypsin deficiency testing would not be the most appropriate next diagnostic test in this patient with a clinical history suggestive of possible CF.
A 69-year-old woman is evaluated for 6 months of progressive dysphagia. She has been previously healthy and takes no medications. On physical examination, vital signs and the remainder of the physical examination are normal. Results of upper endoscopy and biopsy indicate adenocarcinoma of the gastroesophageal junction. The staging evaluation reveals a T3 tumor on ultrasound and no evidence of distant metastatic disease. The patient's tumor is technically resectable. Which of the following is the most reasonable treatment strategy? Adjuvant radiation therapy Neoadjuvant chemotherapy plus radiation therapy Palliative chemotherapy Surgery alone
Neoadjuvant therapy is given before a planned curative-intent surgery to patients with fully resectable gastroesophageal disease to eradicate micrometastatic and increase the chance for cure. The most reasonable treatment strategy is neoadjuvant chemotherapy plus radiation therapy. Although only 30% to 40% of patients with gastroesophageal cancer have potentially resectable disease at presentation, patients with local and locoregional disease are typically treated surgically. Unfortunately, recurrence rates are high, and cure rates with surgical resection alone remain low. Studies have shown that administration of neoadjuvant chemotherapy improves outcome to a modest but statistically significant degree. Neoadjuvant chemotherapy plus radiation therapy is given before a planned curative-intent surgery to patients who are believed to have fully resectable disease to eradicate any micrometastatic disease that might be present outside of the surgical field and thus increase the chance for cure. There is often confusion between neoadjuvant chemotherapy and conversion chemotherapy. The key distinction between neoadjuvant chemotherapy and conversion chemotherapy is that in conversion chemotherapy, the patient's tumor is not believed to be currently resectable but could possibly become resectable if adequate tumor regression can be accomplished with chemotherapy. Thus, both neoadjuvant and conversion chemotherapy are given with a plan for surgery to follow; however, neoadjuvant therapy is administered to patients with micrometastatic disease thought to be resectable, whereas conversion therapy is given to shrink visible tumors in patients with unresectable disease in whom disease might become resectable with adequate tumor regression. Adjuvant therapy is given after curative-intent surgery to eradicate any residual micrometastatic disease and increase the chance for cure. Because of the low cure rates for locoregional therapy for esophageal cancer, chemotherapy has been added to many treatment regimens, and many patients are currently treated with combination chemoradiation following surgery for resectable disease. However, the optimal treatment regimen and the overall effectiveness of different treatment approaches have not yet been established. However, there is no evidential support for adjuvant radiation therapy (surgery followed by radiation therapy) for patients with localized gastroesophageal cancer. Palliative therapy is given to patients with incurable cancer to prolong survival or control symptoms. Palliative care should be considered for patients with poor performance status due to chronic medical comorbidities, for those who have not been able to tolerate initial treatment attempts, or for those who have exhausted standard treatment options. Read Related TextNext Question
A 67-year-old woman is evaluated in December for a 2-day history of runny nose, fever, headache, myalgia, cough, and sore throat. Medical history is notable for heart failure. She lives at home with her husband and is able to eat, drink, and take her oral medications. Her medications are carvedilol and enalapril. She received a standard-dose influenza immunization in October. On physical examination, temperature is 38.6 °C (101.5 °F), blood pressure is 145/62 mm Hg, pulse rate is 98/min, and respiration rate is 20/min. The patient has rhinorrhea, and the pharynx is erythematous. Lungs are clear to auscultation. A chest radiograph shows no infiltrates. A nasopharyngeal swab is positive for influenza B. Which of the following is the most appropriate treatment? Amantadine Oseltamivir Peramivir Rimantadine
Neuraminidase inhibitors (oseltamivir, zanamivir, peramivir) and the endonuclease inhibitor baloxavir are indicated for the treatment of influenza A and B and can be administered through various routes (oral, intranasal, intravenous).
A 67-year-old woman is evaluated in December for a 2-day history of runny nose, fever, headache, myalgia, cough, and sore throat. Medical history is notable for heart failure. She lives at home with her husband and is able to eat, drink, and take her oral medications. Her medications are carvedilol and enalapril. She received a standard-dose influenza immunization in October. On physical examination, temperature is 38.6 °C (101.5 °F), blood pressure is 145/62 mm Hg, pulse rate is 98/min, and respiration rate is 20/min. The patient has rhinorrhea, and the pharynx is erythematous. Lungs are clear to auscultation. A chest radiograph shows no infiltrates. A nasopharyngeal swab is positive for influenza B. Which of the following is the most appropriate treatment? Amantadine Oseltamivir Peramivir Rimantadine
Neuraminidase inhibitors (oseltamivir, zanamivir, peramivir) and the endonuclease inhibitor baloxavir are indicated for the treatment of influenza A and B and can be administered through various routes (oral, intranasal, intravenous). The most appropriate treatment for this patient is the neuraminidase inhibitor oseltamivir. During a confirmed local influenza outbreak, infection can be reliably diagnosed on the basis of clinical criteria alone. When confirmation is needed, nucleic acid amplification tests of respiratory samples from nasopharyngeal swabs detect influenza A and B. Detection of viral nucleic acid by polymerase chain reaction (PCR) is rapid, has high sensitivity and specificity, and can determine the type and subtype of influenza virus. Annual influenza vaccination is the most effective intervention for preventing influenza and is recommended for all persons 6 months or older. Only a few randomized trials have assessed the efficacy of influenza vaccines in older individuals. These trials suggest that the vaccines are approximately 60% effective against influenza in adults 65 years and older. Nevertheless, immunization is likely to prevent hospitalization in older adults. Neuraminidase inhibitors have activity against influenza A and B. They can be given orally (oseltamivir), intranasally (zanamivir), or intravenously (peramivir). Baloxavir, a polymerase acidic endonuclease inhibitor, is another oral treatment option; however, this agent has only been studied in outpatients at low risk for complications and is more expensive than oseltamivir. Antiviral therapy is recommended for severe disease, including all hospitalized patients or those with confirmed or suspected influenza infection at high risk for complications. Older adult patients (older than ≥65 years), young children, pregnant women, and patients with chronic medical conditions (especially chronic lung disease and heart disease) are at higher risk for severe primary influenza complications such as superimposed bacterial pneumonia caused by Streptococcus pneumoniae or Staphylococcus aureus and death. M2 inhibitors, such as amantadine and rimantadine, were the first agents introduced for influenza treatment, but they are only active against influenza A. Therefore, these agents would be ineffective against this patient's infection with influenza B. The Infectious Diseases Society of America guidelines recommend initiating antiviral treatment as soon as possible for hospitalized adults with documented or suspected influenza and for outpatients with severe or progressive illness regardless of illness duration. Treatment is indicated as soon as possible for all other patients at high risk for influenza-related complications. Treatment of otherwise healthy outpatients can be considered if started within 48 hours of symptom onset. Antiviral therapy in patients with documented influenza decreases recovery time, hospitalization rates, and complications. Because this patient can be treated as an outpatient, oseltamivir, baloxavir, or zanamivir would be appropriate choices. Intravenous peramivir is typically reserved for patients who cannot tolerate or are otherwise incapable of taking inhaled or enteral agents.
A 61-year-old woman is admitted to the ICU 1 hour after having a subarachnoid hemorrhage (SAH). An emergent CT scan of the head showed an SAH, and a cerebral angiogram revealed a 6-mm rupture of the anterior communicating artery. The patient has hypertension treated with amlodipine. On physical examination, vital signs are stable, and oxygen saturation is 96% with the patient breathing ambient air. She is somnolent. Nuchal rigidity is present. Subhyaloid hemorrhages are seen on funduscopic examination. No motor or sensory deficits are present. Administration of which of the following medications is the most appropriate treatment? Dexamethasone Magnesium sulfate Nimodipine Nitroglycerin Verapamil
Nimodipine is indicated for all nonhypotensive patients with subarachnoid hemorrhage and is associated with improved neurologic outcomes and survival. This patient should receive nimodipine. She has an aneurysmal subarachnoid hemorrhage (SAH) and is at high risk for neurologic decline from cerebral vasospasm. The risk of vasospasm is greatest 5 to 10 days after SAH onset. Nimodipine is indicated for all nonhypotensive patients with SAH and is associated with improved neurologic outcomes and survival. Because nimodipine may not directly reduce the vasospasm, additional monitoring with repeat neurologic examinations and transcranial Doppler ultrasonography is indicated in a specialized ICU. The presence of vasospasm is suggested by worsening findings on neurologic examination and can be confirmed with CT angiography or catheter angiography, with the latter test having the additional benefit of providing endovascular therapy. Glucocorticoids, such as dexamethasone, are not routinely indicated for any stroke subtype and are ineffective in reducing intracranial pressure in that setting. One randomized study of 95 patients with aneurysmal SAH treated with 3 days of methylprednisolone showed improved 1-year functional outcomes in patients. However, the preponderance of evidence does not support glucocorticoid treatment for SAH. Magnesium sulfate therapy has been used to prevent vasospasm following SAH, but randomized trials have not shown improved clinical outcomes. A 2013 systematic review and meta-analysis concluded that while intravenous magnesium reduced delayed cerebral ischemia, there was no improvement in clinical neurologic outcomes. Nitroglycerin is not an appropriate treatment for this patient. Given the risk of cerebral vasospasm, blood pressure often is allowed to remain elevated in such patients. If her blood pressure were to require treatment, nitroglycerin would be inappropriate because it can increase cerebral venous volume and intracranial pressure, which is a concern in patients with SAH. Verapamil is a calcium channel blocker, as is nimodipine, but verapamil has not been shown to improve outcomes in patients with SAH or vasospasm. Read Related TextNext Question
A 36-year-old woman is evaluated for a 3-year history of fatigue that worsens after activity and does not improve with rest. She also notes intermittent diffuse myalgia and arthralgia, constipation, dizziness, headaches, urinary urgency, memory problems, and paresthesias. Her musculoskeletal symptoms, dizziness, and headache worsen in the upright position and improve when she lies back down. She has almost entirely eliminated social activities. Medical history is significant for episodic migraine and irritable bowel syndrome. Medications are sumatriptan, polyethylene glycol, and hyoscyamine. On physical examination, vital signs are normal. BMI is 24. Neck circumference is 36 cm (14 in). The remainder of the examination is normal. Laboratory studies obtained 6 months ago showed a normal complete blood count, electrolyte levels, kidney function test results, liver chemistry test results, fasting glucose level, serum creatine kinase level, and serum thyroid-stimulating hormone level. Which of the following is the most appropriate diagnostic test to perform next? Antinuclear antibody assay Polysomnography Serum cortisol level measurement No further testing is recommended
No further diagnostic testing is required in this patient. She meets the diagnostic criteria for systemic exertion intolerance disease (SEID), which are fatigue of at least 6 months' duration accompanied by substantial reduction in preillness activities, postexertional malaise, unrefreshing sleep, and either cognitive impairment or orthostatic intolerance. Although the pathophysiology of SEID remains unclear, the phenomenon of central sensitization (the pathophysiologic dysregulation of the thalamus, hypothalamus, and amygdala) is gaining acceptance as a potential cause of SEID as well as of other highly prevalent comorbid conditions, including fibromyalgia, mood disturbances, irritable bowel syndrome, and interstitial cystitis. This patient's history, examination, and previous diagnostic test results point to central sensitization, as demonstrated by the constellation of such symptoms as diffuse arthralgia and myalgia, chronic fatigue, bowel and bladder irritability, chronic headaches, brain fog, paresthesias, and unrefreshing sleep. In patients with SEID, the history and physical examination should guide the choice of diagnostic tests. It is reasonable to obtain a complete blood count, creatine kinase (for myalgia), electrolyte panel, thyroid-stimulating hormone level, fasting glucose level, and kidney and liver chemistry tests; however, unnecessary laboratory, imaging, and invasive studies should be avoided because most patients will have unrevealing findings, which provide no lasting reassurance to patients. In this case, the diagnostic evaluation should be limited unless there is compelling new information to warrant further testing. Antinuclear antibody testing is an effective screening tool for systemic lupus erythematosus; however, myalgia, arthralgia, and fatigue are insufficient reasons to test for antinuclear antibodies unless accompanied by objective findings of systemic lupus erythematosus. Patients at moderate to high risk for obstructive sleep apnea should undergo further testing, including a home sleep study or polysomnography. On the basis of this patient's presentation (female, young, normal BMI and neck circumference, lack of daytime sleepiness), she is considered to be at low risk for obstructive sleep apnea, and further sleep testing is not warranted. Serum cortisol testing is unnecessary in this patient who is not manifesting findings that are suggestive of adrenal failure or insufficiency, such as hypotension, tachycardia, hyponatremia, and hyperkalemia.
A 75-year-old man is evaluated for a 5-year history of hand joint pain and morning stiffness lasting a few minutes. He walks about a mile every day and plays tennis 3 days a week; however, the pain has started to limit him from gripping a tennis racket. He also has noticed fatigue for the past 6 months. He reports no other constitutional symptoms, shortness of breath, or rash. History is also significant for hypertension and diet-controlled type 2 diabetes mellitus. Medications are losartan, acetaminophen as needed, and aspirin, 81 mg/d. On physical examination, vital signs are normal. Bony enlargement of multiple proximal and distal interphalangeal joints of the hands is noted. The remainder of the physical examination is normal. Laboratory studies show an erythrocyte sedimentation rate of 22 mm/h, a hematocrit level of 41%, and a C-reactive protein level of 0.9 mg/dL (9.0 mg/L). Hand radiographs show joint-space narrowing and osteophytes of multiple proximal and distal interphalangeal joints, and similar changes are seen in both first carpometacarpal joints; there are no erosions. Which of the following is the most appropriate next step in management? Anti-cyclic citrullinated peptide antibodies Antinuclear antibodies Rheumatoid factor Serum urate level No further testing
No further testing is currently needed in this 75-year-old man with no clinical findings suggestive of a connective tissue disease. His symptoms of hand joint pain are noninflammatory (pain with use, no warmth or swelling, and only minimal morning stiffness), and the distribution and findings are consistent with a diagnosis of osteoarthritis. Furthermore, the radiographs confirm these findings and do not show changes consistent with an inflammatory rheumatologic disorder (such as erosive joint disease seen in rheumatoid arthritis and psoriatic arthritis, or tophi seen in gout). Therefore, no further testing is indicated. Positive antinuclear antibodies (ANA) in a patient with nonspecific symptoms are difficult to interpret. As with all tests, the positive predictive value of ANA rests upon the pretest probability of disease. In the presence of 0 or 1 clinical manifestations of systemic lupus erythematosus (SLE), a positive ANA is associated with a very low posttest probability of SLE. The American College of Rheumatology's Choosing Wisely list currently recommends against testing ANA and ANA subserologies without a clinical suspicion of immune-mediated disease. The clinical suspicion for a connective tissue disorder such as SLE or rheumatoid arthritis is low in this patient, and ANA, rheumatoid factor, or anti-cyclic citrullinated peptide antibodies should not be tested. Osteoarthritis, psoriatic arthritis, and gout can involve distal interphalangeal joints. However, this patient does not have the rash seen in psoriatic arthritis or symptoms of crystal-induced arthritis such as recurrent acute attacks and/or tophaceous deposits. Serum urate levels are helpful in a patient with symptoms suggestive of gout but are unlikely to helpful here.
A 28-year-old man is evaluated in the emergency department for acute right-sided flank pain and blood in the urine. He reports no prior episodes of hematuria or flank pain. He takes no medications. On physical examination, vital signs are normal. Costovertebral angle tenderness is noted. The abdomen is soft and nontender. Urinalysis shows 3+ blood, trace protein, and too numerous to count erythrocytes. A kidney ultrasound shows normal-appearing kidneys, no hydronephrosis, and no nephrolithiasis. Which of the following is the most appropriate test to perform next? Contrast MRI Contrast-enhanced helical abdominal CT Kidney, ureter, and bladder plain radiography Noncontrast helical abdominal CT
Noncontrast helical abdominal CT is the most appropriate next test for this patient with nephrolithiasis suggested by unilateral flank pain and hematuria. Ultrasonography is an appropriate initial diagnostic test for suspected nephrolithiasis; it is easily available, safe, and relatively inexpensive, and it is the study of choice in pregnant women. Ultrasonography can demonstrate hydronephrosis, kidney size and cortical thickness, echogenicity, and the presence of cysts and tumors. It is useful for uncomplicated nephrolithiasis; a positive ultrasound may be adequate for initial diagnosis. Ultrasonography is less useful in evaluating diseases of the mid or distal ureter, including stones. Furthermore, the absence of hydronephrosis on ultrasound does not rule out kidney stones. Noncontrast helical CT is the gold standard for diagnosis of nephrolithiasis and is appropriate for evaluating renal colic. Most stones can be detected, including small stones and those in the distal ureter not detected on ultrasound. It may provide information regarding stone composition and, because the entire urinary tract and abdomen are visualized, alternative diagnoses may be suggested. MRI with contrast is not as sensitive as CT in detecting suspected kidney stones. Due to lack of radiation, MRI may be useful in pregnant women with stone disease if ultrasound is nondiagnostic. Contrast abdominal CT characterizes renal tumors and cysts, whereas CT urography is the preferred test for patients with unexplained urologic/nonglomerular hematuria. The decision to use contrast depends on the clinical scenario, the patient's risk factors for contrast-induced nephropathy, and the availability and utility of alternative imaging modalities. It is unnecessary in the evaluation of suspected nephrolithiasis and poses greater risk and cost in this situation compared with either ultrasonography or noncontrast helical abdominal CT. Plain abdominal radiography has limited utility due to its inability to detect radiolucent uric acid stones and does not provide as much anatomic information as other modalities. However, it may be useful in assessing stone burden in patients with known radiopaque stones but is not the initial test of choice for acute nephrolithiasis. Read Related TextNext Question
A 65-year-old man is evaluated in the ICU for a 24-hour history of altered mental status with a fluctuating level of consciousness. He was admitted to the hospital 5 days ago for urosepsis and acute kidney injury and developed acute respiratory distress syndrome on hospital day 3. The patient is currently intubated, mechanically ventilated, and receiving continuous hemodialysis. Medications are cefepime, norepinephrine, and fentanyl. On physical examination, temperature is 38.4 °C (101.1 °F), blood pressure is 105/71 mm Hg, pulse rate is 108/min, and respiration rate is 12/min; FIO2 is 0.9. The patient opens his eyes to voice but does not fixate on the examiner or follow commands. Pupils are reactive, and gag and corneal reflexes are present. All limbs move intermittently but not on command. The patient withdraws from painful stimuli in all four limbs. Intermittent twitching of the shoulders and eyelids is noted. Results of laboratory studies show a serum creatinine level of 5.4 mg/dL (477 µmol/L). Glucose, calcium, and electrolyte levels are within normal limits. A 20-minute electroencephalogram shows generalized slow activity, a nonspecific finding compatible with encephalopathy, but no evidence of seizure activity. A head CT scan is normal. Which of the following is the most appropriate next step in management? Continuous (24-hour) electroencephalography Intravenous fosphenytoin Intravenous lorazepam No treatment
Nonconvulsive status epilepticus should be suspected in patients with critical illness who develop altered mental status; the diagnosis is confirmed with continuous (24-hour) electroencephalography. The patient should undergo continuous (24-hour) electroencephalography (EEG). He has fluctuating mental status of unknown origin in the setting of a critical illness, sepsis, and multiorgan dysfunction. These findings raise suspicion of nonconvulsive status epilepticus (NCSE), which is intermittent electrical seizure activity without clinically evident seizure activity. A 20-minute EEG usually is inadequately sensitive to capture seizures, particularly if they are occurring intermittently. NCSE also is increasingly diagnosed in critically ill comatose or stuporous patients with acute neurologic or medical conditions who have not had a convulsive seizure. Similar to NCSE following CSE, NCSE in critically ill populations also is associated with increased morbidity and mortality and requires prompt attention and intervention. The diagnostic test of choice is continuous (24-hour) EEG monitoring. Most critically ill patients with nonconvulsive seizures have their first detectable seizure with EEG monitoring within 24 hours of initiating the recording, but an additional 12% have detectable seizures with 48 hours of continuous monitoring. Cefepime has been known to cause encephalopathy, coma, and status epilepticus in patients with or without epilepsy, especially those with acute kidney injury, and its use should raise the clinician's suspicion of NCSE. Empiric treatment with fosphenytoin or lorazepam is generally not recommended until the diagnosis of NCSE or intermittent nonconvulsive seizures is confirmed by EEG. This approach differs from the treatment of generalized convulsive status epilepticus, which is diagnosed clinically and, optimally, treated without delay. Providing no treatment is inadequate as the next step in management for this patient with altered mental status, a fluctuating level of consciousness, and other abnormalities on neurologic and laboratory testing. The diagnosis of NCSE first must be established (or excluded).
An 83-year-old man is seen in the office for routine follow-up. He has a history of hypertension and atrial fibrillation. Medications are hydrochlorothiazide and warfarin. On physical examination, vital signs are normal. During lung auscultation, a 0.4 × 0.4-cm pink pearly papule with telangiectasias on his back is found. Biopsy of the lesion reveals a basal cell carcinoma with low-risk histology. Which of the follow is the most appropriate treatment? Electrodesiccation and curettage Mohs micrographic surgery Radiation Vismodegib
Noninfiltrating basal cell carcinomas on low-risk areas such as the trunk and extremities are best treated with electrodesiccation and curettage. Electrodesiccation and curettage (ED&C) is a widely used treatment for noninfiltrating basal cell carcinomas on low-risk anatomic sites (trunk and extremities). Treatment of basal cell carcinomas depends on many factors, including the histologic subtype, location, size, cosmetic considerations, and patient's age and comorbidities. Nodular and superficial basal cell carcinomas on the trunk and extremities are often treated with ED&C. Infiltrative and micronodular basal cell carcinomas, especially on the face, are not appropriate for ED&C. For this patient, his lesion is a small, histologically low-risk subtype of basal cell carcinoma on the back that can be treated with ED&C. Mohs micrographic surgery is a specialized surgical procedure that provides margin control while sparing as much normal skin as possible. Indications for Mohs micrographic surgery include tumors with aggressive histologic subtypes (micronodular, morpheaform, infiltrative, perineural involvement); high-risk and cosmetically sensitive locations (face, genitals); large tumors or tumors arising in scar tissue; and tumors in patients who are immunosuppressed. This patient's lesion would not be an appropriate use of Mohs surgery. Radiation therapy for skin cancers is appropriate for patients who refuse surgery or are not optimal surgical candidates. The patient will be able to tolerate an ED&C without any difficulty. Vismodegib is an oral medication that inhibits the hedgehog signaling pathway; this signaling pathway is aberrant in most basal cell carcinomas. It is reserved for locally advanced or metastatic basal cell carcinomas.
A 46-year-old man is evaluated during a follow-up visit for elevated blood pressure. His average blood pressure with home blood pressure monitoring is 126/77 mm Hg. He has no other pertinent medical history and takes no medications. On physical examination, blood pressure is 128/78 mm Hg; other vital signs are normal. BMI is 28. The remainder of the examination is unremarkable. Which of the following is the most appropriate next step in management? Initiate therapy with an angiotensin receptor blocker Initiate therapy with a thiazide diuretic Initiate a trial of lifestyle modification Remeasure blood pressure in 1 year
Nonpharmacologic therapy alone is especially useful for prevention of hypertension, including in adults with elevated blood pressure, and for management of high blood pressure in adults with milder forms of hypertension. The most appropriate next step in management is lifestyle modification. According to the American College of Cardiology/American Heart Association (ACC/AHA) blood pressure guideline, this patient has elevated blood pressure (BP), defined as systolic BP between 120-129 mm Hg and diastolic BP <80 mm Hg. Meta-analysis of observational studies has demonstrated that elevated BP and hypertension (systolic BP >130 mm Hg or diastolic BP >80 mm Hg) are associated with an increased risk of cardiovascular disease, end-stage kidney disease, subclinical atherosclerosis, and all-cause death. Nonpharmacologic therapy alone is especially useful for prevention of hypertension, including in adults with elevated BP, and for management of high BP in adults with milder forms of hypertension. Recommended lifestyle modifications include weight loss in adults with overweight or obesity; a heart-healthy diet that facilitates achieving a desirable weight; reduced sodium intake; high potassium intake (unless contraindicated by the presence of chronic kidney disease or use of drugs that reduce potassium excretion); increase in physical activity; and limiting alcohol consumption of standard drinks to no more than two (men) or one (women) per day. According to the ACC/AHA BP guideline, the use of antihypertensive medications is recommended for secondary prevention of recurrent coronary events in patients with clinical coronary vascular disease and an average systolic BP of ≥130 mm Hg or an average diastolic BP of ≥80 mm Hg. Drug therapy is also recommended for primary prevention in adults with an estimated 10-year atherosclerotic cardiovascular disease risk ≥10% and an average systolic BP ≥130 mm Hg or average diastolic BP ≥80 mm Hg. For initiation of antihypertensive drug therapy, first-line agents include thiazide diuretics, calcium channel blockers, and ACE inhibitors or angiotensin receptor blockers. Because this patient has elevated BP (not hypertension), initiation of drug therapy is not indicated at this time. The ACC/AHA recommends that adults with an elevated BP, such as this patient, should be managed with nonpharmacologic therapy and have a repeat BP evaluation within 3 to 6 months. Patients with normal BP (systolic BP <120 mm Hg and diastolic BP <80 mm Hg) can be reevaluated in 1 year.
A 45-year-old man is evaluated for a 2-month history of a burning sensation starting in his stomach and radiating into his chest, usually occurring 4 to 5 times weekly. He says that he usually eats dinner late and then goes to sleep. He often wakes up with a sour taste in his mouth. He reports no dysphagia or unintentional weight loss. He takes no medication. On physical examination, vital signs are normal; BMI is 34. The remainder of the examination, including abdominal examination, is unremarkable. Which of the following is the most appropriate next step in management? Ambulatory pH testing Barium esophagography Empiric trial of a proton pump inhibitor Upper endoscopy
Patients with a clinical diagnosis of gastroesophageal reflux disease should start an empiric trial of a proton pump inhibitor in conjunction with lifestyle and dietary changes, with no further testing. The most appropriate next step in the management of this patient is the initiation of a proton pump inhibitor (PPI) in conjunction with lifestyle and dietary changes. The diagnosis of gastroesophageal reflux disease (GERD) is made clinically. This patient's symptoms of burning pain in his abdomen and chest and a sour taste in his mouth occurring more than a few times a week are consistent with GERD. Due to the frequency of his symptoms, the best initial treatment is a once-daily PPI for a period of up to 8 weeks. Weight reduction is suggested for patients with recent weight gain or overweight. Interventions such as raising the head of the bed and eliminating meals within 2 to 3 hours of bedtime are helpful for nocturnal GERD. Cessation of alcohol and tobacco use is universally supported. Treatment for less frequent symptoms could include famotidine or other H2 blockers. Ambulatory pH testing is used to determine acid exposure in the esophagus in patients considering antireflux surgery. Indications for surgery include patient preference to stop taking medication, medication side effects, and refractory symptoms despite optimized medical therapy. Laparoscopic fundoplication and bariatric surgery (in patients with obesity) are surgical methods used to treat GERD. This patient has no surgical indications. A barium esophagram is the initial test for evaluation of achalasia. The main symptom of achalasia is dysphagia to both solids and liquids along with regurgitation of undigested food and saliva. Patients may also report unintentional weight loss, chest pain, and heartburn. A barium esophagram is not indicated because this patient's symptoms are more consistent with GERD than with achalasia. Upper endoscopy is indicated in patients with alarm symptoms, such as dysphagia or weight loss, and in patients whose symptoms do not respond to a PPI. Upper endoscopy is not indicated in this patient with typical GERD symptoms because most patients with typical symptoms have normal upper-endoscopy findings. Upper endoscopy is useful for the evaluation of the esophagus to identify damage in the form of erosive esophagitis, stricture, or Barrett esophagus.
A 33-year-old woman is evaluated for redness over her right calf at the site of a scratch that occurred 2 days ago. She has been well otherwise and takes no medications. On physical examination, vital signs are normal. Tender erythema measuring 4 × 3 cm is noted over the right lower leg. No purulence, induration, or fluctuance is present. Which of the following is the most appropriate treatment? Clindamycin Doxycycline Trimethoprim-sulfamethoxazole Vancomycin
Nonpurulent cellulitis without systemic signs of infection is usually caused by streptococci, which can be treated with an oral agent such as clindamycin, penicillin, cephalexin, or dicloxacillin. The most appropriate treatment for this patient is clindamycin. Treatment of skin and soft tissue infections is guided by categorizing the infection as purulent or nonpurulent and by grading the severity of the infection as mild, moderate, or severe. This patient has a nonpurulent cellulitis without systemic signs of infection (mild infection). Nonpurulent infections are typically caused by streptococci, and outpatient treatment with an oral agent active against streptococci, including clindamycin, penicillin, cephalexin, or dicloxacillin, is recommended. After incision and drainage, trimethoprim-sulfamethoxazole or doxycycline would be appropriate empiric therapy for moderate-severity (systemic signs of infection present) purulent skin infections (such as a furuncle or carbuncle) in which the usual cause is Staphylococcus aureus (including methicillin-resistant strains [MRSA]). Because of the increased likelihood of MRSA contributing to infection, intravenous vancomycin would be appropriate to include for empiric treatment of severe purulent skin infections. Severe infections are those that do not improve with incision and drainage plus oral antibiotics or that have specific systemic signs of infection, including temperature greater than 38 °C (100.4 °F), heart rate greater than 90/min, respiration rate greater than 24/min, abnormal leukocyte count (>12,000/µL [12 × 109/L] or <4000/µL [4 × 109/L]), or infection in immunocompromised patients. Intravenous vancomycin is also appropriate for severe nonpurulent necrotizing skin infections (in addition to coverage of gram-negative bacilli and anaerobes). In select patients with cellulitis of moderate severity, vancomycin is recommended for coverage of streptococci and MRSA. These would include cellulitis in patients who are injection drug users, those with nasal MRSA colonization or extracutaneous MRSA infection, or those whose infection is associated with penetrating trauma.
A 19-year-old man is evaluated for multiple episodes of nonbloody diarrhea, fever, occasional vomiting, malaise, and crampy abdominal pain that began yesterday. He is a college student who adopted a pet corn snake 2 months ago; the snake is healthy. On physical examination, temperature is 38.3 °C (100.9 °F), blood pressure is 110/60 mm Hg, pulse rate is 100/min, and respiration rate is 19/min. He appears to be in mild distress. On abdominal examination, bowel sounds are present, as is tenderness to palpation. No rebound or guarding is noted. Stool testing for occult blood is positive. Which of the following is the most likely cause of this patient's diarrheal illness? Chlamydia psittaci Erysipelothrix rhusiopathiae Mycobacterium marinum Nontyphoidal Salmonella species
Nontyphoidal Salmonella is commonly carried asymptomatically by reptiles and amphibians and transferred from the animals' feces to people; human symptoms include crampy abdominal pain, fever, nonbloody diarrhea, and vomiting. The most likely diagnosis is nontyphoidal Salmonella infection. This patient's fever and nonbloody diarrhea are most likely caused by nontyphoidal Salmonella, with infection resulting from contact with a colonized snake. Nontyphoidal Salmonella infection usually results from ingesting fecally contaminated water or food of animal origin, including poultry, beef, eggs, and milk. Intestinally colonized reptiles and amphibians are asymptomatic and intermittently shed the organism in their feces, creating the potential for fecal-oral route transmission. Handling infected snakes, turtles, iguanas, frogs, or toads or anything in the enclosures in which they live can result in infection. Surfaces contaminated by feces may also serve as a source of infection. The incubation period is usually less than 3 days, and symptoms typically include crampy abdominal pain, diarrhea (not usually visibly bloody), fever, headache, nausea, and vomiting. Infection with Chlamydia psittaci is typically acquired by inhaling the organism in feces from a pet bird. The incubation period is about 1 week and the clinical presentation usually consists of chills, dry cough, fever, headache, and myalgia. Diarrhea may be present but is much less common. Chest radiograph abnormalities are common. Erysipelothrix rhusiopathiae is a bacterium that infects animals such as fish, swine, and poultry. Human infection is usually occupationally acquired in butchers, fish handlers, and veterinarians. Localized cutaneous violaceous lesions of the fingers and hands are a classic finding, although more diffuse cutaneous infections, bacteremia, and even infective endocarditis can develop. Mycobacterium marinum is a nontuberculous mycobacterium found worldwide in freshwater and saltwater aquatic environments. Skin infections result from skin trauma and contact with fish tanks ("fish tank granuloma"), fish, or shellfish. Persons predisposed to infection include aquatic sports enthusiasts, fish tank owners, fishermen, and seafood workers. The clinical course is often insidious, manifesting initially as a violaceous or erythematous papule or nodule at the site of inoculation, which may ulcerate. Lesions may be solitary or multiple; occasionally a sporotrichoid distribution along lymphatic vessels may develop. M. marinum does not cause diarrhea.
A 19-year-old man is evaluated for a heart murmur noted on physical examination. He reports no symptoms. Medical history is noncontributory. He takes no medications. On physical examination, vital signs are normal. The patient is of short stature with hypertelorism, neck webbing, and a low hairline. The central venous pressure is elevated with a prominent a wave. Apical impulse is normal, and a prominent impulse is noted at the left upper sternal border. The S1 is normal; the S2 is soft. A grade 4/6 late-peaking systolic murmur is heard best at the left sternal border and second left intercostal space. An ejection click is not audible. The remainder of the examination is unremarkable. Echocardiogram demonstrates a dysplastic pulmonary valve with a peak instantaneous systolic gradient of 65 mm Hg. Pulmonary regurgitation is moderate. The right ventricular size and function are normal, but right ventricular hypertrophy is present. The left heart size and function are normal. Which of the following is the most likely genetic disorder responsible for this patient's findings? Down syndrome Marfan syndrome Noonan syndrome Turner syndrome
Noonan syndrome is commonly associated with congenital cardiac lesions, including pulmonary stenosis. This patient has features of Noonan syndrome, which is an autosomal dominant disorder commonly associated with congenital cardiac lesions, including pulmonary stenosis; the valve is usually dysplastic. Noonan syndrome should be considered in all patients with pulmonary stenosis, particularly those with short stature, variable intellectual impairment, unique facial features, neck webbing, hypertelorism, and other cardiac abnormalities, including hypertrophic cardiomyopathy, atrial septal defect, and ventricular septal defect. This patient has severe pulmonary stenosis demonstrated by the physical examination findings of a palpable, late-peaking systolic murmur located at the second left intercostal space; absence of an ejection click; and features of right ventricular pressure overload. An ejection click may be audible in patients with pulmonary stenosis, but as severity progresses, the click disappears owing to loss of valve pliability. The echocardiogram in this patient confirms severe pulmonary stenosis with a peak systolic gradient of 65 mm Hg. Down syndrome is a genetic disorder caused by the presence of a full or partial extra copy of chromosome 21. Characteristic features include small stature, unique facial features, and intellectual delays. Congenital cardiovascular disease occurs in more than 40% of affected persons, most commonly as a form of atrioventricular septal defect, including ostium primum atrial septal defect, inlet ventricular septal defect, or complete atrioventricular septal defect. Pulmonary stenosis is not typically seen unless associated with tetralogy of Fallot, which is also common in persons with Down syndrome. Marfan syndrome is a genetic disorder caused by a mutation on chromosome 15. Findings typically include ocular, cardiovascular, skeletal, and systemic features that include tall stature with overgrowth of the long bones. The most common cardiovascular feature is aortic sinus dilatation with a propensity to dissection. Turner syndrome is a genetic disorder that affects girls and women and is characterized by complete or partial absence of one of the X chromosomes. Affected persons often demonstrate short stature, webbed neck, low-set ears, low hairline, and primary infertility. Cardiac defects occur in about 45% of affected patients; the most common lesions include bicuspid aortic valve, aortic coarctation, and aortic aneurysm. Although pulmonary stenosis can occur in Turner syndrome, this man could not have Turner syndrome.
A 24-year-old woman is evaluated for progressive muscle weakness of several months' duration. She provides no pertinent personal or family history and takes no medications. On physical examination, temperature is normal, blood pressure is 94/58 mm Hg, pulse rate is 98/min, and respiration rate is 16/min. BMI is 19. The remainder of the examination is normal. Laboratory studies: Serum electrolytes : Sodium 142 mEq/L (142 mmol/L) Potassium 2.8 mEq/L (2.8 mmol/L) Chloride 120 mEq/L (120 mmol/L) Bicarbonate 15 mEq/L (15 mmol/L) Urine electrolytes: Sodium 18 mEq/L (18 mmol/L) Potassium 8.0 mEq/L (8.0 mmol/L) Chloride 32 mEq/L (32 mmol/L) Urinalysis pH 5.0; no blood or protein Which of the following is the most likely cause of this patient's metabolic acidosis? Laxative abuse Surreptitious vomiting Type 1 (hypokalemic distal) renal tubular acidosis Type 4 (hyperkalemic distal) renal tubular acidosis
Normal anion gap metabolic acidosis can be caused by gastrointestinal bicarbonate loss, renal loss of bicarbonate, or the inability of the kidney to excrete acid. Laxative abuse is the most likely cause of this patient's normal anion gap metabolic acidosis. Normal anion gap metabolic acidosis can be caused by gastrointestinal bicarbonate loss, renal loss of bicarbonate, or the inability of the kidney to excrete acid. The normal physiologic response to systemic acidosis is an increase in urine acid excretion. Therefore, an initial diagnostic step in normal anion gap metabolic acidosis is to determine whether the kidney is appropriately excreting acid or whether impaired kidney acid excretion is the cause of the metabolic acidosis. Increased acid excretion by the kidney is reflected as a marked increase in urine ammonium. However, urine ammonium is difficult to measure directly. Because ammonium carries a positive charge, chloride is excreted into the urine in equal amounts with ammonium to maintain electrical neutrality. Therefore, the amount of chloride in the urine reflects the amount of ammonium present, and the urine anion gap can be used as an indicator of the ability of the kidney to excrete acid. The urine anion gap is calculated as follows: Urine Anion Gap = (Urine Sodium + Urine Potassium) - Urine Chloride In the context of increased urinary ammonium excretion, therefore, the urine anion gap will be negative. The negative urine anion gap in this patient (-6 mEq/L [-6 mmol/L]) suggests a gastrointestinal cause of the normal anion gap metabolic acidosis, and laxative abuse is a possible, even likely explanation. In addition, the low urine potassium indicates appropriate renal compensation in context of laxative-induced hypokalemia. Vomiting and gastric acid loss result in metabolic alkalosis, not metabolic acidosis. Renal causes of normal anion gap metabolic acidosis are due to specific defects in renal handling of bicarbonate reclamation (type 2/proximal RTA) or in hydrogen ion secretion (type 1/hypokalemic distal RTA). Type 1 (hypokalemic distal) RTA is caused by a defect in hydrogen secretion and a consequent decrease in ammonium excretion, and is therefore associated with a positive urine anion gap; it is also characterized by high urine potassium secretion and hypokalemia. Type 4 (hyperkalemic distal) RTA is usually caused by aldosterone deficiency or resistance and is characterized by a high serum potassium and positive urine anion gap.
A 25-year-old woman is evaluated in the emergency department for chest pain after a belted motor vehicle accident. She is pregnant at approximately 23 weeks' gestation. She reports no additional symptoms and is otherwise well. Her only medication is a prenatal vitamin. On physical examination, the patient is afebrile, blood pressure is 102/62 mm Hg, and pulse rate is 80/min. Pain and bruising over the left chest wall are noted. Abdominal examination findings are consistent with changes of pregnancy. Laboratory studies are significant for a serum sodium level of 132 mEq/L (132 mmol/L). Which of the following is the most likely cause of this patient's low serum sodium level? Excessive water intake Hypotension-induced antidiuretic hormone release Normal physiologic change in pregnancy Syndrome of inappropriate antidiuretic hormone secretion
Normal physiologic change in pregnancy is the most likely cause of this patient's low serum sodium level. Mild hyponatremia is common in normal pregnancy due to plasma volume increases with water retention (mediated by an increase in antidiuretic hormone levels) greater than sodium retention. An associated drop in serum osmolality of 8 to 10 mOsm/kg H2O and serum sodium concentration of 4 to 5 mEq/L (4-5 mmol/L) may occur. As the serum osmolality and sodium concentration decrease, a new set point is maintained, and thirst occurs in response to osmolality (reset osmostat). No treatment is necessary. Other conditions associated with reset osmostat include quadriplegia, tuberculosis, advanced age, psychiatric disorders, and chronic malnutrition. Primary polydipsia should always be considered in the differential diagnosis of patients with mental illness and hyponatremia, particularly those with schizophrenia who are taking psychotropic drugs. Primary polydipsia presents with hyponatremia, decreased serum osmolality, and decreased urine osmolality, reflecting suppressed antidiuretic hormone (ADH) levels in response to water overload. Primary polydipsia is a rare cause of hyponatremia, and the volume of water intake would need to be very large to induce hyponatremia. This patient is not at risk for primary polydipsia. Hypovolemia causes stimulation of the sympathetic nervous system, activation of the renin-angiotensin-aldosterone axis, and release of ADH. These adaptive responses allow volume maintenance at the expense of a low serum sodium with excessive water intake. Blood pressure in pregnant women begins to lower in the first trimester and reaches a nadir in the second. Furthermore, she is asymptomatic, and ADH release is therefore not likely to be induced by this level of blood pressure. The syndrome of inappropriate antidiuretic hormone (SIADH) secretion may be associated with stress and pain; however, hyponatremia does not develop acutely. Although SIADH could have preceded the patient's car accident, she has no risk factors for SIADH (central nervous system disorders, pulmonary disorders, infection, drugs, postoperative status, tumors), and normal pregnancy is a more likely cause of her low serum sodium level.
A 56-year-old woman is evaluated in follow-up after polysomnography documented an apnea-hypopnea index of 6, a mean oxyhemoglobin saturation of 86.4%, and 38% of sleep time with an oxygen saturation of less than 90% of predicted. She has type 2 diabetes mellitus and hypertension. Current medications are metformin and lisinopril. On physical examination, vital signs are normal. Oxygen saturation is 91% breathing ambient air. BMI is 44. Other than +1 ankle edema, the remainder of the physical examination, including neurological examination, is unremarkable. Laboratory studies reveal a hemoglobin level of 16.9 g/dL (169 g/L). Arterial blood gas studies show a pH of 7.36, a PCO2 of 58 mm Hg (7.7 kPa), and a Po2 of 59 mm Hg (7.8 kPa). Chest radiograph demonstrates clear lung fields. Which of the following is the most likely diagnosis? Amyotrophic lateral sclerosis Central sleep apnea Obesity hypoventilation syndrome Severe obstructive sleep apnea
Obesity hypoventilation syndrome is characterized by daytime hypercapnia, defined as an arterial PCO2 greater than 45 mm Hg that is thought to be a consequence of diminished ventilatory drive and capacity related to extreme obesity. The most likely diagnosis is obesity hypoventilation syndrome. Obesity hypoventilation syndrome is characterized by daytime hypercapnia (arterial PCO2 greater than 45 mm Hg [5.9 kPa]) that is thought to be a consequence of diminished ventilatory drive and capacity related to extreme obesity. Persons with a BMI of 35 or higher are considered at risk for obesity hypoventilation syndrome; it's estimated that more than half of patients with a BMI of 50 or higher have this condition. This patient's BMI of 44, compensated hypercapnic respiratory failure, hypoxemia during wakefulness but more pronounced during sleep, and polycythemia are all consistent with obesity hypoventilation syndrome. Positive airway pressure therapy (continuous positive airway pressure or bilevel positive airway pressure), sometimes with supplemental oxygen, is first-line therapy. Neuromuscular diseases that can affect the respiratory system must be considered in the differential diagnosis of hypoventilation syndromes. Amyotrophic lateral sclerosis (ALS) often leads to hypercapnic respiratory failure. However, this patient has an unremarkable neurologic examination and none of the typical features of ALS, such as muscle weakness and fasciculations and hyperactive deep tendon reflexes. Central sleep apnea (CSA) is defined by intermittent reduced central drive to breathe but is not a hypoventilation syndrome. In fact, the tendency to hyperventilate, as seen with the cyclic ventilatory pattern of Cheyne-Stokes breathing, is a key underlying mechanism of CSA. Patients with CSA are generally normocapnic or slightly hypocapnic on blood gas testing. An apnea-hypopnea index (AHI) of 5 to 15 is indicative of mild obstructive sleep apnea (OSA). This patient has mild OSA based upon an AHI of 6. OSA is typically encountered on sleep testing in those with obesity hypoventilation syndrome, with upper airway collapse superimposed on obesity-related hypoventilation. Severe OSA is defined as an AHI of at least 30; OSA severity is not defined by degree or duration of hypoxemia.
A 37-year-old man is evaluated during a follow-up appointment after resection of a well-differentiated adenocarcinoma of the ascending colon, which was diagnosed 1 month earlier. His family history is significant for endometrial cancer diagnosed in his mother at age 50 years and colon cancer diagnosed in his maternal grandfather at age 49 years. The patient reports that he has been feeling well and takes no medication. All physical examination findings, including vital signs, are normal. Which of the following is the most appropriate next step in management? Colonoscopy in 3 years Fecal immunochemical testing in 1 year Genetic counseling Upper endoscopy and capsule endoscopy
Patients with a family history suggesting Lynch syndrome (three family members are affected with a Lynch syndrome-associated cancer, at least two successive generations are affected, one of the affected family members is a first-degree relative of the other two affected family members, and at least one cancer was diagnosed in a family member younger than age 50 years) should be referred for genetic counseling. This patient should be referred to a genetic counselor for genetic testing. The patient could have Lynch syndrome based on his personal history of colon cancer diagnosed before age 50 years and his family history of colon cancers. His family history meets the Amsterdam II criteria for Lynch syndrome because three individuals in the family are affected with a Lynch syndrome-associated cancer, at least two successive generations are affected, one of the affected family members is a first-degree relative of the other two affected family members, and at least one cancer was diagnosed in a family member younger than age 50 years (the "3-2-1-1" pattern). Additional criteria are that familial adenomatous polyposis has been excluded and tumors have been verified histologically. Guidelines recommend tumor testing (microsatellite instability testing or immunohistochemistry) to screen for Lynch syndrome in all colon cancers. The results of tumor testing can help determine if Lynch syndrome is likely and inform genetic testing. Lynch syndrome is only diagnosed in individuals who meet the Amsterdam criteria and have an identified constitutional mutation in a mismatch repair gene (MLH1, MSH2, MSH6, or PMS2) or the epithelial cell adhesion molecule (EPCAM) gene. Diagnosis of Lynch syndrome will inform colonoscopy surveillance recommendations for this patient. Identification of a Lynch syndrome mutation will also affect the recommendation for screening of other cancers (gastric, skin, urinary tract, and, in women, ovarian and endometrial cancers). In addition, if he has an identifiable mutation associated with Lynch syndrome, genetic testing should be offered to his first-degree relatives. A 3-year interval for colonoscopy cannot be recommended until the results of the genetic evaluation are available. If Lynch syndrome is confirmed, the screening interval is every 1 to 2 years, not every 3 years. Colonoscopy is preferred for screening high-risk patients such as those with a previous colorectal cancer; therefore, screening using fecal immunochemical testing, other fecal-based testing, or colonic imaging is not appropriate. In patients with Lynch syndrome, upper endoscopy is recommended to screen for upper gastrointestinal cancers and to sample for Helicobacter pylori. However, this cannot be recommended until results from genetic testing are available. Patients with Lynch syndrome have an increased risk for small-intestinal cancer. However, routine cancer screening with capsule endoscopy is controversial and not routinely recommended even in patients with confirmed Lynch syndrome.
A 42-year-old woman is evaluated during a follow-up visit. She was diagnosed 4 years ago with stage IIIA high-grade serous ovarian cancer, with a deleterious BRCA1 germline mutation found on genetic testing. She was treated with total abdominal hysterectomy, bilateral salpingo-oophorectomy, and chemotherapy with intravenous and intraperitoneal cisplatin and paclitaxel. Her cancer recurred 18 months after she completed chemotherapy, and she underwent two additional sequential chemotherapies, including carboplatin and paclitaxel, cisplatin and gemcitabine, and bevacizumab. The cancer is progressing on her current treatment. Her performance status is good (Eastern Cooperative Oncology Group = 1). The patient is still taking chemotherapy medications. On physical examination, vital signs are normal. Abdominal distention with lateral dullness is present. The remainder of the examination is normal. Laboratory studies reveal a serum CA-125 value of 145 U/mL (145 kU/L) (1 month ago, 82 U/mL [82 kU/L]). CT scans of the chest, abdomen, and pelvis show increased size and number of peritoneal masses and increasing ascites. There are no pleural effusions. In addition to stopping the current chemotherapy regimen, which of the following is the most appropriate treatment? High-dose chemotherapy with hematopoietic stem cell transplantation Oral poly (adenosine diphosphate-ribose) polymerase inhibitor Repeat intraperitoneal cisplatin chemotherapy Supportive care
Olaparib is FDA approved as monotherapy for patients with germline BRCA-mutated advanced ovarian cancer previously treated with three or more lines of chemotherapy. The most appropriate treatment is an oral poly (adenosine diphosphate [ADP]-ribose) polymerase (PARP) inhibitor. Olaparib is an oral PARP inhibitor that is FDA-approved monotherapy for patients with germline BRCA-mutated advanced ovarian cancer previously treated with three or more lines of chemotherapy. PARP inhibition leads to the formation of double-stranded DNA breaks that in BRCA-competent tumors are repaired by homologous recombination. Tumors with germline BRCA mutations are unable to repair these defects, leading to cell death (referred to as synthetic lethality). A study of 298 patients with germline BRCA1- or BRCA2-associated cancers showed responses in ovarian, breast, pancreatic, and prostate cancers, with a 31% response rate and median duration of response of 7 months in platinum-resistant ovarian cancers. The most common side effects of olaparib were fatigue, nausea, vomiting, and anemia. In addition to the 15% of ovarian cancers with germline BRCA1 and BRCA2 mutations, up to 50% of high-grade serous ovarian cancers may be deficient in homologous recombination repair due to somatically acquired BRCA1 and BRCA2 mutations, epigenetic inactivation of BRCA1, or other defects in the homologous recombination pathway. These ovarian cancers with a "BRCAness" phenotype are also responsive to PARP inhibitors. Olaparib has also been approved for maintenance therapy in patients with BRCA1 or BRCA2 mutations who are in a complete or partial remission after first-line platinum-based chemotherapy. High-dose chemotherapy with hematopoietic stem cell transplantation is not recommended for the treatment of ovarian cancer, neither as initial treatment nor after recurrence. Intraperitoneal chemotherapy is not effective in patients with recurrent ovarian cancer. Given the activity of olaparib in the setting of a germline BRCA1 mutation and this patient's good performance status, it is reasonable to recommend continuing cancer-directed therapy at this time. In patients who did not benefit from previous treatment and with poor performance status (Eastern Cooperative Oncology Group performance status, 3-4) or in those with bowel obstruction in the setting of refractory ovarian cancer, changing to supportive care would be appropriate.
A 73-year-old man is evaluated in the hospital for lightheadedness. He also reports nonbloody, watery diarrhea of 4 months' duration and an unintentional 4.5-kg (9.9-lb) weight loss over the same time period. He has hypertension and hyperlipidemia. His medications are olmesartan and atorvastatin. On physical examination, blood pressure is 100/50 mm Hg and pulse rate is 108/min; other vital signs are normal. Physical examination findings are unremarkable. Results of laboratory studies, including serum creatinine, total IgA, and tissue transglutaminase IgA, are within normal limits. The patient responds to fluid resuscitation with normalization of his pulse and blood pressure. Colonoscopy is grossly normal, and biopsy samples show no evidence of microscopic colitis. Upper endoscopy with duodenal biopsies shows villous flattening and increased intraepithelial lymphocytes. Which of the following is the most appropriate next step in management? Discontinue atorvastatin Discontinue olmesartan Start a gluten-free diet Start prednisone
Olmesartan causes medication-induced enteropathy that can mimic refractory celiac disease.
A 73-year-old man is evaluated in the hospital for lightheadedness. He also reports nonbloody, watery diarrhea of 4 months' duration and an unintentional 4.5-kg (9.9-lb) weight loss over the same time period. He has hypertension and hyperlipidemia. His medications are olmesartan and atorvastatin. On physical examination, blood pressure is 100/50 mm Hg and pulse rate is 108/min; other vital signs are normal. Physical examination findings are unremarkable. Results of laboratory studies, including serum creatinine, total IgA, and tissue transglutaminase IgA, are within normal limits. The patient responds to fluid resuscitation with normalization of his pulse and blood pressure. Colonoscopy is grossly normal, and biopsy samples show no evidence of microscopic colitis. Upper endoscopy with duodenal biopsies shows villous flattening and increased intraepithelial lymphocytes. Which of the following is the most appropriate next step in management? Discontinue atorvastatin Discontinue olmesartan Start a gluten-free diet Start prednisone
Olmesartan causes medication-induced enteropathy that can mimic refractory celiac disease. Discontinuing olmesartan, an angiotensin II receptor blocker, is the most appropriate next step in the management of this patient. The patient's presentation is most consistent with drug-associated enteropathy related to olmesartan. In 2013, the FDA issued a warning that olmesartan medoxomil can cause intestinal symptoms known as sprue-like enteropathy and approved labeling changes to include this concern. The enteropathy may develop months to years after starting olmesartan. Drug-associated enteropathy can mimic refractory celiac disease with findings of villous atrophy and increased intraepithelial lymphocytes in the first part of the duodenum. The clinical presentation can include severe diarrhea, weight loss, and dehydration requiring hospitalization. Most of the reported cases of drug-associated enteropathy are caused by olmesartan, although other angiotensin-receptor blocking drugs have been implicated. In addition to adequate fluid resuscitation, the offending medication should be discontinued. In most cases of drug-associated enteropathy, symptoms and pathological changes resolve when the drug is stopped. Other medications, such as NSAIDs, proton pump inhibitors, antibiotics, colchicine, metformin, and cholesterol-lowering drugs (such as statins), can cause diarrhea and should be considered when evaluating a patient with symptoms of chronic diarrhea (>4 weeks' duration). However, because of the severity of the patient's presentation, the diarrhea is unlikely to be caused by atorvastatin. Additionally, atorvastatin and other statins are not linked to villous atrophy. A gluten-free diet is the mainstay of treatment for celiac disease. Given the patient's normal celiac serology testing, his symptoms are unlikely to be caused by celiac disease and a gluten-free diet is not indicated. Villous atrophy and increased intraepithelial lymphocytes are not specific for celiac disease. Other causes of villous atrophy, such as refractory celiac disease, Crohn disease, and autoimmune enteropathy, are treated with glucocorticoids such as prednisone. This patient's presentation and his older age are not consistent with these causes of villous atrophy, so glucocorticoids are not indicated.
A 38-year-old woman presents for first assessment of bone health 4 months into prednisone therapy. Medical history is significant for treatment of antiphospholipid syndrome with recurrent diffuse alveolar hemorrhage and secondary diabetes mellitus. She is not sexually active. She has had no fractures. Medications are prednisone, cyclophosphamide, neutral protamine Hagedorn (NPH) insulin, calcium citrate/vitamin D3, sulfamethoxazole-trimethoprim, and omeprazole. She is expected to continue prednisone therapy, 7.5 mg or more, for at least the next 6 months. On physical examination, vital signs are normal. BMI is 37. The remainder of the examination is normal. Bone mineral density by dual-energy x-ray absorptiometry shows a lumbar spine Z-score of -2.1 and total hip Z-score of -3.1. Radiograph of the spine shows no vertebral compression. Which of the following is the most appropriate treatment? Alendronate Teriparatide Zoledronic acid No additional therapy
Oral bisphosphonates are recommended as first-line therapy in adult men and women on chronic glucocorticoid therapy with moderate to high fracture risk regardless of age. The most appropriate treatment is alendronate. The American College of Rheumatology recommends that in all adults and children, an initial clinical fracture risk assessment for glucocorticoid-induced osteoporosis should be performed as soon as possible, but at least within 6 months of the initiation of long-term glucocorticoid treatment. Patients are categorized according to fracture risk. High fracture risk in patients younger than 40 years is defined as by a previous osteoporotic fracture. Moderate fracture risk is defined as hip or spine bone mineral density Z score less than -3 or rapid bone loss (>10% at the hip or spine over 1 year) and continuing glucocorticoid treatment at >7.5 mg/day for >6 months. Low risk is defined as no osteoporotic risk factors other than glucocorticoid use. Other criteria are used for defining low, moderate, and high fracture risk in patients age 40 years and older. Oral bisphosphonates are recommended as first-line therapy for patients with moderate to high fracture risk, such as this woman, regardless of age. This includes women of childbearing potential provided they are not planning a pregnancy during the period of bisphosphonate treatment. Optimized calcium and vitamin D intake, lifestyle modifications, and reassessment of fracture risk including bone mineral density testing every 2 to 3 years is recommended over osteoporosis medications for patients younger than 40 at low risk of fracture. However, this patient is not low risk, and treatment with an oral bisphosphonate is indicated. Teriparatide is indicated for the treatment of men and women with osteoporosis associated with sustained systemic glucocorticoid therapy at high risk for fracture. Although it increases bone mineral density at the spine and hip more than oral bisphosphonate therapy, it is less desirable due to expense and the requirement of daily injections. Zoledronic acid is indicated for the treatment and prevention of glucocorticoid-induced osteoporosis in patients who cannot tolerate oral bisphosphonates. Due to uncertain impact on pregnancy outcomes, it is considered a third-line agent in younger women.
A 19-year-old woman is evaluated for irregular menstrual cycles since menarche at 12 years of age, increasing amount of coarse facial hair, and acne. Symptoms have worsened since she stopped playing high school sports and subsequently gained weight. She is most concerned about the hair growth and acne. Medical history is otherwise unremarkable, and she takes no medications. On physical examination, vital signs are normal. BMI is 31. She has coarse terminal hair on the upper lip and chin, acne on the face and back, and non-discolored striae on the abdomen. There is no galactorrhea and no other evidence of virilization such as deepening of the voice, clitoromegaly, or male pattern balding. Laboratory studies show a total testosterone level of 73 ng/dL (2.5 nmol/L), dehydroepiandrosterone sulfate level of 1.8 µg/mL (4.9 µmol/L), and hemoglobin A1c of 5.4%. Other laboratory results are normal. Serum pregnancy test is negative. In addition to exercise and weight loss, which of the following is the most appropriate next step in management? Combined oral contraceptive therapy Metformin Pelvic ultrasound Spironolactone
Oral contraceptive agents are first-line pharmacologic therapy for hirsutism, acne, and menstrual dysfunction unless fertility is desired in a patient with polycystic ovary syndrome. This patient has ovulatory dysfunction with clinical evidence of hyperandrogenism. While this is suggestive of polycystic ovary syndrome, this is a diagnosis of exclusion. The prolonged clinical course and absence of the more concerning findings of virilization also support the diagnosis of polycystic ovary syndrome. Given that this patient is most concerned about hirsutism and acne, oral contraceptive therapy is the first-line therapeutic agent. Oral contraceptive therapy suppresses gonadotropin secretion and resultant ovarian androgen production. Additionally, the estrogen component increases sex hormone-binding globulin resulting in less androgen bioavailability. Oral contraceptives that contain 30 to 35 µg of ethinyl estradiol appear to be more effective in managing hirsutism than formulations containing less ethinyl estradiol. Furthermore, oral contraceptive therapy reduces new terminal hair growth, improves acne, and regulates menses to prevent endometrial hyperplasia. Metformin minimally effects hirsutism and is not recommended for this indication. In patients with polycystic ovary syndrome, metformin could be considered for off-label treatment of prediabetes or treatment of type 2 diabetes, in addition to lifestyle modification. Pelvic ultrasound and adrenal CT should be performed to exclude an ovarian or adrenal neoplasm if the serum total testosterone level is greater than 150 ng/dL (5.2 nmol/L), and adrenal CT is necessary to exclude an adrenal cortisol-secreting and/or androgen-secreting neoplasm if the plasma dehydroepiandrosterone sulfate (DHEAS) level is greater than 7.0 μg/mL (18.9 µmol/L). Pelvic ultrasound and adrenal CT are not indicated in this patient as her testosterone and DHEAS levels are not elevated to the degree that ovarian tumor is a consideration. While spironolactone can reduce the growth of terminal hair, it is used as an add-on treatment to oral contraceptive therapy. This antiandrogen medication may disrupt organogenesis in a male fetus; thus, concomitant reliable contraception is mandated when initiating this treatment.
A 48-year-old woman is evaluated for an 8-hour history of facial paralysis. She reports having "droopiness" on the right side of the face when she awoke this morning. She further says that her lunch tasted "odd" and that sounds on her right seem louder than usual. On physical examination, vital signs are normal. A right facial droop associated with difficulty closing the eye and elevating the eyebrow is noted on the right side. Hearing is intact to whispering, but noises are more pronounced on the right side. The palate is symmetrically elevated, and the tongue protrudes at the midline. Taste is impaired on the right side of the tongue and intact on the left side. Muscle strength is intact on the left side of the face and in both extremities. Shoulder shrug and head turning are normal bilaterally. Facial sensation is intact bilaterally, as are all deep tendon reflexes. The neck is supple, no skin rash is visible, and coordination and gait are normal. Which of the following is the most appropriate initial step in management? Brain MRI Intracranial magnetic resonance angiography Oral prednisone Oral valacyclovir
Oral prednisone administered within 72 hours of symptom onset expedites the speed and rate of full recovery in patients with Bell palsy. Oral prednisone should be given to this patient with Bell palsy within 72 hours of symptom onset to expedite the speed and rate of full recovery. Clinical history and examination findings in this patient are classic for Bell palsy, and thus no initial imaging or laboratory testing is indicated. Unilateral facial weakness involving both upper and lower parts of the face is characteristic of peripheral facial nerve (cranial nerve VII) involvement and distinguishes this condition from a central nervous system process (such as stroke), which typically spares the upper facial muscles because of bilateral innervation. Associated features of classic Bell palsy include alteration in taste (due to involvement of the chorda tympani) and hyperacusis (intolerance of loud noise due to involvement of stapedius muscle); other common features are ipsilateral sensory paresthesia and pain without objective sensory loss. Because this patient does not have subjective or objective hearing loss or any other involvement beyond a unilateral facial nerve, initial brain imaging is not indicated. Neither brain MRI nor intracranial magnetic resonance angiography is indicated in this patient. Sudden painless paralysis of a unilateral facial nerve is unlikely to be caused by aneurysmal compression or bleeding. These imaging tests might be appropriate if the patient had a thunderclap headache (maximum onset of pain within minutes of symptom onset), a slow progressive course of facial weakness, or progressive myoclonic hemifacial spasms. The effectiveness of oral antiviral therapy, such as valacyclovir, in the treatment of Bell palsy is controversial. Currently, no available evidence suggests that adding antiviral therapy improves prognosis.
A 61-year-old woman undergoes routine evaluation. She reports no recent changes in her medications. She has a history of idiopathic deep venous thrombosis. Her only medication is warfarin, initiated 2 months ago. On physical examination, vital signs are normal, and the examination is unremarkable. She has an INR of 7.2. In addition to withholding warfarin, which of the following is the most appropriate management? Administer fresh frozen plasma Administer prothrombin complex concentrate Administer vitamin K Remeasure INR in 2 days
Patients with asymptomatic INR elevation between 4.5 and 10 are managed by simply withholding warfarin. The patient's warfarin therapy should be withheld, and her INR should be remeasured in 2 days. Warfarin acts by inhibiting the synthesis of vitamin K-dependent clotting factors, which include factors II, VII, IX, and X. Variations in INR can occur with dietary changes. Other factors associated with prolonged elevation of INR include age 80 years or older, lower maintenance dose of warfarin, decompensated heart failure, active cancer, and use of medications known to potentiate the effect of warfarin. The 30-day risk of major bleeding is less than 1% with an INR between 5 and 9. Accordingly, warfarin should be withheld in patients with elevated INRs between 4.5 and 10 who are not bleeding and have no major risk factors for bleeding. Approximately one third of patients with an INR greater than 6 will still have an abnormal INR after withholding warfarin for two consecutive doses. Warfarin can be reinstituted when the INR returns to a therapeutic level. The patient is asymptomatic, with no evidence of bleeding. Therefore, neither vitamin K nor fresh frozen plasma should be given. Patients who are not bleeding, are not at high risk for bleeding, and have an INR less than 10 can be managed simply by withholding warfarin. For patients with an INR greater than 10, oral vitamin K, 2.5 mg, should be given. Oral vitamin K and intravenous vitamin K appear equally effective and more effective than subcutaneous vitamin K or placebo for reversing excessive warfarin-induced anticoagulation. The American Society of Hematology recommends against administering plasma or prothrombin complex concentrates for nonemergent reversal of vitamin K antagonists (including situations other than major bleeding, intracranial hemorrhage, or anticipated emergent surgery). However, the 30-day mortality is approximately 13% in patients with major bleeding during treatment with warfarin. In these patients with acute major bleeding as a result of warfarin therapy, in addition to vitamin K, prothrombin complex concentrates are preferred to fresh frozen plasma. Prothrombin complex concentrate is more effective than fresh frozen plasma for reducing 30-day all-cause mortality and the INR in patients with major bleeding or requiring surgical or invasive procedures.
A 65-year-old woman is admitted to the hospital for fatigue and weakness over the last 1 to 2 weeks. Medical history is significant for hypertension, type 2 diabetes mellitus, and rheumatoid arthritis. For the past 3 months, the patient's rheumatoid arthritis has been treated with methotrexate and prednisone. Because of inadequate control, etanercept was added 2 weeks ago. At that time, the patient decided to discontinue prednisone due to increased bruising of her skin. Current medications are methotrexate, etanercept, amlodipine, folic acid, metformin, and aspirin. On physical examination, blood pressure is 110/68 mm Hg sitting and 90/64 mm Hg standing, and pulse rate is 102/min sitting and 110/min standing. Symmetrical synovial bogginess is noted in the metacarpophalangeal joints and wrists bilaterally. Laboratory studies show an 8 AM cortisol level of 2 µg/dL (55.2 nmol/L). Which of the following is the most appropriate management? Initiation of hydrocortisone Initiation of hydrocortisone and fludrocortisone Performance of an adrenocorticotropic hormone (ACTH) stimulation test Performance of an ACTH stimulation test after administration of dexamethasone
Oral, injectable (including joint injections), and even topical glucocorticoids are able to suppress adrenocorticotropic hormone (ACTH) secretion and result in secondary adrenal insufficiency.
A 65-year-old woman is admitted to the hospital for fatigue and weakness over the last 1 to 2 weeks. Medical history is significant for hypertension, type 2 diabetes mellitus, and rheumatoid arthritis. For the past 3 months, the patient's rheumatoid arthritis has been treated with methotrexate and prednisone. Because of inadequate control, etanercept was added 2 weeks ago. At that time, the patient decided to discontinue prednisone due to increased bruising of her skin. Current medications are methotrexate, etanercept, amlodipine, folic acid, metformin, and aspirin. On physical examination, blood pressure is 110/68 mm Hg sitting and 90/64 mm Hg standing, and pulse rate is 102/min sitting and 110/min standing. Symmetrical synovial bogginess is noted in the metacarpophalangeal joints and wrists bilaterally. Laboratory studies show an 8 AM cortisol level of 2 µg/dL (55.2 nmol/L). Which of the following is the most appropriate management? Initiation of hydrocortisone Initiation of hydrocortisone and fludrocortisone Performance of an adrenocorticotropic hormone (ACTH) stimulation test Performance of an ACTH stimulation test after administration of dexamethasone
Oral, injectable (including joint injections), and even topical glucocorticoids are able to suppress adrenocorticotropic hormone (ACTH) secretion and result in secondary adrenal insufficiency. This patient has secondary adrenal insufficiency, and hydrocortisone is the most appropriate treatment. Oral, injectable (including joint injections), and occasionally even topical glucocorticoids are able to suppress adrenocorticotropic hormone (ACTH) secretion. Glucocorticoids prescribed at doses above physiologic replacement for longer than 3 weeks should be tapered when discontinued allowing recovery of the pituitary-adrenal axis; if therapy has lasted less than 3 weeks, no taper is required for pituitary-adrenal axis recovery. The diagnosis of adrenal insufficiency is based on demonstrating inappropriately low serum cortisol levels. Because most assays measure total cortisol, abnormalities in cortisol-binding protein or albumin can trigger spurious results. An early morning (8 AM) serum cortisol of less than 3 μg/dL (82.8 nmol/L) is consistent with cortisol deficiency, whereas values greater than 15 to 18 μg/dL (414.0-496.8 nmol/L) exclude the diagnosis when binding protein abnormalities and synthetic glucocorticoid exposure are excluded. Fludrocortisone in addition to hydrocortisone is unwarranted as fludrocortisone is needed only in primary adrenal insufficiency. There is no mineralocorticoid deficiency in secondary adrenal insufficiency. An ACTH stimulation test is not necessary in this patient since the cortisol level less than 3 µg/dL (82.8 nmol/L) is diagnostic of adrenal insufficiency. If an ACTH stimulation test were necessary, dexamethasone can be given prior to the ACTH stimulation test since dexamethasone is not measureable in the cortisol assay. That is unnecessary given this patient's cortisol level.
A 57-year-old woman is evaluated for progressive right upper leg pain for the past 2 years. The pain is worse with weight bearing. She is postmenopausal and otherwise in good health. She has no family history of fractures. She takes no medications. On physical examination, vital signs are normal. BMI is 36. She has difficulty bearing weight and limps when walking. She has discomfort on palpation over the anterior tibia. The remainder of the physical examination is normal. Laboratory studies: Alkaline phosphatase 150 U/L Calcium 8.2 mg/dL (2.0 mmol/L) Creatinine 0.8 mg/dL (70.7 µmol/L) Phosphorus 2.4 mg/dL (0.8 mmol/L) Parathyroid hormone 176 pg/mL (176 ng/L) 25-Hydroxyvitamin D <6 ng/mL (15.0 nmol/L) Which of the following is the most likely diagnosis? Osteitis fibrosa cystica Osteogenesis imperfecta Osteomalacia Postmenopausal osteoporosis
Osteomalacia related to malabsorption or dietary factors is characterized by low 25-hydroxyvitamin D, calcium, and phosphate levels and elevated parathyroid hormone and alkaline phosphatase levels. The most likely diagnosis in this patient is osteomalacia causing bone pain or fracture of the pelvis or proximal right lower extremity. An insidious, protracted course involving enigmatic pain is typical of osteomalacia, which may be dismissed by patients or symptomatically treated by health care providers. Chronically low levels of vitamin D can lead to rickets in children and osteomalacia in adults. Vitamin D deficiency is caused by factors such as intestinal malabsorption due to gastrointestinal disorders or restricted access to sunlight. In promoting absorption from the gut, vitamin D enables proper bone mineralization by maintenance of calcium and phosphorus levels. Vitamin D also modulates the actions of osteoblasts and osteoclasts to ensure proper bone growth and remodeling. The more common forms of osteomalacia related to malabsorption or dietary factors are characterized by low 25-hydroxyvitamin D (calcidiol), low calcium and phosphate, and elevated parathyroid hormone (PTH) (secondary hyperparathyroidism) and alkaline phosphatase levels. Depending on the duration and severity of vitamin D deficiency, the serum concentration of 1,25-dihydroxyvitamin D may be normal, low, or high and is not helpful in the diagnosis of most forms of osteomalacia. Osteitis fibrosa cystica is due to abnormally high bone turnover that can occur after prolonged exposure of bone to sustained high levels of PTH in hyperparathyroidism. It is associated with very high bone turnover, expansion of osteoid surfaces, and exuberant bone resorption resulting in an increased risk of fracture. Patients can be asymptomatic, or they may have bone pain. Classic skeletal changes on radiograph may include subperiosteal resorption of bone, most prominently at the phalanges of the hands. Osteitis fibrosa cystica is most commonly seen in patients with chronic kidney failure and is rarely associated with severe primary hyperparathyroidism. Osteogenesis imperfecta (OI) comprises four genetic syndromes characterized by autosomal dominant or recessive mutations in COL genes, leading to abnormalities in the structure of type I collagen. OI is associated with bone fractures, short stature, body deformity, hearing loss, and dental deformity. A classic feature of OI is blue sclerae (reflecting visibility of the underlying choroid), but it is not sensitive or specific to OI. The biochemical profile is usually normal in OI. Postmenopausal osteoporosis is a diagnosis of exclusion, made only after having evaluated and eliminated other causes of low bone mineral density such as osteomalacia. Patients with osteoporosis have a normal biochemical profile.
A 67-year-old woman is hospitalized for an ulcer on the bottom of her right foot; it has expanded over the past 2 months. The patient has type 2 diabetes mellitus, hypertension, stage 3 chronic kidney disease, and peripheral neuropathy. Medications are insulin glargine, insulin lispro, lisinopril, and gabapentin. On physical examination, temperature is 37.3 °C (99.1 °F); the remaining vital signs are normal. A deep 3- × 4-cm ulcer is located on the distal medial compartment of the plantar surface of the right foot. The base of the ulcer is necrotic and malodorous; a probe-to-bone test is negative. No surrounding erythema or increased warmth is noted. Both feet are warm with palpable pulses. Erythrocyte sedimentation rate and C-reactive protein level are elevated. Results of a complete blood count are normal. A plain radiograph reveals soft tissue swelling and ulceration; an MRI reveals findings consistent with osteomyelitis of the distal head of the first metatarsal. Which of the following is the most appropriate management? Bone biopsy and culture Forefoot amputation Swabbing and culture of wound base Vancomycin and piperacillin-tazobactam
Osteomyelitis in a patient with a diabetic foot infection and no evidence of skin or soft tissue infection or sepsis requires a bone biopsy before antibiotics are administered. A bone biopsy and culture is the next step in the management of osteomyelitis for this patient. Biopsies can be accomplished by open surgical procedure or percutaneously. Confirming a microbiologic diagnosis is needed before antibiotics can be administered. Indications for amputation include persistent sepsis, inability to tolerate antibiotic therapy, progressive bone destruction despite therapy, and bone destruction that compromises the mechanical integrity of the foot. None of these indications are present in this patient. Surgical debridement of the ulcer may be needed to remove the necrotic tissue, but this can be done at the time of bone biopsy. With the exception of Staphylococcus aureus, microorganisms isolated from culture samples obtained from superficial wounds or sinus tracts correlate poorly with deep cultures from bone; therefore, this practice is of limited value. Bone biopsy with histopathologic assessment and full microbiologic studies is important for diagnosing osteomyelitis, excluding other entities (such as neoplasm), and isolating the causative pathogen(s). Because the patient has no signs of skin or soft tissue infection or of sepsis, antibiotics are not immediately needed; furthermore, the provision of empiric antibiotics would also decrease the yield of a subsequent bone biopsy. Vancomycin and piperacillin-tazobactam might be indicated in the future, pending the results of the bone biopsy. However, a histologic and microbiologic diagnosis confirmation is needed before antibiotics can be administered.
A 32-year-old woman is evaluated during a follow-up visit for management of stage IIIA left breast cancer (3.5-cm, grade 3 invasive ductal carcinoma; estrogen receptor positive, progesterone receptor positive, and HER2 negative; four positive axillary lymph nodes). She has completed neoadjuvant chemotherapy, lumpectomy, and axillary dissection as well as primary breast radiation. She resumed menstruation after completing chemotherapy, and her estradiol levels are in the premenopausal range. She takes no medications. On physical examination, vital signs are normal. There are healed incisions on the left breast and axilla. There are no breast masses. Which of the following is the most appropriate treatment? An aromatase inhibitor Leuprolide Leuprolide and an aromatase inhibitor Tamoxifen
Ovarian suppression along with antiestrogen therapy is recommended to decrease breast cancer recurrence in women with high-risk, early-stage, hormone receptor-positive breast cancer who remain premenopausal after chemotherapy.
A 32-year-old woman is evaluated during a follow-up visit for management of stage IIIA left breast cancer (3.5-cm, grade 3 invasive ductal carcinoma; estrogen receptor positive, progesterone receptor positive, and HER2 negative; four positive axillary lymph nodes). She has completed neoadjuvant chemotherapy, lumpectomy, and axillary dissection as well as primary breast radiation. She resumed menstruation after completing chemotherapy, and her estradiol levels are in the premenopausal range. She takes no medications. On physical examination, vital signs are normal. There are healed incisions on the left breast and axilla. There are no breast masses. Which of the following is the most appropriate treatment? An aromatase inhibitor Leuprolide Leuprolide and an aromatase inhibitor Tamoxifen
Ovarian suppression along with antiestrogen therapy is recommended to decrease breast cancer recurrence in women with high-risk, early-stage, hormone receptor-positive breast cancer who remain premenopausal after chemotherapy. The most appropriate treatment is leuprolide and an aromatase inhibitor. Two prospective randomized clinical trials have shown the superiority of ovarian suppression with leuprolide plus an aromatase inhibitor, such as exemestane, over ovarian suppression plus tamoxifen or tamoxifen alone in premenopausal women with higher-risk hormone receptor-positive early breast cancer. Analyzing the trials together, disease-free survival at 5 years was 91.1% in the ovarian-suppression-plus-exemestane group and 88.3% in the ovarian-suppression-plus-tamoxifen group. Overall survival was the same in both groups. Compared with tamoxifen alone, ovarian suppression with tamoxifen improved disease-free survival, which was further improved by ovarian suppression plus exemestane. The benefit was particularly dramatic in women younger than 35 years who had a 15.7% absolute improvement in 5-year disease-free survival with the use of ovarian suppression and exemestane compared to tamoxifen alone. Adverse effects were seen more frequently in the patients on ovarian suppression, and by 4 years, 21% of patients had stopped ovarian suppression due to side effects. In patients who do not tolerate initial treatment with ovarian suppression and exemestane, ovarian suppression with tamoxifen can be used, transitioning to an aromatase inhibitor after 2 to 3 years if tolerated. An aromatase inhibitor is ineffective in premenopausal women unless concomitant ovarian suppression is given. The benefit of ovarian suppression alone is similar to tamoxifen alone and inferior to the option of ovarian suppression and an aromatase inhibitor. In patients who require initial chemotherapy and remain premenopausal, treatment with tamoxifen alone has a higher risk of breast cancer recurrence than ovarian suppression plus an aromatase inhibitor.
A 52-year-old man is evaluated for a 1-year history of progressive weakness that began as right foot drop and bilateral tingling in the feet. Within the past 2 months, the patient has developed progressive weakness, which makes walking difficult; he also notes weakness in the hands and burning below the knees but no autonomic symptoms. He has hypothyroidism treated with levothyroxine. On physical examination, vital signs are normal. Motor strength is 4/5 in the intrinsic hand and quadriceps muscles and 3/5 in the tibialis anterior and gastrocnemius muscles; bulbar and facial muscle strength is normal. Deep tendon reflexes are absent in the lower extremities. Sensory perception of vibration is severely impaired at the knees. Pinprick testing shows reduced sensation below the ankles. Splenomegaly is present, as are patchy areas of hyperpigmentation and scattered angiomas on the trunk. Gait is broad based and wobbly, and a Romberg test has positive results. Serum immunofixation reveals a λ light chain monoclonal protein. Needle electromyography reveals a demyelinating sensorimotor polyneuropathy. Which of the following is the most likely diagnosis? Amyotrophic lateral sclerosis Chronic inflammatory demyelinating polyradiculoneuropathy Mitochondrial myopathy POEMS syndrome
POEMS syndrome is characterized by the presence of a monoclonal plasma cell disorder, peripheral neuropathy, and one or more of the following: osteosclerotic myeloma, Castleman disease (angiofollicular lymph node hyperplasia), elevated serum vascular endothelial growth factor, organomegaly, endocrinopathy, edema, typical skin changes, and papilledema. The most likely diagnosis is POEMS syndrome (polyneuropathy, organomegaly, endocrinopathy, monoclonal gammopathy, and skin changes). POEMS syndrome is characterized by the presence of a monoclonal plasma cell disorder, peripheral neuropathy, and one or more of the following: osteosclerotic myeloma, Castleman disease (angiofollicular lymph node hyperplasia), elevated serum vascular endothelial growth factor, organomegaly, endocrinopathy, edema, typical skin changes, and papilledema. His clinical presentation, including splenomegaly, skin lesions (hyperpigmentation and angiomas), endocrine disease (hypothyroidism), peripheral neuropathy, and λ monoclonal gammopathy support the diagnosis of POEMS syndrome. POEMS syndrome is typically secondary to an underlying cancer; identification and treatment of the underlying cancer leads to improvement of the neuropathy. Amyotrophic lateral sclerosis (ALS) is characterized by upper motor neuron signs (such as hyperreflexia, spasticity, and an extensor plantar response) coexistent with lower motor neuron findings (such as atrophy and fasciculation). Sensory deficits are characteristically absent, and sensory nerve conduction studies are often normal in ALS. These findings are not consistent with this patient's presentation. The classic presentation of chronic inflammatory demyelinating polyradiculoneuropathy (CIDP) is generalized areflexia and progressive or relapsing symmetric sensory and motor neuropathy. CIDP resembles POEMS syndrome in its neurologic manifestations, but the presence of monoclonal gammopathy, splenomegaly, and skin changes makes this diagnosis unlikely. Mitochondrial myopathy can present with significant variability and may cause fatigue, myalgia, ophthalmoplegia, and various extramuscular manifestations. Mitochondrial myopathies should be suspected in the presence of multiorgan involvement and maternal transmission. In this patient, the presence of sensory deficits and neuropathic findings on needle electromyography are not consistent with a myopathy. Read Related TextNext Question
A 62-year-old man is evaluated in the emergency department for a 3-hour history of transient right arm and right leg weakness. His symptoms resolve while in the emergency department. Medical history is otherwise unremarkable, and he takes no medications. On physical examination, vital signs are normal, and the remainder of the examination is unremarkable. Laboratory studies are notable for an erythrocyte sedimentation rate of 16 mm/h and LDL cholesterol level of 70 mg/dL (1.81 mmol/L). A 12-lead electrocardiogram is normal. Carotid duplex ultrasound shows no evidence of hemodynamically significant plaque. CT of the head without contrast demonstrates no evidence of hemorrhage. MRI of the brain reveals a small left internal capsular infarction. A transesophageal echocardiogram is obtained. Representative mid-esophageal four-chamber images at end-systole (top panel) and end-diastole (bottom panel) are shown. Which of the following is the most likely diagnosis? Bacterial endocarditis Left atrial myxoma Nonbacterial thrombotic endocarditis Papillary fibroelastoma
Papillary fibroelastomas are small, independently mobile cardiac tumors that are typically attached to the left-sided valvular endocardium by a stalk; they may be associated with stroke, transient ischemic attack, angina, myocardial infarction, and peripheral embolization. This patient's clinical history, physical examination findings, and echocardiographic features are most consistent with a papillary fibroelastoma (PFE). The echocardiographic images (shown) reveal a PFE (arrows) arising from the mitral valve. Most PFEs are small (averaging 12 mm × 9 mm), independently mobile, and attached to the endocardium by a stalk. They are more commonly associated with left-sided valves than right-sided valves or other endocardial surfaces. Patients may present with symptoms of embolism (including stroke or transient ischemic attack), angina, myocardial infarction, and peripheral embolization. PFE may also be discovered incidentally on echocardiogram. Surgery is indicated for symptomatic patients with PFE with acceptable operative risk. The treatment of asymptomatic patients with left heart PFE is controversial, although in one large case series, the risks for stroke and death were higher in patients who did not have surgery. Bacterial endocarditis is less likely in this patient given the absence of constitutional symptoms, evidence of systemic inflammation, a predisposing valvular lesion, and risk factors for bacteremia (such as dental cleaning or injectable drug use). Left atrial myxomas may cause cerebral embolic phenomena; however, most myxomas are substantially larger than fibroelastomas and are associated with obstructive symptoms and, in 10% to 15% of patients, an auscultatory "tumor plop." In 75% of cases, myxomas arise within the left atrium attached to the fossa ovalis, not valvular surfaces. Constitutional symptoms, such as fever, anorexia, and weight loss, related to tumor interleukin production may also be present. Nonbacterial thrombotic endocarditis (NBTE), also known as verrucous, marantic, or Libman-Sacks endocarditis, is likely caused by endothelial injury in the presence of a hypercoagulable state. NBTE occurs on the atrial surface of the mitral valve, the ventricular side of the aortic valve, or both locations. Vegetations are typically small (<1 cm), broad based, and irregularly shaped (wart-like) and are most commonly associated with advanced malignancy or connective tissue disorders. This patient does not have evidence of a predisposing condition for NBTE, and the lesion does not have a wart-like or broad-based appearance.
A 67-year-old woman is admitted to the ICU for abdominal distention, vomiting, and hypotension. She had a colectomy 3 weeks ago to treat recurrent diverticular bleeding, and postoperatively she had prolonged anorexia and nausea. She was discharged to an extended care facility on enteral nutrition through a small-bore nasogastric tube. On physical examination, temperature is 36.8 °C (98.2 °F), blood pressure is 100/60 mm Hg, pulse rate is 109/min, and respiration rate is 19/min. BMI is 19. Preoperative BMI was 25. She has temporal wasting, colectomy wound with areas of dehiscence, lower extremity edema, decreased bowel sounds, and distended tympanitic abdomen. Blood glucose is 65 mg/dL (3.6 mmol/L), albumin is 2.4 g/dL (24 g/L), creatinine is 0.6 mg/dL (53 µmol/L), and blood urea nitrogen is 6 mg/dL (2.14 mmol/L). Radiograph and CT scan of the abdomen show dilated loops of small bowel. The nasogastric tube is in the proximal jejunum. Intravenous fluid resuscitation is initiated and surgical consultation is obtained. Which of the following is the most appropriate nutritional management? Maintain current enteral nutrition Measure gastric residual volume Start metoclopramide Switch to parenteral nutrition
Parenteral nutrition should be started as soon as possible for severely malnourished patients or those at high risk of malnutrition for whom enteral nutrition is not possible. This patient should receive parenteral nutrition. Nutrition is an essential part of management for patients in the ICU and can be given enterally or parenterally, with the enteral route preferred. Initiation of enteral nutrition is recommended at 24 to 48 hours following admission if the patient is hemodynamically stable, with advancement to goal by 48 to 72 hours. Benefits include fewer infections and possibly reduced mortality. For patients with adequate nutritional status but who have contraindications to enteral nutrition or do not tolerate enteral nutrition, parenteral nutrition is delayed for 1 to 2 weeks based upon evidence that early parenteral nutrition may increase the risk of infection. For patients with inadequate nutrition (or who are at high risk for malnutrition) who have contraindications or intolerance of enteral nutrition, parenteral nutrition should be initiated as soon as possible. This practice is based on two meta-analyses showing early parenteral nutrition in poorly nourished patients is associated with fewer complications and decreased mortality. This patient has several features consistent with severe malnutrition (temporal wasting, edema, hypoglycemia, poorly healing wounds), as well as ileus and evidence of enteral nutrition intolerance (nausea, vomiting). The patient is obviously not tolerating enteral nutrition, as evidenced by her clinical nutritional status, nausea, vomiting, and distended abdomen. Continuing enteral nutrition would not be appropriate. Measurement of gastric residual volume is no longer recommended for routine monitoring of enteral nutrition because it does not affect outcomes. In this patient, it is clinically apparent that she is not tolerating enteral nutrition, and measurement of gastric residual volume will not add clinically useful information. Metoclopramide improves enteral nutrition tolerance but does not affect patient outcomes and is associated with adverse events (diarrhea, QT prolongation, tardive dyskinesia, cardiac toxicity). Because of this patient's recent surgery, she should first be started on parenteral nutrition and evaluated for a mechanical cause of bowel obstruction. Metoclopramide and other prokinetic agents are contraindicated in the presence of mechanical small bowel obstruction.
A 65-year-old woman is evaluated during a follow-up visit for giant cell arteritis. Prior to confirmation of the diagnosis she was started on 60 mg of prednisone daily and low-dose aspirin. The diagnosis was subsequently confirmed with a temporal artery biopsy. She is otherwise healthy and takes no additional medications. On physical examination, vital signs are normal. There is a healing incision over the right temporal artery. Dual-energy x-ray absorptiometry (DEXA) was performed 6 months ago. Based upon the results of the Fracture Risk Assessment Tool, the patient was classified as medium risk (10%-20%) for a major osteoporotic fracture in the next 10 years. Which of the following is the most appropriate next step in management? Begin alendronate Begin teriparatide Repeat DEXA No additional testing or therapy
Patients at moderate or high 10-year risk for a major osteoporotic fracture taking at least 2.5 mg of prednisone daily for 3 months or more should begin prophylactic bisphosphonate therapy. This patient at risk for glucocorticoid-induced osteoporosis should begin alendronate. One of the many risks and side effects of chronic glucocorticoid therapy is osteoporosis. The Fracture Risk Assessment Tool (FRAX) calculator defines the 10-year fracture risk for patients with T-scores in the −1.0 to −2.5 range. The FRAX calculator () incorporates multiple risk factors, including sex, fracture history, femoral neck bone mineral density, glucocorticoid use, smoking, BMI, age, and alcohol intake to determine projected fracture risk. The American College of Rheumatology recommends that patients over the age of 40 years at moderate or high risk for osteoporotic fractures who are to be on at least 2.5 mg of prednisone daily for 3 months or more should begin prophylactic bisphosphonate therapy with alendronate, risedronate, or zoledronic acid. In patients on chronic glucocorticoid therapy, prophylactic bisphosphonate therapy significantly increases bone mineral density compared with placebo, and these patients also have fewer new vertebral fractures. In patients at high risk for major osteoporotic fracture (10-year risk greater than 20% or a T score ≤−2.5 or a history of a fragility fracture) taking any dose of glucocorticoids for at least 1 month should receive prophylactic treatment (although these patients should be treated regardless of glucocorticoid therapy). Alendronate, risedronate, zoledronic acid, or teriparatide are therapeutic options. This patient is not in the high-risk category, and teriparatide is therefore not a recommended option. Dual-energy x-ray absorptiometry (DEXA) is useful at the outset of chronic glucocorticoid therapy to risk-stratify patients and to provide baseline values. Although there is no consensus on the optimal frequency of monitoring bone mineral density with DEXA, there is no role for repeating the study in less than 1 year. Because of her prolonged need for prednisone and risk for osteoporotic fracture, no additional therapy would be inappropriate management for this patient.
A 25-year-old woman is evaluated in the emergency department for mild shortness of breath and right lateral chest pain of 2 days' duration. She reports no orthopnea or lower extremity edema. Medical history is unremarkable other than anxiety related to meeting the demands of her new job as an engineer. She takes no medications. On physical examination, she is in no apparent distress. Vital signs are normal. Oxygen saturation is 99% breathing ambient air. The lungs are clear to auscultation. No peripheral edema is noted. Her Wells Criteria for Pulmonary Embolism score is 0. A complete blood count, electrocardiogram, and chest radiograph are normal. Which of the following tests should be performed next to rule out pulmonary embolism? CT angiography of the chest D-dimer Echocardiography No further testing
Patients identified as low risk and meeting the Pulmonary Embolism Rule-Out Criteria do not require D-dimer testing to eliminate the need for further diagnostic imaging. The patient requires no further testing for pulmonary embolism. Many patients present to the emergency department with shortness of breath or chest pain. Inappropriate use of D-dimer testing and CT angiography led to the development of new American College of Physicians guidelines regarding the evaluation of patients with possible pulmonary embolism (PE). These guidelines use clinical decision tools (such as the Pulmonary Embolism Rule-Out Criteria [PERC]) to help risk stratify patients who present with shortness of breath or chest pain. The PERC are defined as Age younger than 50 years Heart rate less than 100/min Oxygen saturation 95% or greater No hemoptysis No estrogen use No previous deep venous thrombosis or PE No unilateral leg swelling No surgery or trauma requiring hospitalization in the last 4 weeks To avoid unnecessary testing, patients with a low pretest probability using a validated prediction tool such as the Wells Criteria for Pulmonary Embolism and negative PERC should undergo no further evaluation. In patients with intermediate pretest probability or positive PERC, the D-dimer level should be obtained. Patients with a high pretest probability should proceed directly to imaging, without D-dimer testing. Because this patient has a low pretest probability and the PERC are negative, neither D-dimer testing nor CT angiography of the chest should be performed. A study of multiorgan ultrasonography was shown to be helpful in the diagnosis of PE in adults with a Wells score of 4 or greater or a positive D-dimer test. A positive test was defined by any one of the following: one or more pulmonary subpleural infarctions on lung ultrasonography; right ventricular dilatation or thrombi on heart ultrasonography; or deep venous thrombosis on lower extremity ultrasonography. Multiorgan ultrasonography was found to be 90% sensitive and 86% specific compared with CT angiography. Multiorgan ultrasonography was significantly more sensitive than individual ultrasonographic examinations of the lungs, heart, or lower extremities. Echocardiography is unlikely to be helpful in this patient because her pretest probability of pulmonary embolism was very low, and echocardiography alone is an insensitive test.
A 58-year-old man is assessed for discharge from the hospital. He was admitted 3 days ago with fever and chills. He has non-Hodgkin lymphoma and a tunneled subclavian venous catheter used for chemotherapy infusion. Blood cultures at admission grew vancomycin-resistant Enterococcus faecium. The patient's catheter was removed, a peripherally inserted central catheter (PICC) was placed for intravenous access, and daptomycin therapy was initiated. Blood cultures are now negative, and the patient is afebrile. The patient is ready to be discharged to complete intravenous daptomycin therapy as an outpatient. At the time of discharge, his complete blood count and comprehensive chemistry profile are normal. Which of the following is the most appropriate weekly monitoring of his daptomycin therapy? Electrocardiography and blood glucose Hemoglobin and platelet count Serum amylase and triglycerides Serum creatinine and creatine kinase
Patients receiving daptomycin therapy should undergo baseline measurement of kidney function and creatine kinase level followed by weekly monitoring. Patients receiving outpatient daptomycin therapy should undergo baseline measurement of kidney function and creatine kinase (CK) followed by weekly monitoring. Patients should also be screened for symptoms of myopathy. Daptomycin is commonly used for outpatient parenteral antibiotic therapy (OPAT) because of its safety profile, ease of administration (once daily), and good activity against gram-positive bacteria, including vancomycin-resistant enterococci and methicillin-resistant Staphylococcus aureus. However, daptomycin is known to cause elevated levels of CK and can contribute to the development of myopathy during therapy. Daptomycin should be discontinued in asymptomatic patients if CK levels increase to greater than 10 times the upper limit of normal or the CK level is greater than 5 times the upper limit of normal with symptoms of myopathy. Concomitant treatment with statins (particularly simvastatin and atorvastatin) may increase the chance of developing an elevated CK level; it is suggested that statins be discontinued if possible during daptomycin treatment. If statins cannot be discontinued, or if kidney dysfunction is evident, the CK level should be monitored more frequently than once weekly. Likewise, the creatinine level should also be monitored because daptomycin dosing may require adjustment (lower dose or dosing interval of every other day), and CK may require more frequent monitoring if the creatinine level increases. Daptomycin use does not require electrocardiographic monitoring, and it has no effect on the bone marrow (for example, erythrocyte or platelet suppression), pancreas, lipid levels, or blood glucose level; so weekly amylase, triglyceride, glucose, and hemoglobin measurements and platelet count monitoring are unnecessary (although periodic leukocyte counts may be necessary in some patients for monitoring of the primary infection). It is important for patients undergoing OPAT to have close follow-up to monitor for any adverse effects from antibiotic therapy (including development of vascular access infections) as well as resolution of the infection being treated.
A 65-year-old woman is evaluated for discontinuation of mechanical ventilation. She was placed on mechanical ventilation 5 days ago for respiratory failure secondary to an exacerbation of COPD. Ventilator settings are in the volume-controlled continuous mandatory ventilation mode, with a set respiration rate of 10/min, a tidal volume of 370 mL, an FIO2 of 0.35, and a positive end-expiratory pressure of 5 cm H2O. Current medications are albuterol/ipratropium, levofloxacin, prednisone, and fentanyl. On physical examination, vital signs are normal. She is sleepy but arousable and can follow simple commands. Lung examination reveals distant breath sounds. The remainder of the examination is unremarkable. Arterial blood gas studies show a pH of 7.46, Pco2 of 47 mm Hg (6.25 kPa), and a PO2 of 62 mm Hg (8.25 kPa). Other laboratory studies, including a leukocyte count , are normal. Chest radiograph demonstrates hyperinflation but no infiltrates or evidence of heart failure. Which of the following is the most appropriate test or evaluation to perform next? 30-Minute spontaneous breathing trial Cuff leak test Glasgow Coma Scale Measure negative inspiratory force
Patients should be assessed daily for their readiness to be removed from mechanical ventilation by performing a spontaneous breathing trial; one criterion for success is the ability to tolerate a spontaneous breathing trial for 30 minutes. A 30-minute spontaneous breathing trial (SBT) should be performed using low levels of pressure support (8 cm H2O or less). Weaning from mechanical ventilation can start when the precipitating event or underlying condition that caused respiratory failure has resolved or is resolving. Patients should be assessed daily for their readiness to be removed from mechanical ventilation by performing an SBT. There are several methods used to assess if an SBT is successful. One criterion is the ability to tolerate a weaning trial for 30 minutes (in most patients, SBT failure will occur within approximately 20 minutes). 2-hour SBTs and 30-minute SBTs have a similar ability to recognize patients who are unable to breathe spontaneously. However a 30-minute trial has the benefits of less time on mechanical ventilation and less risk of respiratory muscle fatigue. If the patient successfully completes an SBT, the ability to follow commands, clear secretions, and a patent upper airway are other criteria that should be met to increase extubation success. A "cuff leak" refers to measurable airflow around the endotracheal tube after the cuff of the endotracheal tube is deflated. Absent or minimal cuff leak following deflation of the cuff indicates reduced space between the endotracheal tube and the larynx. Minimal or absent cuff leak may be due to laryngeal edema, laryngeal stenosis, and thick secretions. The test is not standardized and not performed routinely and is not an initial routine test in the process of liberating a patient from mechanical ventilation. It might be considered in a patient who has a successful SBT but is at high risk for edema and stridor following extubation. The Glasgow Coma Scale is pertinent to the actual extubation process, in which lack of awareness and ability to clear secretions and follow simple commands may increase the risk of aspiration and cooperation in the postextubation period. However, this patient should be placed on an SBT before being evaluated for extubation. Negative inspiratory force has been used as a marker of inspiratory muscle strength to identify patients who will be able to be liberated from mechanical ventilation. However, there are technical issues that lead to variable predictive performance. A low negative inspiratory force by itself is not useful; however, serial measurements (for example, in patients with Guillain-Barré or myasthenia gravis) along with other measures (FVC, maximum tidal volume) may give a better picture of muscle strength recovery.
A 66-year-old man is hospitalized in December for a 1-week history of increasing dyspnea on exertion, wheezing, and a nonproductive cough despite outpatient treatment with antibiotics and steroids. He now has awakenings with nocturnal dyspnea, which are only partially relieved by the use of his albuterol inhaler. He has started using his albuterol nebulizer four times a day, which provides only temporary relief. He does not have fever, headache, myalgia, runny nose, sputum production, chest pain, lower extremity edema, or palpitations. He has a history of COPD with an FEV1 of 42% of predicted on spirometry obtained 3 months ago. In addition to albuterol, he takes umeclidinium and vilanterol and was started on azithromycin and prednisone 3 days ago. On physical examination, temperature is 37.1 °C (98.8 °F), blood pressure is 135/80 mm Hg, pulse rate is 110/min, and respiration rate is 22/min. Oxygen saturation is 90% on ambient air. The patient is not using his accessory muscles. He has decreased breath sounds throughout, with diffuse end-expiratory wheezes. Other than tachycardia, the cardiovascular examination is normal without jugular venous distention or edema. Laboratory studies, including complete blood count with differential, basic metabolic panel, B-type natriuretic peptide, and arterial blood gases , are unremarkable. Electrocardiogram demonstrates sinus tachycardia, and a chest radiograph shows hyperinflation but no infiltrates. Which of the following is the most appropriate diagnostic test to perform next? Bedside spirometry CT pulmonary angiography Polymerase chain reaction testing for influenza A and B Sputum culture
Patients who are not responding to typical therapy for COPD exacerbations should be carefully evaluated for heart failure, pneumonia, and pulmonary embolism. This patient should undergo CT pulmonary angiography. Some COPD exacerbations thought to be of unknown cause may actually be due to other medical conditions, including a pulmonary embolism (PE). A meta-analysis suggests that the prevalence of PE in patients hospitalized for an acute COPD exacerbation is as high as 25%. Testing for PE is indicated for patients who are not responding to typical therapy for acute exacerbations unless the pretest probability for PE is unlikely. Other important entities in the differential diagnosis with high risk for mortality include heart failure and pneumonia. These entities are less likely in this patient because of the absence of fever, crackles, edema, normal chest radiograph, and normal B-type natriuretic peptide level. Spirometry usually does not change management during an acute exacerbation. In addition, the patient may not be able to complete the testing given his symptoms and the increased work of breathing. Evaluation for influenza may be useful in patients who present with compatible symptoms, including fever, headache, myalgia, pharyngeal irritation, and respiratory symptoms (nonproductive cough and nasal discharge), particularly during an influenza outbreak. During a confirmed local influenza outbreak, infection can be reliably diagnosed on the basis of clinical criteria alone. When confirmation is needed, polymerase chain reaction testing can be performed. Testing for influenza in this patient is not necessary in the absence of influenza symptoms. Similarly, a sputum culture is usually not indicated as it infrequently changes management of COPD. An antibiotic is often added because infections are the most common triggers for an acute exacerbation.
A 24-year-old woman is evaluated for severe pain in her arms and legs of 5 hours' duration. Medical history is notable for lifelong mild anemia and four episodes of similar pain without precipitating factors, each episode resolving in 3 to 4 days. She has a cousin with sickle cell disease. She takes no medications. On physical examination, vital signs are normal. She has normal range of motion in all joints with no skeletal deformities. Laboratory studies: Hemoglobin 11.2 g/dL (112 g/L) Mean corpuscular volume 76 fL Hemoglobin electrophoresis Hemoglobin A 33% Hemoglobin A2 4% Hemoglobin S 63% Hemoglobin F 0% Which of the following is the most likely diagnosis? Hemoglobin E disease Hemoglobin S β-thalassemia Hemoglobin SS disease Sickle cell trait
Patients who coinherit hemoglobin S and β-thalassemia genes typically have lifelong mild hemolytic anemia with microcytosis and detectable hemoglobin A on electrophoresis; the amount of hemoglobin A is inversely related to the severity of the symptoms and risk of complications. This patient most likely has hemoglobin S β-thalassemia (Sβ-thalassemia). Patients with Sβ-thalassemia generally have milder disease than those homozygous for hemoglobin S (HbS). However, patients with Sβ-thalassemia can have painful crises of varied frequency and intensity. Some of the variability in presentation depends on the relative expression of the β-globin gene (from β+ to β0), which in turn determines the relative amount of hemoglobin A (HbA). Most Black persons with Sβ-thalassemia are β+ and have between 5% and 30% HbA; the more HbA present, the less severe the symptoms and fewer the complications. Occasionally, with a milder disease course, patients do not present until adolescence or early adulthood. Hemoglobin electrophoresis is essential in making a specific diagnosis of a sickling disorder. Patients with Sβ-thalassemia have a slightly reduced hemoglobin level and microcytic erythrocytes. Characteristically, their HbS levels are about 60%. In contrast, patients with sickle cell trait have HbS levels well below 50%, typically closer to 30%, and proportionally more HbA. More notably, patients with sickle cell trait do not experience painful crises and have normal hemoglobin levels. Patients with hemoglobin SS disease (sickle cell anemia) have greater than 90% HbS and no HbA. The clinical expression and severity of sickle cell anemia may be quite variable, and some patients may have few acute pain events and no other major complications, so that the diagnosis may not be suspected until later in life. The level of hemoglobin F detected in these patients may also vary, with increased hemoglobin F levels correlating with milder disease, but Hb A should be undetectable and no microcytosis should be seen. Hemoglobin E disease results from a point mutation in the β-globin gene that causes decreased production of β-globin and results in a thalassemia-like syndrome. Hemoglobin E migrates in a different pattern than HbA and would be revealed on hemoglobin electrophoresis, in addition to increased levels of HbA2, as with other β-thalassemias. Although patients with hemoglobin E disease will have mild anemia, they do not experience acute pain events.
A 71-year-old woman is evaluated in the emergency department 1 hour after acute onset of a severe headache and left-sided weakness. The patient had a myocardial infarction 3 years ago that was treated with a bare metal stent in the right coronary artery. She also has hypertension. Medications are carvedilol, atorvastatin, and aspirin. On physical examination, blood pressure is 200/110 mm Hg, pulse rate 78/min, respiration rate is 20/min, and oxygen saturation is 98% with the patient breathing ambient air. The patient is awake and attentive to both sides and has normal language function. Funduscopic examination shows no papilledema or hemorrhage. Pupils are both 3 millimeters in size and reactive. Left facial weakness, dysarthria, and flaccid paralysis in the left arm and leg with loss of sensation are noted. An emergent noncontrast CT scan shows an acute intracerebral hemorrhage, 1 centimeter in diameter, in the right basal ganglia without intraventricular hemorrhage or midline shift. Which of the following is the most appropriate treatment? Hematoma evacuation Intravenous nicardipine Intravenous nitroprusside Platelet transfusion
Patients who have intracerebral hemorrhage without elevated intracranial pressure whose systolic blood pressure is greater than 180 mm Hg should be treated with an intravenous antihypertensive agent, such as nicardipine, to a target blood pressure of 140 mm Hg. This patient should receive intravenous nicardipine. She has an acute intracerebral hemorrhage whose location suggests hypertension as the cause. Systolic blood pressure on emergent physical examination is greater than 180 mm Hg; this means she is at high risk for hematoma expansion. The systolic blood pressure should thus be lowered to a target of 140 mm Hg with a rapidly acting intravenous agent, such as intravenous nicardipine. A recently completed trial that compared systolic blood pressure targets of 120 mm Hg and 140 mm Hg noted increased adverse renal events in the treatment arm with lower blood pressure. Whatever intravenous antihypertensive agent is chosen, it should be possible to taper its dose quickly if systolic blood pressure becomes too low. Hematoma evacuation is an inappropriate treatment because the patient's intracerebral hemorrhage is not near the cortical surface, and the clinical and imaging findings show no evidence of elevated intracranial pressure (ICP) or cerebral herniation. If she were to have neurologic decline, repeat head CT would be appropriate to evaluate for hematoma expansion. Candidates for surgical evacuation of an intracerebral hematoma include those with evidence of elevated ICP or a cerebellar hemorrhage greater than 3 centimeters in size. Nitroprusside is not a first-line blood pressure treatment for patients with acute hemorrhagic stroke, given its potential for increasing ICP. There is no evidence that platelet transfusion reverses the coagulopathy associated with antiplatelet agents or prevents hematoma expansion. Platelet transfusion also carries the risk of coronary stent thrombosis, volume overload, and transfusion-related reactions.
A 35-year-old woman is evaluated in the emergency department for a 3-week history of worsening cough and dyspnea. She works as a sand mover operator at a hydraulic fracturing site and she notes that there is a large amount of dust that often clogs her respirator. She is otherwise healthy and has no history of fever, chills, sweats, or sick contacts. She has no current medications. On physical examination, pulse rate is 110/min and respiration rate is 26/min. Other vital signs are normal. Oxygen saturation is 92% breathing ambient air. Lung examination reveals inspiratory crackles bilaterally. Cardiovascular examination is normal. Chest radiograph shows patchy bilateral opacities with areas of consolidation in the lower lobes. Bronchoalveolar lavage shows fluid with milky white return but no organisms; bacterial cultures are negative. Which of the following is the most likely diagnosis? Acute interstitial pneumonia Acute silicosis Asbestosis Cryptogenic organizing pneumonia
Patients who work in industries that expose them to silica dust are at risk for silicosis.
A 35-year-old woman is evaluated in the emergency department for a 3-week history of worsening cough and dyspnea. She works as a sand mover operator at a hydraulic fracturing site and she notes that there is a large amount of dust that often clogs her respirator. She is otherwise healthy and has no history of fever, chills, sweats, or sick contacts. She has no current medications. On physical examination, pulse rate is 110/min and respiration rate is 26/min. Other vital signs are normal. Oxygen saturation is 92% breathing ambient air. Lung examination reveals inspiratory crackles bilaterally. Cardiovascular examination is normal. Chest radiograph shows patchy bilateral opacities with areas of consolidation in the lower lobes. Bronchoalveolar lavage shows fluid with milky white return but no organisms; bacterial cultures are negative. Which of the following is the most likely diagnosis? Acute interstitial pneumonia Acute silicosis Asbestosis Cryptogenic organizing pneumonia
Patients who work in industries that expose them to silica dust are at risk for silicosis. This patient most likely has acute silicosis, a fibrotic lung disease caused by the inhalation of silica dust. Silicosis is a spectrum of fibrotic lung diseases related to the inhalation of silica dust. Any occupation that disturbs the earth's crust involves potential risk. Workers in industries that process silica-containing rock or sand are also at risk. The typical disease course of simple silicosis can be accelerated (3 to 10 years after exposure) or latent (greater than 10 years after exposure). This patient has acute silicosis, a rare presentation characterized by onset of cough and dyspnea (but no fever) just a few weeks after intense exposure, patchy bilateral opacities on chest radiograph, and a milky effluent from bronchoalveolar lavage (BAL). Hydraulic fracturing, or fracking, is a process whereby large amounts of water and chemicals are injected into the ground; sand, which contains silica, is used to hold open the fissures created to enhance extraction of natural gas. Acute silicosis portends a poor prognosis, as there is little evidence that any therapies can alter disease course. However, removal from the environment and smoking cessation will prevent further lung injury. Acute interstitial pneumonia develops rapidly during days to weeks and results in progressive hypoxemic respiratory failure. Radiographic examination reveals bilateral alveolar opacities consistent with pulmonary edema. The findings on BAL are nonspecific. This patient's findings are not compatible with acute interstitial pneumonia. Workers in the mining industry are at risk for asbestos exposure, but the typical latency period for asbestosis is decades rather than weeks, making this diagnosis unlikely. In addition, the most common radiographic finding of asbestosis is pleural plaques, which are not present on this patient's radiograph. Patients with cryptogenic organizing pneumonia typically present with cough, dyspnea, fever, and malaise during 3 to 4 weeks that mimic community-acquired pneumonia. The chest radiograph reveals bilateral, patchy or diffuse, consolidative or ground-glass opacities and BAL effluent is not milky opaque.
A 30-year-old woman is evaluated during a new-patient appointment. Her personal medical history is unremarkable and she takes no medication. Her family history includes colon cancer diagnosed in her mother at age 48 years, endometrial cancer diagnosed in a maternal aunt at age 51 years, and colon cancer diagnosed in her maternal grandfather at age 55 years. All physical examination findings, including vital signs, are normal. Genetic testing for MSH2 mutation is positive, consistent with Lynch syndrome. Which of the following is the most appropriate next step? Colectomy Colonoscopy at age 38 Colonoscopy at age 40 Colonoscopy now
Patients with Lynch syndrome should begin screening colonoscopy between ages 20 and 25 years or 2 to 5 years before the earliest age of colorectal cancer diagnosis in the family, whichever comes first, and colonoscopy should be repeated every 1 to 2 years if the baseline examination is normal.
A 30-year-old woman is evaluated during a new-patient appointment. Her personal medical history is unremarkable and she takes no medication. Her family history includes colon cancer diagnosed in her mother at age 48 years, endometrial cancer diagnosed in a maternal aunt at age 51 years, and colon cancer diagnosed in her maternal grandfather at age 55 years. All physical examination findings, including vital signs, are normal. Genetic testing for MSH2 mutation is positive, consistent with Lynch syndrome. Which of the following is the most appropriate next step? Colectomy Colonoscopy at age 38 Colonoscopy at age 40 Colonoscopy now
Patients with Lynch syndrome should begin screening colonoscopy between ages 20 and 25 years or 2 to 5 years before the earliest age of colorectal cancer diagnosis in the family, whichever comes first, and colonoscopy should be repeated every 1 to 2 years if the baseline examination is normal. This patient should undergo a screening colonoscopy now, and if it is normal, have a repeat colonoscopy every 1 to 2 years until age 40 years, and then yearly thereafter. The patient has Lynch syndrome based on her family history and the identification of a deleterious mutation in the MSH2 gene. The Amsterdam II criteria (known as the "3-2-1-1-0 rule") can be used to screen for Lynch syndrome; a diagnosis is warranted if the following criteria are met: Three family members are affected with a Lynch syndrome-associated cancer At least two successive generations are affected One affected family member is a first-degree relative of the other two affected family members One of the cancers was diagnosed before age 50 years Familial adenomatous polyposis has been excluded Tumors have been verified histologically The Amsterdam criteria are specific for Lynch syndrome but lack sensitivity. Additional clinical tools, such as the Bethesda criteria or clinical models such as the PREdiction Model for gene Mutations 5 (PREMM5) (http://premm.dfci.harvard.edu), may be used to screen for Lynch syndrome. Lynch syndrome is caused by germline mutation in one of the DNA mismatch repair genes (MLH1, MSH2, MSH6, PMS2) or the epithelial cell adhesion molecule (EPCAM) gene. The appropriate age to begin screening colonoscopy in patients with Lynch syndrome is between ages 20 and 25 years or 2 to 5 years before the earliest age of colorectal cancer diagnosis in the family, whichever comes first; thus, this patient should undergo a colonoscopy now. The risk for colorectal cancer in patients with Lynch syndrome is elevated significantly compared with the general population and is as high as 80% for some individuals. Women with Lynch syndrome are at increased risk for endometrial cancer. Additional cancers, such as gastric, small intestinal, urothelial, ovarian, and pancreaticobiliary cancers, are also associated with this syndrome. Colectomy and continued annual endoscopic surveillance of the remaining rectum is the recommended management for patients with Lynch syndrome who are found to have colorectal cancer on screening colonoscopy. Colectomy is not a recommended option for managing patients with Lynch syndrome in the absence of a documented cancer. Screening starting at age 40 years or 10 years earlier than the youngest age of colon cancer diagnosis in the family is appropriate for individuals with a family history of colon cancer that does not meet criteria for Lynch syndrome.
A 75-year-old woman underwent total hip arthroplasty 4 hours ago and is now evaluated because she has been unable to void since the operation. Bladder ultrasound reveals 900 mL of urine. Other than manageable postoperative pain, she has no symptoms. Current medications are acetaminophen, oxycodone, and enoxaparin. Which of the following is the most appropriate management? Suprapubic catheter placement Suprapubic warm wet gauze application Tamsulosin administration Urethral bladder catheterization
Patients with postoperative urinary retention and residual bladder volume of 800 mL or more should be treated with bladder decompression and urinary catheterization. Bladder decompression with urinary catheterization is the most appropriate management of this patient who has developed postoperative urinary retention (POUR). POUR is a common complication in the postoperative setting and is characterized by the inability to spontaneously and adequately empty the bladder. Risk factors include type of surgery (incontinence and anorectal surgery, hernia repair, joint arthroplasty), longer surgery, use of regional anesthesia, administration of greater than 750 mL of intraoperative fluids, use of certain postoperative medications (opioids, anticholinergic agents), older age, constipation, pelvic organ prolapse, neurologic disease, history of urinary retention, and history of pelvic surgery. POUR is a urologic emergency. Symptoms of suprapubic pain and the finding of a palpable bladder are insensitive indicators of POUR. Patients may also present with frequent urination of small volumes and overflow incontinence. Reversible causes of POUR, such as medication use, should be addressed. Whenever possible, offending medications, including opioids, anticholinergics, antihistamines, antipsychotics, and calcium-blocking drugs, should be discontinued. Early removal of indwelling urinary catheters and voiding trials are recommended. Retrograde voiding trials are preferred to spontaneous voiding trials because they are more predictive of the need for continued catheterization. A retrograde voiding trial involves infusion of sterile saline, followed by the attempt to void. For patients in whom a voiding trial is unsuccessful, intermittent urinary catheterization should be considered in place of indwelling bladder catheterization. Results of a recent randomized controlled trial in patients undergoing total hip arthroplasty and total knee arthroplasty demonstrated that a catheterization threshold of 800 mL significantly reduced the need for postoperative urinary catheterization and did not increase urologic complications. Placement of a suprapubic catheter would require another surgical procedure and is reserved for situations in which urethral catheterization is not possible or in cases of pelvic trauma. Suprapubic application of hot packs or warm wet gauze may stimulate spontaneous voiding but lacks proof from well-designed clinical trials. Some experts recommend consideration of this technique in patients for a limited time when residual bladder volume is 200 mL to 400 mL. Greater residual volume should be treated with bladder catheterization. There is no role for tamsulosin in the treatment of POUR in women. Randomized studies in men have demonstrated that α1-blockade for the treatment of acute urinary retention is associated with a reduced need for bladder catheterization. α1-Blockers should be continued in men with benign prostatic hyperplasia.
A 59-year-old man is evaluated in the hospital for swelling of his left leg. He was admitted 5 days ago with deep venous thrombosis of the right lower extremity and bilateral pulmonary emboli. Therapy with low-molecular-weight heparin (LMWH) was initiated on hospital day 1. Medical history is significant for chronic heart failure, with a hospitalization 1 month ago for pulmonary edema. Other medications are lisinopril, furosemide, and metoprolol. On physical examination, temperature is 38.1 °C (100.6 °F), blood pressure is 132/84 mm Hg, pulse rate is 104/min, and respiration rate is 24/min. Oxygen saturation is 91% breathing ambient air. The right and left legs are swollen to midthigh. Laboratory studies show a hemoglobin level of 12.8 g/dL (128 g/L), leukocyte count of 11,200/µL (11.2 × 109/L), and platelet count of 78,000/µL (78 × 109/L) (at admission, 168,000/µL [168 × 109/L]). Doppler ultrasonography discloses a new left femoral vein thrombosis. Which of the following is the most appropriate management? Discontinue LMWH and begin argatroban Discontinue LMWH and begin unfractionated heparin Insert an inferior vena cava retrievable filter Overlap LMWH and warfarin for 5 days
Patients with a high probability of heparin-induced thrombocytopenia should have heparin stopped, and therapy with an immediately effective anticoagulant, such as argatroban, should be initiated. The most appropriate management is to discontinue heparin and initiate argatroban. This patient has heparin-induced thrombocytopenia (HIT). He was in the hospital 1 month ago for chronic heart failure and most likely received subcutaneous heparin at that time, so he has a history of heparin exposure. A scoring system for determining the pretest probability of a patient having HIT has been developed (4T score). High probability scores are associated with a 50% decrease in platelet count, onset between days 5 and 10 of heparin exposure, new thrombosis, and no other cause of thrombocytopenia. This patient's platelet count decreased more than 50% from admission 5 days ago, he has a new thrombotic event, and he has no other likely explanation for the thrombocytopenia. HIT can be a life-threatening condition if not addressed rapidly. Heparin must be discontinued and an alternate, immediately effective anticoagulant, such as argatroban, must be initiated. A strongly positive immunoassay for HIT antibody will help confirm the diagnosis. The risk of thrombosis persists after resolution of the thrombocytopenia, so treatment with warfarin should begin when the platelet count normalizes; most experts recommend 3 to 6 months of therapy. Patients with HIT must have all heparin products stopped. Replacing low-molecular-weight heparin with unfractionated heparin does not mitigate the risk of additional thrombosis. Most experts advise delaying warfarin therapy until the platelet count has normalized. Insertion of an inferior vena cava (IVC) filter insertion may be appropriate for select patients with acute VTE and a contraindication to anticoagulation. Although this patient has a contraindication to heparin, he can be safely treated with alternative anticoagulants, including argatroban.
A 60-year-old woman is evaluated during a follow-up visit. She has a lifelong history of intermittent asthma previously provoked by exertion and exposure to cold air. During the past 2 years her symptoms have progressed, and she now has dyspnea after walking one block or going up any incline. COPD was diagnosed 4 days ago after spirometry revealed an FEV1 of 65% of predicted that was only partially reversible with bronchodilator therapy. She has no history of acute exacerbations. She has recently discontinued cigarette smoking. She currently takes albuterol as needed. On physical examination, vital signs are normal; oxygen saturation is 95% breathing ambient air. Lungs are clear to auscultation. Laboratory studies reveal normal hemoglobin concentration and leukocyte count with 8% eosinophils. Chest radiograph shows clear lungs. Which of the following is the most appropriate initial treatment? Chronic macrolide therapy Inhaled glucocorticoid and long-acting β2-agonist Long-acting β2-agonists Long-acting muscarinic agent and long-acting β2-agonists
Patients with a history of asthma-COPD overlap syndrome should not be prescribed a long-acting β2-agonist without concurrent therapy with an inhaled glucocorticoid because of the increased risk of mortality in patients with asthma who are prescribed long-acting β2-agonist monotherapy. The most appropriate treatment is a combination inhaled glucocorticoid and a long-acting β2-agonist. This patient has progressive symptoms, spirometry showing diminished FEV1 that partially reversed with bronchodilation, and eosinophilia. Several diagnostic terms have been used to describe patients with both asthma and COPD, most including the word overlap, but there is no universally agreed upon term or defining diagnostic features for this condition. Given her progression of symptoms, her short-acting inhaler therapy should be augmented. Although a long-acting β2-agonist is indicated in symptomatic patients with COPD and an FEV1 of less than 60% of predicted, this may not be the best treatment for a patient with asthma. Patients with asthma are at increased risk of mortality when a long-acting bronchodilator is prescribed without a controller medication. Experts recommend that patients who have an asthma-COPD overlap syndrome who are receiving a long-acting bronchodilator should ideally also be prescribed an inhaled glucocorticoid. Combination therapy seems to mitigate the excess risk of mortality observed in patients with asthma treated with long-acting β2-agonist monotherapy. Therefore, the patient should be started on a combination therapy of an inhaled glucocorticoid and a long-acting β2-agonist. Chronic macrolide therapy can reduce the incidence of acute exacerbations but this patient has not had any exacerbations to warrant starting a macrolide. Using a long-acting β2-agonist or a long-acting muscarinic agent/long-acting β2-agonist combination inhaler is likely not indicated for this patient with asthma-COPD overlap syndrome based on current guidelines. Long-acting bronchodilators should only be used in combination with inhaled glucocorticoids in patients with a history of asthma. Using long-acting bronchodilators alone has been linked to increased risk of asthma-related deaths.
A 21-year-old man is hospitalized for sudden onset of dyspnea with chest pain that worsens with inspiration. He has a 3-pack-year history of smoking, but his medical history is otherwise unremarkable. On physical examination, vital signs are normal. Oxygen saturation is 95% breathing ambient air. BMI is 18. Lung examination reveals reduced lung expansion, hyperresonance to percussion, and diminished breath sounds on the left side. Chest radiograph demonstrates a large left-sided pneumothorax. A thoracostomy tube is inserted that provides good lung reexpansion. Repeat chest radiograph is normal. Avoidance of which of the following is the most appropriate measure to prevent long-term recurrence of pneumothorax? Air travel Mountain climbing Smoking Strenuous exercise
Patients with a primary spontaneous pneumothorax should be encouraged to stop smoking to prevent recurrence. Smoking cessation is the most effective measure to prevent recurrent pneumothorax. This patient has a primary spontaneous pneumothorax (PSP) (air in the pleural space in someone without underlying lung disease). Cigarette smoking is a significant risk factor for PSP, likely because of airway inflammation. The lifetime risk of pneumothorax for men who are lifelong heavy smokers is 12%, compared to 0.1% for men who have never smoked. Because of the strong association between smoking and occurrence of PSP, smoking cessation may help prevent recurrence. Other risk factors for PSP are family history of PSP and thoracic endometriosis. PSP usually develops when the patient is at rest, and presenting patients are typically in their early 20s. Symptoms include the sudden onset of dyspnea and pleuritic chest pain. Recurrence is estimated at 23% to 50% during the first 5 years. Interventions to prevent recurrence includes chemical and mechanical pleurodesis, which are recommended after the second occurrence of PSP on the ipsilateral side, or first occurrence if the patient has a high-risk occupation such as deep sea diver or airplane pilot. Air travel should be discouraged until resolution of the pneumothorax, but it is not in itself a risk factor for developing a pneumothorax. There is no association between mountain climbing and development of a pneumothorax. There is no association between the onset of pneumothorax and physical activity, with the occurrence being as likely when the patient is sedentary as when active.
A 72-year-old woman is evaluated during a routine visit. She has a 30-pack-year smoking history and quit 5 years ago. She has a history of mild COPD and breast cancer diagnosed 15 years ago, currently in remission. A chest radiograph from 5 years ago showed no signs of disease recurrence. Medications are albuterol and tiotropium inhalers. On physical examination, vital signs are normal. Lung examination reveals prolonged expiration and diminished breath sounds throughout. The breast examination is unremarkable. A screening low-dose chest CT scan shows a peripheral 9-mm solid pulmonary nodule in the left upper lobe and emphysema but no mediastinal or hilar lymphadenopathy and no pleural effusion. A PET/CT scan using fluorodeoxyglucose (FDG) is performed and the nodule is intensely hypermetabolic. There is no evidence of distant uptake. Which of the following is the most appropriate management? Bronchoscopy with biopsy Serial chest CT scans Surgical wedge resection Transthoracic needle aspiration
Patients with a solid indeterminate lung nodule larger than 8 mm and high probability of malignancy should be staged using a PET/CT scan followed by definitive management. Definitive treatment is recommended for this patient and, therefore, a surgical wedge resection is appropriate. She has several risk factors for malignancy, including age, size of the nodule, upper-lobe location of the nodule, smoking history, and history of malignancy. In addition, the PET/CT scan showed fludeoxyglucose avidity, confirming the high probability of malignancy but without evidence of distant metastasis. As with subcentimeter nodules, the availability of previous imaging of the chest to assess the stability or growth of these lesions is helpful. An enlarging or new pulmonary nodule warrants more aggressive evaluation with tissue diagnosis or excision depending on the nodule's pretest probability of malignancy. The first step when evaluating a solid pulmonary nodule that is larger than 8 mm is to estimate the probability of malignancy. This can be done either clinically or using quantitative models and should place the patient in one of three categories: low probability (less than 5%), intermediate probability (5% to 65%), or high probability (greater than 65%). This is most useful when nodules are 8-30 mm. If the lesion is larger than 30 mm, the likelihood of malignancy is so high that it typically is resected; in contrast, when the lesion is smaller than 8 mm, the likelihood of malignancy is low and the patient should undergo routine radiological surveillance with serial CT scans. Biopsy of the nodule or a transthoracic approach is preferred when the probability of malignancy is intermediate (5% to 65%) and would not be appropriate for this patient with a hypermetabolic nodule on PET/CT scan suggesting a high probability of malignancy. Furthermore, the sampling procedure is chosen according to size and location of the nodule, availability, and local expertise. Typically, peripheral nodules are sampled using CT-guided transthoracic needle aspiration, and more central lesions are sampled using bronchoscopic techniques. This lesion is described as peripheral. Radiologic surveillance with serial CT scans is preferred if the probability of malignancy is low (less than 5%). This patient's lung nodule is highly suspicious for malignancy on CT/PET scan so sampling with CT-guided transthoracic needle aspiration is not indicated.
A 55-year-old man is evaluated for progressive dysphagia of 2 years' duration. He reports dysphagia to both solid food and liquids. He has cardiomyopathy, with an ejection fraction of 15%. His medications are pantoprazole, furosemide, valsartan, digoxin, metoprolol, low-dose aspirin, and amiodarone. He is unable to walk up two flights of stairs without stopping. He does not drink or smoke. On physical examination, his blood pressure is 100/65 mm Hg, pulse rate is 90/min, and respiratory rate is 22/min; BMI is 32. His examination is remarkable for fine crackles at the lung bases posteriorly, a third heart sound, and 2+ pitting edema to the knees. Upper endoscopy shows no masses. A barium esophagram is shown achalasia). Which of the following is the most appropriate treatment? Botulinum toxin injection Calcium channel blockers Endoscopic pneumatic dilation Laparoscopic surgical myotomy
Patients with achalasia who are at high surgical risk should be treated with endoscopic botulinum toxin injection. Endoscopic botulinum toxin injection is the best treatment option for this patient because his cardiac status places him at an unacceptable surgical risk. Achalasia is a motility disorder of the esophagus that results in aperistalsis and inadequate relaxation of the lower esophageal sphincter (LES). The cause of achalasia is unknown, but the pathophysiologic process involves ganglion cell and myenteric plexus degeneration in the esophageal body and LES. This nerve imbalance leads to uncontested action by cholinergic nerves and incomplete LES relaxation. The clinical presentation of achalasia consists of dysphagia to both solids and liquids. Additional symptoms may include regurgitation, chest pain, and heartburn. Standard treatments for achalasia include surgical myotomy and endoscopic pneumatic dilation. Botulinum toxin injection inhibits acetylcholine release, resulting in relaxation of the LES and relief of achalasia symptoms in up to 85% of patients. However, approximately 50% of patients will experience recurrent symptoms within 6 to 24 months after injection. Medical therapy with calcium channel blockers or long-acting nitrates is considered third-line therapy because its effectiveness is limited. Medications may be used early in treatment, in patients who decline other interventions, before definitive treatment, and in patients who have a recurrence in symptoms after botulinum toxin injection for achalasia. Studies have shown that endoscopic pneumatic dilation and surgical myotomy result in similar clinical outcomes in patients with achalasia, with choice of therapy depending on local expertise. Pneumatic dilation is the most effective nonsurgical treatment and is more cost-effective than surgical myotomy, but it is associated with serious complications, such as esophageal perforation. Therefore, patients who are not surgical candidates should not undergo endoscopic dilation treatment of achalasia.
A 52-year-old woman is evaluated in the hospital for left arm and left leg weakness of 12 hours' duration. She has a 4-month history of progressive dyspnea on exertion, fatigue, low-grade fever, and arthralgia. She takes no medications. On physical examination, temperature is 38.1 °C (100.5 °F). Other vital signs are normal. On neurologic examination, muscle strength is 4/5 in the left arm and left leg; strength in all other muscle groups is 5/5. Cardiac examination reveals a loud S1, a soft diastolic rumble heard at the apex, and an early diastolic sound. The remainder of the examination is normal. Laboratory studies are significant for an erythrocyte sedimentation rate of 87 mm/h; normocytic, normochromic anemia; and a normal leukocyte count. An electrocardiogram demonstrates normal sinus rhythm. MRI of the brain reveals multiple small infarcts. An echocardiogram is shown. Left ventricular function is normal. Which of the following is the most appropriate treatment? Anticoagulation Chemotherapy and radiotherapy Empiric broad-spectrum antibiotic therapy Percutaenous transesophageal echocardiographic-guided biopsy Surgical excision
Patients with an atrial myxoma may present with constitutional symptoms, embolic phenomena from tumor fragmentation, or symptoms referable to intracardiac obstruction (dyspnea, syncope); treatment is surgical excision. The most appropriate treatment is surgical excision. This patient's history, physical examination findings, and echocardiographic features are all most consistent with a left atrial myxoma. The parasternal long-axis echocardiographic view (shown) demonstrates a large atrial myxoma (AM) prolapsing across the mitral valve from the left atrium (LA) to the left ventricle (LV) during diastole. Myxomas are connective tissue tumors with cells encompassed within mucopolysaccharide stroma. Most are attached to the left atrial wall in proximity to the fossa ovalis. Patients with an atrial myxoma most often present with symptoms related to obstruction or embolization, or constitutional symptoms related to production of tumor-based interleukin 6, such as fatigue, low-grade fever, and arthralgia. Dyspnea may be related to mitral valvular obstruction, which causes a diastolic rumble resembling mitral stenosis and an early diastolic sound ("tumor plop") heard in 10% to 15% of patients. Treatment for an atrial myxoma is urgent surgical excision. Systemic anticoagulation would be appropriate in patients with a thrombus. Right atrial masses in patients with indwelling cardiac catheters are more likely to represent thrombus. PET and contrast-enhanced MRI/CT may be useful in evaluating metabolic activity and vascularity to differentiate a tumor from a thrombus. This patient has no risk factors for intracardiac thrombus, such as cardiac catheter, atrial fibrillation, or poor left ventricular function, making it an unlikely diagnosis. Chemotherapy and radiotherapy are used to treat primary cardiac lymphoma. Suspicion for cardiac lymphoma would be higher if this patient were immunocompromised and had presented with a right atrial mass rather than an intra-atrial mass. Empiric antibiotic therapy would be appropriate for suspected infective endocarditis. Although this patient has a low-grade fever, embolic phenomena, and constitutional symptoms, she has no risk factors for infective endocarditis and no leukocytosis. Additionally, attachment of the tumor to the fossa ovalis, not a valvular surface, makes infective endocarditis unlikely. Percutaneous biopsy with echocardiographic guidance is sometimes useful in establishing the diagnosis of right-sided masses. However, percutaneous biopsy would not eliminate the need for surgical excision in this patient with embolic events.
A 68-year-old woman is evaluated in the emergency department for a 1-hour history of chest pain. Medical history is significant for hypertension and a 20-year history of type 2 diabetes mellitus. Medications are metformin, quinapril, and aspirin. On physical examination, blood pressure is 95/60 mm Hg, pulse rate is 50/min, and respiration rate is 16/min. The patient is alert and conversant. The precordial cadence is not regular. There is no evidence of pulmonary or peripheral congestion, and the extremities are warm. Laboratory studies reveal a serum troponin T level of 1.1 ng/mL (1.1 µg/L). An electrocardiogram is shown. STEMI in inferior leads, Which of the following is the most appropriate next step in management of this patient's arrhythmia? Cardiac catheterization Echocardiography Permanent pacemaker implantation Temporary pacing
Patients with atrioventricular block and evidence of acute coronary syndrome should undergo cardiac catheterization for diagnosis and possible revascularization.
A 68-year-old woman is evaluated in the emergency department for a 1-hour history of chest pain. Medical history is significant for hypertension and a 20-year history of type 2 diabetes mellitus. Medications are metformin, quinapril, and aspirin. On physical examination, blood pressure is 95/60 mm Hg, pulse rate is 50/min, and respiration rate is 16/min. The patient is alert and conversant. The precordial cadence is not regular. There is no evidence of pulmonary or peripheral congestion, and the extremities are warm. Laboratory studies reveal a serum troponin T level of 1.1 ng/mL (1.1 µg/L). An electrocardiogram is shown. STEMI in inferior leads, Which of the following is the most appropriate next step in management of this patient's arrhythmia? Cardiac catheterization Echocardiography Permanent pacemaker implantation Temporary pacing
Patients with atrioventricular block and evidence of acute coronary syndrome should undergo cardiac catheterization for diagnosis and possible revascularization. This patient has Mobitz type 1 second-degree atrioventricular (AV) block and evidence of an acute coronary syndrome, and she should undergo cardiac catheterization for emergent revascularization. This patient's chest pain, ST-segment elevation, and elevated troponin T level all indicate acute myocardial infarction. Additionally, the electrocardiogram demonstrates second-degree AV block, which is characterized by nonconducted P waves. The progressive prolongation of the PR interval before loss of AV conduction is consistent with Mobitz type 1 (Wenckebach block). There are many causes of AV block, including fibrosis and sclerosis of the conduction system, ischemic heart disease, and medication use (β-blockers, calcium channel blockers, digoxin). Reversible causes of AV block should always be identified and treated first. In this patient, cardiac catheterization and revascularization are indicated not only to treat the acute coronary obstruction but also to potentially correct the conduction deficit. Transthoracic echocardiography should be performed in patients with myocardial infarction to evaluate left ventricular function and assess for potential structural complications; however, obtaining an echocardiogram is not the priority in this patient and does not help manage the arrhythmia. The most appropriate next step is cardiac catheterization. Permanent or temporary pacemaker placement is not required in this case because the patient does not have symptomatic or hemodynamically unstable bradycardia or advanced AV block (high-degree AV block, Mobitz type 2 second-degree, or third-degree AV block). If symptomatic or advanced conduction block persists after revascularization, permanent pacemaker implantation would be recommended.
A 45-year-old woman is evaluated during a routine follow-up appointment for cirrhosis due to primary biliary cholangitis. She has chronic symptoms of fatigue that are unchanged since her previous visits. She has no other new symptoms of advancing liver dysfunction, including no ascites, jaundice, or hepatic encephalopathy. A recent upper endoscopy showed small esophageal varices without red wale marks. Her only medication is ursodeoxycholic acid. She is up to date for her influenza, 23-valent polysaccharide pneumococcal, and hepatitis A and B immunizations. On physical examination, vital signs are normal. Scattered spider angiomas are seen on her chest and upper back. The remainder of the examination is normal. An abdominal ultrasound shows changes in the liver consistent with cirrhosis. No liver masses are seen. Which of the following is the most appropriate intervention to perform next? Bone densitometry Colonoscopy Contrast-enhanced abdominal CT Herpes zoster immunization
Patients with cirrhosis are at increased risk for the development of osteoporosis and should be screened using bone densitometry. Bone densitometry to screen for osteoporosis is the most appropriate next intervention for this patient with cirrhosis. Patients with cirrhosis are at twice the risk for the development of osteoporosis as demographically similar patients without cirrhosis, and women (especially those who are postmenopausal) are at higher risk than men. Initial studies of metabolic bone disease in patients with cirrhosis focused on those with primary biliary cholangitis (PBC) or alcoholic liver disease. Subsequently, it has been recognized that all patients with cirrhosis are at increased risk for bone fractures due to osteoporosis. Standard evaluation should include measurement of serum calcium, phosphate, and vitamin D levels. Up to two thirds of patients with cirrhosis are deficient in vitamin D. Dual-energy x-ray absorptiometry is recommended for patients with cirrhosis or PBC (even if noncirrhotic). Patients with primary sclerosing cholangitis (PSC) have a high likelihood of ulcerative colitis with associated risk for colonic dysplasia and colon cancer. Eighty percent of patients with PSC have inflammatory bowel disease, usually ulcerative colitis. This association with ulcerative colitis is not seen in patients with PBC. This patient is at average risk for colorectal cancer, and colonoscopy is not indicated until age 50 years. Contrast-enhanced abdominal CT with arterial and portal venous phase imaging can be helpful in evaluating for hepatocellular cancer; however, in this patient whose ultrasound showed no hepatic mass, a CT scan is not necessary. Patients with cirrhosis who develop an infection have a 30% mortality rate at 1 month, and another 30% die within 12 months; this underscores the importance of preventing infection. However, the herpes zoster vaccine is not indicated in patients younger than age 50 years. The recombinant zoster vaccine is recommended for individuals aged 50 years and older, regardless of whether they have had a previous zoster episode. There is currently no Advisory Committee on Immunization Practices recommendation on the use of recombinant zoster vaccine among pregnant women or adults with immunocompromising conditions. Patients with decompensated cirrhosis are considered immunocompromised due to impaired reticuloendothelial cell function, impaired neutrophil function, and possible concomitant malnutrition.
A 43-year-old man with chronic alcoholic liver disease is hospitalized after massive vomiting of blood. Medical history is notable for alcohol dependence. He takes no medications. On physical examination, temperature is normal, blood pressure is 102/68 mm Hg, pulse rate is 118/min, and respiration rate is 22/min. He is jaundiced and has spider angiomata on the anterior chest. The liver is palpable but not tender. Laboratory studies: Activated partial thromboplastin time 45 s Prothrombin time 17.8 s Hemoglobin 9.8 g/dL (98 g/L) Platelet count 42,000/µL (42 × 109/L) D-dimer 5800 µg/mL (5800 mg/L) Fibrinogen 66 mg/dL (0.6 g/L) He receives volume resuscitation with fluids, erythrocyte and platelet transfusions, and vitamin K. Which of the following should also be provided to manage this patient's coagulopathy? Activated factor VII concentrate ε-Aminocaproic acid Cryoprecipitate Four-factor prothrombin concentrate
Patients with coagulopathy of liver disease and low fibrinogen levels who are experiencing bleeding should receive immediate cryoprecipitate transfusion. This patient should receive a transfusion with cryoprecipitate. He has the coagulopathy of liver disease, characterized by thrombocytopenia, prolonged activated partial thromboplastin time (aPTT) and prothrombin time (PT), elevated D-dimer level, and hypofibrinogenemia. The patient's hemodynamic instability is addressed with fluids and erythrocyte transfusion. Vitamin K will help correct the PT, but he may also need fresh frozen plasma (FFP). Thrombocytopenia is addressed with platelet transfusion. Fibrinogen levels less than 100 mg/dL (1 g/L) associated with active bleeding should be corrected. Cryoprecipitate contains more fibrinogen than FFP and is the blood component of choice for treating this patient. It is often difficult to distinguish between disseminated intravascular coagulation (DIC) and the coagulopathy of liver disease, but a normal or increased factor VIII level points to liver disease instead of DIC. Somewhat paradoxically, factor VIII levels are supranormal in the coagulopathy of liver disease because it is produced in extrahepatic endothelial cells, and a hepatically synthesized factor is required for factor VIII clearance. The management of these coagulopathies is often the same. Recombinant activated factor VII, activated prothrombin complex concentrates, or recombinant porcine factor VIII is recommended to treat patients with acquired hemophilia associated with high titers of antibody directed against factor VIII. Recombinant activated factor VII may also be used to treat uncontrolled bleeding in patients with liver disease unresponsive to replacement of platelets, FFP, vitamin K, and cryoprecipitate. ε-Aminocaproic acid (EACA), an inhibitor of fibrinolysis, has been shown to reduce blood loss after cardiac and orthopedic surgery. Accelerated fibrinolysis plays some role in the coagulopathy of bleeding in liver disease, and EACA may reduce bleeding in some of these patients. No laboratory test correlates fibrinolysis with bleeding risk, and EACA should not be used before treating more objective measures of coagulopathy, such as low fibrinogen levels. Four-factor prothrombin complex concentrate is used to treat patients with warfarin toxicity and life-threatening bleeding. It should not be used more routinely to treat patients with the coagulopathy of liver disease and bleeding until conventional component support proves ineffective.
A 21-year-old man is hospitalized with a 3-day history of fever and cough productive of green sputum. He was in the hospital 2 months ago for giardiasis and a year ago for pneumonia. Medical history is also notable for frequent episodes of sinusitis, bronchitis, and otitis media dating back to childhood. On physical examination, temperature is 38.2 °C (100.8 °F), blood pressure is 118/80 mm Hg, pulse rate is 108/min, and respiration rate is 24/min. Pulmonary examination reveals crackles at the right lung base. Chest radiograph is significant for a right lower lobe consolidation, and empiric antibiotic therapy is initiated. A sputum culture grows Streptococcus pneumoniae. Which of the following is the most likely underlying diagnosis? 1% A AIDS Chronic granulomatous disease Common variable immunodeficiency Myeloperoxidase deficiency
Patients with common variable immunodeficiency are at increased risk of recurrent respiratory tract infections with encapsulated organisms (Streptococcus pneumoniae, Haemophilus influenzae), and they may develop chronic diarrhea because of enteroviruses, norovirus, or Giardia.
A 21-year-old man is hospitalized with a 3-day history of fever and cough productive of green sputum. He was in the hospital 2 months ago for giardiasis and a year ago for pneumonia. Medical history is also notable for frequent episodes of sinusitis, bronchitis, and otitis media dating back to childhood. On physical examination, temperature is 38.2 °C (100.8 °F), blood pressure is 118/80 mm Hg, pulse rate is 108/min, and respiration rate is 24/min. Pulmonary examination reveals crackles at the right lung base. Chest radiograph is significant for a right lower lobe consolidation, and empiric antibiotic therapy is initiated. A sputum culture grows Streptococcus pneumoniae. Which of the following is the most likely underlying diagnosis? 1% A AIDS Chronic granulomatous disease Common variable immunodeficiency Myeloperoxidase deficiency
Patients with common variable immunodeficiency are at increased risk of recurrent respiratory tract infections with encapsulated organisms (Streptococcus pneumoniae, Haemophilus influenzae), and they may develop chronic diarrhea because of enteroviruses, norovirus, or Giardia. This patient has common variable immunodeficiency (CVID). The history of recurrent sinopulmonary and gastrointestinal infections in a young patient should trigger an investigation for an underlying immunodeficiency. CVID refers to a heterogeneous group of disorders that are linked by the presence of low levels of IgG and IgA and an impaired ability to produce antibody to antigenic stimuli. Clinically, patients with CVID are at increased risk for recurrent respiratory tract infections with encapsulated organisms such as Streptococcus pneumoniae and Haemophilus influenzae as well as Mycoplasma pneumoniae. These patients may also develop chronic diarrhea because of enteroviruses, norovirus, or Giardia. Based on this patient's history of previous infection with several of these pathogens, further evaluation with quantitative immunoglobulin levels is warranted. Diagnosis of CVID is important; patients with CVID benefit from replacement therapy with exogenous immune globulin. In addition to recurrent infection, patients with CVID are at increased risk for autoimmune diseases, bronchiectasis, enteropathy, and lymphoma. Patients with advanced HIV infection or AIDS have low levels of CD4 cells and are at risk for opportunistic infections typically controlled through cellular immunity. Mucocutaneous candidiasis infection is one of the most common manifestations of HIV infection. Cryptococcus infection typically manifests as subacute or chronic meningitis. Pneumocystis jirovecii pneumonia is a common complication in patients with HIV infection who have not received prophylaxis. Toxoplasma gondii can cause encephalitis. Tuberculosis and Mycobacterium avium complex infection are the most common mycobacterial infections in patients with AIDS. Cytomegalovirus usually manifests as retinitis, esophagitis or colitis, and polyradiculitis or encephalitis. The nature of this patient's infections and history dating back to childhood make advanced AIDS an unlikely diagnosis. Chronic granulomatous disease (CGD), which is usually diagnosed in childhood, is caused by a defect in neutrophil oxidation. A history of recurrent or unusually severe infections or infections with Aspergillus species, Staphylococcus aureus, Burkholderia cepacia complex, Serratia marcescens, or Nocardia species suggests the diagnosis. This patient's frequent episodes of sinusitis, bronchitis, otitis media, and giardiasis are not characteristic of CGD. Myeloperoxidase is an intraleukocytic enzyme that plays a role in the destruction of fungal organisms. Complete deficiency of myeloperoxidase is rare, and most patients with this syndrome remain asymptomatic. This patient's recurrent episodes of infection since childhood are not compatible with myeloperoxidase deficiency.
A 21-year-old man is hospitalized with a 3-day history of fever and cough productive of green sputum. He was in the hospital 2 months ago for giardiasis and a year ago for pneumonia. Medical history is also notable for frequent episodes of sinusitis, bronchitis, and otitis media dating back to childhood. On physical examination, temperature is 38.2 °C (100.8 °F), blood pressure is 118/80 mm Hg, pulse rate is 108/min, and respiration rate is 24/min. Pulmonary examination reveals crackles at the right lung base. Chest radiograph is significant for a right lower lobe consolidation, and empiric antibiotic therapy is initiated. A sputum culture grows Streptococcus pneumoniae. Which of the following is the most likely underlying diagnosis? AIDS Chronic granulomatous disease Common variable immunodeficiency Myeloperoxidase deficiency
Patients with common variable immunodeficiency are at increased risk of recurrent respiratory tract infections with encapsulated organisms (Streptococcus pneumoniae, Haemophilus influenzae), and they may develop chronic diarrhea because of enteroviruses, norovirus, or Giardia. This patient has common variable immunodeficiency (CVID). The history of recurrent sinopulmonary and gastrointestinal infections in a young patient should trigger an investigation for an underlying immunodeficiency. CVID refers to a heterogeneous group of disorders that are linked by the presence of low levels of IgG and IgA and an impaired ability to produce antibody to antigenic stimuli. Clinically, patients with CVID are at increased risk for recurrent respiratory tract infections with encapsulated organisms such as Streptococcus pneumoniae and Haemophilus influenzae as well as Mycoplasma pneumoniae. These patients may also develop chronic diarrhea because of enteroviruses, norovirus, or Giardia. Based on this patient's history of previous infection with several of these pathogens, further evaluation with quantitative immunoglobulin levels is warranted. Diagnosis of CVID is important; patients with CVID benefit from replacement therapy with exogenous immune globulin. In addition to recurrent infection, patients with CVID are at increased risk for autoimmune diseases, bronchiectasis, enteropathy, and lymphoma. Patients with advanced HIV infection or AIDS have low levels of CD4 cells and are at risk for opportunistic infections typically controlled through cellular immunity. Mucocutaneous candidiasis infection is one of the most common manifestations of HIV infection. Cryptococcus infection typically manifests as subacute or chronic meningitis. Pneumocystis jirovecii pneumonia is a common complication in patients with HIV infection who have not received prophylaxis. Toxoplasma gondii can cause encephalitis. Tuberculosis and Mycobacterium avium complex infection are the most common mycobacterial infections in patients with AIDS. Cytomegalovirus usually manifests as retinitis, esophagitis or colitis, and polyradiculitis or encephalitis. The nature of this patient's infections and history dating back to childhood make advanced AIDS an unlikely diagnosis. Chronic granulomatous disease (CGD), which is usually diagnosed in childhood, is caused by a defect in neutrophil oxidation. A history of recurrent or unusually severe infections or infections with Aspergillus species, Staphylococcus aureus, Burkholderia cepacia complex, Serratia marcescens, or Nocardia species suggests the diagnosis. This patient's frequent episodes of sinusitis, bronchitis, otitis media, and giardiasis are not characteristic of CGD. Myeloperoxidase is an intraleukocytic enzyme that plays a role in the destruction of fungal organisms. Complete deficiency of myeloperoxidase is rare, and most patients with this syndrome remain asymptomatic. This patient's recurrent episodes of infection since childhood are not compatible with myeloperoxidase deficiency.
A 62-year-old woman is evaluated during a routine examination. She feels well and has no exercise limitations. Medical history is significant for hypertension treated with enalapril. She does not smoke. At her last examination 4 years ago, her blood pressure was 122/76 mm Hg. Laboratory studies at that time revealed a serum total cholesterol level of 184 mg/dL (4.77 mmol/L), serum HDL cholesterol level of 42 mg/dL (1.09 mmol/L), and hemoglobin A1c value of 5.4%. At that time, according to the Pooled Cohort Equations, her 10-year risk for atherosclerotic cardiovascular disease was calculated to be 3.8%. She has gained 5 kg (11 lb) since then. On physical examination today, blood pressure is 140/90 mm Hg. BMI is 29. The remainder of the physical examination is unremarkable. Laboratory studies are significant for a serum total cholesterol level of 250 mg/dL (6.47 mmol/L), a serum HDL cholesterol level of 30 mg/dL (0.78 mmol/L), and a hemoglobin A1c value of 6.6%. A subsequent fasting plasma glucose level is 130 mg/dL (7.2 mmol/L). Which of the following confers the highest risk for atherosclerotic cardiovascular disease in this patient? Diabetes mellitus Diastolic blood pressure of 90 mm Hg HDL cholesterol level of 30 mg/dL (0.78 mmol/L) Systolic blood pressure of 140 mm Hg Total cholesterol level of 250 mg/dL (6.47 mmol/L)
Patients with diabetes mellitus have a two to four times increased risk for cardiovascular disease, with more than two thirds of patients with diabetes eventually dying of heart disease. The factor associated with the highest risk for atherosclerotic cardiovascular disease (ASCVD) in this woman is the diagnosis of diabetes mellitus. The presence of diabetes is associated with increased cardiovascular risk, particularly among women. Patients with diabetes have a two to four times increased risk for cardiovascular disease, with more than two thirds of patients with diabetes eventually dying of heart disease. The risk for stroke is increased 1.8- to 6-fold in patients with diabetes. Additionally, patients with diabetes are more likely to have undiagnosed coronary artery disease and have worse outcomes when hospitalized for other cardiovascular diseases, such as heart failure. In this patient, the development of diabetes alone (with all other risk factors held constant) nearly doubled her 10-year risk for ASCVD. Her current 10-year risk, including diabetes, changes in her age, and blood pressure and lipid levels, is 19.4%. Appropriate treatment of cardiovascular risk factors in patients with diabetes is associated with reduced cardiovascular risk. The 2018 American Heart Association/American College of Cardiology Guideline on the Management of Blood Cholesterol recommends that patients aged 40 to 75 years with diabetes receive moderate-intensity statin therapy. If additional risk factors are present, it is reasonable to calculate the 10-year risk for ASCVD. Patients should receive high-intensity statin therapy if 10-year ASCVD risk is 20% or higher. A diastolic blood pressure of 90 mm Hg, HDL cholesterol level of 30 mg/dL (0.78 mmol/L), systolic blood pressure of 140 mm Hg, and total cholesterol level of 250 mg/dL (6.47 mmol/L) have a relatively small effect on 10-year ASCVD risk compared with the presence of diabetes.
A 46-year-old man is evaluated in the emergency department for right flank pain that began 3 hours ago. He describes the pain as sharp and severe with radiation to the right testicle. History is significant for chronic diarrhea from Crohn disease; he has two to three loose bowel movements each day. He reports no nausea, vomiting, or abdominal pain. He has no dysuria. On physical examination, the patient appears uncomfortable. There is right costovertebral angle tenderness. Laboratory studies: Electrolytes : Sodium 138 mEq/L (138 mmol/L) Potassium 3.9 mEq/L (3.9 mmol/L) Chloride 106 mEq/L (106 mmol/L) Bicarbonate 21 mEq/L (21 mmol/L) Urinalysis Specific gravity 1.025; pH 5.5; moderate blood; no protein, leukocyte esterase, or nitrites Kidney ultrasound shows a 6-mm stone at the right ureteral pelvic junction. Which of the following is the most likely composition of this patient's kidney stone? Calcium oxalate Calcium phosphate Cystine Struvite Uric acid
Patients with diarrhea who are volume depleted and have a metabolic acidosis are at increased risk for developing kidney stones, particularly calcium oxalate stones and, less commonly, uric acid stones.
A 46-year-old man is evaluated in the emergency department for right flank pain that began 3 hours ago. He describes the pain as sharp and severe with radiation to the right testicle. History is significant for chronic diarrhea from Crohn disease; he has two to three loose bowel movements each day. He reports no nausea, vomiting, or abdominal pain. He has no dysuria. On physical examination, the patient appears uncomfortable. There is right costovertebral angle tenderness. Laboratory studies: Electrolytes : Sodium 138 mEq/L (138 mmol/L) Potassium 3.9 mEq/L (3.9 mmol/L) Chloride 106 mEq/L (106 mmol/L) Bicarbonate 21 mEq/L (21 mmol/L) Urinalysis Specific gravity 1.025; pH 5.5; moderate blood; no protein, leukocyte esterase, or nitrites Kidney ultrasound shows a 6-mm stone at the right ureteral pelvic junction. Which of the following is the most likely composition of this patient's kidney stone? Calcium oxalate Calcium phosphate Cystine Struvite Uric acid
Patients with diarrhea who are volume depleted and have a metabolic acidosis are at increased risk for developing kidney stones, particularly calcium oxalate stones and, less commonly, uric acid stones. The most likely composition of this patient's kidney stone is calcium oxalate. This patient has classic symptoms of renal colic, including flank pain that radiates to the groin. Stone movement may result in pain migration to the genitalia. Nausea, vomiting, and dysuria may also be present. Microscopic hematuria is usually noted, although its absence does not exclude a stone. Patients with diarrhea who are volume depleted and have a metabolic acidosis are at increased risk for developing a kidney stone, particularly stones composed of calcium oxalate and uric acid. In this patient with Crohn disease and chronic diarrhea, the most likely composition of the stone is calcium oxalate because the chronic metabolic acidosis (suggested by the low serum bicarbonate concentration and relatively low urine pH) increases calcium loss from bone and decreases citrate excretion. Citrate is the major inhibitor of calcium crystallization in the urine. In addition, if there is concomitant fat malabsorption, a common occurrence in inflammatory bowel disease, calcium will bind to fat in the gut, allowing increased absorption of oxalate. Struvite stones are composed of magnesium ammonium phosphate (struvite) and calcium carbonate-apatite and occur in the presence of urea-splitting bacteria, such as Proteus or Klebsiella, in the upper urinary tract. These organisms convert urea to ammonium, which alkalinizes the urine, decreases the solubility of phosphate, and leads to struvite precipitation. Struvite stones can rapidly enlarge to fill the entire renal pelvis within weeks to months, taking on a characteristic "staghorn" shape. Calcium phosphate stones also form in alkaline urine. The low urine pH and absence of signs of infection on urinalysis make these diagnoses unlikely. Cystine stones are caused by cystinuria, a rare autosomal recessive disorder of proximal tubular transport of dibasic amino acids such as cystine that presents at a young age. The main risk factor for uric acid stones is low urine pH, usually ≤5.0, which decreases the solubility of uric acid. Hyperuricosuria is not a consistent finding. Comorbid risk factors for uric acid stones include gout, diabetes mellitus, the metabolic syndrome, and chronic diarrhea.
A 62-year-old man is evaluated during a follow-up visit for difficult-to-control hypertension. He also has chronic kidney disease and hyperlipidemia. Medications are atorvastatin and carvedilol, as well as maximum doses of lisinopril, amlodipine, and hydralazine. On physical examination, the average of three blood pressure measurements is 152/98 mm Hg, and pulse rate is 65/min. There is 1+ pitting pretibial lower extremity edema. The remainder of the examination is normal. Laboratory studies: Creatinine 2.5 mg/dL (221 µmol/L) Potassium 4.8 mEq/L (4.8 mmol/L) Estimated glomerular filtration rate 28 mL/min/1.73 m2 Urine albumin-creatinine ratio 350 mg/g Which of the following is the most appropriate additional treatment? Chlorthalidone Furosemide Hydrochlorothiazide Losartan
Patients with difficult-to-control hypertension typically require the addition of a diuretic, which prevents or corrects extracellular volume expansion in sodium-retentive, edematous conditions (heart failure, liver cirrhosis, chronic kidney disease). The addition of a loop diuretic such as furosemide is the most appropriate treatment. This patient has difficult-to-control hypertension in the setting of stage G4 chronic kidney disease (CKD), and his blood pressure is not at target. The American College of Cardiology/American Heart Association blood pressure guideline recommends a target blood pressure of <130/80 mm Hg for all patients, including those with CKD. The most appropriate treatment is the addition of a loop diuretic. Suboptimal blood pressure therapy in patients with difficult-to-control hypertension is frequently the result of not including a diuretic, which prevents or corrects extracellular volume expansion. Persistent volume expansion, even if not sufficient to produce clinically evident edema, contributes significantly to hypertension. This is particularly important in sodium-retentive, edematous conditions such as heart failure, liver cirrhosis, or CKD. Although thiazide diuretics are frequently used as initial diuretic therapy, they are generally less effective in patients with lower glomerular filtration rates. At estimated glomerular filtration rates <30 mL/min/1.73 m2, loop diuretics tend to be more effective at controlling extracellular volume expansion and should be used instead of (or added to) thiazide diuretics. The dosage of loop diuretics depends on the sodium intake and the severity of CKD. Generally, furosemide doses of 40 to 80 mg twice daily is initiated with a salt-restricted diet and adjusted according to the response. When it is appropriate to add a thiazide diuretic to a blood pressure regimen, chlorthalidone is often preferred because of its longer duration of action. The patient is already on a maximum dose of the ACE inhibitor lisinopril, which is appropriate for the treatment of hypertension in patients with CKD. Although he still has albuminuria, addition of the angiotensin receptor blocker (ARB) losartan is inappropriate because several trials have shown that combination therapy with an ACE inhibitor and ARB may result in adverse events, such as acute kidney injury and hyperkalemia, and does not improve cardiovascular outcomes compared with treatment with an ACE inhibitor or ARB alone.
A 72-year-old man is evaluated for a 4-month history of progressive exertional dyspnea. He has a 3-year history of ischemic cardiomyopathy, and in the past 6 months, he has had two hospitalizations for heart failure exacerbations. He currently has shortness of breath at rest. Medical history is otherwise significant for renal cell carcinoma treated with surgical resection 4 years ago. Medications are enalapril, bisoprolol, furosemide, atorvastatin, and aspirin. On physical examination, temperature is normal, blood pressure is 104/80 mm Hg, pulse rate is 98/min, and respiration rate is 18/min. His neck veins are flat. Cardiac examination reveals an S3. There is no peripheral edema. Extremities are cool. Laboratory studies show a blood urea nitrogen level of 40 mg/dL (14.3 mmol/L) and a serum sodium level of 134 mEq/L (134 mmol/L). The serum creatinine level is 1.7 mg/dL (150.3 µmol/L), which is increased from 1.4 mg/dL (123.8 µmol/L) 3 months ago. An electrocardiogram shows a QRS duration of 90 ms. An echocardiogram shows a left ventricular ejection fraction of 15%, left ventricular diastolic diameter of 8.2 cm (normal 3.9-5.3 cm), and moderate mitral regurgitation. Which of the following is the most appropriate management? Cardiac resynchronization therapy Cardiac transplantation Left ventricular assist device placement Mitral valve repair
Patients with end-stage heart failure should be considered for cardiac transplantation or mechanical circulatory support with left ventricular assist device placement. The most appropriate management is left ventricular assist device (LVAD) placement. This patient has several signs and symptoms of advanced refractory heart failure, including New York Heart Association (NYHA) functional class IV symptoms despite optimal medical therapy, a low sodium level, worsening kidney function, low pulse pressure, cool extremities, and repeat hospitalizations in the past 6 months. In patients with end-stage heart failure, advanced treatments, including cardiac transplantation or mechanical circulatory support with an LVAD, should be considered. This patient is a good candidate for an LVAD as destination therapy. Placement of a continuous-flow LVAD reduces symptoms and improves survival compared with medical therapy in patients with end-stage heart failure. Although LVADs increase survival rates, they are associated with serious complications, including infection, gastrointestinal bleeding, stroke, and pump thrombus. All patients with progressive heart failure symptoms should be engaged in a discussion about their prognosis, treatment options (including hospice), and the risks and benefits of each therapy. Cardiac resynchronization therapy is indicated in patients with an ejection fraction less than or equal to 35%, NYHA functional class II to IV heart failure symptoms despite guideline-directed medical therapy, sinus rhythm, and left bundle branch block with a QRS complex of 150 ms or greater. This patient does not meet the electrocardiographic criteria. Cardiac transplantation remains the gold standard therapy for patients with end-stage heart failure. Indications for cardiac transplantation in these patients include age younger than 65 to 70 years, no medical contraindications (diabetes with end-organ complications, malignancies within 5 years, kidney dysfunction, other chronic illnesses that will decrease survival), and good social support and adherence. Heart transplant is contraindicated in this patient. This patient has significant mitral regurgitation, most likely secondary to his dilated left ventricle; however, mitral valve repair is unlikely to substantially improve his heart failure symptoms.
A 70-year-old man is evaluated for a recent onset of macroscopic hematuria. History is significant for end-stage kidney disease and hypertension. He has been on hemodialysis for 3 years. Urine output is approximately 250 mL/d. Medications are sevelamer, sodium bicarbonate, lisinopril, and amlodipine. On physical examination, blood pressure is 150/90 mm Hg, and pulse rate is 70/min. Bilateral flank tenderness is noted. There is no abdominal mass. Laboratory studies show a hemoglobin level of 15 g/dL (150 g/L). Kidney ultrasound shows several complex cysts and two bilateral solid masses. Which of the following is the most appropriate management? Bilateral partial nephrectomy Bilateral radical nephrectomy Percutaneous kidney biopsy Surveillance ultrasonography
Patients with end-stage kidney disease have a markedly increased risk for renal cell carcinoma, and a high level of suspicion is warranted in patients with symptoms such as new-onset gross hematuria or unexplained flank pain. Bilateral radical nephrectomy is the most appropriate management for this patient with end-stage chronic kidney disease (ESKD) and bilateral kidney solid masses. Acquired kidney cysts often develop in patients with severe chronic kidney disease (CKD) and are frequently detected during routine kidney ultrasound or incidentally noted on abdominal CT or MRI scan. Acquired cystic kidney disease becomes more common and progresses during the course of ESKD, and some studies suggest that it may affect >50% of patients who have had ESKD for >3 years. The epithelial cells lining these cysts may undergo malignant transformation by poorly understood mechanisms. Patients with ESKD have a markedly increased risk for renal cell carcinoma. Although current guidelines do not support routine screening for renal cell carcinoma in all patients with CKD, a high level of suspicion is warranted in patients with symptoms such as new-onset gross hematuria or unexplained flank pain. Partial nephrectomy and nephron-sparing approaches would be indicated for less severe stages of CKD. However, this patient has ESKD, and maintaining residual kidney function is no longer a concern; therefore, radical nephrectomy would be the most appropriate option. Kidney biopsy should be considered in patients with glomerular hematuria, severely increased albuminuria, acute or chronic kidney disease of unclear etiology, and kidney transplant dysfunction or monitoring. The role of kidney biopsy for a suspicious mass is more limited. It may be useful in the evaluation of a small mass if there is suspicion of a renal metastasis or lymphoma and is likely useful to confirm the diagnosis of renal cell carcinoma in patients who cannot tolerate surgery prior to initiating medical therapy. The best approach for this patient with a high likelihood of renal cell carcinoma is bilateral nephrectomy. The excised tissue will provide histological confirmation of the diagnosis and thus guide further therapy. Surveillance ultrasonography would not be the best management of this patient with bilateral solid kidney masses. The new-onset hematuria and kidney masses in the context of advanced CKD are highly suspicious for renal cell carcinoma.
A 58-year-old woman is evaluated for a 4-week history of left lateral hip pain. She describes the pain as a moderate ache that intermittently radiates down the lateral aspect of the left leg. It began insidiously and has gradually worsened. The pain worsens when she is climbing stairs or lying on the affected side. She reports no previous trauma to the area. She has not had any leg weakness or swelling or any constitutional symptoms. She has not taken any analgesics for the pain. On physical examination, vital signs are normal. On palpation, there is tenderness over the left greater trochanter. There is painless full range of motion with abduction, flexion, and external rotation of the left hip. The remainder of the examination is normal. In addition to activity modification, which of the following is the most appropriate management? Ibuprofen Glucocorticoid injection Hydrocodone/acetaminophen Plain radiography of the left hip
Patients with greater trochanteric pain syndrome (trochanteric bursitis) typically have pain localized to the greater trochanter, which may radiate down the lateral leg to the knee, and pain to palpation over the greater trochanter; treatment includes avoiding painful activities, acetaminophen or NSAIDs, and muscle strengthening. This patient's clinical presentation is consistent with greater trochanteric pain syndrome (GTPS; formerly trochanteric bursitis), and first-line therapy is pain relief with acetaminophen or an oral NSAID, such as ibuprofen. Patients with GTPS typically have pain localized to the greater trochanter that may radiate down the lateral leg to the knee. The pain is often exacerbated by lying on the affected side and climbing stairs. Pain onset is usually insidious. GTPS can be differentiated from hip joint pain in that GTPS does not usually radiate to the groin or limit hip range of motion. Diagnosis is made by history and by eliciting pain with palpation over the greater trochanter or reproduction of the pain when the patient takes a step up. Use of pain-relieving agents should accompany activity modification, such as avoiding or minimizing painful activities. Physical therapy to strengthen the muscles of the hip may help with reducing friction and therefore pain. Glucocorticoid injections (frequently combined with a local anesthetic) are reserved for patients with GTPS with persistent symptoms and for those who do not respond to acetaminophen or an oral NSAID. Because this patient has not yet received any therapy, it would be most appropriate to start with an oral agent such as ibuprofen instead of progressing directly to glucocorticoid injection. Most patients respond to acetaminophen, NSAIDs, or glucocorticoid injections. Opioid pain medications are typically unnecessary, have a significant risk profile, and do not have a role in the management of GTPS. Therefore, hydrocodone/acetaminophen would not be an appropriate management option. GTPS is diagnosed clinically based on a consistent clinical presentation. Plain radiographs are typically normal in patients suspected of having GTPS. The role of imaging studies is to evaluate for alternative diagnoses when the diagnosis of GTPS is unclear. In this patient, plain radiography is unnecessary. Read Related TextNext Question
A 62-year-old woman is evaluated during a follow-up appointment 12 weeks after she completed treatment for genotype 1a hepatitis C virus (HCV) infection. She has Child-Turcotte-Pugh Class A cirrhosis (well-compensated cirrhosis). Small esophageal varices were noted on upper endoscopy 1 year earlier. On physical examination, vital signs are normal; BMI is 26. Palmar erythema, spider angiomata over the chest, a firm liver edge 3 cm below the costal margin, and a palpable spleen tip are noted. The examination is otherwise normal. Her HCV RNA is undetectable and her calculated Model for End-Stage Liver Disease score is 8. Which of the following is the most appropriate management for this patient? Liver transplantation evaluation Measurement of HCV RNA in 12 weeks Ultrasonography of the liver every 6 months Upper endoscopy
Patients with hepatitis C viral infection who achieve sustained virologic response have a reduced risk for hepatocellular carcinoma; regardless of virologic response, ultrasonographic surveillance is recommended for patients with stage 3 or stage 4 fibrosis. The most appropriate management for this patient is ultrasonography screening for hepatocellular carcinoma every 6 months. Liver diseases associated with the highest risk for hepatocellular carcinoma are hepatitis B virus (HBV) and hepatitis C virus (HCV) infections and hemochromatosis. Approximately 80% of hepatocellular carcinoma occurs in patients with cirrhosis, but it can develop in the absence of cirrhosis in patients with HBV infection. All patients with cirrhosis from any cause should undergo liver ultrasonography every 6 months with or without α-fetoprotein measurement. Patients with HCV infection who achieve sustained virologic response (which is synonymous with virologic cure) have a reduced risk for hepatocellular carcinoma. Regardless of virologic response, surveillance is recommended for patients with stage 3 or stage 4 fibrosis (stage 4 fibrosis signifies cirrhosis, as found in this patient). Liver transplantation evaluation is not indicated for this patient with Child-Turcotte-Pugh Class A cirrhosis and a very low Model for End-Stage Liver Disease (MELD) score. Indications for liver transplantation are a MELD score of at least 15 or decompensated cirrhosis. Virologic cure in patients with compensated cirrhosis prevents decompensation. However, development of hepatocellular carcinoma would be a reason for referral for liver transplantation evaluation. Measuring HCV RNA again in 12 weeks is not indicated because this patient has achieved virologic cure with undetectable HCV RNA at week 12 after completing treatment for HCV infection. Large studies have demonstrated 98% to 99% concordance between sustained virologic response at 12 and at 24 weeks. Therefore, sustained virologic response at 12 weeks is considered to be consistent with virologic cure, and additional testing is unnecessary. Upper endoscopy is not necessary because the patient had an upper endoscopy 1 year ago. The standard follow-up interval for small varices in a patient who is not taking a nonselective β-blocker is 2 years. Read Related TextNext Question
A 28-year-old man is evaluated in the emergency department for a right ankle sprain that occurred earlier in the day while he was refereeing a soccer game. When running down the field, his right cleat stuck in the turf, and his foot rotated laterally. He was able to bear weight immediately after the injury, but he had to leave the game. The ankle has begun to swell, making it difficult for the patient to ambulate. On physical examination, vital signs are normal. The patient walks with antalgic gait, although he is able to bear weight. Swelling is present over the anterior and lateral distal leg above the right ankle. Pain is reproduced both by squeezing the leg at mid-calf level and by having the patient cross his right leg with the lateral malleolus resting on the left knee. There is no tenderness to palpation over the malleoli, the navicular bone, or the fifth metatarsal base. Which of the following is the most likely diagnosis? Achilles tendon rupture Ankle fracture High ankle sprain Lateral ankle sprain
Patients with high ankle sprains report the acute onset of pain proximal to the ankle, accompanied by swelling and ecchymosis. The most likely diagnosis is a high ankle sprain. High ankle sprains result from excessive dorsiflexion or eversion that causes injury to the tibiofibular syndesmotic ligaments connecting the distal tibia and fibula. Pain can be elicited by compressing the leg at mid-calf (squeeze test), by having the patient cross the legs with the lateral malleolus of the injured leg resting on the other knee (crossed-leg test), or by dorsiflexing and externally rotating the foot with the knee flexed (dorsiflexion-external rotation test). The most common mechanism of injury involves an externally rotated force applied to a dorsiflexed ankle, as is the case with this patient. Patients with high ankle sprains report the acute onset of pain proximal to the ankle. Pain is often accompanied by swelling and ecchymosis. Treatment is similar to that used for other ankle sprains and includes cryotherapy (ice or cold water), mobilization, and analgesics for pain control, but recovery is usually delayed. Achilles tendon rupture most commonly results from sudden, forceful plantar flexion, such as occurs with jumping and sprinting. Patients report sudden onset of heel pain and often hear a popping sound at the time of the injury. On examination, patients have weak or absent plantar flexion. Absent plantar flexion with calf squeezing (Thompson test) also suggests the diagnosis. This patient's clinical presentation is not consistent with Achilles tendon rupture. The Ottawa ankle and foot rules are useful in excluding ankle fractures, with an extremely high sensitivity (>95%). According to these validated rules, radiography should be obtained when a patient is unable to walk four steps both immediately after the injury and during evaluation, and when focal tenderness is present at the posterior aspect of the malleolus, the navicular bone, or the fifth metatarsal base. If these criteria are not met, obtaining radiography is not necessary because the probability of an ankle fracture is exceedingly low, as in this patient. Lateral ankle sprains typically result from inversion injuries to the lateral ankle ligaments (the anterior and posterior talofibular ligaments and the calcaneofibular ligament). Physical examination reveals ecchymosis, lateral ankle tenderness, and swelling, which was not the case with this patient. This patient's symptoms and findings were located above the ankle.
A 26-year-old woman is evaluated for left lateral knee and distal thigh pain that began 6 weeks ago. She is a long-distance runner who trains 6 days per week. The pain began insidiously and has slowly worsened over time. The pain is worst when she is running downhill. She experiences no pain while resting. She has not had any knee trauma and reports no catching, grinding, or locking. On physical examination, vital signs are normal. On palpation, tenderness is noted 2 cm proximal to the lateral femoral condyle. With the patient supine, pain is reproduced with repeated flexion and extension of the knee as thumb pressure is applied to the lateral femoral epicondyle. There is weakness with left hip abduction. There is no joint line tenderness, joint effusion, or ligament laxity with applied stress. Which of the following is the most likely diagnosis? Iliotibial band syndrome Lateral collateral ligament tear Lateral meniscal tear Meralgia paresthetica
Patients with iliotibial band syndrome report diffuse, poorly localized lateral knee and distal thigh pain; there is often tenderness to palpation 2 to 3 cm proximal to the lateral femoral condyle. The most likely diagnosis is iliotibial band syndrome (ITBS). ITBS is a common cause of lateral knee pain in runners and can also occur in patients with significant leg length difference, an excessively pronated foot, genu varum, or gluteal muscle weakness. Patients with ITBS have pain that is poorly localized to the lateral knee and distal thigh. Initially, the pain is present only after prolonged activity (such as running) that involves repeated knee flexion and extension. As the condition progresses, the pain occurs earlier in the course of activity and may eventually be present at rest. On examination, there is often tenderness to palpation 2 to 3 cm proximal to the lateral femoral condyle. Patients also frequently have weakness with hip abduction. Reproduction of the pain with knee extension from 90 degrees to 30 degrees with the examiner's thumb exerting pressure on the lateral femoral epicondyle (Noble test) supports the diagnosis of ITBS. Initial treatment consists of activity modification, ice application, and NSAIDs to reduce inflammation. Once inflammation subsides, stretching and then strengthening exercises are indicated. This patient lacks history of trauma, joint instability, lateral joint line tenderness, or increased laxity with varus force. Lack of these features argues against the presence of a lateral collateral ligament tear. The lack of prior trauma and absence of catching, grinding, and locking all argue against a meniscal tear, as does the absence of an effusion on examination. Meralgia paresthetica is due to entrapment of the lateral femoral cutaneous nerve and causes paresthesias on the anterolateral thigh. Risk factors include diabetes mellitus, obesity, pregnancy, and tight clothing or belts around the waist. This patient's findings are not consistent with meralgia paresthetica.
A 45-year-old man is evaluated in the emergency department for alcohol intoxication, but the source and type of alcohol consumed are unknown. He has a history of heavy ethanol use and occasional isopropanol use. On physical examination, vital signs are normal. Oxygen saturation is 97% on ambient air. He is somnolent but easily aroused. Laboratory studies Bicarbonate 24 mEq/L (24 mmol/L) Blood urea nitrogen 14 mg/dL (5 mmol/L) Chloride 106 mEq/L (106 mmol/L) Creatinine 1.6 mg/dL (141.4 µmol/L) Ethanol Negative Glucose 90 mg/dL (5 mmol/L) Osmolality , plasma 315 mOsm/kg H2O Potassium 4.1 mEq/L (4.1 mmol/L) Sodium 139 mEq/L (139 mmol/L) In addition to monitoring for signs of alcohol withdrawal, which of the following is the most appropriate treatment? Fomepizole Hemodialysis Levetiracetam Supportive care
Patients with isopropyl alcohol poisoning can be treated effectively using supportive care.
A 45-year-old man is evaluated in the emergency department for alcohol intoxication, but the source and type of alcohol consumed are unknown. He has a history of heavy ethanol use and occasional isopropanol use. On physical examination, vital signs are normal. Oxygen saturation is 97% on ambient air. He is somnolent but easily aroused. Laboratory studies Bicarbonate 24 mEq/L (24 mmol/L) Blood urea nitrogen 14 mg/dL (5 mmol/L) Chloride 106 mEq/L (106 mmol/L) Creatinine 1.6 mg/dL (141.4 µmol/L) Ethanol Negative Glucose 90 mg/dL (5 mmol/L) Osmolality , plasma 315 mOsm/kg H2O Potassium 4.1 mEq/L (4.1 mmol/L) Sodium 139 mEq/L (139 mmol/L) In addition to monitoring for signs of alcohol withdrawal, which of the following is the most appropriate treatment? Fomepizole Hemodialysis Levetiracetam Supportive care
Patients with isopropyl alcohol poisoning can be treated effectively using supportive care. The most appropriate management is supportive care. Calculation of the plasma osmolal gap is helpful in assessing the presence of unmeasured solutes, such as ingestion of certain toxins (for example, methanol or ethylene glycol). The plasma osmolal gap is the difference between the measured and calculated plasma osmolality. Plasma osmolality can be calculated using the following formula: Plasma Osmolality (mOsm/kg H2O) = 2 × Serum Sodium (mEq/L) + Plasma Glucose (mg/dL)/18 + Blood Urea Nitrogen (mg/dL)/2.8. When the measured osmolality exceeds the calculated osmolality by greater than 10 mOsm/kg H2O, the osmolal gap is considered elevated. This patient does not have an increased anion-gap metabolic acidosis, thus eliminating methanol and ethylene glycol poisoning. An elevated osmolal gap of 27 and absent blood ethanol support the diagnosis of isopropyl alcohol ingestion (rubbing alcohol), which does not cause a metabolic acidosis. It is metabolized by alcohol dehydrogenase (ADH) to acetone, which can cause a fruity odor on the patient's breath. There are no other toxic metabolites, and the main effect of isopropyl alcohol ingestion is central nervous system depression by both the isopropyl alcohol and the acetone. There is no need to block the action of ADH with fomepizole. Administration of fomepizole would be advised for treatment of methanol or ethylene glycol ingestion, because these alcohols both have toxic metabolites that can lead to blindness, kidney failure, or death. Inhibiting ADH in patients who have ingested isopropyl alcohol only prolongs its elimination. Hemodialysis is not normally needed to clear isopropyl alcohol from the blood, although it is a highly effective modality for eliminating all ingested alcohols. The risks of initiating hemodialysis in this patient who is inebriated but otherwise stable would not be justified. Seizure prophylaxis with levetiracetam is not indicated for isopropyl alcohol ingestion, and this patient shows no clinical signs of withdrawal, a prelude to seizures. As long as he is closely monitored and treated appropriately with benzodiazepine medication for signs of withdrawal, antiepileptic medication should not be needed.
A 55-year-old woman is evaluated in the office for a recent diagnosis of lobular carcinoma in situ and atypical lobular hyperplasia. She wishes to discuss options to decrease her risk of breast cancer. The patient is postmenopausal. Her medical history is significant for a left calf deep venous thrombosis at 25 years of age concurrent with oral contraceptive use. She takes no medications. On physical examination, vital signs are normal. There is a healed left upper breast incision. The remainder of the examination is normal. Which of the following is the most appropriate preventive measure for this patient? Exemestane High-dose vitamin D Low-fat diet Raloxifene Tamoxifen
Patients with lobular carcinoma in situ and atypical hyperplasia are candidates for chemoprophylaxis with antiestrogens. Exemestane is the most appropriate preventive measure. This postmenopausal woman has lobular carcinoma in situ and atypical hyperplasia. These atypical lesions are associated with a 30% to 35% lifetime risk of breast cancer, and patients with these findings on breast biopsy are candidates for chemoprophylaxis with antiestrogens. Antiestrogen options include tamoxifen in both premenopausal and postmenopausal women as well as raloxifene and aromatase inhibitors (such as exemestane or anastrozole) in postmenopausal women. Because this patient had a deep venous thrombosis while taking oral contraceptives in the past, both tamoxifen and raloxifene are contraindicated, and exemestane is the best option if she is willing to take chemoprophylaxis. Exemestane prophylaxis decreased the risk of invasive breast cancer by 65% in the National Cancer Institute of Canada (NCIC) Clinical Trials Group MAP.3 trial. Compared with placebo, there were no significant differences between the two arms in terms of skeletal fractures, cardiovascular events, other cancers, or treatment-related deaths. There were minimal differences in quality-of-life symptoms, such as vasomotor symptoms or arthralgia. Unlike tamoxifen and raloxifene, exemestane does not increase the risk for venous thromboembolic disease or endometrial cancer. Anastrozole, another aromatase inhibitor, has also been shown to decrease the risk of breast cancer in patients with high-risk conditions and is an alternate option for this patient. Although vitamin D deficiency has been associated with a higher risk of breast cancer, there are no studies supporting high-dose vitamin D as an effective method to decrease the risk of breast cancer. A low-fat diet has been shown to decrease breast cancer recurrence in patients with a diagnosis of estrogen receptor-negative invasive breast cancer but has not been shown to be of benefit in decreasing the risk of breast cancer in patients with atypical breast lesions or other high-risk conditions. Both raloxifene and tamoxifen are contraindicated due to this patient's history of a deep venous thrombosis.
A 25-year-old man is evaluated for worsening heartburn of 4 months' duration, despite treatment with twice-daily omeprazole. He has no pertinent personal or family medical history and takes no other medication. On physical examination, vital signs and other findings are normal. Upper endoscopy shows a normal esophagus with a normal gastroesophageal junction. The stomach has 30 small (<10 mm) sessile polyps seen in the fundus. The duodenum is normal. Pathology of a polyp shows it to be a fundic gland polyp. Which of the following is the most appropriate next step in evaluation of this patient? Colonoscopy Gastrectomy Repeat upper endoscopy in 3 months No further evaluation
Patients with multiple fundic gland polyps found at a young age should be evaluated for familial adenomatous polyposis. Colonoscopy is the best next step in the evaluation of this patient. The patient has multiple fundic gland polyps, which can be associated with familial adenomatous polyposis (FAP). Fundic gland polyps related to FAP are usually numerous (>30 polyps) and frequently harbor low-grade dysplasia, but they rarely progress to gastric cancer. Colonoscopy to rule out FAP is recommended in patients younger than age 40 years with dysplastic or numerous fundic gland polyps. Classic FAP results in the development of hundreds to thousands of colorectal adenomas that often manifest by the second decade of life. Without treatment, colorectal cancer typically develops in all patients by age 40 years. Therefore, a colonoscopy is indicated in this patient to evaluate for colonic polyposis. If colonic polyposis identified, the patient should undergo genetic testing to make the definitive diagnosis of FAP. FAP is caused by mutations in the APC gene. Mutations can be inherited in an autosomal dominant pattern with multiple generations affected, but de novo mutations account for up to 25% of cases. If FAP is confirmed in this patient, it is likely due to a de novo mutation because of his unremarkable family history. There is no role for surgical management of fundic gland polyps because they rarely progress to gastric cancer, so gastrectomy is not indicated for this patient. An upper endoscopy in 3 months is not indicated for diagnosis of FAP. If FAP is confirmed, the patient should have regular surveillance of the upper gastrointestinal tract with a forward and side-viewing scope to assess for duodenal polyps. Duodenal polyps occur commonly in FAP, and risk for duodenal cancer is elevated. The interval for surveillance upper endoscopy in FAP depends on the number, size, and pathology of the duodenal polyps. This upper endoscopy also includes surveillance for gastric adenomas and cancer. For sporadic fundic gland polyps, no further evaluation would be needed. Sporadic fundic gland polyps can arise in the setting of proton pump inhibitor (PPI) therapy; however, this usually occurs with prolonged therapy. Because this patient has only been taking a PPI for 4 months, his fundic gland polyposis is unlikely to be caused by PPI therapy. The patient's young age and the number of gastric polyps should prompt evaluation by colonoscopy.
A 65-year-old woman is reevaluated following an initial evaluation for anemia. Other than a gradually increasing sense of fatigue, she has no symptoms. Her only other medical problem is autoimmune thyroid disease, which is being treated with levothyroxine. Her last screening colonoscopy, done 4 years earlier, was normal. Physical examination reveals normal vital signs. There is evidence of conjunctival rim pallor. The remainder of the examination, including thyroid and neurologic examinations, is normal. At the time of her initial evaluation, laboratory studies showed a hemoglobin level of 10 g/dL (100 g/L) and mean corpuscular volume of 104 fL. Serum cobalamin and ferritin levels were low. An antiparietal cell antibody test was positive. Stool testing revealed no evidence of blood. Which of the following is the most appropriate test to perform next? Capsule endoscopy Glucose hydrogen breath test Serum gastrin measurement Upper endoscopy
Patients with newly diagnosed pernicious anemia should be evaluated for gastric adenocarcinoma and gastric carcinoid with upper endoscopy and gastric biopsy. Upper endoscopy with gastric biopsy is the most appropriate next diagnostic study. The two forms of atrophic gastritis are Helicobacter pylori-associated and autoimmune. H. pylori-associated atrophic gastritis typically resolves with H. pylori eradication, whereas autoimmune atrophic gastritis has no cure. This patient's hematologic findings and positive testing for serum antiparietal antibodies confirm the diagnosis of pernicious anemia and suggest the presence of autoimmune atrophic gastritis. Iron deficiency anemia is a common comorbidity of autoimmune gastritis, as is autoimmune thyroid disease. Autoimmune gastritis is associated with parietal cell loss, reduced gastric acid production, and secondary hypergastrinemia. Hypergastrinemia is associated with an increased risk for the development of gastric carcinoid and adenocarcinoma. It is prudent to perform upper endoscopy and gastric biopsy at the time of pernicious anemia diagnosis to evaluate for these cancers; however, the benefit on ongoing surveillance endoscopy is unclear. Autoimmune-related pernicious anemia and associated iron deficiency anemia are likely to require lifelong vitamin B12 and iron replacement, respectively. Capsule endoscopy is typically used to evaluate obscure causes of gastrointestinal bleeding. This patient had a normal colonoscopy 4 years earlier, and stool evaluation is negative for blood. Finally, this patient's iron deficiency anemia is most likely due to atrophic gastritis-related achlorhydria, which decreases iron absorption by impairing the conversion of ferric iron to absorbable ferrous iron; in many patients, iron deficiency precedes vitamin B12 deficiency. Additional testing for iron deficiency beyond upper endoscopy is likely unnecessary. Although small intestinal bacterial overgrowth (SIBO) can be associated with anemia, the diagnosis is unlikely in this patient with no risk factors or typical symptoms of SIBO; therefore, a glucose breath test to assess for SIBO is likely to be of low yield. The serum gastrin level is elevated in patients with any form of atrophic gastritis and has no diagnostic or prognostic value in this setting.
A 34-year-old woman is seeking medical advice. She is pregnant at 24 weeks' gestation. Her husband returned home 1 week ago from a medical mission in the Middle East. Spores of anthrax were discovered in the hospital where he was working. Her husband, who is asymptomatic, has begun taking ciprofloxacin and will receive a vaccination series. The patient questions if she is at risk for infection. Her only medication is a prenatal vitamin. On physical examination, vital signs are normal. The examination is unremarkable. Which of the following is the most appropriate management? Doxycycline and vaccination Nasopharyngeal swab and anthrax blood test Raxibacumab injection No treatment
Patients with no known direct exposure to anthrax do not require treatment or separation from those who may be infected.
A 34-year-old woman is seeking medical advice. She is pregnant at 24 weeks' gestation. Her husband returned home 1 week ago from a medical mission in the Middle East. Spores of anthrax were discovered in the hospital where he was working. Her husband, who is asymptomatic, has begun taking ciprofloxacin and will receive a vaccination series. The patient questions if she is at risk for infection. Her only medication is a prenatal vitamin. On physical examination, vital signs are normal. The examination is unremarkable. Which of the following is the most appropriate management? Doxycycline and vaccination Nasopharyngeal swab and anthrax blood test Raxibacumab injection No treatment
Patients with no known direct exposure to anthrax do not require treatment or separation from those who may be infected. Because this patient has no known direct exposure to anthrax, no treatment is necessary. In cases of proven or suspected anthrax in a family member, no specific treatment or isolation procedures are required for others in the household because spread in health care or household settings has never been demonstrated. In patients with confirmed or suspected bioterrorism-related anthrax exposure, postexposure prophylactic antibiotics, taken for 60 days, should be started as soon as possible. Ciprofloxacin, levofloxacin, and doxycycline are the approved drugs for postexposure prophylaxis in adult patients. In pregnant women, ciprofloxacin is the drug of choice, and although tetracyclines are not recommended during pregnancy, doxycycline can be used with caution when ciprofloxacin is contraindicated. Therapy can be completed with amoxicillin if the isolate is found to be penicillin susceptible. Because of the possibility that residual dormant spores may become active after antibiotics are completed, three subcutaneous injections of anthrax vaccine should be given at 2-week intervals as part of postexposure prophylaxis. No test is available for the detection of anthrax infection in an asymptomatic person, so taking a swab or performing a blood test would provide no useful information. The human monoclonal antibodies against anthrax toxin, raxibacumab (available only from the Centers for Disease Control and Prevention) and obiltoxaximab (FDA approved) can be combined with antibiotics for treatment of inhalation anthrax or for postexposure prophylaxis when alternative preventive therapies are not available or appropriate. In this scenario, they are not indicated for the patient or her husband who has received adequate prophylaxis for anthrax and does not have systemic disease.
A 59-year-old woman is evaluated for a 2-month history of shortness of breath and cough. She has a 55-pack-year smoking history. She takes no medications. On physical examination, vital signs are normal. BMI is 23. Oxygen saturation is normal. Pulmonary examination reveals decreased breath sounds and crackles in the right base posteriorly. Her performance status is good. CT scan of the chest shows a large, centrally located right lower lobe mass, evidence of mediastinal invasion, bulky mediastinal lymph nodes, and hilar lymphadenopathy. Bronchoscopy with biopsy of an endobronchial lesion confirms non-small cell lung cancer. Brain MRI, CT scans of the abdomen and pelvis, and bone scan show no metastatic disease. Which of the following is the most appropriate management? Chemotherapy Combined chemotherapy and radiation Pembrolizumab Radiation Surgical resection of the right lower lobe mass
Patients with non-small cell lung cancer who have extensive primary tumors or bulky, multistation lymphadenopathy in the mediastinum are not candidates for surgery and instead should be treated with combined chemotherapy and radiation. The most appropriate treatment is combined chemotherapy and radiation. Treatment for stage III non-small cell lung cancer, which is defined in most cases by the presence of mediastinal lymphadenopathy, can vary significantly based on the extent of the primary tumor and the extent of associated lymphadenopathy. Patients with extensive primary tumors, such as this patient whose tumor invades the mediastinum, are not candidates for surgery. Patients with bulky, multistation (widespread mediastinal or hilar lymph node involvement) lymphadenopathy in the mediastinum do not fare well after surgery. These patients should be treated with combined chemotherapy and radiation, which has been shown to improve outcomes compared with either chemotherapy alone or radiation alone. In highly selected patients with stage III disease, surgery may be considered an appropriate therapeutic option. For patients with T3N1 disease, surgery can be done as initial therapy, although the procedure should include mediastinal lymph node dissection. In patients with negative mediastinal lymph nodes, adjuvant chemotherapy should be given. In patients with positive mediastinal lymph nodes, sequential chemotherapy and radiation is recommended. However, this patient has bulky mediastinal disease and is clearly not a candidate for surgery. Pembrolizumab is a monoclonal antibody directed against programmed death receptor 1 (PD-1) that is indicated for treatment of recurrent metastatic non-small cell lung cancer. It has no role in the treatment of locally advanced non-small cell lung cancer and would not be indicated for this patient. Radiation alone is inferior to combined chemotherapy and radiation and is not typically recommended. It would be recommended only for patients who were not candidates for combined chemotherapy and radiation based on comorbid conditions. However, this patient does not have any comorbid conditions that would preclude chemotherapy. As noted previously, this patient has bulky mediastinal disease and is clearly not a candidate for surgery; therefore, surgical resection is not appropriate.
A 75-year-old man is evaluated for generalized lymphadenopathy. He is otherwise asymptomatic and takes no medications. On physical examination, all vital signs are normal. There is generalized lymphadenopathy with nodes 1 cm to 2.5 cm in size in his cervical, supraclavicular, axillary, and inguinal areas bilaterally. There is no hepatosplenomegaly. The remainder of the physical examination is normal. CT scan of the chest, abdomen, and pelvis shows mediastinal and retroperitoneal nodes that are 1.5 cm to 2.0 cm in size. Laboratory results, including complete blood count, biochemistry profile, and lactate dehydrogenase level, are normal. Serum urate level is 9.5 mg/dL (0.56 mmol/L). Left inguinal node biopsy shows follicular lymphoma. Which of the following is the most appropriate management? Allopurinol Radiation therapy Rituximab plus cyclophosphamide, doxorubicin, vincristine, and prednisone (R-CHOP) Observation
Patients with nonbulky follicular lymphoma who are not symptomatic may survive many years without the need for therapy. For this 75-year-old man with advanced-stage follicular lymphoma who is asymptomatic and does not have bulky disease, observation is an appropriate management strategy. There is no proof that earlier therapy for stage III to IV (nodes on both sides of the diaphragm [III]; noncontiguous extranodal involvement [IV]) follicular lymphoma provides benefit over deferring therapy until needed, provided the patient is comfortable with a strategy of surveillance. Patients with nonbulky follicular lymphoma who are not symptomatic may survive many years without the need for therapy. Spontaneous tumor lysis syndrome occurs commonly in patients with leukemia and Burkitt lymphoma and after treatment of bulky large B-cell lymphoma or advanced chronic lymphocytic leukemia. Tumor lysis syndrome results in hyperkalemia, hyperphosphatemia, hyperuricemia, hypocalcemia, and disseminated intravascular coagulation. Hyperuricemia can lead to urate nephropathy and acute kidney injury. Prevention of tumor lysis syndrome is accomplished with large-volume intravenous hydration with normal saline plus administration of allopurinol to limit hyperuricemia. Because this patient will not likely be receiving chemotherapy, there is no need for allopurinol to prevent tumor lysis syndrome. Even if he were to be treated, tumor lysis is not common in low-grade lymphomas in the absence of bulky disease or preexisting renal disease or hyperuricemia. Although follicular lymphomas are sensitive to radiation, and radiation therapy may be used for palliation of symptomatic bulky sites of disease or for cure in patients with localized (stage I to II) follicular lymphoma, this patient clearly has disseminated disease with diffuse lymphadenopathy, and systemic therapy would be appropriate if needed. Follicular lymphoma associated with symptomatic systemic disease requires multiagent therapy that traditionally includes rituximab plus cyclophosphamide, doxorubicin, vincristine, and prednisone (R-CHOP); rituximab plus cyclophosphamide, vincristine, and prednisone (R-CVP); or rituximab and bendamustine. This patient is asymptomatic and does not yet require systematic therapy.
A 61-year-old man is evaluated in the emergency department after he collapsed on a hot and humid day. He was playing in a marching band and had to stand in the sun for 2 hours while wearing a heavy uniform. No other medical information is available. On physical examination, temperature is 40 °C (104 °F), blood pressure is 90/45 mm Hg, pulse rate is 110/min, and respiration rate is 20/min. His face is flushed, he is somnolent, and although he is arousable, he is not coherent. There are no signs of trauma. His clothing is removed. Which of the following is the most appropriate treatment? Acetaminophen and a cooling blanket Continuous alcohol sponge bath with cooling fans Ice water immersion Intravenous dantrolene Sprayed water and cooling fans
Patients with nonexertional heat stroke should be treated with evaporative cooling to lower their core temperature to a safe level. This patient should be sprayed with water, and fans should be used to lower his body temperature to a safe level (usually 38.5 °C (101 °F) through evaporative cooling. Heat stroke occurs with high ambient temperature and humidity and is defined by the presence of a temperature greater than 40.0 °C (104.0 °F) and encephalopathy. It is often associated with hypotension, gastrointestinal distress, and weakness. Patients with advanced heat stroke exhibit shock, multiorgan failure, rhabdomyolysis, and myocardial ischemia. Exertional heat stroke typically occurs in healthy individuals undergoing vigorous physical activity in warm conditions. In contrast, most patients with nonexertional heat stroke are older than 70 years of age or have chronic medical conditions that impair thermal regulation. Medications and recreational drugs with anticholinergic, sympathomimetic, and diuretic effects, including alcohol, pose added risk. The primary treatment of nonexertional heat stroke is evaporative, external cooling. This involves removing all clothing and spraying the patient with a mist of lukewarm water while continuously blowing fans on the patient. Evaporative and convective cooling techniques are generally the safest and most effective. Acetaminophen and other centrally acting antipyretics are ineffective in the treatment of heat stroke. A cooling blanket could be used as an adjunct, but it is not as effective as evaporative cooling. Alcohol would evaporate and provide cooling as effective as that from applying water, but it would also be absorbed through the vasodilated skin and could lead to alcohol toxicity similar to that observed in patients who have ingested alcohol and is therefore contraindicated. Although ice water immersion is sometimes used in younger patients with exertional heat stroke to lower the body temperature rapidly, there is evidence for increased mortality when this method is used in older patients. Also, this patient's core temperature of 40 °C (104 °F) is not so severe that more aggressive measures need to be considered. The muscle relaxant dantrolene is ineffective in the treatment of heat stroke. It is used for malignant hyperthermia and sometimes for neuroleptic malignant syndrome, although this is an off-label application.
A 45-year-old man is evaluated for one episode of macroscopic hematuria. He currently does not see blood in his urine. He reports no flank pain and no associated trauma or exertion. He is a nonsmoker and takes no medications. Physical examination and vital signs are normal. Laboratory studies show a normal serum creatinine level; urinalysis shows 1+ blood, no protein, 10-15 isomorphic erythrocytes/hpf, 0-2 leukocytes/hpf, no nitrites, and no leukocyte esterase. Contrast-enhanced CT urogram shows no kidney stones, masses, or cysts. Which of the following is the most appropriate diagnostic test to perform next? Cystoscopy Kidney biopsy Kidney and renal vein Doppler ultrasonography Urine cytology
Patients with nonglomerular hematuria should be stratified as high, intermediate, or low risk for urothelial cancer using factors that include age, gender, tobacco use, extent of hematuria, exposure to urothelial carcinogens, or chronic irritative voiding symptoms; intermediate- or high-risk patients require imaging of the genitourinary tract and cystoscopy. Cystoscopy is the most appropriate diagnostic test to perform next in this patient. Hematuria is frequently encountered among adults in ambulatory care. For hematuria incidentally discovered on dipstick evaluation, clinicians should confirm the presence of hematuria with microscopic urinalysis that demonstrates ≥3 erythrocytes/hpf before initiating further evaluation in asymptomatic adults. Clinicians should also pursue evaluation of hematuria even if the patient is receiving antiplatelet or anticoagulant therapy. Macroscopic hematuria should prompt urology referral even if self-limited. Hematuria is most often of nonglomerular origin. If a nephrologic cause of hematuria is not suggested, hematuria may indicate a malignancy. Guidelines from the American Urological Association recommend that patients with hematuria be stratified as high, intermediate, or low risk for urothelial cancer, with subsequent management guided by that risk classification. This 45-year-old man who has no smoking history, no occupational exposure to urothelial carcinogens (such as aniline dye or cyclophosphamide), no chronic irritative voiding symptoms, and 10 to 15 erythrocytes/hpf on urinalysis would be classified as intermediate risk. This patient needs imaging of the genitourinary tract, in particular to rule out renal cell carcinoma, and cystoscopy. Although concurrent evaluation for nephrologic cause of hematuria may be appropriate if there is suspicion kidney disease is the source of hematuria, this patient has no evidence of glomerular disease (no proteinuria or increased serum creatinine) and negative imaging of the upper genitourinary tract. Kidney biopsy in a patient with hematuria is not indicated in the absence of evidence of glomerular disease or change in kidney function. This patient's normal kidney function and absence of dysmorphic erythrocytes, erythrocyte casts, or proteinuria rules out a significant glomerular process. Furthermore, this patient has no systemic symptoms to suggest systemic lupus erythematosus or other autoimmune disease that would lead to glomerular hematuria. His normal serum creatinine and lack of systemic symptoms make other kidney disease unlikely. Kidney ultrasonography will provide no additional information in this patient. Doppler is not indicated because the risk of renal vein thrombosis as a cause of hematuria in this patient is not suspected (normal kidney function, no flank pain, and no risk of thrombosis). Routine cytologic evaluation of urine is no longer recommended in the initial evaluation of asymptomatic microscopic hematuria, and urine markers approved by the FDA for bladder cancer detection are specifically not recommended for patients with hematuria.
A 62-year-old woman is evaluated for a 2-month history of exertional right leg pain. She has no associated chest pain, shortness of breath, or other symptoms. Medical history is significant for hypertension. Medications are lisinopril and chlorthalidone. On physical examination, vital signs are normal. The right dorsalis pedis and posterior tibialis pulses are faint, and the left dorsalis pedis and posterior tibialis pulses are slightly diminished. Laboratory studies: Total cholesterol 180 mg/dL (4.66 mmol/L) HDL cholesterol 54 mg/dL (1.40 mmol/L) LDL cholesterol 101 mg/dL (2.62 mmol/L) Triglycerides 125 mg/dL (1.41 mmol/L) The right ankle-brachial index is 0.75, and the left is 0.92. The patient's 10-year risk for atherosclerotic cardiovascular disease is 5.4% based on the American College of Cardiology/American Heart Association Pooled Cohort Equations. In addition to aspirin and dietary and exercise counseling, which of the following is the most appropriate treatment? Evolocumab Ezetimibe High-intensity atorvastatin Moderate-intensity pravastatin No additional therapy
Patients with peripheral artery disease should be treated with statin therapy. The most appropriate treatment is high-intensity atorvastatin. This patient with claudication has an ankle-brachial index of less than or equal to 0.90 on the right side, which is diagnostic for peripheral artery disease (PAD). PAD is considered a form of clinical atherosclerotic cardiovascular disease (ASCVD), and as such, this patient should be started on secondary prevention therapy. Additional forms of ASCVD include acute coronary syndrome, history of myocardial infarction, stable or unstable angina, coronary or other arterial revascularization, stroke, or transient ischemic attack. Patients aged 75 years or younger with ASCVD should be started on high-intensity statin therapy, unless a contraindication exists. Options for high-intensity statin therapy include atorvastatin, 40 mg/d or 80 mg/d, and rosuvastatin, 20 mg/d or 40 mg/d. Proprotein convertase subtilisin/kexin type 9 (PCSK9) inhibitors, such as evolocumab, dramatically decrease LDL cholesterol levels. PCSK9 inhibitors are currently approved as adjunctive therapy to diet and maximally tolerated lipid-lowering therapy (statin and ezetimibe) in patients with familial hypercholesterolemia who have not achieved adequate LDL cholesterol reduction. In patients with clinical ASCVD who are at very high risk (multiple ASCVD events, or one ASCVD event and multiple ASCVD risk factors) and have an LDL cholesterol level of 70 mg/dL (1.81 mmol/L) or higher or a non-HDL cholesterol level of 100 mg/dL (2.59 mmol/L) or higher despite maximally tolerated lipid-lowering therapy, it is reasonable to add a PCSK9 inhibitor following a clinician-patient discussion about the net benefit, safety, and cost. Treatment with evolocumab has been associated with a reduction in the composite outcome of cardiovascular death, myocardial infarction, stroke, hospitalization for unstable angina, and coronary revascularization but not cardiovascular mortality or all-cause mortality. Ezetimibe impairs intestinal cholesterol absorption, resulting in a significant reduction in LDL cholesterol levels. Several trials have shown clinical benefit when ezetimibe is added to statin therapy. However, this patient is not yet taking a statin, and ezetimibe is not appropriate as monotherapy. Moderate-intensity statin therapy with pravastatin is not the first choice for patients with clinical ASCVD because high-intensity statin therapy offers superior benefits in patients aged 75 years or younger. Patients older than 75 years can be considered for moderate-intensity statin therapy. Using the American College of Cardiology/American Heart Association risk estimator based on the Pooled Cohort Equations, the 10-year risk for ASCVD can be categorized as low (<5%), borderline (5% to <7.5%), intermediate (≥7.5% to <20%), or high (≥20%). All patients with borderline ASCVD risk and risk-enhancing factors should be engaged in a risk reduction discussion that includes the initiation of moderate-intensity statin therapy. However, this patient with PAD has established ASCVD, and high-intensity statin therapy is indicated.
A 42-year-old woman is evaluated for a 3-year history of palpitations and fatigue. She reports no chest pain, dizziness, near-syncope, or syncope. An exercise stress test and echocardiogram were normal when she was evaluated for palpitations 1 year ago. There is no family history of sudden cardiac death or heart failure. Her only medication is metoprolol. On physical examination, vital signs and the remainder of the examination are unremarkable. An electrocardiogram is shown. Ambulatory 24-hour electrocardiographic monitoring shows frequent monomorphic premature ventricular contractions (22% of all beats) and frequent ventricular bigeminy. An echocardiogram obtained 1 week ago showed mild to moderate global decreased left ventricular function with a left ventricular ejection fraction of 45%. Which of the following is the most appropriate management? Amiodarone Cardiac catheterization Cardiopulmonary exercise testing Catheter ablation of premature ventricular contractions
Patients with premature ventricular contraction-induced cardiomyopathy should be treated with catheter ablation. Catheter ablation is the most appropriate management of this patient with frequent symptomatic premature ventricular contractions (PVCs). PVCs are common and occur in up to 75% of healthy persons. In the absence of high-risk features (syncope, a family history of premature sudden cardiac death, structural heart disease), reassurance is often appropriate management. For patients with bothersome palpitations, the initial diagnostic test is electrocardiography (ECG). If the diagnosis is not established, 24- to 48-hour ambulatory ECG monitoring is used to diagnose and quantify the frequency of PVCs and determine whether they are monomorphic or polymorphic. Symptomatic or frequent monomorphic PVCs (>10,000 PVCs in 24 hours or >10% of all beats) require further evaluation and treatment. Exercise stress testing can be used to evaluate the patient for ischemia and assess the response of PVCs to exercise. In addition, patients with frequent PVCs or polymorphic PVCs should undergo echocardiography or other cardiovascular imaging (such as cardiac magnetic resonance imaging) to evaluate for the presence of structural heart disease. Up to one third of patients with frequent PVCs can develop PVC-induced cardiomyopathy and progressive left ventricular dysfunction, as documented in this patient. First-line therapy for symptomatic or frequent PVCs includes β-blockers or calcium channel blockers. Patients with continued frequent PVCs despite medical therapy or those who develop left ventricular dysfunction should undergo catheter ablation, which resolves most cases of PVC-induced left ventricular dysfunction. Amiodarone is usually effective in suppressing PVCs; however, it carries a high risk for adverse effects and end-organ toxicities. Therefore, amiodarone is best avoided in younger patients, such as this one. This patient had a structurally normal heart and no evidence of ischemia on studies performed 1 year ago, and she has developed no new symptoms. Therefore, cardiac catheterization is not indicated. Cardiopulmonary exercise testing includes assessment of respiratory gas exchange during treadmill or bicycle exercise for a more detailed assessment of functional capacity and differentiation between potential causes of exercise limitation (cardiac, pulmonary, or deconditioning causes versus volitional causes). It would not be the most appropriate next choice in a patient with a known decrease in left ventricular ejection fraction and frequent PVCs.
A 65-year-old woman is evaluated for hypercalcemia that was incidentally discovered on routine blood testing for a new life insurance policy. She has no symptoms. Medical history is unremarkable. Her only medication is calcium carbonate taken as needed for occasional heartburn. On physical examination, vital signs are normal. Height is unchanged from prior measurements. The remainder of her examination is normal. Laboratory studies: Estimated glomerular filtration rate Greater than 60 mL/min/1.73 m2 Calcium 10.6 mg/dL (2.6 mmol/L) Phosphorus 3.1 mg/dL (1.0 mmol/L) Parathyroid hormone 72 pg/mL (72 ng/L) 25-Hydroxyvitamin D 35 ng/mL (87.4 nmol/L) 24-Hour urine calcium 240 mg/24 h (6 mmol/24 h) Kidney-urinary-bladder radiograph is negative for kidney stones. Dual-energy x-ray absorptiometry (DEXA) bone mineral density scan shows femur neck T-score of -1.4, lumbar spine T-score of -1.4, and one-third radius T-score of -1.7. Which of the following is the most appropriate management? Parathyroid sestamibi scan Reevaluate in 6 months Start alendronate Start cinacalcet
Patients with primary hyperparathyroidism who do not undergo surgery require monitoring of serum calcium and creatinine every 6 to 12 months and bone mineral density of the lumbar spine, hip, and distal radius every 2 years. The most appropriate management for this patient is to reassess the patient in 6 to 12 months. Guideline-recommended indications for parathyroidectomy include increase in serum calcium level ≥1 mg/dL (0.25 mmol/L) above upper limit of normal; creatinine clearance <60 mL/min, 24-hour urine calcium >400 mg/day (>10 mmol/day), or increased stone risk by biochemical stone risk analysis; presence of nephrolithiasis or nephrocalcinosis by radiograph, ultrasound, or CT; T-score (on DEXA scan) of less than or equal to −2.5 at any site or evidence of vertebral fracture; and age younger than 50 years. Parathyroidectomy is also indicated in patients in whom medical surveillance is neither desired nor possible, and those with complications of hyperparathyroidism including significant bone, kidney, gastrointestinal, or neuromuscular symptoms. Patients without indications for parathyroidectomy require periodic reassessment that includes serum calcium and creatinine every 6 to 12 months and bone mineral density of the lumbar spine, hip, and distal radius every 2 years. Although imaging tests such as parathyroid sestamibi scan may be performed as part of evaluation of primary hyperparathyroidism, it is most appropriate once surgery is indicated. The results of imaging do not influence the management of nonsurgical patients. Primary hyperparathyroidism is associated with increased bone turnover and decreased bone strength. Although alendronate suppresses bone resorption and improves bone mineral density at the lumbar spine in patients with primary hyperparathyroidism, it has not been shown to reduce fracture risk or to reduce serum calcium levels. Patients at high risk of fracture (T-score ≤−2.5 and/or prevalent fragility fracture) at presentation or during monitoring should undergo parathyroidectomy. Cinacalcet is indicated to treat symptomatic, severe hypercalcemia in adults with primary hyperparathyroidism for whom parathyroidectomy cannot be performed. Although it may be used chronically, cinacalcet is more commonly used until surgery is feasible.
A 62-year-old woman is evaluated for increasing exertional dyspnea during the past 6 months. She is a former smoker who was diagnosed with severe COPD 3 years ago (FEV1 is 35% of predicted). For the past 18 months, she has used tiotropium and salmeterol; inhaled fluticasone was added 4 months ago, but without any perceived benefit. She takes no other medications. On physical examination, blood pressure is 130/79 mm Hg, pulse rate is 88/min, and respiration rate is 18/min. Oxygen saturation is 89% breathing ambient air. Lung examination demonstrates diminished breath sounds. A prominent pulmonic sound is heard on cardiac examination. Arterial blood gas studies breathing ambient air show a pH of 7.41, a PCO2 of 43 mm Hg (5.7 kPa), and a PO2 of 55 mm Hg (7.3 kPa). Chest radiograph reveals hyperinflation. Echocardiography shows an estimated right ventricular systolic pressure of 58 mm Hg. Polysomnography showed an apnea-hypopnea index of 2 and a mean oxygen saturation of 87%. Which of the following is the most appropriate treatment? Bilevel positive airway pressure Prednisone Sildenafil Supplemental oxygen
Patients with pulmonary hypertension secondary to lung disease and associated hypoxemia should be treated with supplemental oxygen. The most appropriate treatment is supplemental oxygen. The clinical assessment and echocardiographic findings are consistent with pulmonary hypertension in the setting of advanced COPD (Group 3 pulmonary hypertension [PH]). The mainstay of treatment of Group 3 PH targets the underlying lung disease. This patient is on maximal inhaler therapy for COPD. Hypoxemia during daytime rest and, in the setting of cor pulmonale or secondary polycythemia, hypoxemia during sleep, is an indication for supplemental oxygen, which has proved benefit in pulmonary hemodynamics and survival in this population. Bilevel positive airway pressure is indicated in patients with hypercapnia in the setting of COPD. Furthermore, in patients with overlap of COPD and sleep disordered breathing, continuous positive airway pressure has been shown to increase quality of life and prolong survival. The arterial blood gas study shows this patient to be normocapnic, and polysomnography demonstrates hypoxemia but no sleep apnea. Therefore, positive airway pressure therapy is not indicated. A short course of prednisone is indicated in acute exacerbations of COPD, which typically present with acute dyspnea, cough, and sputum production. The more insidious course of this patient's symptoms is not consistent with an acute exacerbation of COPD, and there is no role for systemic glucocorticoids in patients with PH due to COPD. Therapy with a vasodilator such as sildenafil is generally not indicated in patients with pulmonary hypertension related to lung disease or hypoxemia. Such drugs may cause harm by worsening ventilation-perfusion matching and further impairing gas exchange.
A 71-year-old man is evaluated during a follow-up appointment for stage II rectal cancer that was resected 3 years ago. He takes no medications. On physical examination, vital signs are normal. BMI is 23. The liver edge is palpable 3 cm below the xiphisternum. A surveillance CT scan shows three new hypodense lesions in the left lobe of the liver consistent with metastatic disease. The largest lesion is 4 cm. No other abnormal findings are found on a contrast-enhanced CT scan of the chest, abdomen, and pelvis. Which of the following is the most appropriate management? Hepatic arterial embolization Needle biopsy of the most accessible liver lesion Palliative chemotherapy Resection of all lesions
Patients with recurrent colorectal cancer in a single organ in an oligometastatic pattern are candidates for surgery with curative intent. The most appropriate management is resection of all lesions. The role of postoperative surveillance in patients with colorectal cancer is to identify surgically curable recurrence, such as oligometastatic liver disease or lung metastases. The liver is the primary metastatic site for patients with colorectal cancer. About one third of patients with metastatic disease will have it confined to the liver and will be amenable to surgical resection and cure, with or without adjuvant chemotherapy. The criteria for surgical resection of hepatic metastases have expanded from the "three lesions or less" rule. Most patients with good performance status can be considered for surgical resection unless they have one of three conditions: tumor involvement of the common artery or portal vein or common bile duct; more than 70% liver involvement, more than six involved segments, or involvement of all three hepatic veins; and a predicted inadequate hepatic reserve after resection. This patient has recurrent colorectal cancer in a single organ in an oligometastatic pattern. As such, he is a candidate for surgery with curative intent. Hepatic arterial embolization is not routinely used to treat colorectal cancer because colorectal tumors tend not to be very vascular. Further, it is a palliative, noncurative intervention. It is more appropriately considered in the management of unresectable neuroendocrine tumors or primary liver cancers. A needle biopsy should not be done because it will not change management. The clinical presentation in this patient is compelling enough to diagnose recurrent colorectal cancer; a negative needle biopsy specimen would be assumed to be a false negative and would not alter management. Surgical resection would be the appropriate intervention regardless of the results of a biopsy. Because this cancer is potentially curable, palliative chemotherapy should not be considered.
A 29-year-old woman is evaluated during a routine office visit. She was recently diagnosed with rheumatoid arthritis, for which she is taking naproxen and methotrexate. She is otherwise well. She does not smoke, drinks one alcoholic beverage daily, and engages in strenuous physical activity 5 days per week for 30 minutes each session. Her mother and father are first-generation Japanese immigrants, both in excellent health. On physical examination, vital signs are normal. Cardiovascular examination is normal. There is evidence of synovitis involving the second, third, and fourth metacarpophalangeal joints bilaterally. Her most recent lipid profile showed a total cholesterol level of 128 mg/dL (3.32 mmol/L) and a calculated LDL cholesterol level of 72 mg/dL (1.86 mmol/L). Which of the following is the most significant risk factor for future cardiovascular disease? Activity level Alcohol consumption Ethnicity Rheumatoid arthritis
Patients with rheumatoid arthritis have a 1.5- to 2-fold elevated risk for coronary artery disease compared with the general population. This patient's most significant risk factor for cardiovascular disease (CVD) is the presence of rheumatoid arthritis. Patients with systemic inflammatory conditions, such as systemic lupus erythematosus and rheumatoid arthritis, have an increased risk for CVD. Patients with rheumatoid arthritis have a 1.5- to 2-fold elevated risk for coronary artery disease compared with the general population. Most deaths in patients with SLE and nearly 40% of deaths in patients with rheumatoid arthritis are cardiovascular and, in particular, heart failure related. The risk for CVD increases with the duration of the underlying inflammatory condition. In patients with rheumatoid arthritis, the risk for CVD increases from two times to three times that of the general population after 10 years' duration of rheumatoid arthritis. The increased atherosclerotic burden is likely a result of both the inflammatory process of the systemic disease, including a prothrombotic state, and the presence of traditional cardiovascular risk factors. The risk for future atherosclerotic CVD is typically calculated by using the American College of Cardiology/American Heart Association CVD risk calculator based on the Pooled Cohort Equations; however, this calculator underestimates risk in patients with nontraditional risk factors, as in this patient with rheumatoid arthritis. Sedentary lifestyle, poor diet, and obesity contribute to increased cardiovascular risk and increased risk for diabetes mellitus. Moderate-intensity exercise for 30 minutes or longer on 5 to 7 days per week is recommended for nearly all persons and is being achieved by this patient. Regular moderate alcohol consumption (one to two drinks daily for men, one drink daily for women) has been associated with a decreased incidence of CVD. Because of the known deleterious effects of drinking, the American Heart Association does not recommend that nondrinking patients consume moderate amounts of alcohol as a measure to decrease the risk for CVD. This patient has one drink per day, which is likely beneficial in reducing her CVD risk. The prevalence of CVD and CVD risk factors in the United States varies by ethnicity. Hawaiians and Pacific Islanders have the highest rate of heart disease (19.1%), followed by American Indians and Alaska Natives (13.7%), non-Hispanic White persons (11.1%), Black persons (10.3%), Hispanic and Latino persons (7.8%), and Asian persons (6.0%). This patient's ethnicity places her in the lowest risk category for CVD.
A 72-year-old woman is evaluated for a 3-month history of large-volume, watery, nonbloody diarrhea with mucus, which occurs even when she is fasting. She reports no abdominal pain or flushing but has experienced an unintentional 3-kg (6.6-lb) weight loss. She has never had a colonoscopy. On physical examination, blood pressure is 100/50 mm Hg and pulse rate is 95/min; other vital signs and examination findings are normal. Laboratory studies show a serum sodium level of 130 mEq/L (130 mmol/L) and a serum potassium level of 3.3 mEq/L (3.3 mmol/L). Hemoglobin and serum creatinine levels are within normal limits. Which of the following types of diarrhea is most compatible with the patient's findings? Infectious Inflammatory Osmotic Secretory
Patients with secretory diarrhea may pass liters of stool daily, causing severe dehydration and electrolyte disturbances, with persistent stooling despite fasting. Secretory diarrhea is the diagnosis most compatible with the patient's presentation. In the evaluation of chronic diarrhea, it is often useful to categorize the condition as secretory or osmotic, as well as to determine whether the cause is due to chronic infection or an inflammatory condition, which will guide further evaluation and treatment. Secretory and osmotic diarrhea can often be distinguished by clinical history. Patients with secretory diarrhea may pass liters of stool daily, causing severe dehydration and electrolyte disturbances, with persistent stooling despite fasting. Patients with osmotic diarrhea often have stool volumes of less than 1 L/d and have cessation of stooling when they are fasting. This patient's large-volume, watery diarrhea that persists despite fasting is compatible with secretory diarrhea. Another clue to this patient's diagnosis is hypokalemia. Hypokalemia from lower-gastrointestinal losses is most common when the losses occur over a prolonged period. Fecal electrolytes can be used to calculate the fecal osmotic gap: 290 - (2 × [stool sodium + stool potassium]) An osmotic gap of less than 50 mOsm/kg suggests secretory diarrhea, and a gap greater than 75 mOsm/kg suggests osmotic diarrhea. Examples of conditions that can cause a chronic secretory diarrhea include medications (colchicine, NSAIDs), hormone-producing tumors (carcinoid, gastrinoma, VIPoma), small intestinal bacterial overgrowth, bile acid malabsorption (short-bowel syndrome), and villous adenoma. Additional testing may be warranted in patients with secretory diarrhea. Patients found to have a small-bowel tumor in the setting of diarrhea as well as symptoms such as flushing should be considered for additional testing, including radioimmunoassays for peptides and/or 24-hour urine 5-hydroxyindoleacetic acid measurement for carcinoid tumors. Colonoscopy can be used to evaluate for villous adenomas and is also indicated if the patient is not up-to-date for colon cancer screening. Infections leading to chronic diarrhea can be caused by parasites such as Giardia. Giardia leads to a malabsorptive condition; it would not present with large-volume diarrhea that persists despite fasting. Inflammatory causes of chronic diarrhea include inflammatory bowel diseases (ulcerative colitis and Crohn disease) and microscopic colitis. Patients with inflammatory causes of chronic diarrhea present with bloody stools (except in microscopic colitis), nocturnal symptoms, and sometimes anemia. This patient is less likely to have an inflammatory cause for her chronic diarrhea given the absence of these signs and symptoms. Blood or leukocytes in the stool suggest an inflammatory cause. Causes of chronic osmotic diarrhea include medications (laxatives), undigested sugars (lactose), celiac disease, and fat maldigestion or absorption leading to steatorrhea (chronic pancreatitis). When laxative abuse is suspected, a stool or urine laxative screen can aid in the diagnosis. Celiac disease can be evaluated with serology testing and small intestinal biopsy. A positive 72-hour stool collection for fecal fat confirms steatorrhea; a random fecal fat assessment may be helpful if a timed collection is not possible.
A 62-year-old man was admitted 12 hours ago with severe acute respiratory distress syndrome (ARDS). Overnight, he developed fever and increasing respiratory distress, was intubated, and has had rapidly increasing ventilator requirements. Current ventilator settings are in the volume-controlled continuous mandatory ventilation mode, and he is receiving appropriate low tidal volume ventilation. Current medications are fentanyl, midazolam, heparin, piperacillin/tazobactam, and norepinephrine. On physical examination, temperature is 38.4 °C (101.2 °F), blood pressure is 89/45 mm Hg, pulse rate is 110/min, and his spontaneous respiration rate is 35/min with a positive end-expiratory pressure of 16 cm H2O and an FIO2 of 1.0. Arterial blood gases: pH 7.3 Pco2 51 mm Hg (6.8 kPa) PO2 55 mm Hg (7.3 kPa) Bicarbonate 24 mEq/L (24 mmol/L) Chest radiograph shows diffuse, bilateral infiltrates throughout the lung fields. Which of the following is the most appropriate management? Perform a recruitment maneuver Start inhaled nitric oxide Transition the patient to high-frequency oscillator ventilation Ventilate the patient in the prone position
Patients with severe acute respiratory distress syndrome have a demonstrated mortality benefit from low tidal volume ventilation in the prone position. This patient should be ventilated in the prone position. In patients with acute respiratory distress syndrome (ARDS) low tidal volume ventilation (6-8 mL/kg ideal body weight) is optimal and is associated with significantly better outcomes than conventional, higher tidal volume ventilation (10-12 mL/kg). In patients with severe ARDS, several adjunctive therapies have been studied, but few have demonstrated additional improved outcomes beyond low tidal volume ventilation. However, in 2013, the PROSEVA trial evaluated patients with ARDS who had an arterial PO2/FIO2 ratio of less than 150 (defining severe ARDS) despite significant ventilator support. Patients were randomized to conventional therapy or low tidal volume ventilation plus prone positioning within 36 hours of developing ARDS and respiratory failure. The results suggested a 16.2% reduction in 28-day all-cause mortality in the prone-position group. The 2017 American Thoracic Society, European Society of Intensive Care Medicine, and Society of Critical Care Medicine (ATS/ESICM/SCCM) guideline makes a strong recommendation that all patients with ARDS receive ventilation with lower tidal volumes (4-8 mL/kg of predicted body weight). Patients with severe ARDS should be also ventilated in the prone position for at least 12 hours per day. Proning should be considered standard management for patients with severe ARDS, not a form of "rescue" or "salvage" therapy. Recruitment is the application of a high level of continuous positive airway pressure to open up collapsed alveoli (for example, continuous pressure to 35 cm H2O for 40 seconds). The ATS/ESICM/SCCM guideline provides a conditional recommendation for recruitment maneuvers based on low to moderate confidence in the small to moderate magnitude of its effect on mortality. The guideline cautions that recruitment should not be used in patients with preexisting hypovolemia or shock due to hemodynamic deterioration, which can occur during the maneuver. This patient is hypotensive and, therefore, not a candidate for recruitment maneuvers. Nitric oxide (NO) selectively dilates the pulmonary vasculature when administered by inhalation. Its use in acute hypoxemic respiratory failure is based on the rationale that inhaled NO may improve ventilation-perfusion mismatch. Studies of inhaled NO in patients with severe ARDS have demonstrated temporary improvement in oxygenation but no improvement in survival. In 2013 two large, prospective randomized trials demonstrated that compared to low tidal volume ventilation, high-frequency oscillator ventilation was either no different or harmful to patients with ARDS. In light of this, the ATS/ESICM/SCCM guideline strongly recommends against routine use of high-frequency oscillatory ventilation.
A 24-year-old man is admitted to the hospital for fever, chest pain, shortness of breath, and cough of 3 days' duration. He is homozygous for hemoglobin S (Hb SS). Medications are folate and hydroxyurea. On physical examination, temperature is 38.9 °C (102 °F), blood pressure is 100/70 mm Hg, pulse rate is 110/min, and respiration rate is 22/min. Oxygen saturation is 90% breathing 40% oxygen. He appears ill. Cardiopulmonary examination reveals a grade 2/6 systolic flow murmur and crackles in the lower lung fields bilaterally. The remainder of the examination is unremarkable. Laboratory studies show a hemoglobin level of 7.8 g/dL (78 g/L) and a leukocyte count of 22,000/µL (22 × 109/L). Bilateral lower lobe infiltrates are seen on a chest radiograph. The patient receives intravenous saline, prophylactic doses of low-molecular-weight heparin, and ceftriaxone and levofloxacin. Incentive spirometry is begun. Six hours later, he has worsening hypoxia and respiratory distress requiring intubation and mechanical ventilation. A repeat chest radiograph shows worsening infiltrates. Which of the following is the most appropriate management? CT pulmonary angiography Exchange transfusion Intravenous amphotericin Intravenous full-dose unfractionated heparin Lower extremity Doppler ultrasonography
Patients with sickle cell disease and acute chest syndrome with respiratory distress requiring mechanical ventilation should receive emergent exchange transfusion. The most appropriate management is to perform immediate exchange transfusion. The patient has acute chest syndrome (ACS), defined clinically by fever, respiratory findings that include tachypnea and hypoxia, and evolving infiltrates on chest radiograph. ACS is one of the most common complications of sickle cell disease and is the most common cause of death. ACS may occur secondary to infection, fat or bone marrow embolism, in situ thrombosis, or any combination of these. ACS remains a clinical diagnosis. Patients should receive supplemental oxygen, incentive spirometry, analgesics if they have concomitant pain, and antibiotics to treat common causes of pneumonia. The severity of ACS determines the need for transfusion and type of transfusion (simple versus exchange transfusion). Mild episodes require no transfusion, moderate episodes are managed with simple or exchange transfusion, and severe episodes require exchange transfusion. The target hemoglobin level is 10 g/dL (100 g/L). Most experts agree that patients requiring mechanical ventilation or experiencing shock or other signs of multiorgan dysfunction should receive exchange transfusion. Venous thromboembolism is not a common precipitant of ACS, and filling defects in the pulmonary vasculature are more likely to be caused by bone marrow or fat emboli than by venous thromboembolism. Without evidence for venous thrombosis of the lower extremity or other risk factors, CT pulmonary angiography should not be obtained, and therapeutic anticoagulation with heparin should not be started. Similarly, duplex Doppler imaging of the lower extremities should not be performed in the absence of clinical features such as unilateral leg swelling or edema and dilatation of the venous pattern that suggest acute venous thrombosis. Acute infection is a common precipitant of ACS, and antibiotics to treat common causes of pneumonia should be routinely administered. Although a broad range of bacterial and viral agents have been linked to ACS, infection with fungal organisms is rare, and amphotericin is not indicated.
A 22-year-old woman is seen for a routine prenatal evaluation; she is 10 weeks pregnant. This is her second pregnancy; the first pregnancy was uncomplicated. Medical history is notable for sickle cell disease requiring one to two hospitalizations per year for painful events. She has no history of stroke or acute chest syndrome. Her only medication is a folic acid supplement. On physical examination, vital signs are normal. Cardiac examination reveals a grade 2/6 systolic flow murmur. She has a gravid uterus. Laboratory studies show a hemoglobin level of 6.4 g/dL (64 g/L; baseline, 5-7 g/dL [50-70 g/L]), leukocyte count of 11,500/µL (11.5 × 109/L), and platelet count of 279,000/µL (279 × 109/L). Which of the following is the most appropriate treatment? Clopidogrel Exchange transfusion throughout pregnancy Hydroxyurea Simple transfusion throughout pregnancy Expectant care
Patients with sickle cell disease and uncomplicated pregnancy should be closely monitored and treatment withheld until symptoms appear or some measurable parameter changes. This patient requires no treatment other than expectant care. She should be closely monitored and treatment should be withheld until symptoms appear or some measurable parameter changes. Sickle cell disease (SCD) is associated with maternal morbidity and mortality from stroke, vaso-occlusive crisis, and acute chest syndrome. Furthermore, the risk of eclampsia is higher in pregnant women with SCD. Fetal complications are also more common in patients with SCD, including low birth weight, spontaneous abortion, early delivery, and growth retardation. No routine interventions have been proven to decrease these complications, so close follow-up by hematologic and obstetric specialists and prompt management of evolving problems is important. Pregnancy outcomes are difficult to predict in pregnant patients with SCD, but one of the best indicators is the outcome from a previous pregnancy, which is encouraging in this patient. Antiplatelet drugs, such as clopidogrel, have been associated with increased risk of bleeding and ablatio placenta without any clear benefit in pregnant patients with SCD. Although the role of transfusion in the management of pregnant patients with SCD lacks consensus, no clear benefit has been seen with either simple or exchange transfusion in patients with uncomplicated disease or pregnancy. However, pregnant patients with SCD who have worsening symptomatic anemia should undergo transfusion. Those with past or new severe obstetric or fetal complications, those with twins, or those with chronic organ dysfunction may also benefit from transfusion. The decision to transfuse must be balanced against the risks of transfusion, including alloimmunization and delayed transfusion reaction, as well as iron and volume overload. Because this patient has an uncomplicated pregnancy, exchange or simple transfusion is not indicated. Hydroxyurea therapy results in decreased mortality in SCD and is indicated in patients with recurrent painful episodes, acute chest syndrome, and symptomatic anemia. However, because of its potential teratogenicity, hydroxyurea should not be administered during pregnancy. Proper contraception before hydroxyurea initiation should be discussed with the nonpregnant patient. This patient has no indication for hydroxyurea treatment.
A 53-year-old man is evaluated after a hemicolectomy for a well-differentiated adenocarcinoma of the descending colon. Final pathologic findings showed a well-differentiated adenocarcinoma penetrating into the serosa (T3). Margins of resection were free of tumor. No lymphovascular or perineural invasion was seen. Twenty-seven lymph nodes were identified, and all were free of cancer. He takes no medications. CT scans of the chest, abdomen, and pelvis showed no evidence of metastatic disease. On physical examination, vital signs are normal. All other examination findings are normal. The postoperative carcinoembryonic antigen (CEA) level is normal. Which of the following is the most appropriate management? FOLFOX (parenteral 5-fluorouracil, leucovorin, and oxaliplatin) Oral capecitabine Radiation therapy No further treatment
Patients with stage II colon cancer without high-risk features are unlikely to benefit from adjuvant treatment. No further treatment is needed. This patient has stage II colorectal cancer (invades full thickness of the bowel [T3]), and lymph nodes are not involved [N0]). Patients with stage II colon cancer do not have a clear survival advantage with administration of adjuvant chemotherapy; consequently, surgery alone is the acceptable standard practice for most patients. Exceptions are patients with stage II colon cancer with characteristics associated with a high risk for recurrence (T4 primary tumor [invasion into adjacent structures or through the peritoneum], lymphovascular invasion, inadequate lymph node sampling [fewer than 12 lymph nodes examined], poorly differentiated histology, elevated postoperative carcinoembryonic antigen (CEA), or clinical perforation or obstruction). In these patients, the prognosis is similar to that of patients with stage III disease, and adjuvant chemotherapy may be appropriate. Exposing this patient to the toxicity of chemotherapy is not warranted, as data do not support a survival benefit in such patients. For patients with stage II disease and a high risk for recurrence, treatment with 5-fluororuracil/leucovorin or capecitabine may be appropriate. Patients with stage II disease with a substantial number of high-risk features may at times be considered for oxaliplatin-containing chemotherapy, such as parenteral 5-fluorouracil, leucovorin, and oxaliplatin (FOLFOX). Patients with stage III disease (lymph node metastases) are routinely treated with oxaliplatin-containing regimens, such as FOLFOX, after resection. The combination of capecitabine plus intravenous oxaliplatin is also an acceptable regimen for adjuvant treatment of patients with stage III colon cancer. Because local recurrence of colon cancer rarely develops and because it can be difficult to isolate the small bowel from the radiation field, radiation therapy, either alone or in combination with chemotherapy, does not have a role in the routine management of patients with stage III colon cancer. In addition, radiation to the small bowel may cause substantial toxicity.
A 63-year-old man is evaluated for increasing fatigue and exertional dyspnea over the past 5 months. Medical history is notable for diabetes mellitus, hypertension, and chronic kidney disease. Medications are insulin glargine, metformin, enalapril, aspirin, and atorvastatin. On physical examination, vital signs are normal. Pallor is noted. Examination findings are otherwise normal. Laboratory studies show a hemoglobin level of 9.5 g/dL (90 g/L) (11 g/dL [110 g/L] 1 year ago), reticulocyte count of 1% of erythrocytes, and serum creatinine level of 2.0 mg/dL (176.8 µmol/L) (1.6 mg/dL [141.4 µmol/L] 1 year ago). Serum haptoglobin, iron, ferritin, and folate levels; total iron-binding capacity; vitamin B12 level; and liver chemistry studies are normal. The stool guaiac test result is negative for occult blood. Which of the following is the most appropriate next step in management? Darbepoetin administration for a target hemoglobin level of 11 g/dL (110 g/L) Darbepoetin administration for a target hemoglobin level of 14 g/dL (140 g/L) Oral iron supplementation and repeat hemoglobin level in 1 month Parenteral iron supplementation and repeat hemoglobin level in 1 month
Patients with symptomatic anemia of chronic kidney disease may be treated with erythropoiesis-stimulating agents to reduce transfusion requirements with a target hemoglobin level of 11 to 12 g/dL (110-120 g/L) to avoid increased risk of adverse cardiovascular events. Darbepoetin should be administered to achieve a hemoglobin level of 11 to 12 g/dL (110-120 g/L). Erythropoiesis-stimulating agents (ESAs) such as erythropoietin and darbepoetin have dramatically reduced the need for transfusion in patients with the anemia of chronic kidney disease (CKD) but have little impact on quality of life or mortality. This patient has symptomatic anemia of CKD, which is determined by ruling out other causes of anemia in patients with reduced kidney function. Although symptomatic anemia occurs commonly in patients with advanced CKD who are receiving hemodialysis, it may also occur in patients with stages III and IV CKD. Regardless of stage, patients with symptomatic anemia of CKD will benefit from ESAs by reducing their transfusion requirements. However, a "normal" hemoglobin threshold is not the preferred target level for patients taking ESAs. Numerous studies have shown that ESAs increase adverse cardiovascular outcomes when the hemoglobin level exceeds 11 to 12 g/dL (110-120 g/L). Additionally, ESAs can cause hypertension and thrombosis. Iron stores must be normal for ESAs to be effective. Patients undergoing dialysis are often treated with additional parenteral iron along with ESAs even if they are not overtly iron deficient. Although supplemental iron with darbepoetin may be required in this patient, iron therapy alone, either by oral or parenteral routes of administration, will not correct anemia in this patient who has no laboratory findings to support the diagnosis of iron deficiency.
A 52-year-old man is evaluated after screening colonoscopy revealed three polyps in the descending colon, measuring 5 mm, 8 mm, and 3 mm in size. Colonoscopy preparation was excellent, and the procedure was complete to the cecum. The polyps were completely excised, and pathology showed all three to be tubular adenomas. The patient has no personal or family history of colorectal cancer. He reports no symptoms and takes no medication. All physical examination findings, including vital signs, are normal. Based on his colonoscopy findings, when should this patient next undergo surveillance colonoscopy? 1 year 3 years 5 years 10 years
Patients with three or more adenomas on screening colonoscopy should undergo surveillance colonoscopy in 3 years. This patient should undergo surveillance colonoscopy in 3 years. Current recommendations emphasize that an adequate bowel preparation and a high-quality colonoscopy that reaches and examines the cecum are required for an adequate screening examination. Screening colonoscopy studies demonstrate that polyps are detected in approximately 60% of average-risk individuals. The prevalence of polyps on average-risk screening colonoscopy is 22% to 25% for adenomas, 12% for hyperplastic polyps, and 0.6% for sessile serrated polyps. The degree of dysplasia in a polyp is reported as high or low. Adenomatous polyps are neoplastic lesions and, therefore, have malignant potential; most colorectal cancers arise from adenomatous polyps. Adenomatous polyps are further defined by their glandular architecture: tubular, villous, or a combination of both. The most common pattern is tubular, followed by tubulovillous, with the least common pattern being villous. After neoplastic polyps (adenomas, sessile serrated polyps, or traditional serrated adenomas) are completely resected, postpolypectomy colonoscopy should be performed; the surveillance interval depends on the size, number, and pathology of the polyp. This patient was found to have three tubular adenomas smaller than 1 cm in size. The presence of three or more adenomas, any adenoma greater than or equal to 1 cm in size, or any adenoma with villous features or high-grade dysplasia has been associated with increased risk for metachronous neoplasia (multiple primary tumors developing at different time intervals), warranting a 3-year surveillance interval. Guidelines recommend screening average-risk individuals beginning at age 50 years. Repeat colonoscopy is based on initial findings. Individuals with no adenomas should have a repeat colonoscopy in 10 years. Individuals who have two or fewer tubular adenomas are also considered low-risk for colon cancer and should repeat screening in 5 to 10 years. Appropriate follow-up recommendations after screening colonoscopy, including avoidance of inappropriate use of colonoscopy after removal of adenomatous polyps, are core gastroenterology quality measures. High-risk findings include more than 10 polyps on baseline colonoscopic examination, which should prompt consideration for a polyposis syndrome. These individuals should have a follow-up colonoscopy in less than 3 years, depending on polyp burden and polyposis syndrome. Any patient who has a polyp removed piecemeal should return within 3 to 6 months for repeat colonoscopy, especially if there is any concern for incomplete polyp removal.
A 55-year-old man is evaluated at a follow-up appointment 4 days after being diagnosed with acute sigmoid diverticulitis. He is on day 4 of oral antibiotics and reports that his pain is almost gone. He has no other symptoms and has never had a colonoscopy. He takes no other medication. His vital signs are normal. Abdominal examination is positive for minimal tenderness on deep palpation in the left lower quadrant without rebound or guarding. The CT scan of the abdomen from the emergency department shows mild sigmoid diverticulitis with no abscess. When should this patient undergo colonoscopy? Now In 1 to 2 months In 6 months In 12 months
Patients with uncomplicated diverticulitis should undergo colonoscopy 1 to 2 months after the episode of acute diverticulitis, when colonic inflammation has resolved. This patient should undergo colonoscopy in 1 to 2 months following treatment of uncomplicated diverticulitis. According to the American Gastroenterological Association Institute Technical Review on the Management of Acute Diverticulitis, patients with a recent episode of acute diverticulitis should undergo colonoscopy 4 to 8 weeks after an episode of acute diverticulitis. Follow-up colonoscopy may identify a few cases of colorectal carcinoma. The risk for perforation after a case of diverticulitis is uncertain. The technical review estimated that 1 in 67 patients with confirmed acute diverticulitis would have a misdiagnosed colorectal cancer found on follow-up colonoscopy. Almost all of the misdiagnosed colorectal cancers included in the technical review were located in the area of diverticulitis. Assessing the patient for underlying inflammatory bowel disease is another reason for performing follow-up colonoscopy. Colonoscopy during the acute phase of illness is contraindicated because acute diverticulitis causes acute colonic inflammation, which may increase the risk for perforation. The risk for perforation has been reported to be as high as 0.3%, but it is generally thought to be less than 0.1%, based on data from large populations undergoing screening colonoscopy. If the results of colonoscopy performed within the past 12 to 24 months are negative, then repeat colonoscopy is not needed. Delaying colonoscopy beyond 1 to 2 months places the patient at risk for a missed diagnosis that could be important and time sensitive, such as a diagnosis of colon cancer.
A 64-year-old woman is evaluated in the emergency department 4 hours after the abrupt onset of sharp, tearing chest and back pain. Medical history is significant for hyperlipidemia. Her only medication is atorvastatin. On physical examination, temperature is 36.8 °C (98.2 °F), blood pressure is 173/99 mm Hg, and pulse rate is 90/min. Blood pressure measurements in both arms are equal. The remainder of the physical examination is unremarkable. CT angiography shows a descending thoracic aortic aneurysm with a maximal diameter of 6.8 cm and aortic dissection originating just distal to the left subclavian artery and extending to just below the diaphragm; there is no involvement of the renal arteries. Which of the following is the most appropriate initial management? Immediate endovascular stenting Immediate open surgical repair Medical therapy Repeat CT angiography in 12 hours
Patients with uncomplicated type B aortic dissection may be initially treated with medical therapy, including β-blockers, sodium nitroprusside, and opioids. The most appropriate initial step in the management of this patient is initiation of medical therapy. Patients with type B acute aortic dissection without evidence of cardiogenic shock should be initially treated with medical therapy to control heart rate and reduce blood pressure. Intravenous β-blockers are first-line treatment, and sodium nitroprusside can be added if hypertension does not completely respond to β-blocker therapy. Current guidelines recommend reducing systolic blood pressure to 120 mm Hg or less in the first hour in patients with aortic dissection. Pain control is best accomplished with intravenous opioids. In patients with ascending aortic dissection (type A aortic dissection), intramural aortic hematoma, or descending aortic dissection (type B aortic dissection) with complications, immediate repair is warranted. Complications are defined as refractory pain, rapid aneurysmal expansion, rupture, or malperfusion syndrome. This patient has a type B aortic dissection arising just distal to the origin of the left subclavian artery with no evidence of complications. Therefore, medical therapy is the most appropriate treatment. Endovascular stenting can be used to treat patients with descending aortic aneurysms; however, there is scant evidence to support this as an initial treatment option for aneurysm with dissection. After medical stabilization, thoracic endovascular aortic repair (TEVAR) with stent grafting can be used; typical indications include descending aortic aneurysm diameter greater than 6.0 cm, rapid growth (>0.5 cm/year), or end-organ damage. TEVAR has the advantage over surgical operation of lower morbidity and shorter hospital stay, although there may be complications, including stroke, spinal ischemia, and aortic graft endoleaks. This patient should be treated with stent implantation only after medical stabilization. Open surgical repair of the descending aorta is not recommended in patients with uncomplicated type B aortic dissection because of high rates of morbidity (such as paraplegia) and mortality. This patient has no signs or symptoms of complications that would prompt immediate surgical repair. Repeat CT angiography at 12 hours is not indicated unless the patient does not stabilize with medical therapy. Serial imaging (usually magnetic resonance angiography) is typically performed in asymptomatic patients at the time of discharge and then periodically in patients who do not have an indication for aneurysmal repair to screen for extension or recurrence of the dissection, aneurysm formation, and leakage.
A 62-year-old woman is evaluated for recurrent breast cancer with bone and liver metastases. She was diagnosed with stage IA estrogen receptor-positive, progesterone receptor-positive, HER2-negative right breast cancer 8 years ago. She underwent lumpectomy and sentinel lymph-node biopsy and primary breast radiation, and completed 5 years of anastrozole therapy 2 years ago. She presented with right hip pain, with lesions on a bone scan consistent with metastases. Biopsy of a bone lesion shows metastatic adenocarcinoma that is estrogen receptor positive, progesterone receptor positive, and HER2 negative, consistent with metastatic breast cancer. She takes no medications. On physical examination, vital signs are normal. Healed incisions of the right breast and axilla are present. Tenderness to palpation of the right lateral hip is noted. The remainder of the examination is normal. Which of the following is the most appropriate treatment? Exemestane Letrozole plus palbociclib Raloxifene Tamoxifen
Patients with widely metastatic hormone receptor-positive, HER2-negative breast cancer can benefit from combination antiestrogen therapy with palbociclib. Letrozole plus palbociclib is the most appropriate treatment for this patient with advanced breast cancer that recurs after completing previous adjuvant aromatase inhibitor therapy. Palbociclib is an oral cyclin-dependent kinase 4 and 6 (CDK4 and CDK6) inhibitor that is synergistic with endocrine therapies in vitro. In vitro studies suggest that breast cancer that has become resistant to hormonal therapy remains dependent on cyclin D1-CDK4 for proliferation. Two studies have shown benefit to the combination of antiestrogen therapy with palbociclib for hormone receptor-positive, HER2-negative advanced breast cancer, with an increased response rate and an approximate doubling of progression-free survival compared to hormonal therapy alone. Therapy with palbociclib was associated with increased adverse effects, including fatigue, nausea, and neutropenia. Based on these studies, palbociclib has been approved for use with either letrozole as first-line therapy or fulvestrant for women with hormone receptor-positive, HER2-negative advanced breast cancer. As first-line therapy, an aromatase inhibitor with palbociclib is typically preferred. The aromatase inhibitor exemestane alone is unlikely to be effective in a patient who has relapsed a few years after receiving treatment with another aromatase inhibitor and is an inferior choice to combination hormonal therapy and palbociclib. Raloxifene can be used as chemoprophylaxis to lower the risk of breast cancer in patients with increased risk due to atypical breast lesions or strong family history but is not an active agent for treatment of breast cancer. Tamoxifen is not typically recommended as first-line therapy for advanced hormone receptor-positive, HER2-negative breast cancer. As a second-line agent after previous aromatase inhibitor treatment, it has an overall response rate of only 10%.
A 22-year-old woman undergoes routine evaluation for chronic anemia, which was diagnosed 6 years ago. Medical history is otherwise unremarkable, but a maternal aunt also has anemia. Her only medication is a combination oral contraceptive pill. On physical examination, vital signs are normal. No hepatosplenomegaly is noted. Laboratory studies: Hemoglobin 10 g/dL (100 g/L) Mean corpuscular volume 67 fL Iron studies Ferritin 200 ng/mL (200 µg/L) Iron 150 µg/dL (27 µmol/L) Total iron-binding capacity 340 µg/dL (61 µmol/L) Hemoglobin electrophoresis reveals a normal pattern of migration of hemoglobin A and normal levels of hemoglobin A2 and hemoglobin F. Which of the following is the most likely diagnosis? Inflammatory anemia Iron deficiency α-Thalassemia silent carrier α-Thalassemia trait β-Thalassemia minor
Patients with α-thalassemia trait have chronic microcytic anemia with hemoglobin levels of approximately 10 g/dL (100 g/L) and a normal hemoglobin electrophoresis pattern. The most likely diagnosis is α-thalassemia trait. This patient has a chronic microcytic anemia. Patients with α-thalassemia trait have inadequate production of two copies of the α gene on chromosome 16 (α−/α− or −−/αα). Such patients have a chronic microcytic anemia with hemoglobin levels of approximately 10 g/dL (100 g/L). Hemoglobin A levels, although reduced in quantity, are otherwise normal in these patients and will migrate in a normal pattern on electrophoresis. Furthermore, there is no increase in the minor hemoglobin components, hemoglobin A2 and hemoglobin F. It is possible to identify specific chromosomal abnormalities that establish the diagnosis of α-thalassemia, but physicians typically make a clinical diagnosis based on family history, complete blood count, peripheral blood smear, and hemoglobin electrophoresis and by ruling out other causes of microcytic anemia. Patients with thalassemia should receive supplemental folate but should not receive iron supplementation; they are not iron deficient and they have an increased ability to absorb iron, which can lead to iron overload. Genetic counseling may be indicated in reproductive planning. This patient's iron study results are normal, and her serum ferritin level is greater than 100 ng/mL (100 µg/L), essentially ruling out iron deficiency. Inflammatory anemia, also known as anemia of chronic disease, is characterized by a low serum iron level and often a concomitant reduction in iron-binding capacity in the absence of iron deficiency. The iron studies in this patient are inconsistent with that diagnosis. Mild microcytosis may be seen in the anemia of inflammation but not as severely as in this patient. Patients who are silent carriers for α-thalassemia have a defect in production of a single α gene. These patients have a normal hemoglobin level and no microcytosis. They are clinically healthy. Patients with β-thalassemia minor also have a chronic microcytic anemia and a normal electrophoretic pattern for hemoglobin A. But in β-thalassemia, the excess α chains link with δ and γ chains to produce increased amounts of hemoglobin A2 and hemoglobin F, respectively.
A 24-year-old woman undergoes routine evaluation. She has not seen a physician recently but reports no symptoms. She was diagnosed with a ventricular septal defect at age 7 months. Regular evaluation was performed during childhood. Medical history is otherwise noncontributory, and she takes no medications. On physical examination, vital signs are normal. The jugular venous pressure and apical impulse are normal. No parasternal impulse is noted. The S1 and S2 are masked by a loud holosystolic murmur noted at the left lower sternal border. The remainder of the examination is unremarkable. An electrocardiogram and chest radiograph are normal. A transthoracic echocardiogram demonstrates a membranous ventricular septal defect with a small left-to-right shunt. The left ventricular size and function are normal, with an ejection fraction of 60%. The right heart chambers and valve function are normal. The estimated pulmonary artery pressure is normal. Which of the following is the most appropriate management? Cardiac catheterization Cardiac magnetic resonance imaging Endocarditis prophylaxis Follow-up in 3 to 5 years
Periodic follow-up with clinical evaluation and imaging are appropriate in patients with a small uncomplicated ventricular septal defect. The most appropriate management for this patient with a small uncomplicated ventricular septal defect (VSD) is follow-up in 3 to 5 years. VSDs are defined by their location on the ventricular septum. The most common type of VSD is perimembranous, making up 80% of cases; these are usually isolated abnormalities. She has no associated symptoms, volume overload of the left heart, pulmonary hypertension, or valve regurgitation; therefore, periodic clinical evaluation and imaging are recommended. If pulmonary vascular disease is present (pulmonary artery systolic pressure >50 mm Hg), patients should be advised against isometric or competitive exercise. In the absence of pulmonary hypertension, pregnancy in women with VSDs is generally well tolerated. It is unnecessary to suggest activity restriction in this patient with a small VSD and normal pulmonary artery pressure, and pregnancy should not be complicated by the VSD. Cardiac catheterization is not indicated for this patient because her clinical presentation and echocardiogram do not demonstrate features of left heart enlargement or pulmonary hypertension. Cardiac catheterization is primarily performed to delineate the shunt ratio and to determine pulmonary pressures if clinical uncertainty exists regarding the degree or impact of a shunt, or if the VSD is incompletely assessed by echocardiographic measures. Cardiac magnetic resonance imaging will usually demonstrate a membranous VSD and can quantitate the impact of the shunt on the left heart; however, it is not indicated in this patient because the clinical and echocardiographic assessment are sufficient to suggest that observation is appropriate. Endocarditis prophylaxis is recommended for patients with unrepaired cyanotic congenital heart disease, including palliative shunts and conduits; a congenital heart defect that has been completely repaired with prosthetic material or device during the first 6 months after the procedure; and repaired congenital heart disease with residual defects. Patients with uncomplicated VSDs without a history of endocarditis do not require endocarditis prophylaxis.
A 36-year-old woman is evaluated in the emergency department for progressive dyspnea. She gave birth 3 weeks ago. The pregnancy and delivery were uncomplicated. She has no history of cardiovascular disease. On physical examination, temperature is normal, blood pressure is 100/72 mm Hg in both arms, pulse rate is 102/min and regular, and respiration rate is 26/min. The estimated central venous pressure is elevated. Cardiac palpation reveals a diffuse apical impulse. The S1 and S2 are soft. An S3 and S4 are present. A grade 2/6 holosystolic murmur is heard at the apex. Crackles are auscultated bilaterally. An electrocardiogram demonstrates sinus tachycardia without ST-T-wave changes. Transthoracic echocardiogram reveals ventricular dilatation with global reduction in contractility; the left ventricular ejection fraction is 30%. Which of the following is the most likely diagnosis? Acute aortic dissection Acute pulmonary embolism Peripartum cardiomyopathy Takotsubo cardiomyopathy
Peripartum cardiomyopathy is left ventricular systolic dysfunction with onset toward the end of pregnancy or in the months following delivery in the absence of another identifiable cause; patients often present with features of heart failure. The most likely diagnosis in this woman who is 3 weeks postpartum is peripartum cardiomyopathy. Peripartum cardiomyopathy is left ventricular systolic dysfunction with onset toward the end of pregnancy or in the months following delivery in the absence of another identifiable cause. Although patients may be asymptomatic, they often present with features of heart failure. Women with peripartum cardiomyopathy should be promptly treated with medical therapy, which may include β-blockers, digoxin, hydralazine, nitrates, or diuretics. ACE inhibitors, angiotensin receptor blockers, and aldosterone antagonists are teratogenic and should be avoided until after delivery. Acute aortic dissection classically presents with sudden onset of chest or back pain that has a tearing or ripping quality. Dyspnea is less common. Physical examination findings include a blood pressure or pulse differential between the upper extremities. A diastolic murmur of aortic regurgitation may be heard at the cardiac base if the aortic valve is involved. Echocardiography demonstrates an enlarged ascending aorta; the dissection flap may also be visible. Ventricular function is usually normal unless the aortic dissection has involved the coronary arteries, although regional abnormalities may be detected. This patient's clinical findings do not support a diagnosis of acute aortic dissection. Pulmonary embolism can occur postpartum, particularly when prolonged bed rest is required during or following pregnancy. Patients with pulmonary embolism frequently present with dyspnea; however, this patient's presentation suggests heart failure, given the elevated venous pressure, pulmonary congestion, and global reduction in ejection fraction. Takotsubo cardiomyopathy, also known as stress cardiomyopathy, is characterized by transient regional cardiac dysfunction, usually involving the apical and mid-portion of the left ventricle. It is usually precipitated by a stressful physical or emotional event. Postpartum cases of takotsubo cardiomyopathy have been reported, especially after cesarean delivery. Patients with takotsubo cardiomyopathy present with features that mimic an acute coronary syndrome: chest pain, ischemic electrocardiographic changes, and elevated cardiac biomarker levels. This patient's clinical picture is inconsistent with takotsubo cardiomyopathy.
A 52-year-old man is evaluated for fragile skin and blisters on the dorsal hands of 2 years' duration. Medical history is significant for a 30-pack-year history of smoking and moderate alcohol consumption. He is a roofer and has had significant sun exposure. He is otherwise in good health and takes no medications. On physical examination, vital signs are normal. He has hypertrichosis on the face. Skin findings are shown. The remainder of the physical examination is normal. Which of the following is the most likely diagnosis? Bullous pemphigoid Bullous tinea Dermatitis herpetiformis Pemphigus vulgaris Porphyria cutanea tarda
Porphyria cutanea tarda presents with skin fragility and small, transient, easily ruptured vesicles in sun-exposed areas, mainly on the hands; these eventually rupture, forming erosions, dyspigmentation, and scarring.
A 45-year-old woman with hypovolemic shock is evaluated for rapid resuscitation in the ICU. She has sickle cell disease with recurrent pain and hemolytic crises, and osteoporosis. On physical examination, temperature is 39 °C (102.3 °F), blood pressure is 70/40 mm Hg, pulse rate is 142/min and weak, and respiration rate is 22/min. Oxygen saturation is 99% breathing ambient air. There is a subcutaneous port in the right anterior chest wall. Which of the following is the most appropriate type of venous access for this patient? Intraosseous port Peripheral wide-bore catheter Subcutaneous intravenous port Triple-lumen central catheter
Peripheral wide-bore venous catheters are the preferred method for rapid intravenous administration of large amounts of fluids The most appropriate treatment is to insert a peripheral wide-bore catheter. This patient is in shock with several possible causes. Flow of fluid through a catheter is inversely proportional to catheter length and proportional to the radius of the catheter to the fourth power. Therefore, the highest flow rates may be achieved through shorter, large-bore catheters. Peripheral intravenous (IV) catheters are shorter and larger than catheters used for central access or peripherally inserted central catheters and can deliver high volumes of fluid rapidly. For this reason, use of larger, shorter peripheral catheters is preferred for fluid resuscitation in patients requiring emergent treatment. However, peripheral IV catheters can sometimes be difficult to insert in patients in shock, and intraosseous ports and central venous catheters are the alternatives. Intraosseous ports provide rapid access, but this patient has osteoporosis, which is a contraindication to this method. When used, an initial dose of lidocaine is needed before infusing because pain levels are very high with initial flushes and infusion. Subcutaneous intravenous ports are long and small bore, which makes them useful for blood draws and small-volume infusion administration but not for rapid, large-volume fluid resuscitation. A triple-lumen central catheter is an acceptable alternative when no other intravenous access can be obtained; however, it takes longer to insert compared to a peripheral wide-bore catheter. When used, care should be taken to choose wider-bore catheters to overcome the flow restriction from longer lengths.
A 52-year-old woman is hospitalized for a toe ulcer and foot pain occurring for 1 month. History is significant for stage G4 chronic kidney disease (estimated glomerular filtration rate , 22 mL/min/1.73 m2) and type 2 diabetes mellitus. Medications are lisinopril, sevelamer, sodium bicarbonate, insulin glargine, and insulin aspart. On physical examination, vital signs are normal. A foul-smelling toe ulcer is present. Probe-to-bone test is positive. A plain radiograph shows changes compatible with osteomyelitis. The patient undergoes wound débridement and bone biopsy. Bone cultures are pending, and empiric antibiotic therapy is to be administered. Which of the following is the most appropriate venous access strategy? Arteriovenous graft creation followed by peripherally inserted central catheter placement in opposite arm Peripherally inserted central catheter in the dominant arm Peripherally inserted central catheter in the nondominant arm Tunneled internal jugular central venous catheter
Peripherally inserted central catheter placement before or after hemodialysis initiation is associated with adverse vascular access outcomes in patients with chronic kidney disease. A tunneled internal jugular central venous catheter is the most appropriate venous access strategy for this patient. In patients with osteomyelitis, 6 weeks of antimicrobial therapy after surgical débridement is the preferred treatment course, and for most patients this requires long-term reliable venous access. However, this patient's advanced degree of kidney disease, relatively young age, and type 2 diabetes mellitus put her at high risk for progressing to end-stage kidney disease, and the upper extremity veins should be protected for future hemodialysis access creation. The American Society of Nephrology recommends nephrology consultation first before placing peripherally inserted central catheter (PICC) lines in patients with stage G3 to G5 CKD. PICCs are long catheters that are inserted peripherally and terminate in the central veins. PICCs are popular because of their ease of insertion and use. However, a national, population-based analysis revealed that PICCs placed before or after hemodialysis initiation were independently associated with lower likelihoods of transition to any working fistula or graft. PICC lines can lead to significant vein trauma and venous stenosis in the veins that may be used for arteriovenous fistula creation. This could impair blood flow through the fistula, impair maturation, and even lead to primary nonfunctioning of the hemodialysis access. Although central lines are associated with central venous stenosis, a central line is a better alternative than a PICC for long-term parenteral antibiotics because central lines are not inserted into a peripheral vein, which would be used for the creation of a hemodialysis site. Arteriovenous fistula or graft creation is inappropriate due to the presence of active infection. Additionally, inserting a PICC line in the arm next to the fistula may also still cause damage to the access itself or damage one of the branching veins, which could impair the maturation of the access.
An 18-year-old man is seen for a physical examination required to join the ice hockey team at his university. He was diagnosed with Kawasaki disease at age 5 years; he does not recall how or if he was treated for the disease. He has medical records from his pediatrician who has provided ongoing care until now. Physical examination, including vital signs, is normal. Which of the following disease complications is most important in determining the nature of his sports evaluation and prognosis? Bundle branch block Coronary artery aneurysm Myocarditis Pericarditis
Persistent coronary artery aneurysm conveys the greatest risk for long-term cardiovascular events in survivors of childhood Kawasaki disease. A history of coronary artery aneurysm is the most important disease complication determining the patient's need for extensive evaluation and his long-term prognosis. Kawasaki disease (KD) is a medium-vessel vasculitis that affects children and is very rare in adults. KD presents as fever, rash, cervical lymphadenopathy, prominent nonexudative conjunctivitis, and oral mucosal and lip changes. Coronary vessel vasculitis, aneurysm formation, and other cardiac complications (such as heart failure, pericarditis, and arrhythmias) may develop. Up to 25% of untreated patients develop coronary artery aneurysms as long-term sequelae of KD; intravenous immunoglobulin (IVIG) treatment reduces the incidence of aneurysms, but 10% to 20% of patients do not respond. Estimates indicate that 4% to 8% of patients treated with IVIG will develop aneurysms. Thrombosis or stenosis of these aneurysms can lead to ischemia and myocardial infarction. It is important to identify and follow patients at risk for long-term cardiac complications of KD, because they may require preventive therapy with aspirin or other antithrombotic medications. With the exception of the complications related to the coronary arteries, most patients with KD completely recover from multisystem acute inflammation. Patients without abnormalities need no specific cardiology surveillance, but those with persistent coronary aneurysm may require periodic electrocardiography, echocardiography, and/or coronary artery imaging as part of a long-term surveillance program. Recommendations about the nature and frequency of continued surveillance of patients recovering from KD, as well as physical activity restrictions, are provided by the American Heart Association and are based upon the coronary artery status of the patient. In the absence of coronary aneurysm, the available data suggest that the risk for coronary artery disease is low. For example, a large cohort study showed that among patients with KD, 5% had persistent coronary aneurysm, and long-term adverse cardiovascular events occurred in 8% with persistent coronary aneurysm versus 0% without persistent aneurysm. However, the nature and frequency of long-term surveillance are still unclear. An ongoing cohort study of Japanese children with a history of KD growing into adulthood will better inform the medical community on the best long-term surveillance and management plan. A past history of bundle branch block, myocarditis, or pericarditis does not appear to be associated with the same risk for long-term cardiovascular events as does the presence of coronary aneurysm.
A 50-year-old woman is evaluated for a 1-month history of pain around the lower part of the inner left knee. The pain is aggravated by ascending stairs and rising from a seated position. She has a history of knee osteoarthritis, for which she takes acetaminophen. Her new pain is different in location and intensity compared with her prior pain. She has no other pertinent personal or family history. On physical examination, vital signs are normal. Tenderness to palpation and slight swelling over the proximal medial tibia about 6 cm below the anteromedial joint margin of the knee are noted. Examination of the knee shows no crepitus, minimal pain along the medial joint line, and no pain on valgus stress; anterior and posterior drawer signs are negative. Which of the following is the most likely diagnosis? Medial collateral ligament tear Meniscal tear Pes anserine bursitis Quadriceps tendonitis
Pes anserine bursitis is an inflammatory condition affecting the bursa at the insertion of the conjoined medial knee tendons into the anteromedial proximal tibia and should be considered when there is localized pain inferomedial to the knee joint. The most likely diagnosis is pes anserine bursitis, an inflammatory condition affecting the bursa at the insertion of the conjoined medial knee tendons into the anteromedial proximal tibia. This condition has been known to affect athletes but can also occur in those with concurrent knee osteoarthritis (usually in obese women) even without a history of trauma or overuse. The diagnosis of pes anserine bursitis is clinical, based on a history of increased medial knee pain worsened with climbing or descending stairs or rising from a seated position. Patients may also note morning pain and stiffness lasting more than 1 hour, and pain may worsen at night. Physical examination showing severe point tenderness at the insertion of the anserine tendon about 3 to 6 cm (2 in) below the medial joint line, often with local edema, supports the diagnosis. Relief with injection of a local anesthetic is also supportive. A medial collateral ligament (MCL) tear may cause medial knee pain, which can also extend to where the MCL inserts on the anteromedial proximal tibia; however, an MCL injury is very painful and may cause difficulty ambulating. MCL injury is suspected when there is pain and/or laxity on valgus stress of the knee. A meniscal tear could cause medial knee pain, but the pain is usually around the joint line; patients may also have unusual sensations in the knee such as clicking, locking, catching, or a sensation that the joint gives way. There are several provocative maneuvers on physical examination that may suggest a meniscal tear. Quadriceps tendonitis occurs as a result of stress placed upon the quadriceps tendon, usually from sports-related activities. Pain can be felt at the lower part of the thigh, just above the patella. There may also be swelling around the quadriceps tendon.
OA tx options?
Physical therapy is appropriate for this patient whose history, physical examination, and radiographic findings are consistent with the diagnosis of osteoarthritis (OA). Physical therapy is an effective intervention for the management of pain and reduced functioning due to OA, with numerous guidelines supporting exercise as an appropriate intervention for all patients with OA. Evidence is most robust for knee OA. Many patients become sedentary due to their symptoms, and physical therapy is often a useful starting point to transition patients to participation in a regular exercise program. Physical therapy can be prescribed at any point in the course of the disease instead of medication, as a supplement to medication that does not adequately reduce pain, or prior to surgery to increase strength and potentially influence surgical outcomes. Glucosamine supplements are the most widely used over-the-counter products worldwide for OA. Many randomized controlled trials have found that pain, function, and radiographic progression improve overall at a level equivalent to the effects of placebo in patients taking glucosamine. Knee replacement surgery is effective for treatment of pain and disability but is not appropriate in a patient who has not been through a trial of physical therapy, particularly a patient who is not regularly exercising. Assessment of the ability to carry out activities of daily living, as well as occupational and recreational activities, will be important in the decision to seek surgery, and a physical therapist can be particularly helpful to the primary care provider in carrying out this assessment. Considerations for surgical referral include the presence of pain at rest or pain that awakens the patient in the middle of the night, both of which are less likely to respond to analgesics, exercise, or physical modalities. Prednisone is an oral glucocorticoid that can be used as an adjunctive medication in the treatment of some forms of inflammatory arthritis. However, insufficient evidence exists to support the use of oral glucocorticoids in the treatment of OA. Read Related TextNext Question
A 19-year-old man is evaluated for 4 to 5 months of persistent itching, thickening of the skin, and odor of the feet. He also has hyperhidrosis of the palms and soles. Medical history is otherwise unremarkable, and he takes no medications. On physical examination, vital signs are normal. Skin findings are shown. There is substantial odor of the feet. There are no changes to the toenails. The axillae and groin are unremarkable. Potassium hydroxide preparation of skin scrapings does not reveal fungal elements. Which of the following is the most likely diagnosis? Aquagenic keratoderma Erythrasma Keratoderma blenorrhagicum Pitted keratolysis Tinea pedis
Pitted keratolysis is a superficial bacterial infection characterized by small pits and punctate erosions primarily on the plantar aspects of the feet; risk factors include increased perspiration (hyperhidrosis) and prolonged occlusion of the feet. Pitted keratolysis is a bacterial infection most commonly caused by Kytococcus sedentarius. The key features are waxy or scaly plaques, thickening of the plantar skin (keratoderma), small punctate erosions in the plaques that may coalesce to form broader erosions, and odor. Risk factors for infection include excess sweating (hyperhidrosis) and prolonged occlusion of the feet in footwear. Usually symptoms are minimal, with most complaints directed toward odor, but minor itching can occur. Treatment is best accomplished with topical antibiotics and keeping the feet dry, including the use of topical antiperspirants. Aquagenic keratoderma presents with wrinkling, translucency, and scaling of the palmar hands and plantar feet shortly after exposure to water and is associated with carrier status of the cystic fibrosis gene. Aquagenic keratoderma does not feature pitting. Erythrasma is also a bacterial infection that tends to involve intertriginous areas such as the axillae and the crural folds. Skin may develop a finely wrinkled, cigarette paper texture, but would not feature pitting. Erythrasma will fluoresce to a coral red color with a Wood lamp examination because of bacterial porphyrin production. Keratoderma blenorrhagicum is a keratoderma of the palms or soles associated with reactive arthritis. With this rash, patients develop psoriasiform red plaques with lamellar scale. Patients may have other features of reactive arthritis such as asymmetric monoarthritis or oligoarthritis in the lower extremities, enthesopathy, dactylitis, and sacroiliitis. Tinea pedis can cause scaling and itching of the feet. Patients typically develop pink or red scaly plaques with a serpiginous or arcuate discrete border. Scale is often emphasized at the edge of plaques. Pustules may be present at hair follicles, but pitting and strong odor are not typical. Potassium hydroxide preparation of skin scrapings should reveal fungal elements, except in partially treated cases.
A 52-year-old woman is evaluated for a 3-week history of new-onset daily headaches. The pain is absent nocturnally and on awakening but starts within 15 minutes of the patient's arising from bed and becomes progressively severe throughout the day. The headache is global, steady, and (when severe) associated with photophobia and mild nausea. Intermittent bilateral tinnitus and brief episodes of horizontal diplopia also have occurred. The pain improves within 15 to 20 minutes of the patient's lying down. Analgesic agents have been unhelpful. She has no other medical problems. On physical examination, vital signs are normal; BMI is 26. Partial right abducens nerve (cranial nerve VI) palsy is noted. An MRI of the brain shows diffuse nonnodular pachymeningeal enhancement, a cerebellar tonsillar descent of 3 mm, and clinically insignificant bilateral subdural fluid collections. Which of the following is the most appropriate first step in management? Acetazolamide administration Epidural blood patch Lumbar puncture Subdural evacuation
Placement of an epidural blood patch is the initial treatment of choice for patients with intracranial hypotension; CT myelography is appropriate for those who do not respond to this treatment.
A 52-year-old woman is evaluated for a 3-week history of new-onset daily headaches. The pain is absent nocturnally and on awakening but starts within 15 minutes of the patient's arising from bed and becomes progressively severe throughout the day. The headache is global, steady, and (when severe) associated with photophobia and mild nausea. Intermittent bilateral tinnitus and brief episodes of horizontal diplopia also have occurred. The pain improves within 15 to 20 minutes of the patient's lying down. Analgesic agents have been unhelpful. She has no other medical problems. On physical examination, vital signs are normal; BMI is 26. Partial right abducens nerve (cranial nerve VI) palsy is noted. An MRI of the brain shows diffuse nonnodular pachymeningeal enhancement, a cerebellar tonsillar descent of 3 mm, and clinically insignificant bilateral subdural fluid collections. Which of the following is the most appropriate first step in management? Acetazolamide administration Epidural blood patch Lumbar puncture Subdural evacuation
Placement of an epidural blood patch is the initial treatment of choice for patients with intracranial hypotension; CT myelography is appropriate for those who do not respond to this treatment. This patient has intracranial hypotension and should undergo placement of an epidural blood patch (EBP). Spontaneous intracranial hypotension classically presents with an orthostatic headache. The interval between postural change and headache development is highly variable. When the headache duration is weeks to months, the orthostatic component may fade completely. Presentation may be thunderclap (maximum onset within 1 minute) or subacute. Intracranial hypotension is the result of cerebrospinal fluid (CSF) leakage that can arise as a result of lumbar puncture, surgery, or trauma or can occur spontaneously. Associated features include tinnitus, diplopia, neck pain, nausea, photophobia, and phonophobia. Clinical examination findings are typically normal but occasionally reveal a "falsely localizing" abducens nerve (cranial nerve VI) palsy. Female sex, middle age, and connective tissue disorders are risk factors. Brain MRI with contrast is abnormal in 80% of affected patients, with possible findings of diffuse nonnodular pachymeningeal enhancement, cerebellar tonsillar abnormalities, and subdural fluid collections. Use of an EBP is the initial treatment of choice. Patients who do not respond to this treatment should undergo CT myelography for potential detection of the precise site of cerebrospinal fluid (CSF) leakage. Subsequent treatments may include a repeat EBP or surgical repair of the identified site. Acetazolamide is indicated in patients with intracranial hypertension but relatively contraindicated in cases of intracranial hypotension. A diagnosis of intracranial hypotension can be confirmed by a CSF opening pressure of less than 60 mm H2O, but lumbar puncture would introduce another site of potential CSF leakage. In the presence of patient history and brain MRI abnormalities that are typical for intracranial hypotension, lumbar puncture is unnecessary and should be avoided. Small subdural fluid collections are commonly seen in patients with intracranial hypotension and do not require surgical evacuation. Typical indications for surgical management include an acute subdural hematoma measuring greater than 10 mm in thickness, a Glasgow Coma Scale score less than 9, and pupillary asymmetry or fixation.
A 34-year-old woman is evaluated for a 6-month history of gradually increasing pain in the left groin with some radiation to the left buttock, particularly with stair climbing. She has a 2-year history of lupus nephritis. Medications are hydroxychloroquine, mycophenolate mofetil, and prednisone. On physical examination, vital signs are normal. Musculoskeletal examination reveals weakness of the left hip flexors. Decreased passive range of motion of the left hip in both external and internal rotation with pain is noted. Which of the following is the most appropriate initial test to evaluate the patient's hip pain? Dual-energy x-ray absorptiometry MRI of the left hip Plain radiography of the left hip Ultrasonography
Plain radiography is the initial radiographic study of choice for osteonecrosis, and MRI is the modality of choice for sensitive evaluation of early disease when plain radiographs are normal.
A 34-year-old woman is evaluated for a 6-month history of gradually increasing pain in the left groin with some radiation to the left buttock, particularly with stair climbing. She has a 2-year history of lupus nephritis. Medications are hydroxychloroquine, mycophenolate mofetil, and prednisone. On physical examination, vital signs are normal. Musculoskeletal examination reveals weakness of the left hip flexors. Decreased passive range of motion of the left hip in both external and internal rotation with pain is noted. Which of the following is the most appropriate initial test to evaluate the patient's hip pain? Dual-energy x-ray absorptiometry MRI of the left hip Plain radiography of the left hip Ultrasonography
Plain radiography is the initial radiographic study of choice for osteonecrosis, and MRI is the modality of choice for sensitive evaluation of early disease when plain radiographs are normal. Plain radiography of the left hip is the most appropriate initial test for this patient with subacute hip pain. She has lupus nephritis that requires treatment with glucocorticoids. Both glucocorticoid treatment and systemic lupus erythematosus are risk factors for osteonecrosis, which causes death of an area of bone due to compromised blood supply. Other common risk factors include prior fracture or radiation exposure, excessive alcohol use, and sickle cell anemia. Osteonecrosis typically affects the ends of long bones, including the femoral head, humeral head, and distal femur. The involved area of bone collapses, often leading to rapidly progressive osteoarthritis. Patients with hip osteonecrosis present with groin pain (that is, true hip pain) occasionally radiating to the buttock, which worsens with ambulation; osteoarthritis can present in the same way, but would not be expected in a 34-year-old patient. Physical findings include reduced range of motion of the hip, altered gait, and pain with weight bearing. Plain radiography is the initial radiographic study of choice, although MRI is the gold standard for diagnosis and may be required if plain radiography is not diagnostic. Early radiographic findings include bone density changes, sclerosis, and, eventually, cyst formation. Subchondral radiolucency producing the "crescent sign" indicates subchondral collapse. End-stage disease is characterized by collapse of the femoral head, joint-space narrowing, and degenerative changes. Dual-energy x-ray absorptiometry is useful to assess bone density in patients on chronic glucocorticoid therapy. Although it may establish the presence of osteoporosis, it cannot determine the cause of pain and has little role in the evaluation of acute symptoms such as hip pain. MRI is the best method for detecting early bone edema caused by osteonecrosis when plain radiographs are normal. MRI is preferred for diagnostic use in patients with nondiagnostic plain radiographs. However, for end-stage disease, MRI is more expensive than plain radiography, and it may be unnecessary if plain radiography is diagnostic.
A 49-year-old man is evaluated for a 10-year history of intermittent low back pain with 30 to 60 minutes of morning stiffness, both of which improve with exercise. The back pain can awaken him from sleep. He also reports occasional pain in the neck, shoulder blades, and buttocks on one side or the other. History is notable for anterior uveitis diagnosed more than 30 years ago, the symptoms of which recurred every 3 to 4 years but have been subsequently controlled by topical glucocorticoids. Current medications are as-needed NSAIDs and ophthalmic glucocorticoid drops. On physical examination, vital signs are normal. Lumbar spine range of motion is restricted for forward flexion. There are no warm, erythematous, or swollen joints. There is no spinal or sacroiliac tenderness. Conjunctivae are without injection. Which of the following is the most appropriate test to perform next? Bone scan CT of the sacroiliac joints MRI of the lumbar spine Radiography of the sacroiliac joints
Plain radiography of the sacroiliac joints should be obtained in patients with suspected ankylosing spondylitis to evaluate for evidence of sacroiliitis, including erosions, sclerosis, and widening, narrowing, or partial ankylosis of the sacroiliac joints. The most appropriate test to perform next is plain radiography of the sacroiliac joints. This patient has features of inflammatory back pain (onset before age 45 years, duration >3 months, insidious onset, morning stiffness >30 minutes, improvement with exercise, no improvement with rest, awaking from pain with improvement on arising, alternating buttock pain), as well as antecedent history of anterior uveitis. He should therefore be evaluated for ankylosing spondylitis. The American College of Radiology appropriateness criteria recommend radiography of the sacroiliac joints and spine as the initial evaluation of patients with inflammatory sacroiliac or back symptoms for evidence of sacroiliitis, including erosions, evidence of sclerosis, and widening, narrowing, or partial ankylosis of the sacroiliac joints. The presence of such findings would confirm the diagnosis of ankylosing spondylitis. A bone scan lacks specificity for the changes of sacroiliitis and is not of value in the diagnosis of ankylosing spondylitis. CT of the sacroiliac joints is not considered an appropriate initial diagnostic test for ankylosing spondylitis. It exposes the patient to unnecessary amounts of radiation. MRI is also unnecessary in most cases of suspected ankylosing spondylitis because the diagnosis can often be confirmed with less costly radiography. Although MRI is more sensitive than radiography or CT in detecting sacroiliitis, it is less specific and can result in a high yield of false-positive test results. MRI may be useful to detect inflammation in patients with high clinical suspicion but normal or equivocal radiographs.
A 48-year-old man is evaluated for a 3-week history of slowly progressive dyspnea on exertion, left-sided chest pain, and fever. He has a 20-year history of anti-cyclic citrullinated peptide antibody-positive rheumatoid arthritis. Prior to starting a tumor necrosis factor α inhibitor, his tuberculin skin test was nonreactive. Medications are methotrexate, etanercept, and folic acid. Despite these medications, he has morning stiffness for 1 hour and has swelling of the joints in his hands, wrists, and feet. On physical examination, temperature is 37.8 °C (100 °F), blood pressure is 148/84 mm Hg, pulse rate is 100/min, and respiration rate is 24/min. Breath sounds are absent at the right base, with dullness on percussion. Joint changes typical of rheumatoid arthritis are present, along with swelling and tenderness of eight joints. The remainder of the examination is normal. Chest radiograph shows a moderate right-sided pleural effusion. Pleural fluid analysis following a thoracentesis shows a leukocyte count of 3500/µL (3.5 × 109/L) with 4% neutrophils, 87% lymphocytes, and 9% monocytes; a pleural fluid glucose level of 6.0 mg/dL (0.3 mmol/L); a pH of 7.2; and a pleural fluid lactate dehydrogenase level of 900 U/L. A pleural fluid adenosine deaminase measurement is low, at 30 U/L. Which of the following is the most likely cause of the pleural effusion? Malignancy Parapneumonic effusion Rheumatoid pleuritis Tuberculosis
Pleuritis is the most common rheumatoid arthritis pulmonary manifestation but is frequently asymptomatic; exudative pleural effusions may occur. Rheumatoid pleuritis is the most likely cause of this patient's pleural effusion. Pleuritis is the most common rheumatoid arthritis (RA) pulmonary manifestation but is frequently asymptomatic; exudative pleural effusions may occur. According to the Light criteria, this pleural fluid is an exudate based upon an elevated pleural fluid lactate dehydrogenase (LDH) concentration greater than two thirds the upper limit of normal for serum LDH. In rheumatoid effusions, pleural leukocyte count is typically less than 5000/µL (5.0 × 109/L), pleural fluid glucose is less than 60 mg/dL (3.33 mmol/L), and pH is less than 7.3. This exudative pleural effusion is compatible with rheumatoid pleurisy, malignancy, and tuberculosis pleurisy. Malignancy is a possible diagnosis accounting for this patient's pleural effusion. However, extremely low pleural fluid glucose concentrations are found only in rheumatoid pleurisy and empyema. Glucose concentration of approximately 30 to 60 mg/dL (1.7-3.3 mmol/L) is more typical of malignancy. In addition, pleural fluid lymphocytosis, particularly with lymphocyte counts representing 85% or more of the total nucleated cells, is not commonly associated with malignancies other than lymphoma. When in doubt, a chest CT with contrast should be performed in patients with pleural effusion of indeterminate cause. The low leukocyte count and lymphocytic predominance is not compatible with an acute parapneumonic effusion. Tumor necrosis factor α inhibitors such as etanercept increase the risk for tuberculosis. However, a previous negative tuberculin skin test and low pleural fluid adenosine deaminase measurement (<40 U/L) effectively rule out this diagnosis.
A 71-year-old man is evaluated for headaches of 2 months' duration. He reports no shortness of breath and has a good energy level. Medical history is significant for hypertension, hypothyroidism, and hypogonadism; he has never smoked. Medications are lisinopril, levothyroxine, and testosterone injections. On physical examination, vital signs are normal except for a blood pressure of 160/92 mm Hg; BMI is 19. He has facial plethora. Cardiopulmonary examination is normal. No hepatosplenomegaly is noted. Laboratory studies: Erythropoietin 40 mU/mL (40 U/L) Hematocrit 56% Hemoglobin 18.9 g/dL (189 g/L) Leukocyte count 7000/µL (7 × 109/L) with normal differential Platelet count 300,000/µL (300 × 109/L) Which of the following is the most likely cause of this patient's findings? Levothyroxine Lisinopril Polycythemia vera Testosterone
Polycythemia is a common adverse effect of testosterone injections, and testosterone supplementation should be interrupted if the hematocrit level exceeds 54%. This patient's polycythemia is likely caused by testosterone injections. Polycythemia is a common adverse effect of testosterone (and anabolic steroid) supplementation, and evaluating the hematocrit level at initiation, 3 to 6 months after testosterone initiation, and annually thereafter is recommended by Endocrine Society guidelines. Guidelines also recommend interrupting the testosterone supplementation if the hematocrit value is greater than 54%. Secondary polycythemia can cause symptoms, such as headaches and elevated blood pressure, and increase the risk of thromboembolism. The risk of venous thromboembolic disease is directly related to hematocrit level. In this patient, stopping the testosterone supplementation until his polycythemia improves and restarting at a lower dose is recommended. In patients with severe symptoms, phlebotomy can also be considered. Testosterone supplementation can also exacerbate undiagnosed sleep apnea, and evaluation for an underlying sleep disorder should be performed in these patients with polycythemia. Anemia rather than erythrocytosis may be seen in patients with both hyperthyroidism and hypothyroidism, and levothyroxine, even if given at inappropriately high or low doses, does not cause polycythemia. Lisinopril does not cause polycythemia. ACE inhibitors, such as lisinopril, are actually used to treat secondary polycythemia after kidney transplantation. Posttransplant erythrocytosis occurs in up to 15% of patients after kidney transplantation. ACE inhibitors reduce erythrocytosis in 90% of patients in a dose-response manner. The mechanism is unknown, but apoptosis of the erythroid precursors has been suggested as well as renin-angiotensin system inhibition that may decrease erythropoietin production. Polycythemia vera (PV) is a disorder of the myeloid and erythroid stem cells that causes erythropoietin-independent proliferation of erythrocytes and splenomegaly. PV should be considered in all patients with polycythemia; however, in this patient who is receiving testosterone supplementation, has an elevated erythropoietin level, and does not have splenomegaly, PV is unlikely. If PV is suspected, mutational analysis for JAK2 V617F should be performed.
A 51-year-old man is evaluated for morning headaches and increasing difficulty with focus and memory. He reports growing problems at work, including making occasional mistakes when updating spreadsheets, missing three recent appointments, and occasionally falling asleep during staff meetings. The patient has no other medical problems. He does not drink alcoholic beverages, smoke, or have a family history of dementia. On physical examination, blood pressure is 150/90 mm Hg but other vital signs are normal; BMI is 33. He scores 25/30 (normal, ≥26) on the Montreal Cognitive Assessment, scoring 2/5 in the recall section, 0/1 in word generation, and 0/1 in letter identification. He scores normally on several clinical depression scales. Other physical examination findings, including those from a neurologic examination, are unremarkable. Which of the following is the most appropriate next step in evaluating this patient? Cerebrospinal fluid analysis Fluorodeoxyglucose PET scanning MRI of the brain Polysomnography
Polysomnography should be performed in a patient with minor symptoms of cognitive impairment and self-reported daytime sleepiness who scores normally on cognitive and depression assessments. This patient should have polysomnography. He has minor symptoms of cognitive impairment, such as memory lapses and calculation difficulties at work. To exclude a slowly progressive dementia and identify any potentially reversible causes of his symptoms, the patient's examination should consist of a neurologic examination, including a cognitive screening evaluation, and evaluation for depression, sleep disorders, alcohol abuse, and family history of dementia. The patient does not use alcohol and has no family history of dementia. His score on the Montreal Cognitive Assessment is borderline abnormal, which would be unlikely if Alzheimer disease or another progressive dementia were the cause of his symptoms, and several depression scales provide no evidence of depression. Because depression and a primary neurodegenerative disorder are unlikely in this patient, additional diagnostic testing is necessary. His hypertension, obesity, and self-reported daytime sleepiness are consistent with a diagnosis of obstructive sleep apnea (OSA). Other neuropsychiatric symptoms of OSA include mood alterations, difficulty concentrating, and problems completing tasks at school or the workplace. Objective testing is required for the diagnosis of OSA. The traditional gold standard of in-laboratory, technician-attended polysomnography is being replaced by portable technology designed for use by the unaccompanied patient and is referred to as out-of-center sleep testing (OCST). This portable testing is typically limited to measurement of oronasal airflow, chest wall excursion, body position, and pulse oximetry, whereas in-laboratory polysomnography also measures brain waves (electroencephalography), muscle activity (electromyography), and heart rhythm (electrocardiography) and occurs in the presence of a technician in case of technical difficulties. OCST performs comparably to polysomnography in patients without comorbid cardiopulmonary disease who have a high pretest probability of moderate to severe OSA. Given the patient's normal findings on neurologic examination and relatively minor neurologic symptoms, cerebrospinal fluid analysis, fluorodeoxyglucose PET scanning, and mild structural brain imaging with MRI are likely to be of low yield.
A 52-year-old man is evaluated for fragile skin and blisters on the dorsal hands of 2 years' duration. Medical history is significant for a 30-pack-year history of smoking and moderate alcohol consumption. He is a roofer and has had significant sun exposure. He is otherwise in good health and takes no medications. On physical examination, vital signs are normal. He has hypertrichosis on the face. Skin findings are shown. The remainder of the physical examination is normal. Which of the following is the most likely diagnosis? Bullous pemphigoid Bullous tinea Dermatitis herpetiformis Pemphigus vulgaris Porphyria cutanea tarda
Porphyria cutanea tarda presents with skin fragility and small, transient, easily ruptured vesicles in sun-exposed areas, mainly on the hands; these eventually rupture, forming erosions, dyspigmentation, and scarring. This patient has porphyria cutanea tarda (PCT). Skin findings seen early in the disease include fragile skin with vesicles and bulla associated with milia (tiny proteinaceous cysts) formation on sun-exposed skin, particularly on the dorsal hands. PCT is caused by an accumulation of photosensitive porphyrins within the bloodstream and tissue. These photosensitizers enter the skin and upon exposure to ultraviolet A light in the 400- to 410-nm range cause damage to the skin. Later in the course of the disease, patients may develop hypo- or hyperpigmentation and hypertrichosis of the face. In some patients, scarring may result in thickening and calcification of the skin. Persons diagnosed with PCT should be evaluated for underlying liver disease, especially hepatitis C and hemochromatosis. Therapy is usually focused on avoiding sun exposure and decreasing the iron overload with phlebotomy, as well as treating the underlying cause of liver disease. Susceptibility factors that can exacerbate PCT to be avoided include alcohol use, smoking, and estrogens. The diagnosis can be made by finding elevated levels of porphyrins in the urine. A 24-hour urine evaluation will show elevated levels of uroporphyrin and coproporphyrin. Bullous pemphigoid is the most frequent cause of autoimmune blistering disease. Patients will have pruritic urticarial red plaques that eventually develop large bullae. These blisters are subepidermal in nature, and milia may occasionally be seen. The blisters occur most frequently on the trunk and can be quite large and tense. Bullous tinea is typically unilateral and associated with an annular scaly patch or patches. A potassium hydroxide examination will reveal fungal hyphae. The blisters are intraepidermal in nature, and milia will not be seen. Dermatitis herpetiformis presents with tiny fragile vesicles or small erosions on the elbows, knees, or scalp. Itching is pronounced, and the condition is worsened by consumption of food containing gluten. Involvement of the dorsal hands would be an unusual clinical finding. Persons with pemphigus vulgaris develop fragile vesicles and bulla without milia formation. The oral mucosa is frequently involved and is often the site of first involvement. Patients with bullous pemphigoid, pemphigus vulgaris, and dermatitis herpetiformis do not develop hypertrichosis.
A 49-year-old man is evaluated for a 6-month history of difficulty walking and frequent falls. He first noticed these symptoms after a prolonged hospitalization for pneumonia that was complicated by sepsis and cardiopulmonary arrest requiring intubation and antibiotic therapy. The patient also has hypothyroidism treated with levothyroxine. On physical examination, vital signs are normal; BMI is 35. Muscle strength is normal in all limbs. At rest, few rapid jerky movements are noted in various limbs. On standing, rapid, nonrhythmic, shock-like movements are noted in the trunk and lower extremities, and there are brief lapses of muscle tone in the legs, which lead to loss of balance. These movements are not suppressible. Cognition, cranial nerves, reflexes, and sensation are normal. Plantar response is flexor bilaterally, and clonus is absent. An MRI of the brain is unremarkable. Which of the following most accurately describes this patient's abnormal movements? Ataxia Chorea Dystonia Myoclonus Orthostatic tremor
Posthypoxic myoclonus, which occurs in patients with a history of hypoxic brain injury, is characterized by prominent action-induced myoclonus that impairs ambulation because of a combination of positive (rapid jerky movements) and negative (lapses in muscle tone) myoclonus. This patient's clinical history and clinical examination findings are most consistent with myoclonus. Myoclonus consists of rapid, nonsuppressible, shock-like, jerky movements that can result from metabolic, endocrine, toxic, infectious, epileptic, autoimmune, and other causes. Posthypoxic myoclonus (Lance-Adams syndrome) occurs in patients with a history of hypoxic brain injury (suggested in this patient by his previous cardiopulmonary arrest) and is characterized by prominent action-induced myoclonus that impairs ambulation because of a combination of positive (rapid jerky movements) and negative (lapses in muscle tone) myoclonus. Other examples of negative myoclonus include hiccups and asterixis. Ataxia is associated with wide-based gait, incoordination, and dysmetria. However, rapid shock-like movements and brief lapses of muscle tone are not features of ataxia. Ataxia is a hallmark of cerebellar damage and can be seen in posthypoxic settings. Chorea consists of random, nonrepetitive, flowing, dance-like movements, unlike the abnormal movements exhibited by this patient. Choreiform movements are not rapid or shock-like. Dystonia involves sustained muscle contractions leading to stereotyped and directional twisting and posturing movements that are absent in this patient. An orthostatic tremor is a high-frequency, rhythmic tremor that emerges in the legs only during standing and resolves with sitting or ambulation. This does not describe the patient's movements, which are nonrhythmic and jerky.
A 26-year-old man is hospitalized for a 3-month history of fever, night sweats, and weakness. Medical history is notable for kidney transplantation 5 years ago. Medications are prednisone and mycophenolate. On physical examination, temperature is 38.1 °C (100.5 °F), blood pressure is 135/62 mm Hg, pulse rate is 68/min, and respiration rate is 20/min. The sclerae are pale. Cervical, supraclavicular, and axillary lymphadenopathy are noted. Abdominal examination reveals hepatosplenomegaly. Laboratory studies: Hemoglobin 8.4 mg/dL (84 g/L) Leukocyte count 2300/µL (2.3 × 109/L) Platelet count 98,000/µL (98 × 109/L) Creatinine (stable) 1.1 mg/dL (97.2 µmol/L) Which of the following is the most likely cause of his symptoms? Cytomegalovirus Epstein-Barr virus Herpes simplex virus type 1 Polyoma BK virus
Posttransplant lymphoproliferative disorder caused by Epstein-Barr virus can present several years after transplantation with fever, pancytopenia, generalized lymphadenopathy, and hepatosplenomegaly. This patient's symptoms are most likely caused by infection with Epstein-Barr virus (EBV). He most likely has posttransplant lymphoproliferative disorder (PTLD). His clinical presentation of fever, pancytopenia, generalized lymphadenopathy, and hepatosplenomegaly is consistent with this diagnosis, considering his kidney transplantation and immunosuppressive therapy. PTLD risk is higher in patients with a history of pre-existing EBV infection treated with lymphocyte-depleting agents and in those receiving sirolimus and tacrolimus compared with those receiving mycophenolate and cyclosporine. PTLD can range from a benign monoclonal gammopathy to a malignant lymphoma. PTLD should be considered in any patient presenting with fever and lymphadenopathy or an extranodal mass; treatment includes reduction of immunosuppression and rituximab or other chemotherapy. Although numerous viral infections can complicate transplantation, cytomegalovirus is the most significant. The risk for reactivation is related to serologic status of the donor and recipient and is most likely in seronegative recipients from a seropositive donor; it is unlikely when donor and recipient are both negative. Like EBV, cytomegalovirus can reactivate and cause pancytopenia, hepatitis, pneumonitis, esophagitis, colitis, or adrenalitis in solid-organ transplant recipients. Cytomegalovirus reactivation is also associated with organ rejection, secondary infection, and an increased risk for graft loss and death. However, cytomegalovirus does not cause generalized lymphadenopathy or hepatosplenomegaly. Cytomegalovirus-seropositive transplant recipients or cytomegalovirus-negative recipients with positive donors should receive prophylactic valganciclovir or undergo routine monitoring for cytomegalovirus viremia. Herpes simplex virus can also reactivate in the setting of immunosuppressive therapy, but it would be more likely to present with oral or genital ulcers. Polyoma BK virus reactivation occurs in approximately 5% of kidney transplant recipients and can cause kidney allograft dysfunction or loss. It can present with a gradual, asymptomatic increase in the serum creatinine level with tubulointerstitial nephritis or, less commonly, with ureteral stenosis. Patients may have polyoma BK virus on polymerase chain reaction of the urine or serum or may have polyoma BK virus inclusion-bearing epithelial cells called "decoy cells." The treatment is to reduce immunosuppression. This patient's stable serum creatinine level argues against this diagnosis.
A 42-year-old woman is evaluated during a follow-up visit for stage IIA cervical squamous cell cancer. She has completed therapy with pelvic external-beam radiation and concomitant weekly cisplatin chemotherapy, with cervical brachytherapy added during the fourth week of therapy. She has had an excellent response to treatment. She is otherwise well and takes no medications. On physical examination, vital signs are normal. Pelvic examination reveals no evidence of tumor, and the remainder of the examination is normal. Which of the following is the most appropriate posttreatment surveillance for this patient? CT of the pelvis every 6 months Complete blood count, serum creatinine, and liver chemistry tests every 3 months History, physical, and gynecologic examination every 3 months Pelvic ultrasonography every 6 months
Posttreatment surveillance of patients with cervical cancer is limited to a thorough history and examination every 3 to 6 months for the first 2 years, then every 6 to 12 months for years 2 through 5; frequency of visits can be tailored to the patient's risk. The most appropriate posttreatment surveillance for this patient with cervical cancer is a history, physical examination, and gynecologic examination every 3 months to monitor for local recurrence. Guidelines for posttreatment surveillance of patients with cervical cancer recommend a thorough history and examination every 3 to 6 months for the first 2 years, then every 6 to 12 months for years 2 through 5. Higher-risk patients, such as those treated with chemotherapy or radiation, should be evaluated every 3 months for 2 years, then every 6 months for years 2 through 5; those with lower-risk disease treated with surgery alone should be evaluated every 6 months for 2 years, then annually through year 5. The main goals of surveillance are to detect central pelvic (not involving the pelvic sidewall) recurrences that might be amenable to curative therapy and to address treatment-related symptoms. Treatment-related symptoms include bladder and bowel dysfunction; dyspareunia; difficulty achieving sexual arousal and orgasm; hot flushes; lymphedema of the lower extremities; fatigue; and mood disturbances related to fear of recurrence, altered body image, and loss of fertility. Patients also should be educated at these visits about symptoms that might indicate recurrence, including vaginal bleeding or new abdominal or pelvic pain. Most recurrences are symptomatic and are detected within 2 years of primary treatment, with nearly all detected by 5 years. Additional laboratory or imaging studies should be directed by signs or symptoms suggesting possible recurrence rather than done as routine studies in asymptomatic patients. Although annual cervical or vaginal cytology, or both, is recommended, it is unlikely that an asymptomatic recurrence will be found by cytology alone. In addition, pelvic radiation may alter tissue histology and make cytology interpretation difficult. There is no evidence that routine imaging studies with either ultrasonography or CT results in better patient outcomes than clinical monitoring, or that the response to palliative therapy is improved by such surveillance techniques. Routine laboratory studies are also not recommended.
A 77-year-old woman is evaluated in the emergency department for new-onset generalized weakness and myalgia 2 days after receiving zoledronic acid for a recent diagnosis of osteoporosis. Medical history is significant for rheumatoid arthritis. She is mainly sedentary and uses a walker for ambulation. She lives alone and prepares her own meals. Her medications are zoledronic acid, methotrexate, prednisone, folic acid, and tramadol. On physical examination, vital signs are normal; however, she develops hand spasm during the blood pressure measurement. BMI is 18. The patient is frail appearing. There is ulnar deviation at the metacarpophalangeal joints of both hands, but no signs of active synovitis. Laboratory studies show a calcium level of 7.5 mg/dL (1.9 mmol/L). Which of the following is the most likely diagnosis? Hungry bone syndrome Hyperphosphatemia Hypoparathyroidism Hypovitaminosis D
Potent antiresorptive drugs can cause severe hypocalcemia by impairing efflux of calcium from the skeleton in patients with vitamin D deficiency; it is important to assess vitamin D levels and correct deficiency before beginning treatment with an antiresorptive drug. The most likely diagnosis is vitamin D deficiency. Special populations will have lower levels of vitamin D owing to medical conditions or medication side effects. Obesity has been correlated with lower vitamin D levels possibly related to fat sequestration. Phenobarbital and phenytoin may increase the metabolism of vitamin D to inactive forms. Glucocorticoids can decrease vitamin D metabolism to active forms. Agents that decrease absorption such as orlistat can decrease vitamin D absorption. Malabsorption disorders, including celiac disease and bariatric surgery, can also result in vitamin D deficiency. This patient has multiple risk factors for vitamin D deficiency including age, possible malnutrition, glucocorticoid use, and being home bound. In the face of ongoing vitamin D deficiency, normocalcemia is maintained through increased bone resorption through increased osteoclastic activity. Antiresorptive drugs, especially the first dose of intravenous bisphosphonates and denosumab, rapidly suppress osteoclastic bone resorption and can precipitate hypocalcemia in these patients. Vitamin D sufficiency should be assessed prior to initiating antiresorptive drugs, especially those administered parenterally. High baseline bone turnover and abrupt alteration in calcium flux between blood and bone are also features of hungry bone syndrome. However, this syndrome specifically occurs after parathyroidectomy for primary hyperparathyroidism. It is caused by rapid influx of calcium from the blood into the skeleton. In the absence of parathyroidectomy, hungry bone syndrome cannot explain this patient's findings. Acute hyperphosphatemia from tumor lysis syndrome or phosphorus-containing bowel preparations may cause acute hypocalcemia. Chronic hyperphosphatemia associated with chronic kidney disease can also result in hypocalcemia. However, these conditions are not present and, in the case of chronic kidney disease, cannot account for the patient's rapid clinical deterioration. In contrast to surgery, hypoparathyroidism due to autoimmunity, radiation, or infiltrative processes develops slowly and serum calcium declines gradually over months to years, which is not consistent with the precipitous drop seen in this patient over 2 days.
A 68-year-old man is evaluated in the emergency department for a 4-week history of severe lower left leg pain, swelling, and redness. The pain is localized to the shin and ankle. He was previously diagnosed with cellulitis, but no improvement was seen with sequential courses of cephalexin and clindamycin. History is also significant for gout and atrial fibrillation. Medications are allopurinol, colchicine, and apixaban. On physical examination, vital signs are normal. The left lower shin is erythematous, warm, and tender to palpation; the erythema is not sharply demarcated. The left ankle is swollen, warm, and tender. There is no ulceration or skin breakdown over lower legs or feet. Solid masses at the extensor surfaces of both elbows are present. Laboratory studies show an erythrocyte sedimentation rate of 90 mm/h and a serum urate level of 6.8 mg/dL (0.40 mmol/L). Blood cultures performed 1 week ago were negative. Plain radiographs of the left shin and ankle show no changes suggestive of osteomyelitis. Aspiration of the left ankle yields no fluid. Which of the following is the most appropriate next step in management? Indomethacin Intravenous clindamycin MRI of the lower leg Prednisone Surgical debridement
Prednisone is appropriate treatment for gouty cellulitis accompanying an attack of acute gouty arthritis. The most appropriate next step in management is treatment with prednisone. This patient has gouty cellulitis accompanying an attack of acute gouty arthritis of the ankle. This well-recognized manifestation is often misdiagnosed as bacterial cellulitis and treated (unsuccessfully) with antibiotics. Prednisone should resolve both the acute gouty arthritis and gouty cellulitis; a typical dose is 40 mg/d for 5 days. NSAIDs such as indomethacin should be avoided in this patient who is anticoagulated with apixaban for atrial fibrillation. Antibiotics such as intravenous clindamycin would be indicated for bacterial cellulitis, for which there is no evidence. There is not an expanding area of skin involvement, and two previous courses of antibiotics have not improved symptoms. Additionally, cellulitis would not explain the inflamed ankle joint. An infected joint with concomitant cellulitis is unlikely because of the protracted time course. MRI of the lower leg would be appropriate to help define chronic osteomyelitis. This is unlikely, however, given the patient's lack of clear predisposing factors such as diabetes mellitus or an overlying skin ulcer, as well as normal plain radiographs after a month of symptoms. Surgical debridement is the initial approach for necrotizing fasciitis. Necrotizing fasciitis, however, has a more fulminant course. It is unlikely in this afebrile, nontoxic-appearing patient with normal vital signs, whose symptoms have not significantly worsened during the past 4 weeks.
A 52-year-old man is evaluated for a rapidly enlarging, painful ulcer on the leg. It started as a small red "pimple" approximately 2 weeks ago and has been expanding over the past week. Medical history is significant for ulcerative colitis. His only medications are sulfasalazine and folic acid. On physical examination, vital signs are normal. Skin findings are shown. Tissue culture is negative. Which of the following is the most appropriate initial treatment? Compression dressings Dapsone Prednisone Surgical debridement Topical hydrocortisone
Prednisone is the preferred initial treatment for pyoderma gangrenosum. This patient has pyoderma gangrenosum, and a glucocorticoid, such as prednisone, is the preferred initial treatment. Typically required in doses of 1 mg/kg, prednisone is often effective in controlling pyoderma. To protect from long-term side effects of prolonged glucocorticoid use, steroid-sparing agents, immunosuppressants, intralesional glucocorticoids, or high-potency topical glucocorticoids are frequently used. Pyoderma gangrenosum is commonly associated with inflammatory bowel disease, hematologic abnormalities, and malignancies. The disease predominately affects the lower legs, but can be seen at any location. The beginning sign is often a tender pustule or red papule or nodule. This will quickly erode into an ulcer. Classic cases will have a rolled purplish border and undermined edges. As the ulcer resolves, it tends to heal with atrophic scarring in a cross-like or cribriform pattern. The ulcer is extremely painful. The diagnosis is one of exclusion, and other causes of ulceration need to be ruled out. Skin biopsies are not diagnostic. Surface cultures will grow myriad bacteria, which are most likely colonizing the ulcer and not causative. Compression can be used in patients with extensive edema, but it is not a mainstay of therapy. Compression is often not tolerated because of pain. Good wound care with nonadherent dressings is important. Avoiding tape on the skin is necessary to prevent pathergy, the occurrence of lesions at sites of trauma, from the skin tape. Dapsone has been used with some success, likely because of its anti-neutrophil effects, but it is not used as first-line treatment of pyoderma. Pyoderma gangrenosum often occurs in the site of trauma (pathergy). For this reason, surgical debridement should be avoided unless a life-threatening infection is suspected. One exception is peristomal pyoderma gangrenosum. Surgical reanastomosis and closure of the ostomy site often results in healing of peristomal pyoderma. Relocating the stoma to another skin site will often result in resolution of the original peristomal ulcer; however, pyoderma will likely develop at the new ostomy site. Potent topical glucocorticoids have been used for small lesions, but topical hydrocortisone is of no benefit as it is a low-potency glucocorticoid. Patients with underlying disease (inflammatory bowel disease, hematologic malignancy, rheumatoid disease) often improve with therapy that is directed at the underlying disease. Active pyoderma in persons with underlying disease often means that the underlying disease is not adequately controlled.
A 37-year-old woman is evaluated for a headache lasting 1 day. She is in the third trimester of her first pregnancy. Until now, the pregnancy has been unremarkable, including blood pressure and urine protein measurements. Her only medication is a prenatal vitamin. On physical examination, blood pressure is 166/115 mm Hg; other vital signs are normal. There is no papilledema. Cardiac examination is normal. On abdominal examination, the patient has a gravid uterus consistent with her stage of pregnancy, and there is no abdominal tenderness. Laboratory studies: Hemoglobin 12.3 g/dL (123 g/L) Platelet count 70,000/µL (70 × 109/L) Alanine aminotransferase 72 U/L Aspartate aminotransferase 80 U/L Bilirubin Normal Creatinine 1.4 mg/dL (123.8 µmol/L) Electrolytes Normal Peripheral blood smear Normal Urinalysis 2+ protein Which of the following is the most likely diagnosis? Chronic hypertension Eclampsia Gestational hypertension HELLP syndrome Preeclampsia
Preeclampsia is defined by new-onset hypertension and proteinuria that occurs after 20 weeks of pregnancy; new-onset hypertension with new-onset end-organ damage (such as liver or kidney injury, pulmonary edema, cerebral or visual symptoms, or thrombocytopenia) are also diagnostic. The most likely diagnosis is preeclampsia. Preeclampsia is defined by new-onset hypertension and proteinuria (≥300 mg/24 h from a timed collection or ≥300 mg/g by urine protein-creatinine ratio) that occurs after 20 weeks of pregnancy. New-onset hypertension with new-onset end-organ damage (such as liver or kidney injury, pulmonary edema, cerebral or visual symptoms, or thrombocytopenia) is also diagnostic of preeclampsia. Severe preeclampsia is identified by persistent systolic blood pressure >160 mm Hg or diastolic blood pressure >110 mm Hg and end-organ damage. Thrombocytopenia, liver enzyme elevation, and kidney involvement are all present in this pregnant patient with new-onset hypertension, making preeclampsia the most likely diagnosis. Chronic hypertension is defined as a systolic blood pressure ≥140 mm Hg or diastolic pressure ≥90 mm Hg starting before pregnancy or before 20 weeks of gestation or persisting longer than 12 weeks' postpartum. In this case, the patient had normal blood pressures earlier in pregnancy. Eclampsia is the presence of generalized tonic-clonic seizures in women with preeclampsia, which are not present in this patient. Gestational hypertension first manifests after 20 weeks of pregnancy without proteinuria or other end-organ damage. Seen in 6% of pregnancies, gestational hypertension resolves within 12 weeks of delivery. Hypertension that persists beyond the 12 weeks is considered chronic hypertension. Of those who develop gestational hypertension, 15% to 25% progress to preeclampsia, and the rate increases to up to 50% in women who develop hypertension before 30 weeks. End-organ involvement in this case is inconsistent with gestational hypertension. HELLP (hemolysis, elevated liver enzymes, and low platelets) syndrome is a life-threatening state that complicates 10% to 20% of cases of preeclampsia. The cause of HELLP syndrome is unknown, but it may be related to placental factors. However, it likely represents a separate disorder from preeclampsia. The diagnosis requires the presence of microangiopathic hemolytic anemia, which is excluded by the normal bilirubin level and peripheral blood smear.
A 28-year-old woman is evaluated at 28 weeks' gestation. This is her first pregnancy. She has chronic hepatitis B virus (HBV) infection acquired through vertical transmission. The patient reports feeling well. Her only medication is a prenatal vitamin. On physical examination, vital signs are normal. The uterus is enlarged, consistent with 28-week intrauterine gestation. No stigmata of chronic liver disease are noted. Laboratory studies are positive for hepatitis B surface antigen and hepatitis B e antigen. The HBV DNA level is 300,000 IU/mL. The results of other studies, including alanine aminotransferase, aspartate aminotransferase, and total bilirubin levels, are within normal limits. Which of the following is the most appropriate next step in management? Cesarean delivery at term HBV DNA measurement in 3 months Pegylated interferon Tenofovir
Pregnant women who have hepatitis B virus DNA levels greater than 200,000 IU/mL at 24 to 28 weeks' gestation should be treated with tenofovir to prevent vertical transmission during delivery. Tenofovir is the most appropriate next step in management of this patient, with the goal of preventing vertical transmission of hepatitis B virus (HBV) infection from mother to child during the course of delivery. Guidelines recommend treatment with lamivudine, telbivudine, or tenofovir for the prevention of vertical transmission in pregnant women who have HBV DNA levels greater than 200,000 IU/mL at 24 to 28 weeks' gestation. There are no head-to-head data comparing these regimens, but tenofovir is preferred over telbivudine and lamivudine due to lower rates of resistance. Tenofovir and telbivudine are the only FDA pregnancy category B agents. Lamivudine and other oral drugs used to treat HBV are category C agents, though there are reasonable data on the safety of lamivudine use in pregnancy in the HIV population. Only a few patients will become hepatitis B surface antigen-negative with treatment; therefore, cure of HBV infection is an unrealistic goal for most chronically infected patients. There are no data suggesting that cesarean delivery is effective at preventing vertical transmission of HBV. All babies born to mothers with chronic HBV infection should receive active HBV vaccination and passive immunization (HBV immune globulin). The risk for developing chronic HBV infection is high (90%) in newborns who acquire HBV. Because the patient's HBV DNA level is high enough (>200,000 IU/mL) to warrant treatment to prevent vertical transmission of HBV, measuring her HBV DNA level again in 3 months would not be appropriate without first instituting treatment. Pegylated interferon is not considered safe in pregnancy and, therefore, would be an inappropriate choice for this patient. Pegylated interferon can be used to treat HBV infection in patients with high alanine aminotransferase levels, low HBV DNA levels, and without cirrhosis. Candidates for interferon have a desire for finite therapy, do not have cirrhosis, are not pregnant, and do not have significant psychiatric disease, cardiac disease, seizure disorder, cytopenia, or autoimmune disease.
A 32-year-old woman is evaluated for urinary frequency and dysuria. She is at 10 weeks' gestation with her first pregnancy. Medical history is notable for chronic immune thrombocytopenic purpura diagnosed 6 years ago. Her only medication is a prenatal vitamin. On physical examination, blood pressure is 110/60 mm Hg; vital signs are otherwise normal. She has a gravid uterus. No petechiae or ecchymoses are noted. Laboratory studies: Hemoglobin 12.2 g/dL (122 g/L) Leukocyte count 9800/µL (9.8 × 109/L) Platelet count 40,000/µL (40 × 109/L); normally 60,000-80,000/µL (60-80 × 109/L) Aminotransferase levels Normal Urinalysis Nitrite +, leukocyte esterase +, small blood; no protein A peripheral blood smear shows large and giant platelets but no clumps or schistocytes. In addition to treating this patient's urinary tract infection, which of the following is the most appropriate management? Begin intravenous immune globulin Begin prednisone Transfuse platelets Monitor platelet count
Pregnant women with immune thrombocytopenic purpura can be safely monitored during their pregnancy without intervention as long as they are asymptomatic and the platelet count remains greater than 30,000/µL (30 × 109/L). No management beyond monitoring her platelet count and treating her urinary tract infection is needed for this patient. Women with immune thrombocytopenic purpura (ITP) can be safely monitored during their pregnancy without intervention as long as they are asymptomatic and the platelet count remains greater than 30,000/µL (30 × 109/L). If the platelet count drops unexpectedly, other diagnoses, in addition to worsening ITP, must be considered, such as pre-eclampsia, HELLP syndrome (Hemolysis, Elevated Liver enzymes, and Low Platelets), thrombotic thrombocytopenic purpura (TTP), or hemolytic uremic syndrome (HUS). This patient's normal blood pressure, absence of proteinuria, and normal liver chemistry tests make pre-eclampsia and HELLP syndrome unlikely. The absence of schistocytes on her peripheral blood smear excludes TTP, HUS, and HELLP syndrome. Her currently decreased platelet count is probably related to her intercurrent infection. However, her platelet count should be monitored closely because it is close to the cut-off for treatment. Intravenous immune globulin is a first-line therapy option in treating ITP in pregnancy, but treatment is unnecessary when the platelet count is greater than 30,000/µL (30 × 109/L). This patient does not require therapy at this time. If treatment is needed, prednisone is another first-line option in ITP. Orofacial abnormalities have been reported in neonates when used in the first trimester, but prednisone is safe to use after the 12th week of gestation. However, this patient is only at 10 weeks' gestation, so prednisone would not be preferred if treatment were needed. Patients with ITP should not receive platelet transfusions because autoantibodies will cross-react with the transfused platelets, interfering with any potential improvement in platelet counts. However, their use in patients with severe bleeding is not disputed.
A 70-year-old man is seen for a preoperative medical evaluation before laminectomy for spinal stenosis. History is also significant for coronary artery disease and a non-ST-elevation myocardial infarction that occurred 5 years ago. He swims for 30 minutes every other day. He reports no chest pain with activity. Medications are aspirin, simvastatin, metoprolol, and lisinopril. Physical examination, including vital signs, is normal. Electrocardiogram performed 2 years ago demonstrated a left anterior fascicular block, several premature atrial contractions, and sinus rhythm. Which of the following is the most appropriate preoperative cardiac testing for this patient? Dobutamine stress echocardiography Electrocardiography Exercise stress testing No further testing
Preoperative electrocardiography is reasonable for patients with known atherosclerotic cardiovascular disease, including coronary artery disease, arrhythmia, peripheral artery disease, cerebrovascular disease, or significant structural heart disease, who are undergoing moderate- to high-risk surgeries; cardiac stress testing should generally be reserved for patients at elevated risk for major adverse cardiac event with a functional capacity less than 4 metabolic equivalents, but only if the results of the test will change perioperative management. The most appropriate preoperative cardiac testing for this patient is electrocardiography (ECG). The 2014 American College of Cardiology/American Heart Association guideline on perioperative cardiovascular evaluation and management of patients undergoing noncardiac surgery states that preoperative ECG is reasonable for patients with known atherosclerotic cardiovascular disease (including coronary artery disease, arrhythmia, peripheral artery disease, cerebrovascular disease, or significant structural heart disease) who are undergoing moderate- to high-risk surgeries. Preoperative ECG also may be considered for other asymptomatic patients, except for those undergoing low-risk procedures. This patient has known coronary artery disease and has not undergone ECG in 2 years. It is reasonable to obtain ECG preoperatively because certain interval findings (for example, new Q waves), additional evidence of conduction disease, or arrhythmia may result in changes in perioperative management. Risk calculators, including the Revised Cardiac Risk Index and the American College of Surgeons National Surgical Quality Improvement Program myocardial infarction and cardiac arrest calculator, can be used to determine the risk for a perioperative major adverse cardiac event (MACE). Patients with low risk (<1% risk for perioperative MACE) may proceed to surgery without preoperative cardiac stress testing, whereas patients with elevated risk (≥1% risk for perioperative MACE) should undergo assessment of functional capacity. Metabolic equivalents (METs) are used to represent the patient's functional capacity based on the intensity of activity able to be performed. If the patient's functional capacity exceeds 4 METs, the patient may proceed to surgery without further testing. Examples of 4 METs of activity include the ability to walk 4 miles per hour on a flat surface, climb one to two flights of stairs without stopping, or perform vigorous housework such as vacuuming. Swimming for 30 minutes also exceeds this threshold. Cardiac stress testing should generally be reserved for patients at elevated risk for MACE with a functional capacity less than 4 METs, but only if the results of the test will change perioperative management. Although this patient has an elevated risk for a MACE perioperatively, his functional capacity exceeds 4 METs; therefore, he does not require preoperative cardiac stress testing with dobutamine or exercise.
A 57-year-old man is evaluated prior to a right partial nephrectomy for renal cell carcinoma. He is asymptomatic. Medical history is significant for stage 3 chronic kidney disease, hypertension, hyperlipidemia, and degenerative joint disease. He has no history of abnormal bleeding and tolerated left knee arthroplasty 3 years ago without complications. Family history is negative for bleeding diatheses. Medications are lisinopril, hydrochlorothiazide, and simvastatin. On physical examination, temperature is normal, blood pressure is 125/75 mm Hg, pulse rate is 63/min, and respiration rate is 18/min. BMI is 29. Cardiac examination reveals a regular rate and rhythm with no murmurs. Lungs are clear to auscultation. There is no lower extremity edema, conjunctival pallor, abdominal mass or tenderness, or hepatomegaly. Which of the following is the most appropriate preoperative testing for this patient? Alanine aminotransferase and aspartate aminotransferase levels Prothrombin time and activated partial thromboplastin time Serum creatinine and electrolyte levels No preoperative testing
Preoperative measurement of serum electrolyte and creatinine levels is recommended in patients with kidney disease and those who are taking medications that may affect kidney function or predispose them to electrolyte abnormalities. This patient should undergo preoperative assessment of serum creatinine and electrolyte levels. Preoperative testing should be ordered selectively according to the patient's symptoms, medical history, medications, and physical examination findings. Serum creatinine and electrolyte levels should be measured preoperatively in patients with kidney disease and those who are taking medications that may affect kidney function or predispose them to electrolyte abnormalities, such as this patient, who has chronic kidney disease and is taking an ACE inhibitor (lisinopril) and diuretic (hydrochlorothiazide). In patients with chronic kidney disease undergoing surgery, electrolyte abnormalities should be corrected, and volume status should be optimized. Serum aminotransferases (alanine aminotransferase and aspartate aminotransferase) should not be routinely ordered in the absence of known liver disease, symptoms suggestive of underlying liver disease, physical examination findings suspicious for liver disease, or history of abnormal liver chemistry results. Coagulation testing is reserved for patients with a history of abnormal bleeding, those taking anticoagulants, and patients with medical conditions that predispose to coagulopathy (such as liver disease or hemophilia). For the patient with a history suggesting a bleeding disorder, preoperative prothrombin time, activated partial thromboplastin time, and platelet count measurements are indicated. Routine preoperative laboratory panels are not recommended because they expose patients to unnecessary testing, risk for incidental findings that are lost to follow-up evaluation, and increased anxiety. However, given this patient's comorbidities and medication use, forgoing preoperative testing would be inappropriate.
A 46-year-old man is evaluated for a 2-month history of right anterior knee swelling. The swelling began insidiously and has gradually worsened. It is now the size of a golf ball and interferes with his ability to kneel, which is vital to his job as a carpet layer. The knee is painful only when he kneels; he has no problems with knee motion or stability. He feels well otherwise. On physical examination, vital signs are normal. He has a 4-cm swelling on the anterior aspect of the right knee. The overlying skin is erythematous and warm. The knee exhibits full range of motion. There is no medial or lateral joint tenderness; knee joint effusion; or laxity with anterior, posterior, valgus, or varus forces. Which of the following is the most appropriate initial management? Activity modification Fluid aspiration Ibuprofen Plain radiography Serum uric acid measurement
Prepatellar bursitis can be caused by repetitive trauma, infection, or gout; fluid aspiration and subsequent analysis should be performed in all patients. This patient has prepatellar bursitis, and the first step in management should be fluid aspiration. Prepatellar bursitis is caused by inflammation of the prepatellar bursa that overlies the patella. Patients present with anterior knee pain and swelling. Physical examination reveals a palpable fluid collection with preserved active and passive range of motion of the knee. The most common cause of chronic prepatellar bursitis is repetitive trauma, as is likely the case in this patient, who must frequently kneel in his job as a carpet installer. Other causes include gout and infection. Most cases of acute prepatellar bursitis are infectious (typically related to skin bacteria), although trauma and gout are other potential causes. Regardless of the duration of the swelling, all patients with prepatellar bursitis should undergo fluid aspiration. Gram stain and culture of the bursal fluid should be obtained and analyzed for leukocyte count and for the presence of crystals to evaluate for the possibility of an underlying infectious cause and gout. Prepatellar bursitis due to repetitive trauma from kneeling is managed with activity modification (avoidance of kneeling), in addition to the use of oral NSAIDs such as ibuprofen. It would be inappropriate to recommend activity modification or NSAIDs before first analyzing bursal fluid to rule out infection or gout. Plain radiography is not usually required for the diagnosis of prepatellar bursitis. It may show soft-tissue swelling on lateral views but rarely aids in establishing the correct diagnosis. Plain radiographs should be obtained when knee osteoarthritis is suspected or there is concern for fracture, neither of which applies to this patient. Prepatellar bursitis caused by gout is diagnosed by bursal fluid analysis and detecting monosodium urate crystals with polarized microscopy. A high serum urate level supports the potential for gout; however, most patients with hyperuricemia do not have gout, and the serum urate level may be low during some acute attacks. Therefore, it would be inappropriate to obtain a serum uric acid level in this patient.
A 64-year-old man is evaluated in the emergency department for a 1-week history of exertional chest pain and onset of rest pain 3 hours ago. During the evaluation, he develops ventricular fibrillation. The patient undergoes rapid defibrillation. He is currently awake with ongoing chest pain. Medical history is significant for hyperlipidemia, hypertension, type 2 diabetes mellitus, tobacco use, and previous stroke. Medications are aspirin, lisinopril, metoprolol, metformin, and simvastatin. On physical examination, the patient is alert and oriented. Temperature is normal, blood pressure is 100/60 mm Hg, pulse rate is 110/min, and respiration rate is 24/min. Oxygen saturation is 98% breathing 4 L of oxygen by nasal cannula. Pulmonary examination reveals bibasilar crackles. An electrocardiogram demonstrates left bundle branch block, a new finding since electrocardiography 8 months ago. Which of the following is the most appropriate next step in management? Intravenous amiodarone Primary percutaneous coronary intervention Therapeutic hypothermia Urgent coronary artery bypass graft surgery
Primary percutaneous coronary intervention is the preferred method of reperfusion in patients with ST-elevation myocardial infarction. Primary percutaneous coronary intervention (PCI) is the most appropriate next step in management. ST-elevation myocardial infarction (STEMI) is caused by a complete occlusion of an epicardial coronary artery by a thrombus at the site of plaque disruption. It is defined by the presence of ischemic chest pain (or an equivalent) and the presence of greater than 1-mm ST-segment elevation in two or more consecutive leads or new left bundle branch block on electrocardiogram. This patient presumably has an anterior myocardial infarction, which is complicated by ventricular fibrillation arrest. The electrocardiogram demonstrates left bundle branch block and is assumed to be new in onset. In some cases, ventricular arrhythmias may signify spontaneous reperfusion events, but the new left bundle branch block is worrisome for persistent left anterior descending artery occlusion, which remains the obvious culprit for the patient's presentation. Prompt reperfusion is required. Primary PCI is the preferred method of reperfusion in patients with STEMI. Amiodarone may be necessary if this patient has recurrent ventricular fibrillation or ventricular tachycardia; however, amiodarone alone is unlikely to control his ischemic-related arrhythmias, and reperfusion is the foremost priority. Furthermore, intravenous amiodarone may precipitate hypotension and shock owing to its acute β-blocking activity, and it must be used carefully in patients with STEMI. Therapeutic hypothermia is an important adjunct therapy to improve outcomes of patients experiencing out-of-hospital cardiac arrest, although hypothermia protocols are not indicated in noncomatose patients. This patient's arrest was a witnessed arrest, and the patient is awake and conversant. Coronary artery bypass graft (CABG) surgery is not commonly performed for STEMI. Urgent CABG surgery is typically reserved for patients in whom thrombolytic therapy or PCI has failed, or for those who develop life-threatening mechanical complications, such as ventricular free wall rupture. Compared with PCI and thrombolytic therapy, urgent CABG surgery is associated with an increased death rate in the first 3 to 7 days after STEMI, and this potential harm must be carefully weighed against the potential benefits. A 2018 systematic review found that supplemental oxygen in the setting of normal peripheral oxygen saturation increases mortality in patients with acute myocardial infarction. A subsequent international guideline strongly recommends that oxygen therapy not be initiated for patients with an acute myocardial infarction and oxygen saturation as measured by pulse oximetry (SpO2) of 93% or higher. If supplemental oxygen is initiated, the guideline recommends not exceeding an SpO2 of 96%. This patient's oxygen saturation is 98%, and his supplemental oxygen should be reduced.
A 63-year-old woman is evaluated for increasing difficulties with communication at work. She teaches mathematics at a local high school. Her most recent work evaluation cited a growing inability to provide clear instructions when assigning homework. She continues to manage the home finances and to drive, cook, and shop without difficulty. On physical examination, vital signs are normal. The patient's speech pattern is fluent, but she has frequent problems with word finding and often repeats herself. She has difficulty understanding and following two-step commands. On the Montreal Cognitive Assessment, she scores 0/3 in the object-naming section, 0/1 in the letter-fluency section, and 4/5 in the recall section. All other physical examination findings are unremarkable. An MRI of the brain is shown. Which of the following is the most likely diagnosis? Alzheimer disease Behavioral-variant frontotemporal dementia Dementia with Lewy bodies Primary progressive aphasia
Primary progressive aphasia is a language-predominant neurodegenerative dementia in which language is often the only cognitive domain affected for years before the development of additional cognitive deterioration; structural brain imaging often shows asymmetric involvement of the left temporal lobe. The most likely diagnosis is language-variant frontotemporal dementia, also known as primary progressive aphasia (PPA). Frontotemporal dementia (FTD) comprises two distinct clinical syndromes. The first, behavioral-variant FTD, is associated with prominent changes in behavior, personality, or executive function. The second, PPA, is associated with prominent and early changes in language function and is further subclassified according to the pattern of language impairment: semantic (or fluent) aphasia, nonfluent (agrammatic) aphasia, and logopenic aphasia. Semantic aphasia, as demonstrated by this patient, is characterized by early prominent difficulty with language comprehension. Nonfluent aphasia is recognized by changes in language production. Logopenic aphasia describes hesitant but grammatically correct speech. Although language decline is common in many dementia syndromes, in PPA it is the symptom noted first, and language is often the only cognitive domain affected for years before the development of additional cognitive deterioration. Structural brain imaging typically reveals asymmetric involvement of the left temporal lobe (which controls language in most patients), as shown (arrow). Persistent forgetfulness is the hallmark of Alzheimer disease, beginning with objects, appointments, names, and other apparently trivial items. As the disease progresses, memories are lost, and problems with word-finding ability become apparent. The most prominent feature of behavioral-variant FTD is an alteration in personality and behavior that typically develops years before the onset of cognitive impairment. Apathy, diminished interest, loss of empathy, lack of initiative, increased emotionality, disinhibition, euphoria, impulsivity, changes in eating behaviors, and compulsiveness are the most common symptoms reported by families. Patients with dementia with Lewy bodies may demonstrate a pattern of cognitive impairment with psychiatric features of personality change, apathy, depression, and psychosis. Abnormal dream enactment behavior during rapid-eye movement (REM) sleep in which patients fight or call out in their sleep (REM sleep behavior disorder) may precede the onset of cognitive decline or motor symptoms by years.
A 55-year-old man is evaluated in the hospital for new-onset ascites. He has a history of cirrhosis due to hepatitis C viral infection. His only medication is propranolol. On physical examination, pulse rate is 58/min; other vital signs are normal. The abdomen is soft and distended consistent with ascites. Laboratory studies show a serum albumin level of 2.5 g/dL (25 g/L), serum total bilirubin level of 3.6 mg/dL (61.6 µmol/L), and serum creatinine level of 1.4 mg/dL (123.8 µmol/L). Paracentesis with analysis of ascitic fluid shows a leukocyte count of 200/µL with 30% neutrophils, albumin level of 0.4 g/dL (4 g/L), and total protein level of 0.9 g/dL (9 g/L). Which of the following is the most appropriate next step in management? Increase propranolol therapy Initiate albumin infusion Initiate ciprofloxacin therapy Initiate lisinopril therapy
Primary prophylactic antibiotic therapy is indicated for patients at high risk for the development of spontaneous bacterial peritonitis, including patients with very low ascitic-fluid protein levels and those with advanced liver failure. Initiation of indefinite primary prophylaxis with ciprofloxacin is the most appropriate next step in the management of this patient with ascites. Patients with ascites are at risk for developing spontaneous bacterial peritonitis (SBP), a common infection in patients with cirrhosis. SBP has a mortality rate of 20%. Long-term primary antibiotic prophylaxis may reduce mortality in patients at high risk for SBP. Criteria for patients at high risk include an ascitic-fluid total protein level less than 1.5 g/dL (15 g/L) in conjunction with any of the following: serum sodium level less than or equal to 130 mEq/L (130 mmol/L), serum creatinine level greater than or equal to 1.2 mg/dL (106.1 µmol/L), blood urea nitrogen level greater than or equal to 25 mg/dL (8.9 mmol/L), serum bilirubin level greater than or equal to 3 mg/dL (51.3 µmol/L), or Child-Turcotte-Pugh class B or C cirrhosis. Patients who have had a bout of SBP should also receive lifelong antibiotic prophylaxis to reduce the risk for recurrence. In the setting of variceal hemorrhage, a limited 7-day course of antibiotics initiated at the time of bleeding is indicated to prevent infectious complications from intestinal bacterial translocation. Increasing this patient's propranolol is not indicated because, although β-blocker therapy can reduce the risk for variceal bleeding, the patient already has a pulse rate of less than 60/min. Nonselective β-blockers such as propranolol may be associated with higher transplant-free survival in patients with cirrhosis overall but may decrease transplant-free survival in the first 6 months after SBP or in patients with refractory ascites, and discontinuation should be considered at that time. Albumin infusion may decrease the frequency of hepatorenal syndrome and improves survival in patients with SBP, but its role in primary prevention of SBP is undefined and its use for primary prevention not recommended. Systemic blood pressure decreases in patients with decompensated cirrhosis resulting in reductions in renal perfusion and glomerular filtration rate. This leads to elevated levels of vasopressin, angiotensin, and aldosterone. ACE inhibitors, such as lisinopril, and angiotensin receptor blockers impair these compensatory efforts to maintain blood pressure and can worsen kidney perfusion in the setting of ascites due to portal hypertension; therefore, initiating lisinopril is inappropriate in this patient.
A 30-year-old man is evaluated for a 6-week history of repeated episodes of sharp pain isolated to the right parietal region lasting less than 10 seconds. The pain is intense, begins and ends spontaneously without a clear trigger, and can recur multiple times each day, sometimes in series. The patient has a history of migraine with aura from age 6 through 12 years but has had no headaches since that time. He has had no associated autonomic features and no symptoms of migraine. He has a family history of migraine with aura. The patient takes no medication. All physical examination findings, including vital signs and those from a neurologic examination, are unremarkable. An MRI of the brain is normal. Which of the following is the most likely diagnosis? Cluster headache New daily persistent headache Primary stabbing headache Trigeminal neuralgia
Primary stabbing headache is characterized by transient localized stabs of head pain that occur spontaneously in the absence of organic disease and typically last seconds; this headache is common among those with a history of migraine
A 30-year-old man is evaluated for a 6-week history of repeated episodes of sharp pain isolated to the right parietal region lasting less than 10 seconds. The pain is intense, begins and ends spontaneously without a clear trigger, and can recur multiple times each day, sometimes in series. The patient has a history of migraine with aura from age 6 through 12 years but has had no headaches since that time. He has had no associated autonomic features and no symptoms of migraine. He has a family history of migraine with aura. The patient takes no medication. All physical examination findings, including vital signs and those from a neurologic examination, are unremarkable. An MRI of the brain is normal. Which of the following is the most likely diagnosis? Cluster headache New daily persistent headache Primary stabbing headache Trigeminal neuralgia
Primary stabbing headache is characterized by transient localized stabs of head pain that occur spontaneously in the absence of organic disease and typically last seconds; this headache is common among those with a history of migraine This patient most likely has a primary stabbing headache (also known as an "ice-pick" headache). The condition is characterized by transient localized stabs of head pain that occur spontaneously in the absence of organic disease and typically last seconds, with some lingering for 1 to 2 minutes; a less-localized dull ache or soreness lasting minutes may follow. The frequency of primary stabbing headache averages less than one per day in most patients, but headache attacks also can occur in series. Periods of activity and remission are common. Pain can occur anywhere on the head, including the eye, but the face is often spared. Primary stabbing headache is more common among those with a history of migraine. No cranial autonomic symptoms are reported. Indomethacin may be helpful during cycles of repeated occurrences but is rarely necessary. Cluster headache is characterized by episodes of severe pain localized to the periorbital or temporal area with associated ipsilateral cranial autonomic features. Attacks of cluster headache may last 15 to 180 minutes and recur one to eight times daily over a span of weeks to months; the shorter duration distinguishes cluster headache from migraine. Many of the attacks are nocturnal, and some may be provoked by alcohol ingestion. The brief duration, parietal location, and absence of autonomic features in this case make cluster headache unlikely. A new daily persistent headache is one with an identifiable and remembered onset. Pain becomes continuous and unremitting within 24 hours, a quality absent in this patient. The pain lacks many characteristic features, but the most common phenotype is similar to that of chronic tension-type headache. This disorder has no known treatment. This patient's pain is occurring in an extratrigeminal location incompatible with the diagnosis of trigeminal neuralgia. Of note, hemicrania continua is characterized by continuous head discomfort isolated and "locked" to one side. Some patients experience brief intense exacerbations of head pain that may be sharp and similar to that of primary stabbing headache. The condition, however, is known to display ipsilateral autonomic features not present in this patient, such as conjunctival injection, lacrimation, nasal congestion, rhinorrhea, forehead and facial sweating, miosis, and ptosis (and/or eyelid edema). An ipsilateral foreign body sensation in the eye is common. This headache is absolutely sensitive to indomethacin.
A 38-year-old woman seeks advice about decreasing her risk for ovarian cancer. Her mother and maternal grandmother were diagnosed with breast cancer at 40 years of age and 38 years of age, respectively. The patient tested positive for a deleterious BRCA2 gene mutation and had bilateral prophylactic mastectomies done at age 35 years. She has a 2-year-old son and would like to have at least one other child. She is premenopausal and takes no medications. There is no family history of ovarian cancer. On physical examination, vital signs are normal. Chest wall and gynecologic examination are normal. The remainder of the examination is normal. Which of the following is the most appropriate recommendation? Annual pelvic examination and serum CA-125 screening studies Bilateral salpingo-oophorectomy by age 40 to 45 years Bilateral salpingo-oophorectomy now Long-term oral contraceptive use
Prophylactic bilateral salpingo-oophorectomy is recommended for both BRCA1 and BRCA2 mutation carriers, which markedly reduces the risk for ovarian cancer and, if done while premenopausal, decreases the risk for breast cancer by 50%. The most appropriate recommendation for this patient is bilateral salpingo-oophorectomy by age 40 to 45 years. This patient has tested positive for a pathogenic BRCA2 mutation. Although she has no family history of ovarian cancer, she has a 12% lifetime risk for ovarian cancer. Prophylactic bilateral salpingo-oophorectomy (BSO) is strongly recommended for both BRCA1 and BRCA2 mutation carriers. Risk-reducing BSO markedly reduces the risk for ovarian cancer and, if done while premenopausal, decreases the risk for breast cancer by 50%. For BRCA1 mutation carriers, the risk for ovarian cancer increases substantially after age 35 years, and thus prophylactic BSO is recommended between the ages of 35 and 40 years and after completion of childbearing. BRCA1 carriers who delay BSO until age 40 years will have a 4% risk of being diagnosed with an ovarian cancer. For BRCA2 carriers, however, the risk of ovarian cancer before age 50 is only 1%. Therefore, it is reasonable to delay risk-reducing BSO until age 40 to 45 years in BRCA2 mutation carriers, particularly if they have already undergone prophylactic bilateral mastectomy. BSO can reduce the risk of ovarian, fallopian tube, or primary peritoneal cancers by 80% and decrease all-cause mortality to age 70 years by 77%. Because this patient desires further childbearing and is only 38 years old, she can safely delay BSO until age 40 to 45 years and until childbearing is complete. Screening with annual or semiannual pelvic ultrasonography and CA-125 is not effective and has been removed as a screening recommendation from BRCA1 and BRCA2 management guidelines. Oral contraceptive use decreases the risk for ovarian cancer by as much as 50% with prolonged use and can be used in BRCA1 and BRCA2 carriers but is not an adequate substitute for prophylactic BSO.
A 56-year-old man is hospitalized 8 hours after developing difficulty speaking and left-sided weakness. An emergent CT scan of the head shows an acute right thalamic hemorrhage with no hydrocephalus or intraventricular hemorrhage; a follow-up CT scan obtained 48 hours later shows no changes. The patient also has hypertension and type 2 diabetes mellitus. Medications before admission were lisinopril, aspirin, and glipizide. On hospital admission, hydrochlorothiazide and lisinopril are continued, aspirin and glipizide are discontinued, and insulin is initiated. On physical examination, blood pressure is 186/94 mm Hg, pulse rate is 74/min and regular, and respiration rate is 12/min. Left facial weakness with dysarthria and left-sided body weakness with no discernible movement are noted. An electrocardiogram shows sinus rhythm without acute changes. Which of the following is the most appropriate next step in management? Administration of low-molecular-weight heparin Administration of warfarin Application of thigh-high graduated compression stockings Resumption of aspirin
Prophylactic heparin can be started 48 hours after a hemorrhagic stroke to prevent development of deep venous thrombosis as long as there is no evidence of active bleeding on neuroimaging
A 56-year-old man is hospitalized 8 hours after developing difficulty speaking and left-sided weakness. An emergent CT scan of the head shows an acute right thalamic hemorrhage with no hydrocephalus or intraventricular hemorrhage; a follow-up CT scan obtained 48 hours later shows no changes. The patient also has hypertension and type 2 diabetes mellitus. Medications before admission were lisinopril, aspirin, and glipizide. On hospital admission, hydrochlorothiazide and lisinopril are continued, aspirin and glipizide are discontinued, and insulin is initiated. On physical examination, blood pressure is 186/94 mm Hg, pulse rate is 74/min and regular, and respiration rate is 12/min. Left facial weakness with dysarthria and left-sided body weakness with no discernible movement are noted. An electrocardiogram shows sinus rhythm without acute changes. Which of the following is the most appropriate next step in management? Administration of low-molecular-weight heparin Administration of warfarin Application of thigh-high graduated compression stockings Resumption of aspirin
Prophylactic heparin can be started 48 hours after a hemorrhagic stroke to prevent development of deep venous thrombosis as long as there is no evidence of active bleeding on neuroimaging This patient should begin receiving low-molecular-weight heparin. He has severe hemiparesis. Two poststroke CT scans obtained 48 hours apart show no evidence of active bleeding. The hemiparesis places him at high risk of deep venous thrombosis (DVT), which is the leading complication of hospitalization for stroke. DVT is found in 11% of immobile patients at 1 week after stroke and pulmonary embolism in about 1% of patients with stroke. Prophylaxis with low-molecular-weight heparin can be started 48 hours after hemorrhagic stroke in patients with no evidence of active bleeding on imaging. Before that time, DVT prophylaxis is possible with external sequential compression devices. Administering warfarin without heparin is not adequate for immediate and short-term prevention of DVT because the anticoagulant effect of warfarin is delayed for 36 to 72 hours. Because of the immediacy of its action, heparin is preferred. A randomized trial of thigh-high graduated compression stockings showed no benefit in preventing venous thromboembolism in patients with all types of stroke and substantial leg weakness. Skin breaks, ulcers, blisters, and skin necrosis were significantly more common in patients wearing stockings. Aspirin should not be resumed in this patient who has no immediate indication for use of antiplatelet agents, such as an acute myocardial infarction or a history of vascular stents. There is no evidence that aspirin is effective prophylaxis for DVT in hospitalized patients. Aspirin can be resumed in patients with intracerebral hemorrhage caused by hypertension 2 to 4 weeks after stroke onset if the hematoma has resolved on imaging and there is an appropriate high-risk indication.
A 25-year-old man undergoes follow-up consultation regarding a positive interferon-γ release assay. He reports working for the past year in Vietnam and having a negative tuberculin skin test before departing. He is asymptomatic. He has had no known exposure to anyone with a history of tuberculosis. He has otherwise been well and takes no medications. On physical examination, vital signs and examination are normal. HIV testing is negative. Posteroanterior and lateral chest radiograph is normal. Which of the following is the most appropriate treatment? Isoniazid and rifapentine once weekly for 24 weeks Isoniazid daily for 9 months Isoniazid, rifampin, pyrazinamide, and ethambutol for 8 weeks followed by isoniazid and rifampin for 4 months No treatment or testing
Prophylactic treatment with isoniazid is a recommended alternative treatment option for persons with latent tuberculosis infection, determined by a newly positive tuberculosis screening test but no signs or symptoms of active disease; 9 months of daily isoniazid can be self-administered. The most appropriate management for this patient is to initiate self-administered treatment with isoniazid. He has traveled to a high-risk area for Mycobacterium tuberculosis. Before departing for Vietnam, he had a negative tuberculin skin test; however, he returned with a positive screening result for tuberculosis. Because the chest radiograph is negative, he requires treatment for latent tuberculosis rather than treatment with the four-drug regimen for active tuberculosis. Several treatment options exist, and the regimen chosen should be based on comorbidities, knowledge of the drug-susceptibility data from the source patient, and possible drug interactions. Self-administered isoniazid for 6 or 9 months is approved as an alternative for the treatment of latent tuberculosis by the Centers for Disease Control and Prevention for patients with and without HIV infection. In patients being treated for latent tuberculosis who have normal baseline aminotransferase levels and no risk factors for liver disease, routine testing of aminotransferase levels during treatment is unnecessary. In 2020, the CDC recommended three preferred treatment regimens for latent tuberculosis: 3 months of isoniazid plus rifapentine, given once weekly; 4 months of rifampin, given daily; and 3 months of isoniazid plus rifampin, given daily. These regimens are preferred because of their effectiveness, safety, and high treatment completion rates. Isoniazid given daily for 6 or 9 months is now considered an alternative therapy for patients unable to take a preferred regimen. Once-weekly isoniazid and rifapentine only needs to be administered for 12 weeks, not 24 weeks. The 12-week regimen can be used instead of 9 months of isoniazid alone for healthy adult patients, including those with HIV coinfection who are not taking antiretroviral agents. It is not recommended for patients suspected of having rifampin- or isoniazid-resistant tuberculosis strains or for patients who are pregnant or plan to become pregnant while taking these agents. Treatment of active tuberculosis usually consists of multiple drugs for 6 to 9 months administered in two phases: initial and continuation. The core first-line antituberculous agents are isoniazid, rifampin, pyrazinamide, and ethambutol. These agents are administered for 8 weeks as part of the initiation phase. Isoniazid and rifampin are then continued for 4 or 7 months as part of the continuation phase. This patient does not have active tuberculosis (absent symptoms and a normal chest radiograph) and does not require therapy with four antituberculosis drugs. This patient has latent tuberculosis infection, as detected by the positive interferon-γ release assay. Providing no treatment would put him at risk of developing active disease or exposing close contacts who may then become ill.
A 62-year-old man is evaluated for a 4-month history of itching all over his body. He has no fatigue, weight loss, or night sweats. His appetite is good. Medical history is significant for hypertension treated with hydrochlorothiazide. He has no risk factors for HIV infection. On physical examination, vital signs are normal. There are a few scattered excoriations on the arms and lower legs. There are no other significant skin findings and no lymphadenopathy. Laboratory studies, including complete blood count with differential, erythrocyte sedimentation rate , liver chemistry tests, serum creatinine , and thyroid-stimulating hormone level, are within normal range. Which of the following is the most appropriate initial management? CT of the chest and abdomen Discontinue hydrochlorothiazide Start topical permethrin Start topical triamcinolone cream
Pruritus in the absence of skin findings should be evaluated for underlying systemic causes; many medications such as hydrochlorothiazide, calcium channel blockers, opiates, or NSAIDs can also cause generalized pruritus without skin findings. The most appropriate initial management for this patient is to discontinue hydrochlorothiazide. Pruritus in the absence of skin findings should be evaluated for underlying systemic causes. These include kidney disease, hyperthyroidism, hepatobiliary diseases, HIV infection, lymphoproliferative disorders (lymphoma), and myeloproliferative disorders (polycythemia vera). Drug-induced pruritus should also be considered. This patient has a normal complete blood count, liver chemistry tests, thyroid-stimulating hormone levels, serum creatinine, and sedimentation rate, making many of the systemic diseases unlikely. However, this patient is taking hydrochlorothiazide for hypertension. Hydrochlorothiazide, calcium channel blockers, opiates, and NSAIDs are all medications that can cause generalized pruritus without skin findings. In patients taking multiple medications, it can be challenging to determine which drug(s) may be contributing to pruritus, as patients may develop medication-induced itch even after months or years on a drug. The most appropriate management in this patient would be to discontinue hydrochlorothiazide and monitor for 2 to 4 weeks to detect improvement of his pruritus. Hodgkin lymphoma is the malignant disease most strongly associated with pruritus. CT of the chest and abdomen could be ordered to evaluate for Hodgkin disease, but he has no other symptoms to suggest lymphoma, such as fever or weight loss, and no lymphadenopathy on physical examination. The high cost of CT imaging and radiation exposure would make this an inappropriate initial management option for this patient. A plain chest radiograph would be a better initial imaging test if Hodgkin lymphoma were suspected. Topical permethrin is prescribed for the treatment of scabies. Although scabies should be considered in any patient with pruritus, there are no skin findings to suggest scabies in this patient, such as erythematous papulonodules and scaling patches, especially in web spaces, wrists, axillae, nipples, waistline, and genitals. Topical triamcinolone can be used for pruritus associated with inflammatory skin conditions, such as atopic dermatitis or psoriasis. This patient has no evidence of atopic dermatitis or psoriasis, such as inflamed, dry, red, itchy skin, or thick silvery scale on erythematous patches.
A 54-year-old man is evaluated in the emergency department for a 5-day history of fever, fatigue, and bleeding gums. He was previously feeling well. He takes no medications. On physical examination, the patient is pale and thin and appears chronically ill. Temperature is 39.0 °C (102.2 °F), blood pressure is 104/62 mm Hg, pulse rate is 108/min, respiration rate is 22/min, and oxygen saturation is 96% breathing ambient air. BMI is 22. Petechiae are present on the conjunctiva, forearms, and distal legs. Cardiac examination reveals tachycardia. There is no hepatosplenomegaly. There is no edema. Laboratory studies: Hemoglobin 8.8 g/dL (88 g/L) Leukocyte count 111,000/µL (111 × 109/L), 98% blasts Platelet count 28,000/µL (28 × 109/L) Creatinine 1.2 mg/dL (106.1 µmol/L) Electrolytes : Sodium 134 mEq/L (134 mmol/L) Potassium 6.4 mEq/L (6.4 mmol/L) Chloride 104 mEq/L (104 mmol/L) Bicarbonate 21 mEq/L (21 mmol/L) Electrocardiogram reveals sinus tachycardia but is otherwise normal. Which of the following is the most appropriate next step in management? Administer intravenous 0.9% saline Administer intravenous calcium gluconate Order a plasma potassium measurement Start inhaled albuterol Start sodium bicarbonate
Pseudohyperkalemia may occur in serum specimens when there are extreme elevations of leukocytes or platelets; repeat plasma potassium measurements can confirm a normal potassium level. The most appropriate next step in management is a repeat plasma potassium measurement. This patient has an elevated serum potassium level of 6.4 mEq/L (6.4 mmol/L) in the setting of leukocytosis likely due to acute leukemia. Hyperkalemia is defined as a serum potassium level >5.0 mEq/L (5.0 mmol/L). Any level >6.0 mEq/L (6.0 mmol/L) can be life-threatening. Because of the potential cardiac effects of hyperkalemia, immediate electrocardiography (ECG) is indicated. The earliest ECG changes of hyperkalemia are peaking of the T waves and shortening of the QT interval. As hyperkalemia progresses, there is prolongation of the PR interval, loss of P waves, and eventual widening of the QRS complexes with a "sine-wave" pattern that can precede asystole. This patient has no ECG evidence of hyperkalemia. Pseudohyperkalemia is therefore the most likely cause of his elevated serum potassium level. Pseudohyperkalemia occurs with a false rise in potassium values from cellular release of potassium during venipuncture due to hemolysis or prolonged tourniquet use. It also occurs with significant leukocytosis or thrombocytosis. During the clotting process, these cells are disrupted with the release of intracellular potassium, and pseudohyperkalemia may occur in serum specimens when there are extreme elevations of leukocytes or platelets. The next step is to obtain a plasma specimen and rapidly separate the plasma from the cells. Normally, the serum potassium is 0.3 to 0.4 mEq/L (0.3-0.4 mmol/L) greater than a plasma specimen obtained at the same time. In extreme cases of leukocytosis or thrombocytosis, the serum potassium can be more than 1.0 mEq/L (1.0 mmol/L) higher. In cases of pseudohyperkalemia, a rapidly processed plasma specimen will be normal. In patients with normal kidney function, intravenous saline, usually with the concomitant administration of a loop diuretic, can increase potassium excretion. Intravenous calcium should be reserved for cases of hyperkalemia exhibiting ECG changes. The effect of calcium on stabilizing the cardiac membrane is a transient event that lasts only 30 minutes. Inhaled albuterol lowers potassium by shifting it into cells. However, this patient has no indication to rapidly reduce potassium levels or stabilize the cardiac membrane because his ECG is normal. The use of sodium bicarbonate in hyperkalemia has fallen out of favor. It has minimal effects on potassium levels and is usually reserved for patients with metabolic acidosis.
A 52-year-old woman is evaluated in the emergency department for increasing redness, scaling, and itchiness of the skin. Over the last 2 days, it has expanded to cover most of her body. She complains of being cold and shivering. Her skin is flaking so badly she is embarrassed to go out in public. Medical history is significant for psoriasis since childhood and COPD for 5 years. She was treated for a COPD exacerbation last week with 5 days of 40-mg prednisone therapy. Medications are triamcinolone ointment, tiotropium, fluticasone/salmeterol, and albuterol as needed. On physical examination, temperature is 37.8 °C (100 °F), blood pressure is 118/70 mm Hg, pulse rate is 100/min, and oxygen saturation is 97% breathing ambient air. BMI is 32. The patient is acutely uncomfortable, covered in many blankets and shivering. Her skin is leathery, indurated, and hot to the touch. Skin findings are shown. There is active bleeding at a few sites where some scale has detached. Nail pitting is present on most of her fingernails. There are no conjunctival, oral, or genital lesions. Which of the following is the most likely cause of the patient's new symptoms? Drug reaction with eosinophilia and systemic symptoms (DRESS) Prednisone Sézary syndrome Stevens-Johnson syndrome
Psoriasis can flare to erythroderma following brief use of systemic glucocorticoids.
A 52-year-old woman is evaluated in the emergency department for increasing redness, scaling, and itchiness of the skin. Over the last 2 days, it has expanded to cover most of her body. She complains of being cold and shivering. Her skin is flaking so badly she is embarrassed to go out in public. Medical history is significant for psoriasis since childhood and COPD for 5 years. She was treated for a COPD exacerbation last week with 5 days of 40-mg prednisone therapy. Medications are triamcinolone ointment, tiotropium, fluticasone/salmeterol, and albuterol as needed. On physical examination, temperature is 37.8 °C (100 °F), blood pressure is 118/70 mm Hg, pulse rate is 100/min, and oxygen saturation is 97% breathing ambient air. BMI is 32. The patient is acutely uncomfortable, covered in many blankets and shivering. Her skin is leathery, indurated, and hot to the touch. Skin findings are shown. There is active bleeding at a few sites where some scale has detached. Nail pitting is present on most of her fingernails. There are no conjunctival, oral, or genital lesions. Which of the following is the most likely cause of the patient's new symptoms? Drug reaction with eosinophilia and systemic symptoms (DRESS) Prednisone Sézary syndrome Stevens-Johnson syndrome
Psoriasis can flare to erythroderma following brief use of systemic glucocorticoids. Psoriasis vulgaris can flare to erythroderma following use of systemic glucocorticoids. The most common cause of erythroderma, or redness over 80% body surface area, is a preexisting condition. Peripheral edema, erosions from excoriations due to severe pruritus, and scaling are common findings. Owing to compromise of the skin barrier, affected patients are at risk for dehydration, electrolyte abnormalities, protein loss, heat loss, and infection, which can be fatal. Short courses of systemic glucocorticoids in a patient with psoriasis can result in a striking increase in skin disease upon abrupt cessation. The presence of lamellar scale that bleeds when peeled away (Auspitz sign) and nail pitting support the diagnosis of erythrodermic psoriasis. Drug reaction with eosinophilia and systemic symptoms (DRESS) can also present with erythroderma. This represents a delayed response to a medication several weeks after initiation. Patients present with rash, striking facial edema, peripheral eosinophilia (or less common atypical lymphocytosis), lymphadenopathy, and evidence of organ involvement such as acute kidney injury or abnormal liver chemistry tests. Sézary syndrome is an acute leukemic variant of cutaneous T-cell lymphoma. Patients often present with erythroderma, ectropion, and lymphadenopathy. The absence of a preceding history of cutaneous T-cell lymphoma makes this diagnosis unlikely. Stevens-Johnson syndrome may cause erythroderma, but it manifests as epidermal necrosis with dusky plaques that slough rather than scale. Mucous membrane involvement is a keystone of diagnosis and is absent in this patient. Most cases of Stevens-Johnson syndrome are caused by a medication.
A 38-year-old man is evaluated for a 6-month history of dyspnea on exertion. He has gastroesophageal reflux disease and Raynaud phenomenon. He does not smoke and has no cough or wheezing. Current medications are lansoprazole and amlodipine. On physical examination, vital signs are normal. Oxygen saturation is 91% breathing ambient air. He has scattered telangiectasias on the face and trunk and sclerodactyly. Lung fields are clear on auscultation. The only abnormality on pulmonary function testing is a DLCO of 43% of predicted. High-resolution CT of the chest shows no evidence of parenchymal lung disease. Which of the following is the most likely diagnosis? Cryptogenic organizing pneumonia Lymphangioleiomyomatosis Lymphoid interstitial pneumonia Pulmonary arterial hypertension
Pulmonary arterial hypertension is commonly associated with connective tissue diseases, such as limited cutaneous systemic sclerosis. The most likely diagnosis is pulmonary arterial hypertension. Pulmonary involvement is frequent (greater than 70%) in patients with systemic sclerosis and can be symptomatic and disabling. The two principal clinical manifestations are interstitial lung disease and pulmonary vascular disease. Pulmonary vascular disease leading to pulmonary arterial hypertension may occur secondary to interstitial lung disease (typically in diffuse cutaneous systemic sclerosis) or as an isolated process (typically in limited cutaneous systemic sclerosis). This patient has several features of limited cutaneous systemic sclerosis with CREST syndrome (a condition defined by calcinosis cutis, Raynaud phenomenon, esophageal dysmotility, sclerodactyly, and telangiectasias). Patients are usually asymptomatic in early disease but later develop dyspnea on exertion and diminished exercise tolerance. Severe disease can lead to right-sided heart failure. Chest imaging is often normal and pulmonary function tests demonstrate a reduced DLCO with normal lung volumes. Organizing pneumonia is a patchy process that involves proliferation of granulation tissue within alveolar ducts, alveolar spaces, and surrounding areas of chronic inflammation. There are many known causes of this pattern, including acute infections and autoimmune disorders such as rheumatoid arthritis. The term cryptogenic organizing pneumonia (COP) is reserved for individuals who have this pattern but do not have a clear associated cause. Patients with COP will typically present with symptoms during 6 to 8 weeks that mimic community-acquired pneumonia. Evaluation typically demonstrates bilateral diffuse alveolar opacities on chest radiograph with normal lung volumes. This patient's presentation is not consistent with cryptogenic organizing pneumonia. Lymphangioleiomyomatosis is a multisystem disease that almost exclusively affects young women. Pulmonary complications are prominent and include diffuse pulmonary cysts, pneumothorax, chylous pleural effusions, and obstructive airways disease. Lymphangioleiomyomatosis would be very unlikely in a patient with an unremarkable chest radiograph. Lymphoid interstitial pneumonia is an interstitial lung disease characterized by lymphocytic infiltration of the pulmonary interstitium. It is observed in patients with Sjögren's syndrome and viral infections, especially HIV. Patients with lymphoid interstitial pneumonia often have crackles on the pulmonary examination, interstitial infiltrates on chest radiography, and decreased lung volumes and diffusing capacity on pulmonary function testing.
A 31-year-old woman undergoes preconception cardiac evaluation. Medical history is significant for tetralogy of Fallot with a palliative right subclavian artery-to-pulmonary artery (Blalock-Taussig) shunt in infancy and subsequent closure of the shunt and enlargement of the right ventricular outflow tract at 3 years of age. She is asymptomatic. On physical examination, vital signs are normal. The estimated jugular venous pressure is elevated with a visible a wave. The apical impulse is normal; a parasternal impulse is noted at the left sternal border. S1 is normal, and S2 is single, with a soft early systolic murmur at the second left intercostal space. A grade 2/6 diastolic decrescendo murmur is noted at the left sternal border that increases with inspiration. The remainder of the examination is normal. A chest radiograph is shown. Which of the following is the most likely diagnosis? Aortic regurgitation Mitral stenosis Pulmonary valve regurgitation Recurrent ventricular septal defect
Pulmonary regurgitation, presenting with a single S2, a parasternal lift, and a soft systolic pulmonary outflow murmur, is the most common structural disorder resulting from tetralogy of Fallot repair. The most likely diagnosis is pulmonary valve regurgitation, which is the most common structural disorder resulting from tetralogy of Fallot (TOF) repair. Clinical findings include features of right heart volume overload with a parasternal lift and a soft systolic pulmonary outflow murmur. A single S2 is heard because pulmonary valve function is sacrificed during TOF repair. TOF repair includes closure of the ventricular septal defect and relief of right ventricular outflow tract obstruction, which in turn results in pulmonary regurgitation, causing the diastolic murmur heard at the left sternal border that increases in intensity with inspiration. This patient's chest radiograph demonstrates evidence of previous sternotomy, a clip in the mediastinum from closure of the Blalock-Taussig shunt, a right-sided aortic arch (present in 25% of patients with TOF), and right heart enlargement (indicated by a rounded cardiac border and uplifted cardiac apex as seen on the frontal view and fullness in the retrosternal airspace [blue arrows] on lateral view). These chest radiograph findings are consistent with repaired TOF with pulmonary regurgitation resulting in right heart enlargement. Aortic regurgitation infrequently occurs late after TOF repair owing to progressive aortic enlargement. The diastolic murmur of aortic regurgitation is generally heard at the left sternal border and decreases in intensity with inspiration. A right ventricular prominence would not be expected in a patient with aortic regurgitation. Mitral stenosis is not expected to occur late after TOF repair. Physical examination findings include increased intensity of S1 and an apical diastolic murmur that increases during expiration. Recurrent ventricular septal defect can occur in patients following TOF repair. However, the physical examination findings would include a systolic murmur heard at the left sternal border, which often obliterates the S1 and S2. Right ventricular prominence would not be expected in a patient with a recurrent ventricular septal defect.
A 64-year-old man is evaluated during a follow-up visit for stable angina. He is currently taking metoprolol and wishes to discontinue this medication owing to side effects. He has normal left ventricular function and no symptoms of heart failure. Medical history is significant for hyperlipidemia, hypertension, and coronary artery disease treated with coronary artery bypass graft surgery. Medications are low-dose aspirin and maximum doses of lisinopril, metoprolol, isosorbide mononitrate, ranolazine, and atorvastatin. On physical examination, temperature is normal, blood pressure is 120/60 mm Hg, pulse rate is 70/min, and respiration rate is 18/min. Oxygen saturation is 98% breathing ambient air. The remainder of the examination is unremarkable. The patient is switched to diltiazem. Which of the following is the most appropriate management of this patient's medication regimen? Decrease ranolazine Increase atorvastatin Switch lisinopril to ramipril No medication adjustment
Ranolazine decreases symptoms of angina and modestly increases exercise times in patients with stable angina; it should not be used with strong CYP3A inhibitors, and dosage should be reduced when used in conjunction with moderate CYP3A inhibitors, such as diltiazem and verapamil. The most appropriate management is to decrease the dosage of ranolazine. This patient with stable angina is being transitioned from metoprolol, a β-blocker, to diltiazem, a calcium channel blocker. He is also currently taking several other antianginal therapies, notably ranolazine. Ranolazine decreases angina and modestly increases exercise times in patients with stable angina. Ranolazine inhibits the late sodium current, which in turn reduces sodium-dependent calcium currents, resulting in reduced wall tension and myocardial oxygen consumption. Ranolazine is primarily metabolized by cytochrome P450 3A4 (CYP3A4); therefore, caution should be exercised when prescribing ranolazine with CYP3A4 inhibitors, which will result in significantly increased plasma levels of ranolazine. Ranolazine should not be used with strong CYP3A4 inhibitors, such as ketoconazole, clarithromycin, and ritonavir. With moderate CYP3A4 inhibitors, including verapamil and diltiazem, the dosage should be decreased by 50% and should not exceed 500 mg twice daily. Ranolazine also has a modest QT-prolonging effect, but no proarrhythmic effect has been directly attributed to ranolazine. The addition of diltiazem to this patient's medication regimen will not require increasing the dosage of atorvastatin. Diltiazem may in fact result in increased levels of atorvastatin, which is a CYP3A4 substrate. There are no significant drug-drug interactions between lisinopril and diltiazem that would warrant switching to another ACE inhibitor. Not adjusting this patient's medication regimen would result in excessive plasma levels of ranolazine and is not an acceptable option.
A 59-year-old man is evaluated for recent cognitive problems resulting in worsening performance in his work as an electrician. He reports having increasing difficulty "running the numbers" when determining the materials needed for a job and increasing problems understanding the layout of plans when working on a new construction. He also notes occasional hand tremors when he works. According to his wife, his gait has slowed somewhat and he seems to have less of a response to things occurring around him in the past few weeks, stating that he "doesn't show any emotion in his face anymore." The patient also has a 5-year history of erectile dysfunction, a 4-year history of constipation, and episodes of lightheadedness when standing for long periods at work. On physical examination, vital signs are normal. A general paucity of movements is noted, as are slowness of movement in the upper and lower extremities and a mildly stooped posture when walking. He scores 20/30 (normal, ≥26) on the Montreal Cognitive Assessment, having significant difficulty with the cube drawing, clock drawing, and sustained attention portions of the test. Which of the following additional clinical findings would be most helpful in establishing the diagnosis in this patient? Delusions Depression Rapid eye movement sleep behavior disorder Repeated falls
Rapid eye movement sleep behavior disorder commonly occurs in dementia with Lewy bodies (DLB) and can help distinguish DLB from Alzheimer disease and other cognitive disorders. Rapid eye movement (REM) sleep behavior disorder commonly occurs in dementia with Lewy bodies (DLB) and can help distinguish DLB from Alzheimer disease and other cognitive disorders. This patient has cognitive and other symptoms suggestive of mild DLB. All the synucleinopathies (Parkinson disease, Parkinson disease dementia, and DLB) have a much higher rate of REM sleep behavioral sleep disorder than other neurologic and neurodegenerative diseases. REM sleep behavior disorder is characterized by the acting out of dreams secondary to loss of normal muscle paralysis during the dream phase of sleep. Symptoms may range from hand gestures to violent thrashing, punching, and kicking that may result in harm to self or bed partner. Patients also may have severe sensitivity to neuroleptic medications, which is more common in DLB than in other dementing illnesses. Delusions occur in both Alzheimer disease (especially the later stages) and DLB; frank hallucinations are more typical of DLB. Therefore, the presence of delusions is unlikely to help distinguish Alzheimer disease from DLB. Depression is a common symptom in most neurodegenerative dementia syndromes; it occurs in as many as 40% to 60% of patients with Alzheimer disease at some point of their disease. Its presence in this patient would not help establish a diagnosis. Repeated falls can occur in DLB but also in vascular cognitive impairment, normal pressure hydrocephalus, and the later stages of Alzheimer disease. Therefore, this finding is unlikely to help in establishing a diagnosis in this patient.
A 53-year-old woman is evaluated for an 8-month history of palpitations, tachycardia, lightheadedness, and sweating with explosive diarrhea and nausea within 15 to 30 minutes of eating. She has lost 6.8 kg (15 lb) during this time period and describes feeling better if she eats smaller portions. She reports no vomiting. Upper endoscopy performed 1 year earlier for refractory gastroesophageal reflux disease incidentally found a small gastrointestinal stromal tumor in her stomach, and she underwent a partial gastrectomy with gastrojejunostomy. Her only medication is omeprazole. On physical examination, vital signs are normal; BMI is 28. Abdominal examination shows a well-healed scar with no abdominal pain on palpation and no organomegaly. Which of the following is the most likely diagnosis? Dumping syndrome Gastroparesis Irritable bowel syndrome Small intestinal bacterial overgrowth
Rapid gastric emptying of hyperosmolar chyme into the small intestine after partial gastric resection can lead to postprandial vasomotor symptoms, abdominal pain, and diarrhea, collectively known as dumping syndrome. Dumping syndrome is the most likely diagnosis in this patient. Rapid gastric emptying of hyperosmolar chyme into the small intestine after partial gastric resection can lead to postprandial vasomotor symptoms known as dumping syndrome. This patient had a gastrojejunostomy 1 year earlier, and her symptoms started after the surgery. Common early symptoms of dumping symptoms are palpitations, tachycardia, diaphoresis, and lightheadedness with abdominal pain and diarrhea presenting within 30 minutes of eating. Late symptoms can occur 1 to 3 hours after eating in 25% of patients and include sweating, tremor, hunger, and difficulty with concentration and cognition (hypoglycemia). A minority of patients have both early and late symptoms. The diagnosis is usually made based on clinical findings. Initial treatment for dumping syndrome is dietary: eating small frequent meals and saving liquids until after a meal, which this patient has already found to help her symptoms. Dumping syndrome is misdiagnosed as gastroparesis in up to 37% of patients. Gastroparesis involves delayed gastric emptying and is typically associated with nausea and vomiting after eating. The prominent vasomotor symptoms this patient describes are uncommon in gastroparesis. Irritable bowel syndrome is more common in younger patients, and rarely causes weight loss. Symptoms that occur after a gastrointestinal surgery should suggest an alternative diagnosis. Small intestinal bacterial overgrowth (SIBO) is an excess number and alteration in type of bacteria cultured from the small intestine. Digestive enzymes and intestinal motility normally limit the growth of excessive bacteria, but SIBO can occur in conditions in which these functions are disrupted, including gastrojejunostomy, which creates a blind loop of small intestine. Clinical features of SIBO are diarrhea, bloating, and weight loss. Patients may have a combination of fat, protein, or carbohydrate malabsorption. Unlike in dumping syndrome, symptoms are not immediately related to eating and are not associated with prominent vasomotor symptoms such as palpitations, tachycardia, diaphoresis, and lightheadedness.
A 59-year-old woman is evaluated for an 8-month history of minimally productive cough. Her cough has persisted despite empiric treatment with intranasal and inhaled glucocorticoids, inhaled bronchodilators, a proton pump inhibitor, antihistamines, and decongestants. She reports no wheezing, heartburn, or nasal discharge. She is a lifelong nonsmoker and has no history of unusual environmental exposures. She has no known allergies and currently takes no medications. On physical examination, vital signs are normal. Tympanic membranes are clear. The remainder of the examination is unremarkable. Chest radiograph, spirometry, bronchial hyperresponsiveness testing, sinus CT scan, and esophageal pH monitoring are normal. Sputum analysis is negative for eosinophils. Which of the following is the most appropriate treatment Azithromycin Inhaled ipratropium Morphine Multimodal speech therapy
Recommended treatments for unexplained chronic cough include multimodality speech pathology therapy and neuromodulators, such as gabapentin.
A 72-year-old woman is evaluated in the hospital for a pneumothorax. The patient has severe, oxygen-dependent COPD complicated by several exacerbations. She was hospitalized 72 hours ago with abrupt onset of chest pain and dyspnea. Chest radiography confirmed the presence of a large left-sided pneumothorax and a thoracostomy tube was placed. She had 90% expansion of the lung following thoracostomy. On physical examination, the patient is frail appearing but comfortable. Vital signs are normal. Oxygen saturation is 96% breathing 3 L/min of oxygen through nasal cannula. Pulmonary examination reveals diminished but present breath sounds bilaterally. A left thoracostomy tube is in place. Chest radiograph demonstrates resolution of pneumothorax with a thoracostomy tube in place. Which of the following is the most appropriate management? Clamp thoracostomy tube Place thoracostomy tube to high suction Pleurodesis Remove thoracostomy tube
Recurrence prevention with pleurodesis is recommended after the first occurrence of secondary spontaneous pneumothorax. The most appropriate management of this patient with severe COPD and secondary pneumothorax is pleurodesis. Pneumothorax (air in the pleural space) can occur spontaneously, as a result of trauma, or iatrogenically. Spontaneous pneumothorax is further characterized as a primary spontaneous pneumothorax (PSP) in a person without underlying lung disease or a secondary spontaneous pneumothorax (SSP) in a person with underlying lung disease. Patients presenting with SSP are at higher risk for persistent air leak, further expansion of the pneumothorax, or pneumothorax recurrence due to their underlying lung disease. Intervention to prevent recurrence includes both chemical and mechanical pleurodesis, which is recommended in all patients with SSP and after the second occurrence of a PSP. In patients with SSP the cause of persistent air leak following pneumothorax is usually subpleural bullae or cysts. Additional interventions are required to close the leak. For patients who are surgical candidates, video-assisted thoracoscopic surgery (VATS) is recommended to locate and staple or resect blebs followed by mechanical pleurodesis. Patients who cannot tolerate surgery are treated with blood patch or chemical pleurodesis. These procedures are designed to seal the leak and prevent recurrence of pneumothorax. A blood patch is performed by injecting a quantity of the patient's blood into the thoracostomy tube. Chemical pleurodesis is performed by instilling tetracycline or one of its derivatives or specialized talc powder through the thoracostomy tube. Success rate for chemical pleurodesis ranges from 60% to 90% but it is not as effective as mechanical pleurodesis using VATS. Clamping or removing the thoracostomy tube would not be appropriate because the patient has a secondary spontaneous pneumothorax and the likelihood of recurrence is high. Placing the thoracostomy tube to high suction would not be appropriate because high levels of suction can increase the risk for reexpansion and pulmonary edema; in addition, the pneumothorax has resolved so no additional suction is needed.
A 48-year-old woman is evaluated for a 2-month history of increased dyspnea, wheezing, and nonproductive cough. She also reports intermittent pain and swelling in the wrists and knees for the past 6 months. She was diagnosed with scleritis of the left eye 1 month ago that improved with glucocorticoid drops. She also had two episodes of right pinna pain, redness, and swelling. Medications are prednisolone acetate ophthalmic and ibuprofen as needed for joint pain. On physical examination, vital signs are normal. There is no rash. Sclerae are normal. Hearing is normal. The trachea is tender. On lung auscultation, wheezing is heard in both lung fields with no crackles or rubs. Swelling of the wrists and knees is present. The ear findings are shown. Laboratory studies show an erythrocyte sedimentation rate of 60 mm/h and a normal complete blood count with differential except for a hemoglobin level of 11 g/dL (110 g/L). Chest radiograph is normal. CT of the chest reveals bronchial thickening with strictures; there is air trapping distal to the strictures. Which of the following is the most likely diagnosis? Cogan syndrome Polyarteritis nodosa Relapsing polychondritis Systemic lupus erythematosus
Relapsing polychondritis is characterized by inflammation and damage of cartilaginous tissues; tissues most commonly affected include the cartilaginous portions of the external and middle ear, nose, tracheobronchial tree, and joints. The most likely diagnosis is relapsing polychondritis (RP). RP is characterized by inflammation and damage of cartilaginous tissues; tissues most commonly affected include the cartilaginous portions of the external and middle ear, nose, tracheobronchial tree, and joints. Auricular involvement affects the helix but spares the earlobes. Nasal chondritis can result in collapse of the nasal bridge (saddle nose deformity), which can also be seen in trauma, granulomatosis with polyangiitis, cocaine use, congenital syphilis, and leprosy. Airway stenosis from tracheal ring involvement and aortitis/large-vessel vasculitis may occur and be life-threatening. RP is diagnosed by its typical clinical manifestations. Laboratory tests are nonspecific; acute phase reactants are elevated in 80%, and mild anemia is present in 44%. Patients with RP should undergo imaging (CT or MRI) to evaluate the large airways for inflammation/stenosis. This patient has a history of recurrent inflammation of the cartilage of the right ear, inflammatory eye disease, inflammatory arthritis, and bronchial strictures due to inflammation of bronchial cartilage, consistent with a diagnosis of RP. Cogan syndrome (interstitial keratitis, with cochlear and vestibular dysfunction) is an unlikely diagnosis because the patient has neither vestibular nor cochlear findings. Polyarteritis nodosa, a medium-vessel vasculitis, affects the mesenteric and renal arteries. Patients usually present with abdominal symptoms, neurologic involvement, and skin findings. Polyarteritis nodosa does not involve the eye or cartilage of the ear, nose, or airways, making this an unlikely diagnosis. Besides the inflammatory arthritis, this patient has no typical symptoms or signs of systemic lupus erythematosus (SLE). In addition, SLE does not typically cause inflammation and destruction of cartilaginous structures.
An 84-year-old woman is evaluated for dementia. She lives alone and is brought to the office by her son, who reports first noticing gradually progressive symptoms 4 years ago when his mother began having episodes of memory loss and a tendency to repeat herself excessively. He also notes increasing hygiene-related problems and says his mother recently called him several times in the middle of the night and asked him to pick her up "so she can go home." She can no longer manage her finances, keep appointments, or remember to take her medications; she stopped driving 1 year ago after getting lost several times. She has sick sinus syndrome and hypothyroidism treated with levothyroxine. On physical examination, blood pressure is 155/60 mm Hg and pulse rate is 55/min and intermittently irregular. She scores 17/30 (normal, ≥24) on the Mini-Mental State Examination, but all other findings from neurologic examination are unremarkable. Results of laboratory studies and brain imaging are normal. Which of the following is the most appropriate treatment at this time? Alprazolam Donepezil Memantine Paroxetine Risperidone
Relative contraindications for the use of cholinesterase inhibitors in the treatment of dementia include sick sinus syndrome, left bundle branch block, uncontrolled asthma, angle-closure glaucoma, and ulcer disease. This patient should receive memantine. She has probable Alzheimer disease, moderate stage, given the degree of functional impairment she exhibits and her Mini-Mental State Examination score. The treatment of Alzheimer dementia is multifactorial and is symptom targeted. Both cholinesterase inhibitors (such as donepezil) and the N-methyl-D-aspartate receptor antagonist memantine are approved for treatment of moderate stages of Alzheimer disease. The cholinesterase inhibitors exert their effect by inhibiting the enzymes responsible for breaking down acetylcholine, thereby increasing the levels of acetylcholine in the neuronal synapse. No clinically significant difference in effectiveness has been shown between the cholinesterase inhibitors. Relative contraindications for their use include (but are not limited to) sick sinus syndrome, left bundle branch block, uncontrolled asthma, angle-closure glaucoma, and ulcer disease. Memantine is believed to reduce glutamate-mediated neurotoxicity in the central nervous system, is not associated with adverse cardiovascular effects, and thus is the most appropriate treatment for this patient. Benzodiazepines (such as alprazolam) and antipsychotic agents (such as risperidone) should be used only in very limited circumstances in patients with Alzheimer disease who have severe psychosis and anxiety. At this point in her disease course, this patient has neither. In patients with mild to moderate Alzheimer disease, donepezil and other acetylcholinesterase inhibitors have had modest benefits in improving cognitive performance without clear improvements in daily functioning. Because of this patient's cardiac history, donepezil is not an appropriate medication. Selective serotonin reuptake inhibitors (such as paroxetine) can be helpful for improving mood and treating psychiatric and behavioral symptoms of dementia. This patient's symptoms are mild at this time, do not warrant use of this drug class, and should be treated with behavioral management.
A 22-year-old man is evaluated for acute onset of fever, chills, dyspnea, and nonproductive cough. He is a college student and spends summers using a combine to harvest wheat. He does not wear respiratory protective equipment. His symptoms start at the beginning of a work week, progress to the point that he must miss several days of work, and then the cycle begins again. On physical examination, temperature is 37.8 °C (100.1 °F), blood pressure is 120/80 mm Hg, pulse rate is 98/min, and respiration rate is 22/min. Oxygen saturation is 94% breathing ambient air. Lung examination reveals diffuse crackles. Chest radiograph demonstrates diffuse upper-lobe micronodular opacities. Which of the following is the most appropriate treatment? Counsel the patient not to return to work Inhaled glucocorticoids Pirfenidone Sirolimus
Removal of the offending antigen is the most appropriate treatment of acute hypersensitivity pneumonitis. This patient has acute hypersensitivity pneumonitis and should be counseled not to return to work. The acute form of hypersensitivity pneumonitis presents within 48 hours of a high-level exposure and will often be associated with fever, flu-like symptoms, cough, and shortness of breath. Radiographic imaging can demonstrate bilateral hazy opacities, whereas high-resolution CT imaging of the chest shows findings of ground-glass opacities and centrilobular micronodules that are upper- and midlung predominant. Symptoms typically wane within 24 to 48 hours after removal from the exposure. Recurrence of symptoms with exposure to the respiratory antigen is the hallmark of this disorder, and careful attention to the history will help identify the cause. Primary treatment of acute hypersensitivity pneumonitis is removal from the offending antigen. Studies indicate that pulmonary function can continue to gradually recover, with initial improvements in oxygen exchange followed by increased FVC and improved chest radiograph findings. Although treatment of acute hypersensitivity pneumonitis with systemic glucocorticoids is appropriate for those with more severe disease, patient response is variable, and prolonged use is associated with significant side effects. There are no data to support the use of inhaled glucocorticoids for acute hypersensitivity pneumonitis and, again, the primary treatment is to remove the offending agent. Idiopathic pulmonary fibrosis (IPF) is the most common idiopathic interstitial pneumonia. It occurs predominantly in older individuals; the diagnosis of IPF is rare in those younger than 50 years of age. Gradual onset of dyspnea and cough during months to years is typical. Pirfenidone is a novel therapeutic agent that regulates transforming growth factor β (TGF-β) and tumor necrosis factor α (TNF-α) activity through an unknown mechanism that is used in the treatment of idiopathic pulmonary fibrosis. Similarly, nintedanib is a tyrosine kinase inhibitor known to block pathways that lead to activation of the fibroblast. Although these therapies are an important step forward in the management of IPF, they are not curative. This patient does not have IPF and treatment with pirfenidone is not indicated. Lymphangioleiomyomatosis is a rare disorder that occurs sporadically in women or in association with tuberous sclerosis. It manifests as a diffuse cystic lung disease due to infiltration of smooth muscle cells into the pulmonary parenchyma. Genetic mutations within the cells lead to activation of the mechanistic target of rapamycin (mTOR) pathway. Diagnosis is based on imaging studies with diffuse thin-walled cysts as well as spontaneous pneumothorax and angiomyolipomas. Treatment is inhibition of the mTOR with sirolimus. Such treatment is not indicated in this patient with hypersensitivity pneumonitis.
A 26-year-old woman is evaluated during a follow-up visit for hypertension diagnosed 1 month ago. She is a marathon runner with previously normal blood pressure. Family history is significant for her mother who died of a ruptured cerebral aneurysm at the age of 50 years. Medications are lisinopril and an oral contraceptive. On physical examination, blood pressure is 146/92 mm Hg, and pulse rate is 59/min. A systolic-diastolic abdominal bruit that lateralizes to the left side is heard. There is no lower extremity edema. The remainder of the examination is unremarkable. Laboratory studies show a serum creatinine level of 1.4 mg/dL (123.8 µmol/L) (1 month ago: 0.8 mg/dL [70.7 µmol/L]). Urinalysis is normal with no blood, protein, or leukocyte esterase. A pregnancy test is negative. A 12-lead electrocardiogram is normal. Which of the following is the most appropriate diagnostic test to perform next? Plasma aldosterone concentration/plasma renin activity ratio Plasma fractionated metanephrines Renal artery imaging Transthoracic echocardiography
Renal artery imaging is the most appropriate diagnostic test to perform next to evaluate for fibromuscular dysplasia in this young woman with new-onset hypertension. Fibromuscular dysplasia is a nonatherosclerotic form of renovascular disease that usually affects the mid and distal portions of the renal artery. This disorder usually occurs in young persons, particularly young women, and abrupt onset of hypertension in a patient under the age of 35 years is suggestive. The elevation of serum creatinine more than 30% from baseline after the initiation of an ACE inhibitor is a clue that renovascular hypertension may be present. Fibromuscular dysplasia is associated with aneurysm and/or dissection in a variety of vascular territories (for example, renal artery, carotid artery, and intracranial arteries). This has resulted in a recommendation that patients with fibromuscular dysplasia undergo one-time, head-to-pelvic cross-sectional imaging. The importance of primary aldosteronism as a cause of hypertension is being increasingly recognized. Testing for primary aldosteronism should be considered in all patients with difficult-to-control hypertension. It should also be performed in patients with hypertension and an incidentally noted adrenal mass, spontaneous hypokalemia, or diuretic-induced hypokalemia. In these cases, the plasma aldosterone concentration/plasma renin activity ratio is obtained. Primary aldosteronism cannot account for this patient's renal bruit or serum creatinine elevation after the initiation of an ACE inhibitor. Plasma fractionated metanephrines are obtained to screen for a pheochromocytoma. The absence of episodic palpitations, headaches, and tachycardia as well as the presence of an abdominal bruit make pheochromocytoma a less likely diagnosis. Transthoracic echocardiography can be used for patients with clinical suspicion for white coat hypertension and/or to evaluate for end-organ damage. There are no pertinent findings from the history, physical examination, or electrocardiogram results to raise clinical suspicion for end-organ damage. More importantly, echocardiography will not be helpful in diagnosing the cause of this patient's hypertension or directing its treatment. Read Related TextNext Question
A 64-year-old woman is hospitalized for progressive fatigue, dyspnea, orthopnea, and peripheral edema that have been present for the last 18 months. During this time, she has been hospitalized several times for heart failure with preserved ejection fraction. Her medical history is otherwise notable for hypertension. Her current medications are spironolactone, furosemide, and amlodipine. On physical examination, temperature is normal, blood pressure is 136/84 mm Hg, pulse rate is 90/min, and respiration rate is 16/min. Jugular venous distention is present. Crackles are noted at the bases of both lungs. A loud S3 is heard at the apex. A grade 2/6 holosystolic murmur is heard at the left lower sternal border and increases in intensity during inspiration. Abdominal ascites and bilateral pitting edema to the knees are present. Laboratory studies reveal a serum ferritin level of 180 ng/mL (180 µg/L) and a B-type natriuretic peptide level of 560 pg/mL (560 ng/L). Serum protein electrophoresis and urine protein electrophoresis are unremarkable. Right heart catheterization demonstrates diastolic equalization of pressures at 18 mm Hg. Simultaneous right and left ventricular hemodynamics demonstrate concordant rise and fall of systolic pressures with respiration. Cardiac magnetic resonance imaging with intravenous gadolinium contrast shows a pericardial thickness of 2 mm without enhancement and marked late gadolinium enhancement of the papillary muscle. Echocardiogram shows symmetric left ventricular wall thickness of 11 mm, normal left ventricular cavity size, a left ventricular ejection fraction of 55%, and severe biatrial dilatation. Right ventricular size and function are normal. Tricuspid regurgitation is noted, and the right ventricular systolic pressure is estimated at 72 mm Hg. There is no pericardial effusion. Which of the following is the most likely diagnosis? Constrictive pericarditis Fabry disease Hemochromatosis Primary restrictive cardiomyopathy
Restrictive cardiomyopathy is distinguished from constrictive pericarditis by an elevated B-type natriuretic peptide level and concordant rise and fall of left and right systolic pressures with respiration. The most likely diagnosis is primary restrictive cardiomyopathy. Patients with constrictive pericarditis and restrictive cardiomyopathy present with similar symptoms and findings on echocardiography, and differentiation is an essential first step in diagnosis. In this patient, several features favor restrictive physiology: the elevated B-type natriuretic peptide level (often <100 pg/mL [100 ng/L] in constrictive pericarditis), the presence of severe pulmonary hypertension, the absence of pericardial thickening on cardiac magnetic resonance imaging, and the presence of delayed enhancement of myocardium consistent with myocardial fibrosis on cardiac magnetic resonance imaging. This patient's simultaneous right and left ventricular hemodynamic recordings demonstrate concordant rise and fall of systolic pressures with respiration. In constriction, an inverse relationship or discordance is present because of ventricular interdependence: right ventricular systolic pressure rises during inspiration coupled with a simultaneous decrease in left ventricular systolic pressure. Fabry disease is a lysosomal storage disorder. Clinical manifestations begin in childhood and progress in an orderly fashion, with up to 70% of the following manifestations appearing by age 40 years: neuropathic pain, telangiectasias and angiokeratomas, and proteinuria and chronic kidney disease. The most common cardiac complications are concentric left ventricular hypertrophy, heart failure, coronary artery disease, left-sided valvular dysfunction, and conduction abnormalities. This patient's late onset of cardiac symptoms and lack of other findings make Fabry disease unlikely. Although wall thickness may be normal in patients with hemochromatosis, the left ventricle is usually dilated, and the serum ferritin level would be greatly elevated in a patient with visceral involvement. Read Related TextNext Question
A 79-year-old man is evaluated for a 2-month history of progressively worsening headaches, nausea, visual disturbance, and difficulty speaking. He also has hypertension and gastroesophageal reflux disease. Medications are lisinopril and omeprazole. On physical examination, vital signs are normal. Right oculomotor nerve (cranial nerve III) and bilateral abducens nerve (cranial nerve VI) palsies are noted, as is right upper and lower facial weakness. An MRI of the brain shows a well-demarcated, homogeneously enhancing, hyperintense lesion that is suspicious for primary central nervous system lymphoma in the brainstem extending from the left pons to the medulla. Mass effect but no edema is present. The lesion obstructs the cerebral aqueduct, and hydrocephalus with enlargement of the lateral and third ventricles is noted. Which of the following is the most appropriate next step in management? Brain biopsy Intravenous dexamethasone Lumbar puncture Vitreous fluid sampling
Slit-lamp examination and vitreous fluid collection with cytologic analysis should be considered in patients with biopsy-inaccessible lesions who have suspected primary central nervous system lymphoma. This patient should undergo vitreous fluid sampling. He most likely has primary central nervous system lymphoma (PCNSL). PCNSL is a non-Hodgkin lymphoma that can affect any part of the central nervous system but commonly presents as a focal supratentorial lesion. Although PCNSL is commonly seen in patients with HIV infection, PCNSL is increasing in incidence among older, immunocompetent patients. An association with Epstein-Barr virus has been noted. Pathologic analysis, usually of a brain biopsy specimen, is required to make a diagnosis of PCNSL. Diffuse large B-cell lymphoma is typical. Cerebrospinal fluid (CSF) cytology can be diagnostic in 10% of patients, although repeated samples often are necessary. Ocular involvement in the vitreous or retina may be seen in 10% to 20% of patients and can be detected with a slit-lamp examination and confirmed by vitreous fluid sampling. In this patient, vitreous fluid collection with cytologic analysis may obviate the need for more invasive and potentially risky testing, such as brain biopsy. Despite being chemo- and radiosensitive, PCNSL typically recurs and has a generally poor prognosis. Because the lesion is in the brainstem, brain biopsy can result in significant neurologic injury and is considered a high-risk procedure. Brain biopsy should be considered only if other tests have been unrevealing, including cytologic analysis of vitreous fluid. When PCNSL is suspected, glucocorticoids should be avoided until the diagnosis is confirmed because they are lymphocytotoxic and can result in false-negative histopathologic or cytopathologic results. Although this patient has hydrocephalus, herniation and brain edema are not present, and thus glucocorticoid therapy may be delayed while the diagnostic evaluation is completed. Lumbar puncture is contraindicated in a patient with mass effect and hydrocephalus because it may precipitate herniation. If mass effect with hydrocephalus were not present, CSF sampling would be a reasonable step because lymphomatous cells may often be present in the CSF and would establish the diagnosis of PCNSL.
A 70-year-old man is seen for treatment recommendations for newly diagnosed colon cancer metastatic to the liver. He also has a long-standing history of diabetes and heart failure for which he takes metformin and lisinopril. The patient spends most of the day on the couch and is unable to walk across the room, dress himself, or do other basic activities of daily living without becoming dyspneic and fatigued. His appetite is poor. On physical examination, vital signs are normal. The patient is thin and has obvious muscle wasting. The liver edge is palpable. The examination is otherwise normal. The patient has read about an ongoing clinical trial of a chemotherapy regimen for metastatic colon cancer that resulted in a response rate of 40% and a progression-free survival of 11 months. Which of the following would this patient be likely to experience with the treatment given relative to the results reported in this clinical trial? Comparable efficacy and comparable toxicity Comparable efficacy but increased toxicity Decreased efficacy and comparable toxicity Decreased efficacy and increased toxicity
Results of clinical trials in patients with cancer, usually encompassing medically fit patients with good performance status, cannot be extrapolated to patients with poor health and performance status. The patient would most likely achieve efficacy results inferior to those in the published trial. Patients who participate in clinical trials typically have a good to excellent performance status, performing most of their normal activities of living. Patients who are sedentary, with limited exercise tolerance and limited caloric intake, are substantially less likely to benefit from treatment than patients who are more medically fit. Patients with a poor performance status, such as this one, would also be more likely to experience greater toxicity with aggressive treatment than would a more medically fit patient. Consequently, the results of clinical trials, which typically comprise medically fit patients with a good performance status, cannot be directly extrapolated to patients with poor health and poor performance status, such as this one.
A 38-year-old woman is evaluated for intermittent palpitations. She reports no other symptoms. Medical history is unremarkable, and she takes no medications. On physical examination, vital signs are normal. The estimated central venous pressure is elevated. Apical impulse is normal, a parasternal impulse is present at the left sternal border, and a soft systolic murmur is heard at the second left intercostal space. Fixed splitting of the S2 is noted throughout the cardiac cycle. The remainder of the physical examination is normal. An electrocardiogram demonstrates right-axis deviation and incomplete right bundle branch block. A chest radiograph shows right heart enlargement. A transthoracic echocardiogram demonstrates a 1.5-cm ostium secundum atrial septal defect. The right heart chambers are enlarged and the estimated pulmonary artery systolic pressure is normal. Which of the following is the best management? Cardiac magnetic resonance imaging Device closure of the atrial septal defect Endocarditis prophylaxis Measurement of functional aerobic capacity
Right heart enlargement and symptomatic disease are indications for device closure of an ostium secundum atrial septal defect. Percutaneous atrial septal defect (ASD) device closure is the best management choice for this patient depending on favorability of anatomic features. Device closure is performed in the catheterization laboratory and is an option for patients with an isolated ostium secundum ASD. Indications for ASD device closure include right heart enlargement and symptoms. Percutaneous device closure is also a reasonable option for asymptomatic patients with shunt-related hemodynamic consequences, for orthodeoxia-platypnea syndrome, and before pacemaker placement because of the increased risk for systemic thromboembolism. An asymptomatic small ASD with no right heart enlargement can be monitored without closure. Surgical closure is indicated for nonsecundum ASDs, large secundum ASDs, anatomy unfavorable for device closure, and a secundum ASD in patients with coexistent cardiovascular disease requiring operative intervention, such as coronary artery disease or tricuspid regurgitation. Cardiac magnetic resonance imaging can be used to detect a secundum ASD and can quantitate the effect of the defect by determining right ventricular volume; however, it is not indicated in this patient because the clinical assessment and echocardiogram are sufficient to guide the decision to close the ASD. Endocarditis prophylaxis is recommended for patients with congenital heart disease characterized by unrepaired cyanotic disease, including palliative shunts and conduits; a congenital defect during the first 6 months after complete repair of the defect with prosthetic material or device; and repaired disease with residual defects. Patients with an uncomplicated ASD without a history of endocarditis do not require endocarditis prophylaxis. Functional aerobic capacity measured by stress testing is not generally used to determine management in patients with ASDs.
A 53-year-old man is evaluated for a 5-year history of recurrent gout attacks involving the base of the great toes, mid feet, and ankles. Episodes are becoming more frequent and severe. History is also significant for hypertension and stage 3 chronic kidney disease. Medications are lisinopril and metoprolol. The patient is of Thai descent. On physical examination, vital signs are normal. There are no tophi or swollen joints. Laboratory studies show a serum urate level of 9.2 mg/dL (0.54 mmol/L). Which of the following is the most appropriate next step in management? Begin allopurinol Begin probenecid Measure antinuclear antibodies Order HLA-B*5801 allele testing
Risk factors for allopurinol sensitivity include diuretic use, chronic kidney disease, and the presence of the HLA-B*5801 allele in certain Asian ethnic groups. HLA-B*5801 allele testing is the most appropriate next step in management. This patient with gout is at high risk for allopurinol sensitivity. His risk factors include ethnicity (Thai descent) and chronic kidney disease. Patients of Thai, Han Chinese, and Korean descent have a higher likelihood of having the HLA-B*5801 allele, which confers a high risk for allopurinol sensitivity. Diuretic use is also a risk factor for allopurinol sensitivity. Allopurinol hypersensitivity is characterized by DRESS (drug reaction, eosinophilia, and systemic symptoms) syndrome and can result in kidney failure and death; allopurinol should be discontinued at the first sign of a rash. The presence of the HLA-B*5801 allele is a contraindication to prescribing allopurinol; therefore, this patient should be screened before beginning urate-lowering therapy. The absence of the HLA-B*5801 allele does not guarantee protection against allopurinol hypersensitivity. Febuxostat is a nonpurine, noncompetitive xanthine oxidase inhibitor that is an option for patients with mild to moderate chronic kidney disease and can be tried after an adverse reaction or failure with allopurinol. Importantly, in February 2019, the FDA mandated a boxed warning for febuxostat regarding the increased risk for cardiovascular death and all-cause mortality with the drug. The FDA has also limited the approved use of febuxostat for patients who are unresponsive to or cannot tolerate allopurinol. Probenecid should be avoided in patients with chronic kidney disease, as the drug requires intact kidney function. There is no suggestion of an antinuclear antibody (ANA)-related disease such as systemic lupus erythematosus, and testing for ANA should not be performed in patients with low pretest probability of disease. The presence of ANA does not correlate with the risk for drug hypersensitivity. Therefore, there is no indication for ANA testing.
A 66-year-old man is hospitalized for progressive dyspnea for the past 10 days. He was diagnosed with granulomatosis with polyangiitis 5 years ago; at that time, cyclophosphamide was initiated, with resolution of all symptoms. He was subsequently switched to maintenance azathioprine therapy. Today he was started on intravenous methylprednisolone. On physical examination, temperature is 37.8 °C (100.0 °F), blood pressure is 140/85 mm Hg, pulse rate is 100/min, respiration rate is 25/min, and oxygen saturation is 90% on 2 L of oxygen by nasal cannula. Bilateral crackles are heard in the lower lung fields. Laboratory studies: Erythrocyte sedimentation rate 90 mm/h Hemoglobin 12 g/dL (120 g/L) Leukocyte count with differential Normal Platelet count 450,000 (450 × 109/L) ANCA Positive with a cytoplasmic pattern and a titer of 1:160; positive proteinase 3 antibodies Urinalysis Normal Chest radiograph shows hazy opacification and a few nodules in the mid and lower lung zones. Sputum and bronchoalveolar lavage, Gram stain, and cultures are negative. Blood cultures are negative. In addition to discontinuing azathioprine, which of the following is the most appropriate treatment? Cyclophosphamide Etanercept Methotrexate Rituximab
Rituximab is appropriate treatment for patients who experience a relapse of granulomatosis with polyangiitis. Rituximab is the most appropriate treatment for this patient who has experienced a relapse of granulomatosis with polyangiitis (GPA), manifesting with pulmonary inflammation and nodules despite maintenance treatment with azathioprine. GPA is a rare systemic autoimmune disease of unknown cause that leads to vasculitis and granulomatosis of small- to medium-sized blood vessels. Without treatment, most patients die within 1 year. With treatment, most patients achieve remission within 6 months. Relapses of GPA are common, occurring in more than 50% of patients within 5 years, and may occur in different organs from the initial presentation. Both rituximab and cyclophosphamide are efficacious for initial induction therapy in patients with severe GPA. The choice of which therapy to use depends upon the individual patient presentation and comorbid conditions. The RAVE (Rituximab versus Cyclophosphamide for ANCA-Associated Vasculitis) trial demonstrated that rituximab is superior to cyclophosphamide in the subgroup of patients with relapse. In this study, remission without prednisone at 6 months was observed in 67% of rituximab-treated patients compared with 42% of cyclophosphamide patients (P = 0.01; NNT = 4). Rituximab was also as effective as cyclophosphamide in the treatment of patients with kidney disease or alveolar hemorrhage. There were no significant differences between the treatment groups with respect to rates of adverse events. In trials of etanercept treatment for GPA, patients manifested an increased risk for the development of solid malignancies, especially if they had previously been treated with cytotoxic drugs. Therefore, etanercept is not considered first-line treatment for GPA. Methotrexate is inadequate as induction therapy for severe disease; it can be used alone either as maintenance therapy after induction, or for mild and limited disease.
Definition of SJS, SJS-TEN, TEN?
SJS involves less than 10%, SJS-TEN overlap involves 10% to 30%, and TEN involves greater than 30%. TEN is almost exclusively caused by medications, whereas SJS can also be triggered uncommonly by vaccines or infection. Patients may have flu-like symptoms for 1 to 3 days prior to the skin eruption. Initially, painful red-purple macules or patches develop on the trunk and extremities, which enlarge and coalesce. Two or more mucosal surfaces, such as the eyes, nasopharynx, mouth, and genitals, are involved in more than 80% of patients. Systemic inflammation can result in pneumonia, hepatitis, nephritis, arthralgia, and myocarditis. Loss of the skin barrier function can lead to infection, hypovolemia, electrolyte disturbances, and death. Prognosis for SJS/TEN can be estimated by application of the SCORTEN assessment tool at 24 hours after presentation. All of the listed answer choices are features of the SCORTEN, but the patient's body surface area has the greatest impact on the score. SCORTEN is a severity-of-illness score validated for TEN. It incorporates blood Sugar (plasma glucose level >252 mg/dL [14.0 mmol/L]), presence of Cancer, Older age (>40 years), heart Rate (>120/min), Ten percent or more body surface area involvement on day 1, Electrolytes (serum bicarbonate <20 mEq/L [20 mmol/L]), and blood urea Nitrogen (>28 mg/dL [10 mmol/L]). Mortality is directly correlated with the number of SCORTEN variables that are fulfilled.
A 28-year-old woman is evaluated in the emergency department for muscle cramps and weakness. She notes a weight loss of 15 kg (33 lb) over the past 3 months; baseline weight was 115 kg (254 lb). She reports no abdominal pain or diarrhea. She has a 1-year history of type 2 diabetes mellitus, for which she takes metformin. On physical examination, temperature is normal, blood pressure is 122/72 mm Hg, pulse rate is 100/min, and respiration rate is 18/min. BMI is 36. Muscle strength of the lower and upper extremities is 4/5. Other than weakness, neurologic examination is normal. Laboratory studies: Electrolytes : Sodium 138 mEq/L (138 mmol/L) Potassium 2.4 mEq/L (2.4 mmol/L) Chloride 92 mEq/L (92 mmol/L) Bicarbonate 34 mEq/L (34 mmol/L) Arterial blood gases : pH 7.50 PCO2 45 mm Hg (6.0 kPa) Urine sodium 40 mEq/L (40 mmol/L) Urine potassium 60 mEq/L (60 mmol/L) Urine chloride 5 mEq/L (5 mmol/L) Which of the following is the most likely diagnosis? Cushing syndrome Gitelman syndrome Primary hyperaldosteronism Surreptitious vomiting
Saline-responsive metabolic alkalosis typically presents with hypovolemia and a low urine chloride of <15 mEq/L (15 mmol/L); the most common causes are vomiting, nasogastric suction, and diuretic use. The most likely cause of this patient's metabolic alkalosis is surreptitious vomiting. Metabolic alkalosis is diagnosed by an elevation in serum bicarbonate concentration. This disorder is caused either by a loss of acid or administration or retention of bicarbonate. Conditions that contribute to the maintenance of metabolic alkalosis include volume contraction, ineffective arterial blood volume, hypokalemia, chloride depletion, and decreased glomerular filtration. Laboratory evaluation of metabolic alkalosis is based on urine chloride concentration. Metabolic alkalosis is considered saline responsive when associated with true hypovolemia and responds to correction of the volume deficit with isotonic saline. Saline-responsive metabolic alkalosis presents with a low urine chloride of <15 mEq/L (15 mmol/L); the most common causes are vomiting, nasogastric suction, and diuretic use. Hypokalemia occurs secondarily due to aldosterone elevation and cation loss as the kidney attempts to lose bicarbonate. Although these patients are usually hypovolemic or normovolemic (with normal or low blood pressures), patients with preexisting chronic hypertension may present with high to normal blood pressures. For those who have a high urine chloride (>15 mEq/L [15 mmol/L]) with elevated blood pressure and hypokalemia and do not appear to be overtly volume overloaded, a mineralocorticoid excess disorder must be considered (saline-resistant metabolic alkalosis). Examples include Cushing syndrome and primary aldosteronism. Neither is a likely diagnosis based on the low urine chloride and absence of hypertension. Rarely, patients with metabolic alkalosis may appear to have clinical features consistent with saline-responsive metabolic alkalosis (normal/low extracellular fluid status, normal/low blood pressure) but have a urine chloride of >15 mEq/L (15 mmol/L). Diuretic use and inherited kidney disorders of sodium and chloride handling, such as Bartter and Gitelman syndromes, can mimic this presentation. These two autosomal recessive genetic disorders of renal sodium and chloride transporters clinically mimic loop diuretic and thiazide diuretic use, respectively. These diagnoses should be considered only after urine diuretic screening. Read Related TextNext Question
A 55-year-old man is evaluated during a follow-up visit after he was diagnosed with type 2 diabetes mellitus based on two hemoglobin A1c measurements of 7.8%. His medical history is significant for dyslipidemia and hypertension. Medications are aspirin, atorvastatin, and lisinopril. On physical examination, vital signs are normal. BMI is 33. The general physical examination, including nondilated eye examination, is normal. The patient will initiate therapeutic lifestyle modifications, and metformin will be started. In addition to spot urine albumin-creatinine ratio testing, which of the following screening tests should be done now? Comprehensive foot examination and dilated eye examination Fasting plasma glucose and 2-hour 75-g oral glucose tolerance test Serum B12 and folate concentrations 24-Hour urine protein and creatinine measurement
Screening for dyslipidemia, hypertension, a dilated eye examination, spot urine albumin-creatinine ratio, and a comprehensive foot examination should be performed at the time of diagnosis of type 2 diabetes. This patient requires a comprehensive foot examination and dilated eye examination. Patients with diabetes mellitus require monitoring for complications. A thorough foot examination should be done annually in patients with type 2 diabetes. Although this patient had an unremarkable visual foot inspection, he has not yet had a comprehensive foot examination, which is indicated at the time of diagnosis of type 2 diabetes and then annually thereafter. A comprehensive foot examination includes inspection of the skin, assessment of foot deformities, neurologic assessment (10-g monofilament testing with at least one other nerve assessment—pinprick, temperature, vibration, or ankle reflexes), and vascular assessment including pulses in the legs and feet. In patients with type 1 diabetes mellitus, a dilated comprehensive eye examination 5 years after diagnosis is appropriate; however, in patients with type 2 diabetes, a dilated comprehensive eye examination is indicated at time of diagnosis. Prediabetes and diabetes can be diagnosed based on the elevated results from one of the following screening tests repeated on two separate occasions: fasting plasma glucose, 2-hour postprandial glucose during an oral glucose tolerance test, or hemoglobin A1c. This patient has two abnormal hemoglobin A1c measurements and additional testing with a fasting plasma glucose test and oral glucose tolerance test for the purposes of diagnosis is not warranted. Although long-term use of metformin may be associated with vitamin B12 deficiency and periodic measurement of vitamin B12 levels should be considered in metformin treatment patients, B12 and folate measurements are not recommended at time of initiation of metformin therapy. Annual screening for albuminuria in patients with type 2 diabetes should be undertaken starting at the time of diagnosis; this is done on a random spot urine sample as planned in this patient, not a 24-hour urine collection for protein and creatinine. Read Related TextNext Question
A 42-year-old woman is evaluated in the office for osteoporosis because she was told her heel ultrasound screening test was abnormal at a health fair. She has no history of fractures. Family history is significant for osteoporosis in her mother, diagnosed at age 68 years; she has no history of fracture. Her only medication is a combination estradiol-levonorgestrel oral contraceptive pill. On physical examination, vital signs are normal. BMI is 19. The remainder of her physical examination is normal. Report of the quantitative heel ultrasound shows a Z-score of -0.5. Which of the following is the most appropriate management? Lifestyle counseling for osteoporosis prevention Dual-energy x-ray absorptiometry (DEXA) scan Evaluation of secondary causes of bone loss Serial quantitative heel ultrasound testing
Screening for osteoporosis in premenopausal women is not indicated in the absence of risk factors. The most appropriate management for this patient is lifestyle counseling for osteoporosis prevention. Lifestyle measures include adequate calcium and vitamin D, exercise, smoking cessation, counseling on fall prevention, and avoidance of heavy alcohol use. Most guidelines recommend screening for osteoporosis with dual-energy x-ray absorptiometry (DEXA) scan in women 65 years of age and older. Screening of younger women may be indicated if one or more risk factors for osteoporosis are present. In premenopausal women without risk factors, assessment of bone mineral density (BMD) for fracture risk is not advised or validated. However, if testing is done in an otherwise healthy person, such as this patient, results that are below age- and gender-matched averages (Z-score <0) generally do not require further evaluation or serial monitoring. The discovery of below average BMD could lead to a discussion regarding osteoporosis prevention with lifestyle modification and assessment of BMD after menopause, but prior to age 65, when screening might otherwise occur. Testing for secondary causes of bone loss is unnecessary when BMD is normal for age. Below average BMD in adults may in fact represent below average peak bone mass rather than loss of bone. Although BMD measured by quantitative heel ultrasound is predictive of osteoporotic fracture in older women and men, BMD measurement by DEXA scan remains the gold standard for diagnosis of osteoporosis and fracture risk assessment. Therefore, abnormal ultrasound results in these populations should be confirmed by DEXA scan. Even so, a DEXA scan should not be performed in this patient given that screening is not indicated and the heel ultrasound result is within the normal range.
A 32-year-old man is evaluated for decreased libido and fatigue. His symptoms have increased over the last 6 months. His medical history is otherwise unremarkable, and he takes no medications. On physical examination, vital signs are normal. BMI is 26. He has gynecomastia. Visual field acuity testing and testicular examination are normal. Smell is intact. Laboratory studies: Follicle-stimulating hormone 4 mU/mL (4 U/L) Luteinizing hormone 5 mU/mL (5 U/L) Prolactin 100 ng/mL (100 µg/L) Testosterone 110 ng/dL (3.8 nmol/L) Which of the following is the most appropriate diagnostic test to perform next? Karyotype analysis Pituitary MRI Screening for anabolic steroid abuse Serum ferritin measurement Sex hormone-binding globulin measurement
Secondary hypogonadism is characterized by low testosterone level and low or inappropriately normal serum luteinizing hormone and follicle-stimulating hormone concentrations; MRI of the pituitary is typically performed to evaluate secondary hypogonadism in the absence of obvious reversible causes such as drugs. The most appropriate test for this patient is a pituitary MRI. With a low testosterone level and low serum luteinizing hormone (LH) and follicle-stimulating hormone (FSH) concentrations, this patient has secondary hypogonadism. Hyperprolactinemia is the most likely cause of his hypogonadism; hyperprolactinemia leads to secondary hypogonadism through suppression of gonadotropin-releasing hormone synthesis and secretion. This patient is on no medications that might cause hyperprolactinemia. In the absence of a culprit drug, the most likely cause of his hyperprolactinemia is a lactotroph adenoma; therefore a pituitary MRI is indicated. A karyotype analysis is not indicated in the evaluation of secondary hypogonadism; however, it should be considered in men and women who have primary hypogonadism. Anabolic steroid use will result in low or normal gonadotropin levels, a low testosterone level, and clinical evidence of hyperandrogenism such as excessive muscle bulk, acne, gynecomastia, and decreased testicular volume. Anabolic steroid abuse, however, would not cause hyperprolactinemia as seen in this patient. Most patients with hereditary hemochromatosis are diagnosed in the presymptomatic phase when iron test results are abnormal. In patients with symptoms, clinical presentation varies and often includes nonspecific findings such as chronic fatigue, weakness, nonspecific abdominal pain, arthralgia, and elevated liver enzymes. Endocrine organs are commonly affected, and diabetes mellitus, hypothyroidism, and gonadal failure may occur. Laboratory evaluation of hemochromatosis-related gonadal failure most commonly demonstrates a hypogonadotropic state (low LH and FSH levels). While this patient is hypogonadotrophic, he has an elevated prolactin level, and a pituitary MRI is the best next diagnostic test. Obesity results in decreased concentrations of sex hormone-binding globulin (SHBG); if SHBG is low, free testosterone should be measured in patients with low total testosterone concentrations. This patient does not have obesity and has evidence of secondary hypogonadism (low testosterone level and low serum LH and FSH concentrations), making determination of SHBG concentration unnecessary.
A 73-year-old woman is hospitalized for an elevated serum creatinine level that has been unresponsive to intravenous fluids. She was evaluated in the emergency department 2 days ago for weakness, myalgia, arthralgia, and cough and admitted to the hospital. She has no other medical history and takes no medications. On physical examination, the patient is afebrile. Blood pressure is 155/95 mm Hg, pulse rate is 70/min, and oxygen saturation is 98% breathing 2 L of oxygen per minute by nasal cannula. Cardiac examination is normal, without evidence of jugular venous distention. Dullness to percussion and diminished breath sounds are present at the posterior lung bases bilaterally. There is pitting lower extremity edema. Laboratory studies: Hemoglobin 9.9 g/dL (99 g/L) Creatinine Baseline 6 months ago: 0.7 mg/dL (61.9 µmol/L); Emergency department: 4.1 mg/dL (362.4 µmol/L); Hospital day 1: 4.3 mg/dL (380.1 µmol/L) Antinuclear antibodies Negative Antimyeloperoxidase antibodies Positive Antiproteinase 3 antibodies Negative Urinalysis 3+ blood; 2+ protein Chest radiograph shows diffuse infiltrates at the lung bases bilaterally. Kidney biopsy shows necrotizing and crescentic glomerulonephritis with linear staining for IgG on immunofluorescence. Which of the following is the most appropriate diagnostic test to perform in this patient? Anti-double-stranded DNA antibodies Anti-glomerular basement membrane antibodies Anti-phospholipase A2 receptor antibodies Antihistone antibodies
Serologic testing for anti-glomerular basement membrane antibodies and kidney biopsy can confirm the diagnosis of anti-glomerular basement membrane antibody disease as the cause of rapidly progressive glomerulonephritis.
A 56-year-old man is seen during a routine evaluation for stage G4 chronic kidney disease (CKD). History is also significant for hypertension. Medications are losartan, labetalol, furosemide, and amlodipine. He has no symptoms and remains physically active. On physical examination, blood pressure is 129/76 mm Hg, and pulse rate is 68/min; other vital signs are normal. The physical examination is otherwise unremarkable. Laboratory studies: Hemoglobin 11 g/dL (110 g/L) Bicarbonate 19 mEq/L (19 mmol/L) Creatinine 3.1 mg/dL (274 µmol/L) Phosphorus 5.7 mg/dL (1.8 mmol/L) Potassium 5.1 mEq/L (5.1 mmol/L) The addition of which of the following will most likely slow progression of this patient's CKD? ACE inhibitor Erythropoiesis-stimulating agent Phosphate binder Sodium bicarbonate
Sodium bicarbonate therapy is the most likely treatment to slow progression of this patient's chronic kidney disease (CKD). Metabolic acidosis frequently occurs in patients with CKD due to defective acid excretion (resulting in reduced bicarbonate generation), most commonly due to impaired ammoniagenesis. Untreated metabolic acidosis can lead to muscle loss and bone loss. In early CKD, metabolic acidosis is typically a normal anion gap hyperchloremic metabolic acidosis. As glomerular filtration rate (GFR) declines, organic and inorganic anions are retained, and an anion gap metabolic acidosis develops. Large observational studies have shown a strong association between lower serum bicarbonate levels and both increased progression of CKD and mortality. Alkali therapy, most commonly sodium bicarbonate or sodium citrate, can delay the progression of CKD. Therefore, the Kidney Disease: Improving Global Outcomes (KDIGO) guidelines recommend starting alkali therapy when the serum bicarbonate is chronically <22 mEq/L (22 mmol/L). The alkali salt therapy dose should be titrated to achieve a serum bicarbonate level within the normal range; excessive alkali therapy in the setting of a reduced GFR may induce a metabolic alkalosis, which is associated with increased mortality. Patients with CKD who are treated with alkali should be monitored for symptoms of volume overload. Blood pressure control, particularly with renin-angiotensin system agents, has been shown to slow progression of proteinuric renal diseases and especially diabetes mellitus. However, this patient is already taking an angiotensin receptor blocker (ARB), and dual therapy with an ACE inhibitor and an ARB may instead be detrimental because studies of dual blockade have shown increased risks of acute kidney disease and hyperkalemia. As the estimated GFR declines below 30 mL/min/1.73 m2 (stages G4-G5), anemia can become symptomatic. Erythropoiesis-stimulating agents (ESAs) are highly effective in raising hemoglobin concentrations and alleviating symptoms but have not been shown to slow progression of CKD. These agents are associated with risks and are expensive. Black box warnings for use of ESAs include the risk of increased mortality and/or tumor progression in patients with active malignancy, increased risk of thromboembolic events for postsurgical patients not on anticoagulant therapy, and increased risk of serious cardiovascular events when ESAs are administered to patients with hemoglobin values >11 g/dL (110 g/L). Although poorly controlled hyperphosphatemia is associated with a greater risk of CKD progression, optimizing mineral metabolism parameters with a phosphate binder has not been shown to slow progression of CKD.
A 73-year-old woman is hospitalized for an elevated serum creatinine level that has been unresponsive to intravenous fluids. She was evaluated in the emergency department 2 days ago for weakness, myalgia, arthralgia, and cough and admitted to the hospital. She has no other medical history and takes no medications. On physical examination, the patient is afebrile. Blood pressure is 155/95 mm Hg, pulse rate is 70/min, and oxygen saturation is 98% breathing 2 L of oxygen per minute by nasal cannula. Cardiac examination is normal, without evidence of jugular venous distention. Dullness to percussion and diminished breath sounds are present at the posterior lung bases bilaterally. There is pitting lower extremity edema. Laboratory studies: Hemoglobin 9.9 g/dL (99 g/L) Creatinine Baseline 6 months ago: 0.7 mg/dL (61.9 µmol/L); Emergency department: 4.1 mg/dL (362.4 µmol/L); Hospital day 1: 4.3 mg/dL (380.1 µmol/L) Antinuclear antibodies Negative Antimyeloperoxidase antibodies Positive Antiproteinase 3 antibodies Negative Urinalysis 3+ blood; 2+ protein Chest radiograph shows diffuse infiltrates at the lung bases bilaterally. Kidney biopsy shows necrotizing and crescentic glomerulonephritis with linear staining for IgG on immunofluorescence. Which of the following is the most appropriate diagnostic test to perform in this patient? Anti-double-stranded DNA antibodies Anti-glomerular basement membrane antibodies Anti-phospholipase A2 receptor antibodies Antihistone antibodies
Serologic testing for anti-glomerular basement membrane antibodies and kidney biopsy can confirm the diagnosis of anti-glomerular basement membrane antibody disease as the cause of rapidly progressive glomerulonephritis. The most appropriate diagnostic test to perform next is anti-glomerular basement membrane (GBM) antibodies. This patient presents with a rapidly progressive glomerulonephritis (RPGN), an acute and steep rise in serum creatinine accompanied by hematuria and proteinuria. The differential diagnosis for RPGN is divided histologically into three patterns on immunofluorescence microscopy of the kidney biopsy: pauci-immune staining (for example, ANCA-associated glomerulonephritis), linear staining (for example, anti-GBM antibody disease), and granular staining (for example, lupus nephritis). The patient's positive antimyeloperoxidase (MPO) antibodies predict that she will have pauci-immune staining on the biopsy, but the linear staining on her biopsy is more consistent with anti-GBM antibody disease. Therefore, testing for anti-GBM antibodies is required to confirm this diagnosis, although treatment for her condition should not be delayed while awaiting results. One in three patients with anti-GBM antibody disease will have positive ANCA serologies, usually MPO antibodies (or p-ANCA in older assays). This combined seropositivity is most commonly seen in the subset of anti-GBM patients who are older women, as in this case. Diagnosing anti-GBM antibody disease could affect treatment decisions in this case, particularly whether to pursue plasmapheresis, which is indicated for all cases of anti-GBM antibody disease. For ANCA-associated glomerulonephritis, plasmapheresis is reserved for those with alveolar hemorrhage and/or severe kidney failure (defined as requiring dialysis or a serum creatinine >5.8 mg/dL [512.7 µmol/L]). This patient has negative antinuclear antibody testing and a biopsy not consistent with lupus nephritis, making testing for anti-double-stranded DNA antibodies unnecessary. Anti-phospholipase A2 receptor (PLA2R) antibodies can be checked in patients suspected of having primary membranous glomerulopathy, which typically presents with the nephrotic syndrome and preserved kidney function, not RPGN. Antihistone antibodies are measured if drug-induced vasculitis or drug-induced lupus erythematosus is suspected. This patient was not taking any medications, making this test unnecessary.
A 72-year-old man is evaluated during a follow-up office visit. Medical history is significant for prostate cancer managed with active surveillance since the age of 69 years. His only symptom is new-onset upper back pain. He takes no medication. On physical examination, vital signs are normal. The upper spine is tender to palpation. Laboratory studies show a serum calcium level of 9.9 mg/dL (2.5 mmol/L), serum creatinine level of 1.2 mg/dL (106.1 µmol/L), and prostate-specific antigen less than 4 ng/mL (4 µg/L). Whole body radionuclide bone scan shows focal increased uptake at T7. There are no other abnormalities. Spine radiograph shows coarsening of trabeculae and expansion of body of T7 without cortical disruption consistent with Paget disease of bone. Which of the following is the most appropriate management? Alkaline phosphatase measurement Bone mineral density testing Thoracic spine CT scan Zoledronic acid therapy
Serum alkaline phosphatase, a marker of increased bone turnover, should be measured after radiographic diagnosis of Paget disease of bone. The laboratory results and findings on radiographs are most consistent with Paget disease of bone. Measurement of serum total alkaline phosphatase is the next step after radiographic diagnosis and delineation of which bones are affected. It reflects the metabolic activity of Paget disease of bone at diagnosis and is used in follow-up evaluation whether treatment is given or not. Occasionally, total alkaline phosphatase may be normal in newly diagnosed patients with radiographic and radionuclide bone scan evidence of Paget disease. This quiescent or "burnt-out" stage of the disease does not require treatment. Many patients with Paget disease are older and may have risk factors for osteoporosis. These two conditions, however, are unrelated, have different treatment endpoints, and different bisphosphonate dosing. If bone mineral density were to be ordered, it would be for reasons independent of the evaluation and treatment of Paget disease. CT imaging of bones provides greater resolution and is superior to conventional radiographs in detecting and characterizing some metabolic bone disorders. With the exception of clarifying the extent of basilar skull involvement and risk for cranial nerve impingement, clinically relevant Paget disease is readily detected and diagnosed by conventional radiographs and radionuclide bone scans. Bisphosphonates, particularly intravenous zoledronic acid, are highly effective, often requiring a single dose to normalize alkaline phosphatase for several years in patients with Paget disease. Given the risk for spinal cord compression or compression fracture, involvement of T7 with Paget disease would be an indication for treatment unless the disease were metabolically inactive. Therefore, zoledronic acid would not be given prior to assessment of serum total alkaline phosphatase.
Tx for severe allergic contact dermatitis? Which of the following is the most appropriate treatment? 7-day course of oral acyclovir 10-day course of oral doxycycline 6-day taper of oral methylprednisolone 21-day taper of oral prednisone Topical hydrocortisone 1%
Severe allergic contact eruptions such as those from poison ivy may necessitate a 2- to 3-week taper of systemic glucocorticoids; because of the risk of rebound dermatitis, shorter courses are not recommended. This patient has severe contact dermatitis from poison ivy (Toxicodendron genus), and the most appropriate treatment is a 21-day taper of oral prednisone. There are two types of contact dermatitis: allergic and irritant. Allergic contact dermatitis is a type IV delayed hypersensitivity reaction. With repeated exposure, a pruritic eczematous dermatitis develops on the exposed area. In exuberant cases, the localized inflammation can lead to a secondary "id" reaction, a generalized acute cutaneous reaction in which pinpoint flesh-colored to red papules develop diffusely on the body. Irritant contact dermatitis is caused by a direct toxic effect on the epidermis from exposure to a chemical such as a cleaning agent, other caustic substances, or repeated wetting and drying, and it is not mediated by the immune system. Urushiol is an allergen in the Toxicodendron genus of plants. Examples of these plants include poison ivy, poison oak, and poison sumac. Typically this rash presents with geometric lines or splatters of red papules and vesicles, with surrounding erythema, especially on exposed areas. In severe cases involving multiple body areas or the face, systemic glucocorticoids are the best treatment. Two- to 3-week courses of glucocorticoids are preferred over shorter courses in order to decrease the risk of rebound dermatitis. A 7-day course of oral acyclovir is the treatment for herpes zoster. Herpes zoster typically presents with painful clusters of vesicles in a dermatomal distribution. The widespread clinical presentation of a pruritic vesicular rash is not consistent with herpes zoster. Oral doxycycline is an antibiotic often combined with incision and drainage for the empiric treatment of moderate purulent skin infections such as furuncles, carbuncles, and abscesses. Although allergic contact dermatitis can become secondarily infected with Staphylococcus aureus or streptococcus, there is no evidence of infection in this patient (for example, extensive and spreading erythema, warmth, drainage, and increasing pain). A 6-day taper of methylprednisolone would increase the risk of rebound dermatitis after completion of the short taper. A longer 14- to 21-day taper of systemic glucocorticoids is preferred. Hydrocortisone 1% is a low-potency topical glucocorticoid and would be ineffective for treating this severe case of contact dermatitis. High-potency glucocorticoids such as clobetasol propionate 0.05% or betamethasone dipropionate 0.05% can be helpful in early and localized disease.
A 24-year-old man is evaluated during a routine examination. He is asymptomatic. He is sexually active with men and has had multiple partners in the past year. He reports using condoms half of the time. Medical history is unremarkable. He takes no medications. On physical examination, vital signs and the remainder of the examination are normal. Results of combination HIV antibody immunoassay/p24 antigen testing are negative. Which of the following is the most appropriate interval for HIV screening in this patient? At least annually Every 2 years Every 3 years Every 5 years No further screening
Sexually active gay, bisexual, and other men who have sex with men and injection drug users should be screened for HIV infection at least annually. The most appropriate interval for HIV screening in this high-risk patient is at least annually. The U.S. Preventive Services Task Force (USPSTF) recommends HIV screening for patients aged 15 to 65 years in all health care settings. The currently recommended method for initial testing is combination HIV antibody immunoassay/p24 antigen testing. An "opt-out" approach is preferred; patients are notified that testing will be performed but can decline. Special consent for testing is not required. HIV testing provides a "teachable moment" to conduct HIV/sexually transmitted infection (STI) prevention counseling and offer pre-exposure prophylaxis for high-risk patients. However, prevention counseling should not be a required activity because it can be perceived as a barrier to screening. The USPSTF suggests that a reasonable approach is one-time screening of adolescent and adult patients and repeated screening of high-risk persons, persons engaged in risky behaviors, and those who reside in or receive medical care in high-prevalence settings for HIV infection. However, the USPSTF found insufficient evidence to determine appropriate or optimal time intervals or strategies for repeat HIV screening. Risk categories may change, and the USPSTF notes that rescreening may not be necessary for persons who have not been at increased risk since they were last tested. Women screened during a previous pregnancy should be rescreened in subsequent pregnancies. In 2015, the Centers for Disease Control and Prevention recommended that providers should offer HIV screening at least annually to all sexually active MSM. Clinicians can also consider the potential benefits of more frequent HIV screening (such as every 3 or 6 months) for some asymptomatic sexually active MSM based on their individual risk factors, local HIV epidemiology, and local policies. The CDC also recommends yearly testing for syphilis, gonorrhea, and chlamydia in MSM; in contrast, the USPSTF has found insufficient evidence to recommend screening for these diseases in men.
Features of WPW?
Short PR interval Slurred QRS complex (delta wave)
A 48-year-old woman is evaluated during a new-patient visit. She reports no symptoms. She is fairly sedentary but is trying to become more active by joining the local health club. She has noticed that she is "out of shape" but can cycle on a stationary bike with moderate intensity to the end of her 30-minute workout. Medical history is otherwise unremarkable. She takes no medications. On physical examination, vital signs are normal. The estimated central venous pressure is 6 cm H2O. The apical impulse is not palpable. Cardiac examination reveals a grade 2/6 midsystolic murmur localized to the left sternal border without radiation. The murmur does not change with respiration or handgrip but does diminish in intensity with standing. The S2 is physiologically split. There are no clicks. The lungs are clear to auscultation. Peripheral pulses are normal in volume and contour. No edema is present. Which of the following is the most appropriate management? Cardiac magnetic resonance imaging Transesophageal echocardiography Transthoracic echocardiography Routine clinical follow-up without imaging
Short, soft systolic murmurs (grade <3) that are well localized to the left sternal border and are not associated with symptoms often do not require further investigation. The most appropriate management is routine clinical follow-up without imaging. Cardiac murmurs result from increased blood flow across a normal orifice (such as with anemia, thyrotoxicosis, pregnancy, or atrial septal defect), turbulent flow through a narrowed orifice (such as with aortic stenosis or mitral stenosis), or regurgitant flow through an incompetent valve (such as with aortic regurgitation or mitral regurgitation). Timing in the cardiac cycle, chest wall location, radiation, intensity, configuration, duration, and pitch all assist in the differential diagnosis. Not all systolic murmurs are pathologic. Asymptomatic patients with short, soft systolic murmurs (grade <3) well localized to the left sternal border and associated with normal heart sounds do not usually require further investigation. Benign murmurs typically decrease in intensity with standing. The presence of any diastolic or continuous murmur, cardiac symptoms (chest pain, dyspnea, syncope), or abnormalities on examination (clicks, abnormal S2, abnormal pulses) requires evaluation by echocardiography. Various interventions may alter the intensity of murmurs. The murmur of hypertrophic cardiomyopathy may increase with standing or Valsalva maneuver; both maneuvers decrease venous return, which decreases left ventricular chamber size and increases the degree of obstruction. The click and murmur of mitral valve prolapse may move earlier in systole and increase in intensity as left ventricular volume decreases (standing or Valsalva maneuver). Aortic outflow murmurs increase in intensity in the beat following a premature ventricular contraction due to increased left ventricular volume. Murmurs of mitral regurgitation, ventricular septal defect, and aortic regurgitation increase with handgrip because of increased cardiac output and peripheral resistance. Right-sided heart murmurs may increase during inspiration due to increased venous return. Characteristics of the S2 may assist in determining the diagnosis or the severity of a valvular lesion. A fixed split of S2 (present during inspiration and expiration instead of only inspiration) results from a delay in right ventricular emptying and is strongly associated with atrial septal defect. A paradoxical split of S2 (present during expiration) indicates a delay in left ventricular emptying, such as with severe aortic stenosis. Presence of a physiologic split (present during inspiration) is helpful for excluding severe aortic stenosis. Because this patient likely has a benign systolic heart murmur and is asymptomatic, imaging with echocardiography or cardiac magnetic resonance imaging is not necessary.
A 32-year-old man undergoes preoperative consultation for a hip replacement. He has an 18-month history of hip pain secondary to avascular necrosis of the left hip. He also has sickle cell disease, requiring three to four hospitalizations per year for vaso-occlusive crisis. Medications are ibuprofen, hydroxyurea, and folic acid. On physical examination, vital signs are normal. Cardiac examination reveals a grade 2/6 systolic flow murmur. He has left hip pain with abduction. Laboratory studies show a hemoglobin level of 5.8 g/dL (58 g/L; baseline, 5-6.5 g/dL [50-65 g/L]). Which of the following is the most appropriate preoperative transfusion management for this patient? Exchange transfusion to 30% hemoglobin S Exchange transfusion to 50% hemoglobin S Simple transfusion to a target hemoglobin level of 10 g/dL (100 g/L) Simple transfusion to a target hemoglobin level of 12 g/dL (120 g/L) No transfusion
Simple transfusion to achieve a hemoglobin level of 10 g/dL (100 g/L) in patients having low- to moderate-risk surgery reduces surgical complications equivalent to exchange transfusion with less risk and cost. The patient should receive preoperative simple transfusion to achieve a hemoglobin level of 10 g/dL (100 g/L). Transfusions may have significant adverse effects, including infection, alloimmunization, and cumulative risk of iron overload. Therefore, persons with sickle cell disease (SCD) should not receive transfusions unless they have significant symptoms or signs of end-organ failure from their anemia or are preparing for surgery. Patients with SCD are at increased risk for perioperative pulmonary, infectious, and thrombotic complications. In 1994, a landmark study was published showing that simple transfusion to a target hemoglobin level of 10 g/dL (100 g/L) was equivalent to exchange transfusion in low- to medium-risk surgeries (low-risk surgeries include adenoidectomy and inguinal-hernia repair; medium-risk surgeries include cholecystectomy and joint replacement) in reducing surgical complications in patients with SCD with less risk, reduced cost, and increased convenience. More recently, guidelines have been published on the care of patients with SCD that strongly recommend simple transfusion to a target hemoglobin level of 10 g/dL (100 g/L) in patients requiring general anesthesia. In addition to transfusion, the use of incentive spirometry has also been shown in a randomized trial to improve surgical outcomes in patients with SCD. Overtransfusion should be avoided in these patients because of complications arising from increased blood viscosity. Typically, hemoglobin levels are kept at or below 10 g/dL (100 g/L). Exchange transfusion is used for acute stroke or retinal artery occlusion or in patients with severe acute chest syndrome. When indicated, the target hemoglobin S level should be less than 30%.
A 60-year-old woman is evaluated for easy bruising and bleeding after minor trauma, and a rash around her eyes. Medical history is unremarkable, and she takes no medications. On physical examination, vital signs are normal. The patient has several ecchymoses located primarily on her arms and legs. Lesions around her eyes are shown. Hepatomegaly is present. The remainder of the physical examination is unremarkable. Urinalysis demonstrates 4+ proteinuria but is otherwise unremarkable. Which of the following is the most likely diagnosis? Amyloidosis Dermatomyositis Hyperlipidemia Sarcoidosis
Skin manifestations of amyloid light chain amyloidosis are present in 30% to 40% of patients and include generalized waxy appearance, ecchymoses with minor pressure ("pinch purpura"), ecchymoses around the eyes ("raccoon eyes"), yellow waxy papules and plaques especially in a periorbital location, dystrophic nails, and macroglossia. This patient likely has amyloidosis. Amyloid light chain amyloidosis is the most common type of amyloidosis; it is a plasma cell dyscrasia-related disease characterized by end-organ damage secondary to tissue deposition of monoclonal free λ or κ light-chain fibrils. Clinical symptoms and manifestations vary and are dictated by the tissue tropism of the amyloidogenic light chain. If amyloid deposits are identified on tissue biopsy, other types of amyloidosis should be excluded by typing, which can be done by κ/λ light-chain immunohistochemistry. Major clinical manifestations include proteinuria with worsening kidney function, restrictive cardiomyopathy, and hepatomegaly. Neurologic findings include a symmetric, distal sensorimotor neuropathy, carpal tunnel syndrome, and autonomic neuropathy with orthostatic hypotension. Skin manifestations are present in 30% to 40% of patients and include generalized waxy appearance, ecchymoses with minor pressure ("pinch purpura"), ecchymoses around the eyes ("raccoon eyes"), yellow waxy papules and plaques especially in a periorbital location, dystrophic nails, and macroglossia. Bleeding caused by acquired factor X deficiency may also occur. A pathognomonic cutaneous feature of dermatomyositis is a heliotrope rash, a distinctive purple or lilac erythema of the eyelids that may be accompanied by edema. It is not yellow or papular as seen in this patient. Xanthelasma is a type of plane xanthoma localized to the periorbital area, most commonly on the upper medial eyelid, and it is characterized by soft, nontender, nonpruritic plaques. Xanthelasma can occur without hyperlipidemia, particularly in older persons, but it is often associated with familial dyslipidemias when seen in a younger person. This patient's periorbital lesions are not consistent with xanthelasma. Classic cutaneous sarcoidosis appears as violaceous papules around the nose including the ala, or periorbitally and around the oropharynx and nasal openings. This patient's yellow periorbital papules are not consistent with sarcoidosis. The combination of the patient's periorbital skin findings, easy bruising and bleeding, hepatomegaly, and proteinuria cannot be explained by dermatomyositis, hyperlipidemia, or sarcoidosis.
A 66-year-old woman comes to the office for a follow-up evaluation 1 month after being hospitalized with an ischemic stroke of the pons. Neurologic deficits have improved steadily in the intervening weeks, but the patient now reports having difficulty with outpatient rehabilitation because of a diminished energy level and fatigue in the absence of depressed mood, anhedonia, or loss of interest. She also has hypertension, dyslipidemia, and depression. Medications are aspirin, amlodipine, chlorthalidone, atorvastatin, and citalopram. On physical examination, vital signs are normal; BMI is 36. Right facial weakness and dysarthria are noted. Muscle strength is 4/5 in the right arm, 5/5 in the right leg, and 4/5 in the right ankle. The patient scores in the normal range on a two-point depression scale. Results of laboratory studies, including a basic metabolic panel, liver chemistry and thyroid function studies, and serum creatine kinase measurement, are normal. Which of the following is the most appropriate next step in management? Discontinuation of atorvastatin Fluoxetine initiation MRI of the brain Polysomnography
Sleep-disordered breathing is highly prevalent in patients with any form of stroke, and the best diagnostic test is polysomnography. This patient should undergo polysomnography. Sleep-disordered breathing is highly prevalent in patients with any form of stroke and is a leading cause of fatigue, headaches, and difficult-to-control hypertension in these patients. Given her recent stroke and elevated BMI, she is at particularly high risk for sleep-disordered breathing, which is best detected by polysomnography. Measurement tools, such as the Epworth Sleepiness Scale, also may help in patients with stroke but have not been well validated in this population. Treatment of sleep-disordered breathing can improve this patient's energy level, fatigue, and blood pressure. Discontinuation of atorvastatin would be reasonable only if the patient were experiencing significant adverse effects, such as myalgia, rhabdomyolysis, or persistently elevated liver transaminase levels. Atorvastatin is effective in reducing the risk of recurrent ischemic stroke and is a key component of secondary risk reduction in this patient. Fatigue may be a side effect of atorvastatin but would not prompt discontinuation of the medication. Fluoxetine should not be initiated in this patient, who already is taking another antidepressant medication and has no symptoms of depression other than fatigue. Although fluoxetine has been studied as an agent to promote motor recovery independent of its effect on depression in the acute stroke setting, the current evidence supporting this indication is lacking. An MRI of the brain may be indicated if there is concern for a specific intracranial structural process, such as a new stroke or an underlying intracranial mass. These processes, however, are usually associated with specific neurologic symptoms or focal neurologic findings on examination. Fatigue is a nonspecific symptom that is not attributable to a specific region of the brain and is unlikely to be the sole clinical manifestation of a new stroke or mass.
A 50-year-old man is evaluated during a routine follow-up visit. History is significant for chronic kidney disease, long-standing hypertension, and HIV infection. His antiretroviral regimen was recently adjusted to a once-a-day dosing, with the integrase inhibitor raltegravir discontinued and dolutegravir started 3 weeks ago. In addition to dolutegravir, current medications are abacavir, lamivudine, and lisinopril. Physical examination and vital signs are normal. Laboratory studies: Serum creatinine 1.5 mg/dL (132.6 µmol/L); baseline, 1.3 mg/dL (114.9 µmol/L) Urinalysis No blood, protein, or erythrocytes Urine albumin-creatinine ratio 100 mg/g (unchanged from baseline) Which of the following is the most appropriate next step in management? Discontinue lisinopril Measure a 24-hour urine creatinine clearance Reassess the serum creatinine level in 1 week No further assessment
Some medications (such as cimetidine, trimethoprim, cobicistat, dolutegravir, bictegravir, and rilpivirine) reduce proximal tubule secretion of creatinine, resulting in increases in serum creatinine that are nonprogressive; repeat serum creatinine measurement is required to confirm stable levels. Reassessing this patient's serum creatinine level in 1 week is the most appropriate next step in management. Some medications (such as cimetidine, trimethoprim, cobicistat, dolutegravir, bictegravir, and rilpivirine) reduce proximal tubule secretion of creatinine, resulting in increases in serum creatinine that are nonprogressive. This patient has chronic kidney disease (CKD), long-standing hypertension, and HIV infection. His antiretroviral medication regimen was recently adjusted to a once-a-day dosing, with the integrase inhibitor raltegravir discontinued and dolutegravir started 3 weeks ago. Dolutegravir is known to interfere with creatinine secretion without affecting glomerular filtration rate. Slight increases in serum creatinine of 0.2 to 0.3 mg/dL (17.7-26.5 µmol/L) may occur. This is more pronounced in those with preexisting CKD, in which creatinine secretion may contribute proportionately more to creatinine clearance. In patients with HIV taking the integrase inhibitors dolutegravir or bictegravir, the non-nucleoside reverse transcriptase inhibitor rilpivirine, or the pharmacokinetic enhancer (CYP3A inhibitor) cobicistat, serum creatinine elevations are nonprogressive and will remain unchanged within 1 to 2 weeks of initiation. Therefore, in the absence of other signs of kidney disease (hematuria, pyuria, or increasing proteinuria), reassessment of the serum creatinine level in 1 week will confirm the drug's effect in this patient. Further increases in serum creatinine levels will require additional evaluation. Discontinuation of the ACE inhibitor lisinopril is not necessary. This patient has long-standing hypertension and is likely benefiting from the lisinopril for his CKD and moderate albuminuria. It is unlikely to have caused this acute increase in serum creatinine because the hemodynamic effects of ACE inhibitors on glomerular filtration occur within days of initiating the medication and then stabilize. A 24-hour urine creatinine clearance will be decreased in this case (due to drug-related lower creatinine secretion) and will not provide additional diagnostic information. Should a repeat serum creatinine level be stable, no further evaluation will be necessary.
A 64-year-old man is hospitalized for persistent low-grade fever, chest discomfort, and lower extremity edema. He underwent triple coronary artery bypass graft surgery 6 weeks ago. His medical history is otherwise notable for hypertension, hyperlipidemia, and previous smoking. His current medications are aspirin, metoprolol succinate, atorvastatin, lisinopril, and furosemide. On physical examination, temperature is 37.7 °C (99.8 °F), blood pressure is 136/84 mm Hg, pulse rate is 96/min, and respiration rate is 14/min. Jugular venous distention is noted to the angle of the mandible. Decreased breath sounds and dullness to percussion are noted at the left base. No crackles are noted. A pericardial rub is present. There is no gallop. The liver is enlarged. There is bilateral pitting edema to the mid shin. Laboratory studies: Erythrocyte sedimentation rate 76 mm/h Leukocyte count 12,000/µL (12 × 109/L) Serum creatinine 1.4 mg/dL (123.8 µmol/L) Troponin (two samples) Normal B-type natriuretic peptide 105 pg/mL (105 ng/L) A 12-lead electrocardiogram demonstrates normal sinus rhythm with nonspecific ST-T-wave abnormalities. A chest radiograph demonstrates bilateral small pleural effusions. An echocardiogram shows no pericardial effusion. A Doppler echocardiogram shows enhanced ventricular interdependence and dilation of the inferior vena cava. Right heart catheterization demonstrates equalization of diastolic pressures in all heart chambers. Gadolinium-enhanced cardiac magnetic resonance imaging demonstrates increased pericardial thickness and evidence of active pericardial inflammation and edema. Which of the following is the most likely diagnosis? Heart failure with preserved ejection fraction Perioperative graft failure with infarction Restrictive cardiomyopathy Transient constrictive pericarditis
Some patients with constrictive pericarditis can have reversible or transient pericardial inflammation that responds to anti-inflammatory agents; most cases are idiopathic, whereas others may be associated with recent cardiac surgery. The most likely diagnosis is transient constrictive pericarditis. This patient has signs of right-sided heart failure accompanied by fever, leukocytosis, an elevated erythrocyte sedimentation rate, and a pericardial friction rub. The echocardiogram demonstrates enhanced ventricular interdependence and equalization of diastolic pressures in all heart chambers. Cardiac magnetic resonance imaging demonstrates pericardial thickening with evidence of active inflammation. These features are consistent with the diagnosis of transient constrictive pericarditis. Transient constrictive pericarditis most often is idiopathic but may follow cardiac surgery. Initial therapy consists of an NSAID plus colchicine. Patients with chronic constrictive pericarditis are sometimes misdiagnosed as having heart failure with preserved ejection fraction. Although this patient has evidence of right-sided heart failure and has a preserved ejection fraction, Doppler echocardiographic findings of ventricular interdependence and the relative low B-type natriuretic peptide level argue against heart failure with preserved ejection fraction. Patients with perioperative graft failure with infarction would be more likely to demonstrate pathologic findings of infarction on electrocardiogram, and serum troponin levels may remain elevated. Although it may be challenging to differentiate restrictive cardiomyopathy from pericardial constriction, the lack of features of heart failure until after coronary artery bypass graft surgery makes the diagnosis of restrictive cardiomyopathy unlikely. Additionally, patients with restrictive cardiomyopathy would not demonstrate evidence of enhanced ventricular interdependence, and the B-type natriuretic peptide level is typically greater than 400 pg/mL (400 ng/L).
A 58-year-old man is evaluated for resistant hypertension. He was first diagnosed with hypertension 10 years ago, and his blood pressure has been increasingly difficult to control. Testing for secondary causes of hypertension will be undertaken. Medical history is otherwise unremarkable. Medications are lisinopril, spironolactone, hydrochlorothiazide, and metoprolol. On physical examination, blood pressure is 149/93 mm Hg. Other vital signs are normal. BMI is 29. The remainder of the physical examination is unremarkable. Which of the following should be discontinued prior to screening for secondary causes of hypertension? Hydrochlorothiazide Lisinopril Metoprolol Spironolactone
Spironolactone and eplerenone can significantly interfere with interpretation of the plasma aldosterone-plasma renin ratio (ARR) and therefore should be discontinued approximately 6 weeks prior to screening for primary aldosteronism.
A 58-year-old man is evaluated for resistant hypertension. He was first diagnosed with hypertension 10 years ago, and his blood pressure has been increasingly difficult to control. Testing for secondary causes of hypertension will be undertaken. Medical history is otherwise unremarkable. Medications are lisinopril, spironolactone, hydrochlorothiazide, and metoprolol. On physical examination, blood pressure is 149/93 mm Hg. Other vital signs are normal. BMI is 29. The remainder of the physical examination is unremarkable. Which of the following should be discontinued prior to screening for secondary causes of hypertension? Hydrochlorothiazide Lisinopril Metoprolol Spironolactone
Spironolactone and eplerenone can significantly interfere with interpretation of the plasma aldosterone-plasma renin ratio (ARR) and therefore should be discontinued approximately 6 weeks prior to screening for primary aldosteronism. Spironolactone can significantly interfere with interpretation of the plasma aldosterone-plasma renin ratio (ARR) and therefore should be discontinued approximately 6 weeks prior to screening for primary aldosteronism. Treatment-resistant hypertension is defined as blood pressure that remains above goal despite concurrent use of three antihypertensive agents of different classes, one of which is a diuretic. Possible situations in which screening for secondary causes of hypertension include: severe or resistant hypertension; young age of onset (in childhood or adolescence), especially in the absence of family history; abrupt worsening of blood pressure in a previously well-controlled patient; or clinical features of an underlying disorder associated with hypertension (for example, cushingoid features). Hyperaldosteronism, usually from an aldosterone-producing adenoma or bilateral idiopathic hyperaldosteronism, may be present in up to 10% of patients with hypertension. Testing for primary aldosteronism should be considered in all patients with difficult to control hypertension. It should also be performed in patients with hypertension and an incidentally noted adrenal mass or spontaneous or diuretic-induced hypokalemia. Spironolactone and eplerenone cause elevation of renin levels and hence can result in a false-negative ARR. On stopping a mineralocorticoid antagonist, the patient may develop hypokalemia if the underlying diagnosis is primary aldosteronism. Potassium should be replaced accordingly prior to screening for primary aldosteronism, as hypokalemia results in lowering of aldosterone levels and hence impacts the ARR. In general, most antihypertensive agents can be continued during screening for primary aldosteronism except for spironolactone, eplerenone, and high-dose amiloride therapy. Specifically, verapamil, doxazosin, and hydralazine have minimal impact on the ARR and, therefore, can be continued during screening for primary aldosteronism. Other antihypertensive agents that have minor effects on the ARR can also be continued during screening for primary aldosteronism as long as the results of the ARR are interpreted with these effects in mind. The hallmark of primary aldosteronism is a suppressed renin level. Any medication that increases renin can result in a false-negative result. On the other hand, a suppressed renin in the presence of a medication that usually would raise renin (an ACE inhibitor) raises the suspicion for primary aldosteronism.
A 57-year-old woman is hospitalized with a 1-month history of diminished concentration, memory, and judgment, with recent mental status fluctuations and gait disorder. Medical history is remarkable for hypertension. Medications are enalapril and hydrochlorothiazide. On physical examination, vital signs are normal. She is somnolent but responds to verbal commands. The neck is supple. Deep-tendon reflexes are increased in the upper extremities and decreased in the lower extremities. Movement of the extremities is associated with myoclonus. The remainder of the examination is normal. Complete blood count, comprehensive metabolic profile, thyroid studies, and vitamin B12 measurements are normal. Lumbar puncture: Opening pressure 70 mm H2O Leukocyte count 4/µL (4 × 106/L) Erythrocyte count 5/µL (5 × 106/L) Glucose 108 mg/dL (6 mmol/L) Protein 57 mg/dL (570 mg/L) Gram stain of cerebrospinal fluid is negative. Polymerase chain reaction of cerebrospinal fluid for herpes simplex virus is negative. Brain MRI shows abnormally increased T2 and FLAIR signal intensity in the putamen and head of the caudate. Which of the following is the most likely diagnosis? Cryptococcal meningoencephalitis Sporadic Creutzfeldt-Jakob disease Tuberculous meningitis Vascular neurocognitive disorder
Sporadic Creutzfeldt-Jakob disease is the most common form of prion disease, involving several neurologic systems and rapid progression of apparent dementia. This patient most likely has sporadic Creutzfeldt-Jakob disease (sCJD). This is the most common form of prion disease and has no evidence of environmental risk factors. Involvement of several neurologic systems and a rapid onset of apparent dementia are classic manifestations of prion disease. On physical examination, ataxia, myoclonus, and a rapidly progressive dementia are present. MRI abnormalities are not specific for sCJD and vary with the clinical syndrome. Patients with increased T2 signal in the caudate and putamen are more likely to have early dementia and shorter survival. No simple, noninvasive assay is available to diagnose sCJD; however, the presence of T-tau or 14-3-3 protein in cerebrospinal fluid (CSF) can be suggestive. The definitive diagnostic test is a brain biopsy demonstrating widespread spongiform changes with gliosis. No definitive treatment exists for any prion disease, and most are rapidly fatal. Cryptococcal meningitis may present initially with altered mentation. Cryptococcal meningitis can be seen in apparently immunocompetent persons; however, it is likely that most patients who develop this infection have some underlying immune deficiency. Lumbar puncture may show a high opening pressure, and the CSF typically shows a lymphocytic pleocytosis. The patient's findings are not compatible with this diagnosis. Tuberculous meningitis may present with waxing and waning mental status changes. However, the CSF in tuberculous meningitis frequently shows high protein (>500 mg/dL [5000 mg/L]) and low glucose (<40 mg/dL [2.22 mmol/L]) levels. Additionally, MRI may reveal basilar pachymeningitis and occasional tuberculomas, which are not seen in this patient. Vascular neurocognitive disorder (VND) is the term now used to describe cognitive impairment of any degree resulting from cerebrovascular disease. The diagnosis is made when neuroimaging or clinical history reveals evidence of a stroke or subclinical cerebrovascular disease that is responsible for impairment of at least one cognitive domain. The absence of infarcts on MRI and rapid progression of this patient's symptoms are not compatible with VND.
A 58-year-old woman is evaluated for a palpable mass in her right groin. She takes no medications. On physical examination, vital signs are normal. A hard, fixed, 3-cm lymph node is palpated in the right inguinal region. Careful inspection of the perianal skin, vulva, vagina, and cervix shows no findings. The remainder of the physical examination is normal. A biopsy of the enlarged node demonstrates poorly differentiated carcinoma. A contrast-enhanced CT scan of the chest, abdomen, and pelvis shows the enlarged inguinal node and no other abnormal findings. Which of the following is the most appropriate diagnostic test to perform next? Anoscopy Bone scan Colonoscopy and upper endoscopy PET scan Serum CEA, CA-15-3, and CA-125
Squamous cell carcinoma or poorly differentiated carcinoma presenting as isolated inguinal lymphadenopathy should prompt a careful examination of the vulva, vagina, and cervix in women, penis in men, and perineal skin and anus in both men and women. The most appropriate diagnostic test to perform is an anoscopy. In the United States, approximately 35,000 patients receive a diagnosis of cancer of unknown primary (CUP) annually. The clinical evaluation should not involve an exhaustive search for a primary site because detection of an asymptomatic and occult primary tumor does not improve outcome. In a patient with CUP, it is important to look for treatable subgroups. Patients with CUP who have lymphadenopathy in a single lymph node or single lymph node region belong to a potentially more treatable subgroup of patients with CUP. Most patients with carcinoma, especially squamous cell carcinoma, involving inguinal lymph nodes have a primary site in the genital or anorectal area. In women this necessitates a careful examination of the vulva, vagina, and cervix. Men require a careful examination of the penis. Men and women should have a careful examination of the perineal skin, a digital rectal examination, and an anoscopy. This patient's examination is complete except for digital rectal examination and anoscopy. The anal region drains directly to the inguinal lymph nodes. Cancers of the anus that metastasize to the inguinal nodes are local-regionally advanced but not distantly metastatic and are potentially curable with a combination of chemotherapy and radiation therapy. A bone scan would not be useful in identifying a treatable primary source and would not be indicated in a patient with an enlarged inguinal lymph node in the absence of specific bone pain. Colonoscopy and upper endoscopy would be indicated in a patient presenting with gastrointestinal symptoms, unexplained anemia, or blood in the stool. Although PET scans may sometimes suggest the location of a primary tumor, these findings are rarely definitive and do not improve long-term outcome. Measurement of serum tumor marker levels, such as carcinoembryonic antigen, CA-19-9, CA-15-3, and CA-125, is rarely helpful and virtually never diagnostic.
A 67-year-old man is evaluated for a carotid bruit detected on routine medical examination. He reports no history of previous focal neurologic symptoms or visual loss. He has type 2 diabetes mellitus and hyperlipidemia treated with metformin, moderate-intensity pravastatin, and aspirin. On physical examination, blood pressure is 128/64 mm Hg, pulse rate is 78/min and regular, and respiration rate is 16/min. A left carotid bruit is heard on cardiac examination. All other physical examination findings, including those from a neurologic examination, are unremarkable. Results of laboratory studies show an LDL cholesterol level of 82 mg/dL (2.12 mmol/L). The carotid ultrasound report describes a mixed-density plaque at the origin of the left internal carotid artery with stenosis estimated to be 60% to 80%. Which of the following is the most appropriate next step in management? Carotid endarterectomy Carotid stenting Magnetic resonance angiography of the neck Replacement of aspirin with clopidogrel No further treatment or intervention
Statin therapy is indicated for asymptomatic carotid stenosis of 60% to 80%. This patient with 60% to 80% stenosis of the left internal carotid artery should receive no further treatment or intervention. The patient's LDL cholesterol level indicates that his atherogenic dyslipidemia is adequately treated with pravastatin, a moderate-intensity statin. Although there is some evidence suggesting that his 10-year risk for a major cardiovascular event is high enough to warrant switching to a high-dose, high-intensity statin, there are no specific guidelines recommending such a change. Asymptomatic carotid artery stenosis was not included in the definition of atherosclerotic cardiovascular disease used in the latest dyslipidemia treatment targets. Data are insufficient among patients with asymptomatic disease to recommend a specific therapy beyond treatment with a statin; there is currently no consensus on which statin and what dose to use. Carotid endarterectomy or stenting is not the best treatment for this patient. The patient has asymptomatic internal carotid artery stenosis of 60% to 80%; the risk of stroke with best medical therapy is very low. Carotid revascularization with either endarterectomy or stenting, on the other hand, has a higher risk of adverse effects, including stroke, and its absolute risk reduction of stroke in asymptomatic patients is small, particularly among patients with stenosis of 80% or less. According to previous studies, predictors of stroke with asymptomatic internal carotid artery stenosis include greater than 80% stenosis, asymptomatic infarcts on brain imaging, an abnormal transcranial Doppler ultrasound study, or rapid progression. Carotid revascularization should be considered in patients at low risk for perioperative cardiovascular morbidity who have greater than 80% stenosis only in the context of a clinical trial. Magnetic resonance angiography (MRA) of the neck is inappropriate because an additional diagnostic test is unlikely to change medical management. The accuracy of MRA without contrast versus carotid ultrasound is likely similar, but neck MRA is associated with patient discomfort and a higher cost. Because there is no clear evidence that clopidogrel is superior to aspirin for the primary prevention of stroke in the setting of asymptomatic internal carotid artery stenosis, replacing this patient's aspirin with clopidogrel is unwarranted.
Indications for stress TTE instead of stress EKG?
Stress testing with adjunctive imaging should be performed in patients with suspected coronary artery disease who have baseline electrocardiographic (ECG) abnormalities that preclude the use of ECG stress testing, such as ST-segment depressions greater than 0.5 mm, left bundle branch block, ventricular paced complexes, digitalis effect, and preexcitation.
A 25-year-old woman is evaluated for anterior neck pain, fatigue, exercise intolerance, excessive sweating, and tremors that began 6 weeks ago. Other than an upper respiratory infection 2 months ago, she has been healthy. Medical history is otherwise unremarkable, and she takes no medications. On physical examination, pulse rate is 105/min. Other vital signs are normal. The patient's thyroid gland is tender to palpation and is without discrete nodules. No thyroid bruit is auscultated. Bilateral lid lag is noted, but there is no proptosis, conjunctival injection, or chemosis. There is a fine tremor of her outstretched hands. Deep tendon reflexes are brisk. Laboratory studies show a serum thyroid-stimulating hormone (TSH) level less than 0.01 µU/mL (0.01 mU/L), a serum free thyroxine (T4) level of 2.8 ng/dL (36.1 pmol/L), and a serum total triiodothyronine (T3) level of 190 ng/dL (2.9 nmol/L). Urine pregnancy test is negative. Which of the following is the most likely diagnosis? Graves disease Molar pregnancy Subacute thyroiditis Toxic multinodular goiter
Subacute thyroiditis is an uncommon cause of thyrotoxicosis that presents following a viral upper respiratory tract infection and is distinguished by a tender or painful thyroid, suppressed thyroid-stimulating hormone, and elevated serum free thyroxine. The most likely diagnosis is subacute thyroiditis. Subacute thyroiditis is an uncommon cause of thyrotoxicosis that presents following a viral upper respiratory tract infection and is distinguished by a tender or painful thyroid. This is a form of destructive thyroiditis resulting from the leakage of stored thyroid hormone from damaged thyroid follicles. The diagnosis can be confirmed by determining radioactive iodine uptake, which would be low (<10%). Management is aimed at controlling symptoms. This includes treatment with β-blockers and pain control with NSAIDs or, less commonly, glucocorticoids. In most cases, thyrotoxicosis typically lasts 2 to 6 weeks. It is followed by a hypothyroid phase after stored thyroid hormone is depleted, typically lasting 6 to 12 weeks. The patient may become clinically hypothyroid and require temporary levothyroxine therapy. Most patients with thyroiditis eventually recover to a euthyroid state. Graves disease is the most common cause of thyrotoxicosis in the United States and most frequently affects young women. This patient does not have pathognomic features of Graves disease (thyroid bruit, eye disease, or dermopathy), making this an unlikely diagnosis. Molar pregnancy is a rare cause of hyperthyroidism resulting from the binding of human chorionic gonadotropin (HCG) to the thyroid-stimulating hormone (TSH) receptor in the setting of very high HCG levels. The negative pregnancy test excludes this diagnosis. Nodular thyroid disease (toxic adenoma and multinodular goiter) is the next most common cause of thyrotoxicosis after Graves disease and is more commonly seen in older adults. This patient lacks palpable thyroid nodules on examination, which is usually seen with hyperthyroidism from nodular thyroid disease. In addition, neither Graves disease nor nodular thyroid disease cause thyroid pain. Read Related TextNext Question
A 45-year-old man is seen for follow-up evaluation for depression and to review the results of laboratory testing. He was seen 1 month ago for a 6-month history of depressed mood, difficulty sleeping, decreased appetite, 2.3-kg (5-lb) weight loss, and fatigue. Major depressive disorder was diagnosed, and escitalopram was prescribed. Today the patient reports a significant improvement in his mood, appetite, and the quality of his sleep since starting treatment. On physical examination, vital signs and physical examination are normal. Screening laboratory studies from 1 month ago show a thyroid-stimulating hormone (TSH) level of 7 µU/mL (7 mU/L) and a free thyroxine (T4) level of 1.0 ng/dL (12.9 pmol/L). Which of the following is the most appropriate management? Measure thyroid peroxidase antibodies Measure thyrotropin receptor antibodies Measure serum triiodothyronine (T3) level Prescribe levothyroxine Repeat serum TSH testing in 2 months
Subclinical hypothyroidism is characterized by a serum thyroid-stimulating hormone (TSH) level above the upper limit of the reference range and normal free thyroxine (T4) level; before making this diagnosis, however, transient elevation of serum TSH should be ruled out by repeating the measurement of TSH in 2 to 3 months. This patient has subclinical hypothyroidism, and the most appropriate management is to repeat serum thyroid-stimulating hormone (TSH) testing in 2 months. Free thyroxine (T4) and thyroid peroxidase antibodies could also be measured at that time. Subclinical hypothyroidism is an early form of primary hypothyroidism affecting up to 10% of the population and is characterized by a serum TSH level above the upper limit of the reference range and normal free T4 level. Before making this diagnosis, however, transient elevation of serum TSH levels should be ruled out by repeating the measurement of TSH in 2 to 3 months. Thyroid peroxidase (TPO) antibodies are frequently seen in the setting of Hashimoto thyroiditis, the most common cause of hypothyroidism in the United States. TPO antibody positivity would support a diagnosis of primary hypothyroidism, predict risk of progression to overt hypothyroidism, and may impact treatment decisions accordingly. However, the most appropriate next step in the management of this patient is to confirm the diagnosis of subclinical hypothyroidism by documenting persistent serum TSH elevation. Thyrotropin receptor antibodies would be more consistent with Graves disease and testing for these antibodies would not be appropriate in this setting. Measurement of serum triiodothyronine (T3) in patients with known or suspected primary hypothyroidism is generally not indicated. Elevation of serum TSH is the earliest biochemical change observed in the setting of primary hypothyroidism. Reductions in serum T3 are not usually seen until after free T4 levels are low. Measuring serum T3 has little to no impact on diagnosis or management decisions. Guidelines differ regarding when to treat subclinical hypothyroidism (elevated serum TSH level with free T4 or total T4 levels within the reference range). For most adults with subclinical hypothyroidism, thyroid hormone replacement will result in no clinical benefits and may have little or no effect on cardiovascular events or mortality. However, treatment should be provided to women who are trying to become pregnant or patients with a TSH level greater than 20 µU/mL (20 mU/L), and treatment may be reasonable for patients with severe symptoms or adults aged 30 years or younger. Guidelines stress that clinicians make individualized, patient-specific decisions about treatment. However, the most appropriate next step is to reassess the patient's TSH level and symptoms before making treatment decisions. Read Related TextNext Question
A 48-year-old woman is evaluated for recurrent pulmonary embolism. Her first episode was 18 months ago; she was treated with warfarin for 3 months. She was hospitalized 9 months ago for pulmonary embolism and has been treated with warfarin since then. She reports progressive exertional dyspnea. She has no chest pain, cough, hemoptysis, or wheezing. Her only medication is warfarin. On physical examination, blood pressure is 108/68 mm Hg, pulse rate is 90/min, and respiration rate is 16/min. Oxygen saturation is 90% breathing ambient air. BMI is 36. The cardiovascular examination shows jugular venous distention, a prominent jugular venous a wave, and widened split S2 with a prominent pulmonic component. Lung examination is unremarkable. INR is 2.8. Echocardiography reveals a normal left ventricle and dilated right ventricle with reduced function. Ventilation-perfusion scan shows multiple mismatched defects. Right heart catheterization reveals a mean pulmonary arterial pressure of 58 mm Hg and a normal pulmonary capillary wedge pressure . Pulmonary angiography is remarkable for pulmonary artery webs, intimal irregularities, and abrupt narrowing of the major pulmonary arteries. Which of the following is the definitive treatment? Apixaban Inferior vena cava filter Nifedipine Pulmonary thromboendarterectomy
Surgical intervention is the only definitive therapy for chronic thromboembolic pulmonary hypertension (CTEPH), and most patients with CTEPH should be referred for evaluation at a specialty surgical center. This patient has chronic thromboembolic pulmonary hypertension (CTEPH) and the preferred treatment is pulmonary thromboendarterectomy. There are two diagnostic criteria for CTEPH: (1) mean pulmonary artery pressure of 25 mm Hg or higher by right heart catheterization in the absence of left heart pressure overload and (2) compatible imaging evidence of chronic thromboembolism. CT pulmonary angiography (CT-PA) may demonstrate proximally located abnormalities such as vascular webs, intimal irregularities, and luminal narrowing but has limited sensitivity in more distal lesions. Ventilation-perfusion scanning is a more sensitive indicator of CTEPH and is generally the preferred first imaging modality. Once CTEPH is suggested by noninvasive testing, conventional pulmonary angiography should be performed to characterize the extent and distribution of organized thrombus and to determine suitability for surgical intervention. Surgical intervention is the only definitive therapy for CTEPH and can prevent irreversible remodeling of the pulmonary arterial vasculature. Surgical evaluation at an experienced center is warranted in all patients with CTEPH; however, only about half of patients will be surgical candidates and fewer than that will opt for surgery. Lifelong anticoagulant therapy, traditionally with warfarin, is indicated in all patients to help prevent further thromboembolism. Experience with direct oral anticoagulants such as apixaban is limited in this patient population. Because this patient's INR is in the therapeutic range, there's no proved advantage of switching from warfarin to apixaban. Inferior vena cava interruption is typically indicated in patients with venous thrombus for whom anticoagulation is ineffective or not tolerated. In patients with CTEPH and coexisting clot in the lower extremities, inferior vena cava interruption can be considered to help prevent further thromboembolism; however, its role in long-term outcomes is not known and of unclear benefit. Calcium channel blockers such as nifedipine are used in the setting of pulmonary arterial hypertension when right heart catheterization reveals acute vasoreactivity. Their role in the treatment of CTEPH is unproved.
A 60-year-old man is evaluated for epigastric pain that occurs intermittently after eating. He was hospitalized twice in the preceding year for idiopathic recurrent acute pancreatitis. He is a former smoker and reports no alcohol use. On physical examination, vital signs are normal, as is the remainder of the examination. Contrast-enhanced CT of the abdomen shows a diffusely dilated main pancreatic duct with normal intrahepatic and extrahepatic bile ducts and a normal gallbladder. No tumor is seen in the pancreas or liver, and there is no peripancreatic inflammation or necrosis. Endoscopic ultrasonography confirms no evidence of a tumor or gallstone, but the main pancreatic duct appears dilated throughout the pancreas. Mucin is seen exuding from the ampulla of Vater during the endoscopy. Which of the following is the most appropriate next step in management? Endoscopic ultrasonography in 1 year MRI of the abdomen in 1 year Oral prednisone Pancreatic resection
Surgical resection is the best management option for high-risk cystic lesions of the pancreas, such as intraductal papillary mucinous neoplasms that involve the main duct. Pancreatic resection is the most appropriate next step in management. Pancreatic cysts are classified as pancreatic cystic neoplasms (the most common), nonneoplastic pancreatic cysts, and pseudocysts. The two most common pancreatic cystic neoplasms are mucinous cystic neoplasms (MCNs) and intraductal papillary mucinous neoplasms (IPMNs), which involve the main duct, branch ducts, or both. Most pancreatic cysts are branch-duct IPMNs. Main-duct IPMNs can be diagnosed based on diffuse dilation of the main pancreatic duct and the characteristic feature of mucin exuding from the ampulla during endoscopic visualization. Main-duct IPMNs carry a greater than 65% risk for malignant transformation. Surgical resection is the only option for treatment of these high-risk cystic lesions of the pancreas in patients who are appropriate surgical candidates. Diagnosis of solitary cysts is challenging. It may require endoscopic ultrasonography and fine-needle aspiration to distinguish between cystic neoplasms (branch-duct IPMNs, MCNs, and serous tumors) and pseudocysts based on cytology. This patient has endoscopic ultrasonographic findings showing characteristic features of main-duct IPMN. The best course of action is pancreatic resection and not endoscopic ultrasound surveillance in 1 year. MRI of the abdomen in 1 year is the standard surveillance recommendation for asymptomatic, low-risk cystic lesions of the pancreas. Main-duct IPMN is considered a high-risk cystic lesion and, therefore, MRI surveillance is not appropriate. Oral prednisone is used to treat autoimmune pancreatitis, a frequent manifestation of IgG4-related disease. The characteristic finding of autoimmune pancreatitis on imaging is a narrowed main pancreatic duct with parenchymal swelling, known as the "sausage-shaped" pancreas, which is not consistent with this patient's findings.
A 38-year-old woman is evaluated for recent onset of visual blurriness when looking straight ahead or reading. She has a 4-mm right posterior communicating artery aneurysm that was first detected 1 year ago. She has not had any recent headaches and is otherwise well. The patient's only medication is an oral contraceptive agent. On physical examination, blood pressure is 138/78 mm Hg; other vital signs are normal. The neck is supple. Findings from funduscopic examination are normal. On neurologic examination, the right pupil is 5-millimeters in diameter and unreactive; left lateral gaze results in diplopia. The left pupil and right lateral gaze are normal. A magnetic resonance angiogram (MRA) of the brain shows that her aneurysm has grown in size to 8 millimeters. Which of the following is the most appropriate next step in management? Lisinopril administration MRA of the neck Neurosurgical intervention Repeat MRA of the brain in 6 months
Surgical treatment with clipping or endovascular coiling can be considered in patients with symptomatic aneurysms or aneurysms of 7 millimeters or greater in the posterior circulation (posterior communicating and basilar arteries).
A 38-year-old woman is evaluated for recent onset of visual blurriness when looking straight ahead or reading. She has a 4-mm right posterior communicating artery aneurysm that was first detected 1 year ago. She has not had any recent headaches and is otherwise well. The patient's only medication is an oral contraceptive agent. On physical examination, blood pressure is 138/78 mm Hg; other vital signs are normal. The neck is supple. Findings from funduscopic examination are normal. On neurologic examination, the right pupil is 5-millimeters in diameter and unreactive; left lateral gaze results in diplopia. The left pupil and right lateral gaze are normal. A magnetic resonance angiogram (MRA) of the brain shows that her aneurysm has grown in size to 8 millimeters. Which of the following is the most appropriate next step in management? Lisinopril administration MRA of the neck Neurosurgical intervention Repeat MRA of the brain in 6 months
Surgical treatment with clipping or endovascular coiling can be considered in patients with symptomatic aneurysms or aneurysms of 7 millimeters or greater in the posterior circulation (posterior communicating and basilar arteries). This patient should be referred for surgical intervention. She has an intracranial aneurysm in the posterior circulation (posterior communicating artery) that is unruptured but now has several features consistent with a high risk of subarachnoid hemorrhage (SAH): size equal to or greater than 7 millimeters, rapid growth, and a cranial nerve deficit, which in this patient is compression of the oculomotor nerve (cranial nerve III) on the right. Although neurosurgical intervention (with clipping or coiling, depending on local expertise) has a potential risk of adverse events, the risk of SAH without surgery is greater. A diagnostic angiogram is likely to be required for operative planning and can be obtained in conjunction with a planned coiling procedure. The patient's blood pressure is already within the appropriate target of less than 140/80 mm Hg. Given the high risk of rupture, further control of blood pressure (with an agent such as lisinopril) at this point is unwarranted and unlikely to be effective. Carotid artery dissection is typically recognized by the presence of unilateral pain of the head, neck, or face; partial Horner syndrome (ptosis, miosis); and cerebral or retinal ischemia. Less than one third of patients will have all three components. The patient has no headache or other features on examination that are consistent with an internal carotid artery dissection. The examination is instead consistent with oculomotor nerve (cranial nerve III) palsy. If internal carotid artery dissection were suspected, a magnetic resonance angiogram (MRA) of the neck would be an appropriate next step. It is not indicated in this patient. A repeat MRA of the brain in 6 months also is inappropriate. This patient has now met several criteria for aneurysm surgery. Additional screening to document further changes in the size of the aneurysm will change nothing, and delaying surgery runs the risk of rupture.
A 27-year-old woman arrives to establish primary care. She feels well. Her last menstrual period was 2 weeks ago, and menses are regular. Medical history is significant for acute lymphoblastic leukemia diagnosed at 5 years of age. She received treatment for 2 years and has been leukemia free since the completion of therapy. She takes no medications. On physical examination, vital signs are normal; BMI is 27. Other examination findings are unremarkable. Complete blood counts performed within the past year were normal. The patient will arrange a treatment summary to be transferred to the office. Which of the following is the most appropriate test to obtain next in this patient? Bone marrow biopsy Estrogen and progesterone levels Exercise stress test Lipid profile and fasting blood glucose Whole genome sequencing
Survivors of acute lymphoblastic leukemia are at increased risk for the metabolic syndrome, including high BMI, truncal obesity, dyslipidemia, insulin resistance, and hypertension. A lipid profile and fasting blood glucose should be performed. Adult survivors of childhood leukemia (primarily acute lymphoblastic leukemia [ALL]) have health risks largely secondary to the cancer therapy they received. Requesting a treatment summary or referral to a survivor clinic at a comprehensive cancer center is the first step in providing care. Increased risks of cardiovascular disease, the metabolic syndrome, and secondary cancer are the most relevant concerns. Compared with control populations, survivors of ALL are more likely to have features of the metabolic syndrome, including high BMI, truncal obesity, dyslipidemia, insulin resistance, and hypertension. Screening for lipid profile, diabetes, and hypertension is recommended by multiple guidelines for adult survivors of childhood ALL. Survivors of ALL in continuous remission for more than 20 years have virtually no risk for ALL recurrence, although they are at risk for therapy-associated myelodysplastic syndrome or acute myeloid leukemia up to 15 years after treatment. With normal complete blood count findings, surveillance bone marrow examinations and whole genome sequencing are not indicated. Leukemia therapy in childhood does not typically compromise ovarian function, and the patient's regular periods attest to the presence of normal ovarian function; obtaining estrogen and progesterone levels is not necessary. Anthracyclines, such as doxorubicin, are the most common cause of dose-dependent chemotherapy-induced cardiomyocyte damage, leading to late and irreversible chronic heart failure. Exposure to anthracycline chemotherapy during childhood can cause heart failure in adulthood. However, echocardiography, not an exercise stress test, is the appropriate test to evaluate for abnormal myocardial function that may be asymptomatic. Echocardiography before pregnancy may be particularly important in women previously treated with anthracyclines because they are at risk for further deterioration of myocardial function during pregnancy.
A 64-year-old man is evaluated for a 9-month history of a swollen and painful right knee without trauma or injury. The knee has been drained twice; synovial fluid leukocyte counts were between 15,000/µL to 20,000/µL (15-20 × 109/L), predominantly lymphocytes. All stains and cultures have been negative. He also reports a low-grade fever in the evening. He has no history of skin rash or tick bites. He had a positive tuberculin skin test 20 years ago and was treated with isoniazid for an unknown duration. He takes no medications. On physical examination, vital signs are normal. The right knee is tender and has a moderate effusion, with reduced passive range of motion. The remainder of the physical examination is normal. Laboratory studies show an erythrocyte sedimentation rate of 48 mm/h and negative Lyme disease serologies. Chest radiograph shows three small calcified granulomas in the left upper lobe. Plain radiograph of the right knee shows joint space narrowing, osteopenia, and small sinus tracts. Which of the following is the most appropriate diagnostic test to perform next? Angiotensin-converting enzyme level CT of the chest HLA-B27 testing Synovial biopsy
Synovial fluid analysis can confirm inflammation but may be inadequate for diagnosis of a chronic inflammatory monoarthritis; synovial biopsy may be required. The most appropriate test to perform next is a synovial biopsy. This patient has persistent inflammatory swelling of the right knee, with synovial fluid leukocyte counts between 15,000 and 20,000/µL (15-20 × 109/L). The differential diagnosis in this patient with chronic inflammatory monoarthritis includes mycobacterial, fungal, or Borrelia burgdorferi infection and other systemic rheumatologic diseases such as sarcoidosis. Frequently in these patients, synovial fluid analysis alone may be inadequate for diagnosis; serologies or other laboratory tests or synovial biopsy is required to establish the diagnosis. In this patient, a synovial biopsy is the most appropriate test to evaluate for all of the conditions in the differential diagnosis. The biggest concern here is Mycobacterium tuberculosis arthritis, given this patient's history of latent tuberculosis and a high risk due to advancing age. Sarcoidosis is characterized by formation of noncaseating granulomas in multiple organs and tissues. These granulomas may increase angiotensin-converting enzyme levels. A history of latent tuberculosis, calcified granulomas in the upper lobe, and a chronically inflamed joint are more consistent with tuberculosis infection. In addition, the angiotensin-converting enzyme level is only 75% sensitive (25% of cases will be missed) and 90% specific (10% of positive results will be false positive), decreasing the usefulness of this test for sarcoidosis. Multiple small pulmonary nodules (<5 mm) found incidentally in a patient without a known malignancy are likely to be nonmalignant. Like solitary pulmonary nodules, multiple pulmonary nodules may be further assessed with helical CT to better characterize their number, location, and morphology. However, in this patient, the previous history of tuberculosis, fever, and a chronic inflammatory monoarthritis strongly suggest tuberculosis, and a tissue diagnosis can be most easily established with synovial biopsy. HLA-B27 testing may be useful in the evaluation of spondyloarthritis, which can present with chronic lower extremity inflammatory monoarthritis. However, this patient has no other findings of spondyloarthritis such as rash, bowel symptoms, or inflammatory back pain. Furthermore, HLA-B27 lacks specificity and is unlikely to yield the correct diagnosis.
A 26-year-old woman is evaluated for a 4-week history of progressive dyspnea on exertion. She has experienced malaise and myalgia for the past 6 months, noting that her arms ache when she does physical activity with them. She was previously healthy. She reports no rashes, headache or jaw claudication, gastrointestinal symptoms, or neurologic symptoms. On physical examination, blood pressure is 120/60 mm Hg in the right arm and 95/50 in the left arm, pulse rate is 80/min, and respiration rate is 18/min. There is no rash. A reduced radial pulse in the left upper extremity is noted. A grade 2/6 decrescendo diastolic murmur at the left sternal border is heard. The lungs are clear. There is no synovitis. Strength is normal. Laboratory studies: Complete blood count Normal, except for anemia Erythrocyte sedimentation rate 90 mm/h Hemoglobin 10 g/dL (100 g/L) Creatinine Normal Antinuclear antibodies Negative Urinalysis Negative Transthoracic echocardiogram shows mild to moderate aortic valve regurgitation, dilated aortic root, normal valve leaflets, left ventricle dilation, and normal left ventricular ejection fraction. Which of the following is the most likely diagnosis? Giant cell arteritis IgA vasculitis Kawasaki disease Polyarteritis nodosa Takayasu arteritis
Takayasu arteritis is a rare chronic granulomatous vasculitis seen in young Asian women that mainly affects the aorta and its major branches as well as the coronary and pulmonary arteries, resulting in claudication, cardiac ischemia, aortic or mitral regurgitation, aortic dissection, and renal artery stenosis. The most likely diagnosis is Takayasu arteritis, a rare granulomatous large-vessel vasculitis affecting the aorta and its major branches and pulmonary arteries. This disease predominantly affects females typically less than age 30 years and of Asian ancestry. Aneurysms and stenoses of large arteries cause symptoms of claudication in the extremities, discrepancies in blood pressure between the arms, and reduced pulses. Involvement of the aorta can lead to aortic insufficiency and heart failure. This rare complication is a leading cause of morbidity and mortality in patients with Takayasu arteritis. This patient has upper extremity claudication, reduced radial pulse, evidence of inflammation (elevated erythrocyte sedimentation rate, anemia) and new-onset aortic insufficiency with dilated aortic root, all consistent with the diagnosis of Takayasu arteritis. Giant cell arteritis (GCA) is a large-vessel granulomatous vasculitis that can also affect the great vessels of the chest, leading to a similar presentation, but is unlikely given this patient's young age; GCA affects individuals over age 50 years. IgA vasculitis is a small-vessel vasculitis mediated by immune complexes predominantly containing IgA. It does not involve the large vessels and almost always causes a rash, not seen in this patient. Kawasaki disease is a medium-vessel vasculitis that begins in childhood, manifesting with a rash and other mucocutaneous findings. It can affect coronary vessels, leading to cardiac complications such as heart failure. This patient developed initial symptoms as an adult, and has no rash, making Kawasaki disease an unlikely diagnosis. Polyarteritis nodosa (PAN) is a medium-vessel vasculitis that can affect multiple organ systems. Cardiac involvement as well as large-vessel involvement is not a typical manifestation of PAN, making this diagnosis unlikely.
A 48-year-old man is evaluated during a follow-up visit for hypertension. He has no symptoms. He received the tetanus toxoid, reduced diphtheria toxoid, and acellular pertussis vaccine 9 years ago and the influenza vaccine during the most recent influenza season. He is a current smoker with a 25-pack-year history. His only medication is chlorthalidone. On physical examination, vital signs are normal, and the remainder of the examination is unremarkable. Which of the following is the most appropriate vaccine to administer to this patient? Recombinant zoster vaccine Tetanus and diphtheria toxoids booster 13-Valent pneumococcal conjugate vaccine 23-Valent pneumococcal polysaccharide vaccine No vaccines are indicated
The 23-valent pneumococcal polysaccharide vaccine should be administered to select immunocompetent patients aged 19 to 64 years, including those with chronic heart, liver, or lung disease; diabetes mellitus; cochlear implants; cerebrospinal fluid leak; alcoholism; or cigarette smoking. This patient should be administered the 23-valent pneumococcal polysaccharide vaccine (PPSV23). Pneumococcal vaccination is recommended in all adults aged 65 years and older and adults aged 19 to 64 years with certain high-risk conditions. Two pneumococcal vaccines are available: PPSV23 and a 13-valent conjugate vaccine (PCV13). The Advisory Committee on Immunization Practices recommends administering PPSV23 alone to select immunocompetent patients aged 19 to 64 years, including those with chronic heart, liver, or lung disease; diabetes mellitus; cochlear implants; cerebrospinal fluid leak; alcoholism; or cigarette smoking. Because this patient is a current smoker, he should be given PPSV23. In adults aged 19 to 64 years with immunocompromise, cochlear implants, or a history of cerebrospinal fluid leaks, PCV13 should be administered in addition to PPSV23. In these patients, a single dose of PCV13 should be given first, followed by a single dose of PPSV23 at least 8 weeks later. All adults aged 65 years and older who have not previously been vaccinated should be considered for PCV13 in addition to PPSV23, and patients should be engaged in a shared decision-making discussion. If given, PCV13 should be given first followed by a dose of PPSV23 1 year later. For immunocompetent adults aged 65 years and older who previously received one or more doses of PPSV23, a single dose of PCV13 should be given at least 1 year after the most recent PPSV23 dose. This patient does not meet the criteria for PCV13 administration. All adults aged 50 years and older, including those with a previous episode of zoster, should receive the recombinant (inactivated) zoster vaccine to reduce the incidence of zoster and postherpetic neuralgia. The recombinant zoster vaccine is recommended in preference to the live attenuated zoster vaccine, and adults who have been previously vaccinated with the live attenuated zoster vaccine should be revaccinated with the recombinant zoster vaccine after a period of at least 8 weeks. This patient should receive the recombinant zoster vaccine in 2 years. In persons who previously received the tetanus toxoid, reduced diphtheria toxoid, and acellular pertussis (Tdap) vaccine, revaccination with a tetanus and diphtheria toxoids (Td) booster is recommended every 10 years. Additionally, for pregnant women, one dose of the Tdap vaccine should be administered during each pregnancy between 27 weeks' and 36 weeks' gestation, regardless of when the last dose of Td or Tdap was given. This patient last received the Tdap vaccine 9 years ago and should receive a Td booster in 1 year. Given the patient's history of cigarette smoking, offering no vaccinations at this time would not be the best strategy.
A 48-year-old woman is evaluated for a 1-year history of 11.3-kg (25-lb) weight gain, fatigue, easy bruising, and difficulty remembering things. She works the night shift at a hotel reception desk. Medical history is significant for hypertension, type 2 diabetes mellitus, and menopausal hot flushes. Medications are lisinopril, estradiol, aspirin, and metformin. On physical examination, blood pressure is 142/88 mm Hg. Other vital signs are normal. BMI is 34. The patient has central obesity, and a dorsocervical fat pad is present. There are no abdominal striae, no proximal muscle weakness, and no supraclavicular fat pads. Which of the following is the most appropriate next step in evaluation of this patient? 1-mg overnight dexamethasone suppression test 24-Hour urine free cortisol measurement Late night salivary cortisol measurement Morning serum total cortisol measurement
The 24-hour urine free cortisol test for Cushing syndrome is not impacted by either estrogen therapy or sleeping patterns. This patient should be screened for Cushing syndrome, and the best screening test in this patient is measurement of 24-hour urine free cortisol to quantify total daily cortisol secretion. Biochemical testing is used to establish the diagnosis of Cushing syndrome. It is critical that the biochemical diagnosis is firmly established prior to any imaging studies due to the relatively high prevalence of clinically insignificant pituitary and adrenal masses. Initial tests include the 1-mg overnight dexamethasone suppression test, 24-hour urine free cortisol, and late-night salivary cortisol. While the 1-mg overnight dexamethasone test and the late-night salivary cortisol test may be more convenient, they are likely to be less accurate in this patient because of her shift work and estrogen use. Measurement of 24-hour urine free cortisol is not impacted by estrogen therapy or sleeping patterns. A threefold or greater increase over normal values is diagnostic of Cushing syndrome if compatible clinical features are present (centripetal obesity, facial plethora, abnormal fat deposition in the supraclavicular or dorsocervical areas, and wide violaceous striae); if this increase is present, test results should be repeated to confirm the abnormal result. The 1-mg overnight dexamethasone suppression test is also not reliable in this patient because it relies on serum cortisol measurement. The late night salivary cortisol test is not a reliable screening test in this patient because she works a night shift and therefore her diurnal cortisol secretion will be reversed. Measurement of morning serum cortisol is unreliable as a screening test for Cushing syndrome because normal secretion of cortisol is pulsatile and the normal range is broad. Hence, there is considerable overlap between serum cortisol levels seen in normal people, those with Cushing syndrome, and those with hypercortisolism due to psychological or medical stressors. In addition, serum cortisol measurement is unreliable in this patient as she is on oral estrogen, which leads to an increase in cortisol binding proteins and subsequent elevation of serum total cortisol levels without impacting free cortisol levels.
A 28-year-old woman is evaluated during a follow-up visit for elevated blood pressure measurements during pregnancy. She is at 12 weeks' gestation of her first pregnancy. She feels well, and the pregnancy has been otherwise uncomplicated. She did not have routine medical care before her pregnancy. Family history is significant for hypertension in her father and sister. Her only medication is a prenatal vitamin. On physical examination, blood pressure is 155/95 mm Hg; other vital signs are normal. Funduscopic, neurologic, and cardiac examinations are normal. Laboratory studies are normal. Which of the following is the most likely cause of this patient's elevated blood pressure? Chronic hypertension Gestational hypertension Normal physiologic changes in pregnancy Preeclampsia
The American College of Obstetricians and Gynecologists defines chronic hypertension as a systolic blood pressure ≥140 mm Hg or diastolic blood pressure ≥90 mm Hg starting before pregnancy or before 20 weeks of gestation or persists longer than 12 weeks' postpartum. The most likely diagnosis in this pregnant patient is chronic hypertension. Hypertension first recognized during pregnancy at <20 weeks' gestation usually indicates chronic hypertension. The American College of Obstetricians and Gynecologists (ACOG) defines chronic hypertension as a systolic blood pressure ≥140 mm Hg or diastolic blood pressure ≥90 mm Hg starting before pregnancy or before 20 weeks of gestation or persists longer than 12 weeks' postpartum. In normal pregnancy, the blood pressure declines during the first trimester, reaches its lowest level in the second trimester, and rises slowly thereafter. A patient with hypertension in the first trimester suggests that the hypertension predates the pregnancy. To avoid overtreatment of hypertension and associated fetal risk, the 2013 ACOG guidelines recommend treating persistent systolic blood pressure ≥160 mm Hg or diastolic blood pressure ≥105 mm Hg in women with chronic hypertension. Blood pressure goals with medications are 120 to 160/80 to 105 mm Hg. Antihypertensive treatment reduces the risk of progression to severe hypertension by 50% compared with placebo but has not been shown to prevent preeclampsia, preterm birth, small size for gestational age, or infant mortality. Gestational hypertension first manifests after 20 weeks of pregnancy without proteinuria or other end-organ damage and resolves within 12 weeks of delivery. This patient's early presentation is not consistent with gestational hypertension. Normal physiologic changes in pregnancy are usually associated with decreased blood pressure in the first trimester with a nadir blood pressure in the second. In this patient, the high blood pressure is inconsistent with normal pregnancy changes. Preeclampsia is defined clinically by new-onset hypertension and proteinuria that occur after 20 weeks of pregnancy. In addition to blood pressure criteria, there must be proteinuria or new-onset end-organ damage, including liver or kidney injury, pulmonary edema, cerebral or visual symptoms, or thrombocytopenia. This patient's early presentation in the first trimester and lack of end-organ involvement is not consistent with preeclampsia.
A 31-year-old man is evaluated during a follow-up visit for depression. He previously experienced two episodes of major depressive disorder that were effectively treated with fluoxetine. Three months ago, he presented with recurrent symptoms of depression. His PHQ-9 score was 14, indicating moderate depression. Fluoxetine was initiated and uptitrated to an effective dosage. The patient now reports significant improvement in his symptoms. His PHQ-9 score is 6, indicating mild depression; he reports no adverse effects from the medication. Which of the following is the most appropriate next step in management? Complete 8 months of fluoxetine therapy Complete 8 months of fluoxetine, then switch to bupropion for long-term maintenance therapy Continue fluoxetine as long-term maintenance therapy Discontinue fluoxetine
The American Psychiatric Association recommends long-term maintenance therapy for patients with three or more episodes of major depressive disorder, persistent depressive disorder, or residual depressive symptoms; the antidepressant dosage that was effective in acute treatment should be continued for long-term maintenance. Long-term continuation of fluoxetine at the current dosage is appropriate for this patient with recurrent depression. Guidelines from the American Psychiatric Association (APA) recommend long-term maintenance therapy for patients with three or more episodes of major depressive disorder, persistent depressive disorder, or residual depressive symptoms. The same antidepressant and dosage that were effective in the treatment of acute depression should be continued for long-term maintenance. Fluoxetine therapy for 8 months should be sufficient for the treatment of the patient's major depressive disorder, but the medication should not be tapered. Because he has had two other episodes of depression, he should be maintained on antidepressant therapy to prevent recurrence. Switching to another antidepressant medication (such as bupropion) for long-term maintenance is not indicated unless the patient develops intolerable adverse effects from the initial medication. Discontinuing fluoxetine is not recommended, even if the patient were not a candidate for long-term therapy. APA guidelines recommend continuing treatment for at least 4 to 9 months after resolution of major depressive disorder, followed by gradual tapering of the antidepressant dosage. Antidepressant drugs should not be stopped abruptly because of the risk for discontinuation syndrome, which is most frequently seen in patients who abruptly stop selective serotonin reuptake inhibitors. The most common discontinuation symptoms include dizziness, fatigue, headache, and nausea. Other symptoms include agitation, anxiety, dysphoria, and irritability. Onset of the syndrome is within 1 to 4 days of abruptly stopping antidepressant therapy or after a rapid taper. Although fluoxetine has the lowest incidence of discontinuation syndrome, therapy should be tapered rather than abruptly stopped.
A 26-year-old man is evaluated during a routine examination. He is asymptomatic. The patient is sexually active with men and has had multiple partners in the past year. He engages in both oral and anal sex, and he reports using condoms most of the time. He does not use illicit drugs. He is unsure about his vaccination status and has never been tested for HIV infection, syphilis, or infectious hepatitis. Medical history is unremarkable. He takes no medications. The physical examination, including vital signs, is normal. Screening is arranged for HIV infection, syphilis, and hepatitis A and B. Which of the following additional screening tests is most appropriate, as recommended by the Centers for Disease Control and Prevention? Anal cytology Hepatitis C antibody assay Nucleic acid amplification test for chlamydia and gonorrhea No additional tests are indicated
The Centers for Disease Control and Prevention recommends that men who have sex with men should be screened at least annually for genital and extragenital chlamydial and gonorrheal infections, syphilis, and HIV infection.
A 26-year-old man is evaluated during a routine examination. He is asymptomatic. The patient is sexually active with men and has had multiple partners in the past year. He engages in both oral and anal sex, and he reports using condoms most of the time. He does not use illicit drugs. He is unsure about his vaccination status and has never been tested for HIV infection, syphilis, or infectious hepatitis. Medical history is unremarkable. He takes no medications. The physical examination, including vital signs, is normal. Screening is arranged for HIV infection, syphilis, and hepatitis A and B. Which of the following additional screening tests is most appropriate, as recommended by the Centers for Disease Control and Prevention? Anal cytology Hepatitis C antibody assay Nucleic acid amplification test for chlamydia and gonorrhea No additional tests are indicated
The Centers for Disease Control and Prevention recommends that men who have sex with men should be screened at least annually for genital and extragenital chlamydial and gonorrheal infections, syphilis, and HIV infection. The most appropriate screening strategy in this patient is nucleic acid amplification testing (NAAT) of urine, rectal, and pharyngeal specimens for Neisseria gonorrhoeae and Chlamydia trachomatis. Multiple studies have demonstrated an increased prevalence of genital and extragenital chlamydial and gonorrheal infections in men who have sex with men (MSM). In one study, prevalence rates of N. gonorrhoeae in MSM were 6.9% (rectal), 6% (urethral), and 9.2% (pharyngeal); for C. trachomatis, prevalence rates in MSM were 7.9% (rectal), 5.2% (urethral), and 1.4% (pharyngeal). Most infections are asymptomatic. The Centers for Disease Control and Prevention (CDC) recommends at least annual gonorrhea screening with NAAT of urethral, pharyngeal, and rectal specimens and at least annual screening for chlamydia with NAAT of urethral and rectal specimens. The CDC notes that commercially available NAATs have not been cleared by the FDA for some of these indications; however, these tests can be used by laboratories that have met all regulatory requirements for an off-label procedure. The CDC also recommends screening for syphilis and HIV at least annually in MSM. In contrast to the CDC, the U.S. Preventive Services Task Force has found insufficient evidence to recommend screening for chlamydia and gonorrhea in men. Human papillomavirus (HPV)-associated conditions (such as anogenital warts and anal squamous intraepithelial lesions) are common among MSM. However, data are insufficient to recommend routine anal cancer screening with anal cytology in MSM. HPV vaccination is recommended for all patients through age 26 years. Sexual transmission of hepatitis C infection can occur in MSM with HIV infection. The CDC recommends that screening should be performed at least yearly for hepatitis C in this population. Hepatitis C screening is also indicated in all past and current injection drug users. Because the patient does not use drugs and his HIV status is unknown, screening for hepatitis C is not indicated at this time. All MSM should also be tested for hepatitis A and B. Vaccination against hepatitis A and B is recommended for all MSM in whom previous infection or vaccination cannot be documented.
A 65-year-old woman is evaluated 3 weeks postoperatively following right hemicolectomy for colon cancer. Pathology of the surgical specimen revealed a 3-cm adenocarcinoma invading into but not through the colonic wall. Four of 17 local-regional lymph nodes examined contained metastatic cancer. All margins of resection were free of tumor. Contrast-enhanced CT scans of the chest, abdomen, and pelvis were unremarkable. She takes no medications. On physical examination, vital signs are normal. Surgical incisions are fully healed. The remainder of the examination is normal. Carcinoembryonic antigen (CEA) is 1.7 µg/L (normal is less than 5.0 µg/L). Which of the following is the most appropriate treatment? 5-Fluorouracil, leucovorin, and oxaliplatin (FOLFOX) Radiation therapy Radiation therapy plus FOLFOX No further therapy
The FOLFOX regimen of 5-fluorouracil, leucovorin, and oxaliplatin or oxaliplatin with oral capecitabine (CAPOX) is the most appropriate adjuvant therapy for stage III colon cancer. The most appropriate treatment is the FOLFOX regimen of 5-fluorouracil (5-FU), leucovorin, and oxaliplatin. This patient has stage IIIC colon cancer. Stage III is defined by metastases to the local-regional lymph nodes, and stage IIIC is defined by metastases to four or more nodes. Stage III colon cancer is potentially curable, although the risk of treatment failure increases with the number of nodes, and four or more positive nodes is a worrisome prognostic indicator. Currently all patients with stage III cancer are recommended to receive adjuvant chemotherapy after definitive surgery, as large randomized trials have shown that this modestly improves the likelihood of cure. For over a decade, the combination of oxaliplatin plus a fluoropyrimidine, such as 5-FU or its oral prodrug, capecitabine, have been the accepted standard management for stage III colon cancer. The drug 5-FU is usually given with the reduced folate leucovorin, which is inactive alone but causes 5-FU to bind more tightly to its target enzyme. Oxaliplatin with oral capecitabine (CAPOX) is also an acceptable regimen for adjuvant treatment of stage III colon cancer. Use of oral capecitabine requires a highly reliable, motivated patient who is able to adhere to a complex oral medication schedule. Local recurrence in colon cancer, unlike rectal cancer, is rare, so radiation therapy is not a routine part of the management of colon cancer. Further, it can be difficult to isolate the small bowel from the radiation field, which results in unnecessary toxicity. Therefore, radiation therapy does not have a role in the routine management of patients with stage III colon cancer, either alone or combined with chemotherapy.
A 27-year-old woman seeks advice on breast cancer screening recommendations. Medical history is significant for stage IIB Hodgkin lymphoma diagnosed at age 19 years, treated with mantle and para-aortic radiation; there has been no evidence of recurrence. The patient is premenopausal and has no family history of breast or ovarian cancer. She takes no medications. On physical examination, vital signs are normal. BMI is 28. There is a healed right supraclavicular incision from a previous lymph node biopsy. Breast examination findings are normal. When should this patient begin breast cancer screening? At age 30 years At age 40 years At age 50 years Now
The International Late Effects of Childhood Cancer Guideline Harmonization Group recommends that for women who received chest wall radiation before the age of 30 years, breast screening should begin at age 25 years or 8 years after completion of radiation therapy, whichever is last. Annual mammography or breast MRI starting now is the most appropriate screening strategy in this patient. She has a history of radiation to the chest wall to treat Hodgkin lymphoma at age 19 years and is at high risk for breast cancer. Among this group, the cumulative breast cancer incidence by age 40 to 45 years ranges from 13% to 20% and is similar to that in BRCA gene mutation carriers. The risk varies with the dose of radiation used, and the strength of the screening recommendation is highest for women who received the highest dose of radiation (20 Gy or higher). For women with low- to moderate-dose chest radiation (less than 20 Gy), the decision to screen should be an individual one, taking into account additional risk factors and patients' values. The increased risk for breast cancer begins within 8 years after treatment. The International Late Effects of Childhood Cancer Guideline Harmonization Group recommends that for women who received chest wall radiation before the age of 30 years, screening should begin at age 25 years or 8 years after completion of radiation therapy, whichever is last. The Guideline Harmonization Group notes that the ideal screening modality is unknown because of a lack of sufficient evidence, but breast MRI, mammography, or a combination of mammography and breast MRI are all reasonable choices. National Comprehensive Cancer Network guidelines recommend annual mammography and MRI starting after age 25 for patients receiving radiation between ages 10 and 30, beginning 8 to 10 years after irradiation. This patient completed radiation at the age of 19 years. According to the Guideline Harmonization Group, her breast cancer screening should start at age 27 years (8 years after radiation or age 25 years, whichever is last), not 30 years. Patients at average risk for breast cancer should start screening every 2 years by age 50 years, with the option to start mammographic screening between ages 40 and 50 years being an individualized decision. This patient, however, is at high, not average, risk.
A 65-year-old woman is evaluated for a 3-month history of increasing fatigue. History is significant for stage G4 chronic kidney disease and hypertension. Medications are sodium bicarbonate, sevelamer, furosemide, losartan, and amlodipine. On physical examination, blood pressure is 120/60 mm Hg, and pulse rate is 75/min; other vital signs are normal. Conjunctival rim pallor is noted. Laboratory studies: Hemoglobin 8.5 g/dL (85 g/L) Mean corpuscular volume 90 fL Ferritin 600 ng/mL (600 µg/L) Transferrin saturation 40% Estimated glomerular filtration rate 25 mL/min/1.73 m2 Stool guaiac testing is negative. Colonoscopy performed within the past 5 years was normal. Which of the following is the most appropriate treatment? Discontinue losartan Schedule packed red blood cell transfusion Start an erythropoiesis-stimulating agent Start intravenous iron
The Kidney Disease: Improving Global Outcomes (KDIGO) guidelines recommend consideration of erythropoiesis-stimulating agents to treat anemia in patients with chronic kidney disease and adequate iron stores who have hemoglobin concentrations <10 g/dL (100 g/L); the dose should be titrated to avoid hemoglobin concentrations increasing above 11.5 g/dL (115 g/L). An erythropoiesis-stimulating agent (ESA) is appropriate to treat anemia in this patient with stage G4 chronic kidney disease (CKD). The prevalence of anemia increases as CKD progresses due to several factors, including impaired erythropoietin production, erythropoietin resistance, and reduced erythrocyte life span. Iron deficiency may contribute to erythropoietin resistance. The Kidney Disease: Improving Global Outcomes (KDIGO) recommendations suggest maintaining transferrin saturation levels of >30% and serum ferritin levels of >500 ng/mL (500 µg/L). The patient's iron stores are adequate based on the patient's transferrin saturation and ferritin levels, and the next step in treatment is to start an ESA. ESAs are highly effective in raising hemoglobin concentrations and alleviating symptoms, and can avoid the need for transfusion. ESAs should not be used in patients with active malignancy, history of malignancy, or history of stroke. ESAs should be started in symptomatic patients with CKD who have a hemoglobin concentration <10 g/dL (100 g/L), and the dose should be titrated to avoid hemoglobin concentrations increasing above 11.5 g/dL (115 g/L). ACE inhibitors and angiotensin receptor blockers (ARBs) inhibit erythropoiesis, and patients on maintenance dialysis taking either ACE inhibitors or ARBs may have increased erythropoietin requirements. However, the mechanisms by which this occurs are poorly defined. Discontinuing losartan is incorrect because renin-angiotensin system blockade in CKD is beneficial in delaying the progression of CKD to end-stage kidney disease, which outweighs any potential interaction with anemia. However, discontinuing the medication could be considered if the patient demonstrated signs of erythropoietin resistance or refractory anemia. Packed red blood cell transfusion is not indicated at this time and would expose the patient to transfusion-related risks and side effects. Additionally, blood transfusions can cause allosensitization, which can lead to antibody formation, and may prolong transplant waiting times. Starting iron is incorrect because the patient is iron replete. Iron repletion is recommended if the transferrin saturation is ≤30% and the ferritin is ≤500 ng/mL (500 µg/L).
When to start statin in CKD patients?
The Kidney Disease: Improving Global Outcomes (KDIGO) guidelines recommend treatment of dyslipidemia with a statin in patients aged ≥50 years with an estimated glomerular filtration rate <60 mL/min/1.73 m2, but not treated with chronic dialysis or kidney transplantation.
A 27-year-old man is evaluated during a routine health maintenance visit with his internist. He asks about reducing his risk for HIV infection because he has sex with men, has multiple partners, and reports using condoms "sometimes." Medical history is unremarkable, and he takes no medications. The physical examination is normal. Laboratory studies show a normal serum creatinine level. A serum rapid plasma reagin test is negative. Nucleic acid amplification testing for gonorrhea and chlamydia from urine, pharynx, and rectum are all negative. An HIV test is negative. Which of the following is the most appropriate preventive measure? Reinforce consistent condom use and avoid antiretroviral therapy Tenofovir and emtricitabine single dose before each sexual encounter Tenofovir and emtricitabine single dose after each sexual encounter Tenofovir disoproxil fumarate and emtricitabine daily
The combination of tenofovir disoproxil fumarate (or tenofovir alafenamide, except in women engaging in receptive vaginal intercourse) plus emtricitabine taken once daily is more than 90% effective, if taken consistently, in preventing HIV acquisition.
A hospital system's initial analysis of costs related to prolonged hospital stays revealed that surgical wound infections account for a large proportion of costs. Assessment of the surgical data for the hospital showed that the postoperative wound infection rate is 19%. Which of the following is the most appropriate tool to assist in reducing postoperative wound sepsis at this institution? Clinical audit Control chart Lean model Model for Improvement
The Model for Improvement relies on identifying a goal to be accomplished with a change, determining how the results of a change will be measured, and deciding on the changes that will bring about an improvement. The most appropriate tool to assist in reducing postoperative wound sepsis at this institution is the Model for Improvement. The Model for Improvement focuses on achieving specific and measurable results in a specified population. This model relies on identifying a goal to be accomplished with a change, determining how the results of a change will be measured, and deciding on the changes that will bring about an improvement. These changes are tested and implemented using the Plan-Do-Study-Act (PDSA) cycle. PDSA cycles are rapid tests of improvement, and additional PDSA cycles are completed until the desired results are achieved. A medical center, for example, may set the specific goal of decreasing central line-associated bloodstream infections and use the PDSA cycle to rapidly implement and assess the impact of changes, such as using a central line bundle. A clinical audit involves measuring current practices against desirable outcomes, which are usually guideline based. Feedback is often provided at the individual level. The audit can identify deviations from desired care (for example, surgical infection rate) but does not establish a goal, an intervention, or a metric to gauge the success of the intervention for the purposes of systematically improving care. A control chart is a commonly used quality improvement tool. Control charts graphically display variation in a process over time and can help determine whether variation is related to a predictable or unpredictable cause. Control charts can additionally be used to determine whether an intervention has had a positive change. This tool could be useful in measuring change but does not involve goal setting or selecting and implementing an intervention, which are required elements in quality improvement models. The Lean model focuses on closely examining a system's processes and eliminating non−value-added activities, or waste, within that system. By using a tool called value stream mapping that graphically displays the steps of a process (and the time required for each step) from beginning to end, inefficient areas (waste) in a process can be identified and addressed. The Lean model would not be particularly helpful in identifying causes of surgical site infection, selecting and implementing an intervention, and measuring the outcome.
A 69-year-old man is evaluated on routine follow-up. He has a history of stage IIIC sigmoid colon cancer that was resected 18 months ago. He received postoperative chemotherapy and has been followed expectantly since that time. He takes no medications. On physical examination, vital signs are normal. Examination is notable for a liver edge palpable 2 cm below the right costal margin. He is now found to have a new elevation in serum carcinoembryonic antigen level. A contrast-enhanced CT scan of the chest, abdomen and pelvis shows numerous hypodense lesions up to 4 cm in diameter in the liver and lungs. Previous postoperative CT scan results were normal. Which of the following is the most appropriate diagnostic test to perform next? Multi-gene sequencing of tumor Needle biopsy of most accessible lesion PET/CT scan RAS mutation status of primary tumor
The RAS mutational status of tumors should be tested in patients with metastatic disease to determine if they are candidates for treatment with an epidermal growth factor receptor inhibitor, such as cetuximab or panitumumab. The RAS mutation status of an archived primary tumor specimen should be tested next. This patient has colon cancer that has metastasized to the liver and lungs in a pattern that is not surgically curable. Chemotherapy is the treatment of choice, and several different agents will be considered. This patient may be a candidate for treatment with an epidermal growth factor receptor inhibitor, such as cetuximab or panitumumab. All patients with metastatic colorectal cancer should have tumor genotyping to identify mutations in the KRAS and NRAS genes because cetuximab and panitumumab are inactive in the 40% to 50% of tumors that harbor these mutations. It is therefore standard practice to obtain RAS mutational status of tumors of patients with metastatic disease. RAS mutations, when they occur, occur early in carcinogenesis, such that the RAS mutation status of the primary tumor and a subsequent recurrence are very highly concordant, and rebiopsy of the tumor for RAS status is not necessary. The absence of KRAS or NRAS mutations does not predict therapeutic success with cetuximab or panitumumab; up to 40% of tumors with wild type (nonmutated) genes will not respond to these agents. Multigene sequencing may open some experimental options, but it does not yield actionable information in terms of standard management options at this time. Thus, the expense is not warranted outside of a potential research setting. The presentation and imaging of this patient, who was at high risk for recurrence, are virtually diagnostic of metastatic colon cancer, and an invasive procedure, such as needle biopsy, for pathologic confirmation is neither required nor appropriate. A PET/CT scan would not provide any additional information to the imaging that is already available and would not change management.
When is PSA indicated?
The U.S. Preventive Services Task Force recommends that clinicians discuss potential benefits and harms of PSA-based screening for prostate cancer in men aged 55 to 69 years. The decision to proceed with PSA level measurement should be individualized based on the patient's beliefs and values. In this 46-year-old patient, obtaining a serum PSA level is not indicated. The patient's family history of prostate cancer at an advanced age does not significantly increase his risk for cancer. He has a low pretest probability of prostate cancer, and a positive test result will likely be a false positive. Nevertheless, patients with positive test results often undergo biopsy with the attendant risks of infection, bleeding, pain, and anxiety.
A 33-year-old man is hospitalized for headache, hypertension, and an elevated serum creatinine level. He has a 10-year history of poorly controlled type 2 diabetes mellitus and hypertension. Medications are insulin glargine, insulin lispro, atorvastatin, amlodipine, and low-dose aspirin. On physical examination, blood pressure is 145/94 mm Hg; other vital signs are normal. Funduscopic examination reveals nonproliferative diabetic retinopathy. There is 1-mm pitting edema of the lower extremities to the ankles, equal on both sides. Dorsalis pedis and posterior tibial pulses are decreased bilaterally, and the feet are insensate. Laboratory studies: Complete blood count Normal Albumin 3.3 g/dL (33 g/L) Creatinine 1.8 mg/dL (159.1 µmol/L) Hemoglobin A1c 8.1% Antinuclear antibodies Negative Hepatitis B virus antibodies Negative Hepatitis C virus antibodies Negative HIV antibodies Negative Urinalysis No blood; 3+ protein Urine protein-creatinine ratio 6700 mg/g Kidney ultrasound reveals mildly increased echogenicity bilaterally, and both kidneys are enlarged at 12 cm. In addition to improved glycemic control, which of the following is the most appropriate management? Add an ACE inhibitor Obtain ANCA titers Obtain serum and urine protein electrophoresis Schedule a kidney biopsy
The addition of an ACE inhibitor or angiotensin receptor blocker (ARB) is the most appropriate management for this patient with diabetic nephropathy. The hallmark clinical features include a proteinuric form of chronic kidney disease in a patient with long-standing diabetes mellitus and evidence of other (microvascular and/or macrovascular) complications of disease. Diagnosis can be made clinically for this patient, and he can be treated with the cornerstone of treatment: improved glucose control, and blockade of the renin-angiotensin system (RAS) with an ACE inhibitor or ARB using the maximal tolerated dose. Combined use of any RAS drug class (ACE inhibitors, ARBs, and direct renin inhibitors) is not recommended; several clinical trials have revealed more adverse events with these combinations (hyperkalemia, hypotension, acute kidney injury), without additional cardiovascular or renal benefits. However, for patients with type 2 diabetes mellitus and chronic kidney disease, the American Diabetes Association recommends that physicians consider use of a sodium-glucose cotransporter 2 inhibitor for patients with an estimated glomerular filtration rate >30 mL/min/1.73 m2 and a urine albumin-creatinine ratio >30 mg/g. The recommendation is more strongly advised in patients with higher-grade albuminuria (urine albumin-creatinine ratio >300 mg/g) in which the sodium-glucose cotransporter 2 inhibitor will reduce progression of kidney disease and/or cardiovascular events. In patients with chronic kidney disease who are at increased risk for cardiovascular events, use of a glucagon-like peptide 1 receptor agonist may reduce risk of progression of albuminuria, cardiovascular events, or both. ANCA-associated glomerulonephritis is associated with the nephritic syndrome. The nephritic syndrome is characterized by hematuria, proteinuria, and leukocytes in the urine sediment. The hallmark is the presence of dysmorphic erythrocytes, with or without erythrocyte casts. Systemic findings may include edema, hypertension, and kidney failure. This patient does not have hematuria, making ANCA testing unnecessary. Serum and urine protein electrophoresis is used to evaluate for kidney failure secondary to dysproteinemia, which is more common in older patients (>65 years of age). Other potential clues to the presence of dysproteinemia-related kidney disease include anemia, hypercalcemia, and evidence of proximal tubular dysfunction such as hypokalemia, metabolic acidosis, hypophosphatemia, glycosuria (with normoglycemia), or hypouricemia. The diagnosis of diabetic nephropathy usually does not require a kidney biopsy if the clinical presentation is consistent with its diagnosis. The best predictors of finding diabetic nephropathy are duration of diabetes for more than 8 years followed by presence of the nephrotic syndrome. Therefore, this patient with a 10-year history of poorly controlled diabetes, nephrotic-range proteinuria, and findings of microvascular and macrovascular complications does not require a kidney biopsy or other testing to confirm the diagnosis of diabetic nephropathy.
A 92-year-old man is hospitalized with a complicated urinary tract infection (UTI). He resides in a nursing home and has a chronic indwelling urinary catheter. In the nursing home, a urinalysis and urine culture were performed for possible UTI, and empiric ciprofloxacin therapy was initiated 1 day before transfer to the hospital. In the emergency department, ciprofloxacin was changed to piperacillin-tazobactam, and blood cultures were obtained. Medical history is notable for dementia, benign prostatic hyperplasia, chronic kidney disease, and recurrent UTIs. Medications are donepezil, memantine, and piperacillin-tazobactam. On physical examination, temperature is 38.5 °C (101.3 °F), blood pressure is 108/70 mm Hg, pulse rate is 100/min, and respiration rate is 16/min. Suprapubic tenderness is noted, and the urinary catheter is draining cloudy urine. Other examination findings are noncontributory. Laboratory studies show a leukocyte count of 15,200/µL (15.2 × 109/L) and a serum creatinine level of 1.9 mg/dL (168 µmol/L). Urinalysis reveals leukocytes too numerous to count but no erythrocytes. Urine culture obtained from the nursing home shows more than 105 colony-forming units of Escherichia coli sensitive to piperacillin-tazobactam, gentamicin, cefepime, and meropenem (resistant to ceftriaxone, ceftazidime, cefotaxime, and ciprofloxacin); it is confirmed to be an extended-spectrum β-lactamase-producing organism. Blood cultures are pending. No infiltrates are seen on the chest radiograph. Which of the following is the most appropriate treatment? Add gentamicin Continue piperacillin-tazobactam Switch piperacillin-tazobactam to cefepime Switch piperacillin-tazobactam to meropenem
The carbapenem class of antibiotics (imipenem, meropenem, doripenem, ertapenem) is the preferred class of agents for treating infections with extended-spectrum β-lactamase-producing organisms. Antibiotic therapy should be switched from piperacillin-tazobactam to meropenem. This patient has a complicated urinary tract infection (UTI), defined by the presence of a chronic indwelling urinary catheter. The pattern of antibiotic susceptibility of Escherichia coli from the urine culture suggests an extended-spectrum β-lactamase (ESBL)-producing organism. ESBL-producing gram-negative organisms are capable of hydrolyzing higher generation cephalosporins that have an oxyimino side chain, including cefotaxime, ceftazidime, ceftriaxone, and cefepime. Laboratory identification of ESBLs is difficult because they are a heterogeneous group of enzymes. The carbapenem class of antibiotics (imipenem, meropenem, doripenem, ertapenem) is the preferred class of agents for treating infections with ESBL-producing organisms. Adding gentamicin would provide no benefit. Additionally, this patient has kidney disease; thus, aminoglycosides should be avoided if at all possible. On laboratory testing, ESBL-producing gram-negative organisms may appear susceptible to piperacillin-tazobactam; however, susceptibility breakpoints do not always reflect clinical success. Thus, piperacillin-tazobactam may be insufficient to treat infections with ESBL-producing organisms. An exception is uncomplicated UTI, in which piperacillin-tazobactam may be effective because high concentrations of the antibiotic are achievable in urine. The oxyimino cephalosporins (such as cefepime) should not be used, even if an ESBL-producing organism appears to be susceptible on laboratory testing. Treatment failures are common, even with higher doses, so carbapenems are the preferred antibiotic.
A 27-year-old man is evaluated during a routine health maintenance visit with his internist. He asks about reducing his risk for HIV infection because he has sex with men, has multiple partners, and reports using condoms "sometimes." Medical history is unremarkable, and he takes no medications. The physical examination is normal. Laboratory studies show a normal serum creatinine level. A serum rapid plasma reagin test is negative. Nucleic acid amplification testing for gonorrhea and chlamydia from urine, pharynx, and rectum are all negative. An HIV test is negative. Which of the following is the most appropriate preventive measure? Reinforce consistent condom use and avoid antiretroviral therapy Tenofovir and emtricitabine single dose before each sexual encounter Tenofovir and emtricitabine single dose after each sexual encounter Tenofovir disoproxil fumarate and emtricitabine daily
The combination of tenofovir disoproxil fumarate (or tenofovir alafenamide, except in women engaging in receptive vaginal intercourse) plus emtricitabine taken once daily is more than 90% effective, if taken consistently, in preventing HIV acquisition. The most appropriate preventive measure for this patient is daily tenofovir disoproxil fumarate and emtricitabine (TDF-FTC). He has multiple risk factors for acquiring HIV infection, including having sex with multiple partners without consistent condom use. Data support the use of pre-exposure prophylaxis (PrEP) in specific populations with ongoing high risk for infection, such as sexual partners of infected persons, men who have sex with men, and injection drug users. PrEP is recommended for persons in such groups who can adhere to the daily regimen. Efficacy has varied depending on adherence to the regimen; rates of reduction in new infections ranged from 42% to 75% and up to 92% in patients whose adherence was documented by monitoring drug blood levels. This patient is in a group shown in clinical trials to benefit from PrEP for HIV. Daily combination tenofovir alafenamide and emtricitabine is approved for PrEP as well, except in women engaging in receptive vaginal intercourse. Risk of kidney dysfunction from TDF is low, but kidney function should be checked every 6 months during PrEP. Patients taking PrEP should be counseled to continue using barrier precautions and should undergo testing for HIV, other sexually transmitted infections (STIs), and pregnancy in women, every 3 months. TDF-FTC given as a single dose before or after each sexual exposure has not been proven effective, and because of concerns for lower effectiveness and possible selection for resistance, these methods should not be used for prevention. Patients should be counseled regarding the need for continued barrier precautions during sex because the effectiveness of PrEP is less than 100% and to reduce transmission of other STIs. However, counseling the patient about consistent condom use without PrEP places the patient at unnecessary risk for HIV infection. PrEP and barrier protection is now the standard of prevention for men who have sex with men at risk for HIV infection.
A 56-year-old woman is evaluated during a wellness examination. She reports no vaginal bleeding, discharge, or other symptoms since reaching menopause at age 52 years. Over the past 3 months, she has noted a lack of interest in sexual activity that has been occasionally distressing for her. She uses vaginal lubricant for intercourse, which reduces the mild discomfort with sexual activity. She has no history of pelvic surgery, sexually transmitted infections, or sexual trauma. Results of screening tests for anxiety and depression are negative. Medical history is otherwise unremarkable, and she takes no medications. On physical examination, the external genitalia are normal. Pelvic examination reveals pale vaginal walls and a decrease in vaginal lubrication. The remainder of the examination is unremarkable. Which of the following is the most likely female sexual disorder diagnosis? Female orgasmic disorder Genitopelvic pain/penetration disorder Sexual interest/arousal disorder No female sexual disorder
The diagnosis of a female sexual disorder requires both significant distress and the persistence of symptoms not explained by a nonsexual mental disorder. This patient does not currently meet the diagnostic criteria for any sexual disorder. Female sexual dysfunction describes sexual difficulties that are persistent, personally distressing to the patient, and not explained by a nonsexual mental disorder. The patient reports that she is only occasionally distressed by these symptoms. Intervention may still be appropriate for patients reporting significant distress, such as sexual health education and/or referral to a sex therapist. Female orgasmic disorder is the persistent or recurrent absence, delay, or diminished intensity of orgasm following a normal excitement phase with at least 75% of sexual encounters. The patient does not describe symptoms compatible with this disorder. Genitopelvic pain/penetration disorder is diagnosed when there is persistent or recurrent difficulty in vaginal penetration during intercourse, marked vulvovaginal or pelvic pain during penetration, fear of pain or anxiety about pain in anticipation of or during penetration, or tightening or tensing of pelvic floor muscles during attempted penetration. For this diagnosis, symptoms must occur more than 75% of the time for at least 6 months and cause clinically significant distress. These symptoms are not present in this patient. Female sexual interest/arousal disorder includes hypoactive sexual desire or arousal dysfunction that is present for a minimum of 6 months and causes significant distress. It is diagnosed if the patient reports at least three of the following symptoms: lack of sexual interest, lack of sexual thoughts or fantasies, decreased initiation of sexual activity or decreased responsiveness to the partner's initiation attempts, reduced excitement or pleasure during sexual activity, reduced response to sexual cues, or decreased genital or nongenital sensations during sexual activity. This patient reports only occasional distress that has been present for 3 months.
A 67-year-old woman is evaluated for history of rhinorrhea, pharyngitis, nonproductive cough, and associated intermittent dyspnea. Her symptoms began 12 weeks ago associated with low-grade fevers, rhinorrhea, and pharyngitis. The fever, rhinorrhea, and pharyngitis resolved but her other symptoms persist. She has no chest pain, palpitations, edema, fever, chills, or orthopnea, and she has never smoked. Her medical history is significant for hypertension treated with an angiotensin receptor blocker. On physical examination vital signs are normal. Cardiac examination, including jugular venous pressure, is normal. Lung examination is normal. Laboratory studies, including complete blood count, are normal. Chest radiograph and spirometry are normal. Which of the following is the most appropriate diagnostic test to perform next? Echocardiography Exhaled nitric oxide testing High-resolution chest CT scan Methacholine challenge testing Nasal swab for influenza polymerase chain reaction
The diagnosis of asthma requires demonstrating reversible airflow obstruction; for a patient with symptoms of asthma and normal spirometry, methacholine challenge testing to evaluate for bronchial hyperresponsiveness is indicated. Methacholine challenge testing is the most appropriate test to perform next for this patient with persistent cough following a presumed viral upper respiratory tract infection. These symptoms can be the initial presentation of asthma, which is common in patients older than 65 years, with a prevalence of 8.1%; this age group also has the highest mortality rate, particularly in low-income Hispanic and black women. However, asthma is currently underdiagnosed and undertreated in patients older than 65 years in the United States. In patients with clinical symptoms suggestive of bronchospastic disease (such as cough or unexplained dyspnea) but with normal spirometry, bronchial challenge testing may be diagnostically helpful. Bronchial challenge testing uses a controlled inhaled stimulus to induce bronchospasm in association with spirometry; a positive test is indicated by a drop in the measured FEV1. This symptomatic patient's spirometry is normal; therefore, methacholine challenge testing to evaluate for bronchial hyperresponsiveness is indicated. Echocardiography could help evaluate cardiac function but the patient does not have findings of heart murmur or heart failure so this test would not be indicated. Exhaled nitric oxide testing is a noninvasive breath test. Nitric oxide is normally present in airways but is increased in certain types of airway inflammation (asthma, eosinophilic airway inflammation). When elevated, it supports the diagnosis of asthma in the appropriate clinical context. Other factors may affect nitric oxide values such as age, sex, atopy, and cigarette smoking so a normal level in an older adult would not rule out asthma. The sensitivity and specificity of exhaled nitric oxide in the diagnosis of asthma are not well defined, particularly in patients with confounding variables, and it is not the preferred next test for this patient. High-resolution CT (HRCT) is indicated if diffuse parenchymal lung disease is suspected. HRCT can help narrow the differential diagnosis based on the distribution of the lung parenchymal abnormalities and the presence or absence of associated findings. HRCT scan of the chest would not help confirm a diagnosis of asthma. Nasal swab for influenza polymerase chain reaction would not be indicated because the patient's symptoms of a viral infection have resolved; furthermore, influenza testing in immunocompetent adults should be performed within 5 days of symptom onset. Read Related TextNext Question
A 17-year-old girl is seen for a health maintenance evaluation. She reports no health issues. In discussing dietary habits, she states that she frequently eats very large amounts of food until she is uncomfortably full. These episodes have occurred at least twice per week for the past year; during them, she feels a loss of control over her eating and guilt about her overconsumption. She often eats large amounts of food despite not being hungry, and she prefers eating alone because she is embarrassed of her eating. She does not take laxatives or induce vomiting after such episodes. She rarely exercises. BMI is 30. Which of the following is the most likely diagnosis? Anorexia nervosa, binging subtype Binge eating disorder Bulimia nervosa Overeating
The diagnosis of binge eating disorder requires at least three of the following characteristics occurring at least once weekly for 3 months: abnormally rapid consumption, consuming large amounts of food when not hungry, eating alone due to embarrassment, eating until uncomfortably full, and feelings of guilt related to overconsumption. This patient most likely has binge eating disorder (BED), which is characterized by impulsive overeating and feeling loss of control around food. The diagnosis of this disorder requires at least three of the following characteristics occurring at least once weekly for 3 months: abnormally rapid consumption, consuming large amounts of food when not hungry, eating alone due to embarrassment, eating until uncomfortably full, and feelings of guilt related to overconsumption. BED is more common than both anorexia and bulimia nervosa and is often accompanied by other psychiatric problems. The primary treatment is cognitive behavioral therapy. Bulimia nervosa and the binging subtype of anorexia nervosa both include episodes of binge eating like BED. The key in differentiating BED from these diseases is the lack of compensatory behaviors (such as induced vomiting and laxative abuse) to avoid weight gain. The major difference between bulimia nervosa and the binging subtype of anorexia nervosa is that patients with anorexia nervosa have a low BMI (usually <18). Many people have episodes of overeating in which they may eat until uncomfortable or feel guilty about their eating. However, simple overeating does not meet the diagnostic criteria for BED and is not accompanied by the feelings of loss of control over food consumption.
A 45-year-old woman is evaluated for episodic nausea, bloating, and epigastric pain of 5 years' duration. In the past 3 months, the nausea has been accompanied by occasional vomiting. She also reports near-daily heartburn symptoms that have not responded to daily omeprazole. She has a 10-year history of type 2 diabetes mellitus that is treated with metformin and glyburide. On physical examination, vital signs are normal; BMI is 29. Abdominal examination shows diffuse tenderness to deep palpation with no guarding. Other findings are normal. Laboratory studies show a blood hemoglobin A1c level of 7.5%. The basic metabolic panel is normal. A complete blood count and liver chemistry tests are normal. Upper endoscopy shows a moderate amount of retained food in the stomach and patchy erythema of the gastric mucosa. Biopsies of the stomach are normal. Which of the following is the most appropriate next step in management? Gastric emptying scintigraphy 24-Hour pH probe Initiation of domperidone Initiation of metoclopramide
The diagnosis of gastroparesis requires the presence of specific symptoms, absence of mechanical outlet obstruction, and objective evidence of delay in gastric emptying into the duodenum. Gastric emptying scintigraphy is the most appropriate next step in management. The diagnosis of gastroparesis requires: (1) the presence of specific symptoms; (2) the absence of mechanical outlet obstruction; and (3) objective evidence of delay in gastric emptying into the duodenum. Commonly reported symptoms include early satiety, postprandial fullness, nausea, vomiting, upper abdominal pain, bloating, and weight loss, but these symptoms correlate poorly with the findings on objective gastric emptying tests. Various other upper gastrointestinal disorders can present with similar symptoms. Exclusion of other upper gastrointestinal disorders, objective documentation of delayed gastric emptying, and an attempt to identify the cause of the gastroparesis are essential before treatment. Retained food in the stomach during upper endoscopy is not objective evidence of delayed gastric emptying. The three tests to objectively demonstrate delayed gastric emptying are gastric scintigraphy, wireless motility capsule, and the gastric emptying breath test. If scintigraphy is pursued, the 4-hour study is preferred over 90- or 120-minute studies due to increased diagnostic accuracy. A 24-hour pH probe may be considered when heartburn symptoms do not respond to a higher dose of acid suppression therapy, such as twice-daily proton pump inhibitor therapy or a proton pump inhibitor plus a histamine receptor antagonist. This patient's medical therapy for heartburn symptoms should be optimized before further testing is pursued, and this test will not explain the patient's predominant symptoms of nausea, bloating, and epigastric pain. Both metoclopramide and domperidone are effective in the treatment of gastroparesis. Metoclopramide is the only FDA-approved agent for the treatment of gastroparesis. Domperidone can be used under a special program administered by the FDA. The side effects of metoclopramide include dystonia, Parkinson-type movements, and tardive dyskinesia. Domperidone can prolong the QT interval on electrocardiography, potentially leading to cardiac arrhythmia. Before initiating treatment for gastroparesis, it is necessary to confirm the diagnosis.
Differential for nephrotic syndrome with AKI?
The differential diagnosis for the nephrotic syndrome with AKI is limited to only a few entities: MCG with acute tubular necrosis or allergic interstitial nephritis, membranous glomerulopathy with bilateral renal vein thrombosis, amyloidosis with cast nephropathy, and collapsing focal segmental glomerulosclerosis.
A 63-year-old woman is evaluated for diarrhea characterized by three to four large-volume, watery stools per day over a period of 14 months with gradually increasing severity and frequency. She now reports occasional urge fecal incontinence and nocturnal diarrhea but no abdominal pain, bloody stools, or weight loss. She has been taking loperamide up to five times daily, but symptoms have persisted. On physical examination, vital signs are normal; BMI is 26. Abdominal examination is normal with no tenderness or distention. Rectal examination reveals no blood or masses. Results of routine laboratory studies are normal. Polymerase chain reaction testing of the stool for Clostridium difficile is negative. Colonoscopy results are normal. Random colon biopsy specimens show lymphocytic infiltration of the mucosa with a subepithelial collagen band. Which of the following is the most appropriate treatment? Bismuth subsalicylate Budesonide Mesalamine Prednisone Probiotics
The first step in the management of microscopic colitis is to discontinue a potentially causative medication, after which supportive treatment with antidiarrheal agents such loperamide can be tried, with budesonide recommended for patients whose symptoms do not respond. Budesonide is the most appropriate treatment for this patient. Her primary symptom of chronic watery diarrhea, colonoscopy results showing normal-appearing mucosa, and biopsy results revealing lymphocytic infiltration and a subepithelial collagen band are diagnostic for collagenous colitis. Collagenous colitis is a subtype of microscopic colitis. It is a clinicopathologic diagnosis made based on clinical presentation, endoscopy features, and histopathology. The condition occurs more commonly in women than in men and typically presents with abrupt or gradual onset of watery diarrhea that has a relapsing and remitting course over months to years. Mild weight loss may occur. The colonic mucosa is macroscopically normal, and inflammatory changes are only appreciated on histopathologic review of colon biopsy specimens. Several medication classes, including NSAIDs, selective serotonin reuptake inhibitors, and proton pump inhibitors, have been associated with the development of microscopic colitis. The first step in management is to discontinue a potentially causative medication. Supportive treatment with antidiarrheal agents such loperamide can be tried as initial treatment. For patients like this one, whose symptoms do not respond to antidiarrheal medication, the American Gastroenterological Association (AGA) strongly recommends, based on moderate-quality evidence, the use of budesonide for induction of clinical remission of microscopic colitis because of its favorable harm-benefit profile and convenience of once-daily dosing. The rate of relapse after discontinuation of budesonide is high, and maintenance therapy with the lowest possible dose to maintain remission may be required. Patients treated with budesonide for longer than 6 months should be monitored for corticosteroid-related adverse effects. Because budesonide is expensive, alternative treatments such as bismuth salicylate may be considered if cost is a determining factor. The AGA conditionally recommends, based on low-quality evidence, bismuth subsalicylate for induction of remission when budesonide therapy is not feasible. Bismuth subsalicylate therapy consists of two to three 262-mg tablets taken orally three to four times daily. The benefit of mesalamine in achieving clinical remission in microscopic colitis is uncertain, and it is recommended as a potential second-line therapy. Prednisone should not be used as first-line treatment of microscopic colitis because of its unfavorable side effects, but it may be considered in patients who have microscopic colitis refractory to budesonide. The AGA conditionally recommends, based on low-quality evidence, against the use of probiotics over no treatment for induction of clinical remission. Various probiotic strains, dosages, and formulations are available, but most have not been evaluated in the treatment of microscopic colitis.
A 38-year-old woman is evaluated for a 2-day history of worsening eye pain and decreasing visual acuity in her left eye. The pain is worse with eye movement. She reports no fever or trauma. Vision loss is mostly central, and her ability to distinguish colors has diminished. She does not feel the sensation of a foreign body in the eyes. She wears contact lenses. Her medical history is otherwise unremarkable. On physical examination, vital signs are normal. Eye movement is intact but painful, with visual acuity of 20/20 in the right eye and 20/60 in the left. Afferent pupillary defect is noted in the left eye. There is no corneal injection or discharge, and the optic discs appear normal. Which of the following is the most likely diagnosis? Corneal abrasion Herpes simplex keratitis Optic neuritis Orbital cellulitis
The hallmarks of optic neuritis are acute vision loss, eye pain with movement, color perception change, and afferent pupillary defect; results of a funduscopic examination may be normal. This patient with acute vision loss and eye pain unassociated with trauma has signs and symptoms suggestive of optic neuritis, including pain with eye movement, loss of color vision out of proportion to the vision loss, and an afferent pupillary defect. Two thirds of optic neuritis cases occur in women. The average age of onset is between 20 and 40 years, and it is often associated with multiple sclerosis. Most of these patients have a normal optic disc on funduscopy, but one third may have a swollen disc or papillitis. An urgent evaluation by an ophthalmologist is required; treatment usually involves high-dose intravenous glucocorticoids. Corneal abrasion can cause sudden onset of pain and foreign-body sensation. It is classically seen in patients who sleep without taking out their contact lenses and then awaken with eye pain and photophobia. If the abrasion is in the central area of the visual axis, visual acuity may be diminished. Corneal abrasion cannot explain the loss of color discrimination and the afferent pupillary defect in this patient. Herpes simplex keratitis typically presents with acute onset of pain, blurry vision, and watery discharge. The absence of discharge and ciliary flush in this case make keratitis unlikely. Ciliary flush is characterized by erythema that is most marked at the limbus, which is the junction of the sclera and cornea. Keratitis would not be associated with loss of color discrimination or an afferent pupillary defect. Orbital cellulitis often presents with eye pain as well as eyelid swelling and erythema, although some cases present without erythema. In the case of inflammation of the extraocular muscles and fatty tissue in the orbit, the patient may experience pain with eye movement. When the condition is severe, visual acuity may be impaired. Orbital cellulitis, however, is more likely to be associated with fever and chemosis, which are not present in this patient, and it would not explain the patient's other findings related to optic nerve damage.
Two new treatments for patients with heart failure with preserved ejection fraction were compared in a randomized controlled trial. The primary outcome was reduction in heart failure-related hospitalizations. Compared with treatment B, treatment A was associated with a statistically significant absolute risk reduction of 6%, and the number needed to treat to prevent one hospitalization was 17. Which of the following is needed to conclude that treatment A is superior to treatment B? Confidence interval Harms and cost of treatment P value Relative risk reduction
The harms and cost of treatment are needed to conclude that treatment A is superior to treatment B. When assessing the clinical impact of an intervention, the number needed to treat (NNT) provides a quantifiable measure of the treatment effect that is easily understood by physicians and patients; it represents the number of patients who must receive a treatment to cause one additional patient to benefit. The acceptability of the NNT as a means of comparing one treatment with another depends on the risks associated with the condition, the cost and side effects of the treatment, and other treatments available. When comparing one treatment with another, head-to-head comparisons provide the best evidence of superiority. In this head-to-head comparison of two treatments, the absolute risk reduction for heart failure-related hospitalizations is 6% with treatment A compared with treatment B. This translates to 17 patients (NNT = 1/absolute risk reduction) who need to receive treatment A to result in 1 less heart failure-related hospitalization compared with treatment B. Although this information is informative, other data, such as cost and harms, must be evaluated before a conclusion that treatment A is superior to treatment B can be reached. If harms are more frequent or more severe with treatment A, the reduction in hospitalization for heart failure may become clinically meaningless. Confidence intervals (CIs) are a method for indicating the range in which a value derived from a study is likely to lie; narrower ranges imply greater confidence, or certainty, that the reported value is closer to the true value. The P value expresses the probability that the findings in a study can be explained by chance alone and represents the level of statistical significance. P values offer less information than do CIs because CIs can demonstrate the plausible range of values for an event or outcome, whereas P values indicate only statistical significance. Although CIs provide more precise information about the range of expected benefit, the P value and CI are of less importance than understanding the harms, costs, and alternative therapies that might be available. A disadvantage of relative comparisons, including relative risk, is the potential for exaggerated outcomes. For instance, interventions that reduce the rate of an outcome from 40% to 20% or from 4% to 2% have a relative risk reduction of 50%. However, the absolute risk reduction for the first case is 20%, whereas the absolute risk reduction for the second case is 2%.
A 37-year-old woman is evaluated for a 6-year history of difficulty initiating sleep and significant daytime sleepiness. She works as a registered nurse, and she notes that her daytime sleepiness is exacerbated when she works several night shifts in a row at the hospital. She does not drink alcohol, use tobacco, or consume caffeinated products. She recently initiated a new exercise program and exercises three to four mornings each week. She has no other symptoms and takes no medications. On physical examination, vital signs are normal. BMI is 30. Oxygen saturation breathing ambient air is 97%. The remainder of the physical examination is normal. Which of the following is the most appropriate next step in management? Multiple sleep latency testing Overnight oximetry Polysomnography Serum ferritin level measurement Two-week sleep diary
The initial evaluation of chronic insomnia involves obtaining a sleep diary to identify adverse environmental factors, inappropriate exposure to electronic screens before bedtime, and sleep patterns. The most appropriate next step in management of this patient with symptoms of chronic insomnia is to obtain a 2-week sleep diary. Symptoms of insomnia vary and may include poor sleep quality, frustration with sleep quantity, difficulty initiating sleep, or an inability to return to sleep after awakening. In patients with symptoms of insomnia, a thorough history, physical examination, and medication review may point to potentially reversible causes, such as sleep apnea or restless legs syndrome. In the absence of specific historical features that are consistent with a primary sleep disorder, the initial evaluation of insomnia additionally involves obtaining a sleep diary to identify adverse environmental factors, inappropriate exposure to electronic screens (computer, phone, tablet, television) before bedtime, and sleep patterns. If the sleep diary reveals red flags for a primary sleep disorder or another condition that may interfere with sleep, further diagnostic testing would be indicated. Use of the Insomnia Severity Index or the Athens Insomnia Scale can also be a helpful component of a comprehensive sleep assessment. In this patient with daytime somnolence, fatigue, and difficulty with sleep initiation, a sleep log may facilitate collection of an accurate sleep history. Polysomnography is indicated in patients for whom there is a strong suspicion of a primary sleep disorder based on the initial history and physical examination. Polysomnography typically involves overnight, laboratory-based testing that is monitored by a sleep technician. Similarly, multiple sleep latency testing is a labor-intensive evaluation designed to identify diagnoses of narcolepsy and idiopathic hypersomnia. Neither study is indicated in the initial evaluation of this patient, whose primary problem appears to be initiating sleep. Overnight pulse oximetry has a high rate of false-positive and false-negative results and has not been validated as a screening tool for obstructive sleep apnea. Normal-appearing results on overnight pulse oximetry may allow for avoidance of further testing in patients with a low pretest probability. However, there is no clinical suspicion for obstructive sleep apnea in this patient, and it would not be indicated in her initial evaluation. Low serum ferritin levels are strongly correlated with restless legs syndrome, which is characterized by discomforting sensations in the legs at rest or when falling asleep, an urge to move the legs, and immediate relief after moving the legs or walking. However, no features in the history suggest restless legs syndrome as a cause of this patient's chronic insomnia.
A 27-year-old woman is hospitalized with a 2-month history of progressive exertional dyspnea and edema. She has no history of recent flu-like symptoms, weight change, arthritis, or chest pain. On physical examination, the patient is afebrile, blood pressure is 118/66 mm Hg, pulse rate is 112/min, and respiration rate is 18/min. Oxygen saturation is 98% breathing ambient air. There is jugular venous distention to the angle of the jaw. Crackles are present at the lung bases. Cardiac examination reveals an S3. Peripheral edema is noted. Complete blood count, serum electrolytes, kidney function tests, liver chemistry tests, glucose level, and lipid levels are normal. An electrocardiogram shows sinus tachycardia with normal intervals and no conduction delay. An echocardiogram demonstrates a left ventricular ejection fraction of 25% and normal left ventricular diastolic diameter. Which of the following is the most appropriate diagnostic test to perform next? Acute and convalescent viral titers Cardiac catheterization Serum iron and ferritin studies Thyroid function studies
The initial laboratory evaluation of patients with new-onset heart failure should include a B-type natriuretic peptide or N-terminal pro-B-type natriuretic peptide assay, complete blood count, serum electrolyte measurement, kidney function tests, liver chemistry tests, and serum thyroid-stimulating hormone measurement. Thyroid studies should be performed next in this patient with new-onset heart failure. Once a diagnosis of heart failure is suspected, echocardiography should be performed to ascertain the left ventricular ejection fraction and to potentially determine the cause and chronicity of the patient's heart failure. A large, dilated left ventricle suggests a chronic process with less chance of recovery. The initial laboratory evaluation of patients with new-onset heart failure should include a B-type natriuretic peptide or N-terminal pro-B-type natriuretic peptide assay, complete blood count, serum electrolyte measurement, kidney function tests, liver chemistry tests, glucose and lipid levels, and serum thyroid-stimulating hormone measurement. Specifically, thyroid-stimulating hormone measurement is indicated to evaluate for occult hypo- or hyperthyroidism as a reversible cause of heart failure. In hyperthyroidism in particular, the predominant manifestation of thyroid dysfunction may be cardiac symptoms, which will abate when the hyperthyroidism is treated. The classic presentation of viral myocarditis includes a viral prodrome with fever, myalgia, and upper respiratory tract symptoms, but a prodrome does not always occur. Patients present with dyspnea, chest pain, and arrhythmias. Electrocardiographic abnormalities are often observed. This patient's presentation is not typical for viral myocarditis, and viral studies are not indicated at this time. Although coronary artery disease is the most common cause of heart failure, the investigation for coronary artery disease by stress testing or cardiac catheterization is not considered part of the routine evaluation of all patients with newly diagnosed heart failure, owing to expense and radiation exposure. Cardiac catheterization should be performed in patients presenting with angina or significant ischemia. Hereditary hemochromatosis results from an autosomal recessive defect in the HFE gene, which leads to increased absorption of dietary iron. Patients may present with heart failure or arrhythmias. However, women do not develop overt iron overload until after menopause, and serum iron and ferritin studies are not indicated in this young woman.
A 46-year-old man is evaluated for abdominal pain in the right upper quadrant and fever of 1 month's duration. He recently emigrated from Mexico. His medical history is unremarkable, and he takes no medication. On physical examination, temperature is 37.7 °C (99.9 °F); other vital signs are normal. Abdominal examination shows tenderness to palpation of the right upper quadrant. No scleral icterus is noted. The remainder of the examination is normal. An ultrasound of the liver shows a fluid-containing structure and complex wall consistent with hepatic abscess. Laboratory studies show a leukocyte count of 10,600/μL (10.6 × 109/L). Testing for Entamoeba histolytica IgG is positive. Which of the following is the most appropriate treatment? Meropenem Metronidazole and paromomycin Percutaneous drainage of the abscess Surgical resection
The mainstay of therapy for amebic liver abscesses is antibiotic therapy, such as metronidazole, plus a luminal agent, such as paromomycin, to eradicate the coexisting intestinal infection. Metronidazole plus paromomycin is the most appropriate treatment for this patient with an amebic liver abscess. The two most common types of hepatic abscess are pyogenic and amebic. Pyogenic liver abscesses are the most common hepatic abscesses in the United States, whereas amebic abscesses are seen more frequently in the developing world. Pyogenic abscesses are typically polymicrobial, originating from gastrointestinal flora in the setting of intra-abdominal infections, malignancies, or procedures. Amebic abscesses are usually caused by the organism Entamoeba histolytica, and the mechanism of formation involves enteric infection with the invasion of amoeba through the intestinal mucosa and via the portal vein. With amebic abscesses, infection of the liver is asymptomatic until hepatic necrosis results in abscess development, at which time abdominal pain, fever, and leukocytosis develop. Amebic liver abscesses are seen in patients from endemic areas, such as India, Africa, or Central or South America, or in patients who have traveled to endemic areas. The typical presentation in travelers occurs 3 to 5 months after infection. Nearly all patients with an amebic abscess have positive serological studies for E. histolytica. The diagnosis is usually established with compatible imaging and serologic testing. Ultrasonography is less sensitive for the detection of pyogenic liver abscesses (CT is preferred) but may be equally sensitive for the detection of amebic abscesses. The mainstay of therapy for an amebic liver abscess is antibiotic therapy, such as metronidazole, plus a luminal agent, such as paromomycin, to eradicate the coexisting intestinal infection. Meropenem is an effective broad-spectrum antibiotic that is often used to treat pyogenic liver abscesses. It has no role in the treatment of an amebic abscess. Percutaneous drainage and surgical resection of an abscess are treatment options indicated for pyogenic liver abscesses. Surgical intervention is usually necessary for abscesses 5 cm or larger in size, complex abscesses, presence of gas-forming organisms, hemodynamic instability, biliary fistulization, or presence of a foreign body. These procedures are typically not needed in the treatment of amebic liver abscesses, which usually resolve with antimicrobial therapy.
A 22-year-old woman is evaluated for intractable pruritus that keeps her awake at night. She is in her 25th week of pregnancy. Her medications are a prenatal vitamin and a folate supplement. On physical examination, vital signs are normal. Scleral icterus is noted. Excoriations are seen on the arms, chest, abdomen, and legs. Laboratory studies: Hematocrit 35% Platelet count 370,000/μL (370 × 109/L) Alanine aminotransferase 55 U/L Aspartate aminotransferase 30 U/L Bile acids Elevated Total bilirubin 2.5 mg/dL (42.8 µmol/L) Testing for hepatitis C and hepatis B viral infections is negative. An ultrasound shows a normal liver with no dilated bile ducts. Which of the following is the most appropriate next step in management? Liver biopsy Peripheral blood smear Topical glucocorticoids Ursodeoxycholic acid
The mainstay of therapy for intrahepatic cholestasis of pregnancy is ursodeoxycholic acid, which is associated with alleviated symptoms and improved liver test abnormalities. Ursodeoxycholic acid therapy is the most appropriate management for this patient. Intrahepatic cholestasis of pregnancy is a liver condition that affects pregnant women in the second or third trimester. This condition spontaneously improves within 48 hours of delivery. Typical symptoms include diffuse pruritus, and there is typically a mild increase in serum transaminase concentrations. Associated risks include premature delivery as well as higher rates of fetal death. Mutations in bile salt transporters have been implicated in the pathophysiology. The mainstay of therapy for intrahepatic cholestasis of pregnancy is ursodeoxycholic acid, which is associated with alleviated symptoms, improved liver test abnormalities but did not change fetal outcomes. Cholestasis of pregnancy can be diagnosed in pregnant women with otherwise unexplained pruritus and abnormal liver chemistry tests. Elevated levels of total serum bile acid in a pregnant woman with pruritus are considered diagnostic for cholestasis of pregnancy in the absence of an alternative diagnosis. In this patient with findings compatible with intrahepatic cholestasis of pregnancy and a normal hepatic ultrasound, a liver biopsy is not needed. HELLP (Hemolysis, Elevated Liver enzymes, and Low Platelets) syndrome presents during the third trimester and is an advanced complication of preeclampsia. HELLP syndrome typically presents with abdominal pain, new-onset nausea and vomiting, pruritus, and jaundice. A peripheral blood smear will reveal schistocytes. This patient is not anemic and does not have thrombocytopenia or hypertension; therefore, a peripheral blood smear to support the diagnosis of HELLP syndrome is not needed. Pruritic urticarial papules and plaques of pregnancy (PUPPP) is the most common specific dermatosis of pregnancy. PUPPP is a clinical diagnosis confirmed by the appearance late in the third trimester of erythematous plaques in the distribution of striae. Persistent and bothersome pruritus is a symptomatic hallmark of the condition. It is not associated with jaundice or elevated transaminase levels. The condition usually resolves shortly after delivery. The first-line option for therapy is usually topical glucocorticoids of low to mid potency. Topical glucocorticoids are of no value in intrahepatic cholestasis of pregnancy.
A 55-year-old man is evaluated after emergent treatment for an episode of hematemesis. Emergency endoscopy was performed in the emergency department, and the bleeding was successfully treated with band ligation. The endoscopy revealed esophageal varices, one of which had stigmata of recent hemorrhage. Treatment with octreotide and a proton pump inhibitor was initiated. The patient has a history of cirrhosis due to hepatitis C viral infection. He has no other medical problems and takes no medication. On physical examination, vital signs and other findings are normal. Laboratory studies show a hemoglobin level of 8.9 g/dL (89 g/L), leukocyte count of 3600/µL (3.6 × 109/L), and platelet count of 80,000/µL (80 × 109/L). Which of the following is the most appropriate next treatment? Blood transfusion Ciprofloxacin Platelet transfusion Transjugular intrahepatic portosystemic shunt placement
The mainstay of therapy for variceal hemorrhage is endoscopic therapy, and adjunctive therapies such as antibiotic therapy improve outcomes. Ciprofloxacin for 7 days is the most appropriate next treatment for this patient with variceal bleeding. Variceal bleeding is a life-threatening complication of portal hypertension. Risk factors for variceal hemorrhage are Child-Turcotte-Pugh class B and C cirrhosis, large varices (>5 mm), and the endoscopic finding of red markings on varices. Approximately 15% to 20% of patients die within 6 weeks of hemorrhage. The mainstay of therapy for variceal hemorrhage is endoscopic therapy. Antibiotic therapy is an important adjunctive therapy for variceal bleeding because bacterial infection occurs in 30% to 40% of patients within 1 week of variceal bleeding. The most common infections seen are spontaneous bacterial peritonitis as well as bacteremia, urinary tract infections, and pneumonia. Antibiotic therapy after variceal bleeding reduces rates of infection and rebleeding as well as mortality after variceal bleeding. There is also benefit to administering antibiotics for patients with cirrhosis who present with nonvariceal upper gastrointestinal bleeding. This patient does not require a blood transfusion. Hemodynamically stable patients with acute upper gastrointestinal bleeding, no acute coronary syndrome, and no history of peripheral vascular disease or stroke should be transfused when the hemoglobin level is 7 g/dL (70 g/L) or less. This transfusion threshold is associated with improved clinical outcomes compared to higher transfusion thresholds. The goal of transfusion is a hemoglobin level of 7 g/dL (70 g/L) to 9 g/dL (90 g/L). In the setting of hemodynamic stability, overtransfusion should be avoided because it can cause an increase of portal pressure and thereby precipitate rebleeding. Platelet transfusion is not warranted because hemostasis has already been achieved. This patient has thrombocytopenia, but a transfusion of platelets also has the potential to cause volume overload leading to an increase in portal hypertension and rebleeding. Patients with thrombocytopenia with clinically significant active bleeding, as well as patients without bleeding in whom nonneuraxial surgery is planned, should be transfused to a target platelet count of 50,000 to 100,000/µL (50-100 × 109/L) depending on the clinical circumstances. Transjugular intrahepatic portosystemic shunt (TIPS) placement is useful in the 10% to 20% of patients with variceal bleeding in whom hemostasis is unable to be achieved by endoscopic therapy. In this patient in whom endoscopic therapy was successful, TIPS placement is not indicated.
A 45-year-old man is evaluated for extreme fatigue accompanied by a 9.1-kg (20 lb) weight loss over the past 2 months. He also has had an occasional productive cough for 1 month. He is unemployed and has been sleeping in homeless shelters for approximately 6 months. He drinks alcohol daily, consuming a fifth of vodka approximately every 2 to 3 days; he does not smoke or use illicit drugs. On physical examination, temperature is 38.3 °C (100.9 °F), blood pressure is 110/60 mm Hg, pulse rate is 90/min, and respiration rate is 18/min. Bilateral crackles are present throughout inspiration over the upper posterior thorax. The remainder of the examination is unremarkable. Interferon-γ release assay is positive. HIV antibody testing is negative. The chest radiograph is shown. Which of the following is the most appropriate management? Initiate isoniazid plus pyridoxine Initiate isoniazid, rifampin, pyrazinamide, and ethambutol Initiate piperacillin-tazobactam Obtain sputum specimen for acid-fast bacilli stain and culture
The microbiologic diagnosis of tuberculosis should be verified by acid-fast bacilli staining of sputum samples and nucleic acid amplification testing before initiating antituberculous therapy. The most appropriate management for this patient is to obtain a sputum specimen and perform acid-fast bacilli (AFB) staining and culture. The patient appears to have active pulmonary tuberculosis, based on an indolent course of cough, fever, fatigue, and weight loss and a posteroanterior chest radiograph showing bilateral upper lobe cavitary infiltrates and a left pleural effusion. Although it is essential to initiate antimicrobial therapy promptly, it is important to verify the microbiologic diagnosis of Mycobacterium tuberculosis infection first by submitting three sputum specimens for AFB staining and culture. If the AFB stain is positive, nucleic acid amplification testing (NAAT) can be used to differentiate M. tuberculosis from other types of mycobacteria, allowing for early initiation of treatment. It is also important to perform susceptibility testing because the frequency of antimicrobial resistance (for isoniazid and rifampin) is increasing. The tuberculin skin test (TST) and interferon-γ release assay (IGRA) are the initial diagnostic studies used to evaluate for tuberculosis infection. However, neither test can distinguish between latent and active tuberculosis. Latent tuberculosis infection is diagnosed when an asymptomatic patient has a positive TST or IGRA result without clinical evidence of active tuberculosis infection by history, physical examination, or chest imaging. Because this patient has active tuberculosis, treatment for latent tuberculosis with isoniazid plus pyridoxine would not be appropriate. This regimen should only be used in patients with latent tuberculosis. For active tuberculosis infection, the Centers for Disease Control and Prevention recommends initiating four-drug therapy with isoniazid, rifampin, pyrazinamide, and ethambutol for 8 weeks. In susceptible isolates, isoniazid and rifampin should then be continued for 18 weeks. This treatment regimen is appropriate only after M. tuberculosis infection has been confirmed with positive stain and NAAT. Piperacillin-tazobactam would be an appropriate antibiotic choice if pneumococcus, Klebsiella pneumoniae, or Pseudomonas aeruginosa were the likely pathogens. However, the presentation of an indolent infection, a positive IGRA, and evidence of fibrocavitary disease in the upper lobes is highly suspicious for pulmonary tuberculosis, not acute bacterial pneumonia.
A 25-year-old woman is hospitalized with a 4-day history of fever and cough productive of brown sputum. She is at 14 weeks' gestation with her first pregnancy. Medical history is significant for mild persistent asthma. Medications are an albuterol inhaler, beclomethasone inhaler, and a prenatal vitamin. On physical examination, temperature is 38.2 °C (100.8 °F), blood pressure is normal, pulse rate is 122/min, and respiration rate is 24/min. Oxygen saturation is 94% breathing ambient air. Crackles are heard at the left lung base on pulmonary auscultation. Chest radiograph shows a left lower lobe infiltrate. Which of the following is the most likely cause of pneumonia in this patient? Escherichia coli Klebsiella pneumoniae Listeria monocytogenes Staphylococcus aureus Streptococcus pneumoniae
The microbiology of community-acquired pneumonia in pregnancy is similar to that seen in the general population; among patients requiring hospitalization, the most common pathogens are Streptococcus pneumoniae, Haemophilus influenzae, and atypical organisms, including Legionella species, Chlamydia pneumoniae, and Mycoplasma pneumoniae. Streptococcus pneumoniae is the most likely cause of this patient's community-acquired pneumonia (CAP). Pneumonia is the most common cause of fatal nonobstetric infection in pregnancy. The microbiology of CAP in pregnancy is similar to that seen in the general population. Among patients requiring hospitalization, the most common pathogens are S. pneumoniae, Haemophilus influenzae, and atypical organisms, including Legionella species, Chlamydia pneumoniae, and Mycoplasma pneumoniae. Empiric treatment of pregnant patients is similar to that in nonpregnant adults, although quinolones and tetracyclines are relatively contraindicated because of the potential for teratogenic effects. In addition to these common bacterial causes of CAP, pregnant women are at increased risk for serious viral pneumonia from influenza virus and varicella-zoster virus, so it is recommended that pregnant women receive seasonal influenza vaccination. Gram-negative bacteria, including Klebsiella pneumoniae, Pseudomonas aeruginosa, Acinetobacter species, Escherichia coli, and Enterobacter species, are rarely implicated in CAP, including among pregnant women hospitalized for pneumonia. Most patients with CAP caused by gram-negative bacteria have a predisposing risk factor, such as bronchiectasis, cystic fibrosis, or COPD, and develop severe pneumonia necessitating admission and care in the ICU. Pregnancy causes a decrease in T-cell function, and pregnant women are at increased risk for severe Listeria infections, including meningitis and sepsis. However, Listeria rarely causes pulmonary infection and would be an unlikely cause of infection in this patient. Staphylococcus aureus is an increasingly recognized cause of CAP, with risk factors including antecedent viral infection or injection drug use. Maternal S. aureus infection can occur perinatally, related to delivery, surgery, or indwelling lines, but remains a rare cause of CAP in the prenatal period.
A 24-year-old woman is evaluated for a breast lump. She has had no breast trauma or discharge from the nipples. She is nulliparous and has regular menstrual cycles. Medical history is otherwise unremarkable. The patient's mother was recently diagnosed with breast cancer at age 58 years; no other family members have breast or ovarian cancer. Her only medication is an oral contraceptive pill. On physical examination, vital signs are normal. BMI is 25. A breast examination reveals no skin changes, with dense breast tissue bilaterally. She has a firm, 2-cm, nontender, mobile mass with well-defined margins in the upper outer quadrant of the left breast. There is no evidence of axillary, cervical, or supraclavicular lymphadenopathy. Which of the following is the most appropriate test to perform in this patient? Biopsy Mammography Mammography and ultrasonography Ultrasonography
The most appropriate diagnostic test for this young woman with a breast mass is ultrasonography. A breast mass is characterized by a lesion that persists throughout the menstrual cycle and differs from the surrounding breast tissue and the corresponding area in the contralateral breast. The differential diagnosis of a palpable breast mass includes abscess, cyst, fat necrosis, fibroadenoma, and neoplasm. Evaluation of a palpable breast mass varies based on the patient's age and risk factors and the degree of clinical suspicion. Mammography and ultrasonography are the initial imaging modalities. Ultrasonography is often preferred in women younger than 30 years because increased breast tissue density in younger women limits the usefulness of mammography. Ultrasonography may also be a better choice for young women and pregnant patients in order to avoid radiation exposure. The main utility of ultrasonography is its ability to differentiate cystic from solid lesions. A cyst is likely to be benign if it has symmetric, round borders with no internal echoes. A solid lesion with uniform borders and uniformly sized internal echoes is consistent with a benign fibroadenoma. In this patient with relatively low-risk clinical symptoms, ultrasonography is preferred. The description of the mass (firm, nontender, mobile mass with well-defined margins and no lymphadenopathy) suggests a benign finding, such as a fibroadenoma or cyst. If the ultrasound shows a simple cyst, no further evaluation is necessary, unless the patient is symptomatic. If the ultrasound reveals a solid lesion, it must be evaluated completely with biopsy. An image-directed core-needle biopsy of a breast mass would be recommended if an ultrasound shows a solid-appearing, suspicious (Breast Imaging Reporting and Data System [BI-RADS] category 4) or highly suspicious (BI-RADS category 5) mass. This patient must undergo ultrasonography to determine the BI-RADS category of the mass before a decision on whether to perform a biopsy can be made. The dense breast tissue often found in young women limits the sensitivity, and hence the effectiveness, of mammography. Therefore, mammography is generally not needed for young women with a low-risk breast mass. For women aged 30 years or older with a palpable breast abnormality, both diagnostic mammography and ultrasonography would be recommended. Because this patient is younger than 30 years, only ultrasonography is warranted at this time.
A 25-year-old man is evaluated for the gradual onset of bilateral low back pain without radiation to the lower extremities daily, with increasing severity over the past year. The pain now awakens him during the night 2 to 3 times per week, with morning stiffness lasting more than an hour. He has improvement with exercise and no improvement at rest. He takes ibuprofen with some improvement. On physical examination, vital signs are normal. Limited lateral bending bilaterally and a reduction in forward flexion at the lumbar spine are noted. The remainder of the examination is normal. An anteroposterior plain radiograph of the pelvis and sacroiliac joints is unremarkable. Which of the following is the most appropriate diagnostic test to perform next? Bone scan CT of the lumbar spine MRI of the sacroiliac joints Radiography of the hip joints
The most appropriate diagnostic test to perform next is MRI of the sacroiliac joints. This patient has symptoms suggestive of back pain due to inflammation, including young age of onset, gradual onset, pain during the night, morning stiffness, improvement with motion, no history of trauma, and no improvement with rest. A single anteroposterior pelvis plain radiograph to view the sacroiliac joints is an appropriate first diagnostic step in this setting, which may reveal joint space widening (early) or narrowing (late), erosions, sclerosis, and ankylosis, and can establish the diagnosis of ankylosing spondylitis. However, plain radiographs may be normal early in the course of disease, as seen in this patient. MRI of the sacroiliac joints can then be utilized, which is more sensitive for detecting early spine and sacroiliac joint inflammation. The finding of bone marrow edema on STIR or T2-weighted images with fat suppression is not specific for ankylosing spondylitis but does suggest active inflammation, particularly if found in characteristic periarticular and subchondral locations. MRI can also identify soft-tissue evidence of inflammation in the entheses, bursae, and tendons, as well as subtle structural abnormalities. Bone scan is sensitive for the detection of osteoblast activity and is the study of choice for detecting bone metastases from various forms of cancer. However, bone scan is not used in the diagnosis of any form of inflammatory arthritis because it lacks specificity. CT of the lumbar spine can be useful in the diagnosis of ankylosing spondylitis when radiography of the sacroiliac joints is normal. However, it is not routinely used in the evaluation of patients for arthritis due to the high dose of radiation required. Hip pathology frequently results in anterior groin pain and would be unlikely to cause back pain. Therefore, radiography of the hip joints is not likely to aid in the diagnosis and is not indicated for this patient.
A 25-year-old woman is evaluated in the emergency department after a suicide attempt. History is significant for major depression. She takes no medication. On physical examination, temperature is normal, blood pressure is 142/92 mm Hg, pulse rate is 110/min, and respiration rate is 22/min. The patient is obtunded. The remainder of the examination is normal. Laboratory studies: Blood urea nitrogen 28 mg/dL (10 mmol/L) Creatinine 2.2 mg/dL (194.5 µmol/L) Electrolytes : Sodium 136 mEq/L (136 mmol/L) Potassium 4.0 mEq/L (4.0 mmol/L) Chloride 100 mEq/L (100 mmol/L) Bicarbonate 12 mEq/L (12 mmol/L) Ethanol Undetected Glucose 90 mg/dL (5.0 mmol/L) Osmolality 314 mOsm/kg H2O Arterial blood gases : pH 7.25 PCO2 28 mm Hg (3.7 kPa) Which of the following is the most appropriate management? Activated charcoal gastric decontamination Intravenous ethanol Intravenous hydration, fomepizole, and hemodialysis Intravenous sodium bicarbonate
The most appropriate management is intravenous hydration, fomepizole, and hemodialysis. This patient has typical findings of ethylene glycol toxicity, including central nervous system depression, an increased anion gap metabolic acidosis, and an increased osmolal gap. In patients with an increased anion gap acidosis, calculation of the serum osmolal gap is helpful in assessing the presence of unmeasured solutes, such as ingestion of certain toxins (for example, methanol or ethylene glycol). The serum osmolal gap is the difference between the measured and calculated serum osmolality. Serum osmolality can be calculated using the following formula: Serum Osmolality (mOsm/kg H2O) = (2 × Serum Sodium [mEq/L]) + Plasma Glucose (mg/dL)/18 + Blood Urea Nitrogen (mg/dL)/2.8 When the measured osmolality exceeds the calculated osmolality by >10 mOsm/kg H2O, the osmolal gap is considered elevated. This patient has an osmolal gap of 27 mOsm/kg H2O. Finally, this patient has kidney failure likely resulting from deposition of calcium oxalate crystals in the renal tubules. Because laboratory confirmation of ethylene glycol intoxication may take days, empiric therapy with fomepizole and aggressive fluid resuscitation with crystalloids (250-500 mL/h intravenous initially) should be instituted in all cases to increase kidney clearance of the toxin and to limit deposition of oxalate in the renal cortex. Hemodialysis to clear the alcohol and toxic metabolites should be instituted in the context of any organ-specific toxicity (central nervous system depression, acute kidney injury, systemic collapse), severe acidemia, or very large ingestions. Activated charcoal gastric decontamination and nasogastric lavage have no role in toxic alcohol poisoning; both ethylene glycol and methanol are typically absorbed too rapidly for either of these modalities to be useful. Intravenous ethanol was traditionally used as a competitive inhibitor of alcohol dehydrogenase. However, fomepizole has been found to be superior to alcohol with few side effects. Therefore, fomepizole is the preferred agent, and there is no additional benefit to coadministration of the two. Adjunct therapy with intravenous sodium bicarbonate therapy in the context of pH <7.30 may reduce penetration of toxic metabolites of ethylene glycol (glycolate, glyoxylate, and oxalate) and methanol (formate) into the tissues and enhance renal excretion of glycolate and formate. However, the initiation of dialysis in this patient is more efficacious.
An 18-year-old man is evaluated in the ICU for oliguric acute kidney injury. Eighteen hours ago he underwent hepatectomy for a giant fibrolamellar hepatic carcinoma. During the procedure he developed coagulopathy and hepatic bleeding and required resuscitation with eight units of packed red blood cells, multiple units of fresh frozen plasma, and several liters of crystalloid fluids. He is receiving cefepime, gentamicin, propofol, and fentanyl. Urine output has decreased to 10 mL/h since ICU admission 14 hours ago. On physical examination, the patient is mechanically ventilated. Blood pressure is 120/70 mm Hg, pulse rate is 115/min, and respiration rate is 12/min. Breath sounds are decreased bilaterally. The abdomen is distended and tense with intact midline incision and wall edema. The remainder of the examination is noncontributory. Laboratory studies: Hemoglobin 10 g/dL (100 g/L) Creatine kinase 1250 U/L Creatinine 1.7 mg/dL (150.3 µmol/L); on admission: 0.9 mg/dL (79.6 µmol/L) Potassium 5.2 mEq/L (5.2 mmol/L) Urine sodium <20 mEq/L (20 mmol/L) Urinalysis Specific gravity 1.030; pH 5.5; 4+ blood; trace protein; too numerous to count erythrocytes; few hyaline casts Kidney ultrasound reveals normal-sized kidneys and no hydronephrosis; a large volume of ascites is noted. Which of the following is the most appropriate diagnostic test to perform next? Fractional excretion of sodium Intra-abdominal pressure measurement Urine myoglobin levels Urine stain for eosinophils
The most appropriate diagnostic test to perform next is intra-abdominal pressure (IAP) measurement. This patient's findings are consistent with abdominal compartment syndrome, which occurs in the setting of abdominal surgery, large volume fluid resuscitation, and multiple transfusions. It can manifest as a distended abdomen, ascites, and sodium-avid acute kidney injury (AKI). The increased IAP causes direct compression of renal parenchyma and vasculature, resulting in oliguria and decreased glomerular filtration rate. The diagnosis of abdominal compartment syndrome is made by an IAP measurement >20 mm Hg and new organ dysfunction. Indirect measurement of IAP can be with intragastric, intracolonic, intravesical (bladder), or inferior vena cava catheters. Management includes supportive therapy, abdominal compartment decompression, and correction of positive fluid balance. The increased specific gravity, low urine sodium, and presence of hyaline casts are consistent with a prerenal AKI. The fractional excretion of sodium would not provide any additional information to aid in the diagnosis. Myoglobin is a heme pigment-containing protein that can cause AKI. The urine is reddish brown, pigmented casts are present, and the urine dipstick is positive for blood in the absence of erythrocytes. Abdominal compartment syndrome is a much more likely cause of this patient's AKI, and urine myoglobin levels do not need to be measured. The patient is on a cephalosporin, which can cause acute interstitial nephritis (AIN). AIN is characterized by hematuria, pyuria, and/or leukocyte casts. However, the timing is too soon for AIN (unless the patient had been previously exposed), and the urine findings do not support it. Moreover, the urine eosinophil stain is neither sensitive nor specific for the diagnosis of AIN and does not need to be performed.
An 18-year-old woman is brought to the emergency department by friends. She is confused and febrile. Her friends state that she took 3,4-methylenedioxymethamphetamine (ecstasy) at a party and was previously well. There is no other medical history. On physical examination, the patient is confused and oriented to her name only. Temperature is 38.9 °C (102.0 °F), blood pressure is 148/94 mm Hg, pulse rate is 108/min, respiration rate is 20/min, and oxygen saturation is 96% breathing 2 L/min oxygen by nasal cannula. The remainder of the examination is unremarkable. Laboratory studies: Blood urea nitrogen 11 mg/dL (3.9 mmol/L) Creatinine 0.8 mg/dL (70.7 µmol/L) Electrolytes : Sodium 118 mEq/L (118 mmol/L) Potassium 3.5 mEq/L (3.5 mmol/L) Chloride 88 mEq/L (88 mmol/L) Bicarbonate 21 mEq/L (21 mmol/L) Glucose 88 mg/dL (4.9 mmol/L) Urine osmolality 405 mOsm/kg H2O Which of the following is the most appropriate initial treatment? 0.9% sodium chloride, 100 mL/h 100-mL bolus of 3% saline Fluid restriction Oral urea Tolvaptan
The most appropriate initial treatment is a 100-mL bolus of 3% saline. The hyponatremia in this young woman who presents confused and febrile is most likely due to ingestion of 3,4-methylenedioxymethamphetamine (ecstasy). Ecstasy is associated with hyponatremia both because it stimulates the release of antidiuretic hormone and because users often drink large quantities of water. When treating hyponatremia, the rate of correction of serum sodium concentration must be carefully considered to avoid the osmotic demyelination syndrome. Brain cells adapt to chronic hyponatremia by reducing intracellular concentration of organic osmolytes, such as myoinositol, to cope with hypotonicity. Acute hyponatremia is associated with an increase in brain water and cerebral edema and should be treated rapidly. Because the brain has not adapted to the hypotonic environment by the release of organic osmolytes, the risk of rapid correction and development of osmotic demyelination is absent. Treatment is with a bolus of 3% saline, and a 100-mL bolus should raise the serum sodium level by 2 to 3 mEq/L (2-3 mmol/L). If symptoms persist, this can be repeated one to two times. In patients with neurologic symptoms, fluid restriction by itself is not an appropriate treatment. The immediate goal is to reduce brain swelling rapidly by the acutely raising the serum sodium level. Even in cases of chronic hyponatremia, the serum sodium can be increased acutely by 2 to 3 mEq/L (2-3 mmol/L) as long as the total change in the serum sodium is <10 mEq/L (10 mmol/L) in a 24-hour period. Except in cases of hypovolemic hyponatremia, the use of 0.9% sodium chloride is not recommended. In patients who are not volume depleted and have syndrome of inappropriate antidiuretic hormone secretion with a fixed urine osmolality, the infused saline can be excreted in the urine in a smaller volume, and thus the serum sodium can actually fall. Both oral urea and tolvaptan are appropriate treatments for chronic hyponatremia, but they do not raise the serum sodium rapidly enough to reverse neurologic abnormalities and are therefore inappropriate for treatment of acute hyponatremia in this symptomatic patient.
A 75-year-old woman is hospitalized for a 3-week history of progressive exertional dyspnea, increasing peripheral edema, and mental status changes. For the past 4 nights, she has been sleeping in a recliner instead of her bed. She reports no chest pain. She has a 6-year history of ischemic cardiomyopathy, for which she takes low-dose aspirin, furosemide, carvedilol, lisinopril, digoxin, spironolactone, and as-needed metolazone. On physical examination, the patient is afebrile, blood pressure is 84/52 mm Hg, pulse rate is 118/min, and respiration rate is 28/min. Oxygen saturation is 95% breathing ambient air. She is confused. Jugular venous distention is present. Cardiac examination reveals an S3. There is ascites on abdominal examination. The extremities are cool, and there is lower extremity edema to the knees. Laboratory studies: Alanine aminotransferase 172 U/L Aspartate aminotransferase 163 U/L Creatinine 2.9 mg/dL (256.4 µmol/L) (baseline, 1.2 mg/dL [106.1 µmol/L]) Potassium 4.7 mEq/L (4.7 mmol/L) Sodium 132 mEq/L (132 mmol/L) (baseline, 140 mEq/L [140 mmol/L]) Digoxin 0.3 ng/mL (0.38 nmol/L) (normal range, 0.5-2.0 ng/mL [0.64-2.56 nmol/L]) An electrocardiogram shows no acute changes. An echocardiogram shows a left ventricular ejection fraction of 20%. Which of the following is the most appropriate initial treatment? Increase carvedilol Increase digoxin Increase lisinopril Start dobutamine
The most appropriate initial treatment of this patient with cardiogenic shock is to start dobutamine. Cardiogenic shock is defined by persistent symptomatic hypotension and end-organ dysfunction. Patients present with acute kidney failure, evidence of liver dysfunction with elevated aminotransferase levels, poor peripheral perfusion with cool extremities, and impaired mental status. Cardiogenic shock secondary to progressive heart failure is generally treated with an inotropic agent, such as dobutamine or milrinone. Patients with peripheral vasoconstriction (increased systemic vascular resistance) often benefit from the addition of a pure vasodilator, such as sodium nitroprusside. In this case, dobutamine would be a better option than milrinone, even in the setting of relative tachycardia, because milrinone is excreted through the kidneys and could build to a toxic level given her current kidney function. β-Blockers such as carvedilol should be continued during a heart failure hospitalization whenever possible. In a patient with volume overload without signs of low cardiac output, β-blockers can often be continued at the maintenance dose while the patient is undergoing diuresis. However, in a patient with low cardiac output such as this one, β-blockers should be discontinued, owing to their negative inotropic effects, even in the setting of tachycardia. Digoxin is a weak inotrope that reduces hospitalizations in patients with heart failure. Although this patient's digoxin level is low, the dose should not be increased in the setting of acute kidney failure. Additionally, increasing digoxin would not be expected to adequately improve cardiac output in this patient. Although afterload reduction is reasonable to decrease myocardial oxygen demand and improve forward flow in patients with acute decompensation, a short-acting intravenous agent, such as nitroprusside or nitroglycerin, would be administered instead of increasing the dosage of an oral ACE inhibitor, such as lisinopril. Read Related TextNext Question
An 81-year-old woman was admitted to the ICU 8 days ago for multisystem organ failure associated with a severe episode of multilobar pneumonia. She has required mechanical ventilation since admission. Efforts to wean the patient from mechanical ventilation have not succeeded, and the patient remains somnolent and unresponsive to verbal stimuli. Medical history is significant for dementia, diabetes mellitus, COPD, chronic kidney disease, and heart failure. The care team concludes and shares with the patient's family that she will not have a meaningful recovery; however, the patient's children request continued ICU-level care. The patient does not have an advance directive, and her wishes are unknown. After a family meeting with the care team to discuss the patient's prognosis, the children continue to request all treatment. Which of the following is the most appropriate management? Consult with the hospital ethics committee Discontinue ICU care in 48 hours if there is no improvement Transfer the patient to another institution Continue current level of care
The most appropriate management is consultation with the hospital ethics committee. A recent policy statement from the Society of Critical Care Medicine recommends that appropriate treatment goals of ICU care include treatment that provides a reasonable expectation of survival outside of the acute care setting with sufficient cognitive ability to perceive benefits of treatment, or palliative care through the dying process in the ICU. Because conflicts between the desire to provide benefit to the patient and the desire to minimize the burden of treatment can be very difficult, one of the most important skills of the physician is the ability to communicate and negotiate a reasonable treatment plan with the patient's family. If these situations become intractable, many organizations recommend initiating a process to resolve the disagreement, including notifying surrogates of the process, seeking a second medical opinion, obtaining review by an interdisciplinary ethics committee, offering the surrogate the opportunity to seek care at another institution, and implementing the decision of the resolution process. This patient's family is requesting treatment that the care team does not think will achieve reasonable goals, and an ethics consultation may lead to conflict resolution. In some situations, the physician and the patient's family may mutually establish a time frame in which care will be withdrawn if there is no improvement; however, these decisions should not be made unilaterally by the care team. A physician should not provide treatment that conflicts with professional obligations and will not meet the goals of care. However, often by communicating his or her concerns, a physician is able to help a family understand the burden of continued, ineffective treatment. If resolution is not possible, family members may seek transfer to another institution; however, the physician is not obliged to initiate such arrangements.
A 26-year-old woman is evaluated for a preconception assessment. She has Marfan syndrome. She reports no cardiovascular symptoms. Her only medication is long-acting metoprolol. On physical examination, blood pressure is 110/60 mm Hg, and pulse rate is 60/min and regular. The patient has facial and skeletal features of Marfan syndrome. The estimated central venous pressure is normal. Cardiac examination reveals a normal apical impulse, normal S1 and S2, and a grade 2/6 late-peaking systolic murmur and midsystolic click over the apex. The lungs are clear to auscultation. No edema is noted. Transthoracic echocardiogram demonstrates a dilated proximal ascending aorta with a dimension of 4.6 cm. No aortic regurgitation is appreciated. Bileaflet mitral valve prolapse is noted with mild mitral regurgitation. The left ventricular size and function are normal. There is no recent echocardiogram for comparison. Which of the following is the most appropriate management? Advise against pregnancy Proceed with mitral valve intervention Repeat echocardiography in 12 months Switch metoprolol to losartan
The most appropriate management is to advise against pregnancy in this patient with Marfan syndrome and a dilated aorta. All women with Marfan syndrome have an increased risk for pregnancy-related aortic dissection and rupture. In women with Marfan syndrome and an ascending aortic diameter of 4.5 cm or greater, aortic repair surgery is recommended before pregnancy to reduce this risk. Some women with Marfan syndrome and an aortic diameter less than 4.5 cm, including those with rapid dilatation of the ascending aorta or a family history of aortic dissection, are at high risk for dissection during pregnancy and are counseled to have surgery before pregnancy. Generally, pregnancy is considered safe if the aortic diameter is smaller than 4.0 cm. This patient is asymptomatic with normal left ventricular size and function and mild mitral regurgitation; therefore, she does not meet the criteria for mitral valve intervention. Although the mitral valve may warrant intervention at some point, this patient's dilated aorta is the major determinant of pregnancy risk. Patients with Marfan syndrome who are considering pregnancy should have an aortic dimension smaller than 4.0 to 4.5 cm, with demonstrated stability for 6 months or longer. This patient does not meet these criteria, and repeat imaging demonstrating aortic stability will not provide reassurance about future pregnancy. Owing to the size of the aorta, this patient should be advised to avoid pregnancy and consider elective surgical repair. If she elects not to proceed with pregnancy or surgical repair at this time, repeat aortic root imaging should be performed in 6 months to confirm stability of the aortic dimensions. Angiotensin receptor blockers, such as losartan, are considered second-line agents to β-blocker therapy for the control of hypertension and slowing aortic root expansion in patients with Marfan syndrome. Furthermore, ACE inhibitors and angiotensin receptor blockers are contraindicated during pregnancy because of fetal toxicity. Fetal exposure to these agents during the first trimester can cause central nervous system and cardiovascular malformations, and exposure during the second trimester can cause developmental malformations of the kidneys and genitourinary system. Read Related TextNext Question
A 27-year-old woman was diagnosed with a 2-cm left cortisol-producing adrenal adenoma. She is admitted to the hospital for an adrenalectomy. Medical history is otherwise unremarkable, and she takes no medications. On physical examination, blood pressure is 152/88 mm Hg; the remainder of the vital signs is normal. Centripetal obesity, facial plethora, fat deposition in the supraclavicular areas, and wide violaceous striae are present. Which of the following is the most appropriate management following adrenalectomy? Epinephrine Fludrocortisone Hydrocortisone Phenoxybenzamine
The most appropriate management of this patient following adrenalectomy for treatment of Cushing syndrome is hydrocortisone therapy. Patients who have undergone adrenalectomy for Cushing syndrome are at risk of secondary adrenal insufficiency due to hypercortisolism-induced suppression of the hypothalamic (corticotropin-releasing hormone [CRH]) and pituitary (adrenocorticotropic [ACTH] hormone) axis. Subsequent recovery of the hypothalamic-pituitary axis following removal of the source of excess cortisol secretion can take time, and hence endogenous cortisol production is impaired. In addition, cortisol-producing cells from the contralateral adrenal gland may have undergone atrophy due to lack of ACTH stimulation. Patients often require higher than physiologic doses of glucocorticoid therapy to prevent "glucocorticoid withdrawal syndrome," an ill-defined complex of symptoms of fatigue, loss of appetite, nausea, and/or myalgia. There is no agreed upon glucocorticoid regimen following adrenalectomy and treatment of Cushing syndrome. The process of tapering and eventual discontinuation of glucocorticoid therapy is guided by patient symptoms and can be a lengthy process, taking up to 1 year or longer for the remaining adrenal gland to produce adequate cortisol. ACTH-dependent causes of Cushing syndrome also lead to suppression of endogenous CRH and ACTH production, and hence patients with these disorders also require glucocorticoid therapy following successful treatment of Cushing syndrome. Epinephrine is also not under ACTH control, and therefore epinephrine replacement is not required after adrenalectomy. In addition, epinephrine replacement is not required following bilateral adrenalectomy, as lack of epinephrine is not known to be associated with clinical disease. Likewise, fludrocortisone therapy is not required following adrenalectomy as mineralocorticoid secretion is not under ACTH control. Therefore, aldosterone secretion from the contralateral adrenal gland is not impacted by Cushing syndrome. Fludrocortisone therapy is required following bilateral adrenalectomy. Phenoxybenzamine is used as a preoperative α-receptor blockade for 10 to 14 days before surgery for pheochromocytomas and paragangliomas to prevent hypertensive crises during surgery and is not indicated following adrenalectomy for Cushing syndrome.
A 74-year-old woman is seen in follow-up for osteoporosis diagnosed 5 years ago. She has been taking alendronate (70 mg weekly) without adverse effect for 5 years. Medical history is otherwise unremarkable. Alendronate is her only medication. On physical examination, vital signs are normal. She has no kyphosis or height loss, and the remainder of her examination is also normal. Her recent bone mineral density by dual-energy x-ray absorptiometry (DEXA) scan showed a lumbar spine T-score of -2.2 and femoral neck T-score of -2.4. Which of the following is the most appropriate management? C-terminal peptide of type 1 collagen (CTx) measurement Continue alendronate for 5 additional years Decrease alendronate dose Discontinue alendronate
The most appropriate management of this patient is to discontinue alendronate. The Fracture Intervention Trial Long-term Extension (FLEX) trial showed that continuing alendronate treatment for 10 years compared with stopping after 5 years resulted in a small decrease in the incidence of clinical vertebral fractures but not nonvertebral fractures. Subject characteristics most predictive of incident fracture after alendronate discontinuation were age older than 76 years, current femur neck T-score below -2.5, and prior osteoporotic fracture. Importantly, women with femur neck T-score below -3.5 or who fractured during the initial 5 years of alendronate therapy were not included in the FLEX trial. The authors concluded that patients at high risk of fracture may benefit from continuing alendronate therapy for up to 10 years. There is inconsistency among expert groups regarding the need to monitor bone mineral density during osteoporosis therapy and at 5 years to inform decision making about discontinuation of therapy. Contrary to other groups, the American College of Physicians recommends against monitoring because data from several studies showed that women treated with antiresorptive treatment benefit from reduced fractures even if BMD did not increase. The antiresorptive effect of alendronate can be assessed by bone turnover markers including serum C-terminal peptide of type 1 collagen (CTx). However, neither CTx levels on alendronate nor change with discontinuation predict which patients will fracture if alendronate is discontinued. Furthermore, serum CTx levels in patients taking alendronate vary widely and cannot be reliably interpreted without pretreatment values. The FLEX trial included postmenopausal women who had taken 5 years of alendronate 5 mg daily (equivalent to 35 mg weekly) or 10 mg daily (equivalent to 70 mg weekly). There was no difference in outcomes based on alendronate dose. Therefore, the management decision to be made after 5 years of alendronate therapy is to continue or discontinue rather than to modify the dose. Read Related TextNext Question
A 67-year-old man is seen for an increase in serum creatinine level and an abnormal urinalysis found during the evaluation of monoclonal gammopathy of undetermined significance. His evaluation revealed an M-protein spike of 1.5 g/dL, <10% clonal plasma cells on bone marrow biopsy, and no evidence of anemia, hypercalcemia, or lytic bone lesions on skeletal survey. Immunofixation revealed IgG as the monoclonal type. He has no constitutional symptoms, no other medical problems, and takes no medications. On physical examination, vital signs are normal. Trace lower extremity edema is noted. The remainder of the examination is unremarkable. Laboratory studies: Albumin 3.6 g/dL (36 g/L) Creatinine 1.6 mg/dL (141.4 µmol/L) Urinalysis pH 5.5; 2+ blood; 3+ protein; 5-8 erythrocytes/hpf Urine albumin-creatinine ratio 400 mg/g Which of the following is the most appropriate next diagnostic test? ANCA testing β2-Microglobulin levels Kidney biopsy Serum free light chains
The most appropriate next diagnostic test is a kidney biopsy. This patient appears to have monoclonal gammopathy of renal significance (MGRS). This is a recently defined set of kidney disorders found in patients who would otherwise meet the criteria for monoclonal gammopathy of undetermined significance but have an abnormal urinalysis and kidney insufficiency. This patient who otherwise meets the diagnostic criteria for monoclonal gammopathy of undetermined significance has an active urine sediment and an increase in the serum creatinine level and thus has underlying kidney disease. MGRS is an increasingly recognized disorder, and a kidney biopsy is necessary to make the diagnosis by demonstrating the presence of monoclonal immunoglobulin deposition in the kidney. MGRS can affect the kidney in various ways, including amyloidosis, proliferative glomerulonephritis, immunoglobulin deposition disease, C3 glomerulopathy, and proximal tubulopathy. Kidney manifestations of MGRS are usually caused by deposition of monoclonal light chains. Patients can present with both nephrotic and subnephrotic proteinuria, hematuria, and elevated serum creatinine. Despite not meeting the definition of multiple myeloma, MGRS increases morbidity and, in most cases, should be treated with therapy designed to eliminate or suppress the immunoglobulin clone. Pauci-immune glomerulonephritis is caused by microscopic vessel vasculitis affecting the kidney, resulting in necrotizing lesions in the glomeruli with few or no immune deposits. The renal lesion may occur with or without systemic vasculitis and is the most common cause of rapidly progressive glomerulonephritis. Most patients have circulating ANCA directed against neutrophils. Other than proteinuria, this patient has a bland urinalysis that is not consistent with a pauci-immune glomerulonephritis, and testing for ANCA in lieu of a kidney biopsy will not establish a diagnosis. β2-Microglobulin levels can be helpful as prognostic indicators in myeloma but cannot be used to make a diagnosis of a plasma cell disorder. Serum free light chains can be used to diagnose myeloma and predict the risk of progression in patients with monoclonal gammopathy of undetermined significance; however, a kidney biopsy, not serum free light chain determination, will confirm the cause of this patient's kidney disease.
26F diagnosed with WPW; next step? Which of the following is the most appropriate next step in management? Atenolol Electrophysiology study Flecainide Verapamil
The most appropriate next step in management is an electrophysiology study. This young woman with a history of palpitations and paroxysms of dyspnea and lightheadedness likely has atrioventricular reciprocating tachycardia associated with an accessory pathway between the atria and ventricles. Accessory pathway conduction is often observed as preexcitation on electrocardiogram (ECG). Because of early ventricular activation over the accessory pathway, the PR interval is shortened, and the initial part of the QRS complex is slurred (delta wave) because of ventricular depolarization adjacent to the pathway. Wolff-Parkinson-White syndrome is characterized by preexcitation on ECG accompanied by symptoms consistent with tachycardia. Symptoms concerning for arrhythmia in a patient with Wolff-Parkinson-White syndrome should prompt referral to a cardiologist or an electrophysiologist. In these patients, it is also important to obtain an echocardiogram to exclude structural heart disease associated with accessory pathways, including Ebstein anomaly. In this case, the patient's symptomatic episodes could be caused by supraventricular tachycardia (orthodromic or antidromic reciprocating tachycardia) or preexcited atrial fibrillation. A diagnostic electrophysiology study is indicated to uncover the cause of her palpitations and to determine her risk for sudden cardiac death. The electrophysiology procedure also affords the opportunity to ablate the accessory pathway and potentially cure the arrhythmia. The cure rate of catheter ablation is as high as 95% to 99% in patients with an uncomplicated accessory pathway. Atenolol and verapamil are atrioventricular nodal blockers and may be unsafe if the patient has anterograde conduction down the accessory pathway during atrial fibrillation. These drugs promote rapid 1:1 conduction from the atria to the ventricles over the accessory pathway during atrial fibrillation, which can lead to very rapid ventricular rates and ventricular fibrillation. Flecainide is an antiarrhythmic drug that is frequently used to treat paroxysmal atrial fibrillation and other arrhythmias. Flecainide is not indicated in this patient because the type and mechanism of the arrhythmia are unknown. This patient requires risk stratification, which can be accomplished with an electrophysiology study. Furthermore, the age of the patient should be considered before antiarrhythmic therapy is initiated. Catheter ablation is preferred in young persons to avoid lifelong use of potentially toxic medications. This young woman may also wish to avoid these types of medications if she will be considering pregnancy in the future.
A 58-year-old woman is evaluated during a follow-up visit 6 months after thyroidectomy for differentiated papillary thyroid cancer. She developed symptomatic hypocalcemia following surgery but is currently asymptomatic. Medications are calcium citrate, calcitriol, and levothyroxine. Vital signs and physical examination are normal. Laboratory studies: Calcium 9.5 mg/dL (2.4 mmol/L) Creatinine 1.0 mg/dL (88.4 µmol/L) Phosphorus 4.5 mg/dL (1.5 mmol/L) Magnesium 2.3 mg/dL (0.95 mmol/L) Parathyroid hormone 10 pg/mL (10 ng/L) Which additional measurement is appropriate now? Bone mineral density 1,25-Dihydroxyvitamin D 24-Hour urine calcium 25-Hydroxyvitamin D Ionized calcium
The most appropriate test to perform now is measurement of 24-hour urine calcium. In chronic hypoparathyroidism, goals of therapy are to eliminate symptoms while avoiding complications of therapy; monitoring of urine calcium excretion is mandatory because hypercalciuria often limits therapy. Without parathyroid hormone (PTH), urinary calcium excretion is higher than normal for any given serum calcium level. Complications of prolonged hypercalciuria include nephrolithiasis and impaired glomerular filtration rate. Serum calcium, magnesium, creatinine, and urine calcium levels should be assessed on a regular basis. The goal calcium levels should be low-normal without hypercalciuria. The magnesium level should ideally be greater than 2 mg/dL (0.83 mmol/L), and creatinine levels should remain in the normal range. If the urine calcium level is greater than 300 mg/24 h (hypercalciuria), calcium and/or vitamin D replacement needs to be decreased. Calcium is usually decreased first if the 25-hydroxyvitamin D level is within the normal sufficiency range (≥30 ng/mL [75 nmol/L]). Thiazide diuretics reduce urine calcium excretion and thus may permit sufficient calcium and vitamin D therapy to achieve goal calcium levels. Hypoparathyroidism slows bone metabolism and is relatively protective against the development of postmenopausal osteoporosis. Further, osteoporosis medications would rarely be indicated in these patients. Therefore, bone mineral density testing is not indicated. The dose of calcitriol administered is titrated by the serum calcium and/or phosphorus level rather than the serum 1,25-dihydroxyvitamin D level. If serum PTH is not detectable 6 months after the onset of surgical hypoparathyroidism, it can be considered chronic. Without PTH, vitamin D activation, specifically the conversion of 25-hydroxyvitamin D to 1,25-dihydroxyvitamin D (calcitriol) in the kidney, is severely impaired. Although vitamin D nutritional status as assessed by 25-hydroxyvitamin D may be relevant for other health conditions, its measurement in not useful in the setting of chronic hypoparathyroidism unless residual PTH production remains. Ionized calcium, rather than total calcium, is the relevant calcium fraction in the blood. However, in an otherwise healthy person in whom serum albumin concentrations are presumed to be normal, measurement of ionized calcium does not contribute to the management of hypoparathyroidism. Read Related TextNext Question
A 59-year-old woman is evaluated for fatigue and weight gain over the past 2 months. Her medical history is significant for a pituitary tumor, treated with surgery followed by radiation therapy, at age 54. She has recently self-initiated calcium and vitamin D and a multivitamin. Her only other medication is levothyroxine. On physical examination, vital signs are normal. BMI is 31.The remainder of the physical examination is normal. Which of the following is the most appropriate next step in management? Increase the levothyroxine dose Measure free thyroxine (T4) level Measure thyroid-stimulating hormone level MRI of the brain
The most appropriate next step in management is measurement of the free thyroxine (T4) level. Thyroid-stimulating hormone (TSH) deficiency leads to secondary or central hypothyroidism. Secondary hypothyroidism symptoms are clinically identical to primary hypothyroidism symptoms. Secondary hypothyroidism is diagnosed by demonstrating a simultaneously inappropriately normal or low TSH and low T4 (free or total). Patients are treated with levothyroxine replacement in the same manner as primary hypothyroidism; however, the serum TSH cannot be used to monitor and assess for adequacy of thyroid hormone replacement dosing. Instead, the levothyroxine dose is adjusted based on free T4 levels with the goal of obtaining a value within the normal reference range. The patient's recent symptoms of fatigue and weight gain may be due to hypothyroidism resulting from impaired absorption of levothyroxine after recently starting a calcium supplement. A low T4 level will confirm the diagnosis. In general the therapeutic goal is to keep the free T4 in the upper half of the normal range. To improve gastrointestinal absorption, levothyroxine should be taken on an empty stomach, 1 hour before or 3 hours after ingestion of food. Medications that would interfere with absorption, such as calcium- or iron-containing supplements should be separated by 4 hours. Increasing the levothyroxine would not be appropriate without first documenting the presence of hypothyroidism by measuring the T4 level. In addition, the preferred initial therapy may simply be to discontinue the calcium, if not warranted, or to separate ingestion of calcium and levothyroxine by at least 2 to 3 hours. The patient has no symptoms suggestive of a pituitary mass effect or other indication to recommend pituitary imaging. Read Related TextNext Question
A 65-year-old woman is evaluated during a routine examination. She was diagnosed with a cardiac murmur in early adulthood. She is active, healthy, and without symptoms. She takes no medications. On physical examination, vital signs are normal. A grade 3/6 holosystolic murmur preceded by multiple clicks is present at the apex. Physical findings are otherwise unremarkable. An echocardiogram demonstrates a left ventricular ejection fraction of 50%. The left ventricle is moderately dilated with an end-systolic dimension of 42 mm. Myxomatous degeneration of the mitral valve is present with severe regurgitation due to posterior leaflet prolapse. Which of the following is the most appropriate next step in management? Serial clinical and echocardiographic evaluations Surgical mitral valve repair Surgical mitral valve replacement Transcatheter mitral valve repair
The most appropriate next step in management is surgical mitral valve repair. Myxomatous degeneration of the mitral valve is common, affecting 1% to 2% of the general population. In 10% of patients, the valvular lesion can progress, become life threatening, and require surgery. The only definitive therapy for severe mitral regurgitation is mitral valve surgery. Options are mitral valve repair, mitral valve replacement with preservation of part or all of the mitral apparatus, and mitral valve replacement with removal of the mitral apparatus. Mitral valve repair is generally preferred to valve replacement because it is associated with improved survival in retrospective studies. Mitral valve repair is strongly recommended for chronic severe primary mitral regurgitation in (1) symptomatic patients with left ventricular ejection fraction greater than 30%, (2) asymptomatic patients with left ventricular dysfunction (left ventricular ejection fraction of 30%-60% and/or left ventricular end-systolic diameter ≥40 mm), and (3) patients undergoing another cardiac surgical procedure. Additionally, mitral valve repair is reasonable in asymptomatic patients with chronic severe primary mitral regurgitation who have new-onset atrial fibrillation or pulmonary hypertension (pulmonary artery systolic pressure >50 mm Hg). Notably, a left ventricular ejection fraction of 60% or less is used in defining left ventricular systolic dysfunction in mitral regurgitation because ventricular emptying into the left atrium contributes to the relatively lower afterload conditions and higher ejection fraction despite impaired left ventricular performance. Serial evaluations every 6 to 12 months are recommended for patients with severe mitral regurgitation who do not have indications for surgery. This patient meets the criteria for surgery; therefore, serial evaluations would not be appropriate at this time. Many patients who could benefit from mitral valve repair are denied surgery because of high surgical risk, advanced age, or comorbid conditions. A catheter-based device can improve mitral valve function by delivering a clip percutaneously to approximate the valve leaflet edges and improve leaflet coaptation at the origin of the mitral regurgitation jet. The device is approved for patients with significant symptomatic degenerative mitral regurgitation for whom mitral valve surgery poses a prohibitive risk. This patient is healthy and does not have comorbid conditions that would significantly increase surgical risk; therefore, transcatheter repair is not indicated.
A 32-year-old woman is evaluated during a domestic airline flight for an episode of weakness and lightheadedness. She is pregnant at 35 weeks' gestation. She has had several contractions since take-off but without regularity. She reports no abdominal pain. She has no medical problems, and her only medication is a prenatal vitamin. On physical examination, the patient appears weak. Temperature is normal, blood pressure is 105/60 mm Hg, pulse rate is 99/min, and respiration rate is 14/min. Her skin is clammy. Cardiovascular examination is unremarkable. Lungs are clear to auscultation. On abdominal examination, she has a gravid uterus. Oxygen, 2 L/min by nasal cannula, is started. An intravenous line is placed, and fluids are initiated. Which of the following is the most appropriate next step in management? Ask the pilot to descend to a lower altitude Connect with the ground-based physician Recommend flight diversion No further management
The most appropriate next step in management is to connect with the ground-based physician. In-flight medical emergencies are relatively common during air travel, occurring in an estimated 1 of 600 flights. Airlines based in the United States are mandated by the Federal Aviation Administration to carry at least one automated external defibrillator; supplemental oxygen; and a medical kit that contains a stethoscope, sphygmomanometer, gloves, airway supplies, intravenous access supplies, and some basic medications. In the case of an in-flight emergency, the physician's role generally involves assessing the patient, establishing a diagnosis when possible, administering basic medical treatments, providing reassurance as appropriate, and recommending flight diversion if necessary. Physicians should practice within their scope of training, be mindful of patient privacy, and document the patient encounter. Although not a Federal Aviation Administration requirement, most airlines have contracts with 24-hour call centers with a ground-based physician to aid in the event of an in-flight emergency. Often, ground-based physicians trained in emergency or aerospace medicine can assist the on-board physician remotely and help direct care, which can be particularly helpful when the medical problem is outside the scope of the physician's practice. The principles of hypobaric hypoxia apply to commercial airplanes, in which cabins are pressurized to the equivalent of 1500 to 2500 meters (approximately 5000 to 8200 feet) in altitude, resulting in an inspired oxygen tension between 110 and 120 mm Hg (about 70% of the levels encountered at sea level). Although this correlates with an arterial PO2 of approximately 60 mm Hg (8.0 kPa) in healthy individuals, those with underlying pulmonary disease are at risk for significant hypoxemia during a flight. This patient will have no difficulty maintaining her oxyhemoglobin saturation above 90%, and asking the pilot to descend to a lower altitude will serve no useful purpose. A better strategy is to contact the ground-based physician. Although this patient is dizzy and weak, her clinical status and vital signs appear stable. She needs further medical evaluation and management; however, flight diversion is probably not indicated at this time. Furthermore, the ground-based medical team can also help determine whether flight diversion is needed. Read Related TextNext Question
A 42-year-old man is evaluated during a follow-up visit for kidney stones. He had his first stone 4 years ago. Despite increasing his water intake, he has had two additional episodes. Stone analysis has revealed only calcium oxalate. He is in otherwise good health. He has no history of urinary tract infections. There is no family history of kidney disease, hyperparathyroidism, or nephrolithiasis. The physical examination and vital signs are unremarkable. The patient weighs 80 kg (176 lb). Laboratory studies: Calcium 9.6 mg/dL (2.4 mmol/L) Creatinine 0.9 mg/dL (79.6 µmol/L) Electrolytes : Sodium 138 mEq/L (138 mmol/L) Potassium 4.1 mEq/L (4.1 mmol/L) Chloride 105 mEq/L (105 mmol/L) Bicarbonate 25 mEq/L (25 mmol/L) Urinalysis Specific gravity 1.008; pH 5.5; no blood, protein, leukocyte esterase, or nitrites 24-Hour Urine Studies: Volume 2945 mL pH 5.2 Calcium 320 mg/24 h (normal range, <320 mg/24 h) Citrate 790 mg/24 h (normal range, 300-1100 mg/24 h) Oxalate 32 mg/24 h (normal range, <40 mg/24 h) Sodium 140 mEq/24 h (normal range, 40-220 mEq/24 h) Uric acid 640 mg/24 h (normal range, <800 mg/24 h) Noncontrast helical CT scan shows a 4-mm stone in the lower pole of the left kidney and a 3-mm stone in the mid pole of the right kidney. Which of the following is the most appropriate next step to decrease this patient's stone recurrence? Add allopurinol Add hydrochlorothiazide Add potassium citrate Increase urine volume Recommend a low calcium diet
The most appropriate next step to decrease this patient's stone recurrence is to add a thiazide diuretic such as hydrochlorothiazide. Hypercalciuria is the most common metabolic risk factor for calcium oxalate stones. In patients with hypercalcemia, increased filtered calcium results in hypercalciuria. However, hypercalciuria is often idiopathic and commonly familial, occurring without associated hypercalcemia. Hypercalciuria due to hypercalcemia is treated by addressing the cause of increased serum calcium. In patients with other forms of hypercalciuria, thiazide diuretics reduce calcium excretion in the urine by inducing mild hypovolemia, triggering increased proximal sodium reabsorption and passive calcium reabsorption. This effect can be enhanced by the addition of sodium restriction. The patient's evaluation reveals a normal uric acid concentration. In calcium stones that form on a uric acid nidus, allopurinol has been associated with a decrease in stone formation. In this patient, however, stone analysis did not reveal a uric acid core and thus would not be the next step in management. Urinary citrate inhibits stone formation by binding calcium in the tubular lumen, preventing it from precipitating with oxalate. Hypocitraturia is seen with diets high in animal protein and metabolic acidosis from chronic diarrhea, renal tubular acidosis, ureteral diversion, and carbonic anhydrase inhibitors (including seizure medications such as topiramate). The patient's urine citrate level is in the high-normal range, and the serum bicarbonate level is normal, thus increasing citrate in the urine would not be beneficial. Although increasing urine volume will reduce the calcium saturation, the present urine volume is acceptable. Urine volume to prevent stone recurrence should be between 2500 and 3000 mL per day. Recommending a low calcium diet is inappropriate for this patient because reducing calcium in the diet would provide less calcium in the gastrointestinal tract to bind oxalate and would increase oxalate absorption, and thus increase urine oxalate concentration and stone formation.
A 50-year-old man is seen for preoperative medical evaluation before left shoulder arthroplasty. History is significant for alcohol-related cirrhosis and osteoarthritis. Medications are lactulose, furosemide, and spironolactone. The patient stopped drinking alcohol 5 months ago but has difficulty with medication adherence. He reports increasing ascites and lower extremity edema. On physical examination, vital signs are normal. There is no jaundice or scleral icterus. Spider telangiectasias are noted on the face and chest. The abdomen is distended with flank dullness. There is 1+ pitting edema to the knees bilaterally. Mental status examination is normal. The calculated Model for End-stage Liver Disease (MELD) score is 22. The patient is instructed to increase his furosemide. Which of the following is the most appropriate preoperative management? Cancel surgery and refer for liver transplant evaluation Delay surgery until after patient achieves 1 year of sobriety Delay surgery until after placement of a transjugular intrahepatic portosystemic shunt Proceed to surgery
The most appropriate preoperative management is to cancel surgery and refer the patient for liver transplant evaluation. Patients with cirrhosis but no complications are referred to as having compensated cirrhosis; they may be asymptomatic or may have nonspecific symptoms, such as fatigue, poor sleep, muscle cramps, feeling cold, or itching. Patients with complications of cirrhosis (hepatic encephalopathy, variceal hemorrhage, ascites, spontaneous bacterial peritonitis, hepatorenal syndrome, jaundice, or hepatocellular carcinoma) are referred to as having decompensated cirrhosis. Referral to a transplant center is indicated for patients with decompensation or a Model for End-stage Liver Disease (MELD) score of greater than 15. Patients with Child-Pugh class C disease or a MELD score greater than 20 are at high risk for death, and all but the most urgent and life-saving surgeries should be avoided until after liver transplantation. The MELD score is an equation that incorporates bilirubin, INR, and serum creatinine levels, and it accurately predicts 3-month survival. This patient, who has decompensated liver disease with a MELD score of 22, which confers a 30-day surgical mortality risk of more than 50%, should avoid elective surgery. Patients with decompensated liver disease have not only a higher perioperative mortality rate but also a significantly increased risk for other complications, including encephalopathy, electrolyte derangements, fluid imbalance, coagulopathy, infection, acute kidney injury, and hepatorenal syndrome. It is reasonable to refer patients at intermediate risk to a hepatologist before proceeding with surgery. Patients with compensated liver disease are often able to proceed with surgery with optimal medical management. The deciding factor in this case is not the patient's duration of sobriety but the presence of decompensated liver disease and an unacceptably high MELD score. Elective surgery should be avoided until these risks are mitigated with liver transplantation. The American Association for the Study of Liver Diseases recommends against perioperative transjugular intrahepatic portosystemic shunts, stating that there is no reliable perioperative evidence of improved clinical outcomes.
A 65-year-old woman is evaluated for a 6-month history of increased facial and body hair and loss of scalp hair. Her voice has become deeper. Medical history is otherwise unremarkable, and she takes no medications. She has been menopausal since age 52 years. On physical examination, blood pressure is 140/95 mm Hg and pulse rate is 82/min. Other vital signs are normal. BMI is 28. There are coarse dark hairs on the upper lip, chin, chest, and abdomen. She also has a deep voice, frontal hair loss, and clitoromegaly. Laboratory studies show a total testosterone level of 89 ng/dL (3.1 nmol/L) and a dehydroepiandrosterone sulfate (DHEAS) level of 890 µg/dL (24.0 µmol/L) (normal <50-450 µg/dL [1.35-12.2 µmol/L]). Which of the following is the most appropriate diagnostic test to perform next? Abdominal CT Adrenal vein sampling Pelvic MRI Pelvic ultrasound
The most appropriate test to perform next is a CT scan of the abdomen. This postmenopausal woman has new-onset hyperandrogenism associated with significant elevation of serum dehydroepiandrosterone sulfate (DHEAS). The clinical picture of rapid-onset hirsutism and signs of virilization indicate an androgen-secreting tumor. Signs of virilization are deepening of the voice, clitoromegaly, hirsutism, and temporal hair loss. Under normal conditions, androgen production in women occurs in both the adrenal glands and ovaries, as well as by peripheral conversion. The major source of DHEAS is the adrenal gland, and an abdominal CT is recommended when serum DHEAS value is above 700 µg/dL (18.9 µmol/L). DHEAS-secreting tumors of the adrenal gland are readily visible on CT imaging and adrenal vein sampling to localize the tumor is rarely required. Pelvic MRI may be more sensitive at detecting small ovarian tumors and is often considered as second-line imaging when pelvic ultrasound is negative. If both pelvic ultrasound and MRI imaging are negative in the setting of suspicion for a testosterone-secreting ovarian tumor, ovarian vein sampling may be required to reveal the location of the tumor. Ovarian vein sampling is reserved for premenopausal women. Postmenopausal women can forego this invasive procedure and proceed directly to bilateral oophorectomy. Significant testosterone secretion can also be due to a non-tumorous condition called ovarian hyperthecosis. Hyperandrogenic symptoms are typically (but not always) of slower onset compared with that seen with androgen-secreting tumors. A pelvic ultrasound is recommended as the first imaging study if testosterone is above 150 ng/dL (5.2 nmol/L). This patient's testosterone level was only mildly elevated, but the DHEAS was quite elevated making a testosterone-producing ovarian tumor less likely than an adrenal tumor. Read Related TextNext Question
A 63-year-old woman is evaluated for a lesion on her nose that is slowly enlarging and nonhealing. She is otherwise in good health and takes no medications. On physical examination, vital signs are normal. The skin examination demonstrates a 0.8 × 0.6-cm pearly ulcerated papule with arborizing telangiectasias. The remainder of the examination is normal. Biopsy of the lesion demonstrates basal cell carcinoma with high-risk micronodular and infiltrative histologic features. Which of the following is the most appropriate treatment of this lesion? Cryotherapy Electrodesiccation and curettage Mohs micrographic surgery Topical 5-fluorouracil Vismodegib
The most appropriate treatment for this lesion is Mohs micrographic surgery. Mohs micrographic surgery is a specialized surgical procedure that provides margin control while sparing as much normal skin as possible. Indications for Mohs micrographic surgery include tumors with aggressive histologic subtypes (micronodular, morpheaform, infiltrative, perineural involvement), high-risk and cosmetically sensitive locations (face, genitals), large tumors or tumors arising in scar tissue, and in patients who are immunosuppressed. Because this patient has a high-risk tumor that is located in a cosmetically sensitive location, Mohs surgery is the most appropriate treatment. Small noninfiltrating basal cell carcinomas can be treated with cryotherapy; however, other surgical methods are more commonly used. To treat basal cell carcinomas appropriately with cryotherapy, the target temperature is -50 °C using liquid nitrogen as the cryogen and this requires local anesthesia. Hypopigmentation is often seen after cryotherapy. The histologic subtype and location of the basal cell carcinoma make cryotherapy inappropriate for this patient. Electrodesiccation and curettage is a widely used treatment for noninfiltrating basal cell carcinomas on low-risk anatomic sites (trunk and extremities). High-risk infiltrative and micronodular basal cell carcinomas are not appropriate for electrodesiccation and curettage. Additionally, because it may result in a cosmetically unappealing scar on the nose, it should not be used for this patient. Topical therapies including 5-fluorouracil and imiquimod are most effective for superficial basal cell carcinomas. Superficial basal cell carcinomas are well-demarcated, irregularly bordered red patches; they tend to enlarge radially rather than invading into deeper structures. Topical therapies are not effective for more aggressive histologic subtypes of basal cell carcinomas. Vismodegib is an oral medication that inhibits the hedgehog signaling pathway. It is reserved for locally advanced or metastatic basal cell carcinomas. There are significant side effects including dysgeusia, alopecia, and muscle cramps. Read Related TextNext Question
A 67-year-old woman is evaluated for urinary incontinence that has progressively worsened over the past 2 years. She experiences incontinence when she laughs or sneezes but reports no dysuria, hematuria, or loss of continence at night. She has been postmenopausal for 12 years. She had four spontaneous term vaginal deliveries between age 24 and 32 years. She has no other problems and takes no medications. On physical examination, all vital signs are normal. BMI is 20. Pelvic examination reveals vaginal atrophy. The remainder of the examination is normal. Which of the following is the most appropriate treatment? Bladder training Oral estradiol Oxybutynin Pelvic floor muscle training (Kegel exercises) Weight loss
The most appropriate treatment for this patient is pelvic floor muscle training (PFMT; also known as Kegel exercises). This multiparous, postmenopausal woman describes classic stress urinary incontinence, which is characterized by urine leakage associated with activities that cause increased intra-abdominal pressure, such as coughing, laughing, or sneezing. The American College of Physicians (ACP) recommends PFMT as first-line therapy for women with stress incontinence. PFMT may also be of benefit in patients with mixed urge and stress incontinence. If performed correctly and diligently, PFMT exercises may strengthen the pelvic floor muscles and enhance urinary retention. The patient is advised to tighten the pelvic muscles as if trying to interrupt urination. Best results require three or four sets of 10 contractions daily, with contractions lasting 10 seconds. The regimen should be continued for a minimum of 15 to 20 weeks. The ACP recommends against pharmacotherapy for this condition. Bladder training and suppressive therapy are recommended by the ACP for urgency and mixed incontinence. With bladder training, patients are instructed to void regularly throughout the day, regardless of urge, and progressively increase the interval between voids. Suppression techniques are used to manage urge to void outside of the schedule. The patient is instructed to contract pelvic floor muscles quickly three or four times, use a distraction technique (counting backwards from 100), and, when the urge passes, walk to the bathroom to urinate. The risks and benefits of systemic hormone replacement therapy, such as oral estradiol, in postmenopausal women must be carefully considered. Its use should be reserved for vasomotor symptoms of menopause at the lowest effective dosage. Estrogen replacement therapy is not recommended for chronic medical problems. Trials of topical estrogen therapy for stress urinary incontinence in patients with vaginal atrophy are of mixed quality at best, and its use is not routinely recommended. Oxybutynin is a treatment for urgency urinary incontinence when bladder training is only partially successful or has failed. It is not recommended for the treatment of stress urinary incontinence. The ACP recommends exercise and weight loss for all women with obesity and urinary incontinence. This patient does not have overweight, and her incontinence would likely not benefit from exercise and weight loss.
A 49-year-old man is evaluated in the emergency department for abdominal pain, vomiting, and nausea after binge drinking. History is significant for alcohol abuse, with numerous hospitalizations for intoxications and withdrawal. On physical examination, temperature is normal, blood pressure is 122/72 mm Hg sitting and 100/62 mm Hg standing, pulse rate is 100/min sitting and 118/min standing, respiration rate is 22/min, and oxygen saturation is 97% breathing ambient air. BMI is 18. Abdominal examination reveals diffuse abdominal tenderness to palpation; there is no rebound tenderness, ascites, or evidence of trauma. Neurologic examination is normal. There is no edema. Laboratory studies: Electrolytes : Sodium 137 mEq/L (137 mmol/L) Potassium 3.7 mEq/L (3.7 mmol/L) Chloride 96 mEq/L (96 mmol/L) Bicarbonate 10 mEq/L (10 mmol/L) Ethanol 10 mg/dL (2.2 mmol/L) Glucose 94 mg/dL (5.2 mmol/L) Lactate 0.8 mEq/L (0.8 mmol/L) Arterial blood gases : pH 7.26 PCO2 23 mm Hg (3.1 kPa) Urinalysis Specific gravity 1.020; pH 5.5; positive ketones; no blood or cells Thiamine and B-complex vitamin are administered. Which of the following is the most appropriate treatment? 0.9% saline 5% dextrose in 0.9% saline 5% dextrose in water with 150 mEq (150 mmol) of sodium bicarbonate Insulin and 5% dextrose in 0.9% saline
The most appropriate treatment is 5% dextrose in 0.9% saline for this patient who most likely has alcoholic ketoacidosis. Alcoholic ketoacidosis occurs in patients with chronic ethanol abuse, frequently with associated liver disease, and develops following an episode of acute intoxication. This patient has an increased anion gap metabolic acidosis (with an anion gap of 31), and ketoacidosis due to acute ethanol intoxication is the most likely cause. The ethanol level may be low or normal at the time of presentation because ingested ethanol may have already been extensively metabolized. Decreased insulin secretion (as a result of starvation) and increased counter-regulatory hormones cause lipolysis and generation of ketones, such as acetoacetate, which result in the anion gap. The urine in this case shows ketones, although ketone test results may be falsely negative in some cases because the nitroprusside reagent in the ketone assay detects only acetoacetate and the ketone β-hydroxybutyrate may predominate. Treatment with dextrose will increase insulin and decrease glucagon secretion, while saline will repair any volume deficit; the combination will correct ketoacidosis. In patients with alcoholism, thiamine should be administered before any glucose-containing solutions to decrease the risk of precipitating Wernicke encephalopathy. Saline alone will correct the volume deficit, but glucose is needed to stimulate insulin secretion to correct the ketoacidosis. Although dextrose with sodium bicarbonate may correct the underlying acidosis, there is no indication for additional bicarbonate in treating the ketoacidosis. In addition, alcoholic ketoacidosis may be associated with metabolic alkalosis due to concurrent vomiting. To determine if there is a concomitant metabolic alkalosis present, the corrected bicarbonate can be calculated. The corrected bicarbonate is the difference between the normal bicarbonate concentration and the delta anion gap. The delta anion gap is the difference between the measured anion gap and the normal anion gap. In this case the corrected bicarbonate concentration is 5 mEq/L (5 mmol/L) [24 − (31 − 12)]. Because the measured bicarbonate concentration (10 mEq/L [10 mmol/L]) is greater than the corrected, or expected, bicarbonate concentration, a concomitant metabolic alkalosis is present. Treatment with bicarbonate would be inappropriate. Insulin treatment is not necessary because dextrose alone will increase insulin levels in patients who do not have diabetes mellitus.
A 52-year-old man is evaluated for a 2-day history of painless red eye, which began on the right side and quickly spread to the left. He reports that his eyes have a thin mucopurulent discharge and that his eyelids are matted shut in the morning upon waking. He has had no photophobia, change in visual acuity, or itching in the eyes, but he has experienced some mild rhinorrhea. He does not use contact lenses. He is sexually monogamous. Medical history is significant for type 2 diabetes mellitus treated with metformin. On physical examination, vital signs are normal. There is redness of the sclerae bilaterally, with a white crust-like residue along the edges of the eyelids. The tarsal vessels are obscured by the conjunctival erythema. Visual acuity is intact, and there is no tenderness around the globes. Which of the following is the most appropriate treatment? Ceftriaxone Levofloxacin ophthalmic drops Olopatadine ophthalmic drops Trimethoprim-polymyxin B ophthalmic drops
The most appropriate treatment is trimethoprim-polymyxin B ophthalmic drops. This patient has acute, painless eye redness and several other signs of bacterial conjunctivitis. Studies have identified features that increase the probability of a bacterial cause of conjunctivitis, including redness of the conjunctival membrane obscuring the tarsal vessels, matting of both eyes in the morning, and purulent discharge. Inability to see redness of the eyes at 20 feet decreases the likelihood of a bacterial cause. Antibiotic treatment of bacterial conjunctivitis with topical trimethoprim-polymyxin B or erythromycin can shorten the duration of symptoms, but overall, bacterial conjunctivitis is a self-limited condition from which most patients recover within 2 weeks. Antibiotics should be enlisted when there is a higher risk for complications, such as in patients who wear contact lenses; immunocompromised patients, such as those with diabetes mellitus; and patients with copious, hyperpurulent discharge of the eye. Ceftriaxone is used to treat gonococcal infection. Typical patients with gonococcal conjunctivitis are young men with copious purulent discharge and marked conjunctival inflammation. Periocular edema and tenderness, gaze restriction, and preauricular lymphadenopathy are common with gonococcal conjunctivitis. Because of concerns about antimicrobial resistance and cost, topical fluoroquinolones (such as levofloxacin) are not first-line therapy for routine cases of bacterial conjunctivitis. Topical fluoroquinolones are indicated for conjunctivitis in contact lens wearers as a result of the high incidence of Pseudomonas infection. Olopatadine ophthalmic drops are used for seasonal allergies; the mucopurulent discharge, morning matting of the eyes, and lack of itching make allergic conjunctivitis a less likely cause of this patient's symptoms.
An 18-year-old woman is evaluated for absence of menarche. She has undergone some breast development (Tanner stage II). Medical history is unremarkable, and she takes no medications. On physical examination, her height is 150 cm (59 in) and weight is 47 kg (103.6 lb). BMI is 21. Vital signs and the remainder of the physical examination, including pelvic examination, are normal. Laboratory studies show a follicle-stimulating hormone level of 74 mU/mL (74 U/L). Serum beta-human chorionic gonadotropin level is undetectable. Thyroid-stimulating hormone and prolactin levels are normal. On pelvic ultrasound, a uterus is present, but ovaries are difficult to visualize. Which of the following is the most likely diagnosis? Functional hypothalamic amenorrhea Primary ovarian insufficiency Turner syndrome Vaginal agenesis
The most common cause of primary amenorrhea is gonadal dysgenesis, most commonly associated with Turner syndrome (45,X0). The most common cause of primary amenorrhea is gonadal dysgenesis caused by chromosomal abnormalities, most commonly those associated with Turner syndrome (TS). TS is caused by loss of part or all of an X chromosome (45,X0) occurring in 1 in 2500 live female births. In some studies, more than 20% of patients are diagnosed after 12 years of age; primary amenorrhea may be the presenting sign. The most consistent physical finding is short stature, as seen in this patient. Other findings may include neck webbing, hearing loss, aortic coarctation, and bicuspid aortic valve. Primary amenorrhea is seen in approximately 90% of women with TS. TS should be considered in women with primary or secondary amenorrhea, particularly those of short stature. Diagnosis is made by karyotype analysis. Functional hypothalamic amenorrhea is caused by a functional disruption of the hypothalamic-pituitary-ovarian axis in which no anatomic or organic disease is identified. Disruption of the pulsatile release of hypothalamic gonadotropin-releasing hormone may occur due to stress, weight loss, or exercise; follicle-stimulating hormone (FSH) levels are not elevated in functional hypothalamic amenorrhea. Primary ovarian insufficiency is considered when a woman younger than 40 years of age develops secondary amenorrhea with two serum FSH levels in the menopausal range (>35 mU/mL [35 U/L]). This condition impacts 1 in 100 women by the age of 40 years. If during the evaluation of secondary amenorrhea, an elevated FSH is found, it should be repeated in 1 month, along with a serum estradiol measurement. In young women, karyotype analysis is also indicated to rule out Turner syndrome. Vaginal agenesis is the second most common cause of primary amenorrhea with an incidence of 1 in 5000. Vaginal agenesis is characterized by congenital absence of the vagina with variable uterine development. These women have a normal female karyotype and ovarian function, thus develop normal secondary sexual characteristics. Presentation is typically after age 15 years due to primary amenorrhea. Examination reveals normal external genitalia with a dimple or small pouch replacing the vagina. On laboratory testing, gonadotropins are unremarkable. This patient's normal pelvic examination eliminates this vaginal agenesis as a cause of primary amenorrhea.
A 28-year-old woman is evaluated for 2 weeks of intense rectal pain that occurs with most bowel movements. She also reports periodic rectal bleeding and occasional constipation symptoms that have worsened since a recent shoulder surgery. Over-the-counter hemorrhoid cream has not alleviated her symptoms. Family history includes colon cancer diagnosed in her grandmother at age 74 years. The patient's medications are acetaminophen with codeine and an over-the-counter stool softener as needed. On physical examination, vital signs are normal. Rectal examination findings are shown. Which of the following is the most appropriate next step in management? Anal botulinum toxin injection Flexible sigmoidoscopy Prednisone suppository Sitz baths and psyllium Topical hydrocortisone and lubiprostone
The most effective treatment approach for anal fissure is daily warm-water sitz baths and the use of the bulk laxative psyllium. Sitz baths and psyllium are the most appropriate next step in the management of this patient, who has an acute anal fissure that is most likely the result of worsening constipation induced by the recent use of opioid analgesic therapy. Anal fissures are tears in the anoderm below the dentate line that can be seen on inspection of the perianal area, often unaided by the use of an anoscope. They are usually in the posterior position and are less often in the anterior midline. The classic symptom is pain with and after defecation, which may be associated with bright red blood on the toilet tissue. Most acute fissures heal spontaneously. The most effective treatment approach is daily warm-water sitz baths and the use of the bulk laxative psyllium. This combination therapy was superior to topical lidocaine or hydrocortisone in a randomized clinical trial. Anal botulinum toxin injection is reserved for chronic anal fissures that do not respond to more conservative treatment measures, including sitz baths, fiber supplementation, and topical therapy (nitroglycerin or calcium channel blocker) applied to the anal canal. This initial acute presentation of anal fissure is best managed by more conservative, less invasive treatment measures first. A flexible sigmoidoscopy is not indicated because the diagnosis of anal fissure can be accurately made by history and examination. Furthermore, the patient's family history is not significant enough to warrant endoscopic evaluation. Although topical anti-inflammatory therapy can alleviate the pain associated with anal fissure, the placement of a suppository may be uncomfortable. In addition, the patient has already tried over-the-counter topical hemorrhoid therapy with no relief of symptoms, so a glucocorticoid suppository is unlikely to help. Topical glucocorticoid therapy is less effective in the treatment of anal fissure than sitz baths and fiber. Lubiprostone is a peripherally acting chloride channel activator with FDA approval for the treatment of opioid-induced constipation. This patient's opioid analgesic use is temporary after shoulder surgery, and lubiprostone use is more appropriate in the setting of more chronic opioid use.
A 52-year-old woman was hospitalized 3 days ago for laparoscopic resection of the sigmoid colon secondary to recurrent diverticulitis. Diet has been advanced to a full diet. She has a 20-year history of hypertension, stage G3 chronic kidney disease, and migraine headaches. Medications are amlodipine, heparin, topiramate, and as-needed intravenous morphine. On physical examination, vital signs are normal. Mild incisional tenderness is present. The remainder of the physical examination is unremarkable. Laboratory studies: On Admission Today Creatinine 1.6 mg/dL (141.4 µmol/L) 1.9 mg/dL (168 µmol/L) Electrolytes : Sodium 140 mEq/L (140 mmol/L) 138 mEq/L (138 mmol/L) Potassium 4.9 mEq/L (4.9 mmol/L) 5.6 mEq/L (5.6 mmol/L) Chloride 102 mEq/L (102 mmol/L) 110 mEq/L (110 mmol/L) Bicarbonate 25 mEq/L (25 mmol/L) 20 mEq/L (20 mmol/L) Glucose 116 mg/dL (6.4 mmol/L) 128 mg/dL (7.1 mmol/L) Urine output during the past 24 hours is 1400 mL. Which of the following is the most likely cause of this patient's elevated serum potassium? Acute kidney injury Heparin Hyperglycemia Metabolic acidosis Topiramate
The most likely cause of this patient's elevated serum potassium level is heparin. Hypoaldosteronism caused by heparin, inhibitors of the renin-angiotensin system, type 4 renal tubular acidosis, or primary adrenal disease can cause hyperkalemia. Both unfractionated and low-molecular-weight heparin use is associated with a decrease in aldosterone synthesis. This occurs more frequently in patients with chronic kidney disease or diabetes mellitus, or in those taking an ACE inhibitor or angiotensin receptor blocker. Major underlying causes of persistent hyperkalemia are disorders in which urine potassium excretion is impaired. This can be due to a marked decrease in glomerular filtration rate, decreased sodium delivery to the distal potassium secretory sites, and hypoaldosteronism. The most common cause is chronic kidney disease with a glomerular filtration rate <20 mL/min/1.73 m2 or acute oliguric kidney injury. Except in these cases, the kidney is able to maintain potassium homeostasis. The patient is not oliguric, and the slight increase in serum creatinine postoperatively is not sufficient to cause hyperkalemia. Extreme elevations in glucose by increasing serum osmolality can directly cause hyperkalemia by pulling water from the intracellular space into the extracellular space, dragging potassium with it. This patient's glucose is only mildly elevated and would have little effect on osmolality and hyperkalemia. In patients with metabolic acidosis caused by mineral acids (such as hydrochloric acid), buffering of intracellular hydrogen ions leads to potassium movement into the extracellular fluid to maintain electroneutrality. This does not occur with organic acids such as lactate or ketoacids. In most cases of hyperchloremic metabolic acidosis, hyperkalemia typically does not develop because there is concomitant urinary and/or gastrointestinal potassium loss. This is the case in patients with hyperchloremic metabolic acidosis with losses of potassium in the stool as the result of diarrhea or in the urine in patients with renal tubular acidosis. Topiramate is a carbonic anhydrase inhibitor. Carbonic anhydrase inhibition results in proximal bicarbonate, sodium, and chloride urinary loss. The increased sodium loss causes hypovolemia and triggers secondary hyperaldosteronism, promoting potassium loss and hypokalemia.
A 40-year-old woman is evaluated for arthralgia, dry eyes, and dry mouth of several weeks' duration. She has been taking naproxen and acetaminophen daily for about 1 week. She has no pertinent personal or family history. On physical examination, vital signs are normal. Mucous membranes and conjunctivae are dry. Bilateral parotid gland enlargement is present. Laboratory studies: Creatinine 0.9 mg/dL (79.6 µmol/L) Electrolytes : Sodium 138 mEq/L (138 mmol/L) Potassium 3.1 mEq/L (3.1 mmol/L) Chloride 118 mEq/L (118 mmol/L) Bicarbonate 12 mEq/L (12 mmol/L) Glucose 74 mg/dL (4.1 mmol/L) Urinalysis pH 7.0; no blood, protein, glucose, erythrocytes, or leukocytes Kidney ultrasound shows echogenic normal-sized kidneys. Which of the following is the most likely cause of the patient's laboratory findings? Naproxen Type 1 (hypokalemic distal) renal tubular acidosis Type 2 (proximal) renal tubular acidosis Type 4 (hyperkalemic distal) renal tubular acidosis
The most likely cause of this patient's laboratory findings is type 1 (hypokalemic distal) renal tubular acidosis (RTA). It is due to a defect in urine acidification in the distal nephron and is most commonly caused by decreased activity of the proton pump in collecting duct intercalated-A cells. Because of the inability to excrete hydrogen ions, patients develop a metabolic acidosis with compensatory hyperchloremia, resulting in a normal anion gap, which is 8 mEq/L (8 mmol/L) in this patient, and the inability to acidify urine below a pH of 6.0, even in the context of an acidemia. The urine anion gap (using the equation: [Urine Sodium + Urine Potassium] - Urine Chloride) would be positive in this case, reflecting decreased acid excretion in the form of ammonium and chloride. The same defects also cause potassium wasting, and the increased proximal resorption of citrate that occurs with metabolic acidosis leads to hypocitraturia and increased risk of calcium phosphate kidney stones and nephrocalcinosis. This patient most likely has Sjögren syndrome (arthralgia, sicca, parotid gland enlargement) with concomitant interstitial nephritis (echogenicity seen on kidney ultrasound), one of the most common diseases associated with a distal RTA. Naproxen, as well as other NSAIDs, can cause an interstitial nephritis usually accompanied by acute kidney injury and proteinuria, neither of which is seen in this case. Type 2 (proximal) RTA involves a proximal tubular defect in reclaiming bicarbonate and is characterized by a normal anion gap metabolic acidosis, hypokalemia, glycosuria (without hyperglycemia), low-molecular-weight proteinuria, and renal phosphate wasting (known as Fanconi syndrome when all features are present). Because distal urine acidification remains intact, the urine pH is usually <5.5 without alkali therapy, and the urine anion gap should be negative, reflecting increased excretion of acid in the form of ammonium and chloride. This patient's high urine pH and absence of other characteristic features make type 2 (proximal) RTA unlikely. Type 4 (hyperkalemic distal) RTA due to aldosterone deficiency or resistance is associated with hyperkalemia and a urine pH <5.5, neither of which is seen in this patient.
An 83-year-old man is evaluated for a 1-week history of poor appetite, myalgia, fatigue, arthralgia, and low-grade fever. He was previously healthy and active. His only medication is acetaminophen as needed. On physical examination, the patient is afebrile. Blood pressure is 155/95 mm Hg, and pulse rate is 80/min; there are no orthostatic changes. There is trace lower extremity edema. A faint red-blue reticular rash is present over the lower extremities. Laboratory studies: Hemoglobin 12 g/dL (120 g/L) Calcium 9.8 mg/dL (2.5 mmol/L) Creatinine Current: 3.1 mg/dL (274 µmol/L); Baseline 2 months ago: 0.9 mg/dL (79.6 µmol/L) Urinalysis 3+ blood; 2+ protein; 20-30 dysmorphic erythrocytes/hpf; 5-10 leukocytes/hpf Chest radiograph shows no acute infiltrates. Kidney ultrasound shows no masses or obstruction. Which of the following is the most likely diagnosis? ANCA-associated glomerulonephritis Anti-glomerular basement membrane antibody disease Minimal change glomerulopathy Myeloma cast nephropathy Proliferative lupus nephritis
The most likely diagnosis is ANCA-associated glomerulonephritis (pauci-immune crescentic glomerulonephritis). This patient's clinical picture suggests a rapidly progressive glomerulonephritis (RPGN), with an acute and steep rise in serum creatinine accompanied by hematuria and proteinuria. The differential diagnosis for RPGN is classically accounted for by three immunofluorescence findings seen on diagnostic kidney biopsy: pauci-immune staining (for example, ANCA-associated glomerulonephritis), linear staining (for example, anti-glomerular basement membrane glomerulonephritis), and granular staining (for example, lupus nephritis). ANCA-associated glomerulonephritis accounts for more than half of all RPGN cases and has a particularly high prevalence in patients >65 years of age presenting with acute kidney injury and active urine sediment. The patient's clinical presentation of acute onset of flu-like symptoms is a classic presentation for vasculitis, and the lacy or reticular rash on examination is likely related to active ANCA-associated vasculitis. He should be tested for antiproteinase-3 and antimyeloperoxidase ANCA antibodies and undergo kidney biopsy to confirm the diagnosis. Anti-glomerular basement membrane antibody disease is a far rarer form of RPGN, with peak incidences in young men and older women. More than half of patients present with concomitant alveolar hemorrhage that may progress to life-threatening respiratory failure. These findings are not present in this patient. Minimal change glomerulopathy is characterized by sudden-onset nephrotic syndrome. It can be associated with acute kidney injury in elderly patients, and the presentation is one of severe nephrotic syndrome without significant hematuria. This is distinctly different from this patient's presentation of rash, hypertension, rapidly progressive kidney failure, and active urine sediment. Kidney dysfunction is found in 29% of patients with multiple myeloma, often due to cast nephropathy (also termed myeloma kidney), a condition in which excess monoclonal free light chains precipitate in the distal tubules and incite tubulointerstitial damage. Patients will typically have concomitant anemia and hypercalcemia. These findings are absent in this patient. Proliferative lupus nephritis is a cause of RPGN that is usually limited to the pediatric and young adult patient population. Read Related TextNext Question
A 64-year-old man is evaluated for an abnormal gait and twitching. He is brought to the office by his wife, who reports that the patient has depression and that he has abused drugs and alcohol for the past 15 years; she further states that within the past year, he has developed impulsivity, imbalance, and incoordination and often appears restless. His father had major depressive disorder and committed suicide at age 60 years. On physical examination, vital signs are normal. Orientation, language, memory, and attention are intact. Speech is dysarthric. The patient cannot perform multistep tasks and cannot sit still on command. He exhibits frequent and variable flowing movements of the limbs and trunk that he incorporates into purposeful gestures. Reflexes are brisk, and gait is rapid, wide based, and uncoordinated. Which of the following is the most likely diagnosis? Creutzfeldt-Jakob disease Frontotemporal dementia Huntington disease Parkinson disease
The most likely diagnosis is Huntington disease. His history of psychiatric disease, impulsivity, and substance abuse and clinical examination findings of executive dysfunction, dysarthria, incoordination, and ataxia are consistent with Huntington disease. His nonsuppressible flowing and variable movements are consistent with chorea, a major manifestation of Huntington disease. A positive family history is common in patients affected by this autosomal dominant disorder, but in its absence, a family history of associated conditions is suggestive. His father's prominent psychiatric disease may have led to suicide before onset of the motor features of Huntington disease; young-onset parkinsonism is a common presentation in younger patients with the disease. Tetrabenazine and deutetrabenazine are the most appropriate treatments of chorea in this disorder. Creutzfeldt-Jakob disease (CJD) is the most common form of prion disease in humans. Onset usually occurs in the seventh decade of life. The most prominent neurologic sign is disordered cognition. Typically, patients also have motor signs, such as ataxia or spasticity, vague sensory problems, or changes in visual perception. Myoclonus is common. Progressive neurologic decline resulting in death occurs rapidly, typically within 6 to 12 months. This patient's duration of symptoms is likely too long for CJD. Behavioral-variant frontotemporal dementia (FTD) is associated with prominent changes in behavior, personality, or executive function. Apathy, diminished interest, loss of empathy, lack of initiative, increased emotionality, disinhibition, euphoria, impulsivity, changes in eating behaviors, hyperorality, and compulsiveness are the most common symptoms. Motor neuron disease, including amyotrophic lateral sclerosis, is seen in approximately 15% of patients preceding, simultaneous with, or after the diagnosis of FTD. Chorea is not typical. The four cardinal signs of Parkinson disease are resting tremor, bradykinesia, cogwheel rigidity, and gait/postural impairment. Resting tremor is characteristically unilateral at onset and remains asymmetric. This patient's clinical features, including impulsivity, ataxia, hyperreflexia, and unprovoked chorea, are not consistent with Parkinson disease. Read Related TextNext Question
A 55-year-old man is evaluated in the emergency department for a 1-month history of low back pain and lower extremity swelling during the past 2 weeks. History is significant for a goiter that was surgically removed 5 years ago; pathology revealed extensive fibrosis. He takes levothyroxine. On physical examination, temperature is normal, blood pressure is 165/90 mm Hg, pulse rate is 88/min, and respiration rate is 18/min. A surgical scar over the anterior neck is noted. There is 2+ lower extremity edema to the mid calf bilaterally. The remainder of the examination is normal. Laboratory studies show a hematocrit level of 34%, a serum creatinine level of 2.1 mg/dL (185.6 µmol/L), and a normal urinalysis. Abdominal ultrasound shows bilateral hydronephrosis. Noncontrast CT of the abdomen and pelvis shows a soft-tissue mass surrounding the infrarenal aorta without any significant lymphadenopathy; the ureters are encased within the mass. Which of the following is the most likely diagnosis? Germ cell tumor IgG4-associated retroperitoneal fibrosis Lymphoma Malignant peritoneal mesothelioma
The most likely diagnosis is IgG4-associated retroperitoneal fibrosis. Most of the conditions that are under the banner of IgG4-related disease (IgG4-RD) are characterized by IgG4-producing plasma cell infiltration and tumefaction of the affected tissue with resultant organ enlargement, fibrosis, and dysfunction. Patients commonly present with a sentinel organ enlargement, but careful evaluation often reveals more extensive disease. Clinical signs include painless enlargement of lymph nodes or the thyroid, parotid, or submandibular glands; proptosis with orbital pseudo-tumor; back or chest pain from aortic involvement; and abdominal pain from pancreatic or biliary tree disease. This patient presents with a classic picture of retroperitoneal fibrosis with back pain and kidney failure from the periaortic mass that is large enough to encase the ureters resulting in obstructive uropathy and kidney injury. In a recent study, almost 60% of patients with retroperitoneal fibrosis were found to have IgG4-RD. Some patients present with an inflammatory aortitis as well. The patient's history of a fibrotic thyroid gland is also a feature of IgG4-RD. Young men with germ cell tumor can present with bulky retroperitoneal or mediastinal lymphadenopathy. A testicular mass may not always be present. Similarly, involvement of retroperitoneal, mesenteric, and pelvic nodes is common in many types of non-Hodgkin lymphoma and can lead to urinary tract obstruction. It is uncommon for either of these conditions to present as a retroperitoneal mass in the absence of lymphadenopathy, and neither germ cell tumor nor lymphoma can account for the fibrous goiter. Mesothelioma is a highly lethal disease that most commonly involves the pleura but also the peritoneum. The most frequently noted symptoms are abdominal pain, weight loss, and increase in abdominal girth due to ascites. Malignant peritoneal mesothelioma does not present as a retroperitoneal mass and does not cause urinary obstruction or a fibrous thyroid goiter.
A 49-year-old man is evaluated a 3-month history of right knee swelling without significant pain. He does a lot of physical activity as a park ranger in Michigan but does not recall trauma. He likes to walk for exercise and does not have significant pain when he walks. He has only minimal discomfort in the knee and stiffness when he squats. He reports no swelling or pain in other joints and no fever, rash, or other symptoms. He takes no medications. On physical examination, vital signs are normal. Examination of the right knee shows a large effusion without erythema, but with minimal warmth and tenderness; full range of active motion is noted. The remainder of the musculoskeletal and physical examination is normal. Knee radiographs confirm a large joint effusion in the right knee; no other abnormalities are seen. Which of the following tests is most likely to confirm the diagnosis? HLA-B27 haplotype testing MRI of the right knee Serologic testing for Borrelia burgdorferi Synovial fluid analysis for crystals
The most likely diagnosis is Lyme arthritis, which can be confirmed by serologic testing for detection of Borrelia burgdorferi-specific antibodies using a two-tiered approach: enzyme-linked immunosorbent assay (ELISA) followed by Western blot. This patient is a park ranger in an area of emerging risk for Lyme disease; thus the diagnosis should be strongly suspected, especially given a monoarthritis of the knee that is not overly painful, which is typical for Lyme arthritis. Lyme arthritis is a late manifestation of Lyme disease; after the first month of infection, at least 5/10 bands should be present on Western blot testing for IgG antibodies to different B. burgdorferi proteins. It is not uncommon for patients to be unaware of or experience manifestations of early stages of disease, such as erythema migrans or constitutional symptoms. Positive HLA-B27 haplotype may be seen in patients who have spondyloarthritis such as ankylosing spondylitis, psoriatic arthritis, or reactive arthritis. Although these conditions commonly involve the peripheral joints, including the knees, the arthritis is usually painful and oligoarticular, not monoarticular. Additionally, inflammatory symptoms such as morning stiffness are prominent. MRI of the knee can be useful to diagnose meniscal, ligamentous, and other soft-tissue abnormalities, as well as synovitis and effusion. It is particularly useful to localize knee pathology in the clinical setting of trauma, which has not occurred in this patient. Degenerative meniscal tears due to knee osteoarthritis may be seen on MRI, without a clinical history of trauma, but this patient does not have clinical or radiographic symptoms/signs of osteoarthritis. In crystal arthropathies, metabolic abnormalities promote the formation and deposition of crystals that stimulate inflammation. Symptoms are typically of an acute painful arthritis, which is not present in this patient. Synovial fluid analysis for crystals is therefore not indicated.
A 26-year-old woman is evaluated in the emergency department for an acute onset of fatigue, chills, and joint pain in the fingers of the left hand, left wrist, and right ankle during the past 3 days. There is no travel history. She is currently taking ibuprofen for her symptoms; she also takes an oral contraceptive pill. On physical examination, temperature is 39.0 °C (102.2 °F), blood pressure is 114/72 mm Hg, and pulse rate is 106/min. Fusiform enlargement of the second and fourth digits of the left hand with pain on extension is noted. Tenosynovitis over the dorsum of the left wrist and at the right ankle is present. A painless lesion on the palm is shown. Which of the following is the most likely diagnosis? Chikungunya infection Disseminated gonococcal infection Reactive arthritis Sarcoidosis
The most likely diagnosis is disseminated gonococcal infection (DGI). This patient has the arthritis-dermatitis syndrome of DGI due to Neisseria gonorrhoeae. This syndrome is characterized by a triad of tenosynovitis, dermatitis (usually painless pustular or vesiculopustular lesions), and polyarthralgia without frank arthritis. Fever, chills, and malaise are common. Inflammation of multiple tendons of the wrists, fingers, ankles, and toes distinguishes this syndrome from other forms of infectious arthritis. This presentation of DGI is associated with positive blood cultures and characteristic skin lesions. Diagnosis is established by blood and synovial fluid cultures, as well as pharynx, cervix, urethra, and rectum cultures, and/or nucleic acid amplification testing. Testing patients and their sexual partners for both gonorrhea and chlamydia is critical. Initial therapy for DGI is usually with ceftriaxone plus either azithromycin or doxycycline, and is rapidly effective. Chikungunya virus is an alphavirus endemic to Asia and Africa, where it is transmitted by mosquito vectors; however, cases are increasingly seen in the United States secondary to travel to endemic countries. In addition to fever, maculopapular rash, and headache, patients may experience synovitis and tenosynovitis of the fingers and wrists that is usually symmetric and may resemble rheumatoid arthritis. The presence of a pustular rash and lack of travel to an endemic area makes this diagnosis unlikely. Reactive arthritis is an acute, asymmetric arthritis that typically follows chlamydial urethritis or infectious gastroenteritis within 6 weeks. It mainly affects lower extremity joints. Enthesitis, particularly at the Achilles tendon, can also occur. Patients also often have conjunctivitis, circinate balanitis, or keratoderma blenorrhagicum. A pustular or vesiculopustular rash is not part of the reactive arthritis syndrome. Acute sarcoid arthritis presents most often as part of Löfgren syndrome, which is characterized by the triad of hilar adenopathy, acute arthritis, and erythema nodosum. The acute polyarthritis is mostly oligoarticular, with symmetric involvement most commonly involving the ankles or other lower extremity joints. This is in contrast to DGI, which is usually asymmetric and involves the upper extremity joints. Sarcoidosis may involve the skin, but lesions may be papular, nodular, plaque-like, or indurated and deep.
A 73-year-old woman is hospitalized for symptoms of heart failure, with progression occurring over the past 2 weeks. She also reports a 6-week history of arm aching that is worse with lifting and reaching, hip aching, morning stiffness, fever, and malaise. She reports no headache or jaw claudication. On physical examination, temperature is 37.5 °C (99.5 °F), blood pressure is normal but systolic pressure is 16 mm Hg less on the right arm than the left arm, pulse rate is 90/min, respiration rate is 20/min, and oxygen saturation is 93% breathing ambient air. There is no rash. There is no temporal tenderness. A bruit is heard over the right supraclavicular fossa. The right radial pulse is reduced. A diastolic decrescendo murmur is heard in the upper right sternal border. Bibasilar crackles are heard. Painful range of motion is noted in the shoulders and hips. No joint swelling is present. Transthoracic echocardiogram shows aortic valve regurgitation with normal leaflets, dilated aortic root, and dilated left ventricle. Which of the following is the most likely diagnosis? Kawasaki disease Polyarteritis nodosa Subcranial giant cell arteritis Takayasu arteritis
The most likely diagnosis is giant cell arteritis (GCA), a large-vessel vasculitis that most commonly affects vessels in the head and neck area (cranial GCA), resulting in jaw claudication, headaches (from involvement of the superficial temporal artery), and visual changes (due to involvement of ophthalmic arteries). Polymyalgia rheumatica (PMR), although not a vasculitis, is an inflammatory disorder that frequently accompanies GCA. Symptoms of PMR (shoulder and hip girdle pain and stiffness) may precede, accompany, or follow the diagnosis of GCA, and may be the only symptom of silent or subclinical GCA. As in this case, GCA less commonly affects the great vessels of the chest (subcranial GCA), causing upper extremity claudication and/or aortitis, as seen in this patient. Aortitis may lead to aortic root dilation, aortic regurgitation, and heart failure, as seen in this patient; this manifestation is usually a late feature of GCA. Kawasaki disease initially presents in childhood, affects medium-sized (coronary) vessels, and can lead to heart failure due to myocardial damage years after the inflammation from vasculitis has resolved. Kawasaki disease does not have an onset in late adulthood. Polyarteritis nodosa is a medium-vessel vasculitis that rarely affects the heart, and when it does, involves coronary vessels, not large vessels; this patient has a large-vessel vasculitis affecting the aorta. In Takayasu arteritis (TA), the affected arteries are primarily the aorta (ascending, descending thoracic, and abdominal aorta) and its major branches. In contrast to GCA, TA is rare (2 cases/million patient-years) and mainly affects young women (9:1 ratio) with a typical age at onset between 15 and 25 years. Histologically, the pathophysiologic processes of TA and GCA are similar, with infiltration of T cells, macrophages, and giant cells in the vessel wall. This patient's age effectively excludes TA as the cause of her symptoms.
A 42-year-old woman is evaluated during a follow-up visit for Raynaud phenomenon of 6 months' duration. She also has gastroesophageal reflux disease. Her only medication is ranitidine. On physical examination, vital signs are normal. There is no sclerodactyly or digital pitting at the fingertips. The remainder of the physical examination is normal. Laboratory studies are significant for positive antinuclear antibodies (titer: 1:640) in a centromere pattern, with a strongly positive anticentromere B antibody level. The nailfold capillaries are shown. Which of the following is the most likely diagnosis? Dermatomyositis Limited cutaneous systemic sclerosis Mixed connective tissue disease Systemic lupus erythematosus
The most likely diagnosis is limited cutaneous systemic sclerosis (LcSSc). This patient has Raynaud phenomenon and abnormal nailfold capillary changes that suggest that the Raynaud disease is part of the features of an underlying connective tissue disease. Raynaud phenomenon (sequential white, blue, and red color changes in the digits precipitated by cold or stress) occurs in almost all patients with systemic sclerosis (SSc). Raynaud phenomenon in LcSSc is initially transient and reversible; later, structural changes develop within small blood vessels, resulting in permanently impaired flow that produces acrocyanosis, digital pitting, and/or ulcerations. Office-based examination of the nailfold capillaries of patients with SSc using a dermatoscope or ophthalmoscope reveals both capillary destruction and dilated capillary loops, which can distinguish SSc from primary Raynaud. Early changes in nailfold capillaries in the setting of a connective tissue disease include dilatation. In the scleroderma spectrum disorders, nailfolds can demonstrate capillary dropout and then more disorganized loops as the vasculopathy progresses. Elevated antinuclear antibodies in a centromere pattern occur in 20% to 40% of patients with SSc and have a 90% specificity for the disease, and in particular for LcSSc. The presence of anticentromere antibodies also increases the risk for developing pulmonary arterial hypertension (PAH). This patient will need to be monitored for progression, including PAH screening. Anticentromere B antibodies are not seen in patients with dermatomyositis or mixed connective tissue disease, neither of which is the likely diagnosis in this patient. The pathognomonic cutaneous features of dermatomyositis are the heliotrope rash and Gottron papules, which are absent in this patient, as is another key feature, proximal muscle weakness. Mixed connective tissue disease (MCTD) is an overlap syndrome that includes features of systemic lupus erythematosus (SLE), SSc, and/or polymyositis in the presence of anti-U1-ribonucleoprotein antibodies. More than 50% of patients with MCTD have hand edema and synovitis at disease onset, which is not present in this patient. Anticentromere B antibodies are rarely seen in SLE (4% or less), and this patient does not have any findings associated with this disorder. SLE should be considered in any patient who presents with unexplained multisystem disease. The most common early SLE manifestations include constitutional symptoms (fever, weight loss, or severe fatigue), arthralgia/arthritis, and skin disease, all of which are absent in this patient.
A 70-year-old man was admitted to the hospital 3 days ago with an ST-elevation myocardial infarction complicated by pulmonary edema and atrial fibrillation. He underwent emergency cardiac catheterization and left anterior descending (LAD) artery stent placement. Today the patient is feeling much better with complete resolution of his initial presenting symptoms. Medications are aspirin, atorvastatin, clopidogrel, lisinopril, metoprolol, and low-molecular-weight heparin. On physical examination, pulse rate is 92/min. Other vital signs are normal. Cardiac examination reveals new findings of an irregularly irregular rhythm and an S3. His physical examination is otherwise normal. Laboratory studies obtained at the time of cardiac catheterization: Thyroid-stimulating hormone (TSH) 0.2 µU/mL (0.2 mU/L) Thyroxine (T4), total 6.5 µg/dL (83.8 nmol/L) Thyroxine (T4), free 1.0 ng/dL (12.9 pmol/L) Triiodothyronine (T3), total 60 ng/dL (0.9 nmol/L) Which of the following is the most likely diagnosis? Central hypothyroidism Heparin-induced thyroid function test abnormality Nonthyroidal illness syndrome Subclinical hyperthyroidism
The most likely diagnosis is nonthyroidal illness syndrome (euthyroid sick syndrome), which is most often seen in critically ill hospitalized patients and is characterized by a reduced serum triiodothyronine (T3) level, low or low-normal serum thyroxine (T4 ) level, and normal or low (but detectable) serum thyroid-stimulating hormone (TSH) level. These findings result from changes in the peripheral uptake of thyroid hormones, reduced levels of thyroid hormone-binding proteins, and alterations in the expression and activity of deiodinases. Very low serum T4 levels are associated with poor overall outcome. Treatment with levothyroxine or liothyronine is not indicated due to lack of evidence of benefit. Central hypothyroidism is not the most likely diagnosis. The patient described here has no signs or symptoms suggestive of hypothyroidism. Characteristic biochemical findings in central hypothyroidism include low or inappropriately normal serum TSH in the setting of low total and free T4 levels and reduced or low-normal serum T3. In contrast to nonthyroidal illness, in which the T3 to T4 ratio is low, a high serum T3 to T4 ratio is seen in central hypothyroidism because conversion of T4 to T3 in peripheral tissues is maintained. Heparin-induced thyroid function test abnormality is also incorrect. A single intravenous heparin injection may increase serum free T4 up to 5 times the baseline value within minutes. This spurious laboratory finding is related to heparin-induced stimulation of lipoprotein lipase and generation of free fatty acids, which displace T4 from binding proteins. Heparin has no effect on serum TSH, total T4, or total T3 values. Subclinical hyperthyroidism is also not the most likely diagnosis. This patient has atrial fibrillation, but no other clinical findings suggestive of thyroid hormone excess. Although the low serum TSH level is consistent with this diagnosis, the total and free T4 levels are near the lower limit of the normal range and total T3 is reduced. Read Related TextNext Question
A 65-year-old man is evaluated for a 1-month history of progressive malaise, myalgia, a 3.6-kg (8.0-lb) weight loss, and numbness and weakness of the right foot; left testicular pain for 1 week; and a painful rash on his legs for 2 days. He was diagnosed 2 months ago with hypertension, for which he takes hydrochlorothiazide. On physical examination, temperature is 37.2 °C (99.0 °F), and blood pressure is 165/90 mm Hg. The left testicle is tender. Small necrotic ulcers are noted on the legs. Numbness of the right lateral ankle and calf is noted, as well as weakness of right foot plantar flexion. Laboratory studies: Erythrocyte sedimentation rate 100 mm/h Hemoglobin 10 g/dL (100 g/L) Leukocyte count 13,000/µL (13 × 109/L) Platelet count 430,000/µL (430 × 109/L) Creatinine 1.7 mg/dL (150.3 µmol/L) ANCA Negative Urinalysis Normal Renal angiogram shows microaneurysms of the renal arteries. A deep skin biopsy (deep dermis and subcutis) shows panmural inflammation with necrosis of a medium-sized artery. Which of the following is the most likely diagnosis? Giant cell arteritis IgA vasculitis Microscopic polyangiitis Polyarteritis nodosa
The most likely diagnosis is polyarteritis nodosa (PAN), a vasculitis affecting medium-sized arteries. This entity may occur in the setting of chronic hepatitis B virus infection, HIV infection, and hairy cell leukemia. The most common symptoms are constitutional, including fever, malaise, and weight loss, and neurologic symptoms such as mononeuritis multiplex. Skin rashes, including purpura and necrotic ulcers, occur in more than half of patients. Kidney involvement manifests as hypertension due to renal artery vasculitis with renal infarction, not glomerulonephritis. Orchitis, an uncommon manifestation, is usually unilateral and due to testicular artery involvement. Mesenteric vasculitis may cause abdominal pain, perforation, and bleeding. This patient has all of these findings except for mesenteric involvement. Diagnosis of PAN is best established by demonstrating necrotizing arteritis in biopsy specimens or finding characteristic medium-sized artery aneurysms and stenoses on imaging studies of the mesenteric or renal arteries. This patient's renal angiogram confirms the presence of vasculitis and skin biopsy confirms medium-vessel vasculitis, consistent with the diagnosis of PAN. Giant cell arteritis causes myalgia and elevated erythrocyte sedimentation rate but involves large vessels usually in the head and neck area or great vessels of the chest, not medium-sized vessels. Furthermore, it usually does not involve nerves or kidneys. IgA vasculitis can cause testicular involvement, mononeuritis, and rash (palpable purpura), but it affects small vessels and causes glomerulonephritis rather than renovascular involvement. Microscopic polyangiitis (MPA) is an ANCA-associated vasculitis affecting small vessels. It can cause neurologic involvement and skin rash, but kidney involvement is due to glomerulonephritis rather than renal artery vasculitis. Furthermore, MPA is associated with a positive ANCA with perinuclear pattern, directed against myeloperoxidase. This patient does not have a positive ANCA or glomerulonephritis, making MPA an unlikely diagnosis. Read Related TextNext Question
A 45-year-old woman is evaluated for a 2-day history of deep boring pain in the right eye. She also describes eye redness and photophobia but no recent trauma to the eye. She has a 10-year history of rheumatoid arthritis, treated with etanercept. On physical examination, vital signs are normal. Diffuse right eye redness is noted, and there is pain on extra-ocular movement testing. Gentle pressure over the eye with the lid closed results in pain. There is no scleromalacia in either eye. There is diminished visual acuity of the right eye. Limited range of motion of the right wrist is noted. There is no swelling of the joints of the upper or lower extremities. Which of the following is the most likely diagnosis? Conjunctivitis Episcleritis Scleritis Subconjunctival hemorrhage
The most likely diagnosis is scleritis in this patient with rheumatoid arthritis (RA). RA is one of the most common diseases associated with scleritis. Typical features include eye pain, pain with gentle palpation of the globe, and photophobia. The deep scleral vessels are involved and may lead to scleromalacia, which is characterized by thinning of the sclera and is seen as a dark area in the white sclera. Scleromalacia may lead to perforation of the sclera called scleromalacia perforans. Scleritis can be vision-threatening and lead to blindness; it is therefore important to urgently refer the patient to an ophthalmologist for care. Conjunctivitis also causes a red eye. Typically, the underlying vessels are visible, a discharge may be seen, and often there is mattering of the eye in the morning. The eye may feel irritated but there is no pain or loss of visual acuity. In general, conjunctivitis is a diagnosis of exclusion. The presence of pain and decreased visual acuity exclude conjunctivitis in this patient. Episcleritis is an abrupt inflammation of the superficial vessels of the episclera, a thin membrane that lies just beneath the conjunctiva. The cause is often unclear; rarely, it is associated with systemic rheumatologic disease. Patients with episcleritis frequently present without pain or decreased visual acuity. On examination, the inflammation appears localized. White sclera can be seen between superficial dilated blood vessels. Episcleritis typically resolves spontaneously. The presence of pain, diffuse redness, and decreased visual acuity make episcleritis an unlikely diagnosis. Subconjunctival hemorrhage is a common disorder and typically benign in origin. It is caused by painless bleeding into the superficial portion of the eye. Examination reveals a blotchy redness (from extravascular blood) that is typically confined to one area of the conjunctiva. Subconjunctival hemorrhage is painless and not associated with loss of vision. Most cases resolve within several weeks without intervention. The patient's findings are not compatible with subconjunctival hemorrhage. Read Related TextNext Question
A 45-year-old woman is evaluated for a 4-month history of diarrhea and a 4.5-kg (10-lb) weight loss. She reports explosive episodes of loose stools that follow most meals. There is a feeling of bloating and pain with the episodes. She has a 10-year history of diffuse cutaneous systemic sclerosis (DcSSc) complicated by Raynaud phenomenon with occasional digital ulceration and gastroesophageal reflux disease. Her only medication is omeprazole. On physical examination, vital signs are normal. BMI is 21. Skin changes associated with DcSSc involve the arms, forearms, and hands. The abdominal examination reveals active and loud bowel sounds but is otherwise unremarkable. Which of the following is the most likely diagnosis? Carcinoid syndrome Chronic mesenteric ischemia Irritable bowel syndrome Small intestinal bacterial overgrowth
The most likely diagnosis is small intestinal bacterial overgrowth (SIBO) associated with systemic sclerosis (SSc). More than 70% of patients with SSc have clinical gastrointestinal involvement. Gastrointestinal motility is compromised in 40% to 90% of patients with systemic sclerosis, especially those with diffuse disease. Because of the decrease in motility of the small bowel, bacterial overgrowth occurs and leads to the symptoms described in this patient history, including diarrhea, bloating, and pain, and can lead to malabsorption. Patients with SSc can also develop chronic pancreatic insufficiency and develop symptoms similar to SIBO, which must be considered in the differential diagnosis. Diagnosis of SIBO can be confirmed with glucose hydrogen breath testing or jejunal aspirate cultures. Treatment is with rotating antibiotics to try to reduce the overgrowth using agents with both aerobic and anaerobic coverage. Probiotics may have some benefit in such patients. It is important to screen such patients for nutritional deficiencies. Carcinoid tumors are neuroendocrine tumors arising from the aerodigestive tract. Although most neuroendocrine tumors are hormonally nonfunctioning, a few produce serotonin and are responsible for the clinical manifestations of the carcinoid syndrome characterized by diarrhea and facial flushing. The diarrhea associated with the carcinoid syndrome is secretory, and symptoms are not confined to periods of food intake; nocturnal diarrhea is a clinical hallmark. Chronic mesenteric ischemia is almost always associated with atherosclerotic disease. Symptoms consist of postprandial pain within 60 minutes after meals, which results in fear of eating and weight loss. It is not associated with postprandial bloating or explosive diarrhea. Irritable bowel syndrome is a symptom complex characterized by abdominal pain and altered bowel habits. Diagnosis is established by fulfilling specific diagnostic criteria, including recurrent abdominal pain or discomfort at least 3 days per month in the last 3 months associated with two or more of the following: improvement with defecation; onset associated with a change in frequency of stool; and onset associated with a change in form (appearance) of stool. The presence of "red flags" such as anemia, fever, or weight loss strongly suggests the presence of an alternative diagnosis. Read Related TextNext Question Bibliography Gyger G, Baron M. Systemic sclerosis: gastrointestinal disease and its management. Rheum Dis Clin North Am. 2015;41:459-73. PMID: 26210129 doi:10.1016/j.rdc.2015.04.007 This content was last updated in March 2019.
A 38-year-old woman is evaluated during a follow-up visit for primary membranous glomerulopathy. Diagnosis was made by kidney biopsy 4 months ago, and she was found to be positive for anti-phospholipase A2 receptor (PLA2R) antibodies. Medications are furosemide, losartan, and simvastatin. Recent age- and sex-appropriate cancer screening tests were normal. On physical examination, vital signs are normal. There is pitting lower extremity edema to the mid shins bilaterally. Laboratory studies: Albumin 2.1 g/dL (21 g/L) Total cholesterol 288 mg/dL (7.5 mmol/L) Creatinine 1.1 mg/dL (97.2 µmol/L) Urine protein-creatinine ratio 9135 mg/g Which of the following complications is this patient at greatest risk for developing? Gout Malignancy Renal cell carcinoma Venous thromboembolism
The nephrotic syndrome can be complicated by clotting manifestations due to a secondary hypercoagulable state, and risk is related to the degree of hypoalbuminemia. The nephrotic syndrome can be complicated by clotting manifestations due to a secondary hypercoagulable state. Of all the nephrotic syndromes, membranous glomerulopathy carries the greatest risk for clotting abnormalities, with some series reporting thrombotic complications in up to 35% of the most severe membranous glomerulopathy cases. The etiology for the hypercoagulable state in membranous glomerulopathy and other forms of heavy nephrosis is multifactorial. In response to the hypoalbuminemia induced by nephrotic-range proteinuria, the liver overproduces proteins. This is most classically seen in the form of hyperlipidemia. In addition, hepatic overproduction of proteins in response to hypoalbuminemia can also lead to increased levels of procoagulant proteins such as factor V, factor VIII, and fibrinogen. Urinary loss of albumin in high volume is also accompanied by similar urinary losses of low-molecular-weight anticoagulants (notably, antithrombin III and protein S) and fibrinolytics (such as plasminogen). In a large retrospective cohort of clotting complications in membranous glomerulopathy, >70% of the clots occurred within 2 years of diagnosis, and the risk of clotting markedly increased once albumin levels dropped below 2.8 g/dL (28 g/L) (OR, 2.53; P = 0.02, compared with albumin ≥2.8 g/dL [28 g/L]). Chronic kidney disease is a risk factor for hyperuricemia and acute gout due to underexcretion of urate by the kidneys. In these patients, hyperuricemia may be due to impaired glomerular filtration and/or defects of urate handling in the renal proximal tubule. This patient's current kidney function does not place her at increased risk for gout. Patients with membranous glomerulopathy have an increased risk of malignancy. Most cancers are diagnosed in men ≥65 years of age and are often solid tumors of the prostate, lung, or gastrointestinal tract. The risk for malignancy seems to be reduced in patients with anti-phospholipase A2 receptor (PLA2R) antibody. Taking into account her negative age- and sex-appropriate cancer screening, young age, and anti-PLA2R antibody status, this patient's risk of malignancy is low. Patients with end-stage kidney disease have a markedly increased risk for renal cell carcinoma. Although current guidelines do not support routine screening for renal cell carcinoma in all patients with chronic kidney disease, a high level of suspicion is warranted in patients with symptoms such as new-onset gross hematuria or unexplained flank pain. In the absence of end-stage kidney disease, this patient is not at increased risk for renal cell carcinoma. Read Related TextNext Question
A 30-year-old man is evaluated for a 1-week history of sore throat and odynophagia. He reports no fever, nausea, vomiting, diarrhea, or other symptoms. He was recently diagnosed with HIV infection and began antiretroviral therapy 2 weeks ago. Medications are tenofovir alafenamide, emtricitabine, dolutegravir, and trimethoprim-sulfamethoxazole. On physical examination, vital signs are normal. Oral examination findings (shown) include whitish plaques on the posterior pharynx. Lymph nodes are palpable in the anterior and posterior cervical regions bilaterally. The remainder of the examination is normal. Laboratory studies at the time of HIV diagnosis showed a CD4 cell count of 55/µL and HIV viral load of 138,855 copies/mL. Which of the following is the most appropriate management? Intravenous caspofungin Oral fluconazole Nystatin swish-and-swallow Upper endoscopy Valganciclovir
The preferred treatment for oropharyngeal candidiasis, including esophageal disease, is oral fluconazole, although esophageal involvement warrants a more prolonged treatment course. Oral fluconazole is the most appropriate management for this patient's likely esophageal candidiasis; he should be treated presumptively and followed for response. The diagnosis of oropharyngeal candidiasis is usually made clinically; although whitish plaques are often prominent, oral candidiasis may also present as diffuse erythema without plaques. The presence of oral candidiasis and painful swallowing symptoms indicates likely esophageal involvement. The preferred treatment is oral fluconazole regardless if the disease is isolated to the oral cavity or extends into the esophagus; however, esophageal involvement warrants a more prolonged course (14-21 days rather than 7-14 days). Clinical response is usually apparent within a few days. Because this patient is able to swallow pills, oral therapy is appropriate, and intravenous therapy is unnecessary. Additionally, fluconazole has higher rates for complete resolution without relapse of disease than the echinocandins and is preferred therapy unless resistance is documented, which would not be expected in a patient who has not been taking long-term azole therapy. Topical agents such as nystatin are less effective than systemic fluconazole for oropharyngeal candidiasis and are especially ineffective for esophageal disease. If presumptive treatment for candida esophagitis is ineffective in improving symptoms, then upper endoscopy is indicated to better define the cause. Cytomegalovirus esophagitis is seen in immunocompromised patients and rarely occurs in patients with an intact immune system. Although herpes simplex virus (HSV) esophagitis can be seen in immunocompetent and immunocompromised patients, it is much more likely to be found in an immunocompromised person. These viral infections usually manifest as esophageal ulcerative lesions rather than plaques. Biopsies of the ulcer should be performed to confirm cytomegalovirus and HSV. Treatment of cytomegalovirus with valganciclovir (or HSV with acyclovir) would not be appropriate without first seeing evidence for it on endoscopy.
A 55-year-old man is evaluated for a 3-week history of progressive joint pain, swelling of the ankles, and occasional dry cough. He also reports a 2-day history of low-grade fever and painful red lumps on his shins. He has no other significant history and takes no medications. On physical examination, temperature is 37.8 °C (100.0 °F); other vital signs are normal. The chest is clear to auscultation. Swelling and warmth of the ankles are noted. There are three raised, erythematous, and indurated subcutaneous nodules on the right anterior shin and one on the left anterior shin. Laboratory studies show a normal complete blood count and an erythrocyte sedimentation rate of 70 mm/h. Chest radiograph shows bilateral hilar adenopathy but is otherwise normal. Which of the following is the most appropriate management? Biopsy of the hilar node Biopsy of a shin lesion Rheumatoid factor testing Synovial fluid cultures No further testing
The presence of acute arthritis, bilateral hilar lymphadenopathy, and erythema nodosum is 95% specific for Löfgren syndrome. No further testing is necessary. This patient has a triad of bihilar adenopathy, arthritis, and erythema nodosum, consistent with Löfgren syndrome, a common rheumatologic manifestation of sarcoidosis. Diagnostic specificity for sarcoidosis is 95% when all three parts of the triad are present, thus making further testing unnecessary. The most common joints involved are ankles, followed by knees and wrists. Löfgren syndrome has a good prognosis with disease remission within 2 to 16 weeks. Biopsy of a hilar lymph node demonstrating noncaseating granulomas consistent with the diagnosis of sarcoidosis is unnecessary in this patient who clinically has Löfgren syndrome. Biopsy of a shin lesion will demonstrate septal panniculitis without primary vasculitis that is consistent with erythema nodosum. The diagnosis of erythema nodosum is usually clinical, but biopsy may be helpful in patients with atypical presentations, such as lesions located in areas other than the anterior shins. In this case, biopsy will not add any further useful information. Rheumatoid arthritis (RA) is a systemic autoimmune disorder that typically presents as a symmetric inflammatory polyarthritis. Characteristically affected joints include the proximal interphalangeal and metacarpophalangeal joints of the hands and feet and the wrists, but other joints can also be involved. Rheumatoid factor is ordered as part of the evaluation of RA but is unnecessary in a patient with acute arthritis, erythema nodosum, and hilar lymphadenopathy. Synovial fluid analysis is likely to demonstrate inflammation but is not specific for sarcoidosis versus other rheumatologic diseases. Joint infection is unlikely in this patient with multiple swollen joints, rather than a single joint, and the presence of other findings specific for Löfgren syndrome.
A 33-year-old man is evaluated in the emergency department for a 10-week history of worsening confusion, memory loss, and difficulty speaking. He has become progressively more disorganized at work and can no longer complete routine tasks. His medical history is otherwise unremarkable, and he takes no medication. On physical examination, vital signs are normal. The patient exhibits decreased attention, is able to follow only simple commands, and is oriented only to person. Speech is dysarthric. Intermittent myoclonic jerking of varying limbs also is noted. Results of laboratory studies show a serum sodium level of 128 mEq/L (128 mmol/L) but are otherwise unremarkable, including a normal complete blood count, comprehensive metabolic panel, serum thyroid-stimulating hormone level, and free thyroxine (T4) level. An urgent electroencephalogram shows evidence of nonconvulsive status epilepticus with focal seizures arising independently from both temporal lobes. A contrast-enhanced brain MRI is normal. Which of the following is the most likely diagnosis? Autoimmune limbic encephalitis Lyme disease Progressive multifocal leukoencephalopathy Rapidly progressive dementia
The presence of new-onset status epilepticus and progressive confusion in a previously healthy patient strongly suggests a diagnosis of autoimmune limbic encephalitis. This patient most likely has autoimmune limbic encephalitis. New-onset status epilepticus in a previously healthy person suggests the presence of a neurologic autoimmune disorder. This patient has a rapidly progressive neurologic syndrome that is most consistent with autoimmune encephalitis, with antibodies directed against the voltage-gated potassium channel receptor complex (the most common target being the LGI1 antibody). This condition is marked by the presence of hyponatremia, myoclonus, and limbic encephalitis (amnesia, temporal lobe seizures, and confusion). Although many neurologic autoimmune disorders are paraneoplastic, primary autoimmune disorders with no accompanying cancer may occur. Lyme disease with neurologic involvement typically presents with meningitis or brainstem encephalitis (multiple cranial nerve palsies and confusion) in a patient with a history of rash and arthralgia. Chronic indolent disease may involve cognitive symptoms, but they are typically not acutely progressive, as they are in this patient. Progressive multifocal leukoencephalopathy typically presents with subacute neurologic symptoms that are focal (unilateral weakness or numbness, aphasia, cranial nerve palsies) in an immunocompromised patient, with characteristic white matter abnormalities on MRI. Rapidly progressive dementia, such as Creutzfeldt-Jakob disease, sometimes presents with symptoms similar to this patient's but usually is not accompanied by hyponatremia or focal seizures.
A 65-year-old man is evaluated for a 2-month history of low back pain. The pain is worse with movement, but it does not radiate. He reports associated fatigue and a 4.5-kg (10-lb) weight loss. He has osteoarthritis and gastroesophageal reflux disease. He has been taking ibuprofen without any pain relief. His only other medication is omeprazole. The physical examination, including vital signs and neurologic examination, is normal. Laboratory studies: Hemoglobin 9.2 g/dL (92 g/L) Creatinine 3.0 mg/dL (265.2 µmol/L) Urinalysis Trace protein; no blood, erythrocytes, leukocytes, leukocyte esterase, or nitrites Urine protein-creatinine ratio 2500 mg/g Ultrasound reveals normal-sized kidneys with slightly increased echogenicity; no hydronephrosis or abnormalities of the collecting system are seen. In addition to discontinuing ibuprofen, which of the following is the most appropriate next step in management? Discontinue omeprazole Obtain a 24-hour urine protein collection Obtain a noncontrast helical abdominal CT Obtain a urine protein electrophoresis
The presence of significant measured proteinuria in the context of minimal proteinuria on urine dipstick suggests the presence of Bence-Jones (light chain) proteinuria, which can be confirmed by a urine protein electrophoresis. The most appropriate next step in management is to obtain a urine protein electrophoresis. Albumin is the predominant protein detected on urine dipstick, which detects albumin excretion graded as trace (10-30 mg/dL), 1+ (30 mg/dL), 2+ (100 mg/dL), 3+ (300 mg/dL), and 4+ (>1000 mg/dL). Highly alkaline urine specimens can produce false-positive results on dipstick testing for protein. The sulfosalicylic acid (SSA) test can be used to detect the presence of not only albumin but also other proteins that are not detected with the urine dipstick, such as urine light chains or immunoglobulins. The possibility of cast nephropathy should be raised in patients with acute kidney injury and anemia when the urine dipstick reads negative or trace for protein, but the urine shows increased positivity for protein by the SSA test or by measuring the urine protein-creatinine ratio. Although these findings are most concerning for myeloma cast nephropathy, light chains are associated with other renal findings such as proximal tubulopathy. Proximal tubule involvement may be associated with hypophosphatemia and glucosuria (Fanconi syndrome). Omeprazole and other proton pump inhibitors are well-described causes of acute interstitial nephritis. Although this is possible, this patient's proteinuria patterns are far more consistent with multiple myeloma. Although a 24-hour urine collection is likely to provide a more accurate assessment of the total urine protein, the urine protein-creatinine ratio is sufficiently accurate to determine the presence of proteinuria. Noncontrast helical abdominal CT may be helpful if the presence of a stone or mass in the kidney is suspected; however, in the absence of flank pain or hematuria, this imaging study is unlikely to provide additional diagnostic information.
A 76-year-old man is evaluated for recent worsening of balance. He has no dizziness or lightheadedness. The patient has an 8-year history of Parkinson disease treated with carbidopa-levodopa and entacapone. On physical examination, vital signs are normal; no orthostatic decrease in blood pressure is noted. The patient has masked facies, a resting tremor, and bradykinesia. Gait assessment reveals mild shuffling without freezing. Findings from cognitive, cerebellar, and sensory examinations are unremarkable. Which of the following is the most appropriate test to determine the patient's risk of falling backward? Head impulse test Pull test Romberg test Tandem gait test
The pull test is the most sensitive predictor of the risk of backward falls in Parkinson disease. This patient's risk of falling backward is best determined by the pull test, which is the most sensitive predictor of risk of backward falls in Parkinson disease. During this test of postural stability, the examiner throws the patient off base by pulling backward on the shoulders; the test is considered positive if the patient topples into the examiner's arms or takes more than two corrective steps. Backward falls are often related to loss of postural reflexes and resultant postural instability. Additional factors, including insufficient control of motor symptoms, dyskinesia, and orthostatic hypotension, also can contribute to falls in Parkinson disease. In this patient, history and gait assessment did not reveal any interference with balance caused by freezing of gait or lower body dyskinesia, and he had no symptoms or findings suggestive of orthostatic hypotension. The head impulse (or thrust) test involves asking the patient to keep eyes focused on a distant object; the examiner then suddenly turns the head approximately 20 degrees. Patients with a normal vestibuloocular reflex remain focused on the object. An abnormal response, indicating a peripheral vestibular lesion, is movement of the eyes off the target followed by jerking of the eyes back to the target (corrective saccade). In this patient, the absence of vertigo and dizziness makes a vestibular cause less likely and testing unnecessary. The Dix-Hallpike test is another bedside test for assessment of peripheral vestibular pathology and is most helpful in the presence of vertigo, which is absent here. A positive Romberg test, defined as severe unsteadiness elicited by eye closure in a patient standing comfortably, indicates impairment of large sensory pathways at the spinal-cord or peripheral nerve level. The absence of abnormal findings on sensory and reflex examinations makes a sensory ataxia less likely in this patient. Tandem gait, in which a patient walks on a straight line, may be intact in the early phase of Parkinson disease. An impaired tandem gait, however, can indicate cerebellar or sensory ataxia or other causes of imbalance but is not a sensitive or specific predictor of backward falls.
An 18-year-old man is evaluated in the emergency department for new-onset seizures. The previous afternoon, he was evaluated in the university health service for acute onset of fever, severe myalgia, headache, and nausea; he was diagnosed with influenza and sent home with oseltamivir and ibuprofen. He received the quadrivalent meningococcal vaccine 7 months ago. Medical history is otherwise noncontributory. Other than oseltamivir and ibuprofen, he takes no medications. On physical examination, temperature is 38.9 °C (102.0 °F), blood pressure is 90/55 mm Hg, pulse rate is 130/min, and respiration rate is 28/min. The patient is confused and lethargic. Photophobia, meningismus, and a diffuse petechial rash are present. CT scan of the head is unremarkable. Microscopic examination of the cerebrospinal fluid demonstrates gram-negative diplococci. Blood and cerebrospinal fluid cultures are pending. Which of the following is the most likely bacterial agent causing this patient's meningitis? Haemophilus influenzae Listeria monocytogenes Neisseria meningitides group B Streptococcus pneumoniae
The quadrivalent meningococcal vaccine does not include coverage for serogroup B Neisseria meningitides, which now accounts for 40% of all meningitis infections in the United States. Neisseria meningitides serogroup B is the most likely cause of this patient's meningitis. Bacterial meningitis classically presents with fever, nuchal rigidity, and altered mental status. However, all three symptoms may not be present in many patients with confirmed disease. Other clinical manifestations that suggest bacterial meningitis include photophobia and headache. Serogroup B N. meningitides accounts for 40% of all bacterial meningitis infections in the United States because the quadrivalent conjugate vaccine (ACYW-135) does not include group B. College students are at higher risk for developing meningococcal meningitis because of risk factors such as living in dormitories, sharing drinks, and kissing multiple partners. Clinical findings that support the diagnosis of meningococcal meningitis include severe myalgia, rapid progression, hemodynamic instability, and early appearance of a petechial or hemorrhagic rash in a previously well person. Finally, the finding of gram-negative diplococci on cerebrospinal fluid (CSF) examination strongly supports the diagnosis. Prevalence of Haemophilus influenzae meningitis has decreased in the pediatric age group since the advent of the type B conjugate vaccine, and it is an uncommon cause of meningitis in adults. Adult patients with functional or anatomic asplenia are at greatest risk. Gram stain of the CSF would show small pleomorphic gram-negative coccobacilli, which is not consistent with this patient's findings. Listeria monocytogenes is responsible for meningitis primarily in immunosuppressed patients with decreased cell-mediated immunity, owing to medications or medical conditions, as well as in the very old or very young. L. monocytogenes appears as gram-positive rods and coccobacilli, which are not consistent with this patient's findings. Streptococcus pneumoniae is the most common bacterial cause of meningitis in adults of all ages followed by N. meningitidis. Since the introduction of the pneumococcal conjugate vaccine in 2000, the rate of pneumococcal meningitis has decreased by about one third, and the greatest reduction is seen in children younger than 5 years. S. pneumoniae is a gram-positive diplococcus and is an unlikely cause of this patient's meningitis.
A 32-year-old woman is evaluated for arthralgia and jaundice. Three months earlier, she traveled to Mexico for a 2-week vacation and developed nausea, vomiting, abdominal pain, fever, and jaundice approximately 3 weeks after returning to the United States. Testing for hepatitis A virus IgM was positive, and she was treated with supportive measures. Her symptoms and jaundice resolved within 3 weeks. A few weeks later, jaundice reappeared along with arthralgia. The patient's medical history is otherwise unremarkable, and she takes no medication. On physical examination, vital signs are normal; BMI is 27. Jaundice is noted. The remainder of the examination is unremarkable. Laboratory studies: Alkaline phosphatase 272 U/L Alanine aminotransferase 775 U/L Aspartate aminotransferase 672 U/L Total bilirubin 5.8 mg/dL (99.2 µmol/L) The results of other studies, including a complete blood count, INR, serum creatinine level, antinuclear antibody, anti-smooth muscle antibody, and total IgG level, are within normal limits. Ultrasonography of the liver is normal. Which of the following is the most likely diagnosis? Autoimmune hepatitis Leptospirosis Malaria Relapsing, remitting hepatitis A viral infection
The relapsing, remitting variant of hepatitis A viral infection is characterized by multiple clinical or biochemical relapses with spontaneous improvement within months to 1 year without intervention. Relapsing, remitting hepatitis A virus (HAV) infection is the most likely diagnosis. This patient returned from Mexico with an icteric illness and tested positive for HAV IgM, consistent with acute HAV infection. Her HAV-infection symptoms resolved and liver chemistry tests normalized, but she developed an icteric illness 3 months later. This presentation is most consistent with an atypical course of HAV infection known as relapsing, remitting HAV infection. It is rare but was reported to occur in up to 10% of patients with HAV infection in one series. Patients may have multiple clinical or biochemical relapses but will spontaneously improve within months to 1 year without intervention. The relapses tend to be milder, are more likely to be associated with cholestasis, and may be associated with extrahepatic manifestations including nephritis, arthralgia, vasculitis, and cryoglobulinemia. In rare cases, HAV infection has been observed to trigger autoimmune hepatitis. However, testing does not show the typical antibodies (anti-smooth muscle or antinuclear antibodies) or elevated IgG levels commonly seen in patients with autoimmune hepatitis. Acute leptospirosis is manifested by high fever, headache, severe myalgia, conjunctival injection, abdominal pain, diarrhea, pharyngitis, and occasionally a pretibial rash. The patient's clinical course is not compatible with leptospirosis. Malarial infection can result in jaundice due to hemolysis or, less commonly, cholestasis. However, Mexico is considered to be a very low-risk travel destination for acquisition of malaria.
A 52-year-old man is evaluated for a 3-year history of slowly progressive worsening gait, left leg weakness, spasticity, and fatigue without periods of improvement. Relapsing-remitting multiple sclerosis was diagnosed 20 years ago after an episode of optic neuritis from which he recovered fully. He takes interferon beta-1a as disease-modifying therapy. His disease was well controlled until symptom onset 3 years ago. On physical examination, vital signs are normal. Muscle strength testing shows 3/5 left hip flexion and ankle dorsiflexion and 4/5 right hip flexion. Finger-to-nose testing elicits dysmetria on the right. The patient ambulates with the assistance of a cane and cannot perform tandem walking. A T2-weighted MRI of the brain obtained 2 weeks ago showed a new lesion in the periventricular white matter that enhanced with gadolinium administration. Which of the following best describes the current status of this patient's multiple sclerosis? Primary progressive, with progression but without activity Primary progressive, without progression but with activity Relapsing remitting, with activity Secondary progressive, with progression and activity Secondary progressive, without progression but with activity
The three core phenotypes of multiple sclerosis can be modified by the presence of activity (clinical relapse or new/enlarging MRI lesion); primary and secondary progressive multiple sclerosis can be further modified as "progressive" if there is ongoing accumulation of neurologic deficits independent of clinical relapses. This patient's clinical course at this time can best be described as secondary progressive multiple sclerosis (MS), with progression and activity. His initial relapsing event of optic neuritis initially would have been diagnosed as relapsing-remitting MS. However, over the past 3 years, the patient has had a slow progressive decline in multiple neurologic symptoms without any clear exacerbations, which is the hallmark of progression in MS. Given the initial relapsing course followed by at least 2 years of disability progression without relapses, the patient's disease would now be considered secondary progressive. Recent revisions to clinical course descriptions in MS have been made to acknowledge that the clinical course in MS is not static and should be redefined constantly. Furthermore, these revised course descriptions have acknowledged the fact that relapses can occur in patients who otherwise have progressive MS and that MRI changes should be seen as a sign of relapsing activity in MS. In summary, clinical relapses or MRI evidence of new or enlarging lesions define "activity," whereas the gradual accumulation of neurologic deficits independent of relapses defines "progression." In light of this patient's recent gradual decline in neurologic function (progression) and MRI changes (activity), his disease status is best described as secondary progressive with progression and activity. If, at a later time, the patient exhibits a pattern of no new neurologic deficits, experiences no relapses, and has no new lesions on MRI, his disease status can then be described as secondary progressive without progression or activity. The patient's initial symptoms involved an acute demyelinating event (optic neuritis), with signs of disability progression coming later. He is not among the 15% of patients with MS who never experience a relapse but instead have progressive disability accumulation from the time of disease onset; these patients have primary progressive MS. Therefore, it would be inaccurate to describe the current status of this patient's MS as primary progressive. Although this patient's initial clinical course was relapsing, the progressive decline with absence of improvement in the past 3 years indicates a transition to progressive MS. The clinical course of his MS thus cannot be described as relapsing remitting, with activity.
A 28-year-old man is evaluated for a 10-month history of dizziness. He describes the dizziness as a sense of nonvertiginous imbalance and notes that it worsens with personal motion, movement of objects around him, and sitting or standing upright. The dizziness has persisted since he experienced a concussion without loss of consciousness while playing soccer 10 months ago. He reports no focal neurologic symptoms. He takes no medications. On physical examination, vital signs are normal. On neurologic examination, cranial nerve examination findings are normal, motor strength is intact, and deep tendon reflexes are normal. Romberg test result is negative. Gait is normal. Findings on a brain MRI are normal. In addition to vestibular and balance rehabilitation therapy, which of the following is the most appropriate treatment? Amitriptyline Canalith repositioning maneuver (Epley maneuver) Lorazepam Sertraline
The treatments of choice for persistent postural-perceptual dizziness are vestibular and balance rehabilitation therapy and medical therapy with selective serotonin reuptake inhibitors or serotonin-norepinephrine reuptake inhibitors. The most appropriate treatment in addition to vestibular and balance rehabilitation therapy (VBRT) is sertraline. Dizziness that remains nonspecific despite a thorough history, examination, and evaluation is referred to as persistent postural-perceptual dizziness (PPPD; formerly known as chronic subjective dizziness). PPPD is described as persistent, nonvertiginous dizziness or imbalance that worsens with personal motion, upright positioning, and movement of objects in the surrounding environment. Symptoms must be present on most days for at least 3 months. PPPD is most often preceded by another vestibular process (benign paroxysmal positional vertigo, vestibular neuronitis, vestibular migraine, stroke), trauma (concussion, traumatic brain injury), infection, or certain psychiatric conditions (anxiety, panic disorder, major depression). Approximately 75% of patients with PPPD have concomitant anxiety or depressive symptoms. The treatments of choice are VBRT and medical therapy, including selective serotonin reuptake inhibitors (SSRIs) or serotonin-norepinephrine reuptake inhibitors (SNRIs). VBRT focuses on balance training, core stabilization, and desensitization exercises; it is often performed by physical and occupational therapists. SSRIs and SNRIs take 8 to 12 weeks to produce a clinical response; if effective, treatment for at least 1 year is recommended. A positive response to these medications does not depend on the presence of psychiatric symptoms. In this patient with a history of concussion, a cause of dizziness has not been identified after thorough evaluation, and he should be treated with VBRT and an SSRI (such as sertraline) or SNRI. Treatment response with other classes of antidepressants has been disappointing, and amitriptyline has not been found to be effective in the treatment of PPPD. The canalith repositioning maneuver (Epley maneuver) is used to treat benign paroxysmal positional vertigo (BPPV). Patients with BPPV have brief episodes of vertigo (10-30 seconds) precipitated by abrupt head movement. This patient's symptoms are not compatible with BPPV, and the canalith repositioning maneuver is not indicated. Lorazepam and other benzodiazepines have been used in the treatment of acute vertigo. This patient does not have vertigo, which is characterized by a spinning, swaying, or tilting sensation that is often accompanied by nausea and vomiting. In addition, long-term treatment with lorazepam can lead to dependence and may suppress vestibular feedback and central compensation mechanisms, resulting in worsening of PPPD symptoms.
A 36-year-old woman is hospitalized for acute kidney injury and hypertension. She has an 18-month history of diffuse cutaneous systemic sclerosis. She was well and taking no medications prior to hospitalization. On physical examination, temperature is 36.6 °C (97.8 °F), blood pressure is 240/130 mm Hg, pulse rate is 100/min, and respiration rate is 18/min. Cardiac examination reveals a prominent S4. Pulmonary auscultation reveals bibasilar crackles. Cutaneous examination reveals sclerodactyly of both hands as well as skin induration of the forearms and anterior chest. Laboratory studies: Hemoglobin 8.0 g/dL (80 g/L) Platelet count 90,000/µL (90 × 109/L) Creatinine 4.9 mg/dL (433.1 µmol/L) Urinalysis 2+ protein; a few hyaline casts Peripheral blood smear shows schistocytes and decreased platelets. Which of the following is the most likely diagnosis? Disseminated intravascular coagulation Hemolytic uremic syndrome Scleroderma renal crisis Thrombotic thrombocytopenic purpura
The typical presentation of scleroderma renal crisis is acute kidney injury and severe hypertension, mild proteinuria, urinalysis with few cells or casts, microangiopathic hemolytic anemia, and thrombocytopenia. The most likely diagnosis is scleroderma renal crisis (SRC) in this patient with systemic sclerosis. Kidney involvement is common in systemic sclerosis. Up to 50% of patients have mild proteinuria, elevation in the plasma creatinine concentration, and/or hypertension, but most do not progress to chronic kidney disease. The most striking and life-threatening manifestation of kidney disease is SRC. SRC occurs in 10% to 15% of patients, and is more frequent in diffuse cutaneous systemic sclerosis (DcSSc) and in patients with anti-RNA polymerase III antibodies. SRC tends to occur early in the disease course; if untreated, it carries a mortality rate approaching 90%. The typical presentation is acute kidney injury and severe hypertension, mild proteinuria, urinalysis with few cells or casts, microangiopathic hemolytic anemia, and thrombocytopenia. Some patients develop pulmonary edema and hypertensive encephalopathy. Occasionally, patients remain normotensive despite kidney dysfunction. ACE inhibitors significantly improve kidney survival and decrease mortality among patients with SRC, regardless of the serum creatinine level. Disseminated intravascular coagulation (DIC) is characterized by abnormal activation of coagulation, generation of thrombin, consumption of clotting factors, and peripheral destruction of platelets. DIC may result from various causes, including sepsis, obstetric emergencies, acute leukemias (especially acute promyelocytic leukemia), severe burns, venoms, and shock. It is not associated with a hypertensive emergency or urgency. Hemolytic uremic syndrome (HUS) is caused by some strains of Escherichia coli, including the O157:H7 strain that produces Shiga-like toxin that targets the vascular endothelium of the glomerulus, causing cell death, breakdown of the endothelium, hemorrhage, and activation of platelets and inflammatory pathways resulting in intravascular thrombosis and hemolysis. It manifests as microangiopathic hemolytic anemia, thrombocytopenia, and acute kidney injury in the setting of dysentery. The absence of dysentery in this patient makes HUS an unlikely diagnosis. Thrombotic thrombocytopenic purpura is characterized by abnormal activation of platelets and endothelial cells, deposition of fibrin in the microvasculature, and peripheral destruction of erythrocytes and platelets. The diagnosis is clinical and requires the presence of thrombocytopenia and microangiopathic hemolytic anemia in the absence of other causes of these findings such as SRC. In this patient with systemic sclerosis, the diagnosis of SRC is much more likely. Read Related TextNext Question
A 34-year-old male is hospitalized for acute respiratory failure following a heroin overdose and aspiration. Current medications are piperacillin/tazobactam, propofol, heparin, and pantoprazole. On physical examination, temperature is 37.7 °C (99.9 °F), blood pressure is 114/77 mm Hg, pulse rate is 74/min, and respiration rate is 16/min. His ideal body weight is 63 kg (138 lb). Ventilator settings are in the volume-controlled ventilation mode with tidal volume of 630 mL, a positive end-expiratory pressure (PEEP) of 8 cm H2O, and FIO2 of 0.5. Arterial blood gas studies on an FIO2 of 0.5 show a pH of 7.33, a PCO2 of 46 mm Hg (6.1 kPa), and a PO2 of 76 mm Hg (10.1 kPa). Chest radiograph shows bilateral opacities. Which of the following is the most appropriate next step? Decrease PEEP Increase PEEP Increase respiration rate Reduce tidal volume
The use of low tidal volume ventilation and positive end-expiratory pressure is associated with prevention of ventilator-associated lung injury and a reduction in mortality related to acute respiratory distress syndrome. The most appropriate next step is to reduce the tidal volume. This patient fulfills the definition of acute respiratory distress syndrome (ARDS) with presentation within 1 week of known insult, arterial PO2/FIO2 ratio of less than 300 with positive end-expiratory pressure (PEEP) of 5 cm H2O or greater, and bilateral otherwise unexplained opacities seen on frontal chest imaging. Common pulmonary causes of ARDS include pneumonia (most common), aspiration, inhalational injury, near drowning, and drugs. Although ARDS mortality remains high, significant reductions in mortality have been attributed to the use of lung protective ventilation strategies. These strategies generally include limiting the tidal volume given in mechanical ventilation to 6 mL/kg of ideal body weight, limiting the plateau pressure in the respiratory cycle to no more than 30 cm H2O, and use of adequate PEEP to prevent the collapse of unstable alveolar units in the expiratory phase of the cycle. In the 2000 ARMA trial, low tidal volume ventilation (LTVV) was associated with a 9% absolute reduction in mortality when patients were ventilated at a goal of 6-8 mL/kg of ideal body weight compared with more liberal tidal volumes of 10-12 mL/kg. However, implementation of LTVV remains challenging for several reasons. Among them, patients who receive LTVV often demonstrate signs of air hunger. This can lead to ventilator dyssynchrony and increased sedation requirements. Current recommendations are to use a PEEP level that achieves adequate oxygenation with an FIO2 of less than 0.6 and does not cause hypotension. These parameters are being met with the current level of PEEP; therefore, adjustment of PEEP is not the most appropriate next step. If the respiration rate remains constant, a lower tidal volume will reduce the total minute ventilation and thus reduce CO2 removal. This results in higher arterial PCO2 and lower arterial pH. In the ARMA trial, LTVV-related hypercapnea was permitted, provided the arterial pH did not go below 7.3. In cases where the pH did drop below 7.3, respiration rate was increased to improve minute ventilation and decrease the PCO2. The patient's Pco2 and pH are acceptable and adjustment of the respiration rate is not the most appropriate next step.
A 42-year-old man is seen to discuss recent test results. He is asymptomatic. He has no known medical problems and takes no medications. He does not use tobacco products. On physical examination, blood pressure is 128/74 mm Hg; other vital signs are also normal. BMI is 24. The remainder of the examination is unremarkable. Laboratory studies: Total cholesterol 270 mg/dL (6.99 mmol/L) LDL cholesterol 170 mg/dL (4.40 mmol/L) HDL cholesterol 40 mg/dL (1.04 mmol/L) Triglycerides 300 mg/dL (3.39 mmol/L) His 10-year risk for atherosclerotic cardiovascular disease based on the Pooled Cohort Equations is 3.4%. Which of the following is the most appropriate treatment of this patient's hyperlipidemia? Low-intensity statin therapy Moderate-intensity statin therapy High-intensity statin therapy Therapeutic lifestyle changes
Therapeutic lifestyle changes, including dietary modification, regular physical activity, weight loss, and smoking cessation, are the initial treatment for hyperlipidemia. This patient with hyperlipidemia and low risk for atherosclerotic cardiovascular disease (ASCVD) should be counseled regarding therapeutic lifestyle changes and followed to monitor progress. Key components of therapeutic lifestyle changes include dietary modification, regular physical activity, weight loss, smoking cessation (if applicable), and addressing risk factors associated with the metabolic syndrome. Patients should be encouraged to adhere to a dietary pattern that focuses on consumption of fruits, vegetables, fiber, and monounsaturated fats and minimizes intake of saturated and trans fats, simple carbohydrates, and red meats. Replacing saturated fats with polyunsaturated fats has been shown to reduce LDL cholesterol levels and cardiovascular mortality. Recommended diets include the American Heart Association (AHA) diet and the DASH (Dietary Approaches to Stop Hypertension) diet. The AHA/American College of Cardiology (ACC) lifestyle management guideline additionally recommends that patients engage in 40 minutes of moderate to vigorous activity 3 to 4 days per week to lower LDL and non-HDL cholesterol levels. According to the 2018 AHA/ACC Guideline on the Management of Blood Cholesterol, groups for which a strong body of evidence supports statin initiation for the primary prevention of ASCVD are patients with an LDL cholesterol level of 190 mg/dL (4.92 mmol/L) or higher, patients aged 40 to 75 years with diabetes mellitus, and patients with a 10-year risk for ASCVD of 20% or higher. Other populations in whom statin therapy should be considered include patients with a 10-year risk for ASCVD of 7.5% to less than 20% accompanied by ASCVD risk-enhancing factors. In this patient with a 10-year risk for ASCVD of 3.4%, statin therapy would not be appropriate. The U.S. Preventive Services Task Force recommends low- to moderate-intensity statin therapy in asymptomatic adults aged 40 to 75 years without ASCVD who have at least one ASCVD risk factor (dyslipidemia, diabetes mellitus, hypertension, or smoking) and a calculated 10-year ASCVD event risk of 10% or higher. In contrast, the U.S. Department of Veterans Affairs and U.S. Department of Defense cholesterol guideline recommends primary prevention with moderate-intensity statin therapy for patients with a 10-year ASCVD risk of 12% or more, an LDL cholesterol level of 190 mg/dL (4.92 mmol/L) or higher, or diabetes. Although this patient has hyperlipidemia, his 10-year ASCVD event risk is 3.4%, and neither low- nor moderate-intensity statin therapy is indicated.
Mechanism of ocular hemorrhages after SAH?
There are several possible pathophysiologic mechanisms for Terson syndrome. Subarachnoid blood may be directly transmitted forward through the optic nerve sheath [4][2]. More commonly, a sudden increase in intracranial pressure leads to rapid effusion of CSF into the optic nerve sheath. The increased pressure wave in the retrobulbar optic nerve sheath mechanically compresses the central retinal vein and venous hypertension results in rupture of thin retinal capillaries. This mechanism is consistent with the fact that Terson syndrome can be seen in patients without intracranial hemorrhage [13].
A 63-year-old man is evaluated for a skin eruption that is itchy and worsening for the past several weeks. Medical history is unremarkable, and he takes no medications. On physical examination, vital signs are normal. There are tense bullae on an erythematous base and scattered erosions on the trunk and extremities. Which of the following is the most appropriate diagnostic test to perform next? Biopsy of lesional skin for direct immunofluorescence Biopsy of lesional skin for histology Biopsy of lesional skin for histology and perilesional skin for direct immunofluorescence Biopsy of perilesional skin for histology and direct immunofluorescence
This patient likely has an autoimmune bullous disease, and the most sensitive method for diagnosis is with two biopsies: one of lesional skin for histology, and one of perilesional skin for direct immunofluorescence, as shown. Autoimmune blistering diseases result from autoantibodies to different antigens in the skin and have similar but distinct presentations. Clinically, they are characterized by persistent pruritic to painful blisters with erosions and variable mucosal and ocular involvement and scarring. These diseases often arise in older persons. Identification and diagnosis of these disorders are important because of the associated morbidity and mortality. Differentiation of the autoimmune blistering diseases can be made based on clinical features, but definitive diagnosis requires histopathologic examination and, in some patients, serologic testing for pathogenic antibodies. Depending on the location of the targeted antigen, flaccid or tense bullae will be present clinically, and the corresponding separation can be appreciated using histopathology. In pemphigus, flaccid blisters correspond with suprabasilar separation, whereas tense bullae correspond with subepidermal blisters in bullous pemphigoid and epidermolysis bullosa acquisita. A skin (shave or punch) biopsy of an intact, early vesicle or bullae with adjacent normal skin should be submitted in formalin and processed for hematoxylin and eosin to examine tissue histology. Each of the autoimmune bullous diseases has characteristic histologic findings that suggest a differential diagnosis. In addition, a perilesional skin biopsy should be submitted for direct immunofluorescence, either in saline or Michel transport medium. The histologic findings together with the pattern on direct immunofluorescence can usually render a diagnosis. Serum from affected patients also can assist in diagnosis. The blood can be reacted with different substrates and will determine if circulating antibodies are present (indirect immunofluorescence). Tests such as serum enzyme-linked immunosorbent assays have been developed that detect the presence of specific antibodies in pemphigus vulgaris, pemphigus foliaceus, and bullous pemphigoid, and may correlate with disease activity. Other studies such as salt-split skin immunofluorescence may be useful in selected instances, but overall serum studies have lower sensitivities than biopsy. Read Related TextNext Question
A 62-year-old woman is evaluated during a follow-up visit for stage G5 chronic kidney disease. She is not a transplant candidate. She has opted for hemodialysis for her eventual dialysis modality; an arteriovenous fistula was created 6 months ago. She is active, has a fair appetite, and is sleeping well. She reports no nausea or vomiting. History is also significant for hypertension and secondary hyperparathyroidism. Medications are furosemide, amlodipine, epoetin alfa, sevelamer, calcitriol, and sodium bicarbonate. On physical examination, blood pressure is 144/85 mm Hg; other vital signs are normal. The left upper extremity arteriovenous fistula appears functioning. There is no asterixis. There is 2+ lower extremity edema. Laboratory studies: Bicarbonate 21 mEq/L (21 mmol/L) Blood urea nitrogen 89 mg/dL (31.7 mmol/L) Creatinine 4.8 mg/dL (424.3 µmol/L) Potassium 4.8 mEq/L (4.8 mmol/L) Estimated glomerular filtration rate 9.7 mL/min/1.73 m2 Which of the following is the most appropriate management for this patient? Delay dialysis until uremic symptoms occur Discontinue diuretics Refer for palliative care Start dialysis now
There is no benefit in starting renal replacement therapy (RRT) in asymptomatic patients or at an arbitrary estimated glomerular filtration rate cutoff compared with careful clinical management and initiating RRT for symptoms or metabolic abnormalities that are refractory to medical treatment. The most appropriate management for this patient with stage G5 chronic kidney disease (CKD) is to delay dialysis until she has uremic symptoms. She is not a candidate for transplant and has opted for hemodialysis for her renal replacement therapy (RRT). The decision of when to start RRT for CKD is complicated and requires frank discussion between providers and the patient and their families. For many years, consensus guidelines suggested that dialysis be initiated on the basis of estimated glomerular filtration rate (eGFR) cutoffs, ranging between 10 and 15 mL/min/1.73 m2. In 2010, results were published from the IDEAL (Initiating Dialysis Early and Late) trial, which demonstrated no significant difference between the early dialysis group (eGFR, 10-15 mL/min/1.73 m2) and late dialysis group (eGFR, 5-7 mL/min/1.73 m2) in the frequency of adverse events (cardiovascular events, infections, or complications of dialysis). Although 76% of the late-start group ultimately initiated dialysis with eGFR >7.0 mL/min/1.73 m2, the median delay in onset of dialysis was 5.6 months compared with the early start group. Thus, this study demonstrated that with careful clinical management, dialysis may be delayed until either the GFR drops below 7.0 mL/min/1.73 m2 or more traditional clinical indicators (such as uremic symptoms or metabolic abnormalities) for the initiation of dialysis are present. Discontinuation of diuretics is inappropriate because the patient does have some evidence of total body sodium and water overload (2+ edema) and thus needs continued diuretic therapy to avoid frank volume overload. Referral for palliative care is not indicated because the patient is functioning well and is without signs of terminal illness or symptoms that require palliation; moreover, she has already chosen hemodialysis for future RRT. However, if her condition worsens over time, it is strongly encouraged to practice "kidney supportive care" or "patient-centered dialysis," in which treatment goals are closely aligned with patient preferences in a shared decision-making process.
A 22-year-old woman is evaluated for loss of appetite and fatigue and 4.5-kg (10-lb) weight loss over the past 6 months. Medical history is otherwise unremarkable, and she takes no medications. On physical examination, blood pressure is 100/70 mm Hg and pulse rate is 94/min. Other vital signs are normal. BMI is 22. The patient looks tanned, even in areas not exposed to the sun, and has buccal and palmar hyperpigmentation. Laboratory studies: Potassium 5.3 mEq/L (5.3 mmol/L) Sodium 134 mEq/L (134 mmol/L) Adrenocorticotropic hormone (ACTH ) 650 pg/mL (143 pmol/L) Cortisol 2.8 µg/dL (77.3 nmol/L) Which of the following is the most appropriate treatment? Dexamethasone twice daily Hydrocortisone twice daily Hydrocortisone twice daily and fludrocortisone once daily Prednisone twice daily Prednisone twice daily and fludrocortisone once daily
This most appropriate treatment for this patient is hydrocortisone twice daily and fludrocortisone once daily. She has primary adrenal insufficiency, which affects all layers of the adrenal cortex, and therefore she requires both glucocorticoid and mineralocorticoid (aldosterone) therapy. Primary adrenal insufficiency is confirmed in this patient by the combination of low serum cortisol level and elevated serum adrenocorticotropic hormone (ACTH) level. Manifestations of aldosterone deficiency are hyponatremia and hyperkalemia. Some patients with primary adrenal insufficiency have normal serum electrolytes and therefore require measurement of plasma renin activity (high) and serum aldosterone (low or inappropriately normal) to confirm mineralocorticoid deficiency. The preferred glucocorticoid for treatment of adrenal insufficiency is hydrocortisone which, because of its shorter duration of action, can be prescribed two or three times daily to better mimic the circadian rhythm of endogenous cortisol secretion. The total daily recommended dose of hydrocortisone ranges from 15 to 25 mg. A higher dose of hydrocortisone (~15 mg) is given in the morning with the remaining dose (~5 mg) given in the afternoon (or in a thrice daily regimen, the second dose of hydrocortisone is given at noon with the third, smaller dose taken later in the afternoon). For patients who have difficulty adhering to a twice-daily medication regimen, prednisone once daily in the morning may be substituted. Dexamethasone has a long duration of action with the potential to cause comorbidities from excess glucocorticoid exposure and therefore should not be used long-term to treat adrenal insufficiency. There are no adequate trial data to direct optimal dose and timing of glucocorticoid replacement. Measurement of serum cortisol and/or ACTH is not helpful. Glucocorticoid dosing is guided by patient symptoms. Administration of doses of glucocorticoid in excess of physiologic replacement can be associated with decreased bone mineral density and features of Cushing syndrome, with increased risk of metabolic syndrome, type 2 diabetes mellitus, hypertension, hyperlipidemia, obesity, and cardiovascular disease. Adequate mineralocorticoid replacement is indicated by normal blood pressure without orthostasis, absence of edema, and normal serum electrolytes. Only glucocorticoid therapy is required in the treatment of secondary adrenal insufficiency. Fludrocortisone replacement is not required as aldosterone secretion is not under the control of ACTH. Hence, patients with secondary adrenal insufficiency do not develop hyperkalemia. Hyponatremia may be present in secondary adrenal insufficiency due to inappropriate antidiuretic hormone secretion and action, with resultant inability to excrete free water.
An 18-year-old man is evaluated for a murmur detected during a college sports physical examination. He reports no symptoms and has no history of cardiac disease. He takes no medications. On physical examination, vital signs are normal. He has a normal central venous pressure, waveform, precordial palpation, and S1. A continuous murmur is heard beneath the left clavicle that envelops the S2. The remainder of the examination is unremarkable. Which of the following is the most likely cause of this patient's murmur? Bicuspid aortic valve with aortic regurgitation Patent ductus arteriosus Pulmonary regurgitation Ventricular septal defect
This patient has a patent ductus arteriosus (PDA). A continuous murmur heard beneath the left clavicle that envelops the S2 is typical of a PDA; it is often described as a "machinery" murmur. A tiny PDA is generally asymptomatic with an inaudible murmur. Patients with a small PDA may have an audible murmur but no other cardiovascular features. Patients with a moderate-sized PDA may have bounding pulses, a wide pulse pressure, left heart enlargement and dysfunction, and clinical heart failure. A large PDA may present with pulmonary hypertension and shunt reversal (Eisenmenger syndrome) in adults. Aortic regurgitation due to a bicuspid aortic valve causes a diastolic murmur, most commonly heard along the left sternal border. A brief systolic murmur is also commonly heard at the second right intercostal space, from increased flow across the bicuspid aortic valve. A systolic ejection click is often heard in patients with bicuspid aortic valve, but a continuous murmur that envelops the S2 is not expected. Pulmonary regurgitation occurs most commonly after balloon or surgical intervention for congenital pulmonary stenosis. It is characterized by a diastolic murmur heard along the left sternal border that increases with inspiration; a systolic ejection murmur is also commonly heard from increased flow across the pulmonary valve. Although the pulmonary component of S2 may be reduced or absent in patients with pulmonary regurgitation, separation between systole and diastole is distinct, and the aortic component should be audible. A small ventricular septal defect (VSD) presents with a loud holosystolic murmur located at the left sternal border that may obliterate the S2; a palpable thrill is often present. The pressure gradient between the ventricles determines the murmur quality and duration. A diastolic component of the murmur is not expected. Small VSDs do not cause left heart enlargement or pulmonary hypertension. Progressive pulmonary hypertension results in shortening of the murmur.
A 25-year-old woman is evaluated for finger pain that began approximately 6 weeks ago. She works as a hairdresser. She is otherwise healthy and takes no medications. On physical examination, vital signs are normal. Skin findings reveal ragged cuticles on most of her digits. On the third digit of the left hand, the nail fold is tender. Skin and nail findings are shown. Which of the following is the most likely cause of this patient's tender finger lesion? Acute paronychia Chronic paronychia Onychomycosis Psoriasis
This patient has chronic paronychia. Chronic paronychia typically is present for 6 weeks or more and affects multiple fingers. Typical manifestations of chronic paronychia include red, swollen, and tender nail folds that lack a cuticle. It can also cause ridging and dystrophy of the nail plate. Chronic irritation from water or chemical contact is the primary cause. Candida species are often isolated. The cause of chronic paronychia is multifactorial, and an important component appears to be an eczematous process, which explains why patients with chronic paronychia respond better to topical glucocorticoid than topical antifungal therapy. The primary management includes minimizing wet work. Acute paronychia is painful swelling of the nail fold, often following minor trauma, and most commonly is caused by Staphylococcus aureus. It typically only affects one nail. Management consists of warm compresses or soaks, incision and drainage, or topical or systemic antibiotics. This patient has chronic paronychia, which tends to be more insidious and involves multiple fingers. Onychomycosis is a fungal infection of the nails. It affects 10% to 20% of adults and is the most common nail infection. Infection is more common in older men with comorbidities such as diabetes mellitus, peripheral vascular disease, and immunosuppression. Dermatophytes cause more than 90% of fungal nail infections. The most common pattern is distal subungual onychomycosis; the distal corner of the nail becomes yellow and lifted, and develops subungual debris. This can then spread proximally and laterally to involve the entire nail plate. Proximal subungual onychomycosis evolves similarly but begins at the proximal nail fold (the cuticle). This is a rare pattern and is associated with HIV infection or other severely immunocompromised patients. This patient's multiple nail involvement manifesting as red, tender, and swollen nail folds and absent cuticles is not compatible with onychomycosis. Nail changes occur in about 75% of persons with psoriasis. The most common nail abnormality observed on both fingernails and toenails is subungual hyperkeratosis. Toenails more often demonstrate thickening and discoloration (whitening or yellowing), whereas pitting and ridging are more common on fingernails. Some of the nail damage can result from concomitant onychomycosis. This patient's findings are not compatible with psoriasis. Read Related TextNext Question
A 48-year-old woman is evaluated in the emergency department for a 1-day history of hearing voices. History is significant for bipolar disorder. Medications are lithium carbonate and quetiapine. On physical examination, the patient is disheveled and looks chronically ill. She is alert and oriented but appears anxious. Blood pressure is 138/78 mm Hg, and pulse rate is 80/min without orthostatic changes. There is no edema. The remainder of the examination is normal. Laboratory studies: Blood urea nitrogen 6 mg/dL (2.1 mmol/L) Creatinine 0.9 mg/dL (79.6 µmol/L) Electrolytes : Sodium 126 mEq/L (126 mmol/L) Potassium 3.5 mEq/L (3.5 mmol/L) Chloride 94 mEq/L (94 mmol/L) Bicarbonate 26 mEq/L (26 mmol/L) Glucose 156 mg/dL (8.7 mmol/L) Urine sodium 12 mEq/L (12 mmol/L) Urine osmolality 96 mOsm/kg H2O Which of the following is the most likely cause of this patient's hyponatremia? Hyperglycemia Nephrogenic diabetes insipidus Polydipsia Syndrome of inappropriate antidiuretic hormone secretion Volume depletion
This patient has isovolemic hypotonic hyponatremia secondary to polydipsia. Isovolemia is documented by the presence of normal vital signs and physical examination findings. Hypotonicity is documented by the low calculated serum osmolality of 263 mOsm/kg H2O, using the following equation: Serum Osmolality (mOsm/kg H2O) = (2 × Serum Sodium [mEq/L]) + Plasma Glucose (mg/dL)/18 + Blood Urea Nitrogen (mg/dL)/2.8 Isovolemic hypotonic hyponatremia is secondary either to impaired dilution of urine or to water intake that exceeds the kidney's ability to excrete dilute urine. Urine osmolality distinguishes between these two entities. Urine osmolality <100 mOsm/kg H2O indicates excessive water intake, as seen with psychogenic polydipsia or poor solute intake. Because the kidney cannot excrete pure water, a minimal solute concentration of 50 mOsm/kg H2O is required. If solute intake is low while liquid intake remains high (as seen in beer potomania or chronic low food intake), water excretion is limited by available urinary solute. Hyperglycemia causes the osmotic translocation of water from the intracellular to the extracellular fluid compartment, which results in a decrease in the serum sodium level by approximately 1.6 to 2.0 mEq/L (1.6-2.0 mmol/L) for every 100 mg/dL (5.6 mmol/L) increase in the plasma glucose above 100 mg/dL (5.6 mmol/L). Although the patient has mild hyperglycemia, her glucose is not elevated enough to lower her sodium to 126 mEq/L (126 mmol/L). Diabetes insipidus, due to either a lack of antidiuretic hormone (ADH) secretion from the posterior pituitary gland or kidney resistance to ADH (nephrogenic diabetes insipidus), will result in low urine osmolality as seen in this patient. In the absence of ADH, excessive water is excreted by the kidneys. Serum sodium is typically normal but may be elevated in patients who do not have access to water. Although lithium can cause nephrogenic diabetes insipidus, the fact that she is hyponatremic rules out this diagnosis. Hyponatremia most often results from an increase in circulating ADH in response to a true or sensed reduction in effective arterial blood volume with resulting fluid retention. Hyponatremia may also be caused by elevated ADH levels associated with the syndrome of inappropriate antidiuretic hormone secretion. Because she has dilute urine indicating a lack of ADH, neither the syndrome of inappropriate antidiuretic hormone secretion nor volume depletion is the cause of her hyponatremia. Volume depletion is also excluded by the normal blood pressure and pulse measurement and absence of orthostatic changes.
A 54-year-old woman is evaluated for lumps that have appeared around the outside of her nostrils over the last 5 months. She has had arthralgias for the past 6 months. She takes no medications. On physical examination, vital signs are normal. Skin findings are shown. The remainder of the skin examination is normal. Skin biopsy shows granulomas without necrosis or lymphocytic infiltration in the dermis. Which of the following is the most likely diagnosis? Lepromatous leprosy Lupus pernio (cutaneous sarcoidosis) Lupus vulgaris (cutaneous tuberculosis) Systemic lupus erythematosus
This patient has lupus pernio, a variant of sarcoidosis that involves granulomatous inflammation of the skin around the nares. Histopathology of sarcoidosis shows granulomatous inflammation of the affected tissue with noncaseating granulomas and minimal lymphocytic infiltration. The term "lupus pernio" is a source of potential confusion. "Lupus" generally refers to systemic lupus erythematosus; "pernio" generally refers to a condition of purple papules on the distal digits exacerbated by cold and moisture; and lupus vulgaris is a form of tuberculosis of the skin. Lupus pernio has little to do with these, however. The typical appearance of lupus pernio includes violaceous subcutaneous plaques or nodules of the central face, often with some overlying scaling, most commonly seen in black persons. Patients with lupus pernio tend to have a chronic, refractory course, and extracutaneous disease. A careful evaluation for the presence of extracutaneous disease is necessary in any patient with cutaneous sarcoid, and treatment depends on other organ involvement and the severity of the clinical disease. Lepromatous leprosy may consist of erythematous macules, papules, and nodules or occasionally simply diffuse infiltration and palpable thickening of the skin. The condition is often generalized in distribution at diagnosis. There is frequent involvement of the nasal mucosa resulting in nasal stuffiness and eventually a saddle-nose deformity. The patient's localized skin findings are most consistent with lupus pernio. The classic findings of lupus vulgaris, a chronic, progressive form of cutaneous tuberculosis, include multiple discrete, red-brown papules that subsequently coalesce to form a slowly growing asymptomatic plaque most typically found on the head and neck. Histopathology will show granulomas with a variable degree of central caseating necrosis; this finding is absent in this patient, making lupus vulgaris an unlikely diagnosis. The most specific skin manifestations of systemic lupus erythematosus are variations on interface dermatitis, with pink-to-violet macules or plaques and varying scale or atrophy. One example is acute cutaneous lupus erythematosus (ACLE), also known as malar or butterfly rash. ACLE consists of erythema and edema over the cheeks and bridge of the nose and sometimes the forehead and chin. Granuloma formation is not a feature of lupus-specific skin disease.
A 60-year-old woman is evaluated for follow-up treatment of long-standing psoriasis. Medical history is significant for hypertension and hypercholesterolemia. Family history includes multiple family members with psoriasis. She has no joint symptoms, and her medications are atenolol and simvastatin. On physical examination, vital signs are normal. There is no joint swelling. She has a total of 30% body surface area involvement with psoriasis without nail involvement. Which of the following is the most appropriate treatment? Methotrexate Oral prednisone Tacrolimus ointment Topical calcitriol
This patient has moderate to severe psoriasis (30% or more body surface area involvement) and should be treated with systemic agents. Patients with psoriasis covering more than 10% body surface area or those with psoriatic arthritis, recalcitrant palmoplantar psoriasis, pustular psoriasis, or psoriasis in challenging anatomic areas (groin, scalp) may be considered for systemic therapy. These include tumor necrosis factor inhibitors, acitretin, methotrexate, IL-23 and IL-17 inhibitors, and phototherapy. Tumor necrosis factor inhibitors, such as etanercept, adalimumab, and infliximab, are excellent options for treating patients with both severe psoriasis and psoriatic arthritis. The newer IL-12/IL-23 and IL-17 inhibitors have shown excellent efficacy in psoriasis treatment. Therapy with any of the systemic agents should be guided by a clinician experienced in their use, including appropriate evaluation for contraindications and careful monitoring. Prednisone is not a good choice for the treatment of psoriasis, as the doses of prednisone required would lead to numerous side effects. Prednisone also has been shown to cause pustular and erythrodermic flares in a subset of patients with psoriasis. Oral glucocorticoids should also be avoided because they may worsen associated comorbidities such as hypertension and dyslipidemia. Topical medications such as vitamin D analogues (for example, calcitriol), immunomodulators, and glucocorticoids can be used for the treatment of psoriasis; however, they are best used as solo agents in the treatment of mild disease with less than 10% body surface area involvement. These medications are often employed in conjunction with systemic therapy to treat small areas of psoriasis that remain after treatment with a systemic agent. Topical immunomodulators, pimecrolimus cream, or tacrolimus ointment are best used on the face or in intertriginous regions to avoid the atrophy seen with topical glucocorticoids. Read Related TextNext Question
Features of stasis dermatitis?
This patient has stasis dermatitis with the characteristic findings of bilateral edema, erythema, scaling, and pruritus of the bilateral lower legs. Crusting and erosions may also be seen. This is due to chronic venous insufficiency or other causes of chronic lower extremity edema. When secondary to chronic venous insufficiency, varicosities, telangiectasias, ulcers, and brown discoloration may also be noted. Symptomatic treatment with topical glucocorticoids and emollients should be used; however, the edema must be addressed with compression stockings and leg elevation for significant improvement to occur. Allergic contact dermatitis is common in patients with stasis dermatitis. It commonly occurs due to the application of topical medications such as over-the-counter triple antibiotic ointment. This patient denies applying any topical medication or emollient to her legs, so her diagnosis would not be allergic contact dermatitis. In patients exposed to potential allergens, a clue to the diagnosis of contact dermatitis is the failure of the patient to respond to appropriate therapy for stasis dermatitis. In these patients, patch testing is often necessary to determine if a secondary allergic contact dermatitis is present. Stasis dermatitis is often misdiagnosed as cellulitis. It would be unusual for cellulitis to present bilaterally. Unlike stasis dermatitis, cellulitis is typically tender and not pruritic. Cellulitis is usually hot to the touch and involves less scaling than stasis dermatitis. Leukocytoclastic vasculitis is inflammation of the small vessels of the skin; it typically presents with nonblanching, violaceous papules (palpable purpura), and macules on the lower extremities. Leukocytoclastic vasculitis can be idiopathic, but it is usually associated with an infection or is often drug induced. Other causes of leukocytoclastic vasculitis include connective tissue disease, inflammatory bowel disease, and malignancies. Psoriasis is characterized by well-demarcated, erythematous plaques with silvery scale. It commonly appears on the knees, elbows, and scalp. There can be associated nail changes (nail pits, onycholysis). This patient's clinical presentation is not consistent with psoriasis. Read Related TextNext Question
A 38-year-old woman is evaluated for a new rash on the chest, arms, and back. It appeared 3 weeks ago. It burns and itches and has not responded to over-the-counter hydrocortisone cream. The rash substantially worsened following a vacation at the beach. Medical history is significant for psoriasis vulgaris with involvement of the elbows and knees with psoriatic arthritis for 3 years' duration. Her only medication is adalimumab, which she started 2 months ago. It cleared her elbows and knees in about 3 weeks. On physical examination, vital signs are normal. Representative skin findings are shown. There is no facial rash, hair loss, rash in the ears, oral ulcerations, or swollen joints. Laboratory studies show antinuclear antibody titer less than 1:40. Which of the following is the most appropriate treatment for this patient? Add hydroxychloroquine Add methotrexate Add oral prednisone taper Discontinue adalimumab
This patient has subacute cutaneous lupus erythematosus (SCLE), and she should discontinue adalimumab. Approximately one third of SCLE cases are drug induced. Tumor necrosis factor inhibitors such as adalimumab are a leading cause of drug-induced SCLE. Unlike psoriasis, SCLE is photodistributed, tends to burn more than itch, and has a lighter pink-to-violet tone when compared with the red tones of psoriasis. Psoriasis improves with sun exposure, whereas SCLE worsens with sun exposure. This patient's rash at presentation is different from the distribution and appearance of her psoriasis, and it is worsened in the sun rather than improved. These features argue against psoriasis treatment failure and point toward drug-induced SCLE. Medications should be evaluated as a cause of all cases of SCLE, particularly those affecting young white men who have a very low risk of developing lupus spectrum disease. Other common causative medications are hydrochlorothiazide, ACE inhibitors, NSAIDs, proton pump inhibitors, and terbinafine. Drug withdrawal typically leads to improvement in drug-induced SCLE. Hydroxychloroquine is a first-line therapy for native cutaneous lupus, but cessation of adalimumab is a better choice for drug-induced SCLE. Methotrexate is an excellent agent to add to tumor necrosis factor inhibitors to prevent or manage the development of human antichimeric antibodies, which may lead to treatment failure or resistance in psoriasis or psoriatic arthritis. Such treatment failure may manifest as a sudden loss of efficacy of therapy with recurrence of skin or joint symptoms. This patient has no evidence of treatment failure, and the best management of the new rash is discontinuation of adalimumab. A short prednisone taper might give the patient brief respite from her symptoms, but they would return shortly if adalimumab is not discontinued. Short prednisone tapers should be avoided in patients with psoriasis as significant rebound skin disease can occur, resulting in erythroderma in some patients. Read Related TextNext Question
A 43-year-old woman is evaluated for painful wheals on the upper legs and back that have been present for 2 weeks. Individual lesions resolve with bruising in 3 to 4 days. The patient also reports some associated joint pain, particularly of the small joints in her hands. Medical history is unremarkable, and she takes no medications. On physical examination, vital signs are normal. There are polycyclic edematous plaques on the back and upper legs. The remainder of the examination, including joint examination, is normal. Which of the following is the most appropriate management? Epicutaneous patch testing Ice cube provocation test Oral prednisone Skin biopsies Topical triamcinolone
This patient has urticarial vasculitis, and the most appropriate management is to obtain skin biopsies to confirm the diagnosis. A routine skin biopsy (for histology) and a biopsy for direct immunofluorescence (for immunoglobulin and complement deposits) should both be obtained. Typical urticarial wheals last for less than 24 hours (with individual lesions lasting only a few hours), resolve without cutaneous sequelae, and are pruritic and not painful. Urticarial vasculitis differs from urticaria in that individual lesions persist longer than 24 hours. Urticarial vasculitis more commonly presents with painful or burning lesions instead of pruritus and leave bruise-like changes when they resolve, which is atypical for usual urticaria. Patients with urticarial vasculitis often have an underlying autoimmune disease, most often lupus erythematosus, but even in the absence of an associated autoimmune disease, patients with urticarial vasculitis (particularly hypocomplementemic urticarial vasculitis) are at risk of multisystem disease, including nephritis. The patient's joint pain suggests the possibility of an additional underlying disorder. Skin biopsies are important in making the diagnosis and guiding further work-up and treatment. Epicutaneous patch testing is performed to diagnose allergic contact dermatitis. Allergic contact dermatitis can occasionally present as typical urticarial, but this patient's history suggests urticarial vasculitis. Physical urticaria is induced by a physical stimulus such as the sun, sweating, physical pressure, or cold temperature. An ice cube provocation test is performed to help diagnose cold urticaria. This test would not be helpful in diagnosing urticarial vasculitis. Neither allergic contact dermatitis nor physical urticaria can explain the patient's joint symptoms. Treatment of urticaria is most effective with long-acting antihistamines, since they help treat active disease and prevent new flares of urticaria. Systemic glucocorticoids and immunosuppressive agents have been used for urticaria, but in most patients, maximizing the dose of the long-acting antihistamines is equally effective and safer. Glucocorticoids are often the mainstay of treatment for urticarial vasculitis, but it would be inappropriate to commit a patient to this therapy without first confirming the diagnosis. Topical glucocorticoids, such as triamcinolone, have limited efficacy in typical and urticarial vasculitis and are not the best management for either condition.
A 27-year-old woman is evaluated for a 6-month history of fatigue, arthralgia, and myalgia. She has a history of urinary tract infections. Medications are an oral contraceptive pill and as-needed naproxen for pain. On physical examination, temperature is 38.2 °C (100.8 °F), blood pressure is 142/90 mm Hg, and pulse rate is 90/min. Cardiac, lung, and abdominal examinations are normal. Laboratory studies show a serum creatinine level of 1.4 mg/dL (123.8 µmol/L); urinalysis shows 2+ blood, 3+ protein, positive leukocyte esterase, no nitrites, 10-15 erythrocytes/hpf, 5-10 leukocytes/hpf, and no crystals. Urine microscopy is shown. Which of the following is the most likely diagnosis? Bladder cancer Glomerulonephritis Tubulointerstitial nephritis Urinary tract infection
This patient likely has glomerulonephritis. Glomerular hematuria typically features brown- or tea-colored urine with dysmorphic erythrocytes (or acanthocytes) and/or erythrocyte casts on urine sediment examination. Erythrocyte casts are recognized by their cylindrical or tubular structure and inclusion of small, agranular spherocytes and, when present, are specific for hematuria of glomerular origin. Isomorphic erythrocytes are of the same size and shape and usually arise from an extraglomerular urologic process causing bleeding into the genitourinary tract, such as a tumor, stone, or infection. Dysmorphic erythrocytes have varying sizes and shapes. Acanthocytes, a specific form of dysmorphic erythrocytes, are characterized by vesicle-shaped protrusions and suggest a glomerular source of bleeding. Acanthocytes and erythrocyte casts are highly specific for glomerulonephritis and exclude an extraglomerular cause of bleeding such as bladder cancer. Sterile pyuria and leukocyte casts are hallmarks of tubulointerstitial nephritis, which can present acutely or may progress indolently and present as chronic kidney disease of unclear duration. Mild subnephrotic proteinuria also can be seen with interstitial nephritis. The cells comprising a leukocyte cast are larger than erythrocytes and appear more granular. The presence of erythrocyte casts excludes the diagnosis of tubulointerstitial nephritis. Although this febrile patient has leukocytes (granular cells larger than erythrocytes) on urinalysis and positive leukocyte esterase on dipstick analysis, the presence of erythrocyte casts is specific for glomerulonephritis. This patient has the nephritic syndrome, which is associated with glomerular inflammation resulting in hematuria, proteinuria, and leukocytes in the urine sediment.
A 51-year-old woman is evaluated in the emergency department for a 1-month history of new-onset abnormal movements, paranoia, hallucinations, and progressive confusion. She was healthy before onset of symptoms and takes no medication. On physical examination, vital signs are normal. The patient is alert but oriented to person and place only, recalls none of three objects after 5 minutes, requires constant redirection to follow commands and sustain attention, is having ongoing visual and auditory hallucinations, and exhibits intermittent slow, writhing (choreiform) movements of the arms. Muscle strength is normal, as are sensation and deep tendon reflexes. Results of standard laboratory studies, including a complete blood count and comprehensive metabolic panel, are unremarkable. Results of brain MRI and lumbar puncture are normal. Subsequent testing for serum anti-N-methyl-D-aspartate receptor antibody is positive. Which of the following is the most likely diagnosis? Breast adenocarcinoma Non-small cell lung cancer Ovarian teratoma Small cell lung cancer
This patient most likely has an ovarian teratoma. An autoimmune condition termed anti-N-methyl-D-aspartate receptor (anti-NMDAR) antibody encephalitis has emerged as an increasingly common cause of encephalitis. Anti-NMDAR antibody encephalitis is associated with ovarian teratomas in more than 50% of patients with the disease because of production of an antibody to a tumor protein that cross-reacts with neuronal tissue. The diagnosis is suggested by the presence of choreoathetosis, psychiatric symptoms, seizures, and autonomic instability and is confirmed by detection of anti-NMDAR antibody in serum. This patient has developed new-onset psychiatric disease and a movement disorder. These symptoms raise suspicion of paraneoplastic or autoimmune encephalitis, and testing confirms the presence of the anti-NMDAR antibody in the serum. Patients are evaluated with CT and/or transvaginal ultrasound. Treatment includes removal of the teratoma to eradicate the immune stimulus and immunosuppression with glucocorticoids or intravenous immune globulin. Paraneoplastic syndromes associated with breast adenocarcinoma can present as ataxia, brainstem encephalitis, ophthalmoplegia, and parkinsonism. They are most often associated with anti-Ri and anti-glutamic acid decarboxylase antibodies rather than anti-NMDAR antibody. Unlike small cell lung cancer, non-small cell lung cancer (NSCLC) typically is not associated with neurologic paraneoplastic syndromes. NSCLC, however, may be associated with other paraneoplastic syndromes, including hypercalcemia and hypertrophic osteoarthropathy. Small cell lung cancer is associated with a host of neurologic paraneoplastic syndromes, including dementia, chorea, ataxia, brainstem encephalitis, and neuropathies, among others. The antibodies associated with these syndromes include anti-Hu, anti-LGI1 (voltage-gated potassium channel), anti-CRMP5 antibodies and rarely the anti-NMDAR antibody. Read Related TextNext Question
An 82-year-old man is evaluated during a routine evaluation. He is accompanied to the visit by his son. The patient lives alone, and his son expresses reservations about his father continuing to drive. The patient no longer drives after dark or on the interstate highway. He limits his driving to within a 10-mile radius of his home and mainly drives for local errands and to church on Sundays. He has had no traffic accidents, but he had two recent incidents in which he misjudged the angle of his car in the grocery store parking lot and ran into the shopping cart stand. Medical history is significant for coronary artery disease, hypertension, and mild cognitive impairment. Medications are atorvastatin, aspirin, hydrochlorothiazide, lisinopril, and metoprolol. On physical examination, blood pressure is 132/82 mm Hg, and pulse rate is 64/min; other vital signs are normal. The patient appears frail with a pleasant demeanor. He wears eyeglasses and hearing aids, and he has impaired hearing as measured by the whispered voice test. On musculoskeletal examination, limited mobility of the cervical spine is noted. He scores 26/30 on the Mini-Mental State Examination. The remainder of the examination is unremarkable. Which of the following is the most appropriate management regarding this patient's driving? Advise the patient to retire from driving Obtain neuropsychological testing Obtain occupational therapy driving evaluation Reassure the patient he is competent to drive with self-imposed limitations
This patient should be advised to retire from driving. Driving assessments are qualitative and rely heavily on clinical judgment. The more risk factors for a motor vehicle accident that an older driver has, the higher the risk for an adverse event while driving. Drivers at highest risk should be counseled to retire from driving. This patient has multiple risk factors for unsafe driving, including cognitive impairment, self-restrictions in driving (does not drive after dark or on the interstate highway, drives within a 10-mile radius of home), minor accidents, and concerns from family members about driving safety. His other risk factors include impaired mobility, hearing decline, and medical conditions with increased risk for loss of consciousness. Physician advice to retire from driving is associated with older drivers appropriately discontinuing driving. Given the risk for depression and social isolation associated with driving retirement, however, this advice should be coupled with suggestions for alternate forms of transportation and follow-up assessment of his mood and quality of life. Detailed neuropsychological testing is especially useful for the following patients: (1) those with milder cognitive symptoms to determine whether cognitive difficulties are within the realm of normal age-associated cognitive decline versus mild cognitive impairment; (2) those with definite dementia, diagnosed on the basis of clinical impression and results of screening cognitive tests, who have clinical features overlapping two or more underlying pathologic processes; and (3) those with cognitive symptoms whose clinical picture is confounded by significant depression. The results of neuropsychological testing are unlikely to change the recommendation to retire from driving considering this patient's multiple risk factors. If a patient is resistant to advice to retire from driving, a formal occupational therapy driving evaluation may be helpful. Given his numerous observable risk factors, this patient should not be reassured that he is competent to drive.
A 22-year-old man is evaluated in the emergency department 40 minutes after having a first-time generalized tonic-clonic seizure. According to his mother, the patient exhibited shaking for 5 minutes. He has never had a seizure previously or any episode of jerking, staring, confusion, or memory loss. He has had no recent illness and has no history of neurologic problems. Birth and development were normal. There is no family history of seizures or epilepsy. He takes no medication and does not use illicit drugs. On physical examination, vital signs are normal. Neurologic examination findings are unremarkable. Results of laboratory studies show a normal complete blood count and comprehensive metabolic panel and a negative urine drug screen. Which of the following is the most appropriate initial step in management? Head CT Intravenous levetiracetam Intravenous lorazepam Lumbar puncture
This patient should have CT of the head. Neuroimaging is indicated in all patients with a first-time seizure to rapidly exclude conditions requiring emergent intervention, including hemorrhage. Head CT is the most appropriate initial study because it typically is readily available in most emergency departments and can be performed rapidly. The patient will later require both MRI and electroencephalography, but CT is the best initial study. Levetiracetam and other antiepileptic drugs are not recommended for first-time seizures until the results of further testing, such as head CT, brain MRI, and electroencephalography (EEG), are known and the recurrence risk of seizures can be determined. He had an unprovoked seizure but does not meet criteria for the diagnosis of epilepsy (at least two unprovoked seizures more than 24 hours apart, or one unprovoked seizure with a high risk of recurrence on the basis of abnormalities found on testing). The 2-year recurrence risk after a single generalized tonic-clonic seizure in patients with normal findings on clinical examination, no epilepsy risk factors, and a normal head CT scan is approximately 40%; that number decreases to about 20% if MRI and EEG are normal. In the setting of two unprovoked seizures or one unprovoked seizure with significant EEG or MRI abnormalities, recurrence risk is at least 60% and treatment is recommended Although this seizure lasted 5 minutes and thus would meet the definition of convulsive status epilepticus (CSE) if seizure activity were ongoing, the patient has recovered completely and has normal findings on clinical examination. Therefore, he does not have CSE, and intravenous lorazepam is not indicated. In adults, lumbar puncture is appropriate when meningitis is clinically suspected (for example, in the presence of neck stiffness, altered mental status, or fever) or when a patient has symptoms highly suggestive of subarachnoid hemorrhage in conjunction with normal noncontrast head CT findings. It otherwise is not indicated after a seizure in adults.
A 50-year-old man is evaluated for a several month history of itchy, scaly feet. It has persisted despite the application of a moisturizing lotion. He has no significant medical history and takes no medications. On physical examination, vital signs are normal. There are erythematous scaly patches on the sides of the feet and maceration between toes. Toenails are normal. Microscopic examination using potassium hydroxide preparation shows branching hyphae in the keratin (scale). Which of the following is the most appropriate treatment? Imidazole cream Nystatin cream Oral ketoconazole Topical betamethasone and clotrimazole
Treatment of tinea of non-hair-bearing skin includes topical antifungal agents such as imidazole, miconazole, clotrimazole, ketoconazole, ciclopirox, or terbinafine; topical nystatin is not effective, and oral ketoconazole should be avoided. Dermatophytosis or tinea of non-hair-bearing skin with limited involvement can be treated with imidazole cream applied once to twice daily for 2 to 4 weeks. Application should extend a few centimeters beyond the advancing border. For tinea pedis, the web spaces between the toes should be treated. Other topical agents including miconazole, clotrimazole, ketoconazole, ciclopirox, or terbinafine can also be used. Over-the-counter preparations are cost-effective options with good efficacy. Most infections will resolve but may recur and require retreatment. In immunosuppressed patients, recognition and treatment of superficial skin fungal infections is essential, as fungal infections can lead to epidermal breakdown and create a portal of entry for invasive pathogens. Tinea pedis is also a potential cause of recurrent bacterial cellulitis. Dermatophytes do not respond to topical nystatin, which is used to treat infections caused by Candida species. Oral antifungal therapy with terbinafine or an azole antifungal agent such as itraconazole or fluconazole may be necessary for treating tinea capitis, onychomycosis, Majocchi granuloma (a granulomatous response to dermatophyte infection in the dermis and hair follicles), or extensive infection. Oral ketoconazole no longer has an indication for superficial fungal infection because of severe and sometimes fatal idiosyncratic liver toxicity. In addition, ketoconazole is a potent inhibitor of CYP3A4 resulting in significant drug interactions. Prolonged use of ketoconazole also may result in adrenal gland suppression. Combination therapy with potent topical glucocorticoids, such as betamethasone, and antifungal creams, such as clotrimazole, should be avoided because of an increased risk of treatment failures, development of skin atrophy with prolonged use, and increased cost without increased efficacy. Read Related TextNext Question
A 22-year-old man is evaluated during a pre-employment examination. The patient is starting a new job as a registered nurse. He is asymptomatic. He received the tetanus toxoid, reduced diphtheria toxoid, and acellular pertussis (Tdap) vaccine 7 years ago and the influenza vaccine during the last influenza season. Approximately 6 months ago, he received one dose of the measles, mumps, and rubella (MMR) vaccine because of lack of documented immunity on serologic testing. Medical history is negative for chronic medical conditions. He is a nonsmoker, and he does not plan to travel outside of the United States in the near future. He takes no medications. Physical examination is normal. Laboratory studies are significant for a positive result on a hepatitis B surface antibody test. Hepatitis B surface antigen, hepatitis B core antibody, and hepatitis A IgG antibody levels are undetectable. Which of the following is the most appropriate vaccination strategy for this health care worker? Administer a second dose of MMR vaccine Administer the hepatitis A vaccine Administer the hepatitis B vaccine Administer the 23-valent pneumococcal polysaccharide vaccine No vaccination at this time
This patient should receive a second dose of the measles, mumps, and rubella (MMR) vaccine. In all immunocompetent adults who lack documented immunity against measles, mumps, and rubella, at least one dose of the MMR vaccine should be administered. Health care workers born after 1957 are at increased risk for acquiring and transmitting measles, mumps, and rubella and should receive a second dose of the MMR vaccine at least 28 days after the first dose. Although other U.S. adults born prior to 1957 are considered immune to measles, this schedule can also be considered in health care workers born before 1957. A second dose should also be administered to postsecondary students, household contacts of immunocompromised patients, and international travelers. For persons who have been previously vaccinated with two doses of a mumps virus-containing vaccine but are at increased risk because of an outbreak, the Advisory Committee on Immunization Practices recommends administering a third dose of mumps virus-containing vaccine to improve protection. This patient's status as a health care worker does not necessitate administration of the hepatitis A vaccine, despite serologic tests indicating that he lacks immunity. The hepatitis A vaccine should be administered to patients who are at increased risk for infection or complications of infection, such as those who work or travel to endemic areas, men who have sex with men, individuals with chronic liver disease, individuals with HIV, users of illicit drugs, patients experiencing homelessness, persons with occupational risk (for example, those who conduct hepatitis A-related research), household or close contacts of adopted children from endemic areas, and those with potential occupational exposure (such as those who work in group homes, nonresidential day-care facilities for developmentally disabled persons, and health care programs serving users of injection or noninjection drugs). Vaccination of individuals at high risk of infection or complications of infection should also be considered during outbreaks of hepatitis A. Hepatitis A vaccination is also indicated in persons who desire vaccination and could be administered to this patient if he wishes. Hepatitis B vaccination is indicated in all health care workers who lack immunity. This patient has a positive hepatitis B surface antibody test result, whereas his surface antigen and core antibody levels are undetectable. This pattern is consistent with prior vaccination with an appropriate immune response. As such, hepatitis B vaccination is unnecessary in this patient. Pneumococcal vaccination is recommended in all adults aged 65 years and older and adults aged 19 to 64 years with certain high-risk conditions. This patient does not have any chronic medical conditions and is also a nonsmoker; therefore, vaccination with the 23-valent pneumococcal polysaccharide vaccine is not indicated.
A 39-year-old woman is evaluated in the emergency department for a 3-day history of worsening imbalance, falling, and vertigo. Multiple sclerosis was diagnosed 5 years ago and has been treated with interferon beta-1a since that time. She also takes gabapentin for neuropathic pain. On physical examination, temperature is 37.3 °C (99.1 °F); all other vital signs are normal. Internuclear ophthalmoplegia is noted on the left. The patient exhibits imbalance in her primary gait and is unable to perform tandem gait or to maintain balance during Romberg testing when she opens her eyes. Which of the following is the most appropriate immediate treatment? High-dose oral prednisone Low-dose oral prednisone Plasmapheresis Vitamin D administration
This patient should receive high-dose oral prednisone, 1250 mg/d for 5 days. She is experiencing an acute exacerbation, or relapse, of multiple sclerosis (MS). Administration of high-dose glucocorticoids is the standard treatment for MS exacerbations. This treatment approach derives from the landmark Optic Neuritis Treatment Trial, in which high-dose intravenous methylprednisolone (1 g/d for 5 days) was compared to prednisone (1 mg/kg/d for 2 weeks) and placebo. Intravenous methylprednisolone was associated with a more rapid recovery of visual function. Subsequent studies have shown that use of a bioequivalent oral high-dose regimen (such as oral prednisone, 1250 mg, or oral methylprednisolone, 1 g) is as efficacious as intravenous methylprednisolone and has no significant differences in adverse effects. Because of their ease of administration and reduced costs, high-dose oral regimens are beginning to replace intravenous methylprednisolone, although either approach is valid. Low-dose oral prednisone, 1 mg/kg/d for 2 weeks, is inappropriate. In fact, low-dose oral prednisone was inferior to high-dose intravenous glucocorticoids and actually resulted in worse outcomes than placebo in the Optic Neuritis Treatment Trial. Although this dose of oral prednisone is used for some other neuroinflammatory conditions (such as Bell palsy), it potentially can harm patients with MS experiencing a flare and should be avoided. A randomized control trial has documented that plasmapheresis may result in clinical improvement in patients with acute exacerbations of MS who do not respond to glucocorticoid therapy. Plasmapheresis is not considered primary, or first-line, therapy for patients with an acute relapse of MS. The links between vitamin D deficiency and MS pathophysiology have been clearly established, with reduced levels of serum vitamin D predicting future accumulation of new lesions on MRI. Administration of vitamin D supplementation has been shown to provide additional disease activity control, with a recent trial finding less MRI activity in patients taking interferon plus vitamin D versus interferon alone. However, there is no role for vitamin D in the management of an acute relapse of MS. Chronic supplementation with vitamin D is reasonable for most patients with MS, especially if vitamin D levels are low.
A 78-year-old woman is evaluated for knee pain that occurs when walking, ascending and descending stairs, and getting up from a seated position. She was diagnosed with bilateral knee osteoarthritis 10 years ago. She has tried acetaminophen, up to 3000 mg/d in divided doses, without significant relief. Previous examinations have documented bony hypertrophy and crepitus on range of motion of the knees with weakness of the quadriceps. She is scheduled to begin physical therapy. Topical diclofenac is prescribed. The patient should be warned about which of the following possible side effects? Eye and mucous membrane irritation Localized lipoatrophy Localized skin hypopigmentation Localized skin rash
Topical NSAIDs are recommended to treat osteoarthritis in patients aged 75 years or older because they provide similar pain relief as oral medications with greater gastrointestinal safety and tolerability, but with an increased incidence of skin irritation and rash. The patient should be warned about localized skin rash as a possible side effect of topical NSAIDs recently prescribed for her knee osteoarthritis (OA). Topical NSAIDs such as diclofenac (available as a solution, spray, gel, or patch) provide similar pain relief as oral medications with fewer gastrointestinal effects. The American College of Rheumatology currently recommends topical NSAIDs rather than oral NSAIDs for patients aged 75 years or older because topical NSAIDs are considered to provide similar OA pain relief as oral medications, with greater gastrointestinal safety and tolerability. However, there are slightly more skin reactions (rash, itch, and burning) than placebo (rate ratio, 1.14 [95% CI, 0.51-2.55]). Topical diclofenac 1% gel may be less prone to produce local skin reactions compared with the solution, but it may be slightly less effective in reducing pain. Simple analgesics are neither anti-inflammatory nor disease modifying but can help relieve pain in patients with arthritis. Topical analgesics (such as capsaicin and lidocaine) may be useful and can limit systemic drug exposure when only a single area is painful. Topical capsaicin is safe to use in the treatment of OA, but it can be associated with local irritation of the eyes and mucous membranes when drug residue is transferred from the fingers to these areas. Intra-articular glucocorticoids can reduce OA knee pain within days to weeks. When a joint effusion is present, glucocorticoid injections can be particularly helpful after drainage of the excessive joint fluid. Local side effects include skin hypopigmentation, subcutaneous tissue atrophy, and joint infection.
A 55-year-old woman is evaluated in the hospital after diagnosis of pulmonary embolism. She was admitted to the hospital 3 days ago for evaluation of a brain mass with imaging features characteristic of glioblastoma. Twenty-four hours after brain biopsy, the patient developed chest pain and dyspnea. Imaging confirmed a filling defect in the right pulmonary artery. On physical examination, blood pressure is 110/60 mm Hg, pulse rate is 100/min, respiration rate is 18/min, and oxygen saturation is 93% with the patient receiving 2 L/min of nasal oxygen. Cardiopulmonary examination shows mild tachycardia but is otherwise normal. Right upper and lower facial numbness and right arm weakness and numbness are noted. Biceps, triceps, and brachioradialis reflexes are increased on the right, as is right-sided hyperreflexia. An extensor plantar response is present on the right. Results of laboratory studies show a serum creatinine level of 1.7 mg/dL (150 µmol/L). Which of the following is the most appropriate treatment? Apixaban Inferior vena cava filter placement Intravenous alteplase Intravenous heparin Subcutaneous low-molecular-weight heparin
This patient should receive intravenous heparin. Venous thromboembolism (VTE) is a common complication in patients with brain tumors, occurring in up to 30% of patients with high-grade glioma. Risk of VTE is correlated with higher-grade malignancies and is associated with release of the potent procoagulant tissue factor. Other factors, including immobilization and recent surgery, increase the risk. The risk of intracranial hemorrhage with the use of anticoagulants complicates the management of VTE in patients with brain tumor, including patients undergoing brain surgery. Although brain tumors have a risk of hemorrhage, therapeutic anticoagulation is generally considered safe. Intravenous heparin is the best choice for this patient because it has a short half-life and is reversible should hemorrhage occur. Although evidence does not support routine use of preanticoagulation neuroimaging to assess for hemorrhage, noncontrast head CT can be considered and is the most cost-effective test in this situation. Apixaban is not indicated. Although non-vitamin K oral anticoagulants are a recommended therapy for VTE, evidence is insufficient to support their use in the setting of central nervous system tumors. Inferior vena cava (IVC) filter placement is not indicated. IVC filters incur an increased risk of subsequent deep venous thrombosis and should be reserved for patients with an absolute contraindication to anticoagulation. The presence of a primary brain tumor and recent biopsy do not absolutely preclude anticoagulation. Alteplase, a tissue plasminogen activator, is contraindicated in most CNS tumors. This class of drugs can be considered in low-risk tumors, such as meningioma, but not high-risk tumors, such as glioblastoma multiforme. Subcutaneous low-molecular-weight heparin (LMWH) is an effective treatment for VTE but should be avoided in this patient because of her kidney dysfunction. LMWH also lacks an effective reversal agent and has a prolonged duration of action, making it a less optimal choice in a patient at increased risk of bleeding. Read Related TextNext Question
A 51-year-old woman is evaluated for a 1-year history of daily afternoon fatigue that necessitates frequent naps and impairs her concentration at the office, where she works as a lawyer. Lifestyle adjustments, such as improving sleep hygiene, getting regular exercise, yoga, and vitamin supplementation have not resolved this symptom. She has a 5-year history of multiple sclerosis. Medications are glatiramer acetate and a vitamin D supplement. On physical examination, vital signs are normal. Depression screening is negative. The remainder of the physical examination is noncontributory. Results of laboratory studies, including hemoglobin and serum thyroid-stimulating hormone levels, are unremarkable. Which of the following is the most appropriate management? Baclofen Memantine Modafinil Substitution of an interferon beta for the glatiramer acetate Tetrahydrocannabinol-cannabidiol combination
This patient should receive modafinil. Chronic fatigue is a common symptom in multiple sclerosis (MS). The fatigue associated with MS can have various causes, such as depression, insomnia, or other comorbid conditions. However, patients with MS without these conditions also can experience significant fatigue, which is often described as a sensation of mental exhaustion, frequently occurring in the midafternoon. Lifestyle adjustments, such as improving sleep hygiene, getting regular exercise, and treating depression, can sometimes resolve this symptom. For those with refractory fatigue, stimulant medications can be used. The most common medications of this type used (off-label) in MS are modafinil, armodafinil, and amantadine. For fatigue that is refractory to these medications, amphetamine stimulants, such as methylphenidate, also can be considered. Spasticity is a frequent consequence of damage to the corticospinal tract in MS. This symptom manifests clinically as increased muscle tone, painful muscle cramps, spasms, and contractures. Spasticity can be reduced by using muscle relaxants, such as baclofen, tizanidine, or cyclobenzaprine. Antispasticity drugs, such as baclofen, are not effective agents for the management of fatigue. Memantine has been evaluated as a means of treating MS-related cognitive deficits but has proved ineffective for this purpose. There is no reported benefit for MS-related fatigue with memantine. Substituting an interferon beta for the glatiramer acetate would not be an appropriate step. Her fatigue is not an adverse effect of glatiramer acetate (which might necessitate a therapeutic switch), but rather a treatable symptom of an MS relapse. Cannabinoids, such as tetrahydrocannabinol-cannabidiol, have been tested in clinical trials for a number of MS-related symptoms but have not improved outcomes. Trials have included the treatment of disease progression, spasticity, pain, and muscle stiffness. There are no data on the effect of cannabinoids on fatigue.
A 27-year-old man is evaluated during a routine follow-up visit for obesity. On his previous two visits, he had a BMI of 38. Despite enrollment in a 6-month comprehensive supervised weight loss program that included nutritional counseling, he has gained 2.3 kg (5.0 lb). He briefly tried orlistat but discontinued this medication because of gastrointestinal side effects. Medical history is significant for type 2 diabetes mellitus and hypertension. Medications are liraglutide, lisinopril, and metformin. On physical examination, blood pressure is 146/91 mm Hg; other vital signs are normal. BMI is 39. The remainder of the physical examination is unremarkable. Laboratory studies reveal a hemoglobin A1c value of 8.2%. Which of the following is the most appropriate management? Bariatric surgery Lorcaserin Referral to a dietician Very-low-calorie diet
This patient with obesity-related comorbid conditions and no weight loss success after a trial of comprehensive lifestyle modifications meets the criteria for referral for bariatric surgery. Bariatric surgery should be considered for patients who do not lose weight with lifestyle modifications, with or without pharmacologic therapy, and have a BMI of 40 or greater or a BMI of 35 or greater with obesity-related comorbid conditions, such as type 2 diabetes mellitus, coronary artery disease, obstructive sleep apnea, or osteoarthritis. Studies comparing bariatric surgery with nonsurgical treatment (diet, exercise, behavioral modification, and medications) have shown that participants randomly assigned to bariatric surgery lost more weight and were more likely to experience remission of type 2 diabetes and metabolic syndrome, improved quality of life, and reduced medication use. Evidence suggests that bariatric surgery is also associated with reduced mortality and improvement of obstructive sleep apnea, osteoarthritis, and other conditions. In 2020, lorcaserin, which was used as adjunctive therapy to comprehensive lifestyle modification in the treatment of overweight and obesity, was withdrawn from the market for safety reasons. Behavioral therapy for obesity may include the use of a trained interventionist, such as a nutritionist or dietician. This patient is already participating in a comprehensive behavioral therapy plan that includes nutritional counseling; a separate referral to a dietician is unnecessary. Behavioral therapy is also less likely than bariatric surgery to be successful in this patient. Very-low-calorie diets are recommended when rapid weight loss is medically indicated. Their use requires close medical supervision with frequent office visits and laboratory monitoring. In this patient, there is no indication for rapid weight loss that would warrant the use of a very-low-calorie diet.
A 44-year-old man is evaluated in the office during a routine visit. Medical history is significant for HIV diagnosed at age 25 years, hypertension, and hyperlipidemia. He is a current smoker. Medications are chlorthalidone, tenofovir-emtricitabine, and raltegravir. On physical examination, the patient is afebrile, and blood pressure is 126/74 mm Hg. Cardiac examination reveals a regular rate and rhythm. S1 and S2 are normal; there is an S4. Laboratory tests are significant for a fasting plasma glucose level of 98 mg/dL (5.43 mmol/L), a total cholesterol level of 210 mg/dL (5.43 mmol/L), and an HDL cholesterol level of 50 mg/dL (1.29 mmol/L). An electrocardiogram shows normal sinus rhythm and left ventricular hypertrophy with repolarization abnormalities. A chest radiograph is normal. To determine his need for statin therapy, his estimated 10-year risk for atherosclerotic cardiovascular disease using the Pooled Cohort Equations will be calculated. Which of the following risk factors will result in underestimation of the risk for atherosclerotic cardiovascular disease in this patient? Age Antihypertensive medication use Blood pressure HDL cholesterol level HIV status
This patient's HIV status will contribute most to the underestimation of his risk for atherosclerotic cardiovascular disease (ASCVD). This young patient has both traditional risk factors (hypertension and smoking) and a nontraditional risk factor (HIV) for ASCVD, which is defined as coronary death or nonfatal myocardial infarction, or fatal or nonfatal stroke. The Pooled Cohort Equations are a risk assessment instrument developed from multiple community-based cohorts; their use as a primary risk assessment tool is recommended in the 2018 American Heart Association (AHA)/American College of Cardiology (ACC) Guideline on the Management of Blood Cholesterol. The AHA/ACC ASCVD risk calculator based on the Pooled Cohort Equations includes age, sex, race, total and HDL cholesterol levels, systolic blood pressure, blood pressure-lowering medication use, presence of diabetes mellitus, and smoking status. Calculated 10-year ASCVD risk can be classified as low (<5%), borderline (5% to <7.5%), intermediate (≥7.5% to <20%), or high (≥20%). This patient's calculated 10-year ASCVD risk of 6.2% is categorized as borderline. However, this patient has HIV infection, an ASCVD risk-enhancing factor. According to the AHA/ACC guideline, patients with borderline risk plus risk enhancers should be engaged in a discussion about initiating moderate-intensity statin therapy. Large observational studies have demonstrated a 1.5- to 2-fold increase in the risk for ASCVD among patients with HIV infection. The increased risk for ASCVD in HIV-infected patients is likely the result of interactions among the viral infection, host factors, traditional risk factors, and therapies for HIV. Traditional risk models, including the AHA/ACC ASCVD risk calculator based on the Pooled Cohort Equations, underestimate the risk for ASCVD in patients with HIV. Alternative risk models, including one based on the D:A:D (Data Collection on Adverse events of Anti-HIV Drugs) study, have been developed, but they lack validation and have not been widely adopted. This patient's age, blood pressure, use of antihypertensive medications, and HDL cholesterol level are accounted for in the AHA/ACC ASCVD risk calculator and are not factors responsible for underestimating his ASCVD risk.
A 23-year-old woman is evaluated for a 2-week history of persistent thick, white vaginal discharge; burning in the vulvar and vaginal regions; and vaginal itching. She has never had these symptoms before. She is in a monogamous sexual relationship. Medical history is otherwise unremarkable, and she takes no medications. On physical examination, vital signs are normal. Pelvic examination reveals vulvar edema with a few excoriations. Speculum examination demonstrates thick, white, curdy vaginal discharge. The remainder of the examination is unremarkable. Laboratory studies reveal a vaginal pH of 4.4; whiff test result is negative. Potassium hydroxide microscopy shows hyphae. Results of tests for Chlamydia trachomatis and Neisseria gonorrhoeae are negative. Which of the following is the most appropriate treatment? Intravaginal clotrimazole Intravaginal nystatin Oral metronidazole Oral voriconazole
Topical antifungal imidazole therapy, such as intravaginal clotrimazole, is an effective treatment for uncomplicated vulvovaginal candidiasis, which is usually caused by Candida albicans. The most appropriate treatment for this patient with symptoms of vulvovaginal candidiasis is intravaginal clotrimazole. Vulvovaginitis describes infectious and noninfectious conditions that cause vulvovaginal symptoms, including abnormal vaginal discharge, vulvar itching, burning, irritation, and malodor. When discharge is associated with abnormal findings, the differential diagnoses most commonly include bacterial vaginosis, trichomoniasis, and vulvovaginal candidiasis. Vaginal irritation also may be caused by dermatologic conditions or allergic reactions, cervical infections, or genitourinary syndrome of menopause. A woman may have more than one type of infection at a time. The diagnosis of vulvovaginal candidiasis is suggested by the presence of vaginal discharge and vulvar pruritus, pain, irritation, and redness. Signs include vulvar edema; fissures; excoriations; and thick, white, curdy vaginal discharge. The diagnosis can be made when a saline or 10% potassium hydroxide wet mount of vaginal discharge shows hyphae, pseudohyphae, or yeast. Because the sensitivity of microscopy is low, empiric treatment of vulvovaginal candidiasis can be considered if symptoms are accompanied by characteristic findings. Several therapeutically equivalent topical and oral drugs are available; among the topically applied drugs, imidazoles (fluconazole, miconazole, clotrimazole) are the most effective. Evidence suggests that topical and oral agents have similar efficacy and that treatment preference should be based on cost, convenience, and patient preference. Intravaginal nystatin, a topical antifungal drug, lacks good supporting evidence for the treatment of vulvovaginal candidiasis and currently is not recommended. Oral metronidazole is used to treat bacterial vaginosis, the most common cause of vaginal discharge, as well as to treat trichomoniasis. Accepted clinical criteria for diagnosing bacterial vaginosis include the presence of three of four characteristics: vaginal pH greater than 4.5, amine ("fishy") odor on the application of 10% potassium hydroxide to vaginal secretions (whiff test), the presence of a thin homogeneous vaginal discharge, and the finding of at least 20% clue cells on a microscopic saline wet mount examination. Although the presentation of trichomoniasis varies, many women develop a copious, malodorous, pale yellow or gray frothy discharge with vulvar itching, burning, and postcoital bleeding. Point-of-care vaginal swab rapid immunoassays and nucleic acid amplification tests for detection of Trichomoniasis vaginalis have replaced microscopy or culture as the gold standard for diagnosis. This patient does not have findings of bacterial vaginosis or trichomoniasis; hence, metronidazole is not indicated. Oral voriconazole should not be used to treat vulvovaginal candidiasis because no data support its use. Case reports suggest that it is ineffective, and there is the added potential for toxicity. Read Related TextNext Question
A 75-year-old woman is hospitalized for a 1-week history of dizziness, nausea, vomiting, increased urination, and decreased appetite. History is significant for hypertension treated with hydrochlorothiazide. She also takes calcium carbonate for bone health. On physical examination, blood pressure is 150/85 mm Hg supine and 122/70 mm Hg standing, pulse rate is 78/min supine and 100/min standing, and respiration rate is 18/min. There is no neck vein distention. Cardiac, pulmonary, and abdominal examinations are unremarkable. There is no lower extremity edema. Laboratory studies: Hematocrit 30% Leukocyte count 3000/μL (3.0 × 109/L) Platelet count 82,000/μL (82 × 109/L) Calcium 12.8 mg/dL (3.2 mmol/L) Creatinine 3.7 mg/dL (327.1 µmol/L) Electrolytes : Sodium 132 mEq/L (132 mmol/L) Potassium 4.9 mEq/L (4.9 mmol/L) Chloride 115 mEq/L (115 mmol/L) Bicarbonate 17 mEq/L (17 mmol/L) Phosphorus 6.2 mg/dL (2.0 mmol/L) Urine sodium 15 mEq/L (15 mmol/L) Urinalysis Specific gravity 1.018; trace protein; few erythrocytes/hpf; occasional leukocytes/hpf; few granular casts; numerous hyaline casts Which of the following is the most likely cause of this patient's hypercalcemia and acute kidney injury? Hydrochlorothiazide therapy Milk alkali syndrome Multiple myeloma Primary hyperparathyroidism
This patient's acute kidney injury (AKI) is due to hypercalcemia from multiple myeloma. Classic symptoms of polyuria, polydipsia, and nocturia sometimes occur with elevated serum calcium levels of 11 mg/dL (2.8 mmol/L) or less. Other symptoms such as anorexia, nausea, abdominal pain, constipation, increased serum creatinine levels, and mild mental status changes are more likely to occur with levels >11 mg/dL (2.8 mmol/L). Kidney dysfunction is found in about 30% of patients diagnosed with multiple myeloma, often due to cast nephropathy (also termed myeloma kidney), a condition in which excess monoclonal free light chains precipitate in the distal tubules and incite tubulointerstitial damage. Hypercalcemia and exposure to nephrotoxic agents are other frequent causes of kidney dysfunction. Hypercalcemia can decrease glomerular filtration rate through renal vasoconstriction, the natriuretic effects of high serum calcium levels, and impaired renal concentrating ability. This patient has orthostatic hypotension, a bland urinalysis with hyaline casts, and a low urinary sodium, consistent with a prerenal AKI from hypovolemia. The constellation of hypercalcemia, normal anion gap metabolic acidosis, pancytopenia, and AKI suggests multiple myeloma as the etiology. Hydrochlorothiazide can cause volume depletion, decreased excretion of calcium and mild hypercalcemia, prerenal AKI, and metabolic alkalosis (due to hypovolemic stimulation of aldosterone release). However, the effect on serum calcium is usually minimal, and the patient has a metabolic acidosis, not metabolic alkalosis as might be seen with thiazide therapy. Milk alkali syndrome occurs with the ingestion of large amounts of calcium and absorbable alkali (for example, calcium carbonate). It presents as hypercalcemia, metabolic alkalosis, and AKI. This patient has a metabolic acidosis, not a metabolic alkalosis, making this diagnosis unlikely. Primary hyperparathyroidism is the most common cause of hypercalcemia in otherwise healthy outpatients and is diagnosed with a simultaneously elevated serum calcium level and an inappropriately normal or elevated intact parathyroid hormone level. Serum phosphorus levels are typically low or low-normal in these patients. This patient's phosphorus level is elevated, making the diagnosis of primary hyperparathyroidism unlikely. Read Related TextNext Question
A 68-year-old man is evaluated for fever, perineal pain, dysuria, frequency, and intermittent straining that began yesterday. Symptoms began 48 hours after a prostate biopsy due to an elevated prostate-specific antigen level detected during routine screening. On physical examination, temperature is 38.7 °C (101.7 °F), blood pressure is 145/82 mm Hg, pulse rate is 105/min, and respiration rate is normal. The prostate is enlarged and boggy, and it is tender to gentle palpation. There is no penile discharge, and no scrotal pain occurs with palpation. Dipstick urinalysis is positive for leukocyte esterase and nitrates. Urine Gram stain reveals gram-negative rods. Urine culture is pending. Which of the following is the most appropriate treatment? Amoxicillin Ceftriaxone and doxycycline Cephalexin Trimethoprim-sulfamethoxazole
This patient's history and physical examination findings indicate acute bacterial prostatitis, and the most appropriate treatment regimen is trimethoprim-sulfamethoxazole. Patient groups at high risk for acute bacterial prostatitis include those with diabetes mellitus, immunosuppression, or cirrhosis. Risk factors include unprotected sexual intercourse, urogenital instrumentation (chronic indwelling bladder catheterization, intermittent bladder catheterization, prostate biopsy), urinary tract manipulation (prostate resection), urinary stasis (obstruction), and benign prostatic hyperplasia. The most common infectious cause for acute bacterial prostatitis is Escherichia coli or other gram-negative bacilli. Diagnosis is typically established with urine Gram stain and culture in patients with a compatible history. The treatment of choice for acute bacterial prostatitis is a prolonged course of trimethoprim-sulfamethoxazole or ciprofloxacin. Data on treatment duration are sparse, but 6 weeks is reasonable and recommended by experts. Given the prolonged duration of antimicrobial therapy required in cases of acute bacterial prostatitis, it is most prudent to select an antibiotic with appropriate coverage, while also attempting to minimize the potential for serious adverse effects. Prolonged ciprofloxacin use has been associated with QT prolongation as well as tendinopathy/tendon rupture, especially in older adults. As such, given the treatment duration needed for acute bacterial prostatitis, trimethoprim-sulfamethoxazole would be the most appropriate choice. Amoxicillin would be an appropriate choice for patients with acute prostatitis and gram-positive cocci in chains. This finding would suggest an enterococcal infection, and treatment with amoxicillin or ampicillin would be appropriate. Neither of these antibiotics would be effective for acute bacterial prostatitis caused by E. coli or other gram-negative bacilli. Men younger than 35 years who are sexually active and men older than 35 years who engage in high-risk sexual behavior should be treated with regimens that cover Neisseria gonorrhoeae and Chlamydia trachomatis. Ceftriaxone and doxycycline, or ceftriaxone and azithromycin, would be appropriate treatment choices in cases of acute epididymitis, specifically targeting C. trachomatis or N. gonorrhoeae. Neither regimen would be appropriate in this case. Cephalexin would be an appropriate choice for patients with acute prostatitis and gram-positive cocci in clusters suggesting infection with Staphylococcus aureus or coagulase-negative staphylococci (Staphylococcus epidermidis or Staphylococcus saprophyticus).
When to refer to ENT for tinnitus?
This patient's tinnitus is gradual in onset and requires further evaluation. However, the patient does not need urgent referral to an otolaryngologist. Patients should be urgently referred when tinnitus is associated with symptoms suggesting serious, reversible underlying pathology, including sudden sensorineural hearing loss, pulsatile tinnitus, vestibular symptoms, ear pain, or drainage or malodor that fails to resolve.
A 57-year-old man is evaluated during a routine visit. History is significant for hypertension. Medications are hydrochlorothiazide, 25 mg/d, and amlodipine, 5 mg/d. On physical examination, blood pressure is 135/86 mm Hg, and pulse rate is 70/min; other vital signs are normal. There is 1+ bilateral ankle edema. The remainder of the examination is normal. Laboratory studies show a serum creatinine level of 1.0 mg/dL (88.4 µmol/L), a serum potassium level of 3.6 mEq/L (3.6 mmol/L), and an estimated glomerular filtration rate >60 mL/min/1.73 m2. Which of the following is the most appropriate treatment? Add hydralazine Add losartan Double the amlodipine dose Double the hydrochlorothiazide dose
Three strategies can be used for antihypertensive dose adjustment in the treatment of hypertension: (1) maximize the medication dose before adding another; (2) add another class of medication before reaching the maximum dose of the first; and (3) start with two medication classes separately or as fixed-dose combinations. The addition of losartan is the most appropriate treatment. As antihypertensive agents are titrated or added when there is inadequate blood pressure control, it is important to recognize that there is a nonlinear and diminishing blood pressure-lowering effect when titrating from 50% maximal dose to 100% maximal dose of any agent. A general rule of thumb is that 75% of an agent's blood pressure-lowering effect may be achieved with 50% of its maximal dose. If blood pressure control requires an additional >5-mm Hg reduction, it is unlikely to be achieved by increasing the single agent from 50% to 100% maximal dose. The better strategy is to add a second drug or a third drug to a two-drug regimen, as seen in this patient. Hydralazine is not the best option because it is a thrice-daily medication and may pose problems with adherence, considering that once-daily medication options have not been exhausted in this patient. In addition, hydralazine is a direct vasodilator and is associated with sodium and water retention and reflex tachycardia; use with a diuretic and a β-blocker is recommended. Hydralazine is typically reserved for patients with resistant hypertension or hypertensive urgencies. Increasing the amlodipine or hydrochlorothiazide dose is not appropriate because there is diminishing return in blood pressure lowering if the dose is titrated up from 50% to 100% of maximum. Also, it is unlikely that increasing the dose from 50% to 100% of maximum will result in an additional >5-mm Hg reduction to bring this patient's blood pressure to a target of <130/80 mm Hg. In addition, titration to maximum doses may result in manifestation of undesirable side effects and decreased adherence. In this case, the patient already has dependent edema, which may increase with uptitration of the amlodipine dose. His serum potassium is borderline at 3.6 mEq/L (3.6 mmol/L), and increasing the dose of hydrochlorothiazide may further decrease his serum potassium and necessitate a potassium supplement, adding yet another medication to his drug regimen.
A 66-year-old woman was admitted to the hospital 24 hours ago with community-acquired pneumonia. Since admission, she has been confused and her oral intake has been poor. Appropriate antibiotics, intravenous fluids, and oxygen have been initiated. She has no other known medical problems. On physical examination, temperature is 39 °C (102.2 °F), blood pressure is 142/88 mm Hg, pulse rate is 98/min, and respiration rate is 20/min. Oxygen saturation is 98% on oxygen, 2 L/min by nasal cannula. Crackles are evident in the right posterior thorax. Laboratory studies show glucose values of 185 to 215 mg/dL (10.3-11.9 mmol/L) and a hemoglobin A1c level is 5.5%. A chest radiograph demonstrates a right lower lobe infiltrate. Which of the following is the most appropriate management of this patient's hyperglycemia? Empagliflozin and sliding-scale insulin Metformin and sliding-scale insulin Scheduled basal insulin and correction insulin Sliding-scale insulin only
To manage in-patient hyperglycemia, scheduled basal insulin or basal insulin plus correction insulin is appropriate for patients who are fasting or who have poor oral intake. The most appropriate management of this patient's hyperglycemia is to initiate scheduled basal insulin and correction insulin. Inpatient hyperglycemia, defined as consistently elevated plasma glucose values above 140 mg/dL (7.8 mmol/L), is associated with poor outcomes. Attempts to decrease morbidity and mortality with tight glycemic control (80-110 mg/dL [4.4-6.1 mmol/L]) have not consistently demonstrated improvements in adverse outcomes and, in some settings, have shown increased rates of severe hypoglycemic events and mortality. As a result, revised inpatient glycemic targets are less stringent than outpatient glucose targets to avoid both hypoglycemia and severe hyperglycemia that can lead to volume depletion and electrolyte abnormalities. Dietary modifications should be made once glucose levels exceed 140 mg/dL (7.8 mmol/L). At persistent glucose levels of 180 mg/dL (10.0 mmol/L) and higher, the American Diabetes Association recommends initiation of scheduled insulin with a blood glucose target of 140 to 180 mg/dL (7.8-10.0 mmol/L) for most critically ill and noncritically ill patients to decrease the risk of adverse outcomes. Scheduled basal insulin or basal insulin plus correction insulin is appropriate for patients who are fasting or who have poor oral intake, such as this patient, with frequent bedside point-of-care monitoring every 4 to 6 hours for insulin adjustments. Scheduled basal and prandial insulin plus correction insulin are appropriate for patients who are eating. The safety of oral antihyperglycemic agents, including empagliflozin, in the hospital setting has not been fully studied or established. In addition, sodium-glucose transporter-2 (SGLT2) inhibitors have been associated with diabetic ketoacidosis and should be avoided in situations that may produce ketone bodies, such as severe illness or prolonged fasting. Scheduled insulin therapy is the recommended treatment regimen for hyperglycemia in the hospital setting. The safety of oral antihyperglycemic agents, including metformin, in the hospital setting has not been fully studied or established. Scheduled insulin therapy is the recommended treatment regimen for hyperglycemia in the hospital setting. The sole use of correction insulin for the management of hyperglycemia is not recommended. It is a reactive approach to hyperglycemia that can lead to large fluctuations in glucose levels coupled with the near universal lag time between measurement of glucose and injection of insulin that occurs in most hospitals.
An 85-year-old woman is evaluated for left knee pain of moderate intensity that occurs with ambulation and at night. Acetaminophen provides no relief. History is significant for coronary artery disease, hypertension, and stage 3 chronic kidney disease. Medications are aspirin, lisinopril, metoprolol, and vitamin D. On physical examination, blood pressure is 152/88 mm Hg; other vital signs are normal. The left knee demonstrates crepitus and decreased passive range of motion; a small effusion is present, and there is no warmth or tenderness. Joint aspiration of the left knee shows a leukocyte count of 1100/µL (1.1 × 109/L). Which of the following is the most appropriate treatment for long-term symptom control? Intra-articular glucocorticoid injections Low-dose prednisone Naproxen Oxycodone Topical diclofenac
Topical diclofenac is the most appropriate treatment for long-term symptom control for this elderly patient with knee osteoarthritis (OA). Acetaminophen has not provided relief; furthermore, the efficacy of acetaminophen for OA is increasingly being questioned because recent controlled trials and meta-analyses demonstrated no benefit from the drug, even at high doses. NSAIDs are available in oral, topical, and intravenous forms. Due to efficacy and cost-effectiveness, oral preparations are usually first-line NSAID therapy in patients without contraindications to treatment. In OA, topical NSAIDs are considered to provide similar pain relief as oral medications with fewer gastrointestinal effects. Furthermore, the American College of Rheumatology currently recommends topical NSAIDs rather than oral NSAIDs for patients aged 75 years or older. However, they are associated with more skin reactions and are significantly more expensive than oral NSAIDs. This elderly woman with unilateral knee OA is an ideal candidate for topical NSAIDs—she has just one area of musculoskeletal pain and has several relative contraindications to oral NSAIDs (age, chronic kidney disease, coronary artery disease, and aspirin intake). When a single symptomatic joint is present, injection directly into the joint may deliver medication to the affected site while minimizing the potential for systemic effects. Intra-articular injections may be used along with or in place of oral or topical analgesics. Intra-articular glucocorticoids are associated with short-term benefit with few side effects. Glucocorticoid injection alone would not likely provide long-term benefit for this patient. Low-dose oral prednisone is associated with modest reductions in short-term knee pain and may increase a 6-minute walk distance in older patients with moderate-to-severe knee OA. However, long-term use is associated with significant complications, including an increased risk for diabetes mellitus, osteoporosis, osteonecrosis, weight gain, fluid retention, hypertension, cardiovascular disease, striae and bruising, and glaucoma and cataracts. Oral naproxen and other oral NSAIDs are relatively contraindicated given the patient's hypertension, chronic kidney disease, and coronary artery disease. Traditional opiates may rarely be warranted to control pain in patients with OA who have not responded to other agents or are poor candidates for other interventions to treat painful joints, such as surgery. Opioid analgesics are inappropriate for this patient because other treatment options are available. In addition, opioids have limited effectiveness for chronic joint pain and are associated with substantial side effects in the elderly, especially increased fall risk, cognitive changes, and constipation. Read Related TextNext Question
A 32-year-old man is evaluated for ongoing intermittent premature ejaculation of 3 years' duration, which he describes as distressing to him and his spouse. He reports no other symptoms. The patient also has depression, which is currently well controlled with amitriptyline. A previous trial of sertraline caused mood instability, prompting its discontinuation. He takes no other medications. On physical examination, vital signs are normal. Growth and pattern of body hair and testicular size are normal. No gynecomastia is noted. Laboratory studies show a normal 8:00 AM serum total testosterone level. Which of the following is the most appropriate additional treatment? Paroxetine Testosterone gel Topical lidocaine No additional treatment is indicated
Topical lidocaine is the most appropriate treatment for this patient's premature ejaculation. Premature ejaculation is defined as ejaculation that occurs sooner than desired and is distressful to either or both partners. The mainstays of therapy include counseling and pharmacotherapy (oral and topical agents). Pharmacologic therapy is the first-line approach, with the combination of a selective serotonin reuptake inhibitor (SSRI) and a phosphodiesterase-5 (PDE-5) inhibitor being most effective; other options, in descending order of efficacy, include topical anesthetics, SSRI or PDE-5 inhibitor monotherapy, and clomipramine. This patient is currently taking a tricyclic antidepressant and has had previous mood instability with an SSRI; therefore, the most appropriate treatment is a regimen of topical lidocaine to help reduce tactile stimulation and thus prolong the time to ejaculation. Topical medications (lidocaine, prilocaine) may be used with or without a condom. Paroxetine therapy is an effective treatment strategy for premature ejaculation. However, it is not appropriate in this case because of the patient's previously reported mood instability when exposed to sertraline. Both sertraline and paroxetine are SSRIs, which can cause deleterious mood changes (a class effect). In a patient with depression that is currently well controlled, adding an additional psychoactive medication is not warranted. Testosterone gel, which is used in the treatment of hypogonadism, is not appropriate for this patient. Hypogonadism (androgen deficiency) can lead to decreased libido and erectile dysfunction, but it has not been shown to be a causative or correlative factor in premature ejaculation. Furthermore, this patient has no examination findings that would raise concern for hypogonadism, such as body hair growth and pattern changes, reduced testicular size, or gynecomastia, and his 8:00 AM serum total testosterone level was normal. Given that the patient's premature ejaculation is distressing to both the patient and his spouse, offering no treatment options could lead to worsening self-confidence, mood, and quality of life. Read Related TextNext Question
A 42-year-old woman is evaluated in the emergency department for right flank pain of 3 hours' duration. History is significant for migraines. There is no family history of kidney stones. Medications are as-needed sumatriptan and daily topiramate. On physical examination, right costovertebral angle tenderness is present. Laboratory studies: Creatinine 0.8 mg/dL (70.7 µmol/L) Electrolytes : Sodium 138 mEq/L (138 mmol/L) Potassium 3.5 mEq/L (3.5 mmol/L) Chloride 104 mEq/L (104 mmol/L) Bicarbonate 21 mEq/L (21 mmol/L) Urinalysis Specific gravity 1.005; pH 6.5; 1+ blood; negative leukocyte esterase; negative nitrites; 20-30 erythrocytes/hpf; 1-3 leukocytes/hpf; amorphous crystals Noncontrast helical CT scan shows a 5-mm stone in the right proximal ureter. Which of the following is the most likely composition of this patient's kidney stone? Calcium oxalate Calcium phosphate Cystine Struvite Uric acid
Topiramate, a carbonic anhydrase inhibitor, causes a decrease in urinary citrate excretion and formation of alkaline urine that favor the creation of calcium phosphate stones. The most likely composition of this patient's kidney stone is calcium phospate. Approximately 80% of kidney stones contain calcium oxalate, calcium phosphate, or both. Calcium stones are radiopaque on plain radiograph. Hypercalciuria, hyperoxaluria, and hypocitraturia are risk factors for calcium stones. Calcium phosphate stones occur when there is persistently elevated urine pH. These stones are therefore commonly associated with distal renal tubular acidosis and hyperparathyroidism. This patient is taking topiramate for migraine prophylaxis, a carbonic anhydrase inhibitor that is associated with calcium phosphate stones. Carbonic anhydrase promotes proximal tubule sodium, bicarbonate, and chloride reabsorption. Inhibitors of carbonic anhydrase produce both sodium chloride and bicarbonate urinary loss. The resultant mild metabolic acidosis causes decreased citrate excretion, and the persistent alkaline urine favors the precipitation of calcium phosphate. Although calcium oxalate is the most common cause of kidney stones, there are no calcium oxalate crystals noted on this patient's urinalysis. The most common crystal formations of calcium oxalate in the urine are the dumbbell-shaped calcium oxalate monohydrate crystals and envelope-shaped calcium oxalate dihydrate crystals. She has amorphous crystals in alkaline urine, which are usually calcium phosphate crystals. Cystine stones occur with cystinuria, which is a genetic disease. This patient has no family history of kidney stones, and the characteristic hexagonal-shaped crystals are not seen on urine microscopy. Struvite stones occur in the presence of urea-splitting bacteria (Proteus, Klebsiella, or, less frequently, Pseudomonas). These bacteria split urea into ammonium, which markedly increases urine pH and results in the precipitation of magnesium ammonium phosphate (struvite). The pH of the urine will be >7.5. Struvite stones commonly produce staghorn calculi (stones that bridge two or more renal calyces) and occur most frequently in older women with chronic urinary infections. Coffin lid-shaped crystals may be seen in the urine. This patient does not demonstrate these findings. Uric acid stones are uncommon (10% of stones), but the incidence increases in hotter, arid climates due to low urine volumes. The main risk factor is low urine pH, which decreases the solubility of uric acid. Hyperuricosuria is not a consistent finding. Comorbid risk factors for uric acid stones include gout, diabetes mellitus, the metabolic syndrome, and chronic diarrhea. Because uric acid stones occur in persistently acidic urine, this is an unlikely diagnosis for this patient.
TRALI Symptoms
Transfusion-related acute lung injury (TRALI) is the leading cause of transfusion-related mortality. It is characterized by the development of acute lung injury within 6 hours of transfusion of erythrocytes, platelets, or fresh frozen plasma. TRALI is mediated by initial priming of neutrophils in the recipient's lung parenchyma followed by their activation by anti-HLA and antineutrophil antibodies present in donor plasma. Signs and symptoms escalate quickly and include dyspnea, hypoxia, fever, chills, and hypotension. The chest radiograph shows diffuse bilateral pulmonary infiltrates. The presence of circulatory overload is not consistent with TRALI. Management of TRALI is supportive.
A 34-year-old transgender woman is evaluated during a routine examination. She desires gender-affirming hormone therapy. Her gender incongruence diagnosis has been made and confirmed by qualified medical providers. She smokes one pack of cigarettes per day, with a 15-pack-year history. Medical history is otherwise unremarkable. She takes no medications. On physical examination, vital signs are normal. She has male hair distribution. Normal male genitalia are present. There are no evident inguinal hernias. In addition to advising smoking cessation, which of the following is the most appropriate next step in management? Initiation of an androgen blocker Initiation of estradiol therapy Refer for gender confirmation surgery consultation Refer for discussion on fertility preservation options Return for treatment 1 year after living in desired gender role
Transgender medicine is the care of persons whose gender identity differs from the sex that was assigned at birth. Gender incongruence is persistent incongruence between gender identity and external sexual anatomy at birth absent of a confounding mental disorder. A transgender man is someone with a male gender identity and a female birth assigned sex; a transgender woman (as in this patient) is someone with a female gender identity and a male birth assigned sex. The most appropriate next step in management is to refer the patient for discussion on fertility preservation options. Gender-affirming hormone therapy is the primary medical intervention sought by transgender people. Criteria for hormone therapy include persistent, well-documented gender dysphoria; capacity to make a fully informed decision; age of majority in a given country; and if present, control of significant medical or psychological conditions. Gender-affirmation hormone therapy limits fertility, thus reproductive options should be discussed with patients prior to initiation of hormone therapy. While feminizing hormone therapy is typically estradiol with an androgen blocker, it would not be appropriate to initiate therapy without first considering its impact on fertility. Additionally, due to the risk of thromboembolic disease with estrogen therapy, smoking cessation must first be undertaken. Gender confirmation surgery is often the last step in the treatment process for gender dysphoria. It is recommended that individuals undergoing irreversible gender-affirming surgery, which affects fertility, engage in at least 1 year of satisfactory social role change as well as consistent and compliant hormone treatment, unless hormone therapy is not desired or medically contraindicated.
A 60-year-old man is evaluated for dry skin and a pruritic rash of 6 months' duration. He is a farmer and has extensive exposure to the sun. The rash is transient, occurring most frequently during the winter and spring when his skin is dry, and is worsened by heat and sweating during the summer. The patient is otherwise well, has no other medical problems, and takes no medications. On physical examination, vital signs are normal. Skin findings on the upper torso are shown. The arms, legs, face, and mucous membranes are unaffected. The remainder of the examination is unremarkable. Which of the following is the most likely diagnosis? Allergic contact dermatitis Atopic dermatitis Lichen planus Transient acantholytic dermatosis
Transient acantholytic dermatosis is characterized by red pruritic papules on the chest, flanks, and back associated with dry skin, heat, and sweating. Transient acantholytic dermatosis, also known as Grover disease, is a benign common eruption that is most often seen in middle-aged to elderly men. The eruption presents as small discrete papules, some of which may be scaly, and papulovesicles on the trunk. Symptomatically there are varying degrees of pruritus. Although the cause and pathogenesis are unknown, a very common association is xerosis. The eruption is frequently triggered by excessive sweating. Skin biopsy shows acantholytic dyskeratosis. Transient acantholytic dermatosis is typically self-limited, but therapy with topical glucocorticoids or moisturizers may be effective. Allergic contact dermatitis is a pruritic eruption of patches and plaques with variable vesiculation. The eruption is found in areas of allergen exposure and typically worsens with subsequent exposures. In exuberant cases, the localized inflammation can lead to a secondary "id" reaction, a generalized acute cutaneous reaction in which pinpoint flesh-colored to red papules develop diffusely on the body. The absence of a known sensitizer and a pruritic rash that is worsened with heat and sweating is not consistent with allergic contact dermatitis. Histologic studies show spongiotic dermatitis with eosinophils. Although atopic dermatitis is pruritic and associated with xerosis, it is clinically distinguished from transient acantholytic dermatosis by the presence of scaly eczematous patches with variable degrees of lichenification. Also, atopic dermatitis in adults appears in the flexures of the extremities rather than the torso. Finally, the prevalence of atopic dermatitis decreases with age, whereas transient acantholytic dermatosis is more common in middle-aged to elderly men. On skin biopsy, there are variable degrees of spongiosis. Lichen planus (LP) is characterized by pruritic, purple, polygonal papules that may coalesce into plaques characteristically involving the ankles and flexor surfaces of the wrist. Fine white lines may be visible on the surface of the papules or plaques. LP can also occur in the mucous membranes (mouth, vaginal vault, and penis) with white plaques that, if uncontrolled, may ulcerate. The eruption can also develop in the nails, leading to thickening and distortion of the nail plate. LP is most commonly idiopathic but may be induced by medications or possibly infection. The purple polygonal papules and plaques of LP are not consistent with this patient's rash. A skin biopsy shows lichenoid dermatitis.
A 26-year-old man is hospitalized for suspected bacterial endocarditis. He has a long history of injection drug use and has used heroin within the past month. Two weeks ago, he began feeling feverish, and over the past 3 days, he has been increasingly short of breath. He takes no medications. On physical examination, temperature is 38.7 °C (101.6 °F), blood pressure is 90/50 mm Hg, pulse rate is 95/min, and respiration rate is 16/min. Bilateral lung crackles are present. Jugular venous distention is noted. A grade 2/6 holosystolic murmur is heard at the apex with an S3. Skin findings are shown. The electrocardiogram is normal. A chest radiograph shows bilateral pulmonary edema and cardiomegaly. Blood cultures are obtained. Which of the following is the most appropriate next step in management? Cardiac CT Chest MRI Transesophageal echocardiography Transthoracic echocardiography
Transthoracic echocardiography is the first-line imaging modality in patients suspected of having infective endocarditis to identify vegetations, determine the severity of valvular lesions, assess ventricular function, and detect complications. The most appropriate next step in management is transthoracic echocardiography (TTE). This patient has symptoms and signs that are suspicious for infective endocarditis. According to the modified Duke criteria for infective endocarditis, he meets three minor criteria (injection drug use, fever with temperature >38 °C [100.4 °F], and vascular phenomena [in this case, erythematous nonpainful macular lesions known as Janeway lesions]), which indicates a possible diagnosis of infective endocarditis. A definitive diagnosis may be made with positive blood cultures and characteristic echocardiographic findings. TTE, as a first-line imaging modality, is indicated to identify vegetations or abscesses (diagnostic for infective endocarditis), determine the severity of valvular lesions, assess ventricular function, and detect complications. The sensitivity of TTE is approximately 75%; therefore, a negative imaging test result does not rule out the presence of infective endocarditis, and other imaging modalities should be used. When the TTE findings are normal or inadequate in patients with a high pretest probability of infective endocarditis, or suspicion remains for undetected complications, further examination with TEE, cardiac CT, or MRI should be considered. The diagnostic sensitivity of TEE for infective endocarditis is approximately 90%. TEE is also superior to TTE in the detection of perivalvular abscesses, leaflet perforation, and other complications. Cardiac CT and chest MRI can be helpful in assessing for the presence of aortic root abscess when TEE is inconclusive, and CT is helpful to identify areas of peripheral embolization, such as in the kidney, spine, spleen, and brain. However, because of cost, accessibility, and invasiveness, TEE, chest MRI, and cardiac CT are not the best initial imaging choices.
A 30-year-old man with a history of alcohol and drug abuse is brought to the emergency department. He was found unresponsive and on arrival he was intubated and placed on mechanical ventilation for poor respiratory effort. Witnesses confirmed that he had been drinking alcohol and had also taken drugs at a party. On physical examination, vital signs are normal. Respiration rate is 14/min on mechanical ventilation. He is not triggering the ventilator. Oxygen saturation is 100% on 35% oxygen. He responds to pain only. His pupils are dilated and reactive to light. He shows no signs of trauma. Blood toxicology screening is positive for alcohol; urine toxicology screening is positive for benzodiazepines. Arterial pH and electrolyte anion gap are normal. Which of the following is the most appropriate management? Administer flumazenil Administer fomepizole Administer hemodialysis Monitor for signs of agitation
Treatment for benzodiazepine overdose is supportive with assurance of adequate ventilation; flumazenil is generally not recommended for benzodiazepine overdose as it can precipitate seizures in chronic users and its short half-life makes it difficult to sustain reversal of long-acting benzodiazepines. This patient should be kept on mechanical ventilation and monitored for signs of agitation. He shows signs of benzodiazepine overdose, combined with alcohol abuse. His airway is secure, and he is easily supported with mechanical ventilation, which can continue until he has metabolized the drug and his mental status has improved. Because of his history of alcohol abuse, he is at risk for alcohol withdrawal. Alcohol withdrawal occurs with chronic heavy alcohol use within hours to days after alcohol cessation. Early withdrawal symptoms occur within a few hours of abstinence and include agitation, anxiety, tremulousness, headache, and symptoms of autonomic hyperactivity (fever, diaphoresis, tachycardia, hypertension). Generalized tonic-clonic seizures may occur usually within 6 to 24 hours and should be treated with benzodiazepines because if left untreated, up to one third of patients may progress to delirium tremens. Flumazenil, a γ-aminobutyric acid (GABA)-receptor antagonist, is the antidote for benzodiazepine toxicity, but reversing the benzodiazepine he took could put him at risk for seizures, especially if he is a chronic user. The short half-life of flumazenil makes it challenging to use in patients requiring sustained reversal of long-acting benzodiazepines, and given the overall low risk of benzodiazepine overdose, it is safer to allow his body to metabolize the benzodiazepine and eliminate it along with the alcohol. Fomepizole inhibits alcohol dehydrogenase. It is used to block the metabolism of ethylene glycol and methanol into toxic metabolites when either of these alcohols is ingested. There is no reason to suspect either agent, especially in a patient with normal blood pH and a normal anion gap. Administration would therefore not be appropriate. Dialysis would not be appropriate because there is no acute indication for this invasive and costly intervention. In principle, hemodialysis is indicated for drug intoxications when the clearance of the drug by hemodialysis is significantly shorter than metabolic clearance and the patient is deteriorating or when measured drug concentrations are predictive of a poor outcome without hemodialysis. Dialysis will remove alcohols effectively, but the patient is stable and does not require an invasive intervention.
A 32-year-old woman is evaluated for a 10-month history of pruritus and scaling of both her hands. She is a child care worker and washes her hands frequently. Medical history is unremarkable, and she takes no medications. On physical examination, vital signs are normal. Skin findings are shown. There is no scale or erythema of the feet. The remainder of the examination is normal. Results of potassium hydroxide microscopy from the scale on her hands are negative. Which of the following is the most appropriate management? Epicutaneous patch testing Oral fluconazole Oral prednisone Thick emollients
Treatment of hand dermatitis includes topical emollients such as petrolatum to repair the skin barrier; hand washing should be minimized. This patient has hand dermatitis likely caused by an irritant contact dermatitis from frequent water exposure, and the most appropriate management would be to minimize hand washing and to apply thick emollients such as petrolatum to repair the skin barrier. Hand dermatitis is characterized by inflamed, scaling, and sometimes fissured skin on the palmar or dorsal hand. The most common causes are over-washing, allergic or irritant contact dermatitis, atopic dermatitis, dyshidrotic eczema (pompholyx), and tinea. Repeated or extended washing with soap causes hand dermatitis by friction, removal of the protective skin barrier, and irritation from the surfactant properties of the soap. People who are required to repeatedly wash their hands (such as medical professionals) or those who wash repeatedly because of obsessive-compulsive or autism spectrum disorders often have extremely pronounced hand dermatitis. This type of irritant dermatitis will be especially marked on the dorsal hands where the stratum corneum is thinner than that of the palms. For irritant hand dermatitis due to over-washing, avoiding the irritant by washing less and moisturizing more can prevent development of a rash. Topical petrolatum jelly is an inexpensive and effective way of repairing the damaged skin barrier. A topical glucocorticoid may be necessary for a short period while the triggers are identified or if the skin is very inflamed. Epicutaneous patch testing can be used to help determine causes of hand dermatitis due to allergic contact dermatitis. In this patient, the most likely cause is hand dermatitis from frequent hand washing, so patch testing would not be the most appropriate management. It could be considered if there was no improvement after minimizing hand washing and the application of emollients. Oral fluconazole is a treatment for fungal infections such as candidiasis. It can be effective in cases of tinea manuum. "Two feet, one hand" tinea is a common presentation of concomitant tinea pedis and tinea manuum. In this patient, the potassium hydroxide was negative and her feet were clear, so treatment with antifungal therapy would be inappropriate. Oral prednisone can improve acute flares of hand dermatitis but often results in relapses of chronic hand dermatitis. Because there are also significant side effects of oral prednisone, it would not be the most appropriate initial treatment for this patient.
A 30-year-old woman is evaluated for a 3-month history of pain and swelling of the proximal interphalangeal (PIP) joints. She has difficulty performing tasks such as gripping or typing. Naproxen provides minimal relief. On physical examination, vital signs are normal. There are no rashes or oral ulcers. Swelling and tenderness of the PIP joints bilaterally are noted. Pain occurs with flexion of the fingers. There is blanching coloration of the fingers and cyanosis at the fingertips. Nailfold examination shows a few dilated capillary loops. There are no cardiopulmonary friction rubs. The remainder of the examination is normal. Laboratory studies performed 1 month ago showed the following: a positive antinuclear antibody titer of 1:320 with a speckled pattern; negative anti-double-stranded DNA antibodies, anti-Smith antibodies, anti-U1-ribonucleoprotein antibodies, anti-Ro/SSA antibodies, anti-La/SSB antibodies, anti-cyclic citrullinated peptide antibodies, and rheumatoid factor; and normal complete blood count with differential, serum creatinine, and urinalysis. Radiographs of the hands are normal. Which of the following is the most appropriate management? Add hydroxychloroquine Discontinue naproxen; begin ibuprofen Obtain plain radiography of the feet Repeat anti-double-stranded DNA antibodies Repeat antinuclear antibodies
Treatment of undifferentiated connective tissue disease is the same as other connective tissue diseases and is based on the manifestations of the individual patient. The addition of hydroxychloroquine is the most appropriate treatment for this patient's arthritis symptoms. This patient has undifferentiated connective tissue disease (UCTD), a term used to describe an autoimmune disease that has clinical manifestations of other specific connective tissue diseases, but not enough positive features to satisfy diagnostic or classification criteria for any one disease. She has Raynaud phenomenon, inflammatory arthritis, and positive antinuclear antibodies, but does not have enough clinical findings to establish a more specific diagnosis. Over time, her condition may evolve into a specific connective tissue disease, most commonly systemic lupus erythematosus. Management of UCTD is the same as other connective tissue diseases and is based on the manifestations of the individual patient. This patient has not responded to a trial of an NSAID and has impaired activities of daily living; thus, she is a candidate for hydroxychloroquine. Discontinuing naproxen and starting ibuprofen is not indicated because ibuprofen is not necessarily more effective than naproxen. Obtaining radiography of the feet in a patient without symptoms or physical findings in the feet is unlikely to show any abnormality and thus will not provide any useful diagnostic information. Repeating anti-double-stranded DNA antibody testing is not appropriate because the results are unlikely to have changed over a 1-month period. Repeat testing could be considered in the future if the patient's clinical condition changes. Repeating antinuclear antibody testing will not provide further insight into the diagnosis, and the titer does not correlate with disease activity.
An 85-year-old woman is evaluated for declining function. According to her son, her memory has been failing, her gait has slowed, she has fallen four times, and she often cries without apparent reason. He further reports that he does most of her cooking and has assumed responsibility for her finances. The patient had a right basal ganglia infarct 5 years ago and left cerebellar infarct 2 years ago. She also has atrial fibrillation and hypertension. Medications are rivaroxaban, hydrochlorothiazide, and metoprolol. On physical examination, blood pressure is 155/80 mm Hg in both arms, and pulse rate is irregularly irregular at 90/min; other vital signs are normal. Cardiac examination shows findings consistent with atrial fibrillation. All other physical examination findings are normal. The patient says she has not been depressed but becomes tearful when discussing her medications. Depression screening has normal findings. She scores 20/30 (normal, ≥26) on the Montreal Cognitive Assessment. Bradykinesia with normal strength is noted in the legs bilaterally, as are increased deep tendon reflexes and an extensor plantar response on the left. Gait examination reveals a shuffling pattern. Results of laboratory studies show normal thyroid-stimulating hormone and vitamin B12 levels. An MRI shows diffuse white matter hyperintensities bilaterally and lacunar infarcts in the bilateral basal ganglia and left cerebellum. Which of the following is most appropriate for this patient's cognitive impairment? Citalopram Donepezil Ginkgo biloba Methylphenidate
Treatment of vascular cognitive impairment should focus on identifying and treating cerebrovascular risk factors; off-label use of acetylcholinesterase inhibitors has shown modest benefit in clinical trials and thus is generally recommended for this condition. The most appropriate medication for this patient's cognitive impairment is the acetylcholinesterase inhibitor donepezil. Given her history of multiple strokes, pseudobulbar affect (a neurologic disorder characterized by involuntary outbursts of laughing and/or crying that are out of proportion to the emotions being experienced), prominent gait problems, and asymmetric neurologic findings, she most likely has vascular cognitive impairment. Treatment of vascular cognitive impairment should focus on identifying and treating cerebrovascular risk factors, such as smoking, diabetes mellitus, hyperlipidemia, hypertension, ischemic heart disease, atrial fibrillation, and hypercoagulable states. Although not yet FDA approved as treatment of vascular cognitive impairment, acetylcholinesterase inhibitors have shown modest benefit in clinical trials and are generally recommended for this type of dementia. Citalopram and other selective serotonin reuptake inhibitors are appropriate to treat the pseudobulbar symptoms exhibited by this patient but are unlikely to improve her cognitive deficits. Ginkgo biloba has not been found to be more effective for dementia than placebo in controlled trials. Because this herbal supplement also is associated with increased bleeding, it is contraindicated in patients taking anticoagulant or antiplatelet agents. Methylphenidate has been shown to be effective in treating the severe apathy often associated with vascular cognitive impairment. This patient has no evidence of apathy. In addition, this agent can raise blood pressure and is inappropriate for this patient with hypertension.
An 82-year-old woman is evaluated for a 3-year history of chronic discomfort and stiffness in the hands, wrists, and shoulders. Some of these joints intermittently become acutely swollen and warm. She currently has 3 days of severe left wrist pain and swelling. She previously had been diagnosed with seronegative rheumatoid arthritis; there was no improvement with sulfasalazine or intravenous infliximab, both of which were discontinued. Short courses of prednisone have alleviated the acute attacks of arthritis. History is also significant for hypertension and stage 3 chronic kidney disease. Medications are hydrochlorothiazide, metoprolol, and losartan. On physical examination, vital signs are normal. Decreased range of motion of both wrists and a few metacarpophalangeal (MCP) joints is noted, and shoulder abduction is limited to 90 degrees passively. The left wrist is swollen and tender. There are no subcutaneous nodules at the elbows, hands, or heels. Laboratory studies: Erythrocyte sedimentation rate 51 mm/h Urate 4.3 mg/dL (0.25 mmol/L) Rheumatoid factor 15 U/mL (15 kU/L) Anti-cyclic citrullinated peptide antibodies Negative Radiographs show osteoarthritic changes at the second through fourth MCP joints bilaterally, both wrists, and glenohumeral joints. Chondrocalcinosis is seen in the knees and both wrists. There are no periarticular erosions in the wrists or hands. Which of the following is the most appropriate treatment? Adalimumab Allopurinol Low-dose prednisone Methotrexate NSAID therapy
Treatment options for chronic calcium pyrophosphate arthropathy include low-dose glucocorticoids, low-dose colchicine, or NSAIDs to prevent inflammatory manifestations of the disease. Low-dose prednisone is the most appropriate treatment for this patient with chronic calcium pyrophosphate arthropathy, which may be present as two patterns: chronic calcium pyrophosphate (CPP) crystal inflammatory arthritis, and osteoarthritis with calcium pyrophosphate deposition (CPPD). Chronic CPP crystal inflammatory arthritis is a rare polyarthritis involving the wrists and metacarpophalangeal joints ("pseudo-rheumatoid arthritis"), with the absence of serologies associated with rheumatoid arthritis. The course may be marked by episodes of acute pseudogout, radiographic chondrocalcinosis, and lack of chronic inflammation on examination. Osteoarthritis with CPPD manifests as typical osteoarthritic findings involving joints not commonly associated with osteoarthritis (such as shoulders or metacarpophalangeal joints). Radiographic findings of osteoarthritis with CPPD (subchondral bone cysts, osteophytes, and subchondral sclerosis, consistent with osteoarthritis) often precede the onset of osteoarthritis. This patient presents with findings on examination and radiograph consistent with both patterns of chronic calcium pyrophosphate arthropathy. Treatment of chronic calcium pyrophosphate arthropathy can be difficult because there is a paucity of data. Options include low-dose glucocorticoids, low-dose colchicine, or NSAIDs to prevent inflammatory manifestations of the disease. For this patient, low-dose daily prednisone is appropriate. Tumor necrosis factor (TNF)-α inhibitors have not been investigated as a treatment for chronic calcium pyrophosphate arthropathy, and beginning a TNF-α inhibitor such as adalimumab in not indicated. In this patient, the complete absence of response to one TNF-α inhibitor is another clue that rheumatoid arthritis was an incorrect diagnosis. Allopurinol is a urate-lowering therapy, but calcium pyrophosphate deposition results from calcium pyrophosphate rather than uric acid deposition. The radiographic presence of chondrocalcinosis and osteoarthritic changes, as well as the absence of classic gouty erosions characterized by punched-out lesions with overhanging edges, argue against the diagnosis of chronic tophaceous gout. Finally, a low serum urate level and the absence of tophi also argue against the diagnosis of chronic tophaceous gout. Methotrexate was recently found to be no better than placebo in a randomized trial for chronic calcium pyrophosphate arthropathy. Although daily NSAID therapy can be a reasonable option, this patient is an elderly woman with hypertension and chronic kidney disease, making NSAIDs relatively contraindicated.
A 40-year-old man is evaluated during a follow-up visit for a kidney transplant he received 2 years ago. History is also significant for hypertension. Medications are tacrolimus, mycophenolate mofetil, prednisone, and nifedipine. On physical examination, blood pressure is 150/95 mm Hg; other vital signs are normal. BMI is 26. The cardiovascular and pulmonary examinations are normal. The abdomen and renal allograft are nontender to palpation. Trace pedal edema is noted. Laboratory studies: Potassium 5.6 mEq/L (5.6 mmol/L) Sodium Normal Estimated glomerular filtration rate 90 mL/min/1.73 m2 Duplex ultrasound of the kidneys shows no evidence of transplant renal artery stenosis. Which of the following is the most appropriate treatment? Chlorthalidone Fludrocortisone Sodium polystyrene sulfonate Spironolactone
Treatment using thiazide diuretics is appropriate for calcineurin inhibitor-induced hypertension and hyperkalemia in kidney transplant recipients. The most appropriate treatment for calcineurin inhibitor-induced hypertension and hyperkalemia in this kidney transplant recipient is a thiazide or thiazide-like diuretic such as chlorthalidone. Patients with kidney transplants must receive immunosuppressive medications to prevent their immune system from rejecting the kidney allograft. Doses are typically highest immediately after transplant and are tapered gradually over several months to minimize toxicities associated with these medications while maintaining adequate immunosuppression. The most commonly used immunosuppressants in the immediate posttransplant period for immunosuppression induction are anti-T-cell and interleukin-2 receptor-blocking antibodies. The most commonly prescribed medications for chronic maintenance immunosuppression include calcineurin inhibitors (tacrolimus or cyclosporine), antimetabolites (mycophenolate mofetil or azathioprine), and glucocorticoids. Although these medications are usually well tolerated, they can have significant side effects. Calcineurin inhibitors activate the sodium chloride cotransporter in the distal convoluted tubule, which reabsorbs sodium and chloride to cause hypertension. Decreased distal tubular flow impairs potassium secretion in the connecting tubule and collecting duct, leading to hyperkalemia. Calcineurin inhibitor-induced hypertension and hyperkalemia share the same phenotype as Gordon syndrome (familial hyperkalemic hypertension), which is due to a dysregulation of the WNK kinases in the distal convoluted tubule. Thiazide and thiazide-like diuretics such as chlorthalidone address the underlying mechanism of calcineurin inhibitor-induced hypertension and hyperkalemia by inhibiting the sodium chloride cotransporter. Although fludrocortisone, a synthetic mineralocorticoid, would treat the hyperkalemia, this medication would exacerbate sodium retention and raise blood pressure in this patient with already uncontrolled hypertension. Sodium polystyrene sulfonate would also treat the hyperkalemia but provides a significant sodium load with each dose and may also raise the blood pressure. These medications do not address the underlying mechanism and are therefore not indicated. Spironolactone may lower the blood pressure but would raise serum potassium and is therefore contraindicated in this patient who is already hyperkalemic.
A 73-year-old man is evaluated for a 3-month history of chronic productive cough, intermittent hemoptysis, night sweats, and 4.5 kg (10 lb) unintentional weight loss. He previously worked as a miner and has a history of chronic silicosis. He also has a 40-pack-year history of smoking, but quit 10 years ago. On physical examination, vital signs are normal. He is thin and appears ill with temporal muscle wasting. Lung examination reveals bilateral upper-lobe crackles. Cardiac examination is unremarkable. Chest radiograph reveals bilateral upper-lobe fibrosis with volume loss of the upper lobes and evidence of cavitation, traction of the hila upwards bilaterally, and bilateral calcified hilar lymphadenopathy. Which of the following is the most appropriate management? Aspergillus IgG antibody test Bronchoscopy with transbronchial biopsy High-resolution CT scan of the chest Sputum sample for acid-fast bacillus
Tuberculosis should be strongly considered when a patient with silicosis develops constitutional symptoms, worsening respiratory impairment, hemoptysis, or changes in the chest radiograph, particularly cavities. The most appropriate management is a sputum sample for acid-fast bacillus. This patient has constitutional symptoms that include night sweats, unintentional weight loss, hemoptysis, and upper-lobe cavitary disease on chest radiograph. Reactivation tuberculosis most commonly involves the apical-posterior segments of the upper lobe; cavitation is present in up to 40% of cases. Tuberculosis should be strongly considered when a patient with silicosis develops constitutional symptoms, worsening respiratory impairment, hemoptysis, or changes in the chest radiograph, particularly cavities. Chronic silicosis adversely affects macrophage function and is clearly associated with the development of infection with tuberculosis. Concomitant silicosis and tuberculosis is associated with a substantially increased risk of mortality. Therefore, a high index of suspicion for this complication of chronic silicosis is essential to ensure early and appropriate medical therapy. Aspergilloma is generally a consequence of colonization of a preexisting pulmonary cavity or cyst or in areas of devitalized lung. Symptoms include cough, hemoptysis, dyspnea, weight loss, fever, fatigue, and chest pain. Radiographic images show a round mass within a pulmonary cavity or cyst. Sputum cultures or IgG antibody are usually positive. This patient's chest radiograph is not consistent with an aspergilloma, and aspergillosis IgG antibody testing is not necessary. Mine workers are often exposed to radon and several kinds of dust, and they have high rates of tobacco use; however, the development of silicosis is also an independent risk factor for the development of lung cancer. Nevertheless, bronchoscopy with transbronchial biopsy would not be appropriate for this patient. Although he has a substantially increased risk of lung cancer, the chest radiograph does not clearly demonstrate a target for biopsy. Furthermore, pursuing bronchoscopy without first ensuring there is not an active tuberculosis infection potentially places health care workers at risk for infection. A high-resolution CT scan of the chest may identify additional findings not visible on the chest radiograph, but it will not change the need to rule out tuberculosis, which is a common comorbidity of chronic silicosis.
A 64-year-old man is evaluated for a 2-month history of increasing fatigue and bilateral swelling of the submandibular region. History is significant for autoimmune pancreatitis treated with prednisone 2 years ago, hypertension, and allergic rhinitis. Medications are losartan and fluticasone propionate. On physical examination, blood pressure is 148/84 mm Hg, and pulse rate is 78/min. There is no rash. Head and neck examination reveals bilateral submandibular gland swelling. Trace edema of the ankles is present. The remainder of the examination is normal. Laboratory studies: Hemoglobin 12 g/dL (120 g/L) Leukocyte count 10,000/µL (10 × 109/L); 33% eosinophils Platelet count 180,000/µL (180 × 109/L) C3 65 mg/dL (650 mg/L) C4 7 mg/dL (70 mg/L) Creatinine 3.1 mg/dL (274 µmol/L); 6 months ago: 1.8 mg/dL (159.1 µmol/L) IgG 2600 mg/dL (26 g/L) IgE 500 U/mL (500 kU/L) Antinuclear antibodies 1:640 Urinalysis Specific gravity 1.010; trace protein; 6-10 leukocytes/hpf Kidney ultrasound demonstrates bilateral markedly enlarged kidneys measuring 15 cm in size with hyperechoic cortex and peripheral cortical nodules. Which of the following is the most likely diagnosis? IgG4-related disease Lupus nephritis Sarcoidosis Sjögren syndrome
Tubulointerstitial nephritis is the most common kidney manifestation of IgG4-related disease and typically presents with pyuria, proteinuria, and elevated serum IgG and IgE levels; kidney imaging may show enlarged kidneys or renal masses. The patient's decline in kidney function is most likely due to IgG4-related disease, a syndrome that mostly affects middle-aged or older men. Plasma cell-rich tubulointerstitial nephritis (TIN) is the most common kidney manifestation. Other manifestations can include autoimmune pancreatitis, allergic rhinitis, submandibular gland swelling, an elevated antinuclear antibody (ANA) titer, low serum complement levels, elevated serum IgG, elevated serum IgE, and peripheral eosinophilia. Kidney imaging may show enlarged kidneys or renal masses that may present as small peripheral cortical nodules. This patient's urinalysis findings of trace protein and pyuria with progressive decline of kidney function are consistent with TIN. Definitive diagnosis of IgG4-related disease requires a tissue biopsy to demonstrate an infiltrate with IgG4-positive plasma cells. IgG4-related disease is readily treatable with glucocorticoids. Although elevated ANA titers and hypocomplementemia are seen in lupus, lupus nephritis is usually characterized by proteinuria and a more active urine sediment (dysmorphic erythrocytes and cellular casts). Furthermore, lupus is more common in women and usually manifests at a much earlier age than in this patient. Sarcoidosis can affect multiple organs and cause TIN. However, sarcoidosis is not associated with autoimmune pancreatitis, hypocomplementemia, elevated serum immunoglobulin levels, or cortical nodules on kidney ultrasound. Although Sjögren syndrome can affect lacrimal and salivary glands and cause TIN, this patient's constellation of symptoms is more consistent with IgG4-related disease. Dryness of the eyes and mouth is extremely common in Sjögren but not in IgG4-related disease. Allergy symptoms, such as asthma or allergic rhinitis, are rarely seen in Sjögren but are common in IgG4-related disease. Hypocomplementemia and the presence of cortical nodules on kidney ultrasound also favor the diagnosis of IgG4-related disease.
Difference between FIT and FOBT?
Two fecal blood detection tests are available: a sensitive guaiac-based fecal occult blood test (gFOBT) and an FIT that uses antibodies to detect human hemoglobin. Sensitive gFOBT requires dietary restriction in order to reduce false-positive results, whereas FIT does not. The FDA has approved a third stool-based screening test that is combined with FIT and detects cancer DNA in the stool (the multitargeted stool DNA test). Mortality data for this screening strategy are not available. Because this patient would prefer not to modify his diet, FIT is the most appropriate screening option.
Options for noninvasive colon cancer screening?
Two fecal blood detection tests are available: a sensitive guaiac-based fecal occult blood test (gFOBT) and an FIT that uses antibodies to detect human hemoglobin. Sensitive gFOBT requires dietary restriction in order to reduce false-positive results, whereas FIT does not. The FDA has approved a third stool-based screening test that is combined with FIT and detects cancer DNA in the stool (the multitargeted stool DNA test). Mortality data for this screening strategy are not available. Because this patient would prefer not to modify his diet, FIT is the most appropriate screening option. The plasma circulating methylated SEPT9 DNA test is an FDA-approved colorectal cancer screening test that holds promise, as blood tests may result in increased screening adherence. However, its sensitivity for detecting colorectal cancer is suboptimal at 48%, and mortality data are lacking.
Type I vs Type II Hepatorenal syndrome?
Type 1 is characterized by acute kidney dysfunction and is usually triggered by a precipitating event such as spontaneous bacterial peritonitis, other infections, gastrointestinal hemorrhage, or a major surgical procedure. Type 2 is more common and is characterized by more slowly progressive kidney failure in patients with refractory ascites. Type 1 hepatorenal syndrome is characterized by a rise in serum creatinine of at least 0.3 mg/dL (26 μmol/L) and/or ≥50% from baseline within 48 hours with a bland urinalysis and normal findings on renal ultrasonography. It is also supported by a lack of improvement in kidney function after withdrawal of diuretics and two days of volume expansion with intravenous albumin. Often patients with hepatorenal syndrome also have low urine sodium, low fractional excretion of sodium, and oliguria. In addition, patients should have no evidence of shock, no current or recent use of nephrotoxic drugs, and no evidence of renal parenchymal disease (proteinuria less than 0.5 g/day, no microhematuria, and normal renal ultrasound). The main treatment of hepatorenal syndrome is the removal of drugs that may reduce kidney perfusion and volume expansion. Ultimately, hepatorenal syndrome is a condition for which the only cure is liver transplantation.
A 68-year-old man is evaluated in the office for increasing shortness of breath, palpitations, difficulty sleeping, fatigue, generalized weakness, and 4.5-kg (10-lb) weight loss over the past month. His medical history is significant for heart failure and atrial fibrillation. For the past 2 years his medications have been metoprolol, lisinopril, amiodarone, and dabigatran. On physical examination, the patient is afebrile, blood pressure is 140/80 mm Hg, pulse is 102/min, and respiration rate is 24/min. Deep tendon reflexes are brisk and symmetric. A fine tremor of his outstretched hands, bilateral lid lag, and an irregularly irregular heart rhythm are noted. Examination of the thyroid gland and remainder of the physical examination are normal. Laboratory studies show a serum thyroid-stimulating hormone (TSH) level less than 0.01 µU/mL (0.01 mU/L), a free thyroxine (T4) level of 3.1 ng/dL (40 pmol/L), and a serum total triiodothyronine (T3) level of 190 ng/dL (2.9 nmol/L). TSH receptor antibodies are undetectable. Other laboratory studies are normal. On thyroid ultrasound, the thyroid lobes and isthmus are normal in size. No thyroid nodules are seen. The background thyroid parenchyma demonstrates no demonstrable vascularity on color flow Doppler. Chest radiograph is normal. In addition to increasing the metoprolol dose, which of the following is the most appropriate initial management? Discontinue amiodarone Begin methimazole Begin prednisone Thyroid scintigraphy with radioactive iodine uptake
Type 2 amiodarone-induced thyrotoxicosis (destructive thyroiditis) can be treated with moderate- to high-dose prednisone that can be gradually tapered over 1 to 3 months. The most appropriate initial management for this patient is to prescribe prednisone. Amiodarone is an antiarrhythmic medication with high iodine content and prolonged half-life of approximately 40 days. Thyrotoxicosis affects 5% of patients taking amiodarone and can occur at any time during or up to 9 months after treatment. Type 1 (hyperthyroidism) amiodarone-induced thyrotoxicosis occurs in patients with underlying multinodular goiter or latent Graves disease and is associated with increased vascularity on color flow Doppler ultrasonography. Type 2 (destructive thyroiditis) usually affects those without thyroid disease and is not associated with increased vascularity on color flow Doppler. Mixed forms can also be seen and making the correct diagnosis can be difficult. The patient's clinical presentation is most consistent with type 2 amiodarone-induced thyrotoxicosis given the absence of structural thyroid disease (no nodules or goiter), absent thyroid-stimulating hormone (TSH) receptor antibody, and absent parenchymal flow seen on Doppler ultrasound. Moderate- to high-dose prednisone is an effective treatment that can be gradually tapered over 1 to 3 months. Discontinuation of amiodarone would not yield any immediate clinical benefit due to its prolonged half-life elimination. The decision to discontinue amiodarone depends on the patient's cardiac status, availability of effective alternatives, and type of thyrotoxicosis, with treatment cessation being more important in type 1 than type 2. Methimazole is most effective in treating type 1 (hyperthyroidism) amiodarone-induced thyrotoxicosis, which occurs in patients with Graves disease or thyroid nodules. Since this patient's presentation is most consistent with type 2 amiodarone-induced thyrotoxicosis (destructive thyroiditis), prednisone is the preferred treatment. Performing thyroid scintigraphy with radioactive iodine uptake is not indicated. Amiodarone has a very high iodine content, which results in high serum iodine levels. This iodine competes with the radioactive isotope used for the test (123I or 131I) resulting in very low radioactive iodine uptake (<1%) in most patients. This test does not discriminate well between type 1 and 2 amiodarone-induced thyrotoxicosis and is not clinically useful.
A 72-year-old man is evaluated in the hospital after developing acute kidney injury 2 days following coronary artery bypass grafting. He is currently on mechanical ventilation and requires vasopressors for hypotension. He underwent coronary angiography 12 hours prior to surgery. The serum creatinine has increased from 0.8 mg/dL (70.7 µmol/L) at baseline to 2.2 mg/dL (194.5 µmol/L), and urine output has decreased to 350 mL/24 h. History is significant for type 2 diabetes mellitus and coronary artery disease. Current medications are intravenous furosemide, insulin, propofol, fentanyl, and norepinephrine. On physical examination, the patient is intubated and mechanically ventilated. A urinary catheter is in place. Temperature is 37.9 °C (100.2 °F), blood pressure is 98/60 mm Hg, pulse rate is 105/min, respiration rate is 28/min, and oxygen saturation is 96% on 30% FIO2. There is no rash. Decreased breath sounds are heard in the lung bases. The remainder of the examination is noncontributory. Which of the following is the most appropriate test to perform next? Examination of urine sediment Fractional excretion of sodium Kidney ultrasonography Measurement of central venous pressure
UA (for muddy casts) or FeUrea (because on lasix) The most appropriate next step is examination of the urine sediment for cells and casts. This patient has developed oliguric acute kidney injury (AKI). He has multiple risk factors for AKI, including type 2 diabetes mellitus, recent coronary arteriography, hypotension, and cardiac surgery. The main consideration is whether the AKI is due to renal hypoperfusion (prerenal AKI) or whether the AKI is due to acute tubular necrosis (ATN). In the setting of diuretics, the fractional excretion of urea (FEUrea) is more accurate than the fractional excretion of sodium because urea excretion is not promoted by diuretics and is still retained in volume-depleted states. FEUrea is calculated using the equation: FEUrea = (UUrea × PCreatinine)/(UCreatinine × PUrea) × 100% FEUrea <35% is suggestive of a prerenal state. The presence of granular casts and/or renal epithelial cells has strong predictive value for ATN. Urine microscopy can also help differentiate other causes of AKI, such as acute interstitial nephritis and glomerulonephritis. The fractional excretion of sodium (FENa) is calculated as follows: FENa = (USodium × PCreatinine)/(UCreatinine × PSodium) × 100% FENa <1% indicates prerenal AKI but can also be seen in contrast-induced nephropathy, acute interstitial nephritis, rhabdomyolysis, glomerulonephritis, and early obstruction because the tubular handling of sodium is intact in these conditions. FENa >2% indicates ATN but is also seen in patients who are prerenal and are on diuretics or who have adrenal insufficiency, bicarbonaturia, or chronic kidney disease due to the increased distal tubular delivery of sodium in these conditions. Kidney ultrasonography should be performed to rule out obstructive uropathy as the cause of AKI. However, the patient already has a urinary catheter, so the likelihood of obstruction as the cause of AKI is low. Central venous pressure is not a reliable indicator of organ perfusion and would not be able to differentiate prerenal AKI from ATN or other causes of AKI. Read Related TextNext Question
A 65-year-old woman comes to the office to establish care. Her medical history is notable for hypothyroidism due to Hashimoto thyroiditis treated with levothyroxine. She does not have any symptoms at this time. There is no history of head or neck radiation exposure. On physical examination, vital signs are normal. The patient's thyroid gland is enlarged. The right lobe is larger than the left, and a mobile 2-cm nodule is palpable in the lower pole. There is no palpable cervical adenopathy. Laboratory studies show a serum thyroid-stimulating hormone level of 2.0 µU/mL (2.0 mU/L). Which of the following is the most appropriate diagnostic test to perform next? CT scan of the neck Fine-needle aspiration biopsy of the thyroid nodule Thyroid uptake and 131I scan Ultrasound of the neck
Ultrasound can confirm the presence of thyroid nodules palpated on examination and based on findings can help to determine if fine-needle aspiration is needed to assess for malignancy. The most appropriate diagnostic test to perform next is ultrasound of the neck. Ultrasound can confirm the presence of thyroid nodules palpated on examination and those detected on other imaging studies. Ultrasound must be performed prior to fine-needle aspiration biopsy (FNAB) to confirm the presence of a nodule, determine that biopsy is indicated, ensure that there are no additional nonpalpable nodules that warrant FNAB, and assess the cervical lymph nodes. In patients with solitary palpable nodules, 15% will have no corresponding nodule on ultrasound, and a similar proportion will have an additional nodule measuring 1 cm or larger. Performing a CT scan of the neck is a more costly test, exposes the patient to unnecessary radiation, and is inferior to ultrasound at assessing the thyroid gland. FNAB should not be performed prior to thyroid/neck ultrasound. Whether or not FNAB is indicated depends on the size and sonographic appearance of the nodule, clinical risk factors for malignancy, and presence of pathologic lymph nodes. Nodules that are predominantly cystic or posteriorly located within the thyroid gland are prone to sampling error. Measurement of serum thyroid-stimulating hormone (TSH) is also part of the initial evaluation of a thyroid nodule. The purpose of measuring TSH is to evaluate for the presence of autonomously functioning or "hot" nodules, which account for 5% to 10% of palpable thyroid nodules. Autonomous nodules may cause hyperthyroidism and are associated with a very low risk of malignancy. Autonomous nodules can be confirmed by performing a thyroid uptake and scan. They concentrate radioactive iodine to a greater extent than normal thyroid tissue, which shows absent or diminished uptake. The TSH in this patient is normal, which does not support a diagnosis of an autonomously functioning thyroid nodule; therefore performing a thyroid uptake and 131I scan is not indicated.
A 48-year-old man is evaluated for increasing dysphagia during the past 2 months. He reports that he first noticed difficulty swallowing solid foods, such as steak, but that this symptom has advanced to occasional difficulty with liquids. He is, however, maintaining adequate hydration and nutrition. He is not currently taking medications. On physical examination, vital signs are normal. There is a 1.5-cm left supraclavicular node and a liver edge that is palpable below the right costal margin. A CT scan of the chest and abdomen shows a mass in the lower third of the esophagus just proximal to the gastroesophageal junction. The liver has multiple hypodense lesions consistent with metastases. Endoscopic evaluation reveals a near-obstructing lesion in the distal esophagus. Biopsy shows adenocarcinoma. Which of the following is the most appropriate diagnostic test to perform next? Assess tumor tissue for BRAF mutation status Assess tumor tissue for HER2 amplification Assess tumor tissue for RAS mutation status Obtain PET/CT scan Obtain upper gastrointestinal imaging (barium swallow)
Upper gastrointestinal tumors should be assessed for human epidermal growth factor receptor 2 (HER2) amplification to guide chemotherapy. This patient's tumor tissue should be assessed for human epidermal growth factor receptor 2 (HER2) amplification. He has metastatic adenocarcinoma of the distal esophagus. Approximately 25% of gastroesophageal cancers demonstrate HER2 amplification; such tumors achieve a superior response to chemotherapy when the anti-HER2 monoclonal antibody trastuzumab is added to standard chemotherapy. For this reason, determination of HER2 amplification status is necessary when initiating chemotherapy for patients with a gastroesophageal tumor. BRAF mutation status is of paramount importance in patients with melanoma, 40% of whom will have BRAF-mutated tumors that are highly likely to respond to tyrosine kinase inhibitors targeting BRAF. However, such mutations are exceedingly rare in upper gastrointestinal malignancies, and BRAF-targeting agents have thus far failed to show activity in BRAF-mutated gastrointestinal malignancies. RAS mutation status is relevant to the use of anti-epidermal growth factor receptor (EGFR) monoclonal antibodies in colorectal cancer. These agents have not been shown to be active in upper gastrointestinal malignancies and are not part of the treatment paradigm. Neither a PET/CT scan nor a barium swallow will add useful information for this patient because the available CT imaging is sufficient for establishing a treatment plan. Any further information regarding extent of disease that might be found on a PET/CT would not be expected to change management. A barium swallow is unlikely to provide clinically useful information about the extent of his tumor or additional information about his swallowing capacity.
A 47-year-old woman is evaluated for a 6-month history of disequilibrium. She often feels "dizzy" and "off-balance," which has led to several falls. She does not experience vertigo. She has a 10-year history of fibromyalgia, and has experienced a 60% improvement in pain and function with graded aerobic exercise, duloxetine, and pregabalin. On a focused physical examination, vital signs are normal. Neurologic examination, including strength testing, sensory examination (light touch, proprioception), tandem walking, and Romberg testing, is normal. Laboratory studies show normal serum folate, thyroid-stimulating hormone, and vitamin B12 levels; hemoglobin A1c level is 5.0%. Which of the following is the most appropriate next step in management? Discontinue duloxetine Discontinue pregabalin Measure methylmalonic acid and homocysteine levels Order an MRI of the brain Schedule vestibular rehabilitation
Use of pregabalin is often limited by side effects, including weight gain, peripheral edema, lethargy, and dizziness; discontinuation may be needed to manage the side effects. The most appropriate next step in management is to discontinue pregabalin. Disequilibrium is an unsteadiness, or sense of imbalance, with standing or walking. Patients may experience disequilibrium as a side effect of medication use. This patient has fibromyalgia, which is characterized by widespread pain, fatigue, disturbed sleep, and cognitive dysfunction. Education, exercise, and psychosocial support are cornerstones of treatment, although pharmacotherapy is often warranted. Pregabalin can be effective for patients with fibromyalgia, but use is often limited by side effects, including weight gain, peripheral edema, lethargy, and especially dizziness (31% of all patients in trials versus 9% on placebo). The patient has fallen several times; therefore, it is imperative to remove any possible offending agent, and pregabalin is the most likely culprit among her medications. Once the troubling side effects resolve off medication, restarting at a lower dose could be considered. Duloxetine can cause dizziness but not nearly as frequently as pregabalin (10% of patients in controlled trials versus 6% on placebo). Because pregabalin is the more likely cause, it should be stopped first; if possible, only one variable should be changed at a time. There is no need to measure methylmalonic acid and homocysteine levels in this patient. Although vitamin B12 deficiency can cause peripheral neuropathy, the normal B12 level and neurologic examination make this a very unlikely diagnosis. Given the high likelihood that pregabalin is causing the dizziness as well as the normal neurologic examination, MRI of the brain is not indicated at this time. It would be most useful if a space-occupying lesion were suspected, but there are no neurologic symptoms or signs referable to the central nervous system. If the patient were to develop neurologic signs or if symptoms did not improve with discontinuation of the pregabalin, this decision would need to be reevaluated. Vestibular rehabilitation may be appropriate as a means of adapting to long-lasting or permanent vestibular damage. It includes exercises and balance training intended to stabilize gait and decrease symptoms and falls. It is not an appropriate first step in this case, however, because drug discontinuation alone is likely to result in improvement of her symptoms.
A 34-year-old man is evaluated for smallpox (variola) exposure. He feels wells. He is an Air Force surgeon and returned 2 days ago from a mission to the Middle East. Two others who accompanied him on his mission are being evaluated for a febrile illness characterized by headache, sore throat, and a vesicular rash on their faces, arms, and legs. The other men have been placed in airborne precautions because their clinical presentation is consistent with probable active variola infection. Medical history is notable for Crohn disease; he is up to date on all recommended immunizations. His only medication is infliximab. On physical examination, vital signs are normal, and other physical examination findings are unremarkable. A complete blood count and comprehensive metabolic profile are normal. Which of the following is the most appropriate treatment? Airborne precautions Tecovirimat Vaccinia immune globulin Vaccinia immunization
Vaccinia immunization is appropriate in the event of possible exposure to smallpox (variola). This man has potentially been exposed to variola and requires active vaccinia immunization to prevent the development of smallpox. Ideally, vaccinia vaccination should be administered no more than 7 days (but preferably within 3) after the presumed exposure. In 1980, the World Health Organization declared that smallpox had been eradicated worldwide. However, because of its ease of deliberate airborne spread, highly contagious nature, and expected significant morbidity and mortality, smallpox has been identified as a member of the A list of potential agents of bioterrorism. Because routine childhood vaccinia immunization is no longer required or recommended in the United States, most of the population is not immune. Smallpox vaccines are available in the event of exposure. Although none of these vaccines contains the actual variola virus, when properly administered the vaccines elicit a significant protective immune response. Although serious adverse events are a greater risk after administration of any live virus vaccine, including those containing vaccinia, persons at high risk for complications from replication-component vaccines are also at higher risk for severe smallpox. Unless the patient is severely immunodeficient (within 4 months of bone marrow transplantation, HIV infection with CD4 cell counts <50/µL, or severe combined immunodeficiency), the vaccine should be given. This patient's use of infliximab would not exclude him from vaccination. The core protein cysteine protease inhibitor tecovirimat, which has proven activity against members of the orthopox genus, has been approved for the treatment of smallpox in the event of a potential outbreak. Airborne precautions after potential exposure would only be indicated if fever or other signs of active infection occurred. Vaccinia immune globulin, available from the Centers for Disease Control and Prevention, consists of pooled human antibodies and is indicated for the treatment of severe vaccinia virus vaccine complications or when vaccination is contraindicated.
A 35-year-old woman is evaluated for severe pain on the left side of the chest for 1 week followed by increased redness and the development of small blisters. Medical history is significant for rheumatoid arthritis. Medications are naproxen as needed, methotrexate folic acid, and an oral contraceptive. On physical examination, skin findings are shown. The remainder of the examination is unremarkable. Which of the following is the most likely diagnosis? Chicken pox (varicella) Eczema herpeticum Herpes simplex virus Herpes zoster
Varicella zoster virus (VZV) is a DNA virus that causes two forms of cutaneous disease. This patient has symptoms of herpes zoster (shingles), recrudescent VZV, which typically presents as a painful, vesicular eruption confined to a single dermatome and is most commonly seen in immunocompromised or elderly patients. The clinical presentation of herpes zoster is characteristic. Prodromal symptoms, such as burning, stinging, or tingling, often occur in a localized region, followed by an eruption of grouped vesicles or pustules on an erythematous base. The outbreak is unilateral and does not cross the body's midline. The most common dermatomes affected are in the thoracic region. Diagnosis is based on clinical presentation but can be confirmed with a Tzanck preparation and/or viral direct fluorescent antibody or polymerase chain reaction testing. Oral antiviral agents (acyclovir, valacyclovir, or famciclovir) are typically used to treat herpes zoster infection and should be started within the first 72 hours of symptom onset for optimal response. Primary infection with VZV causes chickenpox (varicella), a common disease in childhood. It is transmitted by aerosolized droplets; after a 2-week incubation period, it presents as pruritic papules and vesicles with an umbilicated center that appear in crops and heal with crusting. The virus becomes latent within dorsal root ganglia and can reactivate (10% to 20% risk), causing herpes zoster. A vaccine for primary VZV infection was approved for use in the United States in 1995. Eczema herpeticum is a disseminated viral infection most often caused by primary infection of HSV and characteristically seen in patients with atopic dermatitis. It presents with disseminated punched-out erosions with crust, with a predilection for the face. Cutaneous herpes simplex virus (HSV) infections have two common forms: herpes labialis (typically HSV1) or "cold sores/fever blisters" and genital herpes (typically HSV2). Herpes infection presents as vesicles on an erythematous base. Following primary infection, the virus lies dormant and can recur at the original site of infection. A prodrome of pain or burning often precedes recurrences.
A 74-year-old man is noted to have prolonged bleeding from venipuncture sites and new ecchymoses. He was admitted to the hospital 2 weeks ago for antibiotic treatment of a lung abscess. He developed Clostridium difficile-related diarrhea 1 week ago. His oral intake has been poor. Medical history is notable for hyperlipidemia. Medications are ampicillin-sulbactam, oral vancomycin, atorvastatin, and low-dose subcutaneous low-molecular-weight heparin. On physical examination, temperature is 38.2 °C (100.8 °F), blood pressure is 115/68 mm Hg, pulse rate is 96/min, and respiration rate is 24/min. He is thin and chronically ill appearing. On pulmonary examination, crackles are heard at the right lung base. The remainder of the examination is normal. Laboratory studies: Activated partial thromboplastin time 30 s Hemoglobin 11.8 g/dL (118 g/L) Prothrombin time 22 s Alanine aminotransferase 54 U/L Aspartate aminotransferase 56 U/L Factor V 100% Factor VIII 100% Factor X 24% Which of the following is the most appropriate management? Administer vitamin K Discontinue atorvastatin Discontinue heparin and repeat tests in 24 hours Transfuse fresh frozen plasma
Vitamin K deficiency, which can be recognized by a normal activated partial thromboplastin time and an elevated prothrombin time, should be expected in patients who have poor oral intake and are taking prolonged courses of antibiotics. Administering vitamin K is the most appropriate management at this time. This patient has developed vitamin K deficiency from poor oral intake and a prolonged course of antibiotics combined with diarrhea from concurrent Clostridium difficile infection. Although vitamin K is found in many foods, most vitamin K needed for coagulation is derived from saprophytic bacteria in the colon, and antibiotics destroy these bacteria. Diarrhea (from any cause) results in decreased absorption of vitamin K. Vitamin K is needed to activate certain factors (II, VII, IX, and X), and the prothrombin time (PT), which is most sensitive to a decrease in factor VII levels, will be elevated out of proportion to the activated partial thromboplastin time (aPTT), which will be normal or only mildly elevated. Vitamin K deficiency can be managed with oral or parenteral vitamin K, and the coagulopathy will begin to correct within 1 day. Although atorvastatin can cause an elevation in liver enzyme levels, this patient's coagulopathy is not a result of liver disease, because the factor V level is in the normal range. With the exception of factor VIII, all clotting factors are produced in the liver, and liver disease results in a decrease in all factors with corresponding abnormalities in aPTT and PT. Discontinuing this patient's low-dose heparin is unnecessary because this heparin formulation will not prolong the PT. Although fresh frozen plasma will correct the underlying vitamin K deficiency, it is not the safest or most cost-effective treatment option, and the duration of effect is limited by the short half-life of factor VII. Vitamin K is a safer, more cost-effective intervention and will last much longer.
A 43-year-old woman is evaluated during a follow-up appointment for obesity. She has been following a reduced-calorie, low-fat diet and participating in a 45-minute aerobic exercise class three times weekly for the past 6 months. After her last follow-up appointment 1 month ago, she added 30 minutes of brisk walking on the weekend to her exercise regimen. She brings a food diary and weekly weight records to her appointment. She lost an average of 0.2 kg (0.5 lb) per week over the first 3 to 4 weeks, but her weight loss tapered thereafter and her weight has increased by 0.5 kg (1 lb) since her last visit. The total amount of weight lost over the past 8 months is 2.7 kg (6 lb), or 3% of her original weight. The patient reports that she is unable to achieve the calorie reduction recommended, stating that she is "hungry all the time." Medical history is also significant for type 2 diabetes mellitus and hypertension. Medications are amlodipine, lisinopril, and metformin. On physical examination, blood pressure is 141/92 mm Hg, and other vital signs are normal. BMI is 31. The remainder of the examination is unremarkable. Which of the following is the most appropriate next step in treatment? Bariatric surgery Liraglutide Switch to a low-carbohydrate diet Switch to a very-low-calorie diet
Weight-loss medications are recommended when a trial of comprehensive lifestyle modification, including reduced dietary intake, exercise, and behavioral therapy, fails to achieve a 5% to 10% reduction in weight after 3 to 6 months. Liraglutide is the most appropriate next step in treatment. Weight-loss medications are recommended when a trial of comprehensive lifestyle modification, including reduced dietary intake, exercise, and behavioral therapy, fails to achieve a 5% to 10% reduction in weight at 3 to 6 months. This patient has appropriately adhered to dietary caloric restriction, with regular self-monitoring of calorie intake and weight, and is now exercising for more than 150 minutes per week. Her BMI is greater than 27, and she has obesity-related comorbid conditions (type 2 diabetes mellitus and uncontrolled hypertension). Liraglutide has been shown to increase satiety and aid in achieving more than 5% weight loss after 52 weeks of therapy, and it may help the patient feel less hungry. Bariatric surgery is recommended for patients with BMI of 40 or greater, and for patients with BMI of 35 or greater who have obesity-related comorbidities and who have tried all other weight loss therapies without achieving significant weight loss or improvements in comorbid conditions. There is no evidence that low-carbohydrate diets are more effective for reducing weight than low-fat diets. This patient is adherent to her chosen diet but has difficulty reducing her caloric intake to achieve continued weight loss. Clinicians should prescribe a diet with which the patient will adhere (that is, a diet that is palatable and affordable) and that maintains negative energy balance in order to achieve weight loss. Very-low-calorie diets are recommended when rapid weight loss is medically indicated. This patient does not require rapid weight loss, and neither the risk nor the expense of frequent visits and laboratory monitoring are justified in this case.
A 27-year-old woman is evaluated for concern about a 3-week history of a new chemical odor in her workplace. She has a history of mild asthma since childhood that is well-controlled with infrequent use of an albuterol inhaler. The plant safety manager at her job said there is no need for protective equipment, but she is experiencing increased cough and now uses her albuterol inhaler once daily. On physical examination, vital signs are normal. Lungs are clear on auscultation with no wheeze. Office spirometry shows an FVC of 98% of predicted, an FEV1 of 93% of predicted, and an FEV1/FVC ratio of 0.78 of predicted. Which of the following is the most appropriate management? Chest CT scan Obtain hair sample for toxic analysis Review employer Material Safety Data Sheet Transfer patient to other area of plant
When an occupational lung disease is being considered, clinicians should request a Material Safety Data Sheet detailing chemical properties and known health risks associated with substances within the workplace. The most appropriate management is to review the employer Material Safety Data Sheet (MSDS). This patient is concerned that an occupational exposure is causing new respiratory symptoms and potentially may worsen her asthma control. When an occupational lung disease is being considered, clinicians should request the MSDS, which details chemical properties and known health risks associated with substances within the workplace. The U.S. Occupational Safety and Health Administration (OSHA) requires that this information is available upon request for employees who work with potentially harmful materials. Establishing a clear causal link between this patient's symptoms of asthma and an occupational exposure is essential in diagnosis and management. Her history suggests a temporal relationship between the introduction of the new chemical and her cough. In addition, she feels the need to use her albuterol inhaler after exposure. Examples of known respiratory irritants include chlorine gas and sulfur dioxide, which are triggers of bronchospasm. Toluene diisocyanate is associated with allergic sensitization, cough, and bronchospasm that can develop weeks or months after initial exposure. Additional evaluation for occupational illness can include peak flow meter measurements before and after exposure, bronchoprovocation testing after prolonged time away from work and return to work, and, in select cases, specific inhalational challenges. A chest CT scan is usually not needed for the evaluation of a patient with suspected occupational asthma. Exceptions to this rule include patients with abnormal chest radiography or suspected hypersensitivity pneumonitis. This patient has a normal lung examination, normal spirometry, and no symptoms compatible with acute hypersensitivity pneumonitis (fevers, flulike symptoms, cough, and shortness of breath). This patient has no indication for advanced imaging. Although several commercial entities offer testing of hair samples for toxic chemicals, this testing is expensive, unlikely to be covered by insurance, and of questionable validity. It is, therefore, not recommended in the assessment of occupational exposure by primary care internists. Supporting a patient's request to transfer work areas related to a health concern should be based on sound clinical assessment and judgment. A clinician's initial assessment should establish the presence of an occupational illness by assessing exposures, including known chemicals in the workplace, and establishing a temporal relationship between the introduction of the new chemical and symptoms.
A 37-year-old woman is unable to achieve pregnancy despite 7 months of unprotected intercourse. Her menstrual cycles are normal, occurring every 28 days with associated breast tenderness and bloating. There have been no prior pregnancies or attempts to achieve pregnancy by either the patient or her male partner. There is no history of previous sexually transmitted infections. She is otherwise healthy. Medical history is significant for appendicitis at age 26 for which she had an uncomplicated appendectomy. Her only medication is a prenatal vitamin. On physical examination, vital signs are normal. She has a well-healed abdominal scar. Thyroid, skin, and pelvic examinations are all unremarkable. There is no elicitable breast discharge, no signs of hyperandrogenism, and no visual field cuts. Which of the following is the most appropriate management? Obtain midluteal phase serum progesterone level Obtain semen analysis Recommend an additional 5 months of unprotected intercourse Refer for laparoscopy
When evaluating infertility, both female and male factors should be considered concurrently; semen analysis is part of the initial diagnostic evaluation.
A 26-year-old man undergoes follow-up evaluation after hospitalization for deep venous thrombosis last week. He reports no recent travel, surgery, or immobilization. He has a sister who was diagnosed with an unprovoked deep venous thrombosis 1 year ago at 35 years of age. Medical history is otherwise unremarkable. His only medication is rivaroxaban. On physical examination, vital signs are normal. The examination is otherwise unremarkable. The possibility of an inherited thrombophilia is discussed with the patient. After reviewing the risks and benefits of additional testing, he would like to be further evaluated for a possible thrombophilia. Which of the following is the ideal testing strategy? Test now Test in 2 months Test a saved blood sample obtained during hospitalization but before anticoagulation Temporarily stop rivaroxaban in 1 year and test 2 weeks later
With the exception of testing for genetic mutations, thrombophilia testing should not be performed in the acute setting or while receiving anticoagulant therapy and should be delayed at least 2 weeks after discontinuation of anticoagulant therapy to minimize diagnostic error. The ideal testing strategy is to temporarily stop rivaroxaban in 1 year and test 2 weeks later. Long-term anticoagulation is indicated in the setting of idiopathic thrombosis. Because of the unprovoked nature of this patient's thrombotic event, his thrombotic risk remains high even after 3 months of anticoagulation. Therefore, it is safer to continue anticoagulation for 1 year before testing. Testing for inherited thrombophilia remains controversial. Guidelines and systematic reviews recommend against routine population screening or screening in most adults with idiopathic venous thromboembolism (VTE) and in asymptomatic family members of patients with VTE and known factor V Leiden or prothrombin G20210A gene mutation. Most experts agree that thrombophilia evaluation can be considered in select populations, such as patients with thromboses at unusual sites or recurrent idiopathic thrombosis, patients younger than 45 years with unprovoked thrombosis, patients with VTE and a clear family history of thrombosis in one or more first-degree relatives, and patients with warfarin-induced skin necrosis. After an informed discussion of the risks and benefits of testing, patients who desire evaluation should receive genetic counseling before testing. Testing typically focuses on factor V Leiden mutation; prothrombin G20210A mutation; protein C, protein S, and antithrombin deficiencies; and antiphospholipid antibody syndrome. However, the thrombophilia evaluation should not be performed during an acute episode of thrombosis. Additionally, although genetic tests such as factor V Leiden and prothrombin gene mutation can be performed at any time, the other assays can be variably affected by the presence of an acute thrombosis, heparin, and other anticoagulants. For example, during the acute phase of thrombosis, plasma levels of antithrombin and occasionally proteins C and S may transiently decrease, and fibrinogen and factor VIII levels may increase. Depending on the assay used, dabigatran therapy may result in increased measurements of antithrombin and protein S and C levels, whereas the factor Xa inhibitors (for example, rivaroxaban and apixaban) may result only in increased measurements of antithrombin. Warfarin therapy reduces vitamin K-dependent factors, including proteins C and S, which do not resolve for 4 to 6 weeks after discontinuation. Most experts agree that with the exception of testing for genetic mutations, testing should not be done in the acute setting or while receiving anticoagulant therapy and should be delayed at least 2 weeks after discontinuation of anticoagulant therapy to minimize diagnostic error.
A 29-year-old woman is evaluated in the emergency department in January for a 5-day history of fever, severe headaches, delusions, and paranoid behavior. Medical history is noncontributory, and she takes no medications. On physical examination, she is confused and combative. Temperature is 37.9 °C (100.2 °F), blood pressure is 107/48 mm Hg, pulse rate is 102/min, and respiration rate is 20/min. The neck is supple. Occasionally, choreoathetoid movements of the bilateral upper extremities are observed. She has no paralysis. After the examination, she has a generalized seizure in the emergency department. Lumbar puncture is performed, and results show a cerebrospinal fluid leukocyte count of 310/µL (310 × 106/L) with 94% lymphocytes, glucose level of 60 mg/dL (3.3 mmol/L), and protein level of 55 mg/dL (550 mg/L). Cerebrospinal fluid bacterial and fungal cultures are negative. Polymerase chain reaction for herpes simplex virus is negative. An MRI scan of the brain is normal. Which of the following is the most likely diagnosis? Anti-N-methyl-D-aspartate receptor encephalitis Lyme meningitis Tuberculous meningitis West Nile virus encephalitis
When other causes of encephalitis have been ruled out in a patient presenting with associated changes in mood and behavior, anti-N-methyl-D-aspartate receptor encephalitis should be suspected.
A 29-year-old woman is evaluated in the emergency department in January for a 5-day history of fever, severe headaches, delusions, and paranoid behavior. Medical history is noncontributory, and she takes no medications. On physical examination, she is confused and combative. Temperature is 37.9 °C (100.2 °F), blood pressure is 107/48 mm Hg, pulse rate is 102/min, and respiration rate is 20/min. The neck is supple. Occasionally, choreoathetoid movements of the bilateral upper extremities are observed. She has no paralysis. After the examination, she has a generalized seizure in the emergency department. Lumbar puncture is performed, and results show a cerebrospinal fluid leukocyte count of 310/µL (310 × 106/L) with 94% lymphocytes, glucose level of 60 mg/dL (3.3 mmol/L), and protein level of 55 mg/dL (550 mg/L). Cerebrospinal fluid bacterial and fungal cultures are negative. Polymerase chain reaction for herpes simplex virus is negative. An MRI scan of the brain is normal. Which of the following is the most likely diagnosis? Anti-N-methyl-D-aspartate receptor encephalitis Lyme meningitis Tuberculous meningitis West Nile virus encephalitis
When other causes of encephalitis have been ruled out in a patient presenting with associated changes in mood and behavior, anti-N-methyl-D-aspartate receptor encephalitis should be suspected. The most likely diagnosis is anti-N-methyl-D-aspartate receptor (anti-NMDAR) encephalitis. Anti-NMDAR encephalitis has emerged as an increasingly common cause of encephalitis. Anti-NMDAR encephalitis is associated with ovarian teratomas in more than 50% of patients with the disease because of antibody production to a tumor protein that cross-reacts with neuronal tissue. In patients without evidence of teratoma, an inciting antigenic stimulus is rarely identified. The diagnosis is suggested by the presence of choreoathetosis, psychiatric symptoms, seizures, and autonomic instability; detection of anti-NMDAR antibody in serum confirms it. Anti-NMDAR encephalitis should be suspected in young women presenting with encephalitis associated with changes in mood and behavior when initial evaluation for other causes, including herpes simplex virus infection, is negative. Anti-NMDAR encephalitis treatment can include tumor removal (if present), intravenous glucocorticoids, intravenous immune globulin, plasmapheresis, and rituximab. The most common central nervous system manifestation of Lyme disease is lymphocytic meningitis that may be indistinguishable from viral meningitis with fever, headache, and meningismus. An inflammatory encephalomyelitis may very rarely be observed. Lyme meningitis usually occurs in the summer and fall; it is often associated with peripheral facial palsy and is not associated with altered mental status, psychiatric symptoms, choreoathetosis, or seizure as seen in this patient. Tuberculous meningitis is a subacute febrile illness with a predilection for inflammatory changes at the base of the brain. After a 2- to 3-week prodromal period, patients often manifest meningismus and varying degrees of cranial nerve and long-tract signs. Tuberculosis is unlikely in this patient because of the lack of chronicity, normal glucose level in cerebrospinal fluid, absence of cranial nerve and long-track signs, and lack of evidence of basilar meningitis on MRI. West Nile neuroinvasive disease may present with meningitis, encephalitis, or myelitis, either singly or as overlap syndromes in the endemic months between June and October. Limb weakness, which may be symmetric or involve a single extremity, is characteristic. A nonspecific viral exanthema may be found in less than 50% of patients. MRI may show focal lesions of the thalami, basal ganglia, and spinal cord in some patients.
A 26-year-old woman is evaluated in the emergency department for fever and low back pain that has progressed for the past 3 weeks. She injects heroin daily. Medical history is unremarkable, and she takes no medications. On physical examination, temperature is 38.1 °C (100.6 °F), blood pressure is 90/60 mm Hg, pulse rate is 110/min, and respiration rate is 28/min. The lower lumbar spine is tender to palpation. Neurologic examination and other physical examination findings are normal. Aggressive fluid resuscitation is initiated. Complete blood count, metabolic profile, erythrocyte sedimentation rate, and blood culture and urinalysis are obtained. MRI is shown. Which of the following is the most appropriate antibiotic management? Cefazolin and gentamicin Vancomycin Vancomycin and cefepime Withhold antibiotics while awaiting bone biopsy
When sepsis is suspected among patients with osteomyelitis, empiric antibiotic therapy should begin, even when the microbial cause of the infection has not yet been determined. The patient has vertebral osteomyelitis and clinical findings suggestive of sepsis; empiric antibiotic therapy to cover the most likely pathogens should be initiated. A diagnosis of vertebral osteomyelitis should be considered in patients reporting worsening back or neck pain without an alternate explanation. Local tenderness over the site of spinal infection is frequently detected. Radicular pain, motor weakness, and sensory changes may be present. Common sources of hematogenous osteomyelitis are distant foci of infection (for example, skin and soft tissue, genitourinary, gastrointestinal), intravascular catheters, and infective endocarditis. Hematogenous osteomyelitis is typically monomicrobial, and Staphylococcus aureus is the most commonly isolated pathogen; however, aerobic, gram-negative bacilli cause disease in many patients. Certain patient-specific conditions are associated with less common bacterial organisms, including Salmonella osteomyelitis in persons with sickle cell disease and Pseudomonas aeruginosa bone infection in injection drug users. Lengthy parenteral antimicrobial therapy is the mainstay of treatment. An antistaphylococcal agent with methicillin-resistant S. aureus (MRSA) coverage (vancomycin) and a β-lactam with antipseudomonal coverage (cefepime) is an appropriate empiric regimen for suspected osteomyelitis in this patient who uses injection drugs. Cefazolin does not provide coverage for MRSA, and aminoglycosides, such as gentamicin, have poor penetration into bone. Vancomycin alone would not provide coverage for gram-negative organisms, which should be included in the empiric therapy for this patient. Although microbiologic confirmation of the bacterial cause should always be sought, withholding antibiotics in a patient likely to have sepsis (fever, hypotension, tachycardia) is not appropriate. If the patient has positive blood cultures, a presumptive microbiologic diagnosis of vertebral osteomyelitis can usually be made. However, because injection drug users have frequent bacteremia and the patient's bloodstream infection may be unrelated to the vertebral osteomyelitis, some experts would recommend pursuing bone biopsy, even in patients with positive blood cultures.
A 47-year-old man is hospitalized with a 1-month history of increasingly severe low back pain and a 3-day history of fever and chills. He reports injection drug use (last injected 1 week ago). Medical history is otherwise unremarkable, and he takes no medications. On physical examination, temperature is 38 °C (100.4 °F), blood pressure is 110/76 mm Hg, pulse rate is 88/min, and respiration rate is 16/min. Neurologic examination reveals no deficits, and no murmur is heard on cardiac auscultation. Palpation of the spine elicits point tenderness over L2-L4. Laboratory studies show an erythrocyte sedimentation rate of 98 mm/h and a leukocyte count of 18,300/µL (18.3 × 109/L). MRI is positive for spondylodiskitis at L3-L4. Blood samples were obtained, and empiric vancomycin was initiated. Two sets of blood cultures are positive for methicillin-resistant Staphylococcus aureus, with a vancomycin minimum inhibitory concentration of 1.5 µg/mL; repeat blood cultures are pending. The vancomycin trough level is 19 µg/mL. Transesophageal echocardiography shows no valvular vegetations. Which of the following is the most appropriate treatment? Add gentamicin to vancomycin Add rifampin to vancomycin Change vancomycin to daptomycin Continue vancomycin
When treating methicillin-resistant Staphylococcus aureus bacteremia, vancomycin should be used only if the minimum inhibitory concentration is 2 µg/mL or less. The most appropriate management is to continue the vancomycin. Endocarditis and vertebral osteomyelitis are two important complications of Staphylococcus aureus bacteremia. Vancomycin is the preferred antimicrobial agent for treatment of bacteremia with methicillin-resistant S. aureus (MRSA). An alternative to vancomycin should be considered if the organism is not susceptible or if no clinical or microbiologic (such as persistent bacteremia) response occurs despite adequate source control. Daptomycin is an acceptable alternative if the isolate is susceptible. S. aureus with a vancomycin minimum inhibitory concentration (MIC) of 2 µg/mL or less is considered susceptible; intermediate susceptibility is defined as an MIC of 4 to 8 µg/mL. Vancomycin should not be used in the treatment of MRSA bacteremia when the vancomycin MIC is greater than 2 µg/mL because clinical failures have been reported. At MICs close to 2 µg/mL, clinical response (that is, clearance of bacteremia and resolution of fever and leukocytosis) should guide whether to continue vancomycin or change to daptomycin. Source control and appropriate antibiotic therapy are important for improving clinical outcomes. When using vancomycin to treat serious MRSA infections such as bacteremia, drug level monitoring is necessary to ensure successful treatment. Monitoring by trough level with a target of 15 to 20 µg/mL has been shown to result in an increased risk of nephrotoxicity, so monitoring utilizing a calculated target 24-hour time-concentration area under the curve (AUC) to MIC (AUC/MIC) ratio of 400 to 600 mg*h/L is now recommended. Adding rifampin or gentamicin to vancomycin has not been shown to improve outcomes of MRSA bacteremia and should be avoided. Combination therapy also increases the risk for adverse events (nephrotoxicity with gentamicin, hepatotoxicity and drug interactions with rifampin). Combination therapy is sometimes considered with prosthetic device infections, and an infectious diseases specialist should be consulted for guidance in such cases.
A 28-year-old woman is evaluated for preconception counseling. She desires to achieve pregnancy in the next 6 months. Medical history is significant for type 1 diabetes mellitus diagnosed at 12 years of age. She has no known microvascular or macrovascular complications of diabetes. She is up to date on screening for microvascular complications with her last eye examination performed 11 months prior and a normal creatinine level, urine albumin-creatinine ratio, lipid panel, and foot examination 9 months prior. Thyroid-stimulating hormone level was 0.5 µU/mL (0.5 mU/L) 3 months ago. She is currently using a condom for contraception. Medical history also includes autoimmune thyroid disease. Medications are insulin lispro delivered through continuous subcutaneous insulin infusion and levothyroxine. On physical examination, vital signs are normal. Nondilated retinal examination, thyroid examination, and monofilament testing are all normal. Laboratory studies reveal that hemoglobin A1c level is currently 6.7%, improved from 9% 3 months ago. Which of the following is the most appropriate preconception management to perform next? Dilated eye examination Fasting lipid profile Nephrology referral Thyroid-stimulating hormone measurement Urine albumin-creatinine ratio
Women with type 1 or type 2 diabetes mellitus who are planning pregnancy should be counseled on the risk of development or progression of diabetic retinopathy; rapid improvements in glycemic levels during pregnancy can temporarily worsen preexisting retinopathy. The most appropriate preconception management for this patient is a dilated eye examination. Women with type 1 or type 2 diabetes mellitus who are planning pregnancy should be counseled on the risk of development or progression of diabetic retinopathy. Additionally, rapid improvement in glycemic control in the setting of retinopathy is associated with temporary worsening of retinopathy. Given tight glycemic targets in pregnancy, this is often a time of intensified glycemic control for women placing them at greater risk for this complication. Dilated eye examinations should occur before pregnancy or in first trimester if not done prior to pregnancy. Patients should be monitored every trimester and then closely for 1 year postpartum as indicated by the degree of retinopathy. This patient is up to date on lipid screening and additional screening as part of preconception management is not necessary. Thyroid-stimulating hormone (TSH) levels should be monitored closely in pregnancy due to increased level of thyroid-binding globulin in pregnancy resulting in increased levothyroxine needs. This patient has had a TSH measurement with normal results 3 months ago. The dose of levothyroxine may need to be increased on average by 30% to 50% during pregnancy, and patients should have their TSH level checked as soon as a pregnancy test is positive. A referral to a physician experienced in the care of kidney disease should be undertaken in patients with advanced kidney disease, which is not present in this patient as her most recent serum creatinine and urine albumin-creatinine ratio were normal.
A 29-year-old man is evaluated for a 2-month history of night sweats, fevers, a sense of abdominal fullness, and fatigue. He takes no medications. On physical examination, vital signs are normal. The abdomen is distended with diffuse tenderness without rebound or guarding. Bowel sounds are normal. Other examination findings are normal. Laboratory results, including complete blood count and serum bilirubin, creatinine, alkaline phosphatase, aminotransferase, α-fetoprotein, and β-human chorionic gonadotropin levels, are normal. A contrast-enhanced CT scan of the chest, abdomen, and pelvis shows bulky retroperitoneal lymphadenopathy (the largest mass is 10 cm in diameter) with somewhat smaller but still enlarged mediastinal lymphadenopathy. A biopsy specimen of the most accessible enlarged node shows poorly differentiated carcinoma. A testicular ultrasound is unremarkable. Which of the following is the most appropriate treatment? A gastrointestinal cancer chemotherapy regimen A germ cell chemotherapy regimen Radiation therapy to the lymphadenopathy Surgical debulking of the lymphadenopathy
Young men with poorly differentiated carcinoma that is characterized by centrally located bulky retroperitoneal or mediastinal lymphadenopathy may have an unrecognized germ cell tumor and are best treated with a platinum-based chemotherapy regimen. The most appropriate treatment is a germ cell chemotherapy regimen of cisplatin and etoposide. Cancer of unknown primary (CUP) is a diagnosis of exclusion established in patients with a solid metastatic tumor after a detailed medical history and physical examination have been done and imaging studies or other diagnostic studies have not identified a primary tumor site. Diagnostic efforts should focus on identifying whether a patient is among the approximately 20% of patients with CUP who fall into identifiable subgroups with a more favorable prognosis and who can benefit from a specific treatment strategy. A biopsy obtained from the site that can be sampled in the safest, least invasive manner is done, and specimens are often evaluated by immunohistochemical stains consistent with the tumor's pattern of presentation to attempt to establish a diagnosis of a more treatable subtype of CUP. The patient is a young man with a poorly differentiated CUP site that is predominantly midline in presentation. In the absence of an identified primary tumor, these patients should be treated presumptively for metastatic germ cell tumor. The absence of abnormal findings on testicular examination or ultrasonography or the absence of germ-cell specific markers, such as α-fetoprotein and β-human chorionic gonadotropin, does not exclude this diagnosis or this course of action. Even if these evaluation results are negative, an unrecognized germ cell tumor may still exist, and these patients should be treated for this possibility with a platinum-based chemotherapy regimen. A chemotherapy regimen focused on digestive tract malignancies, such as fluorouracil and irinotecan, would be appropriate palliation for a pattern of CUP consistent with a gastrointestinal primary tumor, such as disease predominantly below the diaphragm with liver or peritoneal involvement. However, this patient has bulky lymphadenopathy below and above the diaphragm, making a digestive malignancy unlikely. The extent of retroperitoneal lymphadenopathy and mediastinal lymphadenopathy makes initial treatment with either radiation therapy or surgery inappropriate.
A 62-year-old man arrives for follow-up consultation for multiple myeloma diagnosed 3 weeks ago. A skeletal survey showed multiple lytic lesions in the spine and pelvis, but the patient's vertebral height was maintained. MRI showed abnormalities in the vertebral bodies but no evidence of spinal cord compression. Myeloma therapy was begun with bortezomib, lenalidomide, and dexamethasone. Medical history is otherwise unremarkable. He takes no other medications. On physical examination, vital signs are normal. The lower spine is nontender to palpation. The remainder of the physical examination is unremarkable. Laboratory studies show a serum calcium level of 9.1 mg/dL (2.3 mmol/L) and a serum creatinine level of 0.8 mg/dL (70.7 µmol/L); a complete blood count and serum electrolyte levels are normal. Which of the following is the most appropriate management of this patient's lytic bone lesions? Dual-energy x-ray absorptiometry Radiation therapy Zoledronic acid Current myeloma treatment
Zoledronic acid has been shown to prevent new skeletal-related events and improve survival in patients with multiple myeloma requiring therapy.
A 62-year-old man arrives for follow-up consultation for multiple myeloma diagnosed 3 weeks ago. A skeletal survey showed multiple lytic lesions in the spine and pelvis, but the patient's vertebral height was maintained. MRI showed abnormalities in the vertebral bodies but no evidence of spinal cord compression. Myeloma therapy was begun with bortezomib, lenalidomide, and dexamethasone. Medical history is otherwise unremarkable. He takes no other medications. On physical examination, vital signs are normal. The lower spine is nontender to palpation. The remainder of the physical examination is unremarkable. Laboratory studies show a serum calcium level of 9.1 mg/dL (2.3 mmol/L) and a serum creatinine level of 0.8 mg/dL (70.7 µmol/L); a complete blood count and serum electrolyte levels are normal. Which of the following is the most appropriate management of this patient's lytic bone lesions? Dual-energy x-ray absorptiometry Radiation therapy Zoledronic acid Current myeloma treatment
Zoledronic acid has been shown to prevent new skeletal-related events and improve survival in patients with multiple myeloma requiring therapy. All patients with multiple myeloma (MM) requiring therapy should be given intravenous bisphosphonates, such as zoledronic acid or pamidronate, in addition to antimyeloma therapy. Zoledronic acid and pamidronate have been shown to prevent new skeletal-related events in patients with MM, but only zoledronic acid has been shown to improve survival. Close monitoring of kidney function and calcium level should be performed in all patients taking zoledronic acid; monitoring for pain and swelling that may be early signs of osteonecrosis of the jaw is also essential. Calcium and vitamin D supplementation is usually given to patients taking intravenous bisphosphonates unless a contraindication exists. Patients with MM are given intravenous zoledronic acid every 3 to 4 weeks. Experts have not reached a consensus on duration of therapy, but indefinite bisphosphonate therapy is often used. It is reasonable to consider remission status, extent of skeletal disease, kidney function, and patient preference when determining the duration of bisphosphonate therapy. In patients who stop therapy, bisphosphonates should be restarted at disease relapse. In patients with monoclonal gammopathy of undetermined significance and smoldering MM, dual-energy x-ray absorptiometry (DEXA) is performed to evaluate for osteoporosis or osteopenia in determining bisphosphonate therapy. This patient has MM requiring therapy with lytic lesions. Bone density scanning with DEXA is unnecessary to determine treatment. With the advent of effective antimyeloma therapy, limited use of radiation is recommended to preserve the bone marrow. Radiation therapy is recommended to control pain or prevent impending pathologic fracture or spinal cord compression. In this patient with adequate pain control, no evidence of spinal cord compression, and no neurologic symptoms, radiation therapy is not appropriate. Patients with MM requiring therapy are at high risk for recurrent skeletal-related events. Therefore, continuing with antimyeloma therapy alone without specifically addressing the bone disease is not adequate.
Dysphagia- motility vs nonmotility
Motility- LIQUIDS or solids + liquids Non-moitlity- SOLIDS
A 55-year-old woman is evaluated after screening colonoscopy showed three polyps at the rectosigmoid junction. The three polyps were 3 mm, 5 mm, and 6 mm in size. All three polyps were completely excised and pathology showed them to be hyperplastic. Colonoscopy preparation was excellent and the procedure was complete to the cecum. Family history is significant for colon cancer diagnosed in her paternal grandfather at age 80 years. All physical examination findings, including vital signs, are normal. 1 year 3 years 5 years 10 years
This patient should undergo colonoscopy in 10 years. Serrated polyps are classified into three histologic types: hyperplastic polyps, sessile serrated polyps, and traditional serrated adenomas. Hyperplastic polyps are the most common type of serrated polyp. They are non-neoplastic and are composed of normal mucosal elements; small hyperplastic polyps, often found in the rectosigmoid colon, are believed to have no clinical significance. As a result, the interval until the next screening examination is 10 years, the same as for patients who do not have polyps found on baseline examination. Sessile serrated polyps (also known as sessile serrated adenomas) and traditional serrated adenomas are both neoplastic and are precursors to colorectal cancer; they should be completely excised. Substantial variability has been demonstrated in the ability of a pathologist to differentiate a hyperplastic polyp from a sessile serrated polyp; therefore, guidelines recommend managing large (>10 mm) hyperplastic polyps as if they are sessile serrated polyps. The patient's family history of a second-degree relative with colon cancer diagnosed at age 80 years does not increase her risk for colon cancer or indicate the need for more frequent or early colonoscopy. A 1-year surveillance interval is not appropriate for this patient. It is indicated in patients with more than 10 adenomas found on colonoscopy, those with a diagnosed polyposis syndrome, or those with Lynch syndrome. Lynch syndrome is the term used to describe patients who meet the Amsterdam II criteria for hereditary nonpolyposis colorectal cancer and have an identified germline mutation in one of the four mismatch repair genes (MLH1, MSH2, MSH6, PMS2) or the epithelial cell adhesion molecule gene (EPCAM). A 3-year surveillance interval is recommended for patients who have three or more adenomas (or sessile serrated polyps) found on baseline colonoscopy, one adenoma larger than 10 mm in size, or an adenoma with any degree of villous or high-grade dysplasia. A surveillance interval of 5 years is recommended for patients with two or fewer adenomas (or sessile serrated polyps) found on baseline colonoscopy and for patients with a first-degree relative with colon cancer diagnosed at an age younger than 60 years. Sessile serrated polyps are more frequently found in the proximal colon and may be difficult to detect on colonoscopy due to their flat appearance. Like tubular adenomas, surveillance colonoscopy is based on size and presence of dysplasia.
Treatments for opioid-induced constipation?
*Treatment-specific, only if failed 1st line 1. PO Naloxegol 2. PO naldemedine 3. SC methylnaltrexone First-line laxative therapies, including over-the-counter stool softeners, bulk laxatives (fiber supplements), a stimulant laxative (bisacodyl), and an osmotic laxative (polyethylene glycol [PEG]), have all been ineffective. Current maintenance laxative therapy should be stopped before the initiation of naloxegol and can be added to the naloxegol after 3 days of monotherapy as symptoms dictate. Surfactants such as docusate sodium or docusate calcium are weak laxatives with an excellent safety profile. As such, they are most appropriate for very mild, intermittent constipation and will not be effective in this patient. Adding a second osmotic agent such as lactulose is unlikely to improve the patient's reduced stool frequency, which is the result of slowed colonic motility caused by her chronic opioid analgesic use. Also, lactulose use is likely to lead to bloating.
A 22-year-old man is evaluated for a mass on the left side of his neck. He has not experienced fevers, night sweats, or weight loss. He takes no medications. On physical examination, all vital signs are normal. A left supraclavicular lymph node is enlarged to 4 cm, and axillary lymphadenopathy is noted bilaterally. There is no hepatosplenomegaly. A biopsy specimen of the supraclavicular lymph node reveals Hodgkin lymphoma, nodular sclerosis subtype. Laboratory results, including complete blood count, biochemical profile, and serum lactate dehydrogenase, are normal. CT scan and PET scan show mediastinal lymphadenopathy, with nodes up to 4 cm in size, and left supraclavicular and bilateral axillary lymphadenopathy. No infradiaphragmatic disease is observed. Which of the following is the most appropriate initial management? Bone marrow biopsy Chemotherapy Radiation therapy Staging laparotomy and splenectomy
All patients with classic Hodgkin lymphoma, regardless of stage, receive chemotherapy, usually the doxorubicin, bleomycin, vinblastine, and dacarbazine regimen, obviating the need for staging laparotomy and splenectomy. The most appropriate management for this young man with Hodgkin lymphoma is chemotherapy. Hodgkin lymphoma encompasses four classic histologic subtypes (nodular sclerosing, mixed cellularity, lymphocyte predominant, and lymphocyte depleted) and one nonclassic subtype (nodular lymphocyte-predominant subtype expressing the CD20 cell surface antigen). The presentation is consistent among subtypes and is characterized by palpable, firm lymph nodes and, in some patients, B symptoms (fever, night sweats, weight loss). Other physical examination findings include splenomegaly (30%) and hepatomegaly (5%). Hodgkin lymphoma is a highly curable disease, and the best outcomes are achieved with systemic therapy, sometimes in combination with radiation therapy. In the United States, doxorubicin, bleomycin, vinblastine, and dacarbazine is the most commonly used chemotherapy regimen for classic Hodgkin lymphoma. The need for radiation after chemotherapy for early-stage disease depends on the initial bulk of disease, the response to treatment, and patient and institutional preference. Bone marrow biopsy, in the absence of unexplained blood abnormalities, is not indicated in the evaluation of patients with Hodgkin lymphoma. Staging no longer includes routine bone marrow biopsy, lymphangiography, or staging laparotomy and splenectomy. High-resolution CT, PET, and the routine use of systemic therapy in even early-stage disease have obviated the need for these staging modalities. Patients with extensive lymph node involvement above the diaphragm are now typically treated with chemotherapy; therefore, there is no need for staging laparotomy and splenectomy to determine the presence of microscopic disease below the diaphragm as the treatment would be no different. With this approach, the short-term morbidity of the staging surgery and long-term risk of overwhelming sepsis from asplenia can be avoided. Radiation therapy may have a role in this patient's management; however, it is given after completion of chemotherapy for consolidation. Although radiation therapy alone may have a high cure rate for classic Hodgkin lymphoma, the cure rate is higher and toxicity is reduced when chemotherapy is used as the primary treatment modality.
A 53-year-old man is evaluated for fatigue and arthralgia for the past 2 years. He indicates multiple joints in the hands and lower extremities are involved, but the pain is not exacerbated by joint use and is partially relieved with ibuprofen. He typically consumes two beers per day. His only medication is ibuprofen. On physical examination, vital signs are normal. The right second and third metacarpophalangeal (MCP) joints are swollen. The examination is otherwise unremarkable. Hand radiographs show hook-like osteophytes in the MCP joints. Which of the following is the most appropriate test to perform next? HFE genotyping Liver biopsy Rheumatoid factor and anti-cyclic citrullinated peptide antibodies Transferrin saturation and serum ferritin level
Atypical presentations of rheumatoid arthritis and osteoarthritis, particularly with hook-like osteophytes of the second and third metacarpophalangeal joints, suggest the possibility of hemochromatosis; evaluating the transferrin saturation and serum ferritin level should be considered. The most appropriate test to perform next is transferrin saturation and serum ferritin level. Historically, patients with hereditary hemochromatosis presented with signs of cirrhosis, diabetes mellitus, and skin pigmentation. Because of earlier recognition, fatigue, arthralgias, and loss of libido are more common presenting symptoms. Patients with hemochromatosis have rheumatic symptoms that typically involve the small joints of the hand, especially the second and third metacarpophalangeal (MCP) joints associated with characteristic radiographic findings, including hook-like osteophytes. Guidelines support initial testing with transferrin saturation to diagnosis iron overload. A transferrin saturation greater than 45% is often chosen as the cutoff to warrant additional testing, although this threshold has lower specificity and positive predictive value compared with higher cutoff values. A normal serum ferritin level and transferrin saturation less than 45% have a negative predictive value of 97% to exclude iron overload. Symmetrical involvement of MCP joints suggests the possibility of rheumatoid arthritis (RA), but hook-like osteophytes on the metacarpal heads is a disease-specific finding for hemochromatosis. For these reasons, RA is an unlikely diagnosis, and it is unnecessary to perform rheumatoid factor and anti-cyclic citrullinated peptide antibody testing. The MCP involvement and radiographic changes also make primary osteoarthritis an unlikely diagnosis. HFE genotyping is reserved for patients in whom the serum ferritin level is elevated, a high transferrin saturation is noted, or both. It would not be the appropriate initial test to perform in this patient who has had no other testing. C282Y homozygosity accounts for 80% to 85% of typical patients with hereditary hemochromatosis; C282Y/H63D and C282Y/S65C compound heterozygosity are the next most prevalent genotypes. Since the advent of HFE mutational analysis, the role of liver biopsy has diminished. Liver biopsy is generally reserved to stage liver disease in patients known to be C282Y homozygous or compound heterozygous when the serum ferritin level is greater than 1000 ng/mL (1000 µg/L) with or without elevated liver enzyme levels. Liver biopsy may also be useful when other pathogens, including alcohol and chronic hepatitis, might be causing liver injury.
A 22-year-old man is evaluated for a mass on the left side of his neck. He has not experienced fevers, night sweats, or weight loss. He takes no medications. On physical examination, all vital signs are normal. A left supraclavicular lymph node is enlarged to 4 cm, and axillary lymphadenopathy is noted bilaterally. There is no hepatosplenomegaly. A biopsy specimen of the supraclavicular lymph node reveals Hodgkin lymphoma, nodular sclerosis subtype. Laboratory results, including complete blood count, biochemical profile, and serum lactate dehydrogenase, are normal. CT scan and PET scan show mediastinal lymphadenopathy, with nodes up to 4 cm in size, and left supraclavicular and bilateral axillary lymphadenopathy. No infradiaphragmatic disease is observed. Which of the following is the most appropriate initial management? Bone marrow biopsy Chemotherapy Radiation therapy Staging laparotomy and splenectomy
All patients with classic Hodgkin lymphoma, regardless of stage, receive chemotherapy, usually the doxorubicin, bleomycin, vinblastine, and dacarbazine regimen, obviating the need for staging laparotomy and splenectomy.
APAP vs Bipap indications?
Bilevel positive airway pressure is indicated for hypoventilation syndromes caused by severe COPD or neuromuscular weakness. There is no evidence that bilevel positive airway pressure is superior to continuous positive airway pressure for the treatment of obstructive sleep apnea without hypoventilation. This patient does not suffer from a hypoventilation syndrome, and he is improving on current therapy; therefore, a change in therapy is not indicated. Finally, bilevel positive airway pressure without humidification will not relieve his upper airway symptoms.
A 45-year-old man is evaluated for severe lower back and hip pain related to degenerative joint disease. His pain responds to naproxen, but he was hospitalized 6 months earlier for a bleeding gastric ulcer attributed to daily naproxen use. Naproxen was stopped, and twice-daily omeprazole was initiated at that time. Three months later, the ulcer had healed completely and omeprazole was discontinued. The patient's pain did not respond to trials of acetaminophen. He did not tolerate tramadol. The patient has no other medical problems and currently takes only acetaminophen. Celecoxib Celecoxib and omeprazole Ibuprofen and misoprostol Naproxen and ranitidine
Celecoxib plus omeprazole is the most appropriate treatment regimen for this patient. Patients who have bleeding ulcers while taking NSAIDs should discontinue NSAIDs permanently if possible. In cases where this is not possible, strategies to reduce the risk for recurrent bleeding should be instituted. Selective cyclooxygenase-2 (COX-2) inhibitors, such as celecoxib, preferentially inhibit the COX-2 isoenzyme, which primarily modulates pain and inflammation, and minimally inhibit the COX-1 isoenzyme, which promotes generation of the gastric mucosal protective barrier, decreases gastric acid secretion, and helps to maintain good mucosal blood flow. The risk for gastroduodenal ulcers and ulcer complications is significantly lower in patients taking COX-2 inhibitors compared with nonselective NSAIDs such as naproxen; however, in high-risk patients, such as those with previous peptic ulcer disease, a COX-2 inhibitor alone is no better than a nonselective NSAID coadministered with a proton pump inhibitor in preventing ulcer complications, with rebleeding rates of approximately 4% to 6% within 6 months. A single 12-month randomized study showed an ulcer rebleeding rate of 0% in patients treated with celecoxib plus omeprazole compared with 9% in patients treated with a celecoxib alone. A direct comparison of naproxen plus omeprazole to celecoxib plus omeprazole in the prevention of NSAID bleeding in high-risk patients has not been performed. There is also evidence that COX-2 inhibitors and nonselective NSAIDs, with the possible exception of naproxen, increase the risk for cardiovascular complications; therefore, the decision to use a COX-2 inhibitor requires a harm-benefit analysis that weighs the gastrointestinal risks of an NSAID with the potential cardiovascular risks of a COX-2 inhibitor. The use of ibuprofen, a nonselective NSAID, is no safer than the use of naproxen for lessening the likelihood of development of peptic ulcers. Other gastroprotective agents, such as H2 blockers (including ranitidine) and misoprostol, have been ineffective in preventing NSAID-related peptic ulcers in patients at low or moderate risk; however, their efficacy in high-risk patients (such as this patient with a previous NSAID-induced gastric ulcer) has not been demonstrated. Misoprostol is associated with adverse effects, including diarrhea and abdominal pain. Also, the required twice-daily dosing for ranitidine and four-times-daily dosing for misoprostol may lead to patient nonadherence, which further reduces the efficacy of such therapy.
A 72-year-old woman is evaluated in the hospital for new-onset abdominal pain in the right upper quadrant and fever that developed abruptly on hospital day 5. She was hospitalized 5 days earlier for altered mental status. In the emergency department, she was found to be confused, hypotensive, and tachycardic. She was transferred to the ICU, where she was diagnosed and treated for urosepsis. Within 24 hours, she was hemodynamically stable and the sepsis syndrome resolved. She also has type 2 diabetes mellitus, hypertension, and hyperlipidemia. Her medications are insulin glargine, insulin aspart, lisinopril, atorvastatin, and piperacillin-tazobactam. On physical examination, the patient is alert. Temperature is 38.2 °C (100.8 °F), blood pressure is 90/62 mm Hg, pulse rate is 110/min, and respiration rate is 20/min. Abdominal examination is notable for tenderness to palpation of the right upper quadrant. A soft palpable mass is felt in this area. Laboratory studies show a leukocyte count of 20,000/µL (20 × 109/L) and a serum total bilirubin level of 5 g/dL (85.5 µmol/L). Cholecystostomy tube placement Endoscopic retrograde cholangiopancreatography Hepatobiliary iminodiacetic acid scan MR cholangiopancreatography
Cholecystostomy tube placement is the most appropriate next step in management for this patient. She has risk factors as well as clinical and radiologic findings that are consistent with acalculous cholecystitis. Cholecystectomy is the definitive treatment for acalculous cholecystitis in stable patients. However, this patient is now hemodynamically unstable and, therefore, requires a temporizing cholecystostomy tube to allow time for her to stabilize and for gallbladder inflammation to improve before cholecystectomy. Risk factors for acalculous cholecystitis include diabetes mellitus, sepsis, trauma, burns, vasculitis, cardiovascular disease, mechanical ventilation, and total parenteral nutrition. Acalculous cholecystitis can present with biliary colic symptoms in the alert patient or with unexplained leukocytosis, sepsis, and jaundice in the critically ill patient. On physical examination, a mass may be palpated in the right upper quadrant. Ultrasound may show some of the features seen in this patient (distended gallbladder with wall thickening and pericholecystic fluid), gas bubbles in the fundus ("champagne sign"), or nonvisualization of the gallbladder altogether. No stones or sludge are present in the gallbladder. Management includes bacterial cultures, intravenous antibiotics to cover gram-negative organisms, and cholecystostomy tube placement in the unstable patient or cholecystectomy in the stable patient. The role of endoscopic retrograde cholangiopancreatography with stenting is evolving. It has been used in case series to decompress the gallbladder in patients with acalculous cholecystitis, but it is not yet considered a first- or second-line treatment option due to less robust evidence to support this practice and the need for experienced endoscopists to perform the procedure. A hepatobiliary iminodiacetic acid scan may be used when ultrasonography is equivocal, and it would show nonopacification of the gallbladder in cases of cholecystitis. It is unnecessary in this patient who has clinical and radiologic features consistent with acalculous cholecystitis. This patient's ultrasonographic findings of a distended and thick-walled gallbladder with associated pericholecystic fluid and a lack of gallstones, in addition to her risk factors for acalculous cholecystitis, make further imaging, such as MR cholangiopancreatography, unnecessary and possibly dangerous due to the need for urgent gallbladder decompression.
Use for cryo-poor plasma?
Cryo-poor plasma is derived from fresh frozen plasma in which the cryoprecipitate has been separated. Its use is limited to refractory thrombotic thrombocytopenic purpura, although any advantage over fresh frozen plasma in this disorder has not been proven. It is not appropriate for treatment of most coagulopathies.
A 30-year-old man is evaluated for ongoing symptoms of dysphagia. He was previously diagnosed with eosinophilic esophagitis on upper endoscopy and has completed an 8-week course of swallowed aerosolized fluticasone, which did not alleviate his symptoms. He takes no other medications. On physical examination, vital signs are normal; BMI is 25. Other findings, including those of an abdominal examination, are unremarkable. Upper endoscopy shows an area of high-grade stenosis in the distal esophagus. Increase fluticasone Endoscopy with dilation Omeprazole Oral prednisone
Esophageal stricture in patients with eosinophilic esophagitis requires treatment with endoscopic dilation when symptoms do not respond to medical therapy. Endoscopy with dilation is the most appropriate treatment for this patient, who has eosinophilic esophagitis, refractory symptoms of dysphagia despite fluticasone therapy, and the finding of an esophageal stricture on endoscopy. Eosinophilic esophagitis can cause patients to develop a fibrostenotic esophageal stricture, which can be treated using endoscopy with dilation. Endoscopic dilation relieves the dysphagia but has no effect on underlying inflammation; therefore, medical therapy must be maintained. For unclear reasons, patients with eosinophilic esophagitis are more prone to mucosal tears with dilation than are patients with other stricturing diseases. It is imperative that the extent of dilation be limited in amount to avoid these complications; multiple dilations may be required to adequately treat the dysphagia. Most patients respond quickly after initiation of the fluticasone; therefore, continued or increased fluticasone alone will not alleviate the patient's dysphagia symptoms. Continued fluticasone may be necessary as maintenance therapy for this patient. Eosinophilic esophagitis is a chronic disease that often recurs after treatment is stopped; therefore, repeat or maintenance therapy may be needed. Omeprazole and other proton pump inhibitors (PPIs) are not effective in relieving dysphagia due to stricture. PPIs can reduce inflammation and eosinophil count and are often used before initiating fluticasone therapy to determine if the patient has PPI-responsive eosinophilic esophagitis. Limited data suggest that prednisone may be useful in patients with eosinophilic esophagitis who do not experience relief of symptoms with fluticasone therapy. However, like the swallowed aerosolized glucocorticoids, relapse is common when the medication is stopped, and relatively high doses are typically required, which carry associated risks of immunosuppression and other side effects. Additionally, this patient's esophageal stricture is fibrotic rather than inflammatory, so oral prednisone would not be effective for his dysphagia symptoms.
A 76-year-old man is evaluated for fever and a swollen, painful left knee. He was hospitalized 7 days ago for heart failure and appropriately treated. However, he developed fevers up to 38.0 °C (100.4 °F). On examination, lungs were clear to auscultation. The left knee was hot and swollen; 60 mL of turbid fluid was drained from the knee. Gram stain of the synovial fluid was negative; microscopy revealed needle-shaped intracellular crystals. Gout was diagnosed and the knee was drained again, followed by an injection of 80 mg of methylprednisolone, without improvement. Intravenous methylprednisolone, 60 mg/d for 3 days, did not improve the knee or the fevers. History is also significant for hypertension and gout. Other medications are furosemide, lisinopril, metoprolol, subcutaneous heparin, and morphine as needed. On physical examination today, temperature is 38.0 °C (100.4 °F), blood pressure is 148/92 mm Hg, pulse rate is 116/min, and oxygen saturation is 97% on ambient air. The left knee is warm, swollen, and tender. Blood, urine, and synovial fluid cultures are negative. Anakinra Colchicine Ibuprofen Vancomycin
For patients with severe and refractory gouty attacks or with contraindications to other treatments, off-label use of interleukin-1 inhibitors (anakinra or canakinumab) can be considered. Anakinra is the most appropriate treatment for this patient's knee. He has a persistent fever due to a severe acute gouty attack. Synovial fluid analysis permits definitive diagnosis and can rule out other conditions. Under polarized light, monosodium urate crystals are needle shaped and negatively birefringent. Whereas extracellular crystals confirm a chronic gout diagnosis, crystals within neutrophils define active, gout-induced inflammation. Even in the setting of acute gout, Gram stain and cultures must be obtained to exclude infection because acute gout and joint infection occasionally coexist. This patient has not responded favorably to glucocorticoid therapy, which characterizes some severe acute episodes. In a case such as this, an interleukin-1 inhibitor such as anakinra should be provided as a reliable (although expensive) off-label treatment. Colchicine is unlikely to be effective for an established attack of greater than 12 to 24 hours' duration. It is also unlikely to be useful when intravenous and intra-articular glucocorticoids have been ineffective. The absence of a response to intra-articular or intravenous glucocorticoids necessitates an alternative treatment; however, NSAIDs would not be as effective as the two previously provided failed alternatives. Additionally, NSAIDs have numerous side effects that might be even more likely to occur during critical illness, and they are relatively contraindicated in a patient with heart failure. Blood, urine, and synovial fluid cultures have all been negative, the pneumonia is resolving, and there is no clinical evidence of infection. Vancomycin for methicillin-resistant staphylococcus is unnecessary. Read Related TextNext Question
A 52-year-old woman undergoes perioperative evaluation. She has osteoarthritis of the right hip since sustaining injuries in a motor vehicle accident 15 years ago and is scheduled for elective hip arthroplasty in the next few months. Medical history is otherwise notable for type 2 diabetes mellitus. She is up to date on routine health care. Her last menstrual period was 5 weeks ago. Medications are ibuprofen and metformin. On physical examination, vital signs are normal. She has painful and limited range of motion in the right hip. Laboratory studies: Hemoglobin 10 g/dL (100 g/L) Mean corpuscular volume 81 fL Platelet count 223,000/µL (223 × 109/L) Creatinine 1 mg/dL (88.4 µmol/L) Hemoglobin A1c 7.5% Which of the following is the most appropriate test to perform next? Hemoglobin electrophoresis Iron studies Vitamin B12 level No further evaluation
Iron studies should be performed next. Preoperative anemia is associated with increased perioperative mortality in patients with cardiovascular disease; it is also a significant predictor for perioperative blood transfusion, which itself is associated with postoperative morbidity. The common causes for preoperative anemia are iron deficiency, vitamin B12 deficiency, chronic inflammatory disease, and chronic kidney disease. The hemoglobin level should be measured in the setting of signs or symptoms of anemia or surgery with a large expected blood loss at least 4 weeks before the surgery date. If anemia is identified, laboratory testing should begin with an assessment of iron status. Transferrin saturation less than 15% (or serum ferritin level less than 15 ng/mL [15 µg/L]) is consistent with iron deficiency anemia and should be treated with oral iron. If the response to oral iron is suboptimal because of patient adherence and surgical scheduling, intravenous iron should be used. Iron deficiency in this patient could be attributable to menstrual blood loss, occult gastrointestinal blood loss from NSAID-induced gastritis, or colon cancer. Evaluation and correction of the cause of iron deficiency anemia should take place before elective surgery. Hemoglobin electrophoresis is useful to detect genetic hemoglobinopathies such as sickle cell disease or thalassemia, which are associated with lifelong anemia unlikely to present for the first time at this patient's age. These conditions could be suspected with examination of a peripheral blood smear. Vitamin B12 deficiency is less common than iron deficiency in most patients scheduled to undergo orthopedic surgery, and it is typically associated with macrocytic anemia, which is not present in this patient. Ignoring this patient's anemia is not appropriate, considering the increased risk of perioperative allogeneic transfusion, which carries its own risks and is avoidable if the anemia is treated beforehand.
Fulminant C. diff infection tx?
Fulminant Clostridium difficile infections require oral vancomycin plus intravenous metronidazole; vancomycin enemas may also be added if ileus is present. The most appropriate treatment is oral vancomycin plus intravenous metronidazole. Treatment of Clostridium difficile infection (CDI) should be stratified according to its severity. The Society for Healthcare Epidemiology of America and the Infectious Diseases Society of America define severe CDI by a leukocyte count ≥15,000/µL (15 × 109/L) or a serum creatinine level >1.5 mg/dL (133 µmol/L). The American College of Gastroenterology defines severe CDI by the presence of hypoalbuminemia (<3 g/dL [30 g/L]) plus leukocytosis (≥15,000/µL [15 × 109/L]) or abdominal tenderness. Fulminant CDI is defined as severe infection complicated by ileus, hypotension, shock, or toxic megacolon. Medical therapy for fulminant CDI includes oral vancomycin plus intravenous metronidazole; vancomycin enemas may also be added if ileus is present. Complications are treated on an individual basis, although the presence of toxic megacolon requires surgical consultation to determine if emergent colectomy is indicated. Fecal microbiota transplant is effective for treatment of recurrent episodes of CDI, but it is not recommended for an initial episode of CDI, regardless of severity. Oral metronidazole alone for 10 days can be used for patients who present with an initial episode of nonsevere CDI when oral vancomycin or fidaxomicin is not available. Oral vancomycin alone for 10 days is recommended for treatment of an initial episode of nonsevere and severe CDI. Oral fidaxomicin can also be used. Oral metronidazole does not add benefit when used in addition to oral vancomycin. Read Related TextNext Question
ITP classifications?
ITP is characterized by the duration of the thrombocytopenia. Acute ITP lasts fewer than 3 months, persistent ITP lasts 3 to 12 months, and chronic ITP lasts longer than 12 months. Many patients are asymptomatic until the platelet count decreases to less than 10,000/µL (10 × 109/L).
A 68-year-old man is evaluated in the emergency department for fatigue and exertional dyspnea. He has a 5-year history of chronic lymphocytic leukemia, which has not required therapy. He takes no medications. On physical examination, temperature is 37 °C (98.6 °F), blood pressure is 123/82 mm Hg, pulse rate is 108/min, and respiration rate is 18/min. Oxygen saturation is 95% breathing ambient air. Cervical, axillary, and inguinal lymphadenopathy and splenomegaly are present. Laboratory studies: Hemoglobin 5 g/dL (50 g/L) Leukocyte count 35,000/µL (35 × 109/L) with 85% lymphocytes, 15% neutrophils Platelet count 180,000/µL (180 × 109/L) Reticulocyte count 10% of erythrocytes A direct antiglobulin test result is positive for IgG and C3. The patient is group A-positive and crossmatch incompatible with 5 units of group A-positive blood. Glucocorticoid therapy is started. Which of the following is the most appropriate management? Avoid transfusion until a compatible unit can be found Begin plasma exchange therapy Transfuse crossmatch-incompatible blood Transfuse O-negative uncrossmatched blood
In patients with severe symptomatic autoimmune hemolytic anemia, the autoantibody typically reacts against all erythrocytes, and a completely crossmatch-compatible unit may be impossible to find; these patients should receive ABO and Rh-matched blood even if it is not crossmatch compatible. The most appropriate management is to transfuse with crossmatch-incompatible blood. Although the goal of pretransfusion testing is to ensure the provision of ABO and Rh crossmatch-compatible blood, compatibility might not be achievable in autoimmune hemolytic anemia. Autoimmune hemolytic anemia is caused by the production of autoreactive antibodies against the patient's own erythrocytes. The direct antiglobulin test detects the presence of IgG or complement coating the patient's erythrocytes. In many patients who have IgG autoantibodies, the autoantibodies are of sufficient concentration in the serum or plasma to cause an incompatible crossmatch with donor erythrocytes because the autoantibody is directed against a core antigen on erythrocytes that is present not only on the patient's erythrocytes but also on all donor cells. Although the autoantibody will shorten the survival of transfused cells, they will still survive for days or even weeks and should provide effective therapy for problems related to acute anemia until immunosuppressive therapy becomes effective. A response to immunosuppressive therapy takes 1 to 2 weeks. The major risk in these patients is the failure to identify any additional alloantibody that may be present and could lead to more fulminant acute hemolytic transfusion reactions. In the absence of pregnancy or previous transfusion, the likelihood of non-ABO erythrocyte alloantibody development is extremely low. Therefore, a unit that is ABO and Rh matched is appropriate, even if crossmatch incompatible. It is not appropriate to withhold transfusion when a patient has severe anemia-related symptoms. Plasma exchange therapy is not effective for removing IgG antibodies because of a large extravascular space for IgG distribution that equilibrates back into the vascular pool as plasma is removed. Using "universal" group O-negative or group O-positive blood provides no advantage. It would still be crossmatch incompatible because of the underlying autoantibody. Uncrossmatched blood is reserved for situations of exsanguination in which blood transfusion is urgently required before "type and cross" can be completed.
Pathophys of AOCD?
Inflammatory anemia occurs in response to the inflammatory cytokine IL-6 that leads to hepatic synthesis of hepcidin, which in turn causes proteolysis of the membrane protein ferroportin. As a result, iron absorption from the gut is diminished, as is iron release from macrophages. Patients with chronic infections, such as tuberculosis or osteomyelitis; patients with malignancy; or patients with other chronic inflammatory conditions, such as rheumatologic diseases, can develop inflammatory anemia. Inflammatory anemia is characterized by a hemoglobin level of approximately 10 g/dL (100 g/L) with normocytic or slightly microcytic indices. Characteristically, the serum iron level is low, and the total iron-binding capacity (TIBC) is often low, as well. The serum ferritin level is elevated. In contrast, patients with iron deficiency have a low serum ferritin level and an elevated TIBC. Patients with inflammatory anemia do not require specific therapy. The use of erythropoiesis-stimulating agents, such as erythropoietin or longer acting darbepoetin, should not be used to treat inflammatory anemia in most patients with cancer because of the lack of benefit and accompanying risks, including hypertension, stroke, and tumor progression. Patients with inflammatory anemia do not respond to oral or parenteral iron supplementation.
A 71-year-old man is evaluated during a follow-up visit for sleep-related breathing pauses observed by the hospital staff when he was admitted for implantation of a cardioverter-defibrillator for ischemic cardiomyopathy (left ventricular ejection fraction of 30%). He has recently experienced dyspnea, a few episodes of which have awakened him from sleep. He has no insomnia or daytime sleepiness. He has dyslipidemia, stable coronary artery disease, and hypertension. Current medications are aspirin, atorvastatin, valsartan, metoprolol, and nitroglycerin as needed. On physical examination, vital signs are normal. Oxygen saturation is 93% breathing ambient air. BMI is 23. Lung examination reveals bibasilar crackles, faint end-expiratory wheezing, neck vein distention, and 1+ ankle edema. Polysomnography demonstrates central sleep apnea with a Cheyne-Stokes breathing pattern. Adaptive servo-ventilation Auto-adjusting positive airway pressure Furosemide Supplemental oxygen
Initial treatment of central sleep apnea should target modifiable risk factors; medical optimization of heart failure has been shown to improve central sleep apnea and Cheyne-Stokes breathing and should precede other therapies for sleep apnea. Diuresis with furosemide is the most appropriate treatment option. This patient has central sleep apnea with Cheyne-Stokes breathing in the setting of decompensated heart failure, a state of ventilatory instability. Cheyne-Stokes breathing is an abnormal respiratory pattern characterized by cyclic crescendo-decrescendo respiratory effort during sleep (and sometimes during wakefulness), in the absence of upper airway obstruction. Apnea accompanying the decrescendo effort defines central sleep apnea. The degree of central sleep apnea tends to correlate with left ventricular dysfunction. This patient has evidence on examination of volume overload (crackles and wheezing on lung exam, jugular venous distention, peripheral edema). Optimizing medical management of heart failure and improving fluid balance should precede other therapies for sleep apnea. Adaptive servo-ventilation is a form of positive airway pressure therapy initially designed as a treatment of Cheyne-Stokes breathing. However, a large multicenter trial unexpectedly showed increased mortality in a subset of patients with systolic heart failure (left ventricular ejection fraction less than 45%) and central sleep apnea treated with adaptive servo-ventilation. Auto-adjusting positive airway pressure is not an initial treatment for central sleep apnea. It is used to treat obstructive sleep apnea, where proprietary algorithms deliver varying pressure sufficient to prevent upper airway closure. Supplemental oxygen is sometimes used in advanced heart failure where impaired gas exchange results in hypoxemia. This patient has preserved oxyhemoglobin saturation. Small trials have studied the use of nocturnal supplemental oxygen in the setting of central sleep apnea, with variable results. Such treatment would be premature before optimization of fluid status. Read Related TextNext Question
Tx for toxic megacolon?
Intravenous fluid resuscitation, intravenous high-dose corticosteroids, and broad-spectrum antibiotics (for example, a third-generation cephalosporin with metronidazole) should be initiated in patients with toxic megacolon. Management requires close collaboration with a surgeon; therefore, emergent surgical consultation for consideration of subtotal colectomy is required because of the impending risk for perforation and peritonitis in patients with toxic megacolon. Some patients may respond to medical therapy with high-dose glucocorticoids (in addition to intravenous fluids and broad-spectrum antibiotics), but there should be a low threshold for surgical intervention due to the potential harms associated with toxic megacolon.
When do you treat Asymptomatic Bacteruria?
It is important to recognize that screening for and possibly treating asymptomatic bacteriuria is supported by only two indications: pregnancy and risk mitigation before an invasive urologic procedure. The use of prophylactic antibiotics before minor noninvasive urologic interventions without mucosal bleeding does not provide any benefit and is not recommended. Likewise, screening for and treating ASB in patients about to undergo orthopedic surgery, including total joint arthroplasty, is without proven merit because it is not a cause of postoperative surgical site infection. Data are insufficient to advocate the routine treatment of ASB in kidney transplant recipients or patients with diabetes.
A 65-year-old woman is evaluated for watery, nonbloody diarrhea of 8 months' duration. She reports up to six bowel movements per day with nocturnal awakenings, but no abdominal pain, bloating, flatulence, or weight loss. Her last screening colonoscopy at age 60 years was normal. She has no personal or family history of colon cancer or polyps. She also has osteoarthritis, and her only medication is ibuprofen approximately three times weekly. The patient is not exposed to children and has no recent travel history or engagement in outdoor recreational activities. On physical examination, vital signs are normal. The remainder of the physical examination, including abdominal examination, is normal. Laboratory studies show hemoglobin, serum albumin, total IgA, and tissue transglutaminase IgA levels to be within normal limits. Giardia lamblia infection Microscopic colitis Small intestinal bacterial overgrowth Ulcerative colitis
Microscopic colitis is a cause of nonbloody, watery diarrhea in older adults and is diagnosed by colonoscopy with random biopsies from multiple colonic segments. Based on the patient's age, sex, and clinical presentation, microscopic colitis is the most likely diagnosis. Microscopic colitis is the underlying cause in 10% to 15% of patients with chronic, watery diarrhea. In contrast to inflammatory bowel disease, microscopic colitis is more common in older persons and does not cause endoscopically visible inflammation. The symptoms of microscopic colitis are similar to other chronic causes of nonbloody diarrhea, such as celiac disease and irritable bowel syndrome; therefore, colonic mucosal biopsies are required for diagnosis. Lymphocytic and collagenous colitis are the two subtypes of microscopic colitis, and they are distinguishable only by histology. Random biopsies from multiple colonic segments, including the right and left colon, are recommended to establish the diagnosis because the disease can be patchy. In some patients, certain medications (such as NSAIDs and proton pump inhibitors) have been implicated as causative agents. Microscopic colitis is associated with other autoimmune diseases such as diabetes mellitus and psoriasis. The association with celiac disease is of particular clinical importance because the symptoms of these conditions are similar. Therefore, in patients with celiac disease or microscopic colitis whose symptoms do not respond to appropriate therapy, the other condition must be ruled out.
Post Lyme Disease Syndrome description?
PLDS, a poorly understood sequela of Lyme disease that sometimes is misclassified as "chronic Lyme disease" despite a lack of microbiologic evidence of a persistent viable organism. PLDS is thought to be due to a disordered immunologic response to the preceding infection. Most patients slowly improve over a period of 6 months, and treatment is directed toward symptom amelioration. Randomized controlled trials have shown that patients with PLDS do not respond to prolonged courses of antibiotic therapy, and such treatment is not warranted in this population.
Pathophys of how multiple myeloma --> hypogammoglobulinemia?
Patients with multiple myeloma are at risk for acquired hypogammaglobulinemia from a decrease in normal plasma cells and the effects of treatment. Patients with acquired hypogammaglobulinemia without recurrent infections should not be offered IVIG replacement. Prophylactic antibiotics are also an option in some patients who have recurrent infections with susceptible organisms. In addition to the cost of infusion, IVIG has numerous adverse effects, including anaphylaxis, serum sickness, kidney failure, hypertension, and headache. In patients with recurrent infections, IVIG provides protection from infection by providing passive immunity; the product is collected from healthy donors who likely have antibodies against the causative organisms. In this patient with recurrent sinus infections, a trial of IVIG should be given. Although efficacy is uncertain, vaccination against influenza, pneumococcus, and Haemophilus influenzae is also recommended. Prophylactic acyclovir is often recommended for patients receiving bortezomib therapy (increased risk for herpes zoster) after autologous stem cell transplantation and for patients with recurrent herpetic infections. It is not provided routinely to patients with multiple myeloma to prevent infection and is unlikely to be of benefit in this patient with recurrent bacterial sinusitis. Granulocyte colony-stimulating factor is used in conjunction with certain myelotoxic chemotherapies as infection prophylaxis and in some patients with severe neutropenia, such as severe congenital neutropenia. It is not appropriate in this patient with mild neutropenia. Patients with myeloma are at increased risk for bacterial infection owing to impaired lymphocyte and plasma cell function and hypogammaglobulinemia. The most commonly encountered organisms are Streptococcus pneumoniae, Haemophilus influenzae, and Escherichia coli presenting as sinusitis, pneumonia, and urinary tract infections. Routine prophylaxis against fungal infection with fluconazole is not beneficial.
A 78-year-old woman is evaluated for a 2-year history of gout with progressively more frequent and severe attacks. She currently has pain and swelling in the right second finger. History is also significant for hypertension, chronic kidney disease, nephrolithiasis, and type 2 diabetes mellitus. Medications are lisinopril, furosemide, metformin, and the maximal dose of febuxostat; she is allergic to allopurinol. On physical examination, vital signs are normal. The joint findings are shown. Laboratory studies show an erythrocyte sedimentation rate of 76 mm/h, a serum creatinine level of 1.5 mg/dL (132.6 µmol/L), and a serum urate level of 6.3 mg/dL (0.37 mmol/L). Add probenecid Stop febuxostat; begin pegloticase infusions Stop lisinopril; begin losartan
Pegloticase is strongly recommended for patients with severe recurrent and/or tophaceous gout that is intolerant or resistant to standard therapies. In addition to stopping febuxostat, the most appropriate treatment is pegloticase infusions. This patient has severe recurrent and tophaceous gout that has been resistant to standard therapies, including febuxostat. Pegloticase is an intravenously administered porcine-derived uricase (infused every 2 weeks), which reduces serum urate to nearly zero within hours of administration. If anti-drug antibodies do not form, tophi may resolve over the course of months. Other urate-lowering therapies should be discontinued with initiation of pegloticase because they can mask the development of antibodies that manifest as rising serum urate levels. Patients starting pegloticase should be placed on prophylaxis to prevent acute gout attacks; colchicine, prednisone, or NSAIDs are appropriate. In this case, glucocorticoids and NSAIDs should be avoided because of the concomitant type 2 diabetes mellitus and chronic kidney disease; therefore, low-dose colchicine is the appropriate prophylactic agent. In February 2019, the FDA mandated a boxed warning for febuxostat regarding the increased risk for cardiovascular death and all-cause mortality with the drug. The FDA has also limited the approved use of febuxostat for patients who are unresponsive to or cannot tolerate allopurinol. It is important to note that two recently published guidelines differ regarding the role of pharmacologic urate-lowering therapy in patients with gout. The 2016 American College of Physicians guideline notes a lack of evidence supporting a specific target level for urate lowering; this guideline stresses discussing the risks and benefits of urate-lowering therapy with patients and suggests a "treat to avoid symptoms" approach without specifically considering the serum urate levels. The 2020 American College of Rheumatology (ACR) guideline for the management of gout strongly recommends continuing urate-lowering therapy with xanthine oxidase inhibitors (allopurinol, febuxostat) to achieve and maintain a serum urate target less than 6 mg/dL (0.37 mmol/L). The guideline also strongly recommends switching from xanthine oxidase inhibitors to pegloticase in patients with gout who have not achieved the serum urate target and who have frequent gout flares or nonresolving tophi. The patient's serum urate level is 6.3 mg/dL (0.37 mmol/L) on febuxostat therapy, and she continues to have symptoms and tophi; therefore, escalation of treatment is appropriate according to both the ACP and ACR guidelines. Probenecid is not appropriate for those with an estimated glomerular filtration rate of less than 60 mL/min/1.73 m2 or with a history of kidney stones; this patient has both. Losartan has a modest uricosuric effect, but not significant enough that it would be recommended in a patient with severe tophaceous gout, especially one who remains symptomatic with an elevated serum urate level despite febuxostat therapy.
A 78-year-old man is evaluated in the emergency department for worsening jaundice of 2 weeks' duration, dry mouth, dark urine, and light stools. He has also noticed swelling under his jaw bilaterally. He reports no abdominal pain or weight loss. On physical examination, vital signs are normal. Swollen submandibular glands bilaterally and jaundice are noted. Laboratory studies: Alkaline phosphatase 180 U/L Alanine aminotransferase 66 U/L Aspartate aminotransferase 55 U/L Total bilirubin 6.2 mg/dL (106.0 µmol/L) Direct bilirubin 4.8 mg/dL (82.1 µmol/L) A CT scan of the abdomen shows a narrowed pancreatic duct and enlargement of the pancreas parenchyma, described as a "sausage-shaped" pancreas, as well as findings consistent with retroperitoneal fibrosis. Endoscopic ultrasound-guided biopsy of the pancreas shows more than 10 IgG4-positive cells/hpf and no evidence of malignancy. Which of the following is the most appropriate treatment? Azathioprine Endoscopic retrograde cholangiopancreatography with bile-duct stenting Pancreaticoduodenectomy Prednisone
Prednisone is the most appropriate treatment for this patient. Based on his typical symptom of painless jaundice and the characteristic "sausage-shaped" pancreas on imaging, the patient has type 1 autoimmune pancreatitis, a frequent manifestation of IgG4 disease. He also has associated IgG4-related conditions, sialadenitis, and probable retroperitoneal fibrosis. Autoimmune pancreatitis is rare and has an unclear pathogenesis. Type 1 autoimmune pancreatitis is a systemic fibroinflammatory disease, defined as an inflammatory condition causing tissue damage and scarring. Pancreatic involvement is only one manifestation of IgG4 disease, which is characterized by abundant IgG4-producing plasma cells seen on tissue biopsy. Most IgG4-related conditions are characterized by plasma-cell infiltration of the affected tissue and the clinical consequences that infiltration entails. Almost any organ can be involved; lymph nodes are frequently affected. The initial treatment is oral prednisone with a taper over 2 to 3 months, and symptoms usually resolve within 2 to 4 weeks with treatment. Almost all patients (>90%) enter clinical remission in response to glucocorticoids. Treatment response may be limited by the amount of fibrosis present before initiation of therapy. The relapse rate is approximately 30% in type 1 autoimmune pancreatitis. Azathioprine is an immunosuppressive drug that has been used to treat relapsing IgG4-related disease and can be used as a glucocorticoid-sparing agent. It may take 6 to 8 weeks to reach a therapeutic drug level and provoke a clinical response. Therefore, it is not considered initial first-line therapy for symptomatic IgG4-related disease. Endoscopic retrograde cholangiopancreatography with bile-duct stenting is usually not required in patients with autoimmune pancreatitis because most patients' symptoms respond quickly to oral prednisone. Glucocorticoid therapy also treats the underlying immunologic disorder, making it the preferred first-line treatment. Pancreaticoduodenectomy (Whipple surgery) is not indicated in patients with autoimmune pancreatitis. It may be considered to treat pancreatic adenocarcinoma occurring in the head of the pancreas, which can be mistaken for autoimmune pancreatitis. Because this patient's histology findings confirm IgG4-related disease, a trial of immunosuppressive therapy is the most appropriate treatment.
A 72-year-old man is evaluated in the emergency department after falling when his leg gave way as he tried to arise from bed. He has left hip pain, with the inability to stand and pain at rest. He was recently diagnosed with lymphoma, for which he is receiving chemotherapy. History is significant for a left hip replacement 7 years ago for osteoarthritis. His chemotherapy regimen consists of rituximab plus hyperfractionated cyclophosphamide, vincristine, doxorubicin, and dexamethasone. On physical examination, temperature is 38.2 °C (100.8 °F); other vital signs are normal. The right upper chest is implanted with a venous access port. Warmth and tenderness around the left hip are noted. Pain in the groin is noted. There is limitation of motion in all directions on both active and passive range of motion of the left hip. There are no other joint abnormalities. Laboratory studies show an erythrocyte sedimentation rate of 73 mm/h, a leukocyte count of 13,400/µL (13.4 × 109/L), and a serum urate level of 8.2 mg/dL (0.48 mmol/L). Left hip radiographs show periprosthetic lucency. Gout flare Hemarthrosis Hip dislocation Prosthetic joint infection
Prosthetic joint infection is the most likely diagnosis. This patient has a prosthetic hip and is currently undergoing chemotherapy. Malignancy and immunosuppression are risk factors for prosthetic joint infection, and this patient also has a central venous catheter that increases his risk for bacteremia. The elevated leukocyte count and erythrocyte sedimentation rate, along with periprosthetic lucency on radiographs, are all suggestive of prosthetic joint infection. Prosthetic joint infections are divided into early onset (<3 months after placement), delayed (3 to 24 months postsurgery), and late onset (>24 months after placement). Early and delayed infections are usually related to surgical contamination at the time of the implantation, whereas late infections result from hematogenous seeding of the joint. Early and late prosthetic joint infections typically present with pain, warmth, effusion, and fever. Staphylococcus aureus is a leading causative agent in prosthetic joint infections; treatment involves prompt intravenous antibiotics and sometimes removal of the prosthesis. Gout flare may also cause joint pain and loss of mobility, as well as fever, elevated leukocyte count, and inflammatory markers. However, this patient has no prior history of gout, and the hip is an uncommon place for a gout flare, especially an initial flare. Most initial gout flares affect the foot and/or ankle joints. Hemarthrosis causes bleeding into a joint and is characterized by pain, swelling, warmth, and impaired mobility. It is diagnosed by synovial fluid aspiration. This patient does not have a history or findings suggestive of a bleeding diathesis to put him at risk for a traumatic hemarthrosis. A patient with hip dislocation may present in a similar manner as one with prosthetic joint infection, namely pain and difficulty moving the joint; however, a dislocation would be evident on radiographs.
A 67-year-old man is evaluated for a chronic, nonhealing ulcer on his left foot of 3 months' duration. The patient states he was at a local sauna when he sustained an abrasion to the bottom of his foot after stepping on a sharp object. He subsequently developed an ulcer at the site of the injury and received several courses of antibiotics, including trimethoprim-sulfamethoxazole, doxycycline, and cephalexin, with no improvement. The ulcer continues to expand in size and deepen. Medical history is notable for hypertension and a 40-pack-year smoking history. His only medication is hydrochlorothiazide. On physical examination, the vital signs are normal except for a temperature of 37.3 °C (99.2 °F). The legs and feet are without edema or discoloration. Pedal and popliteal pulses are symmetrical and intact. Lower extremity sensation is intact. He has a 2- × 2-cm ulcerated lesion on the plantar aspect of the metatarsal region of the great toe, with surrounding erythema, yellowish discharge, and firm edges. Which of the following is the most likely cause of his ulcer? Mycobacterium avium Mycobacterium fortuitum Mycobacterium kansasii Mycobacterium leprae
Rapidly growing, nontuberculous mycobacteria, such as Mycobacterium fortuitum, can produce chronic, nonhealing wounds that do not respond to conventional antimicrobial therapy. This patient most likely has an infection with Mycobacterium fortuitum. M. fortuitum is one of the rapidly growing, nontuberculous mycobacteria (NTM) capable of producing chronic, nonhealing wounds anywhere the bacteria has been introduced, including skin, soft tissue, surgical sites, and occasionally on prosthetic devices. Although M. fortuitum is not part of the differential diagnosis of acute wound infections, NTM should always be considered in chronic wounds, especially when conventional antimicrobial therapy has been ineffective. To diagnose an NTM infection, a deep biopsy of chronic wound tissue should be performed. The biopsy specimen should be sent for histopathology to stain for bacteria, mycobacteria, and fungi, and a portion should be sent to the microbiology laboratory for similar stains and cultures. Pulmonary disease is the most common manifestation of NTM infection, and Mycobacterium avium complex (MAC) infection is the most common causative species. MAC is also responsible for most cases of NTM lymphadenitis. Disseminated MAC infection develops in patients with HIV who have CD4 cell counts less than 50/μL and are not receiving MAC prophylaxis. The clinical presentation often includes fever, night sweats, weight loss, and gastrointestinal symptoms. MAC would not be responsible for a solitary, nonhealing cutaneous ulcer. Mycobacterium kansasii most commonly causes a lung infection that mimics tuberculosis, with cough, fever, weight loss, and cavitary lung disease. Risk factors for infection include COPD, cancer, HIV, alcohol abuse, and drug-associated immunosuppression. M. kansasii does not cause isolated chronic skin or soft tissue infections. Leprosy is caused by the acid-fast bacillus Mycobacterium leprae, a slow-growing organism. Leprosy should be considered in the setting of chronic skin lesions that fail to respond to treatment of common skin conditions or when sensory loss is observed within lesions or in extremities. This patient's rapid development of a nonhealing ulcer after an initial injury 3 months ago is not compatible with infection caused by M. leprae.
A 29-year-old woman is evaluated during a new-patient appointment. She was diagnosed with hepatitis B virus (HBV) infection 10 years ago; her mother had HBV infection, and it was presumed that the patient acquired the infection at birth. She reports feeling well. Her medical history is otherwise unremarkable and she takes no medication. On physical examination, vital signs are normal, as is the remainder of the examination. Laboratory studies are positive for hepatitis B surface antigen and positive for hepatitis B e antigen. The serum HBV DNA level is 20,000,000 IU/mL. Alanine aminotransferase and aspartate aminotransferase levels are within normal limits.
Repeat liver chemistry testing in 6 months is the most appropriate next step in the management of this patient. The patient has hepatitis B virus (HBV) infection in the immune-tolerant phase, which can be determined by the likely vertical transmission and the patient's young age, positive hepatitis B e antigen (HBeAg), high viral load, and normal aminotransferase levels. Therefore, the patient only requires serial monitoring of aminotransferase levels. There are four typical phases of HBV infection: (1) immune tolerant, (2) immune active, (3) immune control (inactive), and (4) reactivation. Not all patients go through each phase. Patients with infection in the immune-tolerant phase do not have significant hepatic inflammation and have no fibrosis, and, therefore, do not require treatment. However, infection can progress to the immune-active, HBeAg-positive phase, in which hepatic inflammation, elevated aminotransferase levels, and fibrosis develop, underscoring the need for surveillance of aminotransferase levels. Patients with HBV infection in the immune-active, HBeAg-positive and reactivation, HBeAg-negative phases require treatment if the alanine aminotransferase level is elevated. Antiviral therapy is also required for patients who present with acute liver failure, all patients with cirrhosis, and patients undergoing treatment with certain immunosuppressive or chemotherapy regimens. None of these scenarios apply to this patient, so she does not require antiviral treatment such as tenofovir, entecavir, or pegylated interferon. Patients with HBV infection are at increased risk for hepatocellular carcinoma, even in the absence of cirrhosis. Patients from Southeast Asia should undergo hepatocellular carcinoma surveillance with ultrasonography starting at age 40 years for men and at age 50 years for women, and patients from sub-Saharan Africa should begin at age 20 years. Other indications include persistent inflammatory activity (defined as an elevated alanine aminotransferase level and HBV DNA levels greater than 10,000 IU/mL for at least a few years) and a family history of hepatocellular carcinoma. The preferred surveillance strategy is liver ultrasonography with or without α-fetoprotein measurement. This patient is not yet old enough to warrant hepatocellular carcinoma surveillance, so hepatic ultrasonography is not indicated.
A 25-year-old woman is evaluated for chronic intermittent nonbloody diarrhea with associated abdominal cramping, burping, and bloating. Symptoms began several months ago. She has a history of selective IgA deficiency with recurrent sinopulmonary infections. She has not taken antibiotics in the past 6 months. On physical examination, temperature is 37.3 °C (99.1 °F); the vital signs are otherwise normal. On abdominal examination, bowel sounds are present with minimal diffuse tenderness to palpation. Stool testing for occult blood is negative. Which of the following is the most likely cause of this patient's diarrheal illness? Clostridium difficile Enterohemorrhagic Escherichia coli Giardia lamblia Listeria monocytogenes Nontyphoidal Salmonella
This patient with selective IgA deficiency most likely has chronic diarrhea due to a Giardia lamblia infection. Typical symptoms of Giardia include watery diarrhea that is fatty and foul smelling, bloating, crampy abdominal pain, flatulence, and nausea; fever is uncommon. In immunocompetent hosts, Giardia infection symptoms typically resolve within 2 to 4 weeks, but in patients with humoral immunodeficiency, such as hypogammaglobulinemia or selective IgA deficiency, Giardia infection may be prolonged because of impaired protection against Giardia adherence to the intestinal epithelium. Patients with selective IgA deficiency have impaired humoral immunity but no impairment in neutrophil, T-cell, or complement function. Infectious complications of selective IgA deficiency typically include recurrent respiratory tract infections and chronic diarrhea caused by Giardia. Although Clostridium difficile can cause recurrent disease, this patient does not have a history of recent antibiotic use or any other risk factors for C. difficile infection such as advanced age, chemotherapy, gastrointestinal surgery, inflammatory bowel disease, or gastric acid suppression with proton pump inhibitors. Enterohemorrhagic Echerichia coli (EHEC) infection is usually spread by ingestion of undercooked meat or fecally contaminated food. EHEC typically presents with bloody acute diarrhea, crampy abdominal pain, and no fever. Listeria monocytogenes can cause an acute gastroenteritis syndrome associated with diarrhea, emesis, fever, headache, and nonbloody watery diarrhea associated with pain in muscles and joints. But such an infection typically lasts less than 2 days. Invasive complications of infection, including bacteremia and meningitis, are seen in conditions primarily associated with cell-mediated immune dysfunction such as pregnancy, use of glucocorticoids, and extremes of age (neonates or those older than 65 years). Nontyphoidal Salmonella is the most common cause of foodborne illness. Infection usually results from ingesting fecally contaminated water or food of animal origin. Symptoms are typically self-limited and include crampy abdominal pain, fever, headache, nonbloody diarrhea, nausea, and vomiting. Severe invasive disease may occur in patients with cell-mediated immunodeficiency, but the clinical presentation is not significantly altered in selective IgA deficiency.
How to treat NSTE-ACS?
Risk stratification tools, such as the TIMI risk score, can be used to determine which patients with NSTE-ACS should be treated with an invasive strategy versus an ischemia-guided approach. An early invasive strategy benefits patients with high TIMI risk scores (5-7) and intermediate TIMI risk scores (3-4). This patient has a TIMI risk score of 5, as indicated by the presence of three traditional risk factors for coronary artery disease, aspirin use within the last week, age older than 65 years, two or more angina episodes in the past 24 hours, and significant ST-segment deviation on electrocardiogram. His score places him at high risk for death and cardiac ischemic events, and despite the absence of elevated cardiac biomarker levels, urgent coronary angiography is warranted. -Immediate invasive treatment (within 2 hours) is recommended for patients with NSTE-ACS who have hemodynamic instability, refractory chest pain, heart failure, or ventricular arrhythmias. In patients with an elevated clinical risk score, significant ST-segment deviation, or elevated cardiac biomarkers, cardiac catheterization is usually performed within 24 hours of presentation. The type of revascularization procedure (PCI or CABG) depends on the results of angiography.
A 69-year-old man is evaluated for a 3-year history of dyspnea and chronic productive cough. He was diagnosed with COPD 2 years ago after spirometry confirmed severe airflow obstruction. He discontinued smoking at that time but in the past year he was treated for three COPD exacerbations, one requiring hospitalization. Medications are tiotropium, fluticasone/salmeterol, and albuterol inhalers. On physical examination, vital signs are normal; oxygen saturation is 92% on ambient air. He intermittently coughs during the examination. He has a prolonged expiratory phase. The remainder of the examination is unremarkable. Chest radiograph shows the lungs to be clear. Spirometry demonstrates a postbronchodilator FEV1 of 45% of predicted. Prednison Roflumilast Theophylline Trimethoprim-sulfamethoxazole
Roflumilast, a selective phosphodiesterase-4 inhibitor, is used as add-on therapy in severe COPD associated with chronic bronchitis and a history of recurrent exacerbations to reduce risk and frequency of exacerbations. Roflumilast is the most appropriate treatment. It is used primarily as add-on therapy in severe COPD associated with chronic bronchitis and a history of recurrent exacerbations despite other therapies; it has been shown to improve lung function and reduce risk and frequency of exacerbations in these individuals. However, it is not a bronchodilator, is expensive, and has not been shown to be effective in other groups of patients with COPD. Common side effects include diarrhea, nausea, weight loss, and headache. Recently the FDA has raised concerns regarding psychiatric adverse events with roflumilast (anxiety, depression, insomnia). Roflumilast is contraindicated in patients with liver impairment and has significant drug interactions. Oral glucocorticoids, such as prednisone, are reserved for limited periodic use in treating exacerbations of COPD and may provide some benefit in decreasing hospital readmission rates after exacerbation. Long-term oral glucocorticoid therapy has limited, if any, benefit in COPD and carries a high risk for other significant side effects (such as muscle weakness and decreased functional status) and is generally not recommended. Methylxanthines such as theophylline have shown modest treatment benefit in COPD, likely due to a bronchodilating effect mediated by nonselective inhibition of phosphodiesterase. However, the potential toxicity of this class of drugs coupled with their reduced efficacy has led to increasingly limited use. Although they may be helpful in any classification of COPD, they tend to be used in selected patients with late-stage disease or for patients in whom other preferred therapies have proved ineffective for symptomatic relief; they may also be used when other medications are not available or affordable. Clinical trials have demonstrated that chronic macrolide therapy is associated with a reduction in the rate of exacerbation in patients with moderate to severe COPD despite optimal maintenance inhaler therapy. Macrolide antibiotic therapy and roflumilast have not been directly compared in patients with frequent exacerbations of COPD and the choice among the two is informed by benefits and risks on an individual patient basis. Trimethoprim-sulfamethoxazole has not been shown to prevent exacerbations of COPD.
Diagnostic criteria for IBS? Treatments for IBS?
The diagnosis of IBS requires symptoms of recurrent abdominal pain or discomfort at least 1 day a week for a period of 3 months, along with two of the following three additional criteria: pain relieved by defecation, change in stool frequency, or change in bowel consistency. IBS can then be further subtyped into IBS with predominant constipation (IBS-C), predominant diarrhea, mixed bowel habits, or unclassified. The effects of dietary FODMAPs may contribute to the symptoms of IBS-D. A randomized controlled trial involving 84 adults with IBS-D compared a low-FODMAP diet to a diet based on modified National Institute for Health and Care Excellence (mNICE) guidelines. In this study, more patients on the low-FODMAP diet reported adequate relief of their IBS-D symptoms (52% versus 41%) and response in abdominal pain (51% versus 23%) than those on the mNICE diet. Alosetron is a peripherally acting serotonin type 3-receptor antagonist approved by the FDA for the treatment of IBS-D in women aged 18 years or older after failure of conventional therapy. Due to the risk for adverse events with the use of alosetron, including serious complications of constipation and ischemic colitis, a prescriber must first complete an FDA-mandated Risk Evaluation and Mitigation Strategy training program that is available online (www.alosetronrems.com). Alosetron is not FDA-approved for the treatment of men with IBS-D due to the small number of men involved in the pivotal clinical trials, and it would not be appropriate as an initial treatment for IBS in this male patient. Linaclotide is a peripherally acting guanylate cyclase-C activator approved by the FDA for the treatment of IBS-C. Lubiprostone is a peripherally acting chloride channel activator that is approved by the FDA for the treatment of IBS-C in women aged 18 years or older. In this patient whose predominant bowel symptom is diarrhea, use of either of these agents will worsen his diarrhea and is therefore not indicated. Read Related TextNext Question
A 32-year-old man is evaluated in the hospital for symptoms of persistent asthma. He was evaluated in the emergency department 2 days ago for dyspnea accompanied by wheezing, dysphonia, and upper chest and throat tightness. Symptoms persisted despite use of albuterol inhaler every 3 to 4 hours and intravenous methylprednisolone, and he was hospitalized. He was diagnosed with asthma in high school and generally requires several courses of prednisone per year. Current medications are albuterol and fluticasone/salmeterol inhalers, prednisone, and montelukast. On physical examination, blood pressure is 130/85 mm Hg, pulse rate is 110/min, and respiration rate is 18/min. Oxygen saturation is 100% on 2 L/min of oxygen through nasal cannula. BMI is 25. Chest examination demonstrates monophonic wheezing on inspiration. Laboratory studies, including complete blood count, metabolic panel, and IgE , are normal. Chest radiograph is clear and bedside spirometry is normal. CT scan of the sinuses Increase prednisone dosage Laryngoscopy Polysomnography
The diagnosis of vocal cord dysfunction is suggested by midchest tightness with exposure to particular triggers such as strong irritants or emotions; difficulty breathing in; and symptoms that only partially respond to asthma medications. The most appropriate management is laryngoscopy. This patient has symptoms suggestive of vocal cord dysfunction, which is caused by paradoxical adduction of the vocal cords during inspiration, leading to functional upper airway obstruction. The diagnosis is suggested by dysphonia midchest or throat tightness with exposure to particular triggers such as strong irritants or emotions; difficulty breathing in; and symptoms that only partially respond to asthma medications. Patients may also experience midchest tightness, dyspnea, cough, and dysphonia, and stridor may be detected as inspiratory monophonic wheezing. Vocal cord dysfunction is commonly misdiagnosed as asthma, leading to excessive health care use. Diagnosis is ideally made by visualization of the abnormal vocal cord adduction during laryngoscopy. It may also be diagnosed if spirometry happens to capture a flat inspiratory limb on the flow-volume loop. However, if patients are unable to tolerate laryngoscopy while symptomatic, empiric therapy should be started if there is a high clinical suspicion of vocal cord dysfunction. Treatment consists of speech therapy utilizing cognitive behavioral techniques. Because disorders affecting the upper respiratory tract may affect the lower tract, sinus disorders may be associated with worsening control of asthma. Patients with frequent asthma exacerbations should be evaluated for occult sinus disease, as untreated upper airway inflammation may contribute to poor asthma control. However, CT scan of the sinuses is not appropriate because sinus disease cannot explain the patient's upper chest and throat tightness, dysphonia, inspiratory monophonic wheezing, or unresponsiveness to intensive asthma treatment. Increasing the prednisone dose is not likely to improve the patient's symptoms, which are not likely or entirely due to airway inflammation and will expose the patient to more potential side effects of glucocorticoids. Asthma has been associated with obstructive sleep apnea (OSA). In difficult-to-control asthma, OSA is a significant risk factor for frequent exacerbations. Treatment of OSA improves asthma symptoms. However, this patient has no obvious risk factors for OSA and OSA cannot explain the patient's upper chest and throat tightness, dysphonia, or inspiratory monophonic wheezing. Therefore polysomnography is not indicated.
A 55-year-old man is hospitalized for acute respiratory failure requiring intubation and mechanical ventilation. He also has increasing leg swelling with nonhealing skin ulcers on the legs for the past 4 weeks, and pain with swelling of the bilateral wrists and finger joints for the past 6 weeks. He quit smoking 15 years ago. History is otherwise unremarkable. Prior to hospitalization he was taking no medications. On physical examination, temperature is 37.2 °C (99.0 °F), blood pressure is 150/95 mm Hg, pulse rate is 110/min, respiration rate is 20/min (ventilator set rate, 14/min), and oxygen saturation is 92% on FIO2 of 40%. There is blood in the endotracheal tube. Lung crackles are heard bilaterally. Swelling of the wrists, metacarpophalangeal joints, and proximal interphalangeal joints is noted. Two necrotic ulcers on the left leg and one necrotic ulcer on the right leg are present. There is 2+ pitting edema of the legs. Laboratory studies show normal C3, C4, and rheumatoid factor; negative antinuclear antibodies; positive ANCA with a perinuclear pattern; and 3+ protein on urinalysis. Blood and sputum cultures are negative. Chest radiograph shows diffuse bilateral infiltrates. Biopsy of a skin ulcer shows nongranulomatous, necrotizing small-vessel vasculitis with immunofluorescence negative for immune complexes. Granulomatosis with polyangiitis IgA vasculitis Microscopic polyangiitis Rheumatoid vasculitis Thromboangiitis obliterans
The diagnostic gold standard of microscopic polyangiitis is a biopsy demonstrating nongranulomatous necrotizing pauci-immune vasculitis of small vessels or pauci-immune necrotizing crescentic glomerulonephritis in the kidney. The most likely diagnosis is microscopic polyangiitis (MPA), which characteristically affects the lungs and kidneys, along with other organ systems. Diagnosis is suspected based upon typical clinical findings and positive ANCA. The diagnostic gold standard is a biopsy demonstrating nongranulomatous necrotizing pauci-immune vasculitis of small vessels or pauci-immune necrotizing crescentic glomerulonephritis in the kidney. This patient has pulmonary hemorrhage, pauci-immune small-vessel necrotizing vasculitis of the skin, proteinuria, inflammatory arthritis, and a positive p-ANCA (directed against myeloperoxidase)—all consistent with the diagnosis of MPA. Granulomatosis with polyangiitis (GPA) can cause the same lung, skin, joint, and kidney findings but is usually associated with a positive c-ANCA (directed against proteinase-3). Furthermore, GPA causes granulomatous inflammation; absence of granulomas distinguishes MPA from GPA. IgA vasculitis (Henoch-Schönlein purpura) can affect lungs, skin, joints, and kidneys but it is an immune complex-mediated vasculitis with deposition of IgA containing immune complexes identified on biopsy, which is not present in this patient. Rheumatoid vasculitis usually occurs in long-standing disease with positive rheumatoid factor and is immune complex mediated, none of which is present in this patient. Thromboangiitis obliterans is a smoking-related vasculopathy affecting small to medium vessels with inflammation and thrombosis of vessels in upper and lower limbs, leading to reduced pulses and gangrenous ulcers. This diagnosis is unlikely because this patient no longer smokes, and has involvement of internal organs in addition to the legs.
A 25-year-old woman is evaluated in the hospital for right-upper-quadrant abdominal pain, jaundice, and nausea of 10 days' duration. She is in her 35th week of pregnancy. Her only medication is a prenatal vitamin. On physical examination, the patient is drowsy. Temperature is normal. Blood pressure is 95/60 mm Hg, pulse rate is 108/min, and respiration rate is 22/min. Jaundice is apparent. Abdominal examination shows tenderness to palpation in the right upper quadrant. The uterus is of appropriate size for gestation. Laboratory studies: Hematocrit 34% Leukocyte count 6000/μL (6 × 109/L) Platelet count 155,000/μL (155 × 109/L) INR 2.2 Alanine aminotransferase 115 U/L Aspartate aminotransferase 130 U/L Total bilirubin 6.2 mg/dL (106.0 µmol/L) Glucose 55 mg/dL (3.1 µmol/L) On abdominal ultrasonography, the liver is hyperechoic. Hepatic vasculature is patent, and there is no bile-duct dilation. She is transferred to an ICU setting, and intravenous fluids with glucose are administered. Endoscopic retrograde cholangiopancreatography Immediate delivery of the fetus Lactulose Ursodeoxycholic acid
The fetus should be delivered immediately upon recognition of acute fatty liver of pregnancy. Immediate delivery of the fetus is the most appropriate next step in management. This patient has findings of acute fatty liver of pregnancy, which is a rare but serious condition occurring most commonly in the third trimester. Women with this condition typically present with a 1- to 2-week history of nausea and vomiting, right-upper-quadrant or epigastric pain, headache, jaundice, anorexia, and/or polyuria and polydipsia (due to associated transient diabetes insipidus). Maternal and neonatal mortality rates are high in this setting. This patient's presentation with coagulopathy, hypoglycemia, and somnolence consistent with hepatic encephalopathy are indications of acute liver failure, which can result from acute fatty liver of pregnancy. Immediate delivery of the fetus is indicated to prevent fetal mortality and to reverse the mother's liver failure and improve her condition. Prompt delivery typically results in improvement of the mother's medical condition within 48 to 72 hours. Acute fatty liver of pregnancy can reoccur in subsequent pregnancies. It is also associated with long-chain 3-hydroxyacyl CoA dehydrogenase deficiency, and affected women and their children should be screened for this deficiency. HELLP (Hemolysis, Elevated Liver enzymes, and Low Platelets) syndrome can also occur in the third trimester of pregnancy. It has similarly life-threatening consequences and also requires emergent delivery of the fetus to resolve the condition. HELLP syndrome can present similarly to acute fatty liver of pregnancy in that manifestations of liver failure are present. This patient's ultrasound showed no dilation of bile ducts, and no stones were seen in the common bile duct; therefore, endoscopic retrograde cholangiopancreatography is not indicated. This patient's drowsiness is due to impending liver failure. Lactulose, which is used for treatment of hepatic encephalopathy in patients with chronic liver disease, is not indicated for patients with acute liver failure because it may exacerbate symptoms of ileus, and there is no evidence of benefit in the acute setting. Any symptoms referable to hepatic encephalopathy will resolve with delivery of the fetus, which resolves acute fatty liver of pregnancy. Intrahepatic cholestasis of pregnancy occurs during the second or third trimester and resolves after delivery. The most common laboratory findings are elevated bilirubin and alkaline phosphatase levels. The condition is believed to result from sex hormone-induced inhibition of bile salt export from hepatocytes. It is treated with ursodeoxycholic acid, which can result in alleviation of symptoms. Although the maternal effects of intrahepatic cholestasis of pregnancy are mild, it can cause fetal distress and premature labor. Because this patient's clinical profile is not compatible with intrahepatic cholestasis of pregnancy, ursodeoxycholic acid therapy is not indicated.
A 30-year-old man is evaluated for worsening exertional dyspnea. One week ago, he developed fever, sore throat, and cough. Those symptoms have resolved, but he has become more easily fatigued and short of breath. He had cholecystectomy 2 years ago because of symptomatic cholelithiasis; at that time, he was noted to be anemic and was diagnosed with hereditary spherocytosis. His only medication is a folate supplement. On physical examination, he is pale but in no distress. Temperature is 37.0 °C (98.7 °F), blood pressure is 100/60 mm Hg, pulse rate is 116/min, and respiratory rate is 16/min. Oxygen saturation is 98% breathing ambient air. The spleen is palpable 3 cm below the left costal margin. Other examination findings are normal. Laboratory studies: Hemoglobin 7 g/dL (70 g/L) Leukocyte count 5600/µL (5.6 × 109/L), with a normal differential Mean corpuscular hemoglobin concentration 40 g/dL (400 g/L) Platelet count 213,000/µL (213 × 109/L) Reticulocyte count 1% of erythrocytes Bilirubin Total 6.2 mg/dL (106 µmol/L) Indirect 5.6 mg/dL (95.8 µmol/L) Spherocytes are seen on the peripheral blood smear. A direct antiglobulin test is negative. Which of the following is the most appropriate management? Erythropoiesis-stimulating agent Prednisone Splenectomy Observation
The most appropriate management for this patient is clinical observation. He has hereditary spherocytosis (HS) characterized by a mild lifelong anemia in association with symptomatic cholelithiasis at an early age. The presence of spherocytes is supported by an elevated mean corpuscular hemoglobin concentration frequently seen in this disorder. HS is caused by mutations in several scaffolding proteins that make these cells less distensible and more susceptible to osmotic stress and hemolysis. Patients with this disorder may have mild anemia, an elevated reticulocyte response, and few or no symptoms. The development of pigmented gallstones resulting from excess bilirubin production may result in symptomatic cholelithiasis. Symptoms of anemia may arise when the bone marrow is suppressed, most commonly by an acute infection. In this situation, the reticulocyte count falls, and the patient rapidly develops symptomatic anemia. Parvovirus is most classically linked with bone marrow suppression, but many other viral and infectious agents can have a similar effect. The bone marrow suppression following acute infections is self-limited, but some patients may become symptomatic to the point of requiring blood transfusion. A subset of patients with HS has chronic hemolytic anemia that is much more severe. These patients will benefit from splenectomy. The spleen is the major site of erythrocyte destruction; after splenectomy, the membrane defect remains unchanged, but erythrocyte survival will be significantly prolonged. Splenectomy is not needed in managing an acute, self-limited aplastic crisis. Prednisone would be indicated to treat warm antibody autoimmune hemolytic anemia (WAIHA). Spherocytes are also seen in patients with WAIHA, but the direct antiglobulin test result would be positive. WAIHA would not explain the lifelong anemia in this patient. Erythropoietin deficiency plays no role in the pathogenesis of HS. Endogenous erythropoietin levels are apt to be markedly elevated in this patient, and no evidence indicates that therapy with exogenous erythropoiesis-stimulating agents has any beneficial role.
A 32-year-old man is evaluated in the hospital for a 4-day history of fever and leukocytosis that began 1 week after cranial surgery and a 1-day history of worsening mental status. He experienced traumatic intracranial hemorrhage requiring a craniotomy for drainage and placement of an external ventricular drain. He remains intubated with mechanical ventilation. Medications are empiric vancomycin and cefepime, initiated 4 days ago. On physical examination, temperature is 39.2 °C (102.5 °F), blood pressure is 108/62 mm Hg, pulse rate is 102/min, and respiration rate is 20/min. He was previously awake and able to follow simple commands, but now he is obtunded. Nuchal rigidity is noted. Other examination findings are unremarkable. Cerebrospinal fluid (CSF) evaluation shows a leukocyte count of 134/µL (134 × 106/L) with 50% neutrophils, erythrocyte count of 10,000/µL (10,000 × 106/L), glucose level of 35 mg/dL (1.9 mmol/L), protein level of 112 mg/dL (1120 mg/L), and elevated lactate level of 5.4 mg/dL. Blood cultures taken at onset of fever are negative, as are CSF cultures. A chest radiograph shows no infiltrates. Which of the following is the most appropriate management? Continue current care Discontinue vancomycin and cefepime and begin linezolid Remove the external ventricular drain Switch to intraventricular administration of antibiotics
The most appropriate management in this patient is removal of his ventricular drain. After drain removal, clinical monitoring for intracranial hypertension will be important. Health care-associated ventriculitis or meningitis (HCAVM) or nosocomial meningitis may present a diagnostic and treatment challenge to clinicians. It typically occurs after head trauma or a neurosurgical procedure (craniotomy, lumbar puncture) or secondary to device infection (for example, cerebrospinal fluid [CSF] shunts or drains, intrathecal pumps, deep brain stimulator). Staphylococcus species and enteric gram-negative bacteria are the most common causes, but up to 50% of patients can have negative cultures because more than 50% receive antibiotic therapy before CSF studies are performed, as occurred in this patient. Worsening mental status, new fever, or stiff neck in a patient who recently underwent surgery should raise the possibility of HCAVM. An elevated CSF lactate level greater than 4 mg/dL, an elevated CSF procalcitonin level, or a combination of both, may be useful in the diagnosis of health care-associated bacterial ventriculitis and meningitis in culture-negative cases. Empiric therapy should include vancomycin and a β-lactam with antipseudomonal activity (such as cefepime or meropenem) and device removal, if present. For patients with health care-associated ventriculitis and meningitis caused by staphylococci in whom β-lactam agents or vancomycin cannot be used, treatment with linezolid, daptomycin, or trimethoprim-sulfamethoxazole is recommended with selection based on in vitro susceptibility testing. However, the priority in this patient is removing the ventricular drain, not changing the antibiotic regimen. Intraventricular or intrathecal antimicrobial therapy should be considered for patients with health care-associated ventriculitis and meningitis in whom the infection responds poorly to systemic antimicrobial therapy alone. This strategy is not appropriate in a patient with a potentially infected ventricular drain.
A 68-year-old man is being evaluated for measures to decrease his risk of acquiring a surgical site infection; he is scheduled for coronary artery bypass graft surgery in 5 weeks for limiting chronic angina despite maximal medical therapy. Medical history includes chronic stable angina, hyperlipidemia, hypertension, and diabetes. Medications are low-dose aspirin, propranolol, isosorbide dinitrate, ranolazine, chlorthalidone, lisinopril, and atorvastatin. On physical examination, blood pressure is 126/72 mm Hg; all other vital signs are normal. On cardiac examination, an S4 is present. The remainder of the examination is noncontributory. Which of the following is the most appropriate measure to prevent surgical site infection? Evaluate for Staphylococcus aureus nasal carriage Provide postoperative vancomycin prophylaxis for 7 days Provide preoperative vancomycin prophylaxis Shave patient's chest hair the morning of surgery
The most appropriate measure to prevent surgical site infection is to evaluate for Staphylococcus aureus nasal carriage 2 weeks before surgery and decolonize if positive. S. aureus is the most common pathogen (23%) associated with surgical site infections (SSIs). SSIs after coronary artery bypass graft surgery can be serious and devastating, with mediastinitis related to S. aureus of particular concern. The 2016 World Health Organization guidelines recommend that patients known to be nasal carriers of S. aureus who are scheduled to undergo cardiothoracic or orthopedic surgery should have preoperative decolonization (mupirocin ointment for 5 days with or without chlorhexidine gluconate body wash) to decrease the risk of developing S. aureus-related SSI. Data do not support extending antibiotic prophylaxis beyond 24 hours after cardiac surgery even while drains remain in place. For most other surgeries, no additional doses of antibiotic should be given postoperatively, even in cases of intraoperative spillage of gastrointestinal contents. Postoperative antibiotics are only indicated when treating an active infection. Preoperative antibiotic prophylaxis reduces the risk of SSI by decreasing the concentration of pathogens at or around the incision site. The agent used and the timing of administration are key. For cardiac surgery, cefazolin is recommended unless a patient is known to have methicillin-resistant S. aureus colonization or has a severe (anaphylactic) β-lactam allergy, in which case vancomycin is used. For optimal benefit, the antibiotic should be administered 1 to 2 hours before incision. For procedures lasting more than several hours, the antibiotic should be redosed during surgery (for example, redose at 3-4 hours for cefazolin). Preoperative shaving in the area of the planned incision increases the risk of SSI. Shaving causes microscopic abrasions of the skin, which promotes bacterial proliferation. Recommendations indicate only removing hair from the surgical site if it will interfere with the procedure, in which case clipping is preferred.
A 35-year-old woman is evaluated after laboratory test results showed an elevated LDL cholesterol level during routine screening. Family history is remarkable for myocardial infarction in her father at age 45 years. She takes no medications. On physical examination, vital signs are normal. BMI is 30. The remainder of the examination is unremarkable. Laboratory studies: Alanine aminotransferase 30 U/L Thyroid-stimulating hormone Normal Total cholesterol 294 mg/dL (7.61 mmol/L) LDL cholesterol 195 mg/dL (5.05 mmol/L) HDL cholesterol 55 mg/dL (1.42 mmol/L) Triglycerides 220 mg/dL (2.49 mmol/L) The patient is instructed in therapeutic lifestyle changes to lower her risk for atherosclerotic cardiovascular disease (ASCVD). According to the American Heart Association/American College of Cardiology cholesterol treatment guideline, which of the following is the most appropriate additional treatment for primary prevention of ASCVD in this patient? Evolocumab High-intensity rosuvastatin Moderate-intensity atorvastatin No additional treatment is necessary
The most appropriate treatment for primary prevention of atherosclerotic cardiovascular disease (ASCVD) in this patient is high-intensity statin therapy with rosuvastatin or atorvastatin. According to the 2018 American Heart Association (AHA)/American College of Cardiology (ACC) Guideline on the Management of Blood Cholesterol, patients aged 20 years or older with severe LDL cholesterol elevation (≥190 mg/dL [4.92 mmol/L]) should receive the maximum tolerated statin therapy for primary prevention of ASCVD, regardless of 10-year risk for ASCVD. High-intensity statin therapy is recommended unless there are contraindications to its use. It is reasonable to intensify statin therapy as tolerated to achieve an LDL cholesterol reduction of at least 50%. In contrast to the AHA/ACC recommendation, the U.S. Preventive Services Task Force recommends initiating low- to moderate-intensity statin therapy in adults aged 40 to 75 years without a history of ASCVD who have one or more traditional ASCVD risk factors (dyslipidemia, diabetes mellitus, hypertension, or smoking) and a calculated 10-year ASCVD event risk of 10% or higher. Similarly, the U.S. Department of Veterans Affairs/U.S. Department of Defense cholesterol guideline recommends moderate-intensity statin therapy for patients with a 10-year ASCVD risk of 12% or more, an LDL cholesterol level of 190 mg/dL (4.92 mmol/L) or higher, or diabetes. In the absence of familial hypercholesterolemia or severe hypercholesterolemia despite maximally tolerated cholesterol-lowering therapy with a statin and ezetimibe, proprotein convertase subtilisin/kexin type 9 (PCSK9) inhibitors, such as alirocumab and evolocumab, are not indicated in the primary prevention of ASCVD. Cost, treatment burden (injections), and absence of long-term safety data argue against their use in primary prevention. Such treatment might be considered if LDL cholesterol cannot be sufficiently reduced in the highest-risk patients. In patients with an LDL cholesterol level of 190 mg/dL (4.92 mmol/L) or higher, initial treatment with a moderate-intensity statin is less preferred according to the AHA/ACC guidelines; however, if the patient is unable to tolerate high-intensity therapy, down-titration to moderate-intensity therapy could be considered, especially if adequate LDL cholesterol reduction can be achieved. An evaluation for secondary causes of hyperlipidemia is also indicated in patients with an LDL cholesterol level of 190 mg/dL (4.92 mmol/L) or higher. The most common secondary causes are obesity, hypothyroidism, biliary obstruction, and nephrotic syndrome. Medications can also increase LDL cholesterol level, and some of the most commonly implicated drugs include cyclosporine, HIV medications (such as protease inhibitors), glucocorticoids, and amiodarone. If a secondary cause is not identified, LDL cholesterol level elevation is considered primary, and family members should undergo screening because severe hypercholesterolemia is often genetically determined, as may be the case with this patient who has a first-degree relative with premature ASCVD.
A 39-year-old man is hospitalized for tongue pain, abdominal pain, increased weakness, and a 2-week history of malaise and fever. He reports being in good health previously. He lives in the Ohio River Valley; approximately 1 month ago, he moved his antique business from a barn to an old store in the area, after which he developed "flu-like" symptoms lasting 2 to 3 days. He says that the barn was dusty and had pigeons and bats in the rafters. He also has rheumatoid arthritis. Medications are methotrexate and prednisone. On physical examination, the patient is lethargic. Temperature is 39.7 °C (103.5 °F), blood pressure is 90/50 mm Hg, pulse rate is 128/min, and respiration rate is 24/min. A shallow ulceration is visible on the right buccal mucosa and left lateral tongue. His neck is supple. Lungs are clear to percussion and auscultation. There is moderate hepatosplenomegaly. Results of laboratory studies show a hemoglobin level of 9 g/dL (90 g/L), a leukocyte count of 10,500/µL (10.5 × 109/L), and a platelet count of 90,000/µL (90 × 109/L). Posteroanterior and lateral chest radiographs are unremarkable. Which of the following is the most appropriate treatment? Ceftriaxone and azithromycin Colchicine Itraconazole Liposomal amphotericin B
The most appropriate treatment for this patient is liposomal amphotericin B. He has disseminated histoplasmosis, for which he has numerous risk factors. He lives in an area endemic for histoplasmosis (Ohio River Valley), and because he takes prednisone and methotrexate for his rheumatoid arthritis, he is immunosuppressed. He has had significant exposure working in an old barn with bats. He also is hypotensive and diaphoretic and has oral ulcerations and hepatosplenomegaly, which are typical of disseminated infection, as is pancytopenia. Approximately 10% of patients with histoplasmosis develop disseminated infection; if not diagnosed early, the mortality rate is greater than 90%. The treatment of choice for disseminated histoplasmosis is liposomal amphotericin B initially, with de-escalation to itraconazole for several months. A definitive diagnosis can be established by detection of the urinary antigen for histoplasmosis (95% specificity), blood cultures, or a biopsy of the oral lesions. Ceftriaxone and azithromycin are the recommended drugs of choice for community-acquired pneumonia. The patient's physical examination and chest radiograph are unremarkable, which effectively rules out pneumonia and the need for these antibiotics. Behçet syndrome is a form of vasculitis associated with recurrent painful oral and genital ulcerations; patients also may demonstrate additional distinctive features, including hypopyon and pathergy. Central nervous system involvement can manifest as headaches, stroke, and behavioral changes. Gastrointestinal involvement may be hard to distinguish from inflammatory bowel disease. Low-dose prednisone or colchicine is used for oral or genital ulcers, and high-dose prednisone and immunomodulating agents are used for more severe disease. Behçet syndrome does not explain the patient's fever, hemodynamic instability, pancytopenia, or hepatomegaly, and colchicine therapy is not warranted. Itraconazole is an azole triazole used to treat many endemic fungal infections, including histoplasmosis. However, it is not as effective as liposomal amphotericin B in disseminated infection. It may be used for subacute or chronic histoplasmosis, such as pulmonary histoplasmosis.
A 23-year-old woman is evaluated for depression as she prepares for discharge from the hospital to home hospice care. She was diagnosed with metastatic ovarian cancer 2 years ago, and she progressed through four lines of chemotherapy, a trial of immunotherapy, and a failed attempt at a phase 1 clinical trial. Her life expectancy is measured in weeks. She is currently hospitalized with volume depletion, and after consultation with her oncologist and palliative care team, she has decided to be discharged home with hospice care. On physical examination, the patient exhibits substantial fatigue and poor concentration. She has a flat affect except when intermittently tearful. Previously upbeat despite all of the setbacks, she is now withdrawn and describes feeling hopeless. She has pervasive guilt over the burden she believes she has caused her family. Medications are a fentanyl patch, oxycodone, ondansetron, polyethylene glycol, senna, and zolpidem. Which of the following is the most appropriate treatment? Citalopram Cognitive behavioral therapy Methylphenidate Sertraline
The most appropriate treatment for this patient's depression is methylphenidate. Patients with a serious, life-threatening illness and untreated depression have poorer quality of life, which can lead to increased caregiver stress and burden. Diagnosing depression in terminally ill patients, however, is challenging. Although anticipatory grief is common in patients at the end of life and is considered a normal part of most end-of-life experiences, it can be distinguished from clinical depression by the patient's ability to find enjoyment and a fluctuating mood. Patients with depression at the end of life have symptoms that include hopelessness, pervasive guilt, and worthlessness. Depression in terminally ill patients responds well to both pharmacologic and nonpharmacologic treatments. Tricyclic antidepressants, selective serotonin reuptake inhibitors, serotonin-norepinephrine reuptake inhibitors, and mirtazapine are all effective agents. Prognosis should be taken into account because these medications take weeks to reach peak effect. This patient has symptoms consistent with clinical depression as well as a limited life expectancy. Methylphenidate is a rapid-acting psychostimulant that is well tolerated and may be effective in the treatment of depression; once initiated, results can be seen within 24 to 48 hours. Methylphenidate may also have the benefit of improving cancer-associated fatigue. Selective serotonin reuptake inhibitors, such as citalopram and sertraline, are effective in the treatment of depression; however, they can take many weeks and dose titration to reach effectiveness. Given this patient's limited life expectancy, a more rapid-acting agent is needed. Cognitive behavioral therapy, when available, is an effective therapy for patients with depression and a serious medical illness. However, most trials showing benefit are centered on multiweek, if not several-months-long, interventions and are of limited availability for patients on home hospice. Read Related TextNext Question
A 44-year-old man is evaluated in the emergency department in January for a 3-day history of dyspnea, cough, and fever. Medical history is otherwise unremarkable, and he takes no medications. On physical examination, temperature is 38.5 °C (101.3 °F), blood pressure is 118/80 mm Hg, pulse rate is 113/min, and respiration rate is 33/min. Oxygen saturation is 89% breathing 6 L/min oxygen by nasal cannula. The patient appears uncomfortable and is oriented only to person. Breath sounds are decreased bilaterally with scattered crackles. No rash is noted. Laboratory studies: Leukocyte count 19,000/µL (19 × 109/L) Platelet count 274,000/µL (274 × 109/L) Blood urea nitrogen 36 mg/dL (12.9 mmol/L) Creatinine 1.45 mg/dL (128 µmol/L) Sputum Gram stain shows 2+ polymorphonuclear cells and 1+ epithelial cells; no organisms are seen. A respiratory viral panel from a nasopharyngeal swab is positive for rhinovirus. A chest radiograph is shown. Which of the following is the most appropriate treatment? Azithromycin Ceftazidime plus azithromycin Ceftriaxone plus levofloxacin Piperacillin-tazobactam plus ciprofloxacin
The most appropriate treatment is ceftriaxone plus levofloxacin. This otherwise healthy patient presents with severe community-acquired pneumonia (CAP). The initial evaluation reveals respiratory failure (tachypnea and hypoxia), uremia, altered mentation, and multilobar infiltrates, all of which are minor criteria for severe disease based on Infectious Diseases Society of America/American Thoracic Society (IDSA/ATS) criteria. Patients with at least three minor criteria for severe disease have increased risk of mortality and are best managed in the ICU. Guideline-based recommendations for empiric therapy of CAP requiring ICU care include a β-lactam (ampicillin-sulbactam, cefotaxime, ceftriaxone, or ceftaroline) to treat Streptococcus pneumoniae, gram-negative bacilli, or Haemophilus influenzae plus an agent active against Legionella, such as a macrolide or quinolone. The initial evaluation of this patient was positive for rhinovirus on a respiratory viral panel (RVP). Respiratory viruses are increasingly recognized among hospitalized patients with CAP; however, the significance of this finding is uncertain. Respiratory viruses may predispose patients to a secondary bacterial pneumonia or be present as a copathogen. Considering the severity of this patient's illness, antibiotics should be initiated and continued despite the positive RVP until a bacterial cause is excluded. Azithromycin monotherapy is an appropriate choice for treatment of CAP in a previously healthy outpatient. Patients with more severe CAP are at risk for infections with numerous organisms, as well as with drug-resistant pathogens. For this reason, patients with CAP admitted to the ICU should receive combination therapy with an antipneumococcal β-lactam and either a macrolide or a fluoroquinolone. Ceftazidime is a third-generation cephalosporin that is effective against Pseudomonas but has minimal activity against gram-positive organisms, including S. pneumoniae, and therefore would not be an appropriate choice, even as combination therapy with a macrolide, to treat CAP. Piperacillin-tazobactam plus ciprofloxacin would be indicated when concern for Pseudomonas pneumonia is present; however, in a previously healthy patient with minimal health care interactions or previous antibiotic use, Pseudomonas would be an unlikely pathogen.
A 19-year-old man is evaluated in the emergency department for fever, cough producing blood-tinged sputum, shortness of breath, and headache. He attended a political rally on his college campus 4 days ago. Six other people have been hospitalized with similar symptoms. Medical history is unremarkable, and he takes no medications. On physical examination, the patient is alert and oriented. Temperature is 39.1 °C (102.4 °F), blood pressure is 98/58 mm Hg, pulse rate is 110/min, and respiration rate is 24/min. Oxygen saturation is 92% breathing oxygen 2 L/min by nasal cannula. Neurologic examination is nonfocal, and no meningeal signs are present. Dyspnea, bilateral pulmonary rhonchi, and tubular breath sounds are noted on pulmonary examination. No rash is present, and the abdomen is nontender. Sputum Gram stain reveals many polymorphonuclear cells and abundant gram-negative coccobacilli demonstrating bipolar staining. A chest radiograph shows bilateral patchy infiltrates. Which of the following is the most appropriate treatment? Ceftriaxone and azithromycin Ciprofloxacin Gentamicin Piperacillin-tazobactam and levofloxacin
The most appropriate treatment is gentamicin. This patient most likely has pneumonic plague caused by the bacteria Yersinia pestis, one of the biologic agents classified as an A-list bioterrorism pathogen because of its high potential lethality and ease of dissemination. Sputum Gram stain (and possibly blood smear) may identify gram-negative coccobacilli demonstrating the classic bipolar staining or "safety pin" shape shown. Although most pulmonary involvement occurs through secondary hematogenous spread to the lungs from a bubo or other source, primary pneumonic plague can occur after close contact with another person with plague pneumonia, after animal exposure, or as a result of intentional aerosol release for the purpose of terrorism, as in this case. Recommended first-line treatment is either streptomycin or gentamicin. Anthrax is caused by Bacillus anthracis, a gram-positive, aerobic organism. It appears as a sporulating gram-positive rod on microscopic examination. Patients with inhalational anthrax present with low-grade fever, malaise, myalgia, and headache accompanied by cough, dyspnea, and chest pain. A chest radiograph showing mediastinal widening from hemorrhagic lymphadenitis is characteristic. Ciprofloxacin, levofloxacin, moxifloxacin, or doxycycline should be provided as soon as possible after any actual or suspected case of anthrax that raises concern for a bioterrorism attack. Ambulatory empiric therapy for community-acquired pneumonia (CAP) is directed against Streptococcus pneumoniae, Haemophilus influenzae, and atypical bacteria, even though a significant proportion of patients with CAP infected with viral pathogens do not benefit from antibiotic therapy. The combination of ceftriaxone and azithromycin would be an appropriate choice for the treatment of CAP but would be inadequate for treating plague, which requires either streptomycin or gentamicin. When clinical concern for Pseudomonas is present, dual therapy with two active agents is indicated. Options include an antipseudomonal β-lactam (piperacillin-tazobactam, cefepime, or meropenem) in conjunction with either a quinolone (levofloxacin or ciprofloxacin) or an aminoglycoside. Pseudomonas infection should be considered in immunocompromised patients and patients with underlying structural lung disease (bronchiectasis or cystic fibrosis) or medical conditions requiring repeated courses of antibiotics. The combination of piperacillin-tazobactam and levofloxacin would be ideal for a seriously ill patient with CAP and concern for Pseudomonas infection but not for this patient most likely infected with Yersinia pestis. Read Related TextNext Question
A 72-year-old woman is evaluated for a 2-day history of left facial droop and severe burning and stinging pain on the left ear helix and into the ear canal, with muffled hearing and tinnitus. She received the live-attenuated zoster vaccine at age 60 years. She takes no medications. On physical examination, vital signs are normal. She has a left-sided peripheral facial droop. The tympanic membrane appears normal. No rash is present. Hearing is diminished to a whisper in the left ear. The remainder of the examination is unremarkable. Which of the following is the most likely cause of this patient's findings? Borrelia burgdorferi Herpes simplex virus type 1 Herpes simplex virus type 2 Varicella-zoster virus
Varicella-zoster virus (VZV) is the most likely cause of this patient's findings. VZV reactivation presents with pain or paresthesias in a specific dermatome; the characteristic rash develops several days later. In order of frequency, the thoracic, trigeminal, lumbar, and cervical cutaneous dermatomes are most often involved. More than 50% of cases occur in persons older than 60 years; immunocompromised patients are also at risk. Involvement of the geniculate ganglion may cause herpes zoster oticus, also known as the Ramsay Hunt syndrome, characterized by pain and vesicles in the external ear canal, ipsilateral peripheral facial palsy, and altered or absent taste. Patients may also experience hearing loss, tinnitus, and altered lacrimation. Most experts consider Ramsay Hunt syndrome to be a polycranial neuropathy, with frequent involvement of cranial nerves V, IX, and X. A vesicular rash may be absent in patients with VZV (zoster sine herpete) and should not deter physicians from ordering polymerase chain reaction testing for VZV. Acyclovir is typically prescribed for this syndrome. The live attenuated zoster vaccine has 64% efficacy that decreases to 36% after 6 years. A novel recombinant zoster vaccine, approved in 2017, has 97% efficacy and should be given to this patient. The most common neurologic manifestation of early disseminated Lyme disease is facial nerve palsy, which may be unilateral or bilateral. Treatment of early localized disease prevents progression of infection. Untreated patients may progress to early disseminated infection. This stage typically presents as a febrile illness associated with erythema migrans at multiple sites distant from the initial tick attachment. Constitutional symptoms are common and include fever, myalgia, arthralgia, and headache. Neurologic Lyme disease without a history of tick bite, erythema migrans rash, or systemic symptoms is highly unlikely. Bell palsy is defined as an isolated paralysis of the facial nerve that leads to complete unilateral facial paralysis. Herpes simplex virus (HSV) type 1 reactivation is the likely cause of Bell palsy in most cases. However, most cases of Bell palsy are diagnosed as "idiopathic" because it is difficult in clinical practice to prove that reactivation of HSV-1 is the cause of this peripheral neuropathy. HSV type 2 is more commonly associated with genital ulcers, recurrent aseptic meningitis, or myelitis than with Bell palsy. Neither Lyme disease nor HSV reactivation can explain the burning and stinging ear pain, tinnitus, hearing loss, and facial palsy experienced by this patient as well as VZV reactivation can. Read Related TextNext Question
A 43-year-old man is evaluated in the emergency department for gastrointestinal bleeding. He reports loose, dark stools of 2 weeks' duration. He has an 8-year history of diffuse cutaneous systemic sclerosis (DcSSc) complicated by Raynaud phenomenon and gastroesophageal reflux disease. He does not drink alcohol or take NSAIDs. His only medication is omeprazole. On physical examination, temperature is normal, blood pressure is 100/60 mm Hg, pulse rate is 80/min, respiration rate is 16/min, and oxygen saturation is 94% breathing ambient air. Skin changes associated with DcSSc are noted from the hands to the elbows in the upper extremities and from the feet to the knees in the lower extremities. The abdomen is nontender to palpation. The remainder of the physical examination is normal. Laboratory studies show a normal chemistry panel and a hematocrit level of 26%. Upper gastrointestinal endoscopy demonstrates linear ectatic vessels resembling the stripes found on a watermelon that arise from the pylorus. There is no evidence of a hiatal hernia or other abnormal findings. Cameron lesions Dieulafoy lesions Gastric antral vascular ectasia Portal hypertensive gastropathy
The most likely diagnosis is gastric antral vascular ectasia (GAVE) associated with systemic sclerosis. GAVE is the proliferation of blood vessels typically in the antrum of the stomach; on endoscopy, it has the appearance of watermelon stripes (watermelon stomach). Approximately 60% of patients with GAVE have an underlying autoimmune disease; the remainder have portal hypertension secondary to hepatic cirrhosis. GAVE can be a source of both acute and chronic gastrointestinal bleeding. First-line therapy is argon plasma coagulation or laser coagulation. Cameron lesions are erosions found on the crest of gastric folds within a large hiatal hernia and are thought to be caused by mechanical trauma as the hiatal hernia slides up and down. Up to 5% of patients with known hiatal hernias may have Cameron lesions. There is no evidence of a hiatal hernia, making this diagnosis unlikely. Dieulafoy lesions are submucosal arterioles that intermittently protrude through the mucosa and cause hemorrhage. Dieulafoy lesions are of unknown etiology and account for about 2% of all causes of acute upper gastrointestinal bleeding and are located in the proximal stomach along the lesser curvature. When bleeding they appear as an isolated pumping arteriole in the absence of a mass or ulcer. The patient's findings do not match those of Dieulafoy lesions. Portal hypertensive gastropathy (PHG) commonly occurs with advanced cirrhosis and has a characteristic mosaic appearance on endoscopy, most often seen in the body and fundus. It can be confused with GAVE. This patient has no history of cirrhosis or findings to suggest chronic liver disease, making PHG an unlikely diagnosis.
A 48-year-old woman is evaluated at a follow-up appointment for elevated liver chemistry tests over the preceding 6 months. She reports no symptoms. She has type 2 diabetes mellitus and hypertension. Her family history is unremarkable. She drinks two glasses of wine one or two times per month. Her medications are metformin and lisinopril. On physical examination, vital signs are normal; BMI is 31. No spider angiomata, palmar erythema, splenomegaly, abdominal distention, or lower-extremity edema is noted. Laboratory studies: Alkaline phosphatase 96 U/L Alanine aminotransferase 85 U/L Aspartate aminotransferase 74 U/L Anti-smooth muscle antibody Positive (1:20 titer) Results of other studies, including hepatitis C antibody, hepatitis B surface antigen, iron saturation, α1-antitrypsin phenotype, tissue transglutaminase IgA antibody, antimitochondrial antibody, and IgG, are within normal limits. On ultrasonography, the liver is hyperechoic and enlarged. Antimitochondrial antibody-negative primary biliary cholangitis Autoimmune hepatitis Nonalcoholic fatty liver disease Primary sclerosing cholangitis
The most likely diagnosis is nonalcoholic fatty liver disease (NAFLD). Up to 108 million Americans have NAFLD, which far exceeds the prevalence of any other form of chronic liver disease. Risk factors include obesity, diabetes mellitus, insulin resistance, hypertension, and hyperlipidemia. This patient has three risk factors for NAFLD, as well as an enlarged liver on physical examination. Her elevated alanine aminotransferase and aspartate aminotransferase levels are within the typical range for patients with NAFLD. Alkaline phosphatase (ALP) levels may be slightly elevated as well, typically less than 2 to 2.5 times the upper limit of normal. The finding of a hyperechoic liver on ultrasonography is also consistent with NAFLD. Because other liver diseases may also result in hepatic steatosis, patients with elevated liver chemistries and suspected nonalcoholic steatohepatitis should be evaluated to exclude other causes of chronic liver disease. The diagnosis of primary biliary cholangitis (PBC) is generally made on the basis of a cholestatic liver enzyme profile in the setting of a positive antimitochondrial antibody test. The predominant liver enzyme abnormality is an increase in serum ALP levels, but a mild to moderate increase in aminotransferase levels is also seen. Antimitochondrial antibody-negative PBC accounts for about 10% of cases of PBC, but in suspected antimitochondrial antibody-negative PBC, other PBC-specific autoantibodies (sp100 or gp210) can establish the diagnosis in a patient with a compatible cholestatic liver enzyme profile. This patient's liver chemistry profile is hepatocellular (elevations primarily of aminotransaminase levels), not cholestatic, making PBC unlikely. Although this patient has a positive anti-smooth muscle antibody test, the low titer alone is not diagnostic of autoimmune hepatitis. Autoimmune hepatitis is typically accompanied by higher autoantibody titers and elevated γ-globulin levels. It requires a liver biopsy to establish the diagnosis. Between 20% and 30% of patients with NAFLD exhibit low-titer autoantibodies. This patient has a minimally elevated ALP level, which helps to exclude primary sclerosing cholangitis (PSC) or PBC. In addition, the patient's risk factors and ultrasound findings make these diagnoses unlikely. MR cholangiopancreatography is required to diagnose PSC.
A 49-year-old man is scheduled for total right knee arthroplasty. Medical history is otherwise unremarkable. He takes no medications. On physical examination, vital signs are normal. The right knee demonstrates bony hypertrophy and crepitus with passive movement. Low-molecular-weight heparin and intermittent pneumatic compression will be initiated and continued during the hospital stay. Which of the following is the recommended duration of low-molecular-weight heparin prophylaxis for this patient? Total of 10 days Total of 14 days Total of 35 days Until fully ambulatory Until hospital discharge
The recommended postoperative duration of venous thromboembolism (VTE) prophylaxis with low-molecular-weight heparin (LMWH) following major orthopedic surgery is 35 days in patients who are not at increased bleeding risk and have not experienced perioperative bleeding complications. The American College of Chest Physicians (ACCP) antithrombotic guideline provides recommendations for VTE prophylaxis for both orthopedic and nonorthopedic surgery populations. The ACCP guideline identifies hip arthroplasty, knee arthroplasty, and hip fracture surgery as major orthopedic surgeries. These surgeries pose a high VTE risk, and both pharmacologic and mechanical VTE prophylaxis are recommended during hospitalization. The ACCP recommends LMWH over other pharmacologic agents, although there are other acceptable agents, including aspirin for those unable or unwilling to take heparin. For patients without increased bleeding risk, extended duration of postoperative prophylaxis for up to 35 days is recommended over shorter-duration prophylaxis of 10 to 14 days, which is the minimum recommended duration of pharmacologic VTE prophylaxis in orthopedic surgery. Randomized trials, systematic reviews, and meta-analyses have shown that compared with placebo, aspirin, and warfarin, extended prophylaxis up to 35 days with LMWH reduces the rate of VTE disease without excess bleeding in patients who undergo major orthopedic surgery. If bleeding risk is especially high, mechanical prophylaxis is recommended over no prophylaxis. In patients who decline LMWH injections or who are unable to tolerate LMWH, the oral direct thrombin inhibitor dabigatran, a factor Xa inhibitor (apixaban, rivaroxaban, edoxaban), or a vitamin K antagonist (warfarin) is recommended over alternate forms of prophylaxis. For this patient undergoing major orthopedic surgery, dual perioperative VTE prophylaxis with LMWH and intermittent pneumatic compression is recommended during hospitalization, with LMWH continued for up to 35 days. Because of the elevated risk for VTE in many patients undergoing orthopedic surgery, a short course of VTE prophylaxis, such as 10 or 14 days, is insufficient because thrombotic risk remains elevated beyond this time frame.
A 35-year-old man is evaluated in the emergency department for fever and epistaxis. He has been experiencing malaise for several weeks and fever, chills, and anorexia for the past 3 days. Today he developed epistaxis. He takes no medications. On physical examination, temperature is 38.8 °C (100.4 °F), blood pressure is 105/70 mm Hg, pulse rate is 110/min, and respiration rate is 24/min. He is diaphoretic. Dried blood is noted in the nares, and gingival bleeding is present. No lymphadenopathy or hepatosplenomegaly is noted. He has petechiae bilaterally on the lower extremities. Laboratory studies: Activated partial thromboplastin time 60 s Hemoglobin 9.8 g/dL (98 g/L) Leukocyte count 3600/µL (3.6 × 109/L) with 20% neutrophils, 3% bands, 35% lymphocytes, 23% monocytes, and 18% "atypical" cells Platelet count 17,000/µL (17 × 109/L) Prothrombin time 24 s Fibrinogen 93 mg/dL (0.93 g/L) The peripheral blood smear shows immature leukocytes with prominent granules in the cytoplasm. Which of the following is the most likely diagnosis? Acute promyelocytic leukemia Aplastic anemia Chronic granulocytic leukemia Immune thrombocytopenic purpura
This patient has pancytopenia, immature leukocytes with morphologic features consistent with promyelocytes, and laboratory features of disseminated intravascular coagulation (DIC), all of which are consistent with acute promyelocytic leukemia (APML). Acute leukemia can present with a leukocyte count within or below the normal range. It can include bleeding, as in this patient, from thrombocytopenia that may be accompanied by coagulopathy. APML is a subset of acute myeloid leukemia that often presents with bleeding and coagulopathy; this coagulopathy contributes to early mortality and morbidity. Features of DIC should prompt transfusion with fresh frozen plasma and cryoprecipitate (for hypofibrinogenemia) in addition to platelet transfusion for thrombocytopenia. A bone marrow aspirate and biopsy should be performed urgently to confirm the diagnosis because the coagulopathy of APML responds to therapy with all-trans retinoic acid, which targets the underlying defect in cellular differentiation. Pancytopenia is expected in patients with aplastic anemia, but those patients would not have the atypical cells seen in the peripheral blood of this patient and would not present with DIC. Although immature leukocytes, including myelocytes and promyelocytes, may be seen in the peripheral blood in patients with chronic granulocytic leukemia, these patients have leukocytosis, not leukopenia, and most of the increase comes from mature polymorphonuclear leukocytes. Although severe thrombocytopenia and bleeding are typical of the acute presentation of immune thrombocytopenic purpura, this patient's pancytopenia, coagulopathy, and abnormal leukocytes are inconsistent with this diagnosis. Read Related TextNext Question
A 22-year-old woman is evaluated in the emergency department for a 2-day history of progressive fatigue and shortness of breath. Medical history is significant for sickle cell disease diagnosed at age 6 years. Her sickle cell disease is under good control with rare crisis. Medications are hydroxyurea and folic acid. On physical examination, temperature and blood pressure are normal, pulse rate is 125/min, and respiration rate is 25/min. Oxygen saturation is 93% breathing ambient air. Lungs are clear without wheezing. Other examination findings are unremarkable. Laboratory studies: Hemoglobin 4 g/dL (40 g/L) Leukocyte count 13,000/µL (13 × 109/L) Platelet count 500,000/µL (500 × 109/L) Reticulocyte count 0.1% of erythrocytes Chest radiograph is normal. Which of the following is the most likely diagnosis? Aplastic anemia Hyperhemolysis syndrome Myelodysplastic syndrome Parvovirus B19 infection
This patient has parvovirus B19 infection presenting with pure red cell aplasia (PRCA). Patients with sickle cell anemia have chronic hemolysis and depend on increased erythrocyte production to maintain the hemoglobin level. Parvovirus B19 infection preferentially affects erythrocyte precursors in the bone marrow, causing transient PRCA. This patient's very low reticulocyte count is consistent with the decreased erythrocyte production seen in PRCA. Infection with parvovirus B19 in children can present as erythema infectiosum with fever and rash. Although adults can have flu-like illness and arthralgia, most are asymptomatic. However, in patients with hemolytic anemia, it can present with transient aplastic crisis. Supportive transfusions are usually required to treat symptoms from severe anemia. Aplastic anemia (AA) is a condition characterized by pancytopenia with associated neutropenia, anemia, and thrombocytopenia and a severely hypocellular bone marrow. Isolated cytopenias are uncommon despite the designation of anemia. AA is usually acquired and is caused by toxic, viral, or autoimmune mechanisms. The lack of other cytopenias makes AA an unlikely diagnosis. In some patients with sickle cell disease, a rare condition known as hyperhemolysis is thought to be responsible for an acute hemolytic anemia and reticulocytosis. Episodes have sometimes been associated with delayed transfusion reactions and acute vasoocclusive events. Hyperhemolysis syndrome would be associated with an elevated reticulocyte count, not a reduced count. Myelodysplastic syndrome (MDS) ranges in severity from an asymptomatic disease characterized by mild normocytic or macrocytic anemia to a transfusion-dependent anemia. The incidence of MDS increases with age. Abnormal erythrocyte forms with basophilic stippling or Howell-Jolly bodies and dysplastic neutrophils with decreased nuclear segmentation and granulation may be present. MDS is unlikely to cause a symptomatic anemia developing over a few days in a young patient with sickle cell disease. Read Related TextNext Question
A 73-year-old man undergoes follow-up evaluation after autologous hematopoietic stem cell transplantation (HSCT) 3 weeks ago for multiple myeloma. He had induction chemotherapy 1 year ago. He is doing well, without fever, chills, cough, or localizing symptoms of infection. Medications are acyclovir and omeprazole. On physical examination, vital signs are normal; BMI is 20. He weighs 2.5 kg (6 lb) less than he did 2 months ago before HSCT. Alopecia is noted. In the next 30 days, this patient is at increased risk for which of the following treatment-related complications? Acute graft-versus-host disease Infection Lymphoma Myelodysplastic syndrome
This patient is at increased risk of infection. The most common indications for autologous hematopoietic stem cell transplantation (HSCT) are multiple myeloma and relapsed non-Hodgkin lymphoma. With appropriate supportive care, many older patients are eligible for autologous HSCT. Fatigue, altered taste, diminished appetite, and persistent mild diarrhea are common adverse effects of high-dose chemotherapy, an integral part of HSCT. The most common concern in patients who have undergone HSCT is the increased risk of infection, which persists for 6 to 12 months after HSCT. This risk is attributed to compromised function of neutrophils and lymphocytes and is sometimes associated with mild neutropenia and lymphopenia. For this reason, patients should be counseled to report warning symptoms, such as fever and respiratory or other localizing symptoms of infection, early in the course. Graft-versus-host disease (GVHD) is a major risk of allogeneic HSCT. Acute GVHD occurs when graft T cells recognize the patient's normal gut, skin, and liver sinusoids as foreign. Severe GVHD is life threatening and is treated with high-dose glucocorticoids. Anti-T-lymphocyte immune globulin as part of the myeloablative conditioning regimen can markedly reduce the prevalence of chronic GVHD 2 years after allogeneic HSCT in HLA-matched siblings. GVHD is not a complication of autologous HSCT. Hematopoietic clonal disorders, including myelodysplasia, leukemia, and lymphoma, can occur decades after completion of therapy. Typical agents associated with therapy-related myelodysplasia include alkylators, anthracyclines, and topoisomerase II inhibitors. Likewise, leukemia and lymphoma can develop as a late consequence of exposure to chemotherapy and radiation. But these disorders would not occur in the months immediately following uncomplicated autologous HSCT. Read Related TextNext Question
A 75-year-old man is evaluated for progressive dysphagia of 8 months' duration for both solid food and water, and the necessity to induce vomiting several times each month to relieve his symptoms. He also has experienced chest pain and heartburn symptoms. He has lost approximately 6 kg (13 lb) of weight over the preceding 3 months and a total of 9 kg (20 lb) since his symptoms began. He has a long history of cigarette and alcohol use. His medical history and review of systems is otherwise negative. He has no travel history outside the northeastern United States. He takes no medication. On physical examination, vital signs are normal; BMI is 23. He appears thin and tired. The remainder of the physical examination is unremarkable. Upper endoscopic findings reveal retained saliva, liquid, and food in the esophagus without mechanical obstruction. Manometry demonstrates incomplete lower esophageal relaxation and aperistalsis. Achalasia Chagas disease Eosinophilic esophagitis Pseudoachalasia
This patient likely has pseudoachalasia, and endoscopic ultrasound should be used to diagnose a possible tumor of the distal esophagus or gastric cardia. The other causes of pseudoachalasia are benign disease, such as amyloidosis and sarcoidosis, and postsurgical status (for example, after Nissen fundoplication or bariatric surgery). The clinical presentation of achalasia consists of dysphagia to both solids and liquids along with regurgitation of undigested food and saliva. Other conditions can mimic achalasia and can have identical clinical, barium-imaging, and manometric findings, as well as identical endoscopic appearance. These include pseudoachalasia, secondary achalasia, and Chagas disease. Pseudoachalasia is caused by a tumor at the gastroesophageal junction infiltrating the myenteric plexus causing esophageal motor abnormalities. Tumors capable of infiltrating the myenteric plexus include those of the distal esophageus, gastric cardia, pancreatic, breast, lung, and hepatocellular. Patients with pseudoachalasia are often in their sixth decade of life or older, have a short duration of symptoms, and experience sudden and profound weight loss. Endoscopic ultrasonography can exclude an infiltrating tumor, and guidelines recommend its use in patients with a strong suspicion for malignancy. Achalasia affects men and women equally, with an annual incidence of 1 in 100,000 individuals. It tends to occur between the ages of 30 and 60 years. Typical achalasia has an insidious onset and long duration of symptoms, often measured in years, before patients seek medical attention. This patient's age, short duration of symptoms, and rapid weight loss argue against the diagnosis of achalasia. Chagas disease is caused by infection with the vector-borne parasite Trypanosoma cruzi in rural areas of Latin America. The major manifestations of Chagas disease include enteric myenteric destruction, resulting in achalasia, megacolon, heart disease, and other neurologic disorders. The lack of travel and absence of other manifestations of Chagas disease make this diagnosis unlikely. Eosinophilic esophagitis is inconsistent with this patient's symptoms and endoscopic examination. Its typical presentation is in younger patients with food bolus obstruction. Endoscopy findings in eosinophilic esophagitis include rings and furrows.
A 28-year-old woman undergoes follow-up consultation regarding a pre-employment physical examination. She reports feeling well, with no recent illness. Medical history is notable for gastroesophageal reflux disease. Her only medication is omeprazole. She is Black. On physical examination, vital signs and other examination findings are normal. Laboratory studies: Absolute neutrophil count 1400/µL (1.4 × 109/L) Hemoglobin 13.2 g/dL (132 g/L) Leukocyte count 1867/µL (1.87 × 109/L) with 75% neutrophils, 20% lymphocytes, and 5% monocytes Platelet count 258,000/µL (258 × 109/L) A peripheral blood smear shows decreased neutrophils, normal lymphocytes, normochromic erythrocytes, and normal platelets. Which of the following is the most likely diagnosis? Autoimmune neutropenia Benign ethnic neutropenia Cyclical neutropenia Drug-induced neutropenia
This patient most likely has benign ethnic neutropenia. Isolated neutropenia usually has a hereditary, toxic, or immune cause. Isolated mild neutropenia (1000-1500/µL [1-1.5 × 109/L]) found on routine testing in asymptomatic Black patients, or occasionally in other ethnic groups (Sephardic Jews, West Indians, Arabs of the Middle East), likely has a benign ethnic cause. An absolute neutrophil count less than 500/µL (0.5 × 109/L) is less likely to be a normal variant and more likely to be associated with increased risk for bacterial and fungal infections. Patients with benign ethnic neutropenia usually have good bone marrow reserve and are not prone to developing infections. A detailed evaluation for other causes of neutropenia is usually not required in these patients. Having a previous history of mild neutropenia supports the diagnosis but is not required. Autoimmune neutropenia is caused by destruction of the neutrophils by autoantibodies. It is more commonly seen in conjunction with other autoimmune disorders, such as systemic lupus erythematosus or Felty syndrome. Felty syndrome is a triad of rheumatoid arthritis, splenomegaly, and neutropenia. This patient has no signs or symptoms of an autoimmune disorder. Although idiopathic autoimmune neutropenia without an underlying systemic disorder can occur, benign ethnic neutropenia would be a much more likely diagnosis in this asymptomatic Black patient with mild neutropenia. Cyclical neutropenia is a rare congenital disorder in which the neutrophil count nadirs every 2 to 5 weeks with recurrent infections. Diagnosis requires twice-weekly complete blood counts for 6 to 8 weeks. This patient's clinical history does not indicate recurrent infections or other typical findings of cyclical neutropenia. Drug-induced neutropenia occurs in patients taking medications such as chemotherapy, NSAIDs, carbamazepine, phenytoin, propylthiouracil, cephalosporins, trimethoprim-sulfamethoxazole, or psychotropic drugs. Omeprazole has not been commonly implicated in causing neutropenia. Drug-induced neutropenia is diagnosed by temporal relationship of the neutropenia with starting the medication and improvement with stopping the offending medication. It is important to note that improvement in neutropenia may lag behind by many weeks after stopping the medication.
How to diagnose invasive aspergillosis?
This patient most likely has disseminated aspergillosis with lung and brain involvement. Aspergillus invades blood vessels and causes distal infarction of infected tissue. Patients with invasive pulmonary aspergillosis may have fever, cough, chest pain, and hemoptysis at presentation. Pulmonary infiltrates, nodules, or wedge-shaped densities resembling infarcts may also be seen on chest radiographs; CT scans may show a target lesion with a necrotic center surrounded by a ring of hemorrhage (halo sign). Central nervous system involvement may manifest as a brain abscess or infarction. Other sites of involvement include blood vessels in the heart, gastrointestinal tract, or skin. Risk factors for invasive or disseminated aspergillosis include profound and prolonged neutropenia and hematopoietic stem cell transplantation. The second most common risk group for invasive aspergillosis is solid organ (heart, lung, liver, kidney) transplant recipients. The infection has also increasingly been reported in patients who are critically ill and in ICUs, especially those with exposure to glucocorticoids. The most efficient way to establish a definitive diagnosis (and then initiate antifungal therapy) is with bronchoalveolar lavage (BAL) and biopsy. In patients with disseminated aspergillosis, the serum galactomannan assay has a sensitivity of less than 30% and is not very useful; in contrast, the BAL galactomannan assay has a much higher sensitivity and specificity. Therefore, BAL fluid samples should be collected for analysis, and tissue biopsy should be performed. A brain biopsy is sometimes useful to establish a definitive diagnosis but carries a much greater risk of causing adverse effects than bronchoscopy and BAL. If the lung biopsy is negative, then a brain biopsy should be considered. Fungal blood cultures have a low sensitivity in all invasive mold infections but especially in invasive aspergillosis, in which less than 1% of patients with infection have a positive culture.
A 42-year-old man is evaluated for headache and progressive weakness. He was hospitalized yesterday with a 9-day history of fever, headache, and myalgia. He is a trail runner, and 10 days ago participated in a 10-kilometer race in North Carolina. He is not aware of any tick bite. On physical examination, the patient appears ill. Temperature is 39.4 °C (102.9 °F), blood pressure is 102/78 mm Hg, pulse rate is 102/min, and respiration rate is 24/min. No tonsillar enlargement, cervical lymphadenopathy, hepatosplenomegaly, or skin lesion is noted. Laboratory studies: Leukocyte count 1500/µL (1.5 × 109/L), with 70% neutrophils and no atypical lymphocytes Platelet count 34,000/µL (34 × 109/L) Alanine aminotransferase 667 U/L Aspartate aminotransferase 995 U/L Empiric doxycycline therapy is initiated for a possible tick-borne infection. Within 24 hours, he defervesces, and within 48 hours, the leukocyte and platelet counts normalize. Serologic tests for Rickettsia rickettsii and Ehrlichia chaffeensis obtained on admission have negative results. Which of the following is the most likely diagnosis? Heartland virus infection Human monocytic ehrlichiosis Infectious mononucleosis Rocky Mountain spotted fever
This patient most likely has human monocytic ehrlichiosis (HME), a tick-borne illness primarily caused by Ehrlichia chaffeensis. His symptoms include a nonfocal febrile illness associated with leukopenia, thrombocytopenia, and elevated hepatic enzyme levels, all of which are characteristic of HME. His participation in outdoor activities in wooded areas places him at risk for exposure to ticks. HME is endemic in the mid-Atlantic, southern, and southeastern United States; his clinical presentation is highly suggestive of this illness. The prompt response to doxycycline is classic for a tick-borne rickettsial illness, such as HME; a lack of response within 48 hours of therapy would suggest an alternative diagnosis or a coinfection. The negative serologic results for E. chaffeensis do not disprove the diagnosis and should not prompt early discontinuation of doxycycline. The sensitivity of antibody testing is low in the first week of illness; seroconversion typically occurs within 2 to 4 weeks of symptom onset, and acute and convalescent titers are useful for retrospective confirmation of infection. During the acute illness, buffy-coat staining (to reveal the presence of morulae, which are basophilic inclusion bodies in the cytoplasm of monocytes representing clusters of bacteria [shown]) or polymerase chain reaction of whole blood specimens may allow an early diagnosis. Although many infections present identically to HME, the salient feature in this patient was the rapid response to doxycycline therapy, which essentially excludes a viral process. Heartland virus, a newly described Bunyavirus transmitted by the same vector as E. chaffeensis, is clinically indistinguishable from HME and should be suspected when no improvement is seen within 48 hours of starting doxycycline therapy. Infectious mononucleosis can present with fever, cytopenia, and elevated aminotransferase levels. However, other components of the infectious mononucleosis triad (fever, pharyngitis, and cervical lymphadenopathy) were absent in this patient. The rapid response to doxycycline treatment is consistent with Rocky Mountain spotted fever (RMSF) infection. However, the pronounced leukopenia and absence of a rash more than 1 week into the illness argue against this diagnosis.
A 19-year-old man is hospitalized with a 6-day history of lightheadedness and nightly fevers. He also reports sore throat, headache, joint and muscle aches, and a dry cough. He recalls a blotchy rash on his trunk and arms, which has resolved. He returned home 12 days ago from a trip to Vietnam, for which he did not receive specific immunizations or other prophylaxis. During the second week of his trip, he experienced 2 days of diarrhea; he has had none since, but abdominal discomfort and anorexia persist. On physical examination, the patient is lethargic. Temperature is 39.9 °C (103.9 °F), blood pressure is 98/58 mm Hg, pulse rate is 68/min, and respiration rate is 16/min. No skin rash or evidence of jaundice is present. The abdomen is distended, with diminished bowel sounds, but nontender. Neurologic examination is nonfocal. Which of the following is the most likely diagnosis? Brucellosis Leptospirosis Melioidosis Typhoid (enteric) fever
This patient most likely has typhoid fever, also known as enteric fever, caused by either of the typhoidal Salmonella strains, S. typhi or S. paratyphi. Infection is transmitted by ingestion of food or water contaminated by feces. In resource-poor areas, organisms may be spread in community food or, more frequently, by water. In developed countries, transmission is chiefly by food that has been contaminated during preparation by healthy carriers. Symptoms of enteric fever are generally nonspecific. Fever is the major manifestation, typically rising over 2 to 3 nights and persisting for several days. A pulse-temperature dissociation (relative bradycardia) is often present. A brief period of diarrhea followed by constipation, abdominal discomfort, nonproductive cough, mild confusion, and transient small blanching skin lesions (rose spots) are other clinical features. Ceftriaxone is the preferred empiric antibiotic agent, with ciprofloxacin and azithromycin as additional options if resistance is not encountered. Human brucellosis can develop after exposure to one of four Brucella species through contact with viable organisms in secretions or excretions of infected animals, ingestion of undercooked meat or milk products, or, less often, inhalation. Patients experience numerous nonspecific symptoms as well as recurring or "undulating" waves of fever, but rash, gastrointestinal symptoms, and relative bradycardia are not typical. Humans acquire leptospirosis after contact with infected urine spread by carrier animals. Classically, illness is biphasic, beginning with a septicemic phase followed by an immune phase, which correlates with the appearance of antibodies in serum. Clinically apparent leptospirosis presents with the abrupt onset of fever, rigors, myalgias, and headache. Conjunctival suffusion in a patient with a nonspecific febrile illness should raise suspicion for the diagnosis of leptospirosis. A more severe form, known as Weil syndrome (icteric leptospirosis), consisting of jaundice, azotemia, and anemia, may also occur. Gastrointestinal symptoms are infrequent. The gram-negative bacillus Burkholderia pseudomallei, found in soil and water in endemic areas such as Southeast Asia, is the causative agent of melioidosis. After acquisition through direct skin contact, ingestion, or inhalation, an acute pulmonary, septicemic, or localized suppurative infection may occur. The patient's findings are not compatible with melioidosis.
A 31-year-old man arrives to establish care for newly diagnosed HIV infection. He is asymptomatic. Medical history is otherwise noncontributory, and he takes no medications. On physical examination, vital signs are normal, and the remainder of the examination is unremarkable. A review of his previous laboratory studies shows a normal complete blood count, and chemistries, including glucose, creatinine, and liver enzyme levels, are within normal limits. The HIV-1/2 antigen/antibody combination immunoassay is reactive. The HIV-1/2 antibody differentiation assay is positive for HIV-1 antibody, and HIV-1 RNA is quantified at 27,313 copies/mL. CD4 cell count is 455/µL. Which of the following is the most appropriate next step in management? Genotypic viral resistance testing Glycohemoglobin level Phenotypic viral resistance testing Repeat HIV viral load and CD4 cell count in 1 month
This patient needs baseline genotypic HIV resistance testing. Because of the possibility of transmitted virus having resistance mutations, it is recommended to obtain baseline resistance testing before starting an antiretroviral regimen. If the patient is ready, antiretrovirals can be started the same day, while waiting for resistance testing results, with regimen modification if necessary based on results. Virologic failure of a regimen (rebound of a suppressed viral load or failure to achieve undetectable viral load with therapy) is also an indication for resistance testing to guide the change in regimen. Genotypic testing looks for mutations in the viral genome associated with antiviral drug resistance. Phenotypic testing actually tests the virus's ability to grow in the presence of differing concentrations of the drug and is therefore more useful in the presence of multiple interacting mutations or unclear correlations of mutation and resistance, such as occurs with resistance to protease inhibitors. Genotypic testing is faster and less expensive because all that is necessary is sequencing of the respective genes for the patient's viral isolate. When significant resistance is not expected and information is needed more quickly, genotypic testing would be preferred over phenotypic testing. Some antiretroviral agents have been associated with increased insulin resistance and risk for hyperglycemia, and assessing for this at baseline and during therapy is recommended. This patient, however, already has a normal glucose level at baseline testing, so measuring the glycohemoglobin is not necessary at this time. All patients with HIV should begin antiretroviral therapy as soon as they are ready. Prompt initiation of therapy benefits the patient and reduces the risk of transmission to others, so waiting for repeat viral load and CD4 cell count is inappropriate and unnecessary.
A 45-year-old man is evaluated for shortness of breath with exertion and lower extremity edema of 6 months' duration. He reports no chest pain, fever, or cough. He takes no medications. On physical examination, vital signs are normal. Jugular venous distention is noted. Cardiopulmonary examination reveals crackles at the base of the lungs. He also has 1+ lower extremity edema. Laboratory studies are remarkable for eosinophilia, with an absolute eosinophil count of 2500/µL (2.5 × 109/L). A review of the medical record shows an eosinophil count of 2900/µL (2.9 × 109/L) 1 year ago. An electrocardiogram shows sinus rhythm with low-voltage QRS and nonspecific ST changes. An echocardiogram shows findings compatible with a restrictive cardiomyopathy. Which of the following is the most appropriate initial management? Evaluation for helminth infection Fat pad biopsy Prednisone Repeat laboratory evaluation in 3 months
This patient should be evaluated for hypereosinophilic syndrome (HES), beginning with an evaluation for helminth infection. He has sustained moderate eosinophilia and end-organ damage (restrictive cardiomyopathy), which are characteristic of HES. Organ involvement is more common when the peripheral eosinophil count is greater than 1500/µL (1.5 × 109/L) as in this patient. The numerous neoplastic and nonneoplastic causes of eosinophilia can be recalled using the mnemonic CHINA (Collagen vascular disease, Helminthic infection, Idiopathic, Neoplasia, Allergy/Atopy/Asthma). Helminth infection is a common cause of eosinophilia and should be evaluated and ruled out in all patients. Eosinophilia of any cause can be associated with end-organ damage, and evaluation for organ involvement is indicated. Organs commonly involved are the skin (eczema, erythroderma, urticaria, and angioedema), lungs (parenchymal infiltrates, pleural effusion, lymphadenopathy, and pulmonary emboli), gastrointestinal tract (eosinophilic gastritis, enteritis, colitis, chronic active hepatitis, focal hepatic lesions, eosinophilic cholangitis, Budd-Chiari syndrome), and heart (mitral or tricuspid regurgitation, cardiomegaly, restrictive cardiomyopathy). Fat pad biopsy is performed during evaluation for systemic amyloidosis. Amyloidosis involving the heart can cause restrictive heart disease. However, in this patient with sustained elevation of the eosinophil count, HES is more likely the underlying condition. Prednisone initiation in clinically stable patients with HES without evaluation for the underlying cause of the illness is not recommended because patients with infectious causes of eosinophilia (such as Strongyloides) can experience significant symptom flare. In unstable patients, empiric treatment with glucocorticoids must be initiated emergently with ivermectin if risk factors for exposure to strongyloides are present. Patients with negative infectious evaluations should be evaluated for allergic, immunologic, and neoplastic causes of hypereosinophilia, including myeloproliferative disorders, lymphomas, and solid tumors. Although repeat testing of the eosinophil count is recommended in all patients, at least 1 month apart to confirm sustained elevation, this patient already has two measurements showing sustained hypereosinophilia as well as symptoms of cardiac involvement. Therefore, repeat testing of the eosinophil count in 3 months is not appropriate.
A 55-year-old man is evaluated in the emergency department for abrupt loss of consciousness after a fall. Medical history is notable for atrial fibrillation. He has otherwise been well without additional medical problems. Medications are warfarin and metoprolol. On physical examination, temperature is 37 °C (98.6 °F), blood pressure is 135/85 mm Hg, pulse rate is 83/min and irregular, and respiration rate is 16/min. The patient is obtunded without localizing neurologic findings. Cardiac examination reveals an irregularly irregular rhythm. The remainder of the examination is unremarkable. Head CT scan shows a large subdural hematoma. Laboratory studies show a hemoglobin level of 13 g/dL (130 g/L), platelet count of 183,000/µL (183 × 109/L), and INR of 3.0. Intravenous vitamin K is administered, and plans are made for emergent neurosurgery. Which of the following is the most appropriate treatment? Cryoprecipitate Four-factor prothrombin complex concentrate Fresh frozen plasma Idarucizumab
This patient should be given four-factor prothrombin complex concentrate (4f-PCC). Patients who receive anticoagulation with a vitamin K antagonist have an increased risk for major gastrointestinal and central nervous system bleeding and an increased risk for periprocedural bleeding. Although vitamin K alone can be effective in reversing the effect of warfarin, its hemostatic effect can take several hours, and, in urgent situations, simultaneous replacement of the vitamin K-dependent coagulation factors is necessary. 4f-PCC contains factors II, VII, IX, and X as a lyophilized powder and can be administered quickly in a small reconstituted volume. It provides effective hemostasis 90% of the time and is the preferred option for most patients who require urgent warfarin reversal. Thromboembolism is a potential adverse effect. 4f-PCC should be avoided in patients with a history of heparin-induced thrombocytopenia because it contains residual heparin. Cryoprecipitate would be indicated to treat severe hypofibrinogenemia, usually arising as a consequence of disseminated intravascular coagulation (DIC) or severe liver disease. There is no reason to anticipate DIC in this patient, and he has no history of liver disease. Plasma transfusion is more time consuming (preparation and administration) and is associated with a much higher risk of fluid overload from the volume needed to replace the coagulation factors. It is no longer the best option to reverse warfarin with the availability of 4f-PCC. Idarucizumab is a monoclonal antibody that binds the non-vitamin K antagonist oral anticoagulant dabigatran and causes a rapid reduction in available dabigatran in the body for up to 24 hours. Idarucizumab will not reverse warfarin anticoagulation.
A 38-year-old woman undergoes follow-up evaluation in the office. She was evaluated in the emergency department 3 nights ago with fever and flank pain following 2 days of dysuria. A urine culture and two sets of blood cultures were collected. She was given intravenous ceftriaxone and discharged with a 7-day course of ciprofloxacin. She is now asymptomatic. Medications are ciprofloxacin and an oral contraceptive. On physical examination, vital signs and other findings are normal. Escherichia coli susceptible to ciprofloxacin was isolated from her urine culture and one blood culture. Which of the following is the most appropriate management? Completion of oral ciprofloxacin course Completion of oral ciprofloxacin course with follow-up blood cultures Extended oral ciprofloxacin therapy for 2 weeks Intravenous ceftriaxone Kidney ultrasonography
This patient should complete her prescribed 7-day course of oral ciprofloxacin. She has acute uncomplicated pyelonephritis, which can usually be managed with outpatient oral antimicrobial therapy. Ciprofloxacin for 1 week or levofloxacin for 5 days are the recommended first-line treatment regimens. An initial dose of a long-acting parenteral antibiotic (such as ceftriaxone or aminoglycoside) is suggested when local fluoroquinolone resistance (>10%) is a concern. When a fluoroquinolone antibiotic cannot be used or the bacterial isolate proves resistant, an alternative second-line oral antibiotic should be substituted. Available options include trimethoprim-sulfamethoxazole or the less well-studied oral β-lactam agents. With the exception of pregnancy, follow-up microbiologic cultures and urinalysis are not required or indicated after resolution of infection. Extending the duration of ciprofloxacin therapy beyond 7 days would be warranted for complicated pyelonephritis but should not be influenced by the discovery of the single bloodstream isolate in this otherwise healthy woman. Transient bacteremia does not necessitate hospitalization for parenteral antimicrobial therapy except when the pathogen is found to be multidrug resistant or when complicating features are present (severe illness, obstruction, pregnancy). In adult women with acute kidney infections, urinary tract imaging by ultrasonography or CT is not routinely performed. However, urologic imaging may be useful and is recommended in evaluating patients who do not clinically improve after 72 hours of adequate antimicrobial therapy or when complications such as obstruction or perinephric and renal abscesses are suspected. Such studies should also be considered when evaluating women who experience an excessive number of recurrent urinary tract infections. Read Related TextNext Question
A 40-year-old woman seeks advice on whether she should undergo breast cancer screening with mammography. Her family history is negative for breast and ovarian cancers, and she has no other risk factors for breast cancer. On physical examination, vital signs and the remainder of the examination are normal. The patient is engaged in a discussion of the potential benefits and harms of initiating mammography now, including the potential for false-positive results and overdiagnosis. After the discussion, she states that she is not overly concerned about her risk for breast cancer but is anxious about the potential harms associated with screening. Which of the following is the most appropriate screening test for this patient? Breast self-examination Breast tomosynthesis Screening mammography No testing
This patient should not be screened for breast cancer at this time. In women aged 50 to 74 years, there is a clear benefit to screening mammography, and all breast cancer guidelines recommend screening mammography in this age group. Biennial screening mammography imparts most of the benefit of annual screening mammography with fewer harms, although the recommended screening frequency differs between guidelines. In women younger than 50 years or aged 75 years or older, the balance of benefits and harms is less clear, and screening recommendations vary widely. Most guidelines recommend individualized screening decisions for women aged 40 to 49 years based on patient context and values regarding specific benefits and harms. The American College of Physicians states that the potential harms of screening mammography outweigh the benefits in average-risk patients aged 40 to 49 years and suggests that screening occur only if an informed woman requests it. The U.S. Preventive Services Task Force (USPSTF) concludes that, compared with screening mammography in older women, the number of women in their 40s who benefit from screening mammography is smaller, and the harm is higher; however, the benefit still outweighs the harm. Therefore, the value the patient places on averting death from breast cancer compared with the importance she places on avoiding potential harms (false-positive results, anxiety, and overdiagnosis) can help guide her decision. This patient places more importance on avoiding potential harms and therefore should not pursue screening mammography or breast tomosynthesis at this time; no further testing is the best option. The USPSTF and ACP both recommend against teaching breast self-examination (BSE), as BSE does not reduce breast cancer mortality and is associated with increased rates of breast biopsy. The USPSTF found insufficient evidence to assess the balance of benefits and harms of using breast tomosynthesis, or three-dimensional mammography, as a primary screening method for breast cancer; however, National Comprehensive Cancer Network guidelines indicate that breast tomosynthesis can be considered as an initial screening strategy for average-risk women. In studies, breast tomosynthesis is often associated with double the rate of radiation but lower recall rates. As women progress through their 40s, the number of women who benefit from screening mammography increases, while the chance for harms slightly decreases. A woman's values and preferences may also shift over time; therefore, breast cancer screening should be periodically discussed. Read Related TextNext Question
A 58-year-old man is counseled before undergoing colonoscopy and polypectomy in 10 days' time. A routine screening CT colonography showed two polyps in the descending colon, 10 mm and 8 mm in size. Two years earlier, he had an inferior wall myocardial infarction. His medications are low-dose aspirin, atorvastatin, metoprolol, and enalapril. Which of the following is the most appropriate management of his aspirin therapy? Continue aspirin use until the day of the polypectomy; resume in 48 hours Discontinue aspirin use 7 days before the polypectomy; resume immediately after Discontinue aspirin use 7 days before the polypectomy; resume in 48 hours Do not discontinue aspirin
This patient should not discontinue aspirin use. Aspirin does not need to be discontinued before colonoscopy in any scenario, and data from studies of patients who have undergone polypectomy show no difference in the risk for postprocedure bleeding with discontinuation or continuation of aspirin use. The American College of Gastroenterology's 2016 guidelines for management of lower gastrointestinal bleeding (LGIB) recommend the continuation of aspirin for secondary cardiovascular prophylaxis after polypectomy. Discontinuing aspirin is recommended after polypectomy in patients without established cardiovascular disease who are using aspirin as primary prophylaxis. In patients with established cardiovascular disease, such as this patient, the risks of a potential cardiovascular event outweigh those of potential gastrointestinal bleeding. Holding aspirin for a period of time after a polypectomy, such as 48 hours, has not been shown to reduce postprocedure LGIB and may increase risk for a thromboembolic event in a patient with established cardiovascular disease. *Aspirin for secondary prophylaxis in patients with established cardiovascular disease should be continued after colonoscopy with polypectomy.
A 50-year-old woman is evaluated in the emergency department for fever and tenderness at the site of a cat bite sustained on the dorsum of her right hand 2 days ago. She indicates that she works at an animal shelter. Medical history is significant for a methicillin-resistant Staphylococcus aureus skin infection 6 months ago; she is up to date on all vaccinations, including rabies and tetanus. She takes no medications. On physical examination, temperature is 38.5 °C (101.3 °F), blood pressure is 98/66 mm Hg, pulse rate is 110/min, and respiration rate is 22/min. A tender puncture wound is noted on the dorsum of the right hand; it is warm and surrounded by significant erythema. Some purulent discharge is noted from the wound. Blood cultures are obtained. A plain radiograph of the right hand shows no evidence of gas, foreign body, or bony involvement. In addition to surgical consultation, which of the following antibiotic regimens is the most appropriate treatment at this time? Ampicillin-sulbactam plus vancomycin Ceftriaxone plus metronidazole Ciprofloxacin plus aztreonam Imipenem Vancomycin and clindamycin
This patient should receive antibiotic treatment with ampicillin-sulbactam plus vancomycin in addition to undergoing surgical consultation. She has a cat bite-associated wound infection with accompanying low blood pressure and tachypnea and requires hospitalization. Cat bites are more likely than dog bites to cause infection because of cats' sharp, narrow teeth; infections are caused by organisms from the animal's mouth flora and the host's skin flora. This flora comprises a mix of anaerobic and aerobic organisms, including streptococci, staphylococci, and Bacteroides, Fusobacterium, Porphyromonas, and Pasteurella species. Pasteurella is a facultative anaerobic gram-negative rod that is the most common bacteria in a cat's mouth. Intravenous piperacillin-tazobactam, ampicillin-sulbactam, imipenem, or meropenem would address these organisms but not potential methicillin-resistant Staphylococcus aureus (MRSA). If pus is present, the patient has MRSA risk factors such as a previous MRSA infection or colonization within the last year, or local MRSA prevalence is high, then coverage should also include agents that target MRSA, such as vancomycin or daptomycin. Consequently, the combination of ampicillin-sulbactam and vancomycin would be the best choice for empiric antibiotic coverage because this regimen is effective against serious infections caused by MRSA and the microbiota of a cat's mouth. Ceftriaxone and metronidazole would be effective against many of the organisms encountered in a cat bite-associated skin infection, but it would not cover the possibility of MRSA infection in this patient. Ciprofloxacin and aztreonam also lack coverage for MRSA and some anaerobes likely to be associated with a bite wound. Imipenem is a broad-spectrum antibiotic effective against many of the organisms associated with cat bite wounds, but it does not provide MRSA coverage. Neither vancomycin nor clindamycin provides adequate coverage against Pasteurella species, so this regimen would not be effective for a cat bite-associated infection.
Tx for CO poisoning?
This patient should receive hyperbaric oxygen therapy. Inhaled carbon monoxide has a much higher affinity for hemoglobin binding sites than oxygen and readily forms carboxyhemoglobin, which is an ineffective oxygen transporter and results in reduced tissue oxygen content. Symptoms of carbon monoxide poisoning vary and include headache, confusion, nausea, vomiting, and, in severe cases, loss of consciousness. It is important to understand that carboxyhemoglobin does not lower oxygen saturation measured by standard pulse oximetry or reduce arterial PO2 determined by blood gas analysis. Co-oximetry, which measures carboxyhemoglobin levels, is used to make the diagnosis. She has severe carbon monoxide poisoning due to unvented combustion in a small, enclosed area. Cited indications for hyperbaric oxygen therapy include loss of consciousness, ischemic cardiac changes, neurological deficits, significant metabolic acidosis, or carboxyhemoglobin level greater than 25%. A carboxyhemoglobin level of 50% is critical and needs to be reduced as quickly as possible. Breathing 100% oxygen at normal atmospheric pressure, this patient will clear the carboxyhemoglobin with a half-life of 90 minutes, but hyperbaric oxygen therapy will lower the half-life to 30 minutes. Also, there is considerable risk of delayed neurocognitive impairment following a severe exposure such as this, and hyperbaric oxygen is believed to lower the risk of this long-term complication, although the strength of this evidence is disputed.
A 74-year-old man is evaluated in follow-up for myelodysplastic syndrome diagnosed 3 months ago. Cytogenetic studies and fluorescence in-situ hybridization showed deletion of 5q. It was determined he has low-risk MDS by the Revised International Prognostic Scoring System. He requires erythrocyte transfusions every 2 weeks to prevent symptomatic anemia. He takes no medications. On physical examination, pulse rate is 120/min, and respiration rate is 24/min; other vital signs are normal. Skin pallor is noted. Cardiac examination reveals tachycardia. Laboratory studies show a hemoglobin level of 6.5 g/dL (65 g/L), leukocyte count of 2500/µL (2.5 × 109/L), and platelet count of 220,000/µL (220 × 109/L). Which of the following is the most appropriate treatment? Allogeneic hematopoietic stem cell transplantation Antithymocyte globulin and cyclosporine Imatinib Lenalidomide
This patient with transfusion-dependent myelodysplastic syndrome (MDS) should be treated with lenalidomide. Treatment of MDS has two goals. The first goal is to relieve transfusion dependence; the second is to prevent transformation to acute myeloid leukemia (AML). He has low-risk MDS with chromosome 5q deletion (−5q), and lenalidomide will help with his transfusion-dependent anemia. Patients with infrequent transfusion requirements can be supported with periodic transfusions alone, but in patients requiring frequent transfusions, supplemental treatments to help decrease transfusion requirements should be used to improve quality of life and decrease transfusion-associated iron overload and alloimmunization. In more than 50% of patients with −5q MDS, treatment with low-dose lenalidomide has been shown to achieve transfusion independence and is recommended as first-line therapy. Other second-line treatments for low-risk MDS with −5q include recombinant erythropoietin and the hypomethylating agents azacitidine and decitabine. Hypomethylating agents can reduce transfusion requirements and delay transformation to AML. However, both also worsen blood counts initially and may take up to 6 months to show an effect. Finally, this patient is not at high risk for AML transformation. Allogeneic hematopoietic stem cell transplantation is usually performed in young patients with high-risk MDS. In patients with low-risk disease, early transplantation is not recommended and can be associated with worse survival. Immunosuppression with antithymocyte globulin and cyclosporine, similar to that used for aplastic anemia, has been shown to decrease transfusion requirements in younger patients (age <65 years) and those with hypoplastic bone marrow. However, in this 74-year-old patient with −5q mutation, immunosuppression is not effective and has significant adverse effects. Imatinib is a tyrosine kinase inhibitor that is effective in treating patients with chronic myeloid leukemia and dysregulated tyrosine kinase as a result of the BCR-ABL fusion gene. It is not effective in MDS.
A 27-year-old woman is hospitalized with a 5-day history of intermittent fever, headache, muscle pains, and abdominal cramps. She returned 8 days ago from a 1-week trip to Kenya and Tanzania. She spent time outdoors in the evening and went hiking in a wooded park. She is pregnant at 20 weeks' gestation. She declined pretravel immunizations as well as antimalarial chemoprophylaxis. Her only medication is a prenatal vitamin. On physical examination, temperature is 39.1 °C (102.3 °F), blood pressure is 98/64 mm Hg, pulse rate is 112/min, and respiration rate is 16/min. Her conjunctivae are icteric. Cardiopulmonary examination reveals regular tachycardia. The remainder of the examination is unremarkable. A peripheral blood smear is shown. Which of the following is the most likely causative agent? Plasmodium falciparum Plasmodium knowlesi Plasmodium malariae Plasmodium ovale Plasmodium vivax
This pregnant woman has contracted Plasmodium falciparum malaria after visiting a part of the world where malaria is endemic. Her clinical presentation and peripheral blood smear showing many parasitized erythrocytes demonstrating signet ring forms, together with the absence of trophozoites and schizonts, are typical for infection with P. falciparum. Of returning travelers with acute and potentially life-threatening febrile diseases, P. falciparum malaria accounts for most infections. Furthermore, pregnant women are at increased risk of severe disease and a heightened mortality rate, which is likely related to a reduced immune response. Additionally, effects on the microvasculature and sequestering of organisms in the placenta during pregnancy are known to significantly increase the risk of miscarriage, premature delivery, low-birth-weight neonates, congenital infection, and fetal demise. Accurate identification of P. falciparum and Plasmodium knowlesi is critical because of the risk for severe and potentially lethal infection. P. falciparum should be suspected if the patient traveled to Africa, symptoms begin soon after return from an endemic area, and the peripheral blood smear shows a high level of parasitemia. P. knowlesi is a more recently recognized human pathogen; infection may be severe because of high levels of parasitemia. Examination of the peripheral blood smear reveals all stages of the parasite. The epidemiologic history is helpful because P. knowlesi is not encountered in Africa but rather South and Southeast Asia. Plasmodium malariae, Plasmodium ovale, and Plasmodium vivax are all associated with a low or very low degree of parasitemia, typically less than 2%, and although the risk of recurrence is high, with the exception of P. vivax, the risk for severe disease is low.
Tx for massive DVT?
Thrombolysis should be considered as the initial therapy for patients with iliofemoral deep venous thrombosis with acute limb ischemia. Thrombolytic therapy with alteplase is the most appropriate management for this patient. He has signs and symptoms of extensive proximal deep venous thrombosis (DVT) leading to progressive edema of the leg, compromising arterial blood flow and causing acute ischemic injury. He is at risk for amputation of the leg and increased risk of death. Results of a systematic review of thrombolytic therapy showed greater improvement in venous patency, increased complete clot lysis, and reduced postthrombotic syndrome compared with anticoagulant therapy alone. Thrombolytic therapy was not associated with significant differences in mortality, stroke or intracerebral hemorrhage, pulmonary embolism, or leg ulceration; however, there were few events overall and with wide confidence intervals, and the review could not rule out clinically relevant differences. Bleeding risk with thrombolytic therapy is slightly greater than that associated with anticoagulant therapy, and strict eligibility criteria to reduce the risk of bleeding complications limit the applicability of this treatment. Thrombolytic agents include tissue plasminogen activator, streptokinase, and urokinase. Derivatives of tissue plasminogen activator, such as alteplase, are most commonly used in the setting of thrombosis because they more selectively activate fibrin-bound plasminogen. The parenteral direct thrombin inhibitors argatroban and lepirudin are reserved for the treatment of heparin-induced thrombocytopenia. Argatroban and lepirudin are not thrombolytic. Rivaroxaban, a non-vitamin K antagonist oral anticoagulant, is an accepted monotherapy for DVT (heparin not required). However, rivaroxaban has no immediate effect on thrombus dissolution. Unfractionated heparin potentiates the natural anticoagulant activity of antithrombin and acts as a nonspecific anticoagulant, mostly against thrombin and activated factor X. Heparin will prevent propagation of the thrombus but does not provide an immediate decrease in the size of an existing thrombus; therefore, it will not reverse tissue ischemia. Patients with acute DVT or pulmonary embolism who receive anticoagulant therapy should not undergo inferior vena cava (IVC) filter insertion. Although an IVC filter might decrease the short-term risk of pulmonary embolism, it will predispose to subsequent DVT and increase the risk of postphlebitic syndrome. If a filter were used, it should be inserted after thrombolytic therapy, not following unfractionated heparin administration.
Mechanism of aspirin-induced asthma? What medications induce this?
Treatment with ibuprofen should be discontinued for this patient, and therapy with prednisone should be started. This patient is presenting with signs of aspirin-exacerbated respiratory disease, triggered by his use of ibuprofen. This condition refers to upper and lower respiratory tract reactions to ingestion of substances inhibiting cyclooxygenase-1, which includes aspirin and many NSAIDs. Also known as aspirin-exacerbated respiratory disease or Samter's triad, aspirin-sensitive asthma includes severe persistent asthma, aspirin sensitivity, and hyperplastic eosinophilic sinusitis with nasal polyposis. Asthma is worsened by exposure to aspirin or other NSAIDs, likely because of the inhibition of cyclooxygenase and the resulting increase in leukotriene synthesis. Ingestion of cyclooxygenase-1-inhibiting substances can sometimes cause life-threatening bronchospasm, but patients can also have less severe symptoms, which often cause them to not recognize these substances as a trigger. In addition, the sensitivity to aspirin and other NSAIDs develops over time, generally after the onset of rhinosinusitis. Treatment consists of avoidance of aspirin or other NSAIDs along with typical asthma management. For patients who require aspirin use (such as those with coronary artery disease), an aspirin desensitization procedure can be performed. Successful desensitization down-regulates leukotriene receptors and modifies interleukin sensitivity, which may relieve asthma symptoms in some patients. For this reason, the addition of a leukotriene inhibitor such as montelukast or zafirlukast to an asthma maintenance regimen is helpful for most patients to modify the leukotriene dysregulation thought to contribute to the syndrome; however, addition of a leukotriene inhibitor alone would not be the appropriate next step for this patient. 24-Hour esophageal pH monitoring is helpful to diagnose gastroesophageal reflux disease, but the patient has no symptoms of this; moreover, a trial of empiric proton pump inhibitor therapy would be indicated before invasive testing. A nasal polypectomy may be helpful in the long-term management of this syndrome, but it is not the initial step.
A 58-year-old man is evaluated for follow-up of an asthma exacerbation with no clear trigger, which improved with oral glucocorticoids and a short-acting β2-agonist in addition to his outpatient medications. The patient has a 30-year history of asthma and has had two previous exacerbations during the preceding 12 months. He has no environmental triggers, allergies, atopy, symptoms of reflux, sinus symptoms, snoring, or recent respiratory infections. A recent sleep study was negative for obstructive sleep apnea. Medications are albuterol, budesonide/formoterol, montelukast, tiotropium, and prednisone. On physical examination, vital signs are normal. Oxygen saturation is 97% breathing ambient air. BMI is 36. Pulmonary examination reveals scattered expiratory wheezing. Cardiac examination is normal. Bedside spirometry demonstrates moderate airflow obstruction; FEV1 improves by 15% following inhaled albuterol. Add beclomethasone Perform methacholine challenge testing Start mepolizumab Start omalizumab Supervised weight loss program
Weight loss in patients with obesity-related asthma improves asthma control, lung function, and quality of life; reduces asthma medication use; and should be considered an essential part of the treatment plan. This patient should be referred to a supervised weight loss program. He has poorly controlled asthma despite maximal medical therapy and in the absence of other factors known to exacerbate asthma (environmental triggers, uncontrolled gastroesophageal reflux disease, sinus disease, or obstructive sleep apnea). Obesity is associated with poor asthma control, and the incidence of asthma is 1.47 times greater in patients with obesity than patients without obesity. Weight loss in patients with obesity-related asthma improves asthma control, lung function, and quality of life; reduces asthma medication use; and should be considered an essential part of the treatment plan. Addition of beclomethasone is not appropriate because this patient is already on adequate inhaled therapy and glucocorticoids; increasing the inhaled steroid dose is unlikely to be beneficial. Between attacks and exacerbations of asthma, spirometry can be normal in patients with suspected but undiagnosed asthma. Therefore, a bronchial challenge test, such as a methacholine challenge, may be helpful for diagnosis if positive or make the diagnosis less likely if negative. Methacholine challenge testing is not necessary in this patient because spirometry confirms reversible airflow obstruction (with a 12% or greater improvement in FEV1 or FVC of 200 mL after administration of a bronchodilator), supporting a diagnosis of asthma. Mepolizumab is a monoclonal antibody to IL-5 that has been shown to reduce asthma exacerbations in patients with difficult-to-control asthma and elevated blood eosinophil counts. This patient's eosinophil count was normal; therefore, add-on therapy with mepolizumab is not indicated. Although omalizumab can reduce hospitalizations when added to standard therapy, it is a monoclonal antibody used for treatment of allergic asthma, and it targets elevated levels of IgE. Because this patient does not have a history of allergies and his IgE level is normal, treatment with omalizumab would not be appropriate.